Download as pdf or txt
Download as pdf or txt
You are on page 1of 852

Comprehensive

Objective
PHYSICS
For Competitive Examinations

Volume—II
Comprehensive
Objective

PHYSICS
For Competitive Examinations

Volume—II

By

NARINDER KUMAR
M.Sc. PES (I)
Department of Physics
S.D. Govt. College
Ludhiana

&

Dr. J.K. JUNEJA


M.Sc., M.Phil., Ph.D.
Department of Physics
Hindu College
Sonepat

GOLDEN BELLS
(AN IMPRINT OF LAXMI PUBLICATIONS PVT. LTD.)
l BANGALORE l CHENNAI l COCHIN l GUWAHATI l HYDERABAD
l JALANDHAR l KOLKATA l LUCKNOW l MUMBAI l RANCHI

NEW DELHI
Published by :
GOLDEN BELLS
94-B, Near Hindi Park, Daryaganj
New Delhi-110002.

RS 011-2326 01 90
Phones :
T 011-2328 71 26
RS 011-2326 02 48
Faxes :
T 011-2326 22 79

Branches :
• 129/1, IIIrd Main Road, IX Cross, Chamrajpet, Bangalore-560018 (Phone : 080-26 61 15 61-62)
• 26, Damodaran Street, T. Nagar, Chennai-600017, Tamil Nadu (Phone : 044-24 34 47 26)
• 43/1394 D, St. Benedict’s Road, Ernakulam North, Cochin-682018, Kerala (Phone : 0484-239 70 04)
• Pan Bazar, Rani Bari, Guwahati-781001 (Phones : 0361-254 36 69, 251 38 81)
• 4-2-453, Ist Floor, Ramkote, Hyderabad-500095 (Phone : 040-24 75 02 47)
• Adda Tanda Chowk, ND-365, Jalandhar City, Punjab (Phone : 0181-222 12 72)
• R.D.B. Chambers (Formerly Lotus Cinema) 106/A, 1st Floor, S.N. Banerjee Road, Kolkata-700014 (Phones : 033-22 27 52 47 ;
Fax : 22 27 37 73 E-mail : [email protected])
• 18, Madan Mohan Malviya Marg, Lucknow-226001 (Phone : 0522-220 95 78 ;
E-mail : [email protected] ; Fax : 2204098)
• 142-C, Victor House, Ground Floor, N.M. Joshi Marg, Lower Parel (W), Mumbai (Phones : 022-24 91 54 15, 24 92 78 69)
• Radha Govind Street, Tharpagna, Ranchi-834001 (Phone : 0651-230 77 64)

EMAIL : [email protected]

Website : www.laxmipublications.com

GBC-5009-495-OBJECTIVE PHYSICS II

All rights reserved with the Publisher.

Revised Edition, 2006


Reprint, 2007

C—129/06/06

Laser Typesetted at : Goswami Printers, Delhi Printed at : Ajit Printers, Delhi.


Comprehensive
Objective

PHYSICS
Volume—I
Comprehensive Objective Physics is essential for all the students preparing for various Engineering & Medical En-
trance Examinations given below and also for students preparing for other State Services Exams.
• IIT—JEE
• AIIMS
• DPMT
• BVP, Pune
• CPMT, U.P.
• JEE, Orissa
• CEE, Assam
• CEE, Kerala
• AFMC, Pune
• CMC, Vellore
• CBSE—AIEEE
• CBSE—PMT
• UPSEAT, U.P.
• PET/PMT, M.P.
• BHU, Varanasi
• CMC, Ludhiana
• CEET, Haryana
• CET, Karnataka
• MGIMS, Wardha
• JEE, West Bengal
• PET/PMT, Rajasthan
• DCE, NSIT, Delhi
• JIPMER, Pondicherry
• Aligarh Entrance Exam.
• Manipal Entrance Exam.
• GGSIP University, Delhi
• EAMCET, Andhra Pradesh
• CET/TNPCEE, Tamil Nadu
• Indian School of Mines, Dhanbad
• MLNR, Engineering Entrance Exam.
• CET/PMT, Punjab & UT Chandigarh
• Birla Institute of Technology Mesra, Ranchi
• Other State Level Competitive Examinations
List of Chapters in Volume—I and Volume—II

Volume—I

Unit 0 MATHEMATICAL TOOLS


Unit I INTRODUCTION AND MEASUREMENT
1.01 Introduction, Units, Measurement and Dimension
1.02 Significant Figures and Error Analysis
Unit II DESCRIPTION OF MOTION IN ONE DIMENSION
Unit III DESCRIPTION OF MOTION IN TWO AND THREE DIMENSIONS
3.01 Vectors
3.02 Motion in Two and Three Dimensions
Unit IV LAWS OF MOTION
Unit V WORK, ENERGY AND POWER
Unit VI ROTATIONAL MOTION (Circular Motion, Centre of Mass and Rotational Mechanics)
Unit VII GRAVITATION
Unit VIII PROPERTIES OF MATTER
8.01 Elasticity
8.02 Fluids at Rest
8.03 Surface Tension
8.04 Fluids in Motion
8.05 Kinetic Theory of Gases
Unit IX HEAT AND THERMODYNAMICS
Unit X OSCILLATION AND WAVES
3 Practice Papers

Volume—II

Unit XI ELECTROSTATICS
Unit XII CURRENT ELECTRICITY
Unit XIII MAGNETIC EFFECT OF CURRENT AND MAGNETISM
Unit XIV ELECTROMAGNETIC INDUCTION AND ALTERNATING CURRENT
Unit XV OPTICS
Unit XVI DUAL NATURE OF MATTER AND RADIATIONS
Unit XVII ATOMIC NUCLEUS
Unit XVIII SOLIDS AND SEMICONDCUTOR DEVICES, ELECTROMAGNETIC WAVES AND
PRINCIPLES OF COMMUNICATION
3 Practice Papers
3 Model Test Papers based on Full Syllabus
PREFACE TO THE SECOND EDITION
There is no dearth of competition books for engineering and medical entrance examinations. But there is no single
book which completely satisfies the intelligent student. The present book is a serious attempt in this direction. We have
left no stone unturned to get ‘EXTREMELY SATISFACTORY’ grade from the readers of the book.
Some salient features of the book are as under :
(i) No superfluous question. Only those questions have been included which have either been already set in different
examinations or are expected to be set.
(ii) Most systematic presentation. It is the only book in the market which has systematically categorised the study
material of each unit. The study material has been clearly demarcated in the following seven segments.
1. SYNOPSIS
2. SET I [Contains MCQs based on Memory Work, Hard Facts, Important Terms, Important Formulae etc.]
3. SET II [Contains MCQs based on Important Basic Concepts, Application of Formulae and Brain Teasers]
4. SET III [Contains MCQs with more than one correct alternative]
5. SET IV [Contains MCQs based on Typical Numerical Bank]
6. SELF-EVALUATION TEST I [Contains Expected Questions for Forthcoming Examinations]
7. SELF-EVALUATION TEST II
In addition to the above, ‘Knowledge Plus’ has also been included at some places.
(iii) Time-saving solutions. Special techniques have been used in solving questions. These techniques have been high-
lighted at different places in the book. If the reader tries to grasp these techniques, considerable time will be saved.
The book has been divided into two volumes. The divisions are as under :

Units

Volume—I I—X
Topics included
Mathematical Tools ; Introduction, Units, Measurements and Dimension ; Description of Motion in One Dimension ;
Description of Motion in Two and Three Dimensions (Vectors ; Motion in Two and Three Dimensions) ; Laws of
Motion ; Work, Energy and Power ; Rotational Motion (Circular Motion ; Centre of Mass and Rotational Mechanics) ;
Gravitation ; Properties of Matter (Elasticity ; Fluids at Rest ; Surface Tension ; Fluids in Motion ; Kinetic Theory
of Gases) ; Heat and Thermodynamics ; Oscillations and Waves.

Units

Volume—II XI—XVIII
Topics included
Electrostatics ; Current Electricity ; Magnetic Effect of Current and Magnetism ; Electromagnetic Induction and
Alternating Current ; Optics ; Dual Nature of Matter and Radiations ; Atomic Nucleus ; Solids and Semiconductor
Devices, Electromagnetic Waves and Principles of Communication.
3 Practice Papers are designed in such a way so as to enable the students to evaluate their skills in Volume—I and
Volume—II separately.
Model Test Papers based on full syllabus are included in Volume—II for Self Assessment.

Utmost effort has been made to make this book as best as possible. But it is our firm conviction that there is always
a scope for improvement. So, we invite your suggestions for the further improvement of the book. It would be our
earnest endeavour to incorporate all good suggestions in the next edition.
—AUTHORS
CONTENTS

Unit XI ELECTROSTATICS

11. Electrostatics

• Synopsis 3 Set IV : MCQs based on Typical Numerical 77


• Question Bank with Answers/Solutions Bank (Exclusively for Engineering
Entrance Tests)
Set I : MCQs based on Memory work, Hard 10
Facts, Important Terms, Important • Self-Evaluation Test I with Answers/ 92
Formulae etc. Solutions (Expected Questions for
Forthcoming Examinations)
Set II : MCQs based on Important Basic 33
Concepts, Application of Formulae, • Self-Evaluation Test II with Answers/ 99
Brain Teasers Solutions
Set III : MCQs with one or more than one 73
Correct Alternative

Unit XII CURRENT ELECTRICITY


12. Current Electricity Set IV : MCQs based on Typical Numerical 204
• Synopsis 111 Bank (Exclusively for Engineering
Entrance Tests)
• Question Bank with Answers/Solutions
• Self-Evaluation Test I with Answers/ 211
Set I : MCQs based on Memory Work, Hard 123 Solutions (Expected Questions for
Facts, Important Terms, Important Forthcoming Examinations)
Formulae etc.
• Self-Evaluation Test II with Answers/ 221
Set II : MCQs based on Important Basic 141
Solutions
Concepts, Application of Formulae,
Brain Teasers
Set III : MCQs with one or more than one 201
Correct Alternative

Unit XIII MAGNETIC EFFECT OF CURRENT AND MAGNETISM


13. Magnetic Effect of Current and Set III : MCQs with one or more than one 315
Magnetism Correct Alternative
• Synopsis 231 Set IV : MCQs based on Typical Numerical 317
• Question Bank with Answers/Solutions Bank (Exclusively for Engineering
Entrance Tests)
Set I : MCQs based on Memory Work, Hard 254
• Self-Evaluation Test I with Answers/ 326
Facts, Important Terms, Important
Solutions (Expected Questions for
Formulae etc.
Forthcoming Examinations)
Set II : MCQs based on Important Basic 276
Concepts, Application of Formulae, • Self-Evaluation Test II with Answers/ 333
Brain Teasers Solutions
Unit XIV ELECTROMAGNETIC INDUCTION AND ALTERNATING CURRENT
14. Electromagnetic Induction and Alternating Set III : MCQs with one or more than one 404
Current Correct Alternative
• Synopsis 343 Set IV : MCQs based on Typical Numerical 407
• Question Bank with Answers/Solutions Bank (Exclusively for Engineering
Entrance Tests)
Set I : MCQs based on Memory Work, Hard 358
• Self-Evaluation Test I with Answers/ 409
Facts, Important Terms, Important
Solutions (Expected Questions for
Formulae etc.
Forthcoming Examinations)
Set II : MCQs based on Important Basic 376
Concepts, Application of Formulae, • Self-Evaluation Test II with Answers/ 416
Brain Teasers Solutions

Unit XV OPTICS
15. Optics Set IV : MCQs based on Typical Numerical 602
• Synopsis 429 Bank (Exclusively for Engineering
Entrance Tests)
• Question Bank with Answers/Solutions
• Self-Evaluation Test I with Answers/ 613
Set I : MCQs based on Memory Work, Hard 448 Solutions (Expected Questions for
Facts, Important Terms, Important Forthcoming Examinations)
Formulae etc.
• Self-Evaluation Test II with Answers/ 620
Set II : MCQs based on Important Basic 502
Solutions
Concepts, Application of Formulae,
Brain Teasers
Set III : MCQs with one or more than one 593
Correct Alternative

Unit XVI DUAL NATURE OF MATTER AND RADIATIONS


16. Dual Nature of Matter and Radiations Set III : MCQs with one or more than one 664
• Synopsis 631 Correct Alternative
• Question Bank with Answers/Solutions Set IV : MCQs based on Typical Numerical 666
Bank (Exclusively for Engineering
Set I : MCQs based on Memory work, Hard 637 Entrance Tests)
Facts, Important Terms, Important
• Self-Evaluation Test I with Answers/ 668
Formulae etc.
Solutions (Expected Questions for
Set II : MCQs based on Important Basic 646 Forthcoming examinations)
Concepts, Application of Formulae,
Brain Teasers • Self-Evaluation Test II with Answers/ 674
Solutions

Unit XVII ATOMIC NUCLEUS


17. Atomic Nucleus Set IV : MCQs based on Typical Numerical 741
• Synopsis 683 Bank (Exclusively for Engineering
Entrance Tests)
• Question Bank with Answers/Solutions
• Self-Evaluation Test I with Answers/ 744
Set I : MCQs based on Memory Work, Hard 701 Solutions (Expected Questions for
Facts, Important Terms, Important Forthcoming Examinations)
Formulae etc.
• Self-Evaluation Test II with Answers/ 752
Set II : MCQs based on Important Basic 712
Solutions
Concepts, Application of Formulae,
Brain Teasers
Set III : MCQs with one or more than one 739
Correct Alternative
Unit XVIII SOLIDS AND SEMICONDUCTOR DEVICES, ELECTROMAGNETIC WAVES
AND PRINCIPLES OF COMMUNICATION
18. Solids and Semiconductor Devices, Electro- Set III : MCQs with one or more than one 815
magnetic Waves and Principles of Communi- Correct Alternative
cation Set IV : MCQs based on Typical Numerical 818
• Synopsis 763 Bank (Exclusively for Engineering
Entrance Tests)
• Question Bank with Answers/Solutions
• Self-Evaluation Test I with Answers/ 820
Set I : MCQs based on Memory Work, Hard 779
Solutions (Expected Questions for
Facts, Important Terms, Important
Forthcoming Examinations)
Formulae etc.
• Self-Evaluation Test II with Answers/ 826
Set II : MCQs based on Important Basic 795
Solutions
Concepts, Application of Formulae,
Brain Teasers

PRACTICE PAPERS BASED ON UNITS XI—XVIII


• Practice Paper 1 1
• Practice Paper 2 7
• Practice Papers 3 13

MODEL TEST PAPERS BASED ON FULL SYLLABUS


• Model Test Paper 1 1
• Model Test Paper 2 10
• Model Test Paper 3 18
FREE COUPON
FOR
Booklet containing List of Engineering/Medical Colleges in India with courses offered & seats available, (as per
our records)
Dear student,
In order to receive the above booklet, kindly fill in the coupon below and send it alongwith a self-addressed envelope
affixed with Rs. 10.00 postal stamp to M/s Golden Bells, 94-B, Hindi Park, Daryaganj, New Delhi-2.
We shall send the above booklet to you immediately on receipt of this coupon.

Your Name in Capital Letters ____________________________________________________________________________

Your Father’s/Mother’s Name _____________________________________________________________________________

Your Postal Address _____________________________________________________________________________________

District ________________________________ State _____________________________________

Telephone ________________________________ PIN CODE

Your E-mail _____________________________________________________________________________________________

Other Objective Physics books consulted 1. ________________________________ by __________________________

for the Preparation of Entrance Exams. 2. ________________________________ by __________________________

3. ________________________________ by __________________________

Which of the above stated books you liked most ?

Your frank opinion about Comprehensive Objective Physics


(i) Do you like this book ? Yes/No
(ii) Where do you place this book in rating, as compared to the books you have so far consulted ?

Best Better Good Bad Worse Worst

Names and addresses of Physics teachers under whose guidance you are preparing for the competitions :

1. ______________________________________________________________________________________________________

2. ______________________________________________________________________________________________________

3. ______________________________________________________________________________________________________

Tick booklet required, List of Engineering Colleges or List of Medical Colleges


Name and address of the Coaching Institute you are attending/have attended :
________________________________________________________________________________________________________

_______________________________________________________________________________________________________

Date ____________________ Signature ____________________

*Please give full address of the Coaching Institute and Physics Teacher, with telephone numbers.
Comprehensive Objective Physics—Volume—I
UNIT XI

ELECTROSTATICS

l Frictional electricity l Charges and their conservation l Coulomb’s law l Forces be-
tween two point electric charges l Forces between multiple electric charges l Superposition
principle and continuous charge distribution l Electric field and its physical significance l Electric
field due to a point charge l Electric field lines l Electric dipole l Electric field due to a
dipole and behaviour of dipole in a uniform electric field l Electric potential-Physical
meaning l Potential difference l Electric potential due to a point charge l A dipole and sys-
tem of charges l Equipotential surfaces l Electrical potential energy of a system of two point
charges and of electric dipoles in an electrostatic field l Electric flux l Statement of Gauss’s
theorem and its applications to find field due to an infinitely long straight wire, uniformly charged
infinite plane sheet and uniformly charged thin spherical shell (field inside and outside) l Conductors
and insulators l Presence of free charges and bound charges inside a conductor l Dielectrics
and electric polarisation l General concept of a capacitor and capacitance l Combination of
capacitors in series and in parallel l Energy stored in a capacitor l Capacitance of a parallel
plate capacitor with and without dielectric medium between the plates l Van de Graff generator.

UNIT DETAILS

1. Synopsis Points 65 Plus

2. Illustrations 10 Plus

3. MCQs from Competitive Examinations 280 Plus

4. Self-Evaluation Tests 2

5. Total Number of MCQs 580 Plus

6. Total Number of Solutions 570 Plus


CHAPTER 11

ELECTROSTATICS

SYNOPSIS

1. The existence of two types of charges was discovered 13. Electric field intensity at general point due to a dipole
by Du Fay. is :
2. It was Benjamin Franklin who first assigned positive
1 p
and negative signs to the charges. E= 3 cos2 θ + 1
4 πε0 r 3
3. The fact that like charges repel each other and unlike
charges attract each other can be easily demonstrated Here, θ is the angle between the line joining the ob-
by using pith balls. servation point with the mid-point of the dipole and
4. Electric charge is additive, quantised and conserved. the axial line of the dipole.
5. According to Coulomb’s law of electrostatics, 14. Torque experienced by an electric dipole in uniform
→ → →
1 q1q2 electric field is : τ = p × E .
F=
4 πε r 2 15. The time period of angular SHM of a dipole of mo-
Here, ε = ε0εr , where the letters have usual meanings. ment of inertia I and dipole moment p in electric field
6. When a dielectric is introduced between two point E is
charges, the force between the two point charges is
reduced. I
T = 2π .
7. A material is suitable for use as dielectric if its dielec- pE
tric constant and dielectric strength has large val- 16. According to Gauss’ law in electrostatics, Electric flux,

z
ues.
→ → n
8. 1 emu of charge = c esu of charge 1
or 1 emu of charge = 3 × 1010 esu of charge.
φE =
E
E . dS =
ε0 ∑q i
i= 1
9. 1 coulomb of charge = 3 × 109 statcoulomb, 1 abcoulomb
17. Dimensional Formulae of Electric Field E and
= 10 coulomb.
Electric Flux φ
10. 1 abcoulomb = 10 coulomb.
11. Electric field intensity at a point on the axial line of [Force] [MLT −2 ]
[E] = = = [MLT–3 A–1]
the electric dipole is : [Charge] [ AT]
1 2 pr [φ] = [E] [Area] = [MLT –3 A–1] [L2]
E=
4 πε (r − l2 )2
2 = [ML3 T –3 A–1]
where p is the electric dipole moment, “2l” is the length 18. The electric field due to infinite line charge of linear
of the dipole and r is the distance of the observation 1 2λ
charge density λ is .
point from the mid-point of the dipole. 4 πε 0 r
12. Electric field intensity at a point on the equatorial 19. The electric field due to an infinite plane sheet of charge
line of the electric dipole is :
σ
1 p is , where σ is surface density of charge.
E= 2ε 0
4 πε (r2 + l2 )3 / 2

3
4 COMPREHENSIVE OBJECTIVE PHYSICS

σ 33. Uniformly Charged Non-Conducting Sphere of


20. The electric field near a conductor of any shape is . Charge Q and Radius R
ε0
1 Q
21. Line integral of electric field between two points rep- (i) Electric field (at r > R) =
resents the work done by electric field in moving a 4 πε 0 r 2
unit positive charge between two points in the field.
1
22. Electric potential at a point in an electric field, Note that E ∝
r2
V= −
z

r→ →
E . dr .

23. If the charge is distributed uniformly along a line of


Also note that the charge on the sphere behaves
as if concentrated at the centre.
1 Q
length l and λ is the linear charge density, then (ii) Electric field (at r = R) =

z
4 πε 0 R 2
1 λdl
V= 1 Qr
4 πε 0 l r (iii) Electric field (at r < R) =
4 πε 0 R 3
24. If the charge is distributed uniformly over an area S
and σ is the charge per unit area, then  Note that E ∝ r.

V=
1
4 πε 0 z
S
σdS
r
(iv) Electric potential (at r > R) =
1 Q
4 πε0 r
25. If the charge is uniformly distributed over a volume 1 Q
V and ρ is the charge per unit volume, then (v) Electric potential (at r = R) = .

z
4 πε0 R
1 ρdV
V= . 34. Charged Spherical Conductor of Charge Q and
4 πε 0 V r Radius R
26. Electric potential due to an electric dipole at point on 1 Q
the axial line of the dipole is : (i) Electric field (at r > R) =
4 πε 0 r 2
1 p 1 Q
V= (ii) Electric field (at r = R) =
4 πε 0 r − l2
2
4 πε 0 R2
27. Electric potential due to an electric dipole at point on (iii) Electric field (at r < R) = 0
equational line is zero. 1 Q
28. Electric potential at a general point due to dipole is : (iv) Electric potential (at r > R) =
4 πε0 r
1 p cos θ 1 Q
V= (v) Electric potential (at r = R) = .
4 πε 0 r2 4πε0 R
29. The electric lines of force and equipotential surfaces 1 Q
constitute a mutually perpendicular network. (vi) Electric potential (at r < R) =
4πε0 R
30. No work is required to be done in moving a charge
over an equipotential surface.  Note that the electric potential at every point in-
31. Equipotential surfaces indicate regions of strong and side the conductor is the same as on the surface of the
weak electric fields. conductor.
32. Short dipole 35. Uniformly Charged Ring of Radius r
(i) Potential at equator is zero 1 qx
(i) Electric field at a point on the axis =
1 p 4 πε 0 (r 2 + x2 )
(ii) Electric field at a point on equator =
4 πε 0 r 3 Here x is the distance of the observation point from
the centre of the ring.
1 p
(iii) Potential at a point on axial line = (ii) Electric field at the centre of the charged ring is
4 πε 0 r 2 zero.
1 2p 1 q
(iv) Electric field at a point on axial line = . (iii) Potential at a point on the axis = .
4 πε 0 r 3 4 πε0 r + x2
2
ELECTROSTATICS 5
36. Potential at a point, inside an uniformly charged non-
conducting sphere of charge Q and radius R, at a dis-
1 Q (3R 2 − r 2 )
tance r is .
4 πε0 2 R3 +q –q

37. The potential difference between two points at dis-


tances r1 and r2 from an infinite line charge of linear l l
1 r
charge density is 2λ log e 2 .
4 πε 0 r1
38. Electric field, due to a point charge q, at a distance
d1 + d2 where d1 is thickness of medium of dielectric
constant k1 and d2 is thickness of medium of dielec- Fig. 1
tric constant k2. (ii) Interaction energy of the system mentioned in (i),
1 q 1 q2 q2
E= 2 U= − =− .
4 πε0 (d1 K 1 + d2 K2 ) 4πε 0 2l 8πε0 l
1 q 47. The electric force be-
 Note. Potential, V = ® ®
4 πε 0 d1 k1 + d2 k2 tween two short dipoles p1 p2
of dipole moments p 1
39. The Electric Field at a Point is Equal to the r
and p2 at separation r,
Negative Gradient of the Electric Potential. The Fig. 2
→ when they are co-axial,
negative sign indicates that the direction of E is al-
1 6 p1 p2
ways in the direction of decrease of electric potential. is ‘ – ’.
The SI unit of potential gradient is Vm–1. 4 πε 0 r 4
40. Electrostatic potential energy due to point charges q1 The couple is zero. The potential energy is
1 q1q2 1 2 p1 p2
and q2 separated by a distance r is . − .
4 πε 0 r 4 πε 0 r3
41. Work done in rotating an electric dipole through an 48. The electric force between two
angle θ is : W = pE(1 – cos θ). short dipoles of dipole moments
42. Electric potential energy of an electric dipole in a uni- → →
p1 and p2 at separation r, ® ®
→ → p1 r p2
form electric field is : W = − p . E = – pE cos θ. when they are parallel, is
43. Binding energy of an electric dipole = – (potential en- 1 3 p1 p2
ergy of charges of the dipole) ‘+ ’. The couple is
4 πε 0 r 4
F 1
= − G−
q2I 1 q2 zero. The potential energy is
Fig. 3

H 4πε 0 2lJK
=
4 πε0 2l
.
1 p1 p2
‘ ’.
44. Electrostatic potential energy is the product of charge 4 πε0 r3
and electrostatic potential. 49. The electric force between F
45. The electrostatic self-energy of a charged dielectric two short dipoles of dipole
moments p 1 and p 2, when ®
1 3 q2 they are mutually perpen- p1 ®
sphere of charge q and radius r is . p2
4 πε 0 5r
1 2 p1 p2
dicular, is ± .
46. Method of Electrical Images 4 πε 0 r4
F

(i) Force between point charge ‘+ q’ and an earthed r


conductor = Force between charges + q and – q 1 2 p1 p2
Couple on p1 is . Fig. 4
separated by distance 2l. 4 πε 0 r 3

1 q2 1 p1 p2
= Couple on p2 is . The potential energy is
4 πε0 (2l)2 4 πε0 r 3
zero.
6 COMPREHENSIVE OBJECTIVE PHYSICS

50. DIMENSIONAL FORMULA OF CAPACITANCE C1V1 + C2 V2


and loss of energy is
[Q] [Q] C1 + C2
[C] =
[V]
=
LM OP
W 1 C1C2
QN Q (V – V2)2.
2 C1 + C2 1
or [C] = [W–1 Q2] = [ML2 T –2]–1 [AT]2 →
62. Relation Between P and Surface Density of
= [M–1L–2 T4A2] Induced Charges
51. The capacity of earth or a conductor connected to earth
qi x qi
is infinity. The conductor may be of any shape and P= = = σi
may carry any amount of charge. Ax A
52. The capacity of a parallel plate capacitor is →
So, when P is normal to the surface of the dielectric,
ε0 A the polarisation is numerically equal to σi.
C= K ,
d →
σi = P . n
where K is the dielectric constant of the medium be-
tween the plates. Here, n is a unit normal vector at the surface.
53. The magnitude of the force between the plates of the 63. Electric Susceptibility of Dielectric
Q2 P ∝ E or P = χε0E
capacitor is .
2ε 0 A χ is known as the electric susceptibility of the
dielectric. It describes the behaviour of a dielectric.
54. When capacitors are connected in series, the recipro-
Larger the value of χ, greater will be the polarisation
cal of the combined capacity is equal to the sum of the
of the dielectric in an electric field. In the case of
reciprocals of the individual capacities.
vacuum, the value of χ is zero because there are no
55. When capacitors are connected in parallel, the equiva- molecules in vacuum.
lent capacity is equal to the sum of the individual
64. Battery Disconnected and Dielectric Intro-
capacitances.
duced in Charged Parallel Plate Capacitor (Di-
56. The energy of a charged capacitor is electric fills the space completely)
Q2 1 1 (i) Charge on the plates remains unchanged
or CV 2 or QV .
2C 2 2 (ii) Capacity increases by a factor of K
→ (iii) Potential difference decreases by a factor of K
57. If an electric field E exists at any point in space, then (iv) Electric field decreases by a factor of K
that point can be regarded as a seat of stored energy.
(v) Energy stored in the capacitor decreases by a fac-
The amount of energy stored per unit volume is
tor of K.
1
ε 0E2 . 65. Battery Remains Connected and Dielectric In-
2
troduced (Dielectric fills the space completely)
58. The SI unit of energy density is Jm–3.
(i) Potential difference remains constant
59. The force per unit area on the surface of a conductor
(ii) Capacity increases by a factor of K
σ2 1
due to its own charge is i.e., ε 0E2 . This is called (iii) Charge on the capacitor increases by a factor of K
2ε 0 2
(iv) Electric field remains unchanged
electrostatic pressure. It is sometimes called ‘mechani-
(v) Energy stored in the capacitor increases by a fac-
cal force’.
tor of K.
60. When capacitors are connected in series or parallel,
the energies of the individual capacitors are added 66. COALESCENCE OF n CHARGED DROPLETS
up. Consider n droplets, each charged to a potential V.
61. When two conductors of capacities C 1 and C 2 Let us determine the potential V′ when these droplets coalesce
maintained at potentials V1 and V 2 are brought in to form a large drop.
electrostatic contact, then common potential is Let r be the radius of each droplet. Let R be the radius
of the large drop.
ELECTROSTATICS 7

4 3 4
Then, n × πr = πR 3
3 3
or R = n1/3 r
or C = n1/3c
Also, Q = nq

Dividing,
Q nq
= 1/3
C n c Fig. 5
or V′ = n2/3 V
Sol. The semi-circle shown in Fig. 5 is an equipo-
67. SPLITTING A DROP OF CHARGE Q AND
tential line. So, VPA = VPB = VPC. Since W = QV, therefore
RADIUS R
WA = W B = W C
(i) Suppose the drop is split into n drops, each of ra-
dius r. Let q, v and c be the charge, potential and So, (c) is the right choice.
capacity respectively of each small drop. Illustration 2. In the Fig. 6, the capacitance of each
4 4 capacitor is 3 µ F. The effective capacitance between A and B
Now, n × πr 3 = πR 3
3 3 is
or n1/3 r = R
or n1/3 c = C [∵ capacity ∝ radius]
C
or c = 1/ 3
n
q Q × n1/ 3 V
(ii) Again, v = = = 2/ 3
c nC n
(iii) Let σ and σ′ represent the surface charge densi- Fig. 6
ties of the large drop and small drop respectively.
Q q (a) 3/4 µ F (b) 3 µ F
σ= 2 , σ′ = (c) 6 µ F (d) 5 µ F.
4 πR 4 πr 2
σ Q 4 πr 2 nq × r 2 nr 2 [National Standard Exam. in Physics 1997]
= × = 2 = 2 / 3 2 = n1/3
σ ′ 4 πR 2
q R ×q n r Sol. The equivalent circuit of the given circuit is :
σ
or σ′ = 1/3
n
(iv) Let E and E′ represent the electric fields of the
large drop and small drop respectively.
σ σ′
Then, E= and E′ =
ε0 ε0
E′ σ ′ ε 0
Now, = × Fig. 7
E ε0 σ
Capacity of lower branch is
σ′ σ 1
= = =
σ n1/3 σ n1/3 6×3 18
µF i.e., µF or 2 µF
E 6+3 9
or E′ = 1/3
n
Effective capacity between A and B
ILLUSTRATIONS = (3 + 2) µF = 5 µF
Illustration 1. Consider the situation of Fig. 5. The So, (d) is the right choice.
work done in taking a point charge Q from P to A is WA,
from P to B is WB and from P to C is WC. Illustration 3. The variation of electric field with
(a) WA < WB < WC (b) WA > WB > WC distance x from the centre of a uniformly charged dielectric
sphere of radius “a” is given by
(c) WA = WB = WC (d) WA ≥ WB < WC .
8 COMPREHENSIVE OBJECTIVE PHYSICS

E E closed loop. So, (b) and (d) are ruled out. Again, like charges
repel.
So, (c) is the correct choice.
Illustration 5. In P
Fig. 10 are shown three
d
parallel plates, each of area
A ; the separations are Q d
a a
x x
equal and of magnitude d.
(A) (B)
With the outer plates Wire
E
shortcircuited, the capaci-
Fig. 10
tance between P and Q is
(a) ε0 A/d (b) ε0 A/(2d)
(c) 2ε0 A/d (d) 2πε0 A/d.
[National Standard Exam. in Physics 1996]
Sol. The given arrangement is equivalent to a paral-
a x a x lel combination of two capacitors.
(C) (D)
So, (c) is the right choice.
Illustration 6. Consider q
Fig. 8
+ –
the situation shown in the Fig. 11.
(a) [A] (b) [B] + –
The capacitor A has a charge q
(c) [C] (d) [D]. on it whereas B is uncharged. The + – S
[National Standard Exam. in Physics 1997] charge appearing on the capaci- + –
Sol. Inside the sphere, E varies linearly with distance tor B a long time after the switch A B
from the centre, outside inversely as square of this distance. is closed, is Fig. 11
So, (a) is the right choice. (a) zero (b) q/2
Illustration 4. Three positive charges of equal value (c) q (d) 2q.
q are placed at the vertices of an equilateral triangle. The [IIT Screening 2001]
resulting lines of force should be sketched as in Sol. Conditions for flow of charge are not suitable.
Negative charge is bound charge.
So, (a) is the right choice.
Illustration 7. In the cir- V1 V2
cuit of Fig. 12, the steady state
voltages V 1 and V2 are respec-
tively : 3 mF 2 mF

(a) 12 V, 18 V
(b) 18 V, 12 V 30 V
(c) 15 V, 15 V Fig. 12
(d) 30 V, 30 V.
[National Standard Exam. in Physics 1996]
Sol. C1V1 = C2V2
2 V2
or 3V1 = 2V2 or V1 =
3
Now, V1 + V2 = 30
Fig. 9 2 V2
+ V2 = 30
[IIT Screening 2001] 3
Sol. In (a), electric lines of force end on positive charge. 90
So, (a) is ruled out. In (b) and (d), electric lines of force form or V2 = volt = 18 V
5
ELECTROSTATICS 9

2 and then separated. The final charges Q1 and Q2 on them


V1 = × 18 V = 12 V will satisfy
3
So, (a) is the right choice. Q1 C1 Q1 C1
(a) < (b) =
Illustration 8. A uniform electric field pointing in Q2 C2 Q2 C2
positive x-direction exists in a Q1 C1
C y=+1 (c) > (d) Q1 = C2 .
region. Let A be the origin, B Q2 C2 Q2 C1
be the point on the x-axis at x =
A B E [MP PMT 1999 ; KCET 2000]
+ 1 cm and C be the point on x=+1
the y-axis at y = ± 1 cm. Then Sol. When the spheres are in electrostatic contact,
Fig. 13 they acquire a common potential.
the potentials at the points A,
B and C satisfy : Q1 Q 2 Q C
∴ = or 1 = 1
(a) VA < VB (b) VA > VB C1 C2 Q2 C2
(c) VA < VC (d) VA > VC . So, (b) is the right choice.
[IIT Screening 2001] Illustration 11. A charged ball B +
hangs from a silk thread S, which makes an +
Sol. Potential decreases in the direction of the field. +
angle θ with a large charged conducting sheet P +
So, (b) is the right choice. P, as shown in Fig. 15. The surface charge 
+
Illustration 9. In the circuit shown below, O is density σ of the sheet is proportional to + S
+
connected first to A. It charges capacitor 4 µ F. Now the (a) tan θ (b) sin θ
+
connection of O is switched to B. The charge on the 4 µ F (c) cot θ (d) cos θ. + B
capacitor thereby changes by a factor [AIEEE 2005] Fig. 15
(a) 1 (b) 2/3 Sol. T sin θ = qE
+
(c) 3/4 (d) 1/3. T cos θ = mg +
A B +
qE
tan θ = + 
O mg + T
2 µF  T cos 
4 µF
tan θ =
q σLM OP +
+

Fig. 14 N Q
mg 2ε 0 + T sin 
+
qE

or σα tan θ mg
[National Standard Exam. in Physics 1994]
So, (a) is the right choice. Fig. 16
Sol. Charge on 4 µF distributed over 6 µF.
So, (b) is the right choice.
Illustration 10. Two metal spheres of capacitances
C1 and C2 carry some charges. They are put in contact
10 COMPREHENSIVE OBJECTIVE PHYSICS

QUESTION BANK

MCQs
SET I

based on
Memory Work, Hard Facts, Important Terms,
Important Formulae etc.

Average time allowed per question is 20 seconds.

COULOMB’S LAW & ELECTRIC FIELD 7. An electric dipole of moment p is placed in a uniform
→ → →
electric field E , with p parallel to E . It is then rotated
1. The dielectric constant K of an insulator can be by an angle θ. The work done is
(a) – 1 (b) 0 (a) pE sin θ (b) pE cos θ
(c) 0.2 (d) 5. (c) pE(1 – cos θ) (d) pE(1 – sin θ).
2. The dielectric constant K of an insulator cannot be [Kerala PMT 1999]
(a) 2 (b) 10
8. A cube of side ‘b’ has charge ‘q’ at each of its vertices.
(c) 5 (d) ∞. The electric field at the centre of the cube will be
3. If F is the force between two point charges submerged
q q
in a medium of dielectric constant K , then on with- (a) (b)
drawing the medium, the force between the charges b2 2b2
becomes 32 q
(c) (d) Zero.
(a) F √K (b) FK b2
F F [Karnataka CET 2000]
(c) (d) . [Pb. PMT 2002]
K K
9. An electric field can deflect
4. With a rise in temperature, the dielectric constant K
(a) X-rays (b) neutrons
of a liquid
(c) α-particles (d) γ-rays.
(a) increases (b) decreases
(c) constant (d) none of these. 10. Who of the following was the first to measure force
5. A solid non-conducting sphere of radius R has elec- between charged bodies ?
tric charge uniformly distributed throughout its vol- (a) Franklin (b) Miletus
ume. The electric field at distance r (r < R) from the (c) Newton (d) Coulomb.
centre varies as 11. The SI unit of electric field strength is ......
1 (a) N C –1 (b) coulomb
(a) r (b)
r
(c) abcoulomb (d) newton.
1
(c) (d) 1 . [TNPCEE 2001] 12. Three small spheres, each carrying a positive charge
r2 r3
6. Two identical metal balls with charges + 2Q and – Q Q, are placed on the circumference of a circle of ra-
are separated by some distance, and exert a force F dius ‘r’ to form an equilateral triangle. The electric
on each other. They are joined by a conducting wire, field intensity at the centre of the circle will be
which is then removed. The force between them will 3Q 3Q
(a) (b)
now be r r2
(a) F (b) F/2 Q
(c) (d) zero. [EAMCET 1983]
(c) F/4 (d) F/8. 2 r2
[Haryana PMT 2002]
ELECTROSTATICS 11
13. An electric dipole has the magnitude of its charge as q2 q q2 q
(c) + 3 sin θ (d) + 3 cos θ.
q and its dipole moment is p. It is placed in a uni- b2 a 2 b2 a 2
form electric field E. If its dipole moment is along [AIEEE 2003]
the direction of the field, the force on it and its po-
18. Two identical conducting balls having positive
tential energy are respectively
charges q1 and q2 are separated by a distance r. If
(a) q . E and p . E they are made to touch each other and then sepa-
(b) zero and minimum rated to the same distance, the force between them
(c) q . E and maximum will be
(d) 2q . E and minimum. [All India PM/PD 2004] (a) less than before (b) same as before
14. In the basic Cs Cl crystal Cs
+
Cs
+ (c) more than before (d) zero.
structure, Cs + and Cl – ions + [National Standard Exam. in Physics 2005]
+ Cs
are arranged in a bcc configu- Cs – a
19. An electric dipole placed in a non-uniform electric
Cl
ration as shown in Fig. 17. Cs
+
Cs
+
field experiences
The net electrostatic force ex- Cs
+ a (a) Both, a torque and a net force.
+
erted by the eight Cs+ ions on a Cs
(b) Only a force but no torque.
the Cl– ion is : Fig. 17
(c) Only a torque but no net force.
1 4 e2 1 16 e2 (d) No torque and no net force. [AIIMS 2003]
(a) (b)
4 π ∈o 3a2 4 π ∈o 3 a2
20. A charged oil drop is suspended in a uniform field of
1 32 e2 3 × 10 4 V/m so that it neither falls nor rises. The
(c) (d) Zero. [AIIMS 2004]
4 π ∈o 3 a2 charge on the drop will be (Take the mass of the charge
15. The magnitude of electric intensity E is such that an = 9.9 × 10–15 kg and g = 10 m/s 2)
electron placed in it would experience an electrical (a) 1.6 × 10–18 C (b) 3.2 × 10–18 C
force equal to its weight. E is given by (c) 3.3 × 10–18 C (d) 4.8 × 10–18 C.
e [AIEEE 2004]
(a) mge (b)
mg 21. An electric dipole is placed in an uniform electric field
→ →
mg 2 E such that the dipole moment p makes an angle
e g
(c) (d) . [CPMT 1995] →
e m2 β (≠ 0) with E . The force and torque acting on the
16. Three charges are placed A +Q dipole are respectively given by
at the vertices of an equi- → → →→ → →
(a) 0, p × E (b) p. E, p × E
lateral triangle of side ‘a’
→ → → → → →
as shown in Fig. 18. The (c) p × E, 0 (d) p × E, E × p .
force experienced by the 22. An electric dipole placed in a non-uniform electric
charge placed at the ver- –Q +Q field will experience
a
tex A in a direction normal B C
(a) only a force (b) only a torque
to BC is : Fig. 18
(c) both force and torque (d) neither force nor torque.
(a) Q2/(4π∈0a2) (b) – Q2(4π∈0a2)
23. An electric line of force is leaving a charged spheri-
(c) zero (d) Q2/(2π∈0a2) cal conductor. What is the angle between the surface
[AIIMS 2003] and the electric line of force ?
17. Three charges – q1, + q2 y (a) 0° (b) 30°
– q3
and – q 3 are placed as (c) 45° (d) 90°. [Bihar PMT 1999]
shown in Fig. 19. The x- 24. A large isolated metal sphere of radius r carries a
component of the force on a
fixed charge. A small charge is placed at a distance d
– q1 is proportional to q
from its surface. It experiences a force which is
q q b (a) independent of r and d
(a) 2 − 3 sin θ
b2 a 2 – q1 + q2 x (b) proportional to r2 + d2
q q Fig. 19 (c) proportional to r2
(b) 2 − 3 cos θ
b2 a 2 (d) inversely proportional to (r + d)2.
12 COMPREHENSIVE OBJECTIVE PHYSICS

33. Which of the following is a sure test of electrification ?
25. A dipole of electric dipole moment p is placed in a
→ (a) attraction (b) repulsion
uniform electric field of strength E . If θ is the angle (c) induction (d) friction.
→ →
between positive directions of p and E, then the 34. The angle between electric dipole moment and the
potential energy of the electric dipole is largest when electric field strength due to it on the axial line is :
θ is (a) 0° (b) 90°
(a) zero (b) π /2 (c) 180° (d) None of these.
(c) π (d) π /4. [Pb. PMT 1995]
26. Two plates are 1 cm apart and the potential differ- 35. A positive point charge Q is brought near an isolated
ence between them is 10 volt. The electric field be- metal cube. Which of the following is correct ?
tween the plates is (a) The cube becomes negatively charged
(a) 10 N C–1 (b) 500 N C–1 (b) The cube becomes positively charged
(c) 1000 N C–1 (d) 250 N C–1. (c) The interior becomes positively charged and the surface
becomes negatively charged
27. A charged particle moves with a speed v in a circular
path of radius R around a long uniformly charged (d) The interior remains charge free and the surface gets
non-uniform charge distribution.
conductor
1 [Haryana PMT 1997]
(a) v ∝ R (b) v ∝
R 36. A simple pendulum of time period T is suspended
1 above a large horizontal metal sheet with uniformly
(c) v ∝ (d) v is independent of R.
R distributed positive charge. If the bob is given some
28. A spherical charged conductor has surface density σ negative charge, its time period of oscillation will be
of charge. The electric field on its surface is E. If the (a) > T (b) < T
radius of the sphere is doubled keeping the surface (c) T (d) proportional to its amplitude.
density of charge unchanged, then the electric field [AIIMS 1994]
on the surface of the new sphere will be 37. Charge Q is divided into two parts which are then
(a) E/4 (b) E/2
kept some distance apart. The force between them
will be maximum if the two parts are
(c) E (d) 4 E. [Pb. PMT 1998] (a) Q/2 each (b) Q/4 and 3Q/4
29. A cube of side 5 cm has a charge of 6 C. The surface (c) Q/3 and 2Q/3
density of charge is (d) e and (Q – e), where e = electronic charge.
(a) 100 C m–2 (b) 200 C m–2 [AFMC 1999]
(c) 300 C m–2 (d) 400 C m–2. 38. An electric charge residing on a body produces
30. A positively charged ball hangs from a silk thread. (a) an electric field only (b) a magnetic field only
We put a positive test charge q0 at a point and meas- (c) both electric and magnetic field
ure F/q0. Now, can be predicted that the electric field (d) neither electric nor magnetic field.
strength E 39. Lightning conductor
(a) > F/q0 (b) < F/q0 (a) conducts the charge to the Earth
(b) repels the charge on clouds
(c) = F/q0 (d) cannot be estimated.
(c) removes the clouds
[CPMT 1989] (d) none of the above.
31. If the gold leaves of an electroscope diverge still apart 40. A hollow sphere of charge does not produce an elec-
when a test body is brought near it, then the test tric field at any
body has (a) interior point (b) outer point
(a) a small amount of charge (c) point beyond 2 metre (d) point beyond 10 metre.
(b) no charge [MNR 1985]
(c) a charge of the same nature as already on the leaves 41. The electric field inside a spherical shell of uniform
(d) a charge of the opposite nature as already on the leaves. surface charge density is
32. The dielectric constant of a metal is (a) zero (b) outer point
(a) zero (b) one (c) directly proportional to the distance
(c) K (d) + ∞. (d) none of the above. [CMC LDH 1993]
ELECTROSTATICS 13
42. A positively charged glass rod is brought near the (a) The particle will not move at all
disc of uncharged gold leaf electroscope. The leaves (b) The particle will move along the straight line
diverge. Which of the following statements is correct ? (c) The particle would move along the circular line of force
(a) No charge is present on the leaves (d) Given data is not sufficient to arrive at any conclusion.
(b) Positive charge is induced on the leaves [AFMC 1998]
(c) Negative charge is induced on the leaves
49. ‘All charge on a conductor must reside only on its
(d) Positive charge is induced on one leave and negative on outer surface’. This statement is true
the other.
(a) in all cases
1
43. Two charges of 1 nC and 4 nC are placed m apart. (b) for only solid and hollow spherical conductors
2
The ratio of the Coulomb forces experienced by the (c) for hollow spherical conductors only
two charges is (d) for conductors which do not have any sharp points.
(a) 1 : 1 (b) 1 : 9 →
(c) 9 : 1 (d) 1 : 81.
50. Charge Q is given a displacement r = ai + bj in an

44. Two charges q1 and q2 separated by a dielectric of electric field E = E i + E j . The work done is
1 2
dielectric constant 4 repel each other with a force of
(a) Q(E1a + E2b) (b) Q (E1a) 2 + (E2b) 2
10 N. Another charge q3 is placed between q1 and q2
1 2 2 2 2
(c) Q(E1 + E2 ) a2 + b2 (d) Q (E1 + E2 ) a +b .
such that the distance of q3 from q1 is times the
4
[BHU 1994]
distance of q3 from q2 . Now, the force of repulsion
between q1 and q2 is 51. An uncharged metal object M is insulated from its
(a) 10 N (b) 10 q1 surroundings. A positively charged metal sphere S
is then brought near to M. Which diagram best illus-
4 q2 10 q1
(c) × 10N (d) . [AMU 1993] trates the resultant distributions of charge on S and
q1 q2 M?
45. When 1019 electrons are removed from a neutral
metal plate, the electric charge on it is (coulomb)
(a) 10+19 (b) + 1.6
(c) – 1.6 (d) 10–19.
[Karnataka CET 1999]
46. Two charged spherical conductors, each of radius R,
are at a distance r from each other. The charge on
the first is + q and on the second is – q. If r > 2R,
then the force of attraction is numerically
1 q2 1 q2
(a) equal to (b) more than
4 πε 0 r 2 4 πε 0 r 2

1 q2
(c) less than
4 πε 0 r 2
r
(d) more or less depending upon .
R

47. An electron moves with velocity v in x-direction. An
electric field acts on it in y-direction. The force on
the electron acts in
(a) + ve direction of Y-axis (b) – ve direction of Y-axis
(c) + ve direction of Z-axis (d) – ve direction of Z-axis.
48. An electric line of force in xy plane is given by x2 + y2
= 1. A unit positive charge, initially at rest, at the
Fig. 20
point x = 1, y = 0 in the xy plane, is released.
14 COMPREHENSIVE OBJECTIVE PHYSICS

52. A positive charge and a negative charge of equal (c) Their accelerations will be equal.
magnitude are placed at a short distance apart. (d) The magnitudes of their accelerations will be equal.
Which diagram best represents the associated electric [WB JEE 1999]
field ? 58. Proton is nearly 1836 times more massive than the
electron. The Coulomb force of repulsion between two
protons separated by a finite distance is F. The elec-
trostatic force between two electrons separated by
+ – + – the same distance is
F
(a) F (b)
1837
1837
(c) 1837 F (d) F. [BHU 2002]
( a) (b) 4
59. A thin metallic spherical shell carries a charge Q on
it. A point charge q is placed at the centre of the shell
and another charge q1 is placed outside it as shown
+ – + – in Fig. 22. All the three charges are positive.

(c) (d)
Fig. 21

53. If an electron has an initial velocity in a direction


different from that of an electric field, then the path
of the electron is Fig. 22
(a) a straight (b) a circle
(c) an ellipse (d) a parabola.
The force on the charge at the centre is
(a) towards left (b) towards right
54. A charge is moved against the Coulomb force of an
electric field. Then (c) upward (d) zero. [TNPCEE 2002]
(a) work is done by the electric field 60. Two metal plates having charges Q, – Q face each
(b) energy is used from some outside source other at some separation and are dipped into an oil
(c) the strength of the field is decreased tank. If the oil is pumped out, the electric field be-
(d) the energy of the system is decreased. tween the plates will
[Manipal 1994] (a) increase (b) decrease

55. A million electron are added to a pith ball. Its charge (c) remains the same (d) become zero.
is [Bharati Vidyapeeth 1997]
(a) – 1.6 × 10 –12 coulomb (b) – 1.6 × 10 –13 coulomb 61. Which of the following graphs represents the rela-
(c) – 1.6 × 10 –18 coulomb (d) – 1.6 × 10 coulomb. tionship between the force F between two point charges
56. A charge is uniformly distributed over a large iron and the distance r between them ?
plate. The electric field at a point close to the centre Y Y
of the plate is 40 Vm–1. If the iron plate is replaced by
a copper plate of the same geometrical dimensions
and carrying the same charge, then the electric field
at the same point will be :
(a) 1 Vm–1 (b) 10 Vm–1
F F
(c) 40 Vm–1 (d) 80 Vm–1. [BHU 1996]
57. A proton and an electron are placed in a uniform
electric field. Which of the following is correct ? O r X O r X
(a) The electric forces acting on them will be equal. (a) (b)
(b) The magnitudes of the forces will be equal.
ELECTROSTATICS 15
Y Y (a) 3 F (b) 2 F

(c) 2F (d) 3 F. [KCEE 1995]


66. Which of the following is correct for the case of an
uniform electric filed ?
F F (a) All points are at the same potential.
(b) No two points can have the same potential.
(c) Pairs of points separated by the same distance must have
O r X O r X the same difference in potential.
(c) (d) (d) None of the above.
Fig. 23
67. A positive point charge, which is free to move, is placed
[WB JEE 2001] inside a hollow conducting sphere with negative
62. The angle between the electric dipole moment and charge, away from its centre. It will
the electric field strength due to it on the equatorial (a) move towards the centre
line is (b) move towards the nearer wall of the conductor
(a) 0° (b) 90° (c) remain stationary
(c) 180° (d) 45°. [AIIMS 1995] (d) oscillate between the centre and the nearer wall.
63. Three small spheres, each carrying a charge q are [DCE 2002]
placed on the circumference of a circle of radius R, 68. Two identical simple pendulums, A and B, are sus-
forming an equilateral triangle. If we place another pended from the same point. The bobs are given posi-
charge Q at the centre of the circle, then the force on tive charges, with A having more charge than B. They
Q will be diverge and reach equilibrium, with A and B making
1 qQ angles θ1 and θ2 with the vertical respectively. Which
(a) zero (b) ×
4 π ε0 R2 of the following is correct ?
(a) θ1 > θ2
1 2qQ 1 3qQ
(c) × 2 (d) × 2 . (b) θ1 < θ2
4π ε0 R 4π ε0 R
(c) θ1 = θ2
[RPMT 1993]
(d) The tension in A is greater than that in B.
64. A large non-conducting sheet S is given a uniform
[RPMT 1995]
positive charge density. Two uncharged small metal
plates A and B are placed near the sheet as shown. 69. What is the magnitude of a point charge which pro-
Which of the following is false ? duces an electric field of 2 N C–1 at a distance of 60
cm ? (1/4πε0 = 9 × 109 Nm2C –2)
(a) 8 × 10 –11 C (b) 2 × 10 –12 C
(c) 3 × 10 –11 C (d) 6 × 10 –10 C.
[MP PET 2000]
70. Three point charges are placed at the corners of an
equilateral triangle. Assuming only electrostatic
forces are acting,
(a) the system can never be in equilibrium
Fig. 24
(b) the system will be in equilibrium if the charges rotate
(a) S attracts A (b) S attracts B about the centre of the triangle
(c) A attracts B (d) B attracts A (c) the system will be in equilibrium if the charges have dif-
(e) None of these. ferent magnitudes and different signs
65. Three charges, each equal to + 2 C, are placed at the (d) the system will be in equilibrium if the charges have the
corners of an equilateral triangle. If the force between same magnitude but different signs.
any two charges be F, then the net force on charge [Karnataka CET 2002]
will be
16 COMPREHENSIVE OBJECTIVE PHYSICS

ELECTRIC POTENTIAL q q
(a) (b)
ε0 6ε 0
ELECTRIC FLUX & GAUSS’S LAW
71. An electron travelling in a region of electrostatic q q
(c) (d) .
potential V1 passes into a region of higher potential ε0 l2 4π ε0 l2
V2 . Then the change in its kinetic energy is propor- [TNP CEE 2002]
tional to
78. There is 40 units of positive charge at the centre of a
(a) (V2 – V1)1/2 (b) (V2 – V1)
circle of radius 10 m. The work done in moving 4
(c) (V2 – V1)2 (d) (V2 – V1)/V1. unit of charge around the circle once is
[National Standard Exam. in Physics 2005] (a) Zero (b) 40 units

72. If electric intensity E is along the X-axis, then the (c) 400 units (d) 4 unit.
equipotential surfaces are parallel to 79. Fig. 26 shows a distribution S
(a) XOY plane (b) XOZ plane of charges. The flux of elec-
tric field due to these +q –q
(c) YOZ plane (d) None of these.
charges through the surface
[AIIMS 2002] S is
+q
73. As per Fig. 25, a point charge y (a) 3q/∈o (b) 2q/∈o
+ q is placed at the origin O. (c) q/∈o (d) zero. Fig. 26
A
Work done in taking another [AIIMS 2003]
point charge – Q from the point 80. Equipotential surfaces associated with an electric field
A [Co-ordinates (0, a)] to an- which is increasing in magnitude along the x-direc-
other point B [co-ordinates (a, x
tion are:
O
0) along the straight path AB B (a) Planes parallel to yz-plane.
is : Fig. 25 (b) Planes parallel to xy-plane.
F − qQ 1 I F qQ 1 I (c) Planes parallel to xz-plane.
(a) GH 4π ∈0 a2 JK 2a (b) GH 4π ∈0 a2 JK 2a
(d) Coaxial cylinders of increasing radii around the x-axis.

F qQ 1 I
(c) G
a
[AIIMS 2004]
H 4π ∈0 a2 JK 2
(d) Zero. 81. A charge q is placed at the centre of
[All India PM/PD 2005] the open end of a cylindrical vessel
(Fig. 27). The flux of the electric field
74. The ratio of electric potential due to an electric di-
through the surface of the vessel is
pole in the end-on position to that in the broad side-
(a) zero (b) q/ε0 Fig. 27
on position for the same distance from it, is
(a) ∞ (b) 2 (c) q/2ε0 (d) 2q/ε0 . [RPMT 1997]
(c) 1 (d) zero. [AIIMS 2001] 82. An electron which is initially at rest is accelerated
through a potential difference of one volt. The en-
75. A closed surface encloses an electric dipole. The total
ergy acquired by the electron is
normal electric induction over this closed surface is
(a) 10 –19 J (b) 1.6 × 10 –19 erg
q q
(a) − (b) (c) 1.6 × 10 –19 J (d) zero.
ε0 ε0
83. A charge of 10 coulomb is moved along an equipoten-
q−q q+q
(c) (d) . [AFMC 1993] tial surface having a potential of 2 volt. The work
ε0 ε0
done is
76. If the electric flux entering and leaving an enclosed
(a) 10 Joule (b) zero
surface respectively is φ1 and φ2 , the electric charge
inside the surface will be (c) 2 joule (d) 20 joule.
(a) (φ1 + φ2)ε0 (b) (φ2 – φ1)ε0 84. Which of the following is equivalent to SI unit of
(c) (φ1 + φ2)/ε0 (d) (φ2 – φ1)/ε0 . [AIEEE 2003] potential ?
77. An electric charge q is placed at the centre of a cube (a) N m C–1 (b) N2 m C–1
of side l. The electric flux through one of its faces will (c) N3 m C –2 (d) N m.
be
ELECTROSTATICS 17
85. A sphere is constructed around a positive point charge 93. S1 and S2 are two equipotential surfaces on which
q. The work done in moving a unit positive charge the potentials are not equal. Which of the following
on this sphere is numerically equal to is incorrect ?
(a) capacity of sphere (b) potential of sphere (a) S1 and S2 cannot intersect.
(c) infinity (d) zero. (b) S1 and S2 cannot both be plane surfaces.
86. Two copper spheres of the same radii, one hollow (c) In the region between S1 and S2, the field is maximum
and the other solid, are charged to the same poten- where they are closest to each other.
tial. Which of the two will hold more charge ? (d) A line of force from S1 to S2 must be perpendicular to both.
(a) A (b) B [BHU 2002]
(c) Solid sphere cannot hold any charge 94. In bringing an electron towards the second electron,
(d) Both the spheres will hold the same charge. the electrostatic potential energy of the system
87. The electric potential at the surface of an atomic nu- (a) increases (b) decreases
cleus (Z = 50) of radius 9.0 × 10 –15 m is (c) remains the same (d) becomes zero.
(a) 9 V (b) 9 × 105 V 95. A tin nucleus has charge + 50e. If the proton is at
(c) 8 × 106 V (d) 80 V. a distance 10 –12 m from the nucleus, then the
88. An external agency carries ‘–5 C’ of charge from in- potential V at this position is [charge on the proton
finity to a point in an electrostatic field and performs = 1.6 × 10–19C]
100 joule of work. The potential at the given point is (a) 14.4 × 104 volt (b) 7.2 × 104 volt
(a) 10 V (b) – 10 V (c) 7.2 × 108 volt (d) 14.4 × 108 volt.
(c) 20 V (d) – 20 V. [AFMC 1991]
89. The tangent at any point of an equipotential surface 96. Relation between E and V is
makes an angle θ with the electric intensity vector at →
dV
that point such that (a) E = − (b) E = − K × V
dr

z
(a) θ = 0° (b) θ = 90°
→ → → →
(c) θ = 120° (d) θ = 180°. (c) V 2 = − E. dl (d) V = − E × dl.
90. The electric potential at a point on the axis of an
electric dipole depends on the distance x of the point 97. Earth’s surface is considered to be at
from the mid-point of the dipole as (a) zero potential (b) negative potential
1 1 (c) infinite potential (d) positive potential.
(a) ∝ (b) ∝
x2 x3 98. A charge Q is placed at the mouth of a conical flask.
1 1 The flux of the electric field through the flask is
(c) ∝ 4 (d) ∝ 3 / 2 .
x x Q
(a) zero (b)
ε0
91. Two small spheres, each carrying a charge q, are
Q Q
placed R metre apart. If one of the spheres is taken (c) (d) < .
2ε0 2ε 0
around the other one in a circular path, then the
work done will be equal to [CMC LDH 2002]
(a) Force between them × R 99. Two conducting spheres of radii r1 and r2 are given
the same charge. The ratio of their potentials will be
(b) Force between them/2nR
r r
(c) Force between them × 2nR (a) 2 (b) 1
r1 r2
(d) Zero.
r1 r2
(c) (d) .
92. A cylinder of radius R and length L is placed in a 1 + r2 1 + r1
uniform electric field E parallel to the cylinder axis. 100. In a region with uniform electric field, the number of
The total flux for the curved surface of the cylinder lines of force per unit area is E. If a spherical metallic
is given by conductor is placed in this region, the number of lines
(a) 2πR2E (b) πR2LE2 of force per unit area inside the conductor will be
πR 2 + πR 2 (a) E (b) more than E
(c) (d) zero.
EL (c) less than E (d) zero.
18 COMPREHENSIVE OBJECTIVE PHYSICS

101. An electric dipole of dipole moment p placed in uni- (a) charge potential

form electric field E will have minimum potential (b) uncharged zero
→ →
energy if the angle between p and E is (c) uncharged negative

(a) 0° (b) π (d) negative zero


(e) negative negative.
π 3π
(c) (d) . [CMC LDH 2001] 106. When the separation between two charges is increa-
2 2
sed, the electric potential energy of the charges
102. Two conducting spheres of radii r1 and r2 are at the
same potential. What is the ratio of their charges ? (a) increases (b) decreases
(a) r1 : r2 (b) r2 : r1 (c) remains the same (d) may increase or decrease.
(c) r12 : r22 (d) r22 : r12. [Himachal PMT 2000]
[Karnataka CET 1999] 107. A cloud is at a potential of 10 6 volt relative to the
103. A negatively charged rod is ground. A charge of 6 coulomb is transferred in a
–– ––
held near a conductor X lightning stroke between the cloud and the Earth.
that is Earthed (Fig. 28). X The energy dissipated is
X
is (a) 106 J (b) 3 × 106 J
(a) uncharged and at a nega- (c) 6 × 106 J (d) 12 × 106 J.
tive potential Fig. 28
108. As shown in the Fig. 30, charges + q and – q are
(b) uncharged and at a positive potential
placed at the vertices B and C of an isosceles triangle.
(c) positively charged and at Earth potential The potential at the vertex A is
(d) negatively charged and at Earth potential
(e) negatively charged and at a negative potential.
104. The potential difference between a pair of similar,
parallel, conducting plates is known. What additional
information is needed in order to find the electric
field strength between the plates ?
(a) separation of the plates
Fig. 30
(b) separation and area of the plates
(c) permittivity of the medium and separation of the plates 1 2q
(a) . (b) zero
4π ε0 a + b2
2
(d) permittivity of the medium, separation and area of the
plates.
1 q 1 ( − q)
105. The diagram shows an earthed conducting sphere (c) . (d) . .
4 π ε0 2
a +b 2 4π ε0 a2 + b2
held close to a positively-charged dome.
[MP PET 2000]
+ + 109. A charge of Q coulomb is placed on a solid piece of
Positively- + + +
charged + + Conducting metal of irregular shape. The charge will distribute
dome + + sphere itself
+ +
(a) uniformly in the metal object
+ +
+ + (b) uniformly on the surface of the object
+ + + + (c) such that the potential energy of the system is minimised
(d) such that the total heat loss is minimised. [BHU 2001]
Insulating 110. The electric potential at any point inside a charged
support hollow spherical conductor is
(a) zero
(b) greater than potential on its surface
(c) the same as on its surface
Fig. 29
(d) less than the potential on its surface.
What are the charge and potential of the conducting
[Himachal PMT 1995]
sphere ?
ELECTROSTATICS 19

111. A hollow metal sphere of radius 5 cm is charged such 118. The electric flux through a hemisphere of radius R,
that the potential on its surface is 10 V. The poten- placed in an uniform electric field of intensity E par-
tial at a distance of 2 cm from the centre of the sphere allel to the axis of its circular plane is
is (a) 2π R E (b) 2π R2 E
(a) zero (b) 10 V 4
(c) π R2 E (d) π R3 E.
(c) 4 V (d) 10/3 V. [MP PMT 1996] 3
[Himachal PMT 2002]
112. Two conducting spheres of radii R 1 and R 2 are
charged with charges Q1 and Q 2 respectively. On 119. Two identical rings, each of radius r, are co-axially
bringing them in contact, there is placed. The distance between their centres is r. Same
(a) no change in the energy of the system charge Q is placed on each ring. The work done in
(b) an increase in the energy of the system if Q1R2 ≠ Q2R1
moving a test charge from the centre of one ring to
that of the other is
(c) always a decrease in energy of the system
(d) a decrease in energy of the system if Q1R2 ≠ Q2R1 1 2 q0 Q
(a) . (b) zero
[MP PMT 2001] 4 π ε0 r

113. Two charges of + 1 µC each are kept at a distance of ( 2 − 1) q0 Q 2 q0 Q


1 1
1 m from each other. Their potential energy will be (c) . (d) . .
4 π ε0 2r 4 π ε 0 ( 2 − 1) r
(a) 1 J (b) 1 eV
[DPMT 1999]
(c) 9 × 10 –3 J (d) 9 × 10 –9 J. [BHU 2002]
120. N small drops of the same size are charged to V volt
114. Electric charges are distributed in a small volume.
each. If they coalesce to form a single large drop,
The flux of the electric field through a spherical sur-
face of radius 0.10 m surrounding the total charge is then its potential will be
1.5 V m. The flux over a concentric sphere of radius (a) V/N (b) NV
0.15 m will be (c) VN1/3 (d) VN2/3.
(a) 1.5 V m (b) 5.0 V m [National Standard Exam. in Physics 2003]
(c) 10.0 V m (d) 20.0 V m. [BHU 1995]
121. A proton is about 1840 times heavier than an elec-
115. A particle A of mass m and charge Q moves directly tron. When it is accelerated by a potential difference
towards a fixed particle B, which has charge Q. The of 1 kV, its kinetic energy will be :
speed of A is v when it is far away from B. The mini-
(a) 1840 keV (b) 1/1840 keV
mum separation between the particles is not propor-
tional to (c) 1 keV (d) 920 keV. [AIIMS 2003]
1 122. Electric charges + 10 µC, + 5 µC, – 3 µC and + 8 µC
(a) Q2 (b) 2
v are placed at the corners of a square of side 2 m.
1 1 The potential at the centre of the square is
(c) (d) . [CMC LDH 2000]
v m (a) 18 × 105 V (b) 1.8 × 106 V
116. The electric field strength at a distance r from the (c) 1.8 V (d) 1.8 × 105 V.
centre of a charged sphere of radius R is E. If r > R,
[Karnataka CET 1999]
how much work will be done in bringing a test charge
q0 from infinity to that point ? 123. A hollow metal sphere of radius 5 cm is charged such
that the potential on its surface is 10 volt. The electric
1
(a) q0 RE (b) q RE field at the centre of the sphere will be
2 0
(a) 50 volt/metre (b) 10 volt/metre
1
(c) q0 rE (d) q rE . [DPMT 2002]
2 0 (c) 5 volt/metre (d) Zero. [MP PET 2000]
117. The total electric flux leaving spherical surface of
CAPACITORS
radius 1 m and surrounding an electric dipole is
(a) zero (b) q/ε0 124. The air between the plates of a parallel plate capacitor
is replaced by a medium of dielectric constant K. The
(c) 8 π r2 q/ε0 (d) 2 q/ε0.
capacitance becomes
[All India PM/PD 1993]
20 COMPREHENSIVE OBJECTIVE PHYSICS

1 132. The work done in placing a charge of 8 × 10 –18 C on


(a) K times (b) times
a condenser of capacity 100 mF is
K
(a) 32 × 10–32 J (b) 16 × 10–32 J
1
(c) K times (d) times. (c) 3.1 × 10–26 J (d) 4 × 10–10 J.
K
[AIEEE 2003]
125. In the previous question, the electric intensity between
133. Two capacitors of capacities C1 and C2 are connected
the plates of capacitor becomes
in parallel. If a charge Q is given to the combination,
1 the charge gets shared. The ratio of the charge on
(a) K times (b) times
K the capacitor C1 to the charge on the capacitor C2 is
1 C2
(c) K (d) times. (a) C1C2 (b)
K C1

126. In question number 124 the potential difference C1


(c) (d) C1 + C2.
between the plates of capacitor becomes C2

1 134. A sheet of aluminium foil of negligible thickness is


(a) K times (b) times introduced between the plates of a capacitor. The
K
capacitance of the capacitor.
1 (a) increases (b) decreases
(c) K times (d) times.
K (c) remains unchanged (d) becomes infinite.
127. If the dielectric constant and dielectric strength be [AIEEE 2003]
denoted by K and k respectively, then a material suit- 135. Two capacitors C1 and C2 are charged to 120 V and
able for use as a dielectric in a capacitor must have 200 V respectively. When they are connected in
(a) low K and low k (b) low K and high k parallel, it is found that potential on each one of them
(c) high K and low k (d) high K and high k. is zero. Therefore,
(a) 5C1 = 3C2 (b) 3C1 = 5C2
128. If we treat the Earth as a conducting sphere of radius
6400 km, its capacitance would be nearly : (c) 3C1 + 5C2 = 0 (d) 9C1 = 4C2.

(a) 1 µF (b) 0.7 mF [National Standard Exam. in Physics 2005]


(c) 1 F (d) 103 F. 136. A network of six identical capacitors, each of value
C, is made as shown in the Fig. 31. Equivalent
129. Two capacitors 10 µ F and 25 µ F are charged to a
capacitance between points A and B is
potential of 100 V each. They are connected in paral-
lel. The potential across them are :
(a) 50 V each (b) 200 V each
(c) 100 V each (d) 80 V and 120 V
(e) 20 V and 100 V. [Kerala PMT 2003]
130. Between the plates of a parallel plate capacitor of ca-
pacity C, two parallel plates, of the same material
and area same as the plate of the original capacitor,
are placed. If the thickness of each plate is equal to Fig. 31
1/5th of the distance between the plates of the original (a) C/4 (b) 3C/4
capacitor, then the capacity of the new capacitor is (c) 4C/3 (d) 3C.
5 3 [National Standard Exam. in Physics 2001]
(a) C (b) C
3 5
137. A parallel plate condenser with oil between the plates
3C 10C (dielectric constant of oil K = 2) has a capacitance C.
(c) (d) . [EAMCET 2003]
10 3 If the oil is removed, then capacitance of the capacitor
131. The value of one farad in esu will be becomes
(a) 3 × 1010 (b) 9 × 1011 (a) 2C (b) 2C
C C
1 1 (c) (d) . [BHU 2000]
(c) × 10 −11 (d) × 1010 . 2 2
9 3
ELECTROSTATICS 21

138. Three capacitors of capacities C1 , C2 , C3 are con- 144. The internal and external radii of a cylindrical con-
nected in series. Their total capacity will be denser are both doubled. Its capacity will be
(a) C1 + C2 + C3 (b) 1/(C1 + C2 + C3) (a) reduced to zero (b) reduced to half
(c) doubled (d) unchanged.
(c) (C−11 + C2−1 + C3−1) −1 (d) None of these.
[AIIMS 1996]
[All India PM/PD 1997]

145. A capacitor which has a capacitance of 1 farad will
139. The magnitude of electric field E in the annular region (a) be fully charged in 1 second by a current of 1 ampere
of a charged cylindrical capacitor (b) store 1 coulomb of charge at a potential difference of 1 volt
(a) is same throughout. (c) gain 1 joule of energy when 1 coulomb of charge is stored
(b) is higher near the outer cylinder than near the inner on it
cylinder. (d) discharge in 1 second when connected across a resistor of
(c) varies as 1/r where r is the distance from the axis. resistance 1 ohm. [Haryana PMT 1998]
(d) varies as 1/r 3 where r is the distance from the axis. 146. The energy stored in a capacitor of capacitance C,
[IIT 1996] carrying charge Q with potential difference V between
140. In the circuit C its plates, may be obtained by calculating the area
shown, the under an appropriate graph. Which graph shows the
equivalent ca- correct relationship between a pair of the quantities
pacitance be- C, Q and V, and in addition shows a shaded area
tween the points P which corresponds to the energy stored in the ca-
Q
P and Q is C C C pacitor ?
(a) C/5 V C
(b) C/3
C
(c) C/2 Fig. 32
(d) C. [BHU 1997]
141. In a parallel-plate capacitor, the region between the
plates is completely filled by a dielectric slab. The
capacitor is charged from a battery and then discon-
nected from it. The dielectric slab is now taken out.
Q Q
Which of the following is correct ? (a ) (b)
(a) The potential difference across the capacitor is reduced. C C
(b) The potential difference across the capacitor is increased.
(c) The energy stored in the capacitor is reduced.
(d) No work is done by an external agent in taking the slab
out. [EAMCET 2002]
142. A parallel-plate capacitor is charged from a battery
and then disconnected from it. The separation between
the plates is now doubled. Which of the following is 1/V V
false ? (c) (d)
(a) The potential difference between the plates will be dou- Fig. 33
bled.
147. A dielectric slab of dielectric constant K = 5 is cov-
(b) The electric field between the plates of the capacitor will
not change. ered from all sides with a metallic foil. This system
(c) The energy of the capacitor doubles. is introduced into the space of a parallel plate capaci-
(d) Some work will have to be done by an external agency tor of capacitance of 10 pF. The slab fills almost the
in pulling apart the plates. entire space between the plates, but does not touch
(e) None of the above. the plates. The capacitance will become nearly
143. The capacitance of a spherical conductor of radius r (a) infinity (b) zero
is proportional to (c) 2 pF (d) 50 pF.
(a) r2 (b) r
[National Standard Exam. in Physics 1991]
(c) 1/r (d) 1/r3.
22 COMPREHENSIVE OBJECTIVE PHYSICS

148. What is the area of the plates of a 3 F parallel plate 156. A parallel plate capacitor has circular plates of 0.08
capacitor, if the separation between the plates is m radius and 1.0 × 10 –3 m separation. If a potential
5 mm ? difference of 100 volt is applied, the charge will be
(a) 1.7 × 109 m2 (b) 4.5 × 109 m2 (a) 4.8 × 10–10 C (b) 1.8 × 10–8 C
(c) 9.3 × 109 m2 (d) 12.9 × 109 m2. (c) 11.8 × 10–20 C (d) 0 C. [Dhanbad 1994]
149. In a charged capacitor, the energy is stored in 157. When a slab of dielectric material is introduced be-
(a) the electric field between the plates tween the parallel plates of a capacitor which remains
(b) the edges of the capacitor plates connected to a battery, then charge on plates relative
(c) positive charges to earlier charge
(d) both in positive and negative charges. (a) is less
[Karnataka CET 2000] (b) is same
150. The capacity of a metallic sphere of radius 90 cm is (c) is more
(a) 10.9 F (b) 10.9 µF (d) may be less or more depending on the nature of the mate-
(c) 100 pF (d) 190 pF. rial introduced.
151. Three identical capacitors are combined differently. 158. A capacitor with air as the dielectric is charged to a
For the same voltage to each combination, the one potential of 100 volt. If the space between the plates
that stores the greatest energy is is now filled with a dielectric of dielectric constant
(a) Two in parallel and the third in series with it 10, the potential difference between the plates will be
(b) Three in series (c) Three in parallel (a) 1000 V (b) 100 V
(d) Two in series and third in parallel with it. (c) 10 V (d) 0 V. [AFMC 2002]
[MANIPAL 1999] 159. Two capacitors, each of capacity 2 µ F, are connected
152. Two capacitors of equal capacity are first connected in parallel. This system is connected in series with a
in parallel and then in series. The ratio of the total third capacitor of 12 µ F capacity. The equivalent ca-
capacities in the two cases will be pacity of the system will be
(a) 2 : 1 (b) 1 : 2 (a) 16 µF (b) 13 µF
(c) 4 : 1 (d) 1 : 4. (c) 4 µF [MP PET 1990]
(d) 3 µF.
153. If E be the electric intensity of an electrostatic field, 160. The capacity of a parallel plate condenser is 5 µ F.
then the electrostatic energy density is proportional When a glass plate is placed between the plates of the
to 1
(a) E (b) E2 conductor, its potential becomes th of the original
8
(c) 1/E2 (d) E3. value. The value of dielectric constant will be
[Pb. PMT 2000] (a) 1.6 (b) 5
154. The capacitance of a metallic sphere will be 1 µF, if (c) 8 (d) 40.
its radius is nearly 161. The expression for the capacity of the capacitor formed
(a) 9 km (b) 10 m by compound dielectric placed between the plates of
(c) 1.11 m (d) 1.11 cm. a parallel plate capacitor as shown in Fig. 35 will be
155. Three equal capacitors, each with capacitance C, are (area of plate = A)
connected as shown in Fig. 34. Then the equivalent
cpacitance between A and B is

Fig. 34
(a) C (b) 3C
C 3C
(c) (d) . [CMC LDH 1999] Fig. 35
3 2
ELECTROSTATICS 23
ε0A ε0 A 10 mF 10 mF
(a)
FG d1 + d2 + d3 IJ (b)
FG d1 + d2 + d3 IJ
H K 1 K2 K3 K H K 1 + K 2 + K3 K 5 mF

F AK 1 + AK 2 + AK 3 I .
C
ε A(K 1K 2K 3 )
(d) ε 0 G
(c) 0
H d1 d2 d3 JK
A B
d1d2 d3
[MP PET 1996] 10 mF
162. In Fig. 36, capacitor C1 is first charged to a potential
Fig. 37
V so that it stores an energy E0 . It is then discon-
nected from battery and connected to C2 by closing (a) 1500 V (b) 1000 V
the switch S. The two capacitors now store energies (c) 500 V (d) 400 V.
E1 and E2 respectively. Which of the following rela- 166. Minimum number of capacitors of 2 µ F capacitance
tions is correct ? each required to obtain a capacitor of 5 µF will be
(a) Three (b) Four
(c) Five (d) Six. [BHU 1999]
167. The capacity of a parallel plate condenser is 12 pF. If
the area of both the plates is doubled and the dis-
tance between them is reduced to half, the capacity
of the condenser will be
Fig. 36
(a) 124 pF (b) 48 pF
(a) E0 = E1 + E2 (c) 196 pF (d) 136 pF.
(b) E0 > E1 + E2
[Manipal 2000]
(c) E0 < E1 + E2
168. Five capacitors of 10 µ F capacity each are connected
(d) Law of energy conservation fails in this system.
to a dc potential of 100 volt as shown in Fig. 38. The
[WB JEE 2002]
equivalent capacitance between the points A and B
163. Two capacitors of capacities C1 and C2 are connected will be equal to
in series and potential difference V is applied across (a) 40 µF (b) 20 µF
the combination. Then the potential difference across (c) 30 µF (d) 10 µF.
C1 will be
C2 C1 + C2
(a) V (b) V
C1 C1

C2 C1
(c) V (d) V .
C1 + C2 C1 + C2
164. Two condensers of capacities C and 2C are connected
in parallel and then connected in series with a third
condenser of capacity 3C. The combination is charged
with V volt. The charge on condenser of capacity C is
(in coulomb)
1 Fig. 38
(a) CV (b) CV
2 169. The equivalent capacity between the points A and B
3 in the following Fig. 39 will be
(c) 2 CV (d) CV.
2
[Haryana PMT 1999]
3 µF 3 µF 13 µF
165. In the given circuit, if point C is connected to the
Earth and a potential of + 2000 V is given to the A B
point A, then the potential at B is

Fig. 39
24 COMPREHENSIVE OBJECTIVE PHYSICS

(a) 19 µF (b) 11 µF
(c) 14.5 µF (d) 16 µF. 174. Fig. 42 represents the electric
field between two metal plates.
170. Which one of the following methods will reduce the Which of the graphs given repre-
capacitance of a parallel plate capacitor ? sents the variation of force on a
(a) Connecting another capacitor in series with this positive charge as it is moved
(b) Reducing the potential difference between the plates from point a to point b ?
(c) Introducing a dielectric slab between the plates
(d) Introducing a metal plate of suitable thickness.
Fig. 42
[AMU 2002]
171. A thin metal plate is inserted between
the plates of a parallel plate capaci-
tor of capacitance C in such a way
that its edges touch the two plates.
The capacitance now becomes
C 2C
(a) (b)
2 3
C
(c) (d) ∞.
6 Fig. 40
[AIIMS 1997]
172. A slab of copper of thickness y is inserted in between
the plates of parallel plate capacitor as shown in
Fig. 41. The separation between the plates is d.
If y = d/2, then the ratio of capacities of the capaci-
tor after and before inserting the slab is

Fig. 43
175. In a parallel-plate capacitor, the region between the
plates is filled by a dielectric slab. The capacitor is
connected to a battery and the slab is taken out.
Which of the following is correct ?
(a) Some charge is drawn from the battery
(b) Some charge is returned to the battery
Fig. 41
(c) The potential difference across the capacitor is reduced
(a) 2 :1 (b) 2 : 1 (d) No work is done by an external agent in taking the slab
out. [BHU 1998]
(c) 1 : 1 (d) 1 : 2 . [EAMCET 1997] 176. A parallel plate capacitor is filled with a uniform di-
173. A parallel-plate capacitor is connected to a battery. A electric. Maximum charge that can be given to it does
not depend upon
metal sheet of negligible thickness is placed between
(a) dielectric constant of the dielectric
the plates. The sheet remains parallel to the plates of
(b) dielectric strength of the dielectric
the capacitor. Which of the following is correct ?
(c) separation between the plates
(a) The battery will supply more charge.
(d) area of the plates.
(b) The capacitance will increase. 177. Three capacitors of capacitance 6 µ F each are avail-
(c) The potential difference between the plates will increase. able. The minimum and maximum capacitance,
(d) Equal and opposite charges will appear on the two faces of which may be obtained are
the metal plate. [EAMCET 2002] (a) 6 µF, 18 µF (b) 3 µF, 12 µF
(c) 2 µF, 12 µF (d) 2 µF, 18 µF.
[Bharati Vidyapeeth 1995]
ELECTROSTATICS 25

178. Two spherical conductors A 179. While a capacitor remains connected to a battery, a
and B of radii a and b(b > a) dielectric slab is slipped between the plates :
are placed concentrically in (a) The potential difference between the plates is changed
air. The two are connected by (b) Charges flow from the battery to the capacitor
a copper wire as shown in Fig. A (c) The electric field between the plates increases
44. The equivalent capacitance
(d) The energy stored in the capacitor decreases.
of the system is B [Karnataka CET 2001]
4 πε0 ab
(a) (b) 4πε0 (a + b) Fig. 44
b− a
(c) 4πε0b (d) 4πε 0a. [MNR 1995]

Answers (Set I)
1. (d) 2. (d) 3. (b) 4. (b) 5. (a) 6. (d) 7. (c) 8. (d)
9. (c) 10. (d) 11. (a) 12. (d) 13. (b) 14. (d) 15. (c) 16. (c)
17. (c) 18. (c) 19. (a) 20. (c) 21. (a) 22. (c) 23. (d) 24. (d)
25. (c) 26. (c) 27. (d) 28. (c) 29. (d) 30. (a) 31. (c) 32. (d)
33. (b) 34. (a) 35. (d) 36. (b) 37. (a) 38. (a) 39. (a) 40. (a)
41. (a) 42. (b) 43. (a) 44. (a) 45. (b) 46. (b) 47. (b) 48. (c)
49. (a) 50. (a) 51. (e) 52. (b) 53. (d) 54. (b) 55. (b) 56. (c)
57. (b) 58. (a) 59. (d) 60. (a) 61. (d) 62. (c) 63. (a) 64. (e)
65. (d) 66. (d) 67. (c) 68. (c) 69. (a) 70. (a) 71. (b) 72. (c)
73. (d) 74. (a) 75. (c) 76. (b) 77. (b) 78. (a) 79. (d) 80. (a)
81. (c) 82. (c) 83. (b) 84. (a) 85. (d) 86. (d) 87. (c) 88. (d)
89. (b) 90. (a) 91. (d) 92. (d) 93. (b) 94. (a) 95. (b) 96. (a)
97. (a) 98. (c) 99. (a) 100. (d) 101. (a) 102. (a) 103. (c) 104. (a)
105. (d) 106. (d) 107. (c) 108. (b) 109. (c) 110. (c) 111. (b) 112. (d)
113. (c) 114. (a) 115. (c) 116. (c) 117. (a) 118. (c) 119. (b) 120. (d)
121. (c) 122. (d) 123. (d) 124. (a) 125. (b) 126. (b) 127. (d) 128. (b)
129. (c) 130. (a) 131. (b) 132. (a) 133. (c) 134. (c) 135. (b) 136. (c)
137. (d) 138. (c) 139. (c) 140. (d) 141. (b) 142. (e) 143. (b) 144. (d)
145. (b) 146. (a) 147. (a) 148. (a) 149. (a) 150. (c) 151. (c) 152. (c)
153. (b) 154. (a) 155. (b) 156. (b) 157. (c) 158. (c) 159. (d) 160. (c)
161. (a) 162. (b) 163. (c) 164. (a) 165. (c) 166. (b) 167. (b) 168. (d)
169. (a) 170. (a) 171. (d) 172. (b) 173. (d) 174. (c) 175. (b) 176. (c)
177. (d) 178. (c) 179. (b)
26 COMPREHENSIVE OBJECTIVE PHYSICS

Solutions (Set I)
19. Due to unequal and opposite forces, there is a net force
 For 1 and 2. and torque.
For the case of an insultor, the following val- 20. qE = mg
ues of K are not allowed.
mg 9.9 × 10−15 × 10
(i) Negative values or q= = C = 3.3 × 10–18 C.
E 3 × 104
(ii) Values less than 1
(iii) 0 25. U = – pE cos θ
(iv) ∞. U will be maximum when cos θ = – 1 i.e., θ = π.
10V
3. Removal of dielectric medium increases the force by a 26. |E|=
1
m
factor of K. 100
6. When the two balls are connected by a conducting = 1000 NC–1
wire, the net charge is (2Q – Q) i.e., Q. Since, the
mv2 λ
balls are identical therefore charge Q gets shared 27. =q
equally between the two balls. R 2πε 0R
Now, F ∝ 2Q2 …(i) Clearly, v is independent of R.

FG Q IJ 2
 For 28
F′ ∝
H 2K …(ii)
1 Q 1 Q
Dividing (ii) by (i), we get E= 2
=
4 πε0 R ε 0 4 πR 2
F ′ Q2 1 1 σ
= × 2
= or E=
F 4 2Q 8 ε0
F If σ is constant, then the electric field on the surface
or F′ = . of the sphere does not depend upon the radius of the
8
8. The result follows from considerations of symmetry. sphere.
13. The forces on the dipole are equal and opposite. So,
the net force is zero. 6C
29. σ= −2
→ → → → 6 × 5 × 10 × 5 × 10−2 m2
Again, since p and E are parallel. So, U(= – p . E ) is
minimum. 1
=× 104 C m–2
14. The correct choice is (d). It follows from considera- 25
tions of symmetry. = 400 C m–2
mg 30. Due to electrostatic induction, there will be a slight
15. eE = mg or E =
e decrease in the positive charge of the charged ball.
16. The force will be parallel to BC. No force perpendicu-
E0
lar to BC. 32. K=
E
q1q2
17. Clearly, the x-component of total force 2 The value of E in a metal is zero.
b → →
q q q 34. In the case of axial line, both p and E are in the
is proportional to 22 + 32 sin θ
b a q1q3 same direction.
2
a 35. It is a simple concept of electrostatic induction.
18. In the first case, F ∝ q1q2
Fig. 45 Fe
FG q + q2 IJ 2 36. g′ = g +
m
H K
1
In the second case, F ∝
2 Clearly, due to increase in the effective value of accel-

FG q + q IJ 2
eration due to gravity, T will be reduced T ∝
LM 1 OP .
H 2 K
1 2 > q1q2
MN g PQ
ELECTROSTATICS 27
37. F ∝ q(Q – q)
FG 1 IJ . This eliminates
F will be maximum when
q=Q–q
61. Graph should be non-linear F ∝
r H 2 K
(a) and (b). Again, F should be very large when r is
Q very small. Also, F should be very small when r is
or 2q = Q or q = . very large. This decides in favour of (d).
2
45. q = ne For 62
q = 1019 × 1.6 × 10–19 C = 1.6 C.
46. Think in terms of electrostatic induction.
 The electric field at a point on the equato-
rial line is opposite to the direction of the di-
→ →
47. Fy = − e E y . pole moment of the dipole.
48. Note that the equation x2 + y2 = 12 is the equation of
circle. 63. The result follows from considerations of symmetry.
This equation is purely hypothetical. 64. Charges will be induced on the metal plates A and B
as shown.
Think !
Can we have circular electric lines of force ?

50. W = QE1a + QE2b.


51. Negative charges are attracted to surface of M near S
by electrostatic induction and positive charges are
repelled to surface of M away from S such that entire
M still remains neutral (uncharged).
Charge on S is also redistributed to have more charge
Fig. 46
on surface near to M due to the field set up by nega-
tive charges on M. It is clear from Fig. 46 that (a), (b) (c) and (d) are
Diagram e thus best illustrates the charge distribu- correct.
tion on both S and M. 65. The two forces on any charge will be inclined to each
52. The concept of the electric field is represented in dia- other at an angle of 60°.
gram by lines of force, which form a convenient way 66. Basic concept.
of visualising electric field patterns. The lines of force 67. No field inside the hollow conducting sphere.
are always originated from a positive charge and ended 68. Coulomb forces between unequal charges are equal.
on a negative charge, the direction of which is the
q
direction that a positive charge would move when 69. 2 = 9 × 109
0.60 × 0.60
placed in the electric field.
2 × 0.60 × 0.60
Thus diagram b best represents the electric field pat- or q= C
tern of the two unlike charges. 9 × 109
= 8 × 10–11 C.
53. The motion of the electron is a combination of uni-
form motion and uniformly accelerated motion. 70. No system can be in equilibrium under the influence
of electrostatic forces only.
55. q = 106 × 1.6 × 10–19 C
71. The change in the kinetic energy of the electron is
= 1.6 × 10–13 C.
equal to the change in its potential energy, which in
56. The nature of material of the plate is not important.
turn is proportional to the change in the electrostatic
57. Both the proton and the electron carry equal charges. potential.
58. Coulomb force is determined by charges and not by 72. Electric lines of force and equipotential surfaces con-
masses. stitute a mutually perpendicular network.
59. Note that there is no field inside the metallic shell. 73. A and B are at the same potential.
60. Introduction of dielectric reduces the electric field. 74. Note that electric potential in the broad side-on posi-
Removal would increase the electric field. tion is zero.
28 COMPREHENSIVE OBJECTIVE PHYSICS

76. Using Gauss’s law, +


+ + +
+ –– ––
+
q +
φ2 – φ1 = or q = (φ2 – φ1)∈o. +
+ + + +
∈o
X
q
77. Electric flux emanating from a charge q is . Flux
ε0
Fig. 47
through one face is one-sixth of this flux.
78. The circle around the charge is an equipotential line. Since there is free flow of electrons from the conduc-
79. Equal and opposite charges enclosed. Moreover, tor X to the Earth, the conductor must be at earth
charges present outside the surface make no contri- potential eventually.
bution to electric flux. So, no flux. 104. The electric field strength in a parallel-plate capacitor
80. Equipotential surfaces and electric field form a is given by
mutually perpendicular network. V
E=
81. Symmetry considerations lead to right choice. d
where V is the potential difference between the plates
9 × 109 × 50 × 1.6 × 10−19
87. V= volt and d is the separation of the plates.
9 × 10−15
105. Negative charges are being induced on the surface of
= 8 × 106 volt. the Earthed conducting sphere facing the positively-
charged insulated dome. The electrons from the Earth
100
88. V= − volt = – 20 volt. neutralise the positive charge on the other side of the
5
conducting sphere.
For 89. The electrical potential of the Earth is zero.
 Equipotential surfaces and electric lines of
Positively- + +
+ +
+
force constitute a mutually perpendicular network. charged + +
dome + + Conducting
+ + – sphere
90 to 92. Basic concepts. + + ––

93. Two plane equipotential surfaces can have different + + –

+ + + + –
potentials. ––

9 × 109 × 50 × 1.6 × 10−19
95. V= volt Insulating
10−12 support
= 7.2 × 104 V.
98. Apllication of Gauss’s law and symmetry considera-
Fig. 48
tions lead us to the right choice.
106. The increase/decrease of potential energy will be de-
99. 1.
V∝ cided by the nature of the charges.
r
107. Work done
100. No field inside the metallic conductor.
= 6 C × 106 V
101. U = – pE cos 0°
= 6 × 106 J
or U = – pE
q q
This is the minimum value of potential energy. 108. Potential at A = + −
2 2
a +b a + b2
2
Q
102. V ∝ = zero.
r
V is constant ; so, Q ∝ r. For 109
103. When the negatively charged rod is held near a con-
ductor X that is earthed, the conductor becomes posi-
 Every system in nature tends to have least
potential energy.
tively charged as shown in Fig. 47.
ELECTROSTATICS 29
110. Basic concept. 117. Net charge of an electric dipole is zero.
111. Potential at any point inside the sphere is the same 118. In this question, the effective area is πR2.
as on the surface of the sphere. 119. It follows from symmetry considerations that the
1 Q1 potentials at the centres of two rings are same. No
112. V1 =
4πε0 R 1 potential difference, no work done.
120. If q is the charge and r is the radius of the small
1 Q2 drops, total charge on large drop is Nq and its radius
V2 =
4 πε 0 R 2 1
is N1/3 r. Use the relation for potential V ∝ .
If V1 = V2, then r
Q1 Q 2 121. A proton accelerated through 1 kV acquires 1 keV of
= energy.
R1 R2
122. Distance of corner from centre
or Q1R2 = Q2R1
In this case, there will be no flow of charges. 1
= ( 2 )2 + ( 2 )2 m = 1 m
However, if the given spheres have unequal potentials 2
then the charges shall flow from higher potential to 9 × 109
lower potential. Flow of charges shall be accompanied V= [10 + 5 – 3 + 8] × 10–6 volt
1
by loss of energy.
= 1.8 × 105 volt
9 × 109 × 10−6 × 10 −6 123. No electric field can be present inside a charged
113. U= J = 9 × 10–3 J.
1 conductor.
114. Fig. 49 illustrates clearly the concept involved in the 124 to 126. It is understood that the dielectric fills the whole
given question. of the space between the two plates of the capacitor.
127. A material is suitable for use as a dielectric if both
the dielectric constant and dielectric strength have
large values.
6400 × 1000
128. C= F
9 × 109
6.4
= × 10–3 F = 0.7 mF.
9
129. In parallel combination, the potential difference across
each capacitor is the same as across the combination.
∈o A 5∈o A 5
130. C′ = = = C.
Fig. 49 2d 3d 3
d−
5
Same number of lines of force pass through the two 1C
concentric spheres. 131. 1F=
1V
1 1 Q2
115. mv2 =
2 4 πε0 rmin. 3 × 109 statcoulomb
=
1
statvolt
2Q2 300
or rmin. =
(4 πε 0 ) mv2 = 9 × 1011 statfarad.

1 1 Q2 8 × 10−18 × 8 × 10−18
Clearly, rmin. (i) ∝ Q2 (ii) ∝ (iii) ∝ . 132. W = = J
m v2 2C 2 × 100 × 10 −6
V = 32 × 10–32 J.
116. E= 133. Q = CV, V is constant.
r
or V = rE Q1 C1
∴ =
U = q0V = q0rE. Q2 C2
30 COMPREHENSIVE OBJECTIVE PHYSICS

∈o A (b) E ∝ σ, σ unchanged ; So, E unchanged


134. C =
d−t Q2
In the given problem, t → 0 (c) U=
2C
∈o A C is halved ; So, U is doubled.
∴ C= .
d (d) Note that there is an electrostatic attraction
135. The charges on the capacitors are 120C1 and 200C2 between the oppositively charged plates of the
respectively. Since, the potential becomes zero there- capacitor. Work has to be done against this external
fore the net charge on the combination must be zero. agency.
This requires that the magnitudes of the charges on
each capacitor must be equal. 2πε 0l
∴ 120C1 = 200C2 144. C=
Fr I
or 3C1 = 5C2 log e GH r JK
b

a
136. The network is equivalent to
145. Capacitance of a capacitor C, by definition, is equal to
Q
C=
V
where Q is the stored charge, V is the potential differ-
ence across the capacitor.
Thus capacitor of 1 farad will store 1 coulomb of charge
at a potential difference of 1 volt.
Q 1
146. C = i.e., V = Q and energy stored in a capacitor

z
Fig. 50 V C

Therefore equivalent capacitance. is V dQ = shaded area under the curve of V versus Q


= [2 C series C] || [C series 2 C]

=2
FG 2C × C IJ = 4C
H 2C + C K 3
=
1
C z Q dQ =
1 Q2 1
2 C 2
1
= CV 2 = QV .
2
147. Metal shield is like K = ∞.
137. When dielectric is removed, capacity is reduced.
ε0A
148. C=
1 1 1 1 d
138. = + +
Cs C1 C2 C3 Cd
A=
1 ε0
= C1−1 + C2−1 + C3−1
Cs 3 × 5 × 10 −3
= m2 = 1.7 × 109 m2.
or Cs = [C1−1 + C2 −1 + C3−1] –1 8.85 × 10 −12
140. Think in terms of Wheatstone Bridge.
 Can you keep this 3 F capacitor in your house ?
Q2 Q2
141. Ui = , Uf =
2 KC 2C 149. Field is a storehouse of energy.
150. C = 4πε0R
Uf > Ui
This eliminates (c) and (d). 90 × 10−2
C = F
Again, we know that 9 × 109
V0 or C = 10–10 F
= K or V0 = KV
V = 100 × 10–12 F = 100 pF
This decides in favour of (b).
1
142. (a) Q = CV 151. U= CV 2
2
Q is unchanged ; C is halved ; clearly V will be or U∝C (∵ V is given to be constant.)
doubled.
ELECTROSTATICS 31

U will be maximum when C is maximum. Maximum 162. Since there would be a flow of charges and consequent
capacity is obtained in the case of parallel combina- loss of energy, therefore
tion of three capacitors. (E1 + E2) < E0 or E0 > (E1 + E2).
152. Cp = 2 C C1C2
163. Total capacity =
C C1 + C2
Cs =
2 C1C2
Total charge = V
Cp 2C C1 + C2
= = 4.
Cs C In series combination, the charge on each individual
2 capacitor is the same as the charge on the series com-
1 bination.
153. u= ε E2 So, potential difference across C1
2 0
∴ u ∝ E2. C1C2 V 1 C2
154. C = 4πε0R = . = V.
C1 + C2 C1 C1 + C2
C 164. Clearly, the potential difference V gets equally divided
R=
4 πε0 between the parallel combination (of C and 2 C) and
= 9 × 109 × 1 × 10–6 m capacitor of capacity 3 C.
= 9 × 103 m = 9 km
155. The given network is a parallel combination of three
capacitors.
22
156. A= × 0.08 × 0.08 m2
7
d = 10–3 m

8.85 × 10−12 × 22 × 0.08 × 0.08 Fig. 51 Fig. 52


C= F
7 × 10−3 Also, in parallel combination, the potential difference
= 0.178 × 10–9 F across each individual capacitor is the same as the
Q = 0.178 × 10–9 × 100 C = 1.8 × 10–8 C. potential difference across the combination.
157. Q = CV So, potential difference across capacitor of capacity C
V unchanged ; C increased. So, Q increases. V
is .
V0 100 2
158. Vm = = V = 10 V
K 10 CV
∴ Charge on capacitor of capacity C is .
In this question, it is understood that the dielectric 2
fills the whole of the space between the two plates of
the capacitor. 165.
159. Cp = (2 + 2) µF = 4 µF
4 × 12 48
C= µF = µF or C = 3 µF. Fig. 53
4 + 12 16
V0 V0 Now, Q same ; VAB 15
160. K= or V = = =3
V K VBC 5
1 V0 or VAB = 3VBC
or V = or K = 8.
8 0 K
If VBC = x, then VAB = 3x.
Capacity of parallel combination is 15 µF.
For 161
Now, 4x = 2000 or x = 500
 Note that the effective distance is d . ∴ VAB = 3 × 500 V = 1500 V
K
VA = 2000 V, VAB = 1500 V ; So, VB = 500 V.
32 COMPREHENSIVE OBJECTIVE PHYSICS

166. The required arrangement is shown below. 171. If a conducting path is provided between the two plates
of a capacitor, then any amount of charge may be
given to the capacitor. [This is of course only a
theoretical statement].
172. Effective distance is halved. Capacity is doubled.
173. Since the thickness of the metal sheet is to be taken
as zero therefore (a) and (b) are ruled out. Again, since
the battery remains connected therefore (c) is ruled
out.
174. F = qE
Since E is uniform therefore F is constant.
Fig. 54
175. When slab is taken out, C is reduced. V constant. Q ∝
A 2A C. So, Q has to be reduced. Thus, some charge will be
167. C∝ , C′ ∝ returned to the battery.
d d/2
A C′ Qmax
or C′ ∝ 4
; = 4 176. = Dielectric strength. Clearly, Qmax. does not
d C Kε 0 A
or C′ = 4 × 12 pF = 48 pF. depend upon the separation between the plates.
168. Think in terms of Wheatstone bridge. 177. Cmax. = 3 × 6 µF = 18 µF
169. The given network is a parallel combination of 6
Cmin. = µF = 2 µF.
capacitors. 3
178. All the charge given to the inner sphere shall flow to
For 170
the outer sphere. So, the given system is equivalent
O Capacity does not depend upon the applied volt- to a single spherical conductor of radius b.
age.
O Introduction of dielectric slab/metal plate increases Q
179. V=
capacity. C
V is to be kept constant ; C is increased ; Q has to
increase.

KNOWLEDGE PLUS
O Two point charges + 8q and – 2q are located at x = 0 and x = L respectively. The location of a point on the x axis
at which the net electric field due to these two point charges is zero is
L
(a) 4L (b) 8L (c) (d) 2L. [AIEEE 2005]
4
Ans. (d)
Electric field cannot be zero at any point between A and B. This is because both
8q A B – 2q
the fields will act in the same direction. x=L
x=0
Electric field cannot be zero at a point to the left of A because the electric field of L l
charge 8q shall always dominate the electric field of charge “– 2q”.
Fig. 55
So, electric field can be zero only at a point to the right of charge “– 2q”. Suppose
electric field is zero at a distance l to the right of point B.
1 2q 1 8q
Now, 2
= or 4l2 = (L + l)2 or 2l = L + l or l = L
4 π ∈0 l 4π ∈0 (L + l)2
Distance from A is 2L.
ELECTROSTATICS 33

MCQs
based on
SET II

Important Basic Concepts


+
Application of Formulae
+
Brain Teasers
Average time allowed per question is 50 seconds.

(a) 1.67 × 10–27 N C– 1 (b) 1.02 × 10–7 V m– 1


COULOMB’S LAW AND
(c) 1.6 × 10–19 V m– 1 (d) 1.04 × 10–27 V m– 1.
ELECTRIC FIELD
[National Standard Exam. in Physics 1998]
185. A metallic solid sphere is placed in an uniform elec-
180. The ratio of the forces between two small spheres
tric field. The lines of force follow the path (s) shown
with constant charges (i) in air (ii) in a medium of in Fig. 56 as
dielectric constant K is respectively
(a) 1 : K (b) K : 1
(c) 1 : K2 (d) K 2 : 1. [AFMC 1995]
181. Two point charges placed at a distance r in air expe-
rience a certain force. Then the distance at which
they will experience the same force in a medium of
dielectric constant K is Fig. 56
(a) r/K (b) Kr (a) 1 (b) 2
r (c) 3 (d) 4. [IIT 1996]
(c) (d) r K . [MP PMT 2001]
K 186. A charge q is divided into two parts q1 and (q – q1 ).
182. Let [ε0] denote the dimensional formula of the q
What is the ratio so that the force between two
permittivity of vacuum. If M = mass, L = length, q1
parts placed at a given distance apart is maximum ?
T = time and I = electric current. Then
(a) 1 : 1 (b) 2 : 1
(a) [ε0] = [M– 1 L– 3 T2 I] (b) [ε0] = [M– 1 L– 3 T 4 I2]
(c) 1 : 2 (d) 1 : 4.
(c) [ε0] = [M– 1 L2 T – 1 I – 2] (d) [ε0] = [M– 1 L2 T – 1 I].
187. A pith ball of mass 0.05 g carries a charge of + 100
[National Standard Exam. in Physics 2001] stat C. The charge that must be given to a ball placed
183. A point charge A of charge +4 µC and another point 10 cm directly above the pith ball which will hold the
charge B of charge –1 µC are placed in air at a distance pith ball in equilibrium, is
1 metre apart. Then the distance of the point on the (a) + 49 stat C (b) – 49 stat C
line joining the charges and from the charge B, where (c) – 49 C (d) + 49 C.
the resultant electric field is zero, is (in metre) 188. A plastic globule of mass m and radius r carrying
(a) 1.5 (b) 0.5 charge q moves upwards with velocity v0 in an elec-
tric field E. Which of the following forces is directed
(c) 1 (d) 2.
upwards ?
[Karnataka CET 1998] (a) mg (b) qE
184. The electric field required to exert a force on proton (c) 6 ηπrv0 (d) none of these.
equal to its weight is [EAMCET 1995]
34 COMPREHENSIVE OBJECTIVE PHYSICS

189. Two copper balls, each weighing 10 g, are kept in air C. The direction of rotation is anti-clockwise. If the
10 cm apart. If one electron from every 106 atoms is angle of rotation θ is measured with reference to a
transferred from one ball to the other, the coulomb line which is perpendicular to the electric field, then
force between them is (atomic weight of copper is 63.5) the variation of τ with θ is best represented by curve
(a) 2.0 × 1010 N (b) 2.0 × 104 N (see Fig. 59)
(c) 2.0 × 108 N (d) 2.0 × 106 N.
[Karnataka CET 2002]
190. Two identical point charges are placed at a separa-
tion of r. P is a point on the line joining the charges,
at a distance x from any one charge. The field at P is
E. E is plotted against x for values of x from close to
zero to slightly less than r. Which of the following
best represents the resulting curve ?

Fig. 59

E E (a) I (b) II
(c) III (d) IV.

O x r x r
193. The insulation property of air breaks down at 3 × 106
O
V m– 1. The maximum charge that can be given to a
(a ) (b) sphere of diameter 5 m is nearly
(a) 2 × 10– 2 C (b) 2 × 10– 3 C
(c) 2 × 10– 4 C (d) 2 × 10– 5 C.
194. In Figs. 60 (a) and 60 (b), a particle with small charge
E x E – q is free to move up or down, but not sideways near
r x r
O O a larger fixed charge Q. The small charge q is in
equilibrium because in the positions shown, the
electrical upward force is equal to the weight of the
particle. Which of the following is correct ?

(c) (d )
Fig. 57

[WB JEE 1995]


191. An electric dipole consisting of two opposite charges
of 2 × 10 –6 C each, separated by a distance of 0.03 m, Fig. 60
is placed in an electric field of 2 × 105 NC –1. The
(a) In Fig. 60 (a), – q is in stable equilibrium
maximum torque on the dipole will be
(a) 12 × 10– 1 N m (b) 12 × 10– 3 N m (b) In Fig. 60 (a), – q is in neutral equilibrium

(c) 24 × 10– 1 N m (d) 24 × 10– 3 N m. (c) In Fig. 60 (b), – q is in stable equilibrium


(d) Neither in Fig. 60 (a) nor in Fig. 60 (b) is – q in stable
192. The electric dipole is situ-
equilibrium.
ated in an electric field as
shown in Fig. 58. Both 195. Point charge q moves
90°
the dipole and the elec- from point P to point S
tric field are in the plane –q +q → along the path PQRS
of the paper. The dipole
C E (Fig. 61) in a uniform

90°
is rotated about an axis electric field E pointing
perpendicular to the parallel to the positive di-
plane of the paper and rection of x-axis. The
passing through a point Fig. 58 co-ordinates of the points
Fig. 61
ELECTROSTATICS 35
P, Q, R and S are (a, b, 0), (2a , 0, 0), (0, – b, 0) and (0, 201. Six charges, three positive and P Q
0, 0) respectively. The work done by the field in the three negative of equal magnitude
above process is given by are to be placed at the vertices of a U R
(a) qEa (b) – qEa regular hexagon such that the elec- O
tric field at O is double the electric
(c) qEa 2 (d) qE (2a)2 + b2 . field when only one positive charge T S
of same magnitude is placed at R. Fig. 62
196. A charged particle of mass m and charge q is released
Which of the following arrangements of charges is
from rest in an electric field of constant magnitude
possible for P, Q, R, S, T and U respectively ?
E. The kinetic energy of the particle after time t is
(a) +, –, +, –, –, + (b) +, –, +, –, +, –
2 2 2 2 2 (c) +, +, –, +, –, – (d) –, +, +, –, +, –
2E t E q t
(a) (b)
mq 2m [IIT Screening 2004]
202. A hollow metallic sphere of radius 10 cm is given a
Eq2m Eqm
(c) (d) . charge of 3.2 × 10 –9 coulomb. The electric intensity
2 t2 2t
at a point 4 cm from the centre is
[Karnataka CET 2003] (a) 2.88 V (b) 288 V
197. A charge q is placed at the centre of the line joining (c) 9 × 10 –9 V (d) 0 V. [MP PMT 1990]
two equal charges Q. The system of the three charges 203. The number of electrons to be put on a spherical con-
will be in equilibrium if q is equal to ductor of radius 0.1 m to produce an electric field of
(a) – Q/2 (b) – Q/4 0.036 N C –1 just above its surface is
(c) + Q/4 (d) + Q/2. (a) 2.7 × 105 (b) 2.6 × 105
[IIT 1987 ; Karnataka CET 2001, 2003] (c) 2.5 × 105 (d) 2.4 × 105.
198. Two equal negative charges (– q each) are fixed at the 204. Two point charges placed at a distance of 0.20 m in
points [0, a] and [0, – a] on the y-axis. A positive air repel each other with a certain force. When a di-
charge Q is released from rest at the point [2a, 0] on electric slab of thickness 0.08 m and dielectric con-
the x-axis. The charge Q will stant K is introduced between these point charges,
(a) execute simple harmonic motion about the origin force of interaction becomes half of its previous value.
(b) move to the origin and remain at rest
Then K is nearly
(a) 1 (b) 2
(c) move to infinity
(d) execute oscillatory but not simple harmonic motion. (c) 2 (d) 4.
[IIT 1978 ; Karnataka CET 2001] 205. Two point charges – 2Q and + Q are situated as shown
in Fig. 63. At which point could the resultant electric
199. Four charges equal to – Q are placed at the four cor-
field due to these charges be zero ?
ners of a square and a charge q is at its centre. If the
system is in equilibrium, the value of q is
Q Q
(a) – (1 + 2 2 ) (b) (1 + 2 2 )
2 4 Fig. 63
Q Q (a) A (b) B
(c) – (1 + 2 2 ) (d) (1 + 2 2 ) .
4 2 (c) C (d) D. [Manipal 1995]
[AIEEE 2004]
206. Two spherical conductors B and C having equal ra-
200. Three charges 4q, Q and q are placed in a straight dii and carrying equal charges on them repel each
line of length l at point 0, l/2 and l respectively. What other with a force F when kept apart at some dis-
should be Q in order to make the net force on q to be tance. A third spherical conductor A having same
zero ? radius as that of B but uncharged is brought in con-
tact with B, then brought in contact with C and fi-
1
(a) – q (b) − q nally removed away from both. The new force of re-
2
pulsion between B and C is
(c) – 2q (d) 4q. [AIIMS 1990] (a) F/8 (b) 3 F/4
(c) F/4 (d) 3 F/8. [AIEEE 2004]
36 COMPREHENSIVE OBJECTIVE PHYSICS

207. A charge Q is uniformly distributed throughout the


Y Y
volume of a non-conducting sphere of radius R. The
dielectric constant of the material is K. The electric E E
field intensity at a distance r (< R) from the centre is
1 Q
(a) (b) zero
4 πε 0K r 2
O z
LM 1 − r OP .
O z Z Z
1 Qr Q
(c) (d)
N r2 R3 Q
(a) (b )
4πε0K R 3 4 πε 0K
Y Y
[CET 2002]
E E
208. A metallic particle having no net charge is placed
near a finite metal plate carrying a positive charge.
The electric force on the particle will be
(a) towards the plate (b) away from the plate O z Z O z Z
(c) parallel to the plate (d) zero. [JIPMER 2002]
209. An electric dipole is placed in a uniform electric field.
(c) (d)
The net electric force on the dipole
(a) is always zero Fig. 64
(b) depends on the orientation of the dipole [CET 2002]
(c) can never be zero
214. If the Earth’s surface is treated as a conducting sur-
(d) depends on the strength of the dipole. face with some charge, what should be the charge
[Haryana PMT 1994] per unit area so that a proton remains suspended in
210. A circular metal plate has radius 2.0 m and negligi- space near the Earth’s surface ? Take g = 10 m s– 2.
ble thickness. It is given a charge of 40 mC. The elec- (a) 0.9 × 10 –18 C m–2 (b) 0.9 × 10 –12 C m–2
tric field at a point close to the plate and near its (c) 0.9 × 10 –6 C m–2 (d) 0.9 C m–2.
centre is nearest to 215. Two identical positive charges are fixed on the y-axis,
(a) 0.9 × 108 V m–1 (b) 1.8 × 108 V m–1 at equal distances from the origin O. A negatively
(c) 3.6 × 108 V m–1 (d) 7.2 × 108 V m–1. charged particle starts on the x-axis at a large
[National Standard Exam. in Physics 1991] distance from O, moves along the x-axis, passes
through O and moves far away from O. Its acceleration
211. Three particles have charges + 20 µC each, and they
a is taken as positive along its direction of motion.
are fixed at the corners of an equilateral triangle of
The particle’s acceleration a is plotted against its x-
side 0.5 m. The force on each of the particles has coordinate. Which of the following best represents
magnitude the plot ?
(a) zero (b) 14.4 N

(c) 28.8 N (d) 14.4 3 N. a a


x x
[National Standard Exam. in Physics 1991] O
O
212. In a regular polygon of n sides, each corner is at a
distance r from the centre. Identical charges of mag-
nitude q are placed at (n – 1) corners. The field at the (a) (b)
centre is
q q
(a) k (b) (n − 1) k 2 a a
r2 r x x
n q n−1 q O O
(c) k (d) k 2.
n − 1 r2 n r
213. A circular ring carries a uniformly distributed posi-
tive charge and lies in X-Y plane with centre at ori- (c) (d)
gin of co-ordinate system. If at a point (0, 0, z), the Fig. 65
electric field is E, then which of the following graphs
is correct ? [DCE 1999]
ELECTROSTATICS 37
(a) zero (b) q′ = q
ELECTRIC POTENTIAL, ELECTRIC FLUX r r
(c) q′ = – 1 q (d) q′ = 1 q .
AND GAUSS’S LAW r2 r2

216. The dielectric strength of air at NTP is 3 × 106 V m– 1. [National Standard Exam. in Physics 1998]
Then the maximum charge that can be given to a 222. A charge + q is placed at each of the points x = x0 , x
spherical conductor of radius 3 m is = 3x0 , x = 5x0 , ......., ad infinitum on the x-axis, and
(a) 3 × 10– 1 C (b) 3 × 10– 2 C a charge – q is placed at each of the points x = 2x0 , x
(c) 3 × 10– 3 C (d) 3 × 10– 4 C . = 4x0, x = 6x0 ...... ad infinitum. Here x0 is a positive
217. In the previous question, the maximum potential up constant. Take the electric potential at a point due to
to which the conductor can be charged, is a charge Q at a distance r from it to be Q/(4πε0 r).
(a) 9 × 103 V (b) 9 × 106 V Then the potential at the origin due to the above sys-
(c) 9 × 107 V (d) 9 × 108 V. tem of charges is
218. A infinite number of charges, each numerically equal q
(a) 0 (b)
to q and of the same sign, are placed along the x-axis 8πε0 log e 2
at x = 1, x = 2, x = 4, x = 8 and so on. Then the q log e 2
(c) ∞ (d) .
electrical potential at x = 0 due to this set of charges, 4πε 0 x0
is [IIT Screening 1998]
2q 223. Electric charge is uniformly distributed along a long
(a) 2q (b)
5 straight wire of radius 1 mm. The charge per cm
4q 4q length of the wire is Q coulomb. Another cylindrical
(c) (d) . surface of radius 50 cm and length 1 m symmetri-
3 5
cally encloses the wire as shown in the Fig. 67. The
219. When a charge of 3 C is placed in a uniform electric
total electric flux passing through
field, it experiences a force of 3 × 103 N. Within this
the cylindrical surface is
field, potential difference between two points sepa-
Q
rated by a distance of 0.01 m is (a)
ε0
(a) 10 V (b) 102 V
100Q
(c) 103 V (d) 3 × 103 V. (b)
ε0
220. A point charge q is placed at one corner of a cube of 10Q
edge a. The flux through each of the cube faces is (c)
(πε 0 )
q q 100Q
(a) (b) (d) . [MP PET 2001]
ε0 16 ε 0 (πε 0 )
Fig. 67
q q 224. A ball of mass 1 g and charge 10 –8
C moves from a
(c) (d) . [DCE 2002]
24 ε 0 48 ε 0 point A whose potential is 600 V to the point B whose
potential is zero. Velocity of the ball at the point B is
221. Charge on the outer sphere is q and the inner sphere
is grounded. Then the charge q′ on the inner sphere 20 cm s– 1. The velocity of the ball at the point A is
is q′, for (r2 > r1) (a) 16.7 cm s– 1 (b) 16.7 m s– 1
(c) 2.8 cm s– 1 (d) 2.8 m s– 1.
[Karnataka CET 2001]
225. Two spherical conductors of radii 4 m and 5 m are
charged to the same potential. If σ1 and σ2 be the
respective values of the surface density of charge on
σ
the two conductors, then the ratio 1 is
σ2
25 16
(a) (b)
16 25
5 4
(c) (d) .
4 5
[Karnataka CET 1998]
Fig. 66
38 COMPREHENSIVE OBJECTIVE PHYSICS

226. A hollow metal ball 8 cm in diameter is given a charge


– 4× 10 –8C. The potential on the surface of the ball is
(a) – 9000 volt (b) – 900 volt
(c) – 90 volt (d) zero volt.
227. Fig. 68 shows two points A and B in uniform electric
→ V V
field E . The potential difference between the points
A and B is
R R
(a) Er cos θ
B O O
r r
E
(b) (c) (d )
r cos θ r ®
E
Fig. 69
Er q
(c) A [Bharati Vidyapeeth 1999]
cos θ
Fig. 68 232. A ring of radius 0.5 m carries a total charge of
E 1.0 × 10 –10 C distributed non-uniformly on its
(d) cos θ. →
r
circumference, producing an electric field E . The

z
228. The ratio of the forces between two small conducting
r=0 → →
spheres charged to constant potentials in (a) air (b) a value of − E. dr (r = 0 being the centre of the
medium of K = 2 is r=∞

(a) 1 : 4 (b) 1 : 2 ring) is


(a) + 1.8 V (b) – 6V
(c) 2 : 1 (d) 4 : 1.
(c) – 1.8 V (d) + 6V. [MNR 2002]
229. Charge of + 1 esu, + 2 esu and + 3 esu are placed at
the three corners of a square. What charge should be 233. Two concentric spheres of radii R and r have similar
placed at the fourth corner so that potential at the charges with equal surface charge densities (σ). The
electric potential at their common centre is
centre may be zero ? One side of the square is 2 cm.
σ
(a) + 6 esu (b) – 6 esu (a) σ/ε0 (b) (R − r )
ε0
(c) + 4 esu (d) – 4 esu.
σ σ
[All India PM/PD 2001] (c) (R + r) (d) (R + r) .
ε0 4πε 0
230. Given : Electric potential at a point is : φ = x2y + yz. [AFMC 2002]
The electric field at the point (1, 3, 1) is 234. A charge + Q at A
(a) 7 units (b) 70 units (see Fig. 70) produces
(c) 49 units (d) 490 units. [MNR 2000] electric field E and
electric potential V
231. A solid sphere of radius R is charged uniformly. The
at D. If we now put
electrostatic potential V is plotted as a function of charges – 2Q and
distance r from the centre of the sphere. Which of the + Q at B and C re-
following best represents the resulting curve ? spectively, then the
electric field and po-
tential at D will be Fig. 70
(a) E and 0 (b) 0 and V

V E V
(c) 2 E and (d) and .
2 2 2
V V [National Standard Exam. in Physics 1992]
235. A particle A has charge + q and particle B has charge
R R + 4q with each of them having the same mass m.
O r O r When allowed to fall from rest through the same
(a ) (b) electric potential difference, the ratio of their speeds
vA
will become
vB
ELECTROSTATICS 39
(a) 1 : 2 (b) 2 : 1 when a charge of 40 nano-coulomb is moved from
(c) 1 : 4 (d) 4 : 1. [MNR 1991] infinity to a point which is at a distance of 2 cm from
the surface of the sphere is
236. The electric potential V at any point x, y, z (all in
metre) in space is given by V = 4x2 volt. The electric (a) 14.4 π (b) 28.8 π
field at the point (1 m, 0 m, 2 m) in V m– 1 is (c) 144 π (d) 288 π.
(a) 8 along negative x-axis [EAMCET 2003]
(b) 8 along positive x-axis 242. A thin spherical conducting shell of radius R has a
(c) 16 along negative x-axis charge q. Another charge Q is placed at the centre of
(d) 16 along positive x-axis. [IIT 1992 ; MP PMT 2001] the shell. The electrostatic potential at a point P a

237. Consider the charge configuration q2 R


distance from the centre of the shell is
and a spherical Gaussian surface as
+q1 2
shown in Fig. 71. When calculating (q + Q) 2 2Q
(a) (b)
the flux of the electric field over the –q1 4πε 0 R 4πε 0R
spherical surface, the electric field will 2Q 2q 2Q q
(c) − (d) + .
be due to Fig. 71 4πε 0R 4πε 0R 4πε 0R 4πε 0 R
(a) q2 (b) only the positive charges [AIEEE 2003]
(c) all the charges (d) + q1 and – q1.
243. Two charges q 1 and q 2 are q3
C
[IIT Screening 2004] placed 30 cm apart, as shown
238. A metallic shell has a point charge ‘q’ kept inside its in the Fig. 73. A third charge
cavity. Which one of the following diagrams correctly

40 cm
q3 is moved along the arc of a
represents the electric lines of forces ? circle of radius 40 cm from C
to D. The change in the poten-
q2
(a) (b) tial energy of the system is q1
A 30 cm B D
q3
k , where k is : Fig. 73
4πε 0
(a) 8 q2 (b) 6 q1
(c) (d)
(c) 8 q1
(d) 6 q2. [All India PM/PD 2005]
Fig. 72 244. A, B, C, D, P and Q are
points in a uniform
[IIT Screening 2003] electric field. The
239. A bullet of mass 2 g is having a charge of 2 µC. potentials a these points
Through what potential difference must it be are V (A) = 2 volt. V (P) =
accelerated, starting from rest, to acquire a speed of V (B) = V (D) = 5 volt. V
10 m/s ? (C) = 8 volt. The electric
(a) 50 kV (b) 5 V field at P is
(c) 50 V (d) 5 kV. (a) 10 V m– 1 along PQ
[All India PM/PD 2004] (b) 5 V m– 1 along PC
240. The inward and outward electric flux from a closed (c) 15 2 V m– 1 along PA Fig. 74
surface are respectively 8 × 103 and 4 × 103 units.
(d) 5 V m– 1 along PA.
Then the net charge inside the closed surface is
(a) – 4 × 103 coulomb (b) 4 × 103 coulomb [National Standard Exam. in Physics 1997]
245. Twenty seven drops of water of the same size are
− 4 × 10 3 equally and similarly charged. They are then united
(c) coulomb (d) – 4 × 103 ε0 coulomb.
ε0 to form a bigger drop. By what factor will the electri-
[KCET 2003] cal potential change ?
(a) 9 times (b) 27 times
241. A charged sphere of diameter 4 cm has a charge
density of 10 –4 coulomb/cm2. The work done in joule (c) 6 times (d) 3 times.
[MP PET 1991, 94]
40 COMPREHENSIVE OBJECTIVE PHYSICS

246. Three infinitely charged sheets 2


250. A long string with a charge of λ per unit length passes
are kept parallel to x – y plane 2
+d through an imaginary cube of edge l. The maximum
R R R
having charge densities as possible flux of the electric field through the cube
shown in Fig. 75. Then the C1 d C2 will be
value of electric field at ‘P’ is (a) λl/ε0 (b) 2 λl/ε 0
− 4σ  4σ  +q d –q
(a) k (b) k (c) 6λl2/ε0 (d) 3 λl/ε 0 .
∈0 ∈0
Fig. 75 [CMC LDH 1998]
− 2σ  2σ 
(c) k (d) k.
∈0 ∈0 251. When two uncharged metal balls, each of radius 0.09
[IIT Screening 2005] mm, collide, two electrons are transferred between
247. Fig. 76 shows two equipotential lines in x, y plane for them. The potential difference between them would be
an electric field. The scales are marked. The x- (a) 16 µV (b) 16 pV
component Ex and y-component Ey of the field in the (c) 32 µV (d) 64 µV.
space between these equipotential lines are respectively [Himachal PMT 1999]
252. A and B are two points on the axis and the
perpendicular bisector respectively of an electric
dipole. A and B are far away from the dipole and at

equal distances from it. The fields at A and B are EA

and EB are respectively such that
→ → → →
(a) E A = EB (b) E A = 2 EB
Fig. 76 → → → 1 →
(c) E A = − 2 EB (d) E A = EB .
(a) + 100 V/m, – 200 V/m (b) – 100 V/m, + 200 V/m 2
(c) + 200 V/m, + 100 V/m (d) – 200 V/m, – 100 V/m. [AMU 2000]
[National Standard Exam. in Physics 1991] 253. In the previous question if VA and VB be the potentials
248. A 5 C charge experiences a force of 2000 N when at A and B respectively, then
moved between two points separated by a distance of (a) VA = VB (b) VA = 2VB
2 cm in a uniform electric field. The potential differ- (c) VA ≠ 0, VB = 0 (d) VA = 0, VB = 0.
ence between the two points is
254. Two points are at distances a and b (a < b) from a
(a) 8 V (b) 200 V
long string of charge per unit length λ. The potential
(c) 800 V (d) 20000 V.
difference between the points is proportional to
[MP PET 1996]
249. A system consists of two metallic spheres of radii r1 b b2
(a) (b)
and r2 connected by a thin wire and a switch S as a a2
shown in Fig. 77. Initially S is open and spheres carry b b
charges q1 and q2 respectively. If the switch is closed, (c) (d) log e . [MNR 2000]
a a
the potential of the system is 255. Four charges 2 C, – 3 C, – 4 C and 5 C respectively
q2 are placed at all the corners of a square. Which of the
q1 following statements is true for the point of intersec-
S
tion of the diagonals ?
r1 (a) Electric field is zero but electric potential is non-zero.
r2 (b) Electric field is non-zero but electric potential is zero.
(c) Both electric field and electric potential are zero.
Fig. 77 (d) Neither electric field nor electric potential is zero.
(a) (1/4 πε0) {q1q2/r1r2} (b) (1/4πε0){(q1 + q2)/(r1 + r2)} [CMC LDH 1997]
(c) (1/4πε0){q1/r1 + q2/r2} (d) {1/4πε0){(q1 + q2)/(r1r2)1/2}. 256. A half-ring of radius r has a linear charge density λ.
[National Standard Exam. in Physics 2005] The electric potential at the centre of the half-ring is
ELECTROSTATICS 41

λ λ Qq
(a) (b) (c) (d) Escape is not possible.
4ε0 4 π 2ε 0 r 4 πε 0 mR 2

λ λ 263. Consider a hemispherical surface of radius r in the


(c) (d) . [MNR 2001] electric field of a positive point charge q. The electric
4 πε 0 r 4ε0 r
flux through this hemisphere is
257. Two thin wire rings each having a radius R are placed
q
at a distance d apart with their axes coinciding. The (a) zero (b)
ε0
charges on the two rings are + q and – q. The poten-
tial difference between the centres of the two rings is q 2q
(c) (d) .

q
LM1 1
OP 2ε0 ε0
264. The electron and protons in a singly ionised helium
(a)
MMN
4 π ∈0 R

R 2 + d 2 PQ
P (b) zero
atom are bound at a distance of 2Å. The electrostatic

q M1
L 1
OP potential energy of the system is
(a) – 13.6 eV (b) – 14.4 eV
2π ∈0 M R P
(c) − (d) qR/4π∈0 d2.
MN R 2 + d 2 PQ (c) – 27.2 eV (d) – 40.8 eV.
[AIEEE 2005] 265. The electrostatic potential at a distance of 40 cm from
258. The electric potential in a region along the X-axis the centre of a charged metal sphere of radius 50 cm
is 18 volt. The charge carried by the sphere is
varies with x according to the relation V (x) = 4 +
5x2. Then the incorrect statement is (a) 1 mC (b) 1 µC
(a) potential difference between the points x = 1 and x = – 2 is (c) 1 nC (d) 1 pC.
15 volt →
(b) force experienced by a one coulomb charge at x = – 1 m will 266. Given : E = (10 i + 7j)
 Vm– 1. The electric flux through
2
1m area in XZ plane is
be 10 N
(c) the force experienced by the above charge will be towards (a) 10 V m (b) 7 V m
+ x-axis (c) 100 V m (d) 49 V m.
(d) a uniform electric field exists in this region along the x-axis. 267. If electrostatic potential varies inversely as square of
[AMU 2002] distance of observation point from source charge, then
259. Two charges, each equal to 1 µC, are placed at the the electric field strength at infinity will be
vertices A and B of a triangle ABC. The product of (a) zero (b) negative
AC and BC is 30 cm2. The sum of the sides AC and (c) imaginary (d) unpredictable.
BC is 10 cm. The potential at C is
CAPACITORS
(a) 3 × 105 V (b) 6 × 105 V
(c) 9 × 105 V (d) 18 mV. 268. A parallel plate capacitor with a dielectric constant 3
260. Eight charged water drops, each with a radius of filling the space between the plates is charged to a
1 mm and a charge of 10–10 C, coalesce to form a potential difference V. The battery is then discon-
single drop. The potential of the big drop is nected and the dielectric slab is withdrawn and re-
(a) 3.6 V (b) 36 V placed by another dielectric slab of dielectric constant
(c) 360 V (d) 3600 V. 2. The ratio of energy stored in the capacitor before
and after replacing the dielectric slab by new one is
261. Given : electric potential, φ = x2 + y2 + z2. The modu-
lus of electric field at (x, y, z) is 3 9
(a) (b)
2 4
(a) 2 (x + y + z) (b) 2 x2 + y2 + z2 4 2
(c) (d) .
(c) 2x + y + z (d) xyz. 9 3
262. A spherical conductor of radius R is charged with Q 269. The combined capacity of the parallel combination of
units of negative charge. The escape velocity of a par- two capacitors is four times their combined capacity
ticle of mass m and charge q from the surface of this when connected in series. This means that their ca-
conductor is pacities are
Qq Qq (a) equal (b) infinite
(a) (b) (c) 0.5 µF and 1 µF (d) 1 µF and 2 µF.
2πε 0 mR 4 πε 0 R
42 COMPREHENSIVE OBJECTIVE PHYSICS

270. Eight small drops, each of radius r and having same (a) decreases (b) does not change
charge q, are combined to form a big drop. The ratio (c) becomes zero (d) increases.
between the potential of the bigger drop and the [Karnataka CET 1999]
smaller drop is 277. Electrical energy stored in the given network is
(a) 1 : 8 (b) 2 : 1
(c) 4 : 1 (d) 8 : 1. C2
271. 1000 small water drops, each of radius r and charge
q, coalesce together to form one spherical drop. The C1
C3
V

potential of the big drop is larger than that of the


smaller drops by a factor of
Fig. 79
(a) 1 (b) 10
(c) 100 (d) 1000. 1 (C1C2 + C2C3 + C3C1) V 2
(a) (C1 + C2 + C3) V2 (b)
272. Two metal plates have potential difference of 300 V 2 2 (C3 + C2 )
and are 0.01 m apart. A charged particle of mass C1 C2 C3 V 2
1.96 × 10 –15 kg is held in equilibrium between the (c)
2 (C1 + C2 + C3 )
plates of the capacitor. Then the electric field is
(a) 3 × 102 V m– 1 (b) 3 V m– 1 (C1 C2 + C2 C3 + C3 C1) V 2
(d) .
2 (C1 + C2 + C3 )
(c) 3 × 104 V m– 1 (d) 3 × 10– 4 V m– 1.
[National Standard Exam. in Physics 1998]
273. In the previous question, the charge on the particle
is : Take g = 10 m s– 2 278. A parallel plate air capacitor of capacitance C0 is con-
nected to a cell of emf V and then disconnected from
(a) 1.6 × 10– 19 C (b) 3.2 × 10– 19 C
it. A dielectric slab of dielectric constant K, which
(c) 12.8 × 10– 19 C (d) 6.5 × 10– 19 C.
can just fill the air gap of the capacitor, is now in-
274. When a potential serted in it.Which of the following is incorrect ?
difference of 103 V is (a) The potential difference between the plates decreases K
applied between A times.
and B, a charge of (b) The energy stored in the capacitor decreases K times.
0.75 mC is stored in
1
the system of capaci- (c) The change in energy is C V2 (K – 1)
2 0
tors. The value of C
is (µ F) Fig. 78 1
C0 V 2
1 FG IJ
(a) 3 (b) 2.5
(d) The change in energy is
2 K H
−1 .
K
1 279. A parallel plate con-
(c) 2 (d) . A
2 denser is filled with two
[Karnataka CET 1999] dielectrics as shown in
A/2 A/2

275. The equivalent capacitance of three capacitors of ca- Fig. 80. Area of each
pacitance C1, C2 and C3 connected in parallel is 12 plate is A m 2 and the d K1 K2
units and the product of C1 , C2 , C3 is 48. When the separation is d metre.
capacitors C1 and C2 are connected in parallel, the
The dielectric constants
equivalent capacitance is 6 units. Then the
capacitances are are K 1 and K 2 respec- B
(a) 1, 5, 6 (b) 1.5, 2.5, 8 tively. The capacitance
Fig. 80
(c) 2, 3, 7 (d) 2, 4, 6. in farad, between A and
[Karnataka CET 1999] B, will be
ε A ε A
276. A parallel plate air capacitor is charged to a potential (a) 0 (K 1 + K 2 ) (b) 0 (K 1 + K 2 )
d 2d
difference V. After disconnecting the battery, the dis-
tance between the plates of the capacitor is increased ε0A ε0A
(c) 2 (K 1 + K 2 ) (d) (K 1 − K 2 ) .
using an insulating handle. As a result, potential dif- d 2d
ference between the plates [Pb. PMT 1999]
ELECTROSTATICS 43

280. Three capacitors, each of capacitance 15 µF, are con- 284. Two parallel plate air filled capacitors, each of ca-
nected in series. This combination is connected to a pacitance C, are joined in series to a battery of emf V.
battery of 100 V. The charge drawn from the battery The space between the plates of one of the capacitors
is is then completely filled up with a uniform dielectric
having dielectric constant K. The quantity of charge
(a) 5 × 10– 4 C (b) 15 × 10– 4 C
which flows through the battery is
(c) 25 × 10– 4 C (d) 30 × 10 – 4 C.
[PCET 1998]
FG IJ
CV K − 1 FG IJ
CV K + 1

281. To obtain 3 µ F capacity from three capacitors of


(a)
H K
2 K+1
(b)
H K
2 K−1

2 µ F each, they will be arranged 2 F K − 1I 2 F K + 1I


(a) All the three in series (c) G J
CV H K + 1K
(d) G J.
CV H K − 1K
(b) All the three in parallel
[UGET 1998]
(c) Two capacitors in series and the third in parallel with the
combination of first two 285. A 40 µ F capacitor in a defibrillator is charged to
3000 V. The energy stored in the capacitor is sent
(d) Two capacitors in parallel and the third in series with the
through the patient during a pulse of duration 2 ms.
combination of first two. [MP PMT/PET 1998] The power delivered to the patient is
282. Parallel plate capacitor is constructed using three (a) 45 kW (b) 90 kW
different dielectric materials as shown in Fig. 81. The (c) 180 kW (d) 360 kW. [AIIMS 2004]
parallel plates, across which a potential difference is
286. A parallel plate capacitor is made by stacking n equally
applied are of area A m2 and separated by a distance
spaced plates connected alternatively. If the capaci-
d metre. The capacitance across AB is
tance between any two adjacent plates is ‘C’ then the
resultant capacitance is
(a) n C (b) C
(c) (n + 1) C (d) (n – 1) C. [AIEEE 2005]
287. Eight drops of mercury of equal radii and possessing
equal charges combine to form a big drop. Then the
capacitance of bigger drop compared to each indi-
vidual drop is
(a) 18 times (b) 14 times
(c) 2 times (d) 32 times.
4
288. Capacitance of a parallel plate capacitor becomes
Fig. 81 3
times its original value if a dielectric slab of thick-
(a) 0
LM
ε A K1
+
K 2K 3 OP (b) 0
LM +
OP
ε A K 1 (K 2 + K 3 )
ness t =
d
is inserted between the plates [d is the
d N
2 K2 + K3 Q d N
2 K 2K 3 Q 2
separation between the plates]. The dielectric con-
L O ε0A L 2 K2 + K3 O
(c) 0 M
ε A 2
+
K 2K 3
P
d N K1 K 1 + K3 Q
(d) M + K 1K 3 PQ .
d NK1
stant of the slab is
(a) 4 (b) 8
[All India PM/PD 1999, Modified] (c) 2 (d) 6. [KCET 2003]
283. A 10 µF capacitor and a 20 µ F capacitor are con- 289. In the circuit as shown in Fig. 82, the effective
nected in series across a 200 V supply line. The charged capacitance between A and B is
capacitors are then disconnected from the line and 4 F
reconnected with their positive plates together and A
negative plates together and no external voltage is
applied. The potential difference across each capaci- 2 F
4 F 2 F
tor is
B
400 800 4 F
(a) V (b) V
9 3
Fig. 82
(c) 400 V (d) 200 V. [MP PET 1997]
44 COMPREHENSIVE OBJECTIVE PHYSICS

(a) 2 µF (b) 3 µF 3
(a) 0 (b) CV 2
(c) 8 µF (d) 4 µF. [KCET 2003] 2
290. A network of four capacitors of
1
capacity equal to C1 = C, C 2 = (c) CV 2 (d) CV 2.
2
2C, C3 = 3C and C4 = 4C are con- C2
ducted to a battery as shown in 296. In Q. 295, the potential of B is
C3 C1
Fig. 83. The ratio of the charges (a) 0 (b) V
on C2 and C4 is : C4 (c) 3V (d) V/3.
3 7
(a) (b) 297. In Q. 295-296, the total electrostatic energy stored in
22 4
both the capacitors before and after the introduction
22 4
(c) (d) . V of the dielectric is
3 7
[All India PM/PD 2005] Fig. 83 5 3
(a) (b)
291. In the electric circuit, ca- 3 5
pacitance of each capaci- 2 9
tor is 1 µ F. The effective (c) (d) .
9 2
capacitance between the
points A and B is (in µ F) [IIT 1983, modified]
2 3 298. The equivalent capacity between A and B of the net-
(a) (b) work shown in Fig. 86 is
3 2
1 (a) 1 µF (b) 2 µF
(c) (d) 6. Fig. 84
6 (c) 3 µF (d) 4 µF.
[Karnataka CET 1998]
292. 125 identical drops, each charged to the same poten-
tial of 50 V, are combined to form a single drop. The
potential of the new drop will be
(a) 50 V (b) 250 V
(c) 500 V (d) 1250 V. [MP PET 1997]
293. What fraction of the energy drawn from the charg-
ing battery is stored in a capacitor ?
(a) 100% (b) 75%
(c) 50% (d) 25%.
[Karnataka CET 2002]
294. Fig. 85 shows two identi- S Fig. 86
cal parallel plate capaci- 299. For the arrangement
tors connected to a bat- of capacitors shown in
tery with the switch S V C C Fig. 87, the equivalent
A B
closed (see Fig. 85). The capacitance between P
total energy stored in the and Q is
capacitors is Fig. 85 (a) 4 µF
1 (b) 10 µF
(a) 0 (b) CV 2
2 (c) 2 µF Fig. 87
(c) CV 2 (d) 2 CV 2. (d) 7 µF.
295. In Q. 294, the switch is now opened and the free space 300. A capacitor when filled with a dielectric K = 3 has
between the plates of the capacitors is filled with di- charge Q0 , voltage V0 and field E0 . If the dielectric is
electric of dielectric constant 3. Now, the energy of A replaced with another one having K = 9, the new
is values of charge, voltage and field will be respectively
ELECTROSTATICS 45
(a) 3Q0, 3V0, 3E0 (b) Q0, 3V0, 3E0
V0 V0 E0
(c) Q0 , , 3E0 (d) Q0 , , .
3 3 3
[Manipal 1998]
301. The area of the plates of a parallel plate condenser is
A and the distance between the plates is 10 mm. There
are two dielectric sheets in it, one of dielectric con-
stant 10 and thickness 6 mm and the other of dielec-
tric constant 5 and thickness 4 mm. The capacity of
the condenser is 8 capacitors
12 2
(a) ε0A (b) ε 0 A A B
35 3
Fig. 88
5000
(c) ε0A (d) 1500 ε0A. [AFMC 1998]
7 (a) 1 µF (b) 2 µF
302. A 10 µ F capacitor is charged to a potential difference (c) 0 µF (d) ∞.
of 50 V and is connected to another uncharged ca- 308. Force of attraction between the plates of a parallel
pacitor in parallel. Now the common potential differ- plate capacitor is
ence becomes 20 volt. The capacitance of second ca-
pacitor is q2 q2
(a) (b)
(a) 10 µF (b) 20 µF 2ε 0 AK Kε 0 A

(c) 30 µF (d) 15 µF. [MP PET 1999] q q2


(c) (d) . [AFMC 1998]
303. The capacity of a parallel plate condenser is 10 µ F 2ε 0 A 2ε 0 A 2 K
without dielectric. Dielectric of constant 2 is used to 309. A 6 µ F capacitor is charged from 10 volt to 20 volt.
fill half the distance between the plates. The new Increase in energy will be
capacitance in µ F is
(a) 18 × 10– 4 joule (b) 9 × 10– 4 joule
(a) 10 (b) 20
(c) 4.5 × 10– 4 joule (d) 9 × 10– 6 joule.
(c) 15 (d) 13.33. [EAMCET 1995]
310. When a parallel plate capacitor is connected to a
304. A capacitor of 20 µF and charged to 500 V is con-
source of constant potential difference,
nected in parallel with another capacitor of 10 µ F
charged to 200 V. The common potential is (a) the whole of the charge drawn from the source is stored in
the capacitor.
(a) 100 V (b) 200 V
(c) 300 V (d) 400 V. (b) the whole of the energy drawn from the source is stored in
the capacitor.
305. A number of condensers, each of capacitance 1 µF
(c) the capacity of the capacitor is decreased.
and each one of which gets punctured if a potential
difference just exceeding 500 V is applied, are (d) the potential difference across the capacitor becomes infi-
provided. Then an arrangement suitable for giving a nite.
capacitance of 2 µ F across which 3000 V may be 311. Fig. 89 shows four plates, each of area A and sepa-
applied requires at least ...... component capacitors. rated from one another by a distance d. The capaci-
(a) 6 (b) 12 tance between A and B is :
(c) 72 (d) 3.
306. A technician has only two capacitors. By using them
singly, in series or in parallel, he is able to obtain the
capacitances of 3 µ F, 4 µ F, 12 µ F and 16 µ F. The
capacitances of the two capacitors are
(a) 4 µF and 12 µF (b) 4 µF and 16 µF
Fig. 89
(c) 6 µF and 10 µF (d) 7 µF and 9 µF.
307. Refer to the infinite network of capacitors shown in ε A 2ε 0 A
(a) 0 (b)
d d
Fig. 88. The capacitance of each capacitor is 1 µ F.
The equivalent capacity between A and B is 3ε 0 A 4ε 0 A
(c) (d) . [BHU 1998]
d d
46 COMPREHENSIVE OBJECTIVE PHYSICS

312. By inserting a plate of dielectric material between (a)


3
and 50 volt (b)
7
and 30 volt
the plates of a parallel plate capacitor, the energy is 7 3
decreased five times. The dielectric constant of the 3 7
(c) and 30 volt (d) and 0 volt.
material is 7 3
[MP PMT 1991]
1 1
(a) (b) 318. A dielectric material
25 5
of dielectric constant
(c) 5 (d) 25. [MP PET 1990]
K is introduced in
313. A condenser having a capacity 2.0 microfarad is half the length of a
charged to 200 volt and then the plates of the capaci- parallel plate capaci-
tor are connected to a resistance wire. The heat pro- tor. If area of the
duced will be plates of the capacitor Fig. 91
(a) 4 × 104 joule (b) 4 × 1010 joule is A and the plate separation is d, then the capaci-
(c) 4 × 10 – 2 joule (d) 2 × 10 – 2 joule. tance of the arrangement is
[Karnataka CET 1992] 2 ε0A ε A (1 + K)
(a) (b) 0
314. Two capacitors of capacities 6 µ F and 3 µ F, charged d d
to a potential of 12 V each, are connected to each ε A (1 + K) 3ε 0 A
other with the positive plate of each joined to the (c) 0 (d) .
2d 2d
negative plate of the other. The potential difference 319. Keeping the charge on a capacitor unchanged, we
across each will now be can decrease the energy of the capacitor by
(a) 2 V (b) 13 V (a) increasing the dielectric constant of the medium between
(c) 4 V (d) 16 V. [AIIMS 2001] the plates.
(b) decreasing the dielectric constant of the medium between
315. A dielectric slab of length l, width b, thickness d and
the plates.
dielectric constants K fills all the space inside a par-
(c) making dielectric constant zero.
allel plate capacitor. At t = 0, the slab begins to be
(d) none of the above.
pulled out slowly with speed v. At time t, the capac-
ity of the capacitor is 320. The combined capacitance of the arrangement shown
in Fig. 92 is
ε b ε l
(a) 0 [Kl − (K − 1) vt] (b) 0 [Kl + K vt]
d b
ε0d ε 0b
(c) [(K + 1) vt] (d) [ l − (K + 1) vt] .
b d
[DCE 2002]
316. Two condensers C1
and C2 in a circuit
are joined as shown
Fig. 92
in Fig. 90. The po-
tential of point A is Fig. 90 8
(a) 1 µF (b) µF
V1 and that of B is V2 . 11
The potential of point D will be 30
(c) µF (d) 4 µF
1 C2 V1 + C1V2 11
(a) (V1 + V 2 ) (b) (e) 11 µF.
2 C1 + C2
[London Schools Exam. Deptt.]
C1V1 + C2 V2 C2 V1 − C1V2
(c) (d) . 321. A 500 µ F capacitor is charged at the steady rate of
C1 + C2 C1 + C2
100 microcoulomb per second. How long will it take
[MP PMT 1997] to raise the potential difference between the plates of
317. A condenser having a capacity of 6 µ F is charged to the capacitor to 10 volt ?
100 volt and is then joined to an uncharged condenser (a) 5 s (b) 10 s
of 14 µ F and then removed. The ratio of the charges
(c) 50 s (d) 100 s. [DCE 1998]
on 6 µ F and 14 µ F and the potential of 6 µ F will be
ELECTROSTATICS 47

322. A variable capacitor and an electroscope are connected 328. n identical droplets are charged to V volt each. If
in parallel to a battery. The reading of the electro- they coalesce to form a single large drop, then the
scope would be decreased by potential of the large drop will be
1. Increasing the area of overlap of the plates. V
(a) Vn (b)
2. Placing a block of paraffin wax between the plates. n
(c) Vn1/3 (d) Vn2/3.
3. Decreasing the distance between the plates.
[Karnataka CET 2000, 1995 ; MP PMT 2001]
4. Decreasing the battery potential.
329. The distance between the plates of a parallel plate
State :
condenser is 4 mm and potential difference is 60 volt.
(a) If only (1), (2) and (3) are correct If the distance between the plates is increased to 12
(b) If only (1) and (3) are correct mm, then
(c) If only (2) and (4) are correct (a) the potential difference will remain unchanged
(d) If only (4) is correct. (b) the potential difference will become 20 volt

323. Two identical parallel plate capacitors are placed in (c) the potential difference of the condenser will become 180
volt
series and connected to a constant voltage source of
(d) the charge on condenser will reduce to one-third.
V volt. If one of the capacitors is completely immersed
in a liquid of dielectric constant K, then the potential 330. Two capacitors of capacitances C and 2C, are charged
difference between the plates of the other capacitor to potential differences V and 2V respectively. If the
will change to two positive plates are connected together and the
two negative plates are connected together, then this
K K +1
(a) V (b) V system of capacitors
K+1 K
(a) gains charge but loses energy
2K K+1
(c) V (d) V. (b) gains energy but loses charge
K +1 2K
(c) loses both energy and charge
324. Two capacitors, each having capacitance C and break-
(d) loses charge but energy remains constant
down voltage V, are joined in series. The capacitance
and breakdown voltage of the combination are (e) loses energy but charge remains constant.
respectively 331. An isolated parallel-plate capacitor of plate area A,
(a) 2 C and V (b) 2 C and 2 V plate separation d and capacitance C (= ε A/d) is found
C V C to lose charge slowly by conduction through the
(c) and (d) and 2 V.
2 2 2 dielectric, which has resistivity ρ. What is the
325. If the capacitors in the previous question are joined resistance of the dielectric ?
in parallel, the capacitance and the breakdown volt- ρε
(a) (b) ρCε
age of the combination will be C
εC ρC
(a) 2 C and 2 V (b) C and 2 V (c) (d)
ρ ε
(c) 2 C and V (d) C and V.
ρ
326. Force acting upon a charged particle kept between (e) .

the plates of a charged capacitor is F. If one of the 332. Two condensers A and B of capacities 0.3 µ F and
plates of the capacitor is removed, then the force act- 0.6 µ F respectively are connected in series. The com-
ing on the same particle will be bination is connected across a potential of 6 volt. The
(a) F (b) 0 ratio of energies stored by the condensers A and B
will be
F
(c) 2F (d) . [MP PMT 1991] (a) 2 (b) 4
2
1 1
327. Eight mercury droplets, each having a radius of (c) (d) .
2 4
10– 3 m and a charge of 0.066 pC each, merge to form
one drop. The potential of the drop is 333. Three capacitors, each of capacitance 1 µ F, are con-
nected in parallel. To this combination, a fourth ca-
(a) 1.2 V (b) 2.4 V
pacitor of capacitance 1 µ F is connected in series.
(c) 3.6 V (d) 4.8 V. The resultant capacitance of the system is
48 COMPREHENSIVE OBJECTIVE PHYSICS

(a) 2 µF (b) 4 µF (a) 4 : 3 (b) 3 : 4


4 3 (c) 2 : 3 (d) 3 : 2.
(c) µF (d) µF.
3 4 340. The capacity and the energy stored in a parallel plate
334. A parallel plate capacitor with air as medium between condenser with air between its plates are respectively
the plates has a capacitance of 10 µ F. The area of C0 and W0 . If air is replaced by glass (dielectric con-
capacitor is divided into two equal halves and filled stant = 5) between the plates, the capacity of the plates
with two media having dielectric constants K1 = 2 and the energy stored in it will respectively be
and K2 = 4. The capacitance of the system will now be W0
(a) 5 C0, 5W0 (b) 5 C0,
(a) 40 µF (b) 30 µF 5
(c) 20 µF (d) 10 µF.
C0 C0 W0
(c) , 5W0 (d) , .
335. A photographic flash unit consists of a xenon-filled 5 5 5
tube energised by the discharge of a capacitor previ-
[MP PET 1990]
ously charged by a 1000 V source. The average power
341. The two metallic plates of radius r are placed at a
delivered to the flash tube is 1000 W and the flash
distance d apart and its capacity is C. If a plate of
lasts 0.040 s. The capacitance of the capacitor can be
radius r/2 and thickness d of dielectric constant 6 is
estimated as
placed between the plates of the condenser, then its
(a) 40 × 10– 6 F (b) 80 × 10– 6 F
capacity will be
(c) 160 × 10– 6 F (d) 80 × 10– 3 F
(a) 7C/2 (b) 3C/7
(e) 160 × 10 –3
F.
(c) 7C/3 (d) 9C/4. [CET 2002]
336. There are 8 identical capacitors. The equivalent ca-
342. In the circuit shown in
pacitance when they are connected in series is C. The
Fig. 94, capacitors A and
equivalent capacitance when they are connected in
B have identical geom- 10 V
parallel is
etry, but a material of di-
(a) 8 C (b) 64 C electric constant 3 is
C C present between the plates A B
(c) (d) . [CMC LDH 2002]
8 64 of B. The potential differ-
337. Two metallic charged spheres whose radii are 20 cm ences across A and B are,
Fig. 94
and 10 cm respectively, have each 150 microcoulomb respectively
positive charge. The common potential, after they (a) 2.5 V, 7.5 V (b) 2 V, 8 V
are connected by a conducting wire, is (c) 7.5 V, 2.5 V (d) 8 V, 2 V.
(a) 9 × 106 V (b) 4.5 × 106 V
[National Standard Exam. in Physics 1992]
(c) 1.8 × 107 V (d) 13.5 × 106 V. [AMU 2002]
343. The potential difference across the plates of a con-
338. A capacitor is charged to a voltage V and then dis-
denser is 60 volt and its capacity is 12 µ F. Another
charged through a small dc motor. As the capacitor
condenser of 24 µ F is connected with it in series and
discharges, the motor raises a mass through a height
its free plate is earthed. Then the potential difference
h. The experiment is repeated for several values of V.
across 12 µ F capacitor will be
A constant fraction of the capacitor energy is con-
(a) 60 volt (b) 40 volt
verted to gain of gravitational potential energy.
(c) 20 volt (d) 90 volt.
Which graph would be expected to give a straight
line ? [All India PM/PD 2002]
(a) h against V 2 (b) h against V 344. If the area of each plate of a parallel plate condenser
(c) h against (d) h against 1 / V .
is A, the distance between the plates is d, the dielec-
V
tric medium between the plates is K and the charge
339. Four capacitors, each of ca-
on the plate is q, then the intensity of the electric
pacity 3 µ F, are connected
field between the two plates will be
as shown in Fig. 93. The ra-
tio of equivalent capacitance q Kq
(a) (b)
between A and B and be- K Aε 0 Aε0
tween A and C will be ε0q
K qε 0
(c) (d) . [Pb. PMT 2002]
Fig. 93 Ad KAd
ELECTROSTATICS 49
345. To form a composite 16 µ F, 1000 V capacitor from a 352. Two condensers of capacities 4 µ F and 6 µ F are con-
supply of identical capacitors marked 8 µ F, 250 V, nected in parallel. This combination is now connected
we require a minimum number of capacitors : in series to a third condenser. If the resultant capaci-
(a) 40 (b) 32 2
tance becomes 6 µ F , the capacitance of the third
(c) 8 [MP PET 1996]
(d) 2. 3
346. A parallel plate capacitor has a capacity C. The sepa- condenser is
ration between the plates is doubled and a dielectric (a) 2 µF (b) 3 µF
medium is introduced between the plates. If the ca- (c) 4 µF (d) 20 µF. [CET 2001]
pacity now becomes 2C, the dielectric constant of the 353. In the network shown
medium is in Fig. 96, all the ca-
(a) 2 (b) 1 pacitors are identical.
(c) 4 (d) 8. [CET 1996] When an emf is ap-
347. A parallel plate capacitor has separation t and ca- plied across A, B one
pacitance 100 pF. If a metallic foil of thickness t/3 is of the capacitors re-
introduced between the plates, the capacitance would mains uncharged.
become (in pF units) That capacitor is Fig. 96
(a) 3 × 100 (b) (3/2) × 100 (a) C1 (b) C2
(c) 100 (d) (2/3) × 100. (c) C5 (d) C6.
[National Standard Exam. in Physics 1990 ; [National Standard Exam. in Physics 1993]
Kerala PMT 2003]
354. The capacity of a condenser is 4 × 10–6 farad and its
348. The heat generated when condenser of 100 microfarad potential is 100 volt. The energy released on discharg-
capacity and charged to 200 volt is discharged ing it fully will be
through a 2 ohm resistance, is (a) 0.02 joule (b) 0.04 joule
(a) 2 J (b) 0.002 J (c) 0.025 joule (d) 0.05 joule.
(c) 12 J (d) 2 × 104 J. [MP PET 1996] [Pb. PMT 2001]
349. The outer sphere of a spherical air capacitor is earthed. 355. In the circuit shown in
For increasing its capacitance, Fig. 97, the steady state
(a) vacuum is created between two spheres voltage across capacitor C
is a fraction f of the bat-
(b) dielectric material is filled between the two spheres
tery emf. The fraction is de-
(c) the space between two spheres is increased cided by
(d) the earthing of the outer sphere is removed. (a) R1 only
[MP PET 1991] (b) R1 and R3 only
350. A 10 µ F capacitor is charged by a battery of emf 100 (c) R1 and R2 only Fig. 97
volt. The energy drawn from the battery, and the (d) R1, R2 and R3.
energy stored in the capacitor, are respectively [National Standard Exam. in Physics 1993]
(a) 0.10 J and 0.05 J (b) 0.05 J and 0 J 356. A 4 µ F condenser is connected in parallel to another
(c) 1.0 mJ and 0.5 mJ (d) 0.05 J and 0.05 mJ. condenser of 8 µ F. Both the condensers are then con-
[National Standard Exam. in Physics 1991] nected in series with a 12 µ F condenser and charged
to 20 volt. The charge on the plates of 4 µ F con-
351. The capacities of the
denser is
capacitors are shown
in the Fig. 95. The (a) 3.3 µC (b) 40 µC
equivalent capacitance (c) 80 µC (d) 240 µC. [MP PET 1989]
between the points A 357. The equivalent ca-
and B and the charge pacity in the sys-
on the 6 µ F capacitor tem of capacitors
will be
shown in Fig. 98
(a) 27 µF, 540 µC
Fig. 95 will be
(b) 15 µF, 270 µC
(a) 1 µF (b) 2 µF
(c) 6 µF, 180 µC (d) 15 µF, 90 µC.
(c) 1.5 µF (d) 3 µF.
[RPMT 2002] Fig. 98
50 COMPREHENSIVE OBJECTIVE PHYSICS

358. Between the plates of a parallel plate condenser, a (a) 5 µC (b) 10 µC


plate of thickness t1 and dielectric constant K 1 is (c) 12 µC (d) 15 µC. [MP PMT 1996]
placed. In the rest of the space, there is another plate 363. Two dielectric slabs of
of thickness t2 and dielectric constant K2 . The poten- dielectric constants K 1
tial difference across the condenser will be and K2 have been filled
Q F t1 + t2 I F
ε Q t1 t I in between the plates of
(a) GH K 1 K 2 JK
Aε 0
(b) 0 GH+ 2
A K 1 K2 JK a capacitor as shown in
Fig. 101. What will be
Q F K1 K2 I ε Q
(c) G + t2 JK
Aε 0 H t1
(d) 0 (K 1t1 + K 2t2 ) .
A
the capacitance of the
capacitor ? Fig. 101

359. Two identical capacitors are joined in parallel, charged 2ε 0 A F


(b) 2ε 0 A K 1 + K 2
I
to a potential V and then separated and then con-
(a)
d
(K 1 + K 2 )
d
GH
K1 × K2
JK
nected in series i.e., the positive plate of one is con-
d FK1 K2 I 2ε 0 A F K 1 K 2 I
nected to negative of the other. Which of the follow- (c) GH
2ε 0 A K 1 + K 2
JK (d) G
d H K1 + K2 K
J.
ing is correct ?
(a) The charges on the free plates connected together are [MNR 1985]
destroyed. 364. Three capacitors of capacitances 3 µ F, 9 µ F and
(b) The charges on the free plates are enhanced. 18 µ F are connected once in series and another time
(c) The energy stored in the system increases. in parallel. The ratio of equivalent capacitance in the
(d) The potential difference in the free plates is 2V. FC I
[All India PM/PD 1999]
two cases GH C JK
s
p
will be

360. A capacitor of capac-


S (a) 1 : 15 (b) 15 : 1
ity C1 is charged to
the potential of V0. (c) 1 : 1 (d) 1 : 3. [Manipal 1997]
After disconnecting C1,V0 C2 365. A spherical condenser has inner and outer spheres of
from the battery, it radii a and b respectively. The space between the two
is connected with a is filled with air. The difference between the capaci-
Fig. 99 ties of two condensers formed when outer sphere is
capacitor of capacity
C2 as shown in the Fig. 99. The ratio of energies be- earthed and when inner sphere is earthed will be
fore and after the closing of switch S will be (a) zero (b) 4πε0a
(a) (C1 + C2)/C1 (b) C1/(C1 + C2)
FG b IJ .
(c) C1C2 (d) C1/C2. [MAHE 2002] (c) 4πε0b (d) 4 πε 0 a
H b − aK
361. In the connections shown in Fig. 100, the equivalent
[MP PET 1996]
capacity between A and B will be
366. Four metallic plates
each with a surface area
A of one side are placed
at a distance ‘d’ from
each other. The plates
are connected as shown
in the Fig. 102. Then
the capacitance of the Fig. 102
system between ‘a’ and
Fig. 100
‘b’ is
(a) 10.8 µF (b) 69 µF 3ε 0 A 2ε 0 A
(a) (b)
(c) 15 µF (d) 10 µF. d d
362. Three capacitors of 2.0 µ F, 3.0 µ F and 6.0 µ F are 2ε 0 A 3ε 0 A
(c) (d) . [Manipal 1996]
connected in series to a 10 V source. The charge on 3d 2d
the 3.0 µ F capacitor is
ELECTROSTATICS 51

367. A capacitor of capacity 100 µµ F is charged to 100 372. Two identical parallel plate capacitors are connected
volt. After disconnecting it from battery, it is con- in series to a battery of 100 V. A dielectric slab of
nected in parallel with another condenser. If the fi- dielectric constant 4.0 is inserted between the plates
nal potential is 30 volt, then the capacity of the sec- of second capacitor. The potential difference across
ond condenser will be nearly the first and second capacitors will now be respec-
(a) 233 µF (b) 233 µµF tively
(c) 233 F (d) 233 M F. (a) 50 V, 50 V (b) 80 V, 20 V

368. Two condensers of capacities 2C and C are joined in (c) 20 V, 80 V (d) 75 V, 25 V.


parallel and charged upto potential V. The battery is [MP PMT 1992]
removed and the condenser of capacity C is filled 373. Four condensers are
completely with a medium of dielectric constant K. joined as shown in
The potential difference across the capacitors will now Fig. 104. The capacity
be of each is 8 µ F. The
equivalent capacity be-
3V 3V Fig. 104
(a) (b) tween the points A and
K +2 K B will be
V V (a) 32 µF (b) 2 µF
(c) (d) . [IIT 1988]
K+2 K (c) 8 µF (d) 16 µF. [AFMC 1998]
369. The plates of a parallel plate condenser are pulled 374. The plates of parallel plate capacitor are charged upto
apart with a velocity v. If at any instant their mutual 100 V. A 2 mm thick plate is inserted between the
distance of separation is d, then the magnitude of plates. Then to maintain the same potential differ-
the time rate of change of capacity depends on d as ence, the distance between the plates is increased by
follows 1.6 mm. The dielectric constant of the plate is
(a) 5 (b) 1.25
1 1
(a) (b) (c) 4 (d) 2.5. [MP PMT 1991]
d d2
375. The diameter of each plate of an air capacitor is 4
(c) d2 (d) d. [MP PET 1991]
cm. To make the capacity of this plate capacitor equal
370. In the circuit shown in Fig. 103, the potential differ- to that of 20 cm diameter sphere, the distance be-
ence across the 4.5 µ F capacitor is tween the plates will be
(a) 4 × 10 –3 m (b) 1 × 10 –3 m
3 µF
(c) 1 cm (d) 1 × 10 –3 cm.
4.5 µF [MP PET 1996]
376. Four plates of equal
area A are separated
6 µF by equal distances d
12 V
and are arranged as
shown in Fig. 105.
Fig. 105
Fig. 103 The equivalent capac-
ity is
8
(a) V (b) 4 V
3 2ε 0 A 3ε 0 A
(a) (b)
(c) 6 V (d) 8 V. [MP PET 1992] d d
371. The capacitance of an air capacitor is 15 µ F and the 3ε 0 A ε A
(c) (d) 0 . [AMIU 1997]
separation between the parallel plates is 6 mm. A 2d d
copper plate of 3 mm thickness is introduced sym- 377. The respective radii of the two spheres of a spherical
metrically between the plates. The capacitance now condenser are 12 cm and 9 cm. The dielectric con-
becomes stant of the medium between them is 6. The capacity
(a) 5 µF (b) 7.5 µF of the condenser will be
(c) 22.5 µF (d) 30 µF. [MP PMT 1995]
52 COMPREHENSIVE OBJECTIVE PHYSICS

(a) 240 pF (b) 240 µF (a) 24 µF (b) 18 µF


(c) 240 F (d) None of these. (c) 12 µF (d) 6 µF.
[EAMCET 2001] [All India PM/PD 1999]
378. If three capacitors, each of capacity 1 µ F, are con- 383. Four capacitors are connected in a circuit as shown
nected in such a way that the resultant capacity is in the Fig. 107. The effective capacitance in µ F be-
tween points A and B will be
1.5 µ F, then
(a) all the three are connected in series 2 µF 12 µF

(b) all the three are connected in parallel


(c) two of them are in parallel and connected in series to the
third A
2 µF
B

(d) two of them are in series and then connected in parallel to


the third. [DPMT 2001]
379. A 0.2 F capacitor is charged to 600 V by a battery.
2 µF
On removing the battery, it is connected in parallel
with another parallel plate condenser of 1 F. The po- Fig. 107
tential decreases to 28
(a) (b) 4
(a) 100 V (b) 120 V 9
(c) 300 V (d) 600 V. (c) 5 (d) 18. [MP PET 1996]
380. The capacity of a condenser in which a dielectric of 384. The effective capacity between A and B of the given
dielectric constant 5 has been used, is C. If the di- network is
electric is replaced by another with dielectric con-
stant 20, the capacity will become
C
(a) (b) 4C
4
C
(c) (d) 2C. [MP PMT 1996]
2
381. In a parallel plate capacitor, the separation between
the plates is 3 mm with air between them. Now a 1
mm thick layer of a material of dielectric constant 2
is introduced between the plates due to which the
capacity increases. In order to bring its capacity to Fig. 108
the original value, the separation between the plates
(a) 3C (b) 2C
must be made
C
(a) 1.5 mm (b) 2.5 mm (c) C (d) . [Pb. PMT 1998]
3
(c) 3.5 mm (d) 4.5 mm.
385. Two capacitors of 3 pF and 6 pF are connected in
[Bharati Vidyapeeth 2002] series and a potential difference of 5000 V is applied
382. The effective capacitance between points X and Y is across the combination. They are then disconnected
and reconnected in parallel. The potential between
C1 = 6 µF the plates is nearly
(a) 2250 V (b) 1111 V
C3 = 6 µF C2 = 6 µF (c) 2.25 × 10 V 6
(d) 1.1 × 106 V.

Y
[MP PMT 1992]
X
C5 = 20 µF 386. The distance between the circular plates of a parallel
plate condenser 40 mm in diameter, in order to have
C4 = 6 µF
same capacity as a sphere of radius 1 metre is
(a) 0.01 mm (b) 0.1 mm
Fig. 106 (c) 1.0 mm (d) 10 mm.
[MP PET 1992]
ELECTROSTATICS 53

387. A 2 µ F capacitance has a potential difference of 200


V across its two terminals. It is disconnected from
battery and then another uncharged capacitor is con-
nected in parallel to it. Now, the potential difference
becomes 20 volt. Then the capacity of another ca-
pacitor will be
Fig. 110
(a) 2 µF (b) 4 µF (a) C (b) 2C
(c) 18 µF (d) 16 µF. (c) C/2 (d) C/3. [DCE 1996]
[CMC LDH 2002] 393. A capacitor of capacitance 5 µ F is connected as shown
388. The force between the plates of a parallel plate ca- in the Fig. 111. The internal resistance of the cell is
pacitor of capacitance C and distance of separation 0.5 Ω. The amount of charge on the capacitor plates
of the plates d with a potential difference V between is
the plates, is
CV 2 C2 V 2
(a) (b)
2d 2d2

C2 V 2 V 2d
(c) (d) . [MP PMT 1999]
d2 C

389. Two identical charged spherical drops, each of ca-


pacitance C, merge to form a single drop. The result- Fig. 111
ant capacitance is (a) 0 µC (b) 5 µC
(a) equal to 2C (c) 10 µC (d) 25 µC. [MP PET 1997]
(b) greater than 2C 394. Fig. 112 (a) shows two capacitors connected in series
(c) less than 2C but greater than C and joined to a battery. The graph in Fig. 112 (b)
shows the variation in potential as one moves from
(d) less than C. [AFMC 1993]
left to right on the branch containing the capacitors,
390. An air capacitor of capacity C = 10 µ F is connected if
to a constant voltage battery of 12 V. Now the space
between the plates is filled with a liquid of dielectric V
constant 5. The charge that flows now from battery
to the capacitor is
(a) 120 µC (b) 600 µC
(c) 480 µC (d) 24 µC. [MP PMT 1997] x
C1 C2
391. For the circuit shown in Fig. 109, which of the fol- (a) (b)
lowing statements is true ?
Fig. 112
(a) C1 > C2 (b) C1 = C2
(c) C1 < C2
(d) The information is not sufficient to decide the relation
between C1 and C2. [MP PMT 1999]
Fig. 109
395. What is the equivalent capacitance between A and B
(a) With S1 closed, V1 = 15 V, V2 = 20 V in Fig. 113 (all capacities are in farad) ?
(b) With S3 closed, V1 = V2 = 25 V
(c) With S1 and S2 closed, V1 = V2 = 0
(d) With S1 and S3 closed, V1 = 30 V, V2 = 20 V. [IIT 1999]
392. The equivalent capacitance of the combination shown
in Fig. 110 is
Fig. 113
54 COMPREHENSIVE OBJECTIVE PHYSICS

13 48
(a) F (b) F
18 13
1 240
(c) F (d) F. [BHU 1997]
31 71
396. In the circuit shown in Fig. 114, the equivalent ca-
pacitance between the points A and B is

3 µF
Fig. 116
A B (a) 60 V (b) 45 V
2 µF 15 µF 6 µF
(c) 40 V (d) 30 V. [MQIMS 2002]
4 µF
400. Four metallic plates, each with surface area of one
side A, are placed at a distance d from each other.
Fig. 114 The plates are connected as shown in Fig. 117. Then
10 15 the capacitance of the arrangement between A and B
(a) µF (b) µF is
3 14
2 25
(c) µF (d) µF . [DCE 1994]
5 9
397. Two identical capacitors
are connected as shown in
Fig. 115. Charge on each I
capacitor is q0 . A dielec-
2V0
tric slab is now introduced K
between the plates of one II
of the capacitors so as to
fill the gap, the battery re- Fig. 117
maining connected. The Fig. 115
3ε 0 A 2ε 0 A
charge on each capacitor will now be (a) (b)
d d
2 q0 q0 2 ε0 A 3 ε0 A
(c) (d) .
(a) (b) 3 d 2 d
1 1
1+ 1+ 401. A rectangular parallel-plate capacitor has a dielec-
K K
tric slab which partially fills the space between the
2q0 q0 plates as shown in Fig. 118. The energy of the capaci-
(c) (d) .
1+ K 1+ K tor with charge Q is
[All India PM/PD 1998]
398. A box is known to contain three identical capacitors b
wired together in a circuit containing no other com-
ponents. Two wires lead from this circuit to the out-
side of the box and the measured capacitance between d

these wires is 30 µ F. Which one of following could be


a
the correct capacitance of each capacitor ?
l
(a) 15 µF (b) 20 µF
(c) 40 µF (d) 60 µF Fig. 118
(e) 100 µF. [London Schools Exam. Deptt.] (a) zero (b) infinite
399. In the circuit shown in Fig. 116, a potential differ-
Q2 d 1
ence of 60 V is applied between a and b. The potential (c) (d) None of these.
2ε 0b Kl − (K − 1) a
difference between the points c and d is
ELECTROSTATICS 55

402. In the circuit shown in Fig. 3 mF 6 mF


a b
119, the potential difference C C C C/2
across the 3 µ F capacitor is 2 mF
V. The value of V is
C C C
(a) 20 V (b) 40 V 60 V
(c) 45 V (d) 60 V.
A B
[Haryana PMT 2000] Fig. 119 C C C

403. Refer to network shown in Fig. 120. The effective Fig. 120
capacitance between a and b is C
(a) C (b)
2
C C
(c) (d) .
3 4

Answers (Set II)


180. (b) 181. (c) 182. (b) 183. (c) 184. (b) 185. (d) 186. (b) 187. (b)
188. (b) 189. (c) 190. (d) 191. (b) 192. (b) 193. (b) 194. (c) 195. (b)
196. (b) 197. (b) 198. (d) 199. (b) 200. (a) 201. (d) 202. (d) 203. (c)
204. (d) 205. (d) 206. (d) 207. (c) 208. (a) 209. (a) 210. (b) 211. (d)
212. (a) 213. (c) 214. (a) 215. (b) 216. (c) 217. (b) 218. (a) 219. (a)
220. (c) 221. (c) 222. (d) 223. (b) 224. (a) 225. (c) 226. (a) 227. (a)
228. (b) 229. (b) 230. (a) 231. (d) 232. (a) 233. (c) 234. (a) 235. (a)
236. (a) 237. (c) 238. (c) 239. (a) 240. (d) 241. (a) 242. (d) 243. (a)
244. (c) 245. (a) 246. (c) 247. (b) 248. (a) 249. (b) 250. (d) 251. (d)
252. (c) 253. (c) 254. (d) 255. (b) 256. (a) 257. (c) 258. (d) 259. (a)
260. (d) 261. (b) 262. (a) 263. (c) 264. (b) 265. (c) 266. (b) 267. (a)

268. (d) 269. (a) 270. (c) 271. (c) 272. (c) 273. (d) 274. (c) 275. (d)
276. (d) 277. (b) 278. (c) 279. (b) 280. (a) 281. (c) 282. (a) 283. (a)
284. (a) 285. (b) 286. (d) 287. (c) 288. (c) 289. (d) 290. (a) 291. (a)
292. (d) 293. (c) 294. (c) 295. (b) 296. (d) 297. (b) 298. (a) 299. (a)
300. (d) 301. (c) 302. (d) 303. (d) 304. (d) 305. (c) 306. (a) 307. (b)
308. (b) 309. (b) 310. (a) 311. (c) 312. (c) 313. (c) 314. (c) 315. (a)
316. (c) 317. (c) 318. (c) 319. (a) 320. (d) 321. (c) 322. (d) 323. (a)
324. (d) 325. (c) 326. (d) 327. (b) 328. (d) 329. (c) 330. (e) 331. (a)
332. (a) 333. (d) 334. (b) 335. (b) 336. (b) 337. (a) 338. (a) 339. (a)
340. (b) 341. (d) 342. (c) 343. (a) 344. (a) 345. (b) 346. (c) 347. (b)
348. (a) 349. (b) 350. (a) 351. (c) 352. (d) 353. (c) 354. (a) 355. (c)
356. (b) 357. (a) 358. (a) 359. (d) 360. (a) 361. (d) 362. (b) 363. (d)
364. (a) 365. (c) 366. (d) 367. (b) 368. (a) 369. (b) 370. (d) 371. (d)
372. (b) 373. (a) 374. (a) 375. (b) 376. (a) 377. (a) 378. (d) 379. (a)
380. (b) 381. (c) 382. (d) 383. (c) 384. (a) 385. (b) 386. (b) 387. (c)
388. (a) 389. (c) 390. (c) 391. (d) 392. (b) 393. (c) 394. (c) 395. (d)
396. (a) 397. (a) 398. (b) 399. (d) 400. (c) 401. (c) 402. (b) 403. (c)
56 COMPREHENSIVE OBJECTIVE PHYSICS

Solutions (Set II)

q1q2 Consideration of directions of Fe and mg leads us to


180. Fa = 2 the conclusion that q is negative.
r
1 q1q2 189. Number of electrons, n
Fm =
K r2 6 × 1023 1 6 × 1018
Fa K = × 10 × 6 =
= 63.5 10 63.5
Fm 1
q1q2 qq 6 × 1018 × 1.6 × 10−19
r q= C
181. = 1 22 or K r′2 = r2 or r′ = 63.5
r2 K r′ K
or q = 1.5 × 10–2 C
1 q1q2
182. F=
4 π ε0 r2 9 × 109 × 1.5 × 10−2 × 1.5 × 10−2
F= N
[ q1q2 ] [ I2 T 2 ] FG 10 IJ 2
[ε0] = 2 =
[F r ] [ML3 T –2 ]
= [M–1L–3T4I2]
H 100 K
1 × 10 −6 4 × 10 −6 1 2 = 2 × 108 N.
183. 9 × 109 = 9 × 109 or =
r2 (r + 1)2 r r+1 190. Following arguments decide in favour of (d).
or 2r = r + 1 or r = 1 m. (i) Electric field at mid-point is zero.
(ii) The direction of electric field to the left of mid-
point is opposite to the direction of electric field to
the right of the mid-point.
(iii) E → ∞ when x → 0 and x → r.
191. q = 2 × 10–6 C
2l = 0.03 m
Fig. 121
E = 2 × 105 N C–1
184. eE = mg τ = pE
mg 1.67 × 10−27 × 9.8 = q × 2l × E
or E= = V m–1
e 1.6 × 10−19 = 2 × 10–6 × 0.03 × 2 × 105 N m
= 1.02 × 10–7 V m–1 = 12 × 10–3 N m
185. Following two arguments shall decide the right choice 192. τ = pE sin (90° – θ) = pE cos θ
(i) No electric lines of force can be present inside the So, positive cosine curve is the right choice.
metallic solid sphere. 1 Qmax.
(ii) Electric lines of force should meet the conductor 193. Emax. =
4π ε0 R 2
normally.
186. F ∝ q1(q – q1) 9 × 109 × Q max.
3 × 106 =
F will be maximum if q1(q – q1) is maximum. 2.5 × 2.5
or q1 = q – q1
or q = 2q1 3 × 106 × 2.5 × 2.5
or Qmax. = C
9 × 10 9
q 2
or = = 2 × 10–3 C.
q1 1
194. In Fig. 60 (a), if ‘– q’ is given a small upward displace-
q1q2 ment, then the upward electrostatic force shall domi-
187. mg =
r2 nate the force of gravity.
100q So, (a) and (b) are eliminated.
0.05 × 980 =
100 In Fig. 60 (b), if ‘– q’ is displaced slightly up, then the
q = 49 statcoulomb force of gravity shall dominate electrostatic force. So,
ELECTROSTATICS 57
the charge ‘– q’ shall come back to the original posi-
1 Q2
tion. FB = ,
4 π ε 0 a2
Again, if ‘– q’ is displaced slightly down, then the elec-
trostatic force shall dominate the force of gravity con- 1 Q2
FD =
sequently, charge ‘– q’ shall get back to its original 4 π ε 0 a2
position. Thus, in Fig. 60 (b), q is in stable equilib-
rium. This favours (c) and eliminates (d). 1 2 Q2
FBD = 2 FB =
195. No work will be done corresponding to displacement 4π ε0 a2
along y-axis and z-axis.
1 Q2 1 Q2
Also the work done does not depend upon path. Net FC = =
displacement in the direction of electric field = – a 4 π ε0 ( 2 a)2 4 π ε0 2a2
Work done = q E(– a) = – q Ea. 1 qQ 1 2qQ
FO = =
196. F = qE 4π ε0 F 2a I 2
4 π ε 0 a2

a=
F qE
=
GH 2 JK
m m
For equilibrium, FBD + FC = FO
q Et
v = at = 1 2 Q2 1 Q2 1 2 qQ
m 2
+ 2
=
4π ε0 a 4 π ε 0 2a 4 π ε 0 a2
Kinetic energy
Q = 2q or 2q = Q
1 q Et LM OP 2
E2q2 t2 .
or 2Q+
2 2
(2 2 + 1)
= m
2 m N Q =
2m
or q=
Q
(1 + 2 2 ) .
197. Consider the equilib- A 4
rium of charge Q at A. Q q Q
4Qq 4 q2
2 200. = − or Q = – q.
Qq Q d l2 l2
=−
d2 4d2
2d
Q
or q= − Fig. 122
4
198. From considerations of Y
symmetry, we find that
the net force on Q is di- (0, a) – q
rected towards O. As Q
approaches O, this force Q
Fig. 125
O
increases. As Q moves to (2a, 0)
the left of O, the force is 201. If the charges are arranged according to the option
again directed towards O. (0, – a) – q (d), the electric fields due to P and S and due to Q and
So, the motion is not T add to zero, while due to U and R will be added up.
Y′
SHM. Clearly, the motion 202. Electric field intensity at any point inside the charged
is oscillatory in nature. Fig. 123 sphere is zero.
199. Consider the equilibrium of charge – Q at A. 203. 1 q 1 ne
E= =
FB 4 π ε 0 R 2 4 π ε0 R 2
FBD
1
FC or n = E × 4π ε0R2 ×
–Q –Q e
FD D
A 1 1
= 0.036 × × 0.1 × 0.1 ×
FO q 9 × 10 9
1.6 × 10 −19
a 5
O = 2.5 × 10 .
9 × 109 q1q2
B C 204. F = N
–Q –Q 0.20 × 0.20
Fig. 124
58 COMPREHENSIVE OBJECTIVE PHYSICS

209. The two charges of the dipole shall experience forces


F 9 × 10 9 q1q2
= which are equal in magnitude and opposite in direc-
2 (0.12 + K × 0.08)2
tion.
(0.12 + K × 0.08)2 q
Dividing, 2 = 210. Charge density (notice factor 2).
(0.20)2 2r 2 π
0.12 + K × 0.08 1
or 2= E is times this (conductor case).
0.20 ε0
On simplification, K = 4. 40 × 10−3
205. Electric field due to charge + Q, E+ E= = 20 × 10–3 × 9 × 109
2 × π × 22 ε 0
1 Q = 1.8 × 108 V m–1.
=
4πε0 r+2
Electric field due to charge – 2Q, E– 9 × 109 × 20 × 10−6 × 20 × 10−6
211. F = N
0.5 × 0.5
1 2Q
=− = 14.4 N
4πε 0 r−2
Net force
For net field to be zero,
1 Q F1 2Q I = 14.4 3 N.
4 πε0 r+2
=− GH
4 πε 0 r−2 JK or
1
r+2
=
2
r−2
Fig. 126
212. If there were identical charges at each corner of the
or r–2 = 2r+2 or r– = 2 r+ regular polygon, then the electric field at the centre
The condition is satisfied by point D. would have been zero. We conclude from here that
the field due to one charge is equal in magnitude to
206. In the first case, the charge on each sphere is q.
the combined field of all the remaining charges.
1 q2 213. Following two arguments shall lead us to the right
F= choice.
4 π ε0 d 2
(i) Electric field at the centre of the ring is zero.
When A is brought in contact with B, the charge on
(ii) Electric field is directed away from the ring.
q
each becomes . Now, when A is brought in contact 214. eE = mg
2
q eσ ε mg
+q = mg or σ = 0
2 3q
with C, the charge on each is or . ε0 e
2 4
If F′ is the new force of repulsion, then 8.85 × 10−12 × 1.67 × 10−27 × 10
or σ= C m–2
FG q IJ FG 3q IJ 1.6 × 10−19

F′ =
1 H 2K H 4 K = 0.9 × 10–18 C m–2
215. Following two arguments shall decide the right choice.
4π ε0 d2
(i) Acceleration is zero at the origin and also at points
1 3 q2 3F which are far away from the origin.
or F′ = = .
4 π ε 0 8 d2 8 (ii) The directions of acceleration on the two sides of
207. Enclosed charge the origin are opposite.

4Q Qr 3 9 × 109 × Q max.
= × πr 3 = 3 216. 3 × 106 = or Qmax. = 3 × 10–3 C.
4 3×3
πR 3 3 R
3 9 × 109 × 3 × 10−3
3 217. Vmax. = volt
1 Qr 3
Now, E =
4 πε 0 K R 3 r 2 = 9 × 106 volt.
1 Qr . q q q q
= 218. V= + + + + ......
4πε 0 K R 3 1 2 4 8
208. Negative charge will be induced. LM
1 1 1 OP
N
V = q 1 + + + + ......
2 4 8 Q
ELECTROSTATICS 59

1 C1VR22 R1 R22 R2 5 .
V= q or V = 2q = = × = =
1
1− C2 VR 12 R 2 R12 R 1 4
2
226. r = 4 cm = 4 × 10–2 m
3 × 103 N q = – 4 × 10–8 C
219. E= = 103 N C–1
3C
9 × 109 × 4 × 10 −8
dV V= − V
103 = or dV = 10 volt 4 × 10 −2
0.01
= – 9000 V
220. Following arguments shall decide the right choice.
227. Distance in the direction of electric field is the effec-
(i) At each corner, 8 cubes can be placed symmetri-
tive distance.
q
cally. So, flux linked with each cube will be . 228. To keep potential constant, charge has to be increased
8ε 0 by a factor of K.
(ii) Out of the six faces of the cube, the flux linked
q 1 q1q2
with three faces is zero. So, the flux of is to be Now, F=
8ε 0 4 πε 0 r 2
shared by the remaining three faces. 1 (K q1)(K q2 )
and F′ =
221. Potential at the surface of inner sphere = 0 (grounded) 4 πε0 K r2
1 q 1 q′ 1 K q1q2
+ =0 or F′ =
4 πε0 r2 4πε 0 r1 4 πε 0 r2
− qr1 Again, F 1 1
⇒ q′ = . = =
F′ K 2
r2

q LM 1 1 OP 229. r2 + r2 = 2 or 2r2 = 2
222. V=
4 πε 0 x0 N
1 + + + ......
3 5 Q or r = 1 cm
r r


q LM
1 1 1
+ + + ......
OP Now, 1 2 3 q
+ + + =0
N
4 πε0 x0 2 4 6 Q or
1 1 1 1
q = – 6 esu Ö2 cm

V=
q LM 1 1 1 1 1
1 − + − + − + ......
OP Fig. 127
4 πε0 x0 N 2 3 4 5 6 Q 230.
→ ∂φ  ∂φ  ∂φ 
E=− i− j− k
q ∂x ∂y ∂z
or V= log e 2 ∂ 2
4 πε0 x0 Now, Ex = − (x y + yz) = – 2xy
∂x
x2 x 3 x 4 ∂ 2
 log (1 + x) = x – + − + ...... Ey = − (x y + yz) = – x2 – z
2 3 4 ∂y
∂ 2
1 Ez = − (x y + yz) = – y.
223. ΦE = × enclosed charge ∂z
ε0
1 100 Q E= E x 2 + E y2 + E z 2
= × Q × 100 =
ε0 ε0 = 4 x2 y2 + ( x2 + z2 ) + y2
1 At (1, 3, 1),
224. × 10–3 [0.22 – v2] = 600 × 10–8
2 E= 4 × 12 × 32 + (12 + 1)2 + 32
or 0.04 – v2 = 1200 × 10–5 = 0.012
or v2 = 0.04 – 0.012 = 0.028 = 36 + 4 + 9 = 49 = 7 units.

or v = 0.028 m s–1 = 0.167 m s–1 kQ


231. V= (3R2 – r2)
= 16.7 cm s–1 2R 3
Clearly, the variation of V with r is non-linear. We
σ1 Q1 4 πR22 Q1R22
225. = × = arrive at the right choice quickly if we look at the
2
σ 2 4 πR 1 Q2 Q2R12
60 COMPREHENSIVE OBJECTIVE PHYSICS

region characterised by r < R in (a), (b) and (d). In all q


these figures, a linear variation of V is depicted which 240. Using Gauss’s law, = φE
ε0
is not acceptable.
or q = ε0 [4 × 103 – 8 × 103] = – 4 × 103 ε0.

z
9 × 109 × 1 × 10−10 r
232. V= volt = 1.8 volt. → →
0.5 241. W = – q0 E . dS
1 Q 1 Q ∞
233. V= +

z z
4 πε 0 R 4 πε 0 r 4 × 10 −2 4 × 10 −2
σR 2 q σR 2 1
or V=
1 QLM R+
Q
r
OP = – q0
ε0r 2
dr = – 0
ε0 r2
dr

N
ε0 4 π R 2 4 πr 2 Q ∞ ∞

4 × 10 −2
σ q0 σR 2 1 q0 σR 2 1
= (R + r). =– − = ×
ε0 ε0 r ∞ ε0 4 × 10−2
234. When charges – 2Q and + Q are placed at B and C
q0 σR 2 4π
respectively, =
4 π ε0 4 × 10 −2
V′ =
1 LM
Q 2Q Q
− +
OP = zero
4 πε0 a aN a Q 40 × 10−9 × 10−4 × 104 × (2 × 10 −2 )2
× 4 × 9 × 109 π
This settles in favour of (a). = J
4 × 10−2
1
235. m vA2 = qV
2 40 × 4 × 10−4 × 36π
= J
4 × 10−2
1
m vB2 = 4qV = 14.4π J.
2
Only option d is the correct statement about the elec-
vA 2 qV vA 2 1 vA 1 tric potential at a point as in statement (4).
Dividing, = or = or =
2
vB 4 qV vB2 4 vB 2 242. Total potential at P = Potential at P due to charge q
236. V = 4x2 on the surface of the sphere + Potential at P due to
charge Q at the centre of the shell
dV
= 8x 1 q 1 Q
dx = +
4 π ε 0 R 4 π ε 0 R/2
dV
− = – 8x or E = – 8x q 2Q
dx = + .
or E = – 8(1) = – 8 V m–1. 4 π ε0 R 4 π ε 0R
Clearly, the electric field is 8 V along negative x-axis. 1 q1LM q
+ 2
OP
237. In Gauss’s law, the electric field is due to all the
charges present whether inside or outside the Gaussian
243. VC =
N
4 πε0 0.40 0.50 Q
surface. 1 LM q + q OP
1 2
238. Electric field can not be present inside the metal.
VD =
4 πε0 N 0.40 0.10 Q
Moreover, the electric field and equipotential surfaces
are perpendicular. VD – VC =
q2 LM 1 − 1 OP
1
4 πε0 N 0.1 0.5 Q
239. qV = mv2
2 q2 8q2
= (10 – 2) =
mv 2 4πε0 4 πε0
or V=
2q 8q2 q3
Work done =
2 × 10−3 × 10 × 10 4πε0
= volt
2 × 2 × 10−6 dV
244. Field = − .
= 50 kilo volt. dr
ELECTROSTATICS 61

VA − VP 2e
Field at P = 251. Potential of spheres = ±
AP C
3 30 4e 4e
= = Potential difference = =
0.1 2 2 C 4 πε 0 r
4 × 1.6 × 10−19 × 9 × 109
= 15 2 V m–1 = V
0.09 × 10 −3
4 4 = 640 × 10–7 V = 64 µV.
245. 27 × πr3 = πR3 or R = 3r
3 3 252. Electric field at a point on axial line is twice the elec-
V ′ 27 q r tric field on the equatorial line. The directions are
= × = 9.
V 3r q opposite.
253. Potential at a point on the axial line is not zero. Po-
σ 2σ σ
246. EP = (− k) + (− k) + (− k) tential at a point on the equatorial line is zero.
2ε 0 2ε 0 2 ε0
dV λ dV

=
LM σ +
σ
+
σ OP e− kj 254. E= −
dr 2πε 0 r
=−
dr
N 2ε ε 0 2ε 0 Q
z z
Vb b
0 λ λ 1
or dV = − dr or dV = − dr
=

ε0
− k e j 2πε 0 r
Va
2πε 0
a
r

λ
−2V or Vb – Va = − [loge b – loge a]
247. Ex = − = – 100 V m–1 2πε0
− 2 × 10−2 m
λ b
−2V or Va – Vb = log e .
Ey = − = 200 V m–1 2πε 0 a
1 × 10 −2 m
LM 2 − 3 − 4 + 5 OP = 0
Time-saving solution
255. V = 9 × 109
Nr r r rQ
Clearly, x-component is Electric fields due to the all the charges shall not can-
negative and y-compo- cel out because of different directions.
nent is positive. 256. Charge = πrλ
1 πrλ λ
V= or V = .
Fig. 128 4 πε 0 r 4ε 0
1 q 1 q
2000 257. Potential at C1 = −
248. E= N C–1 = 400 N C–1 4 πε 0 R 4 πε0
5 R + d22

Now, V = Er = 400 ×
2
V=8V q 1 LM 1 OP +d
2

MN PQ
2

100 = − R R R
4 πε 0 R R2 + d2
C1V1 + C2 V2 Potential at C2
C1 d C2
249. Common potential, V =
C1 + C2 1 −q 1 q
= + +q d –q
q1 + q2 4 πε 0 R 4 πε 0 2
R + d2
or V= Fig. 129
n π ε0 r1 + 4 π ε 0 r2
1 q1 + q2 q LM 1 1 OP
or V =
4 πε 0 r1 + r2
=
4 πε 0 MN− R + R + d2
2 PQ
250. Maximum length of string = 3l q 2 LM 2 OP
Maximum enclosed charge = 3 lλ
Potential difference =
4 πε 0 R

MN R2 + d2 PQ
∴ Φ= 3lλ.
q LM
1 2 OP
ε0 =
2 πε 0 RMN

R 2 + d2 PQ
62 COMPREHENSIVE OBJECTIVE PHYSICS

258. (a) V(x) = 4 + 5x2 9 × 10 9 × q


265. 18 = or q = 10–9 C = 1 nC
V(1) = 4 + 5 (1)2 = 9 volt 1/2
V(– 2) = 4 + 5 (– 2)2 = 24 volt
266. φ E = (10i + 7 j ).1 j = 7 V m
Potential difference
= (24 – 9) volt = 15 volt d 1 FG IJ
1
dV
267. E∝
dr r 2
∝ 3
r H K
(b) E= − As r → ∞, E → 0.
dx
d 1 3C0 V 2
=− [4 + 5 x 2 ] = – 10x 268. Ei = (3C0) V 2 =
dx 2 2
Electric field at x = – 1 m 1 FG 3V IJ 2
9C0 V 2
= 10 N C–1
Ef =
2
(2C0)
H2K =
4
So, force on 1 C charge is 10 N. It is towards + x axis.
Ei 3C0 V 2 4 2
1 q LMq OP Ef
=
2
×
9C0 V 2
=
3
259. V= +
N
4 πε0 AC BC Q
4C1C2
9 × 109 × 1 × 10− 6 269. C1 + C2 =
or V= C1 + C2
1
or (C1 + C2)2 – 4C1C2 = 0
× LM 10 × 100OP volt or C12 + C22 – 2C1C2 = 0 or (C1 – C2)2 = 0
N 30 Q Fig. 130 ⇒ C1 = C2.
= 3 × 105 volt. 4 3 4
270. πR = 8 × πr3 or R = 2r or C = 2c
260. Volume is conserved. 3 3
Again, Q = 8q
4 3 4
∴ πR = 8 × πr3 or R3 = 23 r3 or R = 2r Now, CV = 8 cv
3 3
8cv 8cv V 4
1 8q 4 × 9 × 109 × 10−10 or V= = or V = 4v or =
V= = volt C 2c v 1
4 πε 0 2r 1 × 10−3 271. R = 10r ; C = 10 c ; Q = 1000 q ; 10 cV = 1000 cv
= 3600 volt V
→ or = 100 or V = 100 v.
261. ∂φ  ∂φ  ∂φ  v
E=− i− j− k
∂x ∂y ∂z 300V
272. E= = 3 × 104 V m–1
→ 0.01 m
E = − 2 x i − 2 y j − 2 z k
273. q × 3 × 104 = 1.96 × 10–15 × 10

|E| = 2 x 2 + y2 + z2 1.96 × 10−14
or q= C = 6.5 × 10–19 C
1 1 Qq Qq 3 × 104
262. mv2 = or v2 =
2 4πε 0 R 2πε 0 mR (C + 1) 1 3 C+1
274. 0.75 × 10–6 = × 10− 6 or =
C + 1+ 1 4 C+2
Qq or C = 2 µF.
or v=
2πε 0mR
275. C1 + C2 + C3 = 12, C1C2C3 = 48, C1 + C2 = 6
q 48
263. Flux over a total solid angle of 4π steradian is . Now, C3 = 12 – 6 = 6, C1C2 = =8
ε0 6

9 × 109 (2e)(e) C1 – C2 = (C1 + C2 )2 − 4C1C2


264. U= eV
re = 36 − 32 = 4 = 2
9 × 10 9 × 2 × 1.6 × 10 −19 8
= eV = – 14.4 eV. Now, 2C1 = 8 or C1 = 4, C2 = = 2.
2 × 10 −10 4
ELECTROSTATICS 63
276. Distance increases, capacity decreases, charge con-
=
ε0A K 1
+
LM
K2 K3
.
OP
stant, potential increases.
F C2 C3 I
d 2 N
K2 + K3 Q
277. Effective capacity C = C1 + GH C2 + C3
, JK 283. Cs =
10 × 20 200
= =
20
3
µF
10 + 20 30
1 Q = Cs V
stored energy = CV 2.
2 20 4000
= µF × 200 V = µC
1 1 FG V IJ 2
FG
1 1 IJ 3 3
278. Ui = C V 2, Uf = (KC0)
2 0 2 H KK , Uf =
K 2HC0 V 2
K Now, V=
4000 µC
=
4000
V =
400
V
3 × 30 µF 90 9
Change in energy
C
= Uf – Ui 284. Capacity changes from to KC .
1 1 FG IJ 2 K+1
=
2
C0 V 2
K
−1
H K Charge changes from
CV
to
K CV
.
Note that this ‘change of energy’ is negative i.e., there 2 K+1
is a decrease of energy. Quantity of charge flowing through the battery

279.
ε A/2
C1 = K1 0
ε A
= K1 0 =
K CV CV
− =
CV 2K
−1
LM OP
d 2d K+1 2 2 K +1 N Q
C2 = K2
ε0 A/2 ε A
= K2 0 CV K − 1 FG IJ
d 2d
ε0A
=
2 K +1 H K
Now, C = C1 + C2 or C = (K1 + K2) 1
2d × 40 × 10− 6 × 3000 × 3000
285. P= 2 kW
15
280. C= µF = 5 µF 2 × 10−3 × 1000
3
Q = (5 µF) (100 V) = 500 µC = 90 kW.
= 500 × 10–6 C 286. For n plates joined alternately, there will be (n – 1)
capacitors in parallel.
= 5 × 10–4 C
∴ equivalent capacity = (n – 1) C
281. Two capacitors in series would give 1 µF. The third
capacitor in parallel with this series combination 4 3 4
287. πR = 8 × πr3 or R = 2r or C = 2c.
would give 3 µF. 3 3
A ε0 A
ε0 288. C=
2 = ε0A K 1 d
d− +
d
282. C1 = K1
d d 2 2 2K
A
ε0 ε0A ε0A 4 ε0A
C2 = K2 d2 = K 2
or C=
d FG
1 IJ =
3 d
(given)

2
d
2
1+
H
K K
ε0
A
2 = K ε0A
1 FG
1 3 IJ 1 3
C3 = K3
d 3
d

2
1+
H
K
=
4 K or 1 + =
K 2
2
1 3 1
C2 and C3 are in series. or = − 1= or K = 2.

C23 = ε 0
LM
A K2 K3 OP K 2 2
289. Parallel combination of 2 µF and 2 µF gives 4µF. It is
N
d K 2 + K3 Q in series with 4 µF. This gives 2 µF. Again, series
Now, C1 and C23 are in parallel. combination of 4 µF and 4 µF gives 2 µF.
ε A L K K O 1 1 1 1
∴ C123 =
ε0 A K 1
d 2
+ 0
M P
d NK + K Q 2
2 3
3
290.
C123
= + +
C 2C 3C
64 COMPREHENSIVE OBJECTIVE PHYSICS

1 11 6C 4×8
= or C123 = 4 µF and 8 µF are in series. Capacity is µF i.e.,
C123 6C 11 4+8
32 8
6 CV µF i.e., µF
Q123 = 12 3
11
8 8
In series combination, charges on individual capaci- µF and µF are in parallel.
9 3
tors are the same as on the combination. 8 8 32
Capacity of parallel combination is + i. e., µF .
6 CV 9 3 9
∴ Q2 = 32 32
11 Now, µF and µF are in series .
Again, Q4 = C4V = 4CV 23 9
32 32
Q2 6 CV 3 ×
∴ = = Series capacity is 23 9 µF
Q4 11 × 4 CV 22 32 32
+
291. The given arrangement is a series combination of 23 9
1 µF and 2 µF
32 × 32 1024
2 i.e., µF or µF or 1 µF.
∴ C= µF 288 + 736 1024
3
299. 4 µF and 4 µF are in series. Combined capacity is 2
4 4 µF. This 2 µF is in parallel with given 2 µF. This
292. πR3 = 125 × πr3 or R = 5r or C = 5c
3 3 gives 4 µF.
125q 5q 300. It is understood that the battery is “disconnected”.
= So, the charge Q0 remains constant. Again dielectric
V v
constant is increased by a factor of 3. So, both poten-
V = 25 v = 25 × 50 volt = 1250 volt
tial difference and electric field are reduced by a factor
1 of 3.
293. Note that QV is 50% of QV.
2 ε0A
301. C=
1 t1 t
294. Energy = (2C) V2 = CV2 + 2
2 K1 K2
1 t1 6 × 10−3
295. Energy of A = (3C) V2. Now, = = 6 × 10–4
2 K1 10
V V
296. V′ =
= t2 4 × 10−3
K 3 and = = 8 × 10–4
K2 5
297. Total energy stored before dielectric, Ei = CV 2
Total energy stored after dielectric 104 ε 0 A 1000 × 10 ε0 A
∴ C= or C =
3 1 V FG IJ 2
3 1
14 14
Ef =
2
CV 2 + (3C)
2 3 H K =
2
CV 2 + CV 2
6 or C=
5000
7
ε0A.
FG 3 + 1IJ FG 10 IJ = 5 CV 302. C1 = 10 µF, V1 = 50 V, V = 20 V
= CV 2
H 2 6K = CV 2
H 6K 3 2

C1V1 + C2 V2
V=
Ei CV 2 3 C1 + C2
= = .
Ef 5 2 5
CV 10 × 50 + 0
3 20 =
72 10 + C
298. Capacity of 6 µF and 12 µF in series = µF = 4 µF
18 or 200 + 20 C = 500 or 20 C = 300
This 4 µF is in parallel with 4 µF. Total capacity is or C = 15 µF.
8 µF.
303. The given capacitor (without dielectric) is a series com-
8
This 8 µF and 1 µF are in series. Capacity is µF. bination of two capacitors, each of capacity 20 µF.
9 After the introduction of dielectric, the capacity of one
2 µF and 2 µF are in parallel. Capacity is 4 µF.
ELECTROSTATICS 65
becomes 40 µF while the capacity of the other remains 311. The four plates are alternately connected. They form
20 µF. three capacitors in parallel. Capacity of each capaci-
40 × 20 800 ε0A 3 ε0 A
Now, C= µF or C = µF tor is . So, the net capacity is .
40 + 20 60 d d
40 µF 1 Q2
= = 13 µF. 312. U =
3 3 2C
C V + C2 V2 U Q2 1 Q2 Q2
304. V= 1 1 = or × = or K = 5.
C1 + C2 5 2 KC 5 2C 2 KC
20 × 500 + 10 × 200
= volt 1
20 + 10 313. H= CV 2
2
10000 + 2000
= volt 1
30 =× 2 × 10–6 × 200 × 200 joule
2
12000 = 4 × 10–2 joule
= volt = 400 volt.
30 314. If V be the potential difference across each capacitor,
3000 V then
305. Number of capacitors in a row = =6
500 V 6V + 3V = 72 – 36 = 36 or V = 4 volt
1 315. At time t, the system may be regarded as a parallel
Capacity of one row = µF combination of two capacitors :
6
2
Number of rows = = 12
1/6
∴ Required number of capacitors is 6 × 12 i.e., 72.
306. The maximum and minimum values correspond to
parallel and series combination.
1 1 1
307. C = 1+ + + + ......
2 4 8
1
= µF = 2 µF
1
1−
2
σ
308. E=
ε0
Fig. 131
q
E= (i) Capacitor of area bvt having air between its plates.
Kε 0 A
F = qE (ii) Capacitor of area of b(l – vt) having dielectric be-
tween the plates.
q2
F= .
Kε 0 A ε0 bvt K ε0b(l − vt)
∴ C= +
309. Increase in energy d d
1 ε0 b
= C (V22 – V12) or C= [vt + Kl – Kvt]
2 d
ε0 b
1 or C= [Kl – (K – 1) vt].
= × 6 × 10–6 (400 – 100) = 9 × 10–4 J d
2
316. If V be the potential at D, then
310. The energy drawn from the source may partly be con-
sumed by connecting wires and also by the internal V1 − V C2
= or C1V1 – C1V = C2 V – C2 V2
resistance of the source. Capacity depends on geometri- V − V2 C1
cal dimensions and not on the applied voltage. The or C1V1 + C2V2 = (C1 + C2)V
potential difference across the capacitor will be equal C1V1 + C2 V2
to the terminal potential difference. or V=
C1 + C2
66 COMPREHENSIVE OBJECTIVE PHYSICS

317. Total charge = 6 × 100 + 0 = 600 µC V2 KC K


Now, = =
If q1 and q2 are the charges on the capacitors of ca- V1 C 1
pacities 6 µF and 14 µF respectively after connection, V2
then But V1 + V2 = V or + V2 = V
K
q1 q2 q1 6 3 K
= or = = or V2 = V.
6 14 q2 14 7 K +1
Also, q1 + q2 = 600 324. Each capacitor can have a maximum charge of CV.
7q1 Capacity of series combination
Now, q1 + = 600
3 C
=
10 q1 2
= 600 or q1 = 180 µC If V′ is the breakdown voltage then,
3
q1 180 µC C
Again, V1 = = = 30 V. CV =V′ or V′ = 2V.
C1 6 µF 2
325. Capacity of parallel combination = 2C
318. The given arrangement is a parallel combination of
two capacitors : Now, 2 CV = 2 CV′ or V′ = V.
A 326. F = qE
(i) Air capacitor of area If one plate is removed, then E is halved. So, F is
2
halved.
A
(ii) Dielectric capacitor of area
2 4 4
327. 8× πr3 = πR3 or R = 2r = 2 × 10–3 m
ε0 A K ε 0 A ε0A 3 3
∴ C= + or C = (1 + K).
2d 2d 2d 9 × 10 9 × 0.066 × 10 −12 × 8
2 V= volt
Q 2 × 10 −3
319. U=
2C = 2.376 volt ≈ 2.4 volt
When dielectric constant of the medium is increased, 328. Refer to Synopsis.
C is increased and U is decreased.
320. Capacity of lower branch ε0A
329. Q = CV or Q = V or V ∝ d
d
6×3
= µF = 2 µF d is increased by a factor of 3 ; V also increased by a
6+3 factor of 3.
The upper and lower branches are in parallel. So, the ∴ V′ = 3 × 60 volt = 180 volt.
combined capacity is 4 µF. 330. Total energy E1 stored in the capacitors is
321. Q = CV = 500 µF × 10V = 5 × 10–3 C
1 1
E1 = CV 2 + (2C)(2V)2
Q 5 × 10 −3 2 2
Now, Q = qt or t = or t = s
q 100 × 10 −6 9 9 (5CV)2 9 Q2
=CV 2 = =
1 2 2 25C 50 C
= × 1000 s = 50 s After the two capacitors are connected together, total
20
charge remains unchanged as Q. However, the effec-
322. Q = CV
tive capacitance Ce is now
Kε 0 A Ce = C + 2C = 3C
Q= V
d Stored energy E2 is thus
If A increases, Q increases. If K increases, Q increases. 1 Q2 1 Q2 9 Q2
If d is decreased, Q increases. If V is reduced, Q de- E2 = = <
2 (3C) 6 C 50 C
creases and hence the electroscope reading.
∴ E2 < E 1
323. In series combination, charge is ‘constant’.
Hence, after connecting the two capacitors together,
1 this system of capacitors loses energy but charge re-
∴ V∝
C mains constant.
ELECTROSTATICS 67
331. Resistance of the dielectric is given by 1
338. Electric energy of the capacitor, Ec = CV 2
d 2
R= ρ
A Gravitational potential energy gained by the mass,
where ρ is the resistivity Eg = mgh. Since a constant fraction (say k) of the
d is the thickness of the dielectic capacitor energy is converted to gain of gravitational
A is the area of the dielectric. potential energy,
εA d ε 1
But C= or
= . Eg = kEc ⇒ mgh = k CV 2 i.e., h ∝ V 2
d A C 2
d ρε Thus a plot of h against V 2 would give a straight line.
We thus have R = ρ = .
A C FG 3IJ
332. U=
Q2
; Q constant
339. CAB = 3 +
H 3 K
µF = 4 µF

2C 3 3
CBC = + = 3 µF
U 1 C2 0.6 2 2
∴ = = = 2.
U 2 C1 0.3 CAB 4
∴ =
333. Capacity of parallel combination = 3 µF CBC 3
3×1 3 340. The capacity is increased by a factor of K( = 5).
Now, C= = µF.
3+1 4 Q2 U W0
Now, U′ = = = .
10 2(5C0 ) 5 5
334. C1 = 2 × µF = 10 µF
2 ε 0 πr 2
341. Capacity, C =
10 d
C2 = 4 × µF = 20 µF
2 After the dielectric is introduced
C = C1 + C2 = 30 µF. Effective area
335. The energy EC transferred from the charged capaci- πr 2 3πr 2
tor to the flash tube is given by = πr 2 − =
4 4

EC =
1
CV 2 = Pt 3πr2
Capacity of air condenser = ε0
2 4d
where C is the capacitance of the capacitor
πr2
V is the potential difference of the capacitor Capacity with dielectric = 6ε0
4d
P is the average power delivered to the tube
t is the total flash time. ε0 πr 2 9C
Total capacity =(3 + 6) =
4d 4
1
∴ C (1000) 2 = (1000)(0.04) 342. B has 3-fold capacitance. So, 3 : 1 voltage sharing
2 between A and B.
⇒ C = 80 µF 343. In the first case, Q = 12 µF × 60 V = 720 µC
336. If C′ represents the capacity of the individual capaci- In the second case,
tor, then
720 µC
C′ V= = 60 volt
C= or C′ = 8C 12 µF
8 σ q
Cp = 8 × 8C = 64 C. 344. E= =
Kε 0 KAε0
337. Total charge, Q = 2 × 150 × 10–6 C
1000
345. Number of capacitors in a row = =4
0.30 250
C1 + C2 = F
9 × 109 Capacity of one row =
8
= 2 µF
−6 9
4
300 × 10 × 9 × 10 16
∴ V= volt = 9 × 106 volt Number of rows = =8
0.30 2
Total number of capacitors = 4 × 8 = 32
68 COMPREHENSIVE OBJECTIVE PHYSICS

ε0 A 353. Between A, B we have three parallel systems (C1),


346. C= (C3, C4) and (C2, C6).
d
Clearly, C5 is uncharged.
ε0A
Again, 2C = K 1 1
2d 354. U = CV 2 = × 4 × 10–6 × 100 × 100 J = 0.02 J
2 2
K
Dividing, 2 = or K = 4 355. R1 and R2 divide the potential, acting across C. R3
2 does not influence the potential difference, though it
t 2t affects the rate of charging.
347. Effective separation = t – =
3 3 356. Combined capacity of 4 µF and 8 µF is 12 µF. It is in
series with 12 µF. So, the combined capacity is 6 µF.
ε0A ε0 A
Now, 100 = and C′ = Total charge = 6 × 20 µC = 120 µC
t 2t/3
Charge on parallel combination is 120 µC.
C′ 3 3 Now, Q4 4 1
or = or C′ = × 100 pF. = =
100 2 2 Q8 8 2
1 1
348. U= CV 2 ∴ Q4 = × 120 µC = 40 µC.
2 1+ 2
1 357. Parallel combination of 1.5 µF and 1.5 µF gives 3 µF.
= × 100 × 10–6 × 200 × 200 J = 2 J Now, three capacitors, each of capacity 3 µF, are in
2
349. Dielectric increases the capacity. 3
series. So, total capacity is µF i.e., 1 µF.
350. Energy stored in capacitor 3
1 1 ε0A
= CV 2 = × 10 × 10–6 × 100 × 100 J 358. C=
t1 t
2 2 + 2
K1 K2
=
1
J = 0.05 J FG t
1 t2 IJ
20
Battery supplies CV charge at voltage V Now, Q = CV or V =
Q
HK 1
+
K2 K
ε0A
∴ E = CV 2 supplied
359. Using charge conservation,
∴ Energy supplied
CV + CV = CV1 + CV2
= 10 × 10–6 × 100 × 100 J = 0.10 J.
or V1 + V2 = 2 V
351. Clearly, 6 µF and 12 µF are in parallel. Their com-
bined capacity is (6 + 12) µF i.e. 18 µF. It is in series Q2
360. Initial energy =
2C1
with 9 µF. So, combined capacity is 18 × 9 µF i.e.,
18 + 9 Q2
Final energy =
162 2(C1 + C2 )
µF i.e. 6 µF.
27
The given circuit is a series combination of 9 µF and Q 2 2(C1 + C2 ) C + C2
Required ratio is or 1 .
18 µF. The ratio of capacities is 1 : 2. The ratio of 2C1 Q2 C1
1 361. The equivalent circuit of the given circuit is shown
voltages is 2 : 1. So, voltage across 18 µF is × 90 V here.
3
i.e., 30 V.
Clearly, voltage across 6 µF is 30 V.
Now, Q = 6 µF × 30 V
= 180 µC.
20 10 C
352. =
3 10 + C
200 + 20 C = 30 C
or 10 C = 200 or C = 20 µF.
Fig. 132
ELECTROSTATICS 69

Clearly, there is no potential difference between C and


D. Since potential difference between C and D is zero
therefore the capacitor of capacity 24 µF shall not hold
any charge.
1 1 1 1 3+2+1
362. = + + = or C = 1 µF
C 2 3 6 6
(b)
Q = CV = 1 µF × 10 V = 10 µC
In series combination of capacitors, charge on every Fig. 133
capacitor is the same as the charge on the combina-
ε0 A
tion. So, the charge on the 3 µF capacitor is 10 µC. Effective capacity of series combination =
2d
K 1 ε0A Total effective capacity
363. C1 =
d/2
ε0A ε0A 3ε A
K ε A = + = 0
C2 = 2 0 2d d 2d
d/2 367. Using conservation of charge,
The given arrangement is a series combination of C1 (C + 100) 30 = 100 × 100
and C2.
or C + 100 = 333.33 or C = 233.33 µµF

∴ C=
C1 C2 LM
2ε A K 1 K 2
= 0
OP 368. Initial total charge = 2CV + CV = 3 CV
C1 + C2 d N
K1 + K2 Q Final total charge = 2 CV′ + KCV′
3V
1 1 1 1 1 6+2+1 Equating, CV′ (2 + K) = 3 CV or V′ =
364. = + + or = K+2
Cs 3 9 18 Cs 18 ε0A dC d ε0 A FG IJ
or
1
=
9
=
1
or Cs = 2 µF
369. C=
x
or
dt
=
dt x H K
Cs 18 2 d FG 1IJ − 2ε 0 A dx − 2ε 0 A
Again, Cp = (3 + 9 + 18) µF = 30 µF
= ε0A
dt H xK =
x2 dt
=
x2
v

Cs 2 1 dc 1
= = clearly, ∝
C p 30 15 dt d 2
9 × 4.5
ab 370. C= µF or C = 3 µF
365. When outer sphere is earthed, C1 = 4πε0 13.5
b−a Q = CV = 3 × 12 µC = 36 µC
4 πε 0 ab Potential difference across 4.5 µF
When inner sphere is earthed, C2 = 4πε0b + 36
b−a = V=8V
4.5
C2 – C1 = 4πε0b
366. An equivalent of the given arrangement is shown in  Time-saving solution
Fig. 133 (b).
V1 C2 9 2 8
= = = =
V2 C 1 4.5 1 4 Fig. 134
1
– – – – – – a
+ + +2+ + + 371. Effective separation is halved. Capacity is doubled.
+ + + + + +
– – – 3– – –
– – – – – –
b
 Time-saving solution of Q. 372
+ + +4+ + +
C2 4
=
C1 1
(a) V2 1 20
= =
V1 4 80
70 COMPREHENSIVE OBJECTIVE PHYSICS

373. The given capacitors are connected in parallel. 383. The equivalent of the given circuit is as under :
2 2
374. d–2+ + 1.6 = d or = 0.4
K K
2
or K= = 5.
0.4
FG IJ2
2

375.
H K
ε 0π
100
= 4πε0
10
d 100
1 1
4
= or d = 10–3 m.
d × 10 10 Fig. 138
376. It is a case of parallel combination of two capacitors.
rr 4 × 12
377. C = 4πε0 K 2 1 Capacity of upper branch = µF
r2 − r1 4 + 12
6 108 × 10−4 48
= × F = 24 × 10–11 F µF = 3 µF
=
9 × 109 3 × 10−2 16
= 240 × 10–12 F = 240 pF. Total capacity = (3 + 2) µF = 5 µF
378. Capacity of series combination 384. The equivalent of the given network is :
1
= µF
2

FG 1 IJ
Total capacity = 1 +
H 2
µF
K Fig. 135
= 1.5 µF.
1200
379. (0.2 + 1) V = 0.2 × 600 or V = volt
12
= 100 volt
380. K increased by a factor of 4. So, capacity increased by
a factor of 4. Fig. 139
1
381. 3=3–1+ +d
2
d = 0.5 mm
New separation = (3 + 0.5) mm = 3.5 mm.
382. Consider balanced Wheatstone Bridge.

Fig. 140

Fig. 136

Fig. 141
Parallel combination of C and 2C gives 3C.

385.
LM 3 × 6 OP 5000 = (3 + 6) V or V=
10000
≈ 1111 V.
N3 + 6Q 9

Fig. 137
ELECTROSTATICS 71

ε0A A 3 µF 6 µF
386. 4πε0r = or d =
d 4πr
π (20 × 10−3 )2 −6
= m = 400 × 10 m A B
4π × 1 4
–6
= 100 × 10 m = 10 m –4
2 µF 4 µF
= 10–1 mm = 0.1 mm.
387. 2 × 200 = [2 + C] 20 or 2 + C = 20 or C = 18 µF
V Fig. 143
388. Electric field =
d
V 2 µF
Electric field due to one plate =
2d
QV CV 2
Force = = .
2d 2d A B
4 4
389. 2× πr3 = πR3 or R = 21/3 r 4/3 µF
3 3
or C′ = 21/3 C = 1.26 C. Fig. 144
390. Charge that flows from battery to capacitor
= (K – 1) CV FG 4 IJ 10
= (5 – 1) 10 × 12 µC = 480 µC
C = 2+
H 3 K µF =
3
µF

391. With S1 and S3 closed, there would be no change in 397. q0 = C0V0


potential difference.
392. The given arrangement is a parallel combination of Again, Q=
LM K C OP (2V )
two capacitors. NK + 1 Q 0 0

2.5
393. A=1A
I= 2q0 2q0
=
2.5 K+1 = 1
V = 1 × 2 volt = 2 volt 1+
K K
Q = CV 398. 3C = 30 (for parallel combination)
= 5 µF × 2 volt = 10 µC C = 10 µF
394. Q same ; CV = constant ; C less, V more
C
20 Again, = 30 (for series combination)
395. Parallel combination of [8F, 4F] and 4F gives F. 3
3
1 1 3 1 or C = 90 µF
Now, = + +
C 12 20 16 Clearly, we have to consider
series-parallel combination.
1 20 + 36 + 15 1 71 Consider a capacitor connected
or = or =
C 240 C 240 in parallel with a series com-
240 bination of two capacitors. Fig. 145
or C= F.
71 3C
396. The equivalent of the given network is : Now, = 30 or C = 20 µF
2
3 mF 6 mF Q Q Q 2Q
399. + + = 60 or = 60
2C C 2C C
Q
A B or = 30 volt
C
2 mF 4 mF
400. The equivalent of the given arrangement is shown
here.

Fig. 142
72 COMPREHENSIVE OBJECTIVE PHYSICS

Q2 Q2
U= =
2C 2ε 0 b
[Kl − (K − 1) a]
d
Q2 d
=
2ε 0b [Kl − (K − 1)a]
Fig. 146 402. Potential difference across the series combination of 3
µF and 6 µF = 60 volt
Capacity of parallel arrangement is 2C. So, we have
V1 C2 V1 6 2
to find the capacity of series combination of C and 2C. Now, = ; = =
V2 C1 V2 3 1
∴ Equivalent capacity
V1 2 40
(C) (2C) 2C2 2C 2ε A = =
= = = = 0 V1 + V2 3 60
C + 2C 3C 3 3d
This leads us to the right choice.
401. The given arrangement is a parallel combination of
two capacitors (i) air capacitor and (ii) dielectric
capacitor.
 Note the special technique used in the solu-
tion of Q. 402.
ε0ba
Capacity of air capacitor =
d C
403. Capacity of each row =
ε b(l − a) 3
Capacity of dielectric capacitor = K 0
d C C C
Total capacity, Total capacity of three rows = + + =C
3 3 3
C= 0
ε ba Kε0b (l − a)
+ FG C IJ C2
d d
Cab =
(C)
H 2K = 2 = C
ε 0b ε b C 3C 3
= [a + K(l – a)] = 0 [Kl – (K – 1) a] C+
d d 2 2
ELECTROSTATICS 73

MCQs
SET III

with
More than one correct alternative

Average time allowed per question is 50 seconds.

404. A parallel plate capacitor is connected across a bat- (c) During the process, a work is done by external force
tery. A dielectric slab is introduced between the plates, (d) During the process, the internal energy of the battery
the battery is still connected to the plates. Which of shall not change.
the following is incorrect ? 408. A dielectric slab of thickness d is inserted in a paral-
(a) Potential difference remains unchanged lel plate capacitor whose negative plate is at x = 0
(b) Charge on the plates is increased and positive plate is at x = 3d. The slab is equidistant
(c) The electric field is increased enormously from the plates. The capacitor is given some charge.
(d) The energy is decreased. As x goes from 0 to 3d,
(a) the magnitude of the electric field remains the same
405. The electric field and the electric potential at a point (b) the direction of the electric field remains the same
are E and V respectively. Which of the following is
(c) the electric potential increases continuously
incorrect ?
(d) the electric potential increases at first, then decreases and
(a) If E = 0, V must be zero (b) If V = 0, E must be zero
again increases. [IIT 1998]
(c) If E ≠ 0, V cannot be zero (d) If V ≠ 0, E cannot be zero.
409. Which of the following statements about an electric
406. In a parallel-plate capacitor of plate area A, plate sepa- field is correct ?
ration d and charge Q, the force of attraction be- (a) The electric field strength due to a point charge varies as
tween the plates F is proportional to 1/r2 where r is the distance from the charge.
(a) C (b) V 2 (b) Electric field strength is a vector quantity.
1 (c) The electric field strength at a point is a measure of the
(c) (d) All of these.
d force exerted on a unit positive charge at that point.
407. Identical dielectric (d) The electric field strength is zero at all points where the
slabs are inserted potential is zero.
into two identical ca- (e) The electric field strength at a point is a measure of the
pacitors A and B. potential gradient at that point.
These capacitors
410. Mark out the wrong options :
and a battery are
(a) The total charge of the universe is constant.
connected as shown
in Fig. 147. Now, (b) The total positive charge of the universe is constant.
the slab of capacitor (c) The total negative charge of the universe is constant.
B is pulled out with (d) The total number of charged particles in the universe is
battery remaining Fig. 147 constant.
connected. Which of the following is correct ? 411. A deutron and α-particle are placed in an electric
(a) During the process, the charge on the combination begins field. The forces acting on them are F1 and F2 , and
to decrease their accelerations are a1 and a2 respectively. Which
(b) Finally, charge on capacitor B will be same as on capacitor of the following is correct ?
A (a) F1 = F2 (b) F1 ≠ F2
(c) a1 = a2 (d) a1 ≠ a2
74 COMPREHENSIVE OBJECTIVE PHYSICS

412. The electric potential decreases uniformly from (a) If Q1 changes, both E and φ will change
120 V to 80 V as one moves on the X-axis from (b) If Q2 changes, E will change but φ will not change
x = – 1 cm to x = + 1 cm. The electric field at the (c) If Q1 = 0 and Q2 ≠ 0 then E ≠ 0 but φ = 0
origin
(d) If Q1 ≠ 0 and Q2 = 0 then E = 0 but φ ≠ 0.
(a) must be equal to 20 V/cm
(b) may be equal to 20 V/cm 418. Following operations can be performed on a capaci-
(c) may be greater than 20 V/cm tor :
(d) may be less than 20 V/cm. X – connect the capacitor to a battery of emf E
413. Four charges, all of the same magnitude, are placed Y – disconnect the battery
at the four corners of a square. At the centre of the
Z – reconnect the battery with polarity reversed
square, the potential is V and the field is E. By suit-
able choices of the signs of the four charges, which of W – insert a dielectric slab in the capacitor.
the following can be obtained ? (a) In XYZ (perform X, then Y, then Z) the stored electric
(a) V = 0, E = 0 (b) V = 0, E ≠ 0 energy remains unchanged and no thermal energy is de-
veloped.
(c) V ≠ 0, E = 0 (d) V ≠ 0, E ≠ 0.
(b) The charge appearing on the capacitor is greater after the
414. A parallel plate capacitor of plate area A and plate
action XWY than after the action XYW.
separation d is charged to potential difference V and
then the battery is disconnected. A slab of dielectric (c) The electric energy stored in the capacitor is greater after
constant K is then inserted between the plates of the the action WXY than after the action XYW.
capacitor so as to fill the space between the plates. If (d) The electric field in the capacitor after the action XW is
Q, E and W denote respectively the magnitude of the same as that after WX.
charge on each plate, the electric field between the
419. A metal sphere of radius r, carrying charge q, lies
plates (after the slab is inserted), and work done on
inside an uncharged conducting shell of radius 2r. If
the system, in question, in the process of inserting
the slab, then they are joined by a metal wire,
ε AV ε K AV q
(a) Q = 0 (b) Q = 0 (a) charge shall flow from the sphere to the shell.
3
d d
2q
V ε AV 2
(d) W = 0
1 FG IJ (b) charge shall flow from the sphere to the shell
(c) E =
Kd 2d
1−
K
.
H K 3
(c) charge q shall flow from the sphere to the shell
[IIT 1991]
q2
415. Two large, parallel conducting plates are placed close (d) amount of heat will be produced.
16πε 0 r
to each other. The inner surfaces of the two plates
420. Which of the following quantities do not depend on
have surface charge densities + σ and – σ. The outer
the choice of zero potential or zero potential energy ?
surfaces are without charge. The electric field has a
magnitude of (a) potential at a point
(a) 2σ/ε0 in the region between the plates (b) potential difference between two points
(b) σ/ε0 in the region between the plates (c) potential energy of a two-charge system
(c) σ/ε0 in the region outside the plates (d) change in potential energy of a two-charge system.

(d) zero in the region outside the plates. 421. A ring with a uniform charge q and radius r, is placed
in the yz plane with its centre at the origin.
416. From the following statements for equipotential sur-
(a) The field at the origin is zero.
face, the correct statement is
(a) The potential difference between two points on the sur- q
(b) The potential at the origin is k .
face is zero r
(b) The direction of electric intensity is perpendicular to the q
(c) The field at the point (x, 0, 0) is k .
surface at that point x2
(c) No work is done in moving the charge on the surface q
(d) The field at the point (x, 0, 0) is k .
(d) The shape of the surface is always spherical. R2 + x2
417. Charges Q1 and Q2 lie inside and outside respectively 422. Correct statement is
of a closed surface S. Let E be the electric field at any (a) Condensers connected in series have the same charge
point on S and φ be the flux of electric field over the (b) Condensers connected in parallel have the same potential
closed surface. Which of the following is correct ? difference
ELECTROSTATICS 75
(c) Condensers connected in series, the potential across the 425. A parallel plate capacitor is charged to a certain volt-
condenser of lowest capacity is also lowest age. Now, if the dielectric material (with dielectric
(d) In parallel connection of condensers, the charge on the constant k) is removed, then which of the following
condenser of least capacity is minimum. is incorrect ?
423. A parallel-plate capacitor is charged from a cell and (a) The capacitance increases by a factor k.
then isolated from it. The separation between the (b) The electric field reduces by a factor k.
plates is now increased. Which of the following is (c) The voltage across the capacitor decreases by a factor k.
correct ?
(d) The charge stored in the capacitor increases by a factor k.
(a) The force of attraction between the plates will decrease
[National Standard Exam. in Physics 2001]
(b) The field in the region between the plates will not change
426. Potential difference across a capacitor is doubled.
(c) The energy stored in the capacitor will increase
Which of the following is correct ?
(d) The potential difference between the plates will decrease.
(a) The capacity remains unchanged.
424. Charges q1 and q2 are placed inside and outside re-
(b) The charge on the capacitor is doubled.
spectively of an uncharged metallic shell. Their sepa-
(c) The field between the plates of the capacitor is doubled.
ration is r. Which of the following is correct ?
q1q2 (d) The energy associated with the capacitor becomes four
(a) The force on q1 is zero. (b) The force on q1 is k times of its initial value.
r2
q1q2
(c) The force on q2 is k . (d) The force on q2 is zero.
r2
Answers (Set III)
404. (c), (d) 405. (a), (b), (c), (d) 406. (a), (b), (c), (d) 407. (a), (b), (c) 408. (b), (c)
409. (a), (b), (c), (e) 410. (b), (c), (d) 411. (b), (c) 412. (b), (c) 413. (a), (b), (c), (d)
414. (a), (c), (d) 415. (b), (d) 416. (a), (b), (c) 417. (a), (b), (c) 418. (b), (c), (d)
419. (c), (d) 420. (b), (d) 421. (a), (b) 422. (a), (b), (d) 423. (b), (c)
424. (a), (c) 425. (a), (b), (c), (d) 426. (a), (b), (c), (d)

Solutions (Set III)


404. Since battery remains connected therefore V remains (b) Since the capacitors are in series therefore charge
constant. So, (a) is correct. Again, C is increased. on capacitor B will be the same as the charge on
Q (= CV) is increased. capacitor A.
So, (b) is correct. (c) The external force has to do work against the force
Introduction of dielectric reduces electric field. of attraction between the capacitor and the
So, (c) is incorrect. dielectric.
1 (d) Internal energy of the battery shall change due to
Again, U=CV2 the flow of charge into the battery.
2
Since C is increased by a factor of K therefore U is 408. Note that we are moving from negative plate to posi-
increased by a factor of K. So, (d) is incorrect. tive plate.
409. The electric field strength is zero at all points if the
Q2 (CV)2 C (CV 2 ) ε A CV 2
406. F= = = = 0 potential gradient is zero at these points. Zero poten-
2ε 0 A 2ε 0 A 2 ε0 A d 2ε 0 A
tial at these points do not imply that the potential
CV 2 gradient at these points is also zero and hence the
= .
2d statement d is incorrect.
407. (a) When the dielectric is pulled out, the capacity of 410. Universe is to be regarded as an isolated system. So,
capacitor B decreases. The capacity of the combi- total charge has to remain constant.
nation decreases. V remains fixed. So, charge on 411. (b) Same electric field but different charges.
the combination decreases. (c) Think in terms of charge to mass ratio.
76 COMPREHENSIVE OBJECTIVE PHYSICS

412. The electric field may or may not be along X-axis. XYW
413.
–q +q –q –q Q constant.
(c) WXY
C more, Q more
XYW
C more, Q same.
+q –q +q +q (d) In both the cases, the electric field is reduced by a
(a ) (b) factor of K.
Fig. 148 Fig. 149 419. (c) Charge resides on the outer surface.
q2
(d) Initial energy =
+q +q +q –q 2 × 4πε 0r
q2
Final energy =
2 × 4 πε0 (2r)

Heat energy produced q2 q2


= −
2 × 4 πε0 r 2 × 4πε 0 (2r)
+q +q +q +q
(c) (d )
=
q2 LM1 1

OP
Fig. 150 Fig. 151 N
4 πε 0 r 2 4 Q
414. (a) Since the battery is disconnected therefore charge q2 2 − 1 q2
= =
Q remains 4πε 0 r 4 16πε 0 r
ε0A σ
∴ Q = CV = V 423. (b) Note that E=
d ε0
(c) Electric field is reduced by a factor of K
Q2 Q2d
1 V (c) U = or U = .
∴ E= 2C 2ε 0 A
K d When d is increased, U is increased.
1 1 FV I2
(d) W=
2
CV 2 –
2
(KC) GH K JK
2
424. (a) Charge q1 is electrostatically shielded from the in-
fluence of charge q2.

=
1
CV 2 1 − LM OP
1 (c) Following two arguments shall decide the correct-
2 KN Q ness of this choice.

ε 0 AV 2
=
LM1 − 1 OP . (i) Due to electrostatic induction, there will be charge
2d N KQ on the outer surface.
(ii) For an external point, the charge on the sphere
415. In the region between the plates,
behaves as if concentrated at the centre.
σ σ σ
E= + or E = 425. Capacitance will decrease. Both the electric field and
2ε 0 2ε 0 ε0 voltage shall increase. Again, Q = CV. Since C de-
416. Equipotential surfaces and electric lines of force con- creases therefore Q decreases.
stitute a mutually perpendicular network. 426. (a) Capacity is independent of potential difference.
417. Note that the electric field is due to all charges inside
(b) Q = CV ; Since V is doubled therefore Q is doubled.
the closed surface as well as outside.
418. (a) Thermal energy will be evolved due to flow of cur- V
(c) E = ; V is doubled ; E is doubled.
rent. d
(b) XWY 1
(d) U = CV 2 ; V is doubled ; U is quadrupled.
V constant ; C increased, Q increased 2
ELECTROSTATICS 77

SET IV MCQs
based on
TYPICAL NUMERICAL BANK
(Exclusively for Engineering Entrance Tests)

Average time allowed per question is 60 seconds.

427. The capacity of a parallel plate air capacitor is 10 432. In the previous question, the electrical intensity at
µ F and it is given a charge of 40 µC. The electrical x = 0, is
energy stored in capacitor is 2q
(a) 2q (b)
(a) 8 × 10–3 erg (b) 8 × 102 erg 3
(c) 8 × 103 erg (d) 20 × 103 erg. 4q 4q
(c) (d) .
3 5
FG 10 IJ 433. The capacities of two conductors are C1 and C2 and
428. Electric charges of
H 3K × 10 –9 coulomb are placed
their respective potentials are V1 and V2 . If they are
at each of the four corners of a square of side 0.08 m. connected by a thin wire, then the loss of energy will
The potential at the intersection of the diagonals is be given by
(a) 900 volt (b) 900 2 volt C1C2 (V1 + V2 ) C1C2 (V1 − V2 )
(a) (b)
(c) 150 2 volt (d) 1500 2 volt. 2(C1 + C2 ) 2(C1 + C2 )
429. A particle of mass 0.002 kg and a charge 1 µC is held
C1C2 ( V1 − V2 )2 (C1 + C2 ) (V1 − V2 )
at rest on a frictionless horizontal surface at a (c) (d) .
2(C1 + C2 ) C1C2
distance of 1 m from a fixed charge of 1 mC. If the
particle is released, it will be repelled. The speed of 434. The capacities and connection of five capacitors are
the particle when it is at a distance of 10 m from the shown in the Fig. 152. The potential difference be-
fixed charge is tween the points A and B is 60 volt. Then the equiva-
(a) 60 m s–1 (b) 75 m s–1
lent capacity between A and B and the charge on 5
µF capacitance will be respectively
(c) 90 m s–1 (d) 100 m s–1.
430. A charged oil drop of mass 2.5 × 10 –7 kg is in space
between the two plates, each of area 2 × 10 –2 m2, of a
parallel plate capacitor. When the upper plate has a
charge of 5 × 10 –7 C and the lower plate has an equal
negative charge, the oil drop remains stationary. The
charge of the oil drop is : [Take g = 10 m s–2]
(a) 9 × 10 –1 C (b) 9 × 10 –6 C
(c) 8.85 × 10 –13 C (d) 1.8 × 10 –14 C.
431. An infinite number of charges, each numerically equal Fig. 152
to q but consecutive charges having opposite sign, (a) 44 µF, 300 µC (b) 16 µF, 150 µC
are placed along the x-axis at x = 1, x = 2, x = 4, x =
(c) 15 µF, 200 µC (d) 4 µF, 50 µC. [IIT 1984]
8 ...... and so on. Then the electrical potential at x =
435. Five identical plates, each of area A are joined as
0 due to this set of charges is
shown in the Fig. 153. The distance between the plates
2q
(a) 2q (b) is d. The plates are connected to a p.d. of V volts.
3
The charge on plates 1 and 4 will be
4q 4q
(c) (d) .
3 5
78 COMPREHENSIVE OBJECTIVE PHYSICS

CE
(a) CE (b)
R2
CE R 1 CE R 2
(c) (d) .
R2 + r R2 + r
439. In the Fig. 156, charge and the potential difference
across the 4 µ F capacitor will be nearly

Fig. 153
ε AV 2ε 0 AV − ε 0 AV 2ε 0 AV
(a) 0 , (b) ,
d d d d
ε 0 AV − 2ε 0 AV − ε 0 AV − 2ε 0 AV
(c) , (d) , .
d d d d
[IIT 1984]
436. The numerical value of the charge on each plate of
the capacitor (see Fig. 154) is

Fig. 156
(a) 600 µC, 150 volt (b) 300 µC, 75 volt
(c) 800 µC, 200 volt (d) 580 µC, 145 volt.
440. A capacitor of 4 µ F charged to 50 V is connected to
another capacitor of 2 µF charged to 100 V with plates
of like charges connected together. The total energy
before and after connection in multiples of (10 –2 J) is
(a) 1.5 and 1.33 (b) 1.33 and 1.5
(c) 3.0 and 2.67 (d) 2.67 and 3.0.
[MP PMT 1992]
Fig. 154 441. The area of each plate of a parallel plate capacitor is
100 cm2 and the distance between the plates is 1 mm.
(a) 6 µC (b) 12 µC It is filled with mica of dielectric 6. The radius of the
(c) 18 µC (d) 3 µC. equivalent capacity of the sphere will be
437. A charged soap bubble of radius R and surface ten- (a) 47.7 m (b) 4.77 m
sion σ is in equilibrium with outside and inside pres- (c) 477 m (d) 4.7 km.
sures equal. The charge on the soap bubble is
442. A solid conducting sphere having a charge Q is sur-
(a) 32ε 0 π 2R 3σ (b) 128 ε 0π 2R 3σ rounded by an uncharged concentric conducting hol-
low spherical shell. Let the potential difference be-
(c) 128 ε 0 π 2R 3σ (d) 32 ε 0 π 2R 3σ . tween the surface of the solid sphere and that of the
438. Refer to the circuit shown in Fig. 155. The charge on outer surface of the hollow shell be V. If the shell is
the plates of the capacitor C is now given a charge of 3Q, the new potential differ-
ence between the same two surfaces is
(a) V (b) 2V
(c) 4V (d) – 2V. [IIT 1989]

443. Given : E = E0 xi. Consider an imaginary cubical
volume of edge l, with its edges parallel to the axes of
coordinates. The charge inside this volume is
(a) zero (b) ε0E0 l3
1 1
(c) E l3 (d) ε E l2.
ε0 0 6 0 0
Fig. 155
[Vardhman 2002]
ELECTROSTATICS 79
444. A parallel plate A/2 A/2 447. A hollow charged metal sphere has radius r. If the
capacitor of potential difference between its surface and a point
area A, plate K1 K2 d/2 at distance 3r from the centre is V, then the electric
separation d field intensity at distance 3r from the centre is
d
and capacitance V V
C is filled with (a) (b)
K3 6r 4r
three different V V
(c) (d) .
dielectric mate- A = Area of plates 3r 2r
A
rials having di- 448. Two identical thin rings, each of radius R metre, are
electric con- Fig. 157 co-axially placed at a distance R metre apart. Q1 cou-
stants K1 , K2 and K3 as shown in Fig. 157. If a single lomb and Q 2 coulomb are the charges uniformly
dielectric material is to be used to have the same spread on the two rings. The work done in moving a
capacitance C in this capacitor, then its dielectric con- charge q from the centre of one ring to that of the
stant K is given by other is
1 1 1 1 1 1 1 (a) zero (b) q(Q1 – Q2) ( 2 – 1)/ 2 4πε0R
(a) = + + (b) = +
K K 1 K 2 2K 3 K K 1 + K 2 2K 3
2 q (Q 1 + Q 2 ) q(Q1 + Q 2 ) ( 2 + 1)
K 1K 2 (c) (d) .
(c) K = + 2K 3 (d) K = K1 + K2 + 2K3 . 4 πε 0R 2 . 4 πε 0R
K 1 + K2
[IIT 1992]
[IIT Screening 2000]
449. A charge Q is placed at each of two opposite corners
445. Three charges Q,
of a square. A charge q is placed at each of the two
+ q and + q are
opposite corners of the square. If the resultant elec-
placed at the ver-
tices of a right an- tric field on Q is zero, then
gled isosceles tri- q
(a) Q = – (b) Q = − 2 2 q
angle as shown in 2 2
Fig. 158. The net
electrostatic en- (c) Q = – 2q (d) Q = 2 2 q .
ergy of the con- 450. A charge Q is distributed over two concentric hollow
figuration is zero spheres of radii r and R (> r) such that the surface
Fig. 158
if Q is equal to densities are equal. The potential at the common cen-
−q − 2q tre is
(a) (b)
1+ 2 2+ 2 Q(R 2 + r 2 ) Q
(a) (b)
(c) – 2q (d) + q. 4 πε 0 (R + r) R+r
Q(R + r )
[IIT Screening 2000] (c) zero (d) .
4 πε 0 (R 2 + r 2 )
446. Due to a charge inside the cube, the electric field is :
(Fig. 159) Ex = 600x1/2, Ey = 0, Ez = 0. The charge [IIT 1981]
inside the cube is nearly 451. A capacitor is filled with two dielectrics of the same
dimensions but of dielectric constants 2 and 3 as
Y shown first in (a) and then in (b). The ratio of the
capacitances in the two arrangements is
0.1 m

O 2
2 3
X 3

0.1 m
Z (a) (b)
Fig. 160
Fig. 159
(a) 25 : 24 (b) 24 : 25
(a) 600 µC (b) 60 µC
(c) 12 : 13 (d) 13 : 12.
(c) 7 µµC (d) 6 µµC.
80 COMPREHENSIVE OBJECTIVE PHYSICS

452. A capacitor of capacity C is connected with a battery (a) VAB = 100 volt, VBC = 100 volt
of potential V in parallel. The distance between its (b) VAB = 75 volt, VBC = 25 volt
plates is reduced to half at once, assuming that the (c) VAB = 25 volt, VBC = 75 volt
charge remains the same. Then to charge the capaci-
(d) VAB = 50 volt, VBC = 50 volt. [IIT 1979]
tor upto the potential V again, the energy given by
the battery will be 457. The resultant capacitance between A and B in the
(a) CV 2/4 (b) CV 2/2 following network is equal to
(c) 3CV 2/4 (d) CV 2.
453. 100 capacitors, each having a capacity of 10 µF, are
connected in parallel and are charged by a potential
difference of 100 kV. The energy stored in the capaci-
tors and the cost of charging them, if electrical en-
ergy costs 108 paise per kWh, will be
(a) 107 joule and 300 paise (b) 5 × 106 joule and 300 paise
(c) 5 × 106 joule and 150 paise
(d) 107 joule and 150 paise. [MP PET 1996]
454. In the circuit shown
here C1 = 6 µ F, C2 = 3
µ F and battery B = 20 Fig. 163
V. The switch S 1 is (a) 1 µF (b) 3 µF
first closed. It is then (c) 2 µF (d) 1.5 µF.
opened and after-
458. Four capacitors are connected as shown in Fig. 164.
wards S 2 is closed.
The approximate charge and potential across 8 µF
What is the final
capacitance will be respectively
charge on C2 ?
Fig. 161
(a) 120 µC (b) 80 µC
(c) 40 µC (d) 20 µC.
455. The electric field at the centre of an uniformly charged
ring is zero. What is the electric field at the centre of
a half ring if the charge on it be Q and its radius be R ?
1 Q 1 Q
(a) × (b) ×
4 πε 0 πR 2 4 πε 0 R 2
1 2Q 1 2Q Fig. 164
(c) × (d) × .
4πε 0 πR 2 4 πε 0 R 2
(a) 320 µC ; 40 volt (b) 400 µC ; 50 volt
456. In fig. 162, what is the potential difference between (c) 213 µC ; 27 volt (d) 360 µC ; 45 volt.
the point A and B and between B and C respectively
459. A charge q is uniformly distributed over the volume
in steady state ?
of a sphere of radius R. Assuming the dielectric con-
3 mF 1 mF stant to be unity throughout, the potential at the
B
centre of the sphere will be
9q 3 1 q
3 mF 1 mF (a) (b) . .
4πε 0R 2 4 πε 0 R
7q 1 1 q
(c) (d) . . .
1 mF 4πε 0R 7 4πε 0 R
10 W [Vardhman 2001]
460. Two equal point charges are fixed at x = – a and
x = + a on the x-axis. Another point charge Q is placed
A 20 W C
100 V at the origin. The change in the electrical potential
Fig. 162
energy of Q, when it is displaced by a small distance
x along the x-axis, is approximately proportional to
ELECTROSTATICS 81
(a) x (b) x2 R2 − R1 R2 + R1
(a) (b)
(c) x3 (d) 1/x. R 1R 2 R 1R 2
[IIT Screening 2002] R 1R 2 R 1R 2
(c) (d) .
461. Two identical capacitors have the same capacitance R1 + R2 R2 – R1
C. One of them is charged to potential V1 and the [MP PET 2001]
other to V2 . The negative ends of the capacitors are 466. Consider a parallel plate ca-
connected together. When the positive ends are also pacitor of capacity 10 µ F (mi-
connected, the decrease in energy of the combined cro-farad) with air filled in the
system is gap between the plates. Now
1 1 one half of the space between
(a) C(V12 – V22) (b) C(V12 + V22)
4 4 the plates is filled with a di-
1 1 electric of dielectric constant
(c) C(V1 – V2)2 (d) C(V1 + V2)2.
4 4 4, as shown in Fig. 166. The
[IIT Screening 2002] capacity of the capacitor
changes to Fig. 166
462. Two identical metal plates are given positive charges
Q1 and Q2 ( < Q1 ) respectively. If they are now brought (a) 25 µF (b) 20 µF
close together to form a parallel plate capacitor with (c) 40 µF (d) 5 µF. [MP PET 2001]
capacitance C, the potential difference between them 467. Electric potential is given by : V = 6x – 8xy2 – 8y +
is 6yz – 4z2. Electric field at the origin is
(a) (Q1 + Q2)/(2C) (b) (Q1 + Q2)/C
(a) − 6i + 8 j (b) 6i − 8 j
(c) (Q1 – Q2)/C (d) (Q1 – Q2)/(2C).
(c) i + j (d) zero.
[IIT Screening 1999]
468. Figures given below represent roughly the electric
463. Two capacitors C1 = 2 C1 C2
lines of force due to three positive point charges of
µF and C2 = 6 µ F in
equal value. Guess the possibly correct representa-
series, are connected
tion.
in parallel to a third
capacitor C3 = 4 µ F.
This arrangement is C3
then connected to a
battery of emf = 2V, 2V
as shown in Fig. 165.
How much energy is
lost by the battery in Fig. 165
charging the capacitors ?
(a) 22 × 10 –6 J (b) 11 × 10 –6 J

FG 32 IJ × 10 FG 16 IJ × 10 ( a) (b)
H3K H3K
(c) –6 J (d) –6 J.

[MP PET 2001]


464. A capacitor of capacitance C1 = 1 µ F can withstand
maximum voltage V1 = 6 kV (kilo-volt) and another
capacitor of capacitance C2 = 3 µ F can withstand
maximum voltage V2 = 4 kV. When the two capaci-
tors are connected in series, the combined system
can withstand a maximum voltage of
(a) 4 kV (b) 6 kV
(c) 8 kV (d) 10 kV. [MP PET 2001] (c) (d)
465. A solid conducting sphere of radius R1 is surrounded
by another concentric hollow conducting sphere of Fig. 167
radius R2 . The capacitance of this assembly is pro-
portional to
82 COMPREHENSIVE OBJECTIVE PHYSICS

469. Electric charge is uni- (a) 3 × 10 –3 V m–1 (b) 9 × 10 –3 V m–1


formly distributed along (c) 27 × 10 –3 V m–1 (d) zero.
a long straight wire of 476. A charge q is placed at each corner of a cube of side a.
radius 1 mm. The The potential at the centre of the cube is
charge per centimetre
1 16q 1 8q
length of the wire is Q (a) (b)
4 πε 0 3 a 4 πε 0 3 a
coulomb. Another cylin-
1 4q
drical surface of radius (c) (d) zero.
50 cm and length 1 m 4 πε 0 3 a
symmetrically encloses 477. A 20 µ F capacitor is charged by a constant current
the wire as shown in Fig. of 10 m A. If the capacitor is initially uncharged,
168. The total electric how long does it take for the potential difference across
flux passing through the the capacitor to reach 300 V ?
cylindrical surface is (a) 6 × 10 – 4 s (b) 0.6 s
Q 100Q Fig. 168 (c) 15 s (d) 1.5 × 104 s.
(a) (b)
ε0 ε0 478. An isolated solid metal sphere of radius R is given
10Q 100Q an electric charge. Which one of the following best
(c) (d) . represents the way in which the density of charge
πε 0 πε 0
varies with distance r from the centre of the sphere ?
470. A hollow conducting sphere of radius R has charge
(+ Q) on its surface. The electric potential within the
R
sphere at a distance r = from the centre is
3
1 Q
(a) zero (b)
4πε 0 r σ σ
1 Q 1 Q
(c) (d) .
4πε 0 R 4πε 0 r 2
R R
471. Two spheres A and B of radii a and b respectively are r r
at the same electric potential. The ratio of the sur-
face charge densities of A and B is ( a) (b)
a b
(a) (b)
b a
a2 b2
(c) (d) .
2
b a2
472. The electric dipole moment of HCl molecule is σ σ
3.2 × 10 –30 Cm. The distance between the ions is
(a) 2 × 10 –11 m (b) 2 m
R R
(c) 2 mm (d) 0.09 mm. r r
473. Two spheres have surface densities of charge in the (c) (d)
ratio 1 : 8. The charges are in the ratio 2 : 9. The
Fig. 169
ratio of their radii is
(a) 4 : 3 (b) 3 : 7 479. The total electric flux leaving spherical surface of ra-
(c) 4 : 9 (d) 4 : 13. dius 1 cm and surrounding an electric dipole is
474. In a typical lightning flash, the potential difference q
(a) (b) zero
between discharge points is 109 V. The quantity of ε0
charge transferred is 30 C. The energy released is 2q
(c) (d) infinite.
(a) 3 × 10 –8 J (b) 3 × 1010 J ε0
(c) 9 × 1018 J (d) 3 × 1018 J. [Here q represents either charge of the dipole.]
475. A point charge is placed on the y-axis at y = 10 cm. If 480. A capacitor when filled with a dielectric K = 2 has
the electric field at the point y = 40 cm is 27 × 10 –3 charge Q0 , field E0 and voltage V0 . If the dielectric is
Vm–1, then the electric field at y = 100 cm is replaced with another one having K = 4, the new
values of charge, field and voltage will be respectively
ELECTROSTATICS 83
E0 V0 Q0 V (a) 50 pF (b) 75 µF
(a) Q0, , (b) , E0 , 0
2 2 2 2 (c) 100 µF (d) 150 pF.
(c) Q0, E0, 2V0 (d) None of these.
488. Condenser A has a capacity of 15 µ F when it is filled
481. Charges of 1C, – 2C, 3C, – 4C and 5C are placed at with a medium of dielectric constant 15. Another con-
the five corners of a regular hexagon. What charge denser B has a capacity 1 µ F with air between the
placed at the sixth corner would make the potential plates. Both are charged separately by a battery of
zero at the centre of the hexagon ? 100 V. After charging, the battery is disconnected
(a) – 3C (b) + 3C and then the dielectric is removed. Now, the two ca-
(c) + 6C (d) None of these. pacitors are connected in parallel. The common po-
482. Twenty seven identical liquid drops, each charged to tential will now be
a potential of 4 V combine to form a big drop. The (a) 400 V (b) 800 V
potential of the big drop is (c) 1200 V (d) 1600 V.
(a) 4 V (b) 16 V 489. Four particles, each hav-
(c) 24 V (d) 36 V. ing a charge q, are placed
483. Given : a metallic sphere of radius 90 cm. Its capac- at the four vertices of a
ity is regular pentagon. The dis- C
(a) 10.9 F (b) 100 pF
tance of each corner from
the centre is a. The elec-
(c) 900 pF (d) 1800 pF.
tric field at the centre of
484. A dielectric slab of dielectric constant K = 2 is cov- the polygon is Fig. 170
ered from all sides with a metallic foil. This system q q
is introduced into the space of a parallel plate capaci- (a) 2 (b)
4 πε 0 a πε 0 a2
tor of capacitance 2 µ F. The slab fills almost the en-
tire space between the plates but does not touch the 2q q
(c) (d) .
plates. The capacitance will become nearly πε0 a2 πε 0 a
(a) 2 µF (b) 4 µF 490. A cylinder of radius r and length l is placed in an
(c) zero (d) infinite. uniform electric field in such a way that the axis of
485. Two concentric thin metallic spheres of radii R1 and the cylinder is parallel to the field. The flux of the
R2 (R1 > R2 ) carry charges Q1 and Q2 respectively. field through the cylindrical surface is
The potential at point P (distance from common cen- 2rl l
tre is r) is : [Given : R1 < r < R2] (a) (b)
ε0 ε0
Q1 Q2 Q1 Q 2 2πrl
(a) + (b) + (c) (d) zero.
R1 R 2 r R2 ε0
491. A charge q is distrib-
Q1 Q 2 Q Q uted uniformly on a
(c) + (d) 1 + 2 .
R1 r R2 r ring of radius r. A
sphere of equal ra-
486. Two condensers of capacities 0.3 µ F and 0.6 µ F are
dius r is constructed
connected in series. The combination is connected
with its centre at the
across a battery of emf 6 V and zero internal resist-
periphery of the ring
ance. The ratio of the energies stored by the two con-
[see Fig. 171]. The
densers is
electric flux through
2 4 Fig. 171
(a) (b) the surface of the
1 1 sphere is
6 8 q 2q
(c) (d) . (a) (b)
1 1 ε0 ε0
487. A parallel plate capacitor has separation d and a ca-
q q
d (c) (d) .
pacitance of 100 pF. If a metallic foil of thickness 2ε0 3ε 0
3
is introduced between the plates, then the capacitance 492. Which one of the following methods will reduce the
would become capacitance of a parallel plate capacitor ?
84 COMPREHENSIVE OBJECTIVE PHYSICS

(a) Introducing a dielectric slab 496. A potential difference of 150 V is applied to two par-
(b) Introducing a metal plate of suitable thickness allel metal plates. If an electric field of 5000 Vm–1 is
(c) Reducing the potential difference produced between the plates, the plate separation is
(a) 1 cm (b) 2 cm
(d) Connecting another capacitor in series.
(c) 3 cm (d) 4 cm.
493. Fig. 172 shows three arrangements of an electron e
and two protons p, p. Which of the following is incor- 497. A capacitor of capacitance C is charged to a potential
rect ? V. The flux of the electric field through a closed sur-
face enclosing the capacitor is
D CV
(a) zero (b)
ε0
d d D
2CV CV
p e p (c) (d) .
e p p ε0 2ε 0
(a) (b)
498. Fig. 173 shows four situations in which charged par-
e D
ticles are at equal distances from the origin. The mag-
p
nitude of the net electric field at the origin is
d
Y Y
– 5q – 5q
p
(c)
2q – 3q 3q – 2q
Fig. 172
O X O X
(a) The net electrostatic force on the electron due to the pro-
tons is largest in arrangement b.
(b) The net electrostatic force on the electron due to the pro-
(a) (b)
tons is smallest in arrangement b.
(c) In arrangement c, the angle between the net electrostatic
Y Y
force on the electron and the line labelled d is less than –q
45°.
(d) The net electrostatic force on the electron due to the pro-
4q –q q – 4q
tons is largest in arrangement a.
O X O X
494. A parallel plate capacitor is connected to a battery. A
metal sheet of negligible thickness is placed between
the plates. The sheet remains parallel to the plates of 5q 4q
the capacitor. Which of the following is correct ?
(c) (d)
(a) The battery will supply more charge.
(b) The capacitance will increase. Fig. 173
(c) The potential difference between the plates will increase.
(a) same in all situations (b) maximum in (a)
(d) Equal and opposite charges will appear on the two faces of (c) maximum in (b) (d) same in (c) and (d).
the metal plate.
499. A parallel plate capacitor is charged by a battery
495. The electric field outside a charged long straight wire which is then disconnected. Then, the distance be-
5000 tween the plates is decreased. Which of the following
is given by E = – Vm–1. It is radially inward.
r is incorrect ?
The value of VB – VA is : [Given rB = 60 cm and rA = (a) The charge increases.
30 cm]
(b) The potential difference decreases.
(a) 5000 loge 2 volt (b) 0 V (c) The electric field remains unchanged.
(c) 2 V (d) 2500 V. (d) The stored energy decreases.
ELECTROSTATICS 85

Answers (Set IV)


427. (b) 428. (d) 429. (c) 430. (c) 431. (b) 432. (d) 433. (c) 434. (d)
435. (c) 436. (a) 437. (b) 438. (d) 439. (d) 440. (a) 441. (b) 442. (a)
443. (b) 444. (b) 445. (b) 446. (c) 447. (a) 448. (b) 449. (b) 450. (d)
451. (a) 452. (d) 453. (c) 454. (c) 455. (c) 456. (c) 457 (a) 458. (c)
459. (b) 460. (b) 461. (c) 462. (d) 463. (b) 464. (c) 465. (d) 466. (a)
467. (a) 468. (d) 469. (b) 470. (c) 471. (b) 472. (a) 473. (a) 474. (b)
475. (a) 476. (a) 477. (b) 478. (a) 479. (b) 480. (a) 481. (a) 482. (d)
483. (b) 484. (d) 485. (c) 486. (a) 487. (d) 488. (b) 489 (a) 490. (d)
491. (d) 492. (d) 493. (a) 494. (d) 495. (a) 496. (c) 497 (a) 498. (a)
499. (a)

Solutions (Set IV)


Q2 40 × 40 × 10−12 430. qE = mg
427. U= = J
2C 2 × 10 × 10− 6 qσ
= mg
= 80 × 10– 6 J = 80 × 10–6 × 107 erg ε0
= 8 × 102 erg. qQ ε A mg
= mg or q = 0
428. Distance of corner of square from centre of square ε0 A Q
2 × 0.08 8.85 × 10−12 × 2 × 10−2 × 2.5 × 10 −7 × 10
= m = 0.04 2 m = C
2 5 × 10−7
Potential at the intersection of the diagonals = 8.85 × 10–13 C.

4 × 9 × 109 ×
10
× 10 −9 FG q + q + q + ......IJ − FG q + q + q + ......IJ
= 3 volt
431. V=
H 1 4 16 K H 2 8 32 K
0.04 × 2 q
12000 q 2
= × 2 volt = 1500 2 volt. V= −
4×2 1 1
1− 1−
4 4
1 1 q1q2 1 q1q2
429. mv2 + = 4q 4q 8q − 4q 4q 2q
2 4πε 0 r2 4 πε 0 r1 V= − = = =
3 6 6 6 3
1
mv2 =
1 LM
1 1 OP q FGq q IJ
or
2 4 πε 0
q1q2 −
N
r1 r2 Q 432. E=
12 H
+ 2 +
4 162
+ ......
K
1 FG q + q + q IJ
or
2
× 0.002 × v2 −
H 2 8 32
2 2 2
+ ......
K
q
=
9 × 109 × 10− 6 × 10−3 1

LM
1 OP q 4
1 1 10 N Q =
1

1− 1−
1
16 16
2×9×9
or v2 = 16 q q 16 16 q 3 4 q
0.002 × 10 = − × = × = .
15 4 15 15 4 5
1 1 433. Refer to Comprehensive Physics for class XII.
or v2 = 2 × 9 × 9 × −3
×
2 × 10 10 434. 10 µF, 5 µF, and 9 µF are in parallel. The combined
or v2 = 9 × 9 × 102 or v = 90 m s–1 capacity of the parallel combination is 24 µF.
Now, 12 µF, 24 µF and 8 µF are in series.
86 COMPREHENSIVE OBJECTIVE PHYSICS

1 1 1 1 1 1 1 1
∴ = + + 439. = + +
C 12 24 8 C 20 8 12
1 12 + 6 + 18 1 6 + 15 + 10 120
or = or = or C = µF
C 144 C 120 31
144 120
or C= µF or C = 4 µF Now, Q = CV = × 300 µC
36 31
Total charge = 4 µF × 60 V = 240 µC Charge across 4 µF
240 µC 120 × 300
Again, voltage across 24 µF is or 10 V = µC = 580.6 µC
24 µF 31 × 2
∴ Charge on 5 µF capacitor Voltage across 4 µF
= 5 µF × 10 V = 50 µC. 580.6
= V = 145.15 V.
ε A 4
435. C12 = 0
d 440. Total energy before connection
ε 0 AV 1
q1 = C12V = = × 4 × 10–6 × 50 × 50
d 2
ε0 A 1
Now, C23 = + × 2 × 10– 6 × 100 × 100
d 2
ε0 A
C34 = = 5 × 10–3 + 10 × 10–3
d = 15 × 10–3 = 1.5 × 10–2 J
C23 and C34 are connected in parallel.
If V be the common potential after connection, then
2ε 0 A
∴ C14 = (4 + 2)V = 4 × 50 + 2 × 100
d
2ε 0 AV or 6V = 200 + 200
Charge on plate 4 = −
d 400 200
or V= volt or V = volt
This charge is negative because plate 4 is connected 6 3
to the negative terminal of the source of emf. Total energy after connection

436.
5V
I= =1A 1 FG IJ
200
2

5Ω =
2
(4 + 2)
3H K × 10− 6
Voltage across 4 Ω = 4 × 1 volt = 4 volt
200 × 200
Combined capacity of series combination of 3 µF and =3× × 10− 6
3×3
3
3 µF = µF 40000
2 = × 10– 6 = 1.33 × 10–2 J.
3
3
Charge = µF × 4 volt = 6 µC K ε0 A
2 441. = 4πε0 R
d
4σ (Surface charge density)2
437. = KA
R 2ε 0 R=
4πd
4σ 1 Q2 6 × 100 × 10−4 × 7
= = m = 4.77 m
R 2ε 0 16 π 2R 4 4 × 22 × 1 × 10−3
or Q2 = 128 ε0 π2R3σ 442. Let a and b represent the radii of the sphere and shell
respectively.
or Q = 128ε0 π 2R 3σ .
1 Q 1 Q
E V= −
438. I= 4 πε 0 a 4πε 0 b
R2 + r
ER2 or V=
Q LM1 1

OP
Voltage across R2 = IR2 =
R2 + r N
4 πε 0 a b Q
Q = CV When charge ‘– 3Q’ is given to the shell,

Q =
CE R 2
. V′sphere =
1 LM
Q − 3Q
+
OP
R2 + r 4 πε0 aN b Q
ELECTROSTATICS 87

or V′shell =
1 LM
Q − 3Q
+
OP 445.
Qq
+
q2 Q q
+ =0
4 πε0 b N b Q 2a a a
Q
Now, V′sphere – V′shell or +q+Q=0

=
1 LM
Q 3Q Q 3Q
− − +
OP 2
LM OP
4πε0 a Nb b b Q or Q
1
+1 =–q
N Q
1 LM Q − Q OP = V . 2
=
4πε 0 Na bQ or Q=
1+ 2
− 2q
or Q =
− 2q
2+ 2
443. Net flux = E0(x + l)l2 – E0xl2 = E0l3
1
446. q = 600 0.2 × 0.1 × 0.1 – 600 0.1 × 0.1 × 0.1
ε0
q = 8.85 × 10–12 × 6[ 0.2 − 0.1 ]
q = 53.1 × 10–12 [0.447 – 0.316]
q = 53.1 × 0.131 × 10–12 C
q ≈ 7 µµ C.
O 1 q q LM OP
1 3q − q
x 447. V=
4 πε0 r 3r

N or V =
Q
4 πε0 3r
x+l 1 2q
or V=
4 πε 0 3r
Fig. 174 1 q
Again, E=
4 πε 0 9r 2
Using Gauss’s law,
E 1 3 1 V
1 = × = or E =
q = E0l3 or q = ε0 E0l3 V 9r 2 6r 6r
ε0 448. Potential at the centre of ring II
ε A/2 K ε A
444. C1 = K1 0 or C1 = 1 0 √2 R
d/2 d
K ε A
C2 = 2 0 R
d
K ε A 2K 3ε 0 A
C3 = 3 0 =
d/2 d
I II
ε A
C12 = C1 + C2 = 0 (K1 + K2) Fig. 175
d
C12C3 1 Q2 1 Q1
Now, C123 =
C12 + C3 = +
4πε 0 R 4 πε 0 2R
ε0 A 2K 3ε0 A Potential at the centre of ring I
(K 1 + K 2 )
or C123 = d d 1 Q1 1 Q2
ε0 A 2K 3 ε 0 A = +
(K 1 + K 2 ) + 4πε 0 R 4πε 0 2R
d d
Potential difference
2K 3ε 0 A
or Kε 0 A
=
(K 1 + K 2 )
d =
1 Q1LM
+
Q2 Q
− 2 −
Q1 OP
d K 1 + K 2 + 2K 3 4πε 0 R N 2R R 2R Q
2K 3 (K 1 + K 2 ) =
1 LM Q 1 − Q2 Q1 − Q2

OP
or K=
K 1 + K 2 + 2K 3 4πε 0 N R 2R Q
1 K 1 + K 2 + 2K 3 1 (Q 1 − Q 2 )( 2 − 1)
or = =
K 2K 3 (K 1 + K 2 ) 4 πε 0 2R
1 1 1 q(Q1 − Q2 )( 2 − 1)
or = + . Work done =
K 2K 3 K 1 + K 2 2 4πε 0R
88 COMPREHENSIVE OBJECTIVE PHYSICS

1 Q2 1 qQ (iv) Energy given by battery = (CV)V = CV 2.


449. 2
=− 2× 453. C = 1000 µF
4 πε 0 2 a 4 πε 0 a2
V = 105 V
q a
Q 1
U= × 1000 × 10–6 × 1010 J = 5 × 106 J
2
Now, 1 kWh = 103 × 3600 J
a √2 a a
108
Cost = × 5 × 106 paise
103 × 3600
= 150 paise.
Q
a
q 454. Q = 6 µF × 20 V = 120 µC
Q2 C2
Fig. 176 Now, =
Q1 C1
Q
or = − 2 q or Q = −2 2 q Q2
=
C2 Q2 3
2 or or =
Q 1 + Q 2 C1 + C2 120 9
450. q1 = 4πr2σ, q2 = 4πR2σ
or Q = 40 µC.
Q = 4πσ(r2 + R2)
455. ΣdE sin θ = 0
1 q2 1 q1
V= +
4 πε0 R 4πε 0 r
1 4 πR 2σ 1 4 πr 2σ
= +
4 πε0 R 4 πε0 r
4πσ 1 Q
= (R + r) = (R + r)
4πε 0 ε0 4 π(R 2 + r 2 )
Q(R + r)
=
4πε 0 (R 2 + r 2 )
2ε 0 A/2 3ε 0 A/2
451. In fig. (a) C = +
d d
ε0A 5ε 0 A Fig. 177
or C= (2 + 3) or C =
2d 2d
π

z
In fig. (b) +

FG 2ε A IJ FG 3ε A IJ
2
E= d E cos θ
H d/2 K H d/2 K
0 0
π
C′ = −
2 ε 0 A 3ε 0 A 2
+ π

z
d/2 d/2 +
2
ε0A 1 dq
6 E= cos θ
d/2 4 πε 0 R 2
C′ = −
π
5 2
π

z
12ε0 A +
C′ = 2
5d 1 dl Q
= cos θ
C 5ε 0 A 5d 25 4πε 0R 2 πR
= × = −
π
C′ 2d 12ε 0 A 24 2
π

z
452. Following arguments shall decide the right choice. +
2
(i) Distance is halved. Capacity is doubled. R Rdθ cos θ
=
(ii) Since charge remains the same and capacity is 4π ε 0R 2
2
π
R

V 2
doubled therefore potential become . π
2 +
(iii) To restore potential to V, an additional charge CV Q 2
= sin θ
should be given to the capacitor. 4π ε 0R 2
2

π
2
ELECTROSTATICS 89

Q LM
π π FG IJ OP = 1 2Q ∴ V18 =
2
× 40 =
80
V ≈ 27 V
= 2
4π ε 0R 2
sin − sin −
2N 2 H K Q 4πε 0 πR 2
3 3
456. 3 µF and 3 µF are in par- 80 640
Q8 = 8 µF × V= µC ≈ 213 µC
allel. So, the combined 3 3
capacity is 6 µF. 1 µF and 459. Potential at the centre due to thin spherical shell of
1 µF are in parallel. So, radius r and thickness dr, dV
the combined capacity is
1 4 πr 2 dr ρ ρ
2 µF. = = r dr
4 πε 0 r ε0
Now, 6 µF and 2 µF are
in series. Their combined Fig. 178 Potential due to given sphere

z
R R
6×2 12 3 ρ ρ r2
capacity is µF i.e., µF or µF. It is in = r dr =
6+2 8 2 ε0 ε0 2
parallel with 1 µF. Thus, the combined capacity is 0 0

FG 3 + 1IJ µF i.e., 5 R2
2
ρ R 1 q
= =
H2 K 2
µF ε0 2 ε 0 4 πR 3 2
3
Now, Q = CV
3 1 q
=
5 2 4πε 0 R
µF × 100 V = 250 µC
=
2 1 2qq′
Charge in 6 µF – 2 µF branch 460. Ui =
4 πε 0 a
3
= µF × 100 V = 150 µC q q′ q
2 X′ X
–a O +a
150
∴ VAB = V = 25 V
6 Fig. 180
VBC = (100 – 25) V = 75 V
457. The equivalent of the given network is : Uf =
qq′ 1LM+
1 OP
N
4 πε 0 a + x a − x Q
∆U =
qq′ LM 1 + 1 − 2 OP
4 πε 0 N a + x a − x aQ
=
qq′ LM a(a − x) + a(a + x) − 2(a 2
− x2 ) OP
4 πε 0 MN 2
a( a − x ) 2
PQ
qq′ 2 x2
=
4 πε0 a(a2 − x 2 )
Fig. 179
Neglecting x2 in comparison to a2
This network is equivalent to a series combination of
qq′
three capacitors, each of capacity 3 µF. So, the com- ∆U = x2 or ∆U ∝ x2.
2πε0 a3
bined capacity is 3 µF i.e., 1 µF. 461. Loss of energy
3
458. In the given network, 6 µF, 4 µF and 8 µF are in
parallel. Their combined capacity is 18 µF. It is in
1 1 V + V2
C V12 + V22 − (2C) 1
LM OP 2

series with 36 µF. So, the combined capacity is


=
2 2 2 N Q
1× 2 C
18 × µF or 12 µF. = [2V12 + 2V22 – V12 – V22 – 2V1V2]
1+ 2 4
V18 36 2 C C
Now, = = = [ V12 + V22 − 2V1V2 ] = (V1 – V2)2
V36 18 1 4 4
90 COMPREHENSIVE OBJECTIVE PHYSICS

Q1 Q2 Q − Q2 470. Potential at any point within the conducting sphere


462. E= − = 1 ; is the same as on the surface of the sphere.
2ε 0 A 2ε0 A 2ε 0 A
(Q1 − Q2 )d Q1 − Q2 1 Q 1 Q σR
V = Ed = = 471. V= = R= ;
2ε 0 A 2C 4 πε0 R ε0 4 πR 2 ε0
12 3 1 σA b
463. C12 = µF = µF V constant ; σ ∝ ; = .
8 2 R σB a
1 3 1
U = × × 10− 6 × 2 × 2 + × 4 × 10− 6 × 2 × 2 p 3.2 × 10−30
2 2 2 472. p = qd or d = = m = 2 × 10–11 m.
q 1.6 × 10−19
U = 3 × 10–6 + 8 × 10–6
U = 11 × 10–6 J q q r q1 σ2 2 8 4
473. σ= or r ∝ ; 1 = × = × = .
An equivalent amount of energy is lost. 4 πr 2 σ r2 σ1 q2 9 1 3
464. Q1 = 6 × 10–3 coulomb 474. W = qV = 30 C × 109 V = 3 × 1010 J.
Q2 = 12 × 10–3 coulomb 475. In the first case, r1 = 30 cm. In the second case, dis-
Clearly, the charge on the series combination cannot tance is 90 cm. Distance is increased by a factor of 3.
exceed 6 × 10–3 coulomb. Electric field is reduced by a factor of 9.
6 × 10 −3 476. Diagonal of cube = 3a . Distance from centre of cube
Now, V = volt or V = 8 kV.
1× 3
× 10 − 6 3a
1+ 3 to corner is . Potential at the centre due to cor-
2
465. Refer to Comprehensive Physics for class XII.
1 2q
466. C = (5 + 4 × 5) µF = 25 µF. ner charge is . Potential at the centre due
4 πε 0 3 a
∂V ∂ 1 16 q
467. = [6x – 8xy2 – 8y + 6yz – 4z2] to all charges is .
∂x ∂x 4 πε 0 3 a
Here y and z are
to be treated as constants CV CV 20 × 10 −6 × 300
477. I= or t = = s = 0.6 s.
∂ ∂ t I 10 × 10 −3
=6 ( x) − 8 y2 ( x) 478. Charge resides on the outer surface of the metal
∂x ∂x
= 6 – 8y 2 sphere.
FG ∂V IJ 479. Net charge of dipole is zero.
H ∂x K origin
=6
Here x and z are
480. Think in terms of an equivalent situation. Initially,
air. Finally a dielectric of dielectric constant 2.
∂V constants 481. Total given charge is 3C. If the sixth charge is ‘– 3C’,
Again, = – 16x – 8 + 6z
∂y then the net charge would be zero.
FG ∂V IJ 482. [27]2/3 = 32 = 9 ; V′ = 9 × 4 V = 36 V.
H ∂y K origin
=–8
0.9
483. C = 4πε0R = F = 10–10 F
Again,
∂V
= 6y – 8z Here x and y are 9 × 109
∂z = 102 × 10–12 F = 100 pF.
FG ∂V IJ constants
E0
H ∂z K origin
=0 484. K=
E
, E = 0 ∴ K = ∞.

→ ∂V  ∂V  ∂V  485. The observation point is at a distance r from the cen-


Now, E=− i− j− k tre of the inner sphere of radius R2. But it is inside
∂x ∂y ∂z
the sphere of radius R1.
= − 6i − (− 8 j) = − 6i + 8 j .
V1 C2 0.6 2
468. (a) Wrong. Electric lines of force cannot be closed loops 486. Q = CV ; Q same ; CV = constant ; = = =
V2 C1 0.3 1
(b) Wrong. Electric lines of force cannot end on positive
charge 2 1
Clearly, V1 = × 6 V = 4 V and V2 = × 6 V = 2 V.
(c) Wrong. Electric lines of force cannot intersect. 3 3
1
469. Total enclosed charge q = 100 Q coulomb × 0.3 × 4 × 4
Now, U 2 2
q 100 Q
1
= = .
U2 1
φE = = × 0.6 × 2 × 2 1
ε0 ε0 2
ELECTROSTATICS 91

d 2d 2 492. In (a) and (b), capacitance increases. In (c), potential


487. Effective distance = d – = = × initial distance ; difference has no effect on capacity.
3 3 3
3 3 493. The forces due to the two protons add up.
New capacity = × initial capacity = × 100 pF
2 2 494. Think of electrostatic induction.

z
= 150 pF. rB rB
488. Q1 = 1500 µC, Q2 = 100 µC, − 5000
495. VB – V A = − dr = 5000 log r
15 r
rA rA
C1 = µF = 1 µF, C2 = 1 µF
15
Q + Q2 1600 µC = 5000 [loge rB – loge rA] = 5000 log e rB
Now, V= 1 = = 800 V. rA
C1 + C2 2 µF
60
489. By symmetry, the field at the centre would be zero if = 5000 loge = 5000 loge 2 volt.
there is a charge q at the vacant corner. So, electric 30
field due to this imaginary charge must be equal and V V
496. E=
or d =
opposite to the combined electric field of all the four d E
given charges. 150 15000
→ → or d= m or d = cm = 3 cm.
490. ∆S on cylindrical surface perpendicular to E . 5000 5000
→ → 497. Net charge enclosed is zero.
∴ Σ E . ∆S = 0 . 498. Determine fields in each case.
491. Note that the angle at O1 is 120°. Since only one-
499. When distance is decreased, capacity is increased ; V
third of the ring is inside the sphere therefore the
is constant ; charge increased.
q
charge enclosed by the sphere = . Now, use Gauss’s
3
law.

KNOWLEDGE PLUS
O The electric field due to a uniformly charged sphere of radius R as a function of the distance from its centre is
represented graphically by :

E E E E
(a) (b) (c) (d)

O R r O R r O R r O R r

Fig. 180

[AIIMS 2004]
1
Solution. Inside the sphere, E ∝ r. Outside the sphere, E ∝ . So, (b) is the right choice.
r2

O A fully charged capacitor has a capacitance ‘C’. It is discharged through a small coil of resistance wire embedded
in a thermally insulated block of specific heat capacity ‘s’ and mass ‘m’. If the temperature of the block is raised by
‘∆T ’, the potential difference ‘V ’ across the capacitance is
2 ms∆T ms∆T mC∆T 2 mC∆T
(a) (b) (c) (d) [AIEEE 2005]
C C s s
Ans. (a)

Explanation. 1 C V 2 = ms∆T or V = 2 m s ∆T
2 C
92 COMPREHENSIVE OBJECTIVE PHYSICS

SELF-EVALUATION TEST I
Based on UNIT XI

[Expected Questions for Forthcoming Examinations]

1. A point charge q is situated at X between two parallel 3. A gold nucleus (radius r) is represented by the
plates which have a potential difference V and carry symbol 197 Au. Taking e as the elementary charge and
79
charges + Q and – Q. What is the electric field ε0 as the permittivity of free space, what is the elec-
strength at X ? tric field strength at the surface of an isolated gold
nucleus ?
79 e
(a) zero (b)
4 πε0 r 2

197 e 79 e2
(c) (d)
2
4 πε 0 r 4 πε0 r 2

197 e2
(e) .
4 πε0 r 2

4. In an experiment to demonstrate Coulomb’s law in


Fig. 181
electrostatics, the force F between two small charged
V Vq spheres is measured for various distances r between
(a) (b)
d d their centres. A graph is plotted of log F (y-axis)
Q qQ against log r(x-axis). The slope of this graph is
(c) (d) .
4 πε 0 a2 4 πε 0 a2 (a) – 2 (b) + 1/2
2. A positively-charged plate P1 is brought close to a (c) – 1/2 (d) + 2
coplanar and earthed metal plate P2 which is ini- (d) – log 2.
tially uncharged. 5. In the direction indicated by an electric field line,
(a) the electric field strength must increase
(b) the electric field strength must decrease
(c) the potential must remain constant
(d) the potential must increase
(e) the potential must decrease.
6. 20 erg of work is done in moving 4 esu of positive
charge from infinity to a point A in an electric field.
The potential at A is
Fig. 182
(a) 5 esu (b) 20 esu
Which diagram shows the induced charges on P2 ? (c) 80 esu (d) 15 esu.
7. What is the magnitude of the electric field strength
– – + + at a distance r from an isolated stationary nucleus of
– – + +
– – + + proton number (atomic number) Z ?
– – + +
– – + + Ze (Ze)2
(a) (b)
– – + + 4 πε 0 r 4πε0 r 2
Ze
(c) (d) zero
4 πε 0 r 2
(a) (b) (c) (d)
Ze2
(e) .
Fig. 183 4πε 0 r
ELECTROSTATICS 93

8. The diagram shows electric field lines around two 12. 64 drops, each having the capacity C and potential
isolated point charges P and Q. At X, the field V, are combined to form a big drop. If the charge on
strength is zero. the small drop is q, then the charge on the big drop
will be
(a) 2q (b) 4q
(c) 16q (d) 64q.
13. Which of the following plates should be inserted be-
tween the plates of a parallel plate capacitor in order
to obtain maximum capacitance ?
P Q
x (a) 4 mm thick glass plate (K = 7)
(b) 0.20 mm thick mica plate (K = 6)
(c) 2.0 cm thick amber plate (K = 2)
(d) All will give the same capacity.
14. Fig. 185 shows some
of the electric field
Fig. 184
lines corresponding
Which of the following statements is true ? to an electric field.
(a) Q is a smaller charge than P because X is closer to P than Q The figure suggests
that Fig. 185
(b) Field strength is always proportional to the distance from
X (a) EA > EB > EC (b) EA = EB = EC
(c) The potential at Q is less than the potential at P (c) EA = EC > EB (d) EA = EC < EB.
(d) The field lines show that both charges are positive 15. A body acquires positive charge. This means that the
(e) The potential at X is zero.
body
(a) has gained protons (b) has gained positrons
9. If a body is charged by rubbing it, its weight
(c) has gained α-particles (d) has lost electrons.
(a) remains precisely constant [MGIMS 1997]
(b) increases slightly 16. Three point charges, as
(c) decreases slightly shown, are placed at the
(d) may increase slightly or may decrease slightly. vertices of an isosceles
right angled triangle.
[MGIMS 2002]
Which of the numbered
10. You are travelling in a car during thunder storm. In vectors coincides in direc-
order to protect yourself from lightning, you would tion with the electric field
prefer to at the mid-point of the hy-
Fig. 186
(a) remain in the car potenuse ?
(b) take shelter under a tree (a) 1 (b) 2

(c) touch the nearest electric pole (c) 3 (d) 4.

(d) get out of the car and stand in some open space. [National Standard Exam. in Physics 1993]
[JIPMER 2001] 17. A capacitor of capacity C has charge Q and stored
energy is W. If the charge is increased to 2Q, the
11. A parallel plate capacitor has charge Q coulomb, po-
stored energy will be
tential V volt and energy E joule. A dielectric slab is
(a) 2W (b) W/2
now inserted between the two plates. Then
(c) 4W (d) W/4.
(a) V and E both decrease
(b) V and E both increase 18. The capacity of a parallel plate air condenser is 2
µµF. If the distance between the plates is 4 cm and
(c) V decreases, E increases
the area of each plate is 0.01 m2, the value of permit-
(d) V increases, E decreases.
tivity of air and its units are respectively
[National Standard Exam. in Physics 1999]
94 COMPREHENSIVE OBJECTIVE PHYSICS

(a) 8 × 10–12 ; farad metre–1 25. Let V and E be the potential and the field respec-
(b) 5 × 10–13 ; farad metre–1 tively at a point. Which of the following assertions is
(c) 8 × 10–12 ; farad-metre correct ?
(d) 5 × 10–13 ; farad-metre. [MP PET 1990] (a) If V = 0, E must be zero. (b) If V ≠ 0, E cannot be zero.
(c) If E ≠ 0, V cannot be zero. (d) None of these.
19. The charge given to a hollow sphere of radius 5 cm is
20 stat coulomb. The potential inside it will be [AFMC 1998]
(a) Zero (b) 4 esu 26. The intensity of electric field at a point between the
(c) – 4 esu (d) 100 esu. plates of a charged capacitor
(a) is directly proportional to the distance between the plates
20. A pellet carrying charge of 0.5 coulomb is acceler-
ated through a potential of 2,000 volt. It attains a (b) is inversely proportional to the distance between the plates
kinetic energy equal to (c) is inversely proportional to the square of the distance be-
(a) 1000 erg (b) 1000 joule tween the plates
(c) 1000 kWh (d) 500 erg. (d) does not depend upon the distance between the plates.
21. A light bulb, a capacitor [MP PET 1996]
and a battery are con- 27. On bringing 0.4 coulomb of electric charge from in-
nected together as finity to the point P, the work done is 20 joule. Then
shown here, with switch the potential at P will be
S initially open. When (a) 50 volt (b) 8 volt
the switch S is closed, (c) 1000 volt (d) 160 volt.
which one of the follow-
Fig. 187 [Pb. PMT 1998]
ing is true ?
(a) The bulb will light up for an instant when the capacitor 28. If two conducting spheres are separately charged and
starts charging. then brought in contact, then
(b) The bulb will light up when the capacitor is fully charged. (a) the total energy of the two spheres is conserved
(c) The bulb will not light up at all. (b) the total charge on the two spheres is conserved
(d) The bulb will light up and go off at regular intervals. (c) both the total energy and charge are conserved

[MP PMT 1995] (d) the final potential is always the mean of the original
potentials of the two spheres.
22. Value of potential at a point due to a point charge is
[Bharti Vidyapeeth 2000]
(a) inversely proportional to square of the distance
29. A condenser of capacitance 10 µ F has been charged
(b) directly proportional to square of the distance
to 100 volt. It is now connected to another uncharged
(c) inversely proportional to the distance condenser in parallel. The common potential becomes
(d) directly proportional to the distance. [MP PET 1996] 40 volt. The capacitance of another condenser is
23. A spring-block system undergoes vertical oscillation (a) 15 µF (b) 5 µF
above a large horizontal metal sheet with uniform (c) 10 µF (d) 16.6 µF. [MP PET 1992]
positive charge. The time period of the oscillation is
30. A capacitor having capacitance C is charged to a volt-
T. If the block is given a charge Q, its time period of
age V. It is then removed and connected in parallel
oscillation will be
with another identical capacitor which is uncharged.
(a) T (b) > T The new charge on each capacitor is now
(c) < T (a) CV (b) CV/2
(d) > T if Q is positive and < T if Q is negative. (c) 2CV (d) CV/4. [MP PET 1990]
[MNR 1997] 31. The energy stored in a condenser of capacity C which
24. A charge of 10 –9 C is placed on each of the 64 identi- has been raised to a potential V is given by
cal drops of radius 2 cm. They are then combined to
1 1
form a bigger drop. The potential of the bigger drop (a) CV (b) CV 2
2 2
is
(a) 7.2 × 103 V (b) 7.2 × 102 V 1
(c) CV (d) . [RPMT 1999]
2VC
(c) 1.44 × 102 V (d) 1.44 × 103 V.
32. When two charged conductors are connected by a
[MP PET 1997]
thin wire, the change in electrical potential energy
will not occur, if
ELECTROSTATICS 95
(a) the capacity of two conductors is equal 40. A parallel plate capacitor is charged and the charg-
(b) the potential of two conductors is equal ing battery is then disconnected. If the plates of the
(c) the charge on the two conductors is the same capacitor are moved further apart by means of insu-
(d) for both the conductors, the value of C/V is the same.
lating handles, then
(a) the charge on the capacitor increases
[RPMT 2000]
(b) the voltage across the plates decreases
33. Two condensers of capacities 1 µ F and 2 µ F are con-
(c) the capacitance increases
nected in series and the system is charged to 120
volt. Then the potential difference on 1 µ F capacitor (d) the electrostatic energy stored in the capacitor increases.
will be [IIT 1987 ; MP PMT 1996]
(a) 40 V (b) 60 V 41. A table tennis ball which has been covered with con-
(c) 80 V (d) 120 V. [RPMT 2001] ducting paint is suspended by a silk thread so that it
hang between two plates, out of which one is earthed
34. Capacitors connected in series have
and other is connected to a high voltage generator.
(a) same difference of potential across each capacitor
This ball
(b) numerically the same charge on each plate of all capacitors
(a) is attracted towards high voltage plate and stays there
(c) C = C1 + C2 + C3
(b) hangs without moving
(d) None of the above. [TNPCEE 2001] (c) swings backward and forward hitting each plate in turn
35. What is the effect on soap bubble when some charge (d) is attracted to earthed plate and stays there.
is given to it ?
[All India PM/PD 1994]
(a) Its size is increased. (b) Its size is decreased.
42. An electron (charge = 1.6 × 10 –19 coulomb) is acceler-
(c) There is no effect on size. (d) The bubble collapses.
ated through a potential of 1,00,000 volt. The energy
[DCE 2001] acquired by the electron is
36. The electric field at the origin is along the positive X- (a) 1.6 × 10–24 joule (b) 1.6 × 10–14 erg
axis. A small circle is drawn with the centre at the (c) 0.53 × 10 –14
joule (d) 1.6 × 10–14 joule.
origin cutting the axes at points A, B, C and D hav-
[MP PET 1989]
ing co-ordinates (a, 0), (0, a), (– a, 0), (0, – a) respec-
tively. Out of the points on the periphery of the cir- 43. Two charges are placed a finite distance apart. If a
cle, the potential is minimum at glass slab is placed between them, force between them
(a) A (b) B
will
(a) be zero (b) increase
(c) C (d) D.
(c) decrease (d) remain the same.
[Bharti Vidyapeeth 2001]
44. If the potential difference across the ends of a capaci-
37. Work done in moving a positive charge on an equipo-
tor of capacity 4 µ F is 1.0 kilovolt, then its electrical
tential surface is
potential energy will be
(a) finite, positive but not zero
(a) 4 × 10–3 erg (b) 2 erg
(b) finite, negative but not zero
(c) 2 joule (d) 4 joule. [DCE 2001]
(c) zero
45. The energy of a charged conductor is given by the
(d) infinite. [Himachal PMT 1998]
expression (q = charge on the conductor and c = its
38. The work done in carrying a charge of 5 µC from a capacity)
point A to a point B in an electric field is 10 mJ. The
potential difference (VB – VA ) is then q2 q2
(a) (b)
(a) + 2 kV (b) – 2 kV 2c c
(c) + 200 V (d) – 200 V. (c) 2qc (d) q .
[Kurukshetra CEE 1996] 2c2
39. A hollow metallic sphere of radius 3 cm is charged so [MP PMT 1989]
that potential on its surface becomes 5 volt. The po- 46. Three capacitors of capacity 10 µ F, 5 µ F and 5 µ F
tential at the centre of sphere will be (in volt) are connected in parallel. The total capacity will be
(a) Zero (b) 5 (a) 10 µF (b) 5 µF
(c) 3 (d) 10. [MP PET 1995] (c) 20 µF (d) None of these.
[MP PET/PMT 1988]
96 COMPREHENSIVE OBJECTIVE PHYSICS

47. A particle has a mass 400 times than that of the elec- 49. Faraday’s ice-pail experiment shows that
tron and charge is double than that of a electron. It (a) charge resides on the outside of a conductor
is accelerated by 5 V of potential difference. Initially (b) a gold leaf electroscope deflects more if a charge similar to
the particle was at rest. Then its final kinetic energy its own charge is brought near it
will be (c) magnetism is a form of electrical phenomenon
(a) 5 eV (b) 10 eV (d) charge consists of an excess of or lack of electrons.
(d) 100 eV (d) 2000 eV.
[All India PM/PD 1994]
[MP PMT 1990]
50. Two equal positive charges are kept at points A and
48. The capacity of a conductor in the presence of dielec- B. The electric potential at the points between A and
tric medium B (excluding these points) is studied while moving
(a) increases with dielectric medium from A to B. The potential
(b) decreases with dielectric medium (a) continuously increases (b) continuously decreases
(c) proportional to the square root of dielectric constant (c) increases then decreases (d) decreases then increases.
(d) remains unchanged. [AMU 1996] [AIIMS 1998]

Answers
1. (a) 2. (a) 3. (b) 4. (a) 5. (e) 6. (a) 7. (c) 8. (d)
9. (d) 10. (a) 11. (a) 12. (d) 13. (b) 14. (c) 15. (d) 16. (c)
17. (c) 18. (a) 19. (b) 20. (b) 21. (a) 22. (c) 23. (a) 24. (a)
25. (d) 26. (d) 27. (a) 28. (b) 29. (a) 30. (b) 31. (b) 32. (b)
33. (c) 34. (b) 35. (a) 36. (a) 37. (c) 38. (a) 39. (b) 40. (d)
41. (c) 42. (d) 43. (c) 44. (c) 45. (a) 46. (c) 47. (b) 48. (a)
49. (a) 50. (d)

Solutions
1. The electric field strength in a parallel-plate capaci- 4. Force F between two small charged spheres at a dis-
tor with applied voltage of V and separation d is given tance r apart is given by
by 1 Q 1Q 2
F=
V 4 πε r 2
E= .
d where Q1 and Q2 are charges on the charged spheres.
2. Plate P2 is charged by the process of electrostatic in- FG Q Q IJ
H 4πε K
⇒ log F = log 1 2 – 2 log r
duction. The free induced positive charge is neutral-
ised by electrons flowing up from the Earth surface.
The graph of log F against log r is thus a straight line
of gradient equal to – 2.
 While negative induced charge is a bound
5. Since electric field strength E at a point is defined as
charge, the positive induced charge is a free charge.
dV
the negative of the potential gradient , we have
3. Electric field strength E at distance dx
r from the centre of the nucleus is dV
E= −
79e dx
E=
4 πε0r 2 Thus, in the direction indicated by an electric field
line (x-direction), the potential must decrease (dV ↓)
[Charge with the nucleus is 79e.]
for increasing x (∆x ↑).
Fig.188
ELECTROSTATICS 97
6. Potential is work done per unit charge. 14. Crowding of electric lines of force represents a strong
8. From the diagram, we can conclude that electric field.
15. Positive charge means deficiency of electrons.
16. Fields due to – Q and + Q add up to NW side ; that
due to 2Q has same value but on NE side.
Sum northward.
Q2
17. W =
Fig. 189 2C
(a) Q is the larger charge because X is closer to P Q is doubled, W is quadrupled.
than Q ε 0 × 0.01
18. 2 × 10–12 =
qP qQ q F I
r
2 4 × 10 −2
When EP = EQ =
4 πεrP 2
=
4 πεrQ2 qQ GH JK
, ∴ P = P
rQ 2 × 10 −12 × 4 × 10 −2
or ε0 = F m–1
Since rQ > rP, then qQ > qP 0.01
(b) Field strength is always proportional to the in- = 8 × 10–12 F m–1
verse squared distance from X as it can be readily
Q
seen from the expression in (a). 19. V=
(c) The potential at P or Q is infinitely large since the
C
potential at the location of an isolated charge is 20
V= esu = 4 esu
theoretically infinite. 5
(d) Both charges are positive since the field lines are 20. U = qV = 0.5 × 2000 joule = 1000 joule
radially outwards from the charges. 21. Initially capacitor acts like a short-circuit and finally
(e) The potential at X is not zero and is given by like an open switch.
qP qQ 23. Period of oscillation of a spring is independent of the
VX = + . ‘effective value of acceleration due to gravity’.
4 πεrP 4πεrQ
9. During rubbing, there is a transfer of electrons. 4 4
24. 64 × π × 23 = πR3
10. Electrostatic shielding is required. 3 3
11. On introducing the dielectric slab, C increases, Q re- R = 2 × 4 cm = 8 cm
mains constant, hence V decreases (Q = VC) and E
also decreases (E = Q2/2C). 9 × 109 × 64 × 10−9
Now, V= volt
12. Add up the charges on the individual drops. 8 × 10−2
ε0A = 7.2 × 103 volt
13. C=
t
d −t+ σ
K 26. E = ; Clearly, E is independent of the distance
ε0
ε 0A between the plates.
C=
FG t IJ
H
d− t−
K K 27. V =
W 20
= volt =
200
volt = 50 volt.
q 0.4 4
C would be minimum if t – t is minimum.
K 28. Conservation of charge.
4 24 29. 10 × 100 = (10 + C) 40
(a) 4 – = = 3.4
7 7 1000
or C + 10 = or C = (25 – 10) µF
0.2 40
(b) 0.2 – = 0.013
6 or C = 15 µF
20 V
(c) 20 – = 10 30. CV = (2 C)V′ or V′ =
2 2
Clearly, (b) is the right choice. CV
∴ New charge on each capacitor is .
2
98 COMPREHENSIVE OBJECTIVE PHYSICS

1 39. Potential at any point inside the sphere is the same


31. CV 2
U=
2 as on the surface of the sphere.
32. There would be no flow of charge between ‘equal 40. Work done against force of attraction between the
potentials’. plates increases the energy.
V1 C2 41. Due to electrostatic induction, the ball is attracted
33. =
V2 C1 towards one plate. When the ball touches the plate,
V1 C2 the charges are shared and there is repulsion. When
= it hits the other plate, charge is conducted to the earth.
V1 + V2 C2 + C1
42. U = qV = 1.6 × 10–19 × 105 J = 1.6 × 10–14 J.
V1 2 2
= or V1 = × 120 volt 1
120 2 + 1 3 43. F ∝ .
K
= 80 volt.
35. Electrostatic energy is added to the bubble. 1
44. U= × 4 × 10–6 × 103 × 103 J = 2 J.
36. Potential falls in the direction of the field. 2
46. Cp = C1 + C2 + C3
37. Potential difference between any two points on an
equipotential surface is zero. 47. K.E = (2e) 5V = 10 eV.

10 × 10 −3
38. VB – VA = volt = 2 kV.
5 × 10 −6

KNOWLEDGE PLUS
l A charge q is located at the centre of a cube. The electric flux through any face is :
2 πq 4 πq
(a) (b)
6 (4πε 0 ) 6 (4πε 0 )
πq q
(c) (d) [All India PM/PD 2003]
6 (4πε 0 ) 6 (4πε 0 )
q
Solution. Using Gauss’s law, ΦE =
ε0
q 4 πq
Flux through one face, φ= =
6ε 0 6(4 πε 0 )
So, (b) is the right choice.

l Three capacitors each of capacity 4 m F are to be connected in such a way that the effective capacitance is 6 µ F.
This can be done by
(a) connecting all of them in series (b) connecting them in parallel
(c) connecting two in series and one in parallel (d) connecting two in parallel and one in series.
[All India PM/PD 2003]
Solution. It is a parallel combination of 2 µF and 4 µF.
This gives 6 µF. 4 F 4 F
So, (c) is the right choice.
4 F
Fig. 190
ELECTROSTATICS 99

SELF-EVALUATION TEST II
Based on UNIT XI

DIRECTIONS :
(i) MCQs 1 to 24 have one correct alternative.
(ii) MCQs 25 to 30 have more than one correct alternative.
(iii) MCQs 31 to 35 have one or more than one correct alternative.

1. A sphere of 4 cm radius is suspended within a hollow (a) 1.2 × 10–33 N (b) 6.0 × 10–32 N
sphere of 6 cm radius. The inner sphere is charged to (c) 7.5 × 10–17 N (d) 3.8 × 10–15 N
a potential 3 esu when the outer sphere is earthed. (e) 7.5 × 10 –15
N.
The charge on the inner sphere is
6. In the circuit shown, a capacitor of capacitance 3 µ F
1 is charged from a batery of emf 6 V with the switch
(a) esu (b) 30 esu
4 connected to terminal P.
(c) 36 esu (d) 54 esu. [MP PMT 1991] Q
P
2. The capacitance of a spherical condenser is 1 µ F. If
the spacing between the two spheres is 1 mm, then
the approximate radius of the outer sphere is
6V
(a) 3 m (b) 5 cm
3 mF 6 mF
(c) 6 m (d) 30 cm.
3. The radii of the inner and outer spheres of a con-
denser are 9 cm and 10 cm respectively. If the dielec-
Fig. 192
tric constant of the medium between the two spheres
is 6 and charge on the inner sphere is 18 × 10 –9 cou- The switch is now connected to Q. This charges the
lomb, then the potential of inner sphere will be, if the 6 µ F capacitor from the 3 µ F one.
outer sphere is earthed, What is the new potential difference across the com-
(a) 18 V (b) 30 V bination ?
(c) 90 V (d) 180 V. (a) 1 V (b) 2 V
4. A 20 µ F capacitor is charged by a constant current (c) 4 V (d) 6 V.
of 10 mA. If the capacitor is initially uncharged, how 7. Charge is sprayed on to the belt of a Van de Graff
long does it take for the potential difference across generator by a sharp needle P (maintained at + 5 kV)
the capacitor to reach 300 V ? and is transferred to the dome via a second sharp
(a) 6.0 × 10–4 s (b) 0.60 s needle Q, connected to the interior of the dome as
shown in figure below.
(c) 15 s (d) 1.5 × 104 s
(d) 6.0 × 105 s.
5. The potential en-
ergy E P of an
electron in a uni-
form electric field
between two cha-
rged plates 0.02
m apart varies
with displace-
ment x from one Fig. 191
of the plates as
shown in Fig. 191. Fig. 193
What is the force on the electron ?
100 COMPREHENSIVE OBJECTIVE PHYSICS

Assuming that the insulation is perfect, which one of Ag mgA


the graphs below shows how the potential V of the (a) (b)
Qm 4 πQ
dome varies with time t when the belt starts to move
at a steady speed ? gAQ mgA
(c) − (d) − .
D 4 πQ
10. The circuit shown in
Fig. 195 was used to dis-
charge a charged capaci-
tor of 10 µ F capacitance.
For a period of 40 s, the
current was kept con-
stant at 20 µ A by con-
tinuous adjustment of R.
By how much did the po- Fig. 195
tential difference across
the capacitor fall during this period of 40 s ?
(a) 8.0 × 10 –4 V (b) 20 V
(c) 1.3 × 10 –2 V (d) 80 V
(e) 14 V.
11. Two parallel plates X and Y are mounted vertically
and given equal and opposite charges. A light
uncharged conducting sphere is suspended by an
insulating thread from point P vertically above the
mid-point O of the line joining the centre of the plates.

+ –

X Y

Battery
Fig. 194
8. The energy density in the electric field created by a
point charge falls off with the distance from the point Fig. 196
charge as
If the sphere is initially placed in contact with X as
1 1 shown in the diagram, which statement best de-
(a) (b)
r r2 scribes its subsequent motion ?
(a) It remains in contact with plate X
1 1 (b) It moves to plate Y and sticks to it
(c) (d) .
r3 r4 (c) It moves back and forth continuously, touching each plate
in turn
9. An oil drop of weight mg is observed to float freely
between the plates of a parallel plate capacitor, the (d) It moves to point O and quickly comes to rest there
plates being horizontal. The lower plate carries a (e) It oscillates as a simple pendulum.
charge of + Q. The area of each plate is A and the 12. In a system used for spraying cars, a car body is
separation between the plates is d. In cgs system, the positively charged. Neutral droplets of paint are then
charge on the drop is
ELECTROSTATICS 101
attracted to the car because the positive car body capacitance of
induces charge on the droplets of paint. the system is C.
Which diagram best shows the charge pattern ? When a light-
ning flash of
+ mean current I
– + –
+
+ and time dura-
– + – +
+ tion t occurs,
+ +
– + – + + + the electric field
+ Fig. 199
+ – + strength be-
Paint droplet + – tween cloud and Earth is reduced by
Car body
It It
(a ) (b) (a) (b)
Ch C
+ –
+ – + It
(c) (d) CIt
+ – + – + h
+ + – + CIt
– (e) .
+ h
+ – – +
15. Dust particles may be ex- air only
– + – +
+
tracted from air using
(c) (d ) the electric field between
a wire and a metal cyl-
Fig. 197 inder. The electric field wire
13. Which circuit could be used to measure the capaci- removes electrons from
tance of a capacitor using the reed switch S ? some air molecules, thus cylinder
forming ions. These ions
then become attached to
dust particles. The
S
S
mA charged dust particles
then move to the inside
V mA air and dust
V of the cylinder.
Fig. 200
Which combination of
ion charge and potentials must apply ?
(a ) (b)
ion charge wire potential cylinder potential

S S (a) positive + 10 kV 0
mA
(b) positive 0 + 10 kV
V
mA V (c) negative 0 + 10 kV

(d) negative – 10 kV 0
(c) (d )
16. The diagram
100 V 40 V – 20 V – 80 V
shows a uni-
S
form electric
V mA
field in which
the lines of equal
electric field
potential are
spaced 2.0 cm
(e) apart.
line of equal
Fig. 198 What is the 2.0 cm 2.0 cm 2.0 cm potential
value of the elec-
14. A thundercloud and the Earth’s surface may be Fig. 201
tric force which
regarded as a pair of charged parallel plates separated is exerted on a charge of + 5.0 µC when placed in the
by a distance h as shown in the diagram. The field ?
102 COMPREHENSIVE OBJECTIVE PHYSICS

(a) 6.0 × 10– 6 N (b) 1.5 × 10 –2 N 19. A metal sphere of radius 0.1 m was insulated from
(c) 3.0 × 10 N 3
(d) 6.0 × 10 N.
8 its surroundings and given a large positive charge. A
17. Four identical capacitors are connected as shown : small charge was brought from a distant point to a
point 0.5 m from the sphere’s centre. The work done
P against the electric field was W and the force on the
small charge in its final position was F. If the small
charge had been moved to only 1 m from the centre
of the sphere, what would have been the values for
the work done and the force ?
Q work done force
(a) W/4 F/4
(b) W/4 F/2
(i) (ii) (c) W/2 F/4
S (d) W/2 F/2
R (e) W/2 F/√2.
20. An isolated, solid metal sphere of radius R is given
an electric charge. Which one of following best repre-
sents the way in which the density of the charge var-
ies with distance r from the centre of the sphere ?
(iii) (iv)
charge charge
Fig. 202 density density
Which of the following lists the arrangements in or-
der of decreasing capacitance ?
(a) PRSQ (b) PSRQ
(c) QRSP (d) QSRP.
18. Two large plane parallel conducting plates are situ-
ated 40 mm apart as shown. The potential difference 0 R r 0 R r
between the plates is V. (a ) (b)
What is the potential difference between point X and charge charge
point Y ? density density

40 mm

Y 0 R r 0 R r
(c) (d )
0 +V
25 mm
15 charge
mm density
X
15 mm 20 mm 5
mm

Fig. 203

15 20 0 R r
(a) V (b) V
40 40 (e)
25 40
(c) V (d) V. Fig. 204
40 40
ELECTROSTATICS 103
21. Fig. 205 shows the impor- What is the direction of the resultant electric field at
tant components of a large the fourth corner ?
Van de Graff generator op-
erating in air at atmos-
pheric pressure. When the
electric field strength at the
sphere’s surface exceeds
the breakdown field of the
air, the sphere discharges
by sparking. Which one of
the following determines
the maximum potential at-
tained by the sphere ?
(a) the emf of the battery P
Fig. 205
(b) the radius of curvature of Fig. 207
the point Q
(a) A (b) B
(c) the speed of the belt R
(c) C (d) D
(d) the distance of the point S from the belt
(e) E.
(e) the radius of the sphere T.
22. Fig. 206 shows a 25. A point charge is brought in an electric field. The
charged conductor electric field at a nearby point,
resting on an insu- (a) will increase if the charge is positive
lating stand. (b) will decrease if the charge is negative
If at a point P the (c) may increase if the charge is positive
charge density is σ, (d) may decrease if the charge is negative.
the potential is V and 26. Which one statement is correct ? A parallel plate air
the electric field condenser is connected with a battery. Its charge,
strength is E, what potential, electric field and energy are Q0 , V0 , E0 and
are the values of Fig. 206 U0 respectively. In order to fill the complete space
these quantities at point Q ? between the plates a dielectric slab is inserted, the
charge density potential electric intensity battery is still connected. Now the corresponding
(a) >σ >V >E values Q, V, E and U are in relation with the initially
(b) >σ V >E stated as :
(c) <σ V E (a) Q > Q0 (b) V > V0
(d) <σ V <E (c) E > E0 (d) U > U0. [IIT 1985]
(e) <σ <V < E. 27. A spherical conductor A lies inside a hollow spheri-
23. In discussing electric fields, the terms electric field cal conductor B. Charges Q 1 and Q2 are given to A
strength, electric potential and potential gradient are and B respectively.
used. Which one of the following statements about (a) Charge Q1 will appear on the outer surface of A.
these terms is correct ? (b) Charge – Q1 will appear on the inner surface of B.
(a) Unit potential gradient exists between any two points if
(c) Charge Q2 will appear on the outer surface of B.
one joule of work is done in transporting one coulomb of
charge between the points. (d) Charge Q1 + Q2 will appear on the outer surface of B.
(b) The electric potential at a point is the force on unit positive 28. A closed surface S is constructed
charge placed at that point. around a conducting wire con-
(c) Electric potential and potential gradient are both scalar nected to a battery and a switch
quantities. (Fig. 208). As the switch is closed,
(d) Electric field strength at a point is the work done in bring- the free electrons in the wire start
ing unit positive charge from infinity to the point. moving along the wire. In any
(e) The potential gradient at a point is numerically equal to time interval, the number of elec-
the electric field strength at that point. trons entering the closed surface
24. Point charges, each of magnitude Q, are placed at S is equal to the number of elec-
Fig. 208
three corners of a square as shown in the diagram. trons leaving it. On closing the
104 COMPREHENSIVE OBJECTIVE PHYSICS

switch, the flux of the electric field through the closed F


3 mv2 I
surface (a) E = GH
4 qa
JK
(a) is increased (b) is decreased
(c) remains unchanged (d) remains zero. 3 mv3F I
29. An electric dipole is placed at the centre of a sphere.
(b) Rate of work done by the electric field at P is
4 a
GH JK
Mark the correct options :
(c) Rate of work done by the electric field at P is zero.
(a) The flux of the electric field through the sphere is zero
(d) Rate of work done by both the fields at Q is zero.
(b) The electric field is zero at every point of the sphere
[IIT 1991]
(c) The electric field is not zero anywhere on the sphere
(d) The electric field is zero on a circle on the sphere. 33. A point charge is placed at Y in front of an earthed
metal sheet X. P and Q are two points between X and
30. Two insulated charged spheres of radii 20 cm and 25
Y as shown in the diagram.
cm respectively and having an equal charge Q are
connected by a copper wire. Then they are separated. If the electric field strengths at P and Q are respec-
Which of the following is incorrect ? tively EP and EQ , which one of the following state-
ments is correct ?
(a) Both the spheres will have the same charge Q
(b) Charge on the 20 cm sphere will be greater than that on
the 25 cm sphere
(c) Charge on the 25 cm sphere will be greater than that on
the 20 cm sphere
(d) Charge on each of the sphere will be 2Q.
31. The separation between the plates of a charged par-
allel plate capacitor is increased. Which of the follow-
ing quantities will change ?
(a) charge on the capacitor
(b) potential difference across the capacitor
(c) energy of the capacitor
(d) energy density between the plates. Fig. 210
32. A particle of charge (a) EP = EQ (b) EP = 0
y →
+q and mass m mov- E (c) EQ = 0 (c) EP > EQ
v
ing under the influ- P
→ (e) EQ > EP
ence of a uniform B
34. Electric charges q, q and – 2q are placed at the cor-
electric field E i and a
ners of an equilateral triangle of side ‘l’. The magni-
a uniform magnetic tude of electric dipole moment of the system is
field B j follows a Q
x (a) 2ql (b) ql
trajectory from P to 2a 2v
Q as shown in Fig. (c) 4ql (d) 3 ql
209. The velocities at Fig. 209
35. The dimension of (1/2) ε0 E2 (ε0 : permittivity of free
 
P and Q are v i and – 2v j . Which of the following space ; E : electric field) is
statement(s) is/are correct ? (a) MLT –1 (b) ML2T –2
(c) ML–1 T –2 (d) ML2T –1
[IIT Screening 2000]
Answers
1. (c) 2. (a) 3. (b) 4. (b) 5. (e) 6. (b) 7. (a) 8. (d)
9. (b) 10. (d) 11. (c) 12. (d) 13. (e) 14. (a) 15. (a) 16. (b)
17. (b) 18. (b) 19. (c) 20. (a) 21. (e) 22. (d) 23. (e) 24. (b)
25. (c), (d) 26. (a), (d) 27. (a), (b), (d) 28. (c), (d) 29. (a), (c) 30. (a), (b), (d) 31. (b), (c) 32. (a), (b), (d)
33. (e) 34. (d) 35. (c)
ELECTROSTATICS 105

Solutions
4×6 7. The potential V of the dome is given by
1. C= esu of capacity
6−4 Q
V=
or C = 12 esu of capacity C
V = 3 esu of potential Where Q = charge on the dome.
Q = CV = 36 esu of charge. C = capacitance of the dome.
4 πε 0rarb Thus, potential V of the dome is directly proportional
2. C= to the charge arrives on it. When the belt starts to
rb − ra
move at a steady speed, charge is transferred to the
1 r2 dome at a constant steady rate. Hence, potential V
1 × 10 –6 = or r2 = 9 or r = 3 m. increases linearly with t, i.e.,
9 × 109 1 × 10 −3
4 πε 0K rbra 6 0.09 × 010
. λt
3. C= = F V=
rb − ra 9 × 109 0.01 C
= 0.6 × 10–9 F where λ is the rate at which charge is transferred to
the dome, t is the time taken.
Q 18 × 10−9 1
V= = volt = 30 volt. 8. u ∝ E2 and E ∝ .
C 0.6 × 10−9 r2
4. If V is the potential difference across the capacitor of 9. qE = mg
capacitance C at time t, the constant current I charg- mg mgε 0 mg( 4 πε 0 )A
ing the capacitor is given by or q= = =
E σ 4 πQ
Q CV mgA
I ==
t t For cgs system, q = .
4πQ
where Q is the charge stored on the plate
10. The rate at which the potential difference V across
Given I = 10 mA, C = 20 µF, V = 300 V the capacitor is falling is given by
Time taken is
dQ dV
I= =C
( 20 × 10−6 )( 300) dt dt
10 × 10–3 = or t = 0.60 s.
t where C is the capacitance of the capacitor, I is the
5. The force on the electron is the gradient of the graph constant current leaving the capacitor.
of the potential energy EP versus the displacement x. Given I = 20 µA, C = 10 µF, we have
Mathematically, we may write
dV dV
∆EP dEP 20 µ = (10 µ) or = 2 Vs–1.
F = – =− dt dt
∆x dx Over a period of 40 s the potential difference across
Hence, force on the electron is given by the capacitor falls by 40(2) V or 80 V.
FG 1.5 − 0 IJ × 10 −16 1.5 × 10 −16 11. The conducting sphere acquires charge in its initial
F= −
H 0 − 0.02 K N =
0.02
N
position. Once it is released, it moves to plate Y and
keeps in contact with Y momentarily until it loses all
= 7.5 × 10–15 N.
its charge and becomes negatively charged. Its weight
6. Initially at switch position P, the capacitor of capaci- and the repulsive force of plate Y causes it to swing to
tance 3 µF is charged to 6 V carrying a charge of plate X and keeps in contact again with plate X mo-
q = CV = 3 × 6 = 18 µC. mentarily and becoming positively charged. The
When at switch position Q, by conservation of charge, sphere once again moves to plate X carrying positive
the voltage V across the two capacitors is thus given charges and it thus moves back and forth continu-
by ously, touching each plate in turn.
q = q1 + q2 = C1V + C2V = (C1 + C2)V 12. Positive charges on the metal surface of car attracts
18 the negative charges in the droplet towards the drop-
⇒ 18 = (3 + 6)V ⇒ V = = 2 volt let’s surface nearest to the car, and expels the posi-
9
106 COMPREHENSIVE OBJECTIVE PHYSICS

tive charges in the droplet away from the car and Case Capacitance
causes these positive charges distribute themselves
P 4C
on the right surface in the droplet as illustrated in
1
diagram d. Q 4 C
13. The capacitor has to be connected to a supply when it 2
R 5
C
is charged in order that there is flow of current to the
capacitor plates. At discharge, the charged capacitor 3
S 4 C
has to be connected to a passive load (resistor or de-
vice of resistance) to allow the flow of charge from the where C is the capacitance of the four identical ca-
capacitor. pacitors.
(a) Capacitor cannot discharge because it is switched The arrangement in order of decreasing capacitance
to an open circuit (voltmeter). is thus PSRQ.
(b) Capacitor cannot discharge because it is switched 18. The electric field between the two conducting plates
to an open circuit. is constant and given by
(c) Capacitor cannot charge up because it is not con- V
nected to a supply. E=
d
(d) Capacitor cannot charge up because it is not con- where V = potential difference between the plates
nected to a supply.
d = distance between the plates = 40 mm
(e) Capacitor is able to charge up and discharge be- The work required to carry a test charge q0 from point
cause of power source and load connection. X to point Y is thus given by
14. Change in electric field strength, W = q0VXY = (q0E) dXY
∆V where VXY = potential difference between X and Y
∆E =
h dXY = effective distance between points X and Y
But change in potential between thunder cloud and = 20 mm
Earth’s surface, ∆V =
∆Q
and change in charge, ∆Q FG d IJ V = 20 V .
H d K 40
XY
C i.e., VXY = EdXY =
= It.
19. Since the test charge q is small enough that it does
Hence, electric field strength E has reduced by an
not disturb the original field distribution of the charged
amount
metal sphere, we may assume the metal sphere of
∆E = ∆V = ∆Q = It . positive charge Q to be a positive point charge located
h Ch Ch at its centre. Hence, work done W against the field
15. When the ions are attached to dust particles, these and the force F at a point 0.5 m away from the charge
dust particles become positively charged. If these are Q are
attracted to the cylinder surface due to the electric R|W = Qq
field set up between the wire and the cylinder, the
wire must have a higher potential than that of the |S 4πε (0.5) 0

cylinder. ||F = Qq
Thus, (a) gives the best combination of ion charge T 4πε (0.5)0
2

and the potentials of the wire and cylinder. Now, the distance apart is 1 m, the corresponding
16. The electric field strength E is equal numerically to work done W′ and F′ are then given by
the gradient of the potential, i.e., Qq FG IJ
0.5 W
E=
V
=
60
= 3.0 × 103 V/m
W′ =
4 πε 0 (1)
=
1H K
W=
2
d 2 × 10−2 Qq
=G
F 0.5 IJ
F
2

The electric force acting on a charge of + 5.0 µC is F′ =


4 πε 0 (1) 2 H 1K
4
. F=
thus
20. Electric charge placed on an isolated, solid metal
Fe = qE = (5.0 × 10–6)(3.0 × 103) sphere, will initially set up electric fields within the
= 1.5 × 10–2 N. conductor. The fields act on the charge carriers of the
17. The combined or effective capacitance for each of the conductor (electrons) and cause them to move. The
four connections, is as follows :
ELECTROSTATICS 107
charges cease motion when the electric fields in the (b) Electric field strength E at a point is the force on
conductor are everywhere zero. This occurs when the unit positive charge at the point. It is a vector
electric charge is on the outer surface of the conduc- quantity.
tor. Hence, the charge density is zero everywhere in- (a) Unit electric potential exists between any two points
side and outside of the metal sphere except on its sur- if one joule of work is done in transporting one
face. Graph a correctly represents the charge density coulomb of charge between the points. It is a sca-
distribution on the metal sphere. lar quantity.
21. The electric field strength E and the electric potential Only option e correctly describes the relation between
V at the sphere’s surface of a Van de Graff generator potential gradient and electric field strength.
are given by the equations : 24. Due to symmetry, Q
R|E = Q the net resultant
E4 E2 =
4πε0d
2

|S 4πε R 0
2 electric field must be
along the diagonal of
45° +Q

||V = Q the square in direc- E1 =


Q
T 4πε R 0 tion B or D. Result- 4 πε 0d
2

Q
where Q is the charge on the sphere’s surface ant field due to E1 E3 =
8πε0d
2 d
R is the radius of the sphere T. and E2 is
We may thus relate V to E by the equation V = RE. Q 2
E4 = .
At a given value of E, the maximum potential V is 4 πε 0 d2
limited by R, radius of the sphere. The net resultant +Q –Q
22. The surface of the conductor is an equi-potential sur- electric field due to Fig. 211
face since there is free flow of electrons within the the three charges is
conductor. Thus potential at Q is the same as that at
P. That is VP = VQ = V. Q 2 Q
E4 – E3 = 2

The electric field E at a point on the equi-potential 4 πε 0 d 8πε 0 d2
surface of the conductor is inversely proportional to
the square of radius of curvature r at that point. That Q
= (2 2 − 1) in the direction B.
is 8πε 0 d 2
1 26. Refer to synopsis.
E∝ 2 .
r 27. Think in terms of electrostatic induction.
Since point Q has a larger radius of curvature than 28. The net charge is unchanged.
that at point P, the electric field at Q is less than that
29. Net charge enclosed is zero.
at P. That is
30. V same ; C more ; Q more.
EQ < EP = E.
Furthermore, the charge density at a point is propor- 31. Due to change in capacity, V would change. Note that
tional to the electric field at that point on the couductor Q remains same.
surface. The charge density σQ at point Q is thus less 1 1
than that at point P. That is 32. (a) qE × 2a = m(2v)2 – mv2
2 2
σQ < σP = σ.
23. Correct descriptions about the terms electric field 3mv2
E=
strength, electric potential and potential gradient are 4 qa
as follows : (b) P = Fv = qEv
(d) Electric field strength E at a point is the negative
F 3mv I v = 3 mv
2 3

of the potential gradient,


dV
at that point. That
=q GH 4qa JK 4 a .
dx → →
dV (d) At Q, v and E are perpendicular. So, work done
is E = − and it is a vector quantity. →
dx by E is zero. The work done by magnetic field is
(c) Electric potential V at a point is the work done in always zero.
bringing unit positive charge from infinity to the
point. V is a scalar quantity.
108 COMPREHENSIVE OBJECTIVE PHYSICS

33. The diagram shows the electric field distribution from Hence, EQ > EP.
the point charge (positive) at Y to the earthed metal 34. The given system is equivalent
sheet X. to a system shown in Fig. 213.
The point closer to the point charge Y has higher po- Net electric dipole moment
tential and hence its field strength is higher than the
= 3 p = 3 ql .
other point further away from Y.
Fig. 213
35. It has the dimensions of energy per unit volume.

LM 1 ε E OP = [energy]
2

N 2 Q [volume]
0

[ML2T −2 ]
= = [ML –1 T–2].
[L3 ]

Fig. 212

KNOWLEDGE PLUS

l An electron is moving round the nucleus of a hydrogen atom in a circular orbit of radius r. The coulomb force F
between the two is ?
e2 e2
(a) K r (b) – K r
r2 r3
e2 → e2 → 1
(c) K r (d) – K 3
r (where K = ) [All India PM/PD 2003]
3 r 4πε 0
r
Solution. Use coulomb’s law. Also, the force vector and r are oppositely directed.
So, (b) is the right choice.

l A charged particle ‘q’ is shot towards another charged particle ‘Q’, which is fixed, with a speed ‘v’. It approaches ‘Q’
upto a closest distance r and then returns. If q were given a speed of ‘2v’, the closest distance of approach would be

q Q
v r

Fig. 214
(a) r/2 (b) 2r (c) r (d) r/4. [AIEEE 2004]
1 K qQ
Solution. mv2 =
2 r
1
m(2v)2 =
K qQ
or 4
LM 1 mv OP = K qQ
2
or
4KqQ K qQ
= or r′ =
r
2 r′ N 2 Q r′ r r′ 4
So, (d) is the right choice.

l When a sinusoidally varying current flows through a capacitor


(a) electrons tunnel through the dielectric of the capacitor.
(b) electrons get accumulated on one plate and depleted on the other sinusoidally.
(c) electric flux over the area of the plates varies sinusoidally.
(d) the polarization of the dielectric material varies sinusoidally.
[National Standard Exam. in Physics 2005]
Ans. (b), (c), (d).
UNIT XII

CURRENT ELECTRICITY

l Electric Current l Flow of electric charges in a Metallic Conductor l Drift velocity and mobility
and their relation with electric current l Ohm’s law l Electrical Resistance l V-I
characteristics l Exceptions of Ohm’s law (Non-linear V-I Characteristics) l Electrical resistivity and
conductivity l Classification of materials in terms of conductivity l Superconductivity (elementary
idea) l Carbon Resistors l Colour code for carbon resistors l Combination of resistances—Series
and Parallel l Temperature–dependence of resistance l Internal resistance of a cell l Potential
difference and emf of a cell l Combination of cells in Series and in Parallel l Kirchhoff ’s Laws—
Illustration by simple applications l Wheatstone Bridge and its application for temperature
measurements l Metre Bridge—special case of Wheatstone Bridge l Potentiometer—principle and
applications to measure potential difference, and for comparing emfs of two cells l Electric
Power l Thermal effects of current and Joule’s law l Chemical effects of current l Faraday’s
laws of electrolysis l Electro-chemical cells—Primary (Voltaic, Leclanche, dry, Daniel) l Secondary—
Rechargeable cells (Lead accumulator, Alkali accumulators) l Solid states cells l Thermoelectricity—
Origin l Elementary ideas of Seebeck, Thomson and Peltier Effects l Thermocouple l Thermo
emf l Neutral and Inversion Temperatures

UNIT DETAILS

1. Synopsis Points 65 Plus

2. Illustrations 20 Plus

3. MCQs from Competitive Examinations 240 Plus

4. Self-Evaluation Tests 2

5. Total Number of MCQs 590 Plus

6. Total Number of Solutions 580 Plus

C-11\IITS\C12-1 CDR-12-1
CHAPTER 12

CURRENT ELECTRICITY

SYNOPSIS

(A) CURRENT ELECTRICITY & SOURCES OF EMF → →


6. Relation between drift velocity vd and electric field E
1. Just as mass, length, volume etc. of a conductor are

macroscopic quantities, the electric current is also a → eE
macroscopic quantity. In other words, electric cur- is : vd=– τ , where τ is relaxation time, e is elec-
m
rent is a characteristic of the whole of the conductor.
The ‘arrows’ associated with electric current simply tronic charge and m is the mass of the electron.
convey the sense (direction) of flow of electric charge. 7. Relation between drift velocity νd and electric current
Electric current is a scalar quantity. On the other I
hand, current density is a vector quantity. It is a char- I is νd = where n is the number density of charge
neA
acteristic of a certain point in a conductor. In other
carriers i.e., the number of charge carriers per unit
words, it is a microscopic quantity.
volume. A is the cross-sectional area of the conductor.
2. If we study the flow of current in a conductor of vary-
8. Ohm’s Law. The current I flowing through a con-
ing cross-section, it will be observed that the current
ductor is directly proportional to the potential differ-
is the *same at all points. If this were not so, the
ence V across the two ends of the conductor, provided
charge will not be conserved.
physical conditions such as temperature, mechanical
3. (i) The charge carriers in the case of metals are free strain etc. remain the same. V ∝ I or V = IR, where R
electrons. is the resistance of the conductor.
(ii) The charge carriers in the case of liquids are posi- 9. Relation between resistance R and relaxation time τ
tive and negative ions.
ml
(iii) The charge carriers in the case of gases are posi- is R = , where A is the cross-sectional area of
tive ions and free electrons. ne2τA
(iv) The charge carriers in the case of semiconductors the conductor and l is the length of the conductor.
are free electrons and holes. 10. The resistance R of a conductor depends on its length
4. Electric current through the given area of a conduc- l and constant cross-sectional area A through the re-
tor is the net charge passing per unit time through l
lation : R = ρ .
that area. A
q ne Here ρ called resistivity is a property of the material
I= = (for steady flow of charge)
t t and depends on temperature and pressure.
dq 11. RELATION BETWEEN RESISTANCE, MASS
and I = (for non-steady flow of charge). AND LENGTH OF CONDUCTOR
dt
5. Current flowing per unit area is called current density l
J. R=ρ
a
I ne
J= = .
A At l×l l2
R= ρ or R = ρ
*This is in sharp contrast with the flow of liquid in a tube of a×l V
varying cross-section.

111
C-11\IITS\C12-1 CDR-12-1
112 COMPREHENSIVE OBJECTIVE PHYSICS

m m (iv) The total resistance is greater than the greatest


But density, d = or V = individual resistance.
V d
(v) If G is the effective conductance, then
ρdl2 1 1 1 1
∴ R= = + + + ......
m G G1 G 2 G3
12. RELATION BETWEEN RESISTANCE, MASS (vi) If n resistances, each of value R, are connected in
AND CROSS-SECTIONAL AREA OF CONDUC- series, then Rs = n R
TOR 15. Total resistance Rp of n resistors connected in paral-
l lel is given by
R=ρ
a 1 1 1 1
n
1
la V
= +
R p R1 R2
+ ...... +
Rn
= ∑ Ri
R= ρ or R = ρ i= 1
a2 a2 The following physical facts may be noted about
m m the parallel combination of resistances :
But d= or V = (i) Potential difference is the same across each
V d
resistor.
ρm (ii) The total current is the sum of individual currents.
∴ R=
da2 (iii) The individual currents are inversely proportional
13. STRETCHING A WIRE to individual resistances.
(i) Keeping mass constant, length increased n times (iv) The equivalent resistance is less than the least
individual resistance.
ρdl2 ρd (nl)2
R= ; R′ = (v) Effective conductance, G = G1 + G2 + G3 + ......
m m (vi) If n equal resistances, each of value R, are con-
R′ R
= n2 ; R′ = n2R nected in parallel, then Rp = .
R n
(ii) Keeping mass constant, radius increased n times 16. CASE OF n IDENTICAL RESISTANCES
ρm ρm ρm Let R be the value of each resistance. Let Rs be the
R= 2 2
or R = 2 4 ; R′ = 2 equivalent resistance of their series combination. Let Rp be
d (πr ) π dr π d (nr) 4
the equivalent resistance of their parallel combination.
R′ 1 R
= 4 or R′ = 4 R s n2 Rp 1
R n n Then, = or =
Rp 1 Rs n2
(iii) Keeping mass constant, cross-sectional area in-
creased n times 17. CASE OF TWO UNEQUAL RESISTANCES R 1
AND R 2
ρm ρm R
R=
da2
; R′ =
d (na)2
; R′ =
n2 1 LM
R s + R s2 − 4 R s R p OP
14. Total resistance Rs of n resistors connected in series
R1 =
2 N Q
= LR OP
1
2 MN
n
− R s2 − 4 R s R p
is given by Rs = R1 + R2 + ...... + Rn = ∑ Ri
R2 s
Q
i= 1
18. DISTRIBUTION OF ELECTRIC CURRENT IN
The following physical facts may be clearly noted TWO PARALLEL RESISTANCES
about the series combination of resistances :
(i) The current through all the resistances is the I1 IR2 R + R2 I1 R 2
= × 1 or =
same. I2 R 1 + R 2 IR1 I2 R 1
(ii) The total potential difference is equal to the sum
The currents divide themselves in the inverse ratio of
of individual potential differences.
resistances.
(iii) The individual potential differences are propor-
tional to individual resistances. I1 R2 R 2I
Now, = or I1 =
I1 + I2 R 2 + R 1 R1 + R 2
CURRENT ELECTRICITY 113
R1 (iv) Equivalent resistance between adjacent corners =
I1
I1 5R
6
I x y I R

I2 I2
R R
R2

Fig. 1

R 1I
Similarly, I2 =
R 1 + R2 R R
19. A SPECIAL WINDOW ON SOME USEFUL NET-
WORKS R
(i) Equivalent resistance between adjacent corners
Fig. 5
2R
=
3 THINK IT OVER !
Polygon of n sides
Equivalent resistance between adjacent corners
R R
(n − 1) R
=
n

(v) An equilateral triangle made


R from a wire of resistance R.
Fig. 2 Equivalent resistance between
(ii) Equivalent resistance between adjacent corners any two adjacent corners
3R 2 R 2R
= = . =
4 3 3 9 Fig. 6
R
(vi) A square made from
a wire of resistance
R. Equivalent resist-
ance between two
R R
adjacent corners
3 R 3R
= . =
4 4 16 Fig. 7
R
Fig. 3 (vii) A polygon made from a wire
(iii) Equivalent resistance between adjacent corners of resistance R.
4R Equivalent resistance between
= two adjacent corners
5
R R 4 R 4R
= . =
5 5 25
Fig. 8

THINK IT OVER !
R R A Polygon of n sides made from a wire of resistance R.
Equivalent resistance between adjacent corners
(n − 1) R
=
n2
R
Fig. 4
114 COMPREHENSIVE OBJECTIVE PHYSICS

(viii) Effective resistance between the ends of a diago- are ± 5% and ± 10% respectively. If there is no colour,
nal = R. it indicates a tolerance of 20%.
R The first two rings or strips from the end give
the first two significant figures of resistance
in ohm. While the colour of the first ring on the ex-
treme left represents the first significant figure, the
R R second ring represents the second significant figure.
The third ring indicates the decimal multiplier
i.e., the number of zeros that will follow the two sig-
nificant figures.
R The last ring indicates the tolerance in percent
Fig. 9
about the indicated value i.e., it represents the
percentage accuracy.
(ix) If a wire of resistance R is bent in the form of a 21. Ohmic conductors are those conductors which obey
circle, then the effective resistance between the Ohm’s law.
ends of the diameter is R.
22. Non-ohmic conductors are those conductors which do
(x) Skeleton cube made from 12 wires, each of resist- not obey Ohm’s law.
ance R.
23. The variation of resistance of a metallic conductor
(a) Equivalent resistance between diagonally with temperature is given by :
5R Rt = R0(1 + αt)
opposite corners =
6 where R0 is the resistance at 0°C, Rt is the resistance
(b) Equivalent resistance between adjacent at t°C and α is the temperature coefficient of resist-
ance.
7R
corners = . 24. The temperature at which a substance loses all its
12
resistance is called critical temperature and the
(xi) If a skeleton cube is made from 11 wires, then phenomenon is called superconductivity.
equivalent resistance between the corners of the 25. The relation between internal resistance r, emf E,
7R terminal potential difference V and external resist-
vacant edge is .
5 E−V
ance R is : r = R.
20. COLOUR CODE FOR CARBON RESISTANCES V
Letter as Colour Number Multiplier Colour Tolerance 26. When the cell is sending current through the exter-
an aid to nal resistance, then
memory V = E – Ir.
B Black 0 10 0 Gold ± 5% 27. When the cell is being charged,
B Brown 1 10 1 Silver ± 10% V = E + Ir.
28. When n cells, each of emf E and internal resistance r,
R Red 2 10 2 No ± 20%
are connected in series, then the current is given by
colour
O Orange 3 10 3 nE
I= .
nr + R
Y Yellow 4 10 4
29. When internal resistance is negligible, then the cells
G Green 5 10 5
are connected in series to get maximum current.
B Blue 6 10 6 30. For a series combination of cells, the following state-
V Violet 7 10 7 ments apply :
G Grey 8 10 8 (i) The emf of the battery is equal to the sum of the
individual emf’s of the various cells.
W While 9 10 9
(ii) The current in each cell is the same and is identi-
The multipliers in the case of gold and silver are 10–1 cal with the current in the entire series arrange-
and 10–2 respectively. The tolerances (percentage ment.
accuracy or reliability) in the case of gold and silver
CURRENT ELECTRICITY 115
(iii) The total internal resistance of the battery is equal 42. For the determination of the internal resistance of a
to the sum of the individual internal resistances. cell with the help of potentiometer,
31. For a parallel arrangement of identical cells, the fol- l1 − l2
lowing statements are true : r= R.
l2
(i) The emf of the battery is the same as the emf of a
Here l1 and l2 are the balancing lengths correspond-
single cell.
ing to the emf E and terminal potential difference V
(ii) The reciprocal of the total internal resistance is respectively. R is the resistance of the shunt across
equal to the sum of the reciprocals of the resistances the cell.
of the individual cells.
(B) THERMAL EFFECT OF CURRENT
(iii) The current in the external circuit is divided
43. In actual practice, the measurement of electric cur-
equally among the cells.
rent does not include individual measurements of
32. When n cells, each of emf E and internal resistance r, charge and time. The electric current is generally
are connected in parallel, then the current is given by measured by its magnetic effect, chemical effect, heat-
E ing effect etc.
I= . 44. When a current I flows through a passive resistor R
r
R+ for time t, heat Q is produced such that
n
33. When external resistance is negligible, then the cells Q = I2Rt
are connected in parallel to get maximum current. This is Joule’s law of heating.
34. If n cells are connected in a row and m such rows are 45. LAW OF TIME
connected in parallel, then the current is given by Keeping R and I constant, if Q1 and Q2 are the heats
nE produced during times t1 and t2 respectively, then it
I= . will be observed that
nr
R+
m Q1 t1
=
35. When external resistance is comparable to internal Q2 t2
resistance, then the cells are connected in mixed group-
ing to get maximum current. 46. LAW OF ELECTRIC CURRENT
36. According to Kirchhoff ’s current law, the algebraic Keeping R and t constant, if Q1 and Q2 are the heats
sum of currents meeting at a junction is zero. produced by currents I1 and I2 respectively, then it
This law is based on law of conservation of charge. will be observed that
37. According to Kirchhoff ’s voltage law, the algebraic
Q1 I12
sum of potential drops and emfs along any closed path =
Q 2 I22
in a network is zero. This law is based on law of con-
servation of energy.
47. LAW OF RESISTANCE
38. Condition for a Wheatstone bridge to be balanced is :
Keeping I and t constant, if Q1 and Q2 are the heats
P R produced in resistances R1 and R2 respectively, then
= .
Q S
Q1 R1
39. In the case of a metre bridge, the unknown resistance =
Q2 R2
100 − l
is given by X = R , where l represents the bal- 48. In a passive resistor
l
ancing length and R is the resistance. V V
(i) V = IR : I = ,R=
40. In the case of potentiometer, the potential difference R I
V across any length is proportional to the length l. volt = ohm × ampere
41. For comparing emfs with the help of potentiometer, (ii) Power consumed
E1 l1 = heat developed per second, in watt
= . Here l1 and l2 are the balancing lengths ob-
E2 l2
V2
tained with cells of emfs E1 and E2 respectively. P = I2R = IV =
R
116 COMPREHENSIVE OBJECTIVE PHYSICS

(iii) Heat developed in time t. 56. According to Faraday’s second law of electrolysis, the
Electrical energy consumed masses of different substances produced in electrolysis,
= Heat developed in joule by the same quantity of electricity, are proportional
to their equivalent masses.
V2
I2Rt = IVt = t m1 E1
R =
m2 E2
49. In any device whatever
Electrical power consumed E
57. = F, where F is Faraday constant. It is the quan-
= Power developed in other forms Z
P = IV tity of charge required to liberate one gram equiva-
lent of the substance. Its value is 96500 C mol–1.
watt = ampere × volt
58. Electrolysis of CuSO4
50. MAXIMUM POWER RATING OF A RESISTOR
At the cathode
It is the maximum power that can be dissipated with-
out overheating the device. When this rating is ex- Cu++ ions drift to the cathode and are neutralised by
ceeded, the resistance may change unpredictably. In the electrons flowing in from the negative terminal to
more extreme cases, the resistor may melt or even the external source.
explode. In practical applications, the power rating of Cu++ + 2e– → Cu
a resistor is just as important a characteristic as its At the anode
resistance value. Cu dissolves into the solution producing Cu++ and
2e–, the latter flowing to the positive terminal of the
V2
51. Electrical energy = VIt = t = I2Rt source.
R
Cu → Cu++ + 2e–
The practical unit of electrical energy is kWh.
In effect, copper is dissolved off the anode and depos-
1 kWh = 3.6 × 106 J ited at the cathode.
52. The material of a heater element should have high 59. Electrolysis of water
resistivity and high melting point.
At the cathode :
53. The material of a fuse wire should have high resistiv-
4H2O + 4e– → 4OH– + 2H2
ity and low melting point.
At the anode :
54. The heat produced in a conductor is independent of
2H2O – 4e– → 4H+ + O2
the direction of current.
In the electrolyte, H+ and OH– diffuse away towards
(C) CHEMICAL EFFECT OF CURRENT the opposite electrodes and neutralise.
55. According to Faraday’s first law of electrolysis, the 4H+ + 4OH– → 4H2O
mass m of a substance deposited at the cathode or The net reaction is
dissolved off the anode in electrolysis is proportional 2H2O → 2H2 + O2
to the charge Q passing through the electrolyte. which requires 2.9 eV per molecule.
m ∝ Q, m = ZQ = ZIt
Here Z is called electrochemical equivalent.
CURRENT ELECTRICITY 117
60. A COMPARATIVE STUDY OF DIFFERENT CELLS

S. No. Cell Anode Cathode Electrolyte emf Internal resistance Special comment

1. Simple Cu Zn dil. H2SO4 1.08 V depends upon defects Defects : Local action and
Voltaic Polarisation
(Primary)
2. Daniel Cu Zn dil. H2SO4 1.08 V 1 Ω to 4 Ω CuSO4 solution is depolariser
(Primary)
3. Leclanche C Zn NH4Cl 1.5 V 0.25 Ω to 4 Ω MnO2 is depolariser
(Primary)
4. Dry C Zn NH4Cl 1.5 V 0.25 Ω to 4 Ω MnO2 is depolariser
5. Lead PbO2 Pb dil. H2SO4 1.8 V to 0.01 Ω to 0.001 Ω Energy efficiency is nearly
accumulator (Spongy 2.2 V 80 %.
lead)
6. Alkali Ni + NiO2 FeO2 KOH 1.25 V to 0.01 Ω Efficiency is nearly 60%
accumulator 1.35 V
7. Cadmium Hg Hg + Cd CdSO4 1.00 V 500 Ω Hg2SO4 is depolariser

(D) THERMOELECTRIC EFFECT OF CURRENT junction. It depends only


61. If two junction of dissimilar metals are held at differ- upon the nature of the
ent temperatures, an emf develops causing current to metals constituting the
flow. This is Seeback effect. thermocouple.
62. SEEBECK SERIES The temperature
Seebeck studied the behaviour of many a pair of met- ti for which the thermo
als. He arranged them in a series known as Seebeck series emf becomes zero and
or thermoelectric series given below : then changes direction
(becomes inverted) is
1. Antimony (Sb) 2. Arsenic (As) 3. Iron (Fe) Fig. 10
called the temperature of
4. Cadmium (Cd) 5. Zinc (Zn) 6. Silver (Ag)
inversion. It depends upon the temperature of the cold junc-
7. Gold (Au) 8. Molybdenum (Mo) 9. Tin (Sn)
tion. It is as much above the neutral temperature tn as the
10. Lead (Pb) 11. Mercury (Hg) 12. Manganese (Mn)
temperature of cold junction lies below the neutral tem-
13. Copper (Cu) 14. Platinum (Pt) 15. Cobalt (Co)
16. Nickel (Ni) 17. Bismuth (Bi).
perature.
∴ tn – tc = ti – tn or 2tn = ti + tc
The thermoelectric current flows from a metal ear-
lier in the series to a metal later in the series across the cold ti + tc
or tn =
junction. 2
The magnitude of the thermo emf depends upon the
extent of separation of the metals in the series. 64. THERMOELECTRIC POWER
Metals on the right of lead are considered thermo- The rate of change
electrically negative while metals on the left of lead are of thermoelectric emf
thermoelectrically positive. with temperature is
63. VARIATION OF SEEBECK EMF WITH TEM- called thermoelectric
PERATURE power. It is also known
Thermo emf increases with an increase in the tem- as Seebeck coefficient. It
perature of the hot junction, becomes maximum for a par- is expressed in volt per
ticular temperature tn and then again starts decreasing.
degree celsius.
The temperature tn of the hot junction at which the
thermo emf becomes maximum is called the neutral tem- Fig. 11. Thermoelectric
perature. It is independent of the temperature of the cold power line.
118 COMPREHENSIVE OBJECTIVE PHYSICS

(a) 3.2 ampere to the left


dV
∴ Thermoelectric power, S = = α + βt (b) 3.2 ampere to the right
dt
(c) 2.18 ampere to the right
The graph between thermoelectric power and tem-
perature t is a straight line (Fig. 11). This line is called (d) 0.90 ampere to the right. [Pb. PMT 2002]
thermoelectric power line. The intercept of the thermo- [2 × 6.4 × 1018 + 7.2 × 1018 ][16
. × 10 –19 ]
electric line is α and the slope is β. Sol. I = A
1
65. tn AND ti IN TERMS OF α AND β
= 3.2 A
α
tn = − The direction of current is in the direction of flow of
β positive charges.
2α So, (b) is the right choice.
ti = –
β Illustration 2. In the circuit shown, R1 = 10 ohm,
FG α IJ R2 = 20 ohm and R3 = 30 ohm. The potentials of points A, B
Now, ti = 2 −
H βK or ti = 2tn and C are 10 V, 6 V and 5 V respectively.

66. TWO LAWS OF THERMOELECTRICITY


(i) Law of successive temperatures (also called
the law of intermediate temperatures).
For a given thermocouple, the emf for any specified
temperature difference is equal to the sum of the emf ’s
corresponding to any smaller intervals into which the given
range of temperature may be sub-divided.

t t t
∴ Vt n = Vtt2 + Vt 3 + Vtt4 + ....... + Vt n
1 1 2 3 n−1
Fig. 12
(ii) Law of successive metals (also called the law The current through resistance R1 is
of intermediate metals). (a) 0.1 A (b) 0.2 A
The insertion of an additional metal or metals into (c) 0.3 A (d) 0.4 A. [BCET 2002]
any thermoelectric circuit does not change the effective emf
Sol. Loop AOBA
of the circuit, provided that both the ends of each such con-
ductor are at the same temperature.
If A and B are the extreme metals, and C, D, E, .....,
K are the intermediate metals, then

VAB = VAC + VCD + VDE + ...... + VKB

67. When an electric current is passed through a junc-


tion of two dissimilar metals, heat is either absorbed
or released at the junction, depending upon the direc-
tion of current. This is called Peltier effect.
68. Thomson effect refers to the emf that develops between
two parts of a single metal when they are at different
temperatures. Fig. 13

ILLUSTRATIONS INDICATING LATEST EXAMINA- – 10 I – 20 I1 + 4 = 0


TION TRENDS or 5 I + 10 I1 = 2 ...(1)
Loop AOCA
Illustration 1. In an electrolyte 6.4 × 1018 bivalent
positive ions drift to the right per second while 7.2 × 1018 – 10 I – 30(I – I1) + 5 = 0
monovalent negative ions drift to the left per second. Then or 2 I + 6 I – 6 I1 = 1
the current is or 8 I – 6 I1 = 1 ...(2)
CURRENT ELECTRICITY 119
Multiplying (1) by 6 and (2) by 10 and adding, Sol. Clearly, R6 is ‘out’. Applying condition for bal-
30 I + 60 I1 + 80 I – 60 I1 = 22 anced Wheatstone bridge,
110 I = 22 R1 R2
=
I = 0.2 A R3 R4
So, (b) is the right choice.
or R1R4 = R2R3
Illustration 3. The effective resistance between
So, (c) is the right choice.
points P and Q of the electrical circuit shown in the figure is
Illustration 5. In the given circuit, with steady cur-
rent, the potential drop across the capacitor must be

Fig. 14 Fig. 16
(a) 2Rr/(R + r) (b) 8R(R + r)/(3R + r) (a) V (b) V/2
(c) 2r + 4R (d) 5R/2 + 2r. (c) V/3 (d) 2V/3.
[IIT Screening 2002] [IIT Screening 2001]
Sol. Considerations of symmetry lead us to the re- Sol. 2 V – 2 IR – IR – V = 0
moval of the cross-pieces, each of resistance 2R. So, we have
V
a parallel combination of 4R, 2r and 4R. If Rp is the equiva- or I=
3R
lent resistance, then
For the upper loop, V + VC – IR – V = 0
1 1 1 1 V
= + +
R p 4R 2r 4R or VC = IR =
3
1 1 1 1 R+r So, (c) is the right choice.
or = + or = Illustration 6. An ammeter and a voltmeter are
R p 2R 2 r Rp 2Rr
joined in series to a cell. Their readings are A and V respec-
2Rr tively. If a resistance is now joined in parallel with the volt-
or Rp =
R+r meter,
So, (a) is the right choice. (a) A will increase, V will decrease
Illustration 4. (b) A will decrease, V will increase
In the given circuit, it (c) both A and V will decrease
is observed that the (d) both A and V will increase.
current I is independ- [Karnataka CET 2002]
ent of the value of the
Sol. Effective resistance decreases, current increases,
resistance R 6 . Then
and voltage decreases.
the resistance values
must satisfy So, (a) is the right choice.
Fig. 15 Illustration 7. A cell supplies a current of 0.9 A
through a 2 Ω resistor and a current of 0.3 A through a 7 Ω
(a) R1R2R5 = R3R4R6
resistor. The internal resistance of the cell is
1 1 1 1 (a) 2.0 Ω (b) 1.2 Ω
(b) + = +
R5 R6 R1 + R2 R3 + R4 (c) 1.0 Ω (d) 0.5 Ω.
(c) R1R4 = R2R3 [Karnataka CET 2002]
(d) R1R3 = R2R4 = R5R6 . [IIT Screening 2001]
120 COMPREHENSIVE OBJECTIVE PHYSICS

Sol. E = I(R + r) ; E constant ; I(R + r) = Constant 12


∴ 0.9 [2 + r] = 0.3 [7 + r] Sol. Current through R =
500 + R
or 6 + 3r = 7 + r
1 12 R
or 2r = 1 or r = Ω = 0.5 Ω Voltage across R =
2 500 + R
So, (d) is the right choice. Since galvanometer shows zero deflection,
Illustration 8. An energy source will supply a con- 12 R
stant current into the load if its internal resistance is =2
500 + R
(a) non-zero but less than the resistance of the load or 12 R = 1000 + 2 R
(b) zero
or 10 R = 1000 or R = 100 Ω
(c) very large as compared to the load resistance
So, (c) is the right choice.
(d) equal to the resistance of the load.
Illustration 11. If a good conductor is connected
[AIEEE 2005] between a point of an electric circuit and the ground
Sol. In the case of an ideal battery, the internal (a) current flows from the point to the earth.
resistance is zero.
(b) the potential of the point is the same as that of
So, (b) is the correct choice. the earth.
Illustration 9. Two sources of equal emf are con- (c) current flows through the conductor only if one
nected to an external resistance R. The internal resistances more point of the same circuit with some resistance between
of the two sources are R1 and R2 (R2 > R1). If the potential the two is connected to the ground.
difference across the source having internal resistance R2 is
(d) when the current flows through the conductor to
zero, then
the earth the current through the circuit remains unchanged.
(a) R = R1R2/(R2 – R1)
[National Standard Exam. in Physics 2005]
(b) R = R1R2/(R1 + R2) Ans. (b) and (c) [Based on factual informations]
(c) R = R2 – R1 Illustration 12. For the network shown in Fig. 19
(d) R = R2 × (R1 + R2)/(R2 – R1). [AIEEE 2005] the value of the current i is
Sol. V = E – IR2 R
2W
V=0
E = IR2
4W
W

2ER 2 W
4

E= 3
R + R1 + R 2 E, R1 E, R2
or R + R1 + R2 = 2R2 Fig. 17 6W
i
or R = R2 – R1
So, (c) is the right choice.
Illustration 10. In the circuit, the galvanometer G Fig. 19
shows zero deflection. If the batteries A and B have negligi- 18 V 5V
ble internal resistance, the value of the resistor R will (a) (b)
5 9
be
9V 5V
(c) (d) .
500  35 18
G [All India PM/PD 2005]
2V
A Sol. The given network is balanced Wheatstone bridge.
12 V
B
R So, we have a parallel combination of 6 Ω and 9 Ω. This
6×9 18
gives Ω i.e. Ω.
6+9 5
Fig. 18 V 5V
Now, i = =
(a) 1000 Ω (b) 500 Ω 18/5 18
(c) 100 Ω (d) 200 Ω. [AIEEE 2005] So, (d) is the right choice.
CURRENT ELECTRICITY 121
Illustration 13. When a wire of uniform cross-sec- N2
tion a, length l and resistance R is bent into a complete ∴ = 4 or N2 = 36 or N = 6
9
circle, resistance between any two of diametrically opposite
So, (b) is the right choice.
points will be
Illustration 16. A heater coil is cut into two equal
R R parts and only one part is now used in the heater. The heat
(a) (b)
8 2 generated will now be
R (a) halved (b) one-fourth
(c) 4 R (d) .
4 (c) four times (d) doubled. [AIEEE 2005]
[All India PM/PD 2005] V2
Sol. Q= t
R R R R
Sol. The parallel combination of and gives . When R is halved, then Q is doubled.
2 2 4
So, (d) is the right choice. So, (d) is the right choice.
B1 B2
Illustration 14. A 100 W bulb B1 , Illustration 17. The resistance of hot tungsten fila-
and two 60 W bulbs B2 and B 3 , are con- ment is about 10 times the cold resistance. What will be the
nected to a 250 V source, as shown in the resistance of 100 W and 200 V lamp when not in use ?
Fig. 20. Now W1 , W2 and W3 are the output (a) 200 Ω (b) 400 Ω
powers of the bulbs B1 , B2 and B3 , respec- B3
(c) 20 Ω (d) 40 Ω. [AIEEE 2005]
tively. Then
Sol. Rhot = 10 Rcold
(a) W1 > W2 = W3
2
(b) W1 > W2 > W3 250 V Rhot = V = 200 × 200 = 400 Ω
(c) W1 < W2 = W3 P 100
(d) W1 < W2 < W3 . 400
Fig. 20 ∴ Rcold = Ω = 40 Ω
10
[IIT Screening 2002]
So, (d) is the right choice.
V 2 V2
Sol. R1 = , R2 = = R3 Illustration 18. A 5-ampere fuse wire can withstand
100 60 a maximum power of 1 watt in the circuit. The resistance of
(250)2 (250)2 the fuse wire is :
∴ W3 = ; W2 = R ;
R3 (R 1 + R2 )2 2 (a) 0.4 Ω (b) 5 Ω
(250)2 (c) 0.04 Ω (d) 0.2 Ω.
W1 = R
(R 1 + R 2 )2 1 [All India PM/PD 2005]
W3 : W2 : W1 = 64 : 25 : 15 Sol. P = I2R
So, (d) is the right choice. P 1
Illustration 15. A wire of length L and 3 identical
or R=
2
= Ω = 0.04 Ω
I 5×5
cells of negligible internal resistances are connected in se- So, (c) is the right choice.
ries. Due to the current, the temperature of the wire is raised
Illustration 19. Two voltameters, one of copper and
by ∆T in a time t. A number N of similar cells is now con-
another of silver, are joined in parallel. When a total charge
nected in series with a wire of the same material and cross-
q flows through the voltameters, equal amount of metals
section but of length 2L. The temperature of the wire is
are deposited. If the electrochemical equivalents of copper
raised by the same amount ∆T in the same time t. The
and silver are z1 and z2 respectively the charge which flows
value of N is
through the silver voltameter is
(a) 4 (b) 6
(c) 8 (d) 9. z2 z1
(a) q (b) q
[IIT Screening 2001] z1 z2
V2 q q
Sol. t = mc∆T (c) (d) . [AIEEE 2005]
R z z1
1+ 2 1+
For the given data, V 2 ∝ Rm z1 z2
Now, (3V)2 ∝ Rm 1
Sol. m = zq, z ∝
(NV)2 ∝ (2R)(2m) q
122 COMPREHENSIVE OBJECTIVE PHYSICS

z1 q2 Sol. θn – θ0 = θi – θn
= or θ0 = 2θn – θi
z2 q1
= (2 × 300 – 620)°C
Now, q = q1 + q2
= – 20°C
q q
= 1 +1 So, (d) is the right choice.
q2 q2
Illustration 21. The emf developed by a thermo-
q z couple is measured with the help of a potentiometer and
= 2 +1
q2 z1 not by a moving coil millivoltmeter because
q (a) the potentiometer is more accurate than the volt-
or q2 = meter.
z2
+1
z1 (b) the potentiometer is more sensitive than voltme-
So, (c) is the right choice. ter.
Illustration 20. The temperature of inversion of a (c) the potentiometer makes measurement without
thermocouple is 620°C and the neutral temperature is 300°C. drawing any current from the thermocouple.
What is the temperature cold junction ? (d) measurement using a potentiometer is simpler
(a) 20°C (b) 40°C than with a voltmeter.
[National Standard Exam. in Physics 2005]
(c) 320°C (d) – 20°C.
Sol. Note that emf is the terminal potential differ-
[All India PM/PD 2005]
ence when no current is drawn from the source.
Clearly, (c) is the right choice.
CURRENT ELECTRICITY 123

QUESTION BANK

MCQs
SET I

based on
Memory Work, Hard Facts, Important Terms,
Important Formulae etc.

Average time allowed per question is 20 seconds.

R
CURRENT ELECTRICITY (a) nR (b)
n
1. A copper wire is stretched to make it 0.1% longer. n R
(c) (d) . [MP PMT 1998]
The percentage increase in resistance will be R n2
(a) 0.2 (b) 2 8. In the circuit below, bulb B does not light although
(c) 1 (d) 0.1. [MP PMT 2000] ammeter A indicates that the current is flowing. Why
2. In a potentiometer experiment the balancing with a does the bulb not light ?
cell is at length 240 cm. On shunting the cell with a
resistance of 2 Ω , the balancing length becomes
120 cm. The internal resistance of the cell is
A
(a) 2 Ω (b) 4 Ω B
(c) 0.5 Ω (d) 1 Ω. [AIEEE 2005]
3. How many electrons per second constitute a current Variable
resistor Switch
of one micro ampere ?
(a) one electron (b) 10 –6 electrons Battery

(c) 106 electrons (d) 6.25 × 10 12 electrons.


[EAMCET 1991]
4. The thermal velocity of free electrons in a conductor Fig. 21
is of the order of (a) The bulb is fused.
(a) 1 m s–1 (b) 10 m s–1
(b) There is a break in the circuit between bulb and ammeter.
(c) 103 m s–1 (d) 106 m s–1. [Assam 1993]
(c) The variable resistor has too large a resistance.
5. A typical value of drift velocity is
(d) There is a break in the circuit between bulb and variable
(a) 1 mm s–1 (b) 1 m s–1
resistor. [WBJEE 2000]
(c) 10 m s–1 (d) 1000 km s–1.
[Orissa 1992] 9. A piece of wire of resistance 4 ohm is bent through
6. Ten identical resistances, each having a resistance of 180° at its mid-point and the two halves are twisted
1 Ω , are joined in parallel. The combination has the together. Then the resistance is
resistance of (a) 8 ohm (b) 1 ohm
(a) 10 Ω (b) 1 Ω (c) 2 ohm (d) 5 ohm.
(c) 0.1 Ω (d) 0.01 Ω . [Pb. CET 1999] [Himachal PMT 1994]
7. A wire of resistance R is cut into n equal parts. These 10. Which one of the following is best material for mak-
parts are then connected in parallel. The equivalent ing connecting wires ?
resistance of the combination is
124 COMPREHENSIVE OBJECTIVE PHYSICS

(a) manganin (b) constantan What are the voltmeter readings ?


(c) copper (d) nichrome. [AFMC 1997]
11. Three equal resistors are given. How many different
combinations can be made from them ?
(a) three (b) four
(c) five (d) six. [EAMCET 1999]
12. The current in a copper wire is increased by increas-
ing the potential difference between its ends. Which
one of the following statements regarding n, the
number of charge carriers per unit volume in the Fig. 22
wire and v, the drift velocity of the charge carriers is
correct ? X reading Y reading
(a) n is unaltered but v is decreased (a) 0V 0V
(b) n is unaltered but v is increased (b) 0V 240 V
(c) n is increased but v is decreased (c) 40 V 40 V
(d) n is increased but v is unaltered
(d) 240 V 0V
(e) Both n and v are increased.
[Cambridge Local Examinations 18. For which of the following dependence of drift veloc-
Syndicate] ity vd on electric field E, Ohm’s law is obeyed ?
13. A piece of copper and a piece of germanium are cooled (a) vd ∝ E2 (b) vd ∝ E
from the room temperature down to 77 K.
(c) vd ∝ E (d) vd ∝ E0. [DPMT 1998]
(a) The resistance of each of them increases.
19. The example of non-ohmic resistance is
(b) The resistance of each of them decreases.
(a) copper wire (b) carbon resistance
(c) The resistance of copper increases and that of germanium (c) silver wire (d) tungsten wire.
decreases.
20. V-I graphs for parallel
(d) The resistance of copper decreases and that of germa-
and series combination
nium increases. [AIEEE 2003, 2004 ; IIT 1988]
of two metallic resistors
14. If the resistivity of an alloy is ρ′ and that of constitu- are as shown in Fig. 23.
ent metals is ρ, then Which graph represents
(a) ρ′ > ρ (b) ρ′ < ρ parallel combination ?
(c) ρ′ = ρ (a) A
(d) There is no simple relation between ρ and ρ′. (b) B
[Karnataka 1994] (c) A and B both
(d) Neither A nor B. Fig. 23
15. The capacity of a storage cell is 5 ampere hour. The
[MAHE 1999]
maximum current it can supply for ten hour is
(a) 5 ampere (b) 0.5 ampere
21. The current in the arm CD of the network in Fig. 24
is
(c) 50 ampere (d) 0.25 ampere.
[AFMC 2002]
16. Mains electrical equipment is sometimes provided
with a neon indicator lamp to show that the equip-
ment is switched on. Which charge carriers consti-
tute the current in the neon ? Assume that the volt-
age is above 12 V.
(a) electrons only (b) positive ions only
(c) negative ions only (d) holes and electrons
(e) positive ions and electrons.
17. A mains circuit contains six similar bulbs connected Fig. 24
in series. One of the bulbs has a broken filament. (a) I1 + I2 (b) I2 + I3
Voltmeters X and Y of infinite resistance are placed (c) I1 + I3 (d) I1 – I2 + I3.
in the circuit as shown.
[MP PMT 1998, 2000]
CURRENT ELECTRICITY 125
22. A car battery has an emf of 12 volt. Eight or nine (a) 10 Ω (b) 20 Ω
ordinary cells connected in series can give 12 volt. (c) 40 Ω
But this arrangement is not used because (d) None of the above three values. [MP PMT 2000]
(a) the internal resistance of 8 or 9 cells can be very large as
30. A student has 10 identical resistors, each of resist-
compared to the internal resistance of the battery.
ance ‘r’. The minimum resistance made by him from
(b) the cell arrangement cannot supply large current required
given resistors is
to start the car.
r
(c) of both of the above. [Haryana 1999] (a) 10r (b)
10
23. An electric cell does 5 joule of work in carrying 10 C
of charge around a closed electric circuit. The elec- r r
(c) (d) .
tromotive force of the cell is 100 5
(a) 2 V (b) 2.5 V 31. For measurement of potential difference, potentio-
(c) 3 V (d) 0.5 V. [AFMC 1999] meter is preferred in comparison to voltmeter because
(a) Potentiometer is more sensitive than voltmeter
24. Inside the electric cell, charge is transported by
(b) The resistance of potentiometer is less than voltmeter
(a) electrons (b) positive ions only
(c) negative ions only (c) Potentiometer is cheaper than voltmeter
(d) both positive and negative ions. (d) Potentiometer does not takes current from the circuit.
25. A steady current flows in a metallic conductor of non- [MP PET 1983]
uniform cross-section. The quantity/quantities con- 32. Efficiency of alkali accumulator is less than that of
stant along the length of the conductor is/are
lead accumulator on account of
(a) drift speed only
(a) lower emf
(b) current, electric field and drift speed
(b) smaller amount of active material
(c) current and drift speed
(c) higher internal resistance
(d) current only. [IIT 1997]
(d) higher current.
26. Potential difference of a cell in an open circuit is 6
volt but it falls to 4 volt when a current of 2 A is 33. What is the resistance of the portion of a circuit
drawn from it. The internal resistance of the cell is between A and B in the figure given below. Points A
(a) 1 Ω (b) 2 Ω and ‘a’ are connected by a copper wire of negligible
resistance. Similarly points B and ‘b’ are also
(c) 3 Ω (d) 4 Ω. [Pb. PMT 1998]
connected by a copper wire of negligible resistance.
27. Current density is
(a) a scalar quantity
(b) a vector quantity
(c) neither a scalar nor a vector quantity
(d) sometimes scalar and sometimes a vector quantity.
28. The positive terminal of a 12 V battery is connected
to the earth. Then the negative terminal will be at
(a) zero potential (b) – 12 V Fig. 26
(c) + 12 V (d) + 24 V. (a) 3R (b) R/3
29. The effective resistance between points A and B is (c) 3R/2 (d) 2R/3. [AFMC 2000]
34. The diagrams show five different ways in which cur-
rents I1 , I2 , I3 can combine at a junction.

Fig. 25
126 COMPREHENSIVE OBJECTIVE PHYSICS

(c) the number density of free electrons as well as the elec-


tron drift speeds are very large and these compensate for
the very small magnitude of the electron charge
(d) the very small magnitude of the electron charge has to be
divided by the still smaller product of the number density
and drift speed to get the electric current.
[KCEE 1996]
39. A series resistance of 8.5 Ω is required for connect-
Fig. 27 ing an arc lamp having the rated voltage and cur-
rent of 42 V and 10 A respectively to a circuit. The
For which of these junctions, is the following equa-
tion correct ? voltage in the circuit is
I1 + I2 = I3 + I4 (a) 169 V (b) 149 V
(a) A (b) B (c) 127 V (d) 113 V. [EAMCET 1997]
(c) C (d) D 40. A primary cell has an emf of 1.5 volt. When short-
(e) E. [EAMCET 1997] circuited, it gives a current of 3 A . The internal re-
35. The resistance of the circuit shown in Fig. 28 is sistance of the cell is
(a) 4.5 Ω (b) 2 Ω
1
(c) 0.5 Ω (d) Ω. [CPMT 1993]
4.5
41. The resistance of a conductor carrying a current of
2 A , when potential difference across it is 20 V, is
(a) 0.1 Ω (b) 1 Ω
(c) 10 Ω (d) 40 Ω. [Haryana 1996]
42. In the measurement of resistance by a metre bridge,
the known and the unknown resistances are inter-
changed to eliminate
Fig. 28 (a) end errors (b) index error
(c) error due to thermoelectric effect
(a) 18r (b) 9r
(d) random error. [CEE 1988]
(c) 6r (d) 3r.
43. The conductivity of a super conductor is
36. A 50 volt battery is connected across a 10 ohm resis-
(a) infinite (b) very large
tor. The current is 4.5 ampere. The internal resist-
(c) very small (d) zero. [MP PMT 1997]
ance of the battery is
(a) zero (b) 0.5 ohm 44. Carbon resistors used in electronic circuits are
marked for their resistance value and tolerance by a
(c) 1.1 ohm (d) 5.0 ohm. [AMU 1994] colour scheme. A given resistor has a colour scheme
37. When both the length and area of cross-section of a brown, black, green and gold. Its value is (in ohm)
wire are doubled, then its resistance (a) 3.2 × 105 ± 5% (b) 1.0 × 106 ± 10%
(a) will become half (b) will be doubled (c) 1.0 × 106 ± 5% (d) 1.0 × 103 ± 5%.
(c) will remains the same (d) will become four times. [Karnataka 1994]
[MP PET 1989] 45. Three resistors, each of 2 ohm, are connected together
in a triangular shape. The resistance between any
38. We are able to obtain fairly large currents in a con-
two vertices will be
ductor because
(a) 4/3 ohm (b) 3/4 ohm
(a) the electron drift speed is usually very large
(c) 3 ohm (d) 6 ohm.
(b) the number density of free electrons is very high and this
can compensate for the low values of the electron drift
[MP PMT 1993]
speed and the very small magnitude of the electron charge 46. Fig. 29 shows current in a part of electric circuit.
The current I is
CURRENT ELECTRICITY 127
51. Two resistors of resistances R1 and R2 having R1 > R2
are connected in parallel. For equivalent resistance
R, the correct statement is
(a) R > (R1 + R2) (b) R1 < R < R2
(c) R2 < R < (R1 + R2) (d) R < R1. [CPMT 1998]
52. A solenoid is at a potential difference of 60 V and
current flowing through it is 15 ampere. Then the
resistance of coil will be
(a) 4 Ω (b) 8 Ω
Fig. 29 (b) 0.25 Ω (d) 2 Ω. [RPMT 2000]
(a) 1.7 A (b) 3.7 A 53. A piece of wire is cut into four equal parts and the
(c) 1.3 A (d) 1 A. pieces are bundled together side by side to form a
[Karnataka CET 2000] thick wire. Compared with that of the original wire,
the resistance of the bundle is
47. Two unequal resistances are connected in parallel.
Which one of the statements is correct ? 1
(a) the same (b)
(a) The current flowing is same in both 4
(b) More current will flow from higher resistance piece 1 1
(c) (d) . [JIPMER 2001]
(c) The potential drop is same in both 8 16
(d) All of the above. [BHU 2000] 54. What is the number of equal parts into which a con-
48. The instrument for the accurate determination of the ductor having a resistance R0 = 100 Ω should be cut
emf of a cell is to obtain the resistance R = 1 Ω if the parts are con-
(a) voltmeter (b) ammeter nected in parallel ?
(c) potentiometer (d) slide wire bridge. (a) 5 (b) 10
[Karnataka 1993] (c) 20 (d) 2. [AMU 1998]
49. The balance point of a slide wire Wheatstone bridge 55. The resistivity of iron is 1 × 10 –7
Ω m. The resistance
will not be changed by increasing of an iron wire of particular length and thickness is
1. the emf of the driver cell. 1 ohm. If both the length and the diameter of wire
2. the resistance of the slide wire. are doubled, then the resistivity in ohm metre will
3. the galvanometer resistance. be
(a) 1, 2, 3 correct (b) 1 and 2 correct (a) 1 × 10–7 (b) 2 × 10–7
(c) 1 and 3 correct (d) 1 only correct (c) 4 × 10–7 (d) 8 × 10–7. [CPMT 1993]
(e) 3 only correct. [CAMBRIDGE] 56. The resistance of a wire of uniform diameter d and
50. Which of two switches S1 and S2 shown in Fig. 30 length l is R. The resistance of another wire of the
will produce short-circuiting ? same material but diameter 2d and length 4l will be
(a) 2R (b) R
R R
(c) (d) . [BPMT 1999]
2 4
57. Which graph best represents the way in which the
current I through a thermistor depends upon the
potential difference V across it ?

Fig. 30
(a) S1 (b) S2
(c) Both S1 and S2 (d) Neither S1 nor S2.
[Pb. PMT 1999]
128 COMPREHENSIVE OBJECTIVE PHYSICS

(a) 3 V (b) 15 V
(c) – 5.1 V (d) + 5.1 V.
[CMCLDH 1999]
62. The electric resistance of a certain wire of iron is R.
If its length and radius are both doubled, then
(a) the resistance will be halved and the specific resistance will
remain unchanged.
Fig. 31 (b) the resistance will be halved and the specific resistance will
be doubled.
58. The diagram shows two squares, X and Y, cut from
a sheet of metal, of uniform thickness t. X and Y (c) the resistance and the specific resistance will both remain
have sides of length L and 2L respectively. unchanged.
X Y (d) the resistance will be doubled and the specific resistance
2L
will be halved. [All India PM/PD 2004]
63. N identical cells, each of emf E and internal resist-
t
L 2L ance r, are joined in series to form a closed circuit.
L t One cell A is joined with reverse polarity. The poten-
tial difference across each cell except A is
2E FG N – 1IJ E
(a) (b)
(a)
N
(b)
H N K
FG N IJ E (d) FG
N – 2I
H N JK E .
Fig. 32

The resistances RX and RY of the squares are meas-


(c)
H N – 1K
ured between the opposite faces shaded in the dia- [BHU 1999]
RX 64. Resistances n, each of r ohm, when connected in par-
gram. What is the value of ?
RY allel give an equivalent resistance of R ohm. If these
(a) 1/4 (b) 1/2
resistances were connected in series, the combina-
tion would have a resistance in ohm, equal to
(c) 1 (d) 2
(a) R/n2 (b) R/n
(e) 4.
(c) nR (d) n2R
59. In a potentiometer experiment, the balancing length
with a cell is 560 cm. When an external resistance of [All India PM/PD 2004]
10 Ω is connected in parallel to the cell, the balanc- 65. The thermistors are usually made of
ing length changes to 412 cm. The internal resist- (a) metal oxides with high temperature coefficient of
ance of the cell in ohm is resistivity.
(a) 3.6 (b) 2.4
(b) metals with high temperature coefficient of resistivity.
(c) 1.2 (d) 0.6. [EAMCET 2003]
(c) metals with low temperature coefficient of resistivity.
60. In a metre bridge, with a standard resistance of 5 Ω (d) semiconducting materials having low temperature coeffi-
in the left gap, the ratio of balancing lengths on the
cient of resistivity. [AIEEE 2004]
metre bridge wire is 2 : 3. The unknown resistance is
(a) 1 Ω (b) 15 Ω 66. In a Wheatstone’s bridge all the four arms have equal
resistance R. If the resistance of the galvanometer
(c) 10 Ω (d) 3.3 Ω
arm is also R, the equivalent resistance of the combi-
(e) 7.5 Ω [Kerala PMT 2003]
nation as seen by the battery is
61. The potential difference between points A and B in
R R
the Fig. 33 is (a) (b)
4 2
6V
2W
9V
0.7 W
(c) R (d) 2R
3A
[All India PM/PD 2003]
A B

Fig. 33
CURRENT ELECTRICITY 129
67. A group of N cells whose emf var- 1
I (d)
30 (E – 0.5i)
, where i is the current in the potentiometer
ies directly with the internal re- r1 100
sistance as per the equation N r r2
2 wire. [AIEEE 2003]
N
EN = 1.5 rN are connected as 72. The potential difference between the terminals of a
shown in Fig. 34. The current I r3 cell in open circuit is 2.2 volt. With resistance of 5 ohm
in the circuit is : r4 3 across the terminals of a cell, the terminal potential
(a) 5.1 A (b) 0.51 A 4 difference is 1.8 volt. The internal resistance of the
(c) 1.5 A (d) 0.15 A Fig. 34 cell is
[KCET 2003] 9 10
(a) ohm (b) ohm
10 9
68. A potentiometer has uniform potential gradient. The
specific resistance of the material of the potentiometer 7 12
(c) ohm (d) ohm [KCET 2003]
wire is 10 –7 ohm metre and the current passing 12 7
through it is 0.1 ampere. The cross-section of the 73. Resistors, each of value 1 Ω , are arranged as shown
wire is 10–6 m2 . The potential gradient along the in Fig. 36. The equivalent resistance between points
potentiometer wire is A and B is
(a) 10–6+ Vm–1 (b) 10–4 Vm–1
(c) 10 Vm
–8 –1
(d) 10–2 Vm–1. [KCET 2003] A B
69. A wire of length L is drawn such that its diameter is
reduced to half of its original diameter. If the initial
resistance of the wire were 10 Ω, its new resistance
would be : Fig. 36
(a) 40 Ω (b) 80 Ω
1 1
(c) 120 Ω (d) 160 Ω [AIIMS 2003] (a) Ω (b) Ω
2 5
70. The temperature (T) dependence of resistivity (ρ) of a
semiconductor is represented by 5
(c) Ω (d) none of the above
4
(a) (b)
[National Standard Exam. in Physics 2004]
74. A battery of emf E and negligible internal resistance
  is connected to two resistors of resistances R1 and
R2 as shown in the circuit diagram.
O O
T T

(c) (d) R1

 E


O O R2
T T

Fig. 35
[AIIMS 2004] Fig. 37
71. The length of a wire of a potentiometer is 100 cm and
What is the potential difference across the resistor of
the emf of its standard cell is E volt. It is employed resistance R2 ?
to measure the emf of a battery whose internal re-
E(R 1 + R 2 ) ER 2
sistance is 0.5 Ω. If the balance point is obtained at (a) (b)
R1 R1 + R2
l = 30 from the positive end, the emf of the battery is
30 E 30 E E(R 1 + R 2 ) ER 2
(a) (b) (c) (d)
100 100.5 R2 R1
30 E ER1
(c) (e) .
(100 – 0.5) R1 + R2
130 COMPREHENSIVE OBJECTIVE PHYSICS

(a) 1.4 × 10 3 W (b) 1.3 × 103 W


THERMAL EFFECTS OF CURRENT
(c) 1.08 × 103 W (d) 9.68 × 102 W.
75. Three electric bulbs 40 W, 60 W and 100 W are de- 83. A 2 Ω and a 2/3 Ω resistor are connected in parallel
signed to work on a 220 V mains. Which bulb will across a 3 V battery. The energy given out per minute
burn most brightly if they are connected in series is
across 200 V mains ? (a) 60 × 2 × 3 J (b) 60 × 9/2 × 3 × 3 J
(a) 100 W bulb (b) 60 W bulb (c) 60 × 1/2 × 3 × 3 J (d) 60 × 3 × 3 × 2 J.
(c) 40 W bulb 84. A 1°C rise in temperature is observed in a conductor
(d) All bulbs will burn equally brightly. by passing a certain current. If the current is dou-
76. Two electric bulbs, rated for the same voltage, have bled, then the rise in temperature is approximately
powers of 200 W and 100 W. If their resistances are (a) 2.5°C (b) 4°C
r1 and r2 respectively, then (c) 2°C (d) 1°C.
(a) r1 = 2r2 (b) r2 = 2r1 85. If the current in electric bulb decreases by 0.5%, then
(c) r2 = 4r1 (d) r1 = 4r2. the power in the bulb decreases by approximately
77. The water in an electric kettle begins to boil in (a) 1% (b) 2%
15 minutes after being switched on. Using the same (c) 0.5% (d) 0.25%.
mains supply, should the length of the wire used as 86. Two electric bulbs have tungsten filament of same
the heating element be increased or decreased if the length. If one of them gives 60 watt and other
water is to boil in 10 minutes ? 100 watt, then
(a) decreased (b) increased (a) 100 watt bulb has thicker filament.
(c) unchanged (d) none of the above. (b) 60 watt bulb has thicker filament.
78. A secondary battery has a 20 hour charge rate of 10 (c) both filaments are of same thickness.
ampere and delivers 5 ampere for 36 hours on (d) it is not possible to get different wattages unless the lengths
discharge with a terminal voltage of 19.6 V. The are different.
ampere-hour efficiency is 87. Two heating coils, one of the fine wire and the other
(a) 9% (b) 90% of thick wire, made of the same material and of same
(c) 20% (d) 100%. length are connected in series and in parallel. Which
of the following statements is correct ?
79. Watt-hour meter measures
(a) In series, fine wire will liberate more energy while in par-
(a) electrical energy (b) current
allel thick one will liberate more energy.
(c) voltage (d) power.
(b) In series, fine wire will liberte less energy while in parallel
[Karnataka 1994] more energy.
80. An electric bulb rated for 500 W at 100 V is used in a (c) Both will liberate equal energy.
circuit having a 200 V supply. The resistance R that (d) In series, thick wire will liberate more energy while in
must be put in series with the bulb, so that the bulb parallel less energy.
draws 500 W is 88. Constantan wire is used in making standard resis-
(a) 18 Ω (b) 20 Ω tances, because its
(c) 40 Ω (d) 700 Ω. (a) specific resistance is low (b) density is high
[Himachal PMT 2002] (c) temperature coefficient of resistance is negligible
81. The time required by 1 kW heater to raise the tem- (d) melting point is high. [MP PET 1999]
perature of 10 litre of water through 10°C is 89. n identical light bulbs, each designed to draw P power
(a) 42 s (b) 210 s from a certain voltage supply, are joined in series
(c) 420 s (d) 840 s. across that supply. The total power which they will
82. The power of a heater is 1000 W at 1000°C. What will draw is
be its power at 400°C ? Given : temperature coeffi- (a) nP (b) P
cient of resistance of heater-wire is 1.4 × 10 – 4 °C–1. (c) P/n (d) P/n2.
[Karnataka CET 2002]
CURRENT ELECTRICITY 131

90. The heating element of an electric heater should be (a) 20 Ω (b) 10 Ω


made with a material which should have. (c) 40 Ω (d) 30 Ω [KCET 2003]
(a) high specific resistance and high melting point. 99. Time taken by a 836 W heater to heat one litre of
(b) high specific resistance and low melting point. water from 10°C to 40°C is
(c) low specific resistance and low melting point. (a) 150 s (b) 100 s
(d) low specific resistance and high melting point. (c) 50 s (d) 200 s [AIEEE 2004]
[MNR 1980] 100. A constant voltage is applied between the two ends of
91. Two electric bulbs whose resistances are in the ratio a metallic wire. If both the length and the radius of
of 1 : 2 are connected in parallel to a constant voltage the wire are doubled, the rate of heat developed in
source. The powers dissipated in them have the ratio the wire will
(a) 1 : 2 (b) 1 : 1 (a) be halved (b) be doubled
(c) 2 : 1 (d) 1 : 4. [NCERT 1977] (c) remain the same (d) be quadrupled.
92. A resistor R1 dissipates the power P when connected 101. The power rating of an electric motor which draws a
to a certain generator. If a resistor R2 is put in series
current of 3.75 A, when operated at 200 V, is nearly
with R1, the power dissipated by R1
(a) 54 W (b) 1 hp
(a) decreases (b) increases
(c) 500 W (d) 750 hp
(c) remains the same
(d) any of the above depending upon the relative values of R1 102. A cable of resistance 10 Ω carries electric power from
and R2. [CPMT 1985] a generator producing 250 kW at 10000 V. The cur-
rent in the cable is
93. How many calories of heat will be approximately de-
(a) 1000 A (b) 250 A
veloped in a 210 watt electric bulb in 5 minutes ?
(a) 15000 (b) 1050 (c) 100 A (d) 25 A.
(c) 63000 (d) 80000 103. In the previous question, the power lost in the cable
(e) none of the above. [MNR 1985] during transmission is
(a) 3.15 kW (b) 12.5 kW
94. A frill of hundred electric bulbs is connected in series
(c) 6.25 kW (d) 25 kW.
across a 220 V supply. After ten bulbs are fused, the
remaining 90 are connected again in series across 104. The heat generated through 4 Ω and 9 Ω resistances
the same supply. The illumination will be separately, when a capacitor of 100 µF capacity
(a) more with 100 bulbs than with 90.
charged to 200 V is discharged one by one, will be
(b) more with 90 bulbs than with 100. (a) 2 J and 8 J respectively (b) 8 J and 2 J respectively

(c) equal in both the cases. (c) 2 J and 4 J respectively (d) 2 J and 2 J respectively.
(d) in the ratio 1002 : 902. 105. If the length of the filament of a heater is reduced by
95. The fuse wire is made up of 10%, the power of the heater will
(a) Lead-tin alloy (b) tungsten (a) increase by about 9% (b) increase by about 11%
(c) increase by about 19% (d) decrease by about 10%.
(c) copper (d) nichrome.
96. Fuse wire is a wire of [MNR 2002]
(a) high resistance and high melting point 106. A 2 kW boiler used for 1 h every day consumes the
(b) high resistance and low melting point following electrical energy in 30 days
(c) low resistance and low melting point (a) 60 units (b) 120 units
(d) low resistance and high melting point. (c) 15 units (d) none of the above.
[All India PM/PD 2003] [Karnataka 1993]
97. A 220 volt, 1000 watt bulb is connected across a 107. Two bulbs which consume powers P1 and P2 are con-
110 volt mains supply. The power consumed will be nected in series. The power consumed by the combi-
(a) 1000 watt (b) 750 watt nation is
(c) 500 watt (d) 250 watt [AIEEE 2003] (a) P1 + P2 (b) P1P2
98. If a 30 V, 90 W bulb is to be worked in 120 V line, the (c) P1P2 /(P1 + P2) (d) 2P1P2 /(P1 + P2).
resistance to be connected in series with the bulb is
[MNR 1999]
132 COMPREHENSIVE OBJECTIVE PHYSICS

108. Which statement describes the electrical potential (a) 4m (b) m/2
difference between two points in a wire carrying a (c) m/4 (d) 2m.
current ? 114. Two electroplating cells, one silver and another alu-
(a) The force required to move a unit positive charge between minium, are connected in series. After 30 minute op-
the points. eration, the number of silver atoms deposited to the
(b) The ratio of the energy dissipated between the points to number of aluminium atoms deposited is
the current. (a) 3 : 1 (b) 1 : 3
(c) The ratio of the power dissipated between the points to (c) 9 : 1 (d) 1 : 9.
the current.
115. The ratio of the number of hydrogen to that of oxy-
(d) The ratio of the power dissipated between the points to
gen atoms liberated in the electrolysis of acidulated
the charge moved. water is
109. Two cells, each of emf E and internal resistance r, (a) 1 : 4 (b) 1 : 2
are connected in parallel across a resistor R. The (c) 2 : 1 (d) 4 : 1.
power delivered to the resistor is maximum if R is
116. A charge of 4.8 × 10 5
C passes through an electro-
equal to
lytic solution of copper sulphate. The number of Cu++
r ions liberated from the electrolyte is
(a) (b) r
2 (a) 3 × 1024 (b) 1.5 × 1024
(c) 2r (d) 0.[AFMC 1997]
(c) 7.68 × 10 –14 (d) 1.25 × 1013.
110. Consider the following passage, which contains an
117. To deposit two litre of hydrogen at 11.2 atmosphere
error. ‘In POOR conductors ELECTRICAL energy is
from acidulated water, the charge that must be passed
transferred by lattice VIBRATIONS. These travel through is
through the specimen as WAVES which are SCAT- (a) 11.2 C (b) 22.4 C
TERED by the lattice as they progress through it’.
(c) 96.5 kC (d) 193 kC.
What is the error ?
118. A certain charge liberates 0.8 g of oxygen. The same
(a) POOR should be METALLIC.
charge will liberate
(b) ELECTRICAL should be THERMAL. (a) 0.8 g of silver (b) 10.8 g of silver
(c) VIBRATIONS should be IMPERFECTIONS. (c) 108 g of silver (d) 108/0.8 g of silver.
(d) WAVES should be PHOTONS. 119. Faraday constant
(e) SCATTERED should be REFRACTED. (a) depends on the amount of the electrolyte
(b) depends on the current in the electrolyte
CHEMICAL AND THERMOELECTRIC (c) is a universal constant
EFFECTS OF CURRENT (d) depends on the amount of charge passed through the elec-
trolyte.
111. If 1 A of current is passed through CuSO4 solution 120. In a copper voltameter, mass deposited in 30 s is
for 10 s, then the number of copper ions deposited at m g. If time-current graph is as shown (see Fig. 38),
the cathode will be then ECE of copper is
(a) 1.6 × 1019 (b) 3.125 × 1019
(c) 4.8 × 1019 (d) 6.2 × 1019.
112. Silver and zinc voltameter are connected in series
and same current is passed through both of them for
same time. If x kg of silver is liberated, then the
amount of zinc liberated will be nearly
(a) x (b) x/5
(c) x/3 (d) x/2.
113. In an electroplating experiment, m g of silver is de-
posited when 4 A current flows for 2 minutes. The Fig. 38
amount (in g) of silver deposited by 6 A of current for (a) m (b) m/2
40 s will be (c) 0.1 m (d) 0.6.
CURRENT ELECTRICITY 133

121. Electric current is passed through the following so- Peltier effect is caused
lutions. In which case hydrogen will be liberated at (a) due to both A and B.
the cathode ? (b) due to A but not due to B.
(a) Sugar (b) Sodium hydroxide (c) due to B but not due to A.
(c) Sulphuric acid (d) Copper sulphate.
(d) neither due to A nor due to B. [AIIMS 1998]
[Pb. PMT 2002]
129. In an electrolysis experiment, a current I passes
122. If 1 mole of electrons contains 6.02 × 10 23 electrons, through two different cells in series, one containing
then the value of Faraday constant F in C mol–1 is a solution of CuSO 4 and the other a solution of
96320 AgNO 3 . The rate of increase of the weights of the
(a) (b) 96320 × 2
2 cathodes in the two cells will be
(c) 96320 (d) 6.6 × 1019. (a) in the ratio of the densities of Cu and Ag
123. According to Faraday’s laws of electrolysis, the (b) in the ratio of the atomic weights of Cu and Ag
amount of decomposition is proportional to (c) in the ratio of half the atomic weight of Cu to the atomic
(a) 1/time for which current passes weight of Ag
(b) electrochemical equivalent of the substance (d) in the ratio of the half atomic weight of Cu to half the
atomic weight of Ag. [Haryana PMT 2002]
(c) 1/current
(d) 1/electrochemical equivalent.CAPACI[MP PMT 1993]
130. If 1.5 A current liberates x kg of ions, the ions liber-
ated by 6.0 A of current will be
124. The amount of ions liberated by 96500 C of charge
(a) x/4 kg (b) x/2 kg
passed through the electrolyte is called
(c) 2x kg (d) 4x kg.
(a) electrochemical equivalent
(b) chemical equivalent [CMC LDH 1994]
(c) gram equivalent (d) none of the above. 131. In electrolysis, if the duration of the passage of cur-
[Pb. PMT 2001] rent is doubled, the mass liberated is
(a) doubled (b) halved
125. E.C.E.’s of copper were determined by taking copper
(c) increased four times (d) remains the same.
sulphate and copper chloride as electrolytes. If Z1
and Z2 are the E.C.E.’s determined in the two cases, [CMC LDH 2001]
then 132. To deposit one litre of hydrogen at 22.4 atmosphere
(a) Z1 = Z2 (b) Z1 > Z2 from acidulated water, the quantity of electricity that
(c) Z1 < Z2 (d) Z1 = 1/Z2. must pass through is
[Himachal PMT 2000] (a) 1 coulomb (b) 22.4 coulomb

126. What determines the emf between the two metals (c) 96500 coulomb (d) 193000 coulomb.
placed in an electrolyte ? [All India PM/PD 1999]
(a) Relative position of metals in the electrochemical series 133. The electrochemical equivalent of a material depends
(b) Distance between them on
(c) Strength of electrolyte (a) the nature of the material

(d) Nature of electrolyte. [Pb. PMT 2000] (b) the current through the electrolyte containing the mate-
rial
127. How much electric charge should pass through acidu-
(c) the amount of charge passed through the electrolyte
lated water to release 22.4 litre of hydrogen at NTP ?
(d) the amount of this material present in the electrolyte.
(a) 6.02 × 1023 C (b) 193 k C
[Bharati Vidyapeeth 1998]
(c) 22.4 C (d) 1 C. [MANIPAL 1994]
134. Same current passed through different electrolytes
128. Consider the following two statements :
for given time liberates ions in proportional to their
(A) Free-electron density is different in different met- (a) electrochemical equivalent
als. (b) chemical equivalent
(B) Free-electron density in a metal depends on tem- (c) atomic masses
perature. (d) atomic numbers. [BHU 1996]
134 COMPREHENSIVE OBJECTIVE PHYSICS

135. Given that chemical equivalent of a substance is E. 144. In the Peltier effect there is conversion of
To obtain its electrochemical equivalent we should (a) electrical into heat energy at the junction
multiply E with the ECE of (b) electrical into heat energy all along the length of the
(a) Carbon (b) Oxygen thermocouple wires
(c) Hydrogen (d) None of the above. (c) heat into electrical energy at the junctions
[Haryana PMT 1999] (d) None of the above cases. [MGIMS 2001]
136. To deposit one kg equivalent of an element at an elec- 145. Neutral temperature of a thermocouple is
trode, one needs a charge equal to
(a) constant
(a) one C (b) one absolute C
(b) increases with increase in the temperature of cold junction
(c) one stat C (d) 96500000 C.
(c) decreases with increase in the temperature of cold junc-
[BHU 1998]
tion
137. What carries current in an electrolyte ? (d) decreases with decrease in the temperature of cold junc-
(a) Electrons only (b) – ve ions only tion [JIPMER 2002]
(c) + ve ions only (d) Both + ve and – ve ions.
146. If Tc ,Tn and Ti denote the temperatures of cold junc-
[Haryana PMT 2000] tion, neutral temperature and inversion temperature
138. Mass of the ions liberated during electrolysis by 1 A of a thermocouple respectively, then
current in one second is called (a) Tc + Ti = Tn (b) Ti – Tc = 2Tn
(a) equivalent weight (b) chemical equivalent
(c) Tc + Ti = 2Tn (d) Tc – Ti = 2Tn.
(c) electrochemical equivalent
[MP PET 2001]
(d) None. [MANIPAL 1998]
147. In copper-iron thermocouple, the direction of ther-
139. An electrolysis experiment is stopped and the bat-
tery terminals are reversed moelectric current is from
(a) The electrolysis will stop. (a) copper to iron at cold junction
(b) The rate of liberation of material at the electrode will in- (b) iron to copper at cold junction
crease. (c) iron to copper at both junctions
(c) The rate of liberation of material will remains the same. (d) copper to iron at both junctions. [JIPMER 2002]
(d) Heat will be produced at a greater rate.
148. Peltier coefficient for the junction of a pair of metals
[AIIMS 2000] is proportional to
140. The mass of a substance liberated when a charge ‘q’ (a) absolute temperature of the junction
flows through an electrolyte is proportional to
(b) square of absolute temperature of the junction
(a) q (b) 1/q
1
(c) q2 (d) 1/q2. [EAMCET 1996] (c)
temperature of the junction
141. The thermoelectric power at the neutral temperature
1
is (d) .
square of absolute temperature of the junction
(a) – ve (b) zero
[MP PMT 1993]
(c) + ve (d) infinity.
149. In Peltier effect, heat evolved or absorbed at the junc-
142. What is the nature of graph between temperature and tion is proportional to
thermo emf ?
(a) current (b) current
(a) Hyperbola (b) Straight line
(c) Parabola (c) (current)2 (d) (current)0.

(d) Some other curve. [Himachal PMT 1995] [MANIPAL 2002]


143. The heat developed in a system is proportional to the 150. Two ends of a conductor are at different tempera-
current through it. tures. The electromotive force generated between two
(a) It cannot be Thomson heat. (b) It cannot be Peltier heat.
ends is
(a) Seebeck electromotive force (emf)
(c) It cannot be Joule heat.
(d) It can be any of the three heats mentioned above. (b) Peltier electromotive force (emf)
(c) Thomson electromotive force (emf)
[RPMT 1999]
(d) None of the above. [MP PMT 2001]
CURRENT ELECTRICITY 135

151. Inversion temperature for a thermocouple is the tem- 154. The negative Zn pole of a Daniell cell, sending a con-
perature at which thermo-emf stant current through a circuit, decreases in mass
(a) increases (b) remains unchanged by 0.13 g in 30 minute. If the electrochemical equiva-
(c) reverses in sign (d) changes erratically. lents of Zn and Cu are 32.5 and 31.5 respectively, the
increase in the mass of the positive Cu pole in this
[BHU 2001]
time is
152. The neutral temperature for a thermocouple is 300°C.
(a) 0.242 g (b) 0.180 g
If the temperature of cold junction is 30°C, the inver-
(c) 0.141 g (d) 0.126 g [AIEEE 2003]
sion temperature is :
(a) 600°C (b) 570°C 155. The electrochemical equivalent of a metal is 3.3 × 10–7
(c) 630°C (d) 550°C
kg per coulomb. The mass of the metal liberated at
the cathode when a 3 A current is passed for 2 sec-
(e) 500°C [Kerala PMT 2003]
ond will be
153. At the temperature of inversion, the emf in a ther- (a) 6.6 × 10–7 kg (b) 9.9 × 10–7 kg
mocouple is :
(c) 19.8 × 10–7 kg (d) 1.1 × 10–7 kg
(a) half its maximum value (b) maximum
[AIEEE 2004]
(c) minimum (d) reaches infinity
(e) zero [Kerala PMT 2003]

Answers (Set I)
1. (a) 2. (a) 3. (d) 4. (d) 5. (a) 6. (c) 7. (d) 8. (c)
9. (b) 10. (c) 11. (b) 12. (b) 13. (d) 14. (a) 15. (b) 16. (e)
17. (d) 18. (c) 19. (b) 20. (a) 21. (b) 22. (c) 23. (d) 24. (d)
25. (d) 26. (a) 27. (b) 28. (b) 29. (a) 30. (b) 31. (d) 32. (c)
33. (b) 34. (c) 35. (d) 36. (c) 37. (c) 38. (b) 39. (c) 40. (c)
41. (c) 42. (a) 43. (a) 44. (c) 45. (a) 46. (a) 47. (c) 48. (c)
49. (a) 50. (b) 51. (d) 52. (a) 53. (d) 54. (b) 55. (a) 56. (b)
57. (a) 58. (c) 59. (a) 60. (e) 61. (c) 62. (a) 63. (a) 64. (d)
65. (a) 66. (c) 67. (c) 68. (d) 69. (d) 70. (c) 71. (a) 72. (b)
73. (a) 74. (b) 75. (c) 76. (b) 77. (a) 78. (b) 79. (a) 80. (b)
81. (c) 82. (c) 83. (d) 84. (b) 85. (a) 86. (a) 87. (a) 88. (c)
89. (c) 90. (a) 91. (c) 92. (a) 93. (a) 94. (b) 95. (a) 96. (b)
97. (d) 98. (d) 99. (a) 100. (b) 101. (b) 102. (d) 103. (c) 104. (d)
105. (b) 106. (a) 107. (c) 108. (c) 109. (a) 110. (b) 111. (b) 112. (c)
113. (b) 114. (a) 115. (c) 116. (b) 117. (d) 118. (b) 119. (c) 120. (b)
121. (c) 122. (c) 123. (b) 124. (c) 125. (a) 126. (a) 127. (b) 128. (b)
129. (c) 130. (d) 131. (a) 132. (d) 133. (a) 134. (b) 135. (c) 136. (d)
137. (d) 138. (c) 139. (c) 140. (a) 141. (b) 142. (c) 143. (c) 144. (a)
145. (a) 146. (c) 147. (b) 148. (a) 149. (a) 150. (c) 151. (c) 152. (b)
153. (e) 154. (d) 155. (c)
136 COMPREHENSIVE OBJECTIVE PHYSICS

Solutions (Set I)

1. Volume is conserved. 0.1% longer means 0.1% reduc- 16. Typically, below 12 V, the charge carriers are elec-
tion in cross-sectional area. trons emitted by the cathode which has a heater ele-
Percentage increase in resistance = 0.1 + 0.1 = 0.2. ment. The neon lamp will not produce light but can
act as a diode.
l1 − l2 240 − 120
2. r = R = × 2 Ω = 2 Ω. Above 12 V, the noble gas (argon, neon, etc.) breaks
l2 120
down into positive ions and electrons. Then there are
3. Number of free electrons per second two charge carriers. To produce visible light, ionisa-
10−6 tion is required.
= = 6.25 × 1012.
1.6 × 10 −19 17. Since no current flows through the circuit, all the
voltage drops across the bulb that has a broken fila-
1
6. Rp = Ω = 0.1 Ω. ment. i.e., X = 240 V, Y = 0 V.
10
20. Parallel combination reduces resistance.
R
7. Resistance of each part is . 21. Apply Kirchhoff ’s current law.
n
R/n R 5J
Rp = = 2. 23. E= = 0.5 JC–1 = 0.5 V.
n n 10 C
8. It is a case of weak current. 24. Ionisation of electrolyte.
1 1 1 I
9. = + = 1 or Rp = 1 Ω 25. Drift speed, vd = ; Since A is not constant there-
Rp 2 2 neA
fore vd is not constant.
l/2 1 1
Aliter. R′ = ρ or R′ = R = × 4 Ω = 1 Ω.
2a 4 4 E−V 6−4
26. r= = Ω = 1 Ω.
10. Alloys have high resistivity. So, they are unsuitable I 2
for making connecting wires. 27. Refer to synopsis.
11. (i) all three in series (3R)
28. 12 = 0 – V or V = – 12 volt.
FG R IJ
(ii) all three in parallel
H 3K 29. Balanced Wheatstone bridge.

FG 3 RIJ 30.
1 10
= or R =
r
(iii) two in parallel, in series with third
H2 K R r 10
(iv) two in series and third parallel to combination 31. Potentiometer method is a ‘null’ method.
FG IJ
2 33. It is a case of parallel combination of three resistances.
H K
3
R .
34. By Kirchhoff’s second law, the total current entering
12. The situation resembles the flow of water in a pipe. a node must be equal to the total current leaving the
Increased pressure makes water flow faster but does node. The current equations associated with the given
not change water density. figures are summarised as follows.
13. Resistance of a metal decreases with decrease of tem- Figure Current Equation
perature. Resistance of a semiconductor increases with
decrease of temperature. α is + ve for metals and – ve A I1 + I3 + I4 = I2
for semiconductors. B I4 = I1 + I2 + I3
14. The resistivity of alloys is greater than that of met- C I1 + I2 = I3 + I4
als.
D I1 + I2 + I3 + I4 = 0
15. I × 10 = 5
5 E I2 + I4 = I1 + I3
I= A
10
35. Balanced Wheatstone bridge.
I = 0.5 A.
Again, parallel combination of 6r and 6r.
CURRENT ELECTRICITY 137

E – 100
36. E = I(R + r) or r = R 54. = 1 or n = 10.
I n2
55. Resistivity is a property of the material. It does not
50 100
= − 10 = − 10 depend upon the geometrical dimensions.
4.5 9
56. When diameter is doubled, cross-sectional area is in-
10 creased by a factor of 4. Since both l and a are in-
= Ω = 1.1 Ω
9 creased by a factor of 4 therefore there is no effect on
2l the resistance.
37. R′ = ρ = R.
2a 57. Temperature of a thermistor increases as the poten-
39. Voltage across 8.5 Ω tial difference across it increases. As temperature
= 8.5 × 10 V = 85 V rises, resistance of the thermistor decreases and hence
Voltage across bulb graph A best illustrates its I-V relationship.
= 42 V 58. The resistance of the square X is given by
Total voltage L ρ
RX = ρ =
= (85 + 42) V = 127 V. Lt t
1.5 1.5 where ρ is the resistivity of the metal.
40. 3= or r = Ω
r 3 Similarly, for RY ,
1 2L ρ
or r = Ω = 0.5 Ω. RY = ρ = same as before.
2 (2L)t t
20
41. R = Ω = 10 Ω. RX
2 Hence, = 1.
44. Use colour code. RY
45. It is a parallel combination of 4 Ω and 2 Ω.
l1 − l2 560 − 412
4×2 8 4 59. r = R = × 10 Ω
∴ Rp = Ω= Ω= Ω l2 412
4+2 6 3
= 3.59 Ω ≈ 3.6 Ω
46. Current flowing away from junction a = 4A
Current flowing towards junction b = 3A X 3 3
60. = or X = × 5 Ω = 7.5 Ω
Now, I + 1.3 = 3 5 2 2
or I = (3 – 1.3)A = 1.7 A 61. (VA – VB ) – 6 – 3 × 2 + 9 – 3 × 0.7 = 0
47. When resistances are connected in parallel, the po- or VA – VB = 5.1
tential difference across each resistance is the same or VB – VA = – 5.1 volt.
as across the combination. 2l R
48. Note that null methods are more accurate than de- 62. R′ = ρ =
4a 2
flection methods. There will be no effect on specific resistance.
49. What is the equation which represents the balance NE − 2E (N − 2)E
condition ? Does that equation contain any of the given 63. I= =
Nr Nr
factors ? For 2 remember that it is the ratio of the
resistances of the two sections of the slide wire that is (N − 2)E
V = E – Ir = E − r
important. Nr
50. When the switch S2 is closed, the whole of current LM
= E 1−
N−2 OP =
2E
shall flow through the connecting wire only which is
supposed to have zero resistance.
N N Q N
r
51. In parallel combination, the combined resistance is 64. R= or r = nR
less than the least resistance. n
Rs = nr or Rs = n (nR) = n2R
V 60
52. R= = Ω = 4 Ω. 65. Thermistors are devices having high temperature
I 15 coefficient of resistivity. The negative temperature
1 1 coefficient types are made from a mixture of oxides of
53. R′ = R= R.
4 2 16 iron, nickel and cobalt with small amounts of other
substances.
138 COMPREHENSIVE OBJECTIVE PHYSICS

66. The given arrangement is a parallel combination of 77. Since water is to be boiled in lesser time therefore
2R and 2R. This gives an equivalent resistance of R. more heat is required to be produced.
n(n + 1) n(n + 1) V2
67. ΣEn = 1.5r ; Σrn = r We known that P =
2 2 R
ΣEn 1
I= = 1.5 A In the given problem, V is constant. ∴ P ∝
Σrn R
For more P, R should be less. So, l should be less.
V
68. Potential gradient =
l 78. Ampere-hour efficiency = ampere-hour on discharge .
ampere-hour on charge
IR I l Iρ
= = ρ = P
l l a a 80. P = VI, I = 500 W, 100 V 100 V
V
0.1 × 10−7 500 W
= −6
Vm −1 = 10 –2 Vm–1 or I= =5A 5A R
10 100 V
Now, 5 R = 100 200 V
l l×A ρV 1
69. R = ρ =ρ = 2 or R ∝ or R = 20 Ω
A A×A A D4 Fig. 39
81. Pt = mc θ
If D is halved, R is increased by a factor of 16. So, 1000 × t = 10 × 4200 × 10
new resistance is 160 Ω.
10 × 4200 × 10
70. At low temperatures, the resistivity is very large. As t= s or t = 420 s
temperature increases, the resistivity decreases. 1000
71. Using the principle of potentiometer, P0
82. Pt =
V∝l 1 + αt

V l l 30 P0
∴ = or V = E= E Pt ′ =
E L L 100 1 + αt′
72. Fall of potential = 2.2 V – 1.8 V = 0.4 V P0
1000 =
Ir = 0.4 1 + 1000 α
1.8 P0
× r = 0.4 Pt′ =
5 1 + 400 α
2 20 10 Pt ′ 1 + 1000 α
or r= Ω = Ω = Ω =
1.8 18 9
1000 1 + 400 α
E−V Pt′ = 1000(1 + 1000 α) (1 + 400 α)–1
Aliter r= R
V Pt′ = 1000(1 + 1000 α) (1 – 400 α)
2.2 − 1.8 10
= ×5Ω= Ω Pt′ = 1000(1 + 600 α)
1.8 9
74. Current in R1 and R2 is given by = 1000(1 + 600 × 1.4 × 10–4 )
= 1000 × 1.0840 W = 1.084 × 103 W.
E
I= 4
R1 + R 2
1
Potential difference across the resistor of 83. Combined resistance = 3 Ω = Ω
8 2
ER 2 3
resistance R2 = IR2 = .
R1 + R 2
V2 3 × 3 × 60
75. Resistance of 40 W bulb is maximum, current is same Energy = t = J = 60 × 3 × 3 × 2 J
in series connection. R /
12
84. When current is doubled, heating effect becomes four
V2 V2 1
76. P= or R = or R ∝ times. Now, Q ∝ ∆T
R P P
So, ∆T becomes four times.
r2 200
∴ = = 2 or r2 = 2r1 85. P ∝ I2. % decrease in power = 2 × 0.5% i.e., 1%.
r1 100
CURRENT ELECTRICITY 139

V2 l V2
86. P = . If P is more, R is less. R = ρ . For less R, ‘a’ 105. P=
R a l
is more. So, 100 W bulb has thicker element. ρ
a
87. Fine wire has more resistance. So, heat produced is
1
more, in series connection. P∝
l
88. Alloys are used for making standard resistance coils
because of high resistivity and low temperature coef- 1
P′ ∝
ficient of resistance. 10
l− l
nV 2 V 2P P 100
89. Total resistance = , Power = =
P nv2 n P′ l
=
V 2
1 P2 R 1 P l− 1 l
91. P= or P ∝ ∴ = 10
R R P1 R 2
92. Due to increase in resistance, the current decreases. P ′ 10
=
Again, P = I2R. Note that I2 is the dominant term. P 9
210 × 5 × 60 FG P′ − 1IJ × 100 = FG 10 − 1IJ × 100
93. Heat in calories =
4.2
= 15000
HP K H9 K
94. Resistance decreases. Current increases. Current is
P′ − P 100
the dominant term. × 100 = ≈ 11
P 9
V2 106. 2 kW × 30 h = 60 kWh = 60 units.
97. P = . If V is halved, P is reduced by a factor of 4.
R 107. R = R1 + R2
1000 V2 V2 V 2 1 1 1
So, new power is W i.e. 250 W. = + or = +
4 P P1 P2 P P1 P2
V 2 30 × 30 P1P2
98. R = = Ω = 10 Ω or P= .
P 90 P1 + P2
P 90 108. For an electrical potential dif- I
I= = A =3A ference of V between two points
V 30
To work with 120 V line, in a wire carrying current I, V
the power dissipated P is given Fig. 40
120 by
3= or R = 30 Ω
10 + R
P
99. Pt = mSθ Power, P = IV or V =
I
or 836 t = 1 × 4200 × 30 i.e., the ratio of power dissipated between the points
4200 × 30 to the current = V = electrical potential difference.
or t= s ≈ 150 s
836 109. The condition for delivering maximum power is that
2 2 2 2 the external resistance is equal to internal resistance.
V V a V πr
100. P= or P = =
R ρl ρl 110. The corrected passage should read. ‘In POOR conduc-
101. P = 200 × 3.75 W = 750 W ≈ 1 hp tors THERMAL energy is transferred by lattice VI-
P BRATIONS. These travel through the specimen as
102. P = VI ; I = WAVES which are SCATTERED by the lattice as they
V
progress through it’.
250 × 1000
I= A = 25 A 111. Q = 10 coulomb
10000
103. Power lost = 25 × 25 × 10 W = 6250 W = 6.25 kW n × 2 × 1.6 × 10–19 = 10
1 10
104. U= CV2 or n = × 1019
2 3.2
1 = 3.125 × 1019
= × 100 × 10–6 × 200 × 200 J = 2 J
2
140 COMPREHENSIVE OBJECTIVE PHYSICS

112. Using Faraday’s second law of electrolysis, 108


mAg 108
m 32.5
= or m ≈
x 118. = 1 =
mO2 16 8
x 108 3
2
113. Q1 = 4 × 120 C = 480 C 108
or mAg = × 0.8 = 10.8 g
Q2 = 6 × 40 C = 240 C 8
Clearly, charge is halved. So, mass is halved. 1
120. Q= (10 + 30)100 × 10–3 coulomb = 2 coulomb
114. Note that the valencies of Al and Ag are 3 and 1 re- 2
spectively. m = ZQ
115. 4H2O + 4e– → 4OH– + 2H2 m m
Z= =
2H2O → 4H+ + 4e– + O2 Q 2
130. Since current is increased by a factor of 4 therefore
4H+ + 4OH– → 4H2O
charge is also increased by a factor of 4.
Adding, 2H2O → 2H2 + O2 131. Charge is doubled. So, the mass of ions liberated is
116. If n be the number of copper ions, then the number of doubled.
sulphate ions will also be n. 132. See solution of Q. 117.
∴ 2n × 1.6 × 10–19 = 4.8 × 105 152. θi – θn = θn – θc
θi = 2θn – θc = 2 × 300 – 30 = 570°C
4.8 × 105
or n = 154. Using Faraday’s laws of electrolysis
2 × 1.6 × 10−19
mZn ZZn
or n = 1.5 × 1024. =
mCu ZCu
117. 11.2 litre of H2 is liberated, by 96500 coulomb, at one
atmospheric pressure. At 11.2 atmosphere, 1 litre of 0.13 32.5 31.5 × 0.13
= or mCu = g = 0.126 g
H2 will be liberated by 96500 C. For 2 litre, charge mCu 31.5 32.5
will be 2 × 96500 C i.e., 193 kC. 155. m = ZIt
m = 3.3 × 10–7 × 3 × 2 = 19.8 × 10–7 kg

KNOWLEDGE PLUS
l Five equal resistances, each of resistance R, are connected as shown in Fig. 41. A C
battery of V volt is connected between A and B. The current flowing in AFCEB will be
V V R
(a) (b)
R 2R
R F R
2V 3V
(c) (d) [All India PM/PD 2004]
R R A
R B
Solution. Using the principle of Wheatstone bridge, the given network reduces to a
parallel combination of 2R and 2R. So, (b) is the right choice. D R E
Fig. 41
CURRENT ELECTRICITY 141

MCQs
based on
Important Basic Concepts
SET II

+
Application of Formulae
+
Brain Teasers
Average time allowed per question is 50 seconds.

as shown in the Fig. 42. When the galvanometer


CURRENT ELECTRICITY shows no deflection, the values of the resistances are
P and Q.
156. A conductor of resistance 12.8 Ω is cut into a number
of parts. All these parts are connected in parallel.
1
The resistance of the combination so obtained is Ω.
5
The number of parts is
(a) 4 (b) 6
(c) 8 (d) 12.
157. The temperature coefficient of resistance of a wire is
0.00125° C. At 300 K, its resistance is 1 Ω. The resist-
ance of the wire will be 2 Ω at
(a) 1154 K (b) 1100 K
(c) 1400 K (d) 1127 K. Fig. 42
[Karnataka CET 2003]
What is the value of the ratio E2/E1 ?
158. A resistor is in the shape of a truncated right circu-
lar cone. The end radii are r1 and r2 (r2 > r1) and the P P
(a) (b)
altitude is l. If the taper is small, then the resistance Q P+Q
between the plane faces of this resistor is Q P+Q
(c) (d)
Kl Kl P+Q P
(a) (b)
πr12 πr22
P+Q
Kl (e) .
K πr1r2 Q
(c) (d) .
πr1r 2 l
161. The resistances in Wheat-
Here, K represents the resistivity of the material of
stone’s Bridge circuit shown
the resistor.
in the Fig. 43 have different
159. A cell of negligible resistance of emf 2 volt is con- values. The current through
nected to a series combination of 2 Ω, 3 Ω and 5 Ω . the galvanometer is zero. If
The potential difference in volt between the termi-
all thermal effects are negli-
nals of 3 ohm resistance will be
gible, the current through
2 the galvanometer will not be
(a) 0.6 volt (b) volt
3 zero, when
Fig. 43
(c) 3 volt (d) 6 volt. [WB JEE 1996] (a) the battery emf is doubled
160. Two cells of emfs E1 and E2 and of negligible internal (b) the battery and galvanometer are interchanged
resistances are connected with two variable resistors
142 COMPREHENSIVE OBJECTIVE PHYSICS

(c) all resistances in the circuit are doubled 167. A piece of copper is to be shaped into a conductor of
(d) resistances R1 and R2 are interchanged. minimum resistance. The possible values of length
[WB JEE 2001] and diameter are
(a) l, d (b) 2l, d
162. In the circuit shown in Fig. 44, the total resistance
between points X and Y is R0 . The value of resist- l d
(c) , 2d (d) 2l, .
ance R is 2 2
168. The effective resistance between A and B in the net-
work of Fig. 46 is

Fig. 44

R0
(a) R0 (b)
3
R Fig. 46
(c) 0 (d) 3 R0.
2
[WB JEE 2002] 4 3
(a) Ω (b) Ω
163. How many 2.1 V cells with internal resistance 0.20 Ω 3 2
each are to be connected in series to obtain a battery 8
(c) 7 Ω (d) Ω .
so that a conductor of resistance 6.0 Ω would carry a 7
current of 1.5 A ? 169. In Fig. 47, two cells
(a) 4 (b) 5 have equal emf E but
(c) 6 (d) 7. [Pb. PMT 1993] internal resistances
164. A non-conducting ring of radius r has charge q dis- are r 1 and r 2 . If the
tributed uniformly over it. If it rotates with an angu- reading of voltmeter
lar velocity ω, the equivalent current is is zero, then relation
between R, r 1 and r2
qω 2π
(a) (b) is
2π qω
(a) R = r1 – r2
qω Fig. 47
(c) (d) q r ω . (b) R = r1 + r2
2π r
(c) 2 r1 – r2 (d) r1 r2.
[All India PM/PD 1993]
170. B is a 1.5 V, 0.30 A, 5.0 Ω torch bulb working prop-
165. A resistance whose printed value is 2 Ω is observed erly in the circuit shown. The combined resistance of
to have an actual resistance of 2.1 Ω. What length of R and B, between the points X and Y, is
wire of resistance 21 Ω m– 1 must be connected in
parallel with the coil in order that the combined re-
sistance shall be exactly 2 Ω ?
(a) 1 m (b) 0.5 m
(c) 2 m (d) 4 m.
[CMC Vellore 2002]
166. What is the resistance of
the hexagonal circuit
shown in Fig. 45 ?
(a) 1 Ω
Fig. 48
(b) 0.5 Ω
(c) 0.25 Ω (a) 0.45 Ω (b) 1.0 Ω
(d) 0.125 Ω. (c) 3.0 Ω (d) 5.0 Ω. [AIIMS 1994]
171. Four resistors 40 Ω, 60 Ω, 90 Ω and 110 Ω are con-
nected as shown to a battery of emf of 4 V and negli-
Fig. 45
CURRENT ELECTRICITY 143
gible internal resistance. Now, the value of VB–VD across the terminals of the battery when a load re-
is : sistance R ( = r) is connected to its terminals ?
(a) 2E volt (b) E volt
E E
(c) volt (d) volt.
2 4
177. In the network shown in
Fig. 51, each resistance
is equal to 2 Ω . The re-
sistance between the
points A and B is
(a) 1 Ω
(b) 4 Ω
(c) 3 Ω
Fig. 49
(d) 2 Ω. [BHU 2001]
(a) – 0.2 V (b) 0.2 V 178. A copper wire of resist- Fig. 51
(c) 1 V (d) 0.1 V. [BHU 1997] ance R is cut into ten parts of equal length. Two pieces
172. The equivalent resistance between A and B in the each are joined in series and then five such combina-
tions are joined in parallel. The new combination will
network in Fig. 50 is
have a resistance
R
(a) R (b)
4
R R
(c) (d) . [MP PET 1998]
5 25
179. A student connects four cells in series but one cell
has its terminals reversed. If the internal resistance
1
of each cell is Ω , the load resistance is 1 Ω and
4
each cell has an emf of 1.5 volt, then the current
flowing in the circuit is given by
(a) 1.5 A (b) 2 A
Fig. 50
(c) 2.5 A (d) 9 A. [WBJEE 1999]
4 3
(a) Ω (b) Ω 180. In the circuit, the reading
3 2 of the ammeter is (assume
(c) 3 Ω (d) 2 Ω. internal resistance of the
173. Six 1.1 V cells with internal resistance 3.0 Ω each battery be zero)
are connected in three parallel branches of two cells 40
each. If the resistance of external circuit is 2.0 Ω, (a) A
29
then the current is Fig. 52
10
(b) A
(a) 0.33 A (b) 0.44 A 9
(c) 0.55 A (d) 0.66 A. 5
(c) A (d) 2 A. [AMU 1996]
[Haryana PMT 1995] 3
181. In the previous question, if the internal resistance of
174. A cell of emf 4 V, when short-circuited, gives a cur-
the battery is 1 ohm, then what is the reading of
rent of 8 A. The internal resistance of the cell is
ammeter ?
(a) 0.5 Ω (b) 1 Ω
5 40
(c) 2 Ω (d) 4 Ω. (a) A (b) A
3 29
175. An electron in the potentiometer wire experiences a 10
force of 3.2 × 10 –19 N. The length of the potentiometer (c) A (d) 1 A. [AFMC 1999]
9
wire is 4 m. The emf of battery across the wire is 182. A letter ‘A’ is constructed of a uniform wire with re-
(a) 1.6 V (b) 3.2 V sistance 1.0 Ω per cm. The sides of the letter are
(c) 4.8 V (d) 8 V. 20 cm and the cross-piece in the middle is 10 cm long.
176. A battery of internal resistance r having no load re- The apex angle is 60°. The resistance between the
sistance has an emf E volt. What is the observed emf ends of the legs is
144 COMPREHENSIVE OBJECTIVE PHYSICS

(a) 50.0 Ω (b) 26.7 Ω


R 1 kΩ R 2 kΩ R3 kΩ
(c) 2.72 Ω (d) 34 Ω.
[National Standard Exam. in Physics 2005] (a) 1 1 2
183. A Daniel cell is balanced on 125 cm length of a (b) 2 1 2
potentiometer wire. Now the cell is short-circuited (c) 3 2 2
by a resistance of 2 ohm and the balance is obtained (d) 3 2 3
at 100 cm. The internal resistance of the Daniel cell
is 187. How will the read-
A
(a) 0.5 Ω (b) 1.5 Ω ing in the ammeter
A of the figure 55 be
(c) 1.25 Ω (d) 4/5 Ω.
affected if another Mains P Q
[All India PM/PD 2002] identical bulb Q is
184. In the circuit shown in Fig. 53, the magnitude and connected in paral-
direction of the flow of current respectively would be lel to P as shown. Fig. 55
The voltage in the
mains is maintained at a constant value.
(a) The reading will be reduced to one-half.
(b) The reading will not be affected.
(c) The reading will be double of previous one.
(d) The reading will be increased four fold.
188. In the Fig. 56, there is no deflection in the galvanom-
Fig. 53 eter. Then R is equal to
2W RW
7 7
(a) A from a to b via e (b) A from b to a via e
3 3
G
(c) 1 A from b to a via e (d) 1 A from a to b via e.
185. A cell of emf E and internal resistance r is connected
in series with an external resistance nr. The ratio of 10 W 30 W
the terminal potential difference to emf is
1 1
(a) (b)
n n+1

n n+1 Fig. 56
(c) (d) . (a) 2 Ω (b) 30 Ω
n+1 n
2
186. A potential divider is used to give outputs of 2 V and (c) 6 Ω (d) Ω. [DPMT 2001]
3
3 V from a 5 V source, as shown in Fig. 54. 189. When a piece of aluminium wire of finite length is
drawn through a series of “dies” to reduce its diam-
eter to half its original value, its resistance will be-
come
(a) two times (b) four times
(c) eight times (d) sixteen times.
[EAMCET 2001]
190. Equivalent resistance
between A and B will 3W 3W
be a b
(a) 2 ohm
(b) 18 ohm 3W 3W
Fig. 54 3W 3W
(c) 6 ohm
Which combination of resistances, R1 , R2 , R3 gives (d) 3.6 ohm.
the correct voltages ? A 3W 3W B
Fig. 57
CURRENT ELECTRICITY 145
191. The resistance of the series combination of two 198. An electric current is passed through a circuit
resistances is S. When they are joined in parallel the containing two wires of the same material, connected
total resistance is P. If S = nP, then the minimum in parallel. If the lengths and radii of the wires are in
possible value of n is 4 2
(a) 2 (b) 3 the ratio of and , then the ratio of the currents
3 3
(c) 4 [AIEEE 2004]
(d) 1 passing through the wires will be
192. Two wires of the same dimensions but resistivities (a) 8/9 (b) 1/3
ρ1 and ρ2 are connected in series. The equivalent re-
(c) 3 (d) 2 [AIEEE 2004]
sistivity of the combination is :
199. Thirteen resistances, R R
ρ + ρ2 each of resistance R ohm,
(a) 1 (b) ρ1 + ρ2
2 are connected in the cir-
R R
(c) 2(ρ1 + ρ2) (d) ρ1 ρ2 [KCET 2003] cuit as shown in Fig. 60. R R
The effective resistance
193. Two cells with the same emf E and different internal between A and B is
A R B
resistances r1 and r2 are connected in series to an R R
external resistance R. The value of R so that the po- 4R R R
(a) Ω (b) 2 R Ω
tential difference across the first cell be zero is 3

(a) r1r2 (b) r1 + r2 2R R R


(c) R Ω (d) Ω
3 Fig. 60
r +r
(c) r1 – r2 (d) 1 2 [EAMCET 2003] [KCET 2003]
2
200. A uniform wire of resistance
194. n cells, each of emf E and internal resistance r, send
the same current through an external resistance R 36 Ω is bent in the form of a A
whether the cells are connected in series or in paral- circle. The effective resistance
30°
lel. Then : between A and B is B
(a) R = nr (b) R = r (a) 2.75 Ω
(b) 3 Ω
(c) r = nR (d) R = nr
(c) 33 Ω
(e) r = nR [Kerala PMT 2003] Fig. 61
(d) 36 Ω .
195. A 3 volt battery with neg- I 201. In a metre bridge experiment null point is obtained
ligible internal resistance at 20 cm from one end of the wire when resistance X
is connected in a circuit is balanced against another resistance Y. If X < Y,
as shown in Fig. 58. The 3V 3 3 then where will be the new position of the null point
current, I, in the circuit from the same end, if one decides to balance a resist-
will be ance of 4 X against Y ?
1 (a) 40 cm (b) 80 cm
(a) A (b) 1 A 3
3 (c) 50 cm (d) 70 cm [AIEEE 2004]
Fig. 58
(c) 1.5 A (d) 2 A 202. Four resistors are connected as shown in Fig. 62.
[AIEEE 2003]
196. The length of a given cylindrical wire is increased by
100%. Due to the consequent decrease in diameter,
the change in the resistance of the wire will be
(a) 300% (b) 200%
(c) 100% (d) 50%. [AIEEE 2003]
197. For the post office box arrangement B C D
to determine the value of unknown
resistance, the unknown resistance
should be connected between A
(a) B and C
(b) C and D B1 C1
(c) A and D Fig. 59 Fig. 62
(d) B1 and C1 [IIT Screening 2004]
146 COMPREHENSIVE OBJECTIVE PHYSICS

Between which two points does the maximum resist- 20 W 4W


ance of the combination occur ?
(a) P and Q (b) Q and S
(c) R and S (d) S and P.
203. When a resistor of 4 Ω is connected across a cell, the 1.4 A 15 W
potential difference across the resistor is 8 V, but
this falls to 6 V when a second 4 Ω resistor is con-
nected in parallel with the first. The emf and inter-
nal resistance of the cell are respectively 50 W 10 W
(a) 6 V, 4 Ω (b) 12 V, 4 Ω
Fig. 64
(c) 12 V, 2 Ω (d) 6 V, 2 Ω. [MNR 1996]
(a) 1.4 A (b) 0.4 A
204. The circuit shown in Fig. 63 is used to compare the
(c) 1.0 A (d) 0.7 A.
emfs of two cells E1 and E2 (E1 > E2). The null point
is at C when the galvanometer is connected to E1. 209. In the circuit shown in Fig. 65, an ideal ammeter
When the galvanometer is connected to E2 , the null and an ideal voltmeter are used. When key is open,
point will be the voltmeter reads 1.53 V. When the key is closed,
the ammeter reads 1.0 A and the voltmeter reads
B 1.03 volt. The resistance R is

A C
B

E1 Fig. 65
G
(a) 0.5 Ω (b) 1.03 Ω
E2 (c) 1.53 Ω (d) 0.53 Ω. [Pb. PMT 1997]
Fig. 63 210. A number of 100 volt lamps connected in parallel are
(a) to the left of C (b) to the right of C
fed by a 130 V battery having an internal resistance
of 2.6 ohm. If the resistance of each lamp is 200 ohm
(c) at C itself (d) nowhere on AB.
and the resistance of conducting wires is 0.40 ohm,
[MP PMT 1995]
the number of lamps which the battery can light
205. A potential difference V is applied to a conductor of properly is
length l and radius r. When potential difference is
(a) 15 (b) 18
doubled, the drift velocity is
(a) halved (b) unchanged (c) 20 (d) 25. [JIPMER 2000]
(c) doubled (d) quadrupled. 211. G1 , G2 , G3 are the conductances of the three conduc-
206. In the previous question, when l is doubled without tors. When they are joined in series, their equivalent
changing the potential difference, then the drift ve- conductance will be
locity is 1 1 1
(a) G1 + G2 + G3 (b) + +
(a) halved (b) unchanged G1 G 2 G3
(c) doubled (d) quadrupled. G 1G 2G 3
(c) (d) None of these.
207. In Q. 205, when the radius is quadrupled without G 1 + G2 + G3
changing the potential difference, and the length of
the conductor, then the drift velocity is 212. The masses of three wires of copper are in the ratio
(a) halved (b) unchanged
of 1 : 3 : 5. and their lengths are in the ratio of
5 : 3 : 1. The ratio of their electrical resistances is
(c) doubled (d) quadrupled.
(a) 125 : 15 : 1 (b) 5 : 3 : 1
208. In the circuit shown in Fig. 64, the current through
4 Ω resistor is (c) 1 : 3 : 5 (d) 1 : 15 : 125.
CURRENT ELECTRICITY 147
213. What is the ammeter reading in the circuit of Fig. 66 ? (a) 20 ohm (b) 10 ohm
(c) 5 ohm (d) 15 ohm.
[All India PM/PD 2001]
10 V
219. The equivalent resistance of the following infinite
5W network of resistances is
A

5W

Fig. 66
(a) 0.25 A (b) 0.5 A
(c) 1.0 A (d) 2.0 A.
214. A voltmeter of range 0–100 mV is calibrated with the
help of a potentiometer having 4 m wire. It gives full Fig. 68
scale deflection, when connected across the two ends
of the wire. If it reads 35 mV when connected across (a) less than 4 Ω (b) 4 Ω
1.41 m of the wire, what is the error in the reading ? (c) more than 4 Ω but less than 12 Ω
(a) 1 mV (b) 0.5 mV (d) 12 Ω. [BHU 1994]
(c) 0.25 mV (d) No error. 220. Six equal resistances, each of 6 Ω,
215. A straight conductor of uniform cross-section car- are joined to form a network as
ries a current I. Let s be the specific charge on an shown in Fig. 69. The resistance be-
electron. The momentum of all the free electrons per tween any two corners is
unit length of the conductor due to their drift veloc- (a) 2 Ω (b) 3 Ω
ity only, is (c) 4 Ω (d) 5 Ω. Fig. 69
(a) I/s (b) I s 221. A cell of emf E volt with no internal resistance is
F II2
(c) G J
connected to a wire whose cross-section changes. The
H sK (d) I/s . [WB JEE 2002] wire has three sections of equal length. The middle
section has a radius a whereas the radius of the outer
216. How many times will the total resistance of ‘n’ iden-
two sections is 2a. The ratio of the potential differ-
tical resistors be increased if the parallel arrange-
ence across the section AB to the potential difference
ment is changed to series arrangement ?
across the section CA is
(a) n times (b) n2 times
E
(c) n times (d) n3 times. [AIIMS 2001]
217. Two non-ideal batteries are connected in parallel.
Consider the following statements :
(i) The equivalent emf is smaller than either of the two emfs.
(ii) The equivalent internal resistance is smaller than either C l A l B l
of the two internal resistances.
Fig. 70
(a) Both (i) and (ii) are correct
(b) (i) is correct but (ii) is wrong (a) 5 (b) 4
(c) (ii) is correct but (i) is wrong (c) 1/2 (d) 1/4. [EAMCET 1995]
(d) Both (i) and (ii) are wrong. [MP PMT 1999] 222. In the circuit shown, the value of
218. What resistor 14 V
should be con-
nected in parallel
with the 20 Ω resis-
tor in branch ADC 20 W B 10 W
in the circuit
shown so that the A C
potential difference
between B and D 20 W D 5 W
Fig. 71
may be zero ? Fig. 67
148 COMPREHENSIVE OBJECTIVE PHYSICS

(a) R = 15 Ω (b) R = 30 Ω V V
(c) E = 36 V (d) E = 180 V.
[DPMT 2002]
223. The current flowing through a coil of resistance 800
Ω is to be reduced by 80%. What value of shunt should
be connected across the coil ?
(a) 200 Ω (b) 80 Ω
(c) 8 Ω (d) 10 Ω. (c) I (d) I
[CMC Vellore 1999]
Fig. 73
224. The four arms of a Wheatstone Bridge have the fol-
lowing resistances : AB = 6 Ω, BC = 3 Ω, CD = 6 Ω, 227. In Fig. 74, the reading of the ammeter A, when the
DA = 12 Ω . The galvanometer is of 10 Ω resistance. internal resistance of the battery is zero, is
The potential difference of 2 V is maintained across
AC. The current through the galvanometer is

Fig. 74
20 20
(a) A (b) A
3 12
20 FG
20 20 IJ
(c)
4
A (d)
3H +
12
A.
K
228. In Q. 227, the reading of the ammeter A, when the
internal resistance of the battery is 2 ohm is
Fig. 72
(a) 5 A (b) 3 A
(a) 8.7 mA (b) 7.8 mA (c) 1 A (d) 4 A.
(c) 8.7 A (d) 0 A. [EAMCET 1990] 229. Fig. 75 represents a load consisting of three identical
225. A battery has an emf of 12 V and an internal resistance resistances connected to an electric energy source of
of 0.5 Ω. When an external 5.5 Ω resistor is connected emf 12 V and internal resistance 0.6 Ω . The amme-
across the terminals of the battery, the potential ter reads 2 A. The magnitude of each resistance is
difference between the terminals will be
(a) 12 V (b) 11.5 V
(c) 11 V (d) 2 V. [Pb. PMT 1997]
226. The potential difference across the terminals of a cell
varies with the current drawn from the cell accord-
ing to the graph

V V

Fig. 75
(a) 3.6 Ω (b) 7.2 Ω
(c) 16.2 Ω (d) 10.8 Ω.
(a) I (b) I [Himachal PMT 2001]
CURRENT ELECTRICITY 149
230. Refer to the network of Fig. 76. Potential difference 237. Three similar light A
VA – VB is bulbs are connected to
a constant-voltage dc +
A
supply as shown in the –
+
diagram. Each bulb op-
2W –
3W erates at normal
brightness and the am-
meter (of negligible re-
Fig. 77
2A D C sistance) registers a
steady current.
3W 2W
The filament of one of the bulbs breaks. What hap-
pens to the ammeter reading and to the brightness
B of the remaining bulbs ?
Fig. 76 ammeter reading bulb brightness
(a) increases increases
(a) + 2 V (b) + 1V
(c) – 1 V (d) + 4 V (b) increases unchanged

231. A wire has resistance 12 Ω. It is bent in the form of a (c) unchanged unchanged
circle. The effective resistance between the two points (d) decreases unchanged
on any diameter of the circle is (e) decreases decreases.
(a) 3 Ω (b) 6 Ω
238. The current in the circuit shown in Fig. 78 is
(c) 12 Ω (d) 24 Ω.
232. A metal block has length 2 m, width 1 m and thick-
ness 0.5 m. If the resistance across the length is R,
then the resistance across the thickness is
(a) R/16 (b) R/8
(c) R/4 (d) 4 R.
233. The efficiency of a modern lead accumulator is of the
order of
(a) 20 % (b) 50 % Fig. 78
(c) 70 % (d) 100 %. (a) 8.31 A (b) 6.82 A
234. An electric toaster draws 4.8 A, an electric iron draws (c) 4.92 A (d) 2 A.
10.2 A and an electric refrigerator draws 3.5 A from
a 220 volt mains. The three appliances are connected 239. There is a current of 40 ampere in a wire of 10 –6
in parallel. All the three appliances are to be oper- square metre area of cross-section. If the number of
ated at the same time. The current-carrying capacity free electrons per cubic metre is 1029, then the drift
of the fuse should be velocity is
(a) 8 A (b) 10 A (a) 250 mm s–1 (b) 25.0 mm s–1
(c) 13 A (d) 18.5 A. (c) 2.50 mm s–1
235. Five cells, each of emf E and internal resistance r, (d) 125 mm s–1.
are connected in series. If, due to oversight, one cell 240. In the circuit shown in P Q
is connected wrongly, then the equivalent emf and
Fig. 79, P ≠ R. The reading
the internal resistance of the combination is S
of galvanometer is same
(a) 3 E and 5 r (b) 3 E and 3 r
with switch S open or closed.
(c) 5 E and 4 r (d) 5 E and 5 r.
Then R
236. Into how many equal parts is it necessary to cut a G
(a) IR = IG (b) IP = IG
81 Ω conductor so as to obtain a resistance of 1 Ω by
(c) IQ = IG (d) IQ = IR.
connecting these parts in parallel ?
(a) 9 (b) 20 [IIT 1999]
V
(c) 41 (d) 81.
Fig. 79
150 COMPREHENSIVE OBJECTIVE PHYSICS

241. Fig. 80 shows a circuit (a) 4 E (b) 2 E


which may be used to (c) E (d) zero.
compare the resistance l 244. The potential difference between A and C (Fig. 114)
R of an unknown resis-
tor with a 100 Ω stand- is
ard. The distances l X (a) 3 E (b) 2 E
from one end of the (c) E (d) zero.
Y
potentiometer slide- [Bharati Vidyapeeth 2000]
wire to the balance 100 W R Z
point are 400 mm and 245. A current of 4.8 ampere is flowing in a conductor.
588 mm when X is con- The number of electrons flowing per second through
nected to Y and Z re- Fig. 80 the conductor will be
spectively. The length (a) 3 × 1019 electrons per second
of the slide-wire is 1.00 m. (b) 76.8 × 1020 electrons per second
What is the value of resistance R ? (c) 7.68 × 1020 electrons per second
(a) 32 Ω (b) 47 Ω
(d) 3 × 1020 electrons per second. [BHU 1993]
(c) 68 Ω (d) 147 Ω
246. The momentum acquired by the electrons in length l
(e) 150 Ω. [Oxford and Cambridge Schools
of wire when a current I starts to flow is
Examination Board]
(a) mI l/e (b) mI e/l
242. Fig. 81 shows a
Wheatstone’s (c) m I l (d) m I e [Pb. PMT 1996]
net, with P = 247. The capacity of a storage cell is 2.5 Ah. What is the
1000 ohm, Q = maximum current that it can supply for half an hour ?
10.0 ohm, R (a) 0.5 A (b) 1.0 A
(unknown), S (c) 2.5 A (d) 5.0 A.
variable and 248. If two wires made of the same material have lengths
near 150 ohm in the ratio 1 : 2 and radii in the ratio 2 : 3, then the
for balance. If resistances are in the ratio of
the connections Fig. 81 (a) 4 : 9 (b) 9 : 4
across A, C and
(c) 8 : 9 (d) 9 : 8.
B, D are interchanged, the error range in R determi-
249. The potential gradient along the length of a uniform
nation would
wire is 10 V m–1. B and C are the two points at 30 cm
(a) remain unaffected (b) increase substantially and 60 cm point on a metre scale fitted along the
(c) increase marginally (d) decrease substantially. wire. The potential difference between B and C is
[National Standard Exam. in Physics 1990] (a) 3 V (b) 0.4 V
243. Fifty identical cells each having emf E, and internal (c) 7 V (d) 4 V.
resistance r are connected as shown in Fig. 82. The 250. The internal resistance of a cell of emf 2 V is 0.1 Ω. It
potential difference between points A and B is is connected to a resistance of 3.9 Ω . The voltage
across the cell will be
(a) 2 V (b) 0.5 V
(c) 1.5 V (d) 1.95 V.
251. A wire of length L metre is redrawn to a length
2L metre. Then its resistance will
1
(a) increase four times (b) decrease to of original
4
(c) increase two times (d) not change.
[MP PET 2001 ; Karnataka 1992]
252. Six resistors of 3 Ω each are connected along the
sides of a hexagon and three resistors of 6 Ω each are
Fig. 82 connected along AC, AD and AE as shown in Fig. 83.
CURRENT ELECTRICITY 151
The equivalent resistance between A and B is equal 257. A 2 volt battery, a 15 Ω resistor and a potentiometer
to of 100 cm length, are all connected in series. If the
resistance of potentiometer wire is 5 Ω, then the po-
tential gradient of the potentiometer wire is
(a) 0.005 V cm– 1 (b) 0.05 V cm– 1
(c) 0.02 V cm– 1 (d) 0.2 V cm– 1.
[AIIMS 1982]
258. In the circuit shown in Fig. 86, XY is a potentiometer
wire 100 cm long. The circuit is connected up as
shown. With switches S2 and S3 open, a balance point

Fig. 83
(a) 2 Ω (b) 3 Ω
(c) 6 Ω (d) 9 Ω. [AMIU 2000]
253. A current I flows in a wire of circular cross-section
with the free electrons travelling with a drift velocity
v. The drift velocity of electrons when a current 2 I
flows in another wire of twice the radius and of the
same material is
(a) v/2 (b) v
(c) 2 v (d) 2 v. [CMC LDH 1997]
254. A wire of resistance 36 Ω is bent in the form of an
Fig. 86
equilateral triangle. The resistance between two ver-
tices is is found at Z. After switch S1 has remained closed
(a) 12 Ω (b) 24 Ω for some time, it is found that the contact at Z must
(c) 8 Ω (d) 36 Ω. be moved towards Y to maintain a balance. Which of
the following is the most likely reason for this ?
255. In the circuit E1 R1 (a) The cell V1 is running down.
shown here, E 1 =
E2 = E3 = 2 V and (b) The cell V2 is running down.
R1 = R2 = 4 Ω. The (c) The wire XZ is getting warm and its resistance is increas-
current flowing be- E2 ing.
tween points A and A B (d) The resistor R1 is getting warm and increasing in value.
B through battery (e) Polarisation is affecting the emf of V2.
E2 is [Associated Examination Board, Hampshire]
E3 R2
(a) Zero
259. The potentiometer in Q. 258 is balanced as before
(b) 2 A from A to B with S3 open. S3 is then closed and the balance point
Fig. 84
(c) 2 A from B to A again found. What happens to the balance length XZ
(d) None of the above. [MP PET 2001] when S3 is closed if the internal resistance of the cell
256. Two cells A and B, each V2 is NOT negligible ?
of emf 2 V, are connected (a) No change occurs
in series to an external re- (b) XZ increases, the increase being greatest for large values
sistance R = 1 ohm. If the of R2
internal resistance of cell (c) XZ increases, the increase being greatest for small values
A is 1.9 Ω and that of B is of R2
0.9 Ω, what is the poten- (d) XZ decreases, the decrease being greatest for large values
tial difference between the of R2
Fig. 85
terminals of cell A ? (e) XZ decreases, the decrease being greatest for small values
(a) 2 V (b) 3.8 V of R. [Oxford and Cambridge Schools
(c) Zero (d) None of the above. Examination Board]
[MP PET 2001]
152 COMPREHENSIVE OBJECTIVE PHYSICS

260. An ideal ammeter (zero resistance) and an ideal volt- (a) 3.75 V, 7.5 V (b) 3.75 V, 3.75 V
meter (infinite resistance) are connected as shown. (c) 4.25 V, 7.5 V (d) 4.25 V, 4.25 V.
The ammeter and voltmeter readings are 264. In the above question, the net emf is
(a) 8 V (b) 12 V
(c) 2 V (d) 4 V.
265. The current in the circuit of Fig. 121 is
(a) 0.5 A anticlockwise (b) 0.5 A clockwise
(c) 1 A anticlockwise (d) 1 A clockwise.
266. In Q. 263, the potential difference between A and B is
(a) 4 V (b) 8 V
(c) 2 V (d) 1 V.
267. In the circuit shown in
Fig. 90, the resistance R
Fig. 87 is increased. What will be
the effect on the galva-
(a) 6.25 A, 3.75 V (b) 3.00 A, 5 V
nometer reading if the
(c) 3.00 A, 3.75 V (d) 6.00 A, 6.25 V. internal resistance of the
[RPET 1998] battery is zero ?
261. Two resistors of resistances 200 kΩ and 1 MΩ re- (a) It will not change.
spectively form a potential divider with outer junc- Fig. 90
(b) It will increase.
tions maintained at potentials of + 3 V and – 15 V. (c) It will decrease. (d) Data is inadequate.
268. In the above question, if the battery has a finite value
of internal resistance, then what will be the effect on
galvanometer reading ?
(a) It will not change. (b) It will increase.
Fig. 88
(c) It will decrease. (d) Data is inadequate.
What is the potential at the junction X between the
269. Three 10 Ω , 2 W resistors are connected as in Fig.
resistors ?
91. The maximum possible voltage between points A
(a) + 1 V (b) 0 V
and B without exceeding the power dissipation lim-
(c) – 0.6 V (d) – 12 V. its of any of the resistors is
262. N identical cells, each of emf E and internal resist-
ance r, are joined in series. Out of these, n cells are
wrongly connected, i.e., their terminals are connected
in reverse of that required for series connection. The
emf and internal resistance of the resulting battery
are respectively
(a) (N – n) E, (N – n)r (b) (N – 2n) E, (N – 2n)r Fig. 91
(c) (N – 2n) E, Nr (d) (N – n) E, Nr.
(a) 5 3 V (b) 3 5 V
[TNPCEE 2002]
5
263. In the circuit (c) 15 V (d) V.
3
shown in Fig. 89,
the cells E1 and E2 270 Two uniform wires A and B are of same metal and
have emfs 4 V and have equal masses. The radius of A is twice that of B.
8 V and internal The total resistance of A and B when connected in
resistances 0.5 Ω parallel is
and 1 Ω respec- (a) 4 Ω when the resistance of wire A is 4.25 Ω
tively. Then the (b) 5 Ω when the resistance of wire A is 4 Ω
potential differ-
(c) 4 Ω when the resistance of wire B is 4.25 Ω
ence across E1 and
E2 will be Fig. 89 (d) 5 Ω when the resistance of wire A is 4 Ω. [CEE 1994]
CURRENT ELECTRICITY 153

271. A cell of emf E volt has internal resistance of r ohm. (a) E =


j
(b) E = jk
It is connected across external resistance of x ohm. k
The potential difference across the terminals of the k
(c) E = (d) k = jl.
cell is E/2 volt. Then, j
(a) x = r (b) x > r 276. To get maximum current in a resistance of 3 Ω, one
can use n rows of m cells (connected in series) con-
(c) x < r (d) x ≤ r. [BITS 1999]
nected in parallel. If the total number of cells is 24
272. In the circuit shown in and the internal resistance of a cell is 0.5 Ω, then
Fig. 92, cells A and B (a) m = 12, n = 2 (b) m = 8, n = 3
have emf 10 V each and (c) m = 2, n = 12 (d) m = 6, n = 4.
the internal resistance
277. Three resistances 2 Ω, 3 Ω and 5 Ω are connected in
is 5 Ω for A and 2 Ω for
parallel to a battery of 10 V and of negligible internal
B. For what value of R resistance. The potential drop across the 3 Ω resist-
will the potential differ- Fig. 92
ance will be
ence across cell A be zero ? (a) 2 V (b) 3 V
(a) zero (b) 1 ohm (c) 5 V (d) 10 V.
(c) 2 ohm (d) 3 ohm. 278. Five resistances are connected as shown in Fig. 94.
[National Standard Exam. in Physics 1990] The effective resistance between A and B is
273. In the potentiometer circuit shown in Fig. 93, V and
R are set to give a potential gradient 10 mV/cm on
the wire MN. If R′ = 2.000 ohm, and the null point
occurs at MP = 82.8 cm, the current through R′ is

Fig. 94

10 10
(a) Ω (b) Ω
3 17
(c) 40 Ω (d) 45 Ω. [TNPCEE 2001]
279. The equivalent resistance of the arrangement of re-
Fig. 93 sistance shown in Fig. 95, between A and B, is
(a) 0.414 A (b) 0.828 A
(c) 1.656 A (d) 0.207 A.
[National Standard Exam. in Physics 1990]
274. An electrical cable of copper has just one wire of ra-
dius 9 mm. Its resistance is 5 Ω. This single wire is
replaced by 6 different well-insulated copper wires,
each of radius 3 mm. The total resistance of the ca-
ble would now be equal to
(a) 7. 5 Ω (b) 45 Ω
(c) 90 Ω (d) 270 Ω.
275. The electric intensity E, current density j and spe-
cific resistance k are related to each other through
Fig. 95
the relation
(a) 6 ohm (b) 8 ohm
(c) 16 ohm (d) 24 ohm.
[TNPCEE 2000]
154 COMPREHENSIVE OBJECTIVE PHYSICS

280. The dimensions of a block are 1 cm × 1 cm × 100 cm.


If the specific resistance of its material is 2 × 10 –7
Ω m, then the resistance between the opposite rec-
tangular faces is
(a) 2 × 10 – 9 Ω (b) 2 × 10 – 7 Ω
(c) 2 × 10 – 5 Ω (d) 2 × 10 – 3 Ω.
Fig. 97
281. In the previous question, the resistance between the
(a) 1 (b) 2
square faces is
(c) 3 (d) 4 or 5.
(a) 2 × 10 – 9 Ω (b) 2 × 10 – 7 Ω
(c) 2 × 10 – 5 Ω (d) 2 × 10 – 3 Ω.
[Bharati Vidyapeeth 1999]
286. In a potentiometer circuit, there is a cell of emf 2 volt,
282. Two resistances are joined in parallel. The combined
a resistance of 5 ohm and a wire of uniform thick-
6 ness of length 1000 cm and resistance 15 ohm. The
resistance is Ω. One of the resistance wires is bro-
5 potential gradient in the wire is
ken and the effective resistance becomes 2 Ω. Then
the resistance of the wire that got broken was (a) 1/500 V cm– 1 (b) 3/2000 V cm– 1

3 (c) 3/5000 V cm– 1 (d) 1/1000 V cm– 1.


(a) Ω (b) 2 Ω
5 [MGIMS 2002]
6 287. A potentiometer is connected across A and B and a
(c) Ω (d) 3 Ω. [MP PMT 2001]
5 balance is obtained at 64.0 cm. When potentiometer
283. Two square metal plates A and B are of the same lead to B is moved to C, a balance is found at 8.0 cm.
thickness and material. The side of B is twice that of If the potentiometer is now connected across B and
A. These are connected as shown in Fig. 96 (series C, a balance will be found at
connection). If RA and RB are the resistances of A
RA
and B respectively, then is
RB

Fig. 98
(a) 8.0 cm (b) 56.0 cm
(c) 64.0 cm (d) 72.0 cm.
[JIPMER 1999]
288. Two resistances 100 Ω and 200 Ω are connected in
series across a battery of emf 10 V and negligible
internal resistance. To increase the current from the
battery to double the value, a resistance of R must be
Fig. 96 connected across the combination. R is
(a) 1 : 2 (b) 2 : 1 (a) 100 Ω (b) 150 Ω

(c) 1 : 1 (d) 4 : 1. (c) 200 Ω (d) 300 Ω.

284. A battery formed of three lead-acid storage cells in 289. The equivalent resistance of three parallel resistors
series, each having emf 2.2 V, is charged at a rate of is 30 Ω. Their resistances are in the ratio 1 : 3 : 5.
100 A. The potential difference across its terminals The resistances of the resistors are
is measured to be 7.6 V. This same battery is now (a) 30 Ω, 90 Ω, 150 Ω (b) 23 Ω, 69 Ω, 115 Ω
used to deliver a current of 100 A. Its new terminal (c) 46 Ω, 138 Ω, 230 Ω (d) 4 Ω, 12 Ω, 20 Ω.
voltage is 290. Four cells, each of emf 2 V and internal resistance
(a) 9.6 V (b) 5.6 V 1 Ω, are in series across an external resistance of 6
(c) 4.5 V (d) 18.7 V. [AMU 2001] Ω . The current in the external resistance is
285. In Fig. 97, a fuse in one of the bulbs causes all the (a) 0.25 A (b) 0.5 A
other to go out. Which bulb has fused ? (c) 0.8 A (d) 1.33 A.
CURRENT ELECTRICITY 155
291. Two copper wires whose masses are 8 g and 12 g 297. One metre long metallic wire is broken into two un-
have lengths in the ratio 3 : 4. Their resistances are equal parts P and Q. The part P is uniformly ex-
in the ratio of tended into another wire R. Length of R is twice the
(a) 27 : 128 (b) 27 : 32 length of P and the resistance of R is equal to that of
(c) 16 : 9 (d) 4 : 9. Q. The ratio of the resistances of P and R is
(a) 1: 4 (b) 1: 2
292. In the circuit shown in
(c) 3 : 2 (d) None of these.
Fig. 99, the total current 2W 6W
supplied by the battery is [Roorkee 1996]
3W
(a) 2 A 298. In a potentiometer experiment, it is observed that no
6V current flows through the galvanometer when the
(b) 4 A 1.5 W
terminals of the cell are connected across 52 cm of
(c) 1 A the potentiometer wire. If the cell is shunted by a
(d) 6 A. Fig. 99 resistance of 5 Ω , a balance is observed when the cell
[AIEEE 2004 ; DPMT 1998, Modified] is connected across 40 cm of the wire. The internal
resistance of the cell is
293. In the above question, the current through the 6 Ω
200
resistor, is (a) 5 Ω (b) Ω
52
(a) 1 A (b) 2 A
52
(c) 0.5 A (d) 4 A. (c) Ω (d) 1.5 Ω.
8
294. Three resistances, each of 3 Ω , are connected together 299. A battery of four cells in series, each having an emf
in different manners. Each time the combination is of 1.4 V and an internal resistance of 2 Ω, is to be
used at the same potential difference of V volt. The used to charge a small 2 V accumulator of negligible
ratio of the maximum and minimum currents ob- internal resistance. The charging current is
tained in combination will be (a) 0.1 A (b) 0.2 A
(a) 1 : 1 (b) 9 : 1 (c) 0.3 A (d) 0.45 A.
(c) 1 : 9 (d) 1 : 3. [Haryana PMT 2001]
295. In the circuit shown in Fig. 100, every resistance has 300. In the circuit shown in Fig. 102, the current I has a
24 ohm value. The resistance between terminals P value equal to
and Q is 15 W
4W
I 2W 15 W

15 W
4W

E = 10 V
r=1W

Fig. 100 A

(a) 26 ohm (b) 32 ohm Fig. 102


(c) 36 ohm (d) 42 ohm. (a) 1 A (b) 2 A
[Kerala PMT 2003 ; National Standard (c) 4 A (d) 3.5 A. [RPMT 2002]
Exam. in Physics 1994]
301. Variation of current passing D
296. In the circuit shown in 100 W through a conductor as the
Fig. 101, the potential differ-
ence across PQ will be near- voltage applied across its C

est to 80 W ends is varied is shown in B


48 V
(a) 9.6 V (b) 6.6 V 100 W Q
Fig. 103. If the resistance is
V A
(c) 4.8 V (d) 3.2 V.
20 W determined at the points A,
P
B, C and D, we will find that I
[National Standard
resistances at Fig. 103
Exam. in Physics 1995] Fig. 101
156 COMPREHENSIVE OBJECTIVE PHYSICS

(a) C and D are equal (b) B is higher than at A 308. A wire has a resistance of 10 ohm. What will be its
(c) C is higher than at B (d) A is lower than at B. resistance if it is stretched by one tenth of its origi-
302. A solid metal cube having resistance of 4 Ω between nal length ?
its two opposite faces is cut into 64 equal-sized small (a) 12.1 ohm (b) 14.2 ohm
cubes. The resistance of one such small cube between (c) 10 ohm (d) 10.1 ohm.
two opposite faces is [TNPCEE 1999]
(a) 4 Ω (b) 8 Ω 309. The specific resistance of manganin is 50 × 10 –8 Ω m.
(c) 12 Ω (d) 16 Ω . The resistance of a cube of length 50 cm will be
303. A battery of 6 V is connected to the terminals of a (a) 10 –6 Ω (b) 2.5 × 10 – 5 Ω
three-metre long wire of uniform thickness and re- (c) 10 – 8 Ω (d) 5 × 10 – 4 Ω.
sistance of the order of 100 Ω. The potential differ- [MGIMS 2000]
ence between two points separated by 50 cm on the
310. Read the following statements
wire is
Y : The resistivity of a semiconductor decreases with
(a) 1 V (b) 1.5 V
increase of temperature.
(c) 2 V (d) 3 V.
Z : In a conducting solid, the rate of collisions be-
304. The current I in the given circuit is tween free electrons and ions increases with in-
crease of temperature.
Select the correct statement(s) from the follow-
ing :
(a) Y is true but Z is false (b) Y is false but Z is true
(c) Both Y and Z are true
(d) Y is true and Z is the correct reason for Y. [IIT 1993]
311. What is the current flowing through 2 Ω resistance
shown in Fig. 105 ?
Fig. 104

1 1
(a) A (b) A
45 15
1 3
(c) A (d) A. [RPMT 2001]
10 5
305. There are 8 equal resistances R. Two are connected
in parallel. Such four groups are connected in series.
The total resistance of the system will be
(a) R/2 (b) 2 R
(c) 4 R (d) 8 R. [MP PMT 1987] Fig. 105

306. An electron revolves 6 × 1015 times per second in a 2E


(a) 2 E (b)
circular orbit. The current in the loop is 7
(a) 0.96 m A (b) 0.96 µ A E
(c) (d) E. [Pb. PMT 1999]
7
(c) 28.8 A (d) None of these.
312. A technician has only two resistors. By using them
[MLNR 1995]
singly in series or in parallel, he is able to obtain the
307. In Bohr’s hydrogen atom, the electron moves around resistance of values 3, 4, 12 and 16 Ω. What are the
the nucleus in a circular orbit of radius 5 × 10 –11 m resistances of the two coils ?
with a time period of 1.5 × 10 – 16 s. The associated (a) 6 Ω and 10 Ω (b) 4 Ω and 12 Ω
current is
(c) 7 Ω and 9 Ω (d) 5 Ω and 12 Ω.
(a) zero (b) 1.6 × 10 –19 A
[Karnataka 1993]
(c) 0.17 A (d) 1.07 × 10 –3 A. [CEE 1992]
CURRENT ELECTRICITY 157

313. You are given several identical resistances, each of


R = 10 Ω and each capable of carrying a maximum
current of 1 A. It is required to make suitable combi-
nation of these resistances to produce a resistance of
5 Ω which can carry a current of 4 A. The minimum
number of such resistances required is
(a) 4 (b) 10 Fig. 108
(c) 8 (d) 20.
2 8
[All India PM/PD 1998] (a) V (b) V
3 9
314. A constant 60 V dc supply is connected across two
resistors of resistances 400 kΩ and 200 kΩ . 4
(c) V (d) 2 V. [JIPMER 1999]
3
What is the reading of the voltmeter, also of resist-
ance 200 kΩ, when connected across the second re- 317. Two resistors of 15 Ω and 30 Ω are connected in
sistor as shown in the Fig. 106 ? parallel. What should be the value of R to be con-
nected in series with the other two so that the net
resistance will be 20 Ω ?
(a) 5 Ω (b) 10 Ω
(c) 15 Ω (d) 120 Ω.
[All India PM/PD 1998]
318. In the circuit shown in Fig. 109, the current through

Fig. 106
(a) 12 V (b) 15 V
(c) 20 V (d) 30 V
(e) 40 V.
315. Fig. 107 shows a network of 9 identical resistors.
The resistance of the whole circuit is 1.5 Ω . The re- Fig. 109
sistance R is (a) the 3 Ω resistor is 0.50 A (b) the 3 Ω resistor is 0.25 A
(c) the 4 Ω resistor is 0.50 A (d) the 4 Ω resistor is 0.25 A.
[IIT 1998]
319. A potentiometer is connected with a battery of con-
stant emf. The emf of cadmium cell is 1.0182 volt
and is balanced by 33.94 cm length of potentiometer.
The potential gradient of the potentiometer wire is
(a) 0.03 V cm–1 (b) 3 V cm–1
(c) 30 V m–1 (d) 1.0182 V m–1.
[JIPMER 1998]
320. When a current I is flowing through a conductor, the
Fig. 107 drift velocity is v. If 2I current flows through the
(a) 1.1 Ω (b) 3.3 Ω same metal but having double the area of cross-
(c) 1.4 Ω (d) 1.8 Ω. [MGIMS 1999] section, then the drift velocity will be
(a) v/4 (b) v/2
316. The potential difference between points A and B of
network shown in Fig. 108 is (c) v (d) 4v. [EAMCET 2000]
158 COMPREHENSIVE OBJECTIVE PHYSICS

321. In the given circuit, the current in the 2 ohm resistor is (a) 0 (b) 2
(c) 4 (d) 8.
326. The current in a circuit having an external resist-
ance of 3.75 Ω is 0.5 A. When a resistance of 1 Ω is
introduced into the circuit, the current becomes 0.4 A.
The emf of the power source is
(a) 4 V (b) 3 V
(c) 2 V (d) 1 V.
Fig. 110 327. In the previous question, the internal resistance of
(a) 1.4 A (b) 1.2 A the source is
(c) 0.4 A [DPMT 2000]
(d) 1.0 A. (a) 1 Ω (b) 0.85 Ω
322. The resistance between A and B (see Fig. 111) of the (c) 0.5 Ω (d) 0.25 Ω.
network is 328. A current flows in a wire of circular cross-section
with the free electrons travelling at drift velocity v.
What is the drift velocity for the same current in a
wire of the same material but of half the radius ?
v v
(a) (b)
4 2
(c) v (d) 2v
(e) 4v.
329. A 1 µF capacitor
holding a charge of
Fig. 111 1 × 10 –5 C is con-
nected to a 10 Ω re-
R R
(a) (b) sistor via a switch.
2 3 Fig. 113
What current will
(c) 6 R (d) 2 R.
flow after the switch is closed ?
323. A battery of 20 cells (each having emf 1.8 V and in-
(a) zero (b) 10–5 A
ternal resistance 0.1 Ω) is charged by 220 V and the
(c) 1 A (d) 10– 1 A
charging current is 15 A . The resistance to be put in
the circuit is (e) 10 A.
(a) 10.27 Ω (b) 12.27 Ω 330. AB is a potentiomet- R 2V
(c) 8.62 Ω (d) 16.24 Ω. er wire of length
100 cm and its re-
324. Fig. 112 represents a part of closed circuit. The po-
sistance is 10 Ω. It A 40 cm B
tential difference (VA – VB ) is
is connected in se-
ries with a resistance
R = 40 Ω and a bat- E
Fig. 114
tery of emf 2 V and
negligible internal resistance. If a source of unknown
emf E is balanced by 40 cm length of the potentiometer
Fig. 112 wire, then the value of E is
(a) 24 V (b) 0 V (a) 0.8 V (b) 1.6 V
(c) 6 V (d) 18 V. (c) 0.08 V (d) 0.16 V. [MP PET 2001]
325. 32 cells, each of emf 2 V, are connected in series and 331. In the network of resistances shown in Fig. 115, the
kept in a box. Externally, the combination shows a effective resistance between A and B is
potential difference of 56 V. The number of cells re-
versed in the connection is
CURRENT ELECTRICITY 159

Fig. 115
5 8
(a) R (b) R
3 3
(c) 5 R (d) 8 R. [AMU 1999]
332. When a conductor of cross-sectional area 5.0 × 10 –6 Fig. 118
m2 carries a current of 6.0 A, the drift velocity of the
(a) 60 Ω (b) 30 Ω
conduction electrons is 1.2 × 10 – 4 m s–1.
(c) 15 Ω (d) 90 Ω. [EAMCET 1999]
What is the number density (number per unit vol-
ume) of the conduction electrons ? 336. Fig. 119 shows a network of resistors. Each resist-
(a) 4.0 × 10 –28 m – 3 (b) 6.3 × 1028 m – 3 ance is of value 2 Ω. The equivalent resistance be-
(c) 1.6 × 10 –27 m – 3 (d) 1.3 × 1034 m – 3 tween A and B is
(e) 2.5 × 10 27 m – 3.
333. What is current I in the circuit shown in Fig. 116 ?
A

Fig. 119

Fig. 116 (a) 5 Ω (b) 7 Ω


(c) 9 Ω (d) 11 Ω.
(a) 1 A (b) 0.5 A
(c) 1.2 A (d) 2 A.
[Himachal PMT 1999]
337. In Fig. 120, the effective resistance of the network
[All India PM/PD 1998]
between a and b is
334. Five resistances are connected as shown in Fig. 117.
The equivalent resistance between the points A and R R R
B is a
R
b
R R R

Fig. 120
(a) 2 R (b) 4 R
(c) 10 R (d) 5 R/2.
[Haryana PMT 1998]
338. Two cells A and B of elec- E1
tromotive forces 1.3 V A
and 1.5 V respectively are
Fig. 117 arranged as shown in V

(a) 5 Ω (b) 10 Ω Fig. 121. The voltmeter B


(assumed ideal) reads
(c) 15 Ω (d) 1.5 Ω. [MNR 1997] E2
1.45 V. The internal
335. Fig. 118, shows a network of 8 resistors. The equiva- Fig. 121
resistances of cells A and
lent resistance between A and B is B are rA and rB respectively. Which of the following is
correct ?
160 COMPREHENSIVE OBJECTIVE PHYSICS

(a) rA = 2 rB (b) rA = 3 rB (a) VAC = VCB (b) VAC > VCB


(c) rB = 2 rA (d) rB = 3 rA. (c) VAC < VCB (d) VCB = 24 volt.
[Haryana PMT 1998] [AFMC 1998]
339. The value of current I in the circuit shown in Fig. 122 343. Fig. 124 shows a potential di-
is vider circuit which, by adjust-
ment of the position of the
60 W
contact X, can be used to pro-
I vide a variable potential dif-
15 W 5W ference between the terminals
P and Q.
What are the limits of this po-
10 W Fig. 124
tential difference ?
1A 1A (a) 0 and 20 m V (b) 5 mV and 25 m V
(c) 0 and 20 V (d) 0 and 25 V
(e) 5 V and 25 V.
Fig. 122 344. An electric current flows along an insulated strip PQ
of metallic conductor.
(a) 0.1 A (b) 0.5 A
1 1
(c) A (d) A.
3 6
[Karnataka CET 2000]
340. A uniform wire of resistance R is uniformly com-
pressed along its length until its radius becomes n
times the original radius. Now the resistance of the
wire becomes
R R
(a) (b)
n 2 n4 Fig. 125
R
(c) nR (d) . The current density in the strip varies as shown in
n
the graph.
[Karnataka CET 2000]
Which one of the following statements could explain
341. In an electrostatic machine, a belt of width w, hav-
this variation ?
ing surface charge density σ, travels with velocity v.
(a) The strip is narrower at P than at Q.
As the belt passes a certain point, all the charge is
removed and is carried away as an electric current. (b) The strip is narrower at Q than at P.
What is the magnitude of this current ? (c) The potential gradient along the strip is uniform
wσ (d) The current at P is greater than the current at Q.
(a) wvσ (b)
v (e) The resistance per unit length of the strip is constant.
vσ 345. A flash light battery is connected in the circuit shown
(c) wv2 σ (d)
w in Fig. 126. The current through the battery is
(e) 1/2wv2σ.
342. Five resistors are connected between points A and B
as shown in Fig. 123. A current of 10 A flows from A
to B. Which of the following is correct ?

Fig. 126
(a) 0.3 A (b) 0.4 A
(c) 0.5 A (d) 0.6 A.
Fig. 123 [Himachal PMT 1998]
CURRENT ELECTRICITY 161
346. A and B are two points on a 350. Two cells of emfs E1 and E2 (E1 > E2 ) are connected
uniform ring of resistance R. as shown in Fig. 129.
∠ AOB = θ, where O is the
centre of the ring. The equiva-
lent resistance between A and
B is
Fig. 129
R
(a) (2 π − θ) θ When a potentiometer is connected between A and B,
4π2
Fig. 127 the balancing length of the potentiometer wire is
R
(b) × (2 π − 1) θ 300 cm. On connecting the same potentiometer be-
4π2 tween A and C, the balancing length is 100 cm. The
(2π − θ) θ
(c) R (d) R . [Pb. PMT 1996] E1
4π 2π ratio is
E2
347. The potential difference between the points P and Q
in the electric circuit shown in Fig. 128 is (a) 3 : 1 (b) 1 : 3
(c) 2 : 3 (d) 3 : 2.
[All India PM/PD 1997]
351. Three resistances of values 2 Ω , 3 Ω and 6 Ω are to
be connected to produce an effective resistance of 4
Ω. This can be done by connecting.
(a) 6 Ω resistance in series with the parallel combination of 2 Ω
and 3 Ω
(b) 3 Ω resistance in series with the parallel combination of 2 Ω
and 6 Ω
(c) 2 Ω resistance in series with the parallel combination of 3 Ω
and 6 Ω
(d) 2 Ω resistance in parallel with the parallel combination of
3 Ω and 6 Ω. [Karnataka CET 2001]
352. Five resistances are connected as shown in the
Fig. 128 Fig. 130. The equivalent resistance between A and C
(a) 4.5 V (b) 12 V is
(c) 24 V (d) 8 V.
2W 3W
[Karnataka CET 1999]
348. Three similar cells, each of emf 2 V and internal re- A 7W C
sistance r Ω , send the same current through an ex-
ternal resistance of 2 Ω , when connected in series or
4W 6W
in parallel. The strength of the current flowing
through the external resistance is Fig. 130
(a) 0.75 A (b) 1 A 10
(a) Ω (b) 22 Ω
(c) 1.5 A (d) 2 A. 3
[Karnataka CET 1999] (c) 15 Ω (d) 10.6 Ω.
349. A cylindrical metal wire of length l and cross-sec- [Karnataka CET 2001]
tional area S has resistance R, conductance G, resis- 353. Refer to the circuit shown in Fig. 131.
tivity ρ and conductivity σ. Which one of the follow-
ing expressions for σ is valid ?
GR ρlG
(a) (b)
ρ SR
Rl GS
(c) (d)
S l
ρR
(e) .
G
Fig. 131
162 COMPREHENSIVE OBJECTIVE PHYSICS

The ammeter (assumed ideal) reads 0.48 A. The in- I I


ternal resistance of the battery is
(a) 1.0 Ω (b) 0.5 Ω
(c) 1.5 Ω (d) 2 Ω. [MANIPAL 1997]
0 0
354. A wire 3.00 m long, of uniform cross-sectional area 0 V 0 V
(d) (e)
2.00 mm2, has a conductance of 1.25 S. What is the
resistivity of the material of the wire ? [1 S = 1 Ω m]
Fig. 133
(a) 5.3 × 10 – 7 S (b) 2.4 S
(c) 8.3 × 10 – 7 S (d) 1.9 × 106 S 358. 5 rows of 10 identical cells, connected in series, send
(e) 0.80 S.
a current I through an external resistance of 20 ohm.
If the emf and internal resistance of each cell are 1.5
355. Refer to the circuit shown in Fig. 132. The value of R volt and 1 Ω respectively, then the value of I is
is
(a) 0.68 A (b) 0.75 A
(c) 0.25 A (d) 0.14 A.
[Karnataka CET 2000]
359. The resistance of a conductor is 5 ohm at 50°C and 6
ohm at 100°C. Its resistance at 0°C is
(a) 3 ohm (b) 4 ohm
(c) 1 ohm (d) 2 ohm.
[Karnataka CET 2000]
360. Fig. 134 below shows a thick copper rod X and a thin
copper wire Y joined in series. They carry a current
which is sufficient to make Y much hotter than X.
Fig. 132
(a) 8 ohm (b) 16 Ω
(c) 4 Ω (d) 20 ohm. Y
X
[Bharati Vidyapeeth 1997]
Fig. 134
356. A current flows in a wire of circular cross-section
with the free electrons travelling with a drift velocity Which one of the following is correct ?
v. If an equal current flows in a wire of the same
number density mean time between
material but of twice the radius, what is the new
of conduction collisions of the
mean drift velocity ?
electrons electrons
v
(a) (b) v (a) same in X and Y less in X than in Y
4
v (b) same in X and Y same in X and Y
(c) 2 v (d)
2 (c) same in X and Y more in X than in Y
(e) 4 v. (d) more in X than Y less in X than in Y
357. Some early electric light bulbs used carbon filaments, (e) more in X than Y same in X and Y
the resistances of which decreased as their tempera-
361. The long resistor between
ture increased. Which of the following graphs best A
represents the way in which I, the current through A and B has a resistance
such a bulb, would depend upon V, the potential dif- of 300 ohm and is tapped 100 W
ference across it ? at one-third points as D 40 W
shown in figure. Then the
I I I equivalent resistance be- 100 W
tween A and B is 120 W
C
(a) 70 Ω
100 W

25 W

0 0
(b) 475 Ω
0 B
0 V 0 V 0 V (c) 32 Ω
(a) (b) (c)
(d) 20 Ω. [WB JEE 1999] Fig. 135
CURRENT ELECTRICITY 163

362. There are two concentric spheres of radii a and b J


(a > b) and the space between them is filled with a
medium of resistivity ρ. Then the resistance between
the inner and outer coatings of the medium will be
FG IJ
ρ b−a ρFG a − b IJ
(a)
4πH K
ab
(b)
4π H ab K
ρ F 1
(c)
ρ2 F 1 1 I
G − J
4π H b a K
(d) G − 1 IJ .
4 π H b2 a2 K J
0
X (c) Y
J
[WB JEE 1998]
363. In the circuit shown in Fig. 136, the battery E1 has
an emf of 12 volt and zero internal resistance while
the battery E2 has an emf of 2 volt. If the galvanometer
G reads zero, then the value of the resistance Y is
(zero at all points)
0 0
X Y X Y
(d) (e)
Fig. 138

365. The charge flowing in a conductor varies with time


as Q = at – bt2. Which of the following in incorrect ?
(a) The current decreases linearly with time
(b) The current reaches a maximum and then decreases
a
Fig. 136 (c) The current falls to zero after time t =
2b
(a) 10 Ω (b) 100 Ω (d) The current changes at a rate – 2b. [WB JEE 1995]
(c) 500 Ω [WB JEE 1997]
(d) 200 Ω.
366. In the circuit shown in Fig. 139, the resistances R1
364. A cell is connected to a uniform resistance wire XY and R2 are respectively
and Y is earthed as shown in Fig. 137.

Fig. 139
Fig. 137
(a) 14 Ω and 40 Ω (b) 40 Ω and 14 Ω
Which one of the graphs in Fig. 138 shows how the (c) 40 Ω and 30 Ω (d) 14 Ω and 30 Ω.
current density J varies along XY ? [National Standard Exam. in Physics 2001]
J J 367. What is the effective resistance between points A and
B?

Fig. 140
0 0
X (a) Y X Y (a) 4R /3 (b) R
(b)
(c) 2R/3 (d) R/2.
[National Standard Exam. in Physics 2001]
164 COMPREHENSIVE OBJECTIVE PHYSICS

368. The current in the branch AB is 1W


M
P Q

2W
(d)
2W
M
P Q

1W
(e)
Fig. 141
(a) 1 A (b) 2 A Fig. 143
(c) 1.5 A (d) 3 A. 371. In the circuit shown in Fig. 144, what is the value of
[National Standard Exam. in Physics 1999] the unknown resistor R so that the total resistance
369. Three resistors are connected as shown in Fig. 142. of the circuit between points P and Q is also equal to
The points X and Y are connected to a source of di- R?
rect current.
The ratio I1/I3 is

Fig. 144
Fig. 142
(a) 3 Ω (b) 39 Ω
R3 + R1 R2 + R 1
(a) (b) (c) 69 Ω (d) 10 Ω. [MP PET 2001]
R1 R1
372. The equivalent resistance between points A and B in
R 2R 3 the circuit shown in Fig. 145 is
(c)
R 1 (R 2 + R 3 )
(d) dependent on the internal resistance of the source and
independent of R1.
370. In which one of the following arrangements of resis-
tors does the meter M, which has a resistance of 2 Ω,
gives the largest reading when the same potential Fig. 145
difference is applied between points P and Q ?
(a) 5 R (b) 2 R
1W 2W R 6R
M (c) (d) . [MP PET 2001]
P Q 2 5
(a)
373. In the circuit shown in Fig. 146, resistors X and Y,
each of resistance R, are connected to a 6 V battery
1W 1W of negligible internal resistance. A voltmeter, also of
resistance R, is connected across Y.
M M
P Q P Q
2W 2W
(b) (c)
CURRENT ELECTRICITY 165
(a) 111 V (b) 250 V
(c) 125 V (d) 333 V
X
(e) 200 V.
376. A standard cell of emf 1.02 V is used to find the
6V
potential difference across the wire XY as shown in
voltmeter of Fig. 149. It is found that there is no current in the
Y V
resistance R galvanometer when the sliding contact is at S, l1 from
X and l2 from Y.
What is the potential difference across XY ?
Fig. 146
What is the reading of the voltmeter ?
(a) zero (b) between zero and 3 V
(c) 3 V (d) between 3 V and 6 V
(e) 6 V.
374. Fig. 147 shows a simple potentiometer circuit for
measuring a small emf produced by a thermocouple.
The metre wire PQ has a resistance of 5 Ω and the
driver cell has an emf of 2.00 V. If a balance point is
obtained 0.600 m along PQ when measuring an emf
of 6.00 mV, what is the value of the resistance R ?
Fig. 149

F l2 I V
(a) 1.02 V (b) 1.02 GH l1 JK
F l1 + l2 I V Fl I
(d) 1.02 G 1 J V
(c) 1.02 GH l2 JK H l2 K
Fl + l I
(e) 1.02 G 1 2 J V .
H l1 K
377. If the conductivity of the fuse wire is doubled, the
safe current will be
1
(a) 2 times (b) times
Fig. 147 2
(a) 95 Ω (b) 995 Ω 1
(c) times (d) 2 times.
(c) 195 Ω (d) 1995 Ω 2
(e) 495 Ω. [EAMCET 1998]
375. A constant voltage dc source is connected, as shown 378. In the circuit shown in Fig. 150, there is a current of
in Fig. 148, across two resistors of resistances 400 3 A in the 2 Ω resistor.
kΩ and 100 kΩ. What are the values of the current I delivered by,
What is the reading of the voltmeter, also of resist- and the voltage V across the power supply ?
ance 100 kΩ, when connected across the second re-
sistor as shown ?
1000 V
dc supply

V
100 kW

400 kW 100 kW
Fig. 150
Fig. 148
166 COMPREHENSIVE OBJECTIVE PHYSICS

I (in A) V (in V) 20 minute. When the other coil is used, the same
quantity of water takes 30 minute to boil. How long
(a) 3 10.5
will it take for the same quantity of water to boil if
(b) 4 9
the two coils are connected in series ?
(c) 4 12
(a) 25 minute (b) 12 minute
(d) 12 18
(c) 50 minute (d) 40 minute.
THERMAL EFFECTS OF CURRENT 385. In the previous question, have long will it take for
the same quantity of water to boil if the two coils are
379. A tap supplies water at 22°C. A man takes 1 litre of connected in parallel ?
water per minute at 37°C from the geyser. The power (a) 50 minute (b) 25 minute
of the geyser is
(c) 12 minute (d) 18 minute.
(a) 525 W (b) 1050 W
386. A fuse wire with a circular cross-sectional radius of
(c) 1575 W (d) 2100 W.
0.2 mm blows with a current of 5 A. For what value
380. Three equal resistors, connected in parallel with a
of current, another fuse wire made from the same
battery, together dissipate 90 W of power. What power
material but with cross-sectional radius of 0.3 mm
will be dissipated if the same resistances are con-
blow ?
nected in series with the same battery ?
3
10 (a) 5 A (b) 5 × A
(a) 10 W (b) W 2
3
3 27
(c) 90 W (d) 30 W. (c) 5 × A (d) 5 × A.
2 8
381. In the circuit shown in Fig. 151, the heat produced in
5 Ω resistor due to current flowing in it is 10 cal s–1. 387. You are given a resistance wire of length 50 cm and a
The heat produced in 4 Ω resistor is battery of negligible resistance. In which of the fol-
lowing cases is largest amount of heat generated ?
(a) When the wire is connected to the battery directly.
(b) When the wire is divided in two parts and both the parts are
connected to the battery in parallel.
(c) When the wire is divided in four parts and all the four parts
are connected to the battery in parallel.
(d) When only half the wire is connected to the battery.
388. A constant voltage is applied between the two ends of
Fig. 151 a uniform metallic wire. Some heat is developed in
it. The heat developed is doubled if
(a) 1 cal s–1 (b) 2 cal s–1
(a) both the length and radius of wire are halved
(c) 3 cal s–1 (d) 4 cal s–1. [IIT 1985]
(b) both the length and radius of the wire are doubled
382. An immersion heater is rated 418 W. It should heat
(c) the radius of the wire is doubled
a litre of water from 10°C to 30°C in nearly :
(a) 144 s (b) 100 s (d) the length of the wire is doubled. [IIT 1980]
(c) 200 s (d) 400 s. 389. If 2.2 kW power is transmitted through a 10 Ω line at
22,000 V, the power loss in the form of heat will be
383. The maximum power rating of a 20 Ω resistor is
21 kW. The resistor is connected directly across a (a) 0.1 W (b) 1 W
300 V DC source. Now, (c) 10 W (d) 100 W. [MP PMT 1998]
(a) the resistor will be damaged 390. A house is served by a 220 V supply line. In a circuit
(b) the resistor will not be damaged protected by a fuse marked “9 ampere”, the maxi-
(c) the resistor will develop a power of 9 kW mum number of 60 W bulbs that can be safely con-
(d) none of the above.
nected in parallel is
(a) 11 (b) 22
384. An electric kettle has two coils. When one coil is con-
(c) 33 (d) 44.
nected to the AC mains, water in the kettle boils in
CURRENT ELECTRICITY 167
391. Let H1 be the heat generated per second in the fila- 397. Three equal resistances, each of 10 ohm, are con-
ment of 100 W, 250 V lamp and H2 be that in fila- nected as shown in Fig. 153. The maximum power
ment of 200 W, 250 V lamp. Which of the following is consumed by each resistance is 20 W. What is the
correct ? maximum power that can be consumed by the com-
(a) H1 = 2H2 (b) H1 = 4H2 bination ?
(c) 2H1 = H2 (d) 4H1 = H2.
[Pb. PMT 1995]
392. Two bulbs of 500 W and 300 W are manufactured to
operate on a 220 V line. If their resistances are R1
R1
and R2 respectively, the value of is
R2
Fig. 153
5 3
(a) (b)
3 5 (a) 5 W (b) 15 W
25 9 (c) 30 W (d) 60 W.
(c) (d) . [MP PET 1999]
9 25 [National Standard Exam. in Physics 2003]
393. The two head-lamps of a car are in parallel. They 398. The three resistances of equal value are arranged in
together consume 48 W with the help of a 6 V bat- the different combinations shown below. Arrange
tery. The resistance of each bulb is them in increasing order of power dissipation.
(a) 0.67 Ω (b) 3.0 Ω
i i
(c) 4.0 Ω (d) 1.5 Ω. (I) (II)
394. An electric kettle has two coils. When one coil is con-
nected to the A.C. mains, the water in the kettle boils
in 60 minutes. When the other coil is used, the same i i
quantity of water takes 40 minutes to boil. How long (III) (IV)
will it take for the same quantity of water to boil if
two coils are connected in parallel ?
(a) 24 min (b) 12 min Fig. 154
(c) 5 min (d) 1 min. (a) III < II < IV < I (b) II < III < IV < I
395. Three bulbs B1 , B2 (c) I < IV < III < II (d) I < III < II < IV
and B 3 are con- [IIT Screening 2003]
nected to the 399. Two 220 volt, 100 watt bulbs are connected first in
mains as shown in series and then in parallel. Each time the combina-
Fig. 152. How will tion is connected to a 220 volt ac supply line. The
the brightness of power drawn by the combination in each case respec-
bulb B1 be affected tively will be
if B2 or B3 are dis- (a) 50 watt, 100 watt (b) 100 watt, 50 watt
connected from the Fig. 152 (c) 200 watt, 150 watt (d) 50 watt, 200 watt
circuit ?
[All India PM/PD 2003]
(a) Bulb B1 become brighter.
(b) Bulb B1 become dimmer. 400. An electric kettle has two heating coils. When one of
(c) No change occurs in the brightness. the coils is connected to an a.c. source, the water in
the kettle boils in 10 minute. When the other coil is
(d) Bulb B1 becomes brighter if bulb B2 is disconnected and
dimmer if bulb B3 is disconnected. used the water boils in 40 minute. If both the coils
396. Two bulbs of wattage 25 and 100 respectively, each are connected in parallel, the time taken by the same
rated at 220 volt, are connected in series with the quantity of water to boil will be
supply of 440 volt. Which bulb will fuse ? (a) 8 minute (b) 4 minute
(a) 100 watt bulb (b) 25 watt bulb (c) 25 minute (d) 15 minute
(c) None of them (d) Both of them.
[All India PM/PD 2003]
[MNR 1988]
168 COMPREHENSIVE OBJECTIVE PHYSICS

401. Which of the plots shown in Fig. 155 may represent power in R in circuit (i)
the thermal energy produced in a resistor in a given What is the ratio ?
power in R in circuit (ii)
time as a function of the electric current ?
(a) 9.0 (b) 7.2
(c) 5.4 (d) 3.0
(e) 1.8.
406. Two identical batteries, each of emf 2 V and internal
resistance 1 Ω, are available to produce heat in a
resistance R = 0.5 Ω by passing a current through it.
The maximum Joulean power that can be developed
across R using these batteries is
(a) 1.28 W (b) 2.00 W
8
(c) W (d) 3.2 W.
9
Fig. 155 407. Two wires of the same material and having same
(a) a (b) b uniform area of cross-section are connected in series
(c) c [MP PMT 1999]
(d) d. in an electrical circuit. The masses of the wires are
m and 2m. When a current I flows in the circuit, the
402. When three identical bulbs of 60 watt, 200 volt rat-
heats produced by them in a given time are in ratio
ing are connected in series to a 200 volt supply, the
(a) 2 : 1 (b) 1 : 2
power drawn by them will be
(a) 180 watt (b) 10 watt (c) 4 : 1 (d) 1 : 4.
(c) 20 watt (d) 60 watt 408. The main supply voltage to a room is 120 V. The
[All India PM/PD 2004] resistance of the lead wires is 6 Ω. A 60 W bulb is
already giving light. What is the decrease in voltage
403. In India electricity is supplied for domestic use at
across the bulb when a 240 W heater is switched on ?
220 V. It is supplied at 110 V in USA. If the resist-
(a) no change (b) 10 V
ance of a 60 W bulb for use in India is R, the resist-
(c) 20 V (d) more than 10 V.
ance of a 60 W bulb for use in USA will be
(a) 2 R (b) R /4 409. A battery of 10 cells is being charged with a power
(c) R /2 (d) R supply of 150 V. The charging current is 10 A . If the
[All India PM/PD 2004] emf of each cell is 2 V, the internal resistance of each
cell is 0.1 Ω , then the power supplied by the charger
404. A battery is charged at a potential of 15 V for 8 hour
when the current flowing is 10 A. The battery on is
discharge supplies a current of 5 A for 15 hour. The (a) 10 W (b) 20 W
mean terminal voltage during discharge is 14 V. The (c) 150 W (d) 1500 W.
“Watt-hour” efficiency of the battery is 410. A 250 V and 1000 W electric heater and five 250 V
(a) 80% (b) 90% and 100 W bulbs are connected in parallel. On con-
(c) 87.5% (d) 82.5%
necting this combination with 250 V power supply,
[All India PM/PD 2004] the current from the mains will be
405. Three identical cells, each having an emf of 1.5 V (a) 0.5 A (b) 5 A
and a constant internal resistance of 2.0 Ω, are con-
nected in series with a 4.0 Ω resistor R, firstly as in (c) 6 A (d) 15 A.
circuit (i), and secondly as in circuit (ii). 411. The resistors P, Q and R in the circuit have equal
resistance.

Fig. 156 Fig. 157


CURRENT ELECTRICITY 169
The battery, of negligible internal resistance, supplies 418. A battery supplies 150 W and 196 W power to two
a total power of 12 W. What is the power dissipated resistors of 6 Ω and 4 Ω when they are connected
by heating in resistor R ? separately to it. The internal resistance of the bat-
(a) 2 W (b) 4 W tery is
(c) 3 W (d) 6 W. (a) 0.5 Ω (b) 1 Ω
412. Two wires A and B of same material and mass have (c) 2 Ω (d) 2.5 Ω.
their lengths in the ratio 1 : 2. On connecting them, [Karnataka CET 2001]
one at a time, to the same source of potential, the 419. The resistance of a certain circuit element is directly
rate of heat dissipation in B is observed to be 5 W. proportional to the current passing through it. When
The rate of heat dissipation in A is the current is 1.0 A , the power dissipated in the ele-
(a) 5 W (b) 10 W ment is 6.0 W. What is the power dissipated when
(c) 15 W (d) 20 W. the current is raised to 2.0 A ?
(a) 3.0 W (b) 24 W
413. Ten identical electric bulbs, each rated 220 V, 50 W
are used in parallel on 220 V line for 10 hours per (c) 6.0 W (d) 48 W
day in a month of 30 days. Then the electrical energy (e) 12 W.
consumed in kilowatt hour is 420. A condenser having a capacity 2.0 µF is charged to
(a) 15 (b) 150 200 V and then the plates of the capacitor are con-
nected to a resistance wire. The heat produced in
(c) 1500 (d) 15000. [CPMT 1991]
joule will be
414. Two filaments of same length are connected first in (a) 2 × 10 –2 J (b) 4 × 10 –2 J
series and then in parallel. For the same amount of
(c) 4 × 104 J (d) 4 × 1010 J.
main current flowing, the ratio of the heat produced
is [Karnataka 1992]
(a) 2 : 1 (b) 1 : 2 421. In the circuit shown in Fig. 158, the heat produced
(c) 4 : 1 (d) 1 : 4. [MP PMT 1999] by the 6 Ω resistance is 60 cal s–1. What heat per
second is produced across 3 Ω resistance ?
415. An electrical source with internal resistance r is used
2W 3W
to operate a lamp of resistance R. What fraction of
the total power is delivered to the lamp ?
R+r R−r
(a) (b)
R R
6W 4W
R R
(c) (d)
R+r r Fig. 158
r (a) 30 cal (b) 60 cal
(e) .
R
(c) 100 cal (d) 120 cal. [MP PMT 1996]
416. A fuse wire with a radius of 1 mm blows at 1.5 A. If
422. A torch bulb rated at 4.5 W, 1.5 V is connected as
the fuse wire of the same material should blow at
shown in Fig. 159. The emf of the cell needed to make
3.0 A, the radius of the wire must be
the bulb glow at full intensity is
(a) 41/3 mm (b) 2 mm
(c) 0.5 mm (d) 8.0 mm.
[Karnataka CET 2003, 1991]
417. A heater boils 1 kg of water in time t1 and another
heater boils the same water in time t2 . If both are
connected in series, the combination will boil the same
water in time
t1t2 t1t2
(a) (b)
t1 + t2 t1 − t2
(c) t1 + t2 (d) 2 (t1 + t2).
[JIPMER 1998]
Fig. 159
170 COMPREHENSIVE OBJECTIVE PHYSICS

(a) 4.5 V (b) 1.5 V 429. Two 1000 W heaters when connected in parallel across
(c) 2.67 V (d) 13.5 V. [MP PMT 1999] 220 V supply produce heat Q1 in time t. If they are
connected in series across the same power supply,
423. A 25 W, 220 V bulb and a 100 W, 220 V bulb are
joined in series and connected to the mains. Which the heat produced in the same time is Q2. What is
bulb will glow brighter ? Q1/Q2 ?
(a) 0.25 (b) 0.5
(a) 25 W bulb (b) 100 W bulb
(c) 2 (d) 4.
(c) first 25 W bulb and then 100 W bulb
430. Four equal resistors, when connected in series, dis-
(d) both will glow with same brightness. [MP PMT 1999] sipate 5 W power. If they are connected in parallel,
424. A generator produces 100 kW of power at a potential the power dissipated will be
difference of 10 kV. The power is transmitted through (a) 80 W (b) 60 W
cables of total resistance 5 Ω. What is the power loss (c) 40 W (d) 20 W.
in the cables ? 431. Two resistors 2R and R are connected in series in an
(a) 50 W (b) 250 W electric circuit. The ratio of Joule heat in 2R to that
(c) 500 W (d) 1,000 W in R is
(a) 1 : 2 (b) 1 : 9
(e) 50,000 W.
(c) 2 : 1 (d) 4 : 1.
425. Fig. 160 shows a network of three resistances. When
432. Two resistors having equal resistances are joined in
some potential difference is applied across the net-
series and a current is passed through the combina-
work, the thermal powers dissipated by A , B and C
tion. Neglect any variation in resistance as the tem-
are in the ratio
perature changes. In a given time interval,
(a) equal amounts of thermal energy must be produced in the
resistors
(b) unequal amounts of thermal energy may be produced
(c) the temperature must rise equally in the resistors
(d) the temperature must rise unequally in the resistors.
[MP PMT 1999]
Fig. 160
433. In one ordinary electric heater, if the length of the
(a) 2 : 3 : 4 (b) 2 : 4 : 3 coil is reduced to half, a given quantity of water will
(c) 4 : 2 : 3 (d) 3 : 2 : 4. [MGIMS 1998] boil in
(a) more time (b) less time
426. A 220 V and 700 W electric kettle and four 220 V and
(c) same time
100 W bulbs are connected in parallel. On connecting
(d) a time which depends upon the resistivity of the wire.
this combination with 220 V electric supply, the total
[Karnataka 1993]
current will be
434. An electric bulb is designed to draw P 0 power at
(a) 5.5 A (b) 6.9 A
V0 voltage. If the voltage is V, it draws P power. Then,
(c) 5.0 A (d) 0.15 A.
V0 V
(a) P = P0 (b) P = P0
427. Water boils in an electric kettle in 15 minute after V V0
switching on. If the length of the heating wire is de-
F V I2 P FG V0 IJ 2 P0 .
creased to 2/3 of its initial value, then the same
amount of water will boil with the same supply volt-
(c) P =
GH V0 JK 0 (d) P =
HVK
age in [Karnataka CET 2001]
(a) 8 minute (b) 10 minute 435. A 1000 W immersion heater is used to boil 1 litre of
(c) 12 minute (d) 15 minute. water at 20°C. How long will it take to boil water if
428. An electric heater of resistance 6 Ω is run for 10 60% of the available energy is radiated to surround-
minute on a 120 V line. The energy liberated in this ings ?
period of time is (a) 14 minute (b) 16 minute
(a) 7.2 × 103 J (b) 14.4 × 105 J (c) 18 minute (d) 20 minute.
(c) 43.2 × 104 J (d) 28.8 × 104 J.
CURRENT ELECTRICITY 171

436. The same mass of copper is drawn into two wires 444. By means of electric immersion heater connected to
3 mm and 5 mm thick. The two wires are connected 240 V mains, it is required to heat 100 litre of water
in series and current is passed through them. The from 20°C to 70°C in 2 hours. Assuming no heat loss,
ratio of heats produced in the two wires is the current necessary to effect this is
(a) 12/13 (b) 21/8 (a) 6.1 A (b) 12.2 A
(c) 19/7 (d) 625/81. (c) 18.4 A (d) 28.4 A.
437. A house is served by a 220 V supply line. In a circuit 445. A given resistor cannot carry currents exceeding 20 A,
protected by a fuse marked 10 A, the maximum without exceeding its maximum power dissipation
number of 100 W lamps in parallel that can be turned ratings. By forced air cooling suppose that we in-
on is crease the rate at which heat can be carried away by
(a) 66 (b) 33 a factor of 3. Now the maximum current that the
(c) 22 (d) 11. resistor can carry is
438. It takes 16 minute to boil some water in an electric
(a) 20 / 3 A (b) 20 A
kettle. Due to some defect, it becomes necessary to
remove 10% turns of heating coil of the kettle. After (c) 20 3 A (d) 60 A.
repairs, how much time will it take to boil the same 446. The effective wattage of 100 W, 60 W and 40 W lamps
mass of water ? connected in series, is equal to
(a) 13.7 minute (b) 14.4 minute
(a) 200 W (b) 124 W
(c) 17.7 minute (d) 20.9 minute.
(c) 120 W (d) 600/31 W.
439. Two electric bulbs whose resistances are in the ratio
of 1 : 3 are connected in series. The powers dissi- 447. Three equal resistors are connected as shown in
pated in them have the ratio Fig. 161. The maximum power consumed by each re-
(a) 1 : 1 (b) 1 : 3 sistor is 18 W. Then maximum power consumed by
(c) 1 : 6 (d) 3 : 1. the combination is
440. A heater coil is cut into two parts of equal length and
only one of them is used in the heater. The ratio of
the heat produced by this half-coil to that by the origi-
nal coil is
(a) 4 : 1 (b) 1 : 4 Fig. 161
(c) 1 : 2 (d) 2 : 1.
(a) 18 W (b) 27 W
441. An electric kettle, rated 2.1 kW, contains 1 kg of wa-
(c) 36 W (d) 54 W.
ter. Assuming no heat loss and neglecting the ther-
mal capacities of the kettle, how long will it take to 448. A 100-W bulb and a 25-W bulb are designed for the
start to boil the water if its initial temperature is same voltage. They have filaments of the same length
and material. The ratio of the diameter of the 100-W
20°C ?
bulb to that of the 25-W bulb is
(a) 200 s (b) 160 s
(a) 4 : 1 (b) 2 : 1
(c) 120 s (d) 80 s.
442. You have three appliances, each of 500 W, running (c) 2 : 1 (d) 1 : 2. [AMU 1997]
on 220 V a.c. : (i) an electric lamp, (ii) an electric iron 449. The resistors P, Q and R in the circuit (Fig. 162)
(iii) an electric room heater. The electrical resistance have equal resistance. If the battery is supplying a
total power of 12 W, what is the power dissipated as
is
heat in resistor R ?
(a) maximum for electric lamp
(b) maximum for the heater
(c) maximum for electric iron
(d) same for all.
443. How long will it take to heat 2 litre of water from
25°C to 100°C in an electric kettle taking 5 A from a
220 V supply ?
(a) 2078 s (b) 136 s
(c) 573 s (d) 32 s. Fig. 162
172 COMPREHENSIVE OBJECTIVE PHYSICS

(a) 2 W (b) 6 W 455. A copper refining cell consists of two parallel copper
(c) 3 W (d) 8 W plate electrodes 5 cm apart and 1m square, immersed
(e) 4 W. in a copper sulphate solution of resistivity 1.2 × 10 –2
450. All bulbs in fig. 163 are identical. Which bulb lights ohm metre. Then the potential difference which must
more brightly ? be established between the plates to provide a con-
stant current to deposit 0.66 kg of Cu on cathode in
one hour is nearly : Given : Z = 3.3 × 10 –7 kg C –1.
(a) 0.61 V (b) 0.33 V
(c) 33 V (d) 1.5 V.
456. The thermo emf of a thermocouple is 25 µV/°C at
room temperature. A galvanometer of 40 ohm resist-
ance, capable of detecting current as low as 10–5 A, is
connected with the thermocouple. The smallest tem-
perature difference that can be detected by this sys-
Fig. 163
tem is
(a) 1 (b) 2 (a) 20° C (b) 16° C
(c) 3 (d) 4.
(c) 12° C (d) 8° C [AIEEE 2003]
[Haryana PMT 1997]
457. An ammeter in series with a silver voltameter reads
0.81 A. If the weight of Ag deposited in 20 minutes is
CHEMICAL AND THERMOELECTRIC 1.071 g, the error in the reading of the ammeter is
EFFECTS OF CURRENT Given : ECE of Ag = 0.001118 gC –1.
(a) + 0.012 A (b) – 0.012 A
451. A dry cell of emf 1.5 V and internal resistance 0.10 Ω
(c) + 0.12 A (d) – 0.12 A.
is connected across a resistor in series with a very
low resistance ammeter. When the circuit is switched 458. The thermo emf produced in a thermocouple is
3 microvolt per degree centigrade. If the temperature
on, the ammeter reading settles to a steady value of
2.0 A. The steady rate of (i) chemical energy con- of the cold junction is 20°C and the thermo emf is
0.3 milli volt, the temperature of the hot junction is
sumption of the cell (ii) energy dissipation inside the
cell (iii) energy dissipation inside the resistor (iv) (a) 80° C (b) 100° C
power output of source is (c) 120° C (d) 140° C [EAMCET 2003]
(a) (i) 3W (ii) 0.4 W (iii) 2.6 W (iv) 2.6 W 459. The thermo emf of a thermocouple varies with the
(b) (i) 0.4 W (ii) 3 W (iii) 2.6 W (iv) 2.6 W temperature θ of the hot junction as E = aθ + bθ2 in
(c) (i) 2.6 W (ii) 0.4 W (iii) 9 W (iv) 1 W volt where the ratio a/b is 700°C. If the cold junction
(d) None of these. is kept at 0°C, then the neutral temperature is
452. If 100 kWh of energy is consumed at 33 V in a copper (a) 1400°C
voltameter, what is the mass of copper liberated ? (b) 350°C
Given : ECE of Cu = 0.33 × 10 –6 kg C –1. (c) 700°C
(a) 3.6 kg (b) 3.3 kg
(d) No neutral temperature is possible for this thermocouple.
(c) 1 kg (d) 1 m g.
[AIEEE 2004]
453. A Leclanche cell supplies a current of 1 A for
10 minute. ECE of hydrogen = 104 × 10 –7 g C –1. Mass 460. A silver voltameter of resistance 2 ohms and a 3 ohm
of hydrogen liberated is resistor are connected in series across a cell. If a re-
(a) 624 × 10 – 5 g (b) 1248 × 10 –5 g sistance of 2 ohm is connected in parallel with the
(c) 1872 × 10 –5 g (d) 2496 × 10 –4 g. voltameter, then the rate of deposition of silver
454. A Daniel cell has an internal resistance of 1.2 Ω. The (a) decreases by 25% (b) increases by 25%
ratio of amounts of heat produced in the cell for each (c) increases by 37.5% (d) decreases by 37.5%.
gram of Zn consumed in the short-circuit to that [Haryana PMT 1996]
when the external resistance of 1.2 Ω is connected in 461. A Leclanche cell supplies a current of one ampere for
circuit is one hour. Atomic weight of
(a) 2 (b) 1
Manganese = 55 ; Oxygen = 16 ; Zinc = 65
(c) 0.5 (d) 0.25.
CURRENT ELECTRICITY 173

and electrochemical equivalent of hydrogen (a) 8 (b) 10


= 0.0000104 g per coulomb. Then the mass of hydro- (c) 20 (d) 25.
gen liberated is 469. The neutral temperature of copper-iron thermocou-
ple is 270°C. If the temperature of the cold junction
(a) 0.03744 g (b) 1.217 g
is 20°C, then the temperature of inversion will be
(c) 3.258 g (d) indeterminate.
(a) 540°C (b) 520°C
[All India PM/PD 1995]
(c) 490°C (d) 500°C.
462. The atomic mass number of zinc is 66 and its va-
470. A cold-water pipe and a hot-water pipe are both made
lency is 2. How much mass of zinc will be liberated
of copper and are initially electrically isolated. In
by 9.65 A of current in 10.0 second ?
which one of the following arrangements will the
(a) 33 kg (b) 33 g
galvanometer indicate a thermo-electric current ?
(c) 33 mg (d) 33 µg. [MANIPAL 1995]
463. Silver and copper voltameters are connected in par-
allel to a 12 V battery of negligible internal resist-
ance. In 30 minute, 1 g of silver and 1.8 g of copper
are deposited. The rate at which the battery delivers
energy is nearly : (electrochemical equivalent of cop-
per = 6.6 × 10 –4 gC –1 and electrochemical equivalent
of silver = 11.2 × 10 –4 gC –1)
(a) 24 J (b) 2.4 J
(c) 0.24 J (d) 240 J.
[Himachal PMT 1997]
464. A current of 16 ampere flows through molten NaCl
for 10 minute. The amount of metallic sodium that
appears at the negative electrode would be nearly :
(assuming Faraday constant = 9.65 × 104 C/gram
equivalent)
(a) 0.23 g (b) 1.15 g
(c) 2.3 g (d) 11.5 g. [MNR 1995]
465. On passing 96500 coulomb of charge through a
CuSO4 solution, the amount of copper liberated is
(a) 64 g (b) 32 g
(c) 32 kg (d) 64 kg. [MP PMT 2001]
466. Consider the following two statements.
(A) Free-electron density in a metal depends on tem-
perature. Fig. 164
(B) Free-electron density is different in different met- 471. Consider the following two statements.
als. (A) Free-electron density is different in different met-
Thomson effect is caused als.
(a) due to both A and B (b) due to A but not due to B (B) Free-electron density in a metal depends on tem-
(c) due to B but not due to A perature.
(d) neither due to A nor due to B. Seebeck effect is caused
[All India PM/PD 1994] (a) due to both A and B (b) due to A but not due to B
467. How much current should pass through acidulated (c) due to B but not due to A
water for 100 s to liberate 0.224 litre of hydrogen ? (d) neither due to A nor due to B. [JIPMER 1997]
(a) 22.4 A (b) 19.3 A 472. In a thermocouple, minimum current flows at
(c) 9.65 A (d) 1 A. [CMC LDH 1995] (a) neutral temperature (b) temperature of inversion
468. If the temperature of the hot junction of a thermo- (c) twice the neutral temperature
couple changes from 80°C to 100°C, then the percent- (d) twice the temperature of inversion. [TNPCEE 1998]
age change in thermoelectric power is
174 COMPREHENSIVE OBJECTIVE PHYSICS

473. The unit of Peltier coefficient is 478. Two similar thermocouples, made of dissimilar met-
(a) JC–1 (b) JA–1 als A and B are connected as shown in Fig. 165
(c) JV–1 (d) None of these. through a key K and a sensitive galvanometer G.
[RPMT 1998] One of the thermocouples is dipped in a hot bath
maintained at temperature t2 and the other in a cold
474. The unit of Thomson coefficient is
bath at temperature t 1. When the key is pressed, a
(a) joule per coulomb (b) joule per volt
deflection is seen in the galvanometer because
(c) joule per watt (d) joule per coulomb per K.
[AIIMS 1997]
475. A thermocouple is connected across a galvanometer
of resistance 30 Ω. One junction is immersed in wa-
ter at 373 K and the other in ice at 273 K. The emf of
the thermocouple is 90 µV for each 1 K difference in
temperature between the junctions, and the thermo-
couple resistance is 6 Ω.
What current will flow in the galvanometer ?
(a) 1.8 µA (b) 250 µA
(c) 300 µA (d) 1.5 mA
(e) 1.8 mA.
476. Consider the following statements regarding a ther-
mocouple.
(A) The neutral temperature does not depend on the
Fig. 165
temperature of the cold junction.
(B) The inversion temperature does not depend on (a) An emf of the order of a few microvolt is generated which
the temperature of the cold junction. is proportional to (t2 – t1)
(a) Both A and B are correct. (b) An emf is generated the value of which will depend upon
the temperature of the hot bath only
(b) A is correct but B is wrong.
(c) B is correct but A is wrong. (c) An emf of about one volt is generated which will be propor-
tional to (t2 – t1)
(d) Both A and B are wrong. [DPMT 1999]
(d) An emf of a few microvolt is generated which will be pro-
477. A thermocouple produces a thermo emf of 40 µV/K.
portional to t2 only. [AIIMS 1996]
A thermopile is made using 150 such thermocouples.
The temperature of the two junctions is 20°C and 479. When a current passes through a wire whose parts
40°C. The thermo emf generated in the thermopile is are maintained at different temperatures, evolution
(a) 12 mV (b) 1.2 mV or absorption of heat all along the length of wire is
known as
(c) 120 mV (d) 0.24 V.
(a) Joule effect (b) Seebeck effect
[Himachal PMT 1996]
(c) Peltier effect (d) Thomson effect.
[AFMC 1996]

Answers (Set II)


156. (c) 157. (d) 158. (c) 159. (a) 160. (b) 161. (d) 162. (b) 163. (b)
164. (a) 165. (c) 166. (b) 167. (c) 168. (d) 169. (a) 170. (b) 171. (b)
172. (a) 173. (c) 174. (a) 175. (d) 176. (c) 177. (d) 178. (d) 179. (a)
180. (d) 181. (b) 182. (b) 183. (a) 184. (d) 185. (c) 186. (b) 187. (c)
188. (c) 189. (d) 190. (d) 191. (c) 192. (a) 193. (c) 194. (b) 195. (c)
196. (a) 197. (c) 198. (b) 199. (d) 200. (a) 201. (c) 202. (b) 203. (c)
204. (a) 205. (c) 206. (a) 207. (b) 208. (c) 209. (b) 210. (c) 211. (d)
CURRENT ELECTRICITY 175

212. (a) 213. (d) 214. (c) 215. (a) 216. (b) 217. (c) 218. (a) 219. (c)
220. (b) 221. (b) 222. (d) 223. (a) 224. (d) 225. (c) 226. (c) 227. (c)
228. (b) 229. (c) 230. (b) 231. (a) 232. (a) 233. (c) 234. (d) 235. (a)
236. (a) 237. (d) 238. (d) 239. (c) 240. (a) 241. (b) 242. (d) 243. (d)
244. (d) 245. (a) 246. (a) 247. (d) 248. (d) 249. (a) 250. (d) 251. (a)
252. (a) 253. (a) 254. (c) 255. (b) 256. (c) 257. (a) 258. (a) 259. (e)
260. (b) 261. (b) 262. (c) 263. (c) 264. (d) 265. (a) 266. (d) 267. (a)
268. (b) 269. (b) 270. (a) 271. (a) 272. (d) 273. (a) 274. (a) 275. (b)
276. (a) 277. (d) 278. (a) 279. (b) 280. (b) 281. (d) 282. (d) 283. (c)
284. (b) 285. (a) 286. (b) 287. (b) 288. (d) 289. (c) 290. (c) 291. (b)
292. (b) 293. (c) 294. (b) 295. (a) 296. (d) 297. (a) 298. (d) 299. (d)
300. (a) 301. (a) 302. (d) 303. (a) 304. (d) 305. (b) 306. (a) 307. (d)
308. (a) 309. (a) 310. (c) 311. (b) 312. (b) 313. (c) 314. (a) 315. (b)
316. (c) 317. (b) 318. (d) 319. (a) 320. (c) 321. (d) 322. (b) 323. (a)
324. (d) 325. (b) 326. (c) 327. (d) 328. (e) 329. (c) 330. (d) 331. (a)
332. (b) 333. (d) 334. (a) 335. (c) 336. (b) 337. (d) 338. (b) 339. (a)
340. (b) 341. (a) 342. (b) 343. (c) 344. (a) 345. (d) 346. (a) 347. (d)
348. (a) 349. (a) 350. (d) 351. (c) 352. (a) 353. (b) 354. (a) 355. (c)
356. (a) 357. (b) 358. (a) 359. (b) 360. (c) 361. (c) 362. (b) 363. (b)
364. (b) 365. (b) 366. (a) 367. (d) 368. (b) 369. (d) 370. (c) 371. (c)
372. (c) 373. (b) 374. (b) 375. (a) 376. (e) 377. (a) 378. (c) 379. (b)
380. (a) 381. (b) 382. (c) 383. (b) 384. (c) 385. (c) 386. (d) 387. (c)
388. (b) 389. (a) 390. (c) 391. (c) 392. (b) 393. (d) 394. (a) 395. (b)
396. (b) 397. (c) 398. (a) 399. (d) 400. (a) 401. (d) 402. (c) 403. (b)
404. (c) 405. (a) 406. (b) 407. (b) 408. (d) 409. (d) 410. (c) 411. (a)
412. (d) 413. (b) 414. (c) 415. (c) 416. (a) 417. (c) 418. (b) 419. (d)
420. (b) 421. (d) 422. (d) 423. (a) 424. (c) 425. (c) 426. (c) 427. (b)
428. (b) 429. (d) 430. (a) 431. (c) 432. (a) 433. (b) 434. (c) 435. (a)
436. (d) 437. (c) 438. (b) 439. (b) 440. (d) 441. (b) 442. (d) 443. (c)
444. (b) 445. (c) 446. (d) 447. (b) 448. (b) 449. (a) 450. (a) 451. (a)
452. (a) 453. (a) 454. (a) 455. (b) 456. (b) 457. (a) 458. (c) 459. (d)
460. (b) 461. (a) 462. (c) 463. (a) 464. (c) 465. (b) 466. (b) 467. (c)
468. (d) 469. (b) 470. (e) 471. (a) 472. (b) 473. (a) 474. (d) 475. (b)
476. (b) 477. (c) 478. (a) 479. (d)
176 COMPREHENSIVE OBJECTIVE PHYSICS

Solutions (Set II)


12.8 1 R (R + R0 )
156. = or n2 = 12.8 × 5 = 64 or n = 8
n2 5 162. R0 = R +
2R + R0
157. Rt = R0 (1 + αt)
1 = R0 [1 + 125 × 10–5 × 27]
2 = R0 [1 + 125 × 10–5 × t]
1 + 125 × 10 −5 × t
Dividing, 2 =
1 + 125 × 10 −5 × 27
2.0675 = 1 + 125 × 10–5 t
Fig. 168
1.0675 106750
or t= −5
= = 854
125 × 10 125
T = (854 + 273) K = 1127 K.
158. Effective radius, r = r1r2
Fig. 169
or (R0 – R) (2R + R0) = R2 + RR0
r2
or 2RR0 – 2R2 + R02 – RR0 = R2 + RR0
r1
or 3R2 = R02
R0
or R=
3
nE n × 2.1
Fig. 166
163. I= or 1.5 =
R + nr 6 + n × 0.20
159. E = 2 V, r = 0 1 9
9 + 1.5n × = 2.1n or 9 = 1.8n or n = =5
5 1.8
q
164. I =
T
2π 2π
But ω= or T =
T ω

I=

Fig. 167
1 1 1 1 1 1
165. = + or = −
2 2 2.1 R R 2 2.1
Current, I= A
10 1 0.1 4.2
I = 0.2 A or = or R = or R = 42 Ω
R 2 × 2.1 0.1
V3 = 0.2 × 3 V = 0.6 V
160. Potential difference VP across P as determined from Required length = 42 m = 2 m
21
E1 is given
166. Think in terms of two Wheatstone Bridges connected
by VP = FG E IJ P
1
in parallel.
H P + QK .
167. R=ρ
l×4
Potential VP across P as determined from E2 is same πd 2
as E2 , because no current is drawn i.e., VP = E2. l
For minimum resistance, 2 should be minimum.
FG P IJ E2 P
l
d
2l
Therefore, E2 = E1
H P + QK =⇒
E1 P + Q
.
(a) 2
d
(b) 2
d
161. In (a), (b) and (c), the condition for balanced l 2l × 4
Wheatstone Bridge is satisfied. However this condition (c) (d)
is not satisfied if R1 and R2 are interchanged. 2 × 4d2 d2
Clearly (c) is the right choice.
CURRENT ELECTRICITY 177
168. Using symmetry considerations, the given network 90
can be bisected along the line YY′ as shown. Again, VD = V – × 4 or VD = V – 1.8
200
Y
Now, VB – VD = (V – 1.6) – (V – 1.8)
= (1.8 – 1.6) volt = 0.2 volt.
1W
172. The equivalent of the given network is as under :
1W 2
1W

A B

1W

Fig. 170 Fig. 171

An equivalent of the left segment is shown in


Fig. 171.
The equivalent of the above network is as shown in
Fig. 172. Fig. 173
4W The equivalent of the above network is as under :
3

1W
Fig. 172

4
34 Fig. 174
∴ Rp = = Ω
4
+1 7 The equivalent of the above network is a parallel com-
3
bination of 3 Ω, 4 Ω and 6 Ω
Since the two segments are in series therefore the
4 8 1 1 1 1
combined resistance is 2 × Ω or Ω. = + +
7 7 R 3 4 6
169. V = E – Ir1, V = 0
1 8+6+4 24 4
∴ E = Ir1 = or R = Ω= Ω.
R 24 18 3
Total emf = Ir1 + Ir1 = 2Ir1
Total resistance = R + r1 + r2 173. Total emf = 2.2 volt
6
2Ir1 Total internal resistance = Ω=2 Ω
Now, I= 3
R + r1 + r2
Total resistance = 2 Ω + 2 Ω = 4 Ω
or R + r1 + r2 = 2r1
or R = 2r1 – r1 – r2 or R = r1 – r2
170. VXY = 1.5 volt
Voltage across 3 Ω = 4.5 volt
Now, 4.5 = I × 3
4.5
I= A = 1.5 A
3
VXY = 1.5 RXY 2Ω
1.5 = 1.5 RXY or RXY = 1 Ω
Fig. 175
40
171. VB = V – ×4 or VB = V – 1.6 2.2 11
100 Current, I = A= A = 0.55 A.
4 20
178 COMPREHENSIVE OBJECTIVE PHYSICS

E 182.
174. I =
r
4 4
8= or r = Ω = 0.5 Ω
r 8
3.2 × 10−19 × 4
175. E= volt = 8 volt
1.6 × 10 −19
E E
176. V=E– R or V = E – R
R+r R+R
E E
or V=E– or V =
2 2
177. Balanced Wheatstone Bridge.
R
178. Resistance of each part is . When two such parts Fig. 177
10 The equivalent of the above network is :
R
are connected in series, then the resistance is . Now,
5
R
five resistances, each of value , are connected in
5
parallel.
R
∴ Combined resistance is .
25 Fig. 178
179. Total resistance in the circuit
20 × 10
F 1 I
= G 4 × + 1J
Parallel combination of 20 Ω and 10 Ω gives
20 + 10
H 4 K Ω=2 Ω
200
Total emf = 1.5 × 2 V = 3 V Ω i.e., Ω.
30
3V 200 20 80
Current, I = = 1.5 A Now, R = 10 + + 10 = 20 + = Ω = 26.7 Ω.
2Ω 30 3 3
180. Voltage across 5 Ω = 10 V 183. E ∝ 125
10 V ∝ 100
∴ I=
A =2A
5 E 125 5 I (R + r) 5
4×5 29 = = , =
181. Total resistance = + 1= Ω V 100 4 IR 4
4+5 9
2+r 5
10 × 9 90 = or 8 + 4r = 10 or 4r = 2
Current, I = Ω= A 2 4
29 29
1
4 × 90 40 or r= Ω = 0.5 Ω
Current through 5 Ω = A= A 2
29 × 9 29
184. emf = 6 V ; Total resistance = 6 Ω
I1 R 2 I1 R2 6
 = or = I= A=1A
I2 R 1 I R1 + R2 6
For direction of current, look for the direction of emf
of cell of 10 V.
E
185. I=
nr + r
E
I=
(n + 1) r

Fig. 176
V = E – Ir Fig. 179
CURRENT ELECTRICITY 179

E l
V=E– r R = (ρ1 + ρ2 )
nr + r A

LM
V = E 1−
1 OP or
V
=
n Again,
2l
R=ρ
N n+1 Q E n+1 A
Comparing, 2ρ = ρ1 + ρ2
186. For resistors in series connection, current (I) is the
same through the resistors. In other words, ratio of 1
or ρ = (ρ1 + ρ2 )
the voltage drop across each resistor with its resist- 2
ance is the same. That is, 193. V = E – Ir1
5−3 3−2 2 2E
I= = = or V = E− r
R1 R2 R3 r1 + r2 + R 1
i.e. R1 : R2 : R3 = 2 : 1 : 2. But V = 0 (given)
187. Resistance is halved. Current is doubled.
2Er1
R 30 ∴ E=
188. = = 3 or R = 6 Ω. r1 + r2 + R
2 10
or r1 + r2 + R = 2r1 or R = r1 – r2
l
189. R=ρ nE E
πd 2 194. =
R + nr R + r
4 n
When d is halved, l will be quadrupled. nE nE
or =
So, R is increased by a factor of 16. R + nr nR + r
190. Equivalent resistance between a and b is 3 Ω. Νοw, or R + nr = nR + r
the equivalent of the given network is or (n – 1) r = (n – 1)R
or r=R
6×3 18
195. Resistance, R = Ω= Ω=2Ω
6+3 9
3
I=
A = 1.5 A
2
Fig. 180 196. New length is 2l, if the original length is l. Clearly,
9×6 54 18 a
RAB = Ω= Ω= Ω = 3.6 Ω. the new cross-sectional area is , if a is the initial
9+6 15 5 2
cross-sectional area. This is because the volume of
R1R 2
191. S = R1 + R2, P = the wire has to remain constant.
R1 + R 2
2l
∵ S = n P, Now, R′ = ρ = 4R
a/2
nR1R2 Increase in resistance = 4R – R = 3R
∴ R1 + R2 =
R 1 + R2
3R
or (R1 + R2)2 = nR1R2 Percentage increase in resistance = × 100 = 300.
R
For minimum value of n, R1 = R2 197. BC, CD and BA are the known resistances. The un-
∴ (2R1)2 = nR12 known resistance is to be connected between A and D.
or n = 4
I 1 R 2 ρL 2 F IFA I
l
192. R1 = ρ1 , R2 = ρ2
l 198. =
I2 R 1
=
A2
A
× 1 =
ρL 1
L2
L1
GH JK GH A JK
1
2

3 F 2I
A A 2
I1 L 2 πr12 L2 r12
= G J
4 H 3K
R = R1 + R2 or = =
I2 L1 πr22 L1r22
l l
R = ρ1 + ρ2 3 4 1
A A = × =
4 9 3
180 COMPREHENSIVE OBJECTIVE PHYSICS

199. The given circuit is reducible to a parallel combina- I1 R2


208. =
4R 4R 2R I1 + I2 R 2 + R 1
tion of and . So, the effective resistance is .
3 3 3
I1 R2 R2
1 = or I1 = I
200. Section AB is th of the total length of circular wire. I R2 + R1 R 2 + R1
12
Resistance of segment AB is 3 Ω. Resistance of longer 60 84
segment is 33 Ω. or I1 =× 1.4 A = A=1A
84 84
Effective resistance
209. A careful analysis would show that the voltage along
33 × 3 99 11 R is 1.03 V.
= Ω= Ω= Ω = 2.75 Ω.
33 + 3 36 4 1.03 = 1 × R or R = 1.03 Ω.
X 20 100 1
201. In the first case, = 210. Current through each lamp = A= A
Y 80 200 2
4X l If n be the required number of lamps, then
In the second case, =
Y 100 − l
1 130
4 × 20 l n× = or 200 + 3n = 260
2 200
or = or l = 100 – l + 2.6 + 0.4
80 100 − l n
or 2l = 100 cm or l = 50 cm or 3n = 60 or n = 20.
202. The combined resistance for each of the following cases 211. Rs = R1 + R2 + R3
is 1 1 1 1
= + +
Cases Effective resistance Gs G 1 G 2 G3
9 1 G G + G 1G3 + G 1G2
(a) Ω = 2 3
10 Gs G1G 2G 3
5 25 G 1G 2G3
(b) = Ω or Gs =
2 10
G 1G 2 + G 2G 3 + G3G 1
21
(c) Ω l l
10 212. R= ρ ×
a l
12 24
(d) = Ω l2
5 10 R=ρ
V
E−8 m m
203. r= ×4 …(1) But d= or V =
8 V d
E−6 ρl2d
Again, r= ×2 ∴ R=
6 m
2 2 2
1
Equating,
E−8 E−6
= ∴ R1 : R2 : R3 : : 5 : 3 : 1 : : 25 : 3 :
2 3 1 3 5 5
or 3E – 24 = 2E – 12 or E = 12 volt : : 125 : 15 : 1
213. Branch containing ammeter has a potential differ-
12 − 8
Again, from n (1), r = × 4 = 2 Ω. ence of 10 V.
8
10
204. Since E2 < E1, ∴ l2 < l1. I= A = 2 A.
5
eE e V 100
205 to 207. vd = –τ or vd = – τ 214. V= × 1.41 mV = 35.25 mV
m m l 4
When V is doubled, vd is doubled. Error = (35.25 – 35) mV = 0.25 mV.
When l is doubled, vd is halved n Al
vd is not changed if radius of wire is changed. 215. Number of free electrons per unit length = = nA
l
CURRENT ELECTRICITY 181
Momentum per unit length It is equivalent to a parallel combination of 6 Ω and
I I I 6 Ω. This gives 3 Ω.
= (mvd) nA = m nA = = 221. In series combination,
neA e/m s
R s nR I is constant. ∴ V ∝ R
216. = = n2 or Rs = n2 Rp
Rp R l
n ρ
VAB R AB π a2
217. Note that one cell will have a discharging action and = =
VCA RCA l
the other will be charged. ρ
π (2a)2
20R
20 + R 20 20R VAB 4
218. = or = 10 or =
5 10 20 + R VCA 1
or 20 R = 200 + 10 R 222. Current through R2 is (2.25 – 1.5) A i.e. 0.75 A.
or 10R = 200 or R = 20 Ω. Clearly, R2 = 2 × 30 Ω = 60 Ω.
2R 2R Also, R1 = 60 Ω
219. R=2+ + 2 or R = 4 +
2+R 2+R Voltage across R1 = 2.25 × 60 volt = 135 volt
Voltage across parallel combination of 30 Ω and 60 Ω
is 45 V.
∴ E = (135 + 45) V = 180 V.
223. Current ratio, 20% : 80% i.e. 1 : 4. Resistance ratio is
4 : 1 i.e. 800 Ω : 200 Ω. Hence S = 200 Ω.
224. It is a case of balanced Wheatstone Bridge.
Fig. 181 E− V
225. r= R
2R V
or R–4=
2+R 12 − V
0.5 = 5.5
or (R– 4) (2 + R) = 2R V
or 2R – 8 + R2 – 4R = 2R or R2 – 4R – 8 = 0 1 12 − V
=
4 ± 16 + 32 4 ± 48 4±4 3 11 V
or R = = = V = 132 – 11V or 12V = 132
2 2 2
=2± 2 3 V = 11 volt.
226. V = E – Ir
Ignoring –ve sign,R=2+ 2 3
If I = 0, then V = E
R = 2 + 2 × 1.732 As I increases, V decreases.
= 2 + 3.464 = 5.464 Ω. Note that y = c – mx is the equation of a straight line.
220. The given arrangement is equivalent to the following 227. Voltage across 4 Ω = 20 V
network.
20
Current is A.
4
48
6W 6W 228. Parallel combination of 12 Ω and 4 Ω is Ω i.e. 3 Ω
16
6W
20
Now, I= A =4A
3+2
V = 20 – 4 × 2 = 12 volt
6W 6W
12
Current through 4 Ω = A = 3 A.
4
12 R
6W 229. 2= or + 0.6 = 6
R 3
+ 0.6
Fig. 182 3
182 COMPREHENSIVE OBJECTIVE PHYSICS

R 240. The reading of galvanometer is same whether switch


or = 5.4 or R = 16.2 Ω S is open or closed. So, there is no current in the di-
3
agonal arm of the bridge. So, the bridge is clearly
230. Current in each branch is 1 ampere. balanced.
VA = V – 2
P R
VB = V – 3 ∴ =
Q G
Now, VA – VB = (V – 2) – (V – 3) = + 1 volt.
Since P ≠ R, ∴ Q ≠ G
231. Parallel combination of two resistances, each of value
In the balanced position IP = IQ and IR = IG
6 Ω.
But IP ≠ IR and IQ ≠ IG
2 So, (a) is the right choice.
232. Case I. R= ρ
1 × 0.5 241. When X is connected to Y, the balance length l is
R=4ρ Ω proportional to the p.d. across the 100 Ω resistor.
ρ When X is connected to Z, the balance length is pro-
0.5
Case II. R′ = ρ or R′ = Ω portional to the p.d. across the 100 Ω resistor and
2×1 4
resistor R. Assuming that the current through the
R′ 1 R 100 Ω resistor and resistor R at balance is constant
Now, = or R′ =
R 16 16 and unchanged when X is reconnected, then it follows
234. Current-carrying capacity
100 + R
= (4.8 + 10.2 + 3.5) A = 18.5 A. that the resistor ratio is equal to the length
100
235. Loss of emf = 2E. No effect on internal resistance.
ratio.
81 242. R is of the order of 15,000 Ω. The junctions of highest
236. = 1 or n = 9
n2 two and lowest two resistances are A and C, and for
237. Suppose V is the voltage of the supply and R is the better sensitivity, the galvanometer be between these.
resistance of each bulb. So, error will decrease with the suggested interchange.
R 50 E E
then Rp = and current in ammeter, I = V = 3 V 243 & 244. I= =
3 Rp R 50 r r
provided all three bulbs are working properly. Terminal potential difference across each cell, V
If one bulb has broken down, then E
= E – Ir = E –
r=E–E=0
r
R V
Rp = and I = 2 So, potential difference between any two points will
2 R be zero.
∴ Current reading decreases and since current I 4.8
q ne
through each bulb is V/R the same as before, bright- 245. I= = or n = = = 3 × 1019
ness of bulbs is not affected. t 1 e 1.6 × 10−19
238. The given network is a parallel combination of 3 Ω I
246. vd =
and 12 Ω. neA
3 × 12 36 12 mI
∴ R= Ω = Ω= Ω Momentum of 1 electron = mvd =
3 + 12 15 5 neA
Number of electrons in length l = n × A × l
V 4.8 × 5
I= = A=2A mIl mIl
R 12 ∴ Momentum of nAl electrons = nA = .
neA e
40 1
239. vd = 247. I× = 2.5 or I = 5 A.
1029
× 1.6 × 10−19 × 10−6 2
l
40 248. R=ρ
= × 10−4 m s –1 = 2.5 × 10–3 m s–1 πr 2
1.6
= 2.5 mm s–1
R1 l r2 l r
= 12 × 2 = 1 2
LM OP 2
=
LM OP
1 3
2
=
9
R 2 r1 l2 l2 r1 N Q NQ
2 2 8
CURRENT ELECTRICITY 183

V For loop (2)


249. VBC = × l = 10 V m–1 × 30 × 10–2 m = 3 volt – 2 – 2 + 4I2 = 0
l
or 4I2 = 4 or I2 = 1 A
2 2
250. V=2– × 0.1 = 2 – × 0.1 So, current from A to B is 2 ampere.
0.1 + 3.9 4
256. E = 4 volt
= 2 – 0.05 = 1.95 volt Total resistance = (1 + 0.9 + 1.9) Ω = 3.8 Ω
251. Length doubled ; Area of cross-section halved.
4
252. Left-most unit has resistance of 3 Ω. Continuing with Now, I= ampere
3.8
the next units, we arrive at the following network.
Again, terminal potential difference across A
4
=2– × 1.9 = 2 – 2 = 0.
3.8
2 1
257. I= A= A
20 10
1 1
Now, potential difference across wire = ×5V= V
10 2
Fig. 183 Potential gradient

This is a parallel combination of 6 Ω and 3 Ω. 1


V
6×3 18 = 2 = 0.5 Vm–1 = 0.005 V cm–1
∴ R= Ω= Ω = 2 Ω. 1m
6+3 9
I 258. Ignore S3 and R2. A longer balance length corresponds
253. vd = to either V1 being smaller or V2 being larger than at
neA
2I v the start. Just assess which of the two is more likely.
v′d = =
ne (4 A) 2 259. With S3 closed and the balance point located, V2 will
24 × 12 288 deliver current through R2 and S3 only. Since V2 de-
254. R= Ω = Ω=8Ω livers current, then there will be a loss of potential
24 + 12 36
difference across its internal resistance. Will this loss
change the potential difference across the terminals
of V2 ? If the potential difference across the terminals
of V2 changes, then the balance length must change
in proportion (compared with the value when S3 was
open).

FG 5 × 15 + 1.25IJ Ω = FG 75 + 1.25IJ Ω
260. Total resistance =
H 5 + 15 K H 20 K
Fig. 184 = (3.75 + 1.25) Ω = 5 Ω
255. For loop (1) 2 + 2 – 4I1 = 0 20
I= A=4A
or 4I1 = 4 or I1 = 1 A 5
15
2V
4W
Current through 5 Ω = ×4A =3A
I1 20
1 Voltmeter reading = Potential drop across 1.25 Ω
I1 + I2 I1
= 4 × 1.25 volt = 5 volt
A B 261. Current I through the resistors is
I1 + I2
2V 3 − ( − 15)
I2 2 I2 I= mA = 0.015 mA
200 + 1000
4W I2 Potential at X is thus
2V
Vx = 3 – (200 × 103)(0.015 × 10–3) = 0
Fig. 185
184 COMPREHENSIVE OBJECTIVE PHYSICS

262. Loss of emf = Number of wrongly connected cells


× 2E E
E− V E−
Internal resistance remains unchanged. 271. r = R or r = 2 R or r = R = x
V E
263 to 266. Net emf = (8 – 4) V = 4 V
2
Total resistance
6×3 18 272. Potential difference equals the difference of emf and
= 0.5 + 1 + 4.5 + =6+ =8Ω Ir. Data requires I to be 2 A. So, R = 3 Ω.
6+3 9
4 273. Potential difference across R′ is 82.8 × 10 mV i.e. 0.828
I= A = 0.5 A V. Since R′ is 2 Ω therefore we get a current of 0.414
8
A.
This current shall flow in the direction of emf of cell
of higher voltage. This direction is anti-clockwise. 274. Radius one-third, cross-sectional area one-ninth. Re-
sistance becomes 9 times.
3×6
VAB = 0.5 × volt 9×5
3+6 Now, R= Ω = 7.5 Ω
6
1 18
= × volt = 1 volt V
2 9 275. E=
l
Again, V1 = E1 + I r1
= 4 + 0.5 × 0.5 = 4.25 volt IR
E=
Also, V2 = E2 – Ir2 l
= 8 – 0.5 × 1 = 7.5 volt I l
E= ρ
267. Potential difference across galvanometer is not l a
changed. I
268. Current shall decrease. Potential difference across E=ρ or E = ρj
a
battery and hence across the galvanometer shall in-
Here ρ = k ∴ E = jk
crease.
mr
V2 276. =R
269. Power = or V = PR = 2 × 10 = 20 = 2 5 n
R
m R
Clearly voltage across single resistor of 10 Ω cannot =
n r
exceed 2 5 volt. So, voltage across parallel combina- m 3
or = = 6 or m = 6n
tion has to be 5 volt. Note that the resistance of n 0.5
parallel combination is half of 10 Ω. Thus, the maxi- Again, N = mn
24 = (6n)n or 6n2 = 24 or n2 = 4
mum possible voltage between A and B is 3 5 volt.
or n = 2 ∴ m = 6 × 2 = 12.
l l×a 277. If internal resistance is negligible, then terminal po-
270. R=ρ =ρ
a a2 tential difference equals emf of a cell.
V m 278. It is a parallel combination of three resistances, each
R=ρ 2 =ρ
a da2 of 10 Ω.
ρm 1 279. The upper branch is a series combination of 4 Ω and
or R= ;R∝
dπ r 4
r 4 20 Ω. So, total resistance of upper branch is 24 Ω.
4 9 × 18
RA r 1
= 4
= The lower branch is a series combination of Ω
R B 16 r 16 9 + 18
or RB = 16 RA or RB = 16 × 4.25 Ω = 68 Ω and 6 Ω i.e. a series combination of 6 Ω and 6 Ω. So,
4.25 × 68 289 total resistance of lower branch is 12 Ω.
Now, R= Ω= Ω =4Ω
4.25 + 68 72.25 12 × 24 288
Now, R= Ω= Ω = 8 Ω.
12 + 24 36
CURRENT ELECTRICITY 185

2 × 10 −7 × 1 × 10 −2 m m
280. R= Ω = 2 × 10–7 Ω But d= or V =
1 × 100 × 10 −4 V d
l2d
2 × 10−7 × 100 × 10−2 ∴ R=ρ
281. R= Ω = 2 × 10–3 Ω m
1 × 10 −4
R1 l12 m2 12 3 × 3 27
6 2R = × = × =
282. = ⇒ R = 3 Ω R 2 m1 l22 8 4 × 4 32
5 2+R
283. Both the length and cross-sectional area are doubled. 2×6 12
292. R26 = Ω= Ω
So, resistance remains unaffected. 2+6 8
284. 7.6 = 3 × 2.2 + 100 r 3
R26 = Ω
7.6 − 6.6 1 2
or r= Ω= Ω It is in series with 1.5 Ω. This gives 3 Ω. It is in
100 100
1 3
Again, V = 6.6 – 100 × = 5.6 volt parallel with 3 Ω. This gives Ω i.e. 1.5 Ω.
100 2
285. A simple examination of the circuit leads us to the 6
I= A=4A
right choice. 1.5
2 1 I1 2
286. I= A= A 293. =
20 10 2 8
1 2
Now, V= × 15 volt = 1.5 volt or I1 = A = 0.5 A
10 4
Potential gradient I1 6W
1.5 3
= V cm–1 = V cm–1
1000 2000
2A
287. E1 ∝ 64
E1 – E2 ∝ 8 2W
E2 ∝ l Fig. 186
∴ 64 – l = 8 294. Rmax. = 9 Ω
or l = 64 – 8 = 56 cm Imax 9
Rmin. = 1 Ω or =
10 1 Imin 1
288. I= A= A
300 30 295. The given network is equivalent to the network shown
1 10 (300 + R ) in Fig. 187 So, the net resistance is (12 + 8 + 6) Ω i.e.
Now 2× = 26 Ω.
30 300 R
or 2R = 300 + R or R = 300 Ω.
1 1 1 1 Fig. 187
289. = + +
30 k 3k 5k 48 × 20
296. Potential difference across PQ = V = 3.2 V
1 15 + 5 + 3 15k 30 300
or = or = 297. Length is doubled. So, area of cross-section is halved.
30 15 k 23 1
298. E ∝ 52
or k = 46 V ∝ 40
So resistances are 46 Ω, 138 Ω and 230 Ω.
E 52 I (R + r ) 52
290. E = 8 volt, R = 10 Ω = or =
V 40 IR 40
8
I= A = 0.8 A R + r 52 r 52
10 = or 1+ =
R 40 R 40
l l×l l2
291. R=ρ =ρ or R = ρ r 12 12 3
a l×a V or = or r = ×5Ω= Ω = 1.5 Ω
R 40 40 2
186 COMPREHENSIVE OBJECTIVE PHYSICS

4 × 1.4 − 2 3.6 −8
299. I= A= A = 0.45 A 309. R = ρ = 50 × 10 × 2 Ω = 10–6 Ω
4×2 8 l 1
10 310. When temperature of a semiconductor is increased,
300. I= A more holes and free electrons are generated.
15 4
+ +2+1 311. Equivalent resistance of parallel combination is
3 2
10 10 4×2 8 4
= A= A= 1A Ω i.e. Ω or Ω. Ratio of voltage drops is
5+2+3 10 4+2 6 3
4
301. Slope of V – I graph gives resistance ; CD is a straight 1: or 3 : 4. So, voltage across parallel combination
line. 3
4E 4E 2E
302. L3 = 64 l3 is . Current through 2 Ω = = .
7 7×2 7
L 312. Note that 3 Ω and 16 Ω represent minimum and
L = 4l or l =
4 maximum values of the combinations.
l 313. Number of parallel resistances
Again, R=ρ
a Total current to be carried
For the case of a cube of each side l1 , =
Current - carrying capacity of individual resistor
l ρ
R=ρ or R = 4
l 2
l = =4
1
Since l is reduced by a factor of 4 therefore R is in- 10
creased by a factor of 4. So, new resistance is 4 × 4 Ω Resistance of parallel combination = Ω = 2.5 Ω
4
i.e. 16 Ω. Clearly, two such units are required to be connected
6V in series. So, total resistance is 5 Ω.
303. Potential gradient = = 2 V m–1
3m 200
314. Effective resistance across voltmeter = = 100 kΩ
Potential difference between two points separated by 2
1 1 Total resistance across dc supply = 400 + 100
m= × 2 = 1 volt = 500 kΩ
2 2
2 30 3 100
304. I = 5 × 10 A = A= A Thus the voltage across the voltmeter = (60 V)
50 5 500
5 + 10 = 12 V.
R 315. An equivalent of the given network is as under :
305. Resistance of parallel combination is
2
R
Total resistance of series combination = 4 × = 2 R.
2
306. I = 1.6 × 10–19 × 6 × 1015 A = 0.96 × 10 –3 A = 0.96 m A.

1.6 × 10 −19
307. I = A = 1.07 × 10–3 A. Note that the value
1.5 × 10 −16
of radius is not required.
LM l OP = al 10a
308.
N
a′ l +
10 Q or a′ =
11
Fig. 188

l′ l 2R 2 2R
Now, R′ = ρ and R = ρ Combination of 2R and R gives or .
a′ a 3R 3
R ′ l′ a 11 11 2R 5R
The series combination of and R gives .
= × = × = 1.21 3 3
R a′ l 10 10
So, the network reduces to a parallel combination of
or R′ = 1.21 × 10 Ω = 12.1 Ω
5R 5R
, R and .
3 3
CURRENT ELECTRICITY 187

1 3 1 3 3+5+3 322. The equivalent of the given network is as under :


∴ = + + =
Rp 5R R 5R 5R

5R 3 5R
Rp = or =
11 2 11
33 Fig. 189
10 R = 33 or R = Ω = 3.3 Ω.
10
R
2 20 4 It is a parallel combination of R and .
316. I= A= A= A 2
7.5 75 15
FG R IJ
R2
Current in upper branch (R)
2H 2K
R
FG
1 4 2 IJ ∴ Rp =
R
=
3R
=
3
=
H
2 15
A =
15
A
K R+
2 2

2 20 4 220 − 20 × 1.8
VAB = 10 × volt = volt = volt 323. 15 =
15 15 3 20 × 0.1 + R
15 × 30 184
317. R+ = 20 15 [2 + R] = 220 – 36 = 184 or 2 + R =
15 + 30 15
450 184 184 − 30
R = 20 – or R= –2= Ω
45 15 15
R = 10 Ω 154
Ω = 10.27 Ω
=
9 15
318. Total resistance = 9 Ω ; current I = A=1A 324. VA + 3 – 3 – 18 = VB or VA – VB = 18 volt
9
At A, this current is to be distributed between 8 Ω 325. 56 = 32 × 2 – 2n × 2
and the resistance of the remainder circuit which is 8 4n = 64 – 56 = 8 or n = 2
Ω. The current will be equally distributed. So, cur- E
326 & 327. Using I= , we get
1 R+r
rent entering B = A. This current will be distrib-
2 E
uted equally between 8 Ω and the resistance of the 0.5 = …(i)
remaining circuit which is 8 Ω. So, current through 3.75 + r
1 E
4 Ω resistor is A i.e. 0.25 A. 0.4 = …(ii)
4 4.75 + r
1.0182 V Dividing (i) by (ii), we get
319. Potential gradient = = 0.03 V cm–1
33.94 cm 5 4.75 + r
=
I 4 3.75 + r
320. vd =
neA or 18.75 + 5r = 19 + 4r
Both I and A are doubled. So, drift velocity remains or r = 19 – 18.75 = 0.25 Ω
unchanged.
E
321. Condition for balanced Wheatstone Bridge is satis- From Eq. (i), 0.5 =
3.75 + 0.25
fied. So, there will be no current through the galva-
nometer. or E = 4 × 0.5 V = 2 V
I(12) 328. The current I is related to drift velocity v and cross-
Now,
30
= sectional area A by
1.4 12 + 30
I = nAve
1.4 × 30 where e is the charge of 1.6 × 10–19 C.
or I(12) = A=1A
42 n is the number density of the conduction
electrons.
188 COMPREHENSIVE OBJECTIVE PHYSICS

Since I is held constant, we have 333. Total resistance of the parallel combination of 2 Ω
1 1 1 2×6 12 3
v ∝ = or v ∝ . and 6 Ω is Ω i.e. Ω or Ω.
A πr 2 r 2 2+6 8 2
Now, the radius of the wire is half, we have 3
Now, I= A=2A
vA r2
3/2
= 2 = 22 = 4 ⇒ vA = 4 v. 334. The equivalent of the given network is as under :
v rA
329. The voltage across the capacitor is

Q 1 × 10−5
V= = = 10 V
C 1 × 10−6
Thus, the current flowing through the resistor after
the switch is closed will be
V 10
I= = A=1A
R 10
Fig. 191
330. Suppose source of unknown emf is not connected.
The parallel combination of 10 Ω and 10 Ω gives 5 Ω.
2 2 1
Then, I= A= A= A 335. The equivalent of the given network is as under :
10 + 40 50 25
Voltage across 40 cm length of wire
1 10 4
= × × 40 = volt = 0.16 volt
25 100 25
Clearly, E = 0.16 volt
331. Using principle of balanced Wheatstone Bridge, we
arrive at the following equivalent network.

Fig. 192

This is equivalent to a parallel combination of 30 Ω


and 30 Ω. This gives 15 Ω.
336. The equivalent of the given network is as under :

Fig. 190

The equivalent of the above network is a series combi-


2R 5R
nation of R and . This gives .
3 3
332. The number density of the conduction electrons n is
given by I = nAve
where I = current = 6.0 A
A = cross sectional area = 5.0 × 10–6 m2
Fig. 193
v = drift velocity = 1.2 × 10–4 m s–1
Clearly, we have a series combination between A and
e = electrical charge = 1.6 × 10–19 C
B.
⇒ 6.0 = n(5.0 × 10–6)(1.2 × 10–4)(1.6 × 10–19)
FG 2 2 2 IJ
⇒ n = 6.25 × 1028
≈ 6.3 × 1028 m–3
∴ R = 1+
H 3
+2+ + +2
3 3 K Ω=7Ω
CURRENT ELECTRICITY 189
337. The equivalent of the given network is as under : 343. The lower limit is zero volt (0 V) when X is at the
lower end of the 4 kΩ resistor.
The upper limit is the potential difference across the
4 kΩ resistor when X is at the upper end of the 4 kΩ
resistor. That is

FG 25 IJ 4 K = FG 25 IJ 4 = 20 V
Fig. 194
v=
H 1K + 4 KK H 5 K
3R Thus, the limits are 0 and 20 V.
It is a series combination of R and .
2 344. The current density at P is higher than at Q. For the
This gives 5R . same current flowing through the metallic conductor
2 PQ, the cross-sectional area at P is narrower than at
338. IrB = 0.05 Q. The resistance per unit length r is given by
IrA = 0.15 1
r=ρ
rB 1 A
Dividing, =
rA 3 where ρ is the resistivity.
or rA = 3rB A is the cross-sectional area of the conductor
PQ.
1 1 1 1 1 + 3 + 6 10 1
339. = + + = = = Thus, r is inversely proportional to cross-sectional area
R 60 20 10 60 60 6
A of the conductor.
or R=6Ω
6 3×9 27 9
Now, 1 × 6 = 60 I or I = A = 0.1 A 345. R39 = Ω = Ω = Ω
60 3+9 12 4
Total resistance
l
340. R = ρ 9 1 10
πr 2 = + = Ω = 2.5 Ω
4 4 4
l
Now, l′ × π (nr)2 = l × πr2 or l′ = 1.5 3
n2 Now, I= A= A = 0.6 A
2.5 5
l′ ρl 1
R′ = ρ = = R R
π (nr) 2 2
n π (n r )2 2
n4 346. Resistance/length =
2πr
341. In one second, the belt has travelled through the point R Rθ
for a distance v m. Thus, the particular point has Resistance of shorter segment = (rθ) =
2πr 2π
swept through a surface area of vw m2. As a conse-
quence, the charges removed through the point is R
Resistance of longer segment = (r) (2π – θ)
wv σ in one second. Thus, the current collected through 2πr
the point is wv σ. It is the case of a parallel combination.
4×6 24 Rr θ (2π − θ) R
342. R46 = Ω= Ω = 2.4 Ω ∴ Rp = or Rp = (2π – θ)θ.
4+6 10 2πr θ + 2π − θ 4 π2
1 1 1 1 347. Thinking in terms of balanced Wheatstone Bridge,
= + +
R 468 4 6 8 3 Ω resistance is not to be considered.
8 × 16 128 16
1 12 + 8 + 6 Now, R= Ω = Ω or Ω.
= 8 + 16 24 3
R 468 48
3 16
48 24 Again, V = IR = × volt = 8 volt
or R468 = Ω= Ω = 1.85 Ω 2 3
26 13 6 2
348. Is = ; Ip =
In series combination, I is constant. So, V is more for 2 + 3r r
higher resistance. 2+
3
190 COMPREHENSIVE OBJECTIVE PHYSICS

6 6 354. Resistivity of the material is given by


Equating, = or 2r = 4 or r = 2 Ω
2 + 3r 6 + r RA A
ρ= =
l Gl
6 6
∴ Is = A= A = 0.75 A where R is the resistance of the wire,
2 + 3(2) 8
G is the conductance of the wire,
349. The expression for σ in terms of ρ, R, S, l or G is given
A is the cross-sectional area of the wire,
by
l is the length of the wire.
1 l l
σ= = =G
ρ RS S 2.00 × 10−6
∴ ρ= Ω m = 5.3 × 10–7 S
(1.25)(3.00)
l 1 GR
where R=ρ = or σ = 355. The equivalent of the given circuit is as under :
S G ρ
350. E1 ∝ 300
E1 – E2 ∝ 100
E1
= 3 or E1 = 3E1 – 3 E2
E1 − E2
E1 3
or 3E2 = 2 E1 or =
E2 2
6×3 Fig. 196
351. Parallel combination of 3 Ω and 6 Ω gives Ω or
6+3
Current through 4 Ω is clearly 2 A.
18
Ω or 2 Ω. Total current = (2 + 0.5) A = 2.5 A
9
18
Now, 2.5 =
4 × 16
R+
4 + 16
64 18
or R + = = 7.2 or R = 7.2 – 3.2 = 4 Ω
20 2.5
356. The expression for current I in a conduction wire is
Fig. 195 given by
I = nAve
This is in series with 2 Ω. So, we get 4 Ω.
where n is the number density of conduction elec-
352. Condition for Wheatstone Bridge is satisfied. So, 7 Ω
trons.
need not be considered. Resistance of upper branch is
5 Ω. Resistance of lower branch is 10 Ω. A is the cross-sectional area of the wire,
So, combined resistance is v is the drift velocity of the electrons,
e is the electrical charge of 1.6 × 10–19 C.
5 × 10 50 10
Ω i.e., Ω or Ω.
5 + 10 15 3 1 1
If I is constant, then v ∝ = 2
353. Voltage across 2 Ω = 0.48 × 2 volt = 0.96 volt A πr
Voltage across 3 Ω = 0.96 V where r is the radius of the circular cross-sectional
area.
0.96
Current through 3 Ω = A = 0.32 A Now, radius is doubled, so drift velocity is reduced to
3
Current in circuit = (0.48 + 0.32) A = 0.8 A v
.
4
2.8
Now, 0.8 = 357. Resistance is the gradient of V-I graph. If resistance
6
r + 1.8 + decreases with temperature rise (which occurs when
5
28 voltage is increased), the graph becomes less steep in
or 8= or 8r + 24 = 28 the I-axis.
r+3
or 8r = 4 or r = 0.5 Ω.
CURRENT ELECTRICITY 191

10 × 1.5
358. I =
10 × 1
20 +
5
15 15 x
I= A= A = 0.68 A
20 + 2 22
dx b
359. 5 = R0 [ 1 + 50 α] … (1)
6 = R0 [ 1 + 100 α]
5 1 + 50 α
=
6 1 + 100 α
6 + 300 α = 5 + 500 α Fig. 197

1 dx
or 1 = 200 α or α = dR = ρ
200 4 πx2

LM 1 OP Total resistance is given by

z z
From Eq. (1), 5 = R0 1 + 50 ×
N 200 Q R=
a
ρ
dx or R =
ρ
a
x −2 dx
4 πx 2
or 5 = R0 LM1 + 1 OP b

b

N 4Q ρ x −2 + 1 a
ρ 1 LM OP a

or
5×4
R0 = Ω=4Ω
R=
4π − 2 + 1
b
or R = –
4π x N Q b
5
360. The number density n of conduction electrons in the R= LM
ρ 1 1 OP or R =
ρ a−b
copper is a characteristic of the copper and is about N
4π b a

Q 4 π ab
1029 at room temperature for both the copper rod X 363. Clearly, E2 = IY
and the thin copper wire Y. 12
2= Y or 500 + Y = 6 Y
Both X and Y carry the same current I since they are 500 + Y
joined in series. or 5Y = 500 or Y = 100 Ω
From I = nAvq 364. The current I through the resistance wire XY is the
where q is the electron charge of 1.6 × 10–19 C, same. Since the wire is uniform, its cross-sectional
v is the drift velocity in the conductor, area A is constant throughout its length. Hence, the
A is the cross-sectional area of the conductor. current density,
We may conclude that rod X has a lower drift velocity I
J=
of electrons compared to wire Y since rod X has a A
larger cross-sectional area. This is so because the elec- is also uniform through the wire XY.
trons in X collide more often with one another and 365. Q = at – bt2
with the copper ions when drifting towards the posi-
tive end. Thus, the mean time between collisions of dQ
= a – 2bt or I = a – 2bt
the electrons is more in X than in Y. dt
2500 Clearly, the current decreases linearly with time.
361. Parallel combination of 100 Ω and 25 Ω gives When I = 0, a – 2bt = 0
125
Ω i.e. 20 Ω. a
Now, 120 Ω and 120 Ω are in parallel. The combined or 2bt = a or t =
2b
resistance is 60 Ω. Now, we have a parallel combina-
tion of 160 Ω and 40 Ω. dI
Again, = – 2b.
160 × 40 6400 dt
∴ R= Ω= Ω = 32 Ω
160 + 40 200 366. Potential difference across 20 Ω = 20 × 1 = 20 volt
362. Consider a spherical shell of radius x and thickness = Potential difference across R2
dx. Its resistance is given by
192 COMPREHENSIVE OBJECTIVE PHYSICS

Current in R2 = 0.5 A 1 1 1 1
372. = + +
20 Rp R 2R 2R
R2 = = 40 Ω
0.5 1 2 + 1+ 1
or =
Potential difference across R1 = 69 – 20 = 49 V Rp 2R
20 2R R
Current in R1 = 0.5 A + A + 1 A = 3.5 A or Rp = or Rp =
10 4 2
373. The circuit may be redrawn as follows :
49
∴ R1 = = 14 Ω
3.5 I I I
R
367. Effective resistance = (2 R)// R // (2R) = R // R = .
2 R R
6V I
368. Current = =2A 6V 3
3Ω 2 º 6V
º
6V 2R
1
R R 2R

Fig. 198 Fig. 199


369. The potential difference across R2 and R3 is the same 6 4
which is given by Current I is given by I== A
3 R
R
2
Potential difference = R2I2 = R3I3
I 2
⇒ I2/I3 = R3/R2 ∴ Current through the voltmeter is or A.
2 R
Sum of the current I2 and I3 is I1.
2FG IJ
So,
I R
I1/I3 = 1 + 2 + 1 + 3 (independent of R1).
Hence, the reading of the voltmeter is
R H K(R) or 2 V.

I3 R2 374. The voltage per unit length on the meter wire PQ is


370. If V is the potential difference applied across P and Q, 6.00 mV or 10mV/m
the current through M is determined by 0.60 m
Circuit Current Hence, potential across the meter wire PQ is
(10 mV/m) (1m) = 10 mV.
a V/5
Current drawn from the driver cell,
b 3 V/8
10 mV
c V/2 I= = 2 mA
5Ω
d V/3
Resistance of resistor R is R
e V/4
2 V − 10 mV 1990 mV
Hence, circuit arrangement c gives the largest read- = = = 995 Ω.
2 mA 2 mA
ing in ammeter M.
375. Effective resistance across voltmeter = 50 kΩ
10 (3 + R)
371. 3+ =R Total resistance across dc supply = 450 kΩ
13 + R
1000 1
30 + 10R Current drawn from supply = = A
or =R–3 450 K 450
13 + R
Potential difference across voltmeter
or 30 + 10 R = (13 + R) (R – 3)
50 K 1000
or 30 + 10R = 13 R – 39 + R2 – 3R = V= V = 111 V
450 9
or R2 = 69 or R = 69 Ω
CURRENT ELECTRICITY 193
376. Potential across XS is 1.02 V at the length of l1. 382. 418 t = 1 × 4180 × 20 or t = 200 s
F 1.02 V I l F l I 1.02 V.
Potential across SY is GH l JK 1
2 or GH l JK
2
1
Note that in this case, S = 4180 J/kg/° C
J = 4.18 J/cal
Potential across XY = Potential across XS + Potential
across SY
Fl I Fl + l I
= 1.02 + G J 1.02 = G
383.
300 × 300
P= W=
90
kW = 4.5 kW
H l JK
2 20 20
Hl K
1 2
1.02 V.
1 1 Since this power is less than 21 kW therefore the re-
377. I2R = 2πrlh or I2 ρ = constant sistor will not be damaged.
I2 V2 V2 V2
or = constant or σ ∝ I2 or I ∝ σ 384. × 20 = × 30 = t
σ R1 R2 R1 + R 2
If σ is doubled, I becomes 2 times. Clearly, t = (20 + 30) min = 50 min.
378. Current delivered by the power supply 2
V V2 V2
=3A+1A=4A 385. × 20 = × 30 = t
R1 R2 R 1R 2 /(R 1 + R 2 )
+ – 4A
V
20 × 30 600
Clearly, t= = min = 12 min.
20 + 30 50
1. 5 W 6V
3A 2W 386. I ∝ r3/2

I I
=
LM OP
0.3
3/ 2
or I = 5 ×
27
A
N Q
4A
1A 6W
5 0.2 8

6V
Fuse wire
Fig. 200
I2Rt = 2πrl ht where h is heat lost by radiation per
Voltage across the power supply unit area per unit time.
= 6 V + 6 V = 12 V ρl
I2 t = 2 πr lht
379. Pt = mSθ πr 2
or I2 ∝ r3 or I ∝ r3/2.
1 × 4200 × 15
P= W = 1050 W
60
387. Option (c) is a case of minimum resistance and maxi-
Problem of geyser and water mum current.
Use Pt = mSθ l
388. R = ρ ; When both l and r are doubled, resistance
Take S = 4200 J/kg/°C. πr 2
is halved. Current is doubled.
V2 3V 2 V2
380. 90 = = or = 30 P 1 FG 1 IJ 2
R/3 R R 389. I= =
V 10
A, P =
H 10 K × 10 W = 0.1 W

V 2 30 390. Power = 220 × 9 W


Again, P= = W = 10 W
3R 3
220 × 9
381. 10 = I2 × 5 or I = 2 A Number of bulbs = = 33
60
2 1 391. R100 = 2 R200
Current in upper branch shall be A i.e. A,
2 2 ∴ H200 = 2H100 or 2H1 = H2
because the resistance of upper branch is twice the
R 1 P2 300 3
resistance of lower branch. 392. = = =
R 2 P1 500 5
Q
Now, 4 =
FG 1 IJ 2

t H 2K × 4 cal s–1 = 2 cal s–1


194 COMPREHENSIVE OBJECTIVE PHYSICS

6×6 36 3 (220)2 (110)2 110 × 110 R


393. 48 = or R = = ∴ = or R′ = R=
R 48 4 R R′ 220 × 220 4
Resistance of each lamp 404. Watt-hour of battery when it is charged = VIt
= 15 × 10 × 8 Wh = 1200 Wh
3 3
=2× Ω = Ω = 1.5 Ω. Watt-hour supplied by battery during discharge
4 2
= 14 × 5 × 15 Wh = 1050 Wh
394. See solution of Q. 385.
1050
395. If B2 or B3 is disconnected, resistance is increased. ∴ Efficiency = × 100 = 87.5%
1200
396. 25 W bulb shall have a voltage higher than its rated
V2
voltage. 405. Power P =
R
397. The current through the single 10 Ω resistance can- Since R is common, P ∝ V2
not exceed 2 ampere.
398. Power dissipated ∝ Requivalent F V I F 4.5 IJ
Power ratio = G J = G
2 2

H V K H 1.5 K
1
= 32 = 9
(I) Req = R + R + R = 3 R 2
(II) Req = 2R/3 406. For maximum power, internal resistance = 0.5 Ω
(III) Req = R/3 So, the batteries should be connected in parallel.
(IV) Req = 3R/2 2
I= =2A
∴ Increasing order of power dissipation 0.5 + 0.5
III < II < IV < I 1
P=2×2× W=2W
399. When connected in series, each bulb gets 110 V. 2
V2 R1 m1 a22
P= . V halved. P becomes one-fourth. So, each 407. = ×
R R 2 a12 m2
100 Fm I Fa I 2
FG 1IJ (1)
bulb gets
4
W i.e., 25 W.
= GH m JK GH a JK
1
2
2
1
=
H 2K 2 =
1
2
So total power consumed by series combination is
50 W. Q1 I 2R 1t 1
= =
In parallel combination, each bulb gets 220 V. So, Q2 I 2 R 2 t 2
each bulb consumes 100 W. Power drawn by combi-
nation is 200 W. Relation between resistance, mass and
tt 10 × 40 cross-sectional area
400. tp = 1 2 = = 8 minute
t1 + t2 10 + 40 l×a V
R=ρ or R = ρ
401. U ∝ I2. a× a a2
V 2 (200)2 R=ρ
m
=
ρ m LM OP
402. The resistance of each bulb =
P
=
60
Ω . When
da2 d a2 N Q
three identical bulbs are connected in series, their m
I∝
(200) 2 a2
resultant resistance, Rre = 3 ×Ω
60
120 × 120
∴ Power drawn by bulbs, when connected across 408. R60 = Ω = 240 Ω
60
V2 (200)2
200 V supply, P = = W = 20 W 60 W
R re (200)2

60
(220)2
403. In India, P=
R
(110)2 120 V
In USA, P′ = , But P = P′
R′ Fig. 201
CURRENT ELECTRICITY 195

120 120 1
Current = A= A
240 + 6 246 Total resistance = 125 2
1
Voltage across bulb +1
2
120
= × 240 volt = 117.1 volt 1 2 125
246 = 125 × × = Ω
2 3 3
250 × 3
I= A = 6 A.
125
411. Let the resistances of P, Q and R be r.
The total resistances across the battery is thus
r 3
rtotal = r + = r.
2 2
Current through P,

Power 12 8
IP = = 3
= .
rtotal r r
2

Current through R,

1 2
I= IP = .
Fig. 202 2 r
120 × 120 Power dissipated in R is thus
R240 =
240
Ω = 60 Ω
FG 2 IJ r = 2 W
Resistance of parallel combination
PR = I2 r =
H rK
60 × 240 l × l ρ l2
= Ω = 48 Ω 412. R=ρ =
60 + 240 a×l V
Total resistance = (48 + 6) Ω = 54 Ω m m
But d= or V =
V d
120
Current I= A ρ l2 d
54 ∴ R=
Voltage across parallel combination m
R1 1
120 =
= × 48 volt = 106.7 volt R2 4
54
P1 R 2 4 P1
Change in voltage = (117.1 – 106.7) V = 10.4 V. = = or =4
P2 R 1 1 5
409. Power supplied by charger = 150 V × 10 A = 1500 W. or P1 = 20 W
410. Resistance of heater 413. Energy consumed = 10 × 50 × 30 × 10 Wh = 150 kWh.
250 × 250 125 Q1 R 1 2R 4
= = Ω = = =
1000 2 414.
Q2 R2 R/2 1
Resistance of 1 bulb
415. If I is the current drawn from the source, then
250 × 250 total power delivered = I2 (r + R)
= Ω = 625 Ω
100 Power delivered to the lamp with resistance R = I2R
Resistance of 5 bulbs connected in parallel ∴ Fraction of the total power delivered to the lamp
625
= Ω = 125 Ω I2R R
5 = 2
=
I (r + R) r+R
or Power = I2R
196 COMPREHENSIVE OBJECTIVE PHYSICS

In series, circuit I is common i.e. power ∝ R. 424. Current produced by the generator is given by
R 100 kW
∴ Power fraction to lamp = . I= = 10 A
r+R 10 kV

r3 FG 3 IJ 2 Power loss in the cables


416.
13
=
H 1.5 K = 4 or r = (4)1/3 mm = I2R = (10)2 5 = 500 W.
425. Let current flow from b to a as shown.
418. Current through 6 Ω

150
= A=5A
6

196
Current through 4 Ω = A=7A
4
Now, E = 5 [ 6 + r] = 7 [ 4 + r]
or 30 + 5r = 28 + 7r Fig. 203
or 2r = 2 or r = 1 Ω.
FG 2 IIJ 2
FG 1 IIJ 2
419. Power dissipated in a resistor of resistance R with a
current flow of I is given by
Ratio is
H3 K 3R :
H3 K 6R : I2R

Power dissipated, P = I2R 4 2


or : :1 or 4 : 2 : 3.
It is given that R is directly proportional to current I, 3 3
we have 700 100 1100
426. I= +4× = A=5A
220 220 220
P ∝ I2 (I) or I3
V2 V 2a
Given, P = 6.0 W at I = 1.0 A 427. Q= t or Q = t
R ρl
Thus, at I = 2.0 A, the power dissipation is
⇒ t∝l
FG 2.0 IJ 3
If l becomes two-third, t also becomes two-third.
P=
H 1.0 K 6.0 = 48 W
2
∴ Required time is × 15 min or 10 min.
1 1 3
420. CV 2 = × 2 × 10 –6 × 200 × 200 = 4 × 10 –2 J
2 2 V2 120 × 120
428. Q = t= × 10 × 60 J = 14.4 × 105 J
421. I2 × 6 = 60 or I = 10 A R 6
Current through upper branch = 2 10 A. Heat pro-
duced per second across 3 Ω = (2 10 ) 2 × 3 cal 220 × 220 484
429. R= Ω= Ω = 48.4 Ω
= 120 cal. 1000 10
Rp = 24.2 Ω
1.5 × 1.5 Rs = 96.8 Ω
422. Resistance of bulb = Ω = 0.5 Ω
4.5
V2
Resistance of parallel combination, 430. P=
R
1 R is decreased by a factor of 16. So, P is increased by a

2 1 factor of 16.
R= Ω= Ω
1 3 ∴ New Power = 16 × 5 W = 80 W
1+
2
Q1 R1
431. = [∵ I is same]
E−V Q2 R2
Now, r= R
V
432. Both I and R are same.
8 E − 1.5 1
= × or E = 13.5 volt V2 V 2a
3 1.5 3 433. P = = ; When l is halved, P is doubled.
R ρl
423. Higher resistance ; some current.
So, less time is required.
CURRENT ELECTRICITY 197

V2 P P0 100 × 4200 × 50
434. P= or = = A = 12.2 A
R V 2
V0 2 240 × 2 × 60 × 60
40 445. I∝ Q
435. × 1000 × t = 1 × 4200 × 80
100
42 × 80 1 1 1 1
42 × 80 446. = + +
or t= s= min = 14 min. P 100 60 40
4 4 × 60
l×a V 1 6 + 10 + 15 600
= or P = W
436. R=ρ =ρ 2 P 600 31
a×a a
447. If power consumed in resistor I is 18 W, then power
ρm ρm 1 18
or R= or R = or R ∝
da2 L πD OP
dM
2 2 D4 consumed in each resistor II and III is
4
W.

MN 4 PQ
Q1 R 1 D24 54 625
= = = =
Q2 R2 D14 34 81
220 × 10
437. Number of lamps = = 22.
100
V2 V2 Fig. 204
438. Q= t or Q = t or t ∝ l
R l
ρ ∴ Total power consumed
a
10% reduction in length means 10% reduction in time. FG 18 18 IJ
90 144
H
= 18 +
4
+
4 K
W = 27 W.

So, new time is × 16 i.e. min or 14.4 min.


100 10
439. P = I2R. THINK IT OVER !
In the given problem, I is constant.
P1 R 1 1
∴ = =
P2 R 2 3
V2
440. P=
R
when R is halved, P is doubled. Fig. 205
441. Pt = mSθ Repeat Q. 447 for the above arrangement.
2.1 × 1000 × t = 1 × 4200 × 80
4200 × 80
or t= s = 160 s
2.1 × 1000
442. Note that the printed specifications are the same in
all appliances.

Resistance of an electric appliance is always


determined from printed specifications.

443. Pt = mSθ or VIt = mSθ Fig. 206

mSθ 2 × 4200 × 75 Repeat Q. 447 for the above arrangement of infinite


or t= or t = s network. Would your answer be 36 W ?
VI 220 × 5
= 572.7 s ≈ 573 s V2 V2
448. P= = or D ∝ P
mSθ R l
444. VIt = mSθ or I = ρ
Vt πD2
198 COMPREHENSIVE OBJECTIVE PHYSICS

455. m = ZIt, V = IR
D1 100 2

D2
=
25
= 4 =
1 ∴ m=Z
LM V OP t
449. Let resistors P, Q and R have resistor r. The effective
NRQ
resistance across the source is
m=Z M
L V A OP t m ρl

Reff = r + r // r = r +
( r) ( r) r 3r
=r+ =
or
N ρl Q or V =
Z At
r+r 2 2
0.66 × 1.2 × 10 −2 × 5 × 10 −2
Current drawn from the source is Is2 Reff = 12 Now, V= volt
3.3 × 10 −7 × 1 × 3600
12 8
⇒ Is = = A. 0.66 × 1.2 × 50
Reff r = volt = 0.33 volt.
3.3 × 36
Since Q and R have equal resistance r, each draws a
456. If θ is the small temperature difference,
1 2 then E = 25 θ × 10 –6 volt
current of I which is given by I = I = A
2 s r or IR = 25 θ × 10 –6
Heat dissipation in R can now determined which is or 10 × 40 = 25 θ × 10 –6
–5

given by −5
or θ = 10 × 40 = 400 ° C = 16°C
FG 2 IJ r = 2 W. 25 × 10 −6 25
Power dissipated as heat = I2r =
H rK 457. m = ZIt or I =
m
450. Maximum current flows through bulb. Zt
451. (i) Rate of chemical energy consumption 1.071
or I= A
= 1.5 × 2 W = 3 W 0.001118 × 20 × 60
(ii) Rate of energy dissipation inside the cell or I = 0.798 A
= 2 × 2 × 0.1 W Ammeter reading = 0.81 A
(iii) Rate of energy dissipation inside the resistor Error is reading = (0.81 – 0.798) A = + 0.012 A.
= (3 – 0.4) W = 2.6 W dV
458. σ= or dV = σ dT
(iv) Power output of source = (3 – 0.4) W = 2.6 W dT
∫ dV = σ(T2 – T1) or V = σ(T2 – T1)
Some useful facts or 3 × 10–4 = 3 × 10–6 (T2 – 20)
EI represents rate of chemical energy consumption or 100 + 20 = T2 or T2 = 120°C
of the cell. 459. E = aθ + bθ2
I2r represents the rate of energy dissipation inside
the cell.
dE
At neutral temperature, =0
(EI – I2r) represents the power output of the source

of emf. dE a
∴ = a + 2b θn = 0 or θn = −
dθ 2b
Z VIt
452. m = ZIt or m =
V or θn = −
1
× 700 °C = – 350°C LM∵ a
= 700° C
OP
0.33 × 10− 6 × 100 × 1000 × 3600
= kg
2 N b Q
33 As θc = 0°C,
3.3 × 36 ∴ θn > 0°C
= kg = 3.6 kg
33 But here, θn < 0°C, ∴ No neutral temperature is
453. m = ZIt possible for this thermocouple.
m = 104 × 10–7 × 1 × 10 × 60 g 460. Initial resistance = (2 + 3) Ω = 5 Ω
= 624 × 10 –5 g FG 2 × 2 + 3IJ
454. Current in the first case is twice of the current in the
second case.
Final resistance =
H2+2 K Ω=4Ω
CURRENT ELECTRICITY 199

5 465. Equivalent weight of Cu is 32.


Clearly current becomes times. Percentage increase 467. For 22.4 litre, 96500 C of charge is required.
4
5 For 0.224 litre, 965 C of charge is required.
−1
in rate of deposition = 4 × 100 = 25. 965
I= A = 9.65 A.
1 100
461. m = ZIt 468. Percentage change in thermoelectric power = Percent-
m = 0.0000104 × 1 × 3600 = 0.03744 g. age change in temperature
462. Q = 9.65 × 10 C
100 − 80
Q = 96.5 C = × 100 = 25.
80
96500 C of charge deposits 33 g of zinc.
469. θi – θn = θn – θ0
33 θi = 2θn – θ0
So, 96.5 C of charge deposits g i.e., 33 mg.
1000 = 2 × 270 – 20 = 520°C.
463. 1 = 11.2 × 10–4 I1 × 30 × 60
470. The thermocouple works in a closed circuit. Since the
104 two pipes are isolated, only b and e are closed-circuits.
or I1 = A = 0.496 A
11.2 × 30 × 60 In circuit b, the two constantan wires to the cold cop-
per pipe produce 2 opposing currents.
Again, 1.8 = 6.6 × 10–4 I2 × 30 × 60
In circuit e, the copper-constantan junction is main-
1.8 × 104 tained across the cold pipe. A steady current can flow.
or I2 = A = 1.515 A
6.6 × 30 × 60 475. Temperature difference = 373 – 273 = 100 K
I = I1 + I2 emf = (100 × 90 µV) = 9000 µV
= (0.496 + 1.515) A = 2.01 A ≈ 2 A Total resistance = 30 + 6 = 36 Ω
Rate at which battery delivers energy 9000
Thus current = µA = 250 µA.
= 12 V × 2 A = 24 J. 36
464. Q = 16 × 10 × 60 C = 9600 C 476. Inversion temperature depends upon the temperature
96000 C would liberate 23 g of sodium. of cold function.
9600 C would liberate nearly 2.3 g of sodium. 477. E = 150 × 40 × 20 µV = 120 mV.

KNOWLEDGE PLUS
Six equal resistances are connected between points P, Q and R as shown in the Fig. 207.
P
Then the net resistance will be maximum between
(a) P and Q (b) Q and R
(c) P and R (d) any two points [IIT Screening 2004]
R/3 R R/2

Solution. (i) Q R

Fig. 207
P R Q

Fig. 208
R R 5R
Series combination of and gives . Net resistance is
3 2 6
FG 5R IJ R
H6K or
5R 2
×
6
or
5R
or 0.45 R
5R 6 11 R 11
+R
6
200 COMPREHENSIVE OBJECTIVE PHYSICS

R P R/3
FG R IJ FG 4R IJ
(ii)
H KH 3 K
In this case, net resistance is 2 or
4R 2
×
3
or
4R
or
2
R i.e. 0.28 R
4R 2 × 3 7R 2×7 7
R+
Q R 3
R/2
Fig. 209
R Q R/2
FG 3R IJ FG R IJ
(iii) In this case, net resistance is
H 2 K H 3K or
R
×
6
or
3R
or 0.27 R
3R R 2 11 11
+
P R 2 3
R/3
Fig. 210
Clearly, (a) is the right choice.

Find current in 2 Ω resistor


10 V 20 V
(a) 0 (b) 2 A (c) 4 A
(d) 1 A.
5 W 10 W
[IIT Screening 2005]
Solution. Using Kirchhoff’s junction rule, (a) is the right choice.
2W
Fig. 211

Ideal gas is contained in a thermally insulated and rigid container and it is heated through a resistance 100 Ω by
passing a current of 1 A for five minute, then change in internal energy of the gas is
(a) 0 kJ (b) 30 kJ (c) 10 kJ (d) 20 kJ. [IIT Screening 2005]
Sol. 2
∆Q = ∆U = I R∆t = 1 × 1 × 100 × 5 × 60 J = 30 KJ
∆W = 0
Now, ∆Q = ∆U + ∆W = ∆U = 30 KJ
So, (b) is the right choice.
CURRENT ELECTRICITY 201

SET III MCQs


with
More than one correct alternative

Average time allowed per question is 50 seconds.

480. When a potential difference is applied across, the cur- 484. Two voltmeters P and Q give the same reading when
rent passing through connected across a resistor X in a current-carrying
(a) an insulator at 0 K is zero circuit. But when connected across another resistor
(b) a semiconductor at 0 K is zero Y, the reading comes much lower in P than in Q. If
(c) a metal at 0 K is finite P, Q, X, Y represent the respective resistances, the
(d) a p-n diode at 300 K is finite, if it is reversebiased. data shows that
[IIT 1999] (a) X << Y (b) X << P
481. Which of the following statements is true ? (c) P << Q (d) Y << P.
(a) Conductance is the reciprocal of resistance and is meas- [National Standard Exam. in Physics 1990]
ured in siemen
485. Two conductors, made of the same material, have
(b) Ohm’s law is not applicable at very low and very high lengths L and 2L, but have equal resistances. The
temperatures
two are connected in series in a circuit in which cur-
(c) Ohm’s law is applicable to semiconductors
rent is flowing. Which of the following is/are correct ?
(d) Ohm’s law is not applicable to electron tubes, discharge
(a) the potential difference across the two conductors is the
tubes and electrolytes.
same.
482. The resistances in Wheat-
(b) the electron drift velocity is larger in the conductor of
stone’s Bridge circuit
length 2L.
shown in the Fig. 212 have
different values and the (c) The electrical field in the first conductor is twice that in
current through the galva- the second.
nometer is zero. If all ther- [National Standard Exam. in Physics 1990]
mal effects are negligible,
486. A copper strip AB and an iron strip AC are joined at
the current through the
A. The junction A is maintained at 0°C and the free
galvanometer will be zero,
when ends B and C are maintained at 100°C. There is a
Fig. 212
(a) the battery emf is doubled
potential difference between
(b) the battery and galvanometer are interchanged (a) the two ends of the copper strip
(c) all resistances in the circuit are doubled (b) the copper end and the iron end at the junction
(d) resistances R1 and R2 are interchanged. (c) the two ends of the iron strip
483. A current passes through a wire of nonuniform cross- (d) the free ends B and C.
section. Which of the following quantities are inde-
487. In a typical Wheatstone’s network, the resistances in
pendent of the cross-section ?
cyclic order are P = 10 ohm, Q = 5 ohm, S = 4 ohm
(a) the charge crossing in a given time interval
and R = 4 ohm. For the bridge to balance
(b) drift speed
(a) 5 ohm should be connected in parallel with Q
(c) current density
(b) 5 ohm should be connected in series with Q
(d) free-electron density.
202 COMPREHENSIVE OBJECTIVE PHYSICS

(c) 10 ohm should be connected in parallel with P


(d) 10 ohm should be connected in series with P. (b) 5 R/6

[Karnataka CET 2000]


488. Which of the following statements is/are correct ?
(a) In series combination of cells, current through external
resistance is maximum when total internal resistance is
less than external resistance.
(b) In parallel combination of cells, current through external
resistance is maximum when total internal resistance is
more than external resistance.
(c) R/2
(c) In mixed grouping of cells, current through external re-
sistance is maximum when total internal resistance of
battery is equal to the external resistance.
(d) In mixed grouping of cells, the maximum current through
nE mE
external resistance is given by or , where all the (d) R
2R 2r
letters have their usual meaning.
489. Electric current arises from the flow of charged par- Fig. 213
ticles. Now, [National Standard Exam. in Physics 1995]
(a) in metals it is dominantly due to flow of electrons 492. Fig. 214 shows a balanced Wheatstone net. The fol-
(b) in semiconductors, it is jointly due to flow of holes and lowing galvanometer currents appear when resist-
electrons ance changes are made (one change at a time) as de-
(c) in electrolytes, it is due to flow of negative ions only scribed
(d) in discharge tubes containing gases at low pressure it is (i) i1 when R1 is made 101 ohm
due to flow of positive ions only.
(ii) i2 when R2 is made 51 ohm
[National Standard Exam. in Physics 1992] (iii) i 3 when R 1 is made 101 ohm and R 2 is made
490. Electric current is due to flow of charge carriers in 51 ohm
the conductors. Which of the following is/are correct ? (iv) i4 when all resistors are made 2-fold and then R1
(a) The drift speed of charge carriers is a very small fraction raised 1% of new value.
of the mean thermal agitation speed of the same charge
carriers. R4 = 5 W R2 = 50 W
D
(b) The number of charge carriers per unit volume is always –
the same as the number of atoms of the conductor per R3 = 10 W + R1 = 100 W
unit volume. A C
B
(c) The drift velocity is proportional to the electrical field ap-
plied ordinarily.
(d) In an intrinsic semiconductor the charge carriers are ei-
ther electrons only or holes only ; both of them may not Fig. 214
participate in conduction.
Which of the following are correct ? (neglect battery
[National Standard Exam. in Physics 1994] resistance)
491. In the circuit elements given below, all individual (a) i1 is from A towards C
resistors are identical. The resistance between ter- (b) i2 has same magnitude as i1
minals P and Q in the different cases is
(c) i3 has same direction as i1
(d) i4 = i1.
[National Standard Exam. in Physics 1991]
(a) 3 R
CURRENT ELECTRICITY 203

Answers (Set III)


480. (a), (b), (d) 481. (a), (b), (d) 482. (a), (b), (c) 483. (a), (d) 484. (a), (b), (c)
485. (a), (c) 486. (a), (b), (c), (d) 487. (b), (c) 488. (a), (b), (c), (d) 489. (a), (b)
490. (a), (c) 491. (b), (c), (d) 492. (a), (d)

Solutions (Set III)


480. A metal at 0 K shall behave as an insulator. P R P R
487. (b) = ; =
481. Semi-conductors are non-ohmic. Q S Q′ S
482. If R1 and R2 are interchanged, then condition for bal- 10 4
=
anced Wheatstone Bridge will be violated. 5+5 4
483. Both the drift speed and the current density depend (c) If 10 Ω is connected in parallel with P, then P′ = 5
upon area. Ω.
484. The first observation means P much greater than X, P′ R
=
Q also much greater than X. So, (b) is true. The sec- Q S
ond observation means P not much greater than Y, 5 4
though Q is much greater than Y. This can only hap- = .
5 4
pen if Y is much greater than X (which is (a)), and 488. Refer to Synopsis.
also Q is much greater than P (which is (c)) 489. (a) true, dominantly
[Note : (d) could be true, but the data do not give (b) true for pure semiconductors.
clear evidence] (c) no, both + and – flow.
485. (a) Same current, same resistance. Hence potential (d) no, both + and – flow.
difference equal. 490. (a) true (b) ? not true in semiconductors. (c) That is
(b) Longer conductor will have larger cross-section area the basis of ohm’s law. (d) ? Both do participate ; in
(for same R), hence smaller current density (for fact in an intrinsic semiconductor, their numbers are
same i), which means smaller drift velocity. equal.
∴ incorrect. 491. (a) No. In parallels, R′ = R/3
Potential difference (b) First three give R/3 ; next two give R/2. These in
(c) E = . Hence statement is series lead to R′ = 5R/6.
Length
correct. (c) Upper two give 2R lower two also 2R. These in
486. (a) A and B are at different temperatures. parallel give R, and this with the third R leads to
R/2 OK.
(d) By symmetry, the mid-joining R would not carry
any current. So, R′ is only for 2R and 2R in paral-
lel, which is R.
492. (a) Potential fall across BC increases. So C is at lower
Fig. 215
potential than A. True.
(b) Contact potential difference (b) Change 1 in 50 is more than 1 in 100. So more
(c) Different temperatures current this time. False.
(d) B and C have different nature of material. (c) Effect of changing R2 will dominate. So flow oppo-
site to i1. False.
(d) Change 2 in 200 now is same as 1 in 100. PD
across AC same as for (1). True.
204 COMPREHENSIVE OBJECTIVE PHYSICS

MCQs
based on
SET IV

Typical Numerical Bank


(Exclusively for Engineering Entrance Tests)

Average time allowed per question is 60 seconds.

493. Twelve wires of equal resistance x are connected to (a) The Aluminium wire is 3.9 times longer.
form a cube. The effective resistance between two di- (b) The Steel wire is 1.9 times longer.
agonal ends will be (c) The Aluminium wire is 12.6 times longer.
5 6 (d) The Steel wire is 4.2 times longer.
(a) x (b) x
6 5 497. A metal wire of specific resistance 64 × 10 –6 Ω cm
(c) 3x (d) 12x. [RPET 1999] and length 198 cm has a resistance of 7 Ω. The radius
494. In an experiment to measure the internal resistance of the wire will be
of a cell by potentiometer, it is found that the bal- (a) 2.4 cm (b) 0.24 cm
ance point is at a length of 2 m when the cell is shunted (c) 0.024 cm (d) 24 cm.
by a 5 Ω resistance ; and is at a length of 3 m when 498. The circuit diagram shows a network of resistors,
the cell is shunted by a 10 Ω resistance. The internal each of resistance R.
resistance of the cell is
(a) 1.5 Ω (b) 10 Ω
(c) 15 Ω (d) 1 Ω. [KCEE 1996]
495. In the Fig. 216, the potential difference across the
cell

Fig. 217

What is the effective resistance between the points X


Fig. 216 and Y ?
(a) B will be zero (b) A will be zero 2R R
(a) (b)
(c) A and B will be 2 V 7 2
(d) A will be > 2 V and B will be < 2 V. 5R 2R
(c) (d)
8 3
496. Steel and Aluminium wires have equal resistance and
3R
mass. Which of the wires is longer and how many (e) .
4
times ? Given densities of Steel and Aluminium are
499. In the circuit shown, each of the resistors X and Y
7.8 × 10 3 kg m–3 and 2.7 × 103 kg m –3 and their
has resistance 6 Ω. The cell C has emf 12 V and in-
resistivities are 0.15 µ Ω m and 0.028 µ Ω m respec- ternal resistance 3 Ω.
tively.
CURRENT ELECTRICITY 205

Fig. 218 Fig. 221

What is the current in Y ? 503. The equivalent resistance between points A and B of
an infinite network of resistances each of 1 Ω con-
(a) 0.5 A (b) 1.0 A
nected as shown, is
(c) 2.0 A (d) 4.0 A
(e) 8.0 A.
500. Two wires P and Q, each of the same length and the
same material, are connected in parallel to a battery.
The diameter of P is half that of Q.
What fraction of the total current passes through P ?
(a) 0.20 (b) 0.33
Fig. 222
(c) 0.25 (d) 0.50.
(a) Infinite (b) 2 Ω
501. In Fig. 219, the resistance be-
tween points A and B is R. Both 1+ 5
(c) Ω (d) Zero.
the circle and the diameter are 2
A B
made of uniform wire of resist- [Kurukshetra CEE 1996]
ance 1 × 10 –4 ohm per metre. 504. A conducting liquid fills a cylindrical metal case to a
The length AB is 2 metre. The depth x as shown in the Fig. 223.
value of R is Fig. 219
2
(a) × 10 –4 ohm (b) 2π × 10 –4 ohm
3 metal rod
(c) 14.56 × 10 –4 ohm (d) 0.88 × 10 –4 ohm.
cylindrical
502. A wire of length l tapers uniformly from end P to end metal case
Q with the diameter at P twice that at Q.

insulating
base

Fig. 220 Fig. 223


A potential difference is applied across the ends of The resistance between the case and the metal rod is
the wire. Which graph represents the variation with (a) proportional to x2 (b) proportional to x
distance of the drift velocity v of the conduction elec-
(c) independent of x (d) inversely proportional to x
trons ?
(d) inversely proportional to x2.
505. The diagram shows a net-
work of three resistors. Two
of these marked R, are iden-
tical. The other one has a re-
sistance of 5.0 Ω. The resist-
ance between Y and Z is
found to be 2.5 Ω. What is
Fig. 224
the resistance between X and
Y?
206 COMPREHENSIVE OBJECTIVE PHYSICS

(a) 0.21 Ω (b) 0.53 Ω


(c) 1.9 Ω (d) 4.8 Ω
(e) 6.0 Ω.
506. A cell sends a current through a resistance R for
time t, next the same cell sends current through an-
other resistance r for the same time t. If the same
amount of heat is developed in both the resistances,
then the internal resistance of the cell is
R+r R−r
(a) (b)
2 2

Rr Fig. 226
(c) Rr (d) .
2
507. It is well known that resistance of the filament of an 509. A thermocouple is connected across a galvanometer
electric bulb changes with temperature. If an electric of resistance 30 Ω. One junction is immersed in wa-
bulb rated 220 volt, 100 watt is connected to a source ter at 373 K and the other in ice at 273 K. The emf of
of voltage 220 (0.8) volt, then the actual power will be the thermocouple is 90 µV for each 1 K difference in
(a) (100)(0.8) watt temperature between the junctions, and the thermo-
(b) (100)(0.8)2 watt couple resistance is 6 Ω.
(c) more than 100 × (0.8) watt but less than 100 watt What current will flow in the galvanometer ?
(d) more than 100 × (0.8)2 watt but less than 100 × 0.8 watt. (a) 1.8 µ A (b) 1.5 mA
(c) 250 µ A (d) 1.8 mA
[CPMT 1989]
(e) 300 µ A.
508. The diagram shows three resistors of resistances 4Ω,
10 Ω and 6 Ω connected in series. A potential differ- 510. In the diagram, the variable resistor R can be ad-
ence of 10 V is maintained across them, with point Q justed over its full range from zero to 107 Ω.
being earthed.

Fig. 227

What are the approximate limits for the resistance


Fig. 225
between P and Q ?
Which graph represents the change in potential along (a) zero and 104 Ω (b) 10 Ω and 104 Ω
the resistor network ?
(c) 10 Ω and 107 Ω (d) 10 Ω and 1011 Ω
(e) 104 Ω and 10 7 Ω.
CURRENT ELECTRICITY 207

Answers (Set IV)


493. (a) 494. (b) 495. (a) 496. (a) 497. (c) 498. (c) 499. (b) 500. (a)
501. (d) 502. (c) 503. (c) 504. (d) 505. (c) 506. (c) 507. (d) 508. (d)
509. (c) 510. (b)

Solutions (Set IV)


493. E = 6 IR l−3
Also, r= × 10
3
2I
8 7
Equating, 5(l − 2) 10(l − 3)
=
2 3
I 2I
or 3l – 6 = 4l – 12 or l = 12 – 6 = 6 cm
6−2
4
I
3 2I From equation (1), r = ×5Ω
6I 2
I 4
= × 5 Ω = 10 Ω.
2I 5 6
2
I 4 4
495. I= A= A
2I I I 0.2 + 0.3 + 0.1 0.6
1 40 20
2I
2 = A= A
6I 6 3
20
E Again, V=2– × 0.3 = 2 – 2 = 0.
3

l l ρl2
Fig. 228 496. R= ρ × or R =
a l V
Now, apply Kirchhoff’s laws to the mesh 1 – 2 – 6 – 5
m m
– 1 and 5 – 6 – 7 – 8 – 5. But d= or V =
V d
Aliter. The given network may be redrawn as under. 2
∴ R = ρl d
m
In the given problem, ρsls2ds = ρal lal2 dal
lal 2 ρ ds
or
2
= s
ls ρal dal
lal2 0.15 × 10−6 × 7.8 × 103
or = = 15.48
ls2
0.028 × 10−6 × 2.7 × 103
or lal or lal = 3.9 ls.
= 15.48
Fig. 229 ls
E–V l ρl ρl
494. r = r 497. R=ρ = or r2 =
V a πr 2 πR

l−2 64 × 10− 6 × 198 × 7


∴ r= ×5 …(1) = cm 2
2 22 × 7
r = 24 × 10–3 cm = 0.024 cm.
208 COMPREHENSIVE OBJECTIVE PHYSICS

498. The effective resistance across XY is determined as 12 V


3Ω 3Ω
follows :

12 V
6Ω

Fig. 231

12
I is thus given by, I = =2A
6
⇒ I
Current through Y is hence or 1.0 A.
2
500. Resistance of a wire is given by
l 4l
R=ρ =ρ
A πD2
or
5V
I = IQ + IP =
4R

wire Q wire P
V
Fig. 230 R 4R

499. The circuit may be redrawn as IQ = V


R IP = V
4R

Fig. 232

where ρ is its resistivity.


D is its diameter.
If resistance of wire Q is R, then resistance of wire P
R
is or 4 R.
(0.5)2
Thus when P and Q are connected in parallel to a
battery, fraction of total current through P is then
V V
IP 4R 4R 1
= = =
I V V 5V 5 .
+
R 4R 4R
CURRENT ELECTRICITY 209
501. Resistance of each semi-circle = π × 1 × 10 –4 Ω
I
Resistance of diameter = 2 × 1 × 10 –4 Ω
1 1 1 1
Now, use = + + .
R R 1 R2 R 3 I
a
502. I = nevA in the usual b
notation. Since current y
(I), number of electrons
per unit volume (n) and Dy
electron charge (e) are
x
constant quantities, vA
= constant = c and
Fig. 235
c Fig. 233
hence v = or drift t ∆y
A ∆R = ρ =ρ
velocity is inversely proportional to the cross-sectional A (2πy) x
area of the wire. Now radius (r) of the wire decreases ∴ Resistance, R = ∫ dR
linearly with distance x, from say 2r0 at P (corre-
sponds to x = 0), to r0 at Q (corresponds to x = 1). The
area thus decreases parabolicly from π(2r0)2 or 4πr02
ρ ∆y
a 2 πx y
=
ρ
2πx
=ln
b
azohm
b

Hence, the resistance R between the case and the metal


at point P to πr02 at point Q.
rod is inversely proportional to x.
Hence drift velocity of the electrons increases from
505. Given that the resistance between Y and Z is 2.5 Ω,
c c the resistance of R can thus be found where
v0 = to at the other and of the wire at
4 πr0 2 πr02 (R + R) // 5 = 2.5
point Q.
1 1 1
⇒ 2R // 5 = 2.5 ⇒ + =
R 2R 5 2.5
503. R = +1
R +1 ⇒ R = 2.5 Ω
The resistance between X and Y is Rxy where
Rxy = R // (R + 5) = 2.5 //7.5
(2.5)(7.5)
= = 1.825 ≈ 1.9 Ω.
2.5 + 7.5
506. Q = I2Rt
F E I Rt 2

Fig. 234
Q= GH R + r JK i

Q= G
F E I rt 2

H r + r JK
R 2
or R–1= or R – 1 = R
R+1 i

or R2 –R–1=0 R r
Equating, =
2
− (− 1) ± 1 + 4 1+ 5 (R + ri ) (r + ri )2
or R= Ω= Ω.
2 2 R(r + ri)2 = r(R + ri)2
504. If a source is connected across the metal rod and R (r + ri) = r (R + ri)
the metal case, current entering the metal rod from
the source will pass through the liquid and return- r R + ri R = R r + ri r
ing to the source through the metal case. Assume
or ri [ R − r ] = R r − r R
that the radius of the metal rod is a and the radius
of the metal case is b. or ri [ R − r ] = Rr [ R − r ]
For a thin cylindrical sheet of thickness ∆y, the in-
or ri = Rr .
cremental resistance is given by
210 COMPREHENSIVE OBJECTIVE PHYSICS

509. Emf of the thermocouple developed across the junc-


V 2 220 × 220
507. R= = Ω or R = 484 Ω tions
P 100
= (90 µV/K)(373 K – 273 K) = 9000 µV
(220 × 0.8)2 Total resistance across the junctions
P′ =
R = 30 Ω + 6 Ω = 36 Ω
On decreasing voltage, temperature decreases. Resist- Current registered by the galvanometer
ance decreases. Power increases. 9000
= µA = 250 µA .
36
10
508. Current I= = 0.5 A 510. When resistance of R is zero, we have
4 + 10 + 6
10 Ω 0Ω 10 Ω
10 Ω
≈ ≈
P Q P Q P Q
4
4 10 Ω
10 Ω

Fig. 237

When resistance of R is 107 Ω, we have

Fig. 236

Potential across 4 Ω resistor,


VPQ = 4 × 0.5 = 2 V ⇒ VP = 2 V
Potential across 10 Ω resistor
VR = 10 × 0.5 = 5 V ⇒ VR = – 5 V
Fig. 238
Potential across 6 Ω resistor,
VRS = 6 × 0.5 = 3 V ⇒ VS = – 8 V. Hence, the approximate limits for the resistance be-
tween P and Q are 10 Ω and 104 Ω.

KNOWLEDGE PLUS
l Eels are able to generate current with biological cells called + – 0.15 V 0.25 
electroplaques. The electroplaques in an eel are arranged in 100
rows, each row stretching horizontally along the body of the fish
containing 5000 electroplaques. The arrangement is suggestively 5000 electroplaques per row 100
shown in Fig. 239. Each electroplaques has an emf of 0.15 V and rows
internal resistance of 0.25 Ω.
The water surrounding the eel completes a circuit between the
head and its tail. If the water surrounding it has a resistance of
500 Ω, the current an eel can produce in water is about: + – + –
(a)1.5 A (b) 3.0 A
(c) 15 A (d) 30 A [AIIMS 2004]
500 
0.15 × 5000 750 750 Fig. 239
Solution. I = = A≈ A = 1.5 A
0.25 × 5000 500 + 12.5 500
500 +
100
So, (a) is the right choice.
CURRENT ELECTRICITY 211

SELF-EVALUATION TEST I
Based on UNIT XII

[Expected Questions for Forthcoming Examinations]

1. Three 4 ohm resistances are connected in the form of 7. In a conductor, 4 coulomb of charge flows for 2 sec-
a triangle. The effective resistance between any two ond. The value of electric current will be
corners is (a) 4 V (b) 4 A
(a) 8/3 Ω (b) 16 Ω (c) 2 A (d) 2 V.
(c) 64 Ω (d) infinite. 8. Three resistors are connected to form the sides of a
2. Three resistances of one ohm each are connected in triangle ABC. The resistance of side AB is 20 Ω, of
parallel. Such connection is again connected with side BC 30 Ω and of side CA 50 Ω. The effective re-
2/3 Ω resistor in series. The resultant resistance will sistance between points A and B is
be (a) 40 Ω (b) 16 Ω
5 3 (c) 32 Ω (d) 64 Ω.
(a) Ω (b) Ω
3 2 9. In the circuit shown, the electrical resistance between
2 points A and B is
(c) 1 Ω (d) Ω. [MP PMT 1985]
3
3. A cell is connected with an external resistance R.
Then its terminal voltage is
(a) always equal to its emf
(b) always less than its emf
(c) may be less or more depends on the direction of current
through resistance
(d) may be less or more, depends on the internal resistance of
cell whether it is less or more than the resistance R.
4. A metallic wire of 40 Ω resistance is drawn to double
its length. Its new resistance will be
(a) 20 Ω (b) 160 Ω
(c) 80 Ω (d) 320 Ω. [MP PET 1996]
5. Two cells of emfs
1.8 V and 2 V and
1.8 V, 0.6 Ω
internal resistances
0.6 Ω and 0.4 Ω are
connected as
shown in figure. 2 V, 0.4 Ω
What will be the
Fig. 241
reading of the ideal
voltmeter shown in V (a) 24 ohm (b) 12 ohm
Fig. 240 ? (c) 18 ohm (d) 9 ohm.
Fig. 240
(a) 4.8 V 10. In the network shown in Fig. 242,
(b) 1.92 V the ring has zero resistance. The
(c) 2.2 V (d) 2.8 V. equivalent resistance between the
6. On increasing the temperature of a conductor, its points A and B is
resistance increases because (a) 2 R (b) 4 R
(a) relaxation time decreases (c) 7 R (d) 10 R. Fig. 242
(b) mass of the electrons increases 11. The wire of the potentiometer has resistance 4 ohm
(c) electron density decreases and length 1 m. It is connected to a cell of emf 2 V
(d) none of the above.
212 COMPREHENSIVE OBJECTIVE PHYSICS

and internal resistance 1 ohm. The current flowing 18. Four resistors are connected as shown in Fig. 244.
in the potentiometer wire is The voltage between points A and B is 18 V. The
(a) 0.1 A (b) 0.4 A current in 6 Ω resistor is
(c) 0.2 A (d) 0.8 A.
12. The resistivity of a wire
(a) increases with the length of the wire
(b) decreases with the area of cross-section
(c) decreases with the length and increases with the cross-
section of wire
(d) none of the above. [MP PMT 1984]
13. A dry cell has an emf of 1.5 V and an internal resist-
ance of 0.05 Ω. The maximum current obtainable Fig. 244
from this cell for a very short time interval is
(a) 1.7 A (b) 2.5 A
(a) 30 A (b) 300 A
(c) 0.8 A (d) 5 A.
(c) 3 A (d) 0.3 A. [KCEE 1996]
19. We have three conductors of 10 Ω each. To get a
14. In a neon gas discharge tube, 2.9 × 1018 Ne+ ions 15 Ω resistance using all the three we should connect
move to the right through a cross section of the tube (a) all the three in series
each second, while 1.1 × 1018 electrons move to the
(b) all the three in parallel
left in this time. The current is
(c) by connecting two of them in series and the third one in
(a) 1 A to the right (b) 0.288 A to the left
parallel with them
(c) 288 A to the left (d) 0.64 A to the right.
(d) by connecting two conductors in parallel and the third in
15. The reading of ammeter A in the circuit shown be- series.
low is 20. A cell of emf E is connected with an external resist-
ance R. The potential difference across cell is V. The
4Ω
internal resistance of cell will be
4V A (E – V)R (E – V)R
4Ω
(a) (b)
E V
4Ω
(V – E)R (V – E)R
(c) (d) . [MNR 1987]
V E
4Ω
21. Four arc lamps rated for a voltage of 40 V and a
current of 15 A each are connected in series with one
Fig. 243 another and a resistor in a circuit at a voltage of
220 V. The resistance of the resistor is
(a) 0.25 A (b) 0.50 A
(a) 140 Ω (b) 9 Ω
(c) 0.75 A (d) 4 A.
(c) 112 Ω (d) 4 Ω.
16. Sensitivity of potentiometer can be increased by
22. The value of current I in the given circuit is
(a) increasing the emf of the cell
(b) increasing the length of the potentiometer wire
(c) decreasing the length of the potentiometer wire
(d) none of the above.
17. N identical cells are connected to form a battery.
When the terminals of the battery are joined directly
(short-circuited), current I flows in the circuit. To
obtain the maximum value of I
(a) all the cells should be connected in series
(b) all the cells should be connected in parallel
(c) two rows of N/2 cells each should be connected in parallel Fig. 245
(d) N rows of N cells each should be connected in paral- (a) 3 A (b) 13 A
lel, given that N is an integer. (c) 23 A (d) – 3 A.
CURRENT ELECTRICITY 213
23. All the edges of a block with parallel faces are un- (a) 20 W (b) 60 W
equal. Its longest edge is twice its shortest edge. The (c) 100 W (d) 200 W.
ratio of the maximum to minimum resistance be- 31. What is immaterial for an electric fuse wire ?
tween parallel faces is
(a) Length (b) Diameter
(a) 1.5 (b) 4
(c) Specific resistance (d) Current flowing through it.
(c) 7.5
32. In antimony-bismuth thermocouple, the direction of
(d) Data is inadequate because the length of the third edge is
thermoelectric current is from
not specified.
(a) Antimony to bismuth at cold junction
24. The equivalent resistance between the points X and
(b) Bismuth to antimony at cold junction
Y of the following Fig. 246 is
(c) Antimony to bismuth at both junctions
(d) Bismuth to antimony at both junctions.
33. n identical cells, each of emf E and internal resist-
ance r, are joined in series to form a closed circuit.
The potential difference across any one cell is
(a) zero (b) E
E n−1
(c) (d) E.
Fig. 246 n n
34. The resistance between points A and B in the circuit
(a) 10 Ω (b) 22 Ω
is
(c) 20 Ω (d) 50 Ω.
25. The resistance of 20 m long wire is 5 ohm. The wire
is stretched to a uniform wire of 40 m length. The
resistance now will be
(a) 5 Ω (b) 10 Ω
(c) 20 Ω (d) 200 Ω.
26. How much electrical energy in kilowatt hour is con-
sumed in operating ten 60 W bulbs for 10 hours per
day in a month of 30 days ?
(a) 18 (b) 180
(c) 1800 (d) 1800. Fig. 247
27. An electric bulb is rated 220 V and 200 W. Power 17
consumed by it when operated at 110 V is (a) 19 Ω (b) Ω
3
(a) 25 W (b) 50 W
23
(c) 75 W (d) 90 W. (c) Ω (d) 15 Ω.
7
28. Two heater wires of equal resistance are first con- 35. If the resistivity of a potentiometer wire be ρ and
nected in series and then in parallel. The ratio of the area of cross section be A, then what will be the po-
heats produced in the two cases is tential gradient along the wire ? Given : I is the cur-
(a) 1 : 2 (b) 1 : 3 rent through the potentiometer wire.
(c) 2 : 1 (d) 1 : 4. (a) Iρ/A (b) I/ρA
29. A toaster produces more heat than a light bulb when (c) IA/ρ (d) IAρ.
they are connected in parallel. Which of the follow- 36. In the network shown below, the equivalent resist-
ing statements is true ? ance between A and B is
(a) Toaster has more resistance than the light bulb.
(b) Toaster has less resistance than the light bulb.
(c) Both have equal resistance.
(d) Toaster will not be in working order after some time.
30. Two electric lamps of 100 W each are connected in
parallel. The power consumed by the combination
will be Fig. 248
214 COMPREHENSIVE OBJECTIVE PHYSICS

(a) R/2 (b) R


(c) 2R (d) 4R.
37. When a current passes through the junctions of two
different metals, evolution or absorption of heat at
the junction is known as
(a) Joule effect (b) Seebeck effect
(c) Peltier effect (d) Thomson effect. Fig. 249
38. The carrier density (number of free electrons per m3) (a) 5 kΩ (b) 9 kΩ
in metallic conductors is of the order of (c) 13 kΩ (d) 18 kΩ.
(a) 1010 (b) 1016 45. The equivalent resistance between a and b of the cir-
(c) 1022 (d) 1028. cuit shown in the Fig. 250 is
39. Two identical heaters, each marked 1000 W, 250 V
are placed in parallel with each other. These are con-
nected to a 250 V supply. Their combined rate of heat-
ing will be
(a) 250 W (b) 500 W
(c) 1000 W (d) 2000 W.
40. The heaters of Q. 39 are connected in series with
each other and then connected to a 250 V supply.
Their combined rate of heating will be
(a) 250 W (b) 500 W
(c) 1000 W (d) 2000 W.
41. A silver voltameter of resistance 2 Ω and a 3 Ω resistor
are connected in series across a cell. If a resistance of
Fig. 250
2 Ω is connected in parallel with the voltameter, then
the rate of deposition of silver 1
(a) 1 Ω (b) Ω
(a) decreases by 25% (b) increases by 25% 2
(c) increases by 37.5% (d) decreases by 37.5%. 1 1
(c) Ω (d) Ω.
[EAMCET 2001] 3 4
42. Resistance of conductor is doubled keeping potential 46. For the network shown in Fig. 251, the effective re-
difference across it constant. The rate of generation sistance between points P and Q is
of heat will
(a) become one-fourth (b) be halved
(c) be doubled (d) become four times.
[AIEEE 2002]
43. Two cells having the internal resistances 0.2 Ω and
0.4 Ω are connected in parallel. The voltage across
the battery terminals is 1.5 V. The emf of one cell is
1.2 V. The emf of the second cell is
(a) 2.1 V (b) 2.7 V Fig. 251
(c) 3 V (d) 4.2 V. [WB JEE 1994] (a) 15 Ω (b) 40 Ω
44. Two series connected lamps L1 and L2 are placed in (c) 60 Ω (d) 80 Ω.
circuit with voltmeters shown in Fig. 249. The first [National Standard Exam. in Physics 2000]
reads 6.0 V, the second 20 V. The resistance of the
47. In the circuit shown, when R is removed, an addi-
first voltmeter is 4 kΩ. The resistance of the second tional resistance of 72 ohm must be introduced in
voltmeter is nearly
CURRENT ELECTRICITY 215
series with the battery in order to keep the current 49. A potential difference of
through 30 ohm resistance unaltered. Hence R is 220 V is maintained across
a 12000-ohm rheostat, as
shown in the Fig. 253.
The voltmeter has a resist-
ance of 6000 ohm and
point c is at one-fourth of
the distance from a to b.
Therefore the reading of Fig. 253
the voltmeter will be
(a) 32 V (b) 36 V
(c) 40 V (d) 42 V.
[National Standard Exam. in Physics 2002]
Fig. 252
50. A voltmeter of resist-
(a) 15 ohm (b) 18 ohm ance 9800 Ω has a
(c) 20 ohm (d) 21 ohm. least count of 0.2 volt
[National Standard Exam. in Physics 2002] and range (0 – 10)
volt. Instead of con-
48. A copper wire and an iron wire, each having an area necting the voltmeter
of cross section A and lengths L1 and L2 are joined in parallel with one of
end to end. The copper end is maintained at a poten- the 100 Ω resistors, by
tial V1 and the iron end at a lower potential V2. If σ1 mistake, a student
and σ2 are the conductivities of copper and iron re- connected it in series
Fig. 254
spectively, the potential of the junction will be as shown in Fig. 254.
(a) (σ1V1 + σ2V2)/(σ1 /L1 + σ2 /L2) Which of the following statements is correct ?
(a) The voltmeter will get damaged.
(b) (σ1V1 /L1 + σ2V2 /L2)/(σ1 /L1 + σ2 /L2)
(b) The voltmeter will read zero.
(c) (σ1 /L1 + σ2 /L2)/(σ1V1 + σ2V2)
(c) The voltmeter will read 9 volt.
(d) (σ1V1 – σ2V2)/(σ1 /L1 – σ2 /L2).
(d) The voltmeter will read 8.8 volt.
[National Standard Exam. in Physics 2002]
[National Standard Exam. in Physics 2000]

Answers
1. (a) 2. (c) 3. (b) 4. (b) 5. (b) 6. (a) 7. (c) 8. (b)
9. (d) 10. (a) 11. (b) 12. (d) 13. (a) 14. (d) 15. (c) 16. (b)
17. (b) 18. (a) 19. (d) 20. (b) 21. (d) 22. (b) 23. (b) 24. (a)
25. (c) 26. (b) 27. (b) 28. (d) 29. (b) 30. (d) 31. (a) 32. (a)
33. (a) 34. (b) 35. (a) 36. (a) 37. (c) 38. (d) 39. (d) 40. (b)
41. (d) 42. (b) 43. (a) 44. (c) 45. (c) 46. (a) 47. (c) 48. (b)
49. (c) 50. (d)
216 COMPREHENSIVE OBJECTIVE PHYSICS

Solutions
E1 − E2
I=
r1 + r2
1. V = E1 – Ir1
E1 − E2
= E1 − r1
r1 + r2

Fig. 255 E1r1 + E1r2 − E1r1 + E2r1


=
The equivalent of the above r1 + r2
8Ω
network is as follows : E1r2 + E2r1 Fig. 259
=
8×4 32 r1 + r2
R= Ω or R = Ω a b
8+4 12 4Ω Again, V = E2 + Ir2
8 Fig. 256 E1 − E2
or R= Ω. V = E2 + r2
3 r1 + r2
E2r1 + E2r2 + E1r2 − E2r2
or V=
r1 + r2
2.
E1r2 + E2r1
or V=
r1 + r2

6. When the temperature is increased, the average time


Fig. 257 between successive collisions decreases.

The equivalent of the above network is as follows : 1


R∝ .
τ
4C
7. I= =2A
2s
Fig. 258 8. The given network in Fig. 260 is a parallel combina-
tion of 20 Ω and 80 Ω (Fig. 261).
FG 2 + 1 IJ Ω = 3 Ω = 1 Ω.
R=
H3 3 K 3
3. When current flows in the direction of emf, V < E.
l
4. R= ρ
a
2l
R′ = ρ
a/2

R′ = 4 ρ
LM l OP
N aQ Fig. 260 Fig. 261

R′ = 4R = 4 × 40 Ω = 160 Ω. 20 × 80 1600
R= Ω= Ω = 16 Ω.
E1r2 + E2r1 20 + 80 100
5. V= 9. The upper and lower units of the network are bal-
r1 + r2
anced Wheatstone Bridges. So, cross-piece resistances
1.8 × 0.4 + 2 × 0.6 of 12 Ω are not to be considered. So, the given net-
or V= volt
0.4 + 0.6 work is redrawn as under.
= (0.72 + 1.2) volt = 1.92 volt.
CURRENT ELECTRICITY 217
14. n = 2.9 × 1018 + 1.1 × 1018
n = 4 × 1018
q ne
Now, I= =
t t
4 × 1018 × 1.6 × 10−19
I= A
1
I = 0.64 A
The direction of this current is in the direction of flow
of positive charges i.e., to the right.
15. The given circuit may be re-
drawn as under :
Total resistance
Fig. 262 FG 4 + 4 IJ Ω = 16
Using symmetry considerations, the above network
=
H 3K 3

may be redrawn as follows : 4 3


I= A = A = 0.75 A.
16 4 Fig. 265
3
16. Longer the wire of the potentiometer, smaller is the
potential gradient and more is the sensitivity of the
potentiometer.
17. When external resistance is negligible as compared
to internal resistance, the cells should be connected
in parallel to get maximum current.
1 1 1 1
18. = + +
Fig. 263
R p 4 6 12

It is equivalent to a parallel combination of 18 Ω and 1 18 + 12 + 6 36


or = = or Rp = 2 Ω.
18 Ω. So, the net resistance is 9 Ω. Rp 72 72
10. The equivalent of the given network is as under :
Voltage across parallel combination
2 36
= × 18 volt = volt
2 + 1.5 3.5
Current through 6 Ω
36 6
=A= A = 1.7 A.
3.5 × 6 3.5
19. The combined resistance of parallel combination is
5 Ω.
Fig. 264
10 Ω
It is equivalent to a series combination of R and R. So,
the equivalent resistance is 2 R.
2V 10 Ω
11. I= = 0.4 A
5Ω 10 Ω
12. Resistivity does not depend upon geometrical dimen-
sions. Fig. 266

Now, we have a series combination of 5 Ω and 10 Ω.


E 1.5 V
13. I= = = 30 A So, the total resistance is 15 Ω.
r 0.05 Ω
218 COMPREHENSIVE OBJECTIVE PHYSICS

20. V = IR, E = I(R + r), 25. Length is doubled. Resistance will be increased by a
E R+r factor of 4.
=
V R 10 × 60 × 10 × 30
26. Energy consumed = kWh = 180 kWh.
r E r E 1000
or 1 + = or = −1 27. Voltage is halved. Power is reduced by a factor of 4.
R V R V
28. In the first case, resistance is 2R. In the second case,
E– V
or r= R R
V resistance is .
2
40 8 Q1 V 2 R/2 1
21. Resistance of each lamp = Ω= Ω = × =
15 3 Q 2 2R V 2 4
32 V2
Total resistance = R + 29. P=
3 R
R is less. P is more.
220
15 = 30. The powers are added up
32
R+
3 nE E
33. I = =
32 220 44 nr r
or R+ = =
3 15 3 E
V = E – Ir = E –r=E–E=0
44 − 32 12 r
or R= or R = Ω = 4 Ω.
3 3 34. The given network can be redrawn as follows :
22. Applying Kirchhoff ’s current law at the junction a,

Fig. 268

Thinking in terms of Wheatstone bridge, the resist-


ance of 4 Ω need not be considered. So, the network
Fig. 267 may be redrawn as under :
I1 + 8 = 15 or I1 = (15 – 8) A = 7 A
Applying Kirchhoff ’s current law at the junction c,
8 = I3 + 5 or I3 = 3 A
Applying Kirchhoff ’s current law at the junction b,
I2 = I1 + 3 = (7 + 3) A or I2 = 10 A
Fig. 269
Applying Kirchhoff ’s current law at the junction d,
we get Resistance of parallel combination of 8 Ω and 4 Ω is
I = I2 + I3 or I = (10 + 3) A = 13 A. 8×4 32 8
Ω i.e., Ω or Ω.
23. Let l, xl and 2l be the edges in increasing order. Here 8+4 12 3
1 < x < 2. 8
Now, we have a series combination of 2 Ω, Ω and
ρ(2 l) 2ρ 3
Rmax. = 2
= 1Ω.
xl xl So, the combined resistance is
Rmin. =
ρl
=
ρ
FG 2 + 8 + 1IJ Ω or FG 3 + 8 IJ Ω or 17
2xl 2 2xl
H 3 K H 3K 3
Ω.
R max. 2ρ 2 xl 4
Now, = × = V IR Iρl Iρ
Rmin. xl ρ 1 35. = = =
l l lA A
24. Balanced Wheatstone Bridge.
CURRENT ELECTRICITY 219
36. The given network is a parallel combination of three
resistances : R, 2R, 2R and R. V2
42. P=
R
1 1 1 1
= + + Since R is doubled therefore P will be halved.
R p 2R 2R R
R E1r2 + E2r1
1 1 1 2 43. V=
= + = or Rp = r1 + r2
Rp R R R 2
39. P = (1000 + 1000)W = 2000 W. 1.2 × 0.4 + E2 × 0.2
1.5 =
40. Voltage supply to each heater = 125 V 0.4 + 0.2
Power consumed by each heater . × 0.6 − 1.2 × 0.4
15
E2 = volt = 2.1 volt
100 0.2
= W = 250 W
4
Total power consumed E1 − E2
I=
= 250 W + 250 W = 500 W. r1 + r2
V = E1 – Ir1
If the supply voltage is half of the ‘voltage
printed on the device’, then power consumed by E1 − E2
= E1 − r1
the device is one-fourth of the ‘power printed on r1 + r2
the device’.
E1r1 + E1r2 − E1r1 + E2r1
=
41. r1 + r2 Fig. 272

E1r2 + E2r1
= …(i)
r1 + r2
Again, V = E2 + Ir2
E1 − E2
V = E2 + r2
r1 + r2
Fig. 270
E2r1 + E2r2 + E1r2 − E2r2 E r + E1r2
or V= = 21
E r1 + r2 r1 + r2
I=
5 Which is same as (i)
Note that one cell shows discharging action and
the other shows charging action.

6 20 80 40
44. = or R = kΩ = kΩ ≈ 13 kΩ.
4000 R 6 3
45. The given network is a parallel combination of three
resistances, each of value 1 Ω.
Fig. 271 1
∴ Ω
R=
E 3
I′ = 46. The given network is a parallel combination of 30 Ω
4
and 30 Ω. So, the equivalent resistance is 15 Ω.
E
Current through voltameter = 47. With 72 Ω in series, current through 30 Ω resistance
8
Percentage decrease in current 2 1
is = A.
E E 48 + 72 + 30 75
− 300
3 Potential difference across 30 Ω is 0.4 V.
= 5 8 × 100 = × 5 × 100 = = 37.5
E/5 5×8 8 ∴ Potential difference across 48 Ω is 1.6 V.
m 1.6 1
Since m = ZIt therefore is also 37.5%. ∴ Total current = A
t =
48 30
∴ Current through R
220 COMPREHENSIVE OBJECTIVE PHYSICS

FG 1 − 1 IJ A = 1 A FG 1 × 12000IJ Ω
=
H 30 75 K 50 49. Parallel combination of
H4 K and 6000 Ω

0.4 V gives 2000 Ω.


∴ R= = 20 Ω
(1/50) A 220
I= A
9000 + 2000
L1 L2
48. R1 = , R2 = ∴ Potential difference across voltmeter = 40 V.
σ1A σ 2A
50. The current in the circuit = 0.9 mA. Potential differ-
V = potential at the junction. ence across the voltmeter = 8.8 V.
V1 − V V – V2
= .
R1 R2

KNOWLEDGE PLUS
l Express which of the following setups can be used to verify Ohm’s law ?

A A V

V V
(a) (b) A (c) (d)
V A

Fig. 273

[IIT Screening 2003]


Solution. Since ammeter is always connected in series and voltmeter is always connected in parallel therefore (a)
is the right choice.

l In the shown arrangement of the experiment of the metre bridge if AC corresponding to null
deflection of galvanometer is x, what would be its value if the radius of the wire AB is doubled ? R1 R2
(a) x (b) x/4
G
(c) 4x (d) 2x [IIT Screening 2003]
A C B
Solution. If radius is changed, then there is no effect on ratio of resistances. So, balance x
point is not affected. Thus, (a) is the correct choice.
Fig. 274

l Two batteries, one of emf 18 volt and internal resistance 2 Ω and the other of emf 12 volt and
V
internal resistance 1 Ω, are connected as shown in Fig. 275. The voltmeter V will record a
reading of :
(a) 30 volt (b) 18 volt 2W
(c) 15 volt (d) 14 volt. [All India PM/PD 2005] 18 V

6 1W
Solution. Net emf = 6 V ; Total resistance = 3 Ω ; Current, I = A =2A
3 12 V
For 18 V cell, V = (18 – 2 × 2) volt = 14 volt Fig. 275
For 12 V cell, V = (12 + 2 × 1) volt = 14 volt
So, (d) is the right choice.
CURRENT ELECTRICITY 221

SELF-EVALUATION TEST II
Based on UNIT XII

DIRECTIONS :
(i) MCQs 1 to 24 have one correct alternative.
(ii) MCQs 25 to 30 have more than one correct alternative.
(iii) MCQs 31 to 35 have one or more than one correct alternative.

1. In the circuit shown in Fig. 276, the current between


B and D is zero. The unknown resistance is

Fig. 278
(a) 1 A (b) 2 A
(c) 3 A (d) 4 A.
4. V-I graphs for a conductor
(Platinum wire) at tempera-
tures T 1 and T 2 are as
shown in Fig. 279. T2 – T1
is proportional to
Fig. 276
(a) cos 2θ
(a) 4 Ω (b) 2 Ω (b) sin 2θ
(c) 3 Ω (c) cot 2θ
(d) emf of a cell is required to find the value of X. (d) tan 2θ. Fig. 279
2. The potential difference across 8 Ω resistance is 48 V 5. When the external resistance of a circuit is 10 Ω, the
as shown in the Fig. 277. The value of potential dif- current in it is 10 A. When the resistance becomes
ference across X and Y points will be
20 Ω, the current drops to 8 A. The external resist-
X 3Ω ance corresponding to a 4 A current is
(a) 70 Ω (b) 62 Ω
(c) 58 Ω (d) 96 Ω. [WB JEE 2001]
20 Ω 30 Ω 60 Ω
6. The resistance of a wire is 20 ohm. It is so stretched
that the length becomes three times. Then the new
24 Ω 8Ω 48 V resistance of the wire will be
(a) 6.67 Ω (b) 60.0 Ω
(c) 120 Ω (d) 180.0 Ω. [MP PET 1989]
Y 1Ω 7. The potential difference V across a filament lamp is
Fig. 277 related to the current I by V = 2I + 8I 2. The lamp is
connected in one arm of a Wheatstone bridge and
(a) 160 V (b) 128 V other arms are each of constant resistance 4 Ω . The
(c) 80 V (d) 62 V. potential difference which must be applied to the
3. Six resistors, each of 10 Ω, are connected to a 5 volt bridge so that it is balanced is
battery as shown in the Fig. 278. The reading of am- (a) 1 V (b) 2 V
meter is (c) 4 V (d) 8 V.
222 COMPREHENSIVE OBJECTIVE PHYSICS

8. The resistance across two opposite faces of a cube of


side 2 cm is 2 × 10–6 ohm. The specific resistance of
its material in ohm-cm is
(a) 10 –6 (b) 2 × 10 –6
1
(c) 4 × 10 –6 (d) × 10 –6.
2
9. Which of the following is correct ?
(a) Ammeter has low resistance and is connected in series.
(b) Ammeter has low resistance and is connected in parallel.
(c) Voltmeter has low resistance and is connected in parallel.
Fig. 281
(d) None of the above.
(a) 0.62 Ω (b) 0.36 Ω
10. Two conductors when connected in series give 27 Ω,
(c) 0.18 Ω (d) 0.031 Ω.
and in parallel 6 Ω. The two resistances are
(a) 21 Ω, 6 Ω (b) 9 Ω, 18 Ω 16. In Q. 15, the resistance of ‘f b g f ’ branch is
(c) 24 Ω, 3 Ω (d) 15 Ω, 12 Ω. (a) 0.07 Ω (b) 0.37 Ω
(c) 0.47 Ω (d) 0.97 Ω.
11. Resistances of 6 ohm each are connected as shown in
Fig. 280. With the current 0.5 A as shown in figure, 17. In Q. 15, the resistance of ‘a f b g c h’ branch is
the potential difference VP – VQ is (a) 0.37 Ω (b) 0.47 Ω
(c) 0.62 Ω (d) 1.94 Ω.
18. In Q. 15, the equivalent resistance between a and h is
(a) 0.62 Ω (b) 0.97 Ω
(c) 0.47 Ω (d) 1.94 Ω.
19. Fig. 282 shows an infinite wire grid with square cells.
Resistance of each side of the square is 1 Ω. The re-
Fig. 280 sistance between a and b is

(a) 5.6 volt (b) 4.8 volt


(c) 3.0 volt (d) 1.3 volt.
12. The power of a heater is 500 W at 800°C. What will be
its approximate power at 200°C ? Given α = 4 ×
10–4/°C.
(a) 484 W (b) 672 W
(c) 526 W (d) 620 W.
13. The same mass of copper is drawn into two wires 1
mm thick and 3 mm thick. Two wires are connected
in series. The current is passed. The heat produced
in the wires is in the ratio
(a) 3 : 1 (b) 9 : 1
Fig. 282
(c) 81 : 1 (d) 1 : 81.
(a) 4 Ω (b) 2 Ω
14. A house is fitted with 5 tube lights, each of 40 W. If
(c) 1 Ω (d) 0.5 Ω.
all of them are lighted for 20 hours and if the cost of
20. The charge through a conductor of resistance r var-
electrical unit is Rs. 2, then the total cost of electric-
ies with time as
ity consumed is
(a) Rs. 8 (b) Rs. 10 q = at – bt2
(c) Rs. 16 (d) Rs. 32. where a and b are positive constants. The heat pro-
15. Fig. 281 shows a five-pointed regular star made duced in the conductor is
from a uniform wire. The resistance of f j arm is
[cos 72° = 0.31]
CURRENT ELECTRICITY 223
(a) The battery supplies EI power.
a3 r a3r
(a) (b) (b) Heat is produced in R at the rate EI.
9b 6b
a3r a3r FG R IJ .
(c)
3b
(d)
b
. (c) Heat is produced in R at the rate EI
H R + rK
21. Space between two concentric FG r IJ .
spheres of radii a and b(> a) is
filled with a material of resis-
(d) Heat is produced in the battery at the rate EI
H R + rK
26. The constants a and b for the pair silver–lead are
tivity ρ. The resistance be-
2.50 µV/°C and 0.012 µV/(°C)2 respectively. For a
tween the inner and outer sur-
silver–lead thermocouple with colder junction at 0°C,
faces of the material is
(a) there will be no neutral temperature
ρ b−a
(a) (b) there will be no inversion temperature
4 π ab
(c) there will not be any thermo-emf even if the junctions are
Fig. 283
ρ ba kept at different temperatures
(b)
4π b − a (d) there will be no current in the thermocouple even if the
junctions are kept at different temperatures.
4π ab 4π b − a
(c) (d) . 27. The resistance of a semiconductor decreases rapidly
ρ a −b ρ ba
with increasing temperature. The main factor con-
22. A wire of resistance R is bent into a tributing to this effect is the rapid increase, with in-
circular ring of radius r. Equivalent creasing temperature, of
resistance between two points A and
(a) the speed of the random motion of the free charge carriers.
B on its circumference, when ∠AOB
(b) the concentration of the free charge carriers.
is θ, is given by
(c) the drift velocity of the free charge carriers.
Rθ Rθ
(a) ( 2π − θ ) (b) Fig. 284 (d) the frequency of vibration of the atoms of the semicon-
4π2 4 π2 ductor.
Rθ Rθ FG π − θIJ . (e) the amplitude of vibration of the atoms of the semicon-
(c)
π2
( π − θ) (d)
4π2 H2 K ductor.

23. Four resistances carrying a current as shown are 28. The room-temperature resistivity of a typical semi-
immersed in a box containing ice at 0°C. How conductor is several orders of magnitude greater than
much ice must be put in the box every 10 minute that of a typical metal. This is because
to keep the average quantity of ice in the box (a) the drift velocity of charge carriers in a semiconductor is
constant ? Given : J = 4 joule/cal and latent heat very much less than in a metal.
of ice = 100 cal g –1. (b) charge carriers collide with the lattice much more fre-
quently in a semiconductor than in a metal.
(c) the number density of charge carriers is much less in a
semiconductor than in a metal.
(d) in a semiconductor, the effect of electrons flowing in one
direction is almost cancelled by the flow of holes in the
Fig. 285 other ; in a metal, there are only electrons to be considered.
(a) 0.5 kg (b) 0.75 kg (e) impurities are deliberately introduced into semi-
(c) 1 kg (d) 1.5 kg. conductors, and these increase the resistivity.
24. A 2 V potentiometer is used to determine the inter- 29. Two resistors having equal resistances are joined in
nal resistance of a 1.5 V cell. The balance point of the series and a current is passed through the combina-
cell in the open circuit is 75 cm. When a resistor of tion. Neglect any variation in resistance as the tem-
10 Ω is connected across the cell, the balance point perature changes. In a given time interval,
shifts to 60 cm. The internal resistance of the cell is (a) equal amounts of thermal energy must be produced in the
(a) 2.5 Ω (b) 3.5 Ω resistors
(c) 4.5 Ω (d) 9.5 Ω. (b) unequal amounts of thermal energy may be produced
25. A battery of emf E and internal resistance r sends a (c) the temperature must rise equally in the resistors
current I through an external resistance R. Which of (d) the temperature may rise equally in the resistors.
the following is correct ?
224 COMPREHENSIVE OBJECTIVE PHYSICS

30. The charge flowing in a conductor varies with time as What is the value of the direct current I when the
1 2 1 3 current Ig through the milliammeter is 8 mA ?
q = at – bt + ct , (a) 8 mA (b) 14 mA
2 6
where a, b, c are all positive constants. Now, the cur- (c) 20 mA (d) 24 mA
rent (e) 40 mA.
(a) has an initial value a 34. Three resistors of resistances 1 Ω , 2 Ω and 3 Ω re-
(b) attains a minimum value after time b/c spectively are used to make the combinations X, Y
(c) attains a maximum value after time b/c and Z as shown in the diagrams.
(d) has zero value at all times.
31. Two electric bulbs A and B are designed for the same
voltage. Their power ratings are PA and PB respec-
tively, with PA > PB . If they are joined in series across
a power supply
(a) A will draw more power than B
(b) B will draw more power than A
(c) A and B will draw the same power
(d) the ratio of powers drawn by them will depend on the
applied voltage. Fig. 287

32. For a household electric circuit, mark the false state- Which of the following gives the combinations in or-
ments der of increasing resistance ?
(a) All electric appliances drawing power are joined in parallel (a) XYZ (b) XZY
(b) A switch may be either in series or in parallel with the (c) YXZ (d) ZXY
appliance which it controls (d) ZYX.
(c) If a switch is in parallel with an appliance, it will draw 35. Refer to the circuit shown in Fig. 288. Which of the
power when the switch is in the ‘off ’ position following is correct ?
(d) If a switch is in parallel with an appliance, the fuse will
blow off when the switch is put ‘on’.
33. The current that passes through a certain diode var-
ies with the potential difference across it as shown in
Fig. 286 (a) below. PQ is a straight line. Two such
diodes are conencted in parallel with a milliameter
of resistance 100 Ω , as shown in Fig. 286 (b).
current/mA
Q
10 Fig. 288
(a) The heat dissipated in 4 Ω resistance is more than that
dissipated in 12 Ω resistance.
(b) Maximum heat is dissipated in 3 Ω resistance.
P
(c) The heat dissipated in 12 Ω resistance is more than that
0 0.5 1.0 p.d./V A dissipated in 4 Ω resistance.
I Ig
100 Ω (d) The power dissipated in 3 Ω resistance is 3 W.
(a) (b)

Fig. 286
CURRENT ELECTRICITY 225

Answers
1. (b) 2. (a) 3. (b) 4. (c) 5. (a) 6. (d) 7. (b) 8. (c)
9. (a) 10. (b) 11. (c) 12. (d) 13. (c) 14. (a) 15. (a) 16. (c)
17. (d) 18. (b) 19. (d) 20. (b) 21. (a) 22. (a) 23. (c) 24. (a)
25. (a), (c), (d) 26. (a), (b) 27. (b), (d) 28. (b), (c) 29. (a), (d) 30. (a), (b) 31. (b) 32. (b)
33. (b) 34. (b) 35. (a), (b)

Solutions
X 1/ 2 1 5
1. = = 3. Current through each resistace = = 0.5 A
16 4 8 10
8 X = 16 Total current through four resistances to the right of
or X=2Ω ammeter
= 4 × 0.5 A = 2 A.
1 1 1 1 4. R2 – R1 = tan(90° – θ) – tan θ
2. = + +
R p 20 30 60 R2 – R1 = cot θ – tan θ
1 3 + 2+1 6 R0[1 + α t2] – R0[1 + αt1]
= =
Rp 60 60 cos θ sin θ
= −
or Rp = 10 Ω sin θ cos θ
24 × 8 cos2 θ − sin 2 θ 2 cos 2θ
Again, Rp′ = Ω =6Ω = =
24 + 8 sin θ cos θ sin 2θ
The given network may be redrawn as under : or R0 α (t2 – t1) = 2 cot 2θ
or (t2 – t1) ∝ cot 2θ
or (T2 – T1) ∝ cot 2θ
E
5. = 10
r + 10
E
=8
r + 20
r × 20 10 5
= =
r + 10 8 4
5r + 50 = 4r + 80
Fig. 289 E
r = 30 Ω, = 10 or E = 400 volt
Voltage per ohm 40
48 400
= V=8V Again, 4=
6 R + 30
Total resistance = 20 Ω or 4 R + 120 = 400 or 4 R = 280
Voltage between X and Y or R = 70 Ω.
6. 2
R′ = 3 × 20 Ω = 180 Ω
= 20 × 8 V = 160 V
7. V = 2 I + 8 I2
Note the special time-saving technique
used here. Va V LM OP LM OP
V
=2 a +8 a
2

Do not calculate current etc.


2 8 N Q N Q
8
226 COMPREHENSIVE OBJECTIVE PHYSICS

For exact value


P200(1 + 200 α) = P800(1 + 800 α)
P800 (1 + 800 α )
or P200 =
1 + 200 α
500(1 + 800 × 4 × 10 −4 )
= W
1 + 200 × 4 × 10 −4
= 611.11 W.
Fig. 290
Rt = R0(1 + αt)
1 1 Va 1 Va
or = + or = 1 1
2 4 8 4 8 = (1 + αt )
R 0 Rt
or Va = 2 volt.
l V 2 V2
8. R=ρ = (1 + αt)
a R0 Rt
P0 = Pt(1 + αt)
R×a 2 × 10−6 × 2 × 2
ρ= = Ω cm Strictly speaking, P0 corresponds to 0°C only.
l 2
= 4 × 10 –6 Ω cm
l
9. Factual information. 13. R= ρ
a
10. R1 + R2 = 27 …(i)
l×a
R1R2 R1R2 P= ρ
= 6 or =6 a×a
R1 + R2 27
V
or R1R2 = 162 P= ρ
a2
R1 – R2 = (R 1 + R 2 )2 − 4R 1R 2 ρV × 16
R=
π 2D4
= 272 − 4 × 162
1
R∝
= 729 − 648 = 81 = 9 …(ii) D4
Adding (i) and (ii), 2R1 = 27 + 9 = 36 R1 3 × 3 × 3 × 3 81
= =
or R1 = 18 Ω R2 1×1 1
Now, 18 + R2 = 27 or R2 = 27 – 18 = 9 Ω. Now, Q = I2Rt
11. Parallel combination of 6 Ω, 6 Ω and 6 Ω gives 2 Ω. Since wires are connected in series therefore current
12 × 6 is same.
Parallel combination of 12 Ω and 6 Ω gives Ω
12 + 6 ∴ Q ∝ R
72 Q1 81
or Ω or 4 Ω. ∴ = .
18 Q2 1
Total resistance =2Ω+4Ω=6Ω
5 × 40 × 20
V = 0.5 × 6 volt = 3 volt. 14. Energy consumed in kWh = =4
1000
12. P800 = 500 W cost = 4 × Rs. 2 = Rs. 8.
P200 = ? 15. Resistance of f j arm = 2 × 1 × cos 72° Ω
α = 4 × 10–4 /ºC = 2 × 0.31 Ω = 0.62 Ω.
P200 = P800(1 + 600 α) 16. Resistance of f b g f branch is the same as the resist-
= 500[1 + 600 × 4 × 10–4] W ance of parallel combination of 2 Ω and 0.62 Ω.
= 500[1 + 0.24]
2 × 0.62 1.24
= 500 × 1.24 W = 620 W Now, Ω = Ω = 0.47 Ω.
2 + 0.62 2.62
CURRENT ELECTRICITY 227
17. Resistance of a f b g c h branch a3r a 3r a 3r
= 1 Ω × 2 × 0.47 Ω = 1.94 Ω. = + −
2b 6b 2b
18. Note that the star is symmetrical about ah. So, the
resistance between a and h is a parallel combination =
a3rLM 1
1+ −1 =
a 3r OP
of 1.94 Ω and 1.94 Ω. This gives 0.97 Ω. 2b N 3 6b Q
19. If R is the equivalent resistance, then the given net- dl
work has the following equivalent network. 21. dR = ρ
4 πl 2

R=
ρ
4π z a
b
(l −2 )dl

R= −
ρ 1 1

LM OP
4π b a N Q
ρ b−a Fig. 293
R= .
4 π ab
R
22. Resistance/length =
2πr
Resistance of shorter section between A and B
Fig. 291
R Rθ
= (rθ) =
This is further equivalent to the following. 2 πr 2π
Resistance of longer section between A and B
R
=r (2π − θ)
2πr
Both sections are in parallel.
R θ(2π − θ)
∴ Resistance =
2π (θ + 2π − θ)
Fig. 292 R θ( 2π − θ ) Rθ
= = ( 2π − θ ) .
3R 2π 2π 4 π2

R +1 10 × 5
Now, R= 23. Total resistance =2× Ω
3R 10 + 5
1+
R +1 100 20
Ω= = Ω
3R R +1 15 3
or R= × or 4 R + 1 = 3
R + 1 4R + 1 20 1
Q = 10 × 10 × × 10 × 60 × cal = 105 cal
2 3 4
or 4 R = 3 – 1 or R = Ω = 0.5 Ω. 5 3
4 m = Q = 10 = 103 g = 10 kg = 1 kg
d
L 100 103
20. I= (q) = a – 2bt
dt l1 − l2
24. r= R
Clearly, current decreases with time. l2
a 75 − 60 15
When I = 0, t = r= × 10 = Ω = 2.5 Ω
2b 60 6

∴ Q=
z a / 2b
( a − 2 bt )2 r dt
25. Heat produced per unit time in R = I2R
E R

z
0
= (I)(I) R = I
R = EI
a / 2b R+r R+r
Q= (a2 r + 4 b2t 2 r − 4 abtr) dt
0 Heat produced per unit time in battery

= a2 r
LM a OP + 4b r × 1 LM a OP
2
3
LM OP
1 a E r
N 2b Q 3 N ab Q 2
− 4 ab r ×
N Q
2 2b
= I2r = I
R+r
r = EI
R+r
.
228 COMPREHENSIVE OBJECTIVE PHYSICS

a 2.50 V2 V2
26. θn = – =– = – 208.3°C 31. P= or R =
b 0.012 R P
Since PA > PB, ∴ RA < RB
2 × 2.50
Again, θi = – = – 416.3°C. In series combination, Q ∝ R
0.012
∴ QA < QB or QB > QA.
27. In a semiconductor, the increase in thermal energy of
32. Parallel connections and series switching.
the valence electrons due to temeperature rise enables
33. Potential difference across the milliammeter is (8
more of them to break the covalent bonds and become
mA)(100 Ω) or 0.8 V.
free electrons. Thus, more electron-hole pairs are
produced which can act as carriers of current. Hence, From the graph, we notice that the diode nearest to
the ammeter is reversed bias and hence, no current
the electrical resistance of a semiconductor decreases
passes through this diode. The diode further away
with temperature rise. from the ammeter is forward biased with potential
28. In semiconductors, there is a narrow forbidden band difference of 0.8 V. From the graph, it can be easily
between the valence and conduction band. At room determine that the current this diode draws is
temperature, only some valence electrons have suffi-
FG 0.8 − 0.5 IJ 10 = 6 mA. Hence, the total direct current
cient thermal energy to reach the conduction band
where they may become conduction electrons. H 1.0 − 0.5 K
through the diodes and the ammeter is 14 mA.
In metal, however, the valence and conduction bands
can overlap. The electrons in the overlapping region
of energy are conduction electrons. Since, there is a
larger number of conduction electrons in metals than 34.
in semiconductors, metals are thus good conductors
and have low resistivity.
29. Both I and R are same. So, Q is same. The rise in
temperature is to be determined by physical dimen- Fig. 294
sions. Since physical dimensions may or may not be
the same therefore the rise in temperature may or
may not be the same.
d 1
30. I= (q) = a – bt + × c × 3t2
dt 6
Fig. 295
ct2
= a – bt +
2
When, t = 0, I = a
dI c
= − b + × 2t = – b + ct
dt 2
For I to be maximum or minimum,
Fig. 296
dI
=0
dt 4 3 11
Now, Ω< Ω< Ω and hence, only XZY gives
b 3 2 5
∴ – b + ct = 0 or ct = b or t = the combinations in order of increasing resistance.
c
d 2I V2
Now, = c 35. For (a) think in terms of P = .
dt2 R
So, I is minimum. For (b), note that the whole of the current flows throgh
3 Ω.
UNIT XIII

MAGNETIC EFFECTS OF CURRENT


AND MAGNETISM

l Concept of magnetic field l Oersted’s experiment l Biot-Savart law l Magnetic field due
to an infinitely long current carrying straight wire and a circular loop l Ampere’s circuital law and
its application to straight and toroidal solenoids l Force on a moving charge in uniform magnetic
and electric fields l Cyclotron l Force on current-carrying conductor in a uniform magnetic
field l Forces between two parallel current-carrying conductors l Definition of ampere l Torque
experienced by a current loop in a uniform magnetic field l Moving coil galvanometer–its current
sensitivity and conversion to ammeter and voltmeter l Current loop as a magnetic dipole and its
magnetic dipole moment l Magnetic dipole moment of a revolving electron l Magnetic field
intensity due to magnetic dipole (bar magnet) along the axis and perpendicular to the axis l Torque
on a magnetic dipole (bar magnet) in a uniform magnetic field l Bar magnet as an equivalent
solenoid l Magnetic field lines l Earth’s magnetic field and magnetic elements l Para-, dia-
and ferro-magnetic substances with examples l Electromagnets and Permanent magnets

UNIT DETAILS

1. Synopsis Points 110 Plus

2. Illustrations 25 Plus

3. MCQs from Competitive Examinations 390 Plus

4. Self-Evaluation Tests 2

5. Total Number of MCQs 600 Plus

6. Total Number of Solutions 590 Plus

C-11\IITS\C13-1
CHAPTER 13

MAGNETIC EFFECTS OF CURRENT AND MAGNETISM

SYNOPSIS

(A) MAGNETIC EFFECTS OF CURRENT Idl sin θ


8. In cgs units, dB =
1. The source of electric field is electric charge (a scalar → r2
quantity), the source of magnetic field is current ele- 9. The direction of dB is the direction of the vector
→ →
ment (a vector quantity). dl × r . Therefore, at the observation point, the di-
2. Sources of magnetic field : →
rection of dB is perpendicular to the plane of the
(i) Magnets
paper and is directed inwards.
(ii) Current-carrying conductors
10. The magnetic field due to whole conductor is ob-
(iii) Moving charged particle →
tained by integrating dB over the entire length of
(iv) Variable electric field.
the conductor i.e. magnetic field due to the whole
3. The current passes through a conductor producing a conductor at the observation point is given by
magnetic field in the space surrounding it.

z z
4. The magnetic effect of the current was discovered by → →
→ → → µ0 I dl × r
Oersted in 1820. It was mathematically explained by B= dB or B =
Biot-Savart. 4π r3
5. The search for a magnetic monopole (something which
11. Biot-Savart law is an inverse square law and is the
may be called magnetic charge on the ‘lines’ of elec-
magnetic analogue of Coulomb’s law.
tric charge) has failed. All known magnetic fields have

z
been found to be produced either by electric currents →
→ 1 r
or by atomic magnetic moments associated with an- E = dq
gular moments of charged particles or by time-depend- 4 πε 0 r3
ent electric fields (produced by accelerated electric 12. Magnetic field at a point in the direction of current is
charges). zero.

6. The magnetic field is described by the vector B called 13. If we know the magnetic field at a point due to a cur-
magnetic induction, flux density or simply vector field. rent element, then we can vectorially add up the fields

due to different current elements which make up the
7. The magnetic field dB at a distance r from a current conductor. Thus, we can know the magnetic field B
→ at a point due to a current-carrying conductor.
element of length dl carrying a current I is given by
14. The equation of Biot-Savart law is analogous to the
→ → equation of the electric field for a point charge. The
→ µ0 I dl × r
dB = magnetic field obeys the inverse square law of dis-
4π r3 tance just like the electric field of a point charge. How-
µ0 ever, there is a difference between the two. The source
Here is exactly 10 –7 T mA–1. of the magnetic field is not ‘some counterpart of elec-

tric charge’. While the electric charge (source of elec-
tric field) is a scalar, the current element (source of

231

C-11\IITS\C13-1
232 COMPREHENSIVE OBJECTIVE PHYSICS

magnetic field in the problem under consideration) is “Look into the coil along B
a vector. its axis. If the current is
15. The direction of magnetic field depends on direction of flowing anti-clockwise,
current and magnitude depends on the magnitude of then the magnetic field is
current. towards the viewer. If
16. The magnetic field is produced by the motion of charge the current is flowing
clockwise, then the field B
only and not by stationary charges.
(a) (b)
17. In case of current carrying conductor, magnetic field is away from the viewer”.
Fig. 2
is computed either with the help of Biot-Savart law or 24. Magnetic lines of force due to the circular current-
Ampere’s circuital law. carrying loop are shown in Fig. 3.
18. The magnetic field at a distance r from a straight
current-carrying conductor of finite length is
d c
µ I
B = 0 [sin φ1 + sin φ2]
4 πr

I
r f2 a b
f1

Fig. 3

25. Magnetic field due to segment of I


a current loop
Fig. 1 µ 0 Iφ
B=
Caution. While using this equation, do not use sign 4π r
convention with φ1 and φ2. If the loop is semicircular,
i.e. φ=π f r
19. The magnetic field at a distance r from a straight
current-carrying conductor of infinite length is µ0 I
∴ B= P
µ I 4r Fig. 4
B= 0
2πr 26. When a charge q moves in a circular path of radius r
20. The direction of magnetic field due to a long straight with a speed v, it behaves as a current-carrying coil
current-carrying conductor is determined by Right and a magnetic field is produced at the centre of the
hand thumb rule or Maxwell’s cork crew rule. circle.
21. The magnetic field produced by a current-carrying µ 0 qv
conductor is circular. The magnetic lines of force are Magnetic field at centre = .
4π r2
concentric circles with the current-carrying conduc-
tor passing through their common centre. The plane 27. CURRENT LOOP IS A MAGNETIC DIPOLE
of the magnetic lines of force is perpendicular to the The face of the coil in
length of the conductor. which current appears
22. The magnetic field induction at the centre of a circu- to flow anticlockwise
lar current-carrying coil of N turns and radius r is acts as magnetic north
pole.
µ 0 NI (a) (b)
B= The face of the coil in
2r Fig. 5
which current appears
23. The direction of magnetic field at the centre of circu- to flow clockwise acts as magnetic south pole.
lar current-carrying coil is given by right hand palm
rule.
MAGNETIC EFFECTS OF CURRENT AND MAGNETISM 233

In both the cases, the magnetic field at the centre of per unit length of the solenoid and I is the current
the coil is along the normal to the plane of the coil. flowing through the solenoid.
28. The magnetic field at a point on the axis of a circular
1
current-carrying coil is given by At one end of the solenoid, B = µ 0n I .
2
µ 0 NIr 2
B= 30. If the solenoid is of finite length L and the point is on
2 (r 2 + x 2 )3 / 2 the perpendicular bisector of its axis, then
where N is the number of turns, r is the radius of the coil
and x is the distance of the observation point from the centre L
B = µ0 n I
of the coil. L + 4R 2
2

Special Cases :
Here, R is the radius of the solenoid
Case I. At the centre of the loop, x = 0
31. If the solenoid is of finite length and the point is on its
µ 0NIr 2 µ NI axis but near one end, then
∴ B= or B = 0
2r 3 2r
µ0 L
In terms of area A (= πr2) of the circular current loop, B= nI
the above result may be written as under : 2 L + R2
2

µ 0NIAµ 0 NIA µ 2NIA 32. The variation of field with distance along the axis of
B= = or B = 0
2r(πr ) 22π r 3 4 π r3 the solenoid is shown in Fig. 7.
The quantity NIA is known as the magnetic dipole
moment M of the current loop. B
µ 2M
∴ B= 0 3 m0nI
4π r
Case II. If the observation point is far far away from
the coil, then r << x. So, r2 can be neglected in comparison to
x 2.
µ 0NIr 2 – L/2 O L/2
∴ B=
2 x3 Fig. 7
µ NIA µ 0 2NIA
In terms of area, B = 0 = 33. The magnetic dipole moment of a current loop is given
2π x3 4 π x3
by M = NIA, where N is the number of turns, I is the
µ 2M
In terms of magnetic dipole moment, B = 0 3 current flowing through the coil and A is the face-
4π x
area of the loop.
The variation of B on the axis of the coil is shown 34. The Lorentz magnetic force experienced by a charge

here. q moving in uniform magnetic field B is given by
→ → →
B Fm = q ( v × B) .

| Fm | = Bqv sin θ
→ →
where θ is the smaller angle between v and B .
35. Magnetic forces are very much weaker than electro-
static forces.
x=0 x → → →
36. In the equation Fm = q ( v × B) , the angle between
Fig. 6 → → → → →
(i) Fm and v is 90° (ii) Fm and B is 90° (iii) Fm and
→ → → → →
29. The magnetic field induction due to a long current- plane of v × B is 90° (iv) Fm and v × B is zero, if q is
carrying solenoid at a point well within the solenoid positive.
is given by B = µ0nI, where n is the number of turns
234 COMPREHENSIVE OBJECTIVE PHYSICS

37. DIMENSIONAL EXPRESSION OF B direction of magnetic field. It will move with uniform axial
Fm speed of v|| i.e. v cos θ in the direction of the magnetic field.
Fm = Bqv sin θ or B = The superposition of uniform circular motion and
qv sin θ
uniform linear motion will make the path of the charged
[MLT −2 ] →
[B] = = [MA–1T–2] particle helical. The axis of the helix will be parallel to B .
[AT] [LT −1]
Pitch of helix = linear distance covered in a time equal
38. When a charged particle moves perpendicular to a to the period of revolution of the charged particle.
magnetic field, the kinetic energy remains unchanged
but the momentum changes. 2πm
= v|| × T = v cos θ ×
39. A PARTICLE OF MASS m AND CHARGE q IS Bq
PROJECTED INTO A UNIFORM MAGNETIC
→ → 40. MOVING CHARGE IN PERPENDICULAR MAG-
FIELD B WITH A VELOCITY v MAKING AN NETIC AND ELECTRIC FIELDS

ANGLE θ WITH B . The total Lorentz force acting on an electron moving
→ → →
Resolve v into two rectangular components : v|| (= v with velocity v in a uniform magnetic field B and a uni-
→ → → → →
cos θ) in the direction of the magnetic field and v⊥ ( = v sin θ) form electric field E are given by F = – e (E + v × B) .
in a direction perpendicular to that of the magnetic field. → →
If v and B are mutually perpendicular and if the

® plane containing them is itself perpendicular to E , then the
v → → →
® electric force – e E and the magnetic force – e ( v × B) act
v^ ®
q → →
B along the same line. If we adjust E and B in such a way
→ → →
® that E = – v × B , then the electron travels undeflected from
v||
its original direction.
→ →
Fig. 8 For given values of E and B , the above equation will

be satisfied only for a certain speed v such that
Due to v⊥ alone, the charged particle will experience E
a force Bqv⊥ sin 90° i.e. Bqv⊥. Since this force is perpendicu- v=
B
lar to the velocity vector therefore it can change only the →
direction of motion of the charged particle. Thus, the charged 41. The force experienced by a current element I dl placed
→ →
particle shall follow a circular path due to v⊥ alone. If r be in a magnetic field B is given by
the radius of the circular path, then → → →
dF = I (dl × B)
mv⊥ 2 → → →
Bqv⊥ = or F = I ( l × B) or F = BI l sin θ
r
(i) When sin θ = 0 or θ = 0° or 180°, the force on the
Bqr
or v⊥ = current element F = 0 i.e. a current element in a magnetic
m field does not experience any force if the current in it is
v⊥ Bq collinear with the field.
Angular velocity, ω = =
r m ®
B
ω Bq
Frequency, f= = ®
2π 2πm Idl
q = 0°
1 2 πm
Time period, T= = (a)
f Bq
So, the charged particle will complete one revolution q = 180°
®
2πm Idl
in time .
Bq (b)
→ →
Since the angle between and B is zero therefore
v|| Fig. 9
the charged particle will not experience any force in the
θ = 0 or 180°
F = 0 (minimum)
MAGNETIC EFFECTS OF CURRENT AND MAGNETISM 235

(ii) When sin θ = 1 or θ = 90°, the force on the current Weight of balancing mass m = mg
element will be maximum, F = IB dl i.e. the force on a cur-
µ 0 2I2l
rent element in a magnetic field is maximum In equilibrium position, = mg
( = BI dl) when it is perpendicular to the field. 4π r

or I=
LM 4 π mg r OP 1/2

B
N 2µ l Q
0

45. RELATIVE SIZES OF ELECTRICAL AND MAG-


NETIC FORCES
90°
1 q1q2
Fe = ...(1)
→ 4 πε 0 r 2
Idl
µ 0 I1I2
Fm = dl1 dl2 ...(2)
Fig. 10 4 π r2
θ = 90° But I1dl1 = q1v1
dF = IBdl (maximum) and I2dl2 = q2v2
Directed out of page. µ 0 q1v1q2v2
Fm = ...(3)
In this case, the direction of the force is given by 4π r2
Fleming’s Left Hand Rule. Dividing (3) by (1), we get
Statement. If the first Fm µ 0 q1v1q2v2 4 πε0 r 2
finger, the central finger and the = ×
thumb of left hand are stretched
Fe 4 πr 2 q1q2
mutually perpendicular to each Fm
other and the first finger points or = (µ0 ε0) v1v2
Fe
in the direction of the magnetic
field, the central finger in the Fm
or = c–2
direction of the current ; then the Fe
thumb points in the direction of Clearly Fm << Fe
the force on the current-carrying Fig. 11
46. (i) The torque experienced by a current-carrying loop
conductor [Fig. 11].
placed in magnetic field is given by :
42. The force per unit length between two parallel current-
τ = NBIA cos θ
carrying conductors separated by a distance r is
where N is the number of turns, B is the magnetic field
µ0 I1 I2 induction and I is the current flowing through the coil. Here
.
2πr θ is the angle between the plane of the loop and the direction
43. We can formulate the following two laws for force be- of magnetic field.
tween parallel currents : (ii) If θ represents the angle between the normal to
I. Two parallel current-carrying conductors attract the plane of the loop and the direction of magnetic field, then
each other if the currents are in the same direction and re- τ = NBIA sin θ.
pel each other if the currents are in opposite directions. → → →
II. The force between two parallel current-carrying (iii) τ = M × B
conductors is proportional to the product of the current 47. In the case of moving coil galvanometer,
strengths and the length of the conductor under considera- NBIA = kα
tion and varies inversely as the distance between them. where k represents the restoring couple per unit twist and α
44. AMPERE BALANCE OR CURRENT BALANCE represents the deflection of the coil.
If r be the separa- 48. Current sensitivity of a moving coil galvanometer :
tion between two current-
α NBA
carrying wires, then =
I I k
magnetic force on each wire
m 49. Voltage sensitivity of a moving coil galvanometer :
µ 0 2I2l I
r
= , where l is the α NBA
4π r =
Fig. 12 V kR
length of each wire
236 COMPREHENSIVE OBJECTIVE PHYSICS

50. In order to convert a galvanometer of resistance G


into an ammeter of range 0 – I, a low resistance called
shunt resistance S is connected in parallel with the
galvanometer.
GI g
S= I−I
g

where Ig is the current required for full scale deflec-


tion.
51. The resistance of ammeter is given by
GS
Ra = .
G+S
It is a low-resistance instrument.
52. Ammeter is always connected in series in a circuit.
Fig. 13
53. In order to convert a galvanometer of resistance G
into a voltmeter of range 0 – V, a resistance R is to be If t is the time taken by the ion to describe a semi-
connected in series with the galvanometer. circle of radius r1, i.e., a distance of πr1, then
V = Ig (R + G).
πr1 πr1 × m πm
54. The resistance of the voltmeter is given by t= or t = or t=
Rv = R + G v1 Bqr1 Bq
It is a high-resistance instrument. It is clear from the above equation that the time taken
55. Voltmeter is always connected in parallel in a circuit. by the ion to describe a semi-circle depends only on two

56. According to ampere’s circuital law,


where I is the current enclosed.
z → →
B . dl = µ0I factors : (i) Magnetic field (B) ; (ii) Charge-to-mass ratio
of the ion. It is independent of speed of ion and radius of
FG IJ
m
q
H K
57. Magnetic field produced by current in a solid cylinder circle. We may conclude that for a given magnetic field, an
of infinite length : ion will traverse all semi-circles (irrespective of their radii)
(i) At an external point (r > a) in exactly the same time.
If T is the time period of the high frequency oscillator,
µ 0I
B= then for resonance,
2πr
T 2πm
where r is the distance of the observation point from the = t or T = 2t or T=
cylinder of radius a. 2 Bq
(ii) At an internal point (r < a) If ν is the frequency of the oscillator, then
µ 0Ir 1 Bq
B= ν= or ν=
2πa2 T 2πm
58. Inside a current-carrying toroid, the magnetic field is
This equation must be satisfied to achieve resonance.
µ 0NI
2πr Maximum Kinetic energy of the ion
59. CYCLOTRON Let, rm = Radius of the largest semi-circle described
Let us now consider a positive ion of mass m and by the ion

charge q. The magnetic field B acts perpendicular to the vm = Maximum velocity acquired by the ion i.e.
plane of the dees. So, the ion will follow a circular path within the velocity of the ion just before it leaves
the dee. If we consider a circular path of radius r1 being the dees.
traversed by the ion with velocity v1, then
mvm 2 Bqrm
Now, = Bqvm or vm =
mv12 Bqr1 rm m
= Bqv1 or v1 =
r1 m
MAGNETIC EFFECTS OF CURRENT AND MAGNETISM 237

Maximum kinetic energy a b

FG IJ
Y – – – – –
C–

2
1 1 Bqrm
– – – – –
d
mvm2 = m
H K
c
= ®
2 2 m O
®
+ vd I –
X Fm + VH
a´ b´
1 B2 q2 rm2 Z + + +
+ D+
+ + +
= + + + +

2 m c´

Number of complete revolutions made by the


ion Fig. 16
Let,
N = Number of complete revolutions made by Expression for Hall Voltage
the ion
EH
V = Potential difference to which the ion is sub- eEH = Bevd or vd =
jected every time it enters a dee. B
I = neAvd
Clearly, total energy acquired by the ion during N
A = bd
complete revolutions is 2NqV.
But this energy must be equal to the maximum ki- + + + + + + – – – – – –
netic energy of the ion. + + + + + + – – – – – –
eEH eEH
B2 q2 rm2 B2qrm2
∴ 2NqV = or N= b
2m 4 Vm EH – EH +
60. HALL EFFECT
Bevd Bevd
The production of transverse emf in a current-carry- – – – – – – + + + + + +
d
ing conductor when a magnetic field is applied perpendicu- – – – – – – + + + + + +
lar to the direction of current is known as Hall effect. (a) (b)
®
a B b Fig. 17
Y
C
I
c d I = nebdvd or vd =
nebd
O
X I
EH I BI
Z a´ b´ = or EH =
D B nebd nebd
c´ d´
VH
EH =
b
VH BI BI
= or VH =
Fig. 14 b nebd ned

Sign of charge carriers in a conductor FG 1 IJ BI


®
VH =
H ne K d
B
Y a b 1
+ + + + + + The quantity is called the Hall coefficient.
+ + + C++ + d ne
c
O ® VH d
I
X ud e + ∴ Hall coefficient =
®
Z Fm b´

VH BI
a´ – – – – – –
– – – D– – – VH B
c´ d´ Now, Hall resistance, RH = or RH =
I ned

Fig. 15
238 COMPREHENSIVE OBJECTIVE PHYSICS

61. DIFFERENCE BETWEEN ELECTRIC AND MAG- (iii) Wire of infinite length in +∞
NETIC FORCES both sides
In this case, θ1 = 90°
S.No. Electric Force Magnetic Force
and θ2 = 90°
Electric force on a charged Magnetic force on a charged I
1.
∴ At P,
particle is always collinear particle is always perpendicu- r
P
with the field. lar to the field. µ I
B= 0 (sin 90° + sin 90°)
2. Electric force (qE) is inde- Magnetic force is velocity- 4π r
pendent of the state of rest dependent i.e. acts only when
or the motion of the charged the charged particle is in I
B = 2 × 10–7 –∞
particle. motion. r Fig. 20
3. Electric force does work in Magnetic force does no work
displacing the charged when the charged particle is (iv) When wire is terminated r
P
particle. displaced. near the point.
→ →
Magnetic force,
In this case, θ1 = 0°
4. Electric force, F = q E
→ → → and θ2 = 90°
F = q ( v × B)
∴ At P,
= q v B sin θ
µ0 I
When θ = 0° B= (sin 0° + sin 90°)
4π r
Force = 0 (min)
When θ = 90°
I ∞
F = qvB (max) ∴ B = 10–7 Fig. 21
r
5. Electric force may or may not Magnetic force is always non-
be non-central. central.

62. MAGNETIC FIELDS OF SOME IMPORTANT


SYSTEMS
P
(i) Wire of finite length (v) r/2
a1 P
Magnetic field at P,
I I
r/2
I
µ I r q1 r/2 r/2
B= 0 (sin θ1 + sin θ2) q2
P I I
4π r B=0 B=0
I
a2 Fig. 22 Fig. 23
I
or B= 10–7 (sin θ1 + sin θ2) µ 0I 2µ 0I
r (vi) B = 2 × =
Fig. 18 r πr

µ 0I 2
Also, B = (cos α1 + cos α2)
4 πr
(ii) Wire of infinite length in any one side
In this case θ1 = 90° ¥
and θ2 = 0°
P
µ0 I
∴ At P, B = (1 + sin θ)
4π r I I r/2 r/2
r I
P
I q
or B = 10–7 (1 + sin θ)
r
a Fig. 24
µ I The direction of magnetic field is perpendicular to the
Also, B = 0 (1 + cos α)
4 πr plane of paper and directed downwards.
Fig. 19
MAGNETIC EFFECTS OF CURRENT AND MAGNETISM 239

(vii) Direction of magnetic field at P is perpendicular D I C


to the plane of the paper and directed downwards.

a
P

P² x
x P P¢

I A I B
I a
x r–x
r Fig. 27
Fig. 25 (x) Magnetic field at P

BP =
µ 0I 1
+
LM
1 OP ∞
2π x r − xN Q
B
Direction of magnetic field at P′ is perpendicular to
the plane of the paper and directed upwards. I

BP′ =
µ 0I 1

LM
1 OP r
2π x r + xN Q 45°
45°
P

Direction of magnetic field at P″ is perpendicular to r


the plane of the paper and directed upwards.

BP″ =
µ 0I 1

LM
1
.
OP O I A ∞
2π x r + xN Q Fig. 28
(viii) Magnetic field due to current in ABCD at P
= Magnetic field due to AO
+ Magnetic field due to OB
= 2 × Magnetic field due to AO
µ0 I
=2× (sin 90° + sin 45°)
4π r
µ 0 I (1 + 2 )
= × 2.
4π r 2
(xi) Magnetic field at P

a
I P I F
Fig. 26 B E
A
= Magnetic field due to AB + Magnetic field due to BC
a a
+ Magnetic field due to CD
I
µ0 I
=0+ (sin φ1 + sin φ2) + 0 C D
4π r 2a

µ0 I Fig. 29
= (sin φ1 + sin φ2).
4π r
= Magnetic field due to AB + Magnetic field due to
(ix) Magnetic field at the centre of the square BC + Magnetic field due to CD + Magnetic field
Fµ 4I I due to DE + Magnetic field due to EF.
=4× GH 4π0
2a
JK
240 COMPREHENSIVE OBJECTIVE PHYSICS

µ0 I µ I (xvi) Magnetic field due to current-carrying circular seg-


=0+ (sin 0° + sin 45°) + 0 (sin 45° + sin 45°) ment subtending an angle θ at the centre
4π a 4π a
µ 0 Idl
µ I dB =
+ 0 (sin 45° + sin 0°) + 0 4π r2
4π a But dl = rdθ
µ0 I FG 1 IJ × 2 + µ I ( 0 µ0 I ∴
µ0
dB =
Irdθ
=
4π a H 2 K 4π a 2) =
4π a
2 2 4π
µ
r2
Idθ
or dB = 0
D I 4π r Fig. 34
C
µ I
(xii) Magnetic field at Integrating, B = 0 θ.
4π r
centre b P I

µ0 I
= (8 a2 + b2 )
4 π ab I π
A
a
B (xvii) Here θ = 60° =
3
Fig. 30
µ0 I
C ∴ B=
12 r
(Normal to the plane of
(xiii) Magnetic field at the figure and outward)
centre of the equilateral trian- I
gle Fig. 35
P
FG µ bI IJ
H 4π a K
0
=3 A B
I
a
(xviii) Magnetic field at
Fig. 31
centre O = 0.
O
P
I

(xiv) Magnetic field at the Fig. 36


vertex P of the equilateral tri- I I
µ0 I
angle (xix) B =
4r
(At O)

µ0 F 2I I (Normal to the
=
4π GH 3a
JK A I B plane of the paper
a downwards)

Fig. 32 Fig. 37

(xx)
B µ I
= 0 1+
LM1 OP
(xv) Magnetic field at the cen-
I
(At O) 4r N
π Q
B E (Normal to the plane of
tre of the pentagon paper downwards)
L µ 4I sin 36° × tan 36°OP
P

=5 M 0
N 4π a Q
C I D
a

Fig. 33 Fig. 38
MAGNETIC EFFECTS OF CURRENT AND MAGNETISM 241

B µ I (xxvii) When two concentric I B1


I
(xxi) = 0 [π + 2] current carrying
(At O) 4 πr
coils are inclined at q B
(Normal to the plane of O
paper upwards) an angle θ, then
magnetic field at the cen- B2
tre of the coils (At O),
Fig. 45
Fig. 39
B= B12 + B22 + 2B1B2 cos θ
Direction of B is determined by law of addition of
B µ I 1
= 0
LM OP vectors.
(xxii)
(At O) 2r π
−1
N Q (xxviii) In this case when θ = 90°, then the magnetic
field at centre of the coils,
(Normal to the plane of
paper downwards) B= B12 + B22 B1
I2 B
Fig. 40 Now if I1 = I2
and r1 = r2 and n1 = n2.

B= B2 + B2 I1
O
B2

(xxiii) B µ I
= 0 1+
1 LM OP = 2 B= 2
µ 0 nI
(At O) 2r π N Q 2r
µ0 nI
(Normal to the plane of or B = Fig. 46
paper upwards) 2r
Fig. 41

(xxiv) B
= 0 +
LM
µ I n1 n2 OP r1
r2
I
(xxix) B µ Iθ 1 1
= 0 −
LM OP
(At O) 2 r1 N
r2 Q O
I (At O) 4 r1 r2N Q
where n 1 and n 2 are the (Direction of B will be nor-
number of turns in the coil. mal to the plane of paper up-
when n1 = n2 = n, wards)

µ0 nI 1 1
+
LM OP Fig. 42 Fig. 47
then B =
2 r1 r2 N Q ILLUSTRATION OF A BAD PROBLEM WHICH GIVES
→ UNNECESSARY ‘MENTAL STRESS’.
Direction of B will be normal to the plane of the paper
upwards. Illustration 1. A proton moving with a constant
velocity passes through a region of space without change in
(xxv) B µ nI 1 1
= 0 −
LM OP its velocity. If E and B represent electric and magnetic fields
(At O) 2

r1 r2 N Q respectively, this region of space may not have :
(a) E = 0, B = 0 (b) E = 0, B ≠ 0
Direction of B will be nor- (c) E ≠ 0, B = 0 (d) E ≠ 0, B ≠ 0.
mal to the plane of the pa-
Sol. (a) If E = 0 and B = 0,
per upwards. Fig. 43
then no force ; hence straight You can save your time
(xxvi) Helmholtz Coils path. Think of Newton’s first law by thinking on the lines
of motion. of ‘may have’ and then
Magnetic field at the centre
rejecting the options so
µ 0 NI (b) If E = 0, then no force obtained.
= 0.72 due to electric field.
R
The field over a small region If B ≠ 0, then the charge may be moving parallel to
around a point mid-way bet- magnetic field. So, no force due to magnetic field.
ween the coils is uniform. Fig. 44
242 COMPREHENSIVE OBJECTIVE PHYSICS

(c) If E ≠ 0 and B ≠ 0, then there is a possibility that Illustration 5. A cell is con- B


nected between the points A and C
E of a circular conductor ABCD of
Bev = eE or v =
B centre O. ∠AOC = 60°. If B1 and B2 I1
Even in this case, the path would be straight. are the magnitudes of the magnetic O
fields at O due to the currents in
(d) If E ≠ 0 and B = 0, then there must act an electric
ABC and ADC respectively, then 60°
force. Naturally, acceleration would exist.
B
So, (c) is the right choice. the ratio 1 is A C
B2 D
(a) 6 (b) 1 I I2
ILLUSTRATIONS INDICATING LATEST EXAMINA-
TION TRENDS 1
(c) 5 (d).
Illustration 2. A 100 V voltmeter of internal resist- 5
[Karnataka CET 1999] Fig. 49
ance 20 kΩ in series with a high resistance R, is connected
to a 110 V line. The voltmeter reads 5 V. The value of R is µ 0 I1l1 µ I l
Sol. B1 = , B2 = 0 222 ; R ∝ l ; Also, the
(a) 210 kΩ (b) 315 kΩ 4π r2 4π r
(c) 420 kΩ (d) 440 kΩ. longer and the shorter sections are connected in parallel.
[MP PET 1999] So, I1R1 = I2R2 or I1l1 = I2l2
5 B1
∴ =1
Sol. Current, I = A = 0.25 × 10–3 A B2
20 × 103
So, (b) is the right choice.
Now, 105 = 0.25 × 10–3 R or R = 420 kΩ
Illustration 6. A wire
So, (c) is the right choice.
carrying current i is shaped as
Illustration 3. The magnetic field, of a given length shown. Section AB is a quarter i
of wire for single turn coil, at its centre is B. Then its value circle of radius r. The magnetic
for two turns coil for the same wire is field is directed,
B B (a) at an angle π/4 to the
(a) (b) plane of the paper.
B
4 2
r
(c) 4B (d) 2B. (b) perpendicular to the plane
[All India PM/PD 2002] of the paper and directed
i A C
into the paper.
µ (2N) I µ0NI
Sol. B′ = 0
FG IJ
r
=4
2r
= 4B (c) along the bisector of the an- Fig. 50
2
H K
2
gle ACB towards AB.
(d) along the bisector of the angle ACB away from AB.
So, (c) is the right choice.
[Karnataka CET 2002]
Illustration 4. Consider the
Sol. Applying Right hand palm rule, we find that (b)
situation shown in Fig. 48. The
is the correct option.
straight wire is fixed but the loop can
move under magnetic force. The loop Illustration. 7. Two parallel, long wires carry cur-
rents i1 and i2 with i1 > i2. When the currents are in the
will
same direction, the magnetic field at a point midway be-
(a) remain stationary tween the wires is 10 µT. If the direction of i2 is reversed,
(b) move towards the wire the field becomes 30 µT. The ratio i1/i2 is
(c) move away from the wire (a) 4 (b) 3
Fig. 48
(d) rotate about the wire. (c) 2 (d) 1.
[MP PMT 1999 ; IIT 1985] µ 0i1 µ 0i2
Sol. In the first case, − = 10 µT
Sol. Like currents are nearer. The net force is at- 2πr 2πr
traction.
So, (b) is the right choice. µ 0i1 µ 0i2
In the second case, + = 30 µT
2πr 2πr
MAGNETIC EFFECTS OF CURRENT AND MAGNETISM 243

µ 0i1 re m
Adding, = 20 ∴ = e
2πr rp mp
µ 0i2
Subtracting, = 10 So, (b) is the right choice.
2πr
Illustration 11. A moving coil galvanometer has
i1 150 equal divisions. Its current sensitivity is 10 divisions
Dividing, =2
i2 per milliampere and voltage sensitivity is 2 divisions per
So, (c) is the right choice. millivolt. In order that each division reads 1 volt, the re-
Illustration 8. A circular coil of radius 20 cm hav- sistance in ohm needed to be connected in series with the
ing 50 turns carries a current of 2 A. The magnetic moment coil will be :
of the coil is (a) 9995 (b) 99995
(a) 0.4 π A m2 (b) 4π A m2 (c) 10 5 (d) 10 3 [AIEEE 2005]
(c) 40 π A m2 (d) 400 π A m2. Sol. Current required for full scale deflection, Ig
[Pb. PMT 2002] 150
= mA = 15 mA
Sol. M = NIA 10
= 50 × 2 × π (0.20)2 A m2 Voltage corresponding to full scale deflection
= 4π A m2. 150
= mV = 75 mV
So, (b) is the right choice. 2
Illustration 9. A straight conductor carrying a 75 mV
direct current 3I ampere is split into 2I and I as shown in Galvanometer resistance, G = =5Ω
15 mA
Fig. 51. Then the magnetic
2I Voltage to be measured = 150 V
induction B at the centre of
the circular loop of radius r V = Ig (R + G)
r
metre is 3I V
or R= –G
(a) zero O Ig
(b) infinite 150
= –5
µ0 I I 15 × 10−3
(c) tesla = 10000 – 5 = 9995 Ω
2 πr Fig. 51
So, (a) is the right choice.
µ I
(d) 0 . Illustration 12. Two thin, long parallel wires, sepa-
4r rated by a distance ‘d’ carry a current of i. A is the same
µ 0 (2I) µ 0I direction. They will
Sol. B= − (a) repel each other will a force of µ0i2/(2πd2)
4r 4r
(b) attract each other with a force of µ0i2/(2πd2)
µ 0I µ 0I (c) repel each other with a force of µ0i2/(2πd)
B= (2 – 1) or B =
4r 4r (d) attract each other with a force of µ0i2/(2πd).
So, (d) is the right choice. [AIEEE 2005]
Illustration 10. Electron and proton having masses F µ 0 2 I1I2 µ i2
me and mp respectively enter the region of a uniform, per- Sol. = = 0
l 4π d 2πd
pendicular magnetic field with same velocity. The ratio of
the radii of their circular paths inside the field is Since like currents attract therefore force is attractive.
(a) 1 : 1 (b) me : mp So, (d) is the right choice.
Illustration 13. Two concentric coils each of radius
(c) mp : me (d) me : m p . equal to 2 π cm are placed at right angles to each other.
3 ampere and 4 ampere are the currents flowing in each coil
mv2 mv respectively. The magnetic induction in Weber/m 2 at the
Sol. Bqv = or r =
r Bq centre of the coils will be
v, B and q are constants. (µ0 = 4π × 10–7 Wb/A.m)
∴ r∝m (a) 7 × 10 –5 (b) 5 × 10 –5
(c) 10 –5 (d) 12 × 10 – 5. [AIEEE 2005]
244 COMPREHENSIVE OBJECTIVE PHYSICS

µ 0 I1 µ 0 I2 67. Like poles of two magnets repel while unlike poles


Sol. B1 = , B2 = attract.
2r 2r
68. After Oersted’s discovery, it was realised that mag-
B= B12 + B22 netism was an effect of electric charges in motion.
According to Ampere, elementary atomic magnets in
µ0
B= I12 + I22 magnetic materials were tiny circulating currents.
2r They are aligned in the same direction in a bar mag-
4 π × 10−7 net.
B= 9 + 16
2 × 2 π × 10 −2 69. The region around a magnet in which it exerts forces
on other magnets and objects made of iron is called a
=5× 10 –5
Wb m–2
magnetic field.
So, (b) is the right choice.
70. Repulsion is a sure test of magnetism.
Illustration 14. A unifrom electric field and a uni-
form magnetic field are acting along the same direction in a 71. The earth’s magnetic field may be regarded as uni-
certain region. If an electron is projected along the direc- form over a vast region. So, it can be represented by a
tion of the fields with a certain velocity then – set of parallel, equispaced and equidistant lines point-
ing in the same direction.
(a) it will turn towards left of direction of motion
72. The magnetic flux density at a distance r metre from
(b) it will turn towards right of direction of motion
(c) its velocity will increase µ0 m
a magnetic pole is B = , where m is the pole
(d) its velocity will decrease. [AIEEE 2005] 4π r2
Sol. Magnetic field shall not exert any force. How- strength in Am and µ0 is absolute magnetic perme-
ever, the electric field shall exert a force in a direction oppo- ability of free space.
site to the direction of motion of the electron. So, there will m
be decrease in the velocity of the electron. In cgs system, B =
r2
So, (d) is the right choice.
73. The inverse square law of force in magnetism is
Illustration 15. A charged particle of mass m and
charge q travels on a circular path of radius r that is per- µ 0 mm′
pendicular to a magnetic field B. The time taken by the F=
4 π r2
particle to complete one revolution is
74. Magnetic dipole is a pair of equal and opposite mag-
2π m 2π q B netic poles separated by a finite distance.
(a) (b)
qB m 75. Magnetic dipole moment is a vector quantity. Its mag-
2 nitude is equal to the product of pole strength (m) and
2π q B 2 π mq
(c) (d) . [AIEEE 2005] the separation (2l) between the magnetic poles. Its
m B direction is from south pole to north pole.
Sol. mrω2 = Bqv = Bqr ω
→ →
Bq 2 π Bq M = m (2l)
or ω= or =
m T m 76. (i) Magnetic flux density at a point on the axial line of
a magnetic dipole is given by
2πm
or T= µ0 2 Mr
Bq B=
So, (a) is the right choice. 4 π (r 2 − l2 )2
Here r is the distance of the observation point from
(B) MAGNETISM the mid-point of the magnetic dipole. l represents
63. A material which has the property of attracting small half-length of the magnetic dipole.
pieces of iron towards it is called a magnet. Magnets (ii) If the observation point is far far away, then
may be natural or artificial.
µ0 2 M
64. In a magnet, magnetism is strongest at the poles. B=
4 π r3
65. When a magnet is freely suspended, it points along
north-south direction. (iii) The direction of magnetic field is the same as the
direction of magnetic dipole moment of the magnet.
66. Magnetic monopoles are not possible.
MAGNETIC EFFECTS OF CURRENT AND MAGNETISM 245

77. (i) Magnetic flux density at a point on the equatorial Case (ii) If θ = 180°, i.e., if the dipole is displaced
line of magnetic dipole is given by through 180° from its stable equilibrium position, then
µ0 M W = MB (1 – cos 180°) = 2MB
B=
4 π (r2 + l2 )3 / 2 83. The potential energy of a magnetic dipole placed in a
uniform magnetic field is given by
(ii) If the observation point is far far away, then
→ →
µ M U = M .B.
B= 0 3
4π r 84. Magnetic lines of force are continuous closed curves.
(iii) The direction of magnetic field is opposite to the 85. Electric lines of force are discontinuous.
direction of magnetic dipole moment of the mag- 86. If declination is ignored, then the horizontal compo-
net. nent of earth’s magnetic field is from geographic south
78. (i) Magnetic flux density, due to a magnetic dipole, at to geographic north.
a general point is given by 87. Just as the direction of electric dipole moment is from
µ0 M negative charge to positive charge, the direction of
B= 1 + 3 cos2 θ magnetic dipole moment of a magnet is from south to
4 π r3
north.
(ii) For axial line, θ = 0°; For equatorial line, θ = 90º.
88. There are three magnetic elements of earth : declina-
(iii) If θ′ is the angle which B makes with OP (pro-
tion, dip and horizontal component of earth’s mag-
1 netic field.
duced), then tan θ′ = tan θ
2
89. Declination is the angle between the magnetic merid-
B ian and the geographic meridian.

90. Angle of dip is the angle through which the N-pole
P dips down with reference to the horizontal.
 r
M 
BV BH BV
91. tan δ = , cos δ = sin δ =
BH B B
S O N
92. B = BH 2 + B V 2
Fig. 52
93. APPARENT DIP
79. The magnetic field due to a short bar magnet at a If the plane of the scale of the dip circle is not in the
distance r on its axial line is twice the magnetic field
magnetic meridian, then the needle will not indicate the
at the same distance on its equatorial line.
correct direction of the earth’s magnetic field. The angle
80. GAUSS’S THEOREM IN MAGNETISM

z
made by the needle with the horizontal is called the
→ → →
B (r) . dS = 0 apparent dip.
s
→ → Suppose the dip circle is set at an angle α to the mag-
The magnetic field B (r) may be due to some mag- netic meridian. Effective horizontal component in this plane
netic dipole or due to superposition of any number of will be BH cos α. However there will be no effect on the verti-
magnetic fields. cal component BV. Now, the apparent dip δ′ will be given by
81. Torque experienced by a magnetic dipole placed in
BV BV 1
uniform magnetic field is given by tan δ′ = or tan δ′ = .
→ → →
BH cos α BH cos α
r = M×B.
BV
82. Work done in turning a magnetic dipole through an But = tan δ, where δ is the true dip
angle θ from its stable equilibrium position is given BH
by W = MB (1 – cos θ). tan δ
Case (i) If θ = 90°, i.e., if the dipole is displaced or tan δ′ =
cos α
through 90° from its stable equilibrium position, then
W = MB (1 – cos 90°) or W = MB
246 COMPREHENSIVE OBJECTIVE PHYSICS

94. COMBINED FIELD OF BAR MAGNET AND Magnetic field due to the magnet is given S
EARTH, NEUTRAL POINT by
Case I. Magnetic axis of the magnet in the mag-
µ0 2Mr
netic meridian, N-pole of the magnet towards the B=
North of Earth. 4 π (r 2 − l2 )2 r
N
BH
At the neutral point,
µ0 2Mr
= BH
4 π (r 2 − l2 )2 B
Fig. 56
N 95. THE TANGENT LAW
H mH
X X mF × NT = mH × ST N mF
ST q q
S
or F=H
NT F
S
or F = H tan θ mF T

This equation is referred mH


Fig. 57
to as the tangent law.
Fig. 53

If r be the distance of the 96. TANGENT GALVANOMETER


N
neutral point from the centre of BH F = H tan θ (tangent law)
l
the magnet, then the magnetic r
field intensity B at the neutral µ 0NI
∴ = H tan θ
point due to the magnet is given 2r
l

LM 2rH OP tan θ = LMM µHN OPP tan θ


by B S
µ M
B= 0 2 Fig. 54
4 π (r + l2 )3 / 2 or
N Q
I= µ N
0 MN 2r PQ
0

At the neutral point,


µ0 M µ0 N
∴ = BH The quantity is known as the galvanometer
4 π (r 2 + l2 )3 / 2 2r
constant G, because N and r are constants for a galvanometer.
If l2 can be neglected in comparison to r2,
µ0 M H
then = BH ∴ I= tan θ
4 π r3 G
Case II. Magnetic axis of the magnet in the mag- H
netic meridian, S-pole of the magnet towards the The term helps to reduce the deflection θ to current
G
North of Earth. I. So, this is known as reduction factor K of the
galvanometer.
X ∴ I = K tan θ …(1)
97. REDUCTION FACTOR OF TANGENT GALVA-
NOMETER (K)
S 2rH
(i) K =
µ0 N
N
(ii) The reduction factor of a tangent galvanometer is
numerically equal to the current required to produce a de-
flection of 45°.
X

Fig. 55
MAGNETIC EFFECTS OF CURRENT AND MAGNETISM 247

98. VIBRATION MAGNETOMETER If B is in weber m–2, then H is expressed as ampere-


turn per metre. The permeability is expressed in weber per
I ampere-metre. The value of the permeability µ0 of empty
(i) T = 2π
MB space has the value 4π × 10–7 WbA–1m–1.
If B is expressed in gauss, H in oersted then µ is ex-
1 MB pressed in gauss per oersted. The numerical value for the
(ii) Frequency of oscillation, ν =
2π I permeability of empty space in this system is µ0 = 1.
(iii) Comparison of magnetic moments of magnets of 100. INTENSITY OF MAGNETISATION (OR MAG-
the same size NETISATION)
I I M
T1 = 2π and T2 = 2π I=
M1B M2 B V
where V is the volume
of the ferromagnetic ®
T1 M2 T12 M2 M
= or = specimen. If m is the N S
T2 M1 T22 M1
pole strength devel-
(iv) Comparison of magnetic moments of magnets of oped in the specimen
different sizes and 2l is the magnetic Fig. 59
length of the specimen, then
I1 + I2
T1 = 2π m × 2l
(M1 + M2 ) B I=
A × 2l
I1 + I2 where A is the cross-sectional area of the specimen.
T2 = 2π
(M1 − M2 ) B m
∴ I=
A
101. MAGNETIC SUSCEPTIBILITY
S N
I ∝ H or I = xmH
S N
where xm is a “constant” known as the susceptibility of the
(a) Sum combination substance.
If H = 1, then xm = I.
N S Magnetic susceptibility is the intensity of mag-
S N netisation developed in the substance when placed
in a magnetic field of unit strength.
(b) Difference combination
102. MAGNETIC INDUCTION
Fig. 58 The total number of lines of force threading
the specimen per unit area is called magnetic induc-
T1 M1 − M 2 tion. It is also known as magnetic flux density and is de-
Dividing, =
T2 M1 + M 2 noted by B.

M1 T12 + T22 4πm


= ∴ B=H+
M2 T22 − T12 A
(v) Comparison of earth’s magnetic field at two differ-
ent places

I I
T1 = 2π and T2 = 2π
MB1 MB2
S N
T1 B2 T12 B2
Dividing, = or =
T2 B1 T22 B1
99. RELATION BETWEEN B AND H
B = µH
Fig. 60
248 COMPREHENSIVE OBJECTIVE PHYSICS

m 106. DEFLECTION MAGNETOMETER


But =I
A Tan-A Position (end-on position)
∴ B = H + 4πI. The arms of the magnetometer are kept in magnetic
103. MAGNETIC PERMEABILITY east-west direction. The magnetometer box is rotated in its
plane till the pointer reads 0—0. The magnet is now kept as
B
µ= shown in figure. The deflection (θ) in the equilibrium posi-
H tion of the needle is read on the graduated scale. The dis-
For air, the value of µ is 1.0000004. For vacuum, tance r of the centre of the magnet from the centre of the
µ = 1. For iron, the value of µ may exceed 1000. needle is noted.
104. MAGNETIC DIPOLE MOMENT OF A CURRENT
LOOP 90°
N

M = IA N S 0° 0° W E
The direction of dipole moment is perpendicular to 90°
the face area and along the axis of the loop. According to S
right hand rule, it is directed from south to north i.e. out-
Fig. 61
wards from north.
→ → Magnetic field due to the mag-
In vector form, M = IA = IAn net at the location of the needle, Note that the mag-
netic needle is in the
Here n is a unit vector perpendicular to the plane of µ0 2Mr end-on position of
the loop. B=
4 π (r − l2 )2
2
the bar magnet.
The SI unit of dipole moment is A m2. It is the mag- Now, B = BH tan θ
netic moment of one turn loop of an area one square
metre when a current of one ampere is flowing µ0 2Mr
∴ = BH tan θ
through it. 4 π (r 2 − l2 )2
105. ATOM AS A MAGNETIC DIPOLE (ORBITAL M 4 π (r 2 − l2 )2 tan θ
MAGNETIC MOMENT OF AN ELECTRON) or =
BH µ 0 2r
→ e →
In vector form, M=− L All the quantities on the right-hand side are known.
2m
The negative sign indicates that the direction of the M
So, can be determined.
magnetic moment is opposite to the direction of the angular BH
momentum. Tan-B Position
According to Bohr’s theory, an electron (in an atom) (broad side-on position)
is permitted to revolve only in that orbit for which the angu- The arms of the
lar momentum (mr2 ω) is an integral multiple of h/2π. magnetometer are kept in
the magnetic north-south
h direction. The magneto-
∴ mr2 ω = n , N
2π meter box is rotated in its 90°
plane till the pointer reads
where n = 1, 2, 3, ...... 0° 0° W E
0—0. The bar magnet is
e nh eh placed as shown. The de- 90°
Now, M= × or M = n S
2 2πm 4πm flection θ of the needle is
eh noted. The distance r of the
Least value of magnetic moment = magnet from the centre of
4πm
the box is also noted.
This least value is considered as one Bohr magneton.
Magnetic field due to N S
eh the magnet at the location
1 Bohr magneton =
4πm of the needle, Fig. 62

1.6 × 10 −19
× 6.6 × 10 −34 µ M
B= 0 2
= A m2 4 π (r + l2 )3 / 2
4 × 3.142 × 9.1 × 10−31
= 9.2 × 10–24 A m2
MAGNETIC EFFECTS OF CURRENT AND MAGNETISM 249

Now B = BH tan θ Note that the magnetic M 4π


µ0 M needle is in the broad or = (r2 + l2)3/2 tan θ
= BH tan θ BH µ 0
4 π (r + l2 )3 / 2
2 side-on position.

107. TABLE OF MAGNETIC CONSTANTS

S.No. Physical Quantity Symbol Formula SI unit CGS emu Dimensional


conversion formula

M
1. Pole strength m Am 10 [M0LT 0A]
2l
2. Magnetic dipole moment M IA A m2 103 [M0L2T 0A]
3. Magnetic flux φB BA Wb 108 [ML2T –2A–1]

F
4. Magnetic flux density B T 104 [ML0T –2A–1]
qv

B
5. Magnetising force or magnetic field intensity H A m–1 4π × 10–3 Oe [M0L–1T 0A]
µ

M m
6. Intensity of magnetisation I or A m–1 10–3 [M0L–1T 0A]
V a
B
7. Magnetic permeability of medium µ H m–1 107 [MLT –2A–2]
H

µ B
8. Relative magnetic permeability µr or — — —
µ0 B0

I
9. Magnetic susceptibility χm — — —
H

108. IMPORTANT FACT FILE OF PROPERTIES OF (iii) χm – T curve for ferro-


MAGNETIC MATERIALS magnetics is shown in
(i) Examples Fig. 63. According to
Curie-Weiss law,
(a) Ferromagnetic materials. Fe, Co, Ni, Gd χm
(below 16°C), Dy, Fe2O3, Alnico (Al 12%, Ni 1
χm ∝ where Tc is
20%, Co 5%), Cu–Zn ferrite, Permalloy (0.785 T − Tc
Tc
Ni, 0.215 Fe), Mumetal (0.75 Ni, 0.02 Cr, 0.05 Curie temperature. T
Cu, 0.18 Fe) Fig. 63
(b) Paramagnetic materials. Al, Mg, Cr, W, U,
Na, Pt, Gd (above 16°C), Gd2O3, CuCl2, Dy2O3,
(iv) χm – T curve for para-
Oxygen (1 atm), Air (1 atm) magnetics is shown in
(c) Diamagnetic materials. Bi, Hg, Ag, Cu, Fig. 64. According to χm
C2H5OH, N (1 atm), CO2 (1 atm) Curie law,
(ii) Magnetic susceptibility of ferromagnetic substances χm ∝ 1 . T
is high and positive. For paramagnetic substances, T
Fig. 64
χm is low and positive. For diamagnetic substances,
χm is low and negative. (v) Susceptibility of diamagnetic substances does not
depend upon temperature. [Bi at low temperature
is an exception].
250 COMPREHENSIVE OBJECTIVE PHYSICS

(vi) For ferromagnetic substances, µr >>1. For para- Note. The hysteresis loss of the material of a permanent mag-
magnetic substances, µr > 1. For diamagnetic sub- net is immaterial. This is because the permanent magnet need not be
stances, µr < 1. taken through cycles of magnetisation and demagnetisation.
(vii) If B and B0 represent magnetic inductions in mag- 111. CAN STEEL BE USED FOR MAKING PERMA-
netic material and in free space, then
NENT MAGNETS ?
(a) for ferromagnetic substance, B >> B0
Steel does not meet all the requirements for being
(b) for paramagnetic substances, B > B0
used as a permanent magnet. However, it is generally used
(c) for diamagnetic substances, B < B0. for making a permanent magnet.
(viii) Ferromagnetic substances are strongly magnet-
Steel does not have high saturation magnetisation.
ised in the direction of the applied magnetic field.
But the strength of the magnet is increased by making it
Paramagnetic substances are weakly magnetised
in the direction of the applied magnetic field. Dia- laminated. Also, steel does not have high coercivity. In recent
magnetic substances are weakly magnetised in a times, alloys of high retentivity and high coercivity have
direction opposite to the direction of the applied been developed. Examples : Alini and Alnico.
magnetic field. 112. DISTINGUISHING FEATURES OF SOFT IRON
(ix) In the case of ferromagnet- AND STEEL
ic substances, I is in the
direction of H and has a S. No. Parameter Soft iron Steel
large value. I – H curve is I
shown in Fig. 65. 1. Intensity of High Low
H
magnetisation (I)
Fig. 65
2. Magnetic High Low
permeability (µ)
(x) In the case of paramag-
netic substances, I is in 3. Magnetic High Low
the direction of H and has I susceptibility (χm)
a small value. I – H curve
4. Retentivity High Low
is shown in Fig. 66. H
Fig. 66 5. Coercivity Low Slightly larger
than that of
soft iron
(xi) In the case of diamagnetic
substances, I is in a direc- 6. Hysteresis Loss Low High
tion opposite to H and has I
H 7. Area of hysteresis loop Less More
a small value. I – H curve
is shown in Fig. 67. 8. Utility in electro- in permanent
Fig. 67 magnet magnet
(xii) Susceptibility of every magnetic material is inde-
113. SOFT MAGNETIC MATERIALS
pendent of magnetising field H.
These materials generally have low retentivity, low
109. FOUR IMPORTANT PROPERTIES OF THE MA- coercivity and small hysteresis loss. These are suitable for
TERIAL OF AN ELECTROMAGNET
temporary magnetism. These are used for making cores of
1. Low retentivity transformers, motors, generators and electromagnets. Ex-
2. High value of saturation magnetisation amples : soft iron, mu-metal and stalloy.
3. Low coercivity 114. HARD MAGNETIC MATERIALS
4. Small hysteresis loss.
These materials generally have high retentivity, high
Note. Soft iron is highly suitable for making electromagnets.
coercivity and large hysteresis loss. These are suitable for
110. THREE IMPORTANT PROPERTIES OF THE permanent magnetism. Permanent magnets of electric me-
MATERIAL OF A PERMANENT MAGNET ters and loudspeakers are made of hard magnetic materials.
1. High retentivity Examples : Steel and Alnico.
2. High saturation magnetisation
3. High coercivity.
MAGNETIC EFFECTS OF CURRENT AND MAGNETISM 251

115. HYSTERESIS LOOP Sol. Magnetic potential at a point on the broad-side


Lagging of B behind H is called hysteresis. Fig. 68 on position is zero. So, (d) is the right choice.
shows a B–H curve (also called hysteresis loop). OB represents Illustration 19. The number of turns and radius of
residual magnetism, retentivity or remanence. OC represents cross-section of the coil of a tangent galvanometer are dou-
coercive force or coercivity of the specimen. It is a measure bled. The reduction factor K will be
of the magnetic field required to destroy the residual (a) K (b) 2K
magnetism of the specimen.
(c) 4K (d) K/4.
[Haryana PMT 2001]

2rH
Sol. K =
µ0 N
Since both r and N are doubled therefore K would
remain unchanged. So, (a) is the right choice.
Illustration 20. The time of vibration of a dip nee-
dle vibrating in the vertical plane in the magnetic meridian
is 3 second. When the same magnetic needle is made to
vibrate in the horizontal plane, the time period of vibration
is 3 2 second. The angle of dip is
Fig. 68 (a) 30° (b) 45°
The area of the hysteresis loop is a measure of the (c) 60° (d) 90°. [JIPMER 2002]
energy dissipated per cycle per unit volume of the specimen.
I BH
Sol. 3 = 2π
ILLUSTRATIONS INDICATING LATEST EXAMI- MB φ
NATION TRENDS
Illustration 16. A magnetic needle is kept in a non- I
3 2 = 2π B
uniform magnetic field. It experiences MBH
(a) a force but not a torque
(b) a force and a torque 3 BH
Dividing, = Fig. 69
(c) neither a force nor a torque 3 2 B
(d) a torque but not force. [AIEEE 2005]
1 BH
Sol. (b) =
2 B
Illustration 17. If a magnetic needle is vibrated at
neutral point, its time period will be 1
or cos φ = or φ = 60°
(a) Zero (b) Infinity 2
(c) any value (d) T. [Pb. PMT 2002] So, (c) is the right choice.

I Illustration 21. The magnetic field lines due to a


Sol. T = 2π =∞ bar magnet are correctly shown in
M(0)
So, (b) is the right choice. N N
Illustration 18. The ratio of magnetic potential due
to a small bar magnet in the end-on position to that in
broad side-on position for the same distance from it is
1 1
(a) (b)
4 2
S S
(c) 1 (d) ∞. (a) (b)
[All India PM/PD 2002]
252 COMPREHENSIVE OBJECTIVE PHYSICS

N N Sol. Diamagnetic substance is weakly magnetised in


a direction opposite to the direction of magnetising field. This
favours OC.
So, (b) is the right choice.
Illustration 25. A thin rod is bent in the shape of a
S S small circle of radius ‘r’. If the charge per unit length of the
(c) (d) rod is σ, and if the circle is rotated about its axis at a rate of
‘n’ rotations per second, the magnetic induction at a point
Fig. 70
on the axis at a large distance y from the centre is
[IIT Screening 2002]
(a) µ0 π r3n σ/y3 (b) 2µ0 π r3n σ/y3
Sol. Following arguments lead us to the right choice.
(c) (µ0/4π) r n σ/y
3 3
(d) (µ0/2π) r3n σ/y3.
(i) Magnetic lines of force are continuous.
[National Standard Exam. in Physics 2001]
(ii) Inside the magnet, the magnetic lines of force are
from south to north. 2πrσ
Sol. Current I= = 2 πrσn
(iii) Outside the magnet, the magnetic lines of force 1/n
are from north to south. µ 0Ir 2 µ 0Ir 2
So, (d) is the right choice. B= 2 2 3/ 2 = if y >> r
2( y + r ) 2 y3
Illustration 22. A vertical pillar of soft iron par-
µ 0r2 µ 0 πr 3 σn
tially embedded in earth in the Northern Hemisphere is B= (2 πrσn) =
found to be magnetised by the earth’s magnetic field. The 2 y3 y3
polarity at the top end of the pillar will be So, (a) is the right choice.
(a) South pole (b) North pole Illustration 26. Two short magnets have equal pole
(c) Will change from South to North according to strengths but one is twice as long as the other. The shorter
the time of the day magnet is placed 20 cm in tan A position from the compass
(d) None of the above. needle. The longer magnet must be placed on the other side
of the magnetometer for no deflection at a distance equal to
[National Standard Exam. in Physics 1999]
(a) 20 cm (b) 20 (2)1/3 cm
Sol. The pillar is magnetised by induction. In the
phenomenon of induction, the near end is oppositely mag- (c) 20 (2)2/3 cm (d) 20 (2)3/4cm.
netised and far end is similarly magnetised. In the North [CMC LDH 2002]
Hemisphere of earth, it is the “South” pole of earth’s mag- M 2M
net. The top end of the pillar is ‘South’ pole. Sol. For no deflection, 3
=
20 d3
So, (a) is the right choice.
or d = 21/3 × 20 cm
Illustration 23. The relative permeability is repre-
sented by µr and the susceptibility by χ for a magnetic sub- So, (b) is the right choice.
stance. Then for a paramagnetic substance, Illustration 27. A current carrying loop is placed in
(a) µr > 1, χ < 0 (b) µr > 1, χ > 0 a uniform magnetic field in four different orientations, I, II,
(c) µr < 1, χ < 0 (d) µr < 1, χ > 0 III & IV, arrange them in the decreasing order of potential
energy.
[Karnataka CET 2001]
Sol. For a paramagnetic substance, µr is greater than
^n
one. Also, susceptibility is positive.
(I) (II)
So, (b) is the right choice. B B
Illustration 24. The variation of the + I B ^n
intensity of magnetisation (I) with respect A ^n
to the magnetising field (H) in a diamag-
O H
netic substance is described by the graph. (III) (IV)
(a) OD (b) OC C B B
–I ^n
(c) OB (d) OA. D

[Karnataka CET 2002] Fig. 71 Fig. 72


MAGNETIC EFFECTS OF CURRENT AND MAGNETISM 253

(a) I > III > II > IV (b) I > II > III > IV (c) Motion remains S.H. with time period = 4T
(c) I > IV > II > III (d) III > IV > I > II (d) Motion remains S.H. and period remains nearly
[IIT Screening 2003] constant [All India PM/PD 2003]
→ →
Sol. U = – M.B I
Sol. Time period of bar magnet T = 2π where,
So, (c) is the right choice. MH
Illustration 28. The most appropriate I-H curve for mL2
I is moment of inertia = where, m is mass of the magnet
a paramagnetic substance is 12
+I E D mL2
∴ T = 2π ⇒ T∝ m
12 MH
C
∴ If mass is quadrupled then T becomes twice.
O H
So, (b) is the right choice.
B
Illustration 30. If the magnetic dipole moment of
–I A an atom of diamagnetic material, praramagnetic material
and ferromagnetic material are denoted by µd, µ p and µf
Fig. 73 respectively, then :
(a) OA (b) OB (a) µp = 0 and µf ≠ 0
(c) OC (d) OD (b) µd = 0 and µp ≠ 0
(e) OE [Kerala PMT 2003] (c) µd ≠ 0 and µp = 0
(d) µd ≠ 0 and µp = 0 [All India PM/PD 2005]
Sol. Paramagnetic has small susceptibility.
Sol. The individual atoms (or ions or molecules) in a
So, (c) is the right choice.
ferromagnetic material passes a dipole moment, as in a para-
Illustration 29. A bar magnet is oscillating in the magnetic material.
Earth’s magnetic field with a period T. What happens to its The individual atoms (or ions or molecules) of a dia-
period and motion if its mass is quadrupled ? magnetic material do not passess a permanent dipole mo-
T ment of their own.
(a) Motion remains S.H. with time period = Clearly, (b) is the right choice.
2
(b) Motion remains S.H. with time period = 2T
254 COMPREHENSIVE OBJECTIVE PHYSICS

QUESTION BANK

MCQs
SET I
based on
Memory Work, Hard Facts, Important Terms,
Important Formulae etc.

Average time allowed per question is 20 seconds.

(a) 2n B (b) n2 B
MAGNETIC EFFECTS OF CURRENT
(c) n B (d) 2 n2 B [AIEEE 2004]
1. A current of I ampere flows along an infinitely long 6. Two parallel beams of positrons moving in the same
straight thin conductor. The magnetic induction at direction will
any point on the axis of the conductor is (Given : (a) repel each other.
radius of conductor = r).
(b) not interact with each other.
(a) ∞ (b) 0
(c) attract each other.
µ 2I µ I
(c) 0 (d) 0 . (d) be deflected normal to the plane containing the two beams.
4π r 4π r
[AIIMS 2004]
2. A charged particle moves through a magnetic field
in a direction perpendicular to it. Then the 7. A proton and an α-particle, moving with the same
velocity, enter into a uniform magnetic field, acting
(a) speed of the particle remains unchanged
normal to the plane of their motion. The ratio of the
(b) direction of the particle remains unchanged radii of the circular paths described by the proton
(c) acceleration remains unchanged and α-particle is
(d) velocity remains unchanged. (a) 1 : 2 (b) 1 : 4
[All India PM/PD 2003] (c) 1 : 16 (d) 4 : 1 [AIIMS 2004]
3. A galvanometer acting as a voltmeter will have 8. A charge moves in a circle perpendicular to a mag-
(a) a high resistance in series with its coil netic field. The time period of revolution is independ-
(b) a low resistance in parallel with its coil ent of
(c) a low resistance in series with its coil (a) magnetic field (b) charge
(d) a high resistance in parallel with its coil (c) mass of the particle (d) velocity of the particle.
[All India PM/PD 2004] [CMC Vellore 1995]
4. A current i ampere flows along an infinitely long 9. A circular coil of radius R carries an electric current.
straight thin walled tube, then the magnetic induc- The magnetic field due to the coil at a point on the
tion at any point inside the tube is axis of the coil located at a distance r from the centre
of the coil, such that r >> R, varies as
µ 0 2i
(a) . T (b) zero (a) 1/r (b) 1/r3/2
4π r
(c) 1/r 2
(d) 1/r3 [AIIMS 2004]
2i
(c) infinite (d) T [AIEEE 2004] 10. An electron and a proton enter a magnetic field per-
r pendicularly. Both have same kinetic energy. Which
5. A long wire carries a steady current. It is bent into a of the following is true ?
circle of one turn and the magnetic field at the centre (a) Trajectory of electron is less curved.
of the coil is B. It is then bent into a circular loop of (b) Trajectory of proton is less curved.
n turns. The magnetic field at the centre of the coil (c) Both trajectories are equally curved.
will be (d) Both move on straight line path. [MP PET 1999]
MAGNETIC EFFECTS OF CURRENT AND MAGNETISM 255

11. Two very long, straight and parallel wires carry steady 17. A solenoid of 1.5 m length and 2.0 cm radius pos-
currents I and – I respectively. The distance between sesses 10 turns per cm. A current of 5 A is flowing
the wires is d. At a certain instant of time, a point through it. The magnetic field induction inside sole-
charge q is at a point equidistant from the two wires noid is
in the plane of the wires. Its instantaneous velocity v (a) 2π × 10 –3 T (b) 2π × 10 –5 T
is perpendicular to this plane. The magnitude of the (c) 2π × 10 –2 gauss (d) 2π × 10 –5 gauss.
force due to the magnetic field acting on the charge 18. A positive charge is moving upwards vertically. It
at this instant is enters a magnetic field directed to the north. The
force on the charge will be towards
µ 0Iqv µ 0Iqv
(a) (b) (a) north (b) south
2πd πd
(c) east (d) west.
2µ 0Iqv
(c) (d) 0. 19. A wire of length L carrying a current I is bent in the
πd
form of a circle. The magnetic moment is
[IIT 1998 ; National Standard Exam. in
(a) πIL2 (b) IL/4π
Physics 2001]
(c) IL2/4π (d) IL2/2π.
12. The magnetic moment of a current (I) carrying cir-
20. Which of the following graphs correctly represents
cular coil of radius (r) and number of turns (n) varies
the variation of energy density u near the mid-point
as of a long current-carrying conductor with magnetic field
(a) 1/r2 (b) 1/r B?
(c) r (d) r2 [AIIMS 2004]
13. A proton moving in a perpendicular magnetic field
possesses energy E. The magnetic field is increased
8 times. But the proton is constrained to move in the
path of same radius. The kinetic energy will increase
(a) 1/8 times (b) 8 times
(c) 16 times (d) 64 times.
14. A cell is connected between two points of a uniformly
thick circular conductor. The magnetic field at the
centre of the loop will be
(a) zero (b) µ0/2a (i1 – i2)
(c) µ0/2a (i1 + i2) (d) µ0/a (i1 + i2).
15. A straight wire of length π2 m is carrying a current of
2 A and the magnetic field due to it is measured at a Fig. 74
point distant 1 cm from it. If the wire is to be bent 21. A current-carrying power line carries current from
into a circle and is to carry the same current as be- west to east. What will be the direction of the mag-
fore, the ratio of the magnetic field at its centre to netic field at a point above the wire ?
that obtained in the first case would be (a) east to west (b) west to east
(a) 1 : 100 (b) 100 : 1 (c) north to south (d) south to north.
(c) 1 : 50 (d) 50 : 1. 22. The magnetic field inside a toroidal solenoid of ra-
16. An electron moves in a circular orbit with a uniform dius R is B. If the current through it is doubled and
speed v. It produces a magnetic field B at the centre its radius is also doubled keeping the number of turns
per unit length the same, the magnetic field produced
of the circle. The radius of the circle is proportional
by it will be
to :
B B
v v (a) (b)
(a) (b) 4 2
B B (c) 2B (d) 4B.
B B π
(c) (d) .
v v 23. A circular arc of wire subtends an angle at the
2
[All India PM/PD 2005] centre. If it carries a current I and its radius of
256 COMPREHENSIVE OBJECTIVE PHYSICS


curvature is R, then the magnetic field at the centre 30. A proton is moving with velocity v in a direction
of the arc is →
opposite to the direction of magnetic field B . The
µ 0I µ 0I
(a) (b) magnetic force experienced by the proton is
2R R
(a) Bev (b) – Bev
µ 0I µ 0I
(c) (d) . [BHU 1999] (c) Bv [AMU 2000]
(d) zero.
8R 4R
24. A charged particle is released from rest in a region of 31. An electron accelerated through a potential differ-
steady and uniform electric and magnetic fields which ence V passes through a uniform transverse mag-
are parallel to each other. The particle will move in a netic field and experiences a force F. If the accelerat-
(a) straight line (b) circle ing potential is increased to 2V, the electron in the
same magnetic field will experience a force
(c) helix (d) cycloid. [IIT 1999]
(a) F (b) F/2
25. An electron is moving along positive x-axis. To get it
moving on an anti-clockwise circular path in x-y (c) 2F (d) 2F. [AFMC 2001]
plane, a magnetic field is applied 32. A conductor of length 1 m carrying a current of 1 A
(a) along positive y-axis (b) along positive z-axis is placed parallel to a magnetic field of 1 gauss. The
(c) along negative y-axis (d) along negative z-axis. magnetic force acting on the conductor is
(a) zero (b) 1 newton
[MP PMT 1999]
(c) 10–4 newton (d) 1 dyne.
26. The magnetic field at the centre O of the semi-circle
[Fig. 75] due to each straight portion is [Bharati Vidyapeeth 2002]
33. A current-carrying loop is placed in a uniform mag-
netic field. The torque acting on it does not depend
upon
I (a) shape of the loop (b) area of the loop
O (c) value of the current (d) magnetic field.
L L [Manipal 1995]
Fig. 75 34. A charged particle of energy 15 eV moves through a
µ 0I perpendicular magnetic field. The energy of the par-
(a) zero (b) ticle on emerging out of the magnetic field is
4R
µ 0I (a) 15 eV (b) > 15 eV
(c) (d) infinity.
2R (c) < 15 eV
27. In Q. 26, the magnetic field at O due to semi-circle (d) may be more or less than 15 eV depending on the sense of
alone is the applied field. [AFMC 2002]
µ 0I 35. A neutron, a proton, an electron and an α-particle
(a) zero (b)
4R enter a region of uniform magnetic field with equal
µ 0I velocities. The magnetic field is perpendicular to pa-
(c) (d) infinity.
2R per and directed into the paper. The tracks of parti-
28. In Q. 26, the magnetic field at O due to whole of the cles are labelled in Fig. 76. The neutron follows the
wire [Fig. 74] is track
µ 0I
(a) zero (b)
4R C
µ I × × × ×
(c) 0 (d) infinity.
2R × × × × × ×
29. An experiment investigates the variation of the force × × × × ×
F between two long, parallel current-carrying × B × × × ×
conductors a distance d apart. A straight-line graph × × × × ×
should be obtained on plotting A
× × × × D ×
(a) F against d (b) F against 1/d
× × × × ×
(c) F against 1/d2 (d) log F against d
× × × × × ×
(e) F against log d. [AIIMS 2002]
Fig. 76
MAGNETIC EFFECTS OF CURRENT AND MAGNETISM 257

(a) A (b) B (a) attract each other (b) exert no force


(c) C (d) D. (c) deflect perpendicular to the plane of the beams
36. In Q. 35, proton follows the track (d) repel each other. [MP PET 1999]
(a) A (b) B 42. A current of 10 A is flowing through a circular coil of
(c) C (d) D. diameter 1 cm. What is the magnetic induction at its
37. If in a moving coil galvanometer, a current I pro- centre ?
duces deflection θ, then (a) 4π × 10–4 T (b) 2π × 10–4 T
(a) I ∝ tan θ (b) I ∝ θ (c) 4π × 10–8 T (d) 4π × 10–6 T.
(c) I ∝ θ2 (d) I ∝ θ. 43. A positively charged particle projected towards east
is deflected towards north by a magnetic field. The
[MP PET 1999 ; MP PMT 2000, 2001]
field may be
38. In a current-carrying long solenoid, the field produced (a) towards west (b) towards south
does not depend upon (c) upward (d) downward.
(a) number of turns per unit length [DPMT 2001]
(b) current flowing (c) radius of the solenoid
44. Two identical long solenoids carry the same constant
(d) all of the above three. [MP PET 1999] current. When they are very far apart, the flux den-
39. A uniform electric field and a uniform magnetic field sity at the centre of each end of each solenoid is B.
are produced, pointed in the same direction. An elec- The solenoids are now arranged coaxially end-to-end
tron is projected with its velocity pointed in the same with the current in each flowing in the same sense.
direction The flux density on the axis in the region of contact
(a) The electron will turn to its right is
(b) The electron will turn to its left (a) 0 (b) 1/2 B
(c) The electron velocity will increase in magnitude (c) B (d) B 2
(d) The electron velocity will decrease in magnitude. (e) 2B. [Pb. PMT 1999]
[DPMT 1998] 45. A current-carrying straight wire is kept along the
40. Which of the graphs shown in Fig. 77 shows the vari- axis of a circular loop carrying a current. The straight
ation of magnetic induction B with distance r from a wire
long wire carrying a current ? (a) will exert an inward force on the circular loop
(b) will exert an outward force on the circular loop
(c) will not exert any force on the circular loop
(d) will exert a force on the circular loop parallel to itself.
[AMU 1994]
B B 46. An electric charge in uniform motion produces
(a) an electric field only (b) a magnetic field only
r r (c) both electric and magnetic field
(a) (b) (d) no such field at all. [CPMT 1983 ; MNR 1988]

47. A positively charged particle moving with velocity v
enters a region of space having a constant magnetic
induction B. The particle will experience the largest

B B deflecting force when the angle between vector v and

B is
r r (a) 0° (b) 45°
(c) (d) (c) 90° (d) 180°. [CPMT 1990]
48. The magnetic induction associated with current flow-
Fig. 77
ing in a hollow copper tube will be
[MP PMT 1999] (a) only inside (b) only outside
41. Two beams of protons moving parallel and in the (c) both inside and outside (d) neither inside nor outside.
same direction will [National Standard Exam. in Physics 1999]
258 COMPREHENSIVE OBJECTIVE PHYSICS

49. A conducting circular loop of radius r carries a con- 55. To convert a moving coil galvanometer into an am-
stant current I. It is placed in a uniform magnetic meter, one has to connect
→ →
field B such that B is perpendicular to the plane of (a) a small resistance in series
the loop. The magnetic force acting on the loop is (b) a small resistance in parallel
(a) BIr (b) 2πrIB (c) a high resistance in series
(c) zero (d) πrIB. (d) a high resistance in parallel. [JIPMER 1994]
[IIT 1983 ; MP PMT 1999] 56. An electric current i enters and leaves a uniform cir-
50. Which of the following particles will experience maxi- cular wire of radius a through diametrically oppo-
mum magnetic force (magnitude) when projected with site points. A charged particle q moving along the
the same velocity perpendicular to a magnetic field ? axis of the circular wire passes through its centre at
(a) electron (b) proton speed v. The magnetic force acting on the particle
(c) He+ (d) Li++. [CMC LDH 2002] when it passes through the centre has a magnitude
51. A circular loop of area 1 cm2, carrying a current of
(a) qv µ 0i (b) qv µ 0i
10 A, is placed in a magnetic field of 0.1 T perpen- 2a 2 πa
dicular to the plane of the loop. The torque on the µ 0i
loop due to the magnetic field is (c) qv (d) zero. [Pb. PMT 2002]
a
(a) zero (b) 10–4 Nm 57. An electron continues to move in a straight line while
(c) 10–2 N m (d) 1 N m.[CMC LDH 2001] passing through magnetic field. Choose the correct
52. A small coil lies in- statement.
side a larger coil. The (a) It is never possible.
two coils are horizon- (b) It is possible if B is perpendicular to the direction of the
tal, concentric and motion of the electron.
View from
carry currents in the above (c) It is possible if B is parallel to the direction of motion of the
same sense. The electron.
small coil will experi-
(d) It is possible if B is at 45° to the direction of motion of the
ence
electron. [Himachal PMT 2000]
(a) a torque about a hori-
zontal axis Fig. 78 58. An electron of charge 1.6 × 10–19 C and mass 1.0 ×
(b) a torque about a vertical axis 10–30 kg, moves in a circular orbit with a speed of 5.0
(c) an upward force along the axis × 106 m s–1 under the influence of a field of magnetic
(d) a downward force along the axis
flux density 2.0 × 10–2 T. What force, in N, does it
experience ?
(e) no resultant force. [AIIMS 2001]
→ (a) 1.6 × 10–44 (b) 6.4 × 10–28
53. A charge moving with velocity v in X-direction is (c) 3.2 × 10–24 (d) 1.6 × 10–14.
subjected to a field of magnetic induction in the nega-
[MNR 1997]
tive X-direction. As a result, the charge will
59. A proton beam is going from north to south and an
(a) remain unaffected
electron beam is going from south to north. Neglect-
(b) start moving in a circular path in Y-Z plane
ing the earth’s magnetic field, the electron beam will
(c) retard along X-axis be deflected
(d) move along a helical path around X-axis.
(a) towards the proton beam
[CMC LDH 1994] (b) away from the proton beam
54. A charged particle is situated in a region of space (c) upwards (d) downwards.
and it experiences a force only when it is in motion. [All India PM/PD 2002]
It can be deduced that the region encloses
60. The material used for the suspension wire of a mov-
(a) both an electric field and a gravitational field.
ing coil galvanometer is
(b) both a magnetic field and an electric field.
(a) tungsten (b) platinum
(c) both a magnetic field and a gravitational field.
(c) phosphor bronze (d) manganin.
(d) a magnetic field only.
[Bharati Vidyapeeth 2001]
(e) an electric field only. [BHU 1996]
MAGNETIC EFFECTS OF CURRENT AND MAGNETISM 259

61. Two parallel long wires, each carrying a current of The distance between the wires is doubled and the
1 A, are 0.5 m apart. The force per unit length ex- I
erted by one wire on the other is current is decreased to . The force they exert on
3
µ0 µ0 one another, now is
(a) N (b) N
4π 3π (a) F/6 (b) F/9
µ µ (c) 2F/3 (d) F/18. [DPMT 2000]
(c) 0 N (d) 0 N . [AFMC 1996]
2π π
70. An ammeter gives full scale deflection when current
62. A current is passed through a straight wire. The
of 1.0 A is passed in it. To convert it into 10 A range
magnetic field established around it has its lines of
ammeter, the ratio of its resistance and the shunt
force
resistance will be
(a) circular in shape (b) oval in shape and endless
(a) 1 : 9 (b) 1 : 10
(c) straight (d) None of these.
(c) 1 : 11 (d) 9 : 1. [AMU 1993]
[BHU 1994]
71. Fig. 79 shows a rectangu- Z
63. Protons are shot perpendicular to a magnetic field. lar coil of cross-sectional
(a) Magnetic field will have no influence on the motion of area A placed in a uniform
→ I
protons
magnetic field B directed O Y
(b) Protons will continue to move in the same direction but
along Z-axis. The torque
will gain momentum
on coil is
(c) Protons will continue to move in the opposite directions
(a) ABI along + ve Y-axis
but will gain momentum
(b) ABI along – ve Y-axis
(d) They will bend in an arc of a circle. [BHU 2001] (c) ABI along + ve X-axis X
Fig. 79
64. Weber ampere per metre is equal to (d) zero.
(a) joule (b) newton 72. A current I flows along the length of an infinitely
(c) henry (d) watt. [DPMT 1999] long, straight and thin-walled pipe. Then
(a) the magnetic field at all points inside the pipe is the same
65. A rectangular coil of area A of N turns has a current but not zero.
I flowing in clockwise direction when looked at from (b) the magnetic field at any point inside the pipe is zero.
above. The magnetic moment associated with it
(c) the magnetic field is zero only on the axis of the pipe.
(a) points upwards (b) points vertically downwards
(d) the magnetic field is different at different points inside the
(c) is zero (d) is directly proportional to A2. pipe. [Himachal PMT 1994]
[Pb. CET 2000] 73. Which one of the following is not deflected by a mag-
66. To measure total charge displaced through its ter- netic field ?
minals, we use (a) Positron (b) α-particle
(a) an ammeter (b) a tangent galvanometer (c) β-rays (d) γ-rays. [BHU 1993]
(c) a ballistic galvanometer (d) a magnetometer. 74. A straight conductor of length 0.3 metre is placed in
[CMC LDH 1999] a uniform magnetic field of induction 8 × 10–4 tesla
67. If a copper rod carries a direct current, the magnetic parallel to the lines of force. A current of 2.5 ampere
field associated with the current will be flows through it. The mechanical force acting on the
(a) only inside the rod (b) only outside the rod conductor is
(c) both inside and outside the rod (a) 6 newton (b) 6 × 10–4 newton

(d) neither inside nor outside the rod. [Manipal 2001] (c) 3 × 10–4 newton (d) zero. [Manipal 1994]
68. A current is flowing north along a power line. The 75. An electron enters a region where magnetic (B) and
direction of magnetic field above it, neglecting the electric (E) fields are mutually perpendicular to one
earth’s field, is another. Then
(a) north (b) east (a) it will always move in the direction of B

(c) south (d) west. [AFMC 1998] (b) it will always move in the direction of E
(c) it always possess circular motion
69. Two long, thin wires distant a apart exert a force F
on one another when current through each wire is I. (d) it can go undeflected also. [Pb. CET 1993]
260 COMPREHENSIVE OBJECTIVE PHYSICS

76. A charged particle enters in a strong magnetic field. (c) The magnetic field is at 45° with the plane of the coil.
Then its kinetic energy (d) Always in any orientation. [MP PMT 1995]
(a) increases (b) decreases 84. An electron is travelling horizontally towards east.
(c) remains constant A magnetic field in vertically downward direction
(d) first increases and then becomes constant. exerts a force on the electron along
[MP PMT 2001] (a) east (b) west
77. An electron is revolving in a circular orbit of radius r (c) north (d) south. [EAMCET 1984]
in a hydrogen atom. The angular momentum of the 85. The expression for the torque acting on a coil having
electron is l. The dipole moment associated with it is area of cross-section A, number of turns n, placed in
(a) (e/m) l (b) (e/2m) l a magnetic field of strength B, making an angle θ
(c) (m/e) l (d) (2m/e) l. with the normal to the plane of the coil, when a cur-
[All India PM/PD 1993] rent i is flowing in it, will be
78. An α-particle moves at right angles to a uniform (a) n i AB tan θ (b) n i AB cos θ
magnetic field of 0.5 T with a speed of 5 × 106 m s–1. (c) n i AB sin θ (d) n i AB. [MP PET 1996]
The force experienced by α-particle is 86. An electron enters a magnetic field whose direction
(a) 4 × 10 –13 N (b) 8 × 10 –13 N is perpendicular to the velocity of the electron. Then
(c) 4 × 10 –11 N (d) 8 × 10 –11 N. (a) the speed of the electron will increase ;
79. A circular loop carrying a current is replaced by an (b) the speed of the electron will decrease ;
equivalent magnetic dipole. A point on the axis of the (c) the speed of the electron will remain the same ;
loop is now on (d) the velocity of the electron will remain the same.
(a) end-on position (b) broad side-on position [MP PMT 1996]
(c) Both (a) and (b) (d) Neither (a) nor (b). 87. The pole pieces of the magnet used in a pivoted coil
[MNR 1998] galvanometer are
80. To make the field radial in a moving coil galvano- (a) plane surfaces of a bar magnet
meter, (b) plane surfaces of a horse-shoe magnet
(a) the number of turns in the coil is increased (c) cylindrical surfaces of a bar magnet
(b) magnet is taken in the form of horse-shoe (d) cylindrical surfaces of a horse-shoe magnet.
(c) poles are cut cylindrically [MP PMT 1996]
(d) coil is wound on aluminium frame. [MP PET 1993] 88. Two straight parallel wires situated at a distance a,
81. A magnetic field carry currents i 1 and i 2 in the same direction. The
(a) always exerts a force on a charged particle force of attraction between them is
(b) never exerts a force on a charged particle (a) independent of a (b) inversely proportional to a2

(c) exerts a force, if the charged particle is moving across the (c) proportional to (i1 + i2) (d) proportional to (i1 × i2).
magnetic field lines [Himachal PMT 1995]
(d) exerts a force, if the charged particle is moving along the 89. An infinite straight conductor carrying current 2I is
magnetic field lines. [MP PET 1994] split into a coil of radius r as shown in Fig. 80. The
82. A proton and an electron both moving with the same magnetic field at the centre of the coil is
velocity v enter into a region of magnetic field di- I
rected perpendicular to the velocity of the particles.
They will now move in circular orbits such that
(a) their time periods will be same 2I 2I
(b) the time period for proton will be higher O
(c) the time period for electron will be higher
(d) their orbital radii will be same. [MP PET 1995] I
83. A current-carrying rectangular coil is placed in a
Fig. 80
uniform magnetic field. In which orientation, the coil
will not tend to rotate ? µ 0 2 (π + 1) µ 0 2 (π − 1)
(a) (b)
(a) The magnetic field is parallel to the plane of the coil. 4π r 4π r
(b) The magnetic field is perpendicular to the plane of the coil. µ 0 (π + 1)
(c) (d) zero. [Pb. PMT 1993]
4π r
MAGNETIC EFFECTS OF CURRENT AND MAGNETISM 261

90. To reduce the range of voltmeter, its resistance needs a force of 25 N. The distance between the magnets is
to be reduced. A voltmeter has resistance R0 and reduced by a factor of 4. Now, the force of repulsion
range V. Which of the following resistances when con- will be
nected in parallel will convert it into a voltmeter of (a) 4 × 25 N (b) 4 × 4 × 25 N
range V/n ? (c) 4 × 4 × 4 × 25 N (d) 4 × 4 × 4 × 4 × 25 N.
(a) n R0 (b) (n + 1) R0
97. A magnet of magnetic moment M is rotated through
(c) (n – 1) R0 (d) None of these. 360° in a magnetic field. The work done is
91. A current-carrying small loop behaves like a small (a) MH (b) 2 MH
magnet. If A be its area and M its magnetic moment, (c) 2πMH (d) zero.
the current in the loop will be
98. At a certain place, the horizontal component of earth’s
(a) M/A (b) A/M
magnetic field is 3 times the vertical component.
(c) MA (d) A2M.
The angle of dip at the place is
[MP PMT/PET 1998] (a) 75° (b) 60°
92. The magnetic induction at any point due to a long (c) 45° (d) 30°.
straight wire carrying a current is
99. A compass needle is placed at the magnetic pole of
(a) proportional to the distance from the wire earth. It
(b) inversely proportional to the distance from wire (a) points south-east (b) points east-west
(c) inversely proportional to the square of the distance from (c) may stay in any direction (d) becomes vertical.
the wire
100. The magnetic induction and the magnetic field in-
(d) does not depend on distance. [MP PMT/PET 1998] side the iron core of an electromagnet is 1.07 Wb
93. A proton, charge (+ e coulomb) enters in a magnetic m–2 and 150 Am–1. Then, the relative permeability of
field of strength B (B tesla) making an angle 30° with iron is
the direction of magnetic field with speed v. The mag-
107
. 10 −7
netic force on the proton is (a) (b)
150 4π
(a) evB (b) zero
1.07 10 −7 1.07 107
(c) ∞ (d) evB/2. (c) × (d) × .
150 4π 150 4π
94. The magnetic field at 101. The magnetic moment of atomic neon is
the centre of the cube
1
of edge of length a is (a) zero (b) µ
2 B
(a) zero 3
(c) µB (d) µB .
µ0i 2
(b) 8 ×
a 2 102. Which of the following is ferromagnetic ?
(a) aluminium (b) quartz
(c) 8 × µ 0 i 2 (c) nickel (d) bismuth.
a
µ0 i 103. A dip circle is so set that it moves freely in the mag-
Fig. 81
(d) 12 × . netic meridian. In this position, the angle of dip is
2×a 2
40°. Now, the dip circle is rotated so that the plane
in which the needle moves makes an angle of 30°
MAGNETISM with the magnetic meridian. In this position, the
needle will dip
µ0 N (a) 40° (b) 30°
95. In a tangent galvanometer, = 1. Then the value
r (c) > 40° (d) < 40°.
of H is
104. A permanent magnet
K
(a) K (b) (a) attracts all substances
2
(b) attracts only magnetic substances
K K
(c) (d) . [Pb. PMT 1994] (c) attracts magnetic substances and repels all non-magnetic
3 16
substances
96. Two short magnets placed along the same axis with
their like poles facing each other repel each other with (d) attracts non-magnetic substances and repels magnetic sub-
stances. [CMC LDH 1997]
262 COMPREHENSIVE OBJECTIVE PHYSICS

105. Which of the following demonstrates that earth has at a point in tan B position at a distance x from the
a magnetic field ? middle of magnet is
(a) The earth is surrounded by an ionosphere (a shell of (a) 100 gauss (b) 400 gauss
charged particles). (c) 50 gauss (d) 200 gauss.
(b) A large quantity of iron ore is found in the earth.
113. A small piece of unmagnetised substance gets repelled
(c) The intensity of cosmic rays (stream of charged particles when it is brought near a powerful magnet. The sub-
coming from outer space) is more at the poles than at the
stance can be
equator. [JIPMER 1995]
(a) paramagnetic (b) ferromagnetic
106. A pivoted magnetic needle makes 12 oscillations in (c) non-magnetic (d) diamagnetic.
one minute. How many oscillations per minute will
114. The hysteresis cycle for the material of permanent
it make if its pole strength is increased four times ?
magnet is
(a) 12 (b) 24
(a) short and wide (b) tall and narrow
(c) 6 (d) 48. [JIPMER 1993]
(c) tall and wide (d) short and narrow.
107. Pick out the statement which is not true.
115. A dip needle in a plane perpendicular to the mag-
(a) A Tangent galvanometer cannot be used in the polar region.
netic meridian will remain
(b) A short magnet is used in a tangent galvanometer since a (a) vertical (b) horizontal
long magnet would be heavy and may not move easily.
(c) in any direction
(c) Measurements with the tangent galvanometer will be more
(d) inclined at 45° with horizontal. [MP PMT 1995]
accurate when the deflection is around 45°.
(d) While taking readings with a tangent galvanometer, the 116. A sensitive magnetic instrument can be shielded very
readings are repeated by reversing the current to take effectively from outside fields by placing it inside a
care of the fact that the plane of the coil may not be ex- box of
actly along the earth’s magnetic meridian. (a) teak wood (b) plastic material
[Karnataka CET 2001] (c) a metal of high conductivity
108. The value of angle of dip is zero at the magnetic equa- (d) soft iron of high permeability. [CMC LDH 1993]
tor because on it 117. A magnetic needle suspended horizontally by an
(a) V and H are equal (b) the value of V and H is zero unspun silk fibre, oscillates in the horizontal plane
because of the restoring force originating mainly from
(c) the value of V is zero (d) the value of H is zero.
(a) the torsion of the silk fibre
[MP PET 2001]
(b) the force of gravity
109. The total intensity of earth’s magnetic field is given (c) the horizontal component of earth’s magnetic field
by
(d) all the above factors. [All India PM/PD 1994]
(a) I = V + H (b) I = V/H
118. A magnetic needle vibrates in a vertical plane paral-
(c) I = V 2 + H2 (d) I = H/V. lel to the magnetic meridian about a horizontal axis
[CMC Vellore 1993] passing through its centre. Its frequency is f. If the
110. The magnetic field on poles is plane of oscillation is turned about a vertical axis by
(a) only vertical (b) only horizontal
90°, the frequency of its oscillation in vertical plane
will be
(c) both (a) and (b) (d) none of these.
(a) f (b) > f
[DPMT 1993]
(c) < f (d) zero.
111. Magnetic lines of force
119. The tangent galvanometer, when connected in series
(a) always intersect (b) are always closed
with a standard resistance, can be used as
(c) tend to crowd far away from the poles of magnet
(a) an ammeter (b) a voltmeter
(d) do not pass through vacuum.
(c) a wattmeter
[Haryana PMT 1994]
(d) both an ammeter and a voltmeter. [MP PET 1994]
112. The magnetic field due to a short magnet at a point
120. A line passing through places having zero value of
in tan A position at a distance x cm from the middle
magnetic dip is called
point of the magnet is 200 gauss. The magnetic field
MAGNETIC EFFECTS OF CURRENT AND MAGNETISM 263

(a) Isoclinic line (b) Aclinic line (c) induction effect of the sun
(c) Agonic line (d) Isogonic line. (d) interaction of cosmic rays with the current of earth.
[Pb. CET 1997] [CMC LDH 1999]
121. The Mariner’s compass is provided with Gimbals 130. Dip is the angle which the total intensity of earth’s
arrangement so as to magnetic field at a place makes with
(a) give a direct value of dip (a) vertical (b) horizontal
(b) give a direct reading of declination (c) magnetic equator (d) geographic equator.
[BHU 1995]
(c) keep the needle always horizontal. 131. Magnetic susceptibility x is defined as
122. The magnetic moment of a magnet is 5 A m2. If the (a) I/H (b) H/I
pole strength is 25 A m, what is the length of the (c) I × H (d) I2/H2.
magnet ? 132. The magnetic susceptibility for a diamagnetic mate-
(a) 10 cm (b) 20 cm rial is
(c) 25 cm (d) 1.25 m. (a) small and negative (b) small and positive
(c) large and positive (d) large and negative.
123. The north pole of earth’s magnet is near the geo-
graphical 133. Which of the following is paramagnetic ?
(a) east (b) west (a) Bismuth (b) Antimony
(c) Chromium (d) Water.
(c) north (d) south.
[Pb. PMT 1997]
124. Gauss is the unit of
(a) magnetic flux density (b) magnetic flux
134. Which of the following is suitable for the core of the
electromagnets ?
(c) magnetic induction.
(a) Air (b) Soft-iron
125. Current I is flowing in a circular loop of area A placed (c) Steel (d) Cu-Ni alloy.
in a magnetic field B. The maximum couple acting
[DPMT 1994]
on it is
(a) IA/B (b) IBA
135. A ferromagnetic material is placed in an external
magnetic field. The magnetic domains
(c) B/IA (d) IB/A. [BPET 1999]
(a) increase in size (b) decrease in size
126. A long magnetic needle of length 2L, magnetic mo- (c) may increase or decrease in size
ment M and pole strength m units is broken into two (d) have no relation with the field. [AIIMS 1994]
at the middle. The magnetic moment and pole
136. Twists of suspension fibre should be removed in vi-
strength of each piece will be
bration magnetometer so that
(a) M/2, m/2 (b) M, m/2
(a) time period be less (b) time period be more
(c) M/2, m (d) M, m. [AFMC 1993] (c) magnet may vibrate freely
127. The unit of intensity of magnetisation is (d) cannot be said with certainty.
(a) A m–1 (b) A m2
137. Of the following, the most suitable material for
(c) A m (d) Wb m–1. making permanent magnet is
128. A dip needle lies initially in the magnetic meridian (a) Steel (b) Soft iron
when it shows an angle of dip θ at a place. The dip (c) Copper (d) Nickel.
circle is rotated through an angle β in the horizontal 138. The arms of a deflection magnetometer in the tan-
plane and then it shows an angle of dip θ′. Now, gent B position are placed along
tan θ′ (a) the east-west direction (b) the north-south direction
is
tan θ (c) the north east-south west direction
1 1 (d) the north west-south east direction.
(a) (b)
cos β sin β
139. The magnetism of the magnet is due to
1
(c) (d) cos β. (a) the spin motion of electron
tan β
(b) earth
129. The magnetic field of earth is due to
(c) presence of big magnet inside the earth
(a) a magnetic dipole buried at the centre of the earth
(b) motion and distribution of some material in and outside (d) cosmic rays. [JAT Assam 1999]
the earth
264 COMPREHENSIVE OBJECTIVE PHYSICS

140. The magnetic field due to short magnetic dipole of (a) 2M/d3 (b) M/d2
magnetic moment M at a point on the axis of the (c) M/d3 (d) 2M/d2.
dipole and at a distance d from it is given by 149. A paramagnetic material is kept in a magnetic field.
(a) M/d3 (b) 2M/d3 The field is increased till the magnetisation becomes
(c) M/d2 (d) 2M/d2. [CPMT 1994] constant. If the temperature is now decreased, the
141. In the case of a bar magnet, lines of magnetic induc- magnetisation
tion (a) will increase (b) decrease
(a) start from the north pole and end at the south pole (c) remains constant (d) may increase or decrease.
(b) run continuously through the bar and outside 150. Tangent galvanometer is used to measure
(c) emerge in circular paths from the middle of the bar (a) Steady currents (b) Current impulses
(d) are produced only at north pole like ray of light from a bulb. (c) Magnetic moments of bar magnets
[CPMT 1995] (d) Earth’s magnetic field. [MP PET 1993]
142. The time period of a freely suspended magnet does 151. The substances for which the value of magnetic per-
not depend upon meability is just more than unity are called
(a) length of the magnet (a) paramagnetic (b) diamagnetic
(b) the pole strength of the magnet (c) ferromagnetic (d) suspended.
(c) the horizontal component of magnetic field of the earth [Pb. CET 1994]
(d) the length of the suspension 152. A long magnet is cut in two parts in such a way that
143. The ratio of magnetic fields due to a small bar mag- the ratio of their lengths is 2 : 1. The ratio of pole
net in the end-on position to that in broad side-on strengths of both the sections is
position for the same distance from it is (a) equal (b) in the ratio of 2 : 1

(a) 1/4 (b) 1/2 (c) in the ratio of 1 : 2 (d) in the ratio of 4 : 1.

(c) 1 (d) 2. [CPMT 1992] [BITS 1996]


144. If the magnetic flux is expressed in weber, then mag- 153. In a dip circle, the dip needle reads 90°—90°, when
netic induction can be expressed as the plane of rotation of the dip circle is
(a) weber/m2 (b) weber/m (a) horizontal (b) is the magnetic meridian

(c) weber-m (d) weber-m2. (c) vertical


(d) perpendicular to magnetic meridian.
[Bharati Vidyapeeth 1995]
145. The magnetic fields at two points lying at the same [DPMT 1995]
distance from an isolated pole are 154. The period of oscillation of a magnet of a vibration
(a) the same both in magnitude and direction magnetometer is 2.45 s at one place and 4.90 s at the
(b) different both in magnitude and direction other. The ratio of the magnetic fields at the two
(c) the same in magnitude and different in directions places is
(d) different in magnitude but same in direction. (a) 1 : 4 (b) 1 : 2

[AMU 1995] (c) 2 : 1 (d) 4 : 1. [Pb. PMT 1995]


146. If the distance between two similar magnetic poles 155. Relative permeability of iron is 5500. Its magnetic
held one cm apart be doubled, then the force of inter- susceptibility is
action between them will be (a) 5500 × 107 (b) 5499
(a) doubled (b) halved (c) 5501 (d) 5500 × 10 –7.
(c) unchanged [Karnataka CET 2000]
(d) one-quarter of the original value. [Manipal 1996] 156. An example for diamagnetic substance is
147. Time period in vibration magnetometer will be infin- (a) Nickel (b) Aluminium
ity at (c) Copper (d) Iron.
(a) magnetic equator (b) magnetic poles [Karnataka CET 2000]
(c) equator (d) at all places.
157. Orested is a unit of
148. The magnetic field due to a magnetic dipole of mag- (a) magnetic intensity (b) magnet moment
netic moment M at a point on the right bisector of
(c) pole strength (d) dip. [JIPMER 1996]
the dipole and at a distance d from it is given by
MAGNETIC EFFECTS OF CURRENT AND MAGNETISM 265

158. There is no couple acting when two bar magnets are 165. Magnetic lines of force
placed coaxially since (a) cannot intersect at all (b) intersect at infinity
(a) there are no forces on the poles (c) intersect within the magnet
(b) the forces are ⊥ to each other
(d) intersect at the neutral point. [MNR 1993]
(c) the forces are || and the lines of action coincide
166. If a diamagnetic solution is poured into a U-tube and
(d) the forces are || and the lines of action do not coincide with
one arm of this U-tube placed between the poles of a
each other. [Bharati Vidyapeeth 1996]
strong magnet with the meniscus in a line with the
159. A magnetic needle suspended by a silk thread is vi- field, then the level of the solution will
brating in the earth’s magnetic field. If the tempera-
(a) rise (b) fall
ture of the magnet is increased by 500°C, then
(c) oscillate slowly (d) remain as such.
(a) the time period decreases
(b) the time period remains unchanged
[All India PM/PD 1995]
(c) the time period increases (d) the needle stops vibrating. 167. At magnetic poles, the angle of dip is
[Pb. CET 1995] (a) 45° (b) 30°

160. What happens to the force between magnetic poles (c) Zero (d) 90°. [CPMT 1991]
when their pole strength and the distance between 168. Lines of force due to earth’s horizontal magnetic field
them are both doubled ? are
(a) Force increases to two times the previous value (a) parallel and straight (b) elliptical
(b) No change (c) concentric circles (d) curved lines.
(c) Force decreases to half the previous value
[CPMT 1985]
(d) Force increases to four times the previous value.
169. When the S-pole of a bar magnet points towards the
[Haryana PMT 1995] north and the N-pole towards the south, the null
161. Two magnets A and B are identical in mass, length points are at the
and breadth but have different magnetic moments. (a) N and S pole
In a vibration magnetometer, if the time period of B (b) normal bisector of the magnetic axis
is twice the time period of A. The ratio of the mag-
(c) magnetic centre (d) magnetic axis.
netic moments MA/MB of the magnets will be
[AFMC 1995]
(a) 1/2 (b) 2
(c) 4 (d) 1/4. [MP PET 1990] 170. The core of an electromagnet is made of soft iron
because soft iron has
162. Above Curie temperature, a paramagnetic substance
(a) small susceptibility and small retentivity
becomes
(b) large susceptibility and small retentivity
(a) paramagnetic (b) diamagnetic
(c) large density and large retentivity
(c) ferromagnetic (d) super conductor.
(d) small density and large retentivity. [CPMT 1990]
[AFMC 1994]
171. The arms of a deflection magnetometer in the tan-
163. The magnetic field due to a magnetic dipole of mag-
gent A position are placed along
netic moment M at a point at a distance d from its
(a) the east-west direction (b) the north-south direction
centre on a line inclined at an angle θ with the axis is
given by (c) the north east-south west direction

M M (d) the north west-south east direction.


(a) (b)
d3 d2 172. Which material moves from stronger to weaker mag-
2M M netic field ?
(c) (d) 1 + 3 cos2 θ .
d3 d3 (a) Diamagnetic (b) Paramagnetic
[Pb. CET 1994] (c) Ferromagnetic (d) An biferro magnetic.
164. Earth’s magnetic field always has a horizontal com- 173. A small compass needle is placed at the centre of the
ponent except at circular coil in a tangent galvanometer. This is done
(a) equator (b) magnetic pole to ensure that
(c) a latitude of 60° (d) an altitude of 60°.
266 COMPREHENSIVE OBJECTIVE PHYSICS

(a) the compass needle does not produce a strong magnetic (a) zero (b) 45°
field (c) 90° (d) 180°.
(b) the galvanometer does not have a large size 180. Force between two identical bar magnets whose cen-
(c) the moment of inertia is large tres are r metre apart is 4.8 N, when their axes are in
(d) the compass needle moves in the region of uniform mag- the same line. If separation is increased to 2r, the
netic fields. [CMC LDH 1996] force between them is reduced to
174. Two short magnets placed along the same axis with (a) 2.4 N (b) 1.2 N
their like poles facing each other repel each other with (c) 0.6 N (d) 0.3 N. [AIIMS 1995]
a force which varies inversely as 181. Of dia, para and ferromagnetism, the universal prop-
(a) distance (b) square of distance erty of all substances is
(c) cube of distance (d) fourth power of distance. (a) diamagnetics (b) paramagnetics
(c) ferromagnetics (d) all of these.
[CMC Vellore 1999]
175. A magnet of magnetic moment M is freely suspended 182. Domain formation is the necessary feature of
in a uniform magnetic field of strength B. The work (a) diamagnetism (b) paramagnetism
done in deflecting the magnet through an angle θ is (c) ferromagnetism (d) all of these. [BHU 1997]
given by 183. Property possessed by ferromagnetic substance only
(a) MB (b) MB sin θ is
(c) MB cos θ (d) MB (1 – cos θ). (a) attracting magnetic substances
[MP PMT 1996, 1999] (b) hysteresis
176. The strength of the magnetic field in which the mag- (c) susceptibility independent of temperature
net is of a vibration magnetometer is oscillating is (d) directional property. [Karnataka CET 1999]
increased to 4 times its original value. The frequency
184. The incorrect statement regarding the lines of force
of oscillation would then become : →
of the magnetic field B is
(a) twice (b) four times
(a) Magnetic intensity is a measure of lines of force passing
(c) half (d) one-fourth.
through unit area held normal to it.
[All India PM/PD 2000] (b) Magnetic lines of force form a closed curve.
177. Magnetic field of a magnet is similar to which of the (c) Inside a magnet, its magnetic lines of force move from the
following ? north pole of a magnet towards its south pole.
(a) Magnetic field of a straight wire carrying current (d) Due to a magnet, magnetic lines of force never cut each
(b) Magnetic field of a straight wire carrying alternating cur- other. [MP PET 1999]
rent 185. The lines joining the places of the same horizontal
(c) Magnetic field due to a coil of wire carrying current intensity are known as
(d) Magnetic field due to coiled wire carrying alternating cur- (a) Isogonic lines (b) Aclinic lines
rent. [Pb. CET 2000] (c) Isoclinic lines (d) Agonic lines
178. In sum and difference method, the ratio of M1 and (e) Isodynamic lines. [BHU 1998]
M2 is given by
186. The magnetic susceptibility of material is 7.54 ×
M T 2 + T22 M T 2 − T22 10–3. The nature and relative permeability of the sub-
(a) 1 = 12 (b) 1 = 12
M2 T1 − T22 M2 T1 − T22 stance is
(a) diamagnetic (b) paramagnetic, 1.0075
M1 T22 + T12 M1 T22 − T12
(c) = (d) = . (c) ferromagnetic (d) none of these.
M2 T22 − T12 M2 T22 + T12
[CMC Vellore 1996]
[Manipal 1997]
→ 187. Temperature above which a ferromagnetic substance
179. A bar magnet of magnetic moment M is placed in a becomes paramagnetic is called

uniform magnetic field of induction B such that it (a) Critical temperature (b) Boyle’s temperature

makes an angle θ with B . Then the position of stable (c) Debye’s temperature (d) Curie temperature.
equilibrium of the magnet is given by θ equal to [JIPMER 1997]
MAGNETIC EFFECTS OF CURRENT AND MAGNETISM 267

188. Which of the following has highest magnetic perme- 196. The magnet can be completely demagnetised by
ability ? (or magnetic permeability is maximum for) (a) heating it slightly
(a) Paramagnetic substances (b) Diamagnetic substances (b) dropping it into ice cold water
(c) Ferromagnetic substances(d) Vacuum. (c) a reverse field of appropriate strength
[CMC LDH 1995] (d) breaking the magnet into small pieces.
189. If a magnet is divided into 2n parts by cutting along 197. The angle of dip at the magnetic equator is
lines parallel to length, then pole strength of each (a) 0° (b) 45°
part will be (c) 30° (d) 90°.
(a) Same (b) 1/n times 198. The direction of earth’s magnetic field at a place is
(c) 1/2n times (d) 1/4n times. (a) from north to south (b) from south to north
[All India PM/PD 1996] (c) from magnetic north to south
190. Vibration magnetometer is used for comparing (d) from magnetic south to north.
(a) Magnetic field (b) Earth’s field 199. For which of the following substances, the magnetic
(c) Magnetic moments (d) All of these. susceptibility is independent of temperature
[MP PET/PMT 1988] (a) diamagnetics only (b) paramagnetics only
191. When a south pole is placed in the magnetic field of a (c) ferromagnetics only
north pole, the south pole moves towards the north (d) diamagnetics and paramagnetics both.
pole and its potential energy 200. At a neutral point in the neighbourhood of a bar mag-
(a) decreases (b) increases net placed in earth’s magnetic field,
(c) remains unchanged (d) gets doubled. (a) the magnetic field of the magnet is zero
[AMU 1996] (b) the fields of the earth and the magnet are equal in magni-
tude and oppositely directed
192. The susceptibility of a ferromagnetic material is K at
27°C. At what temperature will its susceptibility be (c) a magnetic compass needle will point perpendicular to the
0.5 K ? bar magnet
(a) 54°C (b) 327°C (d) a magnetic compass needle will align itself parallel to the
bar magnet.
(c) 600°C (d) 237°C.
201. The desirable properties for making permanent mag-
[Pb. PMT 1997]
nets are
193. Magnets cannot be made from which of the following
(a) high retentivity and high coercive force
substances ?
(b) high retentivity and low coercive force
(a) Iron (b) Nickel
(c) low retentivity and high coercive force
(c) Copper (d) All of these.
(d) low retentivity and low coercive force. [AIIMS 1997]
[Bihar PET 1984]
202. µ and k stand for the permeability and magnetic sus-
194. The meniscus of a liquid contained in one of the limbs
ceptibility of a susbtance. It is a paramagnetic if
of a narrow U-tube is placed between the pole pieces
(a) µ = 1.00002 and k = 1.71 × 10–6
of an electromagnet with the meniscus in a line with
the field. The liquid is seen to rise to line. This (b) µ = 1.00002 and k = – 1.71 × 10–6
indicates that the liquid is (c) µ = 1.71 × 106 and k = 1.0002
(a) ferromagnetic (b) paramagnetic (d) µ = 1.0002 and k = 1.71 × 10–6.
(c) diamagnetic (d) non-magnetic. 203. The error in measuring the current with a tangent
[MNR 1994] galvanometer is minimum when the deflection is
about
195. A magnet is placed in iron powder and then taken
(a) 0° (b) 30°
out. Then maximum iron powder is at
(c) 45° (d) 60°. [MP PET 2001]
(a) some away from north pole
(b) some away from south pole 204. A current of 2 ampere produces a deflection of 30° in
(c) the middle of the magnet a tangent galvanometer. A deflection of 60° will be
(d) the end of the magnet. produced in it by a current of
268 COMPREHENSIVE OBJECTIVE PHYSICS

(a) 1 A (b) 3 A (c) perpendicular to the field


(c) 4 A (d) 6 A. [MP PMT 2001] (d) in none of the above directions

205. Before using the tangent galvanometer, its coil is set [All India PM/PD 2003]
in 212. The Magnetic Resonance Imaging (MRI) is based on
(a) magnetic meridian the phenomenon of
(a) Nuclear magnetic resonance.
(b) perpendicular to magnetic meridian
(b) Electron spin resonance.
(c) at angle of 45° to magnetic meridian
(c) Electron paramagnetic resonance
(d) It does not require any setting. [MP PMT 2001]
(d) Diamagnetism of human tissues. [AIIMS 2004]
206. The angle of dip at a place on the earth gives
213. A frog can be levitated in a magnetic field produced
(a) the horizontal component of the earth’s magnetic field by a current in a vertical solenoid placed below the
(b) the location of the geographic meridian frog. This is possible because the body of the frog
(c) the vertical component of the earth’s field behaves as
(d) the direction of the earth’s magnetic field. (a) paramagnetic (b) diamagnetic
(c) ferromagnetic (d) antiferromagnetic.
[MP PET 1994]
[AIIMS 2003]
207. Out of the following, which is non-magnetic ? 214. The material used for permanent magnet has
(a) Zinc (b) Steel (a) low retentivity, high coercivity
(c) Copper (d) Cobalt. (b) high retentivity, low coercivity
208. The ultimate individual unit of magnetism in any (c) high retentivity, high coercivity
magnet is called magnetic (d) low retentivity, low coercivity
(a) North pole (b) South pole [Karnataka CET 2003]
(c) Dipole (d) Quadrupole. 215. Liquid oxygen remains suspended between two pole
209. The magnetic lines of force inside a bar magnet faces of a magnet because it is
(a) are from south-pole to north-pole of the magnet (a) diamagnetic (b) paramagnetic
(b) are from north-pole to south-pole of the magnet (c) ferromagnetic (d) antiferromagnetic

(c) do not exist [AIIMS 2004]


(d) depend upon the area of cross-section of the bar magnet. 216. The materials suitable for making electromagnets
should have
[AIEEE 2003]
(a) high retentivity and low coercivity
210. Curie temperature is the temperature above which (b) low retentivity and low coercivity
(a) a paramagnetic material becomes ferromagnetic (c) high retentivity and high coercivity
(b) a ferromagnetic material becomes paramagnetic (d) low retentivity and high coercivity [AIEEE 2004]
(c) a paramagnetic material becomes diamagnetic 217. A bar magnet of magnetic moment M is cut into two
(d) a ferromagnetic material becomes diamagnetic parts of equal length. The magnetic moment of either
[AIEEE 2003] part is
211. A diamagnetic material in a magnetic field moves (a) M (b) 2M
(a) from stronger to the weaker parts of the field (c) M/2 (d) zero.
(b) from weaker to the stronger parts of the field [Himachal PMT 1998]

Answers (Set I)
1. (b) 2. (a) 3. (a) 4. (b) 5. (b) 6. (a) 7. (a) 8. (d)
9. (d) 10. (b) 11. (d) 12. (d) 13. (d) 14. (a) 15. (c) 16. (b)
17. (a) 18. (d) 19. (c) 20. (a) 21. (c) 22. (c) 23. (c) 24. (a)
25. (b) 26. (a) 27. (b) 28. (b) 29. (b) 30. (d) 31. (c) 32. (a)
MAGNETIC EFFECTS OF CURRENT AND MAGNETISM 269

33. (a) 34. (a) 35. (c) 36. (a) 37. (b) 38. (c) 39. (d) 40. (c)
41. (d) 42. (a) 43. (d) 44. (e) 45. (c) 46. (c) 47. (c) 48. (b)
49. (c) 50. (d) 51. (a) 52. (e) 53. (a) 54. (d) 55. (b) 56. (d)
57. (c) 58. (d) 59. (a) 60. (c) 61. (d) 62. (a) 63. (d) 64. (b)
65. (b) 66. (c) 67. (c) 68. (b) 69. (d) 70. (d) 71. (d) 72. (b)
73. (d) 74. (d) 75. (d) 76. (c) 77. (b) 78. (b) 79. (a) 80. (c)
81. (c) 82. (b) 83. (b) 84. (d) 85. (c) 86. (c) 87. (d) 88. (d)
89. (d) 90. (d) 91. (a) 92. (b) 93. (d) 94. (a) 95. (b) 96. (d)
97. (d) 98. (d) 99. (c) 100. (d) 101. (a) 102. (c) 103. (c) 104. (b)
105. (c) 106. (b) 107. (b) 108. (c) 109. (c) 110. (a) 111. (b) 112. (a)
113. (d) 114. (b) 115. (a) 116. (d) 117. (c) 118. (c) 119. (b) 120. (b)
121. (c) 122. (b) 123. (d) 124. (a) 125. (b) 126. (c) 127. (a) 128. (a)
129. (b) 130. (b) 131. (a) 132. (a) 133. (c) 134. (b) 135. (c) 136. (c)
137. (a) 138. (b) 139. (a) 140. (b) 141. (b) 142. (d) 143. (d) 144. (a)
145. (c) 146. (d) 147. (b) 148. (c) 149. (c) 150. (a) 151. (a) 152. (a)
153. (d) 154. (d) 155. (b) 156. (c) 157. (a) 158. (c) 159. (d) 160. (b)
161. (c) 162. (b) 163. (d) 164. (b) 165. (a) 166. (b) 167. (d) 168. (a)
169. (d) 170. (b) 171. (a) 172. (a) 173. (d) 174. (d) 175. (d) 176. (a)
177. (c) 178. (c) 179. (a) 180. (d) 181. (a) 182. (c) 183. (b) 184. (c)
185. (e) 186. (b) 187. (d) 188. (c) 189. (c) 190. (d) 191. (a) 192. (b)
193. (c) 194. (b) 195. (d) 196. (c) 197. (a) 198. (d) 199. (a) 200. (b)
201. (a) 202. (a) 203. (c) 204. (d) 205. (a) 206. (d) 207. (a) 208. (c)
209. (a) 210. (b) 211. (a) 212. (a) 213. (b) 214. (c) 215. (b) 216. (b)
217. (c).

Solutions (Set I)
1.
z → →
B . dl = µ0 × 0 = 0.

2. Magnetic field can not alter the speed of the charged


6. The electrostatic repulsion shall dominate the mag-
netic attraction.
mv2 mv
7. Bqv = or r =
particle. It can merely change the direction of motion r Bq
of the charged particle. v and B are constants.
4. No current is enclosed. Using Ampere’s circuital law, m
B = 0. ∴ r∝
q
µ 0 nI
5. B = rp mp qα mp 2qp 1
2r = × = × =
rα qp mα qp 4m p 2
l
But l = 2πr × n or r = 2πm
2πn 8. T =
Bq
µ0 n2I × 2π
∴ B= or B ∝ n2 Clearly, T is independent of velocity.
2l
This leads us to the right choice.
270 COMPREHENSIVE OBJECTIVE PHYSICS

+ A common misconception
∴ B2 µ 0I 2 π × 1 × 10 −2 = 0.02 = 1
= ×
B1 π µ 0I 50
mv2
Bqv = = mrω2 µ0 I
r 16. B=
LM 4π 2 OP 2r
or mr
MN T2
PQ = Bqv B=
µ0 e
2r T
Can you say that T depends upon v ? If no, why ?
µ 0 ev
B=
2 r 2 πr
µ 0IR 2
9. B = v v
2(R 2 + r 2 )3 / 2 or r2 ∝ or r ∝
B B
If r >> R, then R2 can be neglected in comparison to
N
r2. 17. B = µ0 I
l
2
µ 0IR 1
∴ B= or B ∝ 4 π × 10 −7 × 10 × 5
2r 3 r3 = T = 2π × 10–3 T.
1 × 10 −2
mv2
10. Bqv = 18. Apply Fleming’s left hand rule.
r
mv 19. M = IA = Iπr2
Bq = L
r But L = 2πr or r =
p 2m Ek 2π
or Bq =
r
or Bq =
r LM L OP 2

2m Ek
∴ M = Iπ
N 2π Q
or r= IL2
Bq
or M= .

In the given problem, B, Ek and q are constants.
B2
∴ m r∝ 20. u =
2µ 0
m is more for proton. r is more. So, trajectory is less
or u ∝ B2
curved.
The graph is a symmetric parabola about u-axis and
11. Velocity is parallel or anti-parallel to magnetic field.
passing through the origin.
So, force experienced is zero.
21. Applying right hand thumb rule.
12. M = NIA or M = NI(πr2) or M ∝ r2
22. B = µ0 nI
mv p 2m E k B ∝ I.
13. Bq = = =
r r r
LM OP
1 µ 0I µ I
= 0
or r=
2m Ek
23. B =
4 2R N Q 8R
Bq 24. Magnetic field exerts zero force on stationary charge.
⇒ Ek ∝ B2 25. For the paticle to move
along anticlockwise path, Note the special tech-
14. Magnetic fields due to two parts will be equal and nique adopted in this
opposite. force should be along j . solution.
µ 0I Velocity is along i .
15. B1 =
2π × 1 × 10−2 → → →
Now Fm = – e ( v × B)
µ 0I
Again, B2 = In terms of unit vectors only,
2r
But 2πr = π2 j = – (i × ?)
 or j = ? × i
or 2r = π
Clearly ? is k .
∴ B2 = µ 0 I 26. The magnetic field at a point on the axis of the cur-
π
rent element is zero.
MAGNETIC EFFECTS OF CURRENT AND MAGNETISM 271

FG IJ = µ I .
1 µ 0I 43. Applying Fleming’s left hand rule.
27.
2 2R H K 4R 0
44. Two solenoids in contact coaxially and have the same
current direction act like one long solenoid. If the end
28. Magnetic field at O due to straight wires is zero. flux density is B, the centre density is 2B.
29. It can be shown that the force F between two long, or
parallel current-carrying conductors a distance d apart
The two ends in contact have the same current sense,
µ 0I1I2l therefore the flux densities add in magnitude. These
is F =
2 πd two ends become the centre with density B.
where I1I2 are the currents in the conductors and l 45. Magnetic lines of force are parallel to circular loop.
is the length of each conductor. 47. When θ = 90°, then sin θ has maximum value.
Hence, force F should be plotted against 1/d in order 48. For inside points, enclosed current is zero.

30.
to obtain a straight-line graph.
Fm = Bqv sin θ
In the given problem, θ = 0°.
z → →
B . dl = 0.
49. The magnetic force on each element is balanced by an
31. F = Bqv equal and opposite magnetic force on the diametri-
cally opposite element.
1 2eV
But mv2 = eV or v = 50. Fm = Bqv
2 m
Fm ∝ q.
2eV → → →
∴ F = Bq 51. τ = I (A × B)
m
In the given problem,
F∝ V → →
A || B .
F′ ∝ 2V
52. The larger coil induces magnetic flux density B on
F′ the small coil in the direction pointing into the paper
= 2 or F′ = 2 F.
F as shown by the cross signs. The magnetic flux den-
→ → → sity in turn induce forces on the small coil which are
32. F = I ( l × B) all radially outwards from the small coil.
→ → →
If l || B , then F = 0.
33. Torque does not depend upon shape of coil.
34. Magnetic field does no work.
35. Neutron follows straight path. This is because of zero
force.
mv2
36. Bqv =
r
mv Fig. 82
or r= or r ∝ m
Bq The net resultant force is nil because the force on one
Proton has less mass. So, r is less. point of the small coil is always counter balanced by a
37. NBIA = kθ or I ∝ θ. force of same magnitude on the opposite point along
38. B = µ0nI. Clearly, B does not depend upon radius. the diameter of the small coil.
39. The force on electron will be opposite to the direction → → →
53. Fm = q ( v × B)
of electric field.
→ →
1 v and B are anti-parallel.
40. B ∝ .
r →
41. The electrostatic force dominates the magnetic force. ∴ Fm = 0.
42. r = 0.5 × 10–2m, I = 10 A 54. In an electric field (E), the charged particle (q) will
µ 0I 4 π × 10 × 10−7 always experience a force (qE) whether or not it is in
B= = T = 4π × 10–4 T motion.
2r 2 × 0.5 × 10−2
272 COMPREHENSIVE OBJECTIVE PHYSICS

GM FG IJ 71.
→ →
τ = NI (A × B)

In a gravitational field g =
R2 H K
, the charged parti-
→ → →
Since A || B , ∴ τ = 0.
cle of certain mass (m) will always experience a force
72. No enclosed current.
FG GMm IJ whether or not it is in motion.
H R K 2
73. γ-rays are uncharged.
→ → →
On the other hand, the charged paticle (q) will experi- 74. F = I ( l × B)
→ → →
ence a force (qvB) only when it is in motion (velocity Since l || B , ∴ F = 0.
of v) if it is placed in a magnetic field (B).
75. If Bev = eE, then the electron shall move undeflected.
55. The small resistance is called shunt resistance.
76. Since the work done by magnetic field is zero there-
→ → →
56. B || v ∴ Fm = 0 fore the kinetic energy remains constant.
57. When charge moves parallel to magnetic field, mag- 77. M = IA
netic force is zero. e
M = πr2
58. Fm = Bqv T
= 2 × 10–2 × 1.6 × 10–19 × 5 × 106 N ev
M= × πr2
= 1.6 × 10–14 N. 2πr
evr
M =
+ – 2
l
But l = mvr or r =
59. The two beams constitute mv
like currents. Like currents + – ev l
∴ M = ×
attract. 2 mv
e
+ – or M = l.
2m
78. q = 2 × 1.6 × 10–19 C
Fig. 83 B = 0.5 T
µ 0 2I1I2 µ 2 × 1× 1 v = 5 × 106 m s–1
µ
61. F= = 0 = 0N Fm = Bqv
4π r 4π 1/2 π
63. The magnetic force shall become centripetal force. = 0.5 × 2 × 1.6 × 10–19 × 5 × 106 N
64. F = BIl = 8 × 10–13 N.
Now, 1Wb m–2 A m = 1Wb A m–1 = 1 N. 79. The orientations of the magnetic dipole and current
→ loop are perpendicular.
65. Upper face SOUTH. M from south to north.
2πm
68. Apply Right hand thumb rule. 82. T =
Bq
I2 1A T ∝ m
69. F ∝ A
r Since proton has larger mass therefore time period is
FG I IJ 2
1 more.
F′ ∝
H 3K ×
2r
10 A

9A S
83.
→ →
τ = I ( A × B)

F → → →
F′ ∝ . Fig. 84 Since A || B , ∴ τ = 0.
18
84. Consider an analogous situation. Assume that a posi-
RA 9 tive charge is travelling due west. Now, apply
70. = Fleming’s left hand rule.
S 1
86. Magnetic field cannot change the speed of the charged
particle.
+ In parallel combination, the currents 89. Magnetic fields are equal in magnitude and opposite
divide themselves in the inverse ratio in direction.
of resistances.
MAGNETIC EFFECTS OF CURRENT AND MAGNETISM 273

90. For reducing the range of a voltmeter, we have to 100. 1.07 = µ0µr × 150
reduce the resistance in series with the galvanom- 1.07
eter. This cannot be achieved by connecting a resist- or µr =
µ0 × 150
ance in parallel with the voltmeter.
1.07 1.07 107
= = × .
+ The range of a voltmeter can only be 4 π × 10 −7
× 150 150 4π
increased. It cannot be decreased.
BV tan δ
103. tan δ′ = =
BH cos θ cos θ
91. M = IA
cos θ < 1, tan δ′ > tan δ
M
I= . δ′ > δ.
A
µ I 1 MB
92. B = 0 106. ν=
2πr 2π I
1 When pole strength is increased four times, M is also
B∝ .
r increased four times.
93. F = Bqv sin 30° Clearly, ν is doubled.
Bev 107. A short magnet is used because the magnetic field is
=.
2 uniform over only a small region.
94. Magnetic fields due to two sections cancel out.
112. The magnetic field in tan B position is half of the
2rH magnetic field in tan A position.
95. K =
µ0 N 115. In the given plane, BH = 0.
µ 0N 116. Magnetic screening.
=1 (given)
r 117. Horizontal oscillations due to BH.
K = 2H 118. In the given plane, B is effective. In a plane perpen-
K dicular to the given plane, only BV exists. Also, f ∝
or H=
2 B ; BV < B ; f ′ < f.
1 122. M = m × 2l
96. F ∝ 4 .
r M 5 1
or 2l = = M = m = 20 cm.
m 25 5
+ Force between two magnets is inversely
proportional to the fourth power of the + Note that l represents half-length of the
distance between them.
magnet and not the complete length.
97. The magnet is back to original position.
BV tan θ
128. tan θ′ = =
BH cos β cos β
+ An interesting fact. No work done, if
θ = 2nπ, where n = 0, 1, 2, 3, ...... I
147. T = 2π MBH
BV BV 1 At poles, BH = 0
98. tan δ = = =
BH 3 BV 3 T = ∞.
δ = 30°. 148. Here it is assumed that d is very large.
99. At the magnetic pole of earth, BH = 0. 149. If the temperature is decreased, the thermal vibra-
tions will be reduced. So, there would be no negative
+ In a compass needle, the magnetic nee- effect on magnetisation.
dle oscillates in the horizontal plane.
+ Note that an increase in temperature
promotes “disorder”.
274 COMPREHENSIVE OBJECTIVE PHYSICS

150. Note that ballistic galvanometer shall help measure


1
current impulses. For measuring magnetic moments 180. F′ = F
of bar magnets and earth’s magnetic field, we require 24
vibration magnetometer.
1
151. The magnetic permeability of paramagnetics is only or F′ = × 4.8 N = 0.3 N.
16
slightly greater than one.
152. Only the magnetic moment will be affected. 186. µr = 1 + χ m
= 1 + 0.00754 = 1.00754
+ If you do not disturb the cross-section Since µr is only slightly greater than 1 therefore the
of the magnet, then the number of cur- material is paramagnetic.
rent loops contributing to pole strength
192. 0.5 T = K (273 + 27) or T = 600 K
is not changed.
or t = (600 – 273)°C = 327°C.
153. In a plane perpendicular to the magnetic meridian, 193. Copper is diamagnetic.
BH = 0. Only BV exists. 194. Paramagnetic liquid tends to move to stronger mag-
netic field.
I
154. T = 2π 195. The pole strength is maximum at the end of the mag-
MB
net.
1 196. The methods mentioned in (a), (b) and (d) can merely
T∝
B reduce magnetic field.
B1 T22 4.9 × 4.9 4 197. At the magnetic equator, BV = 0.
or = 2 = = 199. Susceptibility of diamagnetics is independent of tem-
B2 T1 2.45 × 2.45 1
perature.
155. µr = 1 + χm
χm = 5500 – 1 = 5499. + Bismuth at low temperature is an ex-
159. The needle loses its magnetisation. ception to the general property men-
160. The product of pole strengths is increased by a factor tioned in sol. 199.
of 4. The square of distance is also increased by a
factor of 4. 202. In the case of paramagnetics, µ is only slightly greater
than one and k is low but positive.
I
161. T = 2π 203. The tangent galvanometer has maximum sensitivity
MB
when θ = 45°.
1
T ∝ I
M tan 60° 3× 3
204. = = =3
2 tan 30° 1
M A TB2 4
or = 2
= . or I = 6 A.
MB TA 1
213. The frog is diamagnetic as is any other animal.
166. Diamagnetic liquid tends to move “away” from the 216. Materials used for making electromagnets should have
“stronger” magnetic field.
the following properties :
1 MB (i) low retentivity
176. ν =
2π I (ii) high value of saturation magnetisation
ν∝ B (iii) low coercivity
If B is quadrupled, then ν is doubled. (iv) small hysteresis loss.
179. When θ = 0°, then τ = 0. 217. Pole strength is unchanged and length is halved.

+ What is the “acid test” for stable equi-


librium position?
“Minimum Potential Energy”
MAGNETIC EFFECTS OF CURRENT AND MAGNETISM 275

KNOWLEDGE PLUS
l A conducting loop carrying a current I is placed in a uniform magnetic field pointing into y
B
the plane of the paper as shown in Fig. 85. The loop will have a tendency to Ä

(a) contract (b) expand


(c) move towards +ve x-axis (d) move towards – ve x-axis. x
[IIT Screening 2003] I

Sol. It will expand due to radially outward force.


So, (b) is the right choice. Fig. 85

l A galvanometer of resistance 100 Ω is converted to ammeter with a resistance of 0.1 Ω. The galvanometer shows
full scale deflection with a current of 100 µA. Then the minimum current in the circuit for full scale deflection of
galvanometer will be
(a) 100 mA (b) 10.01 mA
(c) 1.001 mA (d) 0.1001 mA. [IIT Screening 2005]
1 1 1 1 1 1 1 1
Sol. = + or = − = − = 10 – 0.01
Ra G S S R a G 0.1 100

1 100
or = 9.99 or S = Ω
S 999
GI g GI g
Now, S= or I – Ig =
I − Ig S

100 × 100 × 999 FG 100 IJ


or I – Ig =
1000 × 100
= 99.9 mA
H
or I = 99.9 +
1000 K
mA = 100 mA

So, (a) is the right choice.

l Select the correct statement(s) from the following :


The path of a charged particle moving with an initial velocity u
(a) parallel to the uniform electrostatic field is a straight line.
(b) perpendicular to the uniform electrostatic field is a parabola.
(c) parallel to the uniform magnetic field is a helix with its axis parallel to the direction of B.
(d) perpendicular to the uniform magnetic field is a circle in a plane normal to the field.
[National Standard Exam. in Physics 2005]
Sol. In the case of (a), the force is in the direction of the field. In (b), it is a combination of uniform motion and
uniformly accelerated motion. In (c), magnetic force is giving the necessary centripetal force.
So, (a), (b) and (d) are the correct options.
276 COMPREHENSIVE OBJECTIVE PHYSICS

MCQ
based on
SET II
Important Basic Concepts
+
Application of Formulae
+
Brain Teasers
Average time allowed per question is 50 seconds.

MAGNETIC EFFECTS OF CURRENT Which one of the following combinations is possible ?


→ → → →
(a) E = 0; B = bi + ck (b) E = ai ; B = ck + ai
218. A wire in the form of a circular loop of one turn carry-
ing a current produces a magnetic field B at the centre. → →
(c) E = 0; B = cj + bk
→ →
(d) E = ai ; B = ck + bj
If the same wire is looped into a coil of two turns and
carries the same current, the new value of magnetic [IIT Screening 2003]
induction at the centre is 222. The seave of a galvanometer of resistance 100 Ω con-
(a) 3 B (b) 5 B tains 25 divisions. It gives a deflection of one division
(c) 4 B (d) 2 B on passing a current of 4 × 10–4 ampere. The resistance
in ohm to be added to it, so that it may become a volt-
[Karnataka CET 2003]
meter of range 2.5 volt is
219. The earth’s magnetic field at a given point is 0.5 × 10–5 (a) 100 (b) 150
Wb m–2. This field is to be annulled by magnetic induc-
tion at the centre of a circular conducting loop of ra- (c) 250 (d) 300. [EAMCET 2003]
dius 5.0 cm. The current required to be flown in the 223. An electron moving with a y
loop is nearly speed u along the positive × × × ×
(a) 0.2 A (b) 0.4 A x-axis at y = 0 enters a re- × × × ×
gion of uniform magnetic × × × ×
(c) 4 A (d) 40 A [AIIMS 2003] →
– u
e
220. A particle of charge –16 × 10 –18
C moving with velocity field B = − B k which exists
0 × × × × x
10 m s–1 along the x-axis enters a region where a mag- to the right of y-axis. The × × × ×
netic field of induction B is along the y-axis and an electron exits from the region
electric field of magnitude 104 Vm–1 is along the nega- after some time with the Fig. 87
tive z-axis. If the charged particle continues moving speed v at co-ordinate y, then
along the x-axis, the magnitude of B is (a) v > u, y < 0 (b) v = u, y > 0
(a) 10 –3 Wb m–2 (b) 103 Wb m–2 (c) v > u, y > 0 (d) v = u, y < 0
(c) 105 Wb m–2 (d) 1016 Wb m–2 [IIT Screening 2004]
[AIEEE 2003] 224. Five very long, straight, insulated wires are closely
221. For a positively charged particle y bound together to form a small cable. Currents car-
moving in a x-y plane initially P ried by the wires are : I1 = 20 A, I2 = – 6 A, I3 = 12 A,
along the x-axis, there is a sud- I4 = – 7 A, I5 = 18 A. {Negative currents are opposite
den change in its path due to the x in direction to the positive]. The magnetic field induc-
presence of electric and/or mag-
tion at a distance of 10 cm from the cable is
netic fields beyond P. The curved
Fig. 86 (a) 5 µ T (b) 15 µ T
path is shown in the x-y plane
and is found to be non-circular. (c) 74 µ T (d) 128 µ T.
[Haryana PMT 1996]
MAGNETIC EFFECTS OF CURRENT AND MAGNETISM 277

225. ABCD is a square loop B C µ 0I


(a) Zero (b)
made of a uniform conduct- 2R
ing wire. The current en- µ 0I µ 0I
(c) (d) . [MP PMT 1997]
ters the loop at A and 4R 6R
leaves at D. The magnetic 230. Two particles X and Y, having equal charges, after be-
field is A D ing accelerated through the same potential differences
(a) zero only at the centre of enter a region of uniform magnetic field and describe
the loop circular paths of radii R1 and R2 respectively. The ra-
(b) maximum at the centre of Fig. 88 tio of the mass of X to that of Y is
the loop
(c) zero at all points outside the loop (a)
LM R1 OP1/2 (b)
R2
(d) zero at all points inside the loop. N R2 Q R1
[Karnataka CET 2001] L R O2
(c) M 1 P (d)
R1
. [IIT 1998]
226. Two particles A and B of
masses m A and m B re-
N R2 Q R2

spectively and having the 231. The magnetic field due to a current carrying circular
A
same charge are moving loop of radius 3 cm at a point on the axis at a distance
in a plane. A uniform of 4 cm from the centre is 54 µT. What will be its value
magnetic field exists per-
B at the centre of the loop ?
pendicular to this plane. (a) 125 µT (b) 150 µT
The speeds of the particles (c) 250 µT (d) 75 µT [AIEEE 2004]
are vA and vB respectively 232. A proton, a deuteron and α-particle with the same K.E.
and the trajectories are as Fig. 89 enter a region of uniform magnetic field B moving at
shown in Fig. 89. Then right angles to the field. What is the ratio of the radii of
(a) mA vA < mB vB (b) mA vA > mB vB their circular paths ?
(c) mA < mB and vA < vB (d) mA = mB and vA = vB. (a) 1 : 2 :1 (b) 1 : 2 : 2
[IIT Screening 2001]
(c) 2 :1:1 (d) 2 : 2 : 1.
227. A coil having N turns is wound tightly in the form of a
233. A β-particle moving with a
spiral with inner and outer radii a and b respectively.
speed of 106 m s–1 enters into
When a current I passes through the coil, the magnetic

B = 0.2 T
a region of uniform magnetic
field at the centre is
field of 0.2 T. The force expe-
µ 0 NI 2µ 0NI rienced by the β-particle is
(a) (b) 30°
b a (a) 2.77 × 10–14 N
µ 0NI b µ 0I N b (b) 1.6 × 10–14 N
(c) ln (d) ln . Fig. 91
2(b − a) a 2(b − a) a (c) 5.54 × 10–14 N
[IIT Screening 2001] (d) 2.77 × 1014 N.
228. A galvanometer of 50 ohm resistance has 25 divisions. 234. Two long conductors, separated by a distance d carry
A current of 4 × 10–4 ampere gives a deflection of one currents I1 and I2 in the same direction. They exert a
division. To convert this galvanometer into a voltme- force F on each other. Now the current in one of them is
ter having a range of 25 volt, it should be connected increased to two times and its direction is reversed.
with a resistance of The distance is also increased to 3d. The new value of
(a) 245 Ω as a shunt (b) 2550 Ω in series the force between them is
(c) 2450 Ω in series (d) 2500 Ω as a shunt 2F F
(a) − (b)
[All India PM/PD 2004] 3 3
229. A uniform wire is bent in the B F
(c) – 2F (d) − [AIEEE 2004]
form of a circle of radius R. A C 3
current I enters at A and A
235. The cyclotron frequency of an electron gyrating in a
leaves at C as shown in magnetic field of 1 T is approximately
O
Fig. 90. If the length ABC is
(a) 28 MHz (b) 280 MHz
half of the length ADC, the
magnetic field at the centre O D (c) 2.8 GHz (d) 28 GHz [AIIMS 2004]
will be Fig. 90
278 COMPREHENSIVE OBJECTIVE PHYSICS

236. A 100 turns coil shown in figure carries a current of 243. A magnetic field of 0.004 k tesla exerts a force of 4i + 3j
2 A in a magnetic field B of 0.2 Wbm– 2. The torque × 10–10 N on a particle having charge of 10–9 C and
acting on the coil is moving in x-y plane. The velocity of particle is
A B (a) (75 i + 100 j ) m s–1 (b) (75 i − 100 j ) m s–1
(c) (− 75 i + 100 j) m s–1 (d) (− 75 i − 100 j ) m s–1.
[MNR 1996]
10 cm

N S
244. A wider conductor strip of width l is bent into a slen-
der tubing of radius r with its two ends forming two
plane extensions. A current I flowing through it gives
D 8 cm C magnetic field in tubular portion given by
Fig. 92 µ 0I µ 0I
(a) (b)
2 πr 2l
(a) 0.32 Nm tending to rotate the side AD out of the page
(b) 0.32 Nm tending to rotate the side AD into the page µ 0I µ 0I
(c) (d) . [MNR 1998]
(c) 0.0032 Nm tending to rotate the side AD out of the page πl l
(d) 0.0032 Nm tending to rotate the side AD into the page. 245. At a distance of 10 cm from a long straight wire carry-
[MP PET 1997] ing current, the magnetic field is 0.04 T. At the dis-
237. A square conducting loop of length L on a side carries tance of 40 cm, the magnetic field will be
a current I. The magnetic field at the centre of the loop (a) 0.01 T (b) 0.02 T
is (c) 0.08 T (d) 0.16 T. [MP PMT 1997]
(a) independent of L
246. A wire of length L is shaped into a circle and then bent
(b) proportional to L2
in such a way that the two semi-circles are perpendicu-
(c) inversely proportional to L
lar. What is the magnetic moment of the system when
(d) linearly proportional to L. [MNR 1995]
current I flows through the system ?
238. A beam of protons with a velocity of 4 × 105 m s–1 en-
ters a uniform magnetic field of 0.3 T at an angle of 2 L2 I 3 L2 I
(a) (b)
60° to the magnetic field. The radius of the helical path 8π 4π
taken by the proton beam is nearly
(a) 1 mm (b) 2 mm L2 I L2 I
(c) (d) . [MNR 2000]
4π 2π
(c) 8 mm (d) 12 mm.
239. In the previous question, the period of revolution is 247. A loop of flexible conducting wire of length l lies in
magnetic field of B normal to plane of loop. A current I
(a) 2.185 × 10 –8 s (b) 21.85 × 10 –8 s
is passed through loop. The tension developed in wire
(c) 21 s (d) 21 m s.
to open up is
240. In Q. 238, the pitch of the helix is π BIl
(a) 4 mm (b) 40 mm (a) BIl (b)
2 2
(c) 43.7 mm (d) 100 mm. BIl
(c) (d) BIl. [Pb. PMT 2000]
241. The magnetic field at two points on the axis of a circu- 2π
lar coil at distances of 0.05 m and 0.2 m from the cen- 248. A wire is bent into three
tre are in the ratio 8 : 1. The radius of the coil is successive quadrants.
(a) 0.1 m (b) 1.0 m The quadrant ab lies in
the xy plane, bc in yz
(c) 0.2 m (d) 0.15 m.
plane and ca in the zx
[Karnataka CET 2001] plane. What is the mag-
242. A milliammeter of range 10 mA has a coil of resistance netic moment of this sys-
1 Ω. To use it as a voltmeter of range 10 V, the resist- tem if a current I flows
ance that must be connected in series with it is through it ? Given : r =
(a) 9 Ω (b) 99 Ω radius of each quadrant.
(c) 999 Ω (d) 1000 Ω. Fig. 93

[Karnataka CET 2001]


MAGNETIC EFFECTS OF CURRENT AND MAGNETISM 279

(a) 0.1 T (b) 0.2 T


3 πr 2 I 2 πr 2 I
(a) (b) (c) 2 × 10 T6
(d) 600 T.
4 4
255. An infinitely long straight
πr 2 I πr 2 conductor is bent into the
(c) (d) . [MNR 1999]
4 4
shape as shown in Fig. 95.
249. An ammeter has a resistance of G Ω and a range of It carries a current I ampere
I ampere. The value of resistance to be used in parallel and the radius of the circu-
to convert it into an ammeter of range n I ampere is lar loop is r metre. Then the
(a) n G (b) (n – 1) G magnetic induction at the
G G centre of the circular loop is
(c) (d) . Fig. 95
n n−1 (a) zero
250. An ammeter is obtained by shunting a 30 Ω (b) ∞
galvanometer with a 30 Ω resistance. The additional µ 0 2I µ 0 2I
shunt required to be connected across it to double its (c) (π + 1) (d) (π – 1).
4π r 4π r
range is [MP PMT 1999]
(a) 15 Ω (b) 30 Ω
256. Which of the following particles will have minimum
(c) 45 Ω (d) 60 Ω. frequency of revolution when projected with the same
251. In an ammeter, 4% of the main current is passing velocity perpendicular to a magnetic field ?
through galvanometer. If the galvanometer is shunted (a) electron (b) proton
with a 5 Ω resistance, then the resistance of the galva- (c) He+ (d) Li+.
nometer is 257. A particle of charge q and mass m moving with a ve-
(a) 4 Ω (b) 5 Ω →
locity v along the x-axis enters the region x > 0 with
(c) 20 Ω (d) 120 Ω.
uniform magnetic field B along the k direction. The
252. Currents of 10 A and 2 A are passed through two par- particle will penetrate in this region in the x-direction
allel wires A and B respectively in opposite directions. upto a distance d equal to
If the wire A is infinitely long and the length of the wire
mv
B is 2 m, then the force on conductor B, which is situ- (a) zero (b)
qB
ated at 10 cm distance from A, will be
2mv
(a) 8 × 10 –5 N (b) 4 × 10 –5 N (c) (d) Infinity. [MP PMT 1997]
qB
(c) 8π × 10 –7
N (d) 4π × 10 –7
N.
258. In Fig. 96, two long paral-
253. Two concentric coils carry the same current in oppo-
lel wires carry equal cur- Y
site directions. The diameter of the outer coil is twice I I
rents in opposite directions.
as compared to the inner coil. If, at its centre, the
Point O is situated midway
smaller coil produces a magnetic field of 2 T, then the O
between the wires and the
magnetic field at the common centre is X
X-Y plane contains the two
(a) 1 T (b) 2 T
wires and the positive Z- Z
(c) 3 T (d) 4 T.
axis comes normally out of 2d
254. A potential difference of 600 the plane of paper. The

volt is applied across the plates +
magnetic field B at O is Fig. 96
of a parallel plate capacitor, + – non-zero along
plates being separated by 3 (a) X, Y and Z axis (b) X-axis
+ –
mm. An electron projected ver-
(c) Y-axis (d) – Z-axis.
tically parallel to the plates + –
259. An electron is accelerated by a potential difference of
with a velocity of 2 × 106 m s–1 + – 12000 V. It then enters a uniform magnetic field of
moves undeflected between the 10 –3 T applied perpendicular to the path of electron.
+
plates. The magnitude of the – – What is the radius of path ? Given mass of electron
magnetic field in the region be- = 9 × 10 –31 kg and charge on electron = 1.6 × 10 –19 C
tween the condenser plates is Fig. 94
(a) 36.7 m (b) 36.74 cm
(c) 3.67 m (d) 3.67 cm. [MP PET 1997]
280 COMPREHENSIVE OBJECTIVE PHYSICS

260. A U tube of uniform square (a) 1 A from R to S (b) 1 A from S to R


cross-sectional side a has (c) 2 A from R to S (d) 2 A from S to R
mercury in it. Current I is (e) 4 A from R to S.
passed between sealed elec- x [Cambridge Local Examinations Syndicate]
trodes x and y. A magnetic 264. A solenoid has 1000 turns wound on a length of 20 cm.
field B is applied across hori- It carries a current of 5 A. The magnetic field in the
zontal section perpendicular y middle of the solenoid is
to plane of diagram. The dif- Fig. 97 (a) 0.031 T (b) 0.31 T
ference in mercury levels is
(c) 0.062 T (d) 0.62 T.
[Given : ρ = density of mercury]
265. In Q. 264, the magnetic field at one end of the solenoid
2 BI 2BI
(a) (b) is
3 ρga ρga
(a) 0.031 T (b) 0.0155 T
BI 4BI (c) 0.00775 T (d) 0.062 T.
(c)
ρga
(d)
ρga
. [MNR 2000]
266. A simple solenoid X is made by winding a 1 m length
261. A circular segment of radius of insulated wire around a hollow card board tube of
10 cm subtends an angle of 9A diameter 1 cm and length 10 cm so as to produce uni-
60° at its centre. A current form spacing of turns in a single layer along the whole
of 9 A is flowing through it. length of the tube. A similar solenoid Y is then made by
The magnitude and direc- winding a 2 m length of wire around another tube of
60°
tion of the magnetic field at
diameter 2 cm and length 20 cm, again so as to pro-
O is
duce a uniform single layer of turns along the whole
(a) 9.43 × 10 –6 N A–1 m–1, per-
pendicular to plane and in- length. How many times must be the current in Y com-
O
wards pared with that in X to produce the same magnetic flux
Fig. 98
(b) π N A–1 m–1 perpendicular density inside each tube ?
to plane and inwards
(a) 21 (b) 1
π
(c) T in the plane (c) 2 (d) 4
2
(d) µ0π T in the plane.
(e) 8. [Northern Ireland Schools
Examination Council]
262. A long vertical wire carries a current of 10 A flowing
upwards through it at a place where the horizontal com- 267. The coil of a ballistic galvanometer is wound on a non-
ponent of the earth’s magnetic induction is 0.3 gauss. metallic frame because the moving system should have
Then the total magnetic induction at a point 5 cm from the least possible
the wire due magnetic north of the wire is (1) moment of inertia, (2) electrical resistance, (3) damp-
(a) 0.7 gauss (b) 0.5 gauss ing
(c) 0.1 gauss (d) 0.4 gauss. (a) 1, 2, 3 correct (b) 1, 2 only correct
(c) 2, 3 only correct (d) 1 only correct
[Pb. PMT 1999]
263. Fig. 99 represents a current-carrying conductor RS of (e) 3 only correct. [Oxford & Cambridge Schools
length 2 m placed at right angles to a magnetic field of Examination Board]
flux density 0.5 T. The conductor experiences a force of 268. Two long parallel wires X X Y
1 N as shown in the Fig. 131. The magnitude and di- and Y carry currents of 3 A
rection of the current is and 5 A respectively. The
force experienced per unit
S
Conductor length by X is 5 × 10 –5
N m–1 to the right as shown
in Fig. 100. The force per Force on X
Magnetic field of unit length experienced by
flux density 0.5 T wire Y is
R
(a) 2 × 10 –5 N m–1 to the left
Force of 1 N (b) 3 × 10 –5 N m–1 to the right
Fig. 99 (c) 3 × 10 –5 N m–1 to the left
Fig. 100
MAGNETIC EFFECTS OF CURRENT AND MAGNETISM 281

(d) 5 × 10 –5 N m–1 to the right 272. A beam of ions with velocity 2 × 105 m s–1 enters nor-
(e) 5 × 10 –5 N m–1 to the left. mally into a uniform magnetic field of 4 × 10–2 T. If the
[Northern Ireland Schools specific charge of the ions is 5 × 107 C kg–1, the radius
Examination Council] of the circular path described will be
269. The magnetic lg (B/µT) (a) 0.10 m (b) 0.16 m
field intensity B
(c) 0.20 m (d) 0.25 m. [JIPMER 1998]
along the axis of a 2.0
small bar magnet 1.7 273. A proton and an α-particle enter a uniform magnetic
was measured at 1.4 field perpendicularly with the same speed. If proton
various distance x 1.1 takes 25 µs to make 5 revolutions, then the periodic
from the magnet time for the α-particle would be
1.3 1.4 1.5 1.6 lg (x/m)
along the axis. (a) 50 µ s (b) 25 µ s
The results gave Fig. 101
the graph shown in Fig. 101. (c) 10 µ s (d) 5 µ s. [MP PET 1993]
It can be deduced from the graph that B is proportional 274. A certain device used the Hall effect in a semiconduc-
to tor to measure magnetic fields. A particular increase
in temperature results in an increase of 2% in the
(a) x–3 (b) x–2
number of charge carriers in the semiconductor. The
(c) e –3x
(d) e2x
sensitivity of the device
(e) x2. [AIIMS 2000] (a) increases by 4% (b) increases by 2%
270. Two solenoids P and Q rns (c) is unchanged (d) decreases by 2%
0 tu
of equal length but dif- 3 0
(e) decreases by 4%. [AIIMS 1995]
ferent numbers of Q
turns, are arranged s 275. A proton and a deuteron both having the same kinetic
turn ×
coaxially as shown in 20 0 energy, enter perpendicularly into a uniform magnetic
Fig. 102. P has 200 P field B. For motion of proton and deutron on circular
turns and Q has 300 path of radius Rp and Rd respectively, the correct state-
turns. ment is
There is a current of (a) Rd = 2 Rp (b) Rd = R p / 2
Fig. 102
1 A in Q. What must be (c) Rd = Rp (d) Rd = 2Rp.
the current in P in order that there is no resultant field [MP PET 1995]
at X midway between the coils ?
276. The diagram shows A
2 3
(a) A (b) A three long straight
3 4 wires P, Q and R nor-
4 mal to the plane of the B
(c) 1 A (d) A P Q
3 paper. Wires P and R
3 carry currents directed
(e) A. [Cambridge Local Examinations
2 into the plane of the
Syndicate] C
paper, and wire Q car- E
271. Two moving-coil galvanometers X and Y are connected ries a current directed D
in series and a current passes through. Their readings out of the paper. All R
are found to be full scale for X and half scale for Y. three currents have the Fig. 103
Given that their scales are identical and their restor- same magnitude.
ing springs of the same strength, but that X’s coil is Which arrow best shows the direction of the resultant
twice as large in area and has twice the number of turns force on wire P ?
compared with Y’s coil, the magnetic field strength of (a) A (b) B
X’s magnet compared with Y’s magnet is (c) C (d) D
1 1
(a) 4 (b) 2
(e) E. [All India PM/PD 1999]
(c) 1 (d) 2 277. The radius of the earth is 6.4 × 106 m and its magnetic
(e) 4.
moment is 6.4 × 10 21 A m2. If it is assumed that this
moment is due to a current carrying loop suspended
[London Schools Examination Department] along the magnetic equatorial line, then the value of
the current will be nearly
282 COMPREHENSIVE OBJECTIVE PHYSICS

(a) 5 × 106 A (b) 5 × 107 A (c) is towards east at both A and B


(c) 5 × 10 A 4
(d) 5 × 10 A. 9
(d) is towards west at both A and B.
278. A long, air-cored solenoid has N turns, is of length l, [CMC Vellore 1997]
has cross-sectional area A and carries current I. The 284. A rectangular cur- 0.4 m
uniform magnetic flux density near the middle of the rent-carrying coil P Q
solenoid is parallel to the axis and has the value is placed in a uni-
µ0NI/l. What is the magnetic flux through the cross-
form magnetic
section in this region ?
field B such that 0.25 m B
µ 0 NIA the sides PQ and
(a) µ0NI (b)
l RS are parallel to
µ 0 NI µ 0 NI B [Fig. 104]. If the S R
(c) (d)
A lA current in the coil Fig. 104
(e) µ0NIA. [All India PM/PD 1998] is 2 ampere and B
279. A proton enters a magnetic field of flux density 1.5 Wb = 2 × 10 –2 T, then torque on the coil is
m–2 with a velocity of 2 × 107 m s–1 at an angle of 30° (a) 2 × 10 –2 N m (b) 4 × 10 –2 N m
with the field. The force on the proton will be (c) 4 × 10 –3 N m (d) 5 × 10 –3 N m.
(a) 2.4 × 10 –12 N (b) 0.24 × 10 –12 N 285. A proton (mass = 1.67 × 10 –27
kg and charge = 1.6 × 10 –19
(c) 24 × 10 –12 N (d) 0.024 × 10 –12 N. C) enters perpendicular to a magnetic field of intensity
[MP PET 1994] 2 weber/m2 with a velocity 3.4 × 107 m s–1. What will
280. A uniform magnetic field acts at right angles to the be its acceleration ?
direction of motion of electrons. As a result, the elec- (a) 6.5 × 1015 m s–2 (b) 6.5 × 1013 m s–2
tron moves in a circular path of radius 2 cm. If the
(c) 6.5 × 1011 m s –2 (d) 6.5 × 109 m s–2.
speed of the electrons is doubled, then the radius of the
circular path will be 286. With a resistance R connected in series with a galva-
(a) 2.0 cm (b) 0.5 cm nometer of resistance 100 Ω, it acts as a voltmeter of
(c) 4.0 cm (d) 1.0 cm. [Manipal 1999] range 0 – V. To double the range, a resistance of 1000 Ω
is to be connected in series with R. Then the value of R
281. A charged particle is whirled in a horizontal circle on
is
a frictionless table by attaching it to a string fixed at
one point. If a magnetic field is switched on in the ver- (a) 1100 Ω (b) 800 Ω
tical direction, the tension in the string (c) 900 Ω (d) 1000 Ω.
(a) will increase (b) will decrease [Karnataka CET 1999]
(c) will remains the same (d) may increase or decrease. 287. A current of 0.1 A circulates around a coil of 100 turns
[Pb. CET 1996] and having a radius equal to 5 cm. The magnetic field
282. A charged particle moves in a uniform magnetic field. set up at the centre of the coil is
The velocity of the particle at some instant makes an (a) 4π × 10 –5 T (b) 8π × 10 –5 T
acute angle with the magnetic field. The path of the
(c) 4 × 10 –5
T (d) 2 × 10 –5 T.
particle will be
(a) a straight line 288. A galvanometer has resistance of 7 Ω and gives a full
(b) a circle
scale deflection for a current of 1.0 A. How will you
convert it into a voltmeter of range 10 V ?
(c) a helix with uniform pitch
(a) 3 Ω in series (b) 3 Ω in parallel
(d) a helix with non-uniform pitch. [MP PMT 1999]
(c) 17 Ω in series (d) 30 Ω in series.
283. A circular loop is kept in that vertical plane which con-
tains the north-south direction. It carries a current that [MP PMT 1999]
is towards north at the topmost point. Let A be a point 289. The earth’s magnetic induction at a certain point is
on the axis of the circle to the east of it and B a point on 7 × 10 –5 Wb m–2. This is to be annulled by the magnetic
this axis to the west of it. The magnetic field due to the induction at the centre of a circular conducting loop of
loop radius 5 cm. The required current in the loop is
(a) is towards east at A and towards west at B
(a) 0.56 A (b) 5.6 A
(b) is towards west at A and towards east at B
(c) 0.28 A (d) 2.8 A. [MP PET 1999]
MAGNETIC EFFECTS OF CURRENT AND MAGNETISM 283

290. An electron (mass = 9 × 10 –31 kg, charge = 1.6 × 10 –19 297. A beam consisting of protons and electrons moving at
C) moving with a velocity of 106 m s–1 enters a region the same speed goes through a thin region in which
where magnetic field exists. If it describes a circle of there is a magnetic field perpendicular to the beam.
radius 0.10 m, the magnetic field must be The protons and the electrons
(a) 1.8 × 10 –4 T (b) 5.6 × 10 –5 T (a) will go undeviated.
(c) 14.4 × 10 –5 T (d) 1.3 × 10 –6 T. (b) will be deviated by the same angle and will not separate.
[BPMT 1992] (c) will be deviated by different angles and hence separate.
291. The coil of a galvanometer consists of 100 turns and (d) will be deviated by the same angle but will separate.
effective area of 1 cm2. The restoring couple is 10 –8 N [Himachal PMT 1999]
m rad –1. The magnetic field between the pole pieces is 298. Fig. 105 shows two
5 T. The current sensitivity of this galvanometer will long conducting wires
be placed at right angles.
(a) 5 × 104 rad/µ A (b) 5 × 10–6 per A They carry currents as
(c) 2 × 10–7 per A (d) 5 rad/µ A. indicated. Assume IA
[MP PMT 1997] that 1 ampere current
292. A charge q coulomb moves in a circle at n revolutions in a long wire pro- d
per second and the radius of the circle is r metre. Then duces magnetic field
magnetic field at the centre of the circle is B′ at a distance d from X d
2πq 2πq the wire. The strength 2A
(a) × 10 –7 N A–1 m–1 (b) × 10 –7 N A–1 m–1 and direction of field
nr r
2πnq 2πq at a point X marked in
(c) × 10 –7 N A–1 m–1 (d) N A–1 m–1. Fig. 105 will be Fig. 105
r r
293. A charged particle of mass m and charge q describes (a) 3B′ into the plane of paper
circular orbit of radius r in a uniform magnetic field of (b) B′ into the plane of paper
strength B. The frequency of revolution is (c) B′ out of the plane of paper
Bq Bq (d) 3B′ out of the plane of paper.
(a) (b)
2πm 2πrm [National Standard Exam. in Physics 1996]
2πm Bm 299. If 10% of the current passes through a moving coil gal-
(c) (d)
2πq
. [MP PET 1997]
Bq vanometer of resistance 99 Ω, then the shunt resist-
294. The resistance of 1 A ammeter is 0.018 Ω. To convert it ance will be
into 10 A ammeter, the shunt resistance required will (a) 9.9 Ω (b) 11 Ω
be (c) 10 Ω (d) 9 Ω.
(a) 0.18 Ω (b) 0.0018 Ω [MP PET 1999 ; Karnataka CET 2003]
(c) 0.002 Ω (d) 0.12 Ω. [MP PET 1992] 300. A long solenoid has 800 turns per metre length of sole-
295. A galvanometer of resistance 25 Ω gives full scale de- noid. A current of 1.6 A flows through it. The magnetic
flection for a current of 10 mA. It is to be changed into induction at the end of the solenoid on its axis is
a voltmeter of range 100 V by connecting a resistance (a) 16 × 10 –4 T (b) 8 × 10 –4 T
of ‘R’ in series with galvanometer. The value of resist- (c) 32 × 10 T
–4
(d) 4 × 10 –4 T.
ance R is
[MP PET 1999]
(a) 10000 Ω (b) 10025 Ω
301. In hydrogen atom, the electron is making 6.6 × 1015
(c) 975 Ω (d) 9975 Ω. [Manipal 2002] rev s–1 around the nucleus in an orbit of radius 0.528
296. Two straight parallel wires, both carrying 10 ampere Å. The magnetic moment (in A m2) will be
in the same direction, attract each other with a force of (a) 1 × 10 –15 (b) 1 × 10 –10
1 × 10 –3 N. If both currents are doubled, the force of
(c) 1 × 10 –23 (d) 1 × 10 –27. [MP PET 1999]
attraction will be
302. Two thin parallel wires separated by a distance b are
(a) 1 × 10 –3 N (b) 2 × 10 –3 N
carrying current I ampere each. The magnitude of the
(c) 4 × 10 –3 N (d) 0.25 × 10 –3 N.
force per unit length exerted by one wire on the other is
[MP PET 1994]
284 COMPREHENSIVE OBJECTIVE PHYSICS

I
µ 0I2 µ 0 I2
(a) (b)
b2 2πb I I
µ 0I µ 0I
(c) (d) . [IIT 1986]
2 πb 2 πb2
303. A length of wire carries a steady current. It is bent first 90°

to form a circular plane coil of one turn. The same


length is now bent more sharply to give a double loop O
of smaller radius. The magnetic field at the centre Fig. 107
caused by the same current is
µ 0 2I µ 0 2I
×
LM 2 + π OP
(a) a quarter of its first value (a)

×
r
2+π (b)
4π r N 4Q
(b) unaltered µ0 I µ0 I
×
LM 2 + π OP .
(c) four times of its first value
(c) ×
4π r
2+π (d)
4π r N 4Q
(d) half of its first value. 309. In Fig. 108, what is the magnetic
field induction at point O ? I
[MP PMT 1999 ; National Standard Exam. in
µ 0I
Physics 1999] (a) I
4 πr
304. A charged particle is projected in a plane perpendicu- µ 0I µ 0I
lar to a uniform magnetic field. The area bounded by (b) + r
O
4r 2π r
the path described by the particle is proportional to µ 0I µ 0I I
(c) +
(a) the velocity (b) the momentum 4r 4 πr
Fig. 108
(c) the kinetic energy (d) none of these. µ 0I µ 0I
(d) − .
4r 4 πr
[MP PMT 1999]
310. A galvanometer of 100 Ω resistance gives full scale de-
305. A galvanometer has a resistance of G ohm and range flection when 10 mA of current is passed. To convert it
V volt. The value of resistance used in series to convert into 10 A range ammeter, the resistance of the shunt
it into voltmeter of range nV volt is required will be
(a) nG (b) (n – 1)G (a) – 10 Ω (b) 1 Ω
G G (c) 0.1 Ω (d) 0.01 Ω. [MP PMT 1985]
(c) (d) . [MP PMT 1999]
n n−1 311. A circular coil of radius 4 cm has 50 turns. In this coil,
306. An electron of mass me under the action of magnetic a current of 2 A is flowing. It is placed in a magnetic
field moves in a circle of radius re at a speed of ve. If a field of 0.1 weber/m2. The amount of work done in ro-
tating it through 180° from its equilibrium position will
proton of mass mp were to move in a circle of same
be
radius in the same magnetic field, then its speed will
(a) 0.1 J (b) 0.2 J
be
(c) 0.4 J (d) 0.8 J. [DPMT 1996]
(a) vp = ve (mp/me) (b) vp = ve (me/mp)
(c) vp = {mp/(me ve)} (d) vp = {me/(mp ve)}. 312. The deflection of a galvanometer falls to 15 th when a
307. The magnetic induction at resistance of 4 ohm is connected in parallel with it. What
the centre O (figure 106) is I
will be the deflection if an additional resistance of 2 Ω
µ 0I µ 0I is connected in parallel with the above shunted galva-
(a) + ⊗ nometer ?
2a 2b a

3µ 0I µ 0I b (a) 1/9th (b) 1/20th


(b) + ⊗ O (c) 1/13th
8a 8b
3µ 0I µ 0I (d) 1/7th of the original deflection. [AFMC 1999]
(c) − ⊗
8a 8b 313. A galvanometer of resistance 10 Ω gives a full scale
Fig. 106
3µ 0I µ 0I deflection for a current of 1.0 mA. What resistance must
(d) + ⊗.
8a 8b be connected in series with it so as to convert it into a
308. What is the magnetic field at the centre O of the arc in voltmeter reading upto 2.5 V ?
the Fig. 107 ? (a) 24.9 V (b) 249 Ω
(c) 24900 Ω (d) 2490 Ω. [MAHE 1999]
MAGNETIC EFFECTS OF CURRENT AND MAGNETISM 285

314. In the above question, with what resistance the galva- 321. An ammeter of 5 ohm resistance can read 5 mA. If it is
nometer should be shunted so as to convert it into an to be used to read 100 volt, how much resistance is to
ammeter that can measure currents up to 2.5 A ? be connected in series ?
(a) 4 × 10 –1 Ω (b) 4 × 10 –2 Ω (a) 19.9995 Ω (b) 199.995 Ω
(c) 4 × 10 Ω–3
(d) 4 × 10 Ω.
–4
(c) 1999.95 Ω (d) 19995 Ω.
315. A galvanometer gives a full-scale deflection when a cur- [MP PMT 2000, 1996 ; MP PET 1991]
rent of 2 mA flows through it and the potential differ- 322. The radius of curvature of the path of the charged par-
ence across its terminals is 4 mV. Which of the follow- ticle in a uniform magnetic field is directly proportional
ing resistors would be most suitable to convert it to give to
a full-scale deflection for a current of 1 A ?
(a) the charge on the particle
(a) 0.004 Ω on series (b) 0.004 Ω in parallel
(b) the momentum of the particle
(c) 0.50 Ω in series (d) 500 Ω in series
(e) 500 Ω in parallel. (c) the energy of the particle

[London Schools Examination Department] (d) the intensity of the field. [MNR 1995]
316. In Fig. 109, there are two semicircles of radii r1 and r2 323. A proton (mass = m and charge = e) of energy 2 MeV is
in which a current I is flowing. The magnetic induc- moving in a circular path in a magnetic field. What
tion at the centre O will be should be the energy of deuteron (mass = 2m and charge
µ 0I = +e), so that it describes the circular path of same ra-
(a) (r1 + r2 )
r r1 dius in the same magnetic field ?
µ 0I (a) 1 MeV (b) 2 MeV
(b) (r1 − r2 )
4 (c) 4.0 MeV (d) 0.5 MeV.
F I
µ 0I r1 + r2 O
(c) GH
4 JK
r1r2
I r2 324. A current I is flowing a loop of
radius r as shown in Fig. 110.
µ I Fr −r I
q
The magnetic induction inten-
(d) 0 G 2 1 J . Fig. 109
4 H r1r2 K sity at the centre O will be
317. An electron moves in a circular arc of radius 10 m at a µ oIθ 2µ oI sin θ
(a) (b) O
constant speed of 2 × 107 m s–1 with its plane of motion 4 πr 4πr 2 r
normal to a magnetic flux density of 10 –5 T. The specific (c)
2µ oI sin θ
(d)
µ oI sin θ
.
charge of the electron is 2r 4r Fig. 110

(a) 2 × 10 C kg
2 –1
(b) 2 × 10 C kg
5 –1 325. Two concentric circular coils of ten turns each are situ-
(c) 5 × 106 C kg –1 (d) 2 × 1011 C kg –1. ated in the same plane. Their radii are 20 cm and 40
cm and they carry respectively 0.2 A and 0.3 A current
[Haryana PMT 1997]
in opposite direction. The magnetic field in Wb m–2 at
318. A proton, a deuteron and an α-particle accelerated the centre is
through the same potential difference enter a region of
µ0 5
uniform magnetic field, moving at right angles to B. (a) (b) µ0
80 4
What is the ratio of their K.E. ?
(a) 2 : 1 : 1 (b) 2 : 2 : 1 7 35
(c) µ0 (d) µ 0 . [MP PMT 1994]
4 4
(c) 1 : 2 : 1 (d) 1 : 1 : 2.
319. In the above problem, if the radius of the proton’s cir- 326. An electron and a proton enter region of uniform mag-
cular path is 10 cm, what is the radius of deuteron’s netic field in a direction at right angles to the field with
path ? the same kinetic energy. They describe circular paths
of radii re and rp respectively. Then
(a) 10 cm (b) 10 2 cm
(a) re = rp (b) re < rp
(c) 10 / 2 cm (d) 20 cm.
(c) re > rp
320. In Q. 318, what is the radius of the path of the α-parti-
(d) r e may be less than or greater than rp depending on the
cle ? direction of the magnetic field.
(a) 10 cm (b) 10 2 cm [Manipal MEE 1995]
(c) 10 / 2 cm (d) 20 cm.
286 COMPREHENSIVE OBJECTIVE PHYSICS

327. If the strength of the magnetic field produced 10 cm (a) AB Y


C B
away from an infinitely long straight conductor is (b) CD I
10 –5 T, the value of the current flowing in the conductor (c) Segment OB only of line AB O I
will be (d) Segment OC only of line CD. X
(a) 5 A (b) 10 A
[MP PMT 1995] A D
(c) 500 A (d) 1000 A. [MP PET 1996]
Fig. 112
328. An electron having charge 1.6 × 10 –19 C and mass
9 × 10–31 kg is moving with speed 4 × 106 m s –1 in a 333. There are 50 turns of a wire in every cm length of a
magnetic field of 2 × 10 –1 T in a circular orbit. The long solenoid. If 4 ampere current is flowing in the so-
force acting on electron and the radius of the circular lenoid, the approximate value of magnetic field along
orbit will be its axis at an internal point and at one end will be re-
(a) 12.8 × 10 –13 N, 1.1 × 10 –4 m spectively
(b) 1.28 × 10 –14 N, 1.1 × 10 –3 m (a) 12.6 × 10 –3 weber/m2, 6.3 × 10–3 weber/m2
(c) 1.28 × 10 –13 N, 1.1 × 10 –3 m (b) 12.6 × 10 –3 weber/m2, 25.1 × 10–3 weber/m2
(d) 1.28 × 10 –13 N, 1.1 × 10 –4 m. [MP PET 1996]
(c) 25.1 × 10 –3 weber/m2, 12.6 × 10–3 weber/m2
329. A rectangular coil 20 cm × 20 cm has 100 turns and
(d) 25.1 × 10 –5 weber/m2, 12.6 × 10–5 weber/m2.
carries a current of 1 A. It is placed in a uniform mag-
netic field B = 0.5 T with the direction of magnetic field [MP PET 1996]
parallel to the plane of the coil. The magnitude of the 334. When charged particle moving with a velocity v enters
torque required to hold this coil in this position is into a region of uniform magnetic field B so that v is
(a) Zero (b) 200 N m perpendicular to B, the particle moves in a circular
path. The angular frequency ω of the particle
(c) 2 N m (d) 10 N m. [MP PMT 1997]
(a) is proportional to the mass of the particle
330. A uniform wire of mass 10 –3
kg cm–1 is held at rest in a
(b) is independent of the velocity of the particle
horizontal position without any support by placing it
(c) is inversely proportional to magnetic field B
in a uniform, perpendicular magnetic field. The wire
(d) is inversely proportional to the charge on the particle.
is carrying a current of 10 A. The magnetic field is [Take
g = 10 m s–2] 335. Two long thin wires ABC and DEF are arranged as
(a) 10 –1
T (b) 1 T shown in Fig. 113. They carry equal currents I as
shown. The magnitude of the magnetic field at O is
(c) 10 –2 T (d) 10 –3 T.
331. Three long, R Q P
straight and par-
allel wires carry-
ing currents are
2 cm 10 cm
arranged as
shown in Fig. 111.
The force experi-
20 A 10 A 30 A
enced by 10 cm
length of wire Q is Fig. 111

(a) 1.4 × 10–4 N towards the right


(b) 1.4 × 10–4 N towards the left
(c) 2.6 × 10–4 N to the right Fig. 113

(d) 2.6 × 10–4 N to the left. [MP PET 1997] (a) zero (b) µ0I/4πa

332. Two very thin metallic wires placed along X and Y- (c) µ0I/2πa (d) µ0I/ 2 2 πa.
axis carry equal currents as shown in Fig. 112. AB and [JIPMER 1999]
CD are lines at 45° with the axes with origin of axes at 336. The dimension of the magnetic field intensity B is
O. The magnetic field will be zero on the line (a) MLT –2 A–1 (b) MT –2 A–1
(c) ML2TA–2 (d) M2LT –2A–1.
[MP PET 2001]
MAGNETIC EFFECTS OF CURRENT AND MAGNETISM 287

337. A wire is bent in the form of a I 342. A long wire A carries a current of 10 A. Another long
circular arc with a straight wire B, which is parallel to A and separated by 0.1 m
portion AB. Magnetic induc- from A, carries a current of 5 A in the opposite direc-
tion at O when current I is O tion to that in A. What is the magnitude and nature of
flowing in the wire, is the force experienced per unit length of B ?
r q q r
µ 0I (a) Repulsive force of 10–4 N m–1
(a) (π – θ + tan θ)
2r (b) Attractive force of 10–4 N m–1
A C B
µ I (c) Repulsive force of 2π × 10–5 N m–1
(b) 0 (π + θ – tan θ) Fig. 114
2 πr (d) Attractive force of 2π × 10–5 N m–1. [MP PET 2000]
µ I µ I
(c) 0 (π – θ + tan θ) (d) 0 (– tan θ + π – θ). 343. The magnetic field at the centre of a circular coil of
2 πr 2r
radius r carrying current I is B1. The field at the centre
338. Due to 10 ampere of current flowing in a circular coil
of another coil of radius 2r carrying same current I is
of 10 cm radius, the magnetic field produced at its cen-
tre is 3.14 × 10 –3 T. The number of turns in the coil will B1
B2. The ratio is
be B2
(a) 5000 (b) 100 1
(a) (b) 1
(c) 50 (d) 25. [MP PET 1996] 2
339. The magnetic field at O due to current in the infinite (c) 2 (d) 4. [MP PET 2001]
wire forming a loop as shown in Fig. 115 is 344. An arbitary shaped closed coil is made of a wire of
length L and a current I ampere is flowing in it. If the
I →
φ1 φ2 plane of the coil is perpendicular to magnetic field B ,
the force on the coil is
θ1 θ2 d
(a) Zero (b) IBL
I I
1
O (c) 2 IBL (d) IBL. [MP PMT 2001]
2
Fig. 115 345. A current I flows in a circular coil of radius r. If the
µ 0I µ 0 2I coil is placed in a uniform magnetic field B with its
(a) (cos φ1 + cos φ2) (b) (tan θ1 + tan θ2) plane parallel to the field, magnitude of the torque that
4 πd 4π d
acts on the coil is
µ0 I µ0 I
(c) (sin φ1 + sin φ2) (d) (cos θ1 + sin θ2). (a) Zero (b) 2πrIB
4π d 4π d
(c) πr2IB (d) 2πr2IB. [MP PET 2001]
340. Two straight long conductors AOB and COD are per-
pendicular to each other and carry currents i1 and i2. 346. A current of 5 A is flowing in a wire of length 1.5 m. A
The magnitude of the magnetic induction at a point P force of 7.5 N acts on it when it is placed in a uniform
magnetic field of 2 T. The angle between the magnetic
at a distance d from the point O in a direction perpen-
field and the direction of the current is
dicular to the plane ABCD is
(a) 30° (b) 45°
µ0 µ0
(a) (i1 + i2 ) (b) (i1 − i2 ) (c) 60° (d) 90°. [MP PET 1997]
2 πd 4 πd
µ0 µ0 Fi1i2 I 347. A proton moves from left to right with a velocity v in
(c)
2 πd
(i12 + i22 )1/2 (d) GH
2πd i1 + i2
. JK horizontal direction. The proton enters a region of space
where electric field E is vertically downwards. What
[AIIMS 1996] should be the direction and magnitude of magnetic field
341. Two infinite, thin, straight wires 1 2 B so that the proton moves undeflected ?
1 m apart carry currents as (a) Horizontal B ⊥ to v and v = E/B
shown in Fig. 116. The distance (b) Horizontal B ⊥ to v and any magnitude
of a point from wire 1, in between (c) E, v and B should be mutually perpendicular
the two wires ; where the net mag- I 4I
(d) B of magnitude E/v and parallel to v. [AFMC 1997]
netic field is zero, is 1m
(a) 0.25 m 348. Two wire loops PQRSP formed by joining two semicir-
(b) 0.2 m
cular wires of radii R1 and R2 carries a current I as
shown in Fig. 117. The magnitude of magnetic induc-
(c) 0.33 m
Fig. 116 tion at centre C is
(d) 0.5 m.
288 COMPREHENSIVE OBJECTIVE PHYSICS

I 353. A solenoid is 1.0 m long and it has 4250 turns. If a


current of 5.0 A is flowing through it, what is the mag-
I netic field at its centre ?
(a) 5.4 × 10 –2 T (b) 2.7 × 10 –2 T
(c) 1.35 × 10 T –2
(d) 0.675 × 10 –2 T.
R2 R1
[MP PMT 1996]
354. The magnetic field at a distance x from a long wire
S R Q P
C carrying current I is 0.4 T. The magnetic field at a dis-
Fig. 117 tance ‘2x’ is
(a) 0.8 T (b) 0.2 T

(a) (µ /4) I M
L R 1 − 1 OP µ0FG IJ LM
1 1 OP (c) 0.1 T (d) 1.6 T.
0
N R 2 R1 Q (b)
4 H K N
I −
R1 R2 Q 355. A current of 1 A is passed through a straight wire of
L 1 − 1 OP
(c) µ 0 I M (d) µ0I (1/R1).
length 2.0 m. The magnetic field at a point in air at a
N R 2 R1 Q distance of 3 m from either end of wire and lying on the
axis of wire will be
[IIT 1983 ; Karnataka CET 2002]
(a) zero (b) µ0/8π
349. A straight wire of length 0.5 m and carrying a current
(c) µ0/4π (d) µ0/2π.
of 1.2 A placed in a uniform magnetic field of induc-
tion 2 T. The magnetic field is perpendicular to the 356. A proton and an alpha particle enter a uniform mag-
length of the wire. The force on the wire is netic field normally with the same velocity. The period
(a) 2.4 N (b) 1.2 N of revolution of the α-particle will be
(a) two times that of proton
(c) 3.0 N (d) 2.0 N. [BHU 1998]
(b) three times that of proton
350. A current of 2 A flows in a long, straight wire of radius
(c) four times that of proton
2 mm. The intensity of magnetic field at the axis of the
wire is (d) same as that of proton.
FG µ0 IJ × 10 tesla FG µ0 IJ × 10 357. Magnetic field induction at the centre of coil of 50 turns,
(a) H πK 3
(b) H 2π K 3
tesla radius 0.5 m and carrying a current of 2 A is
F 2µ0 IJ × 10 tesla
(c) GH
(a) 0.5 × 10 –5 T (b) 1.25 × 10 –4 T
π K
3
(d) Zero. [MP PET 2001] (c) 3 × 10 T–5
(d) 4 × 10 –5 T.
351. A charged particle of mass m [All India PM/PD 1999]
P Q
having kinetic energy K enters 358. A power line lies along the east-west direction and car-
into the region of a uniform ries a current of 10 A. The force per metre due to the
magnetic field between two earth’s magnetic field of 10 –4 T is
plates P and Q as shown in (a) 10 –5 N (b) 10 –4 N
Fig. 118. The charged particle q
(c) 10 –3 N (d) 10 –2 N. [Roorkee 1992]
just misses hitting the plate Q.
359. A voltmeter having resistance of 50 × 103 Ω is used to
The magnetic field in the region
measure the voltage in a circuit. To increase the range
between the two plates is
d of measurement 3 times, the additional series resistance
(a) mK/qd
Fig. 118 required is
(b) 2mK/qd (a) 105 Ω (b) 150 K Ω
(c) mK / qd (d) 2m K /qd . (c) 900 K Ω (d) 9 × 106 Ω.

352. A circular coil ‘A’ has a radius R and the current flow- [MP PMT 2001]
ing through it is I. Another circular coil ‘B’ has a ra- 360. In a circuit 5 percent of total current passes through a
dius 2R and if 2I is the current flowing through it, then galvanometer. If resistance of the galvanometer is G,
the magnetic fields at the centre of the circular coils then value of the shunt is
are in the ratio of (i.e. BA to BB) (a) 19 G (b) 20 G
(a) 4 : 1 (b) 2 : 1 G G
(c) (d) . [MP PMT 2001]
(c) 3 : 1 (d) 1 : 1. 20 19
361. A proton moving with a speed of 107 m s–1 enters the
[All India PM/PD 1993]
region of a uniform, perpendicular magnetic field of
MAGNETIC EFFECTS OF CURRENT AND MAGNETISM 289

0.4 T. The radius of the circular path described by pro- 366. A wire carrying Z
ton is (mass of proton = 1.6 × 10 –27 kg) a current of I A
(a) 0.25 m (b) 0.25 mm has the shape
(c) 4 mm (d) 4.25 mm. shown in Fig. C
120. The radius
[CMC Vellore 1998] R
R of the curved
362. A galvanometer has a resistance of 25 ohm and a maxi- part of the wire
D
B O
mum of 0.01 A current can be passed through it. In is 100 mm. The
Y

order to change it into an ammeter of range 10 A, the linear parts of


shunt resistance required is IA
the wire are very
(a) 5/999 ohm (b) 10/999 ohm long. The mag-
(c) 20/999 ohm (d) 25/999 ohm. netic field induc-
[MP PET 2000] tion at O due to A E

363. A proton of energy 200 MeV enters the magnetic part AB of the X
Fig. 120
field of 5 T. If direction of field is from south to north wire is
and motion is upward, the force acting on it will be µ 0I µ 0I
(a) (b)
(a) zero (b) 1.6 × 10–10 N 2 πR πR
(c) 3.2 × 10–8 N (d) 1.6 × 10–6 N. µ 0I µ 0I
(c) (d) .
[Rajasthan PET 1997] 4 πR 4R
364. Magnetic field due to 0.1 A current flowing through a 367. The magnetic field induction at O due to wire DE in
circular coil of radius 0.1 m and 1000 turns at the cen- Q. 366 is
tre of the coil is
µ 0I µ 0I
(a) 2 × 10–1 T (b) 4.31 × 10–2 T (a) (b)
2 πR 4 πR
(c) 6.28 × 10–4 T (d) 9.81 × 10–4 T.
µ 0I µ 0I
365. A long hollow metallic cylinder of radius R has a cur- (c) (d) .
πR 4R
rent I ampere flowing throught it. The magnetic field
induction at a distance r from the axis varies as shown 368. Magnetic field induction at O due to semi-circular por-
in tion of wire in Q. 366 is
(a) i (b) ii µ 0I µ 0I
(a) (b)
(c) iii (d) iv. 2 πR 4 πR
µ 0I µ 0I
(c) (d) .
πR 4R
369. In Q. 366, the total magnetic induction at O is
µ 0I
(a) µ0I (b)
B B 4 πR
µ 0I µ 0I
(c) ×4 (d) 4 + π2 .
4 πR 4 πR
r r
R R 370. A particle with 10–11 C of charge and 10 –7 kg mass is
(i) (ii) moving with a velocity of 108 m s–1 along the y-axis. A
uniform static magnetic field B = 0.5 T is acting along
the x-direction. The force on the particle is
(a) 5 × 10 –11 N along i (b) 5 × 103 N along k

B B
(c) 5 × 10 –11 N along – j (d) 5 × 10 –4 N along – k .
[MP PMT 1997]
371. Two long parallel wires are at a distance 2d apart. They
R r R r
carry steady equal currents flowing out of the plane of
(iii) (iv) the paper as shown in Fig. 121. The variation of mag-
netic field B along the line XX′ is given by
Fig. 119
290 COMPREHENSIVE OBJECTIVE PHYSICS

376. An electron moving in a circular orbit of radius r makes


n rotations per second. The magnetic field produced at
the centre has magnitude
µ 0ne
(a) Zero (b)
2πr
µ 0ne µ 0 n2 e
(c) (d) .
2r 2r
[Karnataka CET 2001]
377. A and B are two concentric cir- B
cular conductors of centre O
and carrying currents I1 and I2 A
as shown in Fig. 122. The ratio
of their radii is 1 : 2 and ratio O I1
I2
of their flux densities at O due
to A and B is 1 : 3. The value of
I1 Fig. 122
is
I2
Fig. 121
1 1
[IIT Screening 2000] (a) (b)
6 4
372. An ionised gas contains both positive and negative ions. 1 1
If it is subjected simultaneously to an electric field along (c) (d) .
2 3
the + X direction and a magnetic field along the + Z [Karnataka CET 2000]
direction, then
378. A helium nucleus completes one rotation of radius 0.8 m
(a) positive ions deflect towards + Y direction and negative in two second. The magnetic field induction B at the
ions towards – Y direction
centre is
(b) all ions deflect towards + Y direction
(c) all ions deflect towards – Y direction 10 −19
(a) 10 –19 µ0 T (b) T
(d) positive ions deflect towards – Y direction and negative µ0
ions towards + Y direction. [IIT Screening 2000]
2 × 1019
373. In order to convert a milliammeter of range 1.0 mA (c) 2 × 10 –19 µ0 T (d) T.
and resistance 1.0 ohm into a voltmeter of range 10 V, µ0
a resistance of how many ohm should be connected with [WB JEE 2002]
it and in what manner ? MAGNETISM
(a) 999 Ω in series (b) 999 Ω in parallel
(c) 9,999 Ω in series (d) 9,999 Ω in parallel. 379. Two magnets of magnets moments pm and 3 pm are
joined to form a cross (+). When the combination is sus-
[MP PET 2000]
pended freely in a uniform magnetic field, then in the
374. A proton, a deuteron and an α-particle having the same
equilibrium position, the magnet of magnetic moment
kinetic energy are moving in circular trajectories in a
pm makes an angle θ with the magnetic field. The value
constant magnetic field. If rp , rd and rα denote respec-
of θ is
tively the radii of the trajectories of these particles, then
(a) 0° (b) 30°
(a) rα = rp < rd (b) rα > rd > rp
(c) 60° (d) 90°.
(c) rα = rd > rp (d) rp = rd = rα.
[IIT 1997 Re-Examination] 380. A magnetic dipole is under the influence of two mag-
netic fields B1 and B2. The angle between B1 and B2 is
375. In a hydrogen atom, an electron moves in a circular
60°. If the dipole comes to stable equilibrium at an an-
orbit of radius 5.2 × 1011 m and produces a magnetic
gle 15° with B1 , then
induction of 12.56 T at its nucleus. The current pro-
duced by the motion of the electron will be (a) B2 = B1 (b) B2 = B1 sin 105°
(a) 6.53 × 10 A
–3
(b) 13.25 × 10 –10
A B1 sin 15°
(c) B2 = B1 sin 60° (d) B2 = .
(c) 9.6 × 106 A (d) 1.04 × 10 –3 A. sin 45°
[MP PET 1997] [All India PM/PD 1997]
MAGNETIC EFFECTS OF CURRENT AND MAGNETISM 291

381. Fig. 123 shows different combinations of two magnets, 


386. The torque τ acting on a magnet due to a magnetic
each of magnetic moment M. Which combination has →
field B is
largest magnetic moment ?
 → →  → →
N
(a) τ = B . M (b) τ = M × B
N S → →
 B  M
(c) τ = (d) τ = .
→ →
M B
387. The dipole moment of a short bar magnet is 1.25 A m2.
The magnetic field on its axis at a distance of 0.5 m
S S N from the centre of the magnet is
S N N S
I II (a) 1.0 × 10 –4 SI units (b) 4 × 10 –2 SI units
(c) 2 × 10 –6
SI units (d) 6.64 × 10 –8 SI units.
N
[MP PET 1996]
388. The true value of dip at a place is 45°. If the plane of
the dip circle is turned through 60° from the meridian,
then the value of apparent dip is
N S
S
60° (a) 45° (b) 60°
S N S N (c) tan 1 –1
(d) tan–1 2.
III IV 389. The values of the apparent angles of dip in two planes
Fig. 123 at right angles to each other are 30° and 45°. Then the
true value of the angle of dip at the place is
(a) I (b) II
(a) tan–1 1 (b) tan–1 2
(c) III (d) IV.
(c) cot–1 2 (d) cot–1 1.
382. Assume that the terrestrial magnetism is due to a short
390. A short magnet placed perpendicular to the magnetic
magnet at the centre of the earth. Refer to a point on
meridian deflects a compass needle lying at a distance
the surface of the earth. Given : magnetic latitude of
of 10 cm on its axial line through an angle of 45° from
the reference point = λ, δ = angle of dip and R = radius
the meridian. If BH = 3 × 10 –5 Wb m–2, then the mag-
of earth. Then, the value of vertical component V of the
netic moment of the magnet is
earth’s field is
(a) 1.5 × 10 –3 A m2 (b) 15 × 10 –3 A m2
2M sin λ
(a) (b) tan δ (c) 150 × 10 –3 A m2 (d) 1500 × 10 –3 A m2.
R3
(c) tan λ (d) 2 tan λ.
391. A magnet suspended at 30° with the magnetic merid-
ian makes an angle of 45° with the horizontal. The ac-
383. In Q. 382, the value of horizontal component H is
tual value of angle of dip is
2M sin λ M cos λ
(a) 3
(b) (a) tan–1 1 (b) tan–1 3
R R3
M tan λ 3
(c) (d) M tan λ. (c) tan–1 (d) tan–1 2.
R3 2
392. Two magnets of equal mass are
384. In Q. 382, the tangent of magnetic latitude λ is N1 B N2
joined at right angles to each
(a) 2M cos δ (b) M sin δ θ
other. Magnet N1S1 has a mag-
(c) tan δ (d) 0.5 tan δ.
netic moment 3 times that of
385. The time period of a thin bar magnet in earth’s mag- N2S2 . This arrangement is piv-
netic field is T. If the magnet is cut into two equal parts oted so that it is free to rotate S2 S1
perpendicular to its length, the time period of each part in a horizontal plane. When in
in the same field will be equilibrium, θ is Fig. 124
T
(a) (b) T (a) 0° (b) 30°
2
(c) 2 T (d) 2 T. [MP PET 2001] (c) 45° (d) 60°. [Manipal 2002]
292 COMPREHENSIVE OBJECTIVE PHYSICS

393. The B-H curves (a) and (b) 399. When a bar magnet is placed at 90° to a uniform mag-
shown in the Fig. 125 are as- B a netic field, it is acted upon by a couple which is maxi-
sociated with mum. For the couple to be half of the maximum value,
(a) a diamagnetic and a paramag- b the magnet should be inclined to the magnetic field at
netic substance respectively H an angle of
(b) a paramagnetic and a
(a) 45° (b) 30°
ferromagnetic substance re-
spectively (c) 15° (d) 0°.
(c) soft iron and steel respectively 400. A magnetised wire of magnetic moment M and length l
(d) steel and soft iron respec- Fig. 125
is bent in the form of a semi-circle of radius r. Now, the
tively. new magnetic moment is
394. At a certain place, the angle of dip is 50° and the hori- 2M
zontal component of earth’s field is 0.15 T. A beam of (a) M (b)
π
protons is moving vertically upwards with a speed M π
of 3 × 105 m s–1. The force on a proton is [cos 50° = 0.6, (c) (d) .
π M
sin 50° = 0.8]
401. Two bar magnets A and B of the same mass, size and
(a) 4.93 × 10–15 N (b) 5.88 × 10–15 N
shape when suspended one after the other in the stir-
(c) 8.60 × 10–15 N (d) 7.2 × 10–15 N. rup of a vibration magnetometer make 10 vibrations
395. To keep a small magnetic needle perpendicular to mag- and 12 vibrations in one minute respectively. The ratio
netic field at two places A and B, the couples required of their magnetic moments is
are τa and τb respectively. The ratio of magnetic fields (a) 36 : 25 (b) 5 : 6
at two places is
(c) 25 : 36 (d) 6 : 5.
τa τb
(a) (b) 402. A very small magnet is placed in the magnetic merid-
τb τa
ian with its S-pole pointing north. The null point is
1
(c) τb τa (d) . obtained at a distance of 20 cm from the centre of mag-
τb τ a
net. If the earth’s magnetic field at this point is
396. Two magnetic north poles, N 0.3 gauss, the magnetic moment of magnet is
each of m ampere metre, are
(a) 8.0 × 102 emu (b) 2.4 × 103 emu
placed at the two vertices of an
equilateral triangle of side a. a a (c) 1.2 × 10 emu3
(d) 8 × 103 emu.
The resultant magnetic induc- 403. A magnet of magnetic moment 50 i A m2 is placed
tion at the third vertex is → 
along the x-axis in a magnetic field B = (0.5 i + 3.0 j )
µ0 m µ0 2m N a
(a) (b) T. The torque acting on the magnet is
4 π a2 4π a 2
Fig. 126 (a) 175 k N m (b) 150 k N m
µ 3m µ 4m
(c) 0 (d) 0 2 . (c) 75 i N m (d) 25 37 k N m.
4π a2 4π a
397. In the previous question,if one N pole is replaced by S [MP PMT 2000]
pole, then the resultant magnetic induction at the third 404. A bar magnet used in a vibration magnetometer is
vertex is heated so as to reduce its magnetic moment by 19%.
The periodic time of the magnetometer will
µ0 m µ0 2m (a) increase by 19% (b) decrease by 19%
(a) (b)
4 π a2 4π a2
(c) increase by 11% (d) decrease by 11%.
µ0 3m µ 4m [MP PMT 2001]
(c) (d) 0 2 .
4π a2 4π a 405. The earth’s magnetic field at a certain place has a hori-
398. Two north poles of pole strengths 0.4 A m and 6.4 A m zontal component 0.3 gauss and the total strength
are separated by 10 cm. The distance of the neutral 0.5 gauss. The angle of dip is
point from 0.4 A m is 3 3
(a) tan–1 (b) sin–1
(a) 2 cm (b) 4 cm 4 4
4 3
(c) 8 cm (d) 10 cm. [AMU 1997] (c) tan–1 (d) sin–1 . [MP PMT 1995]
3 5
MAGNETIC EFFECTS OF CURRENT AND MAGNETISM 293

406. A tangent galvanometer has a coil with 50 turns and 413. A current of 3 A is flowing in a plane circular coil of
radius equal to 4 cm. A current of 0.1 A is passing radius 4 cm and having 20 turns. The coil is placed in
through it. The plane of the coil is set parallel to the a uniform magnetic field of 0.5 Wb m–2. Then, the di-
earth’s magnetic meridian. If the value of the earth’s pole moment of the coil is
horizontal component of the magnetic field is 7 × 10 –5 (a) 3000 A m2 (b) 0.3 A m2
tesla and µ0 = 4π × 10 –7 WbA–1m–1, then the deflection (c) 75 A m 2
(d) 300 A m2.
in the galvanometer needle will be
[MP PMT 2001]
(a) 45° (b) 48.3°
414. In the previous question, the P.E of the magnetic di-
(c) 50.7° (d) 52.7°. [MP PMT 1993] pole in the position of stable equilibrium is
407. A magnet of magnetic moment 20 cgs units is freely (a) – 1500 J (b) – 0.15 J
suspended in a uniform magnetic field of 0.3 cgs units. (c) + 0.15 J (d) + 1500 J.
The amount of work done in deflecting it by an angle of
415. In Q. 413, to hold the current-carrying coil with nor-
30° in cgs units is
mal to its plane making an angle of 90° with the direc-
(a) 6 (b) 3 3 tion of magnetic induction, the necessary torque is
(c) 3 (2 – 3) (d) 3. [MP PET 1991] (a) 1500 N m (b) 0.15 N m
408. At a place, the horizontal component and the total (c) 15 N m (d) 150 N m.
magnetic induction due to earth are 3 × 10 –5 weber per 416. A magnet is suspended in such a way that it oscillates
metre 2 and 6 × 10 –5 weber per metre2. Then the angle in the horizontal plane. It makes 20 oscillations per
of dip is minute at a place where dip angle is 30° and 15 oscil-
(a) 0° (b) 30° lations per minute at a place where dip angle is 60°.
(c) 45° (d) 60°. The ratio of earth’s magnetic field at two places is
409. In Q. 408, the vertical component of the earth’s mag- (a) 3 3 : 8 (b) 16 : 9 3
netic induction at that place, is
(c) 4 : 9 (d) 2 2 : 3.[MP PMT 1991]
(a) 3 × 10 –5 T (b) 27 × 10 –5 T
417. The angle of dip at a place is 40.6° and the vertical
(c) 6 × 10 –5 T (d) 9 × 10 –5 T.
component of the earth’s magnetic field is 6 × 10 –5 tesla.
410. A coil of 100 turns and area 2 × 10 –2 m2 is pivoted The earth’s magnetic field at this place is [Take sin
about a vertical diameter in a uniform magnetic field 40.6° = 0.6]
and carries a current of 5 A. When the coil is held with
(a) 7 × 10 –5 tesla (b) 6 × 10 –5 tesla
its plane in north-south direction, it experiences a cou-
(c) 5 × 10 –5 tesla (d) 10 –4 tesla.
ple of 0.3 N m. When the plane is east-west, the corre-
sponding couple is 0.4 N m. The value of magnetic in- 418. Fig. 127 shows a short magnet
duction is [Neglect earth’s magnetic field] executing small oscillations in
(a) 0.2 T (b) 0.3 T a vibration magnetometer in
18A
(c) 0.4 T (d) 0.05 T. earth’s magnetic field having
horizontal component 24 µ T.
411. The value of horizontal component of earth’s magnetic
The period of oscillation is 0.1
field at a place is 0.35 × 10 –4 T. If the angle of dip is K
s. When the key K is closed, an
60°, the value of vertical component of earth’s magnetic 0.2 m
upward current of 18 A is es-
field is nearly
tablished as shown. The new
(a) 0.10 × 10 –4 T (b) 0.2 × 10 –4 T Fig. 127
time period is
(c) 0.40 × 10 T
–4
(d) 0.61 × 10 –4 T.
(a) 0.1 s (b) 0.2 s
412. The flux through a certain toroid changes from 0.65 m (c) 0.3 s (d) 0.4 s.
Wb to 0.91 m Wb when the air core is replaced by an-
419. A magnet of magnetic moment M oscillating freely in
other material. The relative permeability and perme-
earth’s horizontal magnetic field makes n oscillations
ability of material are respectively
per minute. If the magnetic moment is quadrupled and
(a) 1.40, 5.6π × 10 –7 H m–1 (b) 1.4, 0
the earth’s field is doubled, the number of oscillations
(c) 0, 1.4 (d) 4π × 10 –7 H m–1. made per minute would be
294 COMPREHENSIVE OBJECTIVE PHYSICS

n n 426. Two bar magnets of same length, breadth and mass


(a) (b)
2 2 2 and having dipole moments 3 pm and 2 pm are allowed
to oscillate in the vibration magnetometer. First we put
(c) 2 2 n (d) 2 n. [MP PET 1991]
the magnets with their north poles in the same direc-
420. The magnetic field at a distance d from a short bar tion and then in the opposite directions. If in the first
magnet in longitudinal and transverse positions are in case, the time period is T, then in the second case, the
the ratio time period will be
(a) 1 : 1 (b) 1 : 2 (a) T (b) 2 T
(c) 2 : 1 (d) 4 : 1. (c) 3 T (d) 5 T .
[Bharati Vidyapeeth 1998]
427. The correct value of dip angle at a place is 45°. If the
421. A bar magnet of magnetic moment 104 J T –1 is free to dip circle is rotated 45° out of the meridian, then the
rotate in a horizontal plane. The work done in rotating tangent of the angle of apparent dip is
the magnet slowly from a direction parallel to a hori- 1
zontal field of 4 × 10 –5 T to a direction 60° from the (a) 1 (b)
2
field will be 1
(c) 2 (d) .
(a) 0.2 J (b) 2.0 J 2
(c) 4.18 J (d) 2 × 102 J. 428. The plane of dip circle is set in the geographic merid-
[MP PET 1995] ian and the apparent dip is θ1. It is then set in a verti-
cal plane perpendicular to the geographic meridian.
422. An iron rod of 0.2 cm2 cross-sectional area is subjected
Now, the apparent dip is θ2. The angle of declination α
to a magnetising field of 1200 A m–1. The susceptibility
at that place is
of iron is 599. The magnetic flux produced is
(a) 0.904 Wb (b) 1.81 × 10 –5 Wb (a) tan α = tan θ1 tan θ2

(c) 0.904 × 10 –5 Wb (d) 5.43 × 10 –5 Wb. (b) tan α = (tan θ 1) 2 + (tan θ2 ) 2


[Himachal PMT 1997] tan θ 1
423. The direction of magnet in tan B position of a deflec- (c) tan α =
tan θ 2
tion magnetometer is tan θ 2
(a) North-South (b) East-West (d) tan α = .
tan θ 1
(c) North-West (d) South-West.
429. A bar magnet of length 10 cm and having the pole
[MP PMT 1988] strength equal to 10 –3 weber is kept in a magnetic field
424. The magnetic needle of an oscillation magnetometer having magnetic induction (B) equal to 4π × 10 –3 tesla.
makes 10 oscillations per minute under the action of It makes an angle of 30° with the direction of magnetic
earth’s magnetic field only. When a bar magnet is induction. The value of the torque acting on the mag-
placed at some distance along the axis of the needle, it net is
makes 14 oscillations per minute. If the bar magnet be (a) 2π × 10 –7 N m (b) 2π × 10 –5 N m
turned so that its poles interchange the positions, the (c) 0.5 N m (d) 0.5 × 102 N m.
new frequency of the needle is [MP PMT 1993]
(a) 2 oscillations per minute 430. In a vibration magnetometer, the time period of a bar
(b) 12 oscillations per minute magnet oscillating in horizontal component of earth’s
(c) 20 oscillations per minute magnetic field is 2 s. When a magnet is brought near
and parallel to it, the time period reduces to 1 s. The
(d) 69.7 oscillations per minute.
ratio H/F of the horizontal component H and the field
425. A magnetic needle vibrates in the vertical plane per- F due to magnet will be
pendicular to the magnetic meridian. Its time period (a) 3 (b) 1/3
is 2 s. If the same needle vibrates in the horizontal plane,
(c) 3 (d) 1/ 3 . [MP PMT 1990]
then the time period is again 2 second. The angle of dip
at that place is 431. The distances of two points on the axis of a magnet
(a) 0° (b) 30° from its centre are 10 cm and 20 cm respectively. The
ratio of magnetic fields at these points is 12.5 : 1. The
(c) 45° (d) 60°.
length of the magnet is
MAGNETIC EFFECTS OF CURRENT AND MAGNETISM 295

(a) 5 cm (b) 25 cm 439. The time period of oscillation of a magnet in a vibra-


(c) 10 cm (d) 20 cm. [AFMC 1999] tion magnetometer is 1.5 second. The time period of
oscillation of another magnet similar in size, shape and
432. If a bar magnet of magnetic moment 80 units be cut mass but having one-fourth magnetic moment than that
into two halves of equal lengths, the magnetic moment of the first magnet oscillating at same place will be
of the each half will be (a) 0.75 s (b) 3.0 s
(a) 80 units (b) 40 units (c) 1.5 s (d) 6.0 s. [MP PMT 1993]
(c) 160 units (d) 20 units. 440. A magnetic dipole is placed in the position of stable
[CMC LDH 1998] equilibrium in a uniform magnetic field of induction
433. A magnetising field of 1600 A m–1 produces a mag- B. If it is rotated through an angle of 180°, then the
netic flux of 2.4 × 10 –5 Wb in an iron bar of cross-sec- work done is
tion area 0.2 cm2. The susceptibility of bar is (a) MB (b) 2MB
(a) 595.6 (b) 1192 (c) MB/2 (d) Zero. [MP PMT 1997]
(c) 298 (d) 1788. 441. In two separate experiments, the neutral points due to
two small magnets are at a distance of r and 2r in broad
[Haryana PMT 1998]
side-on position. The ratio of their magnetic moments
434. A paramagnetic gas consists of atoms with dipole mo- will be
ment M. The temperature of gas is T1 and its volume (a) 4 : 1 (b) 1 : 2
density ρ. Then thermal energy of each dipole compared (c) 2 : 1 (d) 1 : 8. [MP PMT 1985]
with magnetic potential energy in a magnetic field B is
442. The magnetic field at a point x on the axis of a small
given by
bar magnet is equal to the field at a point y on the equa-
3 kT1 2 kT1 tor of the same magnet. The ratio of the distances of x
(a) (b)
2 MB 3 MB and y from the centre of the magnet is
2 k T1 1 kT1 (a) 2 –3 (b) 2 –1/3
(c) (d) .
MB 3 MB (c) 23 (d) 21/3. [MP PMT 1990]
[Bharati Vidyapeeth 2000] 443. Time period for a magnet is T. If it is divided in four
435. A circular coil of diameter 0.07 m has 24 turns of wire equal parts, then time period for each part will be
which carry current of 0.75 A. The magnetic moment (a) 4 T (b) T/4
of the coil is
(c) T/2 (d) T.
(a) 6.9 × 10–2 A m2 (b) 2.3 × 10–2 A m2
444. A bar magnet has coercivity 4 × 103 A m–1. It is desired
(c) 10–3 A m2 (d) 10–2 A m2. [BPET 1994] to demagnetise it by inserting it inside a solenoid 12
436. A magnetic needle lying parallel to a magnetic field cm long and having 60 turns. The current sent through
requires W units of work to turn it through 60°. The the solenoid must be
torque needed to maintain the needle in this position (a) 8 A (b) 6 A
will be (c) 4 A (d) 2 A. [Pb. CET 1997]
(a) 3 W (b) W 445. Two ferromagnetic materials X
3 and Y have hysteresis curves of
(c) W (d) 2 W. [AIEEE 2003] the shapes shown. (Fig. 128) Y
2
(a) X and Y suitable for making a
437. At a certain place, the angle of dip is 30° and the hori-
permanent magnet as well as
zontal component of earth’s magnetic field is 0.50 oer- an electromagnet.
sted. The earth’s total magnetic field (in oersted) is (b) X is more suitable for making O
(a) 3 (b) 1 a permanent magnet while Y is
more suitable for making an X
(c) 1/ 3 (d) 1/2. [CPMT 1992] electromagnet.
438. At a certain place, a magnet makes 30 oscillations/ (c) X is more suitable for making
minute. At another place where the magnetic field is an electromagnet while Y is
double, its time period will be more suitable for making a per-
(a) 4 s (b) 2 s manent magnet.
Fig. 128
(d) neither X nor Y is suitable for
(c) 0.5 s (d) 2 s. making either a permanent magnet or an electromagnet.
296 COMPREHENSIVE OBJECTIVE PHYSICS

446. A short magnet of moment 6.75 A m2 produces a neu- from its centre on opposite sides. The ratio of the mag-
tral point on its axis. If the horizontal component of netic fields at A and B will be approximately equal to
earth’s magnetic field is 5 × 10 –5 Wb m–2, then the dis- (a) 1 : 9 (b) 2 : 9
tance of the neutral point should be (c) 27 : 1 (d) 9 : 1. [CPMT 1989]
(a) 10 cm (b) 20 cm 454. The workdone in turning a magnet of magnetic mo-
(c) 30 cm (d) 40 cm. [AIIMS 1998] ment M by an angle of 90° from the meridian is n times
447. Two small magnets, each of magnetic moment 10 A m2, the corresponding workdone to turn it through an an-
are placed end-on position 0.1 m apart from their cen- gle of 60°. The value of n is given by
tres. The force acting between them is (a) 2 (b) 1
(a) 0.6 × 107 N (b) 0.06 × 107 N (c) 1/2 (d) 1/4. [AMU 1999]
(c) 0.6 N (d) 0.06 N. [AMU 1998] 455. When two tangent galvanometers of the same coil ra-
448. The period of oscillation of a bar magnet in a vibration dii are connected in series, a flow of current in them
magnetometer is 2 second. The time period of oscilla- produces deflections of 60° and 45°. The ratio of the
tion of a bar magnet whose magnetic moment is four number of their turns will be
times that of the first magnet is 4 3
(a) (b)
(a) 1 second (b) 4 second 3 1
(c) 2 second (d) 1/2 second. [CPMT 1997] 3+1 3+1
(c) (d) . [MP PMT 1999]
449. The magnetic field on the axis of a short bar magnet is 1 3−1
1 tesla at a point at a distance of 10 cm from its centre. 456. A short magnetic needle is pivoted in a uniform mag-
What will be the field at a point on its axis at a dis- netic field of strength 1 T. When another magnetic field
tance of 20 cm from its centre ? of strength 3 T is applied to the needle in a perpen-
(a) 0.5 T (b) 2 T dicular direction, the needle deflects through an angle
(c) 0.25 T (d) 0.125 T. [Pb. CET 1998] θ, where θ is
(a) 45° (b) 90°
450. The time period of oscillation of a bar magnet sus-
pended horizontally along the magnetic meridian is T0 . (c) 60° (d) 30°.
If this magnet is replaced by another magnet of the [Karnataka CET 1999]
same size and pole strength, but with double the mass, 457. At a certain place, the horizontal component B0 and
the new time period will be the vertical component V0 of the earth’s magnetic field
are equal in magnitude. The total intensity at the place
(a) T0/2 (b) T0/ 2
will be
(c) 2 T0 (d) 2T0. [DPMT 1997] (a) B0 (b) B02

451. A thin magnetic needle vibrates in the horizontal plane (c) 2B0 (d) 2 B0. [MP PMT 1999]
with a period of 4 second. The needle is cut into two 458. A compass needle placed at a distance r from a short
halves by a plane normal to magnetic axis of the nee- magnet in tan A position shows a deflection of 60°. If
dle. Then the period of vibration of each half needle is the distance is increased to r(3)1/3 then the deflection
approximately of the compass needle, is
(a) 4 second (b) 2 second (a) 30° (b) 60 × 31/3
(c) 8 second (d) 1 second. (c) 60 × 3 2/3
(d) 60 × 33/3.
[MP PMT 1997, 1999] [Himachal PMT 2001]
452. If the ratio of earth’s magnetic field is 2 : 3, then the 459. A magnet of moment 4.0 A m2 is held in a uniform
ratio of time periods of a magnet will be magnetic field 5.0 × 10 –4 T with the magnetic moment
(a) 1 : 2 (b) 9 : 4 vector making an angle of 30° with the field. The work
done in increasing the angle from 30° to 45° is nearly
(c) 4 : 9 (d) 3: 2.
(a) 3.18 × 10 –4 J (b) 4.3 × 10 –4 J
[Bharati Vidyapeeth 1999] (c) 7.3 × 10 –4 J (d) zero.
453. Points A and B are situated perpendicular to the axis [National Standard Exam. in Physics 1997]
of a 2 cm long bar magnet at large distances x and 3x
MAGNETIC EFFECTS OF CURRENT AND MAGNETISM 297

460. If a magnet is suspended at an angle 30° to the mag- their centres 40 cm east and 50 cm west of magnetic
netic meridian, the dip needle makes an angle of 60° needle. If the needle remains undeflected, the ratio of
with the horizontal. The true value of dip is their magnetic moments M1 : M2 is
FG 2 IJ FG 3 IJ (a) 4 : 5 (b) 16 : 25
(a) tan–1
H 3K (b) tan–1
H 2K (c) 64 : 125 (d) 2 : 5 . [MP PET 1990]
(c) tan–1 (3) (d) tan–1 (2). 467. Three identical bar magnets, N S
[Himachal PMT 2002] each of magnetic moment M, are
461. The time period of a vibration magnetometer is T0 . Its placed in the form of an equilat-
magnet is replaced by another magnet whose moment eral triangle with north pole of
one touching the south pole of the
of inertia is 3 times and magnetic moment is 1/3 of the S N
other (Fig. 129). The net manetic
initial magnet. The time period now will be N S
moment of the system is
(a) 3T0 (b) T0 Fig. 129
(a) zero (b) 3M
(c) T0 / 3 (d) T0 /3. [MP PMT 2000] (c) 3M/2 (d) M 3 .
462. A bar magnet A of magnetic moment MA is found to 468. The pole strength of a bar magnet is 48 A m and the
oscillate at a frequency twice that of magnet B of mag- distance between its poles is 25 cm. The moment of the
netic MB when placed in a vibrating magnetometer. couple by which it can be placed at an angle of 30°
We may say that with the uniform magnetic field of flux density 0.15 N
(a) MA = 2 MB (b) MA = 8 MB A–1 m–1 will be
(c) MA = 4 MB (d) MB = 8 MA. (a) 12 N m (b) 18 N m
[Haryana PMT 1999] (c) 0.9 N m (d) None of these.
463. A coil in the shape of an equilateral triangle of side l is 469. The period of oscillation of a magnet at a place is
suspended between the pole pieces of a permanent mag- 4 second. When it is remagnetised, so that the pole

net such that B is in plane of the coil. If due to a cur- strength becomes 4 times the initial value, the period
rent i in the triangle a torque τ act on it, the side l of the of oscillation in second is
tirangle is : (a) 1/2 (b) 1
1 (c) 2 (d) 4. [EAMCET 2003]
FG IJ
1
F
(b) 2 G
τ I 2
(a) 2
3
τ 2
H K
Bi H J
3 Bi K
470. Two short bar magnets of magnetic moments M each
are arranged at the opposite corners of a square of side
d such that their centres coincide with the corners and
2 F τ I
(c) G J
3 H Bi K
(d)
1 τ
3 Bi
. their axes are parallel. If the like poles are in the same
direction, the magnetic induction at any of the other
[All India PM/PD 2005] corners of the square is
464. A certain amount of current when flowing in a prop- µ0 M µ0 2 M
(a) (b)
4 π d3 4π d3
erly set tangent galvanometer, produces a deflection of
µ0 M 2
45°. If the current be reduced by a factor of 3 , the (c) (d) µ 0 M .
deflection would 4 π 2 d3 4 π 2 d3
(a) decrease by 30° (b) decrease by 15° [EAMCET 2003]
(c) increase by 15° (d) increase by 30°.
471. Fig. 130 shows the north-south line
and a bar magnet at 45° from it
[MP PMT 1996] with its polarity shown. The neu-
465. Two normal uniform magnetic field contain a mag- tral points would occur approxi-
netic needle making an angle 60° with F. Then the ra- mately along the line
F (a) OA
tio of is
H
(b) OB
(a) 1 : 2 (b) 2 : 1
(c) OC
(c) 3 :1 (d) 1 : 3.
(d) OD. Fig. 130
466. Two short magnets with their axes horizontal and per-
pendicular to the magnetic meridian are placed with [National Standard Exam. in
Physics 1994]
298 COMPREHENSIVE OBJECTIVE PHYSICS

472. Two identical thin bar magnets, each of length l and 475. A tangent galvanometer has a coil of 25 turns and a
pole strength m, are placed at right angle to each other radius of 15 cm. The horizontal component of the earth’s
with north pole of one touching south pole of the other. magnetic field is 3 × 10 –5 T. The current required to
The magnetic moment of the system is produce a deflection of 45° in it, is
(a) ml (b) 2ml (a) 0.29 A (b) 1.2 A
(c) 3.6 × 10 A
–5
(d) 0.14 A. [MP PMT 2000]
(c) 2 ml (d) ml/2.
476. A magnetic needle of magnetic moment 60 A m2 is di-
[Bharati Vidyapeeth 2000]
rected towards the geographical north at a place. It
473. At a place, the horizontal and vertical components of experiences a torque of 1.2 × 10 –3 N m. If the earth’s
the earth’s field are 4.5 × 10 –5 T and 6 × 10 –5 T horizontal component of that place is 40 µ Wb m–2, then
respectively. The total field of the earth at that place is the angle of declination at that place will be
..... × 10 –5 T.
(a) 30° (b) 45°
(a) 1.5 (b) 7.5
(c) 60° (d) 90°. [Roorkee 1982]
(c) 9.0 (d) 10.5.
477. The length of a magnet is large compared to its width
474. A thin rectangular magnet suspended freely has a pe-
and breadth. The time period of its oscillation in a vi-
riod of oscillation equal to T. Now it is broken into two
bration magnetometer is 2 s. The magnet is cut along
equal halves (each having half of the original length)
its length into three equal parts and these parts are
and one piece is made to oscillate freely in the same
then placed on each other with their like poles together.
T′ The time period of this combination will be
field. If its period of oscillation is T ′, the ratio is
T 2
(a) 2 3 s (b) s
1 1 3
(a) (b) 2
4 2 2
(c) 2 s (d) s [AIEEE 2004]
1 3
(c) (d) 2 [AIEEE 2003]
2

Answers (Set II)


218. (c) 219. (b) 220. (b) 221. (b) 222. (b) 223. (d) 224. (c) 225. (a)
226. (b) 227. (c) 228. (c) 229. (a) 230. (c) 231. (c) 232. (a) 233. (a)
234. (a) 235. (d) 236. (a) 237. (c) 238. (d) 239. (b) 240. (c) 241. (a)
242. (c) 243. (c) 244. (d) 245. (a) 246. (a) 247. (c) 248. (a) 249. (d)
250. (a) 251. (d) 252. (a) 253. (a) 254. (a) 255. (d) 256. (d) 257. (b)
258. (d) 259. (b) 260. (c) 261. (a) 262. (b) 263. (a) 264. (a) 265. (b)
266. (c) 267. (e) 268. (e) 269. (a) 270. (e) 271. (b) 272. (a) 273. (c)
274. (d) 275. (a) 276. (c) 277. (b) 278. (b) 279. (a) 280. (c) 281. (d)
282. (c) 283. (d) 284. (c) 285. (a) 286. (c) 287. (a) 288. (a) 289. (b)
290. (b) 291. (d) 292. (c) 293. (a) 294. (c) 295. (d) 296. (c) 297. (c)
298. (a) 299. (b) 300. (b) 301. (c) 302. (b) 303. (c) 304. (c) 305. (b)
306. (b) 307. (b) 308. (b) 309. (c) 310. (c) 311. (a) 312. (c) 313. (d)
314. (c) 315. (b) 316. (c) 317. (d) 318. (d) 319. (b) 320. (b) 321. (d)
322. (b) 323. (a) 324. (a) 325. (b) 326. (b) 327. (a) 328. (d) 329. (c)
330. (a) 331. (a) 332. (a) 333. (c) 334. (b) 335. (c) 336. (b) 337. (c)
338. (c) 339. (a) 340. (c) 341. (b) 342. (a) 343. (c) 344. (a) 345. (c)
346. (a) 347. (a) 348. (b) 349. (b) 350. (d) 351. (d) 352. (d) 353. (b)
354. (b) 355. (a) 356. (a) 357. (b) 358. (c) 359. (a) 360. (d) 361. (a)
MAGNETIC EFFECTS OF CURRENT AND MAGNETISM 299

362. (d) 363. (b) 364. (c) 365. (a) 366. (c) 367. (b) 368. (b) 369. (d)
370. (d) 371. (b) 372. (c) 373. (c) 374. (a) 375. (d) 376. (c) 377. (a)
378. (a) 379. (c) 380. (d) 381. (d) 382. (a) 383. (b) 384. (d) 385. (a)
386. (b) 387. (c) 388. (d) 389. (c) 390. (c) 391. (c) 392. (b) 393. (c)
394. (d) 395. (a) 396. (c) 397. (a) 398. (a) 399. (b) 400. (b) 401. (c)
402. (c) 403. (b) 404. (c) 405. (c) 406. (b) 407. (c) 408. (d) 409. (b)
410. (d) 411. (d) 412. (a) 413. (b) 414. (b) 415. (b) 416. (b) 417. (d)
418. (b) 419. (c) 420. (c) 421. (a) 422. (b) 423. (b) 424. (a) 425. (c)
426. (d) 427. (c) 428. (c) 429. (a) 430. (b) 431. (c) 432. (b) 433. (a)
434. (a) 435. (a) 436. (a) 437. (c) 438. (d) 439. (b) 440. (b) 441. (d)
442. (d) 443. (c) 444. (a) 445. (b) 446. (c) 447. (c) 448. (a) 449. (d)
450. (c) 451. (b) 452. (d) 453. (c) 454. (a) 455. (b) 456. (c) 457. (d)
458. (a) 459. (a) 460. (b) 461. (a) 462. (c) 463. (b) 464. (b) 465. (d)
466. (c) 467. (a) 468. (c) 469. (c) 470. (a) 471. (a) 472. (c) 473. (b)
474. (c) 475. (a) 476. (a) 477. (c).

Solutions (Set II)


µ 0NI 3I I
218. B = ; N doubled; r halved 225. B1 ∝
2r r
µ I 2Br I I I
219. B = 0 or I = B2 ∝ 3 × .
2r µ0 r
E 4 Clearly, the two fields are
10
220. Bz = y = Wb m −2 = 103 Wb m–2 equal in magnitude and op- 3I
vx 10 posite in direction.
4I
221. Plane of motion must be perpendicular to at least one Fig. 131
2
component of the magnetic field. mv
226. Bqv =
V r
222. R = −G
ig mv
or r=
FG 2.5 IJ Bq
=
H 10
−2 K
− 100 Ω = (250 – 100) Ω = 150 Ω or
Clearly,
r ∝ mv
rA > rB
223. The magnetic field will not do any work. Also, the force
on the electron acts in such a way that it describes a ∴ mA vA > mB vB.
circular path in clockwise direction. 227. Consider an element of thickness dr at a distance r
224. Net current is (20 – 6 + 12 – 7 + 18) A i.e. 37 A. from the centre of the spiral.
10 1 N
r= m= m Number of turns, dN = dr
100 10 b−a
µ 0I N
B= µ0 dr I
2πr b−a
dB =
2r
4 π × 10 −7 × 37 × 10 µ0I N dr
Fig. 132
= T
2π × 1 or dB =
2 b−a r
= 74 × 10–6 T = 74 µ T.
300 COMPREHENSIVE OBJECTIVE PHYSICS

z F I = FG 1 + 16 IJ
b 3/ 2 3/ 2
µ I N 1 B′ x2
B= 0
2 b−a
a
r
dr
B GH JK H 9 K
= 1+ 2
r

µ I N
B= 0 log e r
b
F 25 I = FG 5 IJ = 125
=G J
3/ 2 3

2 b− a a
H 9 K H 3 K 27
µ0 NI 125 125
B= [loge b – loge a] or B′ = B= × 54 µT = 250 µT
2(b − a) 27 27
µ 0 NI b mv2 mv p 2 m Ek
B= loge . 232. Bqv = or r = or r = =
2(b − a) a Bq
r Bq Bq

+ Time-saving solution In the given problem,

(a) is rejected because it does not contain “a”. m


r∝
(b) is rejected because it does not contain “b”. q
(d) is rejected because it is dimensionally inac-
curate. See IN. rp m 2 1
= × =
rα 1 4m 1
228. The total current shown by the galvanometer,
233. θ = 60°
Ig = 25 × 4 × 10–4 A = 10–2 A → →
[Note that θ is the angle between v and B .]
V 25 FG IJ
R=
Ig
−G=
10−2 H
− 50 Ω = 2450 Ω
K F = Bqv sin 60°
3
229. Magnetic fields due to two sections are equal and op- = 0.2 × 1.6 × 10–19 × 106 ×
2
posite.
= 2.77 × 10–14 N.
2
mv I1I2
230. Bqv = 234. F∝
r d
mv – 2I1I2
Bq = F′ ∝
r 3d
1 F ′ −2 −2F
But qV = mv2 = or F′ =
2 F 3 3
2qV qB 1.6 × 10−19 × 1
or v= 235. ν= = Hz
m 2πm 2 × 3.14 × 9.1 × 10−31
m 2qV = 28 × 109Hz = 28 GHz
∴ Bq =
r m 236. τ = NBIA
In the given problem, q, V and B are constants. = 100 × 0.2 × 2 × 0.10 × 0.08 N m
∴ r∝ m or m ∝ r2 = 0.32 N m
Applying Fleming’s left hand rule, we find that AD
m1 R1 F I 2

m2
=
R2
GH JK moves out of the page.
237. Magnetic field at the centre due to the loop, B
µ 0Ir 2 µ 0I
231. B=
2( r 2 + x 2 ) 3 / 2
=4×
L FG IJ [sin 45° + sin 45°]

µ 0I

2 H K
B′ = 1
2r Clearly, B ∝ .
L
B′ ( r 2 + x 2 ) 3 / 2 238. v⊥ = 4 × 105 × sin 60°
=
B r3 = 2 3 × 105 m s–1
MAGNETIC EFFECTS OF CURRENT AND MAGNETISM 301

mv2⊥ or 0.4i + 0.3 j = (vx i + vy j) × 0.004 k


= Bqv⊥
r
= – 0.004 vx j + 0.004 vyi
mv⊥
or r= Comparing,
Bq
0.004 vy = 0.4
1.67 × 10−27 × 2 3 × 105
= m or vy = 100
0.3 × 1.6 × 10−19 and – 0.004 vx = 0.3
= 12.05 × 10–3 m ≈ 12 mm.
0.3
2πm or vx = –
239. T= 0.004
Bq or vx = – 75
2 × 3.14 × 1.67 × 10−27
= s So, the required velocity vector is – 75 i + 100 j .

240. Pitch = v|| T


0.3 × 1.6 × 10 −19
= 21.85 × 10–8 s. 244.
or
z → →
B . dl = µ0 I
Bl = µ0I
= v cos θ T
µ0 I
= 4 × 105 × cos 60° × 21.85 × 10 –8 m or .B=
l
= 43.7 × 10–3 m = 43.7 mm.
1
245. B∝
µ 0I r 2 r
241. B=
2 (r 2 + x 2 )3 / 2 r When r is increased by a factor of 4, the magnetic field
is reduced by a factor of 4.
µ 0I r 2 I
x
8B = 0.04
2 (r + 0.052 )3 / 2
2
∴ T = 0.01 T
B′ =
4
µ 0I r 2 Fig. 133
B= L
2
2 (r + 0.2 ) 2 3/2 246. 2πr = L or r =

Now, 16(r2 + 0.052)3/2 = 2(r2 + 0.22)3/2
1
162/3(r2 + 0.052) = 22/3(r2 + 0.22) Area of semi-circle = πr2
2
2 82/3(r2 + 0.052) = 22/3(r2 + 0.22)
LM OP
2/3
2
1 L L2
or
4r2 + 4 × 0.052 = r2 + 0.22
3r2 = 0.22 – 4 × 0.052
=
2
π
N Q

=

0.04 − 0.01 L2I


or r2 = Magnetic moment of each semi-circle =
3 8π
or r2 = 0.01 m2 Since the two semi-circles are perpendicular there-
fore the magnetic moments are perpendicular.
or r = 0.1 m.
2 L2I
∴ Net magnetic moment = .
Time-saving solution 8π
In the given problem, 247. BIl = 2 T sin θ
B(r2 + x2)3/2 = constant θ is small, sin θ ≈ θ
8(r2 + 0.052)3/2 = (r2 + 0.22)3/2
BIl
or 4(r2 + 0.052) = r2 + 0.22.

V
242. R= –G
Ig l/2
10 T cos q T cos q
= −3
–1
10 × 10 q q
q
= (1000 – 1) Ω = 999 Ω. r
T T
→ → →
243. F = q( v × B) ⇒ (4i + 3 j) × 10–10 T sin q T sin q


Fig. 134
= 10–9 [ v × 0.004 k ]
302 COMPREHENSIVE OBJECTIVE PHYSICS

2Tl / 2 T mv2 mv
∴ BIl = 2Tθ or BIl = or BI = 257. = Bqv or r =
r r r Bq
l 258. The magnetic fields due to both the currents are along
But 2πr = l or r= ‘– Z-axis’. So, they add up to give a net magnetic field

along ‘– Z-axis’.
T2π BIl
∴ BI = or T =
l 2π mv2 mv

259. = Bqv or r =
   r Bq
248. M net = M k + M i + M j
∴ Mnet = M2 + M2 + M2 p 2m E k
or r= or r =
Bq Bq
πr 2 3 πr 2 I
= 3 ×I× =
4 4 2m qV
r=
GI g Bq
249. S=
I − Ig
1 2mV
GI G r=
S= or S = B q
nI − I n−1
15 I 2 × 9 × 10−31 × 12000
250. S= = 15 Ω. = 103 m
2I − I . × 10−19
16
GI g 1.5 × 10–1 m
251. S= =3×
I − Ig
= 0.3 × 1.5 × 100 cm = 36.74 cm.
S(I − I g ) 5 [100 − 4]
or G= = BI
Ig 4 260. (hρg) a2 = BIa or h =
ρga
5
= × 96 Ω = 120 Ω. 1 µ 0I 4 π × 10 −7 × 9
4 261. B= = N A–1 m–1
6 2r 12 × 10 × 10 −2
µ 0I1I2 = 9.43 × 10–6 N A–1 m–1.
252. F= l
2πr
µ 0I 4 π × 10 −7 × 10
−7 262. B= =
4π × 10 × 10 × 2 2πr 2 π × 5 × 10 −2
= × 2 N = 8 × 10–5 N.
2π × 10 × 10−2 = 4 × 10–5 T = 0.4 gauss
2 Now B= 0.42 + 0.32 gauss
253. Magnetic field due to bigger coil = T=1T
2
= 0.16 + 0.09 gauss = 0.5 gauss.
Now, magnetic fields of 2 T and 1T are oppositely di-
rected. So, net field is 1 T. 263. The direction of the current is given by left hand rule.

E N 4π × 10 −7 × 1000 × 5
254. B= 264. B = µ0 I= T
v l 20 × 10−2
= 0.031 T.
600 V 1
= −3
× 6 −1 1
3 × 10 m 2 × 10 m s 265. Bend = [0.031] T = 0.0155 T.
2
= 10 –1 T = 0.1 T 266. Remember that the magnetic flux density along the
µ 0I µ 0I axis of a solenoid is proportional to (i) the number of
255. −
2r 2πr turns per unit length, (ii) the current. Work out what
µ 0I FG
1 IJ µ 0I (π − 1) the turns ratio of Y to X will be, not forgetting that
=
2r
1−
πH K =
2πr
. the circumference of Y is twice that of X. Then con-
sider the turns per unit length for Y compared to X.
Bq Finally remember that the product of turns per unit
256. ν=
2πm length and current must be the same for X as for Y if
Out of the given particles, Li+ has maximum mass. they are to have equal field strengths.
MAGNETIC EFFECTS OF CURRENT AND MAGNETISM 303

267. The galvanometer is said to be ‘ballistic’ because the Tα 4 m q


idea is that a sudden brief flow of charge through the = × =2
Tp 2q m
coil sets the coil off into oscillating motion that should
ideally be of constant amplitude. The charge must pass LM 25 OP µs = 10 µs.
through in a pulse of short duration compared with
the time period of the coil, so a heavy coil is useful
or Tα = 2
N5Q
274. The Hall voltage VH measured from the semiconduc-
since it would have a large time period. With low elec-
tor due to the presence of a magnitude field B is given
trical resistance, problems would arise in ‘search coil’
by
experiments where the total circuit resistance was low,
BI
allowing induced currents to be generated by the coil VH =
motion, so damping the oscillations. Damped oscilla- net
where I is the current in the semiconductor.
tions would make the ‘first throw’ of the galvanom-
eter difficult to measure. Refer to advanced text book n is the number of charge carriers per unit
volume.
for further details.
t is the thickness of the semiconductor.
268. Think of Newton’s third law.
e is the electron charge.
269. From the graph, we may write
Rearranged the terms, we have
lg(B/µT) = mlg(x/m) + C
VH net
2.0 − 1.1 B=
where m = gradient of the line = =–3 I
1.3 − 1.6
Thus, a variation of 2% in n due to temperature
C = interception on vertical axis = 2.0
changes will eventually lead to an uncertainty of 2%
Hence, lg(B/µT) = – 3lg(x/m) + 2.0 in the computed value for B due to the variation of the
⇒ B = (102)(x–3) = 100x–3 or B is proportional to x–3. measured values for VH. Hence, the sensitivity of the
270. The field at X, along the axis, will be proportional to device for measuring magnetic fields decreases by 2%.
the field at the centre of each of P and Q since the mv 2m Ek
coils are of equal length and X is midways between 275. r= =
Bq Bq
them. Equate : field due to P = field due to Q and solve
to find the current in P. Rd 2m
Now = = 2
271. Current sensitivity C i.e. deflection per unit current Rp m
BAN
is equal to where B = magnetic field strength, or Rd = 2 Rp.
k
A = coil area, N = number of turns, and k is the spring 276. The forces FQ and FR on the current-carrying wire P
constant of the restoring couple. With the same cur- due to the currents in Q and R are shown in Fig. 135.
rent and the same spring strength, the field strength The resultant force is F which is best represented by
deflection vector C in the original diagram of the question.
ratio is equal to the ratio of for X to that for
AN
Y. Take Y’s values as deflection = 1, area = 1, turns = FQ P
Q
1 and then write down the X values, and hence the
deflection
value of for X to that for Y.
AN
mv v F FR
272. r=
Bq
or r
FG IJ
q
B
H K
m R

2 × 105 1 Fig. 135


or r= −2 7 m= m = 0.10 m.
4 × 10 × 5 × 10 10
277. M = IA
2πm m 6.4 × 1021 = I × 3.14 × 6.4 × 6.4 × 1012
273. T= or T ∝
Bq q
304 COMPREHENSIVE OBJECTIVE PHYSICS

6.4 × 10 21 mv
I= A = 4.976 × 107 A 290. r=
3.14 × 6.4 × 6.4 × 10 12 Bq
≈ 5 × 107 A.
278. The magnetic flux density near the middle of the sole- mv 9 × 10−31 × 106
B= = T
noid parallel to the axis is rq 0.10 × 1.6 × 10 −19
µ 0NI = 56.25 × 10–6 T = 5.6 × 10–5 T.
B= . 291. kα = NBIA
l
Cross-sectional area of the solenoid is A. α NBA
or =
Thus, magnetic flux through the cross-section in this I k
region is
100 × 5 × 1 × 10−4
µ NIA = rad A–1
Φ = BA = 0 . 10−8
l
= 500 × 104 rad A–1
279. Fm = Bqv sin θ = 5 × 106 rad A–1 = 5 rad/µA.
= 1.5 × 1.6 × 10–19 × 2 × 107 sin 30° N
µ 0I µ0 q
= 2.4 × 10–12 N. 292. B= or B =
2r 2r T
mv
280. r= 4 π × 10−7 × q
Bq B= n
2r
If v is doubled, then r is doubled.
281. The tension shall change because of magnetic force. 2πnq
B= × 10–7 N A–1 m–1.
282. Due to B|| , the path should be straight. Due to B⊥, the r
path should be circular. The combination of the two mv2
293. Bqv = or Bqr = mv or Bqr = mrω
paths will be helical. r
283. Apply right hand palm rule. Bq Bq
or 2πν = or ν = .
284. No force on PQ and SR. m 2πm
Force on PS or QR = BIl GI g 0.018 × 1
294. S= = Ω = 0.002 Ω.
=2× 10–2 × 2 × 0.25 N = 10–2 N I − Ig 10 − 1
τ = 10–2 N × 0.4 m = 4 × 10–3 N m. 295. V = Ig(R + G)
Bqv V 100
285. a=
m or R= –G = – 25
Ig 10 × 10−3
2 × 1.6 × 10 −19 × 3.4 × 107 = (10000 – 25) Ω = 9975 Ω.
= m s −2
1.67 × 10 −27 296. F ∝ I1 I2
= 6.5 × 1015 m s–2. When both I1 and I2 are doubled, the force is increased
286. V = IR, I constant, V ∝ R, clearly if R is doubled, V is by a factor of 4.
doubled. 297. Note that protons and electrons are oppositely
So, R + 100 = 1000 or R = 900 Ω. charged.
298. The magnetic fields due to both the currents are into
µ 0NI 4π × 10−7 × 100 × 0.1 the plane of paper and add up.
287. B= = T
2r 2 × 5 × 10−2 299. 10 × 99 = 90 S or S = 11 Ω.
= 4π × 10–5 T 1
300. B = × 4π × 10–7 × 800 × 1.6 T = 8 × 10–4 T.
288. G = 7 Ω, Ig = 1.0 A, V = 10 volt 2
10 = 1(7 + R) or R = 3 Ω. 301. M = evA
µ 0I 2Br 2 × 7 × 10−5 × 5 × 10−2 22
289. B = , I= = A = 1.6 × 10–19 × 6.6 × 1015 × × (0.528 × 10–10)2 A m2
2r µ0 4 π × 10−7 7
= 5.6 A. = 9.25 × 10–24 = 0.9 × 10–23 ≈ 1 × 10–23 A m2.
MAGNETIC EFFECTS OF CURRENT AND MAGNETISM 305

4
F µ0 (I) (I) µ 0I2
302. = = . G 5
l 2πb 2 πb 312. = =4
4 1
303. Number of turns doubled, B doubled ; Radius halved, 5
B again doubled, Net effect : B increased by a factor or G = 16 Ω
of 4. Again, combined resist-
mv2 mv ance of 4 Ω and 2 Ω is
304. Bqv = or r= 4 Fig. 136
r Bq Ω
3
p 2mEk 4 x
or r= =
Bq Bq x G
Now, = 3 1A
1 − x 16
π2mEk
Area = πr2 = x 4 1
B2 q2 or = =
1 − x 48 12 1–x 4
Clearly, area ∝ Ek. W
or 1 – x = 12x 3
305. In general, V = Ig(G + R)
or 1 = 13x Fig. 137
V
In the given problem, ‘V’ = nV, Ig = 1
G or x=
13
V
nV = (G + R) or Gn = G + R or R = (n – 1)G. 313. V = Ig(R + G)
G
V 2.5
mv2 R= –G = – 10
306. Bqv = Ig 1 × 10−3
r
Bqr = (2500 – 10) Ω = 2490 Ω.
v= or mv = Bqr
m GI g 10 × 1 × 10 −3
Bqr is constant. 314. S= = Ω
I − Ig (2.5 − 1) × 10 −3
∴ mv = constant
me ve 10−2
∴ mpvp = meve or vp = . = Ω = 4 × 10–3 Ω.
mp 2.5

3 µ 0I LM OP
1 µ 0I LM OP 315. G=
4 × 10−3
Ω=2Ω
307. B=
4 2a
+
N Q
4 2b
.
N Q 2 × 10−3

3µ 0I µ 0I GI g 2 × 2 × 10−3
B= + . S= =
8a 8b I − Ig (1 − 2) × 10−3
r
308. Distance of straight conductor from O = 4 × 10 −3
2 = Ω ≈ 0.004 Ω.
(1 − 0.002)

B=
µ 0I × 2 µ 0I
2 πr
+
π
2r 2 × 2π 316. B=
µ 0I µ 0I
+ or B =
LM
µ 0I 1
+
1 OP
µ 0 2I LM πOP
4 r1 4 r2 N
4 r1 r2 Q
or B=
4π r N 2+
4 Q
.
=
LM
µ0I r2 + r1 . OP
309. Magnetic field at O is the sum of the magnetic fields 4 Nr1 r2 Q
due to semi-circle and semi-infinite wire.
mv2 e v e 2 × 107
317. Bev = or = or = C kg–1
GI g 100 × 10 × 10 −3
r m rB m 10 × 10−5
310. S= = ≈ 0.1 Ω.
I − Ig 10 − 10 × 10−3 = 2 × 1011 C kg –1.
311. W = 2MB or W = 2NIAB 318. Ek = qV
In the given problem, V is constant
22
= 2 × 50 × 2 × (4 × 10–2)2 × 0.1 J = 0.1 J. ∴ Ek ∝ q.
7
306 COMPREHENSIVE OBJECTIVE PHYSICS

mv2 2mEk 2mqV m 4 π × 10 −7 × 20 × 10 × 10 × 10 −2


319. Bqv = or r = = or r ∝ . 331. F=
r Bq Bq q 2 π × 2 × 10 −2
When m is increased by a factor of 2, r is increased by 4π × 10−7 × 10 × 30 × 10 × 10−2

a factor of 2 . 2π × 10 × 10−2
m 4π × 10−7 × 10 × 10−2
320. r∝ = [100 – 30]
q 2π × 10−2
When m is increased by a factor of 4 and q is increased = 20 × 10–7 × 70 = 1400 × 10–7
by a factor of 2, then r becomes 2 times. So, the new = 1.4 × 10–4 N towards right.
radius is 10 2 cm. 332. All points on AB are equidistant from the two wires.
So, magnetic fields are equal and opposite. Thus, they
321. 100 = 5 × 10–3 (R + 5) or R = (20000 – 5) Ω cancel out.
= 19995 Ω.
50
mv p 333. B = 4π × 10–7 × × 4 T = 25.1 × 10–3 Wb
322. r = or r = or r ∝ p. 10 −2
Bq Bq
B
At end, B′ = = 12.6 × 10–3.
mv 2m E k 2
323. Bq = or Bq =
r r 2πm Bq
334. T = = ω.
or
B q r 2 2 2
1 Bq m
Ek = or Ek ∝ .
2m m 335. AB and DE do not give any magnetic field at O.
m is doubled, Ek is halved. So, the new energy is µ 0I
1 MeV. Magnetic field at O due to BC =
4 πa
µ 0I θ µ 0Iθ It is directed ‘up’. Magnetic field at O due to EF =
324. B = . or B =
2r 2π 4πr
µ 0I
µ N I1 I2
325. B = 0 −
LM OP = µ N LM 0.2
0

0.3 OP 4 πa
2 r1 r2 N Q 2 N 20 × 10 −2
40 × 10 −2 Q It is also directed ‘up’.
× 10 L
MN1 − 4 OPQ = 4 .
µ0 3 5µ 0
µ 0I
= So, B = .
2 2 πa
F
mv2 p 2mEk 336. F = BIl or B =
326. Bqv = or r = = or r ∝ m Il
r Bq Bq
[MLT −2 ]
Since me < mp , ∴ re < rp. [B] = = [MA–1 T –2].
[ A ][L]
µ 0I 4 π × 10 −7 × I Br µ 0I
327. B = = or I = × 107 337. BAB = [2 sin θ]
2πr 2πr 2 4 π (OC)
10 −5 × 10 × 10−2 But OC = r cos θ
= × 107 A = 5 A.
2 µ 0I µ I
328. F = Bev = 2 × 10–1 × 1.6 × 10–19 × 4 × 106 N ∴ BAB = (2 sin θ) or BAB = 0 tan θ
4 πr cos θ 2πr
= 12.8 × 10–14 N = 1.28 × 10–13 N
Magnetic field due to circular portion,
mv 9 × 10 −31 × 4 × 106 µ 0I 2π − 2θ µ I
Now r = = m = 0 (π – θ)
Bq 2 × 10 −1 × 1.6 × 10 −19 BAB′ =
2r 2π 2πr
= 11.25 × 10–5 m = 1.125 × 10–4 m. Total magnetic field
329. τ = NBIA = 100 × 0.5 × 1 × 400 × 10 –4 N m = 2 N m. µ 0I µ I
= tan θ + 0 (π – θ)
mg −3 2πr 2πr
10 × 10
330. BIl = mg or B = = T = 10–1 T µ I
Il 10 × 1 × 10 −2 = 0 [tan θ + π – θ].
2πr
= 0.1 T.
MAGNETIC EFFECTS OF CURRENT AND MAGNETISM 307

µ 0NI 2rB in the vertically upward


338. B = or N = ®
2r µ 0I direction. Using Fleming’s B
left hand rule, the mag- v ®
2 × 10 × 10 −2 × 3.14 × 10 −3
+e E
= = 50. netic field B should be
4 × 3.14 × 10 −7 × 10 Fig. 140
such as shown in Fig. 140.
µ I
339. B = 0 [sin θ1 + sin θ2]
4 πd 348. B =
µ 0I µ I
− 0
µ 0I 1
or B = −
1LM.
OP
But θ1 + φ1 = 90° or θ1 = 90° – φ1 4R 1 4R 2 4 R1 R2 N Q
sin θ1 = sin (90° – φ1) = cos φ1 349. F = BIl = 2 × 1.2 × 0.5 N = 1.2 N.
Similarly, sin θ2 = cos φ2 350. No enclosed current. No magnetic field.
µ 0I mv2 mv 2 mK
(cos φ1 + cos φ2).
B= 351. Bqv = or r = =
4 πd r Bq Bq
340. The magnetic fields due to two conductors are per-
pendicular. 2mK
Here, r=d ∴ B= .
qd
µ 0i1
Now, B1 = B2 = BA I 2R 1
2 πd µ 0NI = × = .
352. B = or
µ0 2r BB R 2I 1
Net field = (i 2 + i22)1/2.
2 πd 1 4250
353. B = 4π × 10–7 × × 5 T = 2.67 × 10–2 T.
I 4I I 4I 1
341. = or = or 4r = 1 – r or 5r = 1
r 1− r r 1− r 1
354. B ∝
1 r
or r= m = 0.2 m. When r is doubled, B is halved.
5
355. Magnetic field at a point on axis is zero.
2πm 2π(4 m)
−7 356. T = or T′ = = 2 T.
F 4 π × 10 × 10 × 5 Bq B(2 q)
342. = N m–1
l 2π × 0.1 10A 5A 4 π × 10 −7 × 50 × 2
= 10–4 N m–1. 357. B = T = 1.25 × 10–4 T.
2 × 0.5
0.1 m
A F
358. = 10–4 × 10 N = 10–3 N.
µ 0I µ 0I Fig. 138 l
343. B1 = or B2 =
2πr 2π (2r) 359. V = Ig G
B1 2 3V = Ig(G + R)
Now, = .
B2 1 G+R
Dividing, 3=
344. Forces on different sections cancel out. So, net force is G
zero. R
or = 2 or R = 2G = 2 × 50 × 103 Ω = 105 Ω.
345. τ = NBIA = 1 × B × I × πr2 = πr2 I B. G
5
346. F = BIl sin θ G
100 100
F = 7.5 75 1 S 5 1
sin θ = = = 360. = =
BIl 2 × 5 × 15
. 150 2 G 95 19
θ = 30°. G
→ or S= . 95 S
347. Due to E , the proton would v 19
+e ® Fig. 141
experience a vertically E
downward force of eE.
Fig. 139 mv2 mv
For the proton to move 361. Bqv = or r =
r Bq
undeflected, the magnetic
field B should be such that 1.6 × 10−27 × 107
= m = 0.25 m.
it experiences a force Bev 0.4 × 1.6 × 10−19
308 COMPREHENSIVE OBJECTIVE PHYSICS

GI g 25 × 0.01 0.25 25 µ 0I = µ 0 e µ 0 ne
362. S = = = Ω = Ω. 376. B= .
I − Ig 10 − 0.01 9.99 999 2r 2r T = 2r

1 2eV µ I
377. B= 0
363. mv2 = eV or v = 2r
2 m
I ∝ Br
Fm = Bqv
I1 1 1 1
6 −19
∴ = × = .
2 × 200 × 10 × 16
. × 10 I2 3 2 6
= 5 × 1.6 × 10–19 N
. × 10−27
167 µ 0I
378. B=
= 1.6 × 10 –10 N. 2r
µ q µ × 2 × 1.6 × 10−19
4 π × 10 −7
× 1000 × 0.1 B= 0 = 0 T = 10–19 µ0 T.
364. B = T = 2π × 10 –4 T 2r T 2 × 0.8 × 2
2 × 0.1
379. τ = MB sin θ
= 2 × 3.14 × 10 –4 T = 6.28 × 10 –4 T.
τ and B are constants
365. No field inside the cylinder.
∴ M sin θ = constant
366 to 369. Think in terms of magnetic fields due to (i) semi-
infinite wire (ii) semi-circular wire. ∴ pm sin θ = 3 pm sin (90° – θ)
Also, the magnetic fields due to straight and semi- or tan θ = 3 or θ = 60°.
circular portions are perpendicular. 380. τ = MB sin θ
→ → →
370. F = q ( v × B ) = 10–11 (108 j × 0.5 i ) = 5 × 10–4 (− k) . τ and M are constants.
371. At the mid-point of the line joining the conduc- ∴ B1 sin 15° = B2 sin (60° – 15°)
→ or B1 sin 15° = B2 sin 45°
tors, B = 0. As we come close to the wires, the magni-
tude of B increases. The direction of magnetic fields B1 sin 15°
∴ B2 = .
on opposite sides of a wire will be opposite. Again, sin 45°

B = 0, as r → ∞. 381. (a) 2 M (b) zero
372. Electric field is along + x direction. So, the positive (c) zero (d) 3 M.
ions will acquire velocity along + x direction. The nega- 382 to 384. Point is on the axial line of M sin λ.
tive ions will acquire velocity along – x direction.

For + ve ion, v = vi , ‘q’ = + q
→ R
For – ve ion, v = – vi , ‘q’ = – q –l
90 V
→ → → l
Again, F m = q ( v × B) . Both will be deflected along

– y-axis because B = Bk .
l
sin

M
M

373. 10 = 1 × 10–3 [R + 1]
or 104 – 1 = R or R = 9,999 Ω. (a) (b)
This should be connected in series.
Fig. 142
mv2
374. Bqv =
r 2M sin λ H
V= 3
p 2m Ek R
r= = Point is on the equatorial
Bq Bq
line of M cos λ. R
rα 4
= =1 M cos λ
rp 2 ∴ H=
R3
This alone decides (a). V M cos λ
µ I Now, tan δ =
375. B = 0 H Fig. 143
2r M cos λ R3
−11 or tan δ = ×
B × 2r 12.56 × 2 × 5.2 × 10 R3 2M sin λ
I= = A = 1.04 × 10–3 A.
µ0 4 × 3.14 × 10 −7
MAGNETIC EFFECTS OF CURRENT AND MAGNETISM 309

1 1 394. Fm = BH qv = 0.15 × 1.6 × 10–19 × 3 × 105 N


or tan δ = or tan λ = tan δ = 7.2 × 10–15 N.
2 tan λ 2
= 0.5 tan δ.
385. Mass is halved. Length is halved. Magnetic moment + In Q. 394, note that BV would not exert any
is halved. force on the protons.
I 395. τ = MB sin θ
T = 2π
MB τ∝B
mL2 ∴
Ba τ a
= .
T = 2π
12 MB Bb τ b
396. The two magnetic inductions are inclined at 60°.
T′ 1 1 1 2 1 T
= × × × = or T′ = 397. The two magnetic inductions are inclined at 120°.
T 2 2 2 1 2 2
0.4 6.4
386. Refer to synopsis. 398. =
2
r (10 − r)2
µ 2M 2 × 125
.
387. B = 0 3 = 10–7 × T = 2 × 10–6 T. or 4r = 10 – r or 5r = 10 or r = 2 cm.
4π r 0.5 × 0.5 × 0.5
399. τ = MB sin θ
tan δ = tan 45° = 2 τ = τ0 sin θ
388. tan δ′ =
cos θ cos 60° 1
–1 τ = τ0 sin θ
δ′ = tan (2). 2 0
tan δ tan δ 1
389. tan 30° = or cos θ = = 3 tan δ or sin θ = or θ = 30°.
cos θ tan 30° 2
M
tan δ 400. M = m × l or m =
Again, tan 45° = or sin θ = tan δ l
sin θ
M l
Now, sin2 θ + cos2 θ = 1 M′ = m × 2r = ×2×
l π
∴ tan2 δ + 3 tan2 δ = 1
1 1 l 2M
or tan2 δ = or tan δ = πr = l or r = =
4 2 π π
1 1 401. ν∝ M
or = or δ = cot–1(2).
cot δ 2 M1 ν12 100 25 .
= = =
2M M2 ν 22 144 36
390. 3 × 10–5 = 10–7
(10 × 10−2 )3 2M 0.3 × 20 × 20 × 20
402. 0.3 = or M = emu
−5
1.5 × 10 × 10 −3 203 2
or M= A m2 = 150 × 10–3 A m2. = 1.2 × 103 emu.
10−7

tan δ 403. τ = 50i × (0.5i + 3.0 j) = 150 k .
391. tan 45° =
cos 30°
3 F 3I 404. T = 2π
I
or tan δ =
2
or δ = tan–1 GH 2 JK MB

I × 100
392. τ = MB sin θ T′ = 2π
81 MB
τ and B are constants.
10
Now, 3 sin θ = 1 × sin (90° – θ) T′ = T
9
1 Percentage increase
or tan θ = or θ = 30°.
3 10
393. The hysteresis loss for soft iron is low. On the other −1
1
hand, hysteresis loss for steel is high. = 9 × 100 = × 100 ≈ 11.
1 9
310 COMPREHENSIVE OBJECTIVE PHYSICS

413. M = 20 × 3 × 3.14 × 0.04 × 0.04 A m2 = 0.3 A m2.


405. BV = 0.52 − 0.32 gauss
414. U = – MB = – 0.3 × 0.5 J = – 0.15 J.
or BV = 0.8 × 0.2 gauss = 0.4 gauss 415. τ = MB = 0.15 N m.
BV 0.4 4 FG 4 IJ .
tan δ =
BH
= =
0.3 3
or δ = tan–1
H 3K 416. T = 2π
I
MB cos δ
µ 0 NI µ NI
406. = BH tan θ or tan θ = 0 1 MB cos δ
2r 2 r BH ν=
2π I
4 π × 10 −7 × 50 × 0.1 ν∝ B cos δ
= = 1.1214
2 × 4 × 10 −2 × 7 × 10 −5
ν2
or θ = 48.3°. or B∝
cos δ
407. W = MB(1 – cos 30°)
B1 400 cos 60°
F 3 I cgs units = ×
= 20 × 0.3 1 −GH 2 JK B2 cos 30°
16 × 2 1
225

= × = 16 : 9 3 .
= 3(2 – 3 ) cgs units. 9× 3 2
408 & 409. BH = 0.3 gauss 417. BV = B sin δ
B = 0.6 gauss BV 6 × 10 −5
or B = or B = T
sin δ sin 40.6°
BV = 0.62 − 0.32 = 0.9 × 0.3 gauss
6 × 10−5
27 = T = 10–4 T.
= 0.27 gauss = × 10–4 T = 27 × 10–5 T 0.6
10
−7
B 27 × 10 −5 µ 0I = 4π × 10 × 18
418. B= T = 18 µ T
tan δ = V = = 3 2πr 2π × 0.2
BH 3 × 10 −5
δ = 60°. I
N Now, T = 2π
410. NBIA sin θ = τ ...(1) B MBH
θ
N, B, I and A are constants. I
90–θ and T′ = 2π
∴ sin θ ∝ 0.3 W E M(BH − B)
cos θ ∝ 0.4 Dividing
3 T′ BH
tan θ = S = T′ 24
4 or = =2
T BH − B T 24 − 18
From Eq. (1), Fig. 144
T′ = 2 × 0.1 s = 0.2 s.
τ
B= 1 MB
NIA sin θ 5 419. n=
0.3 × 5
3 2π I
B= T
100 × 5 × 2 × 10−2 × 3 q 1 (4 M) (2B)
n′ = or n′ = 2 2 n.
= 0.05 T. 4 2π I
3
sin q = 420. Longitudinal and transverse positions refer to axial
5
and equatorial positions.
Fig. 145
BV 421. W = MB(1 – cos θ)
411. tan δ =
BH = 104 × 4 × 10–5(1 – cos 60°)
or BV = BH tan δ = 0.35 × 10–4 × tan 60° = 2 × 10–1 J
= 0.35 × 1.732 × 10–4 T = 0.61 × 10–4 T. = 0.2 J.
0.91 422. µ = µ0(1 + χ) or µ = 4π × 10–7 (1 + 599)
412. µr = = 1.4
0.65 or µ = 7.536 × 10–4 T m A–1
µ = µ0 µr = 4π × 10–7 × 1.4 H m–1 B = µ H = 7.536 × 10–4 × 1200 T
= 5.6π × 10–7 H m–1. φ = BA = 7.536 × 10–4 × 1200 × 0.2 × 10–4 Wb
= 1.81 × 10–5 Wb.
MAGNETIC EFFECTS OF CURRENT AND MAGNETISM 311

N 429. τ = (m × 2l) B sin θ


= 10–3 × 10 × 10–2 × 4π × 10–3 × sin 30° N m
W E
= 2π × 10–7 N m.
S I
430. 2 = 2π
MH
423.
I
1 = 2π
M(F − H)
N S
2 F−H
Dividing, =
Fig. 146 1 H
F−H
1 MBH or 4= or 4H = F – H
424. ν= H
2π I
H 1
or 3H = F or = .
1 M(BH + B) F 3
ν′ =
2π I µ 0 2M × 10
1 M(BH − B) 431. B1 =
ν″ = 4 π (102 − l2 )2
2π I µ 2M × 20
FG ν′ IJ 2
BH + B
B2 = 0
4 π (202 − l2 )2
H νK =
BH B1 10 (202 − l2 )2
14 × 14 BH + B = 2 2 2
×
= B2 (10 − l ) 20
10 × 10 BH
125 (202 − l2 )2 (202 − l2 )2
or 1.96 BH = BH + B or 0.96 BH = B or = or 25 =
10 2 (102 − l2 )2 (102 − l2 )2
FG ν″ IJ 2
BH − B
Now
H νK =
BH or 5=
400 − l2
100 − l2
or 500 – 5l2 = 400 – l2
(ν″ )2 BH − 0.96 BH 100
or = or (ν″)2 = 4 or ν″ = 2. or 100 = 4l2 or l2 = = 25 or l = 5 cm
100 BH 4
425. Clearly, BH = BV Length = 2l = 10 cm.
∴ tan δ = 1 or δ = 45°.
T′ 2 + T 2 3 + Note that there is no need to convert cm
426. =
T′ 2 − T 2 2 into m.
T′25
or =or T′2 = 5 T2 or T′ = 5 T. 432. No effect on pole strength. However, length is halved.
2
T 1
tan 45° 2.4 × 10−5
427. tan δ′ = = 2. 433. B= Wb m–2 = 1.2 Wb m–2
cos 45° 0.2 × 10−4
tan δ tan δ B 1.2
428. tan θ1 = or cos α = ...(1) µ= = = 7.5 × 10–4 N A–2
cos α tan θ1 H 1600
tan δ µ 7.5 × 10 −4
Again, tan θ2 = µr = = = 596.6
cos (90 − α) µ 0 4 π × 10−7
tan δ Now χ = µr – 1 = 596.6 – 1 = 595.6.
or sin α = ...(2)
tan θ 2 3
434. Ethermal = kT1
(2) 2
gives
(1) Emagnetic = MB
tan θ1 Ethermal 3kT1
tan α = . = .
tan θ 2 Emagnetic 2MB
312 COMPREHENSIVE OBJECTIVE PHYSICS

22 2 × 6.75 2 × 6.75
435. M = 24 × 0.75 × × 0.035 × 0.035 A m2 446. 5 × 10–5 = 10–7 × or r3 =
7 r3 500
= 6.93 × 10–2 A m2. = 27 × 10–3 m
FG 1 IJ or r = 3 × 10–1 m = 0.3 m = 30 cm.
436. W = MB(1 – cos 60°) = MB 1 −
H 2 K µ 0 6 MM′ 6 × 10 × 10
MB 447. F = 4 = 10–7 N = 0.6 N.
= or MB = 2 W 4π r 0.1 × 0.1 × 0.1 × 0.1
2
1
3 448. T ∝
τ = MB sin 60° = (2 W) = 3 W. M
2
When M is increased by a factor of 4, T is halved.
437. BH = B cos δ
1
BH 0.50 1 449. B ∝
or B= = = oersted. r3
cos δ cos 30° 3
When r is doubled, B is reduced by a factor of 8.
60
438. T = s=2s 450. I is increased by a factor of 2. So, T0 is increased by a
30
factor of 2 .
When B is doubled, T is reduced by a factor of 2.
2 1 m FG IJ FG l IJ 2
I
T′ =
2
s= 2 s. 451. I′ =
12 2 H K H 2K or I′ =
8

T′ 1 M M I′ I×2
439. = × = 2 or T′ = 2T = 2 × 1.5 s = 3 s. Also, M′ = or T′ = 2π = 2π
T M/4 1 2 M′ B 8 × MB
440. W = MB(1 – cos 180°) = 2 MB. 1 1
= T = 4 s = 2 s.
µ M µ M2 2 2
441. B = 0 31 or B= 0
4π r 4 π (2r)3 I
452. T = 2π
M1 M2 M1 1 . MB
Equating, = or =
r3 8 r3 M2 8 T1 B2 3
= = .
2M M x 3
x T2 B1 2
442. = 3 or 3 = 2 or = 21/3.
x3
y y y 1
453. B∝ .
443. Effect on magnetic dipole moment r3
m 2l 1 454. MB(1 – cos 90°) = n MB(1 – cos 60°)
2
M′ =
× = M
2 4 FG 1 IJ
Effect on moment of inertia
or 1 = n 1 −
H 2 K or n = 2.

mass l FG IJ 2
N 1 tan 60° 3
I′ =
4
×
2 H K =
1
I
455. =
N 2 tan 45°
=
1
.

12 16
456. 3 = 1 × tan θ or tan θ = 3 or θ = 60°.
Effect on time period
457. B2 = BH2 + BV2 = B02 + B02 = 2B02 or B = 2 B0.
1
I×4 µ 0 2M
16 T
T′ = 2π = . 458. = tan θ or d3 tan θ = constant.
MB 2 4 π d3
N 4 × 103 × 12 × 10−2 ∴ r3 tan 60° = 3r3 tan θ or 3 = 3 tan θ
444. I = 4 × 103 or I = A = 8 A.
l 60 1
445. Steel or an alloy Alnico (Al + Ni + Co) is used for mak- or tan θ = or θ = 30°.
3
ing permanent magnets. Hysteresis loss is high. More-
459. W = – MB(cos θ2 – cos θ1)
over, coercivity is high and retentivity is low.
= – 4 × 5 × 10–4 [cos 45° – cos 30°]
Soft iron is used for electromagnets. Retentivity is
high. Coercivity is low. = – 20 × 10–4 [0.707 – 0.866] = 3.18 × 10–4 J.
MAGNETIC EFFECTS OF CURRENT AND MAGNETISM 313

tan δ 467. All the magnetic moment vectors can be represented


460. tan δ′ =
cos θ both in magnitude and direction by the three sides of
a triangle taken in the same order.
3 3
tan δ = cos θ tan δ′ = cos 30° tan 60° = × 3= 468. τ = (m × 2l) B sin θ
2 2
= 48 × 25 × 10–2 × 0.15 × sin 30° N m = 0.9 N m.
FG 3 IJ .
δ = tan–1
H 2K + F = BIl sin θ
F
3I × 3 B=
461. T′ = 2π = 3T0. Il sin θ
MB
Unit of B is N A–1 m–1.
1 M AB 1 MB B
462. =2×
2π I 2π I I
469. T = 2π
MH
Clearly M A = 4 MB or MA = 4 MB.
463. τ = BiA T1 M2 4M
= = =2
T2 M1 M
1 l2
A= l l2 − T1 4
2 4 T2 = = s=2s
2 2
1 3l 470. Magnetic induction at point A
A= l
2 2 due to magnet at B (axial point) S D
N
C
2 µ 2M
3l B1 = 0 3 (along BA)
A= 4π d
4
Magnetic induction at point A B1 B2
3 l2 due to magnet at D (equatorial
S
∴ τ = Bi A
B N
4 point) d

F I
1
µ0 M Fig. 148
4τ B2 = (along AB)
l=G J
2
or
H 3 BiK
4 π d3
µ0 M
F I Resultant, B = B1 – B2 =
1
τ 4 π d3
GH J
2

3 BiK
or l=2
471. Drawing loops from magnetic N pole towards S, one
464. I ∝ tan θ or I ∝ tan 45° finds that only OA somewhere the loop direction falls
along earth’s NS line and opposed to it in direction.
I
∝ tan θ 472. Magnetic moments are equal in magnitude and per-
3
pendicular to each other.
tan θ 1 1 473. B2 = BH2 + BV2 = (4.5 × 10–5)2 + (6 × 10–5)2
= or tan θ = or θ = 30°
tan 45° 3 3
= 4.52 + 62 × 10–5 T = 20.25 + 36 × 10–5 T
Decrease = 45° – 30° = 15°.
F
60°
= 56.25 × 10–5 T = 7.5 × 10–5 T.
465. H = F tan 60°
H= 3F H mass × (length)2
474. Moment of inertia =
12
F 1
= . When magnet is divided into two equal halves, mass
H 3
Fig. 147 is reduced by a factor of 2 and length is also reduced
by a factor of 2.
M1 d13 40 × 40 × 40 64
466. = = = . 1
M2 d23 50 × 50 × 50 125 So, new moment of inertia is th of the initial mo-
8
ment of inertia.
314 COMPREHENSIVE OBJECTIVE PHYSICS

Also, magnetic moment = pole strength × length


Pole strength is unchanged and the length is halved.
+ l Compass needle is free to rotate in a
So, new magnetic moment is one-half of the initial horizontal plane and points along the
magnetic meridian.
magnetic moment.
l When the compass needle is pointing
I′ I/8 T T along the geographical meridian, it ex-
Now T′ = 2 π = 2π M
= = periences a torque due to the horizon-
M′ B 4 2
B tal component of earth’s magnetic
2
field.
T′ 1 l The angle between the geographic and
∴ =
T 2 magnetic meridians is called angle of
475. F = H tan 45° declination.

µ 0NI 2rH 477. Note that the magnet is cut along length and not
=H or I =
2r µ0 N perpendicular to length.
M
2 × 15 × 10 −2 × 3 × 10 −5 Magnetic moment of each part =
= A = 0.29 A. 3
4 π × 10 −7 × 25
I
Moment of inertia of each part =
τ 3
476. τ = MB sin θ or sin θ =
MB Magnetic moment of given combination

1.2 × 10−3 M
= = 3× =M
= 0.5 or θ = 30°. 3
60 × 40 × 10−6
I
Moment of inertia of combination = 3 × =I
3
I
T′ = 2π =T=2s
MB

KNOWLEDGE PLUS
l A very long straight wire carries a current I. At the instant when a charge + Q at point
y

P has velocity v , as shown, the force on the charge is :
(a) along oy (b) along ox (c) opposite to ox (d) opposite to oy.
Q
x
Sol. The magnetic field due to the wire is along negative z axis. The corresponding P ® O

v
→ → → → →
unit vector is – k . For v , the unit vector is i . Using F = Q ( v × B) , we find F is in
I
the direction of i × (− k) or k × i or j i.e. in the positive direction of y-axis.
So, (a) is the right choice.
Fig. 149

l P, Q and R are long straight wires in air, carrying currents as shown in Fig. 150. The force
P Q R
on Q is directed
(a) to the left (b) to the right
(c) perpendicular to the plane of the diagram (d) along the current in Q. 20 A 40 A 60 A
[National Standard Exam. in Physics 2005]
Sol. The magnetic fields due to both the current-carrying wires P and R are directed into
the plane of the figure at Q and exert a force on Q to the left.
So, (a) is the right choice. Fig. 150
MAGNETIC EFFECTS OF CURRENT AND MAGNETISM 315

SET III MCQs


with
More than one correct alternative

Average time allowed per question is 50 seconds.

478. A microammeter has a resistance of 100 Ω and a full 482. A proton moving with a constant velocity passes
scale range of 50 µ A. It can be used as a voltmeter or as through a region of space without any change in its
a higher range ammeter provided a resistance is added velocity. If E and B represent the electric and magnetic
to it. Pick the current range and the resistance combi- fields respectively, this region of space may have
nations.
(a) E = 0, B = 0 (b) E = 0, B ≠ 0
(a) 50 V, 10 k Ω in series (b) 10 V, 200 k Ω in series
(c) 5 mA, 1 Ω in parallel (d) 10 mA, 1 Ω in parallel.
(c) E ≠ 0, B = 0 (d) E ≠ 0, B ≠ 0. [IIT 1985]
[National Standard Exam. in Physics 2002] 483. A particle of charge + q and mass m moving under the
479. A proton (mass m and charge + e) and an alpha parti- influence of an uniform electric field E i and a uniform
cle (mass 4m and charge + 2e) are projected with same magnetic field B k follows a trajectory from P to Q as
kinetic energy at right angles to a uniform magnetic shown in Fig. 151. The velocities at P and Q are v i
field. Which one of the following statements will be in- and – 2v j respectively. Which of the following
correct ? statement(s) is/are correct ?
(a) The alpha particle will be bent in a circular path with a
Y
smaller radius than that of the proton.
(b) The radius of the path of the alpha particle will be greater
than that of the proton.
(c) The alpha particle and the proton will be bent in a circu- ^
P v ^i Ei
lar path with the same radius.
(d) The alpha particle and the proton will go through the a
field in a straight line. [CPMT 1993] Q
480. If a charged particle goes unaccelerated in a region con- ^ O 2a X
taining electric and magnetic fields, Bk
^
– 2v j
→ →
(a) E must be perpendicular to B
→ →
(b) v must be perpendicular to E Z
→ →
(c) v must be perpendicular to B Fig. 151
(d) E must be equal to vB.
3 mv2
481. The magnetic field at the origin, due to a current ele- (a) E =
→ → 4 qa
ment i dl placed at a position r , is
LM
3 mv3 OP
(a)
→ →
µ 0i dl × r

µ i r × dl
(b) − 0
→ (b) Rate of work done by the electric field at P is
4 MN
a PQ
4π r3 4π r3 (c) Rate of work done by the electric field at P is zero
→ → → → (d) Rate of work done by both the fields at Q is zero.
µ i r × dl µ i dl × r
(c) 0 (d) − 0 . [IIT 1991]
4π r 3 4π r3
316 COMPREHENSIVE OBJECTIVE PHYSICS

484. Which of the following pairs has quantities of the same axis oscillates with period T. If the polarities of the
dimensions ? magnet are reversed, the period of oscillation of the nee-
(a) magnetic field B and magnetising field intensity H dle becomes 2T. If F is field due to the magnet at the
(b) magnetic field B and intensity of magnetisation I needle and H is horizontal component of earth’s mag-
(c) magnetising field intensity H and intensity of magneti- netic field, F/H is given by
sation I (a) 5/3 (b) 3/5
(d) longitudinal strain and magnetic susceptibility.
(c) 4 (d) 1/4.
485. A bar magnet NS is placed horizontally with its axis
[National Standard Exam. in Physics 1990]
north-south. A free magnetic needle P placed along its

Answers (Set III)


478. (b), (c) 479. (a), (b), (d) 480. (a), (b) 481. (c), (d) 482. (a), (b), (d) 483. (a), (b), (d) 484. (c), (d) 485. (a), (b).

Solutions (Set III)


10 V (d) At Q, the forces due to both the fields are perpen-
478. (b) = 50 µA gives full scale deflection. dicular to the direction of motion of the charge. Con-
200 kΩ
(c) 5mA through 1 Ω gives 5 mV drop, which across 100 Ω sequently, the work done by both the fields is zero.
micrommeter gives 50 µA. 484. (c) Both are measured in A m–1.
mv2 mv p 2m Ek (d) Both are dimensionless.
479. Bqv = or Bq = = or Bq = 485. The effective fields at the needle are the sum F + H for
r r r r
the first case (stronger field) and either F – H or H – F
2m Ek m in the second case. The data do not clearly settle the
or r= or r ∝
Bq q choice. For this reason, both (a) and (b) are correct.
rα 4 1 I
= × = 1. (a) T = 2π
rp 2 1 M (F + H)
→ → → I
480. q E = q( v × B) 2T = 2π
M (F − H)
→ → → →
Clearly, E ⊥ B , v ⊥ E .
1 F−H 1 F−H
481. Note that the position of origin with respect to the cur- = or =
→ 2 F+H 4 F+H
rent element is ‘– r ’.
or F + H = 4F – 4H
482. (a) If both E and B are zero, then no force exists.
F 5
(b) If proton is moving parallel to B, then no force 5H = 3F or = .
exists. H 3
(d) If eE = Bev, then no force exists. I
(b) T = 2π
483. (a) Force due to electric field = qE ; Displacement is M (F + H)
2a. Work done = qE × 2a or 2qaE. This work done is
I
3 2T = 2π
equal to increase in kinetic energy, mv2. M (H − F)
2
3 3 mv2 1 H−F 1 H−F
Equating, we get 2qaE = mv2 or E = . Dividing, = or =
2 4 qa 2 F+H 4 F+H
(b) Rate of work done by electric field at P
F 3
F 3 mv I v = 3 . mv
= force × velocity at P = qEv = q G
2 3
or F + H = 4H – 4F or 5F = 3H or = .
H 5
H 4 qa JK 4 a .
MAGNETIC EFFECTS OF CURRENT AND MAGNETISM 317

MCQs
based on
SET IV

TYPICAL NUMERICAL BANK


(Exclusively for Engineering Entrance Tests)

Average time allowed per question is 60 seconds.

486. Consider two straight long parallel conductors spaced 491. The magnetic potential at a point distant 10 cm from
50 cm apart and carrying oppositely directed currents ; the middle point of a magnetic dipole on a line inclined
the first 20 A and the second, 24 A. The observation at an angle of 60° with the axis is 3 emu. Then the
point is separated from the first conductor by a distance magnetic moment of the magnet is
of 40 cm and from the second by 30 cm. The magnetic (a) 300 abampere cm2 (b) 600 abampere cm2
field induction at the observation point is [Given : 3.56 (c) 30 abampere cm 2
(d) 60 abampere cm2.
≈ 1.88] 492. An electron revolves in a circular orbit in a plane per-
(a) 188 × 10 –7 T (b) 1.88 × 10 –7 T pendicular to a uniform magnetic field of induction
1.88 6 × 10–5 weber per metre2. Then the time period of revo-
(c) 2 × 1.88 T (d) T.
2 lution is
487. In the previous question, the magnetic field strength at (a) 5.9 × 10 –5 s (b) 5.9 × 10 –7 s
the observation point is nearly (c) 5.9 × 10 s
–9
(d) 5.9 × 10 –11 s.
(a) 0.15 A m–1 (b) 1.5 A m–1 493. A proton of mass m and charge q is accelerated by a
(c) 15 A m–1 (d) 150 A m–1. potential difference V in a perpendicular magnetic field
488. A horizontal rod of mass 10 g and length 10 cm is placed B occupying space d. The sine of deviation of proton
on a smooth plane inclined at an angle of 60° with the from initial direction is
horizontal, with the length of the rod parallel to the q q
edge of the inclined plane. A uniform magnetic field of (a) Bd (b) Bd
2 mV
2 mV
induction B is applied vertically downwards. If the cur-
rent through the rod is 1.73 A, the value of B for which q 2q
(c) Bd (d) Bd .
the rod remains stationary on the inclined plane is mV mV
[Given : g = 10 m s–2] 494. A uniform magnetic field with a slit system as shown
1 in Fig. 152 is to be used as a momentum filter for high
(a) 1.73 T (b) T
1.73
(c) 1 T (d) 0.5 T.

489. The ratio of the magnetic field at the centre of a circu- B
lar current-carrying wire and the magnetic field at the
centre of a square formed with the same length of wire
when same current flows through it is
(a) π : 8 2 (b) π2 : 8 2
(c) π : 4 (d) 2π : 9.
490. A current of 1 A is flowing in the sides of an equilateral
triangle of side 4.5 × 10 –2 m. Magnetic field at centroid
is Source Detector
(a) 4 × 10 –5 T (b) 8 × 10 –5 T
(c) 12 × 10 –5 T (d) 16 × 10 –5 T. Fig. 152
318 COMPREHENSIVE OBJECTIVE PHYSICS

energy charged particles. With a field B Tesla, it is 501. Three long, parallel, straight wires X, Y and Z are
found that the filter trasmits α-particles each of energy placed in the same plane in a vacuum as shown in Fig.
5 MeV. The magnetic field is increased to 2 B tesla and 153. Given that the force per unit length between two
deuterons are passed into the filter. The energy of each long, parallel, straight wires placed 10 cm apart, each
deutron transmitted by filter is carrying a current of 1 A, is 2 × 10–6 N m–1, what is the
(a) 14 MeV (b) 10 MeV net force per unit length acting on Z ?
(c) 28 MeV (d) 2 MeV. 1A X
495. A circular current-carrying coil has a radius R. The
distance from the centre of the coil, on the axis, where 10 cm
1
the magnetic induction will be th of its value at the 2A Y
8
centre of the coil, is
10 cm
R
(a) 3R (b) 1A Z
3
(c) R2 (d) R8.
Fig. 153
496. A bar magnet suspended by a horse’s hair lies in the
magnetic meridian when there is no twist in the hair. (a) 3.0 × 10 –6
Nm–1
(b) 3.5 × 10 –6 N m–1
On turning the upper end of the hair through 150° from (c) 4.0 × 10 –6 N m–1 (d) 4.5 × 10 –6 N m–1
the meridian, the magnet is deflected through 30° from (e) 5.0 × 10 –6
Nm .
–1

the meridian. Then the angle through which the upper


502. A deuteron of kinetic energy 50 KeV is describing a cir-
end of the hair has to be twisted to deflect the magnet
cular orbit of radius 0.5 metre in a plane perpendicu-
through 90° from meridian is →
lar to magnetic field B . The kinetic energy of the pro-
(a) 450° (b) 360°
ton that describes a circular orbit of radius 0.5 metre
(c) 330° (d) 150°. →
in the same plane with the same B is
497. The field normal to the plane of a wire of n turns and
(a) 25 KeV (b) 50 KeV
radius r which carries a current i is measured on the
axis of the coil at a small distance h from the centre of (c) 200 KeV (d) 100 KeV.
the coil. This is smaller than the field at the centre by 503. A current I ampere flows in a circular arc of wire which
the fraction subtends an angle (3π/2) radian at its centre, whose
3 h2 2 h2 radius is R. The magnetic induction B at the centre is
(a) (b)
2 r 2 3 r2 (a) µ0I/R (b) µ0I/2R
3 r2 2 r2 (c) 2µ0I/R (d) 3µ0I/8R.
(c) (d) .
2 h 2 3 h 2 504. Two circular coil x and y having equal number of turns
498. The magnetic moment of a current carrying loop is and carrying equal currents in the same sense and sub-
2.1 × 10 –25 A m2. The magnetic field at a point on its tend same solid angle at point O. If the smaller coil x is
axis at a distance of 1 Å is midway between O and y, then if we represent the mag-
(a) 4.2 × 10 –2 Wb m–2 (b) 4.2 × 10 –3 Wb m –2 netic induction due to the bigger coil y at O as By and
(c) 4.2 × 10 Wb m–4 –2
(d) 4.2 × 10 –5 Wb m–2.
that due to smaller coil x at O as Bx , then
499. A proton of mass 1.67 × 10 –27 kg and charge 1.6 × 10–19 d
C is projected with a speed of 2 × 106 m s–1 at an angle
d/2
of 60° to the X-axis. If a uniform magnetic field of 0.104
T is applied along Y-axis, then the path of proton is 2r
r
(a) a circle of radius 0.2 m and time period π × 10 –7 s. O
(b) a circle of radius 0.1 m and time period 2π × 10 –7 s.
(c) a helix of radius 0.1 m and time period 2π × 10 –7 s.
x
(d) a helix of radius 0.2 m and time period 4π × 10 –7 s.
y
[IIT 1995]
500. A 2 MeV proton is moving perpendicular to a uniform Fig. 154
magnetic field of 2.5 tesla. The force on the proton is 1
(a) 2.5 × 10 –10 N (b) 8 × 10 –11 N (a) By/Bx = 1/4 (b) By/Bx =
2
(c) 2.5 × 10 –11 N (d) 8 × 10 –12 N. (c) By/Bx = 1 (d) By/Bx = 2.
MAGNETIC EFFECTS OF CURRENT AND MAGNETISM 319

505. A circular wire ABC 5µ 0 I µ 0I


(a) (b)
and a straight conduc- × × × × × × × 2 2 πr 2 πr
tor ADC are carrying I B
8µ 0 I
× × C× × 3µ 0I
current I and are kept (c) (d) .
× × × I × × × 2 πr 2 πr
in the magnetic field D
B. Then, considering × θ × × × × 508. An infinitely long conductor PQR is bent to form a right
points A and C
A angle as shown in Fig. 158. A current i flows through
× × × × × × ×
PQR. The magnetic field due to this current at the point
(a) force as per ABC is
more than ADC Fig. 155 M is H1. Now another infinitely long straight conduc-
(b) force as per ABC is tor QS is connected at Q so that the current is i/2 in
less than ADC QR as well as QS, the current in PQ remaining un-
(c) force as per ABC is equal to that as per ADC changed. The magnetic field at M is now H2. The ratio
(d) any of (a) or (b) or (c). H1/H2 is given by
506. Let B and u denote magnetic field induction and en-
ergy density at midpoint of a long solenoid carrying a M
current I. The graph between u and I will be

Y Y

90°
–∞ i +∞
P Q S
90°
u u

R
O I X O I X
–∞
(a) (b)
Fig. 158
Y Y
(a) 1/2 (b) 1
(c) 2/3 (d) 2.
[IIT Screening 2000]
509. A metallic block carrying current I is subjected to a
u →
u uniform magnetic induction B as shown in Fig. 159.

The moving charges experience a force F given by .....
which results in the lowering of the potential of the face
O I X O I X
....... Assume the speed of the carriers to be v.
(c) (d)

Fig. 156

507. Magnetic induction at point P from shown current-car-


rying long conductors is given by
I P
45° r

Fig. 159
Fig. 157
320 COMPREHENSIVE OBJECTIVE PHYSICS

(a) evB k , ABCD (b) evB k , EFGH (a) 0.50 gauss (b) 0.25 gauss
(c) 0.05 gauss (d) 0.005 gauss.
(c) – evB k , ABCD (d) – evB k , EFGH.
515. A wire ABCDEF (with each side of length L) bent as
[IIT 1996]
shown in Fig. 161 and carrying a current I is placed in
510. Two particles, each of mass m and charge q, are a uniform magnetic induction (field) B parallel to posi-
attached to the two ends of a light rigid rod of length tive Y direction. The force experienced by the wire is
2 R. The rod is rotated at constant angular speed about
a perpendicular axis passing through its centre. The Z
D
ratio of the magnitudes of the magnetic moment of the
system and its angular momentum about the centre of C I
the rod is E
q q F
(a) (b)
2m m Y
2q q O
(c) (d) . [IIT 1998]
m πm A
B
511. A long straight wire along the z-axis carries a current I
in the negative z direction. The magnetic vector field X

B at a point having coordinates (x, y) in the z = 0 plane Fig. 161
is
(a) BIL in positive Y-direction
µ I ( yi − xj) µ I ( xi + yj) (b) BIL in positive Z-direction
(a) 0 (b) 0
2π ( x 2 + y2 ) 2π ( x 2 + y2 ) (c) 3 BIL (d) zero. [IIT 1990]
µ I ( xj − yi) µ I ( xi − yj) 516. A non-planar loop of con-
(c) 0 (d) 0 .
2π ( x 2 + y2 ) 2π ( x 2 + y2 ) ducting wire carrying a z
[IIT Screening 2002] current I is placed as y
shown in Fig. 162. Each of I
512. Fig. 160 shows a cur- I the straight sections of the
rent-carrying wire.
loop is of length 2a. The
The magnetic field in-
magnetic field due to this x
duction at O is r b
loop at the point P (a, 0, a) 2a
LM OP
O a
µ 0I 3 points in the direction Fig. 162
(a) π+1
N Q
4 πr 2
r
1 1
(− j + k) (− j + k + i)
µ 0I L 3 O d (a) (b)
c
2πr MN 2
(b) π + 1P 2 3
Q Fig. 160 1
(i + j + k)
1
(i + k) .
µ 0I L 3 O
(c) (d)
πr MN 2 PQ
(c) 3 2
[IIT Screening 2001]
µ I L 3 O 517. A particle of mass m and charge q moves with a con-
(d) 0 M1 − πP .
2π r N 2 Q stant velocity v along the positive x direction. It enters
513. Two identical magnetic dipoles of magnetic moments a region containing a uniform magnetic field B directed
1.0 A m2 each, placed at a separation of 2 m with their along the negative z direction, extending from x = a to
axis perpendicular to each other. The resultant mag- x = b.The minimum value of v required so that the par-
netic field at a point midway between the dipoles is ticle can just enter the region x > b is
(a) qbB/m (b) q(b – a)B/m
(a) 5 × 10–7 T (b) 5 × 10–7 T
(c) qaB/m (d) q(b + a)B/2m.
(c) 10–7 T (d) None of these.
[IIT Screening 2002]
[Roorkee 1995]
518. Surface charge density on a
514. A uniform magnetic needle is suspended from its centre ν
ring of radius a and width d
by a thread. Its upper end is now loaded with a mass is σ. If it rotates with fre-
of 50 milligram when the needle becomes horizontal. If quency ν about its own axis, σ
the strength of each pole is 98.1 ab-ampere × cm and the magnetic induction at cen- a
g is 981 cm/s2, then the vertical component of the earth’s tre is
magnetic induction is Fig. 163
MAGNETIC EFFECTS OF CURRENT AND MAGNETISM 321

(a) πµ0νσd (b) µ0νσd


π
(c) 2πµ0νσd (d) νσd .
2µ 0 K

519. A long magnet is placed vertically on a table. A neutral


point is located at a distance of 20 cm from it. If
horizontal component of earth’s magnetic field is
5 × 10 –5 Wb m–2, then the pole strength of the magnet
is
(a) 20 A m (b) 40 A m S1 S2
(c) 60 A m (d) 80 A m.
Fig. 164
520. Two identical solenoids S1 and S2 are suspended
(a) One turn in a clockwise direction, and the other in an
coaxially and symmetrically by four long thin wires anticlockwise direction
so that they may swing freely, as shown in Fig. 164. (b) They both turn in a clockwise direction
When the switch K is closed, what happens to the (c) They move away from each other
solenoids ? (d) They move towards each other.

Answers (Set IV)


486. (a) 487. (c) 488. (c) 489. (b) 490. (a) 491. (b) 492. (b) 493. (b)
494. (b) 495. (a) 496. (c) 497. (a) 498. (a) 499. (c) 500. (d) 501. (a)
502. (d) 503. (d) 504. (b) 505. (c) 506. (c) 507. (b) 508. (c) 509. (a)
510. (a) 511. (a) 512. (a) 513. (b) 514. (b) 515. (b) 516. (d) 517. (b)
518. (a) 519. (a) 520. (d).

Solutions (Set IV)


µ0 × 20 20 A θ
486. B1 = c os
2π × 0.40 BIl
40
cm
µ 0 × 24 in
θ θ
B2 = s BIl
mg
50 cm

2π × 0.30
90°
µ FG 20 IJ + FG 24 IJ
2 2 θ mg
B= 0
2π H 0.4 K H 0.3 K 30
cm
Fig. 166
4π × 10−7 24 A
= 2500 + 6400 µ 0I µ 0 πI µ 0 πI
2π Fig. 165 489. B = = =
2r 2πr l
= 2 × 10–7 8900
µ 0I
= 2 × 94.3 × 10–7 = 188.6 × 10–7 T ≈ 188 × 10–7 T. Again, B′ = 4 ×
l FG IJ
× 2 sin 45°

487. H =
188 × 10 −7
A m–1 = 15 A m–1.

8 H K
4 π × 10 −7
8µ 0I 1 8 2 µ 0I
488. For equilibrium, = ×2× =
πl 2 πl
mg
BIl cos θ = mg sin θ or B = tan θ B µ 0 πI πl π2
Il Now = × = .
B′ l 8 2 µ 0I 8 2
10 × 10 −3 × 10 × 1.73
= T = 1 T.
1.73 × 10 × 10 −2
322 COMPREHENSIVE OBJECTIVE PHYSICS

4.5 × 10−2 Ed 1 × 1 4 × 2 × 2
490. tan 60° = Now = × =2
2r Eα 1 2×2×2
∴ Ed = 2Eα = 2 × 5 MeV = 10 MeV.
µ 0IR 2 1 µ 0I
495. =
2(R 2 + x 2 )3 / 2 8 2R
2(R 2 + x2 )3 / 2
16 R =
R2
60° r
or 16 R3
= + 2(R2 x2)3/2 or 8R3 = (R2 + x2)3/2
30°
or 2R = (R2 + x2)1/2 or 4R2 = R2 + x2
4.5 × 10–2 m or x = 3 R.
2
496. MB sin 30° = k(150° – 30°) = 120 k ...(1)
Fig. 167 Again, MB sin 90° = k(θ – 90°) ...(2)
4.5 × 10−2 (2)
or r= gives
2 tan 60° (1)
µ0 × 1 × 2 tan 60° K(θ − 90° ) MB × 2
B=3× [2 sin 60°] =
4 π × 4.5 × 10 −2 120 K MB × 1
3 × 10−7 × 200 3 or 240 K = K(θ – 90°) or θ = 240 + 90 = 330°.
or B= ×2× 3× = 4 × 10–5 T.
4.5 2 µ 0NIr 2 µ 0NI
497. B= or B0 =
M cos θ 2(r 2 + h2 )3 / 2 2r
491. V=
r2 µ 0 NI µ 0 NIr 2
B0 – B = −
M cos 60° 2r 2(r 2 + h2 )3 / 2
3=
10 × 10
abampere cm2
=
LM
µ 0NI 1
− 2
r2 OP
or M = 600 abampere cm2. 2MNr ( r + h2 ) 3 / 2 PQ
492. T =
2πm
=
2 × 3.14 × 9.1 × 10 −31 LM OP
µ NI M 1 PP
Bq 6 × 10 −5 × 1.6 × 10 −19 2
= 0
M −
r

MM r FGH 1 + hr IJK PPP


=
2 × 3.14 × 9.1
× 10–7 s = 5.9 × 10–7 s. 2 Mr 2 3/ 2
3
6 × 1.6
mv2 mv N 2
Q
µ NI L F 3 h + ...I OP
493. Bqv = or r =
M
2
1 − G1 −
H 2 r JK PQ
r Bq 0
=
2r MN
θ r
2
m 2qV 1 2mV
or r= = d
Bq m B q µ0 NI 3 h2 3µ0 NIh2
= =
d θ 2r 2 R 2 4r 3
Now sin θ =
r d
Fractional decrease in magnetic field
d B0 − B 3µ0NIh2 × 2r
or r= 3 h2
sin θ = = = .
B0 4r 3 × µ0 NI 2 r2
d 1 2mV µ 0 2M
∴ = Fig. 168
sin θ B q 498. B =
4 π d3
q 10−7 × 2 × 2.1 × 10−25
or sin θ = Bd . B= Wb m–2 = 4.2 × 10–2 Wb m–2.
2mV (10−10 )3
mv2 499. v⊥ = 2 × 106 × cos 60° = 106 m s–1
494. Bqv =
r v|| = 2 × 106 × sin 60° = 3 × 106 m s–1
mv p 2mEk B2q2 r2
Bq = = = or Ek = mv⊥
r r r 2m r=
Bq
MAGNETIC EFFECTS OF CURRENT AND MAGNETISM 323

1.67 × 10 −27 × 10 6 Y
B2
= m 506. u=
0.104 × 1.6 × 10 −19 2µ 0
= 0.1 m v µ 0 NI
But B=
2πm v|| 2r
T=
Bq B
1 µ 02N 2I2
∴ u=
2π × 1.67 × 10−27 60° 2µ 0 4 r2
= s ∴ u∝I 2
0.104 × 1.6 × 10−19 O v⊥ X
–7
Clearly, the graph between u and I will be a parabola
= 2π × 10 s. Fig. 169 which is symmetric about the u-axis and which passes
500. Fm = Bqv through the origin.
1 2qV d r
But mv2 = qV or v = 507. sin 45° = or d = r sin 45° =
2 m r 2
2qV There will be no magnetic field due to ab.
∴ Fm = Bq
m Magnetic field due to ac,

2 × 1.6 × 10 −19 × 2 × 106


= 2.5 × 1.6 × 10–19 N
1.6 × 10−27
= 2.5 × 1.6 × 2 × 10–12 N = 8 × 10–12 N. d
45°
501. The upward force on Z due to current in X, b a
I r
1 45°
FXZ = (2 × 10 –6) = 1 × 10 –6 N m–1 (↑)
2
I
The downward force on Z due to current in Y,
FYZ = 2(2 × 10 –6) = 4 × 10 –6 N m–1 (↓) c
Net downward force on Z due to currents in X
and Y = 3 × 10 –6 N m–1 (↓).
LMNote : F = µ0I1I2 ∴ F ∝ I and F ∝
1 OP
N l 2 πd d Q Fig. 170

B2q2 r 2 µ 0I µ 0I µ 0I
mv p 2 mE B= = =
502. r = = = or E = 2πd 2πr sin 45° 2 πr
Bq Bq Bq 2m
→ → →
Ep 1 2 508. H 1 = BPQ + BQR
= × =2
Ed 1 1 × 1 →
∴ Ep = 2Ed = 2 × 50 KeV = 100 KeV. But BQR = 0, because M lies on RQ.
→ →
µ I 3π 3µ I ∴ H1 = BPQ
503. B = 0 = 0 .
2R 2 × 2π 8R → → →
2 Again H 2 = BPQ + BQS
µ 0NIr
504. B = 1
2( r 2 + x 2 ) 3 / 2 But current in QS = × current in PQ
Fr d2 I 3/ 2 2

By (2 r)2
GH 2
+
4 JK ∴

BQS =
1→
BPQ
= × 2
Bx (4 r 2 + d2 )3 / 2 r2 → 3→
4 (4 r 2 + d2 )3 / 2 ∴ H2 = BPQ
4 1 2
= 3/ 2 = = .
4 (4 r 2 + d2 )3 / 2 8 2 3 H1 2
505. Whatever may be the shape of the curved conductor ∴ H2 = H or = .
2 1 H2 3
between A and C, force would be the same as on the
→ → →
straight conductor AC. 509. F = q( v × B)
If electrons are the charge carriers, then
324 COMPREHENSIVE OBJECTIVE PHYSICS

→ → → 1m
F = − e ( v × B) B2

F = − e (− vi × Bj ) S

F = Bevk S N
P B1
Since the force is directed along z-axis therefore the
1m N
electrons shall move towards ABCD. So, there will be
lowering of the potential of face ABCD.
Fig. 172
[Note that ABCD is perpendicular to x-axis.]
2q 2qω qω B= B12 + B22 = 5 × 10–7 T.
510. I= = 2qν = =
T 2π π 514. Torque due to weight = W × l
qω Deflecting torque due to magnetic field
M= × πR2 or M = qωR2
π = mBV (2l)
Again L = 2(mR2)ω
For equilibrium,
M qωR 2 q
Now = = . W
L 2(mR 2 )ω 2m mBV (2l) = W × l or BV =

2m
511. Magnetic field |B|
50 × 10−3 × 981
= gauss = 0.25 gauss.
µ 0I P 2 × 98.1
= →
2 π x2 + y 2 B 515. Following arguments shall decide the right choice :
O
Unit vector perpendicular (i) Current-carrying conductors FE and BA are par-
→ allel to magnetic field. So, they do not experience
to position vector OP is
any force.
yi − xj Fig. 171 (ii) Conductors ED, DC and CB are perpendicular to
2 2 the magnetic field. So, they experience forces, each
x +y
equal to BIl.
→ µ 0I
∴ B = ( yi − xj) . (iii) Forces on ED and CB cancel out.
2 π ( x 2 + y2 )
(iv) Applying right hand rule for the cross product of
512. Point O is on the axis of ab. So, ab would not contrib-
vectors or right hand screw rule, we find that the
ute any magnetic field at O. Again, dc is semi-infinite
force on DC is along Z-axis.
current-carrying wire.
516. Magnetic induction at (a, 0, a) due to loop in xy plane
µ 0I
∴ Bdc = is in + k direction. Due to loop in yz plane, the mag-
2πr netic field will be in + i direction. Due to both the
It is directed upwards.

1  
3 loops, the direction of B will be (i + k) .
Again, the circular wire between b and c is th por- 2
4
tion of complete circle. mv2 Bqr Bq(b − a)
517. Bqv = or v = or vmin. = .
3 µ 0I r m m
∴ Bbc = 518. Consider an element of radius
4 2r
It is directed upwards. x and thickness dx
Net magnetic field at O, B dq = (2π x dx) σ
µ 0I 3 µ 0I
+ =
µ I
= 0 1+
3π LM OP Corresponding current
dx
.
N Q
x
4 πr 4 2 r = dqν = 2π x σ νdx
4 πr 2
513. P is on axial line of first magnet. µ 0 [2π x σ νdx]
dB =
µ0 2 × 1 2x Fig. 173
∴ B1 = = 2 × 10–7 T dB = µ0 π σ νdx
4 π 13

z
r
P is on the equatorial line of the second magnet.
B= µ 0 π σ νdx
µ 1
B2 = 0 3 = 10–7 T r−d
4π 1
MAGNETIC EFFECTS OF CURRENT AND MAGNETISM 325

r 520. The polarity of magnetic fields are determined by the


B = µ0 π σ ν x Right-hand cork screw rule for each solenoid. Hence
r−d when switch K is closed, the attractive force causes
B = µ0 π σ ν [r – (r – d)] them to move towards each other.
B = µ0 π σ νd.
I1 I2
µ0 m N
519. BH =
4 π r3 K
r
m BH
5 × 10–5 = 10–7 ×
0.20 × 0.20
m = 500 × 0.20 × 0.20 A m
= 20 A m. Bm
+ – – +
Fig. 174 S1 S N S2

Fig. 175

KNOWLEDGE PLUS
l The magnetic field due to a straight conductor of uniform cross-section of radius a and carrying a steady current is
represented by:

B B B B
(a) (b) (c) (d)

a r a r a r a r

Fig. 176

[AIIMS 2004]
Sol. Magnetic field at a point outside the straight conductor is
1
B∝
(r > a)
r
Magnetic field at a point lying inside the conductor is
B ∝ r (r < a)
So, (a) is the right choice.

l An ammeter reads upto 1 ampere. Its internal resistance is 0.81 ohm. To increase the range to 10 A, the value of the
required shunt is
(a) 0.09 Ω (b) 0.03 Ω (c) 0.3 Ω (d) 0.9 Ω [AIEEE 2003]
GI g 0.81 × 1 0.81
Sol. S = = Ω= Ω = 0.09 Ω
I − Ig 10 − 1 9
So, (a) is the right choice.
326 COMPREHENSIVE OBJECTIVE PHYSICS

SELF-EVALUATION TEST I
Based on UNIT XIII

[Expected Questions for Forthcoming Examinations]

1. An electric charge does not experience a force due to a µ 0 2i µ0 i


(a) B = (b) B =
magnetic field 4π r 4π r
(a) if it is stationary µ0 π i µ 2π i
(c) B = (d) B = 0 .
(b) if it moves parallel to the magnetic field 4π r 4π r
(c) in both the above cases (a) and (b) 6. Fig. 178 shows a plane Y
(d) if it moves perpendicular to the field. OXY with axes OX and OY P
at right angles.
2. The resistance of a moving coil galvanomter is 15 Ω
and it gives full scale deflection of 0.01 A. It can be Which of the following cur-
Q
converted into a voltmeter for 5 V full scale deflection rents in a straight conduc-
O X
by connecting a resistor of tor will produce a magnetic
Fig. 178
(a) 485 Ω in parallel (b) 0.05 Ω in parallel field at O in the direction
(c) 485 Ω in series (d) 500 Ω in series. OX ?
[CMC Ludhiana 1999] (a) at P into the plane of the diagram
3. Four parallel conductors, W X (b) at P out of the plane of the diagram
carrying equal currents, (c) at Q into the plane of the diagram
pass vertically through the
(d) at Q out of the plane of the diagram
four corners of a square
WXYZ. In two conductors, O (e) at P parallel to OX.
the current is flowing into resultant 7. Suppose that a proton travelling in vacuum with veloc-
the page, and in the other magnetic ity v1 at 90° to a uniform magnetic field experience twice
field
two out of the page. the force (from this field) that an alpha particle experi-
In what directions must Z Y ences when it is travelling along the same path with
the current flow to produce Fig. 177 v1
a resultant magnetic field velocity v2. The ratio is
in the direction shown at v2
(a) 0.5 (b) 4
O, the centre of the square ?
(c) 2 (d) 1.
into the page out of the page
8. The restoring couple in the moving coil galvanometer
(a) W and X Y and Z
is due to
(b) W and Y X and Z
(a) current in the coil
(c) W and Z X and Y
(b) magnetic field of the magnet
(d) X and Z W and Y
(c) material of the coil
(e) Y and Z W and X.
(d) twist produced in the suspension wire.
[AMU 2002]
9. In Fig. 179, Q is a circu-
4. In order to increase the current sensitivity of a moving lar coil of wire carrying a Q
coil galvanometer, clockwise current I as
(a) the suspension wire should be made stiff shown. P is a long, straight
P I
(b) the area of the coil should be reduced wire carrying a current
(c) the magnetic field should be increased perpendicularly into the
(d) the number of turns in the coil should be reduced. plane of the paper through
the centre of the coil.
5. A current i ampere flows along an infinitely long straight Fig. 179
conductor. If r metre is the perpendicular distance of a Because of the current in
point from the lower end of the conductor, then the mag- P, each part of the wire Q
netic induction B is given by experiences
MAGNETIC EFFECTS OF CURRENT AND MAGNETISM 327

(a) a force into the plane of the paper 13. A charged particle is moved along a magnetic field line.
(b) a force out of the plane of the paper The magnetic force on the particle is
(c) a force towards P (a) along its velocity (b) opposite to its velocity
(d) a force away from P (c) perpendicular to its velocity
(e) no force in any direction. (d) zero. [CMC LDH 2000]
10. Two parallel conductors carry equal sinusoidal alter- 14. When a tangent galvanometer is properly set, then the
nating currents differing in phase by π rad. Which one
two magnetic fields acting on the horizontal plane are
of the following graphs shows how F, the mutual force
(a) earth’s horizontal component and the field due to cur-
of attraction, varies with time t ?
rent in the coil
(b) earth’s horizontal component and earth’s vertical com-
ponent
(c) earth’s field and the field due to compass needle
(d) earth’s vertical component and the field due to current
in the coil. [AMU 2001]
15. If an electron describes half a revolution in a circle of
radius r in a magnetic field B, the energy required by it
is
1
(a) zero (b) mv2
2
1
(c) mv2 (d) πr × Bev.
4
16. An electric field of 1500 V m–1 and a magnetic field of
0.4 Wb m–2 act on a moving electron to produce no field.
The speed of the electron is
1500 0.4
(a) m s–1 (b) m s–1
0.4 1500
1
(c) m s–1 (d) 1500 × 0.4 m s–1.
1500 × 0.4
17. A wire is placed parallel to the lines of force in a mag-
netic field and a current flows in the wire. Then
(a) the wire will experience a force along the direction of mag-
Fig. 180 netic field.
11. Which of the following particles will describe the small- (b) the wire will not experience any force.
est circle when projected with the same velocity perpen- (c) the wire will experience a torque.
dicular to a magnetic field ? (d) the wire will experience a perpendicular force.
(a) electron (b) proton 18. A straight section PQ of a circuit lies along the X-axis
a a
(c) He+
(d) Li . +
from x = – to x = and carries a steady current I.
2 2
12. Four copper wires of same The magnetic field due to the section PQ at a point
B
length and same cross-sec- X = + a will be
tional area are connected in (a) proportional to a (b) proportional to a2
the form of a square ABCD. a
1
On passing currents in them A
a a
C (c) proportional to (d) zero.
O a
as shown in Fig. 181, the mag-
a 19. A proton is projected horizontally eastward in a uni-
netic field at the centre O of the
square is form magnetic field which is horizontal and southward
(a) Zero D in direction. The proton will be deflected
µ I (a) upward (b) downward
(b) 4 . 0
2π a (c) southward (d) northward.
2µ 0 I µ 0I Fig. 181
(c) (d) . [AFMC 2002]
2πa 2 πa
328 COMPREHENSIVE OBJECTIVE PHYSICS

20. In Fig. 182, a straight conductor 24. In a coaxial, straight cable, the central conductor and
carrying current is shown. The di- the outer conductor carry equal currents in opposite
rection of the magnetic field at P directions. The magnetic field is zero
is P
(a) outside the cable
(a) normal to the plane of the paper, (b) inside the inner conductor
downwards
(c) inside the outer conductor
(b) normal to the plane of the paper,
upwards (d) in between the two conductors.
Fig. 182
(c) to the left of the plane of the pa- 25. An electron and proton of equal momentum enter a
per uniform magnetic field normal to the lines of force. If
(d) to the right of the plane of the paper. the radii of circular paths be re and rp respectivity, then
21. An electron is moving with a speed of 108 m s –1 r re m p
perpendicular to a uniform magnetic field of intensity (a) e = 1 (b) =
rp rp me
B. Suddenly intensity of the magnetic field is reduced
to B/2. What is the radius of the path if the original r mp r me
(c) e = (d) e = .
radius is r ? rp me rp mp
(a) No change (b) r/2
26. Fig. 184 represents an area A = 0.5 m2 situated in a
(c) 2r (d) r/9.
uniform field B = 2.0 T and making an angle of 60°
[MP PET 1993] with respect to the magnetic field. The value of the mag-
22. Of the following graphs, the one which shows the rela- netic flux through the area A would be equal to
tionship between the resistance R of a multirange mov-
ing coil voltmeter and its voltage range V is

B
60°

Fig. 184

3
(a) 2.0 Wb (b) Wb
2
(c) 3 Wb (d) 0.5 Wb.
[MP PMT 1999]
27. The sensitiveness of a moving coil galvanometer can
be increased by decreasing
(a) the number of turns in the coil
(b) the area of the coil
(c) the magnetic field
(d) the couple per unit twist of the suspension.
Fig. 183 [MP PMT 1996]
(a) a (b) b 28. A circular coil of wire carries a P
(c) c (d) d. current ; PQ is part of a very long
23. A vertical wire carries a current in upward direction. wire carrying a current and
An electron beam sent horizontally towards the wire passing close to the circular coil.
will be deflected If the directions of the currents
are those as shown in Fig. 185,
(a) towards right (b) towards left
what is the direction of the force
(c) upwards (d) downwards. Q
acting on PQ ?
[Haryana PMT 2001] Fig. 185
MAGNETIC EFFECTS OF CURRENT AND MAGNETISM 329

(a) parallel to PQ, towards P µ 0I


(c) , directed towards you
(b) parallel to PQ, towards Q 3a
(c) at right angles to PQ, to the right µ I
(d) 0 , directed into the plane of the paper.
(d) at right angles to PQ, to the left. 3a
29. The permanent magnetic moment of the atoms of a 36. In Q. 34, the net magnetic field at C due to the current-
material is not zero. The material carrying loop is
(a) must be paramagnetic (b) must be diamagnetic µ 0I
(a) zero (b)
(c) must be ferromagnetic (d) may be paramagnetic. a
µ 0I
30. The angle between geographical and magnetic merid- (c)
9a
ian is called
µ I F 3 − 1I , directed into the plane of the paper.
(a) angle of dip (b) angle of declination (d) 0
2a
GH π 3 JK
(c) latitude (d) longitude.
37. The permanent magnetic moment of the atoms of a ma-
[MP PMT 2000]
terial is zero. The material
31. Ferromagnetics owe their properties due to
(a) must be paramagnetic (b) must be diamagnetic
(a) filled inner subshells
(c) must be ferromagnetic (d) may be paramagnetic.
(b) vacant inner subshells
[Haryana PMT 2002]
(c) partially filled inner subshells
38. A battery of electro motive force E is connected in series
(d) all the sub-shell equally filled.
with a resistance R and a voltmeter. An ammeter is
32. A beam of charged particles is passing through a mag- connected in parallel with the battery.
netic field. The work done on the beam by the magnetic (a) Both ammeter and voltmeter are likely to be damaged
field (b) Neither the ammeter nor the voltmeter will be damaged
(a) is zero. (c) Only ammeter is likely to be damaged
(b) will depend on the velocity of the beam. (d) Only voltmeter is likely to be damaged.
(c) will depend on the deflection of the beam. [Karnataka CET 2001]
(d) will depend on the strength of the magnetic field.
39. Two magnets are held together in a vibration
33. A 10 turn circular coil of radius 2 cm carries a current
magnetometer and are allowed to oscillate in the earth’s
of 0.5 A. The magnetic induction at the centre of the magnetic field. With like poles together, 12 oscillations
coil is
per minute are made but for unlike poles together, only
(a) 1.57 × 10–6 T (b) 1.57 × 10–5 T
4 oscillations per minute are executed. The ratio of their
(c) 1.57 × 10–4 T (d) 1.57 T.
magnetic moments is
[CMC LDH 2001]
(a) 3 : 1 (b) 1 : 3
34. A wire loop carrying a
(c) 3 : 5 (d) 5 : 4. [MP PMT 1996]
current I is shown in
Fig. 186. The magnetic a 40. A bar magnet is oscillating in earth’s magnetic field
field induction at C due to with period T. What happens to its period and its mo-
120°

straight part is I I C tion if its mass is quadrupled ?


(a) Motion remains simple harmonic with the new period
3 µ 0I a
= 4T
(a) , directed into
2π a
(b) Motion remains simple harmonic with the new period
the plane of the paper
= T/4
µ 0I Fig. 186 (c) Motion does not remain simple harmonic and period is
(b) , directed into the
6a approximately constant.
plane of the paper
(d) Motion remains simple harmonic with the new period
µ I = 2T.
(c) 0 , directed towards you
6a
[CMC Vellore 2002]
µ 0I F 3 − 1I towards you.
(d)
2a
GH π 3 JK 41. Consider the following :
Assertion (A) : The poles of a magnet cannot be
35. In Q. 34, the magnetic field at C due to curved part is separated by breaking it into two
µ 0I pieces.
(a) , directed into the plane of the paper
6a Reason (R) : The magnetic moment will be re-
µ I
(b) 0 , directed towards you duced to half when a magnet is bro-
6a ken into two equal pieces.
330 COMPREHENSIVE OBJECTIVE PHYSICS

In these statements 46. A rod PQ carrying current I1


(a) both A and R are true and R is the correct explanation of is placed near an infinitely P Q
A long conductor carrying cur- I1
(b) both A and R are true but R is not the correct explanation rent I2. The rod will I2
of A (a) remain stationary
(c) A is true but R is false
(b) translate only
(d) A is false but R is true. [CMC Vellore 2001]
(c) rotate only
42. Which one of the following is the correct unit of mag- Fig. 189
(d) translate and also rotate.
netic flux ?
(a) tesla (b) Wb m–1 47. Same current I
flows in two paral- a
(c) Wb m –2
(d) T m . 2
lel conductors dis-
43. A small coil with N turns is mounted on one end of a tant 2d as shown in 2d P
balance beam and introduced between the poles of an Fig. 190. The
electromagnet. The cross-sectional area of coil is A and strength of mag- a
the length of balance beam is l. The balance is in equi- netic field at a point
librium when no current is passed in the coil. On pass- P equidistant from Fig. 190
ing current I through the coil, the equilibrium has to both conductors is
be restored by putting additional counter weight of mass
µ 0Id µ 0Id
∆m on the balance pan. The magnetic induction at the (a) (b)
4 πa 2πa
spot where coil is located is
µ 0Id µ 0Id
∆mg l ∆mg l (c) (d) .
(a) (b) 4πa2 πa2
NIA 2NIA
∆mg l
48. On the axis of a long, uniform current-carrying sole-
N ∆mg l
(c) (d) . noid, the ratio of flux density at an end to the flux den-
IA NIA 4
sity at the centre is
44. Two straight infinitely A B (a) 1 : 2 (b) 2 : 1
long and thin parallel
0.1 m (c) 4 : 1 (d) 1 : 1.
wires are spaced 0.1 m
apart and carry a current Fig. 187 49. A long straight wire car- Y
of 10 A each. The mag- ries a current of 10 A
netic field at a point distant 0.1 m from both wires when directed along negative
currents are in the same direction is direction of y-axis as
(a) 2 3 × 10–5 T (b) 2 × 10–5 T shown in Fig. 191. A O
uniform magnetic field ® X
(c) 4 × 10 T–5
(d) zero. B
of induction 10–6 T is di-
45. Thick and thin wires give rise to 3 different squares x, rected along + ve x-axis.
y and z as shown in Fig. 188. The magnetic field at the The resultant magnetic Z
centre of induction in tesla at I
point (0, 0, 2 m) is
x y z Fig. 191
(a) 10–6
(b) 0
(c) ∞ (d) none of these.
50. In the previous question, the resultant magnetic induc-
Fig. 188 tion in tesla at (0, 2 m, 0) is
(a) x, y and z is zero (b) x and z is zero (a) zero (b) 10 –6
(c) y and z is zero (d) x only is zero. (c) 2 × 10 –6 (d) 3 × 10 –6.

Answers
1. (c) 2. (c) 3. (a) 4. (c) 5. (b) 6. (b) 7. (b) 8. (d)
9. (e) 10. (d) 11. (a) 12. (a) 13. (d) 14. (a) 15. (a) 16. (a)
MAGNETIC EFFECTS OF CURRENT AND MAGNETISM 331

17. (b) 18. (d) 19. (b) 20. (a) 21. (c) 22. (b) 23. (c) 24. (a)
25. (a) 26. (d) 27. (d) 28. (d) 29. (d) 30. (b) 31. (c) 32. (a)
33. (c) 34. (a) 35. (b) 36. (d) 37. (b) 38. (c) 39. (d) 40. (d)
41. (b) 42. (d) 43. (a) 44. (a) 45. (c) 46. (d) 47. (d) 48. (a)
49. (b) 50. (b).

Solutions
1. Fm = Bqv sin θ 9. The current in P induces magnetic
Whether v = 0 or θ = 0, Fm = 0. field B along the circumference of
2. 5 = 0.01 [15 + R] or R = (500 – 15) Ω = 485 Ω. the circular coil of wire carrying a
clockwise current I as shown in
3. The direction of the resultant field for each of the given
Fig. 192. Since the magnetic field
case : current flow is tabulated in the table below
B is parallel to the current I in each
part of the wire Q, the wire thus ex-
into the out of resultant
page the page magnetic field periences no force (F = BIl sin θ, Fig. 192
θ = 0 ⇒ F = 0) in any direction.
A W and X Y and Z ←0
10. The force per unit length F/l between the two parallel
B W and Y X and Z (zero) l 0 conductors carrying sinusoidal alternating currents
C W and Z X and Y 0↓
differing in phase by π, say I0 sin ωt and I0 sin (ωt + π)
respectively and are at a distance d apart is given by
D X and Z W and Y (zero) l 0
µ0
E Y and Z W and X 0→
F/l = − I 2 sin2 (ωt)
2 πd 0
which is appropriately represented in graph d.
NBA
4. Current sensitivity = mv2 mv
k
11. Bqv = or r =
Clearly, increase of B increases sensitivity. r Bq
5. It is a case of semi-infinite current-carrying wire. In the given problem, r ∝ m.
6. Following table summarises the directions of the mag- 12. Magnetic fields due to ABC and ADC are equal in
netic fields produced by the various currents at loca- magnitude and opposite in direction.
tions P or Q 13. θ = 0°, Fm = Bqv sin 0° = 0.
15. No work is done.
Current Magnetic field
E 1500
At P into the plane At O in the direction 16. v = = m s–1.
of the diagram opposite to OX B 0.4
→ →
At P out of the plane At O in the direction OX 17. l | | B .
of the diagram 18. Magnetic field at a point on the axis is zero.
At Q into the plane At O in the direction OY 19. Apply Fleming’s left hand rule.
of the diagram
20. Apply right hand thumb rule.
At Q out of the plane At O in the direction
of the diagram opposite to OY 1
21. r ∝ .
At P parallel to OX At O into the plane
B
of the diagram 22. V = IR.
23. Apply Fleming’s left hand rule.
7. Fp = 2Fα
mv p
25. r = =
v1 4 Bq Bq
Bqv1 = 2 B(2q) v2 or = .
v2 1 In the given problem, p, B and q are constants.
332 COMPREHENSIVE OBJECTIVE PHYSICS

1 φ
26. φ = B A cos θ = 2 × 0.5 × Wb = 0.5 Wb. 42. B = , φ = BA
2 A
So, SI unit is T m2.
NBA
27. Sensitivity =
k ∆mgl
43. ∆mg × l = NBIA or B =
If k is decreased, the sensitivity can be increased. NIA
B1
28. Magnetic field due to circular coil is perpendicular to µ 0I
plane and directed upwards. Now, use Fleming’s left 44. B1 = B2 =
2 πa 60°
hand rule. −7
4 π × 10 × 10 B2
µ NI 4π × 10−7 × 10 × 0.5 = T
33. B= 0 = T 2π × 0.1
2r 2 × 2 × 10−2 = 2 × 10–5 T
3.14
= × 10–4 T = 1.57 × 10–4 T. Now B= 3 × 2 × 10–5 T A B
2 Fig. 194
=2 3 × 10–5
T.
µ 0I 45. In y and z, equal division of current. Equal and oppo-
34. B = × 2 sin 60°
4 π (a cos 60° ) a site magnetic fields.
µ 0I 3 µ 0I 46. The rod is in non-uniform magnetic field.
= tan 60° = 60°
2πa 2πa µ 0I
47. B1 = B2 =
Applying right hand thumb rule, 2 πa
we find that the magnetic field
is perpendicular to the plane and
B1
directed inwards. a
θ
Fig. 193 B1 sin θ
µ 0I 120° µ 0I θ
35. B = × = 2d +
2a 360° 6 a θ B2 sin θ
Applying right hand palm rule, we find that the mag- a θ
netic field is directed perpendicular to the plane and B2
outwards.
36. Net magnetic field

=
3 µI µ 0 I
− = 0
LM
µ I 3 1

OP
.
Fig. 195

2 πa 6a 2a πMN 3 PQ B = 2B1 sin θ = 2 ×


µ 0I d
×
2 πa a
µ Id
= 02 .
πa
It is directed into the plane of the paper.
38. Ammeter is a low resistance instrument. When con- 1
φ e Be µ 0 nI
nected in parallel with a battery, a large current shall 2 1
48. = = = .
flow through the ammeter. φ c Bc µ 0nI 2
M1 T12 + T22 49. Magnetic field due to current-carrying conductor
39. =
M2 T12 − T22 µ 0I 4 π × 10 −7 × 10
= = T = 10 –6 T
60 60 2π r 2π × 2
T1 = s = 15 s, T2 = s=5s
4 12 It is directed along – ve x-axis.
M1 152 + 52 225 + 25 250 5 . Net field = 10–6 (− i ) + 106 i = zero.
= =
M2 15 − 52 2 225 − 25 = 200 = 4 50. The magnetic field due to current-carrying conductor
I is clearly zero.
40. T = 2π
MB ∴ Net magnetic field = 10–6 T along x-axis.
When mass is quadrupled,the moment of inertia is
increased by a factor of 4. So, T is doubled.
MAGNETIC EFFECTS OF CURRENT AND MAGNETISM 333

SELF-EVALUATION TEST II
Based on UNIT XIII

DIRECTIONS :
(i) MCQs 1 to 24 have one correct alternative.
(ii) MCQs 25 to 30 have more than one correct alternative.
(iii) MCQs 31 to 35 have one or more than one correct alternative.

1. A thin wire carrying a cur- 6. Two parallel wires carry currents of 20 A and 40 A in
rent of 8 A is bent as shown opposite directions. Another wire carrying a current
in Fig. 196. The curvature antiparallel to 20 A is placed midway between the two
radius is 10 cm. The magni- wires. The magnetic force on it will be
tude of the force vector act- O (a) towards 20 A
ing on a unit length of the Fig. 196 (b) towards 40 A
thin wire at O is
(c) zero
(a) 0.20 mN m–1 (b) 20 N m–1
(d) perpendicular to the plane of the currents.
(c) 200 N m–1 (d) 200.1 × 10 –19 N m–1.
7. A plotting compass is placed near a solenoid.
2. A metal wire of mass m slides without friction on two
When there is no current in the solenoid, the compass
rails spaced at a distance d apart. The track lies in a
needle points due north as shown in Fig. 197.
vertical uniform field of induction B. A constant cur-
rent I flows along one rail, across the wire and back
down the other rail. The velocity of the wire as a func-
tion of time, assuming it to be at rest initially is
(a) BId (b) BIdt
BIdt
(c) Bm Idt (d) .
m X Y
3. The plane of a rectangular loop of wire with sides 0.05 m
Fig. 197
and 0.08 m is parallel to a uniform magnetic field of
induction 1.5 × 10 –2 T. A current of 10 A flows through When there is a current from X to Y, the magnetic field
the loop. If the side of length 0.08 m is normal and the of the solenoid at the compass is equal in magnitude to
side of length 0.05 m is parallel to the lines of induc- the Earth’s magnetic field at that point.
tion, then torque is (in N m) In which direction does the plotting compass set ?
(a) 0 (b) 6000
A B C D
(c) 6 × 10 –4 (d) 1.2 × 10–2.
4. The magnetic field midway between two parallel cur-
rent-carrying wires, carrying currents I and 2I is B. If
the current in the wire with current I is switched off, Fig. 198
the magnetic field will become
(a) A (b) B
(a) B/3 (b) 2B
(c) C (d) D.
(c) B/2 (d) B/4.
8. A moving coil galvanometer has a resistance of 50 Ω
5. An ammeter has a resistance of 5.0 Ω. It can measure
and gives a full-scale deflection for 10 mA. How could
a maximum current of 1.5 A. Its range may be increased
it be converted into an ammeter with a full-scale de-
to 6.0 A by using a shunt of
flection for 1 A ?
(a) 3/5 Ω (b) 5/3 Ω
50 50
(c) 8/3 Ω (d) 3/8 Ω. (a) Ω in series (b) Ω in parallel
99 99
[All India PM/PD 1999] (c) 0.01 Ω in series (d) 0.01 Ω in parallel.
334 COMPREHENSIVE OBJECTIVE PHYSICS

9. A wire of length 3.0 cm is placed at right angles to a at a speed of 3.0 × 106 m s–1. If a proton of mass
magnetic field of flux density 0.040 T. The wire carries 1.8 × 10–27 kg were to move in a circle of the same
a current of 5.0 A. radius and if it were acted upon by the same mag-
netic field, then its speed will be
× × × × × × × (a) 3.0 × 106 m s–1 (b) 1.5 × 103 m s–1
4
(c) 6 × 10 m s –1
× × × × × × ×
region of uniform (d) cannot be estimated from the given data.
× × × × × × ×
magnetic flux 14. A milliammeter has a resistance of 12 Ω and gives a
× × × × × ×
5.0 A full-scale deflection for a current of 0.01 A. The instru-
× × × × × × × ment is to be converted into a voltmeter reading up to
× × × × × × × 3 V. The resistance that must be put in series with it is
(a) 102 Ω (b) 288 Ω
Fig. 199 (c) 300 Ω (d) 412 Ω.
15. A charged particle moves in a region having a uniform
What is the magnitude of the force which the field ex-
magnetic field and a parallel uniform electric field. At
erts on the wire ?
some instant, the velocity of the particle is perpendicu-
(a) less than 0.006 N (b) 0.0060 N lar to the field direction. The path of the particle will
(c) greater than 0.006 N but less than 0.6 N be
(d) 0.60 N (e) greater than 0.6 N. (a) a straight line
10. Two identical coils carry equal currents, have a com- (b) a circle
mon centre, and their planes are at right angles to each (c) a helix with uniform pitch
other. What is the ratio of the magnitudes of the result- (d) a helix with non-uniform pitch.
ant magnetic field at the centre and the field due to one 16. Consider a long, straight wire of cross-sectional area A
coil alone ? carrying a current i. Let there be n free electrons per
(a) 2 : 1 (b) 1 : 1 unit volume. An observer places himself on a trolley
(c) 1 : 2 (d) 2 : 1. moving in the direction opposite to the current with a
11. A long straight wire XY lies in i
X speed v = and separated from the wire by a dis-
the same plane as a square loop nAe
tance r. The magnetic field seen by the observer is very
of wire PQRS which is free to P Q
nearly
move. The sides PS and QR are
initially parallel to XY. µ0 i
(a) (b) zero
2πr
The wire and loop carry steady
currents as shown in Fig. 200. µ0 i 2µ 0 i
(c) (d) . [MP PET 1994]
S R πr πr
What will be the effect on the
loop ? 17. A meter of resistance 5 Ω, fitted with a shunt resist-
Y
ance of 1 Ω, is in a circuit in which the current is 0.09
(a) It will move towards the long Fig. 200
wire.
A. The combined resistance of the meter and the shunt
(b) It will move away from the long
is
wire. 1
(a) 6 Ω (b) Ω
(c) It will rotate about an axis parallel to XY. 5
(d) It will be unaffected. 5 6
(c) Ω (d) Ω .
(e) It will contract. 6 5
12. A bar magnet 8 cm long is placed in the magnetic me- 18. In the above question, the potential difference across
ridian with the N-pole pointing towards geographical the meter is
north. Two neutral points separated by a distance of (a) 0.45 V (b) 0.075 V
6 cm are obtained on the equatorial axis of the magnet. (c) 0.54 V (d) 0.09 V.
If eBh = 3.2 × 10–5 tesla, then the pole strength of the
19. In Q. No. 17, the current through the shunt is
magnet is
(a) 0.018 A (b) 0.45 A
(a) 5 abampere cm (b) 10 abampere cm
(c) 0.54 A (d) 0.075 A.
(c) 2.5 abampere cm (d) 20 abampere cm.
20. A battery is connected between two points A and B on
13. An electron of mass 0.90 × 10–30 kg under the action the circumference of a uniform conducting ring of
of a magnetic force moves in a circle of 2.0 cm radius radius r and resistance R. One of the arcs AB of the
MAGNETIC EFFECTS OF CURRENT AND MAGNETISM 335

ring subtends and angle θ at the centre. The value of nBIl sin θ
the magnetic induction at the centre due to the current
nBIl sin θ
in the ring is
θ
(a) Proportional to 2 (180° – θ) N S N S
θ
(b) Inversely proportional to r
nBIl sin θ
(c) Zero, only if θ = 180°
nBIl sin θ
(d) Zero for all values of θ. [IIT 1996]
21. A magnet when placed perpendicular to a uniform field (c) (d )
of strength 10 –4 Wb m–2, experiences a couple of moment
4 × 10 –5 N m. What is its magnetic moment ? nBIl

(a) 0.4 A m 2
(b) 0.2 A m2

(c) 0.16 A m2 (d) 0.04 A m2


N S
(e) 0.06 A m2. θ
22. The ampere is defined as the current which, flowing in
two straight, thin, parallel wires of length l separated nBIl
by a distance d in a vacuum, generates a force F per
unit length on each wire. What are the correct values of (e)
l, d and F ?
l/m d/m F/N m–1 Fig. 202
(a) ∞ 1 2× 10 –7 Which one of the diagrams (Fig. 202) correctly shows
(b) ∞ 1 4π × 10 –7 the magnitude and direction of the forces acting on the
(c) ∞ 2π 4π × 10 –7 vertical sides of the coil ?
(d) 1 1 2 × 10 –7 24. A current I flows in the network shown in Fig. 203.
(e) 1 2π 4π × 10 –7. Resulting magnetic induction at point N is
23. A current I is carried by a square coil of n turns and A F a N
side l suspended vertically as shown in a uniform hori- a
zontal magnetic field of flux density B.
a a
coil suspension
a
2a I E D

N S
q a

l B C
2a

Fig. 203
direction
of current µ 0I 2 µ 0I
(a) (b) −
4 πa 8π a
Fig. 201 8 µ 0I 2 µ 0I
(c) − (d) .
2 πa 8 πa
nBIl cos θ
25. Two ions have equal masses but one is singly-ionised
nBIl cos θ and the other is doubly-ionised. They are projected from
θ the same place in a uniform magnetic field with the
N
θ
S N S same velocity perpendicular to the field.
(a) Both ions will go along circles of equal radii
nBIl cos θ
(b) The circle described by the singly-ionised charge will have
nBIl cos θ a radius double that of the other circle.
(c) The two circles do not touch each other
(a ) (b)
(d) The two circles touch each other.
336 COMPREHENSIVE OBJECTIVE PHYSICS

26. If a charged particle projected in a gravity-free room 32. There will be a force of repulsion between
deflects, (a) two parallel stream of electrons moving in the opposite
(a) there must be an electric field direction
(b) there must be a magnetic field (b) two parallel wires carrying current in the opposite
(c) both fields cannot be zero direction
(d) both fields can be non-zero. (c) two parallel electron streams going in the same direction
27. A long, straight wire of radius R carries a current dis- (d) two parallel wires carrying current in the same direc-
tributed uniformly over its cross-section. The magni- tion.
tude of the magnetic field is 33. A steady electric current is flowing through a cylindri-
(a) maximum at the axis of the wire cal conductor.
(b) minimum at the axis of the wire (a) The electric field at the axis of the conductor is zero
(c) maximum at the surface of the wire (b) The magnetic field at the axis of the conductor is zero
(d) minimum at the surface of the wire. (c) The electric field in the vicinity of the conductor is zero.
28. A long straight current-carrying wire is along X-axis. (d) The magnetic field in the vicinity of the conductor is zero.
There are points in the YZ plane such that 34. Which of the following is correct ?
(a) the directions of magnetic field are same (a) A magnetic dipole experiences maximum torque when it
(b) the directions of magnetic field are opposite is placed normal to the magnetic field
(c) the field may be regarded as uniform (b) The magnetic moment of a ring of radius R, carrying
(d) current at those points is non-zero. charge q, rotating about an axis perpendicular to the plane
29. Susceptibility is positive for of the ring and passing through the centre of the ring is
(a) paramagnetic substances 1
ωq R2
(b) ferromagnetic substances 2
(c) The relation between angular momentum and magnetic
(c) non-magnetic substances
moment is true for every finite size body.
(d) diamagnetic substances. [CPMT 1990] (d) The minimum potential energy of a magnetic dipole is
30. In SI, the unit for magnetic moment is zero.
(a) T J –1 (b) J T –1 35. A charged particle goes undeflected in a region con-
(c) A m –1 (d) N m T –1. taining electric and magnetic field. It is possible that
31. Mark out the correct options : → → → →
(a) Diamagnetism occurs in all materials. (a) E | | B , v | | E
(b) Diamagnetism results from the partial alignment of per- → →
manent magnetic moment. (b) E is not parallel to B
(c) The magnetising field intensity H is always zero in free → → → →
space. (c) v | | B but E is not parallel to B
(d) The magnetic field of induced magnetic moment is oppo- → → → →
site to the applied field. (d) E | | B but v is not parallel to E .

Answers
1. (a) 2. (d) 3. (c) 4. (b) 5. (b) 6. (b) 7. (a) 8. (b)
9. (b) 10. (d) 11. (a) 12. (a) 13. (b) 14. (b) 15. (d) 16. (a)
17. (c) 18. (b) 19. (d) 20. (d) 21. (a) 22. (a) 23. (e) 24. (b)
25. (b, d) 26. (c, d) 27. (b, c) 28. (a, b) 29. (a, b) 30. (b, d) 31. (a, d) 32. (a, b)
33. (b, c) 34. (a, b, c) 35. (a, b).
MAGNETIC EFFECTS OF CURRENT AND MAGNETISM 337

Solutions
µ 0I 4 π × 10 − 7 × 8 the resultant field is B as indicated in the diagram.
1. B= = T = 8π × 10 –6 T Deflection of needle in compass is thus best illustrated
4r 4 × 10 × 10 −2
in diagram A.
F = BIl
F 64 π
= BI = 8π × 10 –6 × 8 N m–1 = mN m–1
l 1000
= 0.20 mN m–1
2. F = BId
BId
acceleration =
m
BIdt
Velocity =
m Fig. 206
3. F = BIl = 1.5 × 10–2 × 10 × 0.08 N = 1.2 × 10–2 N
50 × 10 × 10 −3 0.5 × 100 50
8. S = Ω= Ω = Ω.
0.05 m 1 − 10 × 10 −3 99 99
9. Force acting on wire has a magnitude given by
0.08 m

10 A

Fig. 204

τ = 1.2 × 10–2 × 0.05 N m = 6 × 10 –4 N m.


µ 0 2I µ 0I µ 0I
4. B = − = Fig. 207
r r r
µ 0 (2I) F = IlB = (5.0) (0.03) (0.040) = 0.006 N.
Now B′ = = 2B.
r
10. B1 = B2 + B2 = 2B2 = 2B
GI g 5 × 1.5 7.5 5
5. S = = Ω = Ω = Ω. B1 2
I − I g 6 − 1.5 4.5 3 = .
B 1
6. Like currents attract and unlike currents repel.
11. The forces on PQ and RS are equal in magnitude and
opposite in direction. Hence, no net resultant force in
the XY direction. The force per unit length on PS due
to currents in XY and PS is given by

20A

20A 40A

r r

Fig. 205

7. The magnetic field set up by current in the coil is as


follows :
Since Bh, magnetic field experienced by the compass
is numerically equals to Bv, the Earth’s magnetic field,
Fig. 208
338 COMPREHENSIVE OBJECTIVE PHYSICS

F2 µ 0I1I2 16. Note that v is very small.


= towards XY.
l 2πa GS 5×1 5
The force per unit length on QR due to current in XY 17. = Ω = Ω.
G+S 5+1 6
and QR is given by
F3 µ 0I1I2 5
= away from XY. 18. V = 0.09 × volt = 0.075 volt.
l 2πb 6
Hence, the net resultant force on wire PQRS is 0.075
19. I = A = 0.075 A.
µ I I (b − a) 1
F = F2 – F3 = 0 1 2 towards XY.
2πab 20. Equal and opposite magnetic fields due to currents in
Therefore, the wire loop PQRS will move towards the two parts.
long wire. 21. τ = MB sin 90°
m×8
12. 0.32 = τ 4 × 10−5
5×5×5 M= = A m2 = 0.4 A m2.
B 10−4
22. The ampere is defined as follows :
5 cm
4 cm “It is that current, which flowing in each of two infi-
nitely-long parallel straight wires of negligible cross-
sectional area separated by a distance of 1 metre in
3 cm vacuum, produces a force between the wires of
2 × 10–7 newton metre–1.”
µ0 = 4π × 10–7 H m–1
µ 0I1I2
That is, from F =
Fig. 209
2πd
where d=1m
0.32 × 125 I1 = I2 = 1 A
m= ab ampere cm
8
m = 5 ab ampere cm. (4 π × 10 −7 ) (1) (1)
We have F=
2 π (1)
mv2 Bqr
13. Bqv = or v = = 2 × 10–7 newton metre–1
r m
In the given problem, B, q and r are constants. We thus conclude that
1 l=∞m
∴ v∝
m d=1m
vp me 0.9 × 10−30 1 F = 2 × 10–7 N m–1.
= = −27 =
ve mp 1.8 × 10 2000 23. Only the forces induced on the vertical arms of the
6
coil cause the coil to rotate, the force on each arm is
3 × 10 easily computed as
or vp = m s –1 = 1.5 × 103 m s–1.
2000
14. V = Ig (R + G)
F = nBIl
V
R+G=
Ig B
V
N θ S
R= –G
Ig
3 F = nBIl
R= – 12 = (300 – 12) Ω = 288 Ω.
0.01
15. Due to magnetic field, the particle shall move in a cir- Fig. 210
cular path. Due to electric field, the particle shall fol- F = B(nI) l sin 90° = nBIl.
low a straight path with increasing velocity. The com- Its direction is determined via the right-hand screw
bined effect would result in a helical path with non- rule as shown above.
uniform pitch.
MAGNETIC EFFECTS OF CURRENT AND MAGNETISM 339

24. CD and AF would not contribute any magnetic induc- mv2


tion. 25. Bqv =
r
Magnetic induction at N due to AB and BC,
mv
µ 0I r=
B1 = 2 × sin 45° = 2µ0I . Bq
4 π (2a) 8πa m, B and v are constants.
It is perpendicular to plane and directed up 1
Magnetic induction at N due to DE and EF, ∴ r∝
q
µ 0I 2µ 0I Double the charge, half the radius.
B2 = 2 × sin 45° =
4 πa 4 πa 26. (c) and (d) are basically same.
It is perpendicular to plane and directed downwards. µ 0Ir
Net induction, B = B1 – B2 27. B=
2 πa2
2µ 0I 2µ 0I 2µ 0I . For axis, r=0
= − =−
8πa 4πa 8πa For surface, r = a.

KNOWLEDGE PLUS

l A particle of mass M and charge Q moving with velocity v describes a circular path of radius R when subjected to a
uniform transverse magnetic field of induction B. The work done by the field when the particle completes one full
circle is
F Mv I 2π R
2
(a) BQv2πR (b) GH R JK (c) zero (d) BQ2πR [AIEEE 2003]

Sol. Work done by magnetic field is zero.


So, (c) is the right choice.

l A wire of certain length carries a steady current. It is first bent to form a circular coil of one turn. The same wire is
next bent to form a circular coil of three turns. The ratio of magnetic induction at the centre of the coil in the two cases
is
(a) 9 : 1 (b) 1 : 9 (c) 1 : 3 (d) 1 : 1 [Kerala PMT 2003]
Sol. Ist case :
Number of turns in the coil = 1
l
∴ 1 × 2πr1 = l (length of wire) or r1 =

µ0 n1I µ 0I µ πI
B1 = = = 0
2r1 2(l / 2π) l
IInd case :
Number of turns in the coil = 3
l
∴ 3 × 2πr2 = l or r2 =

µ0 n2I 3µ 0I 9µ 0πI
B2 = = =
2r2 2(l / 6π) l
∴ B1 = B2 = 1 : 9
So, (b) is the right choice.
UNIT XIV

ELECTROMAGNETIC INDUCTION
AND ALTERNATING CURRENT
l Electromagnetic induction l Faraday’s law l Induced emf and current l Lenz’s law l Eddy
currents l Self and mutual inductance l Alternating currents l Peak and rms value of alternating
current/voltage l Reactance and impedance l LC oscillations l LCR series circuit (Phasor
diagram) l Resonant circuits and Q-factor l Power in AC circuits l Wattless current l AC
generator and Transformer

UNIT DETAILS

1. Synopsis Points 45 Plus

2. Illustrations 10 Plus

3. MCQs from Competitive Examinations 240 Plus

4. Self-Evaluation Tests 2

5. Total Number of MCQs 385 Plus

6. Total Number of Solutions 375 Plus


CHAPTER 14

ELECTROMAGNETIC INDUCTION
AND ALTERNATING CURRENT

SYNOPSIS

1. MAGNETIC FLUX ® 4. FARADAY’S LAWS OF ELECTROMAGNETIC


B
→ → INDUCTION
φ = N B. A
φ = NBA cos θ (i) Whenever magnetic flux linked with a circuit
q ® changes, an induced emf is set-up.
For acute value of θ, A
cos θ is positive. So, φ is posi- (ii) The magnitude of induced emf is directly propor-
tive. For obtuse value of θ, tional to the rate of change of magnetic flux linked with the
cos θ is negative, So, φ is nega- circuit.
tive. Fig. 1
5. LENZ’S LAW
2. UNITS OF MAGNETIC FLUX
cgs unit is maxwell The direction of induced emf or induced current is
such that it opposes the change that is producing it.
SI unit is weber.
1 Wb = 108 maxwell. 6. INDUCED EMF BY CHANGING THE RELATIVE
3. FARADAY’S EXPERIMENT ORIENTATION OF COIL AND MAGNETIC
When a magnet moves towards a stationary coil, an FIELD
induced emf is set up. If coil is part of a closed circuit, then (i) If time is counted
an induced current begins to flow from the instant the coil is
S. Motion of Polarity of face Direction Nature of
perpendicular to the mag-
No. magnet of coil opposite of induced magnetic netic field i.e., at t = 0, θ = 0,
to the approa- current force then
ching pole of φ = NBA cos ωt Fig. 2
the magnet
and E = NBA ω sin ωt
1. N pole North anticlockwise Repulsive
approaching or E = E0 sin ωt
the coil Here, E0 = NBAω
2. N pole South clockwise Attractive (ii) If time is counted
receding from from the instant the coil is
the coil parallel to the magnetic field
wt
3. S pole South clockwise Repulsive i.e., at t = 0, θ = 90°, then E
approaching φ = NBA sin ωt
the coil
E = NBA ω cos ωt Fig. 3
4. S pole North anticlockwise Attractive
or E = E0 cos ωt
receding from
the coil
343
344 COMPREHENSIVE OBJECTIVE PHYSICS

Important data is summarised in the following table :

S. Time in terms Angle θ between the normal Position of coil w.r.t. Magnetic flux φ Induced emf
No. of period of to the plane of the coil magnetic field
revolution and the direction of
T magnetic field

1. 0 0° Plane of coil normal to B NBA (max. value) 0

T
2. 90° Plane of coil parallel to B 0 NBAω (max. value)
4

T
3. 180° Plane of coil normal to B NBA (max. value) 0
2

3T
4. 270° Plane of coil parallel to B 0 – NBAω
4
5. T 360° Plane of coil normal to B NBA (max. value) 0

7. INDUCED EMF DUE TO ROTATION OF CONDUCTING ROD OF LENGTH L IN UNIFORM PERPEN-


DICULAR MAGNETIC FIELD B

1
E= Bl2ω where ω is the uniform angular velocity of the rotating rod.
2

8. MOTION OF A CONDUCTOR IN A MAGNETIC FIELD AND FORMULAE FOR MOTIONAL EMF, IN-
DUCED CURRENT AND INDUCED CHARGE
All the important data is summarised in the following table :

S. No. Name of Quantity Formula Circuit Dependence SI Unit


Open/Closed upon resistance


1. Induced emf E=– open or closed No volt
dt

1 dφ
2. Induced current I=– closed Yes ampere
R dt

1
3. Induced charge dQ = – dφ closed Yes coulomb
R

E2 B2l 2v2
4. Power required to pull a loop out of a P= = I2 R = open or closed Yes watt
R R
magnetic field (= Induced power)

× × × × × × × × ×
F B
× × × × × × 2× × × I
× × × × × × × × ×
×F × × × × × × × ×
1
× × × × × × × × l
× × × × × × × × ×
× × × × × × × × ×
× × × × × × × × ×
F v
× × × × × × 3× × ×

Fig. 4
ELECTROMAGNETIC INDUCTION AND ALTERNATING CURRENT 345
l When a straight conductor of length l moves with 11. SELF-INDUCTANCE OF A PLANE COIL
constant velocity v in a magnetic field B, the induced
emf, E = Blv when l and v are both 90° to B. µ 0πN2r
L= henry
l When l or v is parallel to B, then E = 0.
2
l Fleming’s left hand rule is used to determine the di- 12. SELF-INDUCTANCE OF A SOLENOID
rection of force on a current-carrying conductor.
l Fleming’s right hand rule is used to determine the
µ 0N2 A
L=
direction of induced current/induced emf. l
Stretch the Motion of
If no iron or similar material is nearby, then the value
thumb and the first two conductor of self-inductance depends only on the geometrical factors
fingers of your right (length, cross-sectional area, number of turns).
hand in mutually per- Thumb A coil having a sufficiently high inductance is called
pendicular directions. If First ‘inductor’ just as a conductor having a high resistance is
the first finger points in finger called ‘resistor’ and a condenser of high capacitance is called
the direction of the mag- ‘capacitor’.
netic field, the thumb in Field
the direction of motion of 13. ENERGY ASSOCIATED WITH SELF-INDUCT-
the conductor, then the ANCE
central finger points in Central dW = – EIdt, where E is the induced emf
the direction of the in- finger dI
duced emf or induced dW = L Idt = LIdI
Current dt
current in the conductor The total work done in maintaining a steady current
[Fig. 5]. Fig. 5
I0 in the coil is given by

z z
Illustration 1. State (Yes/No) whether emf is in- I0 1
duced in the following cases : W= dW = LIdI = LI02
(i) A conductor falling freely in NS direction. 0 2
(ii) A conductor falling freely in EW direction. If I0 = 1, then L = 2W.
(iii) A metallic satellite orbiting in a plane which is So, the self-inductance of a coil is equal to twice the
also the equatorial plane of earth. work done in establishing unit current in the coil.
(iv) A metallic satellite orbiting in a plane inclined to 14. To calculate emf induced at break is, in general, a
the equatorial plane of earth. complicated business. But it can be easily done for
(v) Aeroplane/Train moving horizontally. one important practical circuit. In order to prevent
Answer. (i) No (ii) Yes (iii) No (iv) Yes (v) Yes. sparking at the contacts of the switch in an inductive
Explanation. (i) l || BH and v || BV circuit, a capacitor is often connected across them.
(ii) v, l and BH are mutually perpendicular When the circuit is broken, the current continues to
(iii) v || BH, l || v (iv) Resolve v into two rectangular flow for a brief time by charging the capacitor. Since
components the current does not decay rapidly therefore back emf
(v) Axle of train and wings of aeroplane shall decide. never rises so high as to cause sparking.
The energy equation for such a circuit is
9. RELATION BETWEEN HENRY AND ABHENRY
1 1
CV2 = LI2.
1 volt 108 abvolt 2 2
1 henry = =
1 ampere / second 1 abampere / second 15. The coefficient of coupling of two coils is a measure of
10
or 1 henry = 109 abhenry the coupling between the two coils. If L1 and L2 are
the coefficients of self-induction of the two coils and M
The smaller units of self-inductance are millihenry
is the coefficient of mutual induction of the two coils,
and microhenry.
then the coefficient of coupling is given by
1 mH = 10–3 H and 1 µH = 10–6 H
M
K= .
10. RELATION BETWEEN HENRY AND WEBER L1L2
In Fig. 7, the value of coefficient of coupling is mini-
φ 1 Wb
L= ∴ 1H= = 1 Wb A–1 mum. This value is maximum in Fig. 8. In Fig. 9, it
I 1A has intermediate value.
Note. The value of coefficient of coupling is always less
than one.
346 COMPREHENSIVE OBJECTIVE PHYSICS

16. MUTUAL INDUCTANCE OF TWO LONG SOLE-


NOIDS M = µ0npnsAl
LM∵ np =
Np
and ns =
Ns OP
µ 0 N pN s A
N l l Q
M= If the core of the solenoid is made of a magnetic ma-
l terial of relative magnetic permeability µr,
If np and ns denote the number of turns per unit then M = µ0µrnpnsAl or M = µnpnsAl
length, then

KNOWLEDGE PLUS
l If the cross-sectional areas of two co-axial solenoids are different, then the cross-sectional area of the inner solenoid is
to be considered in the numerical calculation of mutual inductance.
l If the lengths of the two co-axial solenoids are different, then the length of the bigger solenoid is to be considered in the
numerical calculation of M.

17. MUTUAL INDUCTANCE OF TWO PLANE L


COILS (ii) The term has the dimensions of time. It is
R
Consider two plane coils placed co-axially near to each known as inductive time constant or time constant of
other. Let Np and Ns be the number of turns in the primary LR circuit.
and secondary coils respectively. Let Ip ampere be the cur- L
rent flowing in the primary coil. The magnetic field at the When L is in henry and R is in ohm, then is meas-
R
µ 0 N pI p ured in second.
centre of the primary coil is given by B = , where rp L
2 rp When t = (time constant), then
R
is the radius of the primary coil. This magnetic field spreads
FG 1 IJ
to the secondary coil also.
Magnetic flux through secondary, φ = BANs
H
I = I0 (1 – e–1) = I0 1 −
e K
F µ N I I AN FG 1 IJ
=G
or
H
I = I0 1 −
K ( ∵ e = 2.718)

H 2 r JK
0 p p 2.718
s
p or I = I0 (1 – 0.368) = 0.632 I0
But φ = MIp The time constant of LR circuit is the time dur-
ing which the current grows from zero to 0.632 times
µ 0N p N s the maximum value.
∴ M= A = k NpNs
2 rp (iii) I-t graph for growth of current in LR cir-
cuit
where k is a constant which depends upon the area of the
coils.

18. GROWTH OF CURRENT IN LR CIRCUIT


(i) Helmholtz equation for growth of current is
F I
GG JJ
R
− t

H
I = I0 1 − e
K
L

Fig. 7
Fig. 6
ELECTROMAGNETIC INDUCTION AND ALTERNATING CURRENT 347
(iv) Theoretically, current takes infinite time to grow (ii) When t = CR (time constant), then
to its maximum value.
FG 1IJ
19. DECAY OF CURRENT IN LR CIRCUIT
Q = Q0 (1 – e–1) = Q0 1 −
H e K
(i) Helmholtz equation for the decay of current is as FG 1 IJ = Q
under : H
= Q0 1 −
2.718 K 0 (1 – 0.368) = 0.632 Q0
R
− t The time constant of CR circuit is the time taken
I = I0 e L
by the capacitor to get charged to 0.632 times the
L maximum charge.
(ii) When t = (time constant), then I = 0.368 I0. (iii) Q-t graph for charging of capacitor in CR
R
circuit
The time constant of LR circuit may also be
defined as the time taken by the current to fall from
maximum to 0.368 times the maximum value of
current.
(iii) I-t graph for decay of current in LR circuit

I0

Fig. 10
L2 > L1 (iv) The current at any instant during the charging of
Current

L2 the capacitor is given by


L1
0.368 I0 t

I = I0 e CR

(v) I-t graph for charging of capacitor in CR


circuit
L1 L2 Time
R R

Fig. 8

(iv) Theoretically, current takes infinite time to be-


come zero.

20. CHARGING OF CAPACITOR IN CR CIRCUIT


(i) The equation for the growth of charge on capaci-
F I
GG JJ
1
− t Fig. 11
tor in CR circuit is Q = Q0 1 − e
H K
CR
21. DISCHARGING OF CAPACITOR IN CR CIR-
CUIT
(i) The equation for the decay of charge in CR circuit
is
1
− t
Q = Q0 e CR

(ii) When t = CR (time constant), then Q = 0.368 Q0.


The time constant of CR circuit is the time taken
Fig. 9 by the charge on the capacitor to fall from maximum
to 0.368 times its maximum value.
348 COMPREHENSIVE OBJECTIVE PHYSICS

(iii) The voltage during the discharging of the capaci-


T FG IJ
tor is given by
t
charge in a circuit in the time of half cycle i.e.
2 H
as is sent
K

V = E0 e CR by the given alternating emf in the same circuit in the same
(iv) The current at any instant during the discharg- FG T IJ
ing of the capacitor is given by H
time i.e.
2
.
K
t

I = – I0 e CR 24. ROOT MEAN SQUARE VALUE OF AC
(v) Q-t graph for discharging of capacitor Root Mean Square value (rms value) or Virtual value
or Effective value of AC is that steady current which would
produce the same heat in given resistance in a given time as
is done by the alternating current when passed through the
same resistance for the same time.

Average/Virtual values of AC
(i) Average or mean value of alternating current over posi-
2
tive half cycle is I or 0.637 I0 or 63.7% I0.
π 0
(ii) Average or mean value of alternating current over nega-
2
Fig. 12 tive half cycle is – I or – 0.637 I0 or – 63.7% I0.
π 0
(vi) I-t graph for discharging of capacitor. (iii) Average or mean value of alternating current over one
complete cycle is zero.
(iv) Average or mean value of alternating emf over positive
2
half cycle is E or 0.637 E0 or 63.7% E0.
π 0
(v) Average or mean value of alternating emf over negative
2
half cycle is – E or – 0.637 E0 or – 63.7% E0.
π 0
(vi) Average or mean value of alternating emf over one com-
plete cycle is zero.
1
(vii) Virtual value of alternating current is times (or
2
0.707 times) the peak value of current.
1
(viii) Virtual value of alternating emf is times (or 0.707
2
Fig. 13 times) the peak value of emf.

22. AVERAGE VALUE OF AC OVER HALF CYCLE Illustration 2. Deter-


2A
mine the rms value of alternat-
Average or Mean value of alternating current over
ing current shown in Fig. 14.
half cycle is that steady current which will send the same
I O
FG IJ
T
Ans. 2 A. t

H
amount of charge in a circuit in the time of half cycle i.e.
K
2
Explanation.
2A
as is sent by the given alternating current in the same cir- 22 + 22 + 22
Iv = A
cuit in the same time FG i.e. IJ .
T 3 Fig. 14
H 2K =2A

23. MEAN OR AVERAGE VALUE OF ALTERNAT- 25. AC THROUGH A RESISTOR


ING EMF (i) If I = I0 sin ωt, then
Average of mean value of alternating emf over half E = E0 sin ωt
cycle is that steady emf which will send the same amount of
ELECTROMAGNETIC INDUCTION AND ALTERNATING CURRENT 349
27. AC THROUGH AN INDUCTOR
L
(i) If I = I0 sin ωt, then
FG π IJ
H
E = E0 sin ωt +
2 K
Fig. 19
Fig. 15 (ii) The variation of E and I with time is shown below :
(ii) The variation of E and I with ωt is graphically
represented as follows :

Fig. 20

(iii) The phasor diagram is shown below :


Fig. 16

(iii) The phasor diagram is as shown below :

Fig. 21
Fig. 17
Illustration 4. What would be the effect on current
Illustration 3. What would be the effect on current in a purely inductive AC circuit if the frequency of the alter-
in a purely resistive AC circuit if the frequency of the alter- nating emf is increased ?
nating emf is increased ? Ans. Current will decrease.
Ans. No effect. Explanation. When frequency is increased,
Explanation. R is not affected by the frequency of FG = E IJ
v
the applied voltage.
XL ( = 2πf L) increases. So Iv
H XK L
decreases.

26. POWER IN A PURELY RESISTIVE CIRCUIT 28. INDUCTIVE REACTANCE (XL)


(i) P = Ev Iv
(ii) The variation of (i) XL = ωL = 2πf L, where
power with time f is frequency of AC
is graphically EI (ii) XL is measured in ohm
represented as (iii) The variation of X L XL
follows. with f is shown in
t Fig. 22.
Fig. 18 f
Fig. 22
350 COMPREHENSIVE OBJECTIVE PHYSICS

29. POWER IN A PURELY INDUCTIVE CIRCUIT Illustration 6. What would be the effect on current
The average power consumed over one full cycle is in a purely capacitive AC circuit if the frequency of the al-
zero. ternating emf is increased ?
Ans. Current increases.
30. AC THROUGH A CAPACITOR
Explanation. When the frequency is increased,
C FG = 1 IJ FG = E IJ
H 2πfC K
(i) If I = I0 sin ωt, then v

FG π IJ
XC decreases. So, Iv
H XK C
increases.

E = E0 sin ωt −
H 2 K 32. POWER IN A PURELY CAPACITIVE CIRCUIT
Fig. 23 The power consumed over one cycle is zero.
(ii) The variation of E E = E0 sin wt
33. AC THROUGH LR CIRCUIT
and I with ωt is shown be-
low. In this representation,
L R
E = E0 sin ωt E, I If I = I 0 sin ωt,
®
FG π IJ wt then
and I = I0 sin ωt +
H 2 K E = E0 sin (ωt + φ)

I = I0 sin (wt + p/2) Fig. 28


Fig. 24
34. INDUCTIVE IMPEDANCE
I0 I
ZL = R 2 + L2ω 2
(iii) The phasor diagram
is shown in Fig. 25. The reciprocal of impedance is called admittance.
E E0
90° 35. POWER IN LR CIRCUIT
wt
O X R
Fig. 25 Average power = Ev Iv
R + L2ω 2
2

31. CAPACITIVE REACTANCE


36. AC THROUGH CR CIRCUIT
1 1 C R
(i) XC = = , where f is
ωC 2πfC
the frequency of AC XC If E = E 0 sin ωt,
(ii) XC is measured in ohm then
(iii) The variation of XC with f f I = I0 sin (ωt + φ)
is shown in Fig. 26 :
Fig. 26
Illustration 5. Why hot wire ammeter of circuit Fig. 29
shown in Fig. 27 (a) shows deflection while the same amme- 37. CAPACITIVE IMPEDANCE
ter does not show deflection in circuit shown in Fig. 27 (b) ?
1
C C ZC = R2 +
R R C ω2
2

38. AC THROUGH LCR CIRCUIT


L C R

If I = I 0 sin ωt,
(a) (b) then
Fig. 27 E = E0 sin (ωt + φ)
Ans. Capacitor allows alternating current to flow
through it but does not allow direct current to flow through. Fig. 30
ELECTROMAGNETIC INDUCTION AND ALTERNATING CURRENT 351
39. Phasor representations. Let us consider the phasor 1
diagram of Fig. 31. It shows the maximum current (iii) When Lω = , then
Cω I E
I0, the current I and the phase angle (ωt – φ) at any tan φ = 0 or φ = 0°.
instant of time. In this case, the current
Fig. 32 shows three phasors representing the voltages and emf are in the same phase
across the three circuit elements at any instant of time. [Fig. 36].
Fig. 36

40. Impedance of LCR Circuit. The current amplitude


(maximum) in LCR circuit is given by
E0
I0 =
FG 1 IJ 2

H
R 2 + Lω −
Cω K
FG 1 IJ 2
The term R2 + Lω −
H Cω K represents the effective

resistance offered by LCR circuit to the flow of alternating


current through it. It is known as impedance of LCR circuit
Fig. 31 Fig. 32 and is denoted by Z.
Fig. 33 shows phase relationship between voltage and
FG 1 IJ 2
current. ∴ Z=
H
R2 + Lω −
Cω K
Note. Let VR, VL and VC be the potential differences across
resistance R, inductance L and capacitance C respectively. It may be
noted that
VR + VL + VC ≠ V
where V is the total potential difference across the three. The reason
is that these voltages are not in the same phase with the current.
Whereas VR is in phase with I, VL leads I by π/2 and VC lags behind I by
π/2. Different phase relationships are vectorially represented in
Fig. 38. The voltages VL and VC are out of phase. So, their net value is
(VL – VC). Here, it is assumed that VL is greater than VC. VR is in phase
with current. So, V is the resultant of VR and (VL – VC).

Fig. 33

Special Cases
1 E
(i) If Lω > , then φ is

+ ve. f
In this case, the emf leads I
the current by a phase angle φ. This
is represented vectorially in Fig. Fig. 34
34.

1 Fig. 37 Fig. 38
(ii) If Lω < , then φ is I

– ve. ∴ =V2 VR2
+ (VL – VC)2
f
In this case, the emf lags be- Thus, if VR is 40 volt, VL is 60 V and VC is 30 V, then
hind the current by a phase angle E V ≠ 40 + 60 + 30 volt, i.e., V ≠ 130 volt.
φ. This is represented vectorially in
Fig. 35. Fig. 35 But V= ( 40)2 + ( 60 − 30)2 volt = 50 volt
352 COMPREHENSIVE OBJECTIVE PHYSICS

VL − VC Power factor is defined as the ratio of true


The phase lead φ of V is given by tan φ = power to apparent power.
VR
If I be the current flowing in the circuit at any time, R
Power factor = cos φ =
then
F 1 IJ FG 1 IJ 2

I ( Lω ) − I G
H Cω K Lω −
1 H
R 2 + Lω −
Cω K
tan φ = or tan φ = Cω
IR R Special Cases
41. POWER IN LCR CIRCUIT
Case (i) If the AC circuit contains pure resistance,
Average power over complete cycle, then φ = 0°.
W E0I0 E I
Pav = = cos φ = 0 × 0 cos φ = EvIv cos φ Ev
2
T 2 2 2 ∴ Pav = EvIv cos 0° = EvIv =
R
R
Also, Pav = EvIv Case (ii) If the AC circuit contains pure inductance,
F
+ G Lω −
1 I
2
then φ = 90°. ∴ Pav = 0
R2
H Cω JK Case (iii) If the AC circuit contains pure capacitance,
then φ = 90°. ∴ Pav = 0
Ev
or Pav = Ev × Case (iv) If the AC circuit contains L and R, then
FG 1 IJ 2

H
R2 + L ω −
Cω K cos φ =
R

2 R + L2ω 2
2
R Ev R
× =
F
+ G Lω −
1 I
2
FG 1 IJ 2
2

H Cω JK
R Ev R
R 2
H
R 2 + Lω −
Cω K ∴ Pav = EvIv
R2 + L2ω 2
=
R + L2ω 2
2

Average power is also


Case (v) If the AC circuit contains C and R, then
known as true power. The
2

quantity EvIv is called the appar-


– 1
Cw

R 2
Ev R
ent power or virtual power. cos φ = ∴ Pav =
1 1 1
Lw

It is customary to express true Lw – R2 + R2 + 2 2


Cw 2 2
Cω Cω
+
2

power in kW and apparent power


ÖR

in kVA.
f
cos φ is called the power fac-
R
tor of LCR circuit. Its value var-
ies from zero to 1. Fig. 39

42. IMPORTANT DATA ON AC CIRCUITS

S.No. AC circuit containing emf and correspond- Opposition Phase relation between Average Power factor
ing current offered emf and current power

1. Resistance only (R) E = E0 sin ωt R E and I in phase Iv2R cos φ = 1


I = I0 sin ωt
2. Inductance only (L) E = E0 sin ωt XL = ωL I lags behind E by 90° Zero cos φ = 0
I = I0 sin (ωt – 90°)

3. Capacitance only (C) E = E0 sin ωt 1 I leads E by 90° Zero cos φ = 0


I = I0 sin (ωt + 90°) XC =
ωC
ELECTROMAGNETIC INDUCTION AND ALTERNATING CURRENT 353

4. Inductance and resist- E = E0 sin ωt Current lags emf by EvIv cos φ R


ance (LR) I = I0 sin (ωt – φ) ZL = R2 + X L 2 cos φ =
XL R + X L2
2
angle φ. tan φ =
R
5. Capacitance and re- E = E0 sin ωt Current leads emf EvIv cos φ R
sistance (CR) I = I0 sin (ωt + φ) ZC = R2 + X C2 cos φ =
by angle φ. 2
R + X C2
XC
tan φ =
R
6. Inductance, capaci- E = E0 sin ωt If XL > XC, voltage leads
Z= R 2 + (X L − X C )2
tance and resist- I = I0 sin (ωt ± φ) current by phase angle
ance (LCR) φ. If XL < XC, voltage
lags current by phase
angle φ,
X − XC
tan φ = L
R
or
X − XL
tan φ = C
R
EvIv cos φ
cos φ
R
=
R2 + (X L − X C )2

43. WATTLESS CURRENT OR IDLE CURRENT So, there is no phase difference between current and
If the voltage and current differ in phase by π/2, then voltage.
This is case of resonance in AC circuit.
power factor, cos φ = cos 90° = 0
1 1 1
In this case, the current has no power. Such a cur- Now, Lω = or ω2 = or (2πf)2 =
Cω LC LC
rent is, therefore called wattless current. Since this cur-
rent does not perform any work therefore this current may 1 1
or f2 = or f =
also be called idle current. (2π )2 LC 2 π LC

44. RESONANCE
(a) Series Resonance Circuit
A series resonance circuit is that circuit in which in-
ductance L, capacitance C and resistance R are connected in
series. The impedance of this circuit has a minimum value
and the current through the circuit is maximum.

FG
R 2 + Lω −
1 IJ 2
Impedance in LCR circuit, Z =
H Cω K
1
Lω −
The phase angle φ is given by tan φ = Cω
R
1
When Lω = , then Z = R

In this case, LCR circuit is a purely resistive circuit Fig. 40
and has no impedance. The current in the circuit is maxi- which gives the frequency at which resonance occurs. This
mum. frequency is called series resonance frequency or simply,
Also, tan φ = 0 or φ = 0° resonance frequency. It is denoted by fR. The variation of
current with frequency is as shown in Fig. 40.
354 COMPREHENSIVE OBJECTIVE PHYSICS

(b) Parallel Resonance Circuit The peak of the curve corresponds to maximum power
In this circuit, coil of in- Pm at ωr. It is further clear from the graph that P is small
ductance L and a capacitor of for very low and very high frequencies.
capacitance C are connected in (ii) Half maximum power points. The points A
parallel with the source of AC and B on the graph of Fig. 43 have special significance. These
supply as shown in Fig. 41. Par-
points are known as ‘half maximum power points’. This is
allel resonance circuits are used
due to the fact that power corresponding to these points is
in radio receivers for the pur-
pose of tuning. A brief analyti- one-half of that corresponding to the peak of the graph.
cal treatment of parallel reso- Fig. 41 The ratio of resonance frequency and band width is
nant circuit is given here : known as quality factor (Q).
E0 FG π E IJ
+ 0 sin ωt +
π FG IJ ωr ω L
Current, I =
XL H
sin ωt −
2 XC K 2 H K ∴ Q=
2∆ω
= r
R
F 1 + 1I
= E0 cos ωt − GH X X JK
L C
Q factor is a measure of the sharpness of resonance.
Resonance will be sharp if the value of bandwidth (2∆ω) is
F 1 + CωIJ
cos ωt G −
small. This is of course possible only when the power-
= E0
H Lω K frequency curve falls steeply around ω = ωr.
(iii) Variation of current amplitude with angu-
1
When = Cω, then I = 0. lar frequency. Fig. 44 shows a graph of angular frequency

So, at parallel resonance fre- ω versus current amplitude I0. At the resonance frequency
ωr, we get the maximum current amplitude (Iom). Points
F = 1 I , the current is
quency GH 2π LC JK Fig. 42
where I0 =
Iom
are half maximum power points.
2
zero.
Clearly, parallel resonant circuit offers maximum
impedance at parallel resonant frequency. Parallel resonant
circuits are called rejector or filter circuits or anti-resonance
circuits.

45. THE Q-FACTOR


(i) Variation of power with angular frequency
in LCR series circuit

Fig. 44

(iv) Practical use of LCR circuits and the role


of Q-factor. LCR circuits find extensive use for tuning
purposes in radio set circuits. The radio receiver circuit
contains a variable capacitor. When we turn the knob, the
1
capacity of the variable capacitor changes. Consequently
LC
1
Fig. 43 changes. When the value of equals the angular
LC
R2 frequency of the incoming electromagnetic wave, the current
P = Pm
LM 2 F 1 I OP
2 induced by the electromagnetic wave becomes maximum and
R + G Lω − J resonance takes place. Loud sound is heard.
MN H Cω K PQ
ELECTROMAGNETIC INDUCTION AND ALTERNATING CURRENT 355

There are many radio stations in the world which S. No. Step-down transformer Step-up transformer
relay programmes on different but close frequencies. If the
Q-factor of the receiver circuit is not sufficiently high, then 1. It decreases the alternat- It increases the alternating
ing voltage. (Es < Ep) voltage. (Es > Ep)
it would be very difficult to tune them separately. As an
example, the Q-value should be of the order of 100 in a me- 2. Secondary current is Secondary current is weaker
dium wave receiver set. stronger than primary than primary current.
current. (Is > Ip) (Is < Ip)
46. TRANSFORMER
3. The number of turns in the The number of turns in the
primary is more than the secondary is more than the
number of turns in the number of turns in the pri-
secondary. (Np > Ns) mary. (Ns > Np)

4. The turns ratio


F I is
FK = Ns I GH
The turns ratio K =
Ns
JK
GH N p JK is less than Np
more than one.
one.

Fig. 45 5. The secondary coil is made The primary coil is made of


of thick wire. thick wire.
Let Ep = alternating emf applied to primary,
np = number of turns in the primary, 6. The impedance of second- The impedance of secondary
ary is less than that of pri- is more than that of primary.
dφ mary. (Zs < Zp) (Zs > Zp)
and = rate of change of magnetic flux through each turn
dt
of primary. E sI s E sI s
7. η= × 100% . η= × 100% .
E pI p E pI p

Then, Ep = – np …(i)
dt
Let Es and ns be the corresponding quantities for the 47. DC MOTOR
secondary coil so that (i) When the coil starts rotating in the magnetic field,
dφ it itself becomes a sort of a DC generator. The induced emf,
Es = – ns …(ii) as given by Lenz’s law, acts in a direction opposite to the
dt
direction of the applied emf E. This opposing emf set up dur-
Es n ing the rotation is e, the back emf. Thus the net current
Dividing (ii) by (i), we get = s = K (transforma- flowing in the coil of the armature, while the motor is run-
E p np
tion ratio) ning, is given by
Now, input electrical power = EpIp E−e
,
I= …(1)
output electrical power = EsIs r
where r is the resistance of the circuit. It is quite small in
∴ EpIp = EsIs (Law of conservation of energy) magnitude. Because of this back emf, the current in the
circuit is also quite small. However, a complication arises
Es I p just when we switch on the current. At that time, the motor
∴ =
E p Is is not operating and e = 0. Thus, the current at the start
If Es increases, then Is decreases. Thus whatever we E
Istart = …(2)
gain in voltage, we lose in current. r
Note. If R be the resistance of primary coil, then becomes quite large. It may actually burn out the motor. To
prevent this, we have a starting resistance, or a starter, R

Ep = – np + IpR. in series with the circuit. In the beginning, R is very high.
dt So as we switch on the motor, the current is given by
E
Istart = …(3)
r+R
356 COMPREHENSIVE OBJECTIVE PHYSICS

This current is of reasonable small value because of (iv) Efficiency of Motor delivering maximum out-
the increased resistance. As the motor starts picking up speed, put
we go on reducing the value of R, till at full speed R may be e
zero. In any model, the starter is automatic in functioning We know that η=
E
so that as we stop the motor, the moving arm of the starter
E
swings back automatically to the high R side. Thus the mo- When output is maximum, e=
tor is saved from damage. This is the need of the starter 2
resistance. E/2 1 1
∴ η= = or η = × 100 = 50%
(ii) Efficiency. It is ratio of the output mechani- E 2 2
cal power to the input mechanical power. So, a motor delivering maximum output has an effi-
Let, R = resistance of armature coil, E = applied emf, ciency of only 50%.
e = back emf, I = current in the armature (v)
E−e S. No. Generator/Dynamo Electric Motor
Now, I=
R
Electric power supplied by the external source i.e. in- 1. It converts mechanical en- It converts electrical energy
ergy into electrical energy. into mechanical energy.
put electrical power = EI
Power wasted as heat = I2R 2. It is based on the principle It is based on the fact that a
∴ Output mechanical power = EI – I2R of electromagnetic induc- torque acts on a current-car-
tion. rying coil placed in magnetic
I(E − IR) E − IR
∴ efficiency, η= = field.
EI E
e 3. Coil is rotated in a mag- A coil is placed in a magnetic
or η= (∵ e = E – IR) netic field to induce emf in field. A current is passed
E the coil. through the coil to rotate it.

back emf 4. The armature coil consists The armature coil consists of
∴ efficiency = of a large number of turns a large number of turns of in-
applied emf
of insulated copper wire. sulated copper wire.
It is clear from the above equation that the efficiency
increases with increase in back emf. If the back emf is equal
ILLUSTRATIONS INDICATING LATEST EXAMINA-
to the applied emf, then the efficiency is maximum i.e. 100%.
TION TRENDS
(iii) Condition for maximum output mechanical
power Illustration 7. As P
Q
Output mechanical power = EI – I2R shown in the Fig. 46, P and Q
are two coaxial conducting
= I(E – IR) = eI (∵ e = E – IR) loops separated by some dis- E
FG E − eIJ tance. When the switch S is
=e
H R K closed, a clockwise current IP
flows in P (as seen by E) and
S

For maximum output power, an induced current IQ1 flows in Battery


d LM FG IJ OP = 0
E− e LM
d eE e2 OP = 0 Q. The switch remains closed Fig. 46
N H KQ
e −
de R
or
MN
de R R PQ for a long time. When S is
opened, a current IQ2 flows in Q. Then the direction of IQ1
d L eE O d L e O 2
or − M P =0
de MN R PQ de MN R PQ
or
E 2e

R R
=0
and IQ2 (as seen by E) are
(a) respectively clockwise and anti-clockwise
E (b) both clockwise
or E = 2e or e= (c) both anti-clockwise
2
(d) respectively anti-clockwise and clockwise.
So, the motor delivers maximum mechanical power
[IIT Screening 2002]
when the back emf is half the applied emf. If this condition
Explanation. Using Lenz’s law, we find that the ris-
is satisfied, then the current in the armature may become
ing current induces inverse current and the diminishing
very large because the resistance is small. So, in actual
current induced similar current.
practice, we do not try to get maximum output mechanical
power. So, (d) is the right choice.
ELECTROMAGNETIC INDUCTION AND ALTERNATING CURRENT 357

Illustration 8. Two circular coils can be arranged (a) free electrons in the conductor drift from higher
in any of the three situations shown in Fig. 47. Their mu- potential end to lower potential end.
tual inductance will be (b) free electrons in the conductor drift from lower
potential end to higher potential end.
(c) free electrons in the conductor do not drift along
its length, since there is no electric field inside
the conductor.
(A) (B) (C) (d) free electrons in the conductor do not drift along
Fig. 47
its length, since the force on each electron due to
the electric field is balanced by the force produced
(a) maximum in situation (A) on it by the magnetic field.
(b) maximum in situation (B) [National Standard Exam. in Physics 2002]
(c) maximum in situation (C) Explanation. There is an electric field in the con-
(d) the same in all situations. [IIT Screening 2001] ductor.
Explanation. Maximum flux linkage in (a) is maxi- So, (a) is the right choice.
mum. Illustration 12. A short-circuited coil is placed in a
So, (a) is the right choice. time-varying magnetic field. Electrical power is dissipated
Illustration 9. A metallic due to the current induced in the coil. If the number of
square loop ABCD is moving in turns were to be quadrupled and the wire radius halved,
its own plane with velocity v in a A B the electrical power dissipated would be
uniform magnetic field perpen- (a) halved (b) the same
dicular to its plane as shown in 
v (c) doubled (d) quadrupled
Fig. 48. An electric field is induced
[IIT Screening 2002]
(a) in AD, but not in BC D C
2
(b) in BC, but not in AD e dB l
Explanation. P = , e = – NA ,R∝ 2
(c) neither in AD nor in BC R dt r
Fig. 48
(d) in both AD and BC. N2 r 2
∴ P∝
[IIT Screening 2001] l
→ →
Explanation. In the case of AB and DC, l || v . So, Clearly, P is unchanged when N is doubled and r is
no induced emf. halved.
So, (d) is the right choice. So, (d) is the right choice.
6W
Illustration 10. The figure × × × × × × × × Illustration 13. In the A
shows a wire sliding on two parallel × × × × × × × × circuit shown in Fig. 50, the a.c.
l v
conducting rails placed at a separation × × × × × × × source gives a voltage V = 20 cos 5 mH, 4 W 50 mF
× × × × × × × ×
l. A magnetic field B exists in a direc- (2000 t). Neglecting source resist-
× × × × × × × ×
tion perpendicular to the plane of the ance, the voltmeter and amme-
rails. The force required to keep the Fig. 49 V
ter readings will be
wire moving at a constant velocity v will be Fig. 50
(a) 0V, 0.47 A
µ Bv
(a) evB (b) 0 (b) 1.68 V, 0.47 A
4π l
(c) Blv (d) zero. (c) 0 V, 1.4 A (d) 5.6 V, 1.4 A.
[National Standard Exam. in Physics 2002] [KCET 2002]
Explanation. No change in flux. So, no force is Explanation. XL = Lω = 5 × 10–3 × 2000 Ω = 10 Ω
required. 1 1 106
XC = = = = 10 Ω
So, (d) is the right choice. Cω 50 × 10−6 × 2000 105
Illustration 11. A conductor of length l is moving Clearly XL = XC. This is a case of resonance.

perpendicular to a uniform magnetic field of induction B 20 2

Iv = = = 2 A = 1.44 A
and perpendicular to its length with a uniform velocity v . 2 × 10 2
As a result, an emf ε = Blv is induced between its ends. Voltmeter reading = 1.4 × 4 volt = 5.6 volt.
Under this condition, So, (d) is the right choice.
358 COMPREHENSIVE OBJECTIVE PHYSICS

QUESTION BANK

MCQs
SET I

based on
Memory Work, Hard Facts, Important Terms,
Important Formulae etc.

Average time allowed per question is 20 seconds.

5. Two inductors L1 and L2 are placed at sufficient dis-


ELECTROMAGNETIC INDUCTION tance from each other. If they are joined in parallel,
then the equivalent inductance is
1. A 0.1 m long conductor carrying a current of 50 A is
perpendicular to a magnetic field of 1.25 mT. The L 1L 2
(a) L1 + L2 (b)
mechanical power required to move the conductor L1 + L2
with a speed of 1 m s–1 is (c) L1L2 (d) L1 – L2 .
(a) 6.25 m W (b) 6.25 W
6. A horizontal straight conductor of length l placed
(c) 0.625 W (d) 1W. [AMU 1993] along south-north direction falls freely under grav-
2. A thin ring of radius r carries a uniformly distrib- ity. If H be the horizontal component of earth’s mag-
uted charge. The ring rotates at a constant angular netic field, then the induced emf along the length of
speed of n revolutions per second about an axis pass- the conductor at the instant when its velocity is v is
ing through centre and perpendicular to plane. If B (Given: δ = angle of dip)
be the magnetic field induction at the centre, then (a) zero (b) Hlv
the charge carried by the ring is
(c) Hlv tan δ (d) Hlv/tan δ. [DPMT 1993]
2rB µ 0n
(a) (b) 7. The energy stored in an inductance is 1 joule when a
µ 0n 2rB
current of 0.1 ampere is established in it. The self-
(c) µ0n (d) 2rB. [JIPMER 1998] inductance of the coil is
3. Two coils are placed close to each other. The mutual (a) 200 H (b) 50 H
inductance of the pair of coils depends upon (c) 25 H (d) 2.59 H. [AFMC 1999]
(a) the current in the two coils
8. The areas of cross-section of two solenoids A and B
(b) the rates at which currents are changing in the two coils
of equal length are equal but the number of turns in
(c) relative position and orientation of the two coils the solenoid A is double than that of B. If the coeffi-
(d) the materials of the wires of the coils [AIEEE 2003] cient of self-induction of B is 6 mH, then the coeffi-
4. The magnetic flux through a circuit of resistance R cient of self-induction of A will be
changes by an amount ∆φ in a time ∆t. Then the total (a) 12 mH (b) 3 mH
quantity of electric charge Q that passes any point in (c) 24 mH (d) 1.5 mH. [BHU 2000]
the circuit during the time ∆t is represented by
9. A metal conductor of length 1 m rotates vertically
∆φ ∆φ about one of its ends at angular velocity 5 radian per
(a) Q = (b) Q =
R ∆t second. If the horizontal component of earth’s mag-
∆φ 1 ∆φ netic field is 0.2 × 10 – 4 T, then the emf developed
(c) Q = R. (d) Q = .
∆t R ∆t between the two ends of the conductor is
[All India PM/PD 2004] (a) 5 mV (b) 50 µV
(c) 5 µV (d) 50 mV [AIEEE 2004]
ELECTROMAGNETIC INDUCTION AND ALTERNATING CURRENT 359

10. A copper ring is held horizontally and a bar magnet 17. What voltage is developed across the axle of the wheels
is dropped through the ring with its length along the of a train ? Given : speed of train = 72 km h–1, the
axis of the ring. The acceleration of the falling mag- horizontal component of earth’s magnetic field = 0.40
net is × 10 –4 T, angle of dip = 30°, length of axle = 1.5 m.
(a) equal to that due to gravity
20 × 0.4 × 1.5 × 10 −4
(b) less than that due to gravity (a) volt (b) 3 × 20 × 0.4 × 5 × 10 –4 V
3
(c) more than that due to gravity
(c) 0 (d) 0.4 × 10 –4 × 72 × 0.1 V.
(d) depends on the diameter of the ring and the length of the
[Pb. CET 1997]
magnet. [MP PMT 2001 ; MP PET 1990]
18. A coil of area 5 cm2
and having 20 turns is placed in
11. In an induction coil, the coefficient of mutual induc-
a uniform magnetic field of 103 gauss. The normal to
tion is 4 henry. It a current of 5 ampere in the pri-
1 the plane of the coil makes an angle of 60° with the
mary coil is cut off in s, the emf at the termi- magnetic field. The flux in maxwell through the coil
1500
nals of the secondary coil will be is
(a) 15 kV (b) 60 kV (a) 105 (b) 5 × 104
(c) 10 kV (d) 30 kV. (c) 2 × 104 (d) 5 × 103. [CPMT 1998]
[Bharati Vidyapeeth 1997] 19. When a metallic sphere is moved in a magnetic field,
12. The coefficient of mutual induction of two coils is it gets heated due to
6 mH. If the current flowing in one coil is 2 ampere, (a) direct current (b) eddy currents
then the induced emf in the second coil will be (c) alternating current (d) additional current.
(a) 12 mV (b) 3 mV
20. When a magnet is being moved towards a coil, the
(c) 3 V (d) zero. [Pb. CET 1996] induced emf does not depend upon
13. Direction of induced emf is determined by (a) the number of turns of the coil
(a) Fleming’s left hand rule (b) Fleming’s right hand rule (b) the motion of the magnet
(c) Maxwell’s rule (d) Ampere’s rule of swimming.
(c) the magnetic moment of the magnet
[SCRA 1996]
(d) the resistance of the coil. [Manipal 1996]
14. The momentum in mechanics is expressed as m × v.
The analogous expression in electricity is 21. If the rate of change of current in the coil is unity,
(a) I × Q (b) I × V
then the induced emf is equal to
(a) coefficient of self-induction
(c) L × I (d) L × Q. [BITS 1994]
(b) magnetic flux linked with the coil
15. A copper ring is sus-
pended by a thread in (c) number of turns in the coil
a vertical plane. The (d) thickness of the coil.
north pole of a magnet 22. The ‘henry’ is a unit of
is brought near the
(a) capacitance (b) magnetic field
ring in horizontal di-
rection as shown in Fig. 51 (c) inductance (d) resistance.
Fig. 51. What will be ef- [CMC Vellore 1993]
fect on the ring ? 23. Keeping all other factors same, the number of turns
(a) Ring will be attracted towards the magnet
of a coil is doubled. The magnetic flux linked with
(b) Ring will be repelled away the coil is
(c) Ring will make simple harmonic motion (a) doubled (b) halved
(d) No change in the position of ring. [WB JEE 1995] (c) trebled (d) quadrupled.
16. The average emf induced in a coil in which the cur- [JIPMER 1995]
rent changes from 2 ampere to 4 ampere in 0.05 sec-
24. Induction furnace is based on the heating effect of
ond is 8 volt. What is the self inductance of the coil ?
(a) eddy current (b) magnetic field
(a) 0.1 H (b) 0.2 H
(c) electric field (d) gravitational field.
(c) 0.4 H (d) 0.8 H. [MP PMT 1986]
[AFMC 2002]
360 COMPREHENSIVE OBJECTIVE PHYSICS

25. A coil having an area A0 is placed in a magnetic field 34. The north pole of a long horizontal bar magnet is
which changes from B0 to 4 B0 in time interval t. The being brought closer to a vertical conducting coil along
emf induced in the coil will be the perpendicular direction. The direction of the in-
(a) 3 A0 B0/t (b) 4 A0 B0/t duced current in the conducting coil will be
(c) 3 B0/A0t (d) 4 B0/A0t. [MNR 2002] (a) Horizontal (b) Vertical
(c) Clockwise (d) Anticlockwise.
26. To reduce the loss of energy as heat due to eddy
currents, the soft iron core is laminated. The angle [MP PMT 1994]
between the planes of these sheets and the magnetic 35. Which of the following does not depend upon the
induction is magnetic effect of some sort ?
(a) zero (b) 45° (a) Moving coil galvanometer (b) Hot wire ammeter

(c) 60° (d) 90°. [MP PMT 1985] (c) Dynamo (d) Electric motor

27. When a wire loop is rotated in a magnetic field, the [AFMC 1993]
direction of induced emf changes once in each 2
36. A coil of area 80 cm and 50 turns is rotating with
(a) 1 revolution (b) 1 revolution 2000 revolutions per minute about an axis perpen-
4 2
dicular to a magnetic field of 0.05 T. The maximum
(c) 1 revolution (d) 2 revolutions.
value of the emf developed in it is
[MP PMT 1991]
10π
28. A current increases from zero to 1 ampere in 0.1 sec- (a) 200 π V (b) V
ond in a coil of 5 mH. Then the magnitude of the 3
induced emf will be 4π 2
(a) 0.005 V (b) 0.05 V (c) V (d) V. [MP PMT 1994]
3 3
(c) 0.5 V (d) 5 V.
37. Fig. 52 shows two bulbs B1 and B2 , resistor R and
[MP PET/PMT 1998]
an inductor L. When the switch S is turned off,
29. The unit of magnetic flux is
(a) Wb m–2 (b) Wb
(c) H (d) Am–1.
[MP PMT 1994 ; MP PET 1995]
30. The current passing through a choke coil of 5 henry
is decreasing at the rate of 2 A s–1. The emf develop-
ing across the coil is
(a) 10 V (b) – 10 V
(c) 2.5 V (d) – 2.5 V. [MP PMT 1990]
31. In a circuit, the induced emf increases if
(a) the flux linked are more
(b) the rate of change of flux is greater
(c) there is no change in the flux Fig. 52
(d) none of the above. [Manipal 1994] (a) both B1 and B2 die out promptly.
32. Lenz’s law is consequence of the law of conservation (b) both B1 and B2 die out with some delay.
of (c) B1 dies out promptly but B2 with some delay.
(a) Charge (b) Momentum
(d) B2 dies out promptly but B1 with some delay.
(c) Mass (d) Energy.
[MP PMT 2000] [CPMT 1998]
33. A metallic rod falls under gravity with ends pointing 38. An emf of 5 volt is produced by a self inductance,
in the direction east and west. Then when the current changes at a steady rate from 3 to
(a) an emf is induced in it as it cuts H 2 ampere in 1 millisecond. The value of self induct-
(b) no emf is induced at all ance is
(c) two emf’s of equal but opposite signs are induced giving no (a) zero (b) 5 henry
net emf.
(c) 5000 henry (d) 5 millihenry.
(d) the gravitational field opposes its downward motion.
[BHU 1994] [MP PMT 1991]
ELECTROMAGNETIC INDUCTION AND ALTERNATING CURRENT 361
39. An electron moves along the Y Y
line AB, which lies in the
same plane as a circular loop
of conducting wire as shown
in the diagram. What will be
the direction of current ind- |e| |e|
uced if any, in the loop ? Fig. 53
(a) No current will be induced
(b) The current will be clockwise O 1 2 X O 1 2 X
(c) The current will be anticlockwise t t

(d) The current will change direction as the electron pass by. (c) (d)

[AIIMS 1987] Fig. 55


40. In electromagnetic induction, the induced charge in [WB JEE 2002]
a coil is independent of
43. The coil of area A is kept perpendicular in a magnetic
(a) Change in the flux (b) Time
field B. If coil is rotated by 180°, the change in flux
(c) Resistance in the circuit (d) None of these. will be
[AIIMS 1982] (a) BA (b) Zero
41. The magnetic flux through a circuit of resistance R (c) 2BA (d) 4BA.
changes by an amount ∆φ in time ∆t. Then the total [Haryana PMT 1994]
quantity of electric charge q which passing during 44. A conducting rod of length l is falling with a velocity
this time through any point of the circuit is given by v perpendicular to a uniform horizontal magnetic field
∆φ ∆φ B. The potential difference between its two ends will
(a) q = (b) q = ×R
∆t ∆t be
∆φ ∆φ (a) 2 Blv (b) Blv
(c) q = – +R (d) q = . [AIIMS 1986]
∆t R 1
(c) Blv (d) B2l2v2. [MP PMT 1994]
42. A flexible wire bent in the form of a circle is placed in 2
a uniform magnetic field perpendicular to the plane 45. A conducting square loop of side L and resistance R
of the coil. The radius of the coil changes as shown moves in its plane with a uniform velocity v perpen-
in Fig. 54. The graph [Fig. 55] of magnitude of in- dicular to one of its sides. A magnetic induction B
duced emf in the coil is represented by constant in time and space, pointing perpendicular
and into the plane of the loop exists everywhere. The
current induced in the loop is
× × × × × ×
× × ×
× × × × ×
® ®
× × × × × v × B
× × × × ×

× × × × × ×

Fig. 54 Fig. 56
Blv Blv
Y Y (a) clockwise (b) anticlockwise
R R
2Blv
(c) anticlockwise (d) zero. [IIT 1989]
R
46. “The direction of the induced current in a circuit is
|e| |e| always such that it opposes the cause due to which it
is produced.” This Law is named as
(a) Ohm’s law (b) Lenz’s law
O 1 2 X O 1 2 X
(c) Kirchhoff ’s law (d) Faraday’s law.
t t
[MP PET 1996]
(a) (b)
362 COMPREHENSIVE OBJECTIVE PHYSICS

47. An emf of 100 millivolt is induced in a coil when the 50. A copper ring is suspended by a long, light rod piv-
current in another nearby coil becomes 10 ampere oted at X so that it may swing as a pendulum, as
from zero in 0.1 second. The coefficient of mutual shown in the diagram below. An electromagnet is
induction between the two coils will be mounted so that the ring passes over it as it swings.
(a) 1 millihenry (b) 10 millihenry
The ring is set into oscillation with switch K open.
(c) 100 millihenry (d) 1000 millihenry.
What happens to the motion after switch K has been
[MP PET 1996] closed ?
48. X and Y are solenoids wound on cardboard tubes. X (a) The periodic time will decrease.
carries a constant current I as shown below and
(b) The amplitude will increase because the ring is acceler-
moves with constant speed away from Y along the ated towards the magnet.
common axis of the two tubes.
(c) The ring will be brought to rest with the rod inclined to the
As a result of electromagnetic induction, a current
vertical.
will flow in the straight wire MN and there will be a
force between X and Y. Which one of the following X
correctly describes both the current and the force ?
Suspending rod
X

Copper ring

Electromagnet
I

K
Fig. 58
(d) The oscillation will continue at constant amplitude while
the battery can supply energy
(e) The oscillation will be heavily damped.
M N 51. The diagram represents an aircraft of length l, wing-
span, x, flying horizontally at speed v in a region
Fig. 57 where the Earth’s magnetic field, of uniform flux
Nature and direction of Nature of density B, is inclined at an angle θ to the Earth’s
current in straight wire MN force surface.
(a) diminishing, N to M attraction
(b) diminishing, M to N repulsion
(c) diminishing, N to M repulsion
(d) constant, M to N repulsion
(e) constant, N to M attraction. x
49. A 10 m long copper wire while remaining in the east-
west horizontal direction is falling down with a speed
of 5.0 ms–1. If the horizontal component of the earth’s l
v
magnetic field is 0.3 × 10 –4 weber/m2, the emf devel- θ
oped between the ends of the wire is
(a) 0.15 volt (b) 1.5 volt B

(c) 0.15 millivolt (d) 1.5 millivolt.


[MP PET 2000] Fig. 59
ELECTROMAGNETIC INDUCTION AND ALTERNATING CURRENT 363

Which expression gives the magnitude of the emf gen- 55. An artificial satellite with a metal surface is moving
erated between the wingtips by electromagnetic in- about the earth in a circular orbit. A current will be
duction ? induced in the satellite, if
(a) Blv sin θ (b) Bxv (a) the plane of the orbit coincides with the plane of the equa-
tor.
(c) Blv (d) Bxv sin θ.
(b) the plane of the orbit is inclined to the plane of the equator.
52. In Fig. 60 (a) and Fig. 60 (b), two air-cored solenoids (c) the plane of the orbit coincides with the plane of satellite
P and Q have been shown. They are and the speed of satellite is more than 8 km/s
(d) the plane of the orbit coincides with the plane of the equa-
tor and the speed of satellite is less than 8 km s–1.
[AIIMS 2001]
56. A force of 10 newton is required to move conducting
loop through a non uniform magnetic field at 2 m s–1.
The rate of production of internal energy in watt in
loop is
Fig. 60 (a) (10/2) × 2 (b) 10/2
(c) 10 (d) 10 × 2.
placed near each other. In Fig. 60 (a), when Ip , the
current in P, changes at the rate of 5 A s–1, an emf of [All India PM/PD 1993]
2 mV is induced in Q. The current in P is then 57. A solenoid has n turns. Its coefficient of self induct-
switched off, and a current changing at 2 A s–1 is fed ance L varies with n as
through Q as shown in diagram. What emf will be (a) L ∝ n (b) L ∝ n2
induced in P ? (c) L ∝ n–1 (d) L ∝ n–2. [DPMT 2000]
(a) 8 × 10 –4 V (b) 2 × 10 –3 V 58. A conducting rod y Q
(c) 5 × 10 –3 V (d) 8 × 10 –2 V PQ move parallel to
× × ×
(e) 8 × 10 –1 V. x-axis in a uniform
magnetic field, ®
53. When the current in a coil is reduced uniformly from v
pointing in the nega- × × ×
20 A to zero in 0.50 s, an emf of 4.0 V is induced in a
tive z-direction. The
nearby coil. What is the value of the mutual induct-
end P of the rod gets × × ×
ance between the two coils ?
(a) 0.10 H (b) 0.40 H (a) positively charged P
O x
(c) 2.5 H (d) 40 H (b) negatively charged Fig. 61

(e) 160 H. (c) neutral


(d) first positively charged and then negatively charged.
54. The north pole of a bar magnet is moved swiftly down-
ward towards a closed coil and then second time it is [Himachal PMT 1999]
raised upwards slowly. The magnitude and direc- 59. As shown in the figure, a magnet is moved with a
tion of the induced currents in the two cases will be fast speed towards a coil at rest. Due to this, induced
of electromotive force, induced current and induced
First case Second case charge in the coil are E, I and Q respectively. If the
(a) low value higher value speed of magnet is doubled, the incorrect statement
clockwise anti-clockwise is
(b) low value equal value S N
clockwise anti-clockwise
(c) higher value low value
clockwise clockwise
(d) higher value low value G
anti-clockwise clockwise Galvanometer
[MP PET 1996] Fig. 62
364 COMPREHENSIVE OBJECTIVE PHYSICS

(a) E increases (b) I increases


TRANSIENT CURRENT
(c) Q remains the same (d) Q increases.
AND ALTERNATING CURRENT
[MP PET 1995]
60. A circular coil of radius 5 cm has 500 turns of a wire. 64. In a LCR series circuit, at resonant frequency
The approximate value of the coefficient of self-in- (a) the impedance and the current are maximum.
duction of the coil will be
(b) the impedance is maximum.
(a) 25 millihenry (b) 25 × 10–3 millihenry
(c) the current and voltage are out of phase.
(c) 50 × 10–3 millihenry (d) 50 × 10–3 henry.
(d) the current is maximum.
[MP PET 1996]
(e) the current and voltage are maximum.
61. Which of the following statements is correct ?
(a) A circular ring is rotated about its own axis in a uniform
[Kerala PMT 2003]
magnetic field. No emf is induced in it. 65. A choke is a :
(b) A circular ring is rotated about its own axis in a non- (a) capacitor with a large self inductance
uniform magnetic field. An emf is induced in it. (b) resistor with a large self inductance
(c) A circular ring is rotated about one of its diameter in a (c) inductor with a large self inductance
uniform magnetic field and induction of the magnetic field
(d) inductor with a small self inductance
is increasing at a constant rate. emf, induced in the ring
may be equal to zero for an elemental time interval (e) capacitor with a large self capacitance

(d) None of the above. [CMC LDH 2002] [Kerala PMT 2003]
62. A rectangular coil ABCD is rotated anticlockwise with 66. In an ideal parallel LC circuit, the capacitor is charged
a uniform angular velocity about the axis shown in by connecting it to a dc source which is then discon-
the figure. The axis of rotation of the coil as well as nected. The current in the circuit
the magnetic field B are horizontal. The induced emf (a) becomes zero instantaneously.
in the coil would be minimum when the plane of the (b) grows monotonically.
coil (c) decays monotonically.
(d) oscillates instantaneously. [AIIMS 2003]
67. In an oscillating LC circuit the maximum charge on
the capacitor is Q. The charge on the capacitor when
the energy is stored equally between the electric and
magnetic field is
Q
(a) Q (b)
2
Q Q
(c) (d) [AIEEE 2003]
3 2

Fig. 63 68. A resistance of 40 ohm and an inductance of 95.5


millihenry are connected in series in a 50 cycle/
(a) is horizontal.
second AC circuit. The impedance of this combination
(b) makes an angle of 45° with the direction of magnetic field.
is very nearly
(c) is at right angle to the magnetic field.
(a) 30 ohm (b) 40 ohm
(d) makes an angle of 30° with the magnetic field.
(c) 50 ohm (d) 60 ohm.
[CPMT 1988]
[MP PET 2000]
63. In an induction coil, the coefficient of mutual
69. The power factor in an AC series L-R circuit is
induction is 5 H. If current of 5 A in the primary coil
is cut off at 10 –3 s, the emf at the terminals of the (a) R2 + L2ω 2 (b) L/R
secondary coil will be
(a) 2.5 kV (b) 25 kV (c) R R2 + L2ω 2 (d) R R2 + L2ω 2 .
(c) 2.51 kV (d) zero. [Raj. PET 1996]
[MP PET 2000]
ELECTROMAGNETIC INDUCTION AND ALTERNATING CURRENT 365
70. Alternating current can not be measured by DC am- 79. The average value of alternating current over one
meter because complete cycle is
(a) average value of current for complete cycle is zero. I max
(a) (b) 2 I max
(b) AC changes direction. 2
(c) AC can not pass through DC ammeter. (c) 0.63 Imax (d) zero. [Manipal 1996]
(d) DC ammeter will get damaged. [AIEEE 2004] 80. An AC source is connected to resistive circuit. What
71. In a LCR circuit capacitance is changed from C to of the following is true ?
2 C. For the resonant frequency to remain unchanged, (a) Current leads ahead of voltage in phase.
the inductance should be changed from L to (b) Current lags behind voltage in phase.
(a) L/2 (b) 2 L (c) Current and voltage are in same phase.
(c) 4 L (d) L/4 [AIEEE 2004] (d)Any of the above may be true depending upon the value of
resistance. [All India PM/PD 1996]
72. In an LCR series ac circuit, the voltage across each
of the components, L, C and R is 50 V. The voltage 81. The peak voltage of 220 V AC mains is
across the LC combination will be (a) 155.6 V (b) 220 V

(a) 100 V (b) 50 2 V (c) 311 V (d) 440 V.


[MP PET 2000 ; MP PMT 1999]
(c) 50 V (d) 0 V (zero) [AIEEE 2004]
82. The time constant of LR circuit is
73. A coil of inductance L has an inductive reactance of
R L
XL in an AC circuit in which the effective current is I. (a) (b)
L R
The coil is made from superconducting material and 2L L
has no resistance. The rate at which power is dissi- (c) (d) 0.632 .
R R
pated in the coil is 83. What is time constant of CR circuit ?
(a) 0 (b) I XL C
(a) CR (b)
(c) I2 XL (d) I XL2. [MP PMT 1999] R
74. If L and R denote inductance and resistance respec- C+R R
(c) (d) .
tively, which of the following has the dimensions of R C
frequency ? 84. Current in the LCR circuit becomes extremely large
R L
when
(a) (b) (a) frequency of AC supply is increased
L R
(b) frequency of AC supply is decreased
R L (c) inductive reactance becomes equal to capacitive reactance
(c) (d) . [MP PMT 2000]
L R (d) inductance becomes equal to capacitance.
75. Given : E = 100 sin (100t) volt and I = 100 sin [MP PMT 1995]

FG 100 t + π IJ mA. Power is 85. A resonant AC circuit contains a capacitor of capaci-


H 3K
tance 10–6 farad and an inductor of inductance 10–4
henry. Then the frequency of electrical oscillations is
(a) 104 W (b) 10 W (a) 105 Hz (b) 10 Hz
(c) 2.5 W (d) 5 W. 5
10 10
76. The rate of heating of 10 A AC is the same as the rate (c) Hz (d) Hz.
2π 2π
of heating of DC of
86. The diagram shows a
(a) 10 2 A (b) 10 A source of alternating
10 voltage connected to a
(c) A (d) 5 A.
2 capacitor and a resistor.
77. An AC series circuit contains 40 Ω of resistance and Which of the following
30 Ω of inductive reactance. Impedance of circuit is phasor diagrams cor-
(a) 70 Ω (b) 10 Ω rectly describes the
phase relationship be-
(c) 50 Ω (d) 70 2 Ω . tween Ic, the current be-
78. The frequency of AC mains in India is tween the source and Fig. 64
(a) 30 Hz (b) 50 Hz the capacitor, and I R ,
(c) 500 Hz (d) 5000 Hz. the current in the resistor ?
366 COMPREHENSIVE OBJECTIVE PHYSICS

13 A rms. When the ac


supply is replaced with
a 120 V dc source, an
identical fuse breaks
the circuit if the current
just exceeds
13 Fig. 67
(a) A (b) 13 A
2
13
(c) A (d) 13 2 A
2
(e) 26 A.
1
91. The inductive reactance of an inductor of H at 50 Hz
Fig. 65 π
frequency is
87. Which of the following plots may represent the reac- 50 π
tance of a series LC combination ? (a) Ω (b) Ω
π 50
(c) 100 Ω (d) 50 Ω. [MP PMT 2001]
I
92. A generator provides an output voltage V = V 0 sin
2πt, the amplitude V0 is directly proportional to f.
Reactance

When the output terminals are connected to a resis-


II III
tor and the frequency is varied, the amplitude of the
Frequency current is
(a) inversely proportional to f 2
(b) inversely proportional to f
IV
(c) independent of f (d) proportional to f
Fig. 66 (e) proportional to f 2.

(a) I (b) II 93. A constant current of 2.8 A exists in a resistor. The


(c) III (d) IV. rms current is
(a) 2.8 A (b) about 2 A
88. The ratio of mean value over half cycle to rms value
(c) 1.4 A
of AC is
(d) undefined for a direct current.
(a) 2 : π (b) 2 2 : π
94. A coil of inductance 40 henry is connected in series
(c) 2:π (d) 2 : 1. with a resistance of 8 ohm and the combination is
joined to the terminals of a 2 volt battery. The time
89. A capacitor of capacitance C farad is charged by a
constant of the circuit is
battery of emf V0 volt. The battery is then discon-
(a) 40 second (b) 20 second
nected and a pure inductor of L henry is connected
across it so that LC oscillations are set up. Then the (c) 8 second (d) 5 second.
value of the maximum current in the inductor coil [All India PM/PD 2004 ; MP PET 2000]
is 95. A steady current I dissipates a certain power in a
V0 variable resistor. The resistance has to be halved to
(a) V0 LC (b)
LC obtain the same power when a sinusoidal alternat-
ing current is used. What is the rms value of the
C L
(c) V0 (d) V0 . alternating current ?
L C
(a) 1/2 I (b) 2I
90. When an ac supply of 240 V rms is connected to the
(c) 1/2 I (d) 2I
terminals PQ in the circuit shown in Fig. 67, the
fuse F breaks the circuit if the current just exceed (e) I.
ELECTROMAGNETIC INDUCTION AND ALTERNATING CURRENT 367
96. A sinusoidal current is represented by the equation What is the frequency and the phase relationship
I = I0 sin (θt) between the voltages ?
Which equation represents the sinusoidal current with frequency/Hz phase lead of N over M/rad
both its frequency and amplitude doubled ? π
(a) 0.4 −
(a) 2I = I0 sin (2θt) (b) I = I0 sin (2θt) 4
(c) I = 2I0 sin (2θt) (d) I = I0 sin ( 21 θt) π
(b) 2.5 −
2
(e) I = 2I0 sin ( 21 θt) . π
(c) 2.5 +
97. An AC source is rated 220 V, 50 Hz. The average 2
voltage is calculated in a time interval of 0.01 s. It π
(d) 2.5 −
(a) must be zero (b) may be zero 4
(c) is never zero (d) is (200/ 2 ) V. π
(e) 2.5 + .
98. An AC supply is connected to a resistor. When the 4
peak value of the emf of the supply is V0 and the 102. A capacitor acts as an infinite resistance for
frequency is f, the mean power dissipated in the re- (a) DC (b) AC
sistor is P. The supply frequency is then changed to (c) DC as well as AC (d) neither AC nor DC.
2f, the peak value of the emf remaining as V0 . What 103. Which of the following curves correctly represents
is now the mean power in the resistor ? the variation of capacitive reactance (X C) with fre-
(a) P (b) 2 P quency f ?
(c) 2P (d) 4P.
99. A group of electric lamps having a total power rating
of 900 watt is supplied by an AC voltage
e = 300 sin (314t + 60) . Then the rms value of the
circuit current is
XC XC
(a) 12 A (b) 12 2 A
(c) 6 2 A (d) 6 A. [Pb. PMT 2002]
100. The power dissipated in a resistor is the same for a f f
constant potential difference V as for a sinusoidal (a ) (b)
potential difference with peak value V0 . Which of the
following is the correct relationship between V and
V0 ?
(a) V0 = V/2 (b) V0 = 2 V

(c) V0 = V/ 2 (d) V0 = 2V XC XC
(e) V0 = V.
101. Two sinusoidal voltages of the same frequency are
shown in Fig. 68. f f
(c) (d )

Fig. 69

104. A sinusoidal alternating current of peak value I 0


passes through a heater of resistance R. What is the
mean power output of the heater ?
I 2R I 2R
(a) 0 (b) 0
2 2
(c) I02 R (d) 2 I0 2 R
Fig. 68 (e) 2I02 R.
368 COMPREHENSIVE OBJECTIVE PHYSICS

105. Power factor is one for 110. The armature current in a DC motor is maximum
(a) pure resistor (b) pure inductor when the motor has
(c) pure capacitor (a) picked up maximum speed
(d) either an inductor or a capacitor. (b) just started
106. A capacitor of capacitance 1µ F is used in an alter- (c) intermediate speed
106 (d) just been switched off.
nating current circuit of frequency Hz. The
2π 111. A step-down transformer is employed to reduce the
capacitative reactance is main supply of 200 V to 11 V. The primary draws 5A
106 of current and the secondary draws 90 A. The effi-
(a) 10–6 Ω (b) Ω
2π ciency of the transformer is
1 (a) 95% (b) 90%
(c) Ω (d) 1 Ω.
2π (c) 88% (d) 40%. [MP PMT 2001]
200 112. The armature of a DC motor has 20 Ω resistance. It
107. An AC source is of volt, 50 hertz. The value of
2 draws a current of 1.5 A when run by 200 V DC sup-
1 ply. The value of back emf induced will be
voltage after second from the start is
600 (a) 150 V (b) 170 V
200 (c) 180 V (d) 190 V.
(a) 200 volt (b) volt
2
113. The core of a transformer is laminated to reduce en-
(c) 100 volt (d) 50 volt.
ergy losses due to
[JIPMER 1994] (a) Eddy currents (b) Hysteresis
108. An inductor L, a capacitor C and ammeters A1, A2 (c) Resistance in winding (d) None of these.
and A3 are connected to an oscillator in the circuit as
[Karnataka CET 2001 ; MP PMT 2000]
shown in Fig. 70. When the frequency of the oscillator
is increased, then at resonant frequency, the ammeter 114. Large transformers, when used for sometime, become
reading is zero in the case of hot and are cooled by circulating oil. The heating of
transformer is due to
C (a) heating effect of current alone.
A1 (b) hysteresis loss alone.
(c) both the hysteresis loss and heating effect of current.
(d) none of the above. [MP PMT 2001]
A2
115. If the angular speed of rotation of an armature of
L
A3 alternating current generator is doubled, then the
induced electromotive force will be
(a) twice (b) four times
(c) eight times (d) sixteen times.
Fig. 70 116. There is a current of 10 milliampere through an ideal
choke coil when connected to 220 volt, 50 hertz sup-
(a) Ammeter A1 (b) Ammeter A2
ply. The power consumed is
(c) Ammeter A3 (d) all the three ammeters.
(a) zero (b) 220 × 10 × 10–3 watt
[Bharati Vidyapeeth 2001]
220 × 103
(c) 220 × 50 watt (d) watt.
10
ELECTRICAL MACHINES AND DEVICES [BHU 1999]

109. The back emf in a DC motor is maximum when 117. The armature of an eight polar dynamo rotates mak-
ing 750 revolutions per minute. The frequency of AC
(a) the motor has picked up maximum speed.
produced is
(b) the motor has just started moving.
(a) 8 Hz (b) 4 Hz
(c) the speed of motor is still on the increase.
(d) the motor has just been switched off. (c) 12.5 Hz (d) 50 Hz.
ELECTROMAGNETIC INDUCTION AND ALTERNATING CURRENT 369

118. A device which converts electrical energy into me- 124. A transformer steps down 200 volt to 22 volt to operate
chanical energy is a device with an impedance of 220 ohm. Then the
(a) induction coil (b) electric motor current drawn from the mains by the primary of the
transformer, is
(c) generator (d) dynamo.
(a) 1 A (b) 0.1 A
[Karnataka 1999] (c) 0.01 Α (d) 0.001 A.
119. The ratio of the number of turns in primary and sec- [All India PM/PD 1997]
ondary coils of a transformer is 1 : 20. The ratio of 125. In a transformer, the number of turns of primary
the currents in the primary and secondary coils will coil and secondary coil are 5 and 4 respectively. If
be 220 V is applied on the primary coil, then the ratio of
(a) 1 : 20 (b) 20 : 1 primary current to the secondary current is
(c) 1 : 400 (d) 400 : 1. (a) 9 : 5 (b) 5 : 9

120. Energy is stored in the choke coil in the form of (c) 5 : 4 (d) 4 : 5. [BHU 2001]
(a) Heat (b) Electric energy 126. The number of twins in primary and secondary
windings of a transformer are 1000 and 100
(c) Magnetic energy (d) Electro-magnetic energy.
respectively. If 200 V dc voltage is impressed across
[MP PMT 2000] the primary terminals, the voltage across the
121. In a step-up transformer, the turns ratio is 1 : 2. A secondary terminals is
Leclanche cell (emf 1.5 V) is connected across the
primary. The voltage across the secondary is
(a) 3.0 V (b) 0.75 V
(c) zero (d) 1.5 V. [MP PMT 2000]
122. The core of any transformer is laminated so as to
(a) increase the secondary voltage.
(b) reduce the energy loss due to eddy currents.
(c) make it light weight.
(d) make it robust and strong. [AIEEE 2003]
123. The ratio of the secondary to the primary turns in a
transformer is 3 : 2 and the output power is P. Ne-
glecting all power losses, the input power must be
(a) P/2 (b) P
Fig. 71
(c) 2P/3 (d) 3P/2
(a) zero (b) 22 V
[Karnataka CET 2003] (c) 220 V (d) 2200 V.

Answers (Set I)
1. (a) 2. (a) 3. (c) 4. (a) 5. (b) 6. (a) 7. (a) 8. (c)
9. (b) 10. (b) 11. (d) 12. (d) 13. (b) 14. (c) 15. (b) 16. (b)
17. (a) 18. (b) 19. (b) 20. (d) 21. (a) 22. (c) 23. (a) 24. (a)
25. (a) 26. (a) 27. (b) 28. (b) 29. (b) 30. (a) 31. (b) 32. (d)
33. (a) 34. (d) 35. (b) 36. (c) 37. (c) 38. (d) 39. (c) 40. (b)
41. (d) 42. (b) 43. (c) 44. (b) 45. (d) 46. (b) 47. (a) 48. (a)
49. (d) 50. (e) 51. (d) 52. (a) 53. (a) 54. (d) 55. (b) 56. (d)
57. (b) 58. (b) 59. (d) 60. (a) 61. (a) 62. (c) 63. (b) 64. (d)
370 COMPREHENSIVE OBJECTIVE PHYSICS

65. (c) 66. (d) 67. (d) 68. (c) 69. (d) 70. (a) 71. (a) 72. (d)
73. (a) 74. (a) 75. (c) 76. (b) 77. (c) 78. (b) 79. (d) 80. (c)
81. (c) 82. (b) 83. (a) 84. (c) 85. (c) 86. (a) 87. (d) 88. (b)
89. (c) 90. (b) 91. (c) 92. (d) 93. (a) 94. (d) 95. (b) 96. (c)
97. (b) 98. (a) 99. (c) 100. (b) 101. (b) 102. (a) 103. (c) 104. (a)
105. (a) 106. (d) 107. (c) 108. (c) 109. (a) 110. (b) 111. (b) 112. (d)
113. (a) 114. (c) 115. (a) 116. (a) 117. (d) 118. (b) 119. (b) 120. (c)
121. (c) 122. (b) 123. (b) 124. (c) 125. (d) 126. (a).

Solutions (Set I)
1. P = Fv = BIlv = 1.25 × 10–3 × 50 × 0.1 × 1 W 6. Length is parallel to BH. So, no induced emf due to
= 6.25 × 10–3 W = 6.25 mW BH. Velocity vector is parallel to BV. So, no induced
emf due to BV.
+ An interesting alternative
1 2E 2×1
P = EI = (Blv) I 7. E = LI2 or L = 2 = H = 200 H.
2 I 0.1 × 0.1
µ 0I µ
2. B = = 0 qn + Current I corresponding
8. L =
µ 0N2 A
2r 2r to circulating charge l
2 rB I = q ν, where ν is frequency.
or q= µ0, A and l are constants.
µ0n
∴ L ∝ N2
∆φ
4. Induced emf, V = ∴ LB = 2 × 2 × LA = 4 × 6 mH = 24 mH
∆t
Q ∆φ 1 1
Current, I = = × 9. emf developed = Bωl2
∆t ∆t R 2
Where Q is the total charge in time ∆t 1
= × 0.2 × 10 −4 × 5 × 1 × 1 V
∆φ 2
∴ Q=
R = 50 × 10 –6 V = 50 µV
5. i = i1 + i2
10. Induced current opposes the motion of the magnet.
5
11. E = 4 volt = 4 × 5 × 1500 = 30000 V = 30 kV.
1/1500
12. Steady current cannot produce induced emf.
LMNote that dI = 0OP
N dt Q
14. φ = LI
Mass and velocity are analogous to inductance and
current respectively.
Fig. 72 15. Use Lenz’s law.
e e e 1 1 1 2
= + or = + 16. 8 = L
ωL ωL1 ωL2 L L1 L2 0.05
L1L2 8 × 0.05
or L= . L= H = 0.2 H.
L1 + L2 2
ELECTROMAGNETIC INDUCTION AND ALTERNATING CURRENT 371

17. E = BV lv 42. In the r – t graph, it is clear that, from a to b, there is


no change in radius and hence no change in area and
BV magnetic flux. Same is the situation from c to d.
Also, tan δ =
BH
d
∴ E = BH lv tan δ Now, |e| = (φ)
dt
5 d
= 0.40 × 10–4 × 1.5 × 72 × × tan 30° |e| = B (πr2)
18 dt
20 × 0.4 × 1.5 × 10 −4 dr
= volt |e| = 2πrB
3 dt
18. φ = NBA cos θ dr
Since r ∝ t, ∴ = constant
= 20 × 103 × 10 –4 × 5 × 10 –4 × cos 60° Wb dt
1 1 ∴ | e | ∝ r.
= × 10–3 Wb = × 10–3 × 108 maxwell 43. ∆φ = BA (cos 180° – cos 0°) = BA [– 1 – 1]
2 2
= 5 × 104 maxwell. = – 2 BA or |∆φ| = 2BA.
44. Refer to Synopsis.
4B0 A 0 − B0 A 0
25. E = 45. There is no change of magnetic flux.
t
46. Refer to Synopsis.
3B0 A 0
or E = .
t 10
47. 100 × 10–3 = M
1 01
.
28. E = 5 × 10–3 × V = 0.05 V. or M = 10–3H = 1 mH.
01
.
48. The magnetic flux density B linking coil Y is shown
29. SI unit of φ is weber.
in the diagram.
30. E = – 5 (– 2) volt = 10 volt
X
31. Faraday’s law of electromagnetic induction.
32. Phenomenon of electromagnetic induction does not
violate the law of conservation of energy.
33. Magnetic field, length and velocity vector are mutu- I
ally perpendicular. B

34. The face of the coil facing the magnet has to acquire
north polarity to repel the magnet. This is possible if
Y
the induced current is anti-clockwise.
35. Hot wire instruments are based on heating effect of
current.
I I
–4
2000
36. E0 = NBAω = 50 × 0.05 × 80 × 10 × 2π × V
60 M N

4π Fig. 73
= volt.
3 As X moves away from coil Y in the direction indi-
37. Note that an inductance is present with B2. cated by the arrow, the flux φ linking coil Y decreases
2−3 due to decrease in B. The decrease in φ causes an emf
38. 5 = – L = 1000 L or L = 5 × 10–3 H = 5 mH. E induced in coil Y and hence a current flows in the
1 × 10−3
coil. By Lenz’s law, the induced current is as shown.
39. Apply lenz’s law. Thus, it flows from N to M in coil Y. Since the flux
change of magnetic flux linkages φ eventually reduced to zero, the current in
40 & 41. Induced charge = . Y thus diminishes to zero eventually. The nature of
resistance
force is attractive since the flux φ is decreased due to
372 COMPREHENSIVE OBJECTIVE PHYSICS

the movement of coil X. The magnetic force tends to is the case when the north pole of the magnet moves
hold coil X in its original position to maintain the away from the coil.
flux φ. → →
55. In (b), the angle between v and B is non-zero. So, a
49. E = Blv = 0.3 × 10–4 × 10 × 5 volt = 1.5 × 10–3 volt current is induced.
= 1.5 mV. 56. P = Fv.
50. When the switch K is closed, current flows through 57. Self-inductance is directly proportional to the square
the electromagnet thus sets up a magnetic flux den- of the number of turns.
sity B in the electromagnet. When copper ring passes 58. Following are the two ways to arrive at the correct
over the electromagnetic, the flux φ linking the ring choice :
which is directly proportional to B and the area of the (i) Use Fleming’s Right Hand Rule and find direc-
copper ring, changes with time. By Lenz’s law, the tion of induced emf.
induced emf in the copper ring opposes it motion and → → →
hence the oscillation of the copper ring is heavily (ii) Use Fm = q ( v × B )
damped. The copper ring will eventually come to a 59. With the increase in the time rate of change of flux,
stop with the rod vertically downward. both the induced emf and current increase. But the
51. By Faraday’s law of electromagnetic induction, the induced charge does not depend upon time. So, it would
induced emf is given by remain the same.

dφ d d µ 0πN2r 4 π × 10−7 × π × 500 × 500 × 5


emf = – =− (BA sin θ) = − (Bxt sin θ) 60. L = = H
dt dt dt 2 2 × 100
= 25 mH.
FG dt IJ
= – Bx sin θ
H dt K = – Bxv sin θ 61. Following arguments shall help decide the right
choice :
Hence, induced emf has a magnitude of Bxv sin θ. (i) Whether the magnetic field is uniform or non-uni-
form, if their is no change in area and consequently
52. The mutual inductance M between solenoid P and Q
no change in magnetic flux, then no induced emf
is given by emf induced in Q due to changing current
will be produced. So, (a) is correct and (b) is wrong.
in P = M × (rate of change of current IP in P)
(ii) In (c), induced emf will be zero at two instants.
⇒ 2 × 10–3 = M × (5) ∴ M = 4.0 × 10–4 H However, it will not be zero if an elemental time is
Similarly, emf induced in P due to changing current considered.
in Q = M × (rate of change of current IQ in Q) 62. When the plane of the coil is perpendicular to the
⇒ induced emf in P = (4.0 × 10–4) × (2) magnetic field, then φ is maximum and induced emf
is minimum.
= 8.0 × 10–4 V.
53. The mutual inductance M between two coils is given 5
63. E = 5 × volt = 25 kV.
by : emf induced in one coil by changing current in 10−3
the other coil = M × (rate of change of current in the 1 Q 2 q2 Q
other coil) 67. = or q =
2 2C 2C 2
dI A –3
68. XL = 95.5 × 10 × 2 × 3.14 × 50 Ω = 29.987 Ω ≈ 30 Ω
or EB = M
dt
2
dI A ZL = R2 + X L = 1600 + 900 = 2500 Ω = 50 Ω.
20.0
Now, = = 40 A s–1
dt 0.5 Lω
Ea = 4.0 V
EB 4.0 R 2

Thus, M= = = 0.10 H. 69. cos φ = 2


ω
dIA /dt 40 R2 + L2ω 2 2 +L
√R
54. More the rate of change of magnetic flux, more will be
the induced current. Again, when the north pole of φ
the magnet approaches the coil, the coil repels the
R
magnet. Clearly, the upper face shall have north po-
Fig. 74
larity. So, the current flows anti-clockwise. Reverse
ELECTROMAGNETIC INDUCTION AND ALTERNATING CURRENT 373

1 Since a rms current value of 13 A produces a mean


71. ωres. = amount of heating effect which is the same as that
LC
produces by a direct current of 13 A, the fuse will
If ωres. is to remain unchanged, then the product LC break if a direct current exceeding 13 A flows in it.
should not change. As C changes to 2C, therefore L
L 1
should be changed to . 91. XL = × 2π × 50 = 100 Ω.
2 π
72. The voltages across L and C are in opposite phase. So, 92. The current I through the resistor R is given by
they cancel out.
V V0
73. No power would be consumed in the super-conduct- I= = sin 2 πft = I0 sin 2 πft.
R R
ing material. Also, it is a purely inductive circuit.
L V0
74. Time constant = The amplitude of the current is thus I0 = .
R R
Since V0 is directly proportional to f, the amplitude of
R
Clearly, has the dimensions of frequency. the current is also directly proportional to f as the
L
frequency is varied (the magnitude of current is ∝ f 2.)
E0 I0 π 93. Refer to the definition of rms current.
75. P = Ev Iv cos φ = × cos
2 2 3
L 40
−3 94. Time constant = = s = 5 s.
100 × 100 × 10 1 R 8
= × W = 2.5 W.
2 2 95. If R is the resistance of resistor when the steady cur-
76. Refer to the definition of virtual value of alternating rent I dissipates power P in it and I0 is the rms value
current. of the sinusoidal alternating current dissipates the
same amount of power in the resistor of half the re-
77. ZL = R2 + X L2 = 402 + 302 = 1600 + 900 Ω
sistance value of R/2, then
= 50 Ω.
FG R IJ
81. E0 = 2 × 220 V = 311.08 V ≈ 311 V. P = I2R = I02 H 2K
1 ∴ I02 = 2I 2 or I0 = 2 I.
85. ν =
2π LC
96. Now, I = I0 sin (θt) = I0 sin (2πft) where f is the fre-
1 quency.
ν =
2π 10−4 × 10− 6 Since f and I0 are both doubled, the corresponding
sinusoidal current is thus
105 I = (2I0) sin [2π (2f)t] = 2I0 sin [2(2πf)t]
ν = Hz.
2π = 2I0 sin (2 θt).
86. Since the capacitor is connected in series to the resis- 1
tor, the current IC from the supply and IR through the 97. T= s = 0.02 s
resistor is in phase as represented by choice a. 50
87. At low frequencies, Lω dominates. At higher frequen- Note that the voltage becomes zero twice in each cycle.
1 98. Mean power P, dissipated in a resistor is directly pro-
cies, dominates.
Cω portional to peak value of the emf of the ac supply V0
Im 2 2 2 2 and is independent of the frequency f of the supply.
88. = I0 × = . Since peak of emf V0 remains unchanged and frequency
I rms π I0 π
f has altered, mean power in the resistor thus remains
1 1 C unchanged.
89. LI02 = CV02 or I0 = V0.
2 2 L
300
90. The fuse F breaks if the current through it produces 99. 900 = Iv cos 60° or Iv = 6 2 A.
sufficient heating effect to the metal wire inside it. 2
374 COMPREHENSIVE OBJECTIVE PHYSICS

100. The power dissipated in a resistor for a constant po- 200 π


tential difference V is the same as the for a sinusoidal = 2× sin volt = 100 volt.
2 6
alternating potential difference of rms value V.
108. It is a case of parallel resonant circuit. Maximum
Now, the sinusoidal potential difference has peak value
impedance and zero current.
of V0, we thus have
109. e0 = NBAω
V0 E−e
V= or V0 = 2 V. 110. I =
2 Ra
101. Period of the sinusoidal voltage, T = 0.4 s E
The frequency for the voltages is thus At the start, e = 0 ∴ Istart =
Ra
1 1 Clearly, current is maximum when the motor has
f= = = 2.5 Hz
T 0.4 just started.

FG 01. IJ 2 π or 111. Input power = 220 V × 5A = 1100 W


N lags M by 0.1 s which is equivalent to
H 0.4 K Output power = 11 V × 90 A = 990 W
900
π Efficiency = × 100 = 90.
π 1100
rad. Thus, the lead of N over M is – rad.
2 2
220 − e
1 112. 1.5 = or 220 – e = 30
102. XC = 20
2πfC
or e = (220 – 30) V = 190 V.
When f = 0, XC = ∞. 115. When the angular speed is doubled, the time rate of
1 1 change of magnetic flux is doubled.
103. XC = or XC ∝ . 116. Ideal choke coil has zero resistance.
2πfC f
104. Root mean square current of the sinusoidal waveform, 750 75
117. ν = =
I0 60 6
I = . Eight-polar dynamo has 4 coils.
2
Power output of the heater, 75
∴ Effective value is × 4 Hz or 50 Hz.
6
P = I2R =
FG I IJ
0
2 2
I0 R
H 2K R=
2
.
119.
Ip
Is
=
ns
np
=
20
1
.
R
105. Power factor = 121. Transformer is essentially a DC device.
F
+ G Lω −
1 I
2

H Cω JK
R 2 123. Input power = Output power.
22 × 22
1 124. Ip × 220 = or Ip = 0.01 A.
For a purely resistive circuit, Lω – =0 220

∴ Power factor = 1. Ip ns 4
125. = = .
1 1 1 Is np 5
106. XC = = = Ω = 1 Ω.
Cω 2πf C 106 126. Transformer is essentially an AC device.
2π × × 10 −6

1
107. E = E0 sin ωt = 2 Ev sin 2π × 50 × volt
600
ELECTROMAGNETIC INDUCTION AND ALTERNATING CURRENT 375

KNOWLEDGE PLUS
l When the current changes from + 2A to – 2A in 0.05 second, an emf of 8 V is induced in a coil. The coefficient of
self-induction of the coil is
(a) 0.1 H (b) 0.2 H (c) 0.4 H (d) 0.8 H [AIEEE 2003]
4
Sol. 8=L or L = 2 × 0.05 H = 0.1 H
0.05
So, (a) is the right choice.

l The magnetic flux linked with a coil at any instant ‘t’ is given by φ = 5t3 – 100t + 300, the emf induced in the coil
at t = 2 second is
(a) 40 V (b) – 40 V (c) 300 V (d) 140 V [Karnataka CET 2003]
d
Sol. E= − (φ) = – (15t2 – 100)
dt
Et = – (15 × 2 × 2 – 100) V = 40 V
So, (a) is the right choice.

l One conducting U tube can slide inside another as shown in figure, maintaining electrical × × × B
× A ×
contacts between the tubes. The magnetic field ‘B’ is perpendicular to the plane of the
v
figure. If each tube moves towards the other at a constant speed V, then the emf induced ×
× ×
v
in the circuit in terms of B, l and V where l is the width of each tube, will be × × × ×
(a) 2BlV (b) zero (c) – BlV (d) BlV D × × × C
[AIEEE 2005]
Fig. 75
Sol. Two emfs, each of value BlV, are produced. Both the emfs tend to send current in the
same direction. So, they are added up.
So, (a) is the right choice.
π
l The phase difference between the alternating current and emf is . Which of the following cannot be the constituent
2
of the circuit ?
(a) L alone (b) L, C (c) R, L (d) C alone [AIEEE 2005]
π
Sol. The presence of R forbids a phase difference of .
2
So, (c) is the right choice.

l A circuit has a resistance of 12 ohm and an impedance of 15 ohm. The power factor of the circuit will be
(a) 0.125 (b) 1.25 (c) 0.4 (d) 0.8 [AIEEE 2005]
R 12
Sol. cos φ = = = 0.8
Z 15 Z
So, (d) is the right choice.

R
Fig. 76
376 COMPREHENSIVE OBJECTIVE PHYSICS

MCQs
based on
Important Basic Concepts
SET II

+
Application of Formulae
+
Brain Teasers
Average time allowed per question is 50 seconds.

ELECTROMAGNETIC INDUCTION (a) 2π × 10 –6 weber (b) π × 10 –9 weber


(c) π × 10 –9
cos ωt weber (d) zero.
127. A coil having n turns and resistance R Ω is connected
130. In the previous question, the induced emf is
with a galvanometer of resistance 4R Ω. This combi-
(a) π × 10 –9 cos ωt volt (b) π × 10 –9 ω cos ωt volt
nation is moved in time t second from a magnetic field
W1 weber to W2 weber. The induced current in the cir- (c) 0 volt (d) π × 10 –9 ω sin ωt volt.
cuit is 131. In Q. 129, the induced current is (in ampere)
(W2 − W1) n( W2 − W1) π π
(a) − (b) − (a) × 10 –9 ω sin ωt (b)
Rnt 5Rt 2 2
(c) 10 –9 (d) π × 10 –9.
(W2 − W1) n( W2 − W1)
(c) − (d) − 132. The variation of induced emf (ε) with time (t) in a coil
5Rnt Rt
if a short bar magnet is moved along
[AIEEE 2004] its axis with a constant velocity is best
128. In a uniform magnetic field of induction B, a wire in represented as
the form of a semicircle of radius r rotates about the
diameter of the circle with an angular frequency ω. The (a)  (b) 
axis of rotation is perpendicular to the field. If the total
resistance of the circuit is R, the mean power gener- t
ated per period of rotation is t
2 2 2
(Bπrω) (Bπr ω)
(a) (b)
2R 8R (c)  (d) 

Bπ r 2 ω (Bπrω 2 )2
(c) (d) [AIEEE 2004]
2R 8R
t t
129. The inner loop has an area
of 5 × 10–4 m2 and a resist-
ance of 2 Ω. The larger cir- Fig. 78
cular loop is fixed and has [IIT Screening 2004]
1A
a radius of 0.1 m. Both the 1A
133. A copper rod of length l is rotated about the end
loops are concentric and
perpendicular to the uniform magnetic field B with
coplanar. The smaller loop
constant angular velocity ω. The induced emf between
is rotated with an angular
its two ends is
velocity of ω rad s–1 about Fig. 77 (a) zero (b) Bωl2
its diameter. The flux
1 1
linked with the smaller loop is (c) Bωl2 (d) Bωl2.
2 4
ELECTROMAGNETIC INDUCTION AND ALTERNATING CURRENT 377

134. A circuit has a self-inductance of 1 henry and carries a 139. The mutual inductance between two coils is 1.25 henry.
current of 2 A. To prevent sparking when the circuit is If the current in the primary changes at the rate
switched off, a capacitor which can withstand 400 V is of 80 ampere/second, then the induced emf in the
used. The least capacitance of the capacitor connected secondary is
across the switch must be (a) 12.5 V (b) 64.0 V
(a) 12.5 µ F (b) 25 µ F (c) 0.016 V (d) 100.0 V. [MP PET 1990]
(c) 50 µ F (d) 100 µ F. 140. A small magnet is along the axis of a coil and its
135. A coil of wire of a certain radius has 600 turns and a distance from the coil is 80 cm. In this position, the
self-inductance of 108 mH. The self-inductance of a IInd flux linked with the coil is 4 × 10 –5 weber. If the coil is
similar coil of 500 turns will be displaced 40 cm towards the magnet in 0.08 second,
(a) 74 mH (b) 75 mH then the induced emf produced in the coil will be
(a) 0.5 mV (b) 1 mV
(c) 76 mH (d) 77 mH. [MP PMT 1990]
(c) 7 mV (d) 3.5 mV. [MNR 1997]
136. The coefficient of mutual-inductance of two circuits A
and B is 3 mH and their respective resistances are 141. The magnetic flux linked with a coil is changed from
10 ohm and 4 ohm. How much current should change 1 weber to 0.1 weber in 0.1 s. The induced emf is
in 0.02 second in the circuit A, so that the induced cur- (a) 9 V (b) 10 V
rent in B should be 0.006 ampere ? (c) 0.009 V (d) 91 V.
(a) 0.24 A (b) 1.6 A
142. In Q. 141, the induced charge is (given that the resist-
(c) 0.18 A (d) 0.16 A.
ance of the coil is 10 ohm)
[CMC Vellore 2001] (a) 9 C (b) 0.9 C
137. Two coils,a primary of 400 turns and a secondary of (c) 0.09 C (d) 0.009 C.
20 turns are wound over an iron core of length 20 π cms
143. A coil whose diameter is 0.40 m is in a variable mag-
and cross-section of 2 cms radius. If µr = 800, then the
netic field. As the magnetic induction of the field
coefficient of mutual induction is approximately changes by 127.4 T during 2 s, an emf of 200 V is in-
(a) 1.6 × 107 H (b) 1.6 × 10 –2 H duced in the coil. Then the number of turns in the coil
(c) 1.6 × 103 H (d) 1.6 H. [AFMC 2000] is approximately
138. A circuit ABCD is held perpendicular to uniform mag- (a) 20 (b) 25
netic field of B = 5 × 10 –2 T extending over the region (c) 30 (d) 50. [AIIMS 1999]
PQRS and directed into the plane of the paper. The 144. The coefficient of mutual induction of two circuits is
circuit is pulled out of the field at a uniform speed of 4 mH. If in a circuit, the current changes by 0.60 am-
0.2 m s–1 for 1.5 s. During this time, the current in the pere to 0.61 ampere in 0.002 second, then induced emf
5 Ω resistor is in the second circuit will be
(a) 0.8 V (b) 0.008 V
P Q
(c) 0.02 V (d) 0.2 V. [AMU 1998]
0.5 m
A B 145. Two different coils have self-inductances L1 = 8 mH
and L2 = 2 mH. The current in one coil is increased at
® 5W
0.3 m B a constant rate. The current in the second coil is also
increased at the same constant rate. At a certain in-
–1
0.2 m s
stant of time, the power given to the two coils is the
D C
same. At that time, the current, the induced voltage
S R and the energy stored in the first coil are I1, V1 and W1
respectively. The corresponding values for the second
Fig. 79 coil are I2 , V2 and W2 , at the same instant, respectively.
(a) 0.6 mA from B to C (b) 0.9 mA from B to C Now,
I 1 I
(c) 0.9 mA from C to B (d) 0.6 mA from C to B. (a) 1 = (b) 1 = 4
I2 4 I2
[National Standard Exam. in Physics 1997]
I 2 I
(c) 1 = (d) 1 = 1.
I2 1 I2
378 COMPREHENSIVE OBJECTIVE PHYSICS

146. In Q. 145, × B × × × × ×

V1 1 V1 × × × × × × ×
(a) = (b) =4 P
V2 4 V2
× × × × × × ×
®
V 2 V v M
(c) 1 = (d) 1 = 1. × × × × × × ×
V2 1 V2 N
× × × × × × ×
147. In Q. 145,
W2 W1 × × Q × × × ×
(a) =4 (b) =4
W1 W2 Fig. 80
W1 W1 (a) plate M will be positively charged
(c) =1 (d) = 2.
W2 W2 (b) plate N will be positively charged
148. Flux φ in weber in a closed circuit of resistance 10 Ω (c) both plates will be similarly charged
varies with time t in second according to the equation (d) no charge will be collected on plates. [CPMT 1997]
φ= 6t2 – 5t + 1 153. The mutual inductance of a pair of coils if a current of
What is the magnitude of the induced current at 3 ampere in one coil causes the flux in the second coil
of 2000 turns to change by 6 × 10–4 weber per turn of
1 the secondary coil is
t= s?
4 (a) 6 × 10–4 H (b) 2 × 10–4 H
(a) 1.2 A (b) 0.8 A (c) 0.4 H (d) 4 H. [BHU 1996]
(c) 0.6 A (d) 0.2 A.
154. A 50 turns circular coil has a radius of 3 cm. It is kept
149. Two coils X and Y are placed in a circuit such that in a magnetic field acting normal to the area of the
when current changes by 2 A in coil X, the magnetic coil. The magnetic field B is increased from 0.10 T to
flux changes by 0.4 Wb in Y. The mutual inductance of 0.35 T in 2 m s. The average induced emf in the coil is
the two coils is (a) 1.77 V (b) 17.7 V
(a) 0.2 Wb (b) 0.2 H
(c) 177 V (d) 0.177 V. [MP PET 1994]
(c) 0.3 H (d) 0.3 Wb.
155. There is an aerial 1 m long in a car. It is moving from
150. A straight conductor of length 4 metre moves at a speed east to west with a velocity 100 km h–1. If the horizon-
of 10 m s–1 when the conductor makes an angle of 30° tal component of earth’s magnetic field is 0.18 × 10–4
with the direction of magnetic field of induction 0.1 T. T, then induced emf is
Then the induced emf is (a) 0.50 mV (b) 0.25 mV
(a) 1 volt (b) 2 volt (c) 0.75 mV (d) 1 mV. [JIPMER 1997]
(c) 4 volt (d) 8 volt. [BHU 2001] 156. A circular iron core supports N turns. If a current I
151. Two coils have mutual inductance 0.005 H. The cur- produces a magnetic flux φ across the core’s cross
rent changes in the first coil according to equa- section, then the magnetic field energy is
tion I = I0 sin ωt where I0 = 10 A and ω = 100 π 1
radian per second. The maximum value of emf in the (a) NIφ (b) NIφ
2
second coil (in volt) is
(c) N2Iφ (d) NI2φ. [WB JEE 1997]
(a) 2 π (b) 5 π
157. A search coil of 1000 turns, resistance 5 Ω and cross-
(c) π (d) 4 π.
sectional area 5 cm2 is connected in series with a gal-
[All India PM/PD 1998] vanometer of resistance 45 Ω. The coil is placed in be-
152. A rod PQ is connected to the capacitor plates. The rod tween the pole pieces of an electromagnet. When the
is placed in a magnetic field (B) directed downward coil is withdrawn from the pole pieces in 1 ms, the
perpendicular to the plane of the paper. If the rod is charge flowing in galvanometer is 2 mC. The magnetic

pulled out of magnetic field with velocity v as shown field of the electromagnet is
in Fig. 80 (a) 0.1 T (b) 0.2 T
(c) 102 T (d) 2 × 102 T. [DPMT 1999]
ELECTROMAGNETIC INDUCTION AND ALTERNATING CURRENT 379
158. A horizontal telegraph wire 2.5 km long runing east 166. An electric current I is passed
and west is a part of a circuit whose resistance is 35 Ω. through a circular loop of
The wire falls to the ground from a height of 10 m. If folded copper wire as shown
g = 9.8 m s–2 and eBh = 2 × 10 –5 T, then the current in Fig. 81. The magnetic in-
induced in the circuit is duction at the centre of the
(a) 0.7 A (b) 0.02 A loop will be
(c) 0.01 A (d) 0.04 A. [AFMC 1996] (a) zero
2µ 0 I
159. A coil having an area 2 m2 is placed in a magnetic (b)
field which changes from 1 Wb/m2 to 4 Wb/m2 in a r I I
interval of 2 second. The emf induced in the coil will be µ 0I Fig. 81
(c)
(a) 4 V (b) 3 V r
µ 0I
(c) 1.5 V (d) 2 V. [AMU 1996] (d) . [National Standard Exam.
2r
160. If a current of 3.0 A flowing in the primary coil is re- in Physics 2000]
duced to zero in 0.001 second, then the induced emf in 167. The self-induced emf in a 0.1 H coil when the current
the secondary coil is 15000 V. The mutual inductance in it is changing at the rate of 200 ampere/second is
between the two coils is (a) 125 V (b) 20 V
(a) 0.5 H (b) 5 H
(c) 8 × 10 –4 V (d) 8 × 10 –5 V.
(c) 1.5 H (d) 10 H.
[MP PAT 1990]
[MP PMT 1989, 91]
168. A coil of area 100 cm2 has 500 turns. Magnetic field of
161. A copper disc of radius 0.1 m is rotated about its centre
0.1 weber/metre2 is perpendicular to the coil. The field
with 10 revolutions per second in a uniform magnetic
is reduced to zero in 0.1 second. The induced emf in the
field of 0.1 tesla with its plane perpendicular to the
field. The emf induced across the radius of disc is coil is
π 2π (a) 1 V (b) 5 V
(a) volt (b) volt
10 10 (c) 50 V (d) Zero. [MP PMT 1991]
(c) π × 10 –2 volt (d) 2π × 10 –2 volt. 169. If a current of 2 ampere gives rise a magnetic flux of
[MP PAT 1990] 5 × 10–5 weber through a coil having 100 turns, then
162. A magnet is moved towards a coil (a) quickly (b) slowly, the magnetic energy stored in the medium surround-
then the induced emf is ing the coil is
(a) larger in case (a) (b) smaller in case (a) (a) 5 J (b) 5 × 10 –7 J
(c) equal in both cases (c) 5 × 10 –3 J (d) 0.5 J. [Pb. CET 1999]
(d) larger or smaller depending upon the radius of the coil.
170. An emf of 6 V is introduced in a given coil when the
163. In Q. 162, the induced charge is current in it changes at the rate of 30 A/min. The in-
(a) larger in case (a) (b) smaller in case (b) ductance of the coil is
(c) equal in both cases (a) 0.2 H (b) 5 H
(d) larger or smaller depending upon the radius of the coil. (c) 12 H (d) 180 H.
164. In Q. 162, the work done is [All India PM/PD 1999]
(a) larger in case (a)
171. The magnetic flux through a coil varies with time as
(b) smaller in case (b)
φ = 5t2 + 6t + 9. The ratio of emf at t = 3 s to t = 0 s will be
(c) equal in both cases
(a) 1 : 9 (b) 1 : 6
(d) larger or smaller depending upon the material of the coil.
(c) 6 : 1 (d) 9 : 1.
[CPMT 1985]
[All India PM/PD 2002]
165. The inductance of a closed-packed coil of 400 turns is
172. A rectangular coil 0.20 m × 0.10 m, of 100 turns, ro-
8 mH. A current of 5 mA is passed through it. The mag-
tates in a magnetic field of 3 × 10 –3 T with a frequency
netic flux through the coil is approximately
of 20 Hz, about an axis normal to the magnetic field.
(a) 0.1 µ0 Wb (b) 0.2 µ0 Wb
What is the maximum value of induced emf ?
(c) 1.0 µ0 Wb (d) 2.0 µ0 Wb.
(a) 0.12 π V (b) 0.18 π V
[Himachal PMT 2001]
(c) 0.24 π V (d) 0.30 π V. [AMU 2002]
380 COMPREHENSIVE OBJECTIVE PHYSICS

173. When the number of turns in a coil is doubled without 180. Fig. 82 shows a short coil wound over the middle part
any change in the length of the coil, its self-inductance of a long solenoid.
becomes
(a) four times (b) doubled
(c) halved (d) squared.
[CMC Vellore 2000]
174. An air-cored coil has a self-inductance of 0.1 H. A soft-
iron core of relative permeability 1000 is introduced
and the number of turns is reduced to 1/10th. The value
of self-inductance now is
(a) 0.1 H (b) 1 mH Fig. 82

(c) 1 H (d) 10 mH. [DPMT 2001] The solenoid current I is varied


175. A solenoid (air core) has 400 turns, is 20 cm long and with time t as shown in Fig. 83.
has a cross section of 4 cm2. Then the coefficient of self As a consequence, the flux density
induction is approximately of the magnetic field due to the
(a) 4 × 10 –5 H (b) 4 × 101 H solenoid varies with time. The
relation between B and I is B =
(c) 4 × 10 –4 H (d) 4 H. [JIPMER 2001]
µ0nI.
176. A bulb of 100 W is connected in parallel with an ideal Which graph shows how the emf Fig. 83
inductance of 1 H. This arrangement is connected to a E induced in the short coil varies
90 V battery through a switch. On pressing the switch, with time ?
the
(a) bulb does not glow (b) bulb glows E E
(c) bulb glows after a short time and then continues to glow
(d) bulb glows for a short time and then stops glowing.
[BITS 1999]
177. A coil having resistance 40 Ω, number of turns 100 O t O t
and radius 6 mm is connected to an ammeter of resist- (a ) (b)
ance 160 Ω. The coil is placed perpendicular to the mag-
netic field. When the coil is taken out of the field, a E E
charge of 32 µC passes through it. The intensity of
magnetic field will be
(a) 6.55 T (b) 5.66 T
(c) 0.655 T (d) 0.566 T. O t O t
[Raj. CET 1998] (c) (d )

178. A helicopter rises vertically with a speed of 10 ms–1. It E


can be assumed to be a horizontal linear conductor of
length 10 m. If the horizontal component of the earth’s
magnetic field be 1.5 × 10 –3 Wb/m2, the emf induced
between the tip of the nose and the tail of the helicopter O
t
in volt is
(a) 0.15 (b) 5
(c) 125 (d) 130. (e)
[Himachal PMT 1997]
Fig. 84
179. The magnetic flux through each turn of a 100 turn coil
181. A small coil of radius r is placed at the centre of a large
is (t3 – 2t) × 10 –3 Wb, where t is in second. The induced
coil of radius R, where R >> r. The two coils are
emf at t = 2 s is
coplanar. The mutual inductance between the coils is
(a) – 4 V (b) – 1 V proportional to
(c) + 1 V (d) + 4 V. (a) r/R (b) r2/R
[CMC LDH 1999] (c) r2/R2 (d) r/R2. [Roorkee 1995]
ELECTROMAGNETIC INDUCTION AND ALTERNATING CURRENT 381

182. In a circular conducting coil, when current increases 186. A coil having 500 square loops, each of side 10 cm, is
from 2 A to 18 A in 0.05 s, the induced emf is 20 V. The placed normal to a magnetic field which increases at a
self-inductance of the coil is rate of 1.0 Ts–1. The induced emf (in volt) is
(a) 62.5 mH (b) 6.25 mH (a) 0.1 (b) 0.5

(c) 50 mH (d) None of these. (c) 1.0 (d) 5.0. [CPMT 1990]
[MP PET 2001] 187. Which of the following statements, about a solenoid, is
correct ?
183. A galvanometer is connected to the secondary coil. The
(a) When a current flows through a solenoid, it has ten-
galvanometer shows an instantaneous maximum
dency to increase its radius if no external magnetic field
deflection of 7 divisions when current is started in the exists in the space.
primary coil of the solenoid. Now if the primary coil is
(b) When a current flows through a solenoid, it always pos-
rotated through 180°, then the new instantaneous sesses tendency to increase its radius if an external
maximum deflection will be magnetic field exists in the space.
(a) 7 units (b) 0 units (c) When a current flows through a solenoid, it always pos-
(c) 14 units (d) 21 units. [CPMT 1991] sesses tendency to decrease its radius if an external
magnetic field exists in the space.
184. At a place, the value of horizontal component of the
earth’s magnetic field H is 2 × 10 –5 weber/m2. A me- (d) None of the above. [AIIMS 2002]
tallic rod AB of length 2 m placed in East-West direc- 188. A mutual inductor
tion, having the end A towards east, falls vertically consists of two coils X
downward with a constant velocity of 50 m/s. Which and Y as shown in
end of the rod becomes positively-charged and what is Fig. 86 in which one
the value of induced potential difference between the quarter of the mag-
netic flux produced
two ends ? Fig. 86
by X links with Y, giv-
(a) End A, 3 × 10 –3 millivolt (b) End A, 3 millivolt ing a mutual inductance M.
(c) end B, 3 × 10 –3 millivolt (d) End B, 2 millivolt. What will be the mutual inductance when Y is used as
[MP PET 1996] the primary ?
185. A square metal wire loop of side 10 cm and resistance (a) M/4 (b) M/2
1 Ω is moved with constant velocity v0 in a uniform (c) M (d) 2M
magnetic field of induction B = 2 Wb m–2, as shown in (e) 4M.
Fig. 85. The magnetic field lines are perpendicular to 189. A square coil of 0.01 m2 area is placed perpendicular
the plane of loop and directed into the paper. The loop to the uniform magnetic field of induction 10 3 T. The
is connected to network of resistors, each of value 3 Ω. magnetic flux linked with the coil is
The resistance of lead wires is negligible. The speed of (a) 10 Wb (b) 10 –5 Wb
loop so as to have a steady current of 1 mA in the loop (c) Zero (d) 100 Wb.
is [MP PMT 2001]
190. A conducting wire xy of length l and mass m is sliding
× × × × v0 without friction on vertical conducting rails ab and cd
3W 3W as shown in Fig. 87.
× × × × P 3W
a R c
× × × ×
O
3W 3W
× × × ×
x y
l
Fig. 85
(a) 2 m s–1 (b) 2 cm s–1 b d
(c) 10 m s–1 (d) 20 m s–1.
Fig. 87
[CMC LDH 2000]
382 COMPREHENSIVE OBJECTIVE PHYSICS

A uniform magnetic field B exists perpendicular to the


plane of the rails. xy moves with a constant velocity of
mgR mgR
(a) (b)
Bl Bl 2

mgR mgR
(c) 2 2
(d) . [MNR 1991]
B l B2l
191. Consider the situation × × × × × ×
shown in Fig. 88. The A
× × × × × × × Fig. 90
wire AB is slid on
×v ×
fixed rails with a con-
× × × × × R µ 0 A(N1/N 2 ) µ 0A(N1N2 )
(a) (b)
stant veloicty. If the × × × × × × × L L
wire AB is replaced by ×B
× × × × ×
µ 0AN12N2
(c) µ0A N1N2 L (d) .
semi-circular wire, Fig. 88 L
the magnitude of in- [MNR 1994]
duced emf will 194. Magnetic flux φ (in weber) linked with a closed circuit
(a) increase (b) decrease of resistance 10 Ω varies with time t (in second) as
(c) remain the same φ = 5t2 – 4t + 1
(d) increase or decrease depending on whether the semi- The induced electromotive force in the circuit at t = 0.2
circle buldges towards the resistance or away from it. second is
[MP PMT 1999] (a) 0.4 V (b) – 0.4 V
192. A dc electric motor that has a permanent magnet as its (c) – 2.0 V (d) 2.0 V. [MP PMT 2001]
field magnet is joined to a battery of constant emf and 195. Two coils have mutual inductance M. The second coil
negligible internal resistance. When the motor is used (of negligible resistance) is connected to a resistor of
to drive various loads, the corresponding values of its resistance R and the current in the first coil is changed
speed of rotation ω and the current I passing through at a steady rate dI/dt. What is the power dissipated in
it are measured. Which one of the following graphs most the resistor ?
nearly shows how I varies with ω ?
M2
(a) (dI/dt)2 (b) M2R (dI/dt)2
I I I R
R M
(c) (dI/dt)2 (d) (dI/dt)
M R
(e) M2R (dI/dt).
196. Current in a coil changes at a constant rate of 2 A in 50
0 0 0 ms. A back-emf of 4 V is induced in the coil. What is
0 w 0 w 0 w
the self-inductance of the coil ?
(a) (b) (c)
(a) 0.025 H (b) 0.10 H
(c) 0.40 H (d) 40 H
I I
(e) 160 H.
197. An aluminium ring hangs vertically from a thread with
its axis pointing east-west. A coil is fixed near to the
ring and coaxial with it.
0 0
0 w 0 w
thread
(d) (e)

N
Fig. 89
193. A long solenoid of length L, cross section A having N1 Common
W E
turns has wound about its centre a small coil of N2 axis
turns as shown in Fig. 90.Then the mutual inductance coil ring
S
of two circuits is
Fig. 91
ELECTROMAGNETIC INDUCTION AND ALTERNATING CURRENT 383

What is the initial motion of the aluminium ring when


the current in the coil is switched on ?
(a) remains at rest (b) moves towards S
(c) moves towards W (d) moves towards N
(e) moves towards E.
198. A magnetic field is applied perpendicular to the plane
of a flat coil of copper wire. The time variation of the
magnetic flux density is given by B0 sin (2πt/T), as
shown graphically in Fig. 92.

Fig. 92 Fig. 94
At which of the following values of t is the magnitude 201. A conducting wire of mass m
of the emf induced in the coil is maximum ? slides down two smooth con-
T T ducting bars, set at an angle
(a) (b) θ to the horizontal as shown
8 4
in Fig. 95.
3T T
(c) (d) The separation between the
8 2
bars is equal to l. The system
3T is located in a uniform mag-
(e) .
4 netic field of induction B, per- Fig. 95
199. Two pure inductors, each of self inductance L, are con- pendicular to the plane of the
nected in parallel but are well separated from each sliding wire and bars. The velocity of the wire is
other. Then the total inductance is mg R sin θ mg R sin θ
(a) (b)
(a) L (b) 2L B 2l 2 Bl 3
(c) L/2 (d) L/4. [MNR 1991] mg R θ mg R sin θ
(c) (d) .
200. The magnetic flux φ through a coil varies with time t B2l5 Bl 4
as shown in Fig. 93. [WB JEE 2000]
202. A short bar magnet passes at a steady speed right
through a long solenoid. A galvanometer is connected
across the solenoid.

Fig. 93

Which graph best represents the variation with t of the


emf E induced in the coil ?
Fig. 96
384 COMPREHENSIVE OBJECTIVE PHYSICS

Which graph best represents the variation of the gal- 206. A small square loop of wire of side l is placed inside a
vanometer deflection θ with time t ? large square loop of wire of side L (L >> l). The loops
are coplanar and their centres coincide. The mutual
inductance of the system is proportional to
(a) l/L (b) l2/L
(c) L/l (d) L2/l. [IIT 1998]
207. The current flowing in a coil of self inductance 0.4 mH
is increased by 250 mA in 0.1 s. The emf induced will be
(a) + 1 volt (b) = 1 volt
(c) + 1 mV (d) – 1 mV.
[MP PMT 1994]

TRANSIENT CURRENT
AND ALTERNATING CURRENT

208. When an AC source of emf e = i


E 0 sin (100t ) is connected e
across a circuit, the phase
difference between the emf e
and the current i in the circuit
Fig. 97 is observed to be π/4 as shown
in Fig. 98. If the circuit Fig. 98
203. A metal rod of resistance R is fixed along a diameter of consists possibly only of R-C
a conducting ring of radius r. There is a magnetic field or R-L or L-C in series, find the relationship between
of magnitude B perpendicular to the plane of the loop. the two elements.
The ring spins with angular velocity ω about its axis. (a) R = 1 kΩ, C = 10 µF (b) R = 1 kΩ, C = 1 µF
The centre of the ring is joined to its rim by external (c) R = 1 kΩ, L = 10 H (d) R = 1 kΩ, L = 1 H
wire W. The ring and W have no resistance. The current
[IIT Screening 2003]
in W is
209. In an RC circuit while charg- ln I
Br 2 ω Br 2 ω ing, the graph of ln I versus
(a) (b)
R 2R time is as shown by the dotted S
line in the adjoining diagram
2Br 2 ω R
(c) (d) zero. where I is the current. When the
R
value of the resistance is dou- Q
[Bharati Vidyapeeth 2002] bled, which of the solid curves P
t
204. If L, Q and R represents the inductance, charge and best represents the variation of
resistance respectively, then the units of ln I versus time ? Fig. 99
(a) P (b) Q
QR Q2 R3
(a) will be of current (b) will be of energy (c) R (d) S [IIT Screening 2004]
L L3
210. An emf of 15 volt is applied in a circuit containing
QL Q3R 2
(c) will be of current (d) will be of power. 5 henry inductance and 10 ohm resistance. The ratio
R L of the currents at time t = ∞ and at t = 1 second is
[Pb. PMT 2002]
e1/ 2 e2
205. A coil is wound on a frame of rectangular cross-sec- (a) (b)
e1/ 2 − 1 e2 − 1
tion. If all the linear dimensions of the frame are in-
(c) 1 – e–1 (d) e–1. [MP PMT 1994]
creased by a factor of 2 and the number of turns per
unit length of the coil remain unchanged, the self in- 211. An alternating emf of angular frequency ω is applied
ductance of the coil increases by a factor of across an inductance. The instantaneous power devel-
(a) 2 (b) 4
oped in the circuit has an angular frequency
ω ω
(c) 8 (d) 16. (a) (b)
4 2
[All India PM/PD 2001] (c) 2 ω (d) 4 ω. [EAMCET 2002]
ELECTROMAGNETIC INDUCTION AND ALTERNATING CURRENT 385
212. The magnetic field energy in an inductor changes from 217. In a circuit containing a capacitor, an inductor and a
maximum value to minimum value in 5.0 ms when resistor in series, VC , VL and VR represent the poten-
connected to an AC source. The frequency of the source tial differences across those components and I repre-
is sents the current through them. Which of the following
(a) 20 Hz (b) 50 Hz statements is true ?
(c) 200 Hz [AIIMS 2001]
(d) 500 Hz. 1. VC and I are 180° out of phase
213. 5 cm long solenoid having 10 ohm resistance and 2. VR and I are 90° out of phase
5 mH inductance is joined to a 10 volt battery. At steady
3. VL and VC are 180° out of phase.
state, the current through the solenoid in ampere will
be (a) if 1, 2, 3 are correct (b) if 1, 2 are correct

(a) 5 (b) 1 (c) if 2, 3 are correct (d) if 1 only

(c) 2 (d) Zero. [MP PET 1995] (e) if 3 only. [London School of Examinations]
214. An inductive coil has a resistance of 100 Ω. When an 218. In the circuit shown below, what will be the reading of
AC signal of frequency 1000 Hz is fed to the coil, the the voltmeter V3 and ammeter A ?
applied voltage leads the current by 45°. What is the
inductance of the coil ?
(a) 10 mH (b) 12 mH
(c) 16 mH (d) 20 mH. [EAMCET 1999]
215. In the circuit of Fig.
100, the AC source has
voltage 20 cos (ωt) volt
with ω = 2000 rad/s.
The amplitude of the
current will be nearest Fig. 102
to
Fig. 100 (a) 800 V, 2 A (b) 300 V, 2 A
(a) 2 A (b) 3.3 A
(c) 220 V, 2.2 A (d) 100 V, 2 A.
(c) 2/ 5 A (d) 5 A. [Manipal 2002]
[National Standard Exam. in Physics 1995] 219. The reactance of a coil when used in the domestic AC
216. In a circuit, if the rms voltage of the AC supply remains power supply (220 volt 50 cycles per second) is 50 ohm.
constant when the frequency is varied between 50 Hz The inductance of the coil is nearly
and 50 kHz, which of the graphs below best illustrates (a) 2.2 H (b) 0.22 H
the variation of current through the ammeter with fre- (c) 1.6 H (d) 0.16 H. [MP PMT 2000]
quency ?
220. An rms voltage of 110 V is applied across a series cir-
cuit having a resistance 11 Ω and impedance 22 Ω.
The power consumed is
(a) 275 W (b) 366 W
I I I
(c) 550 W [BHU 2002]
(d) 1100 W.
f f f 221. An inductor, a capacitor and a resistor are connected
(a) (b) (c) in series to an AC supply. When measured with an AC
voltmeter, the potential difference acros the inductor,
capacitor and resistor are respectively 90 volt, 60 volt
and 40 volt. Then the supply voltage is
I I (a) 190 volt (b) 100 volt
(c) 130 volt [DPMT 2002]
(d) 50 volt.
f f 222. In a LCR circuit having L = 8.0 Henry, C = 0.5 µF and
(d) (e) R = 100 Ω in series, the resonance frequency is
Fig. 101 (a) 900 radian s–1 (b) 500 Hz
[London School of Examinations] (c) 700 radian s–1 (d) None of these.
[CPMT 1990]
386 COMPREHENSIVE OBJECTIVE PHYSICS

223. An alternating voltage is connected in series with a re- VC VL


sistance R and an inductance L. If the potential drop VL VC

across the resistance is 200 volt and across the induct-


ance is 150 volt, then the applied voltage is
(a) 350 V (b) 250 V v v

(c) 500 V (d) 300 V.


224. In the circuit shown here, R is a pure resistor, L is an
f f
inductor of negligible ressitance (as compared to R), S
(1) (2)
is a 100 V, 50 Hz AC
source of negligible re- VC VC
VL
sistance. With either key VL
K 1 alone or K 2 alone
closed, the current is I0 .
If the source is changed v v
to 100 V, 100 Hz, the
current with K1 alone
closed and with K 2 f f
alone closed will be re- Fig. 103 (3) (4)
spectively,
Fig. 105
I0
(a) I0, (b) I0, 2I0 (a) 1 (b) 2
2
(c) 3 (d) 4.
I0
(c) 2I0, I0 (d) 2I0, . [National Standard Exam. in Physics 1993]
2
[National Standard Exam. in Physics 1990] 228. In an L-R circuit, an inductance of 0.1 H and a resist-
ance of 1 Ω are connected in series with an ac source of
225. In a series resonant circuit, the AC voltages across R,
voltage V = 5 sin 10 t. The phase difference between the
inductance L and capacitance C are 5 V, 10 V, 10 V
current and applied voltage will be
respectively. The AC voltage applied to the circuit will
(a) π/2 (b) π/4
be
(c) π/6 (d) 0. [AMU 2001]
(a) 25 V (b) 20 V
229. A transmitter transmits at a wavelength of 300 metre.
(c) 10 V (d) 5 V.
A condenser of capacitance 2.4 µF is being used. The
[National Standard Exam. in Physics 1992] value of the inductance for the resonant circuit is ap-
226. In a domestic radio, we have a band switch S and a proximately
tuning knob T so as to receive signals from wide range (a) 10 –4 H (b) 10 –6 H
of frequencies. If L and C refer to the inductance and (c) 10 –8 H (d) 10 –10 H.
capacitance of the resonant circuit, then
[CMC LDH 2001]
(a) S changes C ; T also changes C
230. An inductance has a high resistance to AC and low to
(b) S changes L ; T also changes L DC. When a DC voltage source having some AC com-
(c) S changes C ; T changes L ponent superimposed on it sends current through an
(d) S changes L ; T changes C. inductance to load resistance,
[National Standard Exam. in Physics 1992] (a) the AC voltage falls appreciably across the load and DC
voltage falls by only a small amount
227. A series R, L, C circuit is
(b) the DC voltage falls appreciably across the load and AC
shown here. The source fre-
VL VC component falls by only a small amount
quency f is varied, but the cur-
(c) both AC and DC voltages fall to nearly zero
rent is kept unchanged. Which
(d) both AC and DC voltages fall by the same small per-
of the curves showing changes L C
centage.
of VC and VL with frequency R
would be valid for the circuit
V 231. In a LCR series circuit, the AC voltages across R, L
under consideration ? Fig. 104 and C come out as 10 V, 10 V and 20 V respectively.
The voltage across the entire combination will be
ELECTROMAGNETIC INDUCTION AND ALTERNATING CURRENT 387

V0I0
(a) 30 V (b) 10 3 V (a) V0I0 (b)
2
(c) 20 V (d) 10 2 V. V0I0
(c) (d) zero. [Manipal 2001]
2
[National Standard Exam. in Physics 1990]
236. A generator produces a voltage that is given by V = 240
232. An alternating current of 1.5 mA rms and angular sin 120 t volt, where time t is in second. The frequency
frequency ω = 100 rad s–1 flows through a 10 kΩ resistor and rms voltage are
and a 0.50 µ F capacitor in sereis. The rms potential
(a) 19 Hz and 120 volt (b) 19 Hz and 170 volt
difference across the capacitor is
(c) 60 Hz and 240 volt (d) 754 Hz and 170 volt.
(a) 4.8 V (b) 15 V
[CPMT 1994]
(c) 30 V (d) 34 V
237. An AC circuit consists of an inductor of inductance
(e) 190 V.
0.5 H and a capacitor of capacitance 8 µF in series.
[Oxford and Cambridge Schools Examination] The current in the circuit is maximum when the angu-
233. In the circuit shown in Fig. 106, the rms currents I1 , I2 lar frequency of AC source is
and I3 are altered by varying the frequency f of the (a) 5000 rad s–1 (b) 4000 rad s–1
oscillator. The output voltage of the oscillator remains (c) 2 × 105 rad s–1 (d) 500 rad s–1.
sinusoidal and has a fixed amplitude. [CPMT 1995]
238. In a series LCR circuit, R = 10 Ω and the impedance
Z = 20 Ω. Then the phase difference between current
and voltage is
(a) 45° (b) 90°
(c) 60° (d) 30°.
[Karnataka 1999]
239. An rms current of 0.6 A flows when an inductance of
40 mH is connected to an AC source of 0.24 V (rms).
The angular frequency of the source is
(a) 1 rad/s (b) 2 rad/s
(c) 5 rad/s (d) 103 rad/s.
Fig. 106 Fig. 107 [CMC LDH 2002]
Which curves in Fig. 107 indicate correctly the varia- 240. An inductor of 2 henry and a resistance of 10 ohm are
tions with frequency of the currents I1 , I2 and I3 ? connected in series with a battery of 5 volt. The initial
I1 I2 I3 rate of change of current is
(a) Q Q Q (a) 0.5 A s–1 (b) 2.0 A s–1
(b) R Q Q (c) 2.5 A s–1 (d) 0.25 A s–1.
(c) Q P R
(d) Q R P
[MP PMT 2001]
(e) P Q Q. 241. A 1 M Ω resistor and an initially uncharged 10 µF
234. A resistance of 20 Ω is connected to a source of an capacitor are connected in series with a switch and a
alternating potential V = 220 sin (100 πt). The time 100 V dc supply. What are the charge Q on the capacitor
taken by the current to change from its peak value to and the charging current I immediately after the switch
rms value, is is closed ?
(a) 0.2 s (b) 0.25 s Q/C I/A
(c) 2.5 × 10 –3 s (d) 2.5 × 10 –3 s. (a) 0 0

[MP PET 2001] (b) 0 10 –3


(c) 0 10 –4
235. The current and voltage in an AC circuit are respectively
given by I = I0 sin ωt, V = V0 sin ωt. The power consumed (d) 10 –3 0
in the circuit is (e) 10 –4 0.
388 COMPREHENSIVE OBJECTIVE PHYSICS

242. A resistor and an inductor are connected in series to a Y Y


220 volt AC supply. When measured with an AC volt-
meter, the potential difference across the resistor is 132
volt. Then the potential difference across the terminals
of the inductor is e e
(a) 88 volt (b) 352 volt
(c) 176 volt (d) 220 × 132 volt.
O i X O i X
[DPMT 1998]
(a ) (b)
243. An alternating current I/A varies with time t/s accord- Y Y
ing to the equation
I = 5 sin (100 πt).
What is the mean power developed by the current in a
resistive load of resistance 10 Ω ? e e
(a) 125 W (b) 160 W
(c) 250 W (d) 500 W.
O i X O i X
244. An alternating current flows through a resistor. The
(c) (d )
variation with time of
Fig. 111
this current is shown in [Raj. CET 1994]
Fig. 108. 246. The reactance of a circuit is zero. It is possible that the
Which graph of Fig. 109 circuit contains
shows the variation with (a) an inductor and a capacitor
the time of the power dis- Fig. 108
(b) an inductor but no capacitor
sipated in the resistor ?
(c) a capacitor but not inductor
(d) neither an inductor nor a capacitor.
power

power

247. A sinusoidal supply of frequency 100 Hz and rms


voltage 12 V is connected to a 2.2. µF capacitor. What
O O
time/s time/s is the rms value of the current ?
(a) 5.5 µA (b) 2.6 mA
(a ) (b) (c) 26 µA (d) 17 mA
(e) 0.42 mA.
power

power

248. An LCR circuit contains resistance of 100 ohm and a


supply of 200 volt at 300 radian s–1 angular frequency.
O
time/s
O
time/s If only capacitance is taken out from the circuit and
the rest of the circuit is joined, current lags behind the
voltage by 60°. If on the other hand only inductor is
(c) (d )
taken out, the current leads by 60° with applied volt-
Fig. 109 age. The current flowing in the circuit is
(a) 1 A (b) 1. 5 A
245. Switch S of the
circuit shown in (c) 2 A (d) 2.5 A. [Raj. CET 1996]
Fig. 110 is closed at 249. A coil of inductance 0.20 4.0 Ω
t = 0. If e denotes the H is connected in series
induced emf in L with a switch and a cell
and i, the current of emf 1.6 V. The total re-
flowing through the sistance of the circuit is 1.6 V 0.20 H
circuit at time t, Fig. 110 4.0 Ω.
then which of the What is the initial rate of
following graphs correctly represents the variation of e growth of current when the Fig. 112
with i ? switch is closed ?
ELECTROMAGNETIC INDUCTION AND ALTERNATING CURRENT 389
(a) 0.050 A s–1 (b) 0.40 A s–1 at a mean rate P. A diode having zero resistance in the
(c) 0.13 A s–1 (d) 8.0 A s–1 forward direction and infinite resistance in the reverse
(e) 0.32 A s–1. direction is inserted in the circuit in series with the
250. At t = 0, an inductor of zero resistance is joined to a cell resistor.
of emf E through a resistance. The current increases What is the new rate of energy dissipation ?
with a time constant τ. The emf across the coil after P
time t is (a) zero (b)
4
(a) E t/τ (b) E e–t/τ
P P
(c) E e–2 t/τ (d) E ( 1 – e–t/τ). (c) (d)
2 2 2
[AIIMS 2000]
P
251. An alternating current having peak value 14 A used to (e) .
heat a metal wire. To produce the same heating effect, 2
a constant current i can be used where i is 255. A solenoid has an inductance of 60 henry and a resist-
(a) 14 A (b) about 20 A ance of 30 ohm. If it is connected to a 100 volt battery,
(c) 7 A (d) about 10 A. e −1
how long will it take for the current to reach
[AIIMS 1998] e
252. The voltage of an AC supply varies with time (t) ≈ 63.2% of its final value ?
as V = 120 sin 100 π t cos 100 πt.The maximum voltage (a) 1 second (b) 2 second
and frequency respectively are (c) e second (d) 2e second.
120 [MP PET 2000]
(a) 120 V, 100 Hz (b) V, 100 Hz
2 256. Radio waves with wavelength 300 π metre are trans-
(c) 60 V, 200 Hz (d) 60 V, 100 Hz. mitted from a transmitter. An inductive coil connected
[MP PMT 2001] in series with a capacitor of 1.0 µF to receive these
253. The circuit shows a waves. The inductance of coil in henry is
bridge rectifier with a (a) 2.5 × 10 –9 (b) 2.5 × 10 –8
sinusoidal alternating (c) 1 × 10 –7 (d) 2.5 × 10 –7.
voltage applied to it, the [Haryana PMT 2002]
output terminals P and 257. A coil of resistance 100 ohm and self-inductance
Q being joined by a load 0.5 henry is connected to an AC source of frequency
resistor.
100
If diode X were removed hertz. The phase difference between voltage and
Fig. 113 π
leaving a break in the current is
circuit, which of the following traces would be seen on (a) 0° (b) 30°
a cathode ray oscilloscope connected across PQ ?
(c) 45° (d) 90°. [Pb. PMT 1995]
258. An AC ammeter is used to measure current in a cir-
cuit. When a given DC passes through the circuit, the
(a ) (b) AC ammeter reads 3 A. When another AC passes
through the circuit, the AC ammeter reads 4 A. Then
the reading of this ammeter if DC and AC flow through
the circuit simultaneously is
(a) 3 A (b) 4 A
(c) (d ) (c) 7 A (d) 5 A.

(e) ELECTRICAL MACHINES AND DEVICES

Fig. 114 259. A shunt generator has a field resistance of 240 Ω and
an armature resistance of 0.6 Ω. The generator gets
254. A sinusoidal alternating supply is connected across the
overheated if the armature loss just exceeds 240 W.
terminals of a resistor causing energy to be dissipated Maximum armature current is
390 COMPREHENSIVE OBJECTIVE PHYSICS

(a) 400 A (b) 20 A (a) 220 volt (b) 225 volt


(c) 1 A (d) 21 A. (c) 215 volt (d) 300 volt. [AFMC 2001]
260. In Q. 259, if the emf is 150 V, then the terminal potential 268. The overall efficiency of a transformer is 90%. The
difference is transformer is rated for an output of 9000 W. The pri-
(a) 150 V (b) 162 V
mary voltage is 1000 volt. The ratio of turns in the pri-
mary to secondary coil is 5 : 1. The iron losses at full
(c) 138 V (d) 144 V.
load are 700 W. The primary coil has a resistance of 1
261. In Q. 259, the current in the load resistor is Ω. The voltage in the secondary coil is
FG 138 IJ (a) 1000 V (b) 5000 V
(a) 20 A
H
(b) 20 −
240
A
K (c) 200 V (d) zero volt.

FG 20 + 138 IJ A 138
269. In Q. 268, the current in the primary coil is
(c)
H 240 K (d) 20 ×
240
A. (a) 9 A
(c) 1 A
(b) 10 A
(d) 4.5 A.
262. In Q. 259, the efficiency of the generator is 270. In Q. 268, the copper loss in the primary coil is
138 × 19.425 138 × 0.575 (a) 100 W (b) 700 W
(a) × 100 (b) × 100
150 × 20 150 × 20 (c) 200 W (d) 1000 W.
138 19.425 271. In Q. 268, the copper loss in the secondary coil is
(c) × 100 (d) × 100.
150 20 (a) 100 W (b) 700 W
263. In a step-up transformer, the turns ratio is 1 : 10. A (c) 200 W (d) 1000 W.
resistance of 200 ohm connected across the secondary
272. In Q. 268, the current in the secondary coil is
is drawing a current of 0.5 A. What is the primary volt-
(a) 45 A (b) 46 A
age and current ?
(a) 50 V, 1 A (b) 10 V, 5 A (c) 10 A (d) 0 A.
(c) 25 V, 4 A (d) 20 V, 2 A. 273. In Q. 268, the resistance of the secondary coil is nearly
[MP PET 2000] (a) 0.01 Ω (b) 0.1 Ω
264. A 50 Hz alternating current of peak value 1 ampere (c) 0.2 Ω (d) 0.4 Ω.
flows through the primary coil of a transformer. If the 274. An electric motor has a back emf of 110 V and arma-
mutual inductance between the primary and secondary ture current of 90 A. The armature is making 24.5 revo-
be 1.5 henry, then the peak value of the induced voltage lutions per second. The torque on the armature is
is 110 × 90 2 π × 24.5
(a) 75 V (b) 150 V (a) Nm (b) Nm
2π × 24.5 110 × 90
(c) 225 V (d) 300 V. [Pb. CET 1998]
π × 24.5 90 π
265. The number of turns in the primary coil of a trans- (c) Nm (d) N m.
110 110 × 24.5
former is 200 and the number of turns in the second-
275. An electric motor runs on a DC source of emf 200 volt
ary coil is 10. If 240 volt AC is applied to the primary,
and draws a current of 10 ampere. If the efficiency is
the output from the secondary will be
40%, then the resistance of the armature is
(a) 48 V (b) 24 V
(a) 5 ohm (b) 12 ohm
(c) 12 V (d) 6 V. (c) 120 ohm (d) 160 ohm.
[All India PM/PD 2000] [Haryana PMT 2001]
266. A simple electric motor has an armature resistance of 276. The diagram shows an iron-cored transformer assumed
one ohm and runs from a DC source of 12 volt. When to be 100% efficient. The ratio of the secondary turns to
unloaded, it draw a current of 2 ampere. When a cer- the primary turns is 1 : 20.
tain load is connected, its speed becomes one-half of its
unloaded value. Then the current in ampere it draws
is
Primary
(a) 7 (b) 6
240 V coil 6.0 W
(c) 2 (d) 4. [CMC Vellore 1998] Secondary
coil
267. A dynamo dissipates 20 watt when it supplies a cur-
rent of 4 A through it. If the terminal potential differ-
ence is 220 volt, then the emf produced is Fig. 115
ELECTROMAGNETIC INDUCTION AND ALTERNATING CURRENT 391

A 240 V ac supply is connected to the primary coil and (a) 200 V (b) 40 V
a 6.0 Ω resistor is connected to the secondary coil. (c) 1000 V (d) zero V.
What is the current in the primary coil ? 284. In Q. 283, if the efficiency is 100%, then the current in
(a) 0.10 A (b) 0.14 A the primary coil is
(c) 2.0 A (d) 40 A. (a) 10 A (b) 0.4 A
277. A DC motor has an internal resistance 4 ohm. It is (c) 2 A (d) 0 A.
operated at 220 volt and draws 5 A current. The back 285. In a laboratory experiment to test a transformer, a stu-
emf produced is dent used the circuit shown in the diagram to take meas-
(a) 80 V (b) 160 V urements.
(c) 200 V (d) 120 V.
278. In the above problem, the power given to the motor is Ip Is
A A
(a) 550 W (b) 100 W
(c) 1000 W (d) 1100 W.
Np Ns
Vp Vs
279. In Q. 277, the useful mechanical power developed is turns turns
(a) 550 W (b) 100 W
(c) 1000 W (d) 1100 W.
280. When an electric motor is run at 120 volt, 10 A current
Fig. 116
flows through it and the induced back emf is 115 V.
What will be the current flowing in the coil at the time Two of the original entries in the student’s results ta-
of switch off ? ble are missing as shown :
(a) 230 A (b) 10 A
(c) 240 A (d) zero.
Vp/V Ip/mA Np turns Vs/V Is/mA Ns turns

281. An electric motor driven from a constant voltage sup- 240 2.0 ? ? 50 50
ply is used to raise a load. If the load is increased, which
one of the following sets of changes occurs ? Assuming the transformer was 100% efficient, what are
speed of rotation induced emf in coil current in coil the missing results ?
(back emf) Np turns Vs/V
(a) decreases decreases increases (a) 2 6000
(b) increases increases decreases (b) 50 9.6
(c) decreases decreases decreases (c) 480 1.0
(d) increases decreases increases (d) 1250 9.6
(e) decreases increases decreases (e) 1250 240.
282. A generator produces 100 kW of power at a potential 286. An ideally efficient transformer has a primary power
difference of 10 kV. The power is transmitted through input of 10 kW. The secondary current when the trans-
cables of total resistance 5 Ω. How much power is dis- former is on load is 25 ampere. If the primary : second-
sipated in the cables ? ary turns ratio is 8 : 1, then the potential difference
(a) 50 W (b) 250 W applied to the primary coil is
(c) 500 W (d) 1000 W 104 × 82 104 × 8
(a) V (b) V
(e) 50000 W. 25 25
283. A step-up transformer operates on a 200 volt line and 104 104
(c) V (d) V. [AFMC 2002]
supplies a current of 2 ampere. The ratio of primary 25 × 82 25 × 8
and secondary winding is 1 : 5. The output voltage in
the secondary is
392 COMPREHENSIVE OBJECTIVE PHYSICS

Answers (Set II)


127. (b) 128. (b) 129. (c) 130. (d) 131. (a) 132. (b) 133. (c) 134. (b)
135. (b) 136. (d) 137. (b) 138. (a) 139. (d) 140. (d) 141. (a) 142. (c)
143. (b) 144. (c) 145. (a) 146. (b) 147. (a) 148. (d) 149. (b) 150. (b)
151. (b) 152. (a) 153. (c) 154. (b) 155. (a) 156. (b) 157. (b) 158. (b)
159. (b) 160. (b) 161. (c) 162. (a) 163. (c) 164. (a) 165. (a) 166. (a)
167. (b) 168. (b) 169. (c) 170. (c) 171. (c) 172. (c) 173. (b) 174. (c)
175. (c) 176. (a) 177. (d) 178. (a) 179. (b) 180. (e) 181. (b) 182. (a)
183. (c) 184. (d) 185. (b) 186. (d) 187. (a) 188. (c) 189. (a) 190. (c)
191. (c) 192. (e) 193. (b) 194. (d) 195. (a) 196. (b) 197. (e) 198. (d)
199. (c) 200. (d) 201. (a) 202. (a) 203. (c) 204. (a) 205. (c) 206. (b)
207. (d) 208. (a) 209. (b) 210. (b) 211. (c) 212. (b) 213. (b) 214. (c)
215. (a) 216. (e) 217. (e) 218. (c) 219. (d) 220. (a) 221. (d) 222. (d)
223. (b) 224. (a) 225. (d) 226. (d) 227. (a) 228. (b) 229. (c) 230. (a)
231. (d) 232. (c) 233. (d) 234. (d) 235. (b) 236. (b) 237. (d) 238. (c)
239. (d) 240. (c) 241. (c) 242. (c) 243. (a) 244. (c) 245. (b) 246. (a)
247. (d) 248. (c) 249. (d) 250. (b) 251. (d) 252. (d) 253. (b) 254. (c)
255. (b) 256. (d) 257. (c) 258. (d) 259. (b) 260. (c) 261. (b) 262. (a)
263. (b) 264. (d) 265. (c) 266. (a) 267. (b) 268. (c) 269. (b) 270. (a)
271. (c) 272. (b) 273. (b) 274. (a) 275. (b) 276. (a) 277. (c) 278. (d)
279. (c) 280. (a) 281. (a) 282. (c) 283. (c) 284. (a) 285. (d) 286. (b).

Solutions (Set II)


ndφ n(W2 − W1) d d
127. I = − =− 130. E = – (φ) = – (π × 10–9 cos ωt)
R ′ dt 5 Rt dt dt
F πr I cos ωt
2 = – ( – π ω sin ωt × 10–9) = π × 10–9 ω sin ωt.
128. φ = BA cos θ = B GH 2 JK π × 10−9 ω sin ωt π
131. i = = × 10–9 ω sin ωt.
d 1 2 2
E= − φ = Bπr 2ω sin ωt
dt 2 132. As the magnet moves towards the coil, the magnetic
E2 B2π 2r 4ω 2 sin 2 ωt flux increases (non-linearly). Also there is a change in
P= = polarity of induced emf when the magnet passes on to
R 4R
the other side of the coil.
Now, < sin2 ωt > = 1/2
133. Average emf
(Bπr2ω)2
∴ <P>= 0 + Bvl 1 1 1
8R = = Bvl = B (lω)l = Bl2ω.
2 2 2 2
129. Magnetic field due to larger loop
1 1
µ I 4 π × 10−7 × 1 134. × C × 400 × 400 = ×1×2×2
= 0 = T = 2π × 10 –6 T 2 2
2r 2 × 01
.
4 1
Now, magnetic flux linked with the smaller loop, φ or C= F = × 10 –4 F = 0.25 × 10 –4 F
400 × 400 4
= NBA cos ωt = 1 × 2π × 10 –6 × 5 × 10 –4 cos ωt
= 25 × 10 –6 F = 25 µF.
= π × 10 –9 cos ωt.
ELECTROMAGNETIC INDUCTION AND ALTERNATING CURRENT 393

L2 500 × 500 25 dI1 dI2


135. = = But = (given)
L1 600 × 600 36 dt dt
25 V1 L2 8 4
or L2 = × 108 mH = 75 mH. ∴ V1 = L1 and V2 = L2 or = = =
36 V2 L1 2 1
136. Induced current in B = 0.006 A = 6 × 10–3 A
Again, same power is given to the two coils.
Induced emf in B
= 6 × 10 –3 × 4 volt = 24 × 10 –3 volt I1 V2 1
∴ V1I1 = V2I2 or = =
dI I2 V1 4
Now, M = 24 × 10 –3
dt 1 2
Again, energy = LI
24 × 10 −3 × 0.02 2
or dI = A = 0.16 A
3 × 10 −3 1
W2 2 L2I2
2
FG L IJ = FG I IJ
2 2
2
2 4
(4)2 = .
137. M=
µ 0µr N pNs A
l

W1
=
1
L1I1
2
=
HL K HI K
1 1
=
8 1
2
4 π × 10−7 × 800 × 400 × 20 × π ( 0.02 )2 d
= H 148. E=– (φ) = – (12t – 5)
20π × 10 −2 dt
8 × 4 × 2 × π × 0.02 × 0.02 − (12 t − 5)
= H = 1.6 × 10 –2 H. I=
5 10
Blv 1
138. I= When t = s, then
R 4
5 × 10−2 × 0.3 × 0.2 FG1 IJ
I=
5
A = 0.6 mA.
I=
H
− 12 × − 5
4 K
A = 0.2 A.
Area is decreasing. Flux is decreasing. So, current flows 10
to increase the flux. Clearly, current should be clock- 0.4
149. M= H = 0.2 H.
wise. So, it flows from B to C through 5 Ω. 2
dI 1
139. E = M = 1.25 × 80 volt = 100 volt. 150. E = Blv sin θ = 0.1 × 4 × 10 × volt = 2 volt.
dt 2
FG d IJ 3
FG 80 IJ 3
151. E = M
dI
= 0.005 (10 ω cos ωt) = 0.05 ω cos ωt
H 40 K × 4 × 10−5 Wb
1
140. φ2 =
Hd K
2
φ1 = dt
E0 = 0.05 ω = 0.05 × 100 π = 5π.
= 8 × 4 × 10 –5 Wb = 32 × 10–5 Wb
152. Consider the force on an electron in PQ. This electron
28 × 10 −5 experiences a force towards Q. Clearly, all free elec-
e=– volt = – 3.5 × 10 –3 volt = – 3.5 mV. trons in PQ tend to move towards Q.
0.08
. −1
01 2000 × 6 × 10−4
141. E = – volt = 9 volt. 153. M = H = 0.4 H.
01
. 3
φ 0.9 22
142. q = = C = 0.09 C. 154. ∆φ = NA [B2 – B1] = 50 × × (3 × 10–2)2 [0.25]
R 10 7
= 353.57 × 10–4 Wb
dB
143. | E | = NA 353.57 × 10−4
dt E= volt = 17.7 volt.
2 × 10 −3
|E| 200 × 2
or N = = = 24.997 ≈ 25.
dB 3.14 × 0.2 × 0.2 × 127.4 5
A 155. E = 0.18 × 10–4 × 1 × × 100 V
dt 18
dI 0.01 = 5 × 10–4 V = 0.5 × 10–3 V = 0.5 mV.
144. | E | = M
dt
= 4 × 10–3 ×
0.002
volt = 0.02 volt.
156. E =
1
LI 2 =
LM OP I
1 Nφ 2 =
1
NφI.
145 to 147. V1 = L1
dI1
and V2 = L2
dI2 2 2 I N Q 2
dt dt
394 COMPREHENSIVE OBJECTIVE PHYSICS

dφ NBA dI 30 1
157. dq = or dq = 170. = A s −1 = A s − 1
R R dt 60 2
−3 1
dq × R 2 × 10 × 50
or B= = T = 0.2 T. Now, 6 = L × or L = 12 henry.
NA 1000 × 5 × 10−4 2
d
158. E = B 2gh l = 2 × 10 –5 2 × 9.8 × 10 × 2.5 × 103 V 171. E=– (φ) = – (10t + 6)
dt
= 2 × 14 × 2.5 × 10 –2 V = 0.7 V Et = 3s = – (10 × 3 + 6) = – 36
0.7 Et = 0s = – 6
I= A = 0.02 A.
35 − 36
2 × 4 − 2 ×1 8−2 Required ratio is or 6.
159. E = = volt = 3 volt. −6
2 2 172. E0 = NBA ω
dI E 15000 E0 = 100 × 3 × 10–3 × 0.20 × 0.10 × 2π × 20 V
160. E = M or M = =
dt dI/dt 3 = 0.24 π volt.
0.001
µ 0 π N2r
15000 1 173. L=
= × H = 5 H. 2
3 1000 When N is doubled, r would be halved.
161. E =
1
× 0.1 × 0.1 × 0.1 × 2 × π × 10 volt FG IJ
r
2
= π × 10 –2 volt. L′ =
µ 0 π (2N)2
2H K
2
162. More the rate of change of magnetic flux, larger is the µ 0 πN 2 r
induced emf. L′ = 2 or L′ = 2 L
2
163. Induced charge does not depend upon time.
µ N2 A
1 174. 0.1 = 0
164. W = LI 2 l
2
Induced current is more in case (a). So, work done is
µ0 A NLM OP 2
1000
more in case (a).
L = µr
l 10N Q or L =
100
× 0.1 H = 1 H.

Nφ LI 4 π × 10−7 × 400 × 400 × 4 × 10 −4


165. L= or φ = 175. L = H
I N 20 × 10 −2
8 × 10−3 × 5 × 10 −3 = 4.019 × 10 H ≈ 4 × 10 –4 H.
–4

or φ= Wb = 10 –7 Wb
400 176. The inductance acts as a
4 π × 10 −7
1 short-circuit. The whole of the
= Wb = µ Wb ≈ 0.1 µ0 Wb. current flows through the in-
4π 4π 0
ductance. Since no current
166. The two loops produce equal and opposite magnetic
flows through the bulb there-
fields.
fore the bulb does not glow.
167. E = 0.1 × 200 volt = 20 volt.
168. φ = 500 × 0.1 × 100 × 10 –4 = 0.5 Wb NBA Fig. 117
177. q=
0.5 R
E= volt = 5 volt.
0.1 qR 32 × 10 −6 × 200
or B= = T = 0.566 T.
1 NA . × ( 6 × 10−3 )2
100 × 314
169. E= LI 2
2 178. E = 1.5 × 10–3 × 10 × 10 volt = 0.15 volt.
Nφ dφ
But Nφ = LI or L = 179. E=–N = – 100(3t2 – 2) × 10–3
I dt

LM OP I
1 Nφ 2 1 (E)t = 2s = – 100 (3 × 4 – 2) × 10–3 V = – 1 V.
E=
2 IN Q or E =
2
NφI
180. The induced emf is directly proportional to
1 dB dI
or E= × 100 × 5 × 10–5 × 2 J = 5 × 10–3 J. = µ0n .
2 dt dt
ELECTROMAGNETIC INDUCTION AND ALTERNATING CURRENT 395

dI dI for the solenoid to increase its radius. If an external


Hence induced emf E ∝ . The graph of is given magnetic field helps the field of the solenoid, then the
dt dt
by tendency is more. If the external magnetic field can-
cels the magnetic field of the solenoid, then the sole-
I dI
E∝ noid has no tendency to increase its radius.
dt
188. The mutual inductance, M, between two coils A and B
is defined by the equation :
emf induced in B (or A) by changing current in A (or
O t O t B) = M × rate of change of current in A (or B).
Hence the mutual inductance M remains the same
(a ) (b) whether X or Y is used as the primary.
Fig. 118 189. φ = 103 × 0.01 Wb = 10 Wb.

µ 0I Bvl mg R
181. Magnetic field at centre of large coil = 190. BIl = mg or B l = mg or v = 2 2 .
2R R B l
Magnetic flux linked with 191. Shape is immaterial. Only the end points are relevant.
smaller coil 192. The back-emf E induced in the armature coil of the
µ I motor is related to the current Ia from the source of
= 0 × πr2
2R potential difference V by the equation :
φ µ 0 πr 2
M= = V−E
I 2R Ia = where Ra = armature coil resistance.
Ra
r2
∴ M∝ . Now the back-emf is proportional to the angular speed
R Fig. 119
of the armature coil ω
E 20
182. L= = H or E = kω, where k is the constant of proportionality.
dI/dt 16 /0.05
Thus, we have
20 × 0.05
= H = 62.5 × 10–3 H = 62.5 mH. V k
16 I= − ω
R a Ra
183. When coil is rotated through 180°, the rate of change
of magnetic flux is doubled. a linear equation such that the current I from the
source decreases linearly with the speed of rotation ω
184. | E | = BHlv
of the motor. That is best represented by graph e.
= 2 × 10–5 × 2 × 50 volt
µ 0N1I1
= 2 × 10 –3 volt 193. φ2 = N2B1A or φ2 = N2 A
L
= 2 millivolt
Using Fleming’s left µ 0 N1N2 A
or φ2 = I1
hand rule, we find that L
Fig. 120
electrons shall experi- µ 0 N1N2 A
ence force towards A. B will be deficient in electrons. Comparing with φ2 = MI1, we get M = .
L
So, B will be positively charged.
194. ∴ E = – (10t – 4)
185. Effective resistance is 4 Ω.
At t = 0.2 second, E = – (10 × 0.2 – 4) volt
E Blv IR E = 2 volt.
I= = or v =
R R Bl 195. The induced emf on the second coil is given by
1 × 10 −3 × 4 dI
or v= −2
m s–1 E=M .
2 × 10 × 10 dt
or v = 0.02 ms–1 = 2 cm s–1. Hence the power dissipated in the resistor R is given
186. | E | = 500 × 100 × 10 –4 × 1 = 5 volt by
187. Due to magnetic field of the solenoid, the wire experi- E2 M2 dI FG IJ 2

ences a radially outward force. So, there is a tendency


P=
R
=
R dt H K .
396 COMPREHENSIVE OBJECTIVE PHYSICS

196. The back-emf E is related to the rate of change of cur- 1 1 1 L


199. += or L′ = .
dI L ′
L L 2
rent and the self-induced of the coil L by the equa-
dt 200. From the given graph of φ, we may define φ by the
tion : equation,
E=L
dI
FG 2 πt IJ
dt
dI 2
φ = φ0 sin
HTK
Given that E = 4 V, = = 40 A s–1 The induced emf E in the coil is thus given by
dt 50 × 10−3
dφ 2π 2 πtF I
Thus, we have
E
=
4
L= = 0.10 H.
E=–
dt
= – φ0
T
cos
T H K
dI/dt 40 i.e. a negative cosine function which is best represented
197. When the switch is closed, a current flows through in graph d.
the coil as shown thus sets up a magnetic flux φ 201. Component of weight along the inclined plane
through = mg sin θ
I B 2l 2v
Blv
Again, F = BIl = B l=
R R

N B 2l2v mg R sin θ
Now, = mg sin θ or v = .
R B2 l 2
f
202. The variation of θ can be described in the stages :
B W E (i) When the magnetic bar enters the solenoid, by
Lenz’s law, the current in the coil is induced such
that it opposes the increase in flux due to the
magnetic field by the magnetic bar. The induced
S
current is indicated by the deflection θ as in the
Fig. 121
diagram.
the coil linking the ring. The motion of the ring will be
such that, by Lenz’s law, it opposes the sudden in-
crease in flux linkage from the west. Hence, the ring
moves towards E (east).
198. Assume the area of the flat coil of copper wire to be A.
Then the flux linking the coil is
FG 2 πt IJ
φ = BA = B0A sin
HTK
Thus, the induced emf in the coil is given by

dφ 2πB0 A 2 πt FG IJ
E=–
dt
=−
T
cos
T H K Fig. 122

2 πt (ii) When the magnetic bar moves inside the solenoid,


Maximum of E occurs when cos =–1 flux change due to N-pole is cancelled by that due
T
to S-pole. Hence, no deflection is indicated, i.e.
2 πt T (θ = 0).
or = π i.e. t = .
T 2 (iii) When the magnetic bar leaves the solenoid, again
by Lenz’s law, the current in the coil is induced
dφ dB
(Note. since E = – =−A , maximum of E occurs when such that it tends to maintain the flux in the coil
dt dt
due to the magnetic field by the magnetic bar. Thus,
dB an opposite induced current flows through the coil
negative gradient of B or – is the maximum it is obvious
dt and is indicated by the deflection θ in the diagram
from the graph that this occurs at t = T/2, t = 0 and t = T.) below.
ELECTROMAGNETIC INDUCTION AND ALTERNATING CURRENT 397
–q
l2
Clearly, M ∝ .
L
250 × 10 −3
207. E = – 0.4 × 10 –3 × V = – 1 mV.
N S 01.
208. E = E0 sin 100 t
FG π IJ
Fig. 123
I = I0 sin 100t ±
H 4 K
Hence, the variation of θ is best represented in graph XC XC
tan φ = or tan 45° = or XC = R
(a). R R
R 1 1 1 1
203. Note that each radius has a resistance of . Also, Now, X C = = or C = =
2 ωC 100C 100 X C 100R
the two radii are in parallel. So, the equivalent resist-
R This condition is satisfied only when R = 1000 Ω =
ance is . 1 kΩ and C = 10 µ F.
4
209. Current during a charging RC circuit,
1
Br 2ω
2 2 Br 2 ω E − t / RC
I=
∴ I= = . e
R/4 R R
QR Q charge t E
= = ln I = − + ln
204. = current. RC R
L L/R time
when R is doubled, slope of the curve increases. Also,
205. Following arguments shall helps us to arrive at the at t = 0, the current will be less for a increased value
right choice. of resistance.
(i) Length is doubled but number of turns/length re- 210. It = ∞ = I0
mains the same. Clearly, the total number N of
F I
turns is doubled.
GG 1 − e −
10
×1
JJ
(ii) Since the linear dimensions are doubled therefore
area of cross-section becomes four times.
It = 1s = I0
H 5
K
FG 1 IJ
(iii)
µ N2 A
L= 0
= I0 (1 – e–2) = I0 1 − H e2 K
l
It = ∞ I0 e 2 e2
µ 0 ( 2N) (4A) 2
µ N A 2 ∴ = = .
L′ = =8 0 = 8L. It = 1s I0 ( e 2 − 1) e2 − 1
2l l
206. Let a current I flow through 211. P = EI
the square loop of L. P = E0 sin ωt I0 cos ωt
Magnetic field at centre C of E0I0
loop P= [2 sin ωt cos ωt]
2
µ0 I P ∝ sin 2ωt
=4×

LLM OP [2 sin 45°]
Clearly, the angular frequency for instantaneous
2 N Q power is 2ω.

2µ 0 I 2 2 2 µ 0I Fig. 124
T
= × = 212. Clearly, = 5 × 10–3 s or T = 2 × 10–2 s
πL 2 πL 4
Magnetic flux linked with small square loop of side l, φ 1
ν= × 102 Hz = 50 Hz.
2
2 2 µ 0I 2
= l 10
πL 213. I0 = A = 1 A.
If M be the mutual inductance, then 10
214. Clearly, XL = R
2 2 µ 0 l2
φ = MI ∴ M = or L × 2 × 3.14 × 1000 = 100
πL
398 COMPREHENSIVE OBJECTIVE PHYSICS

100
or L= H 223. V= 2002 + 1502
2 × 3.14 × 1000
= 15.9 × 10–3 H = 15.9 mH ≈ 16 mH. or V = 10 400 + 225 volt = 250 volt.
215. Lω = 5 × 10–3 × 2000 = 10 Ω 224. At 50 Hz, Lω = R ; At 100 Hz, Lω = 2R.
So, current remains unchanged in R. However, it be-
1 1 100 comes half in L.
= −6
= Ω = 10 Ω
Cω 50 × 10 × 2000 10
225. V = 52 + (10 − 10)2 = 5 volt.
1 226. T can change C continuously but S can change L only
Since Lω = ,
Cω by steps.
∴ Z = R = 6 + 0.1 + 4 = 10.1 Ω 1
227. VC ∝ and VL ∝ f.
E 20 f
I0 = = A ≈ 2 A.
Z 10.1 ωL 10 × 01
.
228. tan φ = = =1
216. I is maximum at a certain frequency. This frequency R 1
is called resonance frequency.
∴ φ = π/4.
217. VC and I are 90° out of phase. So, ‘1’ is wrong. VR and
I are in same phase. c 3 × 108
229. ν= = Hz = 106 Hz
218. Voltages across L and C cancel out. λ 300
So, voltage across R is 220 V. 1 1
ν= ⇒ ν2 = 2
2π LC 4 π LC
220
Again, I0 = A = 2.2 A.
100 1
or L=
22 4 π ν2 C
2
219. 50 = L × 2 × × 50
7 1
or L= H
7 4 × 9.87 × 1012 × 2.4 × 10−6
L= H = 0.16 H.
44
220. P = Ev Iv cos φ 10−6
= H = 0.01 × 10–6 H = 10–8 H.
4 × 9.87 × 2.4
Ev R
P = Ev 230. The inductance offers large opposition to AC but small
Z Z opposition to DC. This small opposition to DC is due
Ev2R 110 × 110 × 11 to the fact that the coil is not resistanceless.
or P= 2 = W = 275 W.
Z 22 × 22
231. V= 102 + ( 20 − 10)2

221. V= 402 + ( 90 − 60)2 = 102 + 102 = 10 2 volt.

1600 + 900 volt = 50 volt. 232. The given circuit is a series CR circuit. Determine the
or V=
capacitive reactance. ‘rms’ potential difference = rms
1 current × capacitive reactance.
222. ν= Hz 233. Reactance of the inductor L is given by XL = 2πf L.
2 × 3.14 8 × 0.5 × 10−6
∴ rms current through the inductor L is I2
103 V 1
= Hz = 79.6 Hz = ∝ where V is the rms of the supply voltage.
4 × 3.14 2πfL f
So, (b) is ruled out
1 1
Again, ω = rad s–1 Reactance of the capacitor C is given by XC = .
2πfC
8 × 0.5 × 10−6
∴ rms current through the capacitor is
103
= rad s–1 = 500 rad s–1
2 V
I3 = = 2πf CV ∝ f.
So, (a) and (c) are ruled out. XC
ELECTROMAGNETIC INDUCTION AND ALTERNATING CURRENT 399
The total current I1 is given by I1 = I2 + I3 abruptly. Thus, immediately after the switch is
FG V − 2πf CVIJ . closed, the capacitor remains unchanged and hence
=
H 2 πf L K Q (t = 0) = 0 C.
The charging current I can be computed as follows :
Thus, I2 is best represented by curve R. t
I3 is best represented by curve P. V0 − τ
I= e
R
I1 is best represented by curve Q.
where τ = time constant = RC
234. Both V and I are in same phase. So, let us calculate
V0 = 100 V dc
the time taken by the voltage to change from peak
value to rms value. R = resistance = 1 MΩ
Now, 220 = 220 sin 100 πt1 V0 100
At t = 0, I = = 10 –4 A.
π 1 R 1 × 106
or 100 πt1 = or t1 = s
2 200
242. VL = 2202 − 1322 volt
220
Again, = 220 sin 100 πt2
2 = ( 220 + 132) ( 220 − 132) volt
1 π = 352 × 88 volt = 30976 volt = 176 volt.
or = sin 100 πt2 or 100 πt2 =
2 4 5
1 243. The rms current is Irms = (A).
or t2 = s 2
400 The mean power developed across a resistive load of
Required time = t1 – t2
2
10 Ω = Irms R =
FG 5 IJ 2

=
1

1
=
2 −1
=
1
s H 2K × 10 = 125 W.
200 400 400 400
244. Let R be the resistance of the resistor and if we repre-
= 2.5 × 10–3 s sent the current by I = I0 sin (2πft), the power dissi-
235. P = Vv Iv cos φ pated in the resistor is then given by
Here, power factor = 1 Power, P = I2R
V0 I0 V0I0 1 2
∴ P= . = . = I02R sin2 (2πft) = I R [1 – cos (4 πft)]
2 2 2 2 0
236. ω = 120, 2πν = 120 i.e. variation of power as function of time has a fre-
120 60 × 7 quency double that of the current function.
ν= = Hz ≈ 19 Hz. 245. e = E – iR
2π 22
Clearly, the graph is a straight line with negative
1 1
237. ω = = = 500 rad s–1. slope.
LC 0.5 × 8 × 10− 6 1
246. If Lω = , then Z = 0.
R 10 1 Cω
238. cos φ = = =
Z 20 2 1
φ = 60°. 247. Reactance of the capacitor is XC =
2 πfC
239. Ev = Iv XL = Lω Iv 1
= = 723 Ω
Ev 24 2π (100)( 2.2 × 10−6 )
ω= = rad s–1 = 103 rad s–1.
Iv L 0.6 × 40 × 10−3 The rms value of the current is thus
dI V (rms) 12
240. E – L = RI I= = = 16.6 mA or 17 mA.
dt XC 723
At t = 0, I = 0 1
248. Clearly, Lω =
dI dI E 5 Cω
∴ E=L or = = A s–1 = 2.5 A s–1.
dt dt L 2 So, circuit would behave as a purely resistive circuit.
241. By conservation of energy, the voltage/charge on an Ev 200
initially uncharged capacitor can be increased Iv = = = 2 A.
R 100
400 COMPREHENSIVE OBJECTIVE PHYSICS

249. When the switch is closed (at t = 0 s), no current flows, During the negative cycle of the alternating voltage
voltage drop across the inductor is the same as the supply, there is a break in continuity due to removal
supply voltage of 16 V. Hence, by writing down the of the diode X. Hence, no potential difference across
voltage equation for the circuit, we have PQ is observed.
dI
V = RI + L
dt
dI
⇒ 1.6 = 4I + 0.20
dt
where I is the current drawn from the source.
At t = 0 s, I = 0, we thus have
dI dI 1.6
16 = 0.20 or (t = 0+) = = 8.0 A s–1.
dt dt 0.2
F t I
250. GG
I = I0 1 − e

τ
JJ
H K Fig. 126
t
t dI LI0 − τ The observation of the potential difference across PQ
dI I 0 − τ or E = L = e
= e dt τ on the oscilloscope is best represented by graph (b).
dt τ
254. Since the current in the resistor is half-wave rectified
t t t
LE − τ LE − τ
− by the diode current in series with the resistor, the
= e = L e = Ee τ
τR 1
R
R rms value for the current is that of the original
2
14 14 value.
251. i = A≈ = 10 A
2 1.4
Original rms current = I
120 value current
252. V= [2 sin 100 πt cos 100 πt] power dissipation,
2 I0 2
P=I R
or V = 60 sin 200 πt
Comparing with V = V0 sin ωt,
V0 = 60 volt and ω = 200 π t/s
or 2πν = 200 π or ν = 100 Hz
253. During the positive cycle of the alternating voltage
supply, the diode nearest to P on the upper branch
and the diode nearest to O at the lower branch are Original current in resistor
turned on and potential difference across PQ is near
to follow that of the input waveform. Fig. 127

Current
rms current =
I0 1 2 1
I = I
√2 √2

t/s

Rectified current in resistor

Fig. 128
Fig. 125
ELECTROMAGNETIC INDUCTION AND ALTERNATING CURRENT 401
Thus, the power dissipation in the resistor with recti-
fied current flow is now LM
138 20 −
138 OP
F 1 IIJ R = 1 (I R) = P
2 (iv) Efficiency = N 240 Q
× 100
P =G 150 × 20
2
H 2 K 2
L 2
F I R 138 × 19.425
I = I G1 − e JJ − t = × 100 .
255. 0GH K
L 150 × 20
263. Vs = IsR = 0.5 × 200 = 100 volt
F I R R
I = I G1 − e J
e −1 e −1
− t − t Vp np 1
e 0G
H 0J
K
or
e
L = 1−e L
Vs
=
ns
=
10
1 1 R Vs 100
or = R or t=1 or Vp = = V = 10 V
e t L 10 10
eL
L 60 Ip ns
or t= = s = 2 s. Again = = 10
R 30 Is np
1 or Ip = 10 Is = 10 × 0.5 A = 5 A.
256. f=
2π LC 1
264. T = s.
c 1 50
= For the current to rise from zero to peak value,
λ 2π LC
T 1
3 × 108 1 required time is i.e. s.
= 4 200
300 π 2π L × 10−6 1−0
Now, E = 1.5 V
1/200
1
(2 × 106)2 = = 1.5 × 200 V = 300 V.
10−6 L
Ns 10
265. Es = × Ep = × 240 volt = 12 volt.
1 1 Np 200
or 10 –6L = × 10 –12 or L = × 10 –6
4 4
12 − e
or L = 2.5 × 10 –7 H. 266. 2 = or e = 10 volt
1
Lω When speed is halved, the induced emf is also halved.
257. tan φ =
R ∴ e′ = 5 volt
100 12 − 5
0.5 × 2 × π × Now, I= A = 7 A.
π = 1 or φ = 45°. 1
=
100 267. 20 = 42 × R
20
258. I = 32 + 42 , A = 25 A = 5 A. or R= Ω
16
Note. Think in terms of heating effect of current.
I = 4A
259 to 262. (i) 240 = I2 × 0.6 ∴ V = E – IR
240 20
or I2 = = 400 or I = 20 A or 220 = E – 4 ×
0.6 16
(ii) V = 150 – 20 × 0.6 or E = 225 volt.
= (150 – 12) volt = 138 volt
Es N 1
(iii) Current in the load resistance 268 to 273. (i) = s =
Ep N p 5
= Armature current – Field current
FG 138 IJ or Es =
1
× 1000 volt = 200 volt
= 20 −
H 240 K A 5
402 COMPREHENSIVE OBJECTIVE PHYSICS

9000 90 279. Power loss = 5 × 5 × 4 W = 100 W


(ii) =
Input power 100 Usefull mechanical power
or Input power = 10000 W = (1100 – 100) W = 1000 W
or Ep Ip = 10000 120 − 115 5 1
280. 10 = or R = = Ω
10000 R 10 2
or Ip = A = 10 A At the time of switch off,
1000
(iii) Copper loss = Ip2 Rp = 10 × 10 × 1 W = 100 W 115
I= A = 230 A.
(iv) Power loss = (10,000 – 9,000) W = 1000 W 1/2
Total copper loss and iron losses 281. If V is the supply to a motor and Ia is the current, then
= (700 + 100) W = 800 W we have the power supplied to the motor is VIa and
Copper loss in secondary = (1000 – 800) W = 200 W. RSE = V − R I
b a a
(v) Current in the secondary coil Tand E ∝ ω
b
9000 + 200 9200 where Eb is the back-emf
= A= A = 46 A
200 200 Ra is the armature coil resistance.
274. P = τω or EI = τω ω is the speed of rotation of the motor.
EI 110 × 90 Since the load is increased, more power is drawn from
or τ= = Nm the supply, therefore Ia increases. From the first equa-
ω 2π × 24.5
tion, we see that the back-emf decreases and hence
Output power 40 the speed of rotation also decreases.
275. =
200 × 10 100 282. Current through the cables,
or Output power = 800 W
Power 100
Loss = (200 × 10 – 800) W I= = A = 10 A
Potential difference 10
= (2000 – 800) W = 1200 W
Power dissipated in cables
Now, 1200 = 10 × 10 R
= I2R = (10)2 5 = 500 W.
or R = 12 Ω
ns 5
276. The equivalent primary load is 283. Es = × E p = × 200 V = 1000 V.
np 1
FN I 2
FG 20 IJ 2
R1 = GH N JK
1
2
R2 =
H1K (6.0) = 2400 Ω
284. Ip =
EsIs 1000 × 2
= A = 10 A.
Ep 200
285. Assume the transformer to be tested is an ideal one,
then
Vp Np Is
= = .
Vs Ns Ip
Given Vp = 240 V Ns = 50 turns,
Is = 50 mA. Ip = 2.0 mA.
240 N p 50
⇒ = = = 25.
Vs 50 2
Fig. 129 ∴ Vs = 9.6 V and Np = 1250 turns.
Current in the primary coil 104
N2 1 286. 25Es = 104 or Es = volt
240 240 = 25
= = = 0.1 A. N1 20
R1 2400 Ep np 8
or N1 : N2 = 20 : 1 Now, = =
Es ns 1
277. e = 200 – 5 × 4
104
= (220 – 20) V = 200 V. or Ep = 8 × volt.
25
278. P = 220 V × 5A = 1100 W
ELECTROMAGNETIC INDUCTION AND ALTERNATING CURRENT 403

KNOWLEDGE PLUS
l Two coils have self-inductances L1 = 4 mH and L2 = 1 mH respectively. The currents in the two coils are increased at
the same rate. At a certain instant of time, both coils are given the same power. If I1 and I2 are the currents in the two
coils, at that instant of time respectively, then the value of (I1/I2) is
(a) 1/8 (b) 1/4 (c) 1/2 (d) 1. [EAMCET 2003]
dI
Sol. P = EI or P = L I
dt
dI
Both P and are constants
dt
∴ LI = constant
I1 L 2 1
∴ = =
I 2 L1 4
So, (b) is the right choice.

l The self inductance of the motor of an electric fan is 10 H. In order to impart maximum power at 50 Hz, it should be
connected to a capacitance of
(a) 2 µ F (b) 1 µ F (c) 8 µ F (d) 4 µ F. [AIEEE 2005]
1
Sol. ν=
2π LC
1 1
50 = or 2500 =
2π 10C 40 π 2C
1
or C= F = 10 –6 F = 1 µF
2500 × 40 × 10
So, (b) is the right choice.

l A coil of inductance 300 mH and resistance 2 Ω is connected to a source of voltage 2 V. The current reaches half of its
steady state value in
(a) 0.3 s (b) 0.15 s (c) 0.1 s (d) 0.05 s. [AIEEE 2005]
I0
Sol. L = 300 × 10 –3 H ; R = 2 Ω ; I =
2
−R
t
Using I = I0 (1 – e L )
−R −R
I0 t 1 L
t
= I0 (1 − e L ) or =1– e
2 2
−R −R
L
t 1 L
t −R
or e = or e =2 or t = loge 2 = 0.693
2 L

0.693 × 300 × 10−3


or t= s = 0.1 s
2
So, (c) is the right choice.
404 COMPREHENSIVE OBJECTIVE PHYSICS

SET III MCQs


with
More than one correct alternative

Average time allowed per question is 50 seconds.

287. An AC source rated 100 V (rms) supplies a current of 290. A series LCR circuit is operated at resonance. Which of
10 A (rms) to a circuit. The average power delivered by the following statements is correct ?
the source (a) Impedance is minimum.
(a) must be 1000 W (b) may be 1000 W
(b) Voltage across R is minimum.
(c) may be greater than 1000 W
(c) Power transferred is maximum.
(d) may be less than 1000 W.
(d) Current amplitude is minimum.
288. In the AC circuit shown in Fig. 130, the supply voltage
291. A wire lmn moves
has a constant rms value V but variable frequency f.
along the bisector
At resonance, the circuit
of angle θ with
constant velocity
v in a uniform
magnetic field B
perpendicular to
the plane of the
paper and di- Fig. 131
rected down-
wards. Which of the following is correct ?
θ
(a) The effective length between l and n is 2L sin .
2
Fig. 130 θ
(b) The emf between a and b is 2B Lv sin .
2
(a) has a resonant frequency 500 Hz.
(c) The shape is immaterial. End points are important.
V (d) Data is not adequate.
(b) has a current I = .
R
292. In Fig. 132, a charged capacitor discharges when the
(c) has a voltage across the capacitor which is 180° out of switch is closed. In Fig. 133, an uncharged capacitor is
phase with that across the inductor.
V
(d) has a current I = .

R2 +
1 1 FG IJ 2
+
π π H K
289. A resistance of 1000 Ω, capacitance of 1.0 µF and an
inductance of 2 H are connected in series with a source
of 200 sin 1000 t volt. Mark the correct statement(s)
(a) Current = 0.1 A (b) Impedance = 1414 Ω
(c) Power factor = 0.707
(d) The voltage leads the current by 45°. Fig. 132

4
ELECTROMAGNETIC INDUCTION AND ALTERNATING CURRENT 405

charged when the switch is closed. When the switches 1


are closed, one of the five meters gives a reading θ which 293. An alternating emf of angular frequency ω = is
LC
varies with time, as shown below (Fig.134). applied to a series LCR circuit. For this frequency of
applied emf
(a) the circuit is at resonance and its impedance is made
up only of a reactive part
(b) the current in the circuit is in phase with the applied
emf
(c) the sum of the p.d. across the inductance and capaci-
tance equal the applied emf which is 180° ahead phase
of the current in the circuit.
ωL 1
(d) The quality factor of the circuit is or and
R ω CR
Fig. 133 this is a measure of the voltage magnification of the
circuit.
294. An inductor-coil having some resistance is connected to
an AC source. Which of the following quantities have
zero average value over a cycle ?
(a) current (b) induced emf in the inductor
(c) Joule heat
(d) magnetic energy stored in the inductor.
295. In an AC circuit, the power factor
(a) is unity when the circuit contains an ideal resistance only
Fig. 134 (b) is unity when the circuit contains an ideal inductance
only
Which meter records in this way ? (c) is zero when the circuit contains an ideal resistance only
(a) A1 (b) A2 (d) is zero when the circuit contains an ideal inductance only.
(c) V1 (d) V2 296. A resistance of 30 Ω, a capacitor with XC = 20 Ω and an
(e) V3. inductor with XL = 60 Ω, are connected in series to a
100 V, 50 Hz power source. Then
(a) current = 2.0 A (b) current = 3.33 A
(c) power factor = 0 (d) power factor = 0.6.

Answers (Set III)


287. (b), (d) 288. (a), (b), (c) 289. (a), (b), (c), (d) 290. (a), (c) 291. (a), (b), (c) 292. (a), (e) 293. (b), (d)
294. (a), (b) 295. (a), (d) 296. (a), (d).

Solutions (Set III)


287. If power factor = 1, then 1000
P = Ev Iv = 100 × 10 W = 1000 W. ν= Hz = 500 Hz.
2
1 (b) At resonance, the circuit is a purely resistive
288. (a) ν = Hz circuit.
1 1
2π × × 10−6
π π
406 COMPREHENSIVE OBJECTIVE PHYSICS

1 1 θ
289. XC = = Ω So, effective length is 2 L sin.
Cω 10−6 × 1000 2
= 103 Ω = 1000 Ω 292. The graph of θ versus time is the graph for the charg-
ing of a uncharged capacitor C through a resistor R in
XL = Lω = 2 × 1000 Ω
series.
= 2000 Ω
For the given circuit arrangements, only meter V3
Now,
Fig. 135 records the charging of the capacitor C.
Z= 10002 + ( 2000 − 1000)2 Ω 293. (a) Impedance is purely resistive.
(b) No phase difference between current and emf.
= 2 × 1000 Ω = 1.414 × 1000 Ω = 1414 Ω
(c) The sum of the voltages across L and C is zero.
200 (d) Q-factor is a measure of the voltage amplification.
I= A = 0.1 A
2 × 2 × 1000 294. Think in terms of sine curves of current and emf.
R 1000 1 R
Power factor, cos φ = = = = 0.707 295. cos φ =
Z 2 × 1000 2 Z
Also φ = 45°. R
When circuit is purely resistive, cos φ = =1
290. At resonance, impedance is minimum, current is maxi- R
mum, voltage across R is maximum and power trans- 0
ferred is maximum. When circuit is purely inductive, cos φ = = 0.
XL
θ y 100
291. sin = 296. I= A
2 L
30 + ( 60 − 20)2
2
θ
or y = L sin 100 100
2 or I= A= A=2A
900 + 1600 50

Fig. 136 R 30
Power factor, cos φ = = = 0.6.
Z 50

KNOWLEDGE PLUS
l A charged particle moves along the line AB which lies in the same plane of a circular loop
of conducting wire as shown in Fig. 137. Then,
(a) no current will be induced in the loop.
(b) the current induced in the loop will change its direction as the charged particle passes
by.
(c) the current induced will be anticlockwise.
A B
(d) the current induced will be clockwise. [Karnataka CET 2003] Fig. 137
Sol. Charge flowing along AB constitutes a current. According to Right hand thumb rule,
magnetic field passing through the loop, due to current I, will be perpendicular to the
plane of the page pointing upwards. The direction of current in the loop will be such as to
oppose the increase of this field (Lenz’s law).
So, (d) is the right choice.

A I I B
Fig. 138
ELECTROMAGNETIC INDUCTION AND ALTERNATING CURRENT 407

MCQs
based on
SET IV

TYPICAL NUMERICAL BANK


(Exclusively for Engineering Entrance Tests)

Average time allowed per question is 60 seconds.

297. A circular loop of radius R, carrying current I, lies in B 2l2v2 Blv


xy plane with its centre at origin. The total magnetic (a) (b)
R R
flux through xy plane is
(a) direclty proportional to I B 2lv2 B 2l2v
(c) (d) .
(b) directly proportional to R R R
300. A cathode-ray 1 cm
(c) inversely proportional to R
oscilloscope is fitted
(d) zero. [IIT 1999] with a graticule 1 cm
298. A thin semicircular con- ruled in cm. The Y-
× × × × × ×
ducting ring of radius R sensitivity is set at
is falling with its plane
B 10 V cm –1 and the
× × × × × ×
vertical in a horizontal time-base at 0.5 ms
cm –4. When an
magnetic induction B × × ×N× × ×
alternating emf is
(Fig. 139). At the posi-
× × × × × × applied to the Y-
tion MNQ, the speed of input the wave trace
ring is V and potential × × ×V × × × obtained is as Fig. 141
difference developed M Q shown in the diagram below (Fig. 141).
across the ring is Fig. 139 Which one of the following correctly gives both the peak
(a) zero emf and the frequency of the emf ?
(b) BV πr2/2 and M is at higher potential peak emf/V frequency/Hz
(c) πRBV and Q is at higher potential (a) 15 5000
(d) 2RBV and Q is at higher potential. [IIT 1996] (b) 15 2000
299. Two parallel wires AL (c) 30 5000
and KM placed at a (d) 30 2000
distance l are connected (e) 30 500.
by a resistance R and 301. A length of wire carries a steady current. First it is
placed in a magnetic bent to form a circular coil of one turn. Then it is bent
field B which is per- more sharply to form a loop of two turns but of smaller
pendicular to the plane radius. Now the magnetic flux density at the centre
containing the wire as caused by same current is found to be such that
shown in Fig. 140. 1
(a) magnetic flux density of double loop = that of single
Another wire CD now Fig. 140 4
turn coil
connects two wires
(b) it is four times that of single turn coil
perpendicularly and made to slide with velocity v. The (c) it is same and remains unaltered
work done needed to slide the wire CD will be (d) it is one-half of the first value.
408 COMPREHENSIVE OBJECTIVE PHYSICS

302. The magnetic flux through a stationary loop with


resistance R varies during interval of time T as
φ = at (T – t). The heat generated during this time
neglecting the inductance of loop will be
a 2T3 a 2T2
(a) (b)
3R 3R
a2T a 3T2
(c) (d) .
3R 3R
303. A plane spiral with a large number N of turns wound
tightly to one another is located in a uniform magnetic Fig. 142
field perpendicular to the plane of the spiral. The out-
(a) 4π Na2 B0ω (b) 7π Na B02 ω
side radius of the spiral’s turns is equal to a. The mag-
netic induction varies with time as B = B0 sin ωt, where 1 1
(c) π Na2 B0 ω (d) µ π N2 a B0ω.
B0 and ω are constant. The amplitude of induced emf 3 3 0
in the spiral is

Answers (Set IV)


297. (d) 298. (d) 299. (a) 300. (b) 301. (c) 302. (a) 303. (c).

Solutions (Set IV)


297. Magnetic lines of force form closed loop. So, magnetic d
lines of force moving to ‘– Z’ direction shall eventually 302. |E| = [atT – at2] = aT – 2at

z
dt
move to ‘ + Z’ direction. So, the net magnetic flux E2 a2 T
through the xy plane will be zero. Now dQ = dt ⇒ Q = (T − 2t)2 dt

zL
R R 0
298. Shape immaterial. End points M and Q are important.
a2 T 2
Effective length = 2R. Induced emf = Blv = B(2R)v. or Q= (T + 4t 2 − 4 Tt) dt

MN z z z
Using Lenz’s law, we find that current flows clockwise. R 0
So, it would flow from Q to M. a2 T
T2 dt + 4
T
t2 dt − 4 T
T
tdt
OP
Clearly, Q is at a higher potential.
or Q=
R 0 0 0 Q
F EI E
Work done/time = EI = E G J =
2
B2l2v2 a2 L
MM
4T 3 F T I OP
− 4T G
2
H RK R H 2 JK PQ
299. = . 3
or Q= R T + 3
R
N
300. The peak of the emf reads 1.5 cm. Since the Y-sensi-
tivity is set to 10 V cm–1, this correspond to a peak
LM
a2 4 T 3
− T3
OP
a2 T 3 a2 T 3
value of (1.5) (10) or 15 V. Similarly, the period of the
or Q =
R 3 MN R 3 PQ or Q =
or Q =
3R
.
wave reads 1 cm or corresponds to 0.5 ms since the 303. Let us consider dN number of turns of radius r and
time-base is set at 0.5 ms cm–1. Thus, the frequency of thickness dr. If dE is the corresponding induced emf,
1 then
the wave is or 2000 Hz.
0.5 ms d d
|dE| = dN (πr2B) or |dE| = πr2dN (B sin ωt)
µ NI µ NI dt dt 0
301. φ = BA or φ = 0 × πr2 or φ = 0 πr
2r 2 or |dE| = πr2ω B0 dN cos ωt
µ NI µ I N
or φ= 0 (2πr) or φ = 0 [N × 2πr] or |dE| = πr2ω B0 cos ωt dr
4 4

z
a
µ 0Il
or φ= N a
4 |E| = πω B0 cos ωt r 2 dr
Neither I nor l changes. So, φ is unchanged. a 0
N a3 1
= πω B0 cos ωt = π Na2 B0 ω.
a 3 3
ELECTROMAGNETIC INDUCTION AND ALTERNATING CURRENT 409

SELF-EVALUATION TEST I
Based on UNIT XIV

[Expected Questions for Forthcoming Examinations]

1. Refer to circuit of Fig. 154. E = 10 V, R1 = 2 Ω, R2 = 6. Which voltmeter will give zero reading at resonance ?
3 Ω, R3 = 6 Ω and L = 5H. The current I1 just after
L C
pressing the switch S is R

10 10
(a) A (b) A
4 5 V1 V2
10 10 V3
(c) A (d) A.
12 6
2. In Q. 1, the current I1 long after pressing the switch S
is
Fig. 144
(a) V1 (b) V2
(c) V3 (d) None.
7. For a fixed inductance, increasing the capacitance of a
tuning radio set, a listener will be tuning to stations of
(a) higher f (b) lower λ
(c) lower f (d) None of these.
8. An emf will be induced between the ends of the conduc-
tor shown in Fig. 145 when conductor moves in the
direction.
Fig. 143
M
10 10
(a) A (b) A L
4 5
10 10 Q
(c) A (d) A. N S
12 6
3. In Q. 1, the current I2 long after pressing the switch S
P
is
10 10 Fig. 145
(a) A (b) A
4 5 (a) P (b) Q
10 10 (c) L (d) M. [AIIMS 1992]
(c) A (d) A.
12 6
9. The dimensions of magnetic flux are
4. In Q. 1, the current I1 just after releasing the switch S
(a) MLT –2 A–2 (b) ML2T –2 A–2
is
(c) ML2T –1 A–2 (d) ML2T –2 A–1.
10 10
(a) A (b) A [MP PMT 1994]
4 5
10 10. The equivalent inductance of two inductances is
(c) 0 A (d) A. 2.4 henry when connected in parallel and 10 henry
6
when connected in series. The difference between the
5. In Q. 1, the current I2 just after releasing the switchs is
two inductances is
10 10 (a) 2 henry (b) 3 henry
(a) A (b) A
4 5 (c) 4 henry (d) 5 henry.
10
(c) A (d) zero. [MP PMT 2000]
12
410 COMPREHENSIVE OBJECTIVE PHYSICS

11. The pointer of a dead-beat galvanometer gives a steady The magnetic field is perpendicular to the plane of the
deflection because coil. The emf generated in it is
(a) eddy currents are produced in the conducting frame over (a) 104 V (b) 1.2 V
which the coil is wound.
(c) 1.0 V (d) 10–2 V.
(b) its magnet is very strong.
[MP PMT 1994]
(c) its pointer is very light.
20. A coil of resistance R and inductance L is connected to
(d) its frame is made of ebonite. [MP PMT 1994]
a battery of E volt emf. The final current in the coil is
12. The time required for a current to attain the given value
E E
in a DC circuit containing L and R, depends upon (a) (b)
R L
(a) R only (b) L only
E EL
L (c) (d) .
(c) R 2 + L2 R 2 + L2
R
(d) the applied potential difference. 21. Two identical circular loops of metal wire are lying on
a table without touching each other. Loop A carries a
13. Two circuits have coefficient of mutual induction of
current which increases with time. In response, the loop-
0.09 henry. Average emf induced in the secondary by a
B,
change of current from 0 to 20 ampere in 0.006 second
(a) remains stationary (b) is attracted by the loop A
in the primary will be
(a) 120 V (b) 180 V (c) is repelled by the loop A

(c) 200 V (d) 300 V. [MP PET 1992] (d) rotates about its CM, with CM fixed. [IIT 1999]
14. An emf of 12 volt is induced in a given coil when the 22. The variation with time of the current through, and of
current in it changes at the rate of 48 ampere per the potential difference across, a resistor are shown
minute. The self inductance of the coil is below.
(a) 0.25 henry (b) 15 henry
(c) 1.5 henry (d) 9.6 henry.
[MP PMT 2000]
15. An inductance of negligible resistance whose reactance
is 22 Ω at 200 Hz is connected to 200 V, 50 Hz power
line. The value of inductance is
(a) 17.5 H (b) 1.75 H
(c) 0.175 H (d) 0.0175 H.
16. A circuit has an inductance of 10 mH and a resistance
of 10 Ω. It is connected to a DC source of emf 100 V. The Fig. 146
value of steady state current is Which graph best represents the variation with time of
(a) 0 (b) 0.1 A the power dissipated in the resistor ?
(c) 10 A (d) 104 A.
17. Which of the following is not an application of eddy
currents ?
(a) Induction furnace (b) Galvanometer damping
(c) Speedometer of automobiles
(d) X-ray crystallography.
18. In a coil of area of 10 cm2 and 10 turns, the magnetic
field is directed perpendicular to the plane and is chang-
ing at the rate of 108 gauss s–1. The resistance of the
coil is 20 Ω. The current in the coil will be
(a) 5 A (b) 0.5 A
(c) 0.05 A (d) 5 × 108 A.
19. The magnetic field in a coil of 100 turns and 40 cm2
area is increased from 1 tesla to 6 tesla in 2 second.
Fig. 147
ELECTROMAGNETIC INDUCTION AND ALTERNATING CURRENT 411

23. In an AC circuit, the current lags behind the voltage by 32. AC can be measured with the help of
π/3. The components in the circuit are (a) hot wire ammeter (b) hot wire voltmeter
(a) R and L (b) R and C (c) moving coil galvanometer
(c) L and C (d) Only R. (d) tangent galvanometer.
[MP PMT 2000] 33. A hot wire ammeter reads 10 A in an AC circuit. The
24. The voltage of an AC source varies with time accord- peak value of current is
ing to the equation (a) 10 2 A (b) 5 πA
V = 100 sin 100 πt cos 100 πt 10
2
where t is in second and V is in volt. The (c) A (d) A. [KCET 2000]
π 2
(a) the peak voltage of the source is 100 volt. 34. The magnetic flux linked with the coil varies with
(b) the peak voltage of the source is 50 volt. time as φ = 3t2 + 4t + 9. The magnitude of induced emf
(c) the peak voltage of the source is 100/ 2 volt. at t = 2 second is
(a) 9 V (b) 16 V
(d) the frequency of the source is 50 hertz.
[MP PMT 2000] (c) 3 V (d) 4 V. [KCET 2000]
25. The voltage measured across the AC main terminals is 35. An inductor, a resistor and a capacitor are joined in
210 V. Then the peak to peak variation of voltage be- series with an AC source. As the frequency of the source
tween the terminals will be is slightly increased from a very low value, the reac-
tance
(a) 210 2 V (b) 420 V (a) of the inductor increases (b) of the resistor increases
(c) 420 2 V (d) 420 / 2 V. (c) of the capacitor increases (d) of the circuit increases.
26. Current in a circuit is wattless if 36. Two coils A and B having turns 300 and 600 respec-
(a) current is alternating tively are placed near each other. On passing a current
(b) inductance in the circuit is zero of 3.0 A in A, the flux linked with A is 1.2 × 10 –4 Wb
(c) resistance in the circuit is zero. and with B it is 9.0 × 10 –5 Wb. The mutual inductance
(d) resistance and inductance both are zero. of the system is
27. Energy required to establish a current of 4 A in a self- (a) 2 × 10 –5 H (b) 3 × 10 –5 H
inductance L = 200 mH is (c) 4 × 10 –5
H (d) 6 × 10 –5 H.
(a) 0.16 J (b) 0.40 J
[MP PMT 2001]
(c) 0.80 J (d) 1.6 J.
37. Primary coil of a transformer (turns ratio 1 : 1) is con-
28. When 100 volt DC is applied across a solenoid, a cur- nected to AC source E = E0 sin ωt. Then the form of
rent of 1 A flows in it. When 100 volt AC is applied secondary voltage during one complete cycle will be
across the same coil, the current drops to 0.5 A. If the
frequency of AC source is 50 Hz, the impedance and
the inductance of solenoid are
(a) 100 Ω and 0.93 H (b) 100 Ω and 0.86 H
(c) 200 Ω and 0.55 H (d) 200 Ω and 1.0 H.
29. If the instantaneous current in an AC circuit is given
by i = 2 cos (ωt + φ) ampere, then the rms value of the
alternating current is
(a) 0 A (b) 2A

(c) 2 A (d) 2 2 A.
30. A series LCR AC circuit contains L = 8.0 H, C = 0.5 µF,
and R = 100 Ω. Then the resonant frequency will be
(a) 5 rad s–1 (b) 50 rad s–1
(c) 500 rad s –1
(d) 1500 rad s–1. Fig. 148
31. The frequency of AC is 50 Hz. How many times current
(a) A (b) B
becomes zero in one second ?
(c) C (d) D.
(a) 25 times (b) 50 times
(c) 100 times (d) 200 times. [National Standard Exam. in Physics 1998]
412 COMPREHENSIVE OBJECTIVE PHYSICS

38. A 200 volt DC motor draws current 5 A at start, but 41. In series LCR AC circuit, the capacitance is changed
only 3 A while it is running. The back emf developed is from C to 4C. For the same resonant frequency, the
(a) 200 × 3/5 volt (b) 200 × 5/3 volt inductance should be changed from L to
(a) L/4 (b) L/2
(c) 200 × 1/3 volt (d) 200 × 2/5 volt.
(c) 2L (d) 4L.
[National Standard Exam. in Physics 1992]
39. A solenoid of length l metre has self inductance L henry. 42. In series LCR circuit, the voltages across R, L and C
are shown in the circuit. The voltage of applied source
If number of turns is doubled, its self inductance
is
(a) remains same (b) becomes 2L henry
L
(c) becomes 4L henry (d) becomes henry.
2
[MP PMT 2001]
40. Fig. 149 shows current versus time t in a primary cir-
cuit, and Fig. 150 shows conceivable emf ε induced in
the secondary for the changes in Fig. 149. The correct
one among them is

Fig. 151
(a) 10 V (b) 50 V
(c) 70 V (d) 110 V.
43. The potential difference V and current i flowing through
an instrument in an AC circuit are given by V = 5 cos
ωt volt and i = 2 sin ωt ampere respectively. Then the
power dissipated in the instrument is
(a) 0 W (b) 2.5 W
(c) 5 W (d) 10 W.
44. A train is moving towards north with a speed of 180
Fig. 149 kilometre per hour. If the vertical component of the
earth’s magnetic field is 0.2 × 10 –4 T, the emf induced
in the axle 1.5 m long is
(a) 15 mV (b) 1.5 mV
(c) 5.4 mV (d) 54 mV. [KCET 2001]
45. In the circuit shown in Fig. 152, neglecting source
resistance, the voltmeter and ammeter readings will
respectively be

R = 30 W XL = 25 W XC = 25 W

A
Fig. 150 240 V, 50 Hz
(a) p (b) q
Fig. 152
(c) r (d) s.
(a) 150 V, 3 A (b) 0 V, 3 A
[National Standard Exam. in Physics 1995]
(c) 0 V, 8 A (d) 150 V, 8 A. [KCET 2001]
ELECTROMAGNETIC INDUCTION AND ALTERNATING CURRENT 413

46. A coil of inductance 8.4 mH and resistance 6 Ω is


(a) V2/R (b) V2/ ( R 2 + ω 2L2 )
connected to a 12 V battery. The current in the coil is
1.0 A at approximately the time (c) V2R/(R2 + ω2L2) (d) zero.
(a) 500 s (b) 20 s
[National Standard Exam. in Physics 2001]
49. A coil has ohmic resistance r, inductance L and
(c) 35 ms (d) 1 ms.
reactance XL = 2πfL. When an alternating current of
[IIT Screening 1999] rms value I flows in it, the power consumed will be
(a) I2 r. (b) I2(r + XL)
47. The voltage of AC main is represented by V = 200 2
sin 100 πt volt where t is in second. The frequency of AC (c) I2XL. (d) I2 r 2 + X L 2 .
is [National Standard Exam. in Physics 2000]
(a) 50 Hz (b) 50 π Hz 50. In a series LCR circuit the voltage across R, C and L is
(c) 100 Hz (d) 100 π Hz. 10 volt each. The value of the applied emf is
(a) 30 V (b) 20 V
48. A coil of resistance R and inductance L is connected
across an ac power supply of rms voltage V. The power (c) 300 V (d) 10 V.
dissipated in the coil is [National Standard Exam. in Physics 1997]

Answers
1. (b) 2. (a) 3. (c) 4. (c) 5. (c) 6. (b) 7. (c) 8. (d)
9. (d) 10. (a) 11. (a) 12. (c) 13. (d) 14. (b) 15. (d) 16. (c)
17. (d) 18. (a) 19. (c) 20. (a) 21. (c) 22. (d) 23. (a) 24. (b)
25. (c) 26. (c) 27. (d) 28. (c) 29. (b) 30. (c) 31. (c) 32. (a)
33. (a) 34. (b) 35. (a) 36. (b) 37. (c) 38. (d) 39. (c) 40. (d)
41. (a) 42. (b) 43. (a) 44. (b) 45. (c) 46. (d) 47. (a) 48. (c)
49. (a) 50. (d).

Solutions
1. R3 and L are not to be considered. I2 1 1 I2 1
= = or =
10 10 I 2 + I3 2 + 1 3 I1 3
Now, I= A= A.
2+3 5 1 1 10 10
or I1 = ×
I2 = A= A.
2. L is not to be considered. This is because the current 3 3 4 12
has become steady. 4 & 5. Resistance ‘reacts’ immediately. Inductance ‘reacts’
3×6 18 slowly.
R23 = Ω= Ω=2Ω 6. At resonance, the series combination of L and C gives
3+6 9
zero impedance.
R123 = (2 + 2) Ω = 4 Ω
10
I1 = A.
+ At resonance, the voltages across L and C
4 are equal but opposite in phase.
3. We have to distribute I1 between 3 Ω and 6 Ω
I2 3 1 1
= = 7. f=
I3 6 2 2π LC
When C increases, f decreases.
414 COMPREHENSIVE OBJECTIVE PHYSICS

8. Along P, v || l ; Along L and Q, v || B.


Along M, the three quantities B, l and v are mutually
perpendicular.
9. φ = BA
Fm Fm
But Fm = Bqv or B = ∴ φ= A
qv qv
[ML T−2 ][L2 ]
[φ] = = [ML2T –2A–1]. Fig. 154
[AT][LT −1 ]
L1L2 23. In LR circuit, the current lags emf.
10. = 2.4 100
L1 + L2 24. V= [2 sin 100 πt cos 100 πt]
L1 + L2 = 10 ∴ L1L2 = 24 2
Now, (L1 – L2)2 = (L1 + L2)2 – 4L1L2 V = 50 sin 200 πt
= 100 – 4 × 24 = 100 – 96 = 4 Clearly, the peak voltage of the source is 50 volt.
∴ L1 – L2 = 2. 25. E0 = 210 2 V
dI 20 2E0 = 420 2 V
13. | E | = M = 0.09 × volt = 300 volt.
dt 0.006
48 60
14. 12 = L × or L = H = 15 H.
60 4
22
15. 22 = L × 2 × × 200
7
7
L= H = 0.0175 H.
400 Fig. 155
E 100 26. When resistance is zero, power consumed is zero.
16. I0 = = A = 10 A
R 10
1 2 1
27. LI = × 200 × 10 –3 × 4 × 4 J = 1.6 J.
2 2
+ In the case of steady state current, L has
100
no role to play. 28. R= = 100 Ω
1
100
dB ZL = = 200 Ω
18. I = NA R = 10 (10 × 10 –4) (104)/20 = 5 A. 0.5
dt Now, 2002 = 1002 + L2 × 4 × 9.87 × 2500
dB 5
19. | E | = NA = 100 × 40 × 10 –4 × V = 1 V. 300 × 100
dt 2 or L2 = = 0.304 or L = 0.55 H.
4 × 9.87 × 2500
20. L has no role to play in final current. This alone de-
29. i0 = 2
cides (a).
21. Using Lenz’s law, we find 2
iv = = 2 A.
that if the ‘increasing cur- 2
rent in A is clockwise, then 1 1
1
the induced current in B 30. ω = = = rad s–1
will be anti clock-wise. LC 8 × 0.5 × 10 −6 2 × 10−3
Fig. 153
Clearly, B is repelled. 1000
[Note that there is repulsion between unlike currents.] = rad s–1 = 500 rad s–1.
2
22. If I is alternating current in the resistor and R is its 31. In one cycle, current becomes zero twice.
resistance, power dissipated is given by
32. The measurement of alternating current is based on
the heating effect of current.
ELECTROMAGNETIC INDUCTION AND ALTERNATING CURRENT 415

33. I0 = 2 Iv 43. Phase difference is 90°.


34. φ = 3t2 + 4t + 9 5
44. E = Bvlv = 0.2 × 10 –4 × 1.5 × 180 × V
dφ 18
|E|= = 6t + 4
dt = 15 × 10 –4 V = 1.5 × 10 –3 V = 1.5 mV.
At t = 2 s, 45. Since XL = XC therefore the voltmeter reading is zero.
| E | = 6 × 2 + 4 = 16 volt. Also, the circuit is a resonant circuit. It behaves as a
35. XL = 2πf L or XL ∝ f. purely resistive circuit.
φB 9 × 10−5 Ev 240 V
∴ Iv = = = 8 A.
36. M = = H = 3 × 10 –5 H. 30 30 Ω
IA 3
37. According to Faraday’s law, induced emf is propor- F R I
tional to the rate of change of flux (slope of E vs t graph 46. GG
I = I0 1 − e

L
t
JJ
of primary) and opposite to the charge. H K
12 F I
38. Resistance being the same, the current fall is due to R R
2 1= G1 − e
6 GH

L
t
JJ or e

L
t
=
1
back emf. So, back emf is 200 ×
2
5
volt.
K 2
39. L∝N. R
t R
40. E is proportional to slope of I versus time curve. The e L = 2 or t = loge 2 = 0.693
slopes have ratios 1 : 2 : 0 : 1. L
Only graph has them. 0.693 × 8.4 × 10−3
or t= s
1 6
41. νr = = 9.7 × 10 –4 s = 0.97 × 10 –3 s ≈ 1 ms.
2π LC
47. ω = 2πf ⇒ 100 π = 2πf ⇒ f = 50 Hz.
In the given problem, LC = constant.
If C is increased to 4C, then L has to be changed to
FG V IJ 2
V 2R
L
4
.
48. P = I2R =
H ZK R=
R 2 + ω 2L2
.

42. V2 = VR2 + (VL – VC)2 49. Power is dissipated only in the ohmic resistance.
or V2 = 402 + (50 – 20)2 50. VC = VL suggests resonance. Purely resistive circuit.
or V2 = 1600 + 900 = 2500 Applied emf = VR = 10 volt.
or V = 50 volt.

KNOWLEDGE PLUS
l The time taken by AC of 50 Hz in reaching from zero to the maximum value is
(a) 50 × 10 –3 s (b) 5 × 10 –3 s (c) 1 × 10 –2 s (d) 2 × 10 –2 s [Karnataka CET 2003]
1 1 1
Sol. t = T= = s = 5 × 10–3 s
4 4 ν 200
So, (b) is the right choice.
416 COMPREHENSIVE OBJECTIVE PHYSICS

SELF-EVALUATION TEST II
Based on UNIT XIV

DIRECTIONS :
(i) MCQs 1 to 24 have one correct alternative.
(ii) MCQs 25 to 30 have more than one correct alternative.
(iii) MCQs 31 to 35 have one or more than one correct alternative.

1. The input to the CR circuit shown is a square-wave 2. In the diagram


potential difference as shown in Fig. 156. The varia- (Fig. 158, the so-
tion of potential difference V across the resistor with lenoid, of length l
time t is best represented by closely uniformly
wound, carries an
alternating cur-
rent of constant
amplitude. A
search coil is Fig. 158
placed in different
positions along the solenoid.
Fig. 156
Which one of the following graphs most nearly shows
how the amplitude of the emf E induced in the search
coil varies with its position ?

E E

O l X O l X
(a ) (b)
E E

O l X O l X
(c) (d )

O l X
(e)

Fig. 159
Fig. 157

[Oxford and Cambridge Schools


Examination Board]
ELECTROMAGNETIC INDUCTION AND ALTERNATING CURRENT 417

3. A capacitor discharges through a resistance wire with 6. A uniformly wound solenoidal coil of self inductance
time constant τ1. When the wire is replaced by another 1.8 × 10 –4 henry and resistance 6 ohm is broken up
wire of the same material and length but double the into two identical coils. These identical coils are then
diameter, the time constant is τ2. What is the ratio connected in parallel across 12 V battery of negligible
τ1 : τ2 ? resistance. The steady state current through the bat-
(a) 4 : 1 (b) 2 : 1 tery is
(c) 1 : 1 (d) 1 : 2 (a) 8 A (b) 4 A
(e) 1 : 4. (c) 6 A (d) 2 A. [IIT 1989]
4. A capacitor of capacitance 1.0 µF carries a charge of 7. A coil of wire having finite inductance and resistance
15 mC. It is discharged through a 25 Ω resistor. What has a conducting ring placed coaxially within it. The
is the maximum current during the discharge ? coil is connected to a battery at time t= 0, so that a time-
(a) 0.60 A (b) 1.2 A dependent current I1 (t) starts flowing through the coil.
(c) 300 A (d) 600 A If I2 (t) is the current induced in the ring, and B (t) is
the magnetic field at the axis of the coil due to I1 (t),
(e) 1200 A.
then as a function of time (t > 0), the product I2 (t) B (t)
5. In the circuit shown in Fig. 160, the potential differ- (a) increases with time (b) decreases with time
ence V across the capacitor C is measured using a volt-
(c) does not vary with time (d) passes through a maximum.
meter of infinite resistance. HK and LM are parallel
fine wires. [IIT Screening 2000]
8. A uniform but time -
varying magnetic field
B(t) exists in a circu-
lar region of radius a
and is directed into the
plane of the paper as
shown in Fig. 162. The
magnitude of the in-
duced electric field at
point P at a distance r
Fig. 160 from the centre of the Fig. 162
A missile is projected in such a way as to break HK circular region
and then LM at times t1 and t2 respectively. Which (a) is zero (b) decreases as 1/r
graph best represents the variation of V with time t ? (c) increases as r (d) decreases as 1/r2.
[IIT Screening 2000]
V V V
9. A rectangular coil of
wire, initially placed
as shown in Fig. 163,
is rotated with
constant angular
O t1 t2 t O t1 t2 t O t1 t2 t velocity in a magnetic
(a ) (b) (c) field which acts in the
direction of XX′. The
V V sinusoidal emf rep-
resented by Fig. 164 Fig. 163
is observed to be
produced across the emf
ends of the coils.
O t1 t2 t O t1 t2 t Which one of the
(d ) (e) following movements
would have given this 0
0 time
Fig. 161 result ?
Fig. 164
418 COMPREHENSIVE OBJECTIVE PHYSICS

(a) rotation of a quarter revolution about axis YY′. (a) 1 µV (b) 2 µV


(b) rotation of a half revolution about axis XX′. 1 1
(c) rotation of a half revolution about axis ZZ′ (c) µV (d) µV.
2 4
(d) rotation of a quarter revolution about axis XX′ 14. A conducting circular loop is placed in a uniform mag-
(e) rotation of a half revolution about axis YY′. netic field B with its plane perpendicular to the field.
10. A semi-circular wire of radius a rotates about an axis The radius of the loop starts shrinking at a constant
OO′ with an angular velocity ω in a uniform magnetic rate
dr
. At an instant, when the radius of the loop is r,
field of induction B. The axis of rotation is perpendicu- dt
lar to the direction of field. The mean amount of ther- the induced emf in the loop is
mal power being generated into the loop during one com- dB
plete rotation is [Given R = resistance] (a) πr2 dt (b) 2πrB

dr
(c) 2πrB (d) zero.
dt
15. Sliding contact in circuit shown in Fig. 167 is pulled to
the right continuously so that the resistance in the cir-
cuit is continuously increasing. Value of resistance at
the instant shown is 4 Ω. Now, which of the following
is correct ?
9 mH
Fig. 165

F
1 πω a2B I2 FG IJ
1 πω B
(a)
2RGH 2 JK (b)
H K
2R 2a2

1 F πω 2 B I 1 F πω B2 I
2R GH 2a JK 2R GH 2a JK
(c) (d) .
2V

11. A constant potential Fig. 167


difference is applied (a) Current in the circuit is 0.5 A.
across a solenoid of (b) Current in the circuit is greater than 0.5 A
inductance L. The (c) Current in the circuit is less than 0.5 A
variation of current (d) There is small current of 9 mA in the circuit.
with time is as shown
16. Large alternating currents in a cable can be measured
in Fig. 166.
by monitoring the emf induced in a small coil situated
Which of the following near the cable. This emf is induced by the varying mag-
Fig. 166
can be deduced from netic field set up around the cable.
the shape of the graph ? In which arrangement of coil and cable will the emf
(a) L decreases as the rate of change of current decreases. induced be a maximum ?
(b) L increases as the rate of change of current decreases.
(c) The current is proportional to the rate of change of the
induced emf.
(d) The induced emf is proportional to the current.
(e) The solenoid has resistance as well as inductance.
12. Which of the following is a unit of magnetic flux ?
(a) m s–1 A (b) kg m2 s–2 A–1
(c) m s–2 A–1 (d) kg ms2 A–1.
13. A wheel with 3 spokes is rotated in a plane normal to
the earth’s magnetic field. An emf of 1 µV is induced Fig. 168
between the axle and the rim of the wheel. What will be
(a) A (b) B
the induced emf if the number of spokes is made 6 ?
(c) C (d) D.
ELECTROMAGNETIC INDUCTION AND ALTERNATING CURRENT 419

17. A nonconducting ring of radius r has charge Q. A mag- Region I


X
Region II
netic field perpendicular to the plane of the ring changes
× × × × ×
at the rate of dB/dt. The torque experienced by the ring ω
A
is × × × × ×
1 2 dB 1 2 dB
(a) r Q (b) r Q × × × × ×
2 dt 3 dt C
r O B
2 dB × × × × ×
(c) r Q (d) zero.
dt D
18. A metal rod moves at a constant velocity in a direction × × × × ×
perpendicular to its length. A constant, uniform mag- Y
netic field exists in a space in a direction perpendicu-
lar to the rod as well as its velocity. Select the correct Fig. 170
statement (s) from the following :
(a) The entire rod is at the same electric potential. Y Y
(b) There is an electric field in the rod.
(c) The electric potential is highest at the centre of the rod
and decreases towards its ends.
(d) The electric potential is lowest at the centre of the rod
e e
and increases towards its ends. [IIT 1998]
19. A coil P is connected to a 50 Hz alternating supply of
constant peak voltage. Coil P lies close to a separate
O t X O t X
coil Q which is connected to the Y-input terminals of
an oscilloscope. A sinusoidal trace appears on the screen (a ) (b)
of the oscilloscope.
Y Y

X X
e t e t

O O
(c) (d )

Fig. 169 Fig. 171

What would be the effect on the trace of linking the coils 21. The bob of a simple pendulum
by a soft-iron core ? is replaced by a magnet. The os-
height of trace number of cycles on screen cillations are set along length of
(a) increases increases the magnet. If a copper coil is
(b) increases stays the same now added so that one pole of
(c) stays the same increases the magnet passes in and out of
(d) stays the same stays the same.
it, and the coil is short-
20. XY is a boundary region separating two regions as circuited, the effect will be
shown in Fig. 170. There is no magnetic field in region (a) increases of amplitude
I but a uniform magnetic field in region II. ACD is a
(b) decreases of period
semi-circular conductor of radius r lying in region I. It
is rotated with a constant angular speed ω about an (c) no effect on period
(d) quicker dying of oscillations. Fig. 172
axis passing through O and perpendicular to the plane
of the page. Which one of the following graphs [Fig. 171] [National Standard Exam. in Physics 1992]
represents correctly the variation of induced emf with
time ?
420 COMPREHENSIVE OBJECTIVE PHYSICS

22. A pair of coils of turns n1 and n2 are kept close together. ing quantity ?
If a current passing through the first is reduced at rate (a) Curve I for R (b) Curve II for current I
r, an emf 2 mV is developed across the other coil. If the (c) Curve III for XL (d) Curve IV for XC.
second coil carries current which is then reduced at
the rate 3r, the emf produced across the first coil will
be
(a) 6n1/n2 mV (b) 6n2/n1 mV2
(c) 6 mV (d) 3/2 mV.
[National Standard Exam. in Physics 1992]
23. Assuming that the magnetic field B due to a current-
carrying solenoid is uniform throughout the volume V
of the solenoid, then the magnetic energy stored in the
solenoid is
Fig. 173
B2 B2
(a) (b) [National Standard Exam. in Physics 2001]
µ0 2µ 0
B2 29. The bulbs B1 and B2 , as shown in Fig. 174, are identi-
(c) V (d) Data is not sufficient. cal.
2µ 0
24. An alternating current is given by
B2
i = i1 cos ωt + i2 sin ωt. L = 10 mH
The rms current is given by
i1 + i2 i1 + i2
(a) (b) B1
2 2
500 µF

i12 + i2 2 i12 + i2 2
(c) (d) .
2 2
25. Which of the following factor(s) is/are responsible for
deciding the mutual inductance of two coils ?
220 V, 50 Hz
(a) The number of turns of each coil.
(b) The shape of each coil. Fig. 174
(c) Current through each coil.
(a) B2 will be brighter than B1
(d) Separation between the coils.
(b) As frequency supply voltage is increased, brightness of
26. To convert mechanical energy into electrical energy, one
B1 will increase and that of B2 will decrease
can use
(c) Their brightness will be the same
(a) DC dynamo (b) AC dynamo
(d) Only B2 will glow because the capacitor has infinite im-
(c) motor (d) transformer.
pedance.
27. Resonance occurs in a series LCR circuit when the fre-
30. In an AC series circuit, the instantaneous current is
quency of the applied emf is 1000 Hz. Then
zero when the instantaneous voltage is maximum. Con-
(a) when f = 900 Hz, the circuit behaves as a capacitive cir-
nected to the source may be a
cuit
(a) pure inductor (b) pure capacitor
(b) the impedance of the circuit is maximum at f = 1000 Hz
(c) pure resistor
(c) at resonance the voltage across L and voltage across C
(d) combination of an inductor and a capacitor.
differ in phase by 180°.
31. The current in a LR circuit builds up to 3/4th of its
(d) if the value of C is doubled, resonance occurs at f = 2000
steady state value in 4 second. The time constant of
Hz. [National Standard Exam. in
this circuit is
Physics 1997]
1 2
28. In a series LCR circuit, different physical quantities (a) s (b) s
ln 2 ln 2
vary with frequency ω. Which of the following curves
represent correct frequency variation of the correspond- 3 4
(c) ln 2 s (d) ln 2 s.
ELECTROMAGNETIC INDUCTION AND ALTERNATING CURRENT 421

32. An inductive reactance, XL = 100 Ω, a capacitive reac- (a) the bulb stops glowing
tance, XC = 100 Ω and a resistance, R = 100 Ω, are (b) momentarily the bulb becomes dim and then stops glow-
connected in series with a source of 100 sin 50 t volt. ing
(c) momentarily the bulb becomes very bright and then stops
Mark the correct statement(s).
glowing
(a) The net impedance of the circuit is 100 Ω
(d) the bulb continues to glow for some time and then stops
(b) The maximum voltage across the series is 100 V glowing. [National Standard Exam. in
(c) The maximum voltage across the capacitor is 100 V Physics 2000]
(d) The maximum voltage across the inductance is 100 V. 35. For the circuit shown in Fig. 176 the ammeter A2 reads
33. A solenoid coil of length l with n turns has self induct- 1.6 A and ammeter A3 reads 0.4 A. Then
ance L. If both n and l are doubled, the self inductance
will be
(a) L (b) 2L
(c) L/2 (d) 4L.
[National Standard Exam. in Physics 2001]
34. For the circuit ar-
rangement shown in
Fig. 175, the circuit
is switched on and
the variable resist-
ance is adjusted
such that the bulb Fig. 176
just glows. Now if 4 2π
(a) ω0 = (b) f2 =
the circuit is LC LC
switched off Fig. 175 (c) the ammeter A1 reads 1.2 A
(d) the ammeter A1 reads 2 A.

Answers
1. (a) 2. (d) 3. (e) 4. (d) 5. (e) 6. (a) 7. (d) 8. (b)
9. (a) 10. (a) 11. (e) 12. (b) 13. (a) 14. (c) 15. (b) 16. (a)
17. (a) 18. (b) 19. (b) 20. (d) 21. (d) 22. (c) 23. (c) 24. (c)
25. (a), (b), (d) 26. (a), (b) 27. (a), (c) 28. (a), (c) 29. (a), (b) 30. (a), (b), (d) 31. (b) 32. (a), (b), (c), (d)
33. (b) 34. (c) 35. (c).

Solutions
1. The potential difference across a resistor is propor- φ = BA
tional to the current. Each time that the input changes
R|(µ nI ) A cos ωt for x near t (centre of the solenoid)
S| µ nI A cos ωt
from low to high, C charges up thus taking a current 0 0 2
= 0 0
that drops to zero as C charges. When the input
changes from high to low, C discharges. So, a discharge T 2 for x = 0 or t (ends of the solenoid)

current flows through R (in the reverse direction to where A is the cross-sectional area of the coil
the charging current). n is the number of turns per meter.
2. The magnetic flux along the axis of the solenoid is given I0 cos ωt is the alternating current with am-
by plitude I0.
422 COMPREHENSIVE OBJECTIVE PHYSICS

Thus, the emf induced in the search coil varies with 6. We are interested in the steady-state current. So, let
its position is given by us focus our attention only on resistance.


R|[ωµ nI A] sin ωt for x near
0 0
t Each ‘half coil’ has a resistance of 3Ω. Combined re-
S|L ωµ nI A O sin ωt for x = 0, t
2
E=– = sistance is 1.5 Ω.

TMN 2 PQ
dt 0 0
12
∴ I0 = A = 8 A.
1.5
The amplitude of the emf induced is the one in brack-
ets [ ] which is most appropriately represented by 7.
LM
I1(t) = I0(t) 1 − e

R
t OP
MN PQ
L
graph d.
Note. The amplitude of the induced emf at the two ends is
half of that at the centre of the solenoid. LM
B(t) = B0 1 − e

R
t OP
MN PQ
L
3. The time constants τ1 and τ2 are given by
RS τ
1 = CR1 R
Tτ2 = CR2 I2(t) =
d
dt
B R − t
[ B(t )] = 0 e L
L
where C is the capacitance of the capacitor.
R1 is the resistance of the wire with time constant τ1. B 2R − LM R
t −
2R
t OP
Now, I2(t) B(t) = 0
MN PQ
e L −e L
R2 is the resistance of the wire with time constant τ2. L
Since resistance R is inversely proportional to the
square of the diameter d (or area) of the wire, we have At t = 0, I2 (t) B (t) = 0

R1 d22 2 FG IJ 2 At t = ∞, I2 (t) B(t) = 0


= 2 =
H K =4 Clearly, the product passes through a maximum.

z
R2 d1 1
→ → dφ dB
Hence, the ratio τ2 : τ1 is given by 8. E . dl = or E × 2πr = A
dt dt
τ 2 R1 1
= = or 1 : 4. dB a 2 dB 1
τ1 R2 4 or E × 2πr = πa2 or E = or E ∝ .
dt 2r dt r
4. The voltage across the capacitor before discharge takes
9. The flux linkage in the coil is given by
place is
Q 15 × 10 −3
V= = = 15 × 103 V
C 1.0 × 10 −6
The maximum current during the discharge is the cur-
rent flowing through the resistor of 25 Ω immediately
after the switch is closed.
V 15 × 10 −3
∴ Imax = = = 600 A.
R 25
5. During the time from O to t1 second, the potential dif-
ference across the capacitor is the same as the volt-
age of the source.
Between time t1 to t2 second when HK is broken, dis-
charge of capacitor C through resistor in branch LM
Fig. 177
occurs.
During this period, the potential difference V decays φ = BNA cos θ
exponentially. where N is the number of turns in the coil.
After time t2 second when LM is also broken, the dis- A is the cross-sectional area of each turn
charge from C stops and the potential difference V B is the magnetic field in XX′ direction.
maintains constant at the same level just before t2. Thus, the induced emf E in the coil is given by
This variation of V is best represented in graph e. dφ dθ
E=– = + (BAN sin θ)
dt dt
ELECTROMAGNETIC INDUCTION AND ALTERNATING CURRENT 423

which is dependent on the value of sin θ and the angu- For each of the cases (A to E), the values of cos θ and
dθ dθ
lar speed . may be summarised as follows :
dt dt

case θ dθ /dt sin θ E

A. rotation of a quarter revolution changes from θ to π/2 rad changes from 0 to 1 increases from 0 to

about axis YY′ = ω = constant maximum
dt
angular velocity

B. rotation of a half revolution about θ = 0 throughout the rota- 0 no induced emf (E = 0)



axis XX′ tion =0
dt
C. rotation of a half revolution about changes from 0 to π/2 and to dθ changes from 0 to 1 increases from 0 to
axis ZZ′ π rad = ω = constant and back to 0 maximum and back
dt
angular velocity to 0

D. rotation of a quarter revolution θ = 0 throughout the rota- 0 no induced emf (E = 0)



about axis XX′ tion =0
dt
E. rotation of a half revolution about changes from 0 to π/2 and changes from 0 to 1 increases from 0 to

axis YY′ to π rad. = ω = constant and back to 0 maximum and back
dt to 0
angular velocity

emf emf emf emf emf

O time O time O time O time O time


A B C D E

Fig. 178

d d 1 LM FG
πa 2 ω cos ωt
IJ OP 11. When a constant potential difference is applied across

N H K
10. e=– [φ] = – B a solenoid of inductance L, the induced emf across the
dt dt 2 Q coil E is given by the equation
1
= πBa2ω sin ωt dI
2 E=L
e dt
i= where L = constant of proportionality = inductance
R
Thermal power at any instant = e i of the coil, L.

F eI = e 2
LM
1 1 OP 2 dI
= rate of change of current through the coil.
= (e)
H RK R =
R 2NπBa 2 ω sin ωt
Q dt

F 1 πBa ωI 2 L is property of the solenoid and hence does not vary


H2 K
z
2
T
with the current through the coil. Thus, options a and
Mean power, < P > = sin 2 ωt dt b are ruled out.
RT 0
From the definition of induced emf, we also rule out
LM
1 πωa2B OP 2
1 πωa 2 B LM OP 2
options c and d. Thus we can deduced from the graph
=
2R 2 N =
2R Q 2 MN PQ that the solenoid has resistance (real-world solenoid)
as well as inductance.
In this problem, note that we have not taken into
account the magnetic field of induced current.
424 COMPREHENSIVE OBJECTIVE PHYSICS

In fact it can be shown that, if V is the constant µ 0I


potential difference applied to the solenoid and R is B=
2πr
the resistance of the solenoid, we may write
µ0 = free space permeability.
dI
V=L + RI where r = distance from centre of cable.
dt
Thus B is stronger at distance near the cable
1
since B ∝ .
r
To have maximum emf induced in the coil brought near
the cable, the area of flux linkage A and the magnetic
field linking coil B should be maximised since, emf
induced is given by
dφ d
emf induced = – =− (AB).
dt dt
Area A is maximum when coil is placed with its plane
Fig. 179
parallel with the cable and the distance should be as
Solving the equation, we have closed as possible near the cable to maximise B link-
I = I0 (1 – e– t/τ) ing the coil.
L The best arrangement is thus by diagram A.
where τ= = time constant of the variation in
R Note. In diagram B, the magnetic field linking the coil is
current. zero since flux linking the left-half circle of coil is equal and
V

z
opposite to that linking the right-half circle of coil.
I0 = = steady state current through the solenoid.
R → → dφ
F F 17. E . dl = [Ignoring –ve sign]
12. φ = BA, F = Bqv, B = or φ = A dt
qv qv
d π r 2 dB r dB
2 ∴ E × 2πr = [πr2B] or E = =
Nm dt 2 πr dt 2 dt
SI unit is
As ms−1 1 dB
F = QE = rQ
Nm 2 dt
or or 1 kg m s–2 m A–1 or kg m2 s–2 A–1.
A 1 2 dB
13. All the spokes are in parallel. τ = Fr = r Q .
2 dt
14.

15.
|E|=

I=
E 2
d
dt
[B × πr2] = 2πrB

= A = 0.5 A
dr
dt
.
18.
z → →
E . dl = −
Clearly, E ≠ 0

dt

R 4 19. The soft-iron core would enhance the magnetic flux


Due to increase in resistance, there will be a decrease linkage φ between coils P and Q. Since magnitude of φ
of main current. So, according to Lenz’s law, the in- has increased, magnitude of the induced emf would
duced current shall flow in the direction of the main also increase, though its frequency remains unchanged
current. Thus, the current will be greater than 0.5 A. by Faraday’s law of electromagnetic induction.
16. Magnetic field B induced by alternating current I in a 20. When the loop is entering, the magnetic flux linked
cable is described by the equation with the loop is increasing.
According to Lenz’s law, the direction of induced cur-
rent is anticlockwise.
When the loop is coming out of the magnetic field, the
magnetic flux is decreasing.
According to Lenz’s law, the direction of induced cur-
rent is clockwise.
The above considerations lead us to the right choice
(d).
21. Induced emf generated ; shorting gives induced
current ; hence damping.
Fig. 180
ELECTROMAGNETIC INDUCTION AND ALTERNATING CURRENT 425

22. Mutual inductance is same. So, emf is proportional to 4 2


or = loge 22 = 2 loge 2 or = loge 2
dI τ τ
only.
dt 2
or τ= .
23. Energy stored in volume V = energy density × volume loge 2
B2 32. Clearly, resonance circuit.
V. = Resistive circuit.
2µ 0
∴ Z=R
24. The correct choice follows from the definition of root
mean square value. 100
Maximum current = A=1A
25. All the three contribute towards magnetic flux linkage. 100
26. Simple fact. VL and VC are equal and 180° out of phase.
∴ VR = 100 × 1 V = 100 V
1
27. At 900 Hz, > ωL, capacitive circuit. So, maximum voltage is 100 volt.
ωC
Impedance is minimum at resonance. Maximum voltage across capacitor = 1 × 100 V
28. R is independent of frequency. = 100 V
Current becomes maximum at resonance. Maximum voltage across inductance = 100 V.
XL = ωL = 2πf L µ 0N2 A
33. L=
1 1 l
XC = = .
ωC 2πf C µ 0 (2N)2 A
L′ = = 2L.
29. At low frequencies, Lω is low. 2l
30. In (a), (b) and (c) systems, the phase difference is 90°. 34. When the current is switched off, large induced emf
F t I appears across the bulb making it glow brightly.
31. GG
I = I0 1 − e

τ
JJ After a short time, the stored energy is dissipated and
H K the bulb stops glowing.

F 4 I 4 35. The current of 1.6 A lags emf in phase by


π
. The
3
I0 = I0 GG1 − e −
τ
JJ or e

τ 3 1
= 1− = 2
4 H K 4 4
current of 0.4 A leads emf in phase by
π
. So, these
4 2
two currents are 180° out of phase with each other.
or eτ = 4
∴ Net current, I1 = (1.6 – 0.4) A = 1.2 A.

KNOWLEDGE PLUS
l Input power at 22000 V is to be stepped down to 220 V by a transformer with a winding of 4400 turns in the primary.
What should be the number of turns in the secondary of the transformer ?
(a) 440 (b) 44 (c) 220 (d) 5 [Kerala PMT 2003]
Ns 220
Sol. = or Ns = 44
4400 22000
So, (b) is the right choice.
UNIT XV

OPTICS

l Refraction of light l Total internal reflection and its applications l Spherical


lenses l Thin lens formula l Lens maker’s formula l Magnification l Power of a
lens l Combination of thin lenses in contact l Refraction and dispersion of light due to a
prism l Scattering of light l Blue colour of the sky and reddish appearance of the sun at sun-
rise and sun-set l Optical instruments l Compound microscope l Astronomical telescope
(refraction and reflection type) and their magnifying powers l Wavefront and Huygens’
principle l Reflection and refraction of plane wave at a plane surface using wave fronts (qualitative
idea) l Interference l Young’s double slit experiment and expression for fringe
width l Coherent sources and sustained interference of light l Diffraction l Diffraction due
to a single slit l Width of central maximum l Difference between interference and
diffraction l Resolving power of microscope and telescope l Polarisation l Plane polarised
light l Brewster’s law l Use of plane polarised light and polaroids

UNIT DETAILS

1. Synopsis Points 130 Plus

2. Illustrations 30 Plus

3. MCQs from Competitive Examinations 580 Plus

4. Self-Evaluation Tests 2

5. Total Number of MCQs 1275 Plus

6. Total Number of Solutions 1270 Plus


CHAPTER 15

OPTICS
SYNOPSIS

1. Luminous flux is the amount of light energy emitted


1
per second by a source. radiant intensity in that direction of watt per
683
2. One lumen is the luminous flux emitted per unit solid
angle by a uniform point source of luminous intensity steradian.
1 candela. 11. Photometer is an instrument for comparing the
3. One lumen is the luminous flux emitted per unit solid luminous intensities of light sources.
angle by a uniform point source of one candle power. 12. Luminance is the luminous flux per square metre
4. One lumen is the luminous flux through 1 m2 of coming from a surface in the given direction.
surface of 1 m radius when a source of one candle 13. One nit is the luminance of a plane surface, each
power is placed at the centre of curvature of the square metre of which radiates a flux of luminous
surface. intensity of 1 candela in the direction normal to it.
5. If φ represent the total luminous flux emitted by a 14. Reflection coefficient of a surface is the ratio of lumi-
point source, then nance to illuminance.
total luminous flux φ 15. The luminance depends upon the reflection coefficient
1 lumen = = of a surface.
total solid angle 4π
16. Luminance deals with the luminous flux reflected from
or φ = 4π lumen unit area of surface. On the other hand, illuminance
So, the total luminous flux emitted by a source of one deals with luminous flux incident on unit area.
candle power is 4π lumen. Clearly, the total luminous 17. Illuminance or Intensity of illumination of a surface
flux due to a source of x candle power will be 4πx lumen. at any point may be defined as the luminous flux inci-
6. Relative luminosity of a wavelength dent normally on a unit area of the surface held at
that point. It is generally denoted by E.
Luminous flux of a source of given wavelength
= 18. If luminous flux ∆φ falls normally on an area ∆A of a
Luminous flux of a 555 nm source of same power surface, then illuminance is given by
7. Since the eye is less sensitive to blue light than to ∆φ
green light therefore the blue light source must radi- E= .
∆A
ate more power than a green source. 19. The illuminance depends upon the following factors :
8. Luminous intensity of a source in a given direction is (i) Illuminance is directly proportional to the lumi-
defined as the luminous flux per unit solid angle in nous flux. Larger the luminous flux, larger is the
that direction. illuminance of the given surface.
9. The standards of luminous intensities are maintained (ii) Illuminance follows inverse square law of dis-
at National Standards Laboratories. tance. When the distance from the source is dou-
bled, the illuminance decreases by a factor of four.
10. The candela is defined as the luminous intensity, in a
given direction, of a source that emits monochromatic (iii) Lesser the obliquity of the incident rays, more is
the illuminance. We get maximum illuminance if
radiation of frequency 5.40 × 1014 Hz and that has a
the incident light is normal to the surface.

429

C-11\IITS\C15-1 CDR\C-151a
430 COMPREHENSIVE OBJECTIVE PHYSICS

(iv) Illuminance is directly proportional to the efficiency 30. Radiant flux through an area is the total amount of
of the light source, provided the sources have the energy flowing normally per second through that area.
same input energy. It includes all radiations whether visible or not. It
20. Phot or centimetre candle is the illuminance produced represents the radiant power of the source.
at the inner surface of a sphere of one centimetre 31. Radiant flux is measured in watt.
radius when a source of one candela is placed at its 32. The luminous flux per unit radiant flux is called
centre. luminous efficiency.
21. The intensity of illumination of surface is called Total luminous flux
1 phot (or a centimetre candle) if it is held at a distance 33. Luminous efficiency = .
of 1 cm from a source of one candela in such a way Total radiant flux
that light rays fall normally on the surface. 34. The efficiency of a light source is the ratio of the lumi-
22. Lux or metre candle is the illuminance produced at nous flux emitted and the input electrical power.
the inner surface of a sphere of one metre radius when 35. The efficiency of a light source is the ratio of the out-
a source of one candela is placed at its centre. put power in the visible region to the input electrical
23. The intensity of illumination of a surface is called power.
1 lux (or a metre candle) if it is held at a distance of 36. The unit of luminous efficiency is lumen/watt.
1 m from a source of one candela in such a way that 37. The efficiency of an electric bulb is nearly four times
light rays fall normally on the surface. Its symbol less as compared to the efficiency of a fluorescent tube.
is lx. So, for the same consumption of power, the amount of
24. 1 phot = 1 lm cm–2 light received from a bulb is four times less than the
1 lux = 1 lm m–2 amount of light received from a fluorescent tube.
25. Relation between phot and lux 38. RATIO OF INTENSITIES OF ILLUMINATION AT
THE CENTRE AND EDGE OF A CIRCULAR
1 phot = 1 lm cm–2 = 1 lm (10–2m)–2 = 104 lm m–2
TABLE
1 phot = 104 lux
I
26. Relationship between illuminance and EC = θ
luminous intensity I for a point source h2

2
r +
I cos θ
p I cos θ h
E= EE =

h
2
r 2 r2 + h2
^n
If the surface is perpen- I h
dicular to the light beam, = × E
r r + h2
2
r + h2
2 C r
then θ = 0° and cos θ = cos
θ
0° = 1. Ih
In that case, or EE = Fig. 2
(r + h2 )3 / 2
2

I O
E=
Now, EC I (r 2 + h 2 ) 3 / 2
r 2 = 2 ×
Fig. 1 EE h Ih
27. Lambert’s cosine law
(r 2 + h2 )3 / 2
(r 2 + h2 )3 / 2 r F I
2 3 /2
∴ E ∝ cos θ
The illuminance of the surface is directly proportional h3
= 2 3/ 2
(h )
= 1+ 2
h
. = GH JK
to the cosine of the angle of inclination i.e., the angle 39. RATIO OF INTENSITIES OF ILLUMINATION AT
between the direction of the central ray and the nor- THE CENTRE AND CORNER OF A SQUARE
mal to the surface. TABLE
28. At noon, the rays of light fall normally. So, θ = 0°. I
2
h

EO = θ
This increases E considerably.
+ 2
2

h2
l

29. In the southern hemisphere of earth, the angle of I cos θ Ih


h
EC = = C
incidence of sun rays on earth is minimum in
December and maximum in June. According to h +
l
2
2
LMh 2 l O
+ P
2 3/ 2

MN 2 PQ
l
Lambert’s cosine law, the intensity of heat radiation 2
O 2
on the surface of earth will be maximum in December
and minimum in June. So, in the southern LMh 2
+
l2 OP 3/ 2

hemisphere, there is summer in December and winter EO


=
I
× N
M 2 PQ l
in June. EC h2 Ih Fig. 3
OPTICS 431

the angles of incidence and reflection become (θ + φ).


LMh
2 l2 OP 3/ 2
LMh 2 l2 OP 3/ 2
The deviation is [180° – 2(θ + φ)]. The change in direc-

= N
M +
2 PQ = N
M +
2 PQ = 1+
LM l2 OP 3/ 2
.
tion is 2φ. Thus, the reflected ray is turned through
h3 [ h2 ]3 / 2 MN 2 h2 PQ twice the angle described by the mirror.
42. When three plane mirrors are at right angles to each
40. RATIO OF INTENSITIES OF ILLUMINATION AT
other, one image is produced by each plane mirror
A POINT ON THE CENTRE OF THE EDGE AND
and one by each of the three pairs of combination of
THE CORNER OF SQUARE TABLE
two mirrors each.
I cos θ1 Ih 43. The image in a plane mirrorr undergoes depth inver-
EC = = 3
r12 r1 sion. So, the time on the image clock is as much ahead
h q1
l2 q2
of 12 hour zero minute as the time on actual clock is
But 2
r1 = h2 + behind 12 hour zero minute.
4 r2
r1 44. The size of the mirror has no effect on the nature of
Ih C¢
∴ EC = the image. However, it affects the brightness of the
LMh 2 l O
+ P
2 3/ 2 image.
MN 4 PQ
C
45. NUMBER OF IMAGES FORMED BY TWO IN-
CLINED PLANE MIRRORS
I cos θ2 Ih
EC′ = = Consider two plane mirrors inclined to each other at
r22 r23 an angle θ degrees. When an object is placed between
Fig. 4 them, multiple images are formed. This is due to
l2
But r22 = h2 + multiple successive reflections.
2
360
Ih If is even, then the number of images is given by
∴ EC′ = θ
LMh 2
+
l2 OP 3/ 2
360
MN 2 PQ n=
θ
–1

360
LMh 2
+
l2 OP 3/ 2
LM h 2 l2 OP 3/ 2 If
θ
is odd, then the following two situations arise :

Now, EC = Ih
×N
M 2 PQ =M
+
2 PP (i) If the object lies symmetrically, then
LMh MM h
PPQO
3/ 2 l2
EC ′
2
+
l2 Ih
N
2
+ PQ n=
360
–1
MN 4
4 θ
(ii) If the object lies asymmetrically, then
41. ROTATING A MIRROR 360
A ray of light striking the mirror at an angle of inci- n=
θ
dence θ is reflected at the same angle. The deviation
i.e. the angle between incident and reflected rays is 360
If the two plane mirrors are parallel, then n =
180° – 2θ. 0
= ∞. So, infinite number of images is formed in this
If the direction of the incident ray is unchanged and
case.
the mirror is rotated through an angle φ, then each of

Angle between two mirrors, θ 15° 30° 45° 60° 72° 90° 120° 180° 360°
360
24 12 8 6 5 4 3 2 1
θ
Number of images

For symmetric placement 23 11 7 5 4 3 2 1 0

For asymmetric placement 23 11 7 5 5 3 3 1 1


432 COMPREHENSIVE OBJECTIVE PHYSICS

1 1 1 52. The radius of curvature of a concave mirror is meas-


46. The mirror formula + = is applicable to both ured towards the left of the pole. So, the radius of
u v f
concave and convex mirrors. curvature of a concave mirror is taken as negative.
For the same reason, the focal length of a concave
v
47. The formula m = – is applicable to both concave mirror is taken as negative.
u
and convex mirrors. 53. If the image is enlarged, then the numerical value of
48. While solving numerical problems, the numerical magnification is greater than one. On the other hand,
values of the given quantities are substituted with if the image is diminished, then the numerical value
proper signs. No sign is to be given to the unknown of magnification is less than one.
quantity. 54. When m is positive, the image is erect (virtual).
49. Real image is formed in front of the mirror. It is in- 55. When m is negative, the image is real (inverted).
verted. Magnification, in this case, is negative. 56. For enlarged image, m > 1.
50. Virtual image is formed at the back of the mirror. It 57. For diminished image, m < 1.
is erect. Magnification, in this case, is positive. 58. When an extended two-dimensional object is placed
51. Since the radius of curvature of the convex mirror is in such a way that its surface is perpendicular to the
measured towards the right of the pole therefore the axis of the mirror, then the area of the image is
radius of curvature of the convex mirror is positive. magnified.
For the same reason, the focal length of the convex v2
mirror is taken as positive. Superficial magnification = 2 , where the letters have
usual meanings. u

59. Concave Mirror


Position Position Real/ Inverted/ Magnification Sign of RAY DIAGRAM
of object of image virtual erect and size of image magnification

at infinity at focus (v = f) real inverted m<1 negative


(u = ∞) greatly
F
diminished B¢
P
C

beyond 2f between f and 2f real inverted m<1 negative


(u > 2f) (f < v < 2f) diminished

at 2f at 2f (v = 2f) real inverted m=1 negative A


(u = 2f) same size
B
P
C B′ F

A′

between beyond 2f real inverted m=1 negative


f and 2f (v > 2f) magnified
A
(f < u < 2f) B′ C B P
F
A′
OPTICS 433

at f (u = f) at infinity real inverted m=∞ negative


(v = ∞) magnified A
C B
P
F

between pole behind the virtual erect m>1 positive A′

and focus mirror magnified A


(u < f) C

F B P B′

60. Convex Mirror


at infinity at focus virtual erect m<1 positive
(u = ∞) (v = f) diminished
[at the back of
the mirror]
P F

anywhere between pole virtual erect m<1 positive


A
between and focus diminished
infinity [at the back of A′

and pole the mirror] B P B′ F C

61. The refractive index of denser medium w.r.t. rarer 64. The optical fibres can transmit light beam from one
medium is the reciprocal of the refractive index of end to the other due to the repeated total internal
rarer medium w.r.t. denser medium. reflections even if the fibre is bent or twisted.
1 65. The optical illusion that water is present at some dis-
aµb = . tant place is called inferior mirage. This generally
µ
b a
occurs on very hot summer days. This is due to total
62. The ray of light will leave the parallel glass slab at internal reflection.
the same angle at which it entered the glass slab on
66. The optical illusion of objects floating in air is called
the opposite side. However, it gets laterally displaced.
superior mirage. It is also known as looming. This
t sin (i1 − r1) occurs in very cold regions. This is due to total inter-
Lateral displacement =
cos r1 nal reflection.
Here i1, is the angle of incidence, r1 is the angle of 67. The refractive index of denser medium b w.r.t. rarer
refraction and t is the thickness of the slab. medium a is the ratio of real depth and apparent
depth.
63. The angle of incidence in the denser medium for which
the angle of refraction is 90° is called critical angle. Real depth
aµb =
Apparent depth
1
µ= Apparent depth
sin ic or bµa = .
Real depth
434 COMPREHENSIVE OBJECTIVE PHYSICS

68. For refraction from rarer to denser medium 76. The distance of the second principal focus of a lens
from its optical centre is called the second principal
µ1 µ 2 µ 2 − µ 1
+ = . focal length of the lens.
−u v R
69. For refraction from denser to rarer medium
µ2 µ µ − µ2
+ 1= 1 . F2
−u v R C F2 C

70. FIRST AND SECOND PRINCIPAL FOCAL


LENGTHS OF A SPHERICAL SURFACE f2 f2
µ 1R
f1 =
µ 2 − µ1 Fig. 7 Fig. 8

µ 2R F I
f2 =
µ 2 − µ1
1
f2
= (µ − 1)
1
GH

1
R1 R2
JK
µ 2 − µ1 77. Linear magnification produced by a lens,
Power of refracting surface, P =
(in dioptre).
R v f −v f
71. FORMULA FOR MAGNIFICATION IN THE m= = = .
u f f +u
CASE OF REFRACTION AT SPHERICAL SUR-
FACES
 In the case of convex lens, linear magnifica-
µ v tion is negative in the case of real image and
Transverse magnification, m = 1 .
µ 2v positive in the case of virtual image.
72. LENS MAKER’S FORMULA FOR THIN LENS In the case of concave lens, magnification is
1 F I F1 I always positive.

f
= (µ – 1)
1
GH

1
R1 R 2
JK or P = (µ – 1) GH R
1

1
R2
JK
.
78. POWER OF A LENS
73. First Principal focus of a lens is that point on the 1
principal axis of the lens at which if an object is placed, P=
f
the image would be formed at infinity.
Since focal length of a converging lens is positive there-
74. The distance of the first principal focus of a lens from fore its power is positive. Similarly, the power of a diverging
its optical centre is called the first principal focal length lens is negative.
of the lens. Opticians express the power of a lens in terms of a
unit called the dioptre. It is regarded as the SI unit of optical
power.
The power of a lens is said to be one dioptre if the focal
F1
F1 length of the lens is 1 metre.
C C
100
When focal length is in cm, P =
dioptre.
f
f1 f1
79. LENS EQUATION/LENS FORMULA
1 1 1
Fig. 5 Fig. 6 − =
v u f
F I This equation holds good for both convex and concave
1
f1
= (µ − 1)
1
− GH
1
R1 R 2
JK lenses, whether the image formed is real or virtual.
80. DISPLACEMENT METHOD FOR FINDING THE
75. Second principal focus of a lens is that point on the FOCAL LENGTH/POWER OF A CONVEX LENS
principal axis of the lens where the image is formed (i) In this method, the distance between the object
when the object is at infinity. and the screen must be greater than 4 f, where f is the focal
length of the convex lens.
OPTICS 435
(ii) The image on the screen can be formed correspond- 2 2
1 1 1
ing to two different positions of the lens. = − = −
f v u D + d − (D − d)
(iii) Fig. 9 shows the magnified image of size I1 for the
position L1 of the lens. [Note that object distance is taken as negative.]
I1 v 1 2 2
m1 = = …(1) or = +
O u f D+d D−d
u v
1 2D − 2d + 2D + 2d
or =
O f D2 − d 2

I1 1 4D D2 − d 2
or = 2 or f=
L1 f D − d2 4D

Fig. 9 4D
Also, P=
(iv) Fig. 10 shows the diminished image of size I2 for D − d2
2

the position L2 of the lens.


81. FOCAL LENGTH OF COMBINATION OF TWO
u v THIN LENSES IN CONTACT
1 1 1
= −
f1 v1 u
O
1 1 1
= −
I2 f2 v v1

Adding,
1 1 1 1 …(1)
v u + = −
f1 f2 v u
L1 L2

d
D

Fig. 10
L1 L2
I u
m2= 2 = …(2) O I I1
O v
From (1) and (2), u v
v1
I1I2 v u
= × =1
O2 u v

I 1I2 Fig. 11
or O=
If the given combination is replaced by a single lens of
I1 v v v2 (D + d)2 focal length F which forms the image of the given point object
Also, = × = 2 =
I2 u u u (D − d)2 at I, then

m1 v2 1 1 1 …(2)
(v) = = −
m2 u2 F v u

(vi) d = v – u and D = v + u 1 1 1
Comparing (1) and (2), = +
D+d F f1 f2
2v = D + d or v =
2
If P is the power of the combination, then P = P1 + P2
D−d where P1 and P2 are the powers of the individual lenses.
Again, 2u = D – d or u =
2
436 COMPREHENSIVE OBJECTIVE PHYSICS

82. Convex Lens


Position Position Real/ Inverted/ Magnification Sign of RAY DIAGRAM
of object of image virtual erect and size magni-
of image fication

at infinity at focus (v = f) real inverted m<1 negative


(u = ∞) greatly
diminished

A
beyond between f and 2f real inverted m<1 negative
2f (u > 2f) (f < v < 2f) diminished
F B´ 2F
B 2F F

at 2f at 2f (v = 2f) real inverted m=1 negative A


(u = 2f) same size

B F 2F
2F F B´

between beyond real inverted m>1 negative A


f and 2f 2f (v > 2f) magnified
(f < u < 2f) F 2F B´
2F B F

at f (u = f) at infinity real inverted m=∞ negative


(v = ∞) magnified
F
F


between at a distance virtual erect m>1 positive
optical centre greater than the magnified
and focus object distance A
(u < f) and on the same B´ F
side as object 2F F B
(v > u)
OPTICS 437
83. Concave Lens
at infinity at focus virtual erect m<1 positive
(u = ∞) (v = f) diminished

between between optical virtual erect m<1 positive


infinity and centre and focus diminished A
optical centre A´

B F B´

84. A substance in which the speed of a wave varies with For an achromatic combination of two prisms,
wavelength is said to exhibit dispersion. (µv – µr)A + (µv′ – µr′) A′ = 0
85. The phenomenon of splitting a ray of white light into Also, ωδ + ω′δ′ = 0.
its constituent colours (wavelength) is called dispersion. 97. Dispersion without deviation
86. The band of colours, from red to violet, that emerges δ + δ′ = 0
from the prism is called spectrum. (µ – 1)A + (µ′ – 1)A′.
87. The difference in the deviations suffered by two colours 98. A prism is a wedge-shaped body made from a trans-
in passing through a prism gives the angular parent refracting medium bounded by two plane
dispersion for these colours. surfaces inclined to each other at some angle.
88. Angular dispersion is defined as the rate of change of 99. The line along which the refracting faces of the prism
angle of deviation with wavelength. meet is called the refracting edge of the prism.
89. The angle between the emergent rays of any two
100. The angle between the two refracting faces of the prism
colours is called angular dispersion between those
is called the angle of the prism. It is denoted by A.
colours.
101. The minimum value of the angle of deviation of a ray
90. Angular dispersion = δv – δr = (µv – µr) A
of light passing through a prism is called angle of
91. The dispersive power of the material of a prism for minimum deviation.
any two colours is defined as the ratio of the angular
102. The refractive index of the material of the prism is
dispersion for these two colours to the mean deviation
given by :
produced by the prism.
It may also be defined as dispersion per unit deviation. FG A + δ IJ
H 2 K
m
δ − δ r µv − µ r sin
92. Dispersive power = v =
δ µ−1
µ=
F AI
sin G J
Here, δ = v
δ + δr
and µ = v
µ + µr H 2K
2 2
103. In the position of minimum deviation,
93. Chromatic aberration = ω f.
A + δm A
94. The condition for achromatism is : i= and r =
ω1 ω2 2 2
+ = 0. 104. Inside a prism, the refracted ray bends towards the
f1 f2
base.
95. Spherical aberration can be minimised by using two
105. Dense hot gases and white hot solids give continuous
plano-convex lenses separated by a distance d such
spectrum.
that
106. Line spectrum is given by substances in atomic states.
d = f1 – f2.
Examples : sodium vapour lamp, mercury vapour lamp
96. Deviation without dispersion
and gases in discharge tubes.
438 COMPREHENSIVE OBJECTIVE PHYSICS

107. Incandescent vapours in the molecular state give band


D
spectra. Examples : calcium or barium salts in the Me = 1 = ,
flame of bunsen burner. fe
108. The angular magnification or magnifying power of a where fe is the focal length of the eyepiece.
simple microscope is the ratio of the angle subtended vo
at the eye by the image at the near point and the Mo =
uo
angle subtended at the unaided eye by the object at
the near point. where v0 is the distance A′B′ from the objective and
D uo is the distance of the object from the objective.
M=1+ …(1)
vo F D I
f
In case the eye is placed behind the lens at a distance
M=
uo
GH
1+
fe
JK
‘a’, then the above relation is modified as follows : Since the object is placed very close to the principal
focus of the objective therefore uo is nearly equal to fo.
D−a
M=1+ Moreover, the focal length of the eyepiece is small. So,
f the image A′B′ is formed very close to the eyepiece. So
So, smaller the focal length of the lens, greater will be vo is nearly equal to the length L of the microscope
the magnifying power. tube. Note that L is the separation between the two
The simple microscope may be used in such a way lenses.
that the image is formed at infinity. In this case, L D F I
D
M=–
fo
1+
fe
GH JK
M= …(2)
f It is clear from the above equation that the smaller
the focal lengths of the objective and eyepiece, larger
It is clear from comparison of equations (1) and (2)
is the magnifying power.
that (i) the maximum angular magnification is ob-
tained when the image is at the near point and (ii) the 1 1 1
Again, − =
minimum angular magnification is obtained when the vo uo fo
image is at infinity.
vo vo vo
109. Angular magnification or magnifying power of a com- or − =
pound microscope is defined as the ratio of the angle vo uo fo
subtended by the final image at the eye to the angle vo v vo v
subtended by the object seen directly, when both are or – = − 1+ o or = 1− o
uo fo uo fo
placed at the least distance of distinct vision.
The magnification produced by the compound F vo I F1 + DI
microscope is the product of the magnifications ∴ M = 1− GH fo
JK GH f JK
e
produced by the eyepiece and objective.
When the final image is formed at infinity, then
L D
M = Mo × Me = – ×
vo fo fe
A A´´´
Fo a In this case, the microscope is said to be in normal
B Fo B² Fe B¢ b adjustment.
fo fo
uo 110. Best position for the eye. The best position for the
O A¢ eye is at the real image of the objective formed by the
E eyepiece or eye-ring. All the rays from the object pass
through this image. In fact, the position of the eye-
A² D ring is such that the light coming through the whole
L
of the eye-lens can enter the small pupil of the eye.
Fig. 12 111. The telescope is said to be in normal adjustment when
M = Me × Mo the final image is formed at infinity.
where Me and Mo are the magnifying powers of the 112. Magnifying power of a telescope in normal adjustment
eyepiece and objective respectively. is the ratio of the angle subtended by the image at the
OPTICS 439

eye as seen through the telescope to the angle sub- The adjustment of the telescope shown in Fig. 15 is
tended by the object as seen directly, when both the called normal adjustment.
object and the image are at infinity. Length of telescope = fo + 4fa + fe
(a) When the final image is formed at infinity
fo
Magnifying power =
fe
fo
fe
If the final image is formed at the least distance of
distinct vision, then magnifying power is
α
α
Fo, Fe
FG
fo f IJ
C1 I β C2
feH1+ e .
D K
114. GALILEAN TELESCOPE
E Length of telescope tube is fo – fe , where fo and fe
t
O
ag
ea represent the focal lengths of the objective and eyepiece
Im inity
in f respectively. Magnifying power is the ratio of the focal lengths
of the objective and eyepiece.
Fig. 13

fo
M=–
fe
Clearly, for high angular magnification, the objective
should have a large focal length and the eyepiece a small
focal length. It may be noted that the separation of the lenses
is fo + fe.
(b) When the final image is formed at the least distance
of distinct vision O E
fe
fo

C Fe Fo C´ Fig. 16
α
α I β The main disadvantage of the Galileo telescope is that
the field of view of this telescope is small as compared to the

other terrestrial telescope and the astronomical telescope.
D 115. PRISM BINOCULAR

Fig. 14 O

fo f FG IJ
M=–
fe
1+ e .
D H K P1

113. TERRESTRIAL TELESCOPE


P2

A1
2fa

Fo E
B B1
Fig. 17
2fa
A
Auxiliary 116. In YDSE, the distance of nth bright fringe from the
lens Eyepiece
nDλ
Objective centre of interference pattern is , where D is the
d
Fig. 15
440 COMPREHENSIVE OBJECTIVE PHYSICS

distance of the screen from the slits and d is the dis- produce separate images of two objects very close to-
tance between the two coherent sources producing gether is called resolving power. The term “resolv-
interference. ing power” is used in two senses : (1) when the pur-
117. In YDSE, the distance of nth dark fringe from the pose is to see as separate two objects close together or
(2n + 1)Dλ when the fine structure is seen in a microscope or
centre of the interference pattern is . telescope ; and (2) when the purpose is to see two spec-
2d
tral lines as distinct from each other.

118. In YDSE, the fringe width is . 128. Resolving power of a lens. The resolving power of
d
a lens is its ability to resolve two points that are close
119. If a thin film of mica of thickness t is introduced in to each other. In other words, the resolving power of a
the path of one of the interfering rays, then the shift lens is its ability to separate the images of two points
D that are close together. The resolving power is directly
of the central bright fringe is (µ – 1) t. However,
d proportional to the aperture of the lens and inversely
there is no effect on fringe width. So, the fringe pat- proportional to the wavelength of light used. Thus,
tern remains unchanged. increasing the aperture of the lens increases its
120. In diffraction due to a single slit, the condition for nth resolving power since it decreases the diameter of the
secondary minimum is diffraction disc and thus two images can be closer
d sin θn = nλ together without overlapping by as much as the radius
121. The condition for nth secondary maximum is : of the disc.
129. Resolving power of an optical instrument. The
λ
d sin θn = (2n + 1) resolving power of an optical instrument is the angle
2 subtended at the objective by two point objects which
122. In diffraction at a single slit, the secondary maxima are just resolved. Smaller the angle, higher is the
are less intense than central maximum. resolving power of the optical instrument. The resolv-
123. Fresnel distance is that distance from the slit at which ing power may also be defined as the ratio of wave-
the spreading of light due to diffraction becomes equal length λ to the smallest difference in wavelength dλ
to the size of the slit. It is generally denoted by DF or that can be resolved.
ZF . 130. Rayleigh criterion. According to Rayleigh criterion,
124. According to Malus’ law, I = I0 cos2
θ where θ is the when the central maximum in the diffraction pattern
angle between the planes of transmission of analyser of one point source falls over the first minimum in
and polariser. the diffraction pattern of the other point source, then
125. According to Brewster’s law, µ = tan ip, where ip is the two point sources are said to have been resolved
the polarising angle. by the optical instrument.
126. Eye as an optical instrument has a limitation. 131. Resolving power of a microscope. The resolving
When two objects or their images are very close power of microscope is its ability to form separate im-
together, then it may not be possible for the eye to see ages of two point objects lying close together. It is
them as separate. The eye can see two objects as sepa- determined by the least distance between two point
rate only if the angle subtended by them at the eye is objects which can be distinguished. This distance is
greater than 1′. This is on the assumption that the λ
pupil of the eye is nearly 2 mm in diameter. given by ∆d = , where λ is the wavelength of
2µ sin θ
127. Need of telescope or microscope with reference
light used to illuminate the object and µ is the refrac-
to resolution and resolving power. If two nearby
tive index of the medium between the object and the
objects subtend an angle less than 1′ at the eye, they
objective. The angle θ is the half-angle of the cone of
cannot be seen as separate. However, they may be
light from the point object i.e. it is the angle which a
seen as separate from each other by using either a
marginal ray makes with the axis of the microscope.
lens or a telescope or a microscope or prism etc. The
The term µ sin θ is called the numerical aperture of
method of seeing such objects as separate is called
the objective.
resolution. The ability of an optical instrument to
OPTICS 441

132. IMPORTANT FACTS ABOUT THEORIES OF NATURE OF LIGHT

Name of Year Name of Recti- Reflec- Refrac- Double Inter- Diffrac- Polarisa- Disper- Photo Compton
theory scientist linear tion tion refrac- ference tion tion sion electric effect
propaga- tion effect
tion
Corpus- 1675 Newton Yes Yes Yes No No No No No No No
cular theory
of light
Wave theory 1678 Huygens Yes Yes Yes Yes Yes Yes Yes Yes No No
of light
Electro- 1873 Maxwell Yes Yes Yes Yes Yes Yes Yes Yes No No
magnetic
wave theory
of light
Quantum 1905 Einstein Yes Yes Yes No No No No No Yes Yes
theory of
light
Dual theory 1923 de Broglie Yes Yes Yes Yes Yes Yes Yes Yes Yes Yes
of light

ILLUSTRATIONS ray is incident at an angle of 30° at a point just inside one


end of A. The plane of incidence coincides with the plane of
Illustration 1. Fig. 18 shows the
L the figure. The maximum number of times the ray under-
vertical section of a tunnel. A lamp L is
fixed on the ceiling. If its emission is iso- goes reflections (including the first one) before it emerges
tropic, and the section is circular, the A out is
ratio of illuminances at vertical part of (a) 28 (b) 30
the wall (A) to that at the floor (B) is B (c) 32 (d) 34.
(a) 1 (b) 2 [IIT Screening 2002]
Fig. 18 Sol. Length required for one reflection = x (say)
(c) 2 2 (d) 2 .
[National Standard Exam. in Physics 1993]

I cos 45° 2r 30°


0.2 m

EA ( 2 r) 2
Sol. = 30°
EB I r
(2r)2
A
r
x
r
1 1 2×2 Fig. 21
= × × = 2
2 2 1 B x
Fig. 19
Now, tan 30° =
So, (d) is the right choice. 0.2
0.2
Illustration 2. Two plane mirrors A and B are or x = 0.2 tan 30° = m
aligned parallel to each other, as shown in Fig. 20. A light 3
Total number of reflections
2Ö3 m B Total length 2 3× 3
= = = 30.
x 0.2
So, (b) is the right choice.
30°

0.2 m
Illustration 3. The focal length of a concave mir-
ror is f and the distance from the object to the principal
A focus is p. The ratio of the size of the real image to the size
Fig. 20 of the object is
442 COMPREHENSIVE OBJECTIVE PHYSICS

(a) f/p (b) (f/p)2 Sol. It is a simple question based on factual informa-
(c) (f/p)1/2 (d) p/f. tion.
[National Standard Exam. in Physics 2005] So, (a) is the right choice.
Illustration 8. An equi-convex
Sol. The object distance is (f + p) and the magnifica-
glass lens has a focal length f and power
v P. It is cut into two symmetrical halves
tion is .
u by a plane normal to the principal axis.
So, (a) is the right choice. Then the focal length of each half piece is :
Illustration 4. The distance between an object and (a) P (b) P/2
a screen is 1 m. A lens produces an image on the screen (c) 2/P (d) Zero.
when placed at either of the positions 0.40 m apart. The Fig. 22
[AMU 2003]
power of the lens is nearly :
(a) 2.4 D (b) 4.8 D FG 2 IJ
(c) 7.2 D (d) 9.6 D. [SCRA 1994]
Sol. P = (µ – 1)
H RK
4D P′ = (µ – 1) FG 1 IJ
Sol. P=
D2 − d2 H RK
4×1 4 P′ 1 P
= = D = 4.76 D ≈ 4.8 D Dividing, = or P′ =
1 − 0.4 2
0.6 × 1.4 P 2 2

So, (b) is the right choice. 1 2


Now, f′= =
Illustration 5. A conve x lens of focal length P P
40 cm is in contact with a concave lens of focal length So, (c) is the right choice.
25 cm. The power of the combination is : Illustration 9. The image of a real object formed
(a) – 1.5 D (b) – 6.5 D by a convex lens placed in air is
(a) always real, erect and magnified
(c) + 6.5 D (d) + 6.67 D.
(b) always real, inverted and magnified
[JIPMER 2003 ; IIT 1982]
(c) always real, inverted and diminished
1 1 (d) may be real or virtual, erect or inverted, magni-
Sol. P= +
f1 f2 fied or diminished. [Bharati Vidhyapeeth 2003]
100 100 Sol. It is a simple question based on factual
= − = 2.5 – 4 = – 1.5 D
40 25 informatoin.
So, (a) is the right choice. So, (d) is the right choice.
Illustration 6. The focal length of combination of Illustration 10. A convex lens of focal length 20 cm
two lenses separated by distance d produces a virtual image twice the size of the real object.
Then the distance of the object from the lens is
(a) increases as their separation is increased
(a) 20 cm (b) 30 cm
(b) decreases as the separation is increased
(c) is less than the focal length of either lens for all d (c) 10 cm (d) 60 cm
(d) is less than the focal length of one of them if d is [CMC Vellore 2003]
greater. [CMC Vellore 2003] f
Sol. m =
1 1 1 d f +u
Sol. = + −
F f1 f2 f1 f2 20
+2= or 20 + u = 10
1 20 + u
As d is increased, decreases. So, F increases
F or u = (10 – 20) cm = – 10 cm
So, (a) is the correct choice. So, (c) is the correct choice.
Illustration 7. Which of the following is used in Illustration 11. A convex lens of focal length 20 cm
optical fibres ? produces a real image twice the size of the object. Then the
(a) Total internal reflection distance of the real object from the lens is
(b) Scattering (c) Diffraction (a) 20 cm (b) 30 cm
(d) Refraction. [AIEEE 2002] (c) 10 cm (d) 60 cm. [Manipal 2002]
OPTICS 443

f
Sol. m =
f +u
(a) R1 R2 (b) R ∞
20
–2=
20 + u
R1 ≠ R2
or – 40 – 2u = 20 or – 2u = 20 + 40 = 60
or u = – 30 cm
So, (b) is the right choice.
(c) R R (d) R ∞
Illustration 12. An observer can see through a pin-
hole the top end of a thin rod of height h, placed as shown
in Fig. 23. The beaker height is 3h and its radius h. When
Fig. 25
the beaker is filled with a liquid up to a height 2h, he can
[IIT Screening 2002]
see the lower end of the rod. Then the refractive index of
the liquid is FG 1 − 1 IJ
Sol. dP = dµ
H R RK = 0.

So, (c) is the right choice.


Illustration 14. A ray of light passes through four
transparent media with refractive indices µ1 , µ2 , µ3 and
µ4 as shown in the Fig. 26. The surfaces of all media are
parallel. If the emergent ray CD is parallel to the incident
3h ray AB, we must have

D
h
µ1 µ2 µ3 µ4
2h
C
Fig. 23 B

(a) 5/2 (b) (5/2)


(c) (3/2) (d) 3/2. A
[IIT Screening 2002]
Fig. 26
1 sin i
Sol. = (a) µ1 = µ2 (b) µ2 = µ3
µ sin r
r
(c) µ3 = µ4 (d) µ4 = µ1.
h 1 h [IIT Screening 2001]
But sin i = =
5h 5 Sol. Using Snell’s law, µ4 sin θ = µ1 sin θ
h
Ö2 or µ4 = µ1
h h i
and sin r = r
So, (d) is the right choice.
2h
h

2h Illustration 15. A thin glass (refractive index 1.5)


Ö5

1 1 2 lens has optical power of – 5 D in air. Its optical power in a


∴ = ×
µ 5 1 h liquid medium with refractive index 1.6 will be
(a) – 25 D (b) 25 D
5 (c) – 1 D (d) 1 D
or µ= . h
2
Fig. 24 5
So, (b) is the right choice. (e) D. [AIEEE 2005, Modified]
8
Illustration 13. Which one of the following spherical Fµ IF 1 I
lenses does not exhibit dispersion ? The radii of curvature Sol. P = GH µ2
1
−1 JK GH R
1

1
R2
JK
of the surfaces of the lenses are as given in the diagrams.
444 COMPREHENSIVE OBJECTIVE PHYSICS

FG 1.5 − 1IJ FG 1 1 I
JK
1 1 1
= + =
1
+
1
=−
2
–5=
H 1 K HR 1

R2 f u v − 40 − 40 40
−5 1 1 40
= − or f=– cm = – 20 cm
0.5 R 1 R2 2
So, (c) is the right choice.
1 1
or − = – 10 Illustration 18. Refractive index
R1 R2
of glass is 1.5 and that of water 4/3. A
F 1.5 − 1IJ
B A

P = GH
light ray incident normally on the face AB q
Again,
1.6 K (– 10)
of the glass prism shown in the figure is
(− 0.1) (− 10) 5 totally reflected to reach the face BC if
= = D (a) θ ≥ sin–1 8/9
1.6 8 C
So, (e) is the right choice. (b) θ ≤ sin–1 2/3
Illustration 16. A thin concave and a thin convex (c) θ ≥ sin–1 2/3
lens are in contact. The ratio of the magnitude of power of Fig. 28
(d) θ ≤ sin–1 8/9.
two lenses is 2/3 and focal length of combination is 30 cm,
then the focal length of individual lenses are [National Standard Exam. in Physics 2005]
(a) – 15 cm, 10 cm (b) – 75 cm, 50 cm Sol. Angle of incidence on the hypotenuse face of the
prism is θ which must be greater than or equal to critical
(c) 75 cm, – 50 cm (d) 75 cm, 50 cm. angle θc = sin–1 8/9.
[IIT Screening 2005] So, (a) is the right choice.
P′ 2 f −2 3 Illustration 19. By placing a convex lens of focal
Sol. =− , = or f ′ = – f
P 3 f′ 3 2 length equal to 15.0 cm between an object and a screen sepa-
rated by a distance of 75.0 cm, the sizes of the images ob-
1 1 1 tained are 6.0 cm and 2/3 cm. The size of the object must
Now, = +
F f f′ be
1 1 2 3−2 1 (a) 2.0 cm (b) 4.0 cm
= − = =
30 f 3 f 3f 3f (c) 3.0 cm (d) 1.5 cm.
or 3f = 30 cm or f = 10 cm [National Standard Exam. in Physics 2005]
3 Sol. Use the formula :
Again, f′=– × 10 cm = – 15 cm
2 Size of image = d1d2 , where d1 and d2 are the sizes
So, (a) is the right choice.
of reduced and magnified images.
Illustration 17. A tank So, (a) is the rigth choice.
of height 33.25 cm is completely 15 cm
Illustration 20. A given ray of light suffers mini-
filled with liquid (µ = 1.33). An
object is placed at the bottom of mum deviation in an equilateral prism P. Additional prisms
Q and R of identical shape and of the same material as P
tank on the axis of concave 25 cm 33.25
mirror as shown in the Fig. 27. are now added as shown in Fig. 29. The ray will now suffer
Image of the object is formed 25
cm below the surface of the
Fig. 27 Q
liquid, then focal length of the
mirror is P R

(a) 10 cm (b) 15 cm
(c) 20 cm (d) 25 cm. Fig. 29
[IIT Screening 2005] (a) greater deviation (b) no deviation
RD RD 33.25 (c) same deviation as before
Sol. µ = , AD = = cm = 25 cm
AD µ 1.33 (d) total internal reflection.
[IIT Screening 2001 ; Karnataka CET 2003]
For concave mirror,
Sol. Since the ray suffers minimum deviation in prism
u = – 40 cm, v = – 40 cm, f = ?
P therefore the ray becomes parallel to the base of the prism.
OPTICS 445
Again, prisms Q and R are of the same shape and same (a) parabola (b) straight line
material therefore the deviation remains the same as before. (c) circle (d) hyperbola.
So, (c) is the right choice. [AIEEE 2005]
Illustration 21. Two parallel light Sol. S2P – S1P = constant P
rays are incident at one surface of a prism 30° This is the equation of a hy-
as shown in Fig. 30. The prism made of perbola with focii S1 and S2. S1
glass of refractive index 1.5. The angle
So, (d) is the right choice.
between the rays as they emerge is nearly 30° S2
(a) 19° (b) 37° Fig. 32
(c) 45° (d) 49°. Fig. 30 Illustration 25. In YDSE, an electron beam is used
[National Standard Exam. in Physics 2005] to obtain interference pattern. If speed of electron is in-
Sol. If angle of emer- creased then
gence is r, then sin r = 0.75 B r (a) no interference pattern will be observed.
giving r = 48° 36′. f (b) distance between two consecutive fringes will
Referring to the figure q A increase.
φ = 120° – 48° 36′ = 71° 24′ (c) distance between two consecutive fringes will
giving decrease.
C
θ = 180° – 2(71° 24′) (d) distance between two consecutive fringes remains
≈ 37° same. [IIT Screening 2005]
Fig. 31 Sol. De-Broglie wavelength of electron,
So, (b) is the right
choice. h
Illustration 22. In Young’s double slit experiment, λ=
mv
the fringe width is β. If the entire arrangement is placed in
When v increases, λ decreases.
a liquid of refractive index n, the fringe width becomes
β λD
(a) nβ (b) ∴ Fringe width decreases.
n+1 d
So, (c) is the right choice.
β
(c) (d) β/n. Illustration 26. In young’s double slit experiment
n−1
the angular position of a point on the central maxima whose
[Karnataka CET 2003]
intensity is one-fourth of maximum intensity is
Dλ ′ Dλ β
Sol. β′ = = = . (a) sin–1 (λ/d) (b) sin–1 (λ/2d)
d dn n
So, (d) is the right choice. (c) sin–1 (λ/3d) (d) sin–1 (λ/4d).
Illustration 23. In an interference experiment, third [IIT Screening 2005]
bright fringe is obtained at a point on the screen with a φ
Sol. I = I0 cos2
light of 700 nm. What should be the wavelenght of the light 2
source in order to obtain 5th bright fringe at the same point ? I0 φ
= I0 cos2
(a) 630 nm (b) 500 nm 4 2
(c) 420 nm (d) 750 nm φ 1
or cos =
[Karnataka CET 2003] 2 2
nDλ φ π
Sol. yn = ; nλ = constant or cos = cos
d 2 3
∴ 5 × λ = 3 × 700 2π
or φ=
3 × 700 3
or λ= nm = 420 nm
5 2π 2π
So, (c) is the right choice. Now, (d sin θ) =
λ 3
Illustration 24. A young’s double slit experiment λ
or d sin θ =
uses a monochromatic source. The shape of the interfer- 3
ence fringes formed on a screen is
446 COMPREHENSIVE OBJECTIVE PHYSICS

λ is because the area of the central dffraction band would be-


or sin θ = come one-fourth.
3d
LM λ OP So, (c) is the right choice.
or θ = sin–1
N 3d Q Illustration 30. Two point white dots are 1 mm
apart on a black paper. They are viewed by eye of pupil
So, (c) is the right choice.
diameter 3 mm. Approximately, what is the maximum
Illustration 27. When interference pattern is ob-
distance at which these dots can be resolved by the eye ?
tained by illuminating the slit of a biprism with white light,
[Take wavelength of light = 500 nm]
(a) the central band will be white.
(a) 3 m (b) 6 m
(b) the interference band next to the central band
will be red. (c) 1 m (d) 5 m. [AIEEE 2005]
(c) the interference band next to the central band will 1.22 λ x
Sol. =
be violet. a d
(d) a completely dark band can not be observed. xa
or d=
[National Standard Exam. in Physics 2005] 1.22 λ
Sol. The interference patterns due to different 1 × 10 −3 × 3 × 10 −3
component colours of white light overlap incoherently. The = m
1.22 × 500 × 10 −9
central bright fringes for different colours are at the same
≈5 m
position. So, the central fringe is white. Since violet colour
has the minimum wavelength therefore the fringe closest on So, (d) is the right choice.
either side of the central white fringe is violet ; the farthest
is red. After a few fringes, no clear fringe pattern is seen. A detailed view
The pattern of bright and dark areas is sharply de- Rayleigh’s criterion suggests that two objects are
fined only if light of a single wavelength is used. on the verge of resolvability if the central diffraction
maximum of one is at the first minimum of the other.
So, (a), (c) and (d) are correct choices.
Their angular separation must then be at least.
Illustration 28. Yellow light is used in single slit
λ
diffraction experiment with slit width 0.6 mm. If yellow θR = 1.22 (Rayleigh’s criterion)
d
light is replaced by X-rays, then the pattern will reveal that
in which d is the diameter of the operture through which
(a) no diffraction pattern light passes.
(b) that the central maxima narrower
(c) less number of fringes Illustration 31. The angular resolution of a 10 cm
diameter telescope at a wavelength of 5000 Å is of the order
(d) more number of fringes.
of :
[Karnataka CET 2003]
(a) 10–6 rad (b) 10–2 rad
Sol. For diffraction to take place, the size of the slit
(c) 10–4 rad (d) 106 rad.
should be the of order of wavelength. In the present case, the
[All India PM/PD 2005]
size of the slit is very large as compared to the wavelength of
Sol. Angular resolution,
light.
1.22 λ
So, (a) is the right choice. ∆θ =
d
Illustration 29. If I0 is the intensity of the principal
1.22 × 5000 × 10 −10
maximum in the single slit diffraction pattern, then what = rad
10 × 10 −2
will be its intensity when the slit width is doubled ?
= 6.1 × 10–6 rad ~ –6
− 10 rad
I0
(a) (b) I0 So, (a) is the right choice.
2
Illustration 32. When an upolarized light of inten-
(c) 4I0 (d) 2I0. [AIEEE 2005]
sity I0 is incident on a polarizing sheet, the intensity of the
Sol. When the slit width is doubled, the half angular light which does not get transmitted is
λ (a) I0 (b) zero
width of central maximum which is reduces to half. The
a
1 1
intensity of the central maximum will become 4 times. This (c) I (d) I. [AIEEE 2005]
4 0 2 0
OPTICS 447

Sol. 50 % of light is not transmitted. z-components are absorbed, half the intensity I0 of the
So, (d) is the right choice. original light is lost. The intensity I of the emerging

A detailed analysis of one-half rule I0


polarised light is then given by I =
2
Let us resolve the electric field oscillations of the
unpolarised light into y and z components. We can CAUTION. One-half rule can be used only when
arrange for the y-axis to be parallel to the polarising the light reading a polarising sheet is unpolarised. If
direction of the sheet. Then only the y-components of the original light is initially polarised, the transmitted
the light’s electric field are passed by the sheet ; the z- intensity depends on the angle θ between the polarisa-
components are absorbed. The sum of the y-components tion direction of the original light and the polarising
and the sum of the z-components are equal. When the direction of the sheet. I = I0 cos2 θ.

KNOWLEDGE PLUS
In each question, a statement of assertion (A) is given and a corresponding statement of reason (R) is given just below
it. Of the statements, mark the correct answer as :
(A) If both A and R are true and R is the correct explanation of A.
(B) If both A and R are true but R is not the correct explanation of A.
(C) If A is true but R is false.
(D) If both A and R are false.
(E) If A is false but R is true.
1. Assertion (A). Light waves can be polarised.
Reason (R). Light waves are transverse in nature.
2. Assertion (A). Light waves propagate in straight lines and case shadows of the obstacles.
Reason (R). Ordinarily, the diffraction of light is almost negligible.
589
3. Assertion (A). The wavelength of light coming from a sodium source is 589 nm. Its wavelength in water is nm.
1.33
Reason (R). Frequency of light remains unchanged as light travels from one medium to another medium.
4. Assertion (A). The blue light received from the sky is completely polarised.
Reason (R). When unpolarised light is scattered by small particles, the scattered light is partially polarised.
Key 1. (A) 2. (A) 3. (B) 4. (E)
448 COMPREHENSIVE OBJECTIVE PHYSICS

QUESTION BANK

MCQs
SET I

based on
Memory Work, Hard Facts, Important Terms,
Important Formulae etc.

Average time allowed per question is 20 seconds.

(a) one-fourth (b) half


PHOTOMETRY
(c) same as before (d) doubled.

1. Our eye is most sensitive for which of the following [MANIPAL 2001]
wavelength ? 7. The illumination on a screen 5 metre away from a
(a) 4500 Å (b) 5500 Å source of light is 10 lux. What is the luminous
(c) 6500 Å
intensity of the source ?
(a) 5 candela (b) 10 candela
(d) Equally sensitive for all wavelengths of visible spectrum.
(c) 50 candela (d) 250 candela.
[MANIPAL 1994]
[MANIPAL 2002]
2. Luminous intensity of a source depends upon
(a) its distance from the observation point
8. The distance between a point source of light and a
screen is made twice. The intensity of light will be
(b) colour of light emitted
(a) four times its initial value (b) twice its initial value
(c) area of surface on which light falls
(c) half of its initial value (d) one-fourth of its initial value.
(d) none of the above factors.
[MANIPAL 1995]
[Bharati Vidyapeeth 1995]
9. A point source of light moves in a straight line paral-
3. The sensitivity of eye varies with wavelength of light. lel to a plane table. Consider a small portion of the
The eye is most sensitive with ordinary intensity in table directly below the line of movement of the
the source. The illuminance at this portion varies with
(a) red light (b) blue light its distance r from the source as
(c) yellow-green light (d) red-green light. 1 1
(a) ∝ (b) ∝
[BHU 1995] r r2
4. What is the ratio of luminous intensity of two sources, 1 1
(c) ∝ (d) ∝ . [AIIMS 2000]
which produce shadows of equal intensities at dis- r3 r4
tances 25 cm and 50 cm from the photometer screen ? 10. Light from a point source falls on a screen. If the
(a) 1 : 4 (b) 4 : 1 separation between the source and the screen is
(c) 1 : 2 (d) 2 : 1. increased by 1%, the illuminance will decrease
[Himachal PMT 1996] (nearly) by
5. The surface area of a frosted electric lamp is 40 cm2. (a) 0.5% (b) 1%
Its illumination at distance of 1 m is 2 lm m–2. What (c) 2% (d) 4%. [AIIMS 2002]
is the luminous flux from the lamp ? 11. A point source of 25 candela is placed at the focus of
(a) 2 π lm (b) 4π lm a convex lens of focal length 5 m. Then the illumi-
(c) 8 π lm (d) 10 π lm . [AMU 1997] nance on a screen placed normal to the emergent
6. If we double the aperture of a photographic camera, beam of light is
the new exposure time should be
OPTICS 449
(a) 1 lux (b) 1 phot 20. The unit of luminous efficiency of electric bulb is
(c) 5 lux (d) 5 phot. (a) watt (b) lumen
[CMC LDH 1993] (c) lumen/watt (d) lux. [CMC LDH 1997]
21. A point isotropic light source emits a flux φ = 1257 lm
12. A standard 48-cd lamp placed 36 cm from the screen
in all directions. What is the intensity I of the light of
of a photometer produces the same illumination there this source ?
as a lamp of unknown intensity located 45 cm away.
(a) 60 cd (b) 100 cd
The luminous intensity of the latter lamp is
(c) 80 cd (d) 120 cd.
(a) 48 cd (b) 45 cd
22. Two stars situated at the distances of 1 and 10 light
(c) 48 × 45 cd (d) 75 cd. [AFMC 2000] years respectively from the earth appear to posses
13. The luminous intensity of a light source is 500 can- the same brightness. The ratio of their real bright-
dela. The intensity of illumination on a surface placed ness is
at a distance of 10 m when light is falling normally is (a) 1 : 10 (b) 1 : 100
(a) 1 lux (b) 2 lux (c) 10 : 1 (d) 100 : 1.
(c) 3 lux (d) 5 lux. [BHU 1998] 23. The unit of luminous intensity is
(a) watt (b) photon
14. The light from an electric bulb is incident on a surface
normally. If the surface is inclined at an angle of 60° (c) candle power (d) candela.
with this position, then illuminance will be [Pb. PMT 1993]
(a) reduced to one half (b) increased to double 24. A 50 W lamp has a luminous intensity of 25 candela.
(c) trebled (d) quadrupled. Then its efficiency is
(a) 3.14 lumen W–1 (b) 6.28 lumen W–1
[Bharati Vidyapeeth 1997]
(c) 12.56 lumen W–1 (d) 9.42 lumen W–1.
15. Inverse square law for illuminance is valid for
(a) Isotropic point source (b) Cylindrical source
[BHU 1993]
(c) Search light (d) All types of sources. 25. As the wavelength is increased from violet to red,
the luminosity
[Haryana PMT 1998]
(a) continuously increases (b) continuously decreases
16. Five lumen/watt is the luminous efficiency of a lamp (c) increases then decreases (d) decreases then increases.
and its luminous intensity is 35 candela. The power
of the lamp is [AIIMS 1994]
(a) 80 watt (b) 176 watt 26. The intensity produced by a long cylindrical light
source at a small distance r from the source is
(c) 88 watt (d) 36 watt.
proportional to
[MP PMT 1996] 1 1
(a) (b)
17. The time required for making a print at a distance of r2 r3
0.25 metre from a 60 watt lamp is 5 second. If the (c)
1
(d) none of these.
distance is increased to 40 cm, the time required in r
second to make a similar print is [AIIMS 1996]
(a) 3.1 (b) 8 27. What is the illumination at a distance of 2 m from a
(c) 12.8 (d) 16. 100 candela lamp ?
(a) 50 lux (b) 25 lux
[All India PM/PD 1994]
(c) 12.5 lux (d) none of the above.
18. A lux is equal to [AFMC 1997]
(a) 1 lumen/m2 (b) 1 lumen/cm2 28. Fig. 33 shows a glowing mercury tube. The intensities
(c) 1 candela/m2 (d) 1 candela/cm2. at points A, B and C are related as
[CPMT 1999] (a) B > C > A (b) A > C > B
19. A 400 cd lamp is separated by a distance of 1.0 m (c) B = C > A (d) B = C < A.
from a screen. At what distance behind the lamp
should a plane mirror parallel to the screen be placed
for the illuminance at the screen centre to increase
● ● ●
by 100 lux ? C A B
(a) 1 m (b) 0.5 m
Fig. 33
(c) 0.25 m (d) 0.75 m. [BHU 2001] [AIIMS 2001]
450 COMPREHENSIVE OBJECTIVE PHYSICS

29. Three light sources A, B and C emit equal amount of (a) increased by 4% (b) decreased by 4%
radiant energy per unit time. The wavelengths emit- (c) increased by 2% (d) decreased by 2%.
ted by the three sources are 450 nm, 555 nm and 700 38. An electric bulb emits 68.5 lumen W–1. Its luminous
nm respectively. The brightness sensed by an eye for efficiency is
the sources are XA, XB and XC respectively. Then,
(a) 2.5% (b) 5%
(a) XA > XB, XC > XB (b) XA > XB, XB > XC
(c) 10% (d) 20%. [MANIPAL 1997]
(c) XB > XA, XB > XC (d) XB > XA, XC > XB.
39. In a photometer, two sources of light when placed at
[AIIMS 1998]
30 cm and 50 cm respectively produce shadows of equal
30. An isotropic source produces a light flux of 6π lumen. intensities. Their candle powers are in the ratio of
What is the candela-rating of this source ?
9 16
(a) 1.5 candela (b) 2 candela (a) (b)
25 25
(c) 2.5 candela (d) 3 candela. 3 5
(c) (d) .
[Pb. PMT 1997] 5 3
31. In a cinema hall, the distance between the projector [Himachal PMT 1997]
and the screen is increased by 2.5%. The intensity of 40. An isotropic source of 2 candela produces a light flux
illumination on the screen will be decreased by equal to
(a) 2.5% (b) 5% (a) 2 π lumen (b) 4 π lumen
(c) 7.5% (d) 9.5%. (c) 6 π lumen (d) 8 π lumen.
32. In a room containing smoke particles, the intensity 41. The one parameter that determines the brightness of
due to a source of light will a light source sensed by an eye is
(a) follow the law of inverse square (a) energy of light entering the eye per second
(b) fall off faster with distance from the source than the inverse (b) wavelength of the light
square
(c) total radiant flux entering the eye
(c) increase with distance from the source than the inverse
(d) total luminous flux entering the eye. [DCE 1998]
square
(d) remain constant at all distances. 42. A book can be read if it is placed at a distance
of 50 cm from a source of 1 candela. At what distance
33. A lamp is hanging at a height of 40 cm from the
should the book be placed if the source is of 16
centre of the table. If its height is increased by 10 cm,
candela ?
the illuminance of the table will decrease by
(a) 8 m (b) 4 m
(a) 10% (b) 20%
(c) 2 m (d) 1 m. [Pb. PMT 1996]
(c) 27% (d) 36%.
43. The unit of luminous flux is
34. A point source of 36 candela is placed at the focus of
a convex lens of focal length 6 metre. Then the illu- (a) candela (b) lux
mination on a screen placed normal to the emergent (c) phot (d) lumen. [AMU 1995]
beam of light is
(a) 1 lux (b) 6 lux
REFLECTION
(c) 1/6 lux (d) 216 lux. 44. A man run towards the plane mirror at 2 m s–1. The
35. A point source of 100 candela is held 5 metre above a relative speed of his image w.r.t. him will be
sheet of blotting paper which reflects 75 per cent of (a) 2 m s–1 (b) 4 m s–1
light incident upon it. Then the illuminance on the (c) 8 m s–1 (d) 10 m s–1.
blotting paper is 45. A concave mirror has a focal length of 2 metre in
(a) 4 lux (b) 4 phot vacuum. What would be its focal length when used
(c) 3 lux (d) 3 phot. in a medium of refractive index 2.759 ?
(a) 2 m (b) 2 × 2.759 m
36. In Question Number 35, the luminance of the blot-
ting paper is 2
(c) m (d) Data is not complete.
(a) 4 lux (b) 4 phot 2.759
(c) 3 lux (d) 3 phot. 46. A child walks towards a fixed plane mirror at a speed
37. In a cinema hall, the distance between the projector of 5 km h–1. The velocity of the image is
and the screen is increased by 2%. The intensity of (a) 5 km h–1 (b) – 5 km h–1
illumination on the screen will be (c) 10 km h–1 (d) – 10 km h–1.
OPTICS 451
47. The time taken by a ray of light in travelling from 57. A dice is placed with its one edge parallel to the prin-
one point to another is cipal axis between the principal focus and the centre
(a) minimum (b) maximum of curvature of a concave mirror. The shape of the
(c) zero (d) data is inadequate. image is
(a) cube (b) cuboid
[Haryana PMT 1995]
(c) barrel (d) spherical.
48. A clock hung on a wall has marks instead of numer-
als in its dial. On the adjoining wall, there is a plane 58. A person standing in front of a mirror finds his im-
mirror and the image of the clock in the mirror indi- age larger than himself. This implies that the mirror
cates the time 4.20. Then the time on the clock is is
(a) plane (b) convex
(a) 7.40 (b) 4.20
(c) concave (d) cylindrical.
(c) 2.40 (d) 4.07.
59. The largest distance of the image from a convex mirror
49. In a plane mirror, an object is 0.5 m in front of the
mirror. The distance between object and image is of focal length 10 cm can be
(a) 0.5 m (b) 1 m (a) 20 cm (b) infinite

(c) 0.25 m (d) 0.75 m. (c) 10 cm


50. While using an electric bulb, the reflector for street (d) depends on the position of the object.
lighting should be 60. An object is approaching a plane mirror at 10 cm s–1.
(a) concave mirror (b) convex mirror
A stationary observer sees the image. At what speed
(c) cylindrical mirror (d) paraboloidal mirror.
will the image approach the stationary observer ?
51. Two plane mirrors are inclined to each other at an
(a) 10 cm s–1 (b) 5 cm s–1
angle θ. A ray of light is reflected first at one mirror
(c) 20 cm s–1 (d) 15 cm s–1.
and then at the other. The total deviation of the ray
is [CMC Vellore 1997]
(a) 2θ (b) 240° – 2θ 61. A car is fitted with a convex mirror of focal length 20
(c) 360° – 2θ (d) 180° – θ. cm. A second car 2 m broad and 1.6 m high is 6 m
52. An object 0.5 m tall is in front of a plane mirror at a away from the first car. The position of the second
distance of 0.2 m. The size of the image formed is car as seen in the mirror of the first car is
(a) 0.2 m (b) 0.5 m (a) 19.35 cm (b) 17.45 cm
(c) 0.1 m (d) 1 m. (c) 21.48 cm (d) 15.49 cm.
53. A plane mirror is approaching you at 10 cm s–1. Your
62. In the above question, the breadth and height of the
image shall approach you with a speed of
second car seen in the mirror of the first car, are
(a) + 10 cm s–1 (b) – 10 cm s–1
respectively
(c) + 20 cm s–1 (d) – 20 cm s–1.
(a) 5.79 cm, 6.9 cm (b) 6.45 cm, 5.16 cm
54. A ray of light is incident on a plane mirror at an
(c) 2.7 cm, 4.8 cm (d) 0.1 m, 0.3 m.
angle of incidence of 30°. The deviation produced by
the mirror is 63. In Q. 61, if the second car is overtaking at a relative
(a) 30° (b) 60° speed of 31 m s–1, how fast will the image be moving ?
(c) 90° (d) 120°. (a) – 1 m s–1 (b) 0.5 m s–1
[Himachal PMT 1995] (c) 0.3 m s–1 (d) – 0.032 m s–1.
55. When a convergent beam of light is incident on a [MNR 1996, modified]
plane mirror, the image formed is 64. A plane mirror reflects a pencil of light to form a real
(a) upright and real (b) upright and virtual image. Then the pencil of light incident on the mir-
(c) inverted and virtual (d) inverted and real. ror is
56. A convex mirror is used to form an image of a real (a) parallel (b) convergent
object. Which of the following is wrong ? (c) divergent (d) any of these.
(a) The image lies between the pole and the focus 65. A person is looking at the image of his face in a mirror
(b) The image is diminished in size by holding it close to his face. The image is virtual.
(c) The image is erect (d) The image is real. When he moves the mirror away from his face, the
image is inverted. What type of mirror is he using ?
452 COMPREHENSIVE OBJECTIVE PHYSICS

(a) plane mirror (b) concave mirror (a) plane or convex (b) plane or concave
(c) convex mirror (c) necessarily convex (d) necessarily concave.
(d) combination of mirrors and lenses. [National Standard Exam. in Physics 2002]
66. For a real object, a convex mirror always forms an 74. A spherical mirror forms an image of magnification
image which is 3. The object distance, if focal length of mirror is 24
(a) real and inverted (b) virtual and inverted cm, may be
(c) virtual and erect (d) real and magnified. (a) 32 cm, 24 cm (b) 32 cm, 16 cm
(c) 32 cm only (d) 16 cm only.
67. You stand with your nose 20 cm in front of a plane
mirror. Your right eye is blue and your left eye is 75. Two plane mirrors are inclined to one another at an
green. Which of the following is correct ? angle of 40°. A point object is placed in between them.
The number of images formed due to reflection at
(a) Your eye should focus at 40 cm.
both mirrors is
(b) You would see a green right eye.
(a) Infinite (b) 9
(c) You would see a blue left eye. (c) 8 (d) 6. [Haryana PMT 1996]
(d) All of the above. 76. A light ray incident normally on a plane mirror suf-
68. A convex mirror forms an image one-fourth the size fers a deviation of
of the object. If object is at a distance of 0.5 m from (a) Zero (b) π/2
the mirror, the focal length of mirror is (c) π (d) 2π.
(a) 0.17 m (b) – 1.5 m 77. A concave mirror of focal length f forms an image of
(c) 0.4 m (d) – 0.4 m. the same size as the object. The distance of the object
from the mirror is
69. A dentist has a small mirror of focal length 16 mm.
(a) f (b) f/2
He views the cavity in the tooth of a patient by hold-
ing the mirror at a distance of 8 mm from the cavity. (c) 2f (d) 4f.
The magnification is 78. The sun (diameter d) subtends an angle θ radian at
(a) 1 (b) 1.5 the pole of a concave mirror of focal length f. The
diameter of the image of sun formed by mirror is
(c) 2 (d) 3.
θ
70. A concave mirror has radius of curvature of 1 m. (a) θf (b) f
2
Light from a distant star is incident on the mirror.
θ
The distance of the image of the star from the mirror (c) 2θf (d) f .
π
is 79. A point object is placed at a distance of 30 cm from a
(a) 0.5 m (b) 1 m convex mirror of focal length 30 cm. The image will
(c) 2 m (d) 0.25 m. form at
[JIPMER 1997] (a) infinity (b) pole
71. It is necessary to illuminate the bottom of a well by (c) 15 cm behind the mirror
reflected solar beam when the light is incident at an (d) no image will be formed. [EAMCET 2002]
angle of α = 40° to the vertical. At what angle β to the 80. A candle is placed before a thick plane mirror. When
horizontal should a plane mirror be placed ? looked obliquely in the mirror a number of images
(a) 70° (b) 20° are seen from the surfaces of the plane mirror. Then
(c) 50° (d) 40°.[AMU 2003] (a) first image is brightest (b) second image is brightest
72. A small object is placed 10 cm in front of a plane (c) third image is brightest
mirror. If you stand behind the object, 30 cm from (d) all images beyond second are brighter.
the mirror and look at its image, for what distance [EAMCET 2003]
must you focus your eyes ?
(a) 20 cm (b) 60 cm 81. A boy stands in front of a plane mirror with his hand
at 0.3 m from the mirror. In order to see the image
(c) 80 cm (d) 40 cm.
of his hand clearly, he must focus his eyes at a dis-
[Karnataka CET 2001] tance of
73. A person 6 feet in height can see his full size erect (a) 0.3 m (b) 0.45 m
image in a mirror 2 feet in height. This mirror has to (c) 0.6 m (d) 0.75 m.
be
OPTICS 453
82. Which of the following cannot produce a virtual (d) Two images will be formed, one due to each exposed half.
image ? [Karnataka CET 1991, 2001]
(a) Plane mirror (b) Concave mirror 89. An endoscope is employed by a physician to view the
(c) Convex lens internal parts of a body organ. It is based on the
(d) All of the above can produce a virtual image. principle of :
83. If two mirrors are kept at 60° to each other, then the (a) refraction (b) reflection
number of images formed by them is (c) total internal reflection (d) dispersion [AIIMS 2004]
(a) 5 (b) 6 90. The critical angle for light going from medium X into
(c) 7 (d) 8. [AIEEE 2002] medium Y is θ. The speed of light in medium X is v.
84. How many images of himself does an observer see if The speed of light in medium Y is
two adjacent walls of rectangular room are mirror (a) v (1 – cos θ) (b) v/cos θ
surfaced ? (c) v cos θ (d) v/sin θ
(a) 3 (b) 5 (e) v sin θ.
(c) 7 (d) 9. 91. One surface of a lens is convex and the other is con-
85. An object is placed at a distance of 10 cm from a cave. If the radii of curvature are r1 and r2 respec-
concave mirror of radius of curvature 0.6 m. Which tively, the lens will be convex, if
of the following statements is incorrect ? (a) r1 > r2 (b) r1 = r2
(a) The image is formed at a distance of 15 cm from the mirror. (c) r1 < r2 (d) r1 = 1/r2.
(b) The image formed is real. [National Standard Exam. in Physics 1999]
(c) The image is 1.5 times the size of the object.
92. The refractive index of the material of a prism is 2
(d) The image formed is virtual and erect.
and its refracting angle is 30°. One of the refracting
REFRACTION surfaces of the prism is made a mirror inwards. A
beam of monochromatic light entering the prism from
86. Light travels through a glass plate of thickness ‘t’
the other face will retrace its path after reflection
and refractive index ‘n’. If c is the velocity of light in
vacuum, then the time taken by light to travel the from the mirrored surface if its angle of incidence on
thickness of the plate will be the prism is
(a) nt/c (b) t/nc (a) 60° (b) 0°
(c) tc/n (d) c/nt. (c) 30° (d) 45°
[MP PMT 1996, 1999, 2001] [All India PM/PD 2004]
87. Monochromatic light of wavelength λ1 travelling in 93. An object is immersed in a fluid. In order that the
medium of refractive index n1 enters a denser medium object becomes invisible, it should
of refractive index n2. The wavelength in the second (a) behave as a perfect reflector.
medium is
(b) absorb all light falling on it.
Fn I
(a) λ 1 G 1 J
Fn I
(b) λ 1 G 2 J (c) have refractive index one.
H n2 K H n1 K (d) have refractive index exactly matching with that of the
F n − n1 I
(d) λ 1 G 2
[AIIMS 2004]
H n1 JK
surrounding fluid.
(c) λ1
94. A, B and C are three optical media of respective criti-
F n − n1 I .
(e) λ 1 G 2
cal angles C1 , C2 and C3 . Total internal reflection of
H n2 JK light can occur from A to B and also from B to C but
88. How will the image formed by a convex not from C to A. Then the correct relation between
lens be affected, if the central portion of the critical angles is
the lens is wrapped in black paper, as (a) C1 < C2 < C3 (b) C3 > C1 > C2
shown in the Fig. 34 ? (c) C1 = C2 = C3 (d) C1 > C2 > C3.
(a) No image will be formed. [Karnataka CET 1997]
(b) Full image will be formed but it is less bright. 95. A converging lens has a focal length of 1 m. The
(c) Full image will be formed but without the minimum distance between a real object and its real
central portion. Fig. 34 image formed by this lens is
454 COMPREHENSIVE OBJECTIVE PHYSICS

(a) 0.5 m (b) 1 m (a) Concave, | f | = 1 m (b) Convex, | f | = 1


(c) 2 m (d) 4 m. (c) Concave, | f | = 2 m (d) Convex, | f | = 2 m.
[All India PM/PD 2000] [DCE 1994]
96. A cut diamond sparkles because of the 104. A glass concave lens is placed in a liquid in which it
(a) hardness (b) high refractive index behaves like a convergent lens. If the refractive indi-
(c) emission of light by the diamond ces of glass and liquid be aµg and aµl respectively, then
(d) absorption of light by the diamond. [CPMT 1996] (a) aµg = aµl (b) aµg < aµl
97. A ray of light travels through a transparent slab with (c) aµg > aµl (d) aµg ≥ 3 aµl .
a speed of 2 × 1010 cm s –1. This implies that the 105. The critical angle for a medium with respect to air is
refractive index of the slab material is
45°. The refractive index of that medium with respect
(a) 1.5 (b) 0.667 to air is
(c) 2.0 (d) 6.0. [EAMCET 1998]
(a) 3 /2 (b) 2/ 3
98. When a wave passes from one medium to another,
there is change in (c) 2 (d) 1/ 2 .
(a) frequency and velocity 106. It is possible to observe total internal reflection when
(b) frequency and wavelength a ray travels from
(c) wavelength and velocity (a) air into water (b) air into glass

(d) frequency, wavelength and velocity. (c) water into glass (d) glass into water.

[National Standard Exam. in Physics 1996] [EAMCET 1995]


99. A lens of large aperture is found to give a blurred 107. A mark is made on the bottom of a vessel and over
image of a point object. A black paper with a small this mark, a glass slab of thickness 3.5 cm and
hole in its centre is placed in front of the lens. Then refractive index 7/4 is placed. Now water (refractive
(a) there is no image.
index, 4/3) is poured into the vessel so that the
surface of water is 8 cm above the upper surface of
(b) the image remains unaffected.
the slab. Looking down normally through the water,
(c) the image becomes more intense but less sharp.
the apparent depth of the mark below the surface
(d) the image becomes more sharp but less intense. of water will be
100. Velocity of light in water, glass and vacuum have the (a) 6.33 cm (b) 7.5 cm
values vw , vg and vv respectively. Which of the fol- (c) 8 cm (d) 10 cm. [AFMC 1999]
lowing relations is true ?
108. Two thin lenses of focal lengths f1 and f2 are placed
(a) vw = vw = vv (b) vw > vg but vw < vv in contact with each other. The focal length of the
(c) vw = vg but < vw < vv (d) vv = vw but vw < vg . combination will be given by
101. The curved face of a plano-convex lens has a radius f1 f2
(a) (b) f1 f2
of curvature of 250 mm. The refractive index of lens f1 − f2
material is 1.5. The power of the lens is f1 f2 f + f2
(c) (d) 1 . [MAHE 1996]
(a) 0.2 D (b) – 0.2 D f1 + f2 2
(c) + 2 D (d) – 2 D. [EAMCET 1996] 109. Light of wavelength 7200 Å in air has a wavelength
102. An object 15 cm high is placed 10 cm from the optical in glass (µ = 1.5) equal to
centre of a thin lens. Its image is formed 25 cm from (a) 7200 Å (b) 7201.5 Å
the optical centre on the same side of the lens as the (c) 4800 Å [CPMT 1996]
(d) 10800 Å.
object. The height of the image is 110. A lens will be invisible in vacuum when the refrac-
(a) 2.5 cm (b) 0.2 cm tive index of the lens is
(c) 16.7 cm (d) 37.5 cm. [MP PET 1993] (a) negative (b) one
(c) more than one (d) less than one.
103. A lens forms a virtual, diminished image of an object
placed at 2 m from it. The size of image is half of the [Aligarh 1993]
object. Which one of the following statements is correct 111. When the distance between the object and the screen
regarding the nature and focal length of the lens ? is more than 4f, we can obtain image of the object on
the screen for the two positions of the lens. It is called
OPTICS 455
displacement method. In one case, the image is mag- 120. A concave lens of focal length 20 cm produces an
nified and in the other case, it is diminished. If I1 image half in size of the real object. The distance of
and I2 be the sizes of the two images, then the size of the real object is
the object is
(a) 20 cm (b) 30 cm
I1 (c) 10 cm (d) 60 cm.
(a) I1I 2 (b)
I2 121. A biconvex lens can form a virtual image if the object
I + I2 is placed
(c) I1 – I2 (d) 1 .
2 (a) between the lens and its focus
112. The sun is visible to us a little before the actual sunrise (b) at the focus of the lens (c) between f and 2 f
and a little after the actual sunset. This is because of (d) at infinity.
atmospheric 122. A convex lens of focal length 1.0 m and a concave
(a) reflection (b) refraction
lens of focal length 0.25 m are 0.75 m apart. A paral-
(c) scattering (d) diffraction. lel beam of light is incident on the convex lens. The
[National Standard Exam. in Physics 1990] beam emerging after refraction from both lenses is
113. The time taken by a ray of light in travelling from (a) parallel to principal axis (b) convergent
one point to another is (c) divergent (d) none of these.
(a) minimum (b) maximum
123. A convex spherical surface forms an inverted image
(c) practically zero (d) data not adequate.
of an object. The object is placed in air and image is
114. The number of cardinal points in a lens is formed in glass (µ = 1.5). If the object and image
(a) 8 (b) 6 distances from the spherical surface are 0.2 m and
(c) 4 (d) 2. 0.3 m respectively, the magnification is
115. The axial or longitudinal magnification of a lens is (a) 1.5 (b) 0.67
v 2 (c) 1 (d) 0.45.
v
(a) (b)
u 2
u 124. A convex lens of focal length f produces a virtual image
2 u n times the size of the object. Then the distance of
2v
(c) (d) .
u2 v the object from the lens is
116. As an object gets closer to the focal point of a (a) (n – 1) f (b) (n + 1) f
converging lens from infinity, its image
FG n − 1IJ f FG n + 1IJ f .
(a) becomes smaller (b) remains of the same size (c)
H n K (d)
H n K
(c) gets farther from the lens
125. Two thin lenses of focal lengths 20 cm and 25 cm are
(d) gets closer to the lens. [MP PET 1993]
placed in contact. The effective power of the
117. A convex lens of glass is immersed in water. Com- combination is
pared to its power in air, its power in water will (a) 45 D (b) 9 D
(a) diminish for red light and increase for blue light
1
(b) not change at all (c) D (d) 6 D.
9
(c) increase (d) diminish. [All India PM/PD 1990]
118. A convex lens of power + 6 dioptre is placed in contact 126. A plane glass slab is placed over letters of various
with a concave lens of power – 4 dioptre. What will colours, then
be the nature and focal length of this combination ? (a) green letter appears to be normal
(a) Concave, 25 cm (b) Convex, 50 cm (b) red letter appears to be raised least
(c) Concave, 20 cm (d) Convex, 100 cm. (c) violet letter appears to be raised least
[MNR 1993] (d) green letter appears to be raised least.
119. A virtual object between the optical centre and the 127. A wavefront AB moving in air is incident on a plane
focus of a concave lens produces glass surface XY, as given in Fig. 35. Its Position CD
(a) a real and erect image (b) a real and inverted image after refraction through a glass slab is shown along
(c) a virtual and erect image with normals drawn at A and D. The refractive index
(d) a virtual and inverted image. of glass w.r.t. air is given by
456 COMPREHENSIVE OBJECTIVE PHYSICS

133. When a light ray approaches a glass-air interface from


the glass side at the critical angle, the angle of
refraction is
(a) 0° (b) 45°
(c) 90°
(d) equal to the angle of incidence.
Fig. 35
sin θ sin φ 134. A beam of light propagating in medium A with index
(a) (b) of refraction n(A) passes across an interface into
sin θ′ sin θ
medium B with index of refraction n(B). The angle of
sin θ AB
(c) (d) . incidence is greater than the angle of refraction. v(A)
sin φ′ CD
and v(B) denote the speeds of light in A and B
128. If in a plano-convex lens, the radius of curvature of
respectively. Then which of the following is true ?
the convex surface is 10 cm and the focal length is 30
(a) v(A) > v(B) and n(A) > n(B)
cm, the refractive index of the material of the lens
will be (b) v(A) > v(B) and n(A) < n(B)
(a) 1.5 (b) 1.66 (c) v(A) < v(B) and n(A) > n(B)
(c) 1.33 (d) 3. [MNR 1996] (d) v(A) < v(B) and n(A) < n(B).
129. A microscope is focussed on a mark on a piece of 135. A rectangular tank of depth 8 m is full of water
paper and then a slab of glass of thickness 3 cm and (µ = 4/3). The bottom is seen at the depth
refractive index 1.5 is placed over the mark. How (a) 6 m (b) 8/3 m
should the microscope be moved to get the mark again (c) 8 cm (d) 10 cm. [MP PMT 1987]
in focus ?
136. Total internal reflection of a ray of light is possible
(a) 2 cm upward (b) 1 cm upward
when the (ic = critical angle, i = angle of incidence)
(c) 2.5 cm downward (d) 3 cm downward.
(a) ray goes from denser medium to rarer medium and i < ic .
130. If iµj represents refractive index when a light ray goes (b) ray goes from denser medium to rarer medium and
from medium i to medium j, then the product 2µ1 × i > ic .
3µ2 × 4µ3 is equal to (c) ray goes from rarer medium to denser medium and i > ic .
(a) 3µ1 (b) 3µ2 (d) ray goes from rarer medium to denser medium and i > ic .

(c)
1
(d) 4µ2.
[MP PMT 1994 ; NCERT 1997]
1µ 4 137. The length of the optical path of two media in contact
131. A light ray of frequency ν and wavelength λ enters a of lengths d1 and d2 of refractive indices µ1 and µ2
3 respectively is
liquid of refractive index . The ray travels in the
2 (a) µ1d1 + µ2d2 (b) µ1d2 + µ2d1
liquid with
FG 2 IJ λ d1d2 d1 + d2
(a) frequency ν and wavelength H 3K (c)
µ 1µ 2
(d)
µ 1µ 2
.

F 3I
(b) frequency ν and wavelength GH JK λ
138. A thin double convex lens has radii of curvature each
2 of magnitude 40 cm and is made of glass with
(c) frequency ν and wavelength λ µ = 1.65. The focal length of the lens is nearly
FG 3 IJ ν and wavelength λ. (a) 30 cm (b) 31 cm
(d) frequency
H 2K (c) 40 cm (d) 41 cm.
132. A converging lens is used to form an image on a 139. Two lenses of powers + 12 D and – 2 D are placed in
screen. When the upper half of the lens is covered by contact. What will be the focal length of combination ?
an opaque screen, (a) 10 cm (b) 12.5 cm
(a) half the image will disappear. (c) 16.6 cm (d) 8.33 cm.
(b) complete image will be formed. [MNR 1987 ; MP PET 1990]
(c) intensity of the image will increase. 140. An object is placed at a distance of 20 cm from a
(d) intensity of the image will decrease. [IIT 1986] convex lens of focal length 10 cm. The image is formed
on the other side of the lens at a distance
OPTICS 457
(a) 20 cm (b) 10 cm 148. A convex lens makes a real image 4 cm long on a
(c) 40 cm (d) 30 cm. screen. When the lens is shifted to a new position
without disturbing the object, we again get a real
141. A monochromatic beam of light passes from a denser
image on the screen which is 16 cm tall. The length
medium into a rarer medium. As a result, of the object must be
(a) its velocity increases. (b) its velocity decreases. (a) 1/4 cm (b) 8 cm
(c) its frequency decreases. (d) its wavelength decreases. (c) 12 cm (d) 20 cm. [MP PET 1991]
142. The minimum distance between an object and its real 149. A convex lens and a concave lens of 10 cm focal length
image using a convex lens of focal length f is combine. The combination lens behaves as
(a) zero (b) 2f (a) convex lens (b) concave lens
(c) 4f (d) 6f. (c) as a slab of glass (d) as convex mirror.
[National Standard Exam. in Physics 1989] 150. When an object placed before a convex lens between
143. What happens to the speed, frequency and wavelength its focal length F and infinity is displaced towards
of light when it enters glass from air ? 2F, then its image on the other side
speed frequency wavelength (a) moves from 2F towards infinity and increases in size
(b) moves from F to 2F and decreases in size
(a) decreases increases unchanged
(c) moves from 2F to F and decreases in size
(b) increases unchanged increases
(d) moves from F to 2F and increases in size.
(c) unchanged decreases decreases
(d) decreases unchanged decreases. 151. A concave lens has a focal length of 20 cm. A
convergent beam of light converges to a point 20 cm
144. Critical angle of light passing from glass to air is
behind the concave lens on the principal axis. The
minimum for image is formed at
(a) red (b) green (a) Infinity (b) 20 cms
(c) yellow (d) violet. (c) 10 cms (d) 40 cms.
145. In Fig. 36, the liquids L1 , 152. A ray of light passes through a plane glass slab of
L2 and L3 have refractive thickness t and refractive index µ. The angle between
indices 1.55, 1.50 and 1.20 the incident and emergent ray is
respectively. Therefore, (a) 0° (b) 30°
L1 L2 L3
the arrangement corre- (c) 45° (d) 90°.
sponds to 153. The critical angle for a ray of light suffering total
(a) biconvex lens internal reflection will be smallest for light travel-
(b) biconcave lens ling from
Fig. 36
(c) concavo-convex lens (a) water to air (b) glass to air
(d) convexo-concave lens (c) glass to water (d) water to glass.
[National Standard Exam. in Physics 2000] 154. A double convex air bubble in water would behave as
146. A convex lens A of focal length 20 cm and a concave a
lens B of focal length 5 cm are kept along the same (a) divergent lens (b) convergent lens
axis with the distance d between them. If a parallel (c) concave mirror (d) plane mirror.
beam of light falling on A leaves B as a parallel beam, 155. The focal length of a lens depends on
then distance d in cm will be (a) the radii of curvature of its surfaces.
(a) 25 (b) 15 (b) the refractive index of its material.
(c) 30 (d) 50. (c) the refractive index of the medium surrounding the lens.
[IIT 1985 ; MNR 1990] (d) all the above factors. [BHU 2002]
147. For a plano-convex lens made of glass (µ = 1.5), the 156. A mark at the bottom of a beaker 0.1 m deep appears
relation between focal length f, and radius of curva- raised by 0.01 m when the beaker is filled till brim
ture R is with a liquid. Refractive index of liquid is
(a) R/2 = f (b) f = R (a) 10 (b) 1/10
(c) f = 3R/2 (d) f = 2R. (c) 9/10 (d) 10/9.
458 COMPREHENSIVE OBJECTIVE PHYSICS

157. If ε0 and µ0 are respectively the electric permittivity 164. A bucket contains some transparent liquid and its
and the magnetic permeability of free space, ε and µ depth is 40 cm. On looking from above, the bottom
the corresponding quantities in a medium, the appears to be raised up by 8 cm. The refractive index
refractive index of the medium is of the liquid is
µε (a) 5/4 (b) 5
µε
(a) (b) (c) 4/5 (d) 8/5.
µ 0ε 0 µ0 ε0
165. When a monochromatic ray of light travels from a
µ0ε0 µµ 0 medium of refractive index n1 to a medium of refrac-
(c) (d) .
µε εε 0 tive index n2 (n2 > n1), its
158. The radii of curvature of the two surfaces of a lens (a) speed increases by a factor n2/n1
are 20 cm and 30 cm and the refractive index of the (b) speed decreases by a factor n2/n1
material of the lens is 1.5. If the lens is concave-convex, (c) frequency decreases by a factor n2/n1
then the focal length of the lens is (d) wavelength increases by a factor n2/n1.
(a) 24 cm (b) 10 cm
166. Two thin lenses are in contact and the focal length of
(c) 15 cm (d) 120 cm. the combination is 80 cm. If the focal length of one
159. A rod of flint glass when immersed in carbon lens is 20 cm, then the power of the other lens will be
disulphide becomes almost invisible because (a) 1.66 D (b) 4.00 D
(a) flint glass gets dissolved in carbon disulphide. (c) – 1.00 D (d) – 3.75 D. [CPMT 1991]
(b) flint glass has the same refractive index as carbon 167. In displacement method, the lengths of images in
disulphide.
two positions of lens between object and screen are
(c) of total internal reflection. 9 cm and 4 cm respectively. The length of object must
(d) of scattering of light. be
160. Using a convex lens, a clear image of a candle flame (a) 6.25 cm (b) 3/2 cm
is produced on a screen. How many other clear images (c) 6 cm (d) 36 cm. [CPMT 1993]
can be received on this screen if only the lens is to be
shifted ? 168. A concave lens is used to form an image of a real
(a) A large number (b) Only one more object. The maximum distance of the image from the
(c) Two more (d) None.
optical centre is
(a) 4 f (b) 2 f
161. A convergent beam of light converges to a point
20 cm behind a concave lens on the principal axis. (c) f (d) ∞ .
An inverted image of the same size is formed. Then 169. A real object is placed in front of a concave lens of
the focal length of the lens is focal length f, at a distance equal to f from the lens,
(a) 20 cm (b) 10 cm then the image is formed at
(c) 40 cm (d) 30 cm.
(a) infinity (b) a distance 2/f
162. A lens forms a virtual image 4 cm away from it when
(c) a distance f/2 (d) a distance 2f.
an object is placed 10 cm away from it. The lens is a
...... lens of focal length ...... 170. A ray of light passes from glass having a refractive
(a) concave, 6.67 cm (b) concave, 2.86 cm index of 1.6, to air. The angle of incidence for which
(c) convex, 2.86 cm the angle of refraction is twice the angle of incidence
(d) may be concave or convex, 6.67 cm.
is
FG 4 IJ FG 3 IJ
163. If the space between the lenses in the lens
combination shown were filled with water,
(a) sin–1
H 5K (b) sin–1
H 5K
what would happen to the focal length and FG 5 IJ FG 2 IJ .
power of the lens combination ?
(c) sin–1
H 8K (d) sin–1
H 5K
1
Focal Length Power 171. A convex lens of focal length m forms a real,
3
(a) Decreased increased
inverted image twice in size of the object. The distance
(b) Decreased unchanged
Fig. 37 of the object from the lens is
(c) Increased unchanged (a) 0.5 m (b) 0.166 m
(d) Increased decreased. (c) 0.33 m (d) 1 m.
OPTICS 459
172. An object is placed at a distance of f/2 from a convex 177. Two thin lenses of focal lengths f1 and f2 are in contact
lens of focal length f. The image will be and coaxial. The combination is equivalent to a single
(a) at one of the foci, virtual and double its size lens of power
(b) at 3f/2, real and inverted f1 f2 1
(a) (b) ( f1 + f2 )
(c) at 2f, virtual and erect (d) none of these. f1 + f2 2
[CPMT 1997] f1 + f2
(c) (d) f1 f2 .
173. Consider an equiconvex lens of radius of curvature f1 f2
R and focal length f. If f > R, the refractive index µ of 178. A convex lens forms an image of an object placed
the material of the lens 20 cm away from it at a distance of 20 cm on the
(a) is greater than zero but less than 1.5 other side of the lens. If the object is moved 5 cm
(b) is greater than 1.5 but less than 2.0
towards the lens, the image will move
(a) 5 cm towards the lens (b) 5 cm away from the lens
(c) is greater than one but less than 1.5
(c) 10 cm towards the lens (d) 10 cm away from the lens.
(d) none of these.
179. A converging lens is to project image of a lamp
174. Two convex lenses placed A B
in contact form the 4 times the size of the lamp on a wall at a distance of
image of a distant object 10 m from the lamp. The focal length of the lens is
at P. If the lens B is (a) 1.6 m (b) 2.67 m
moved to the right, the (c) 4.4 m (d) – 1.6 m.
P
image will
180. A thin lens produces an upright image of the same
(a) move to the left
size as the object. Then from the optical centre of the
(b) move to the right lens, the distance of the object is
(c) remain at P Fig. 38
(a) Zero (b) 4f
(d) move either to the left or right, depending upon focal
(c) 2f (d) f/2.
lengths of the lenses.
181. A lens behaves as a converging lens in air and a
175. A double convex thin lens made of glass (refractive
diverging lens in water. The refractive index of the
index, µ = 1.5) has both radii of curvature of magni-
material of the lens is
tude 20 cm. Incident light rays parallel to the axis of
(a) equal to unity (b) equal to 1.33
the lens will converge at a distance L such that
(c) between unity and 1.33 (d) greater than 1.33.
(a) L = 20 cm (b) L = 10 cm
[CPMT 1991]
(c) L = 40 cm (d) L = 20/3 cm. [MNR 1991]
182. A convex lens of focal length f 1 is placed in contact
176. The distance v of the real image formed by a convex
with a concave lens of focal length f2 . The combina-
lens is measured for various object distances u. A
tion will act as a convex lens if
graph is plotted between v and u. Which one of the
following graphs is correct ? (a) f1 > f2 (b) f1 = f2
(c) f1 < f2 (d) f1 > 2f2.
v
183. For a spherical surface of radius of curvature R, sepa-
rating two media of refractive indices µ1 and µ2, the
(a) (b) two principal focal lengths are f1 and f2 respectively.
Which one of the following relations is correct ?
(a) f1 = f2 (b) f2/µ2 = f1/µ1
u
(c) f2/µ2 = – f1/µ1 (d) f2/µ1 = f1/µ2.

v
v 184. Time taken, by light to pass through a water-filled
l metre long pipe when speed of light in air is c and
refractive index of water is µwater , is
(c) (d)
µ water × l × 10 −8 µ water × l × 10 −8
(a) (b)
2 3
u u
l 10 −8 l µ water
Fig. 39 (c) × (d) .
3 µ water 3 × 106
460 COMPREHENSIVE OBJECTIVE PHYSICS

185. The image of an object formed by a device is always (a) u > f > v (b) v > f < u
virtual and small. The device may be (c) less than u as well as v (d) more than u as well as v.
(a) convex lens (b) concave mirror 191. A convex lens is dipped in a liquid whose refractive
(c) a glass plate (d) concave lens. index is equal to the refractive index of the lens. Then
[CPMT 1993] its focal length will
186. A fish is vertically below a flying bird moving (a) become zero. (b) become infinite.
vertically down towards water surface. The bird will (c) become small, but non-zero.
appear to the fish to be (d) remain unchanged. [All India PM/PD 2003]
192. An equiconvex lens is cut Y
into two halves along (i)
XOX′ and (ii) YOY′ as
shown in Fig. 42 . Let f,
f ′, f ″ be the focal lengths X′ X
of the complete lens, of O
each half in case (i), and
of each half in case (ii),
respectively.
Fig. 40 Y′
Choose the correct state- Fig. 42
(a) moving faster than its real speed and also away from the ment from the follow-
real distance.
ing :
(b) moving faster than its real speed and nearer than its real
(a) f ′ = f, f ″ = 2f (b) f ′ = 2f, f ″ = f
distance.
(c) f ′ = f, f ″ = f (d) f ′ = 2f, f ″ = 2f.
(c) moving slower than its real speed and also nearer than its
real distance. [All India PM/PD 2003]
(d) moving slower than its real speed and away from the real 193. An object is placed in air at a distance of 1 m from
distance.
the curved end of a long glass rod (µ = 1.5). The image
187. A convex lens is placed in contact with a mirror as of an object placed in air is formed inside the glass
shown. If the space rod. The image formed is erect and half the size of
between them is filled
the object. The curved end of the rod is
with water, its power will
(a) convex, of radius of curvature 0.5 m.
(a) decrease
(b) concave, of radius of curvature 0.5 m.
(b) increase Fig. 41
(c) remain unchanged (c) convex, of radius of curvature 6 m.

(d) increase or decrease depending on the focal length. (d) concave, of radius of curvature 6 m.

188. A convex lens of focal length 10 cm and concave lens 194. A screen is placed at a distance 100 cm away from an
of focal length 20 cm are kept 5 cm apart. The focal illuminated object. A convex lens is placed between
length of the equivalent lens is the source and the screen and it is attempted to form
120 the image of the source on the screen. If no position
(a) cm (b) 18 m
13 could be obtained, the focal length of the lens
40 (a) must be less than 25 cm (b) must be greater than 50 cm
(c) 30 cm (d) cm.
3 (c) must be greater than 25 cm
189. Decreasing the radii of the two surfaces of a double (d) must be equal to 25 cm.
convex or double concave lens
195. In the above problem, if only one such position is
(a) increases its focal length (b) decreases its focal length obtained, then the focal length of the lens
(c) neither increases nor decreases the focal length (a) must be less than 25 cm (b) must be greater than 50 cm
(d) increases or decreases the focal length depending upon (c) must be greater than 25 cm
change.
(d) must be equal to 25 cm. [JIPMER 2001, modified]
190. A convex lens forms a real image of an object placed 196. A parallel beam of light is incident on a converging
at a distance u from the lens, at a distance v from it. lens parallel to its principal axis. As one moves away
The focal length f of the lens is from the lens on the other side on its principal axis,
the intensity of light
OPTICS 461

(a) remains constant (b) continuously increases (a) depends upon the material of the prism.
(c) continuously decreases (b) depends upon both material and angle of prism.
(d) first increases and becomes maximum at the focus then (c) depends only upon refracting angle of prism.
decreases. [CMC Vellore 1998, modified] (d) is same for all colours of white light. [MP PET 1999]
205. The black lines in the solar spectrum during solar
DISPERSION AND ABERRATIONS eclipse can be explained by
(a) Planck’s law (b) Kirchoff’’s law
197. To obtain achromatic combination of convex and
(c) Boltzmann’s law (d) Solar disturbances.
concave lens, the two lenses chosen should have
[MP PMT 1989]
(a) equal power (b) equal refractive indices
206. Fraunhoffer lines are produced by
(c) equal dispersive powers
(a) the element present in the photosphere of sun.
(d) equal product of their powers and dispersive powers.
(b) the elements present in the chromosphere of the sun.
198. An achromatic combination of lenses produces (c) the vapour of the element present in the chromosphere of
(a) highly enlarged image (b) coloured images the sun.
(c) images in black and white (d) the carbon dioxide present in the atmosphere.
(d) images unaffected by variation of refractive index with 207. A ray of light is incident normally on one of the faces
wavelength. of a prism of angle 30° and refractive index 2 . The
199. Two thin lenses of focal lengths f1 and f2 and of mate- angle of deviation of the ray is
rials with dispersive powers ω1 and ω2 when placed (a) 0° (b) 12.5°
in contact, form an achromatic combination if (c) 15° (d) 22.5°.
ω1 ω2 [Karnataka CET 2001]
(a) ω1 + ω2 = 0 (b) + =0
f1 f2 208. The dispersive power of the material of prism is given
by
(c) f1 + f2 = 0 (d) ω 1 × f1 + ω 2 × f2 = 0 .
µv + µr
(a) (b) µ vµ r
[CPMT 1995] 2
200. For a thin positive lens, fr , fv and fm are the focal µ−1 dµ
(c) (d) .
lengths for red, violet and mean colours respectively. µv − µr µ−1
Then, 209. A white ray of light is passing through a parallel glass
(a) fr = fv (b) fr < fv slab. The emergent ray
(a) undergoes dispersion only.
f
(c) fr > fv (d) fr = m . (b) undergoes deviation only.
fv
(c) undergoes both dispersion and deviation.
[National Standard Exam. in Physics 1998]
(d) neither undergoes dispersion nor deviation.
201. Light from sodium lamp is passed through sodium
vapours. The spectrum of transmitted light consists 210. When white light is allowed to pass through sodium
of vapour, the spectrum of the rays so obtained will be
(a) only continuous spectrum.
(a) a line at 5890 Å (b) a line at 5896 Å
(b) on the background of continuous spectrum few black strips.
(c) sodium doublet lines (d) no spectral features.
(c) on the background of the continuous spectrum, some black
[MP PET 1989] lines.
202. Continuous emission spectrum is obtained from (d) on the black background some more intensive yellow lines.
(a) incandescent bulb (b) carbon arc 211. When white light passes through the achromatic
(c) red hot solid (d) all of these. combination of prisms, then what is observed ?
203. The number of different colours, according to fre- (a) Only deviation (b) Only dispersion
quency in a spectrum, is (c) Deviation and dispersion (d) None of these.
(a) 5 (b) 6 [MP PMT 1989]
(c) 7 (d) infinite. 212. Colour of light having maximum speed in air is
204. The refracting angle of a prism A is small. The correct (a) red (b) yellow
statement for the dispersive power of a prism is that (c) violet (d) blue.
dispersive power
[Karnataka CET 1997]
462 COMPREHENSIVE OBJECTIVE PHYSICS

213. In any band of the band spectrum, there are (c) have different anisotropic properties while travelling
(a) Only two lines (b) No lines through the prism
(c) Infinite number of lines (d) None of these. (d) travel with different speeds. [MP PET 1993]
214. At the time of total solar eclipse, the spectrum of 222. If the refractive indices of crown glass for red, yellow
solar radiation would be and violet colours are respectively µr , µy and µv, then
(a) A large number of dark Fraunhofer lines the dispersive power of this glass would be
(b) A less number of dark Fraunhofer lines µv − µ y µv − µr
(a) (b)
(c) No lines at all µr − 1 µy − 1
(d) All Fraunhoffer lines changed into brilliant colours. µv − µ y µv − µr
[MP PMT 1990] (c) (d) −1.
µ y − µr µy
215. Solar spectrum is an example of
[MP PMT 1996]
(a) band absorption spectrum
(b) line absorption spectrum 223. Dispersive power depends upon
(a) The shape of prism (b) Material of prism
(c) continuous absorption spectrum
(c) Angle of prism (d) Height of the prism.
(d) continuous emission spectrum.
[Karnataka CET 1999] 224. Continuous spectrum is not due to
(a) Hydrogen flame (b) Electric bulb
216. A neon sign does not produce
(c) Kerosene oil lamp flame (d) Candle flame.
(a) Line spectrum (b) An emission spectrum
225. Rainbow is formed due to
(c) An absorption spectrum (d) Photons.
(a) scattering and refraction
[MP PET 1996]
(b) total internal reflection and dispersion
217. When seen in green light, the saffron and green
(c) reflection only
portions of our National Flag will appear to be
(d) diffraction and dispersion. [CBSE PMT 2000]
(a) black (b) black and green respectively
226. Two lenses have powers P1 and P2 respectively and
(c) green
their dispersive powers are ω1 and ω2 respectively.
(d) green and yellow respectively. [Manipal MEE 1995] They will together form an achromatic combination
218. Missing lines in a continuous spectrum reveal if
(a) defects of the observing instrument (a) ω1 P1 = ω2 P2 (b) ω1 P1 + ω2P2 = 0
(c) ω1 + P1 = ω2 + P2 (d) ω1 – P1 = ω2 – P2.
(b) absence of some elements in the light source
(c) presence in the light source of hot vapours of some
227. At sunrise or sunset, the sun looks more red than at
elements
mid-day because
(a) the sun is hottest at these times
(d) presence of cool vapours of some elements around the
(b) of the scattering of light
light source. [MP PET 1995]
(c) of the effects of refraction
219. Line spectrum contains information about
(d) of the effects of diffraction. [AFMC 1995]
(a) The atoms of the prism (b) The atoms of the source
228. If refractive indices for the material of the prism
(c) The molecules of the source
are µv = 1.6 and µr = 1.4, then the dispersive power
(d) The atoms as well as molecules of the source. will be
[MP PET 1995] (a) 3 (b) 1.6
220. In impure spectrum, (c) 0.4 (d) 1.
(a) order of colours is reverse 229. When white light passes through a glass prism, one
(b) order of colours is irregular gets spectrum on the other side of the prism. In the
emergent beam, the ray which is deviating least is
(c) colours are overlapped (d) no colours is present.
(a) Violet ray (b) Green ray
221. We use flint glass prism to disperse polychromatic (c) Red ray (d) Yellow ray.
light because lights of different colours
230. The spectrum of molecular form of the substance is
(a) travel with same speed called
(b) travel with same speed but deviate differently due to the (a) Band spectrum (b) Line spectrum
shape of the prism (c) Absorption spectrum (d) Continuous spectrum.
OPTICS 463
231. An achromatic combination is to be obtained using a 239. Line spectrum was first of all theoretically explained
convex and concave lens. The two lenses chosen by
should have (a) Swan (b) Fraunhoffer
(a) Their powers equal (b) Their refractive indices equal (c) Kirchoff (d) Bohr.
(c) Their dispersive powers equal 240. The width of the spectrum obtained by prism does
(d) The product of their powers and dispersive powers equal. not depend upon
[MP PMT 1998] (a) Angular dispersion (b) Material of the prism
232. An achromatic combination of lenses is formed by (c) Angle of the prism (d) Incident angle.
joining 241. Band spectrum is produced by
(a) 2 convex lenses (b) 2 concave lenses (a) H (b) He
(c) 1 convex lens and 1 concave lens (c) H2 (d) Na. [CPMT 1978]
(d) convex lens and plane mirror. [BHU 1995] 242. Out of a convex, a plano-convex and a concave lens
233. The angle of deviation is the angle between the of equal focal length, which one can obtain a sharp
(a) incident and refracted rays image of an object ?
(b) incident ray and the normal (a) Any lens (b) The convex lens
(c) refracted ray and the normal (c) Plano-convex lens (d) The concave lens.
(d) incident and the emergent rays. 243. To remove the chromatic aberration, the combina-
234. Solar spectrum at the time of total solar eclipse is tion of lenses should be such that
(a) FR + FV = 0 (b) FR > FV
(a) line emission spectrum (b) band absorption spectrum
(c) FR < FV (d) FR – FV = 0.
(c) continuous emission spectrum
(d) line absorption spectrum. [Karnataka CET 2000] [MP PET 1900]
235. The spectrum of iodine gas under white light will be 244. Let spherical aberration and chromatic aberration
(a) only violet (b) bright lines
be denoted by S and C respectively. In case of a concave
mirror
(c) only red lines
(a) S and C both may be present
(d) some black bands in continuous spectrum.
(b) both S and C do not exist.
236. A prism CANNOT be used
(a) to produce divergence of rays. (c) S may be present but C cannot exist.
(b) to produce dispersion of light. (d) C may be present but S cannot exist.
(c) to turn the direction of a ray through 90°. 245. The angle of refraction of a prism is 4.5° and its
(d) to turn the direction of a ray through 180°. refractive index is 1.52. The angle of minimum
[National Standard Exam. in Physics 2000] deviation will be
(a) 1.5° (b) 2.34°
237. If fV and fR are the focal lengths of a convex lens for
violet and red light respectively and FV and FR are (c) 4.5° (d) 2°.
the focal lengths of a concave lens for violet and red 246. In the formation of primary rainbow, the sunlight
light respectively, then we must have rays emerge at minimum deviation from rain-drop
after
(a) fV > fR and FV > FR (b) fV < fR and FV > FR
(a) one internal reflection and one refraction
(c) fV > fR and FV < FR (d) fV < fR and FV < FR.
(b) one internal reflection and two refractions
[CBSE PMT 1996] (c) two internal reflections and one refraction
238. Two lenses of powers P1 and P2 placed in contact (d) two internal reflections and two refractions.
form an achromatic doublet. The ratio of their [MP PET 1989]
dispersive powers is
247. In solar spectrum while going from violet colour to
P1 P2 red colour, for every coloured ray
(a) (b)
P2 P1 (a) deviation decreases, refractive index decreases
(b) deviation decreases, refractive index increases
P1 P2
(c) (d) . (c) deviation increases, refractive index increases
P2 P1
(d) none of the above.
464 COMPREHENSIVE OBJECTIVE PHYSICS

248. Refractive index is (c) a convex lens of smaller focal length and concave lens of
(a) directly proportional to wavelength of light large focal length.
(b) inversely proportional to wavelength of light (d) both cannot be concave.
(c) inversely proportional to square of wavelength of light 257. In order to reduce the spherical aberration in optical
(d) directly proportional to the square of wavelength of light.
instruments, one should use
(a) Plano-convex lenses (b) Concave lenses
249. Finger prints on paper are identified by sprinkling
fluorescent powder on it and observing it under (c) Spherical mirrors (d) Plane mirrors.
(a) Mercury light (b) Sun light 258. Fig. 43 represents three cases of a ray passing through
(c) Infrared light (d) Ultraviolet light. a prism of refracting edge A. The case corresponding
to minimum deviation is
[MP PMT 1989]
250. Chromatic aberration of a lens is
(a) not dependent of focal length
(b) directly proportional to focal length
(c) inversely proportional to focal length
1 2 3
(d) inversely proportional to dispersive power.
[MP PMT 1989] Fig. 43
251. Two thin lenses of the same material and mean focal (a) 1 (b) 2
lengths f1 and f2 form an achromatic combination (c) 3 (d) none of these.
when the lenses are placed co-axially at a distance
[CPMT 1989]
apart such that
259. White light is incident on thin-walled hollow glass
2 f1 f2 2 f1 f2
(a) d = (b) d = prism. The emergent light is
f1 + f2 f1 − f2
(a) White
f +f
(c) d = 1 2 (d) d = f1 – f2. (b) Coloured with violet deviated least
2
(c) Coloured with red deviated least
252. What will be the colour of sky as seen from the earth,
if there were no atmosphere ? (d) None of the above.
(a) Black (b) Blue 260. Two lenses of focal lengths f1 and f2 are available.
(c) Orange (d) Red. [MP PMT 1992] What must be the separation between them so as to
minimise spherical aberration ?
253. The dispersive power is maximum for the material
(a) f1 + f2 (b) f1 – f2
(a) Flint glass (b) Crown glass
f1 f2
(c) Mixture of both (d) None of these. (c) f1f2 (d) .
f1 + f2
254. Fraunhofer lines are obtained in
261. Refractive index, frequency and deviation are maxi-
(a) Solar spectrum
mum for
(b) The spectrum obtained from neon lamp (a) green colour (b) red colour
(c) Spectrum from a discharge tube (c) violet colour (d) black colour.
(d) None of the above. [MP PMT 1989] 262. Light is dispersed by a prism because
255. The defect in the image arising because of marginal (a) it is made of glass. (b) it is triangular.
and paraxial rays undergoing different deviations in (c) refractive index of the prism material is different for
a lens is known as different colours.
(a) distortion (b) spherical aberration (d) light is of seven colours.
(c) coma (d) chromatic aberration. 263. The spectrum of light emitted by a glowing solid is
256. Tick the wrong statement : (a) continuous spectrum. (b) line spectrum.
To obtain a convex achromatic combination of two (c) band spectrum. (d) absorption spectrum.
lenses of the same material placed at some suitable
264. By placing the prism in minimum deviation position,
distance amongst the component lenses,
images of the spectrum
(a) both may be convex.
(a) become inverted. (b) become broader.
(b) a convex lens of greater focal length and concave lens of
(c) become distinct. (d) become intensive.
smaller focal length.
OPTICS 465

265. Just before the time of sun-set, the sun appears to be 272. The refractive index of a given piece of transparent
bigger because quartz is greatest for
(a) the sun changes its shape at that time (a) Red light (b) Violet light
(b) of the scattering of light (c) of the effects of refraction (c) Green light (d) Yellow light.

(d) of the effects of diffraction. [NCERT 1971] 273. Presence of calcium in sun is estimated because
(a) absorption lines of calcium are present in solar spectrum.
266. Which of the following is true statement ?
(b) emission spectrum lines of calcium are present in solar
(a) The order of colours in the primary and the secondary spectrum.
rainbows is the same
(c) calcium is present in earth’s atmosphere.
(b) The intensity of colours in the primary and the secondary (d) calcium is present in sun rays.
rainbows is the same
274. When white light passes through a hollow prism, then
(c) The intensity of light in the primary rainbow is greater
(a) there is no dispersion and no deviation
and the order of colours is the same than the secondary
rainbow (b) dispersion but no deviation
(c) deviation but no dispersion
(d) The intensity of light for different colours in primary rain-
bow is greater and the order of colours is reverse than the (d) there is dispersion and deviation both.
secondary rainbow. [MP PMT 1987]
267. Emission spectrum of CO2 gas 275. When a prism is placed in the position of minimum
(a) is a line spectrum (b) is a band spectrum deviation, the ray of light within the prism
(c) is a continuous spectrum (a) goes parallel to the base.
(d) does not fall in the visible region. [MP PET 1992] (b) goes perpendicular to the base.
(c) makes minimum angle with the base.
268. For pure spectrum, the emergent rays should be
focussed by an achromatic lens so that (d) direction is not fixed relative to the base.

(a) intensity of the spectrum may be increased 276. The refracting angle of a prism is A and refractive
index of material of a prism is cot A/2. The angle of
(b) all light rays may be focussed at one point
minimum deviation is
(c) spherical aberrations can be removed (a) 180° – 3 A (b) 180° + 2 A
(d) same-coloured rays may be focussed. (c) 90° – A (d) 180° – 2 A.
269. In a thin prism of glass (refractive index 1.5), which 277. In the position of minimum deviation, when a ray of
of the following relations between the angle of mini- yellow light passes through the prism, then its angle
mum deviations δm and angle of refraction r will be of incidence is
correct ? (a) less than the emergent angle
(a) δm = r (b) δm = 1.5 r (b) greater than the emergent angle
r (c) sum of angle of incidence and emergent angle is 90°
(c) δm = 2r (d) δm = . [MP PMT 1990]
2 (d) equal to the emergent angle.
270. In deviation without dispersion, 278. If the refractive indices of a prism for red, yellow and
(a) all the coloured rays will be parallel to the incident ray violet colours be 1.61, 1.63 and 1.65 respectively, then
(b) all the coloured rays will be mutually parallel but not par- the dispersive power of the prism will be
allel to the incident ray 1.65 − 1.62 1.62 − 1.61
(a) (b)
(c) the direction of different coloured rays will be different 1.61 − 1 1.65 − 1
(d) all the rays of different colours will coincide together but
1.65 − 1.61 1.65 − 1.63
not parallel to the incident ray. (c) (d) .
1.63 − 1 1.61 − 1
271. When light emitted by a white hot solid is passed
[MP PET 1991]
through a sodium flame, the spectrum of the emer-
gent light will show 279. The chromatic aberration in a lens is because of
(a) the D1 and D2 bright yellow lines of sodium (a) some manufacturing defect
(b) two dark lines in the yellow region (b) the large size of the lens
(c) all colours from violet to red (c) the object being very close to the lens

(d) no colours at all. [MP PMT 1992] (d) dispersion of light in the lens material.
466 COMPREHENSIVE OBJECTIVE PHYSICS

280. The angle of minimum deviation for a prism is 40° 286. Which of the following element was discovered by
and the angle of the prism is 60°. The angle of inci- study of Fraunhoffer lines ?
dence in this position will be (a) Hydrogen (b) Oxygen
(a) 10° (b) 20° (c) Helium (d) Ozone.
(c) 50° (d) 100°. [MP PAT 1990] 287. The refractive index (µ) of a material varies with
281. Which one of the following represents correctly the wavelength (λ) as
variation of angle of deviation (δ) with angle of inci- (a) µ = Aλ + B (b) µ = A + Bλ2
dence (i) for refraction at prism ? (c) µ = A + B/λ2 (d) µ = A + B/λ4.
δ δ 288. The angle of prism is 6° and its refractive index for
green light is 1.5. If a green ray passes through it,
the deviation will be
(a) 30° (b) 15°
(a) (b)
(c) 9° (d) 3°.
289. A thin prism of glass is placed in air and water
i i successively. If aµ g = 3/2 and aµ w = 4/3, then the
O O
δ ratio of the deviations produced by the prism for a
d small angle of incidence when placed in air and water
is
(a) 9 : 8 (b) 4 : 3
(c) (d) (c) 3 : 4 (d) 4 : 1.
290. The failure of the paraxial and marginal rays pass-
O
i i ing through the same point on reflection at a spheri-
O
cal mirror or refraction through a lens is known as
Fig. 44 (a) chromatic aberration (b) spherical aberration
282. In dispersion without deviation, (c) astigmatism (d) coma.
(a) the emergent rays of all the colours are parallel to the 291. Chromatic aberration in the formation of images by
incident ray. a lens arises because
(b) yellow coloured ray is parallel to the incident ray.
(a) of non-paraxial rays
(c) only red coloured ray is parallel to the incident ray.
(b) radii of curvatures of the two sides are not the same
(d) all the rays are parallel, but not parallel to the incident
ray. (c) of the defect in grinding

283. A convex lens, a glass prism and a solid sphere all (d) the focal length varies with wavelength.
are made of the same glass. The dispersive power 292. A combination is made of two lenses of focal length f
will be and f ′ in contact. The dispersive powers of the mate-
(a) in the solid sphere and prism rial of the lenses are ω and ω′. The combination is
(b) in the lens and solid sphere achromatic when
(c) only in prism (d) in all the three. (a) ω = ω0, ω′ = 2 ω0, f ′ = 2f
[CPMT 1986] f
(b) ω = ω0, ω′ = 2 ω0, f ′ =
284. Minimum deviation is observed with a prism having 2
angle of prism = 60°, angle of deviation = 30°, angle FG f IJ
of incidence = i and angle of emergence = e. We have
(c) ω = ω0, ω′ = 2ω0, f ′ = – H 2K
(a) i = 45°, e = 30° (b) i = 30°, e = 45° (d) ω = ω0, ω′ = 2 ω0, f ′ = – 2f.
(c) i = 45°, e = 45° (d) i = 30°, e = 30°. 293. Spherical aberration in a thin lens can be reduced by
285. The angle of minimum deviation of a flint glass prism (a) using a monochromatic light
relative to crown glass prism having the same angle (b) using a doublet combination
of refraction for the same wavelength will be
(c) using a circular annular mask over the lens
(a) less (b) greater
(d) increasing the size of the lens.
(c) equal
(d) depends upon incidence angle. 294. A real image of a distant object is formed by a plano-
convex lens on its principal axis. Spherical aberration
OPTICS 467
(a) is absent 303. Longitudinal chromatic aberration for an incident
(b) is smaller if the curved surface of the lens faces the object parallel beam of a lens of unit focal length is numeri-
(c) is smaller if the plane surface of the lens faces the object cally equal to
(d) is the same whichever side of the lens faces the object. (a) its focal length (b) one
295. What is the angle of incidence for an equilateral prism (c) its dispersive power (d) infinity.
of refractive index 3 so that the ray is parallel to 304. Sodium lamps are used in foggy conditions because:
the base inside the prism ? (a) yellow light is scattered less by the fog particles.
(a) 30° (b) 45° (b) yellow light is scattered more by the fog particles.
(c) 60° (d) either 30° or 60°. (c) yellow light is unaffected during its passage through the
296. Two lenses in contact form an achromatic doublet. fog
Their focal lengths are in the ratio 2 : 3. Then their (d) wavelength of yellow light is the mean of the visible part of
dispersive powers must be in the ratio the spectrum [AIIMS 2004]
(a) 2 : 3 (b) 1 : 3
305. Two convex lenses of same focal length are made of
(c) 3 : 1 (d) 3 : 2.
crown and flint glass respectively. The axial chro-
297. A prism of refractive index µg deviates the incident matic aberration is
ray towards its base. If it is immersed in a transpar-
(a) equal for the two lenses.
ent liquid of refractive index µl such that µl > µ g ,
then the prism would (b) greater for the crown glass lens.
(a) deviate the ray towards its base (c) greater for the flint glass lens.
(b) deviate the ray away from its base (d) sometimes greater for crown glass lens and sometimes for
(c) not deviate the ray at all flint glass lens.
(d) nothing can be said.
298. A combination of a convex and a concave lens of same OPTICAL INSTRUMENTS
focal length is made. The combination behaves as
306. In a compound microscope, the intermediate image
(a) an achromatic converging lens
is
(b) an achromatic diverging lens
(a) virtual, inverted and magnified
(c) an achromatic cylindrical lens
(b) real, inverted and diminished
(d) an achromatic plane slab.
(c) virtual, erect and magnified
299. Line spectrum contains information about
(d) virtual, erect and diminished
(a) the atoms of the prism (b) the atoms of the source
(c) the molecules of the source (e) real, inverted and magnified. [Kerala PMT 2003]
(d) the atoms as well as molecules of the source. 307. The numerical value of the length of a Galileo tel-
[MP PET 1995] escope for normal vision is (assuming f o and fe as
300. When a lens forms coloured images of a point object positive length)
emitting white light, the separation between the violet (a) fo + fe (b) fo – fe
and red images on the principal axis is known as (c) fo/fe (d) fo × fe.
(a) chromatic aberration (b) axial chromatic aberration 308. The largest telescope in the world has a reflector with
(c) transverse chromatic aberration an aperture 200″, to get
(d) spherical aberration. (a) low dispersive power (b) least spherical aberration
301. The mean focal length of a lens is 15 cm. If the dis- (c) high resolving power
persive power of its material is 0.02, the axial chro- (d) high accommodation power.
matic aberration produced by the lens is
309. The resolving limit of healthy eye is about
(a) 0.0013 cm (b) 0.60 cm
(a) 1′ (b) 1″
(c) 0.30 cm (d) none of these.
1
302. For two lenses of the same material to achieve achro- (c) 1° (d) . [MP PET 1999]
60″
matism, one should put them
(a) in contact (b) a finite distance part 310. If fo and fe are the focal length of the objective and
eye-piece respectively for a telescope, its magnifying
(c) in any manner (d) nothing is certain.
power is
468 COMPREHENSIVE OBJECTIVE PHYSICS

f 318. With a simple microscope, if the lens is held at a


(a) e (b) fo × fe
fo distance d from the eye and the image is formed at
f the least distance of distinct vision D from the eye,
(c) o (d) fo × fe. then the magnifying power is
fe
D D
[Karnataka CET 2000] (a) (b) 1 +
f f
311. A reflecting telescope utilises
D–d D+d
(a) A concave mirror (b) A convex mirror (c) 1 + (d) 1 + .
f f
(c) A prism (d) A plano-convex lens.
319. For relaxed eye, the magnifying power of a micro-
312. How should people wearing their spectacles work with
scope is
a microscope ?
v D v f
(a) They should keep on wearing their spectacles. (a) – o × (b) – o × e
uo fe uo D
(b) They should take off their spectacles.
uo D uo F D I.
(c) They may either put on their spectacles or they may take
off their spectacles ; it makes no difference.
(c) ×
vo fe
(d)
vo GH
× −
fe JK
(d) They cannot use the microscope at all. [CBSE PMT 1998]
313. The magnifying power of telescope can be increased 320. An astronomical telescope of tenfold angular magni-
by fication has a length of 44 cm. The focal length of the
(a) increasing focal length of eye-piece. objective is
(b) increasing the distance of object. (a) 4 cm (b) 40 cm
(c) fitting eye-piece of low power. (c) 44 cm (d) 440 cm.
(d) fitting eye-piece of high power. [CBSE PMT 1997]
[All India PM/PD 1997] 321. When the length of a microscope tube is increased,
314. On which of the following does the magnifying power its magnifying power
of a telescope depend ? (a) decreases (b) increases
(a) The focal length of the objective only (c) does not change (d) may increase or decrease.
(b) The focal length of the objective and that of the eye-piece. [MNR 1986]
(c) The diameter of aperture of the objective only 322. The magnifying power of a simple microscope can be
increased if we use eye-piece of
(d) The diameter of aperture of the objective and that of the
eye-piece. [Bharati Vidyapeeth, 1996] (a) higher focal length (b) smaller focal length
(c) higher diameter (d) smaller diameter.
315. An observer looks at a tree of height 15 metre with a
telescope of magnifying power 10. To him, the tree 323. An astronomical telescope has objective and eye-piece
of focal lengths 200 cm and 2 cm respectively. Then
appears
the magnifying power of the telescope for normal
(a) 10 times taller (b) 10 times nearer
vision is
(c) 15 times taller (d) 15 times nearer. (a) 201.85 (b) 100
[BHU 1996] (c) 108 (d) 202.
316. A boy uses a pinhole camera to photograph his school. 324. The length of a simple astronomical telescope in nor-
An exposure time of about 30 minutes is required. In mal adjustment is equal to
order to reduce this time to less than 1 second and (a) difference of the focal lengths of the two lenses.
still get a successful photograph, he could (b) half the sum of focal lengths.
(a) make several holes around the original one (c) the sum of the focal lengths.
(b) make the hole about 1 cm in diameter (d) the product of the focal lengths.
(c) enlarge the hole and place a suitable diverging lens over it 325. With a simple microscope, if the final image is located
(d) enlarge the hole and place a suitable converging lens over at infinity, then its magnifying power is
it. (a) D/f (b) 1 + D/f
317. The final image produced by a simple microscope is (c) f/D (d) D × f.
(a) erect (b) inverted 326. For an optical instrument (like a telescope) given
(c) real and erect (d) real and inverted. magnifying power is
OPTICS 469
(a) maximum for normal adjustment. 334. The focal length of the objective of a microscope is
(b) maximum for near point adjustment. (a) greater than the focal length of the eye-piece.
(c) same for both the above cases. (b) less than the focal length of the eye-piece.
(d) none of the above. (c) equal to the focal length of the eye-piece.
327. If a Galilean telescope has objective and eye-piece of (d) none of the above. [MNR 1986]
focal lengths 200 cm and 4 cm respectively, then the
335. The resolving power of a telescope is
magnifying power of the telescope for normal vision
is (a) directly proportional to the diameter (aperture) of the ob-
jective lens and inversely proportional to the wavelength
(a) 42 (b) 50
of light used.
(c) 58 (d) 196.
(b) directly proportional to the diameter of the objective lens
328. An electron microscope is superior to an optical and also directly proportional to the wavelength of the
microscope in light used.
(a) having better resolving power (c) directly proportional to the wavelength of light used and
(b) being easy to handle (c) low cost inversely proportional to the diameter of the objective lens.
(d) quickness of observation. (d) none of the above.
329. Which of the following statements about an astro- 336. The image of a distant object as seen through an
nomical telescope is incorrect ? astronomical telescope is
(a) The objective is of large aperture. (a) erect (b) inverted
(b) The objective has less focal length than the eye-piece. (c) perverted (d) none of these.
(c) When the length of tube is minimum, the eye is most strained.
337. An endeooscope is a
(d) The difference between the focal lengths of the two lenses
(a) narrow telescope (b) type of camera
is large.
(c) simple microscope (d) photometer.
330. The distance between the eye-lens and crosswires in
Ramsden’s eye-piece where field lens has focal length 338. The magnifying power of compound microscope in
f is given by terms of the magnification mo due to objective and
magnifying power mE by the eye-piece is given by
11 12
(a) (b) f (a) mo/mE (b) mo × mE
12f 11
(c) mo + mE (d) mE/mo. [CPMT 1995]
11 1.1
(c) f (d) f. 339. You are supplied with four convex lenses of focal
12 12
lengths 100 cm, 25 cm, 3 cm and 2 cm. For design-
331. For a reading lens, we require ing an astronomical telescope, you will use lenses of
(a) short focus concave lens (b) long focus concave lens focal lengths
(c) short focus convex lens (d) long focus convex lens. (a) 100 cm and 25 cm (b) 100 cm and 3 cm
332. The image formed by the objective of a compound (c) 25 cm and 2 cm (d) 100 cm and 2 cm.
microscope is
340. The ratio of the focal lengths of the objective to the
(a) virtual and enlarged (b) virtual and diminished
focal length of the eye-piece is greater than 1 for
(c) real and diminished (d) real and enlarged.
(a) telescope (b) microscope
[AIEEE 2003]
(c) both telescope and miscroscope
333. An astronomical telescope in normal adjustment has (d) neither telescope nor microscope.
a converging eye-piece of focal length 5 cm separated
by 85 cm from the objective lens of focal length fo. 341. In order to increase the magnifying power of a
Which one of the following pairs correctly gives the microscope,
position of the image and the value of fo ? (a) the focal powers of the objective and the eye-piece should
position of image fo be large.

(a) 90 cm from the eye 80 cm (b) objective should have small focal length and the eye-piece
should have large focal length.
(b) infinity 80 cm
(c) both should have large focal lengths.
(c) infinity 90 cm
(d) the objective should have large focal length and eye-piece
(d) least distance of distinct vision 80 cm
should have small focal length.
(e) least distance of distinct vision 90 cm.
470 COMPREHENSIVE OBJECTIVE PHYSICS

342. The magnifying power of an astronomical telescope (a) 16 : 25 (b) 9 : 1


is 8 and the distance between the two lenses is 54 cm. (c) 4 : 5 (d) 5 : 4. [AIEEE 2002]
The focal lengths of the eye-lens and objective lens
will be respectively HUMAN EYE
(a) 6 cm and 48 cm (b) 48 cm and 6 cm
(c) 8 cm and 64 cm (d) 64 cm and 8 cm. 349. In myopia,
[MP PMT 1991] (a) image is formed in front of the retina.
(b) image is formed behind the retina.
343. A fly is sitting on the objective of a telescope pointed
towards the moon. What effect is expected in a pho- (c) image is formed on the retina.
tograph of the moon taken through the telescope ? (d) no image is formed. [AFMC Pune 1988]
(a) The entire field of view is blocked. 350. The normal eye is most relaxed when it is focussed
(b) There is an image of the fly on the photographs. on an object at
(c) There is no effect at all. (a) 25 cm. (b) infinite distance.
(d) There is a reduction in the intensity of the image. (c) 25 metre. (d) 25 mm away.
344. A single converging lens is held close to the eye for 351. While taking photographs with a camera, if one wants
use as a magnifying glass. For maximum magnify- that every object beyond one metre should be in focus,
ing power, the lens must be positioned so that one should
(a) the object is just beyond the principal focus of the lens. (a) give short exposure without changing aperture.
(b) the object is in the focal plane of the lens. (b) reduce the aperture and increase the time of exposure.
(c) the image is in the focal plane of the lens. (c) increase the aperture and reduce the time of exposure.
(d) the image is at the near point of the eye. (d) give long exposure without changing the aperture.
(e) the image distance is just less than the focal length of the 352. A myopic patient uses
lens. (a) convex lens (b) concave lens
345. The objective of a telescope, after focussing for infin- (c) cylindrical lens (d) bifocal lens.
ity, is taken out and a slit of length L is placed in its 353. Even in absolutely clear water, a diver cannot see
position. A sharp image of the slit is formed by the very clearly
eye-piece at a certain distance from it on the other
(a) because rays of light get diffused in water
side. The length of this image is l. Then magnifica-
(b) because velocity of light is reduced in water
tion of telescope is
(c) because the rays of light passing through water make it
l 2L turbid
(a) (b)
2L l (d) because the focal length of the eye-lens in water gets
l L changed and the image is no longer focussed sharply on
(c) (d) . the retina.
L l
346. An astronomical telescope has a large aperture to 354. Loss of the ability of eye to focus on near and far
object with advancing age is called
(a) reduce spherical aberration.
(a) presbyopia (b) astigmatism
(b) have high resolution.
(c) hypermetropia (d) myopia.
(c) increase span of observation.
355. The ability of eye to focus on both near and far objects
(d) have low dispersion. [AIEEE 2002]
is called
347. The aperture of the objective lens of a telescope is (a) myopia (b) hypermetropia
made large so as to (c) presbyopia (d) accommodation.
(a) increase the resolving power of the telescope.
356. When a person sees a distant object, the focal length
(b) increase the magnifying power of the telescope. of the eye-lens is f. When he focusses on a near ob-
(c) to focus on distant objects. ject, the focal length of the eye-lens will be
(d) make image aberration less. [KCET 2003] (a) less than f (b) more than f
348. Wavelengths of light used in an optical instrument (c) f (d) any one of these.
are λ1 = 4000 Å and λ2 = 5000 Å. Then ratio of their 357. A defect of vision in which the points in one plane of
respective resolving powers (corresponding to λ1 and an object appear in focus while those in another plane
λ2) is are out of focus is called
OPTICS 471
(a) myopia (b) hypermetropia (d) because in doing so, the focal length of the eye-lens is
(c) astigmatism (d) distortion. effectively increased.

358. A person suffering from hypermetropia uses


INTERFERENCE
(a) convex lens (b) concave lens
(c) cylindrical lens (d) bifocal lens. 368. In a double slit experiment, instead of taking slits of
359. The focal length of the lens in the human eye is maxi- equal widths, one slit is made twice as wide as the
other. Then, in the interference pattern
mum when it is looking at an object at
(a) the intensities of both the maxima and the minima
(a) infinity (b) 25 cm from the eye
increase.
(c) 100 cm from the eye
(b) the intensity of the maxima increases and the minima has
(d) a very small distance from the eye. zero intensity.
360. Astigmatism can be corrected by using (c) the intensity of the maxima decreases and that of the
(a) bifocal lenses (b) concave spherical lenses minima increases.
(d) the intensity of the maxima decreases and the minima has
(c) plano-convex lenses (d) cylindrical lenses.
zero intensity. [IIT Screening 2000]
[CBSE PMT 1990]
369. Bright colours exhibited by spider’s web, exposed to
361. The f-number of a camera lens is 4.5. This means sunlight, are due to
that the (a) resolution (b) interference
(a) focal length of the lens is 4.5. (c) polarisation (d) diffraction.
(b) reciprocal of the focal length is 4.5. [Karnataka CET 1998]
(c) ratio of focal length to the aperture is 4.5. 370. In Young’s double slit experiment, the separation
(d) aperture of the lens is 4.5. between the slits is halved and the distance between
362. An individual with one eye is likely to the slits and the screen is doubled. The fringe width
(a) have stereoscopic vision. (b) have binocular vision. will
(a) be doubled (b) be quadrupled
(c) misjudge distance. (d) none of these.
(c) remain the same (d) be halved.
363. In the human eye, the focussing is done by
[IIT 1988 ; Karnataka CET 1993, 2000]
(a) to and fro movement of the eye-lens.
371. When a thin metal plate is placed in the path of one
(b) to and fro movement of the retina.
of the interfering beams of light,
(c) change in the convexity of the lens.
(a) the fringes become brighter
(d) change in the refractive index of the eye fluids.
(b) the fringes disappear
364. For normal vision, the eye is focussed on an object at (c) the fringe width increases
(a) infinite distance (b) 25 cm away
(d) the fringes become blurred. [Karnataka CET 1999]
(c) 25 mm away (d) 25 metre away.
372. In a Young’s double slit experiment, constructive
365. A cylindrical lens is required to correct interference is produced at a certain point P. The
(a) myopia (b) presbyopia intensities of light at P due to the individual sources
(c) hypermetropia (d) astigmatism. are 4 and 9 units. The resultant intensity at point P
366. For distinct vision, the eye is focussed on an object will be
at (a) 13 units (b) 25 units
(a) infinite distance (b) 25 cm
(c) 97 units (d) 5 units.
(c) 25 mm (d) 25 m.
367. Far-sighted people, who have lost their spectacles, [National Standard Exam. in Physics 2000]
can still read a book by looking through a small 373. The maximum number of possible interference
(3 — 4 mm) hole in a sheet of a paper maxima for slit-separation equal to twice the wave-
(a) because the fine hole produces an image of the letters at a length in Young’s double-slit experiment is
longer distance.
(a) three (b) five
(b) because in doing so, the distance of the object is increased.
(c) infinite (d) zero [AIEEE 2004]
(c) because in doing so, the focal length of the eye-lens is
effectively decreased. 374. A monochromatic beam of light is used for the for-
mation of fringes on the screen by illuminating the
472 COMPREHENSIVE OBJECTIVE PHYSICS

two slits in the Young’s double-slit interference ex- (c) bright fringe is formed at the centre and violet fringes are
periment. When a thin film of mica is interposed in closer to the centre than red fringes.
the path of one of the interfering beams, then (d) fringes of same colour are clearly observed.
(a) the fringe width increases 379. Which one of the following statements must be true
(b) the fringe width decreases about two waves of monochromatic light arriving at
(c) the fringe width remains the same but the pattern shifts a point on a screen if the waves are coherent ?
(d) the fringe pattern disappears [AIIMS 2004] (a) They are in phase.

375. Coherent light is incident on two fine parallel slits S1 (b) They have a constant phase difference.
and S2 as shown in Fig. 45. If a dark fringe occurs at (c) They have both travelled paths of equal length.
P, which of the following gives possible phase differ- (d) They have approximately equal amplitudes.
ences for the light waves arriving at P from S1 and 380. A double slit arrangement produces interference
S2 ? fringes for sodium light (λ = 589 nm) that have an
angular separation of 3.50 × 10–3 radian. For what
wavelength would the angular separation be 10%
greater ?
S1 (a) 527 nm (b) 648 nm
P (c) 722 nm (d) 449 nm.
S2
381. In Young’s double slit experiment, the intensity of
central maximum is I0 . If one slit is closed, the in-
tensity at the same site is
I
(a) I0 (b) 0
Fig. 45 16
I I
(a) 2 π, 4 π , 6 π ... (b) 1/2 π, 5/2 π, 9/2 π ... (c) 0 (d) 0 . [Pb. PMT 1999]
4 2
(c) π, 3 π, 5 π ... (d) 1/2 π, 3/2 π, 5/2 π ...
382. In the pattern of interference fringes, we observe
(e) π, 2 π, 3 π ...
(a) the fringes of different width.
376. If white light is used in Young’s double slit experi- (b) the fringes of equal width.
ment,
(c) the fringes of equal width near the centre only.
(a) a very large number of coloured fringes can be seen
(d) the fringes of unequal width near the centre only.
(b) a very large number of coloured fringes with a central
383. A monochromatic beam of light is used to illuminate
white fringe can be seen
the two slits in Young’s Experiment. Fringes are seen
(c) very few coloured fringes can be seen, with first order
on the screen. When a thin glass plate is interposed
violet fringes being closer to the central white fringe
in one of the beams, then
(d) very few coloured fringes can be seen with first order red
(a) fringes width increases (b) fringe width decreases
fringes being closer to the central white fringe.
(c) fringe pattern disappears
[Karnataka CET 2001]
(d) fringe width remains the same.
377. To demonstrate the phenomenon of interference, we
[National Standard Exam. in Physics 1998]
require two sources which emit radiation
(a) of the same frequency and having a definite phase 384. Soap bubble appears coloured due to the phenom-
relationship. enon of
(b) of nearly the same frequency. (a) interference. (b) diffraction.
(c) of the same frequency. (c) dispersion. (d) reflection. [AFMC 1995]
(d) of different wavelengths. 385. I is the intensity due to a source of light at any point
[Karnataka CET 1990, 1993 ; AIEEE 2003] P on the screen. If light reaches the point P via two
different paths (a) direct (b) after reflection from a
378. In a double slit experiment, white light is used. Then
plane mirror then path difference between two paths
(a) dark and bright fringes are formed alternately.
is 3λ/2, the intensity at P is
(b) white fringe is formed at the centre and red f.ringes are (a) I (b) zero
farther to the centre than violet fringes.
(c) 2I (d) 4I.
OPTICS 473

386. Imagine a Young’s double slit experiment using elec- (a) no interference
tron waves produced by an electron gun. To increase (b) interference with only the bright bands
the width of the interference fringes, (c) interference with only the dark bands
(a) accelerating voltage in electron gun should be decreased. (d) a slight increase in the fringe width.
(b) accelerating voltage in electron gun should be increased. 394. Intensity of maximum due to interference between
(c) distance between the slits should be increased. two waves of equal amplitude is ...... times that due
(d) distance between slits and screen should be decreased. to a single wave.
387. If the intensities of the two interfering beams in (a) 2 (b) 4
Young’s double slit experiment be I1 and I2 , then the (c) 6 (d) 8.
contrast between the maximum and minimum 395. Fringe width observed in the Young’s double slit
intensity is good when experiment is β. If the frequency of the source is
(a) I1 is much greater than I2 (b) I1 is much smaller than I2 doubled, the fringe width will
(c) either I1 or I2 is zero (d) I1 = I2. (a) remain β. (b) become β/2.
388. If the Young’s double slit experiment is performed (c) become 2β. (d) become 3β/2.
with white light, then [Karnataka CET 1995]
(a) the central maximum will be white
396. In an interference experiment, the spacing between
(b) there will not be completely dark fringe successive maxima or minima is
(c) the fringe next to the central will be red λd λD
(d) the fringe next to the central will be violet. (a) (b)
D d
389. Two coherent beams of light of same wavelength su- dD λd
perpose in a certain region of space. If the intensity (c) (d) .
λ 4D
of one beam is 4 times that of the other, then the
397. In Young’s double slit experiment, the distance
ratio of intensity at a bright point to that at a dark
point is between the two slits is 0.1 mm and the wavelength
of light used is 4 × 10–7 m. If the width of the fringe
(a) 16 : 1 (b) 2 : 1
on the screen is 4 mm, then the distance between
(c) 5 : 3 (d) 9 : 1.
screen and slit is :
[National Standard Exam. in Physics 1997]
(a) 0.1 mm (b) 1 cm
390. Two interfering beams of monochromatic light have
(c) 0.1 cm (d) 1 m
intensities in the ratio of 16 : 9. The ratio of the maxi-
(e) 10 m.
mum to the minimum intensity of fringe system
formed on the screen is 398. Monochromatic green light of wavelength 5 × 10–7 m
(a) 7 : 1 (b) 25 : 7 illuminates a pair of slits 1 mm apart. The separation
(c) 256 : 81 (d) 49 : 1.
of nearest bright fringes in the interference pattern
on a screen 2 m away is
391. In double slit experiment, for light of which colour,
(a) 0.25 mm (b) 0.5 mm
the fringe width will be minimum ?
(c) 0. 1 mm (d) 1.0 mm.
(a) Violet (b) Red
(c) Green (d) Yellow. [MP PMT 1994]
[Karnataka CET 1992]

392. Which of the following decides about the contrast 399. In Young’s double slit experiment, yellow light is
between bright and dark fringes in an interference replaced by red light of the same intensity. The fringe
experiment ? width shall
(a) wavelength (a) increase (b) decrease

(b) distance between two coherent sources (c) remain unchanged (d) become zero.

(c) fringe width (d) intensity ratio. 400. If the distance between the slits is halved and distance
between the slits and screen is also halved, then the
393. Interference is observed in a chamber containing air.
The experiment is performed now after evacuating fringe width will be
the chamber. On careful observation, we shall now (a) same (b) doubled
see (c) four times (d) halved.
474 COMPREHENSIVE OBJECTIVE PHYSICS

401. Young’s experiment establishes that With which two waves is interference possible ?
(a) light consists of waves. (b) light consists of particles. (a) In (i) and (iii) (b) In (i) and (iv)
(c) light consists of neither particles nor waves. (c) In (iii) and (iv)
(d) light consists of both particles and waves. (d) Not possible with any combination.
[MP PET 1994] 409. If two waves of same frequency and same amplitude
402. Interference fringes are obtained using two coherent respectively, on superposition, produce a resultant
sources whose intensities are in the ratio 4 : 1. Then wave of the same amplitude, the waves differ in phase
the ratio of the intensities of the bright and dark by
bands is (a) π/3 (b) 2π/3
(a) 4 : 1 (b) 5 : 1 (c) π/4 (d) zero.
(c) 9 : 1 (d) 25 : 9. 410. In Young’s experiment, the ratio of maximum to
[Karnataka 1990] minimum intensities of the fringe system is 4 : 1.
The amplitudes of the coherent sources are in the
403. When monochromatic light is replaced by white light
ratio
in Fresnel’s biprism arrangement, the central fringe
(a) 4 : 1 (b) 3 : 1
is
(a) dark (b) white (c) 2 : 1 (d) 1 : 1. [MP PET 2000]
(c) coloured (d) none of these. 411. The displacements of interfering light waves are
y1 = 4 sin ωt and y2 = 3 sin (ωt + π/2). What is the
404. If yellow light emitted by sodium lamp in Young’s
amplitude of the resultant wave ?
double slit experiment is replaced by monochromatic
blue light of the same intensity (a) 5 (b) 7
(a) fringe width will decrease. (c) 1 (d) zero.
(b) fringe width will increase. [Rajasthan PMT 1996]
(c) fringe width will remain unchanged. 412. If the source of light used in a Young’s Double Slit
(d) fringes will become less intense. [CBSE PMT 1992] Experiment is changed from red to blue, then
(a) the fringes will become brighter.
405. A double slit arrangement produces fringes for light
of λ = 5890 Å which are 0.2° apart. If the whole (b) consecutive fringes will come closer.

4 (c) the number of maxima formed on the screen increases.


arrangement is fully dipped in a liquid of µ = , the (d) the central bright fringe will become a dark fringe.
3
angular fringes separation is 413. Which of following is conserved when light waves
(a) 0.15° (b) 0.30° interfere ?
(c) 0.10° (d) 0.25° (a) Intensity (b) Energy
406. Two waves y1 = A1 sin (ωt – β1) and y2 = A2 sin (c) Amplitude (d) Momentum.
(ωt – β2) superimpose to form a resultant wave whose [Haryana CEET 1998]
amplitude is 414. In the set up shown in
(a) A 12 + A 22 + 2A 1 A 2 cos (β1 − β2 ) Fig. 46, the two slits S1 and
S2 are not equidistant from S1
(b) A 12 + A 22 + 2A 1 A 2 sin (β 1 − β2 ) the slit S. The central fringe S O
at O is then
(c) A1 + A2 (d) | A1 + A2 |.
(a) always bright
[CMPMT 1999] S2
(b) always dark
407. In a wave, the path difference corresponding to a (c) either dark or bright depend-
phase difference of φ is ing on the position of S
Fig. 46
π π (d) neither dark nor bright.
(a) φ (b) φ
2λ λ 415. Two waves originating from S1 and S2 having
λ λ zero phase difference and common wavelength λ
(c) φ (d) φ . [MP PET 2000]
2π π will show completely destructure interference at P if
408. Four different independent waves are represented by (S1 P – S2P) is
(i) y1 = a1 sin ωt (ii) y2 = a2 sin 2ωt (a) 5 λ (b) 0.75 λ
(iii) y3 = a3 cos 3ωt (iv) y4 = a4 sin 5 (ωt + π/3) (c) 0.6 λ (d) 5.5 λ.
OPTICS 475

416. If Young’s interference experiment be performed in The linear separation of fringes on a screen placed in
water instead of air, then the focal plane of the lens is
(a) fringes will be smaller in number. (a) 3 mm (b) 4 mm
(b) fringes will be broader. (c) fringes will be narrower. (c) 2 mm (d) 1 mm.
[National Standard Exam. in Physics 1991]
(d) no fringes will be obtained.
423. In Young’s double slit experiment, the two slits act
417. Two light waves having the same wavelength λ in
as coherent sources of equal amplitude A and of
vacuum are in phase initially. Then the first ray
wavelength λ. In another experiment with the same
travels a path of length L1 through a medium of
set-up, the two slits are sources of equal amplitude A
refractive index µ1. The second ray travels a path of
and wavelength λ, but are incoherent. The ratio of
length L2 through a medium of refractive index µ2.
intensity of light at the mid-point of the screen in the
The two waves are then combined to observe
first case to that in the second case is
interference effects. The phase difference between the
(a) 1 : 1 (b) 1 : 2
two, when they interfere, is
(c) 2 : 1 (d) 4 : 1.
2π 2π
(a) (L 1 − L 2 ) (b) (µ 1L 1 − µ 2L 2 )
λ λ 424. In a double slit experiment, interference is obtained

(c)

(µ 2L 1 − µ 1L 2 ) (d)
LM
2π L 1 L 2
− .
OP from electron waves produced in an electron gun
supplied with voltage V. If λ is wavelength of the beam,
λ N
λ µ 1 µ2 Q D is the distance of screen, d is the spacing between
418. Young’s double slit experiment gives interference by coherent sources, h is Planck’s constant, e is charge
division of on electron and m is mass of electron, then fringe
(a) wavelength (b) frequency width is given as
(c) phase (d) wavefront. hD 2hD
419. The light from a sodium vapour lamp passes through (a) (b)
2 meV d meV d
a single narrow slit and then through two close par-
hd 2 hd
allel narrow slits. The single slit lies on the perpen- (c) (d) .
2 meV D meV D
dicular bisector of the line joining the two close par-
allel slits. An observer looking towards the lamp 425. The path difference between two interfering waves
through the double slit will see corresponding to a phase difference π is
(a) a continuous yellow band (b) a continuous black band (a) λ (b) 3λ
(c) alternate black and yellow bands (c) λ/2 (d) λ/4. [JAT (Assam) 1999]
(d) a coloured spectrum. 426. In Young’s double slit interference experiment, if the
420. If torch is used in place of monochromatic light in slit separation is made 3 folds, the fringe width
Young’s experiment, what will happen ? becomes
(a) Fringe will occur as from monochromatic source. (a) 6 fold (b) 3 fold
(b) Fringe will appear for a moment and then it will disappear. (c) 3/6 fold (d) 1/3 fold. [CPMT 1985]
(c) No fringes will appear. 427. A thin film of air between a plane glass plate and a
(d) Only bright fringe will appear. convex lens is irradiated with parallel beam of mono-
chromatic light and is observed under a microscope.
421. The width of one of the two slits in a Young’s double
You will see
slit experiment is 2 times that of the other. Assume
(a) field crossed over by concentric bright and dark rings.
that the amplitude of wave coming from a slit is
proportional to the slit width. The ratio of maximum (b) field crossed over by parallel bright and dark fringes.
to minimum intensity in the interference pattern will (c) complete darkness. (d) uniform brightness.
be 428. Two light sources are said to be coherent if they are
(a) 2 (b) 3 obtained from
(c) 6 (d) 9. (a) two independent point sources emitting light of the same
[National Standard Exam. in Physics 1991] wavelength
422. Light of wavelength λ = 5890 Å falls on a double slit (b) a single point source (c) a wide source
arrangement having separation d = 0.2 mm. A thin (d) two ordinary bulbs emitting light of different wavelengths.
lens of focal length f = 1 m is placed near the slits. [MP PET 1996]
476 COMPREHENSIVE OBJECTIVE PHYSICS

429. Two coherent sources of light produce destructive (c) cannot be obtained by any of these sources
interference when phase difference between them is (d) since contrast suffers when sources 3 and 4 are used so
(a) 2π (b) π coherence cannot be obtained by using sources 3 and 4.
(c) π/2 (d) 0. [MP PET 1996] 438. In Young’s double slit experiment, the band width of
fringe is independent of
430. In a biprism experiment, if the equal angles of the
two component prisms are slightly increased, the (a) distance between the coherent sources
fringes pattern will (b) distance of the screen from the sources
(a) remain unchanged (b) shrink (c) wavelength of light emitted by the sources
(c) gets enlarged (d) remains unchanged. (d) order of the fringe.
431. In Young’s double slit experiment, if one of the slit is 439. As a result of interference of two coherent sources of
closed fully, then in the interference pattern light, energy is
(a) a bright slit will be observed, no interference pattern will (a) redistributed and the distribution does not very with time
exist (b) increased
(b) the bright fringes will become more bright (c) redistributed and the distribution changes with time
(c) the bright fringes will become fainter (d) decreased. [KCET 2003]
(d) none of the above. 440. The fringe width in Young’s double slit experiment
432. For constructive interference to take place between can be increased if we decrease
two monochromatic light waves of wavelength λ, the (a) the separation of the slits (b) width of the slits
path difference should be (c) distance between slit and screen
λ λ (d) wavelength of light.
(a) (2n – 1) (b) (2n – 1)
4 2 441. Two sources of light are said to be coherent if waves
λ produced by them have the same
(c) nλ (d) (2n + 1) .
2 (a) wavelength. (b) amplitude.
433. If L is the coherence length and c the velocity of light,
(c) frequency and constant phase difference.
then the coherent time is
(d) amplitude and same wavelength.
L
(a) cL (b)
c 442. The Young’s experiment is first performed with
c 1 sodium light and then replaced with neon light of
(c) (d) . [MP PMT 1996] the same intensity. Which of the following effects will
L Lc
434. Interference can be observed with the help of a be noticed ?
(a) spectrometer (b) biprism (a) The fringes will disappear.
(c) photometer (d) prism. (b) The fringes will become less distinct.
435. In Young’s double slit experiment, the intensity of (c) The fringe width will increase.
light at a point on the screen where the path differ- (d) The fringe width will decrease.
ence is λ = I. The intensity of light at a point where 443. Two powerful 100 W bulbs are used to study the in-
the path difference becomes λ/3 is terference of light. How will interference pattern be
I I affected ?
(a) (b)
4 3 (a) Fringes will become narrow.
I
(c) (d) I. (b) Fringes will become broader.
2
(c) It would vanish.
436. In a biprism experiment, when the set up is shifted
from air to the inside of a still, clear lake water, the (d) Pattern will be coloured green- yellow.
fringe pattern 444. If one of the two slits of a Young’s double slit experi-
(a) disappears (b) gets shrunk ment is painted so that it transmits half the light
(c) gets enlarged (d) remains unchanged. intensity as the second slit, then
437. Sources 1 and 2 emit lights of different wavelengths (a) the fringe system will altogether disappear
whereas 3 and 4 emit lights of different intensities, (b) the bright lines will become brighter and the dark fringes
the coherence will become darker
(a) can be obtained by using sources 1 and 2 (c) both dark and bright fringes will become darker
(b) can be obtained by using sources 3 and 4 (d) dark fringes will become brighter and bright fringes darker.
OPTICS 477
445. In a Young’s double slit experiment, a thin film is (a) dispersion (b) diffraction
introduced in the path of one interfering ray. Now, (c) interference (d) refraction [AIIMS 2004]
the fringe width shall
452. Pick out the correct statements
(a) decrease (b) increase
(a) diffraction is exhibited by all electromagnetic waves but
(c) remain unchanged (d) become zero. not by mechanical waves
446. Young’s double slit experiment is being performed in (b) diffraction cannot be observed with a plane polarised light
air. The fringe width is β. What shall be the fringe (c) the limit of resolution of a microscope decreases with
width if the whole apparatus is immersed in a liquid increase in the wavelength of light used
of refractive index µ ? (d) the width of central maximum in the diffraction pattern
(a) β (b) µβ due to single slit increases as wavelength increases.
β [National Standard Exam.n in Physics 1997]
(c) (d) 0.
µ 453. A lens of focal length f gives diffraction pattern of
Fraunhofer type of a slit having width a. If wave-
DIFFRACTION length of light is λ , the distance of first dark band
and next bright band from axis is given by
447. Yellow light is used in a single slit diffraction experi-
ment with slit width of 0.6 mm. If yellow light is a λ
(a) f (b) f
replaced by X-rays, then the observed pattern will λ a
reveal λ
(a) that the central maximum is narrower. (c) (d) aλf. [MNR 1995]
af
(b) more number of fringes.
454. If white light is used in diffraction at a single slit, the
(c) less number of fringes.
central maximum will be
(d) no diffraction pattern. [IIT Screening 1999]
(a) white (b) coloured
448. In a spectrometer experiment, monochromatic light (c) black (d) none of these.
is incident normally on a diffraction grating having
4.5 × 105 lines per metre. The second order line is 455. At the first minima, the path difference between two
seen at an angle of 30° to the normal. What is the waves starting from the two ends of a slit in a single
wavelength of the light ? slit Fraunhofer diffraction experiment is
(a) 200 nm (b) 556 nm λ
(a) (b) λ
(c) 430 nm (d) 589 nm 2
(e) 500 nm. 3
(c) (d) 2λ. [CMC LDH 2000]
449. A diffraction grating is ruled with 600 lines per mil- 2λ
limetre. When monochromatic light falls normally
456. The penetration of light into the region of geometrical
on the grating, the first-order diffracted beams are
shadow is called
observed on the far side of the grating each making
(a) polarisation (b) interference
an angle of 15° with the normal to the grating.
What is the frequency of the light ? (c) diffraction (d) refraction. [CPMT 1999]
(a) 1.2 × 1013 Hz (b) 4.7 × 1013 Hz 457. Bending of light about corner of the obstacle’s sharp
(c) 1.9 × 1014 Hz (d) 3.6 × 1014 Hz edge is called
(e) 7.0 × 1014 Hz. (a) deviation (b) dispersion
450. A beam of light of wavelength 600 nm from a distant (c) polarisation (d) diffraction.
source falls on a single slit 1.00 mm wide and the 458. The conditions for obtaining Fraunhofer diffraction
resulting diffraction pattern is observed on a screen from a single slit is that the light wavefront incident
2 m away. The distance between the first dark fringes on the slit should be
on either side of the central bright fringe is (a) spherical (b) cylindrical
(a) 1.2 cm (b) 1.2 mm (c) elliptical (d) planar.
(c) 2.4 cm (d) 2.4 mm. [Pb. PMT 1998] 459. For producing a broad central maximum by diffrac-
451. When a compact disc is illuminated by a source of tion from a single slit,
while light, coloured ‘lanes’ are observed. This is due (a) slit must be narrow (b) slit must be broad
to (c) source must be narrow (d) sources must be broad.
478 COMPREHENSIVE OBJECTIVE PHYSICS

460. To observe diffraction, the size of an obstacle (c) sound is perceived by the ear
(a) should be of the same order as wavelength (d) sound waves are longitudinal.
(b) should be much larger than the wavelength [Karnataka CET 1995, 2000]
(c) has no relation to wavelength 467. The wavelength of light can be experimentally found
using
(d) should be exactly λ/2. [CPMT 1982]
(a) ripple tank (b) diffraction grating
461. While both light and sound show wave character, (c) plane mirror (d) glass prism.
diffraction (bending round corners) is much harder
[Karnataka CET 1989, 1992]
to observe in light. This is because
468. In the experiment of diffraction at a single slit, if the
(a) speed of light is far greater slit width is decreased, the width of the central maxi-
(b) wavelength of light is far smaller mum
(c) light does not require a medium (a) decreases in Fresnel diffraction but increases in
Fraunhofer diffraction.
(d) waves of light are transverse.
(b) increases in Fresnel diffraction but decreases in
[National Standard Exam. in Physics 1992] Fraunhofer diffraction.
462. For a single slit of width d, the first diffraction mini- (c) decreases in both Fresnel and Fraunhofer diffraction.
mum using light of wavelength λ will occur at an (d) increases in both Fresnel and Fraunhofer diffraction .
angle of [Karnataka CET 2001]
FG λ IJ FG λ IJ 469. The frequency of light can be experimentally found
(a) sin–1
H 2d K (b) sin–1
H dK using
FG d IJ FG 2d IJ . (a) ripple tank (b) diffraction grating
(c) sin–1
HλK (d) sin–1
HλK (c) plane mirror (d) glass prism.
463. Direction of the first secondary maximum in the [Karnataka CET 1988]
Fraunhoffer diffraction pattern at a single slit is given
470. Fig. 47 represents the intensity distribution produced
by (a is the width of the slit)
on a screen by the diffraction of light.
λ 3λ
(a) a sin θ = (b) a cos θ =
2 2
Intensity


(c) a sin θ = λ (d) a sin θ = .
2
[Karnataka 1999]
464. An opaque object 10 cm wide casts a shadow when
placed in a beam of light but has little effect on a
beam of sound emitted by a small source of frequency
500 Hz. This is because
(a) sound is a pressure wave whereas light is an electromag-
netic wave O
distance across screen
(b) sound travels much more slowly than light
Fig. 47
(c) sound waves are longitudinal whereas light waves are
transverse This can be achieved by using
(d) sound waves have a much longer wavelength than light (a) one single wavelength and a single slit
waves. (b) one single wavelength and a double slit
465. The phenomenon of diffraction of light was discov- (c) two separate wavelengths and a single slit
ered by (d) two separate wavelengths and a double slit
(a) Garibaldi (b) Fresnel (e) two separate wavelengths and a diffraction grating.
(c) Newton (d) Huygens. 471. A parallel beam of wavelength λ = 450 × 10–9 m passes
[Karnataka CET 1997, 2000] through a long slit of width 2 × 10–4 m. The angular
466. Diffraction effects are easier to notice in the case of divergence for which most of light is diffracted is
sound waves than in the case of light waves because (a) 5.4 × 10–4 rad (b) 4.5 × 10–3 rad
(a) sound waves are of longer wavelength (c) 3.5 × 10 rad –3
(d) 2.8 × 10–4 rad.
(b) sound waves are mechanical waves [JIPMER 1995]
OPTICS 479
472. Light of wavelength λ is incident on a slit of width d. 479. Polarisation of light takes place due to many proc-
The resulting diffraction pattern is observed on a esses. Which of the following will not cause polarisa-
screen at a distance D. The linear width of the prin- tion ?
cipal maxima is then equal to the width of the slit. D (a) Reflection (b) Double refraction
equals (c) Scattering (d) Diffraction
d 3λ (e) Absorption [Kerala PMT 2003]
(a) (b)
2λ d 480. A ray of light is incident on a glass plate at an angle
d2 2λ2 of 60°. What is the refractive index of glass if the
(c) (d) .
2λ 7d reflected and refracted rays are perpendicular to each
[All India PM/PD 2002] other ?
473. Diffraction pattern is obtained using a beam of blue (a) tan 60° (b) cot 60°
light. What happens if red light is used in place of (c) sin 60° (d) cos 60°.
blue light ? [CMC Vellore 1999]
(a) Bands become broader and farther apart. 481. Two polaroids are kept crossed to each other. Now
(b) Bands become narrower and crowded. one of them is rotated through an angle of 45°. The
(c) No change. (d) Bands disappear. percentage of incident light now transmitted through
474. Given : width of aperture = 3 mm and λ = 500 nm. the system is
For what distance ray optics is good approximation ? (a) 15% (b) 25%
(a) 18 m (b) 18 nm (c) 50% (d) 60%
(c) 18 Å (d) 18 light years. (e) 75%. [Kerala PMT 2003]
475. In diffraction experiment with light waves, the size 482. A polariser reduces the intensity of an unpolarised
of obstacle in path should be of the order of light by
(a) 1 mm (b) 0.1 mm (a) 80% (b) 60%
(c) 10 mm
–4
(d) 1 cm. (c) 50% (d) 40%.
476. If in the experiment of diffraction at a single slit, the 483. Consider the following statements A and B and iden-
width of slit is equal to twice the wavelength of light tify the correct answer.
used, the angle of diffraction for the first minimum A. Polarised light can be used to study the helical
will be surface of nucleic acids.
(a) 30° (b) 45° B. Optic axis is a direction and not any particular
(c) 60° (d) 90°. line in the crystal.
477. When a monochromatic beam of light is passed (a) A and B are correct
through a plane transmission grating at normal (b) A and B are wrong
incidence, the position of the direct image is obtained (c) A is correct but B is wrong
at D. The diffracted images A (d) A is wrong but B is correct. [EAMCET 2003]
and B correspond to first and 484. Unpolarised light is incident on a dielectric of refrac-
second order diffraction. What
tive index 3 . What is the angle of incidence if the
shall happen if the source is D A B reflected beam is completely polarised ?
replaced by a source of longer
wavelength ? Fig. 48 (a) 30° (b) 45°
(a) A and B shall shift away from D. (c) 60° (d) 75°.

(b) A and B shall shift towards D. 485. A polaroid is placed at 45° to an incoming light of
intensity I0 . Now the intensity of light passing
(c) All the three shall shift to the right.
through polaroid after polarisation would be
(d) All the three shall shift to the left.
(a) I0 (b) I0/2
POLARISATION (c) I0/4 (d) zero.
486. A polaroid is being used as an analyser of plane
478. A Nicol prism can be used polarised light. In one complete rotation of the crystal,
(a) only to analyse polarised light the maximum intensities will be observed only
(b) for producing and analysing polarised light (a) once (b) twice
(c) only to produce polarised light (c) thrice (d) data is inadequate.
(d) none of the above. [AlI India PM/PD 1994]
480 COMPREHENSIVE OBJECTIVE PHYSICS

487. The angle of incidence at which reflected light is to- 495. A beam of light AO is in- A B
tally polarised for reflection from air to glass (refrac- cident on a glass slab (µ =
tive index n), is 1.54) in the direction
O
(a) tan–1 (1/n) (b) sin–1 (1/n)
shown. The reflected ray 33° 33°
OB is passed through a Glass slab
(c) sin–1 (n) (d) tan–1 (n) [AIEEE 2004] Nicol prism. On viewing
Fig. 49
488. Unpolarised light converts to partially or plane pola- through a Nicol prism, we
rised light by many processes. Which of the follow- find on rotating the prism that
ing does not do that ? (a) the intensity is reduced down to zero and remains zero
(a) reflection (b) diffraction (b) the intensity reduces down somewhat and rises again
(c) double refraction (d) scattering. (c) there is no change in intensity
(d) the intensity gradually reduces to zero and then again
[National Standard Exam. in Physics 1992]
increases.
489. If the light is polarised by reflection, then the angle 496. A ray of unpolarised light is incident on a glass plate
between reflected and refracted light is at the polarising angle of incidence 57°. Now,
(a) π (b) π/2 (a) both the reflected and transmitted rays will be completely
(c) 2π (d) π/4. polarised
490. If a wave can be polarised, it must be (b) the reflected ray will be completely polarised and
transmitted ray will be partially polarised
(a) an electromagnetic wave (b) a longitudinal wave
(c) the reflected ray will be partially polarised and transmitted
(c) a sound wave (d) a stationary wave ray will be completely polarised
(e) a transverse wave. (d) both the reflected and the transmitted rays will be partially
491. A ray of light falls on a transparent glass plate. Part polarised.
of it is reflected and part is refracted. The reflected 497. Which effect provides direct experimental evidence
and refracted rays can be perpendicular to each other that light is a transverse, rather than a longitudinal
for wave motion ?
(a) no angle of incidence (a) Light can be diffracted.
(b) Two coherent light waves can be made to interfere.
(b) angle of incidence equal to 90°
(c) The intensity of light from a point source falls off inversely
(c) more than one angle of incidence
as the square of the distance from the source.
(d) only one angle of incidence. [CMC Vellore 2003] (d) Light can be polarised.
492. When light is incident from air to glass at an angle 498. Polaroid sheets are often used for making sun glasses.
57°, the reflected beam is completely polarised. If the This is because polaroid glasses
same beam is incident from water to glass, the angle (a) cut off glare
of incidence at which reflected beam is completely (b) absorb more light than coloured glasses
polarised will be (c) are light-weight
(a) θ = 57° (b) θ > 57° (d) remove the polarisation of direct sunlight.
(c) θ < 57° (d) cannot be determined. [Karnataka CET 1989]
[National Standard Exam. in Physics 2001] 499. θ is the polarising angle for two optical media, whose
critical angles are C1 and C2. Then the correct relation
493. Waves that cannot be polarised are
is
(a) electromagnetic waves (b) light waves
sin C2 sin C2
(c) longitudinal waves (d) transverse waves. (a) sin θ = (b) θ =
sin C1 sin C1
[KCET 2003] sin C1 sin C1
(c) tan θ = (d) sin θ = .
494. The phenomenon of rotation of plane polarised light sin C2 sin C2
is called [Karnataka CET 1997]
(a) Kerr effect (b) double refraction 500. If a wave can be polarised, it must be
(a) an electromagnetic wave (b) a longitudinal wave
(c) optical activity (d) dichroism. [KCET 2003]
(c) a progressive wave (d) a stationary wave
(e) a transverse wave.
OPTICS 481

501. When light incident on a refracting surface is polar- (a) stationary (b) moving towards earth
ised by reflection, then angle between the refracted (c) moving away from earth (d) Information is incomplete.
and reflected rays is 509. When there is a relative motion of an observer from
(a) 45° (b) 48° a source of light, the apparent change in its wave-
(c) 90° (d) 98°. length is termed as
[Karnataka CET 1984] (a) Raman effect (b) Seebeck effect
(c) Doppler’s effect (d) Gravitational effect.
502. The correct expression relating polarising angle and
510. If the context of Doppler effect in light, the term red
refractive index is
shift signifies
(a) µ sin ip = 1 (b) µ cot ip = 1
(a) decrease in frequency (b) increase in frequency
(c) µ tan ip = 1 (d) µ cosec ip = 1.
(c) decrease in intensity (d) increase in intensity.
[Karnataka CET 1984]
503. If the speed of light and the polarising angle for a MISCELLANEOUS
given medium are cm and i respectively, then from
511. When light waves travel from one medium into an-
Brewster’s law we find
other, its
(a) cm = cosec i (b) cm = cosh i (a) velocity does not change
(c) cm = cos i (d) cm = c × cot i. (b) wavelength does not change
[Karnataka CET 1985] (c) frequency does not change
504. The experimental proof that light is a transverse wave (d) velocity, frequency and wavelength do not change
may be obtained through (e) velocity, frequency and wavelength changes

(a) demonstration of polarisation phenomenon


[Kerala PMT 2003]
512. Which of the following is correct ?
(b) the photoelectric effect
(a) Light and sound are both transverse waves.
(c) demonstration of interference phenomena
(b) Light of coherent sources passing through two slits causes
(d) the Doppler effect. [Karnataka CET 1988] interference.
505. The planes of vibration and polarisation are (c) Interference of light is evidence for transverse character
of light.
(a) orthogonal (b) parallel
(d) Rainbow appears because of scattering of light.
(c) non-existent (d) orthogonal and parallel.
[Karnataka CET 1996]
[Karnataka CET 1984, 86] 513. Which one of the following waves cannot be polar-
ised ?
DOPPLER EFFECT (a) Radio waves (b) X-rays
(c) Sound waves (d) Light waves.
506. A star is going away from the earth. An observer on
the earth will see the wavelength of light coming from [Karnataka CET 1992]
the star 514. Scattering will be more for the light whose
(a) decreased (b) increased (a) wavelength is more (b) wavelength is less
(c) it is equal for all wavelengths
(c) neither decreased nor increased
(d) scattering does not depend on wavelength.
(d) decreased or increased depending upon the velocity of the
515. Waves that cannot be polarised are
star. [MP PMT 1999]
(a) electromagnetic waves (b) light waves
507. Red shift is an illustration of (c) longitudinal waves (d) transverse waves.
(a) low temperature emission [Karnataka CET 1984, 2000]
(b) high frequency absorption 516. One important similarity between sound and light
(c) doppler effect waves is that both
(d) unknown phenomenon. (a) can pass through even in the absence of any medium
[National Standard Exam. in Physics 1999] (b) are transverse waves
(c) travel at the same speed in air
508. If the shift of wavelength of light emitted by a star is
(d) can show interference effects.
towards violet, then this shows that star is
[Karnataka CET 1989]
482 COMPREHENSIVE OBJECTIVE PHYSICS

517. Which one of the following waves is longitudinal 526. One cannot see through fog because
wave ? (a) fog absorbs light
(a) Radio wave (b) X-ray (b) light is scattered by the droplets in fog
(c) Sound wave (d) Light wave. (c) light suffers total reflection at the droplets in fog
[Karnataka CET 1989] (d) the refractive index of fog is infinity. [AFMC 2002]
518. Lights of different colours propagates through vacuum 527. Intensity of scattered light is
(a) with the velocity of air (b) with different velocities (a) equal to the wavelength.
(b) proportional to the square of wavelength.
(c) with the velocity of sound
(c) proportional to the fourth power of wavelength.
(d) having the equal velocities.
(d) inversely proportional to the fourth power of wavelength.
519. The ratio of the refractive index of red light to blue
528. Which of the following methods can be used to meas-
light in air is
ure the speed of light in laboratory ?
(a) less than unity (b) equal to unity
(a) Roemer method (b) Fizeau method
(c) greater than unity (c) Focault method (d) Michelson method.
(d) less as well as greater than unity depending upon the 529. Which of the following characters of light show in-
experimental arrangement. [CPMT 1978] terference and diffraction ?
520. θ is the optical rotation produced by sugar solution (a) transverse nature (b) wave nature
of concentration C and taken in a glass tube of length (c) particle nature (d) ray nature.
0.2 m and radius r. Then the solution is transferred 530. The locus of all the points in the same state of vibra-
to another glass tube of length 0.3 m and radius r tion in the same phase according to wave theory is
and the remaining space is filled with distilled wa- known as
ter. The optical rotation produced by this solution is (a) a wavefront (b) a half wave zone
(a) 3θ (b) 2 θ (c) a full wave zone (d) nothing can be said.
(c) θ (d) 1.5 θ. 531. On reflection from a denser to a rarer medium, a
[Karnataka CET 1999] wave suffers a path difference of
521. Newton postulated his corpuscular theory on the basis (a) λ/2 (b) λ/4
of (c) λ (d) zero.
(a) dispersion of white light 532. It is possible to distinguish between transverse and
(b) rectilinear propagation of light longitudinal waves by studying the property of
(c) colours of thin films (d) Newton’s rings. (a) interference (b) diffraction
[Karnataka CET 2001] (c) reflection (d) polarisation.
522. Optically active substance among the following is 533. A cylindrical wavefront is spreading out from a line
(a) Sodium chloride (b) Quartz source such as a long narrow slit. When the wavefront
is at a distance r from the source, the amplitude is
(c) Tourmaline (d) Calcite.
proportional to
[Karnataka CET 1997]
1 1
523. Laser is a device to produce (a) (b)
r r
(a) a beam of white light (b) coherent light
1 1
(c) microwaves (d) X-rays. (c) 2 (d) 3 / 2 .
r r
[Karnataka CET 1989]
[All India PM/PD 2001]
524. For a certain light, there are 2 × 103 waves in 1.5
534. If there were no atmosphere, then the length of the
mm in air. The wavelength of light is day on the earth will
(a) 75 Å (b) 750 Å (a) decrease (b) increase
(c) 750 nm (d) 75 × 10–7 m. (c) remain the same (d) depend upon the weather.
525. In Question no. 524, the frequency of the light is [AFMC 2003]
(a) 4 × 105 Hz (b) 4 × 1016 Hz 535. Red colour seems red because
(c) 4 × 1017 Hz (d) 4 × 1014 Hz. (a) it is coated red (b) absorbs red colour
[CMC Vellore 2002, modified] (c) scatters red colour (d) reflects red colour.
OPTICS 483
536. Wavelength of laser beam can be used as a standard 545. The velocity of light was first determined by
of (a) Gallileo (b) Newton
(a) angle (b) time (c) Roemer (d) Michelson.
(c) length (d) temperature.
546. Light passes through a closed cylindrical tube con-
537. Which of the following statements indicates that light taining a gas. If the gas is gradually pumped out, the
waves are transverse ? speed of light inside the tube will
(a) Light waves can travel in vacuum.
(a) increase (b) decrease
(b) Light waves show interference.
(c) remain constant
(c) Light waves can be polarsed.
(d) first increase and then decrease. [BHU 1994]
(d) Light waves can be diffracted.
547. Signals of danger are made red because
[MP PMT 1995 ; AFMC 1996]
(a) our eye is most sensitive for red colour
538. Which of the following phenomena is not explained
(b) scattering is minimum for red colour
by Huygens’ construction of wavefront ?
(c) scattering is maximum for red colour
(a) Refraction (b) Reflection
(d) red colour is internationally accepted colour for danger.
(c) Diffraction (d) Origin of spectra.
548. A book looks red when seen through a piece of red
539. A calcite crystal is placed over a dot on a piece of
glass. Then the cover must be of
paper and rotated. On seeing through the calcite, one
(a) Red colour (b) White
will see
(c) Green (d) Red or white.
(a) one dot
[CPMT 1987]
(b) two stationary dots
549. If a cloth of red colour is seen in the light of mercury
(c) two rotating dots
vapour lamp, it will appear
(d) one dot rotating about the other.
(a) Black (b) Red
540. µ (refractive index) depends upon (c) Yellow (d) Blue. [CPMT 1986]
(a) angle of prism (b) wavelength of light
550. Crossed lenses are
(c) intensity of light (d) amplitude of light
(a) two lenses mutually at right angles so as to form a cross
(e) all of the above. (b) a convex and a concave lens placed together
541. Which of the following is not common to light and (c) specially designed pair of lenses, their radii of curvature
sound waves ? are such that rays passing through every zone, undergo
(a) interference (b) diffraction same amount of deviation
(c) coherence (d) polarisation. (d) two cylindrical lenses. [AIIMS 1997]
542. Beams of red light and green light are sent on the 551. A ray is incident at an angle of incidence i on one
same site of the screen. Now, the screen will appear surface of prism of small angle A and emerges
(a) yellow (b) violet normally from the opposite surface. If the refractive
(c) green (d) black. index of the material of the prism is µ, the angle of
543. A wavefront AB passing A D incidence is nearly equal to
through a system C (a) µA (b) A/µ
C
emerges as DE. The system (c) µA/2 (d) A/2µ.
?
C could be [All India PM/PD 1992]
E
(a) a slit B
Fig. 50 552. Light takes 8 minute 20 second to reach from sun on
(b) a biprism the earth. If the whole atmosphere is filled with wa-
(c) a prism (d) a glass slab. ter, the light will take the time (aµw = 4/3)
[National Standard Exam. in Physics 1990] (a) 8 minute 20 second (b) 8 minute
544. The wavefronts of light coming from a distant source (c) 6 minute 11 second (d) 11 minute 6 second.
of unknown shape are nearly 553. Ray optics is valid, when characteristic dimensions
(a) plane (b) elliptical are
(c) cylindrical (d) spherical. (a) of the order of one millimeter
[JIPMER 1993] (b) much larger than the wavelength of light
484 COMPREHENSIVE OBJECTIVE PHYSICS

(c) much smaller than the wavelength of light 562. According to corpuscular theory of light, the differ-
(d) of the same order as the wavelength of light. ent colours of light are due to
[All India PM/PD 1994] (a) different electromagnetic waves
554. To an observer on the earth, the stars appear to (b) different force of attraction among the corpuscles
twinkle. This can be ascribed to (c) different size of the corpuscles
(a) the fact that stars do not emit light continuously (d) none of the above.
(b) frequent absorption of star light by their own atmosphere
563. The equation of a light wave is written as y = A sin
(c) frequent absorption of star light by the earth’s atmosphere
(kx – ωt). Here, y represents
(d) the refractive index fluctuations in the earth’s atmosphere.
(a) displacement of ether particles
[Himachal PMT 1998]
(b) pressure in the medium (c) density of the medium
555. Blue colour of sea water is due to
(d) electric field. [AIIMS 1995]
(a) Reflection of light and scattering of light from water particles
564. Fig. 51 shows a wavefront P passing through two
(b) Bottom of sea is blue
systems A and B, and emerging as Q and then as R.
(c) Large depth of sea water
The systems A and B could, respectively, be
(d) Sea water is saltish.
556. Though quantum theory of light can explain a
number of phenomena observed with light, it is nec-
essary to retain the wave nature of light to explain
the phenomenon of R

(a) photoelectric effect (b) diffraction


(c) Compton effect (d) black body radiation.
P Q B
557. The frequency of light having a wavelength 3000 Å is A
(a) 90 Hz (b) 3000 Hz
(c) 9 × 1013 Hz (d) 1015 Hz. Fig. 51
[Pb. PMT 2002] (a) a prism and a convergent lens.
558. Huygens’ conception of secondary waves (b) a convergent lens and a prism.
(a) allows us to find the focal length of a thick lens. (c) a divergent lens and a prism.
(b) is a geometrical method to find a wavefront. (d) a convergent lens and a divergent lens.
(c) is used to determine the velocity of light. [National Standard Exam. in Physics 1991]
(d) is used to explain polarisation. 565. When light is transmitted from air to glass, its
559. Speed of light is maximum in (a) wavelength increases. (b) wavelength decreases.
(a) Water (b) Air (c) frequency increases. (d) there happens no change.
(c) Glass (d) Diamond. [IIT 1980]
[CPMT 1990 ; MP PMT 1994 ; AFMC 1996] 566. Optically active substances are those substances
560. A ray of light of frequency ν in air enters into glass of which
refractive index µ . The correct statement is (a) produce polarised light
(a) frequency of light in glass will change
(b) rotate the plane of polarisation of polarised light
(b) both frequency of light and its wavelength in glass will
(c) produce double refraction
change
(d) convert a plane polarised light into circularly polarised
(c) frequency, wavelength and intensity of light all will change
light.
in glass
(d) frequency of light in glass will not change. 567. Light has the following wave property
[MP PET 1999] (a) sometimes longitudinal, sometimes transverse
(b) neither transverse nor longitudinal
561. The velocity of light will be minimum in
(a) Vacuum (b) Air (c) longitudinal (d) transverse.
(c) Water (d) Glass. 568. The idea of the quantum nature of light has emerged
in an attempt to explain
OPTICS 485
(a) Interference (b) Diffraction 575. In the previous question, the ratio of the speeds of
(c) Radiation spectrum of a black body light is
(d) Polarisation. (a) 1 (b) n
569. Evidence for the wave nature of light cannot be 1 1
(c) (d) .
obtained from n n2
(a) reflection (b) doppler effect 576. Who gave wave theory of light ?
(c) interference (d) diffraction. (a) Newton (b) Huygens
[MP PET 1996] (c) Fermat (d) Maxwell.
570. The astronaut in a space ship sees the sky away from 577. Which of the following cannot be explained by wave
the sun as theory of light ?
(a) Red (b) Black (a) Total internal reflection (b) Refraction
(c) White (d) Blue. (c) Diffraction (d) Photoelectric effect.
571. The speed of light depends 578. Air has µ = 1.0003. The thickness of air column, which
(a) on elasticity of the medium only will have one more wavelength of yellow light (6000
(b) on inertia of the medium only Å) than in the same thickness of vacuum is
(c) on elasticity as well as inertia (a) 2 mm (b) 2 cm
(d) neither on elasticity nor on inertia. (c) 2 m (d) 2 km.
572. Which of the following statements is wrong ? 579. The wavefronts of light wave travelling in vacuum
(a) Light travels faster in vacuum than air. are given by x + y + z = c. The angle made by the
(b) The wavelength of light is longer than the wavelength of light ray with the x-axis is
sound. (a) 0° (b) 45°
(c) Sound travels neary 330 metre in one second.
F 1I.
(d) Speed of sound is ‘Mach number one’. (c) 90° (d) cos–1 GH 3 JK
573. The wavelength is shortest for
(a) violet (b) blue 580. In a film projector, a convex lens is usually placed
(c) yellow (d) red. between the bulb and the film. The purpose of this
574. Light is refracted from air into a medium of refrac- lens is to
tive index n. The ratio of the wavelengths of the inci- (a) make the image brighter
dent and refracted waves is (b) laterally invert the image
(a) 1 (b) n (c) produce an image of the film on the screen
1 1 (d) correct any distortions in the image.
(c) (d) .
n n2

Answers (Set I)
1. (b) 2. (b) 3. (c) 4. (a) 5. (c) 6. (a) 7. (d) 8. (d)
9. (c) 10. (c) 11. (a) 12. (d) 13. (d) 14. (a) 15. (a) 16. (c)
17. (c) 18. (a) 19. (b) 20. (c) 21. (b) 22. (b) 23. (d) 24. (b)
25. (c) 26. (c) 27. (b) 28. (d) 29. (c) 30. (a) 31. (b) 32. (b)
33. (d) 34. (a) 35. (a) 36. (c) 37. (b) 38. (c) 39. (a) 40. (d)
41. (d) 42. (c) 43. (d) 44. (b) 45. (a) 46. (b) 47. (a) 48. (a)
49. (b) 50. (b) 51. (c) 52. (b) 53. (c) 54. (d) 55. (a) 56. (d)
57. (b) 58. (c) 59. (c) 60. (a) 61. (a) 62. (b) 63. (d) 64. (b)
65. (b) 66. (c) 67. (d) 68. (a) 69. (c) 70. (a) 71. (a) 72. (d)
73. (c) 74. (b) 75. (c) 76. (c) 77. (c) 78. (a) 79. (c) 80. (b)
486 COMPREHENSIVE OBJECTIVE PHYSICS

81. (c) 82. (d) 83. (a) 84. (a) 85. (b) 86. (a) 87. (a) 88. (b)
89. (c) 90. (d) 91. (c) 92. (d) 93. (d) 94. (a) 95. (d) 96. (b)
97. (a) 98. (c) 99. (d) 100. (b) 101. (c) 102. (d) 103. (c) 104. (b)
105. (c) 106. (d) 107. (c) 108. (c) 109. (c) 110. (b) 111. (a) 112. (b)
113. (a) 114. (b) 115. (b) 116. (c) 117. (d) 118. (b) 119. (a) 120. (a)
121. (a) 122. (a) 123. (c) 124. (c) 125. (b) 126. (b) 127. (c) 128. (c)
129. (b) 130. (c) 131. (a) 132. (d) 133. (c) 134. (b) 135. (a) 136. (b)
137. (a) 138. (b) 139. (a) 140. (a) 141. (a) 142. (c) 143. (d) 144. (d)
145. (c) 146. (b) 147. (d) 148. (b) 149. (c) 150. (d) 151. (a) 152. (a)
153. (b) 154. (a) 155. (d) 156. (d) 157. (a) 158. (d) 159. (b) 160. (b)
161. (b) 162. (a) 163. (d) 164. (a) 165. (b) 166. (d) 167. (c) 168. (c)
169. (c) 170. (b) 171. (a) 172. (a) 173. (c) 174. (b) 175. (a) 176. (d)
177. (c) 178. (d) 179. (a) 180. (a) 181. (c) 182. (c) 183. (c) 184. (b)
185. (d) 186. (a) 187. (a) 188. (d) 189. (b) 190. (c) 191. (b) 192. (a)
193. (a) 194. (c) 195. (d) 196. (d) 197. (d) 198. (d) 199. (b) 200. (c)
201. (d) 202. (d) 203. (d) 204. (a) 205. (b) 206. (c) 207. (c) 208. (d)
209. (d) 210. (c) 211. (a) 212. (a) 213. (c) 214. (d) 215. (b) 216. (c)
217. (b) 218. (d) 219. (b) 220. (c) 221. (c) 222. (b) 223. (b) 224. (a)
225. (b) 226. (b) 227. (b) 228. (c) 229. (c) 230. (a) 231. (d) 232. (c)
233. (d) 234. (a) 235. (d) 236. (a) 237. (b) 238. (b) 239. (d) 240. (d)
241. (c) 242. (c) 243. (d) 244. (c) 245. (b) 246. (b) 247. (a) 248. (c)
249. (d) 250. (b) 251. (c) 252. (a) 253. (a) 254. (a) 255. (b) 256. (b)
257. (a) 258. (c) 259. (a) 260. (b) 261. (c) 262. (c) 263. (a) 264. (c)
265. (c) 266. (d) 267. (b) 268. (d) 269. (a) 270. (d) 271. (b) 272. (b)
273. (a) 274. (a) 275. (a) 276. (d) 277. (d) 278. (c) 279. (d) 280. (c)
281. (d) 282. (b) 283. (d) 284. (c) 285. (b) 286. (c) 287. (c) 288. (d)
289. (d) 290. (b) 291. (d) 292. (d) 293. (c) 294. (b) 295. (c) 296. (a)
297. (b) 298. (d) 299. (b) 300. (b) 301. (c) 302. (b) 303. (c) 304. (a)
305. (c) 306. (e) 307. (b) 308. (c) 309. (a) 310. (c) 311. (a) 312. (b)
313. (d) 314. (b) 315. (b) 316. (d) 317. (a) 318. (c) 319. (a) 320. (b)
321. (a) 322. (b) 323. (b) 324. (c) 325. (a) 326. (b) 327. (b) 328. (a)
329. (b) 330. (c) 331. (c) 332. (d) 333. (b) 334. (b) 335. (a) 336. (b)
337. (a) 338. (b) 339. (d) 340. (a) 341. (a) 342. (a) 343. (d) 344. (d)
345. (d) 346. (b) 347. (a) 348. (d) 349. (a) 350. (b) 351. (b) 352. (b)
353. (d) 354. (a) 355. (d) 356. (a) 357. (c) 358. (a) 359. (a) 360. (d)
361. (c) 362. (c) 363. (c) 364. (a) 365. (d) 366. (b) 367. (c) 368. (a)
369. (b) 370. (b) 371. (d) 372. (b) 373. (b) 374. (c) 375. (c) 376. (c)
377. (a) 378. (b) 379. (b) 380. (b) 381. (c) 382. (b) 383. (d) 384. (a)
385. (b) 386. (a) 387. (d) 388. (d) 389. (d) 390. (d) 391. (a) 392. (d)
393. (d) 394. (b) 395. (b) 396. (b) 397. (d) 398. (d) 399. (a) 400. (a)
401. (a) 402. (c) 403. (b) 404. (a) 405. (a) 406. (a) 407. (c) 408. (d)
OPTICS 487

409. (b) 410. (b) 411. (a) 412. (c) 413. (b) 414. (c) 415. (d) 416. (c)
417. (b) 418. (d) 419. (c) 420. (c) 421. (d) 422. (a) 423. (c) 424. (a)
425. (c) 426. (d) 427. (a) 428. (b) 429. (b) 430. (b) 431. (a) 432. (c)
433. (b) 434. (b) 435. (a) 436. (b) 437. (b) 438. (d) 439. (a) 440. (a)
441. (c) 442. (c) 443. (c) 444. (d) 445. (c) 446. (c) 447. (a) 448. (b)
449. (e) 450. (d) 451. (b) 452. (d) 453. (b) 454. (a) 455. (b) 456. (c)
457. (d) 458. (d) 459. (a) 460. (a) 461. (b) 462. (b) 463. (d) 464. (d)
465. (a) 466. (a) 467. (b) 468. (d) 469. (b) 470. (a) 471. (b) 472. (c)
473. (a) 474. (a) 475. (c) 476. (a) 477. (a) 478. (b) 479. (d) 480. (a)
481. (b) 482. (c) 483. (a) 484. (c) 485. (b) 486. (b) 487. (d) 488. (b)
489. (b) 490. (e) 491. (d) 492. (c) 493. (c) 494. (c) 495. (d) 496. (b)
497. (d) 498. (a) 499. (c) 500. (e) 501. (c) 502. (b) 503. (d) 504. (a)
505. (a) 506. (b) 507. (c) 508. (b) 509. (c) 510. (a) 511. (c) 512. (b)
513. (c) 514. (b) 515. (c) 516. (d) 517. (c) 518. (d) 519. (a) 520. (c)
521. (b) 522. (b) 523. (b) 524. (c) 525. (d) 526. (b) 527. (d) 528. (c)
529. (b) 530. (a) 531. (a) 532. (d) 533. (a) 534. (a) 535. (d) 536. (c)
537. (c) 538. (d) 539. (d) 540. (b) 541. (d) 542. (a) 543. (c) 544. (a)
545. (c) 546. (a) 547. (b) 548. (d) 549. (b) 550. (c) 551. (c) 552. (d)
553. (b) 554. (d) 555. (a) 556. (b) 557. (d) 558. (b) 559. (b) 560. (d)
561. (d) 562. (c) 563. (d) 564. (b) 565. (b) 566. (b) 567. (d) 568. (c)
569. (a) 570. (b) 571. (d) 572. (b) 573. (a) 574. (b) 575. (b) 576. (b)
577. (d) 578. (a) 579. (d) 580. (a).

Solutions (Set I)
I1 r12 25FG IJ1
2 1
4. = 2=
I2 r2 50 H K
=
4
10. E∝
r2
5. φ = 4πI = 4π × 2 lm = 8π lm. 25
11. E= = 1 lux.
6. When the aperture is doubled, the light-gathering 52
power becomes four times. So, the time of exposure
becomes one-fourth. I1 FG
45 cm IJ 2

7. Luminous intensity in candela


12.
48
=
H
36 cm K
= Illumination in lux × square of distance I 500
= 10 × 5 × 5 13. E= 2
= = 5 lux
r (10)2
= 250 candela. 1
14. E′ = E cos θ and cos 60° =
8. Intensity of light is inversely proportional to square 2
of distance. luminous flux
16. Power =
I cos θ Ih luminous efficiency
9. E= E=
r2 r3 4 πI 4 π × 35
r h = =
θ
luminous efficiency 5
1
or E∝ 3 22
r = 4× × 7 = 88 watt.
7
Fig. 52
488 COMPREHENSIVE OBJECTIVE PHYSICS

I I 31. Intensity of illumination is inversely proportional to


17. t1 = 2 t2
r12 r2 square of distance.
32. The scattering of light reduces intensity at a faster
r22
t1 = 40 FG IJ 2
F 8I
5 = G J
2
64 rate than demanded by inverse square law of distance.
or t2 =
r12 25 H K H 5K 5=
5
s = 12.8 s
I′ 40 × 40 16
19. In the first case, 33. = =
I 50 × 50 25
400
E== 400 lux I′ 16 9
12 1– = 1− =
I 25 25
In the second case, an additional 100 lux is required.
If d is the distance of the plane mirror from the lamp, then I − I′ 9
or × 100 = × 100 = 36.
the image of the lamp would be at a distance of (2x + 1) I 25
metre
I 36
400 1 1 34. E= = lux = 1 lux
Now, 100 = or = f2 6×6
(2 x + 1)2 4 (2 x + 1)2
100
or 2x + 1 = 2 or 2x = 1 35. E= cos 0° = 4 lux
5×5
or x = 0.5 m.
36. Luminance
21. 1 cd = 4π lumen = 12.57 lumen
= fraction of light reflected × illuminance
∴ 1257 lumen = 100 cd
22. Think in terms of inverse square distance. 75
= × 4 lux
100
24. Luminous efficiency
= 3 lux.
Luminous flux
= 1
Power 37. E∝ .
4 πI 4 × 3.14 × 25 r2
= = = 6.28 lm W–1
Power 50 38. Note that 685 lumen/watt corresponds to 100%
25. The variation of relative luminosity with wavelength efficiency. So, 68.5 lumen/watt corresponds to 10%
is shown here. efficiency.
39. When the screen is equally illuminated,
1.0 E1 = E2
I1 I2
or =
r12 r22

I1 r12 30 × 30 9
or = 2 = =
luminosity

I2 r2 50 × 50 25
Relative

40. 1 candela produces a flux of 4π lumen.


1 16
42. =
400 700 502 d2
Wavelength (in nm)
or d2 = 502 × 16
Fig. 53 or d = 50 × 4 cm
100 = 200 cm = 2 m.
27. Illumination = lux = 25 lux.
2×2 44. Relative speed of image is actual speed of image plus
28. E ∝ cos θ. the reversed velocity of the object.
29. Refer to the relative luminosity curve. 45. The focal length of the mirror is independent of the
30. 4πx lumen is produced by x candela. So, 6π lumen medium in which the mirror is placed.
x 46. Velocity is a vector quantity. So, proper sign need be
will be produced by × 6π i.e. 1.5 candela.
4 πx used.
OPTICS 489

47. According to Fermat’s principle of least time, the light 57. Lateral magnification is different from axial magnifi-
takes minimum time to travel from one point to an- cation.
other even after a number of reflections or refractions. 60.
48. In a plane mirror, the image undergoes depth inver-
sion. So, the time in the image clock is as much ahead
of 12 hours zero minute as the time on the actual O O¢ I¢ I
clock is behind 12 hours zero minute. So, time on 10 cm 10 cm
actual clock is 12 h 0 min – 4h 20 min x y
Observer
= 7h 40 min.
x + y – 10
49. Note that the image shall be formed at a distance of Fig. 57
0.5 m from the mirror.
As is clear from Fig. 57 the distance of image with
50. Note that convex mirror is used as reflector for street
reference to observer reduces by 10 cm in one second.
lighting but concave mirror is used as reflector in
1 1 1
table lamps. 61. + =
v − 600 20
51. Total deviation
= (180° – 2α) + (180° – 2β) 1 31
or =
= 360° – 2(α + β) v 600
But 90° – α + 90° – β + θ 600
or v= cm = 19.35 cm.
31
β
β

= 180° 90 – β
α
or θ = α + β α v 600 1 1
62. m=− =– × =
u 31 − 600 31
∴ Total deviation θ 90 – α
= 360° – 2θ Breadth of image
Fig. 54 1
= × 200 cm = 6.45 cm
52. The size of the image is the same as the size of the 31
object. 1
Height of image = × 160 cm = 5.16 cm.
31
53. 1 1 1
63. + =
O I O I u v f
1 du 1 dv
x x
− 2 dt
− 2 =0
x–v x–v u v dt
dv v2 du
Fig. 55 or =− 2
dt u dt
As is clear from Fig. 55, the new distance is 2x – 2v. 1
The distance of image from object is reduced by an = − × 31
31 × 31
amount 2v in one second.
54. Deviation = 180° – 2i = 180° – 60° = 120° 1
=– m s–1
31
55. = – 0.032 m s–1.
64.

Fig. 56

Fig. 58
490 COMPREHENSIVE OBJECTIVE PHYSICS

65. Clearly, plane mirror and convex mirror cannot pro- if m = + 3, then
duce inverted image.
− 24
67. Based on properties of a plane mirror. 3=
− 24 − u
f 1 f
68. m= = = or – 24 – u = – 8 or u + 24 = 8
f −u 4 f − (− 0.5)
or u = (8 – 24) cm = – 16 cm
or 4f = f + 0.5 or 3f = 0.5 If m = – 3, then
0.5 − 24
or f= m = 0.17 m. –3=
3 − 24 − u
f − 16 − 16
69. m= = = =2 u + 24 = – 8
f −u − 16 − (− 8) −8 or u = – 32 cm.

 Note. That the mirror used is concave mirror.  Note that the magnification is greater than one. So,
70. Image is at principal focus. mirror cannot be convex.

R 1
f= = − m = 0.5 m 360 360
2 2 75. − 1= − 1 = 9 − 1 = 8.
θ 40
71. Clearly, i + r = i + i = 140°
76. δ = 180° – 2(0°) = 180°
or i = 70°
= π radian
Clearly, plane mirror makes an angle of 20° with ver-
tical and 70° with horizontal.
78.
θ
d
40°
θ
I
20°

70°

70° Fig. 61

I f
=
d u
d
or I=f or I = θf.
Fig. 59 u
72. Clearly, the distance of image from observer is 40 cm. 1 1 1
79. + =
− 30 v 30
1 2 1
O I or = =
v 30 15
Observer
10 cm 10 cm or v = 15 cm.

30 cm  Since v is positive therefore we know that the


Fig. 60 image is formed behind the mirror.

73. The image is erect and diminished. So, the mirror is 80. The first image is due to reflection from the front
necessarily convex. surface i.e. unpolished surface of the mirror. So, only
a small fraction of the incident light energy is reflected.
 In the case of concave mirror, the image is
The second image is due to reflection from polished
erect and magnified.
surface. So, a major portion of light is reflected. Thus,
the second image is the brightest.
f
74. m=
f −u
OPTICS 491
90. Clearly, X is denser medium.
 Let us knock out a common misconception Medium x
about this question. sin θ 1 speed v
Now = θ
Whether the first image is brightest or the second sin 90° µ
Y X
one is brightest shall be determined only by the v 90°
angle of incidence. If angle of incidence is large, = X µY = Medium y
v′
the reflectance will be maximum. Maximum light speed v′
v
will be reflected from the front surface. So, the first or v′ = .
image will be brightest. sin θ Fig. 62
1 F 1 1I
= (µ − 1) G − J
81. The distance of image from the boy is 0.6 m. 91.
f Hr r K 1 2
360 F 1 1I
For lens to be convex, G − J > 0
83. n= − 1= 5
60
84. Angle between two mirrors is 90°.
Hr r K 1 2

1 1
0.6 or > or r1 < r2
85. (a) f =− m = – 0.3 m = – 30 cm r1 r2
2
92. Clearly, r = 30°
1 1 1
+ = sin i
v − 10 − 30 Now, µ=
sin 30°
1 1 1
= − 2 = 2 sin i
v 10 30
1
1 3−1 or sin i = or i = 45°
= 2
v 30
93. If the refractive index of two media are same, the sur-
30 face of separation does not produce reflection or re-
or v= cm = 15 cm.
2 fraction which helps in visibility.
v 15 94. Clearly, A is denser than B and B is denser than C.
(c) m=– =− = 1.5 .
u − 10 ∴ µ1 > µ 2 > µ 3
(d) Object lies between principal focus and pole. ∴ C1 < C2 < C3.
So, the image is virtual and erect. 95. Minimum distance between an object and its real
image is 4f i.e. 4 × 1 m = 4 m.
t nt
86. Time = = .
c c 1
96. sin ic =
n µ
c νλ λ µ is more. sin ic is less. ic is less.
87. n1 = = =
v1 νλ 1 λ1
3 × 1010
97. µ= = 1.5
c νλ λ 2 × 1010
n2 = = =
v2 νλ 2 λ 2
FG 1 − 1 IJ
Now,
n1 λ 2
=
101. P = (1.5 – 1)
H 0.25 ∞ K
n2 λ 1 P = 0.5 × 4 D = 2 D
Fn I λ
or λ2 = GH n JK
1
2
1 102.
I
=
O u
v

88. Only the light – gathering power is reduced. I − 25


=
89. This is made of optical fibres. Optical fibres are based 15 − 10
on total internal reflection.
I = 15 × 2.5 cm = 37.5 cm
492 COMPREHENSIVE OBJECTIVE PHYSICS

f 1 f Since power of the system is zero therefore the inci-


103. m= , = dent parallel beam of light will remain parallel after
f +u 2 f −2
emerging from the system.
or 2f = f – 2 or f = – 2 metre
| f | = 2 metre + Note the special technique employed in the solution.
Since the image is virtual as well as diminished there-
fore the lens is concave. µ1v 1 × 0.3
123. m= = =1
104. A convex lens shall behave like a concave lens if the µ 2u 1.5 × 0.2
refractive index of the material of the lens is less than f f
the refractive index of the medium surrounding the 124. m= n=
f +u f +u
lens.
1 1 f
105. µ= = = 2 f+u=
sin 45° 1/ 2 n
f 1− n FG IJ
106. One of the conditions for total internal reflection to
take place is that incident light should be in denser
or u= − f =
n n H K
f

medium. F n − 1IJ f , | u | =
u=−G
n−1
107. Using the relation apparent depth =
real depth
, we
or H n K n
f.
µ
get apparent depth + Magnification is positive in the case of virtual
3.5 8 image.
= + = (2 + 6) cm = 8 cm.
7/4 4/3
1 1 100 100
1 1 1 1 f1 + f2 125. P= + = +
= f f f1 f2 20 25
108. = + or or f = 12 .
f f1 f2 f f1 f2 f1 + f2 = (5 + 4) D = 9 D

109. λg =
λ a 7200 Å
= = 4800 Å FG 1IJ
µ 1.5
126. The letters are raised through a distance t 1 −
H µ K
,

I1 v I2 u I1I2 where t is the actual thickness.


111. = , = , = 1 , O = I1I2
O u O v O2 For red, λ is maximum. Using Cauchy’s relation
118. Power of combination = (6 – 4) D = 2 D b
µ=a+ , µ is minimum.
100 λ2
Power = 127. Note that θ is the angle of incidence and φ′ is the
f in cm
angle of refraction.
100
2=
f in cm + The angle of incidence is the angle between the
or f in cm = 50 incident wavefront and the refracting surface. The
angle of refraction is the angle between the refracted
Since the net power is positive therefore the combina-
wavefront and the refracting surface.
tion shall behave like a convex lens.
Note that these definitions considerably simplify
f the solution.
120. m=
f +u

1 − 20 1 FG IJ
1
=
2 − 20 + u
128.
30
= (µ − 1)
H K
10
or – 20 + u = – 40 1
or µ − 1=
or u = – 40 + 20 3
or u = – 20 cm.
1 4
122. Power of system or µ = 1+ = = 1.33
3 3
1 1 d 1 1 0.75
= + − = + − 3
f1 f2 f1 f2 1 − 0.25 (1)(− 0.25) 129. Apparent depth = cm = 2 cm
1.5
=1–4+3=–3+3=0 So, the microscope has to be raised upward by 1 cm.
OPTICS 493
130. 2µ1 × 3µ2 × 4µ3 d 1 1 4−1 3
= − = =
v1 v2 v3 100 5 20 20 20
= × × or d = 15 cm
v2 v3 v4
1 1 1 FG IJ
v
= 1 = 4 µ1 =
1 147.
f
= (1.5 − 1) −
R ∞ H K
v4 1µ 4 1 1
or = or f = 2R
c νλ f 2R
131. µ= =
v νλ ′ 148. O = I1I2 = 4 × 16 cm
3 λ 2λ = 8 cm
= or λ′ =
2 λ′ 3 100 100
149. P= − =0
Note that the frequency remains unchanged. 10 10
132. Light-gathering power of the lens is reduced. So, the 1 1 1
151. − =
intensity of the image decreases. v u f
134. ‘‘Light travels faster in rarer medium’’. This alone 1 1 1 1 1
decides in favour of option (b).
= + = + =0
v u f 20 − 20
8 ⇒ v = ∞.
135. Apparent depth = m = 6 m.
4 152. The emergent ray is parallel to the incident ray.
3 1
153. sin ic =
136. Based on conditions of total internal reflection. µ
137. Optical path is the product of refractive index and Largest µ. Minimum value of sin ic. Minimum ic.
geometrical path. 154. The medium surrounding the air lens is denser.
1 2 FG IJ 155. Think of lens-maker’s formula.
138.
f
= (1.65 − 1)
40 H K 156. Apparent shift
FG 1 IJ ; 0.01 = 0.1 FG 1 − 1IJ
or
1 0.65
f
=
20
or f =
20
0.65
cm H
= t 1−
µ K H µK
1 0.01 1
= 30.77 cm ≈ 31 cm. or 1– = =
µ 0.10 10
139. P = (12 – 2)D = 10 D
1 1 9
1 or = 1− =
1 µ 10 10
= 10 or f = m = 10 cm.
f 10 10
140. Note that the object is at 2F. or µ=
9
141. Light travels faster in rarer medium. c
157. Refractive index =
1 b v
144. sin ic = and µ = a + 2
µ λ 1 µε µε
= × =
For violet, λ minimum, µ maximum, sin ic minimum, µ 0 ε0 1 µ 0 ε0
ic is minimum. F I
145. As seen from a rarer medium (L2 or L3) the interface 158.
1
f
GH
= (µ − 1)
1

1
R1 R 2
JK
L1L2 is concave and L2L3 is convex. The divergence
produced by concave surface is much smaller than 1 1 1 FG IJ
the convergence due to the convex surface. Hence the
arrangement corresponds to concavo-convex lens.
f
= (1.5 − 1) −
20 30 H K
1 FG
30 − 20 IJ
146. P=
1
+
1

d
f1 f2 f1 f2
f
= 0.5
600 H K
1 1 10 1
1 1 d or = × = or f = 120 cm.
0= − − f 2 600 120
20 5 20(− 5)
494 COMPREHENSIVE OBJECTIVE PHYSICS

159. Power of flint glass w.r.t. carbon disulphide is zero. 16 9 3


161. Clearly, 2f = 20 cm or sin i = 1 − = =
25 25 5
or f = 10 cm.
FG 3IJ
162.
1 1 1
= −
or i = sin–1
H 5K
f v u
m
1 1 1 171. m=
= − f +u
f − 4 − 10
1
1 1 1 1 2−5 3
or = − or = =− –2= 3
f 10 4 f 20 20 1
+u
20 3
or f=– cm = – 6.67 cm
3 2 1
The negative sign indicates that the lens is concave. – − 2u =
3 3
1 F1 I 1 2 1
163. P = (µ − 1)
1 R 2
GH R − JK or – 2u = + = 1
3 3
or u=−
2
m = – 0.5 m

µ decreases. P decreases. f increases. 1 1 1


172. − =
FG 1 IJ v − f /2 f
164. Apparent shift = t 1 −
H µ K 1 1 2
= − =−
1

FG 1 IJ 1 1 or
v f
v=–f
f f
8 = 40 1 −
H µ K or
5
= 1−
µ
v −f
1 4 5 Again, m= = =2
or = or µ = u − f /2
µ 5 4 Clearly, the image is virtual and double the size.
165. When light travels from rarer to denser medium, the
1 2 FG IJ
n
speed of light decreases by a factor of µ. µ = 2 .
n1
173.
f
= (µ − 1)
R H K
R
100 100 1 or f =
166. = + 2(µ − 1)
80 20 f2
Now, f>R
1 100 100
or = − = (125
. − 5) D R
f2 80 20 ∴ >R
2(µ − 1)
or power = – 3.75 D
1
167. O= I1I 2 = 9 × 4 cm = 6 cm or > 1 or 2(µ – 1) < 1
2(µ − 1)
1 1 1
169. − = 1
v −f −f or µ–1<
2
1 2 f
or =− or v = – FG 1 IJ
v f
sin i
2 or µ < 1+
H 2 K
170. g µa = or µ < 1.5.
sin r
174. Power of the system decreases due to separation
1 sin i between the lenses. So, the focal length increases.
=
aµ g sin 2i
1 FG IJ
2
1
=
sin i
or
1
=
10 5
=
175.
f
= (µ − 1)
RH K
1.6 2 sin i cos i 2 cos i 16 8
1 F 2I
= (1.5 − 1) G J
or
1
=
cos i 4
5
or cos i =
4
5
f H RK
1 2
Now, sin i = 1 − cos2 i or = 0.5 × or f = R
f R
OPTICS 495
Since the object is at infinity therefore the image would 3 × 10 8 l µl
be formed at focus. 184. = or t= × 10−8
µ t 3
176. Think in terms of rectangular hyperbola.
µ 1 µ2 µ2 − µ1
1 1 186. + =
177. P= + −u v R
f1 f2
For a plane surface, R = ∞
f1 + f2
or P= µ1 µ2
f1 f2 ∴ + =0
−u v
178. Clearly, 2f = 20 cm
µ2 µ1 µ 1
or f = 10 cm or = or =
v u v u
Now, u = – 15 cm, v = ?
or v = µu
f = 10 cm
Clearly, to the fish, the bird appears farther than its
1 1 1
− = actual distance.
v − 15 10
dv du
1 1 1 1 1 1 Again, =µ
or + = or = − dt dt
v 15 10 v 10 15 or Apparent speed of bird
1 3−2 1
or = = = refractive index × actual speed of bird.
v 30 30
187. The power of the given system is a combination of the
or v = 30 cm
positive power of the convex lens, negative power of
The change in image distance is (30 – 20) cm the plano-concave lens of water and zero power of the
i.e. 10 cm. plane mirror. Clearly, the power of the system
v decreases.
179. m=
u 1 1 1 5
188. = + −
v F 10 − 20 10(− 20)
–4= or v = – 4u
u
1 1 1 5
Now, v – u = 10 or –5u = 10 = − +
or u=–2m F 10 20 200
v=8m 1 20 − 10 + 5 200
= or F= cm
1 1 1 F 200 15
Again, = − 40
f v u = cm.
3
or
1 1
= −
1
or
1 1 1 1+ 4 5
= + = = 1 FG IJ
2
f 8 −2 f 8 2 8 8 189.
f
= (µ − 1)
RH K
8 If R decreases, f decreases.
or f = m = 1.6 m.
5
F I
180. m=
f 191.
1
f
= (1 − 1) GH
1

1
R1 R 2
JK
f +u
f 1
1= or f + u = f or =0 ⇒ f =∞
f +u f
or u = 0. 192. In the first case, neither the radii of curvature nor
181. Since the lens behaves as a converging lens in air the material of the lens is affected.
therefore the refractive index of the material of the In the second case,
lens is greater than 1. Since the lens behaves as a
1 FG IJ
1
diverging lens in water therefore the refractive index
of the material of the lens is less than 1.33. f″ H K
= (µ − 1)
R
182. If focal length of convex lens is less, then power of 1 F 2I 1
= (µ − 1)G J =
1
convex lens will be more. or
f″ 2 H R K 2f or f ″ = 2 f.
496 COMPREHENSIVE OBJECTIVE PHYSICS

197. Condition for achromatism is 259. The outer faces of the prism
behave like hollow plates.
ω ω′ ω ω′
+ =0 or =− or Pω = – P′ω′.
f f′ f f′

1 1 1 Fig. 63
199. = +
F f1 f2 269. δm = (µ – 1)A = (1.5 – 1)(2r) = r
dF − df1 df2 274. See solution 259.
– = −
F2 f12 f22 FG A + δ IJ
H 2 K
m
sin
(For combination to be achromatic, dF = 0) A
276. cot = A
ω1 ω2 2 sin
df1 − df2
∴ – + = 0 or + =0 2
f12 f22 f1 f2
FG IJA A + δm
207. The angle of incidence on the second face is clearly
30°.
or
H
cos
K2
= sin
2

1 1 F π A I F A + δ IJ
sin G − J = sin G
1 sin 30°
H2 2K H 2 K
or m
= or sin e = 2× =
µ sin e 2 2
or e = 45° π A A δm
or − = +
2 2 2 2
Now, δ = i + e – A = 0 + 45° – 30° = 15°
or δ m = π − 2A
c c
212. v= = or δm = 180° – 2A
µ a+ b
λ2 60° + 40°
280. i= = 50°
2
λ is maximum for red. So v is maximum for red.
283. Dispersive power depends on the nature of the
1.6 + 1.4 3 material.
228. µ= = = 1.5
2 2 284. In minimum deviation position, the angle of incidence
is equal to angle of emergence.
µv − µr 1.6 − 1.4 0.2
ω= = = = 0.4 . 288. δm = (1.5 – 1) 6° = 3°.
µ−1 1.5 − 1 0.5
3 FG IJ
237. We can arrive at the right choice if we take into
δm 2
−1 A
H 1 K
account the sign convention. 289.
δ′ m
=
FG
3 3
= ×8=4
2 IJ
238. For achromatism,
ω1P1 + ω2P2 = 0
2 4H
× −1 A
K
292. For achromatic combination of two thin lenses in con-
ω1 P tact,
or ω1P1 = – ω2P2 or =− 2
ω2 P1 ω ω′ ω ω′
+ =0 or =−
242. When the curved surface of a plano-convex lens faces f f′ f f′
the incident rays, the total deviation of the rays is f′ ω′ f′ 2ω0
or =− or =− =−2 or f ′ = – 2f.
distributed at both the surfaces. This reduces spherical f ω f ω0
aberration and improves the sharpness of the image.
FG 60° + δ IJ
H 2 K
m
243. FR – FV represents longitudinal chromatic aberra- sin
tion. 295. 3=
sin G
F 60° IJ
244. There is no dispersive medium. H2K
245. δm = (µ – 1)A 3 F 60° + δ IJ
= sin G
H 2 K
m
= (1.52 – 1)4.5° = 0.52 × 4.5° = 2.34° 2

sin 60° = sin FG


60° + δ I
H 2 JK
258. In Fig. 43(3), note that the refracted ray is parallel to m
base.
OPTICS 497

60° + δ m 321. Magnifying power of a compound microscope


or = 60°
2 = Magnifying power of objective × Magnifying power
A + δ m 60° + 60° of eye-piece
or δm = 60°
2
=i=
2
= 60°
=
v0 LM fe OP
296. The ratio of dispersive powers same as the ratio of
focal lengths.
N
u0 fe + ue Q
If the length of microscope tube is increased, ue is
302. System of two lenses separated by distance d will be increased. So, magnifying power of microscope de-
free from chromatic aberration if the distance, creases.
ω 1 f1 + ω 2 f2 + A serious mistake
d=
ω1 + ω 2
LF D I
and if the lenses are made of same material, M=− GH
fo
1+
fe
JK
i.e., ω1 = ω2 = ω then,
M∝L
f1 + f2 M increases with increase in L.
d=
2
D
l The combination has same focal length for all colours. 322. M=1+
fe
l Since d is always positive, f1 + f2 must be positive.
If fe is small, then M is large.
Therefore, for such a combination either both the
lenses should be convex or the lens with greater focal fo 200
323. = = 100
length should be convex. fe 2
l The combination is apparently free from lateral chro-
327. If the final image is formed at infinity, the magnify-
matic aberration.
ing power of the Galilean telescope is
304. In the given problem, the Rayleigh scattering which fo 200
1 M= = = 50
is proportional to is not true. This is due to the fe 4
λ4
fo
large size of the scattering object. The size of the 329. M=
raindrops, large dust or ice particles is much larger fe
than the wavelength of the visible light. Yellow colour fo should be large for higher M.
is scattered less in foggy conditions. Moreover, the 2f f 8f + 3f 11f
sodium lamps emit light very near to the peak 330. Required distance is + or or .
3 4 12 12
sensitivity of the human eye under normal viewing
conditions. 333. From the diagram it can be observed that the final
image occurs at infinite. The focal length of the objec-
306. In a compound microscope, the object is placed just
tive lens is 80 cm.
beyond the focus point of the objective. Therefore, the
image formed by the objective is real, inverted and Objective lens Eye-piece
magnified.
313. Higher the power, lesser the focal length.
fo
Also, M= .
fe
fo
314. M= .
fe
319. In this case, the final image is at infinity.
fo fo 80 cm 80 cm 5 5
320. 10 = or fe = cm cm
fe 10
Telescope

Now, fo + fe = 44 Fig. 64

fo 11fo fo
or fo + = 44 or = 44 or fo = 40 cm. 339. Look for maximum value of .
10 10 fe
498 COMPREHENSIVE OBJECTIVE PHYSICS

341. Smaller the focal lengths of the objective and eye-piece, where
higher is the magnifying power of the compound mi- m = positive integers, m = 0 1, 2, ...
croscope. In other words, the objective and eye-piece
1 FG IJ
should have large optical powers.
344. Converging lens L is used as a magnifying glass to
phase difference = m+
2

H K
view the object O. An erect, magnified image is ob- = 2 mπ + π, m = 0, 1, 2.
tained when the object O is near to L than its focal 376. For the central fringe, all the colours have zero path
length, and the observer has to move the lens until difference.
the image at I is situated at his or her near point = D nDλ
from the eye. Again, yn =
d
λ is least for violet.
L 377. Think in terms of coherent sources.
nDλ
378. yn = .
h′ d
h For red colour, λ is more.
λ
I O 380. Angular separation =
F d
For angular separation to be 10% greater, λ should be
D 10% greater.
FG 589 IJ
Fig. 65
New wavelength is 589 +
H 10
nm
K
or (589 + 58.9) nm i.e. 647.9 nm i.e. 648 nm.
fe
345. m= 381. I0 α (a + a)2
u − fe
When objective is replaced by a slit, or I0 ∝ 4a2
u = fo + fe Again, I ∝ a2

l f 1 L I0 I0
∴ = e = or m = ∴ = 4 or I=
L fo m l I 4

348. Take the inverse ratio of wavelengths. λ


385. Note that the path difference is odd multiple of .
368. Imin. ∝(a – a)2 2
I′min. ∝(2a – a)2 h h h
386. λ= = =
Clearly, the intensity of minima increases. mv 2m E k 2meV
Again, Imax. ∝(a + a)2
As V is decreased, λ increases.
I′max. ∝(2a + a)2
Clearly, the intensity of maxima increases. Again, Dλ
β=
d
(2D)λ Dλ
370. β′ = =4 Dλ
d/2 d 388. yn = n .
d
or β′ = 4β.
λ is minimum for violet.
372. I = I1 + I2 + 2 I 1 I2 cos 0°
a1 2 Imax. (2 + 1)2 9
I=4+9+2 4 9 389. = = =
a2 1 Imin. (2 − 1)2 1
= 13 + 2 × 2 × 3 = 25. a1 4 Imax. 49
375. By principle of superposition at P, the optical differ- 390. = , =
ence for two coherent light waves from S1 and S2 must a2 3 I min. 1
1FG IJ Dλ
be m +
2 H K
λ if dark fringes occurs at P, 391. β=
d
λ = wavelength of the monochromatic light wave. λ is minimum for violet.
OPTICS 499

393. There would be a slight increase in fringe width due 411. A= 32 + 42 = 25 = 5


to slight increase in wavelength.
412. Due to decrease in wavelength, the fringe width
I′ (a + a)2 4 decreases. So, more fringes can be accommodated on
394. = =
I a2 1 the screen.
395. Wavelength is halved. λ
415. Path difference is 11 .
396. Refer to definition of fringe width. 2
416. Decrease in λ.

397. β= 417. Effective path difference is (µ1L1 – µ2L2).
d
420. For interference, coherent sources are required.
βd 4 × 10−3 × 0.1 × 10−3 Imax. (2 + 1)2 9
D= = m 421. = =
λ 4 × 10−7 Imin. (2 − 1)2 1
= 1 m. Dλ 1 × 5890 × 10−10

422. β= = = m
2 × 5 × 10−7 d d 0.2 × 10−3
398. β= m = 10 × 10–4 m = 1 mm
1 × 10−3 = 0.29 × 10–2 m
= 2.9 mm ≈ 3 mm.
399. λ increases.
FG D IJ λ 423. In the first case,

400. β′ =
H 2K =

= β. or
I = I0 + I0 + 2 I0 cos 0°
I = 4I0
d d
2 In the second case,
a1 2 I′ = I0 + I0 = 2I0
402. =
a2 1 I 4I0 2
∴ = =
Imax. (2 + 1) 9 2 I′ 2I0 1
= =
Imin. (2 − 1)2 1 Dλ
424. β=
404. There is a decrease of wavelength. d
3 h
405. New wavelength is times the old wavelength. So, and λ=
4 2meV
3 425. Phase difference 2π corresponds to a path difference of
the new angular separation is × 0.2° or 0.15°.
4 λ.
406. Note that phase difference is β1 – β2. Dλ β
426. β′ = =
407. Phase difference of 2π corresponds to path difference 3d 3
λ 429. Condition for destructive interference is that the phase
of λ. So, unit phase difference corresponds to .
2π difference should be an odd multiple of π.
Clearly, the phase difference φ corresponds to a path
λ 1
difference of φ . 430. β∝
2π α
408. If phase difference is time-dependent, the waves can- When α is increased, β decreases.
not be coherent. So, interference is not possible.
431. Two coherent sources are required to produce inter-
409. a2 = a2 + a2 + 2a2 cos θ ference.
1 2π 432. The condition for constructive interference is that the
or cos θ = – or θ = 120° = rad.
2 3 path difference should be an even multiple of λ/2.
a1 + a2 2 φ
410. = 435. I ∝ 4a2 cos2
a1 − a2 1 2
a1 3 In the first case, φ = 2π
3a2 = a1 or =
a2 1 ∴ I ∝ 4a2 ...(1)
500 COMPREHENSIVE OBJECTIVE PHYSICS

2π 2Dλ 2 × 2 × 600 × 10−9


In the second case, φ = 450. = m
3 d 1 × 10 −3
2π = 24 × 10–4 m = 2.4 mm.
∴ I′ ∝4a2 cos2 or I′ ∝ a2
6 451. The fine rulings, each 0.5 µm wide, on a compact disc
I′ 1 I function as a diffraction grating. When a small source
= or I′ =
I 4 4 of white light illuminates a disc, the diffracted light
436. Decrease in wavelength. forms coloured “lanes” that are the composite of the
diffraction patterns from the rulings.
437. Different wavelengths would correspond to different
frequencies. Lights of different intensities can give 2Dλ
coherence even if contrast is poor. 452. Width of central maximum =
d
438. All the interference fringe are of equal width. As λ increases, width increases.
1 Dλ f λ
440. β∝ 453. β= =
d d a
442. λ increases, β ∝λ . 471. Most of the light is diffracted between the two first
443. Independent sources cannot produce interference. order minima.
444. The contrast between bright and dark fringes is ∴ Required angle = 2θ
determined by intensity ratio.

2Dλ = 2 sin θ =
447. β= d
d
β is reduced due to decrease in λ. 2 × 450 × 10 −9
=
448. The angle of diffraction θ normal to the grating for 2 × 10−4
nth-order diffraction pattern is given by = 450 × 10–5 radian
d sin θ = nλ = 4.5 × 10–3 radian.
where d is the spacing of the slits 2Dλ d2
472. =d or D =
λ is the wavelength of the source. d 2λ
1 Dλ
Now d= m, θ = 30° and n = 2. 473. β=
4.5 × 105 d
We thus have λ increases. β increases. The fringe pattern broadens.

d sin θ sin 30° d2


λ= = ≈ 556 nm. 474. DF =
n 2(4.5 × 105 ) λ
449. The angle of diffraction θn for nth order diffraction (3 × 10−3 )(3 × 10 −3 )
DF = m
pattern of the light is given by 500 × 10−9
d sin θn = nλ 90
= m = 18 m
where d = spacing between adjacent slits in diffraction 5
476. d sin θ = λ
10 −1
grating = m. or 2λ sin θ = λ
600
n = 1 for first-order diffracted beam. 1
or sin θ = or θ = 30°
2
θ1 = 15°
477. When the given source is replaced by another of longer
d sin θ1 10−3 wavelength, the direct image remains unaffected. But
⇒ λ = = sin 15° = 4.3 × 10–7 m the diffracted images A and B would shift away from
1 600
D.
Thus frequency of light,
480. 60° is polarising angle. Use Brewster’s law.
c 3 × 108
f = = = 7.0 × 1014 Hz. µ = tan ip = tan 60°.
λ 4.3 × 10−7
OPTICS 501
481. According to Malus’ law as the square of distance from the source, but cannot
be polarised.
I0 I I
cos2 θ = 0 cos2 45° = 0
FG 1 IJ 2
I0 Light, a transverse wave, can be diffracted, can be
I=
2 2 2 H 2K =
4 made to interfere, whose intensity falls of inversely
∴ 25% of incident light is now transmitted through as the square of distance from the source, can also be
the system. polarised.
µ2 1 sin C1 sin C1
484. 3 = tan ip 499. tan θ = = × =
µ1 sin C2 1 sin C2
485. I = I0 cos2 45° 500. Transverse waves can be polarised. These include all
I0 electromagnetic waves such as light, radio waves and
I= . X-rays. On the other hand, longitudinal waves such
2
as sound cannot be polarised.
487. According to Brewester’s law,
503. µ = tan i
n = tan ip or ip = tan–1 (n).
c
488. Reflection causes partial polarisation. Double = tan i
refraction produces two polarised beams. Scattering cm
produces partial polarisation. c
or cm = or cm = c cot i
490. Only transverse wave (inclusive of light wave, tan i
electromagnetic wave and others) can be polarised. 511. Frequency is a characteristic of source of light and
Longitudinal wave (such as sound wave) and not the medium.
stationary wave cannot be polarised.
543. A slit would give divergent ; a biprism would give
491. This ‘one angle of incidence’ is called polarising angle. double ; a glass slab would give a parallel wavefront.
492. wµg < aµg and tan θ = µ. Only a prism would turn the wavefront ; refracting
496. 57° is polarising angle. edge is downwards.
497. Sound, a longitudinal wave, can be diffracted, can be 564. P to Q convergence increasing ; Q to R direction
made to interfere, whose intensity falls off inversely changing.

KNOWLEDGE PLUS
In each question, a statement of assertion (A) is given and a corresponding statement of reason (R) is given just below it.
Of the statements, mark the correct answer as :
(A) If both A and R are true and R is the correct explanation of A.
(B) If both A and R are true but R is not the correct explanation of A.
(C) If A is true but R is false.
(D) If both A and R are false.
(E) If A is false but R is true.
1. Assertion (A). The bees cannot only distinguish unpolarised light from polarised light but can also determine the
direction of polarisation.
Reason (R). The human eye does not distinguish between an unpolarised light and a polarised light.
2. Assertion (A). The diffraction of light can be observed if the size of the obstacle or aperture is of the order of
10–7 m.
Reason (R). The order of wavelength of visible light is 10–7 m. [AIIMS 2001]
3. Assertion (A). In Young’s double slit experiment, the interference pattern disappears when one of the slits is closed.
Reason (R). Interference occurs due to the superposition of light waves from two coherent sources.
[HPMT 2000]
4. Assertion (A). Average energy in the interference pattern is the same as it would be if there were no interference.
Reason (R). Interference is the only rare phenomenon in which law of conservation of energy does not hold good.
[AIIMS 1997]
Key 1. (B) 2. (A) 3. (A) 4. (C).
502 COMPREHENSIVE OBJECTIVE PHYSICS

MCQs
based on
SET II

Important Basic Concepts


+
Application of Formulae
+
Brain Teasers
Average time allowed per question is 50 seconds.

(a) 12 π watt (b) 24 π watt


PHOTOMETRY
(c) 5 π watt (d) 30 π watt.
[MANIPAL 1996]
581. A photographic plate placed at a distance of 5 cm
from a weak point source is exposed for 3 s. If the 585. A small screen is illuminated by three closely spaced
plate is kept at a distance of 10 cm from the source, candles which are 1.2 m away from the screen. One
the time needed for the same exposure is of the candles is put out. By what distance must the
(a) 3 s (b) 12 s screen be brought nearer to the candles for its illu-
(c) 24 s (d) 48 s. minance to remain the same ?
582. Two point sources A and B of luminous intensities 1 (a) 21 cm (b) 39 cm
candela and 16 candela respectively are placed 100 (c) 9 cm (d) 13 cm. [AMU 1998]
cm apart. A grease spot sereen is placed between the 586. A screen receives 3 W of radiant flux of wavelength
two sources. For the grease spot to become indistin- 6000 Å. One lumen is equivalent to 1.50 × 10–3 W of
guishable from both the sides, it should be placed at monochromatic light of wavelength 5550 Å. If rela-
(a) 80 cm from 16 candle lamp and 20 cm from 1 candle lamp tive luminosity for 6000 Å is 0.685 when that for 5550
(b) 20 cm from the 16 candle lamp and 80 cm from 1 candle Å is 1.00, then the luminous flux of the source is
lamp (a) 2 × 103 lm (b) 1.37 × 103 lm
400 100 (c) 3 × 103 lm (d) 4 × 103 lm.
(c) cm from 16 candle lamp and cm from 1 candle
3 3
lamp [Haryana PMT 2000]
587. Correct exposure for a photographic print is 8 sec-
100 400 ond at a distance of 16 cm from a 25 candela lamp.
(d) cm from 16 candle lamp and cm from 1 candle
3 3 For what time will you expose the same print at a
lamp. [Pb. PMT 1999] distance of 24 cm from a 40 candela lamp ?
583. An electric bulb illuminates a plane surface. The (a) 7.25 s (b) 9.25 s
intensity of illumination on the surface at a point (c) 11.25 s (d) 13.25 s.
2 m away from the bulb 5 × 10–4 phot (lumen cm–2). [Himachal PMT 2000]
The line joining the bulb to the point makes an angle 588. Two stars which are 4 and 12 light years away from
of 60° with the normal to the surface. The intensity Earth produce equal illumination on the Earth. What
of the bulb in candela (candle power) is is the ratio of their luminous intensities ?
(a) 40 × 10–4 (b) 40 (a) 9 : 1 (b) 1 : 9
(c) 2 : 3 (d) 3 : 2 [AFMC 2001].
(c) 40 3 (d) 20. [IIT 1980]
589. Rays of light from a source of luminous intensity
584. The luminous efficiency of a lamp is 5 lumen/watt 40 cd are incident at 60° on a screen placed 2 m from
and its luminous intensity is 30 candela. The power the source. Illumination produced on the screen is
of the lamp is (in lux)
OPTICS 503
(a) 20 (b) 30 1 FG 5 IJ 3/ 2
(c) 5 (d) 10. [Karnataka 1999] (a)
2
(b)
H 4K
590. A flux of 100 lumen falls normally on a study table FG 4 IJ F 4I
(d) GH JK .
0.5 m × 1 m. The illumination on the table is (c) H 3K 5
[Pb. CET 1997]
(a) 200 lux (b) 100 lux 597. Correct exposure for a photographic print is 10 second
(c) 50 lux (d) 25 lux. [JIPMER 1998] at a distance of 2 metre from a point source of 80
591. Two 250 cd lamps hang one above the other at heights candela. For an equal fogging of the print placed at a
2.5 m and 3.5 m from the ground. The opproximate distance of 3 metre from a 36 candela source, the
illuminance on the ground at a distance of 2.5 m necessary time of exposure is
from the point on the ground above which the lamps (a) 100 s (b) 25 s
hang is (c) 50 s (d) 75 s.
(a) 15 lux (b) 25 lux [CMC Vellore 1997]
(c) 35 lux (d) 5 lux. [JIPMER 1998] 598. The light output from a 100 watt lamp is 15 lumen
per watt. Its luminous intensity is very near to
592. A fluorescent tube which is equivalent to a line source
(a) 100 cd (b) 120 cd
of 100 candle m–1 is hung horizontally 5 m above the
table. Then the illuminance at a point vertically below (c) 150 cd (d) 200 cd.
the tube is [Bharati Vidyapeeth 1998]
(a) 40 lux (b) 40 phot 599. A point source of yellow light rated 12 watt has an
(c) 8 lux (d) 8 phot. [Pb. CET 1999] overall efficiency of 10 lumen per watt. Then the lu-
593. The separation between the screen and a concave minous flux through a solid angle of π steradian is
mirror is 2r. An isotropic point source of light is placed (a) 120 lumen (b) 60 lumen
exactly midway between the mirror and the screen. (c) 30 lumen (d) 15 lumen. [AMU 1999]
Mirror has a radius of curvature r and reflects 100% 600. The separation between the screen and a plane mir-
of the incident light. Then the ratio of illuminance
ror is 2r. An isotopic point source of light is placed
on the screen with and without the mirror is
exactly mid way between the mirror and the screen.
(a) 10 : 1 (b) 2 : 1
Assume that mirror reflects 100% of incident light.
(c) 10 : 9 (d) 9 : 1. [Pb. CET 2001] Then the ratio of illuminance on the screen with and
594. A battery-operated torch is adjusted to send an almost without the mirror is
parallel beam of light. It produces an illuminance of (a) 10 : 1 (b) 2 : 1
40 lux when the light falls on a wall 2 m away. The (c) 10 : 9 (d) 9 : 1. [BHU 1999]
illuminance produced when it falls on a wall 4 m
away is close to 601. Time of exposure for a photographic print is 10 s
when a lamp of 50 cd is placed at 1 m from it. Then
(a) 40 lux (b) 20 lux
another lamp of luminous intensity I is used, and is
(c) 10 lux (d) 5 lux. [JIPMER 1998]
kept at 2 m from it. If the time of exposure now is 20
595. A lamp rated at 100 cd hangs over the middle of a s, then the value of I is (in cd).
round table with a diameter 3 metre at a height of (a) 20 (b) 200
2 metre. It is replaced by a lamp of 25 cd and the
(c) 25 (d) 100.
distance to the table is changed so that the
illumination at the centre of the table remains as [Karnataka CET 1990]
before. The illumination at edge of the table becomes 602. A table is illuminated by two identical bulbs sus-
X times the original. Then X is nearly pended at the same height h, one of them being on
1 16 the normal passing through the middle of the table
(a) (b)
3 27 and the other, on a straight line forming an angle of
1 1 45° with this normal and at distance h from the lamp.
(c) (d) . [MANIPAL 2000]
4 9 The second bulb has fused. By what distance should
596. A lamp is hanging 1 m above the centre of circular the first bulb be brought closer to the table to retain
table of diameter 1 m. The ratio of illumination at the same illuminance at the middle of the table ?
the centre and the edge is
504 COMPREHENSIVE OBJECTIVE PHYSICS

(a) By 0.019 h (b) By 0.038 h 609. A source of 500 candela is placed at the centre of a
(c) By 0.057 h (d) By 0.14 h. [MNR 1997] piece spherical surface of area 0.5 m2. If the radius of
603. In a grease spot photometer, light from a lamp with the surface be 5 m, the luminous flux through the
dirty chimney is exactly balanced by a point source surface is
distant 10 cm from the grease spot. On clearing the (a) 5 lumen (b) 10 lumen
dirty chimney, the point source is moved 2 cm to (c) 25 lumen (d) 50 lumen.
obtain a balance again. Then the percentage of light
[Pb. CET 1994]
absorbed by the dirty chimney is nearly
(a) 64% (b) 36% 610. An ordinary 100 watt source of light P radiates iso-
(c) 44% (d) 56%.
tropically, and a 2 mW laser source Q emits in a cone
of semiangle 10–4 radian. The intensity ratio IQ/IP of
[Bharati Vidyapeeth 2000]
the beams is close to
604. A photographic plate is placed directly in front of a (a) 4π/5 (b) (4π/5) × 103
small diffused source in the shape of a circular disc.
(c) (4/5) × 104 (d) 8 × 103.
It takes 12 s to get a good exposure. If the source is
[National Standard Exam. in Physics 1990]
rotated by 60° about one of its diameters, the time
needed to get the same exposure will be 611. To prepare a print with 40 watt lamp at 25 cm, it
(a) 6 s (b) 12 s
requires 3 second. If the distance is increased to
50 cm, how much time will it require to prepare the
(c) 24 s (d) 48 s. [AIIMS 2002] print ?
605. An electric bulb is hanging over a table at a height of (a) 6 s (b) 9 s
1 m above it. The illuminance on the table directly (c) 12 s (d) 15 s. [Manipal 1998]
below the bulb is 40 lux. The illuminance at a point
612. A lamp is hanging along the axis of a circular table of
on the table 1 m away from the first point will be
radius r. It is desired that the intensity of illumina-
about
tion at the edge of the table be 1/8th of that at its
(a) 10 lux (b) 14 lux centre. To achieve this, the height at which the lamp
(c) 20 lux (d) 28 lux. should be placed is
[Himachal PMT 2002] (a) r/ 2 (b) r/ 3
606. In a yard, at the same height of 3.0 m, three 200 cd
(c) r/3 (d) r 3 .
lamps hang. All the lamps are arranged at distance
of 2.5 m from each other. The approximate [National Standard Exam. in Physics 2002]
illuminance under each lamp is 613. If the distance between the projector and the screen
(a) 22 lux (b) 32 lux is decreased by 25%, the intensity of illumination
(c) 42 lux (d) 12 lux. will be increased by
(a) 12.5% (b) 25%
[Haryana PMT 2001]
(c) 50% (d) more than 75%.
607. An underground tunnel has a circular cross section.
[Haryana PMT 2000]
An electric lamp is placed at the upper point of the
arch. Then the ratio of the illumination produced by 614. A lamp hanging 4 metre above the table is lowered
the lamp at the lowest point and a point lying on the by 1 metre. The illumination on the table
intersection of the horizontal diameter with the cir- (a) decreases by 25% (b) increases by 25%
cumference of the circular cross section of the tunnel (c) decreases by 77.7% (d) increases by 77.7%.
is [Karnataka CET 2003]
(a) 2 : 1 (b) 2 : 1
(c) 1 : 2 (d) 1 : 2.
REFLECTION
[All India PM/PD 1998] 615. A perfectly reflecting mirror has an area of 1 cm 2.
608. A small lamp is hung at a height of 8 feet above the Light energy is allowed to fall on it for one hour at
centre of a round table of diameter 16 feet. The ratio the rate of 10 W cm –2. The force that acts on the
of intensities of illumination at the centre and at mirror is
points on the circumference of the table will be (a) 2.4 × 10–4 N (b) 1.34 × 10–7 N
(a) 1 : 1 (b) 2 : 1 (c) 3.35 × 10–8 N (d) 6.7 × 10–8 N.
(c) 2 2 : 1 (d) 3 : 2. [Pb. CET 1995]
OPTICS 505
616. With a concave mirror, an object is placed at a dis- 623. A convex mirror has a focal length f. A real object is
tance x1 from the principal focus, on the principal placed at a distance f/2 in front of it from the pole.
axis. The image is formed at a distance x2 from the The mirror produces an image at
principal focus. The focal length of the mirror is (a) infinity (b) f
x + x2 (c) 2f [Pb. PMT 2001]
(d) f/3.
(a) x1 x2 (b) 1
2 624. A concave mirror of focal length 200 cm is used to
x1 obtain the image of the sun which subtends an angle
(c) (d) x1x2 . of 30′ . Then the diameter of the image of the solar
x2
disc is
617. A convex mirror has a focal length of 20 cm. A real
(a) 13.48 cm (b) 1.75 cm
object is placed at a distance of 20 cm in front of the
(c) 9.87 cm (d) 20.0 m.
mirror from the pole. The mirror produces an image
at [Haryana PMT 1997]
(a) infinity (b) 20 cm 625. A beam of light of wavelength λ is totally reflected at
normal incidence by a plane mirror. The intensity of
(c) 40 cm (d) 10 cm .
the light is such that photons hit the mirror at a rate
618. Two vertical plane mirrors are inclined at an angle n. Given that the Planck constant is h, the force ex-
of 60° with each other. A ray of light travelling hori- erted on the mirror by this beam is
zontally is reflected first from one mirror and then
(a) nhλ (b) nh/λ
from the other mirror. Then the resultant deviation
(c) 2nhλ (d) 2nλ/h
is
(a) 60° (b) 120° (e) 2nh/λ.

(c) 180° (d) 240°. [Pb. PMT 2000] 626. The focal length of a concave mirror is 12 cm. Where
should an object of length 4 cm be placed so that an
619. A man has a concave shaving mirror of focal length image 1 cm long is formed ?
0.2 m. How far should the mirror be held from his (a) 48 cm (b) 3 cm
face in order to give an image of two fold magnifica- (c) 60 cm (d) 15 cm.
tion ? 627. The focal length of a concave mirror is 20 cm. Where
(a) 0.1 m (b) 0.2 m an object must be placed to form an image magnified
(c) 0.3 m (d) 0.4 m. two times when the image is real ?
[Bharati Vidyapeeth 1996] (a) 30 cm from the mirror (b) 10 cm from the mirror
(c) 20 cm from the mirror (d) 15 cm from the mirror.
620. A concave mirror, forms a real image of an object. If
the whole arrangement is immersed in water. Then 628. In the previous question, if the magnified image is
the image will virtual, then the distance of the object from the mir-
ror must be
(a) disappear (b) shift towards the mirror
(a) 30 cm (b) 10 cm
(c) shift away from the mirror (c) 20 cm (d) 15 cm.
(d) not suffer any change in position. 629. The image formed by a convex mirror of focal length
[National Standard Exam. in Physics 1999] 20 cm is half the size of the object. Then, the distance
of the object from the mirror is
621. A mirror forms a real image of unit magnification.
(a) 20 cm (b) 10 cm
Its nature is
(c) 30 cm (d) 60 cm.
(a) plane (b) parabolic
630. A ray reflected successively from two plane mirrors
(c) convex (d) concave.
inclined at a certain angle undergoes a deviation of
[Karnataka 1999] 300°. The number of observable images is
622. A luminous object is placed 20 cm from the surface (a) 60 (b) 12
of a convex mirror. A plane mirror is set so that the (c) 11 (d) 5.
virtual images formed in the two mirrors coincide. If 631. An object 3 cm tall is placed on the principal axis of a
the plane mirror is at a distance of 12 cm from the concave mirror of focal length 9 cm at a distance of
object, then the focal length of the convex mirror is 12 cm from it. What is the nature and size of the
(a) 10 cm (b) 5 cm image ?
(c) 20 cm (d) 40 cm. [AMU 2001] (a) real, 9 cm (b) virtual, 9 cm
(c) real, 1 cm (d) virtual, 1 cm.
506 COMPREHENSIVE OBJECTIVE PHYSICS

632. An object 5 cm tall is placed 10 cm from a convex (a) real and will remain at C
mirror of radius of curvature 30 cm. What is the (b) real and located at a point between C and ∞.
nature and size of the image ? (c) virtual and located at a point between C and O
(d) real and located at a point between C and O.
(a) real, 3 cm (b) virtual, 7.5 cm
[IIT 1998]
(c) virtual, 3 cm (d) real, 7.5 cm.
640. An object is placed asymmetrically between two plane
633. A ray of light travelling parallel to the principal axis
mirrors inclined at an angle of 72°. The number of
of a concave mirror strikes the mirror at an angle of
images formed is
incidence θ. If radius of curvature of the mirror is R,
(a) 5 (b) 4
then after reflection, the ray meets the principal axis
at a distance x from the centre of curvature. Then x (c) 2 (d) infinite.
is 641. It is desired to photograph the image of an object
R R placed at a distance of 3 m from a plane mirror. The
(a) (b) camera which is at a distance of 4.5 m from the mir-
2 2 tan θ
ror should be focussed for a distance of
R R
(c) (d) . [MNR 1998] (a) 3 m (b) 4.5 m
2 cos θ 2 sin θ
(c) 6 m (d) 7.5 m.
634. A man 180 cm high stands in front of a plane mirror.
His eyes are at a height of 170 cm from the floor. 642. A cubical room is formed with six plane mirrors. An
Then the minimum length of the plane mirror for insect moves along the diagonal of the floor with
him to see his full length image is uniform speed. The velocities of its image in two
(a) 90 cm (b) 180 cm adjacent walls are 20 2 cm s–1. Then the velocity of
(c) 45 cm (d) 360 cm. the image formed by the roof is
635. In the previous question, the lower edge of the mirror (a) 20 cm s–1 (b) 20 2 cm s–1
should be above the ground at a height of
(c) 40 cm s–1 (d) 10 2 cm s–1.
(a) 85 cm (b) 90 cm
(c) 170 cm (d) 180 cm.
643. A plane glass mirror of thickness 3 cm of material of
636. A man 2 m tall stands 5 m in front of a large vertical 3
µ = is silvered on the back surface. When a point
plane mirror. Then the angle subtended at his eye by 2
his image in the plane mirror is object is placed 9 cm from the front surface of the
(a) 0.4 radian (b) 0.2 radian mirror, then the position of the brightest image from
(c) 0.2 degree (d) 0.4 degree. the front surface is
637. Rays of light striking a horizontal plane mirror at (a) 9 cm (b) 11 cm
45° get reflected from a second mirror horizontally. (c) 12 cm (d) 13 cm. [EAMCET 2003]
The angle of inclination of the second mirror with 644. A convergent beam of light converges to a point 20 cm
the first mirror is behind the convex mirror on the principal axis. An
(a) 45° (b) 60° inverted image of the same size is formed coincident
(c) 67° 30′ (d) 135°. with the virtual object. Then, the focal length of the
638. An object is placed at a distance of 2f from a concave convex mirror is
mirror of focal length f. Light reflected from the (a) 20 cm (b) 10 cm
mirror falls on a plane mirror. The distance of the (c) 40 cm (d) 30 cm.
plane mirror from the concave mirror equals f. The [CMC Vellore 1999]
distance of the final image (due to reflection at both
645. Two parallel plane mirrors M1 and M2 have a length
concave and plane mirror) from the concave mirror
of 2 m each and are 10 mm apart. A ray of light is
is
incident on one end of mirror M2 at an angle of 30°.
(a) f (b) f/2
The number of reflections light undergoes before
(c) 2f (d) Zero. [Pb. PMT 1996] reaching the other end is
639. A concave mirror is placed on a horizontal table, with (a) 173 (b) 134
its axis directed vertically upwards. Let O be the pole (c) 80 (d) 20.
of the mirror and C its centre of curvature. A point [CMC Vellore 2001]
object is placed at C. It has a real image, also located 646. Sun subtends an angle of 0.5° at the centre of curva-
at C. If the mirror is now filled with water, the image ture of a concave mirror of radius of curvature 15 m.
will be
OPTICS 507
The diameter of the image of the sun formed by the 653. A cube of side 2 m is placed in front of a concave
mirror is mirror of focal length 1 m with its face A at a distance
(a) 8.55 cm (b) 7.55 cm of 3 m and face B at a distance of 5 m from the mirror.
(c) 6.55 cm (d) 5.55 cm. [DCE 2003] The distance between the images of face A and B and
647. A convex mirror has a focal length 20 cm. A conver- height of images of A and B are respectively.
(a) 1 m, 0.5 m, 0.25 m (b) 0.5 m, 1 m, 0.25 m
gent beam of light converges to a point 20 cm behind
(c) 0.5 m, 0.25 m, 1 m (d) 0.25 m, 1 m, 0.5 m.
the convex mirror on the principal axis. The image is
formed at
(a) infinity (b) 20 cm 2m
B A
(c) 40 cm (d) 10 cm. [EAMCET 2000]
2m
648. A point source of light B is placed at a distance L in
3m
front of the centre of a mirror
B
of width d hung vertically on
a wall. A man walks in front d
of the mirror along a line
Fig. 67
parallel to the mirror at a
distance 2L from it as shown [JIPMER 2000]
in Fig. 66. The greatest L 654. An object is placed 21 cm in front of a concave mirror
distance over which he can 2L
of radius of curvature 10 cm. A glass slab of thickness
see the image of the light
Fig. 66 3 cm and µ = 1.5 is then placed close to the mirror in
source in the mirror is
the space between the object and the mirror. The
d
(a) (b) d position of final image formed is
2
(c) 2d (d) 3d. (a) – 3.94 cm (b) 4.3 cm

[IIT Screening 2000] (c) – 4.93 cm (d) 3.94 cm. [AMU 2003]
649. Two plane mirrors are inclined to one another at an 655. A vehicle has a driving mirror of focal length 30 cm.
angle θ. The incident ray on mirror M1 is parallel to Another vehicle of dimensions 2 × 4 × 1.75 m3 is 9 m
mirror M 2. The reflected ray from M 1 undergoes away from the mirror of first vehicle. Position of the
reflection at mirror M2. The final reflected ray is second vehicle as seen in the mirror of first vehicle is
parallel to mirror M1. Angle θ is nearly
(a) 30° (b) 45° (a) 30 cm (b) 60 cm
(c) 60° (d) 90°. [MANIPAL 2002] (c) 90 cm [Pb. PMT 1995]
(d) 9 m.
650. A spherical mirror forms diminished virtual image 656. A ray of light makes an angle of 20° with the horizon-
of magnification 1/3. Focal length is 18 cm. The tal and strikes a plane mirror which is inclined at an
distance of the object is angle θ to the horizontal. The angle θ for which the
(a) 18 cm (b) 36 cm reflected ray becomes vertical, is
(c) 48 cm (d) ∞. (a) 40° (b) 80°

[All India PM/PD 1999] (c) 55° (d) 100°. [Pb. CET 1996]
651. The field of view is maximum for 657. Fig. 68 shows two plane
(a) plane mirror (b) concave mirror mirrors and an object O
(c) convex mirror (d) cylindrical mirror. placed between them. What O
[Orissa 1995] will be distance of the first 8 cm 2 cm
three images from the mir-
652. An object is placed at a distance of 25 cm from the
ror M2 ?
pole of a convex mirror and a plane mirror is set so
(a) 2 cm, 8 cm, 14 cm
that the virtual images formed by two mirrors do M1 M2
not have any parallax. The plane mirror is 20 cm (b) 2 cm, 12 cm 18 cm
Fig. 68
from the object. The focal length of the mirror is (c) 2 cm, 18 cm, 22 cm
(a) 37.5 cm (b) – 7.5 cm (d) 2 cm, 24 cm, 38 cm. [MNR 2001]
(c) – 37.5 cm (d) + 75 cm. [AMU 2002] 658. When an object is kept at a distance of 30 cm from a
concave mirror, the image is formed at a distance of
508 COMPREHENSIVE OBJECTIVE PHYSICS

10 cm. if the object is moved with a speed of 9 m s–1, 665. Two plane mirrors are placed at
the speed with which image moves is right angle to each other. A ray
(a) 0.1 m s–1 (b) 1 m s–1 strikes one mirror at an angle
(c) 3 m s–1 (d) 9 m s–1. of incidence i, such that it is also
[National Standard Exam. in Physics 2001] reflected from the second
659. Two plane mirrors are in- mirror. The reflected ray is
M2
clined to one another at an (a) parallel to the ray incident on the
angle of 60°. A ray is inci- first mirror and the two are in the
dent on mirror M 1 at an opposite directions. Fig. 70
angle i. The reflected ray (b) inclined at an angle 2i with ray incident on first mirror.
from mirror M2 is parallel i
i (c) parallel to the ray incident on the first mirror and the two
to mirror M1 as shown in 60° are in the same direction.
Fig. 69. The angle of inci- O M1
dence i is (d) inclined at an angle i with incident ray on first mirror.
Fig. 69
(a) 20° (b) 10° [CMC Vellore 1998]
(c) 30° (d) 40°. [AFMC 1995] 666. A spherical mirror forms a real image of a point object
660. A person standing in the centre of a room, 12 m high placed in front. The distance of the image and object
looks into a plane mirror fixed on the wall. Then the from the mirror is 30 cm and 0.2 m respectively. The
minimum length of the plane mirror for him to see the focal length and nature of the mirror are
full length image of the wall behind him, is equal to (a) 120 mm ; concave (b) 120 mm ; convex
(a) 12 m (b) 8 m (c) 60 cm ; concave (d) 60 cm ; convex.
(c) 4 m (d) 6 m. [MNR 1999]
[All India PM/PD 1993]
667. A short linear object of length b lies along the axis of
661. A convex mirror of radius of curvature 1.6 m has an a concave mirror of focal length f at a distance u
object placed at a distance of 1 m from it. The image from the pole of the mirror. The size of the image is
is formed at a distance of equal to
(a) 8/13 m in front of the mirror
(b) 8/13 m behind the mirror
FG u − f IJ 1/2 FG f IJ 1/2
(c) 4/9 m in front of the mirror
(a) b
H f K (b) b
Hu− f K
(d) 4/9 m behind the mirror. [AMU 1993] F u − f IJ
(c) b G (d) b G
F f IJ 2 .
662. In the above question the magnification is H f K H f − uK [IIT 1988]

(a) 4/9 (b) – 4/9 668. An object 5 cm tall is placed 1 m from a concave
(c) 9/4 (d) 8/13. spherical mirror which has a radius of curvature of
663. The sun has a diameter of 864100 mile and is on an 20 cm. The size of the image is
average at a distance of 92900000 mile from the earth. (a) 0.11 cm (b) 0.50 cm
A concave mirror of radius 6 m is used to obtain its (c) 0.55 cm (d) 0.60 cm.
image. The diameter of the image of the solar disc is [CMC LDH 2002]
(a) 28 m (b) 28 cm
669. A concave mirror is used to form an image of the
(c) 28 mm (d) 28 × 10–1 mm.
Sun on a white screen. If the lower half of the mirror
[PB.CET 1998] were covered with an opaque card, the effect on the
664. A convex mirror of focal length f produces an image image on the screen would be
1/n th of the size of the object. The distance of the (a) negligible
object from the mirror is
(b) to make the image less bright than before
f (c) to make the upper half of the image disappear
(a) (b) (n – 1) f
n−1 (d) to make the lower half of the image disappear
F n − 1IJ f.
(d) G (e) to prevent the image from being focussed.
(c) nf – 1
H n K
[National Standard Exam. in Physics 2002]
OPTICS 509
670. From a spherical mirror, the graph of 1/v vs 1/u is 675. A small piece of wire bent into an L shape with up-
right and horizontal portions of equal lengths, is
v 1/v placed with the horizontal portion along the axis of
the concave mirror whose radius of curvature is
10 cm. If the bend is 20 cm from the pole of the mir-
ror, then the ratio of the lengths of the images of the
upright and horizontal portions of the wire is
(a) 1 : 2 (b) 3 : 1

O (a) u O (b) 1/u (c) 1 : 3 (d) 2 : 1. [MNR 2000]


676. A concave mirror of focal length f forms an erect im-
1/v
1/v age of twice the size of the object. The object distance
from the mirror is
(a) f/2 (b) f/4
(c) 3f/2 (d) 2f.
677. The image formed by convex mirror of focal length
30 cm is a quarter of the size of the object. Then the
O (c) 1/u 1/u
O (d) distance of the object from the mirror is
(a) 30 cm (b) 90 cm
Fig. 71
(c) 120 cm [WB JEE 2003]
(d) 60 cm.
671. A convex mirror has a focal length f. A real object is 678. An object is placed symmetrically between two plane
placed at a distance f in front of it from the pole. The mirrors inclined at an angle of 90°. Then the total
mirror produces an image at number of images formed is
(a) Infinity (b) f (a) 5 (b) 3
(c) f/2 (d) 2f. (c) 2 (d) Infinite. [WB JEE 1999]
672. A convex mirror and a concave mirror of radius 679. The image of an object placed on the principal axis of
10 cm each are placed 15 cm apart facing each other. a concave mirror of focal length 12 cm is formed at a
point which is 10 cm more distant from the mirror
An object is placed midway between them. If the
than the object. The magnification of the image is
reflection first takes place in the concave mirror and
then in convex mirror, the position of the final image (a) 8/3 (b) 2.5
is (c) 2 (d) – 1.5. [JIPMER 2002]
(a) on the pole of the convex mirror 680. For a convex mirror, the variation of u vs v is given
(b) on the pole of the concave mirror by
(c) at a distance of 10 cm from convex mirror v v
(d) at a distance of 5 cm from concave mirror.
673. The image of a luminous point object O placed on the
principal axis of a concave mirror distant 24 cm from
it coincides with itself. When a few drops of a liquid
are placed on the mirror, the object has to be placed
15 cm from the mirror to make its image coincide
with it. The refractive index of the liquid is O (a) u O (b) u
24 24 − 15 v
(a) (b) v
15 15
24 − 15 15
(c) (d) . [CPMT 2000]
15 + 24 15 + 24
674. A driving mirror consists of a cylindrical mirror of
radius 10 cm and the length, over the curved sur-
face, of 10 cm. If the eye of the driver be assumed to
be at a very large distance from the mirror, then the O (c) u O (d) u
field of view is
(a) 6 radian (b) 4 radian Fig. 72

(c) 2 radian (d) 1 radian. [DCE 1995] [EAMCET 1996]


510 COMPREHENSIVE OBJECTIVE PHYSICS

681. An object of length 4 cm is kept on the principal axis (a) – 3R (b) + 3R


of a convex mirror at a distance f. The size of the (c) – R (d) + R.
image formed is
[National Standard Exam. in Physics 1998]
(a) 2 cm (b) 8 cm
688. A light ray is incident perpendi-
(c) 6 cm (d) 4 cm.
cularly to one face of a 90°
[Bharati Vidyapeeth 2002] prism and is totally internally
682. A point object is placed midway between two plane reflected at the glass-air inter- 45°
mirrors distance ‘a’ apart. The plane mirrors form face. If the angle of reflection
45°
an infinite number of images due to multiple reflec- is 45°, we conclude that the re-
tions. The distance between nth order image formed fractive index n Fig. 74
in the two mirrors is
1
(a) na (b) 2 na (a) n > (b) n >
2
na
(c) (d) n2 a. 2
2
[Haryana PMT 2002] 1
(c) n < (d) n < 2 [AIEEE 2004]
683. To get three images of a single object, one should 2
have two plane mirrors at an angle of 689. A plano-convex lens of refractive index 1.5 and ra-
(a) 30° (b) 60° dius of curvature 30 cm is silvered at the curved sur-
(c) 90° (d) 120°. [AIEEE 2003] face. Now this lens has been used to form the image
of an object. At what distance from this lens an ob-
REFRACTION ject be placed in order to have a real image of the size
of the object ?
684. White light is incident on the
(a) 60 cm (b) 30 cm
interface of glass and air as
shown in Fig. 73. If green light (c) 20 cm (d) 80 cm [AIEEE 2004]
Air Green
is just totally internally re- Glass 690. A point object O is placed on the principal axis of a
flected then the emerging ray convex lens of focal length 20 cm at a distance of
ite

in air contains 40 cm to the left of it. The diameter of the lens is


Wh

(a) yellow, orange, red 10 cm. If the eye is placed 60 cm to the right of the
Fig. 73
(b) violet, indigo, blue lens at a distance h below the principal axis, then the
(c) all colours (d) all colours except green. maximum value of h to see the image will be
[IIT Screening 2004] (a) 0 (b) 5 cm
685. A point object is placed at the centre of a glass sphere (c) 2.5 cm (d) 10 cm.
of radius 6 cm and refractive index 1.5. The distance [MP PMT 1999]
of the virtual image from the surface of the sphere is 691. A ray of light from a denser medium strikes a rarer
(a) 2 cm (b) 4 cm medium at an angle of incidence i. The reflected and
(c) 6 cm (d) 12 cm refracted rays make an angle of π/2 with each other.
[IIT Screening 2004] If the angles of reflection and refraction are r and r′,
686. A convex lens of refractive index n behaves as a con- then the critical angle will be
vex lens of smaller power in a liquid of refractive (a) tan–1 (sin i) (b) sin–1 (sin r)
index n1 and as a concave lens in another liquid of (c) sin–1 (tan i) (d) sin–1 (tan r′).
refractive index n2. The relation between n, n1 and
[MP PMT 1999]
n2 is
692. Two optical media of refractive indices n1 and n2 con-
(a) n2 > n > n1 (b) n2 < n < n1
tain x and y waves of the same colour in the same
(c) n = n1 = n2 (d) n > n1 > n2. thickness. Then their relative refractive index 1n2 is
[Karnataka 1999] equal to
687. A clear transparent glass sphere (µ = 1.5) of radius R y
(a) x – y (b)
is immersed in a liquid of refractive index 1.25. A x
parallel beam of light incident on it will converge to y−x x
(c) (d) .
a point. The distance of this point from the centre x y−x
will be [Karnataka 1999]
OPTICS 511
693. The plane faces of two identical plano-convex lenses, (a) 60° (b) 45°
each having focal length of 40 cm, are placed against (c) 90° (d) 180°.
each other to form a common convex lens. The dis- [CMC Vellore 2000]
tance from this lens at which an object must be placed 699. By how much an ink dot appears to be raised when
to obtain a real, inverted image with magnification covered by a glass plate 4.5 cm thick if the velocity of
equal to unity is light in glass is 2 × 1010 cm s–1 and in air 3 × 1010 cm
(a) 80 cm (b) 40 cm s–1 ?
(c) 20 cm (d) 16 cm. [MP PMT 1999] (a) 1.5 cm (b) 3 cm
694. An equiconvex lens of glass of focal length 0.1 metre (c) 4.5 cm (d) 0.
is cut along a plane perpendicular to principal axis 700. A ray of light enters a rectangular glass slab of
into two equal parts. The ratio of focal lengths of
refractive index 3 at an angle of incidence 60°. It
new lenses formed is
travels a distance of 5 cm inside the slab and emerges
(a) 1 : 1 (b) 1 : 2
out of the slab. The perpendicular distance between
(c) 2 : 1 [MP PET 1999]
(d) 2 : 1/2. the incident and the emergent rays is
695. A lens of refractive index n is put in a liquid of refrac- 5
tive index n′. If focal length of lens in air is f, its focal (a) 5 3 cm (b) cm
2
length in liquid will be
3
fn′ (n − 1) f (n′ − n) (c) 5 cm (d) 5 cm.
(a) (b) 2
n′ − n n′ (n − 1)
701. Rays from a lens are converging towards a point.
n′ (n − 1) fn ′n What thickness t of glass (µ = 1.5) must be interposed
(c) (d) . [MP PET 1999]
f (n′ − n) n − n′ in order that the image shall be shifted away through
696. Refractive index of water relative to air is 1.33 and
1″
that of an oil with respect to air is 1.45. The refrac- ?
tive indices of oil with respect to water and of water 8
with respect to oil are 3′′ 3′′
(a) (b)
(a) 0.91, 1.09 (b) 1.09, 0.92 4 8
(c) 0.75, 1.07 (d) 1.07, 0.75. 3′′ 2′′
(c) (d) .
[Pb. CET 1993] 7 7
702. A ray of monochromatic light is incident on the
697. A fish in a pond of water (µ = 1.33) views the setting
refracting face of a prism (angle 75°). It passes through
sun. According to fish, the sun makes an angle of
the prism and is incident on the other face at the
(with the horizon) :
critical angle. If the refractive index of the prism is
[sin–1 (1/1.33) = 48.6°]
2 , then the angle of incidence on the first face of
(a) 0° (b) 90°
the prism is
(c) 48.6° (d) 41.4°. [AFMC 1993]
(a) 15° (b) 30°
3 FG IJ (c) 45° (d) 60°.
698. A ray of light travelling in glass µ =
2 H
is incident
K 703. A layer of benzene (µ = 1.5) 6 cm deep floats on water
on a horizontal glass air surface at the critical angle FG µ = 4 IJ , 4 cm deep. When viewed vertically through
4 FG IJ H 3K
θC . If a thin layer of water µ =
3 H K
is now poured on
air, the apparent distance of the bottom of the vessel
the glass air surface, the angle at which the ray below the free suface of benzene is
emerges into air at the water-air surface is (a) 14 cm (b) 7 cm
Water (c) 21 cm (d) 3.5 cm.
704. The focal length of a convex lens is f. An object is
placed at a distance x from its first focal point. The
R
Glass ratio of the size of real image to that of the object is
R f x2
(a) 2
(b)
x f
f x
(c) (d) .
Fig. 75 x f
512 COMPREHENSIVE OBJECTIVE PHYSICS

705. A point source is located 275 cm below the surface of 711. Two transparent media A and B are separated by a
a lake. The area of the surface that transmits all the plane boundary. The speed of light in medium A is
light that emerges from the surface is 2.0 × 108 m s–1 and in medium B is 2.5 × 108 m s–1.
(a) 0.304 m2 (b) 3.04 m2 The critical angle for which ray of light going from A
(c) 30.5 m2 (d) 304 m2. to B is totally internally reflected is
(a) sin–1 1/2 (b) sin–1 2/5
706. A plano-convex lens when silvered on the plane side
(c) sin–1 5/2 (d) sin–1 4/5.
behaves like a concave mirror of focal length 60 cm.
However, when silvered on the convex side it behaves 712. A fish is a little away be- Air
water
low the surface of a lake.
like a concave mirror of focal length 20 cm. Then the θ
If the critical angle is 49°
refractive index of the lens is
49°, then the fish could
(a) 3.0 (b) 1.5 see things above water
(c) 1.0 (d) 2.0. [MNR 2002] surface within an angu-
Fig. 77
707. An object is placed at a distance of 15 cm from a lar range of θ° where
convex lens of focal length 10 cm. On the other side (a) θ = 49° (b) θ = 98°
of the lens, a convex mirror is placed at its focus 1
(c) θ = 24 ° (d) θ = 90°.
such that the image formed by the combination coin- 2
cides with the object itself. The focal length of the 713. A light source placed at the bottom of a water beaker
convex mirror is 9 cm deep forms an illuminated circle of radius 11
(a) 20 cm (b) 10 cm cm at its surface. If the depth of the water in the
beaker is increased to 18 cm, the radius of the illumi-
(c) 15 cm (d) 30 cm. [AMU 2000]
nated circle will be
708. ACB is a right-angled prism with other angles as 60° (a) 16 cm (b) 30 cm
and 30°. Refractive index A B (c) 22 cm (d) 44 cm.
of prism is 1.5. AB has a 60° 30°
[CMC LDH 1998]
thin layer of liquid on it as
714. What is the relation be-
shown. Light falls nor- µ3
C tween refractive indices
mally on the face AC. For
µ1, µ2 and µ3 if the behav- µ1 µ2
total internal reflection, Fig. 76
iour of light rays is as
maximum refractive index
shown in Fig 135.
of liquid is
(a) µ3 < µ2, µ2 = µ1
(a) 1.4 (b) 1.3
(b) µ2 < µ1, µ2 = µ3
(c) 1.2 (d) 1.6. [AMU 2001] Fig. 78
(c) µ3 < µ2 < µ1
709. The distance between the convex lens and plane (d) µ3 > µ2 > µ1.
mirror is 5 cm. The parallel rays of light are incident
715. A lens of power + 2 D is placed in contact with a lens
on the convex lens. After reflection from the plane
mirror, the final image is formed at the optical centre of power – 1D. The combination will behave like
of the lens. The focal length of the lens is (a) convergent lens of focal length 50 cm
(a) 10 cm (b) 20 cm (b) convergent lens of focal length 100 cm
(c) 40 cm (d) 5 cm. (c) divergent lens of focal length 50 cm
[Bharati Vidyapeeth 1999] (d) divergent lens of focal length 100 cm. [RPMT 1995]
710. A rectangular block of glass of refractive index µ and 716. A ray of light falls on the surface of a spherical glass
thickness d is placed over an ink dot made on a piece paper weight making an angle alpha with the normal
of paper lying on a table. When viewed from above, and is refracted in the medium at an angle beta. The
the dot will appear to be raised by an amount given
angle of deviation of the emergent ray from the di-
by
rection of the incident ray is
d d
(a) (µ – 1) (b) (µ + 1) (a) (alpha – beta) (b) (beta – alpha)
µ µ
1
µd µd (c) (alpha – beta) (d) 2 (alpha – beta).
(c) (d) . 2
(µ − 1) (µ + 1) [MP PET 1995]
OPTICS 513
717. Shown in the Fig. 79 here µ = 1.5 µ = 1.6 723. Two thin convex lenses of focal lengths 10 cm and
is a convergent lens placed 15 cm are separated by 25 cm. The optical interval or
inside a cell filled with a separation between the two lenses is
liquid. The lens has focal (a) 0 (b) ∞
length + 20 cm when in air f + f2
and its material has (c) f1 + f2 (d) 1 .
d
refractive index 1.50. If the
724. A bird in air looks at a fish vertically below it and
liquid has refractive index
inside water. x is the height of the bird above the
1.60, the focal length of the
surface of water and y is the depth of the fish below
system is Fig. 79 the surface of water. The distance of the fish as ob-
(a) + 80 cm (b) – 80 cm served by the bird is : (Given : µ = refractive index of
(c) – 24 cm (d) – 100 cm. water w.r.t. air)
[National Standard Exam. in Physics 1991, 1994] y
(a) x + y (b) x +
718. Light is incident from a medium into air at two µ
possible angles of incidence (1) 20° and (2) 40°. In the (c) µx + y (d) µx + µy.
medium, the light travels 3 cm in 0.2 ns. The ray
will 725. In the previous question, the distance of the bird as
(a) suffer total internal reflection in both cases 1 and 2
observed by the fish is
(b) suffer total internal reflection in case 2 only y
(a) x + y (b) x +
µ
(c) have 100% transmission in case 1
(c) µx + y (d) µx + µy.
(d) have partial reflection and partial transmission in case 2.
726. An air bubble in a glass slab (µ = 1.5) is 6 cm deep as
719. A liquid of refractive index 1.6 P Q
viewed from one face and 4 cm deep as viewed from
is introduced between two iden-
the other face. The thickness of the glass slab is
tical plano-convex lenses in two
(a) 6.67 cm (b) 10 cm
ways P, Q as shown. If lens ma-
(c) 15 cm (d) Data is incomplete.
terial has refractive index 1.5,
the combination is 727. The focal length of a convex lens of glass (µ = 1.5) is
(a) convergent in both 2 cm. The focal length of the lens when immersed in
a liquid of refractive index 1.25 will be
(b) divergent in both
(a) 5 cm (b) 2.4 cm
(c) convergent in Q only
Fig. 80 (c) 1 cm (d) 4 cm.
(d) convergent in P only.
728. A and B are two identical beakers. Beaker A is filled
[National Standard Exam. in Physics 1992]
till brim with liquid (µ = 1.3) and beaker B is filled
720. Two convex lenses of powers 4 D and 6 D are sepa- with some liquid (µ = 1.6). Both beakers are viewed
rated by a distance of
1
m. The power of the optical from directly above. The ratio of the apparent depths
6
of A and B is
system so formed is
(a) 1.3/1.6 (b)1.6/1.3
(a) – 6 D (b) + 6 D
(c) 1 (d) 1.3 × 1.6.
(c) 10 D (d) 2 D .
[JIPMER 1996]
721. A concave lens of focal length 1 metre produces an
1 729. Monochromatic light of wavelength λ gets refracted
image times the size of the object. The distance, in from vacuum to a medium of refractive index µ. The
y ratio of the wavelength of the incident and the
metre, of the object from the lens is
refracted wave is
(a) y (b) y + 1
(a) 1 : µ (b) 1 : 1
1
(c) y – 1 (d) . (c) µ : 1 (d) µ2 : 1. [AFMC 1998]
y
730. The distance between an object and a screen is
722. For two different positions of the lens in the displace-
ment method, the sizes of the images are 1 unit and 100 cm. A lens produces an image on the screen when
3 unit. The size of the object is placed at either of the two positions 40 cm apart. The
(a) 4 unit (b) 3 units power of the lens is nearly
(a) 3 D (b) 5 D
(c) 3 units (d) 2 units.
(c) 7 D (d) 9 D.
514 COMPREHENSIVE OBJECTIVE PHYSICS

731. A point source of light is placed 4 m below the sur- 739. The critical angle between an equilateral prism and
5 air is 42°. If the incident ray is perpendicular to the
face of water of µ = . The minimum diameter of a refracting surface, then
3
(a) after deviation, it will emerge from the second refracting
disc, which should be placed over the source, on the surface
surface of water to cut off all light coming out of
(b) it is totally reflected on the second surface and emerges out
water, is perpendicularly from third surface in air
(a) 1 m (b) 6 m
(c) it is totally reflected from the second and third refracting
(c) 4 m (d) 3 m.
surfaces and finally emerges out from the first surface
732. The radius of curvature of convex surface of a thin (d) it is totally reflected from all the three sides of prism and
plano-convex lens is 15 cm and µ is 1.6. The power of never emerges out. [MP PMT 1986]
the lens is
740. Angle of prism is A and its one surface is silvered.
(a) + 1 D (b) – 2 D
Light rays falling at an angle of incidence 2A on first
(c) + 3 D (d) + 4 D. surface return back through the same path after
733. A convexo-concave lens has faces of radii 3 cm and suffering reflection at second silvered surface.
4 cm respectively and is made of glass of µ = 1.6. The Refractive index of the material of prism is
focal length of the lens is (a) 2 sin A (b) 2 cos A
(a) 20 cm (b) 30 cm 1
(c) 40 cm (d) 50 cm. (c) cos A (d) tan A. [AIIMS 1995]
2
734. Two symmetric double convex lenses A and B have 741. A luminous object is separated from a screen by a
the same focal length but the radii of curvature dif- distance D. What is the greatest focal length a lens
fer so that RA = 0.9 RB. If µA = 1.63, then µB is could have to focus the object on the screen ?
(a) 1.1 (b) 1.5 D D
(a) (b)
(c) 1.6 (d) 1.7. 4 2
(c) D (d) 4D.
735. A ray of light passes from vacuum into a medium of
refractive index µ . The angle of incidence is twice the [Karnataka CET 2001]
angle of refraction. Angle of incidence is 742. The focal length of a plano-convex lens is equal to its
FG IJ µ µ radius of curvature. The value of the refractive index
(a) cos–1
H K 2
(b) 2 cos–1
2
of its material is
F µI
(c) 2 sin G J
(a) 1.33 (b) 1.6
H 2K–1
(d) 2 sin–1 (µ). (c) 1.5 (d) 2.
736. When a ray is refracted from one medium into another [Karnataka CET 1997]
medium, the wavelength changes from 6000 Å to 743. Light of wavelength 500 nm travelling with a speed
4000 Å. The critical angle for a ray from second 2.0 × 108 m s–1 in a certain medium enters another
medium will be medium of refractive index 5/4 times that of the first
(a) cos–1 (2/3) (b) sin–1 (2/3) medium. What are the wavelength and speed in the
(c) tan (3/2)
–1
(d) sin–1 (2/ 13 ). second medium ?
737. If the critical angle for total internal reflection from wavelength/nm speed/m s–1
a medium to vacuum is 30°, the velocity of light in (a) 400 1.6 × 108
the medium is (b) 400 2.5 × 108
(a) 3 × 10 m s 8 –1
(b) 1.5 × 10 m s
8 –1
(c) 500 2.5 × 108
(c) 6 × 108 m s–1 (d) 3 × 108 m s–1. (d) 625 1.6 × 108
[AMU 1996] (e) 625 2.0 × 108.
738. The relation between n1 and 744. Light passes from air into a liquid. The angle of
n2 if the behaviour of light incidence is 60°. The deviation produced is 15°. The
ray is as shown in the Fig. 81 refractive index of the liquid is
n1 n2
is (a) 1.5 (b) 1.33
(a) n2 > n1 (c) 1.22 (d) 1.63.
(b) n1 > > n2 Lens 745. The angle of refraction of a prism is 35°. The critical
(c) n1 > n2 Fig. 81 angle is 30°. A ray should pass through a prism such
(d) n1 = n2. [Karnataka CET 2000]
OPTICS 515
that there should not be total internal reflection. The 750. A parallel beam of light falls axially on a thin con-
minimum angle of incidence should not be less than verging lens of focal length 20 cm. The emergent light
(a) 30° (b) 20° falls or a screen placed 30 cm beyond the lens. An
(c) 15° (d) 10°. opaque plate with a triangular aperture, side 1 cm, is
in contact with the lens. See Figs. 83 (a) and (b).
[All India PM/PD 1997]
f = 20 cm
746. A hollow double concave lens is made of very thin
transparent material. It can be filled with air or either Triangular
aperture
of two liquids L1 or L2 having refractive indices n1
and n2 respectively (n2 > n1 > 1). The lens will diverge
a parallel beam of light if it is filled with Opaque
(a) air and placed in air (b) air and immersed in L1 plate Screen Aperture side
1.0 cm
(c) L1 and immersed in L2 (d) L2 and immersed in L1. 30 cm
(a) (b)
[IIT Screening 2000]
747. Two thin lenses are placed 5 cm apart along the same Fig. 83
axis and illuminated with a beam of light parallel to Which one of the following diagrams best shows to
that axis. The first lens in the path of the beam is a appearance of the patch of light seen on the screen.
converging lens of focal length 10 cm whereas the
second is a diverging lens of focal length 5 cm. If the
second lens is now moved towards the first, the emer-
gent light
(a) remains parallel (b) remains convergent
(c) remains divergent
(d) changes from parallel to divergent. 1.5 cm 1.0 cm 0.5 cm
(e) changes from convergent to divergent. (a) (b) (c)
748. Two similar plano-convex lenses are combined
together in three different ways as shown in Fig. 82.
The ratio of the focal lengths in three cases will be

0.5 cm 1.5 cm
(d) (e)
Fig. 84
(i) (ii) (iii) 751. A lens forms a real image of an object. The distance
from the object to the lens is x cm and that from the
Fig. 82 lens to the image is y cm. The graph (Fig. 85) shows
(a) 2 : 2 : 1 (b) 1 : 1 : 1 the variation of y with x. It can be deduced that the
(c) 1 : 2 : 2 (d) 2 : 1 : 1. lens is
[CMC LDH 1994]
10
749. An object is placed 30 cm to the left of a diverging
lens whose focal length is of magnitude 20 cm. Which
one of the following correctly states the nature and 20
position of the virtual image formed ?
nature of image distance from lens 40
(a) inverted, enlarged 60 cm to the right
(b) erect, diminished 12 cm to the left 20

(c) inverted, enlarged 60 cm to the left


10
(d) erect, diminished 12 cm to the right 20 40 20 10
(e) inverted, enlarged 12 cm to the left Fig. 85
516 COMPREHENSIVE OBJECTIVE PHYSICS

(a) converging and of focal length 10 cm. 1. speed of light in medium Y is 3 times that in
(b) converging and of focal length 20 cm. medium X.
(c) converging and of focal length 40 cm 2. speed of light in me-
0.2
(d) diverging and of focal length 20 cm. dium Y is 1/ 3 times
Sin r
(e) diverging and of focal length 10 cm. that in medium X.
30° Sin i
[London Schools Exam. Deptt.] 3. total internal reflec- 0 0.2 0.4
tion will occur above a
752. The focal power in dioptres of the contact combina- certain i value. Fig. 88
tion of a convex lens of focal length 10 cm and a (a) 2 and 3 (b) 1 and 3
concave lens of focal length 25 cm is
(c) 2 only (d) 3 only.
(a) 4 (b) 0.25
[National Standard Exam. in Physics 1989]
(c) 6 (d) 10. 757. A thin symmetric convex lens of refractive index 3/2
[National Standard Exam. in Physics 1989] and radius of curvature 30 m is put in water
753. The distance between an object and a divergent lens FG refractive index = 4 IJ . Its focal length is
is ‘m’ times the focal length of the lens. The linear
magnification produced by the lens will be equal to
H 3K
(a) 0.15 m (b) 0.30 m
(a) m (b) 1/m
(c) 0.45 m (d) 1.20 m.
(c) m + 1 (d) 1/(m + 1).
[All India PM/PD 1996]
[National Standard Exam. in Physics 2001]
758. A 16 cm long image of an object is formed by a con-
754. Fig. 86, shows positions of an
vex lens on a screen. On moving the lens towards the
image I of an object ‘O’ formed
screen, a 9 cm long image is formed again on the
by a lens. This is possible if
screen. The size of the object is
(a) a convex lens is placed to the left Ι
O (a) 9 cm (b) 11 cm
of O
Fig. 86 (c) 12 cm (d) 13 m.
(b) a concave lens is placed to the left
of O. [Bharati Vidyapeeth 2001]
(c) a convex lens is placed between O and I 759. A convex lens of focal length 12 cm is made of glass
3
(d) a concave lens is placed to the right of I. of µ = . What will be its focal length when immersed
2
[National Standard Exam. in Physics 2000] 5
in liquid of µ = ?
755. A liquid of Refractive index 1.6 4
Glass (a) 6 cm (b) 12 cm
is contained in the cavity of a
(c) 24 cm (d) 30 cm. [MP PMT 1995]
glass specimen of refractive index
1.5 as shown in Fig. 87. If each 760. The wavelength of light in two liquids ‘x’ and ‘y’ is
of the curved surfaces has a
Liquid 3500 Å and 7000 Å. Then the critical angle of x rela-
radius of curvature of 0.20 m, the tive to y will be
Fig. 87
arrangement behaves as a (a) 60° (b) 45°
(c) 30° (d) 15°.
(a) converging lens of focal length 0.25 m.
(b) diverging lens of focal length 0.25 m. [Himachal PMT 2001]
(c) diverging lens of focal length 0.17 m. 761. Fig. 89 (a) shows two plano-
(d) converging lens of focal length 0.72 m. convex lenses in contact as
shown. The combination has
[National Standard Exam. in Physics 1997]
focal length 24 cm. Fig. 89 (b)
756. Light is incident from a medium X at angle of shows the same with a liquid
incidence i and is refracted into a medium Y at angle introduced between them. If
of refraction r. The graph sin i versus sin r is shown refractive index of glass of the
in Fig. 88. Which of the following conclusions would lenses is 1.50 and that of the
fit the situation ? liquid is 1.60, the focal length
(a) (b)
of system in Fig. 89 (b) will be
Fig. 89
OPTICS 517
(a) – 120 cm (b) 120 cm 767. In a slide show programme, the image on the screen
(c) – 24 cm (d) 24 cm. has an area 900 times that of the slide. If the dis-
tance between the slide and the screen is x times the
[National Standard Exam. in Physics 1995]
distance between the slide and the projector lens, then
762. When a lens made of glass (refractive index 1.5) is (a) x = 30 (b) x = 31
immersed completely in water (refractive index 1.3) 1
its focal length changes from f to (c) x = 500. (d) x = .
30
(a) (15/13)f (b) (15/4)f [National Standard Exam. in Physics 2000]
(c) (13/4)f (d) (5/3)f. 768. For a colour of light, the wavelength in air is 6000 Å
[National Standard Exam. in Physics 1989] and in water the wavelength is 4500 Å. Then the speed
763. Fig. 90 shows different (II)
of light in water will be
(III) (a) 5.0 × 1014 m s–1 (b) 2.25 × 108 m s–1
graphs in case of a thin con-
vex lens for the formation (I) (c) 4.0 × 10 m s8 –1
(d) Zero.
of a real image. ‘u’ and ‘v’ (IV) [Haryana PMT 1999]
y
are the object and image
769. A combination of two thin lenses with focal lengths
distances respectively. Find f1 and f2 respectively forms an image of distant ob-
out the correct identifica- ject at distance 60 cm when lenses are in contact.
tion for the different com- The position of this image shifts by 30 cm towards
x
binations from the the combination when two lenses are separated by
Fig. 90
followings 10 cm. The corresponding values of f1 and f2 are
(a) graph I, if x is ‘v’ and y is ‘u/v’ (a) 30 cm, – 60 cm (b) 20 cm, – 30 cm
(b) graph II, x is ‘u’ and y is ‘v’ (c) 15 cm, – 20 cm (d) 12 cm, – 15 cm.
(c) graph III, if x is ‘v’ and y is ‘v/u’ [AIIMS 1995]
(d) graph IV, if x is ‘1/u’ and y is ‘1/v’.
770. The image of a star (effectively a point source) is made
[National Standard Exam. in Physics 2001] by a convergent lens of focal length 50 cm and diam-
764. A parallel beam of light falls normally on the plane eter of aperture 5.0 cm, if the lens is ideal, and the
surface of a Plano convex lens of refractive index effective wavelength in image formation is taken as
3/2. If the radius of the curved surface of the lens is 5 × 10–5 cm, the diameter of the image formed will be
20 cm, the beam will be focussed at which of the nearest to
following distances from the lens ? (a) zero (b) 10–6 cm
(a) 20 cm (b) 30 cm (c) 10 cm–5
(d) 10–3 cm.
(c) 40 cm (d) 50 cm. [National Standard Exam. in Physics 1994]
[CMC Vellore 1995] 771. A convex lens of focal length 20 cm and a concave
765. The velocity of light in a medium is half its velocity lens of focal length f are mounted coaxially 5 cm apart.
in air. If a ray of light emerges from such a medium Parallel beam of light incident on the convex lens
into air, the angle of incidence, at which it will be emerges from the concave lens as a parallel beam.
totally internally reflected, is Then f in cm is
(a) 15° (b) 30° (a) 35 (b) 25

(c) 45° (d) 60°. [BHU 2000] (c) 20 (d) 15. [MANIPAL 1999]
766. A thin equiconvex lens has focal length 10 cm, 772. A lens is placed between a source of light and a wall.
refractive index 1.5. One of its faces is now silvered, It forms images of area A1 and A2 on the wall, for its
and for an object placed at a distance u in front of it, two different positions. The area of the source of light
the image coincides with the object. The value of u is is
A1 + A2
(a) 10 cm (b) 5 cm (a) A 1A 2 (b)
2
(c) 20 cm (d) 15 cm.
A1 − A2 1 1
[National Standard Exam. in Physics 1988] (c) (d) + .
2 A1 A 2
[All India PM/PD 1995]
518 COMPREHENSIVE OBJECTIVE PHYSICS

A
773. A glass prism has refractive index 2 and refracting 0.8 × 10 −14 sin θ = 0.8
angle 30°. One of the refracting surfaces of the prism µ(λ) = 1.20 + θ
λ2
is silvered. A beam of monochromatic light will
Which of the following statements
retrace its path if its angle of incidence on the
is correct ?
unsilvered refracting surface of the prism is
(a) Light of λ = 400 nm undergoes total
(a) 0 (b) π/6 internal reflection B C
(c) π/4 (d) π/3. (b) Light of λ = 500 nm undergoes total
Fig. 91
[All India PM/PD 1992] internal reflection
(c) Neither of the two wavelengths undergo total internal
774. When a glass slab is placed on a cross made on a
reflection
sheet, the cross appears raised by 1 cm. The thick-
(d) Both wavelengths undergo total internal reflection.
ness of the glass is 3 cm. The critical angle for glass
[All India PM/PD 1995]
is
(a) sin–1 (0.33) (b) sin–1 (0.5) 781. A rectangular block of glass is placed on a printed
page lying on a horizontal surface. Then the mini-
(c) sin–1 (0.67) (d) sin–1 ( 3 /2). mum value of refractive index of glass for which the
775. A diver at a depth of 12 m in water (µ = 4/3) sees the letters on the page are not visible from any of the
sky in a cone of semivertical angle vertical faces of the block is
(a) sin–1 (4/3) (b) tan–1 (4/3) (a) equal to 2 (b) more than 2
(c) sin–1 (3/4) (d) 90°. [MP PMT 1995] (c) less than 2 (d) > = < 2.
776. A diminished image of an object is to be obtained on [CMC LDH 1996]
a screen 1.0 m from it. This can be achieved by 782. The sun subtends an angle of 30′ on the surface of
appropriately placing the earth. A convex lens of focal length 100 cm is
(a) a convex mirror of suitable focal length used to form its image. The diameter of the image of
(b) a concave mirror of suitable focal length the solar disc is
(c) a concave lens of suitable focal length (a) 1.74 cm (b) 0.87 cm
(d) a convex lens of suitable focal length less than 0.25 m. (c) 0.435 cm (d) 100 cm.
783. If a ray of light in a denser medium strikes a rarer
[IIT 1995]
medium at an angle of incidence i, the angles of re-
777. Light of λ = 589 nm traverses a tank of height 20.06 m flection and refraction are respectively r and r′. If
first filled with glycerine (µ = 1.47) and then carbon the reflected and refracted rays are at right angles to
disulphide. The difference in the times taken to each other, the critical angle for the given pair of
traverse the tank is 1.07 × 10 –8 s. The refractive index media is
of carbon disulphide is (a) sin–1 (tan r′) (b) sin–1 (tan r)
(a) 1.60 (b) 1.62 (c) tan–1 (sin i) (d) cot–1 (tan i).
(c) 1.58 (d) 1.63. [BHU 1997] [Karnataka CET 2003]
778. Monochromatic light of frequency 5 × 1014 Hz travel- 784. A light wave enters from I medium into II medium.
ling in vacuum enters a medium of refractive index Its velocity in second medium is double that in I
1.5. Its wavelength in the medium is medium. For the phenomenon of total internal re-
(a) 4000 Å (b) 5000 Å flection to take place, the angle of incidence must be
(c) 6000 Å (d) 5500 Å. greater than a certain value. This value is
[MP PET 1995 ; MP PMT 1995] (a) 20° (b) 30°

779. The power of a thin convex lens (ang = 1.5) is + 5.0 D. (c) 45° (d) 60°. [AMU 1994]
When it is placed in a liquid of refractive index anl , 785. A fish rising vertically
then it behaves as a concave lens of focal length up towards the surface
100 cm. The refractive index of the liquid anl will be of water with speed 3
(a) 5/3 (b) 4/3 m s –1 observes a bird
diving vertically down
(c) 3 (d) 5/4. towards it with speed
780. Parallel beam containing light of λ = 400 nm and 9 m s –1. The actual
500 nm is incident on a prism as shown in Fig. 91. velocity of bird is :
The refractive index µ of the prism is given by the LM
Given : µ =
4 OP Fig. 92
relation
N 3 Q
OPTICS 519
A A
(a) 4.5 m s–1 (b) 5.4 m s–1
(c) 3.0 m s–1 (d) 3.4 m s–1. 90° 90°
[CMC LDH 1994] 45°
45° 45° 45°
786. A point source S is placed at the bot-
n=3
tom of different layers as shown in n=2 B (a) C B (b) C
Fig. 93. The refractive index of n=1
bottommost layer is µ0. The refrac- A A
S
tive index of any other upper layer 90° 90°
Fig. 93
is
45° 45°
µ0
µ(n) = µ0 – where n = 1, 2, ...... 45° 45°
4n − 18
B (c ) C B (d ) C
A ray of light starts from the source S as shown.
Total internal reflection takes place at the upper Fig. 94
surface of a layer having n equal to [EAMCET 1985]
(a) 3 (b) 5 791. An equiconvex lens is made from glass of refractive
index 1.5. If the radius of each surface is changed
(c) 4 (d) 6. [WB JEE 1997]
from 5 cm to 6 cm, then the power
787. A fish, looking up through the water, sees the out- (a) remains unchanged (b) increases by 3.33 D
side world contained in a circular horizon. If the re- (c) decreases by 3.33 D (d) decreases by 5.5 D.
4 [JIPMER 1994]
fractive index of water is and the fish is 12 cm
3 792. A convex lens produces an image of a real object on a
below the surface of water, the radius of the circle in 1
centimetre is screen with a magnification of . When the lens is
2
12 × 3 moved 30 cm towards the object, the magnification
(a) (b) 12 × 3 × 5
5 of the image on the screen is 2. The focal length of
the lens is
12 × 3 (a) 30 cm (b) 60 cm
(c) (d) 12 × 3 × 7 .
7 (c) 20 cm (d) 15 cm.
[Karnataka CET 2003 ; AIEEE 2005] [Himachal PMT 1994]
788. A concave lens with unequal radii of curvature made 793. A layered lens as shown in Fig. 95 is
of glass (µg = 1.5) has a focal length of 40 cm. If it is made of two types of transparent
immersed in a liquid of refractive index µl = 2, then materials indicated by different
(a) it behaves like convex lens of 80 cm focal length. shades. A point object is placed on
its axis. The object will form
(b) it behave like a convex lens of 20 cm focal length.
(a) 1 image (b) 2 images
(c) its focal length becomes 60 cm
(c) 3 images (d) 9 images.
(d) nothing can be said. [EAMCET 1999] [BHU 1996] Fig. 95
789. A fish in water sees an object 24 cm above the surface 794. An object is put at a distance of 5 cm from the first
of water. The height of object above the surface of focus of a convex lens of focal length 10 cm. If a real
water that will appear to fish is image is formed, its distance from the lens will be
(a) 24 cm (b) 32 cm (a) 15 cm (b) 20 cm

(c) 18 cm (d) 48 cm. [CPMT 1993] (c) 25 cm (d) 30 cm. [AMU 1995]
795. A lens has a power of 10 D, when placed in air. When
790. The refractive index of a material of a prism of angles
45° – 45° – 90° is 1.5. The path of the ray of light 4 FG IJ
incident normally on the hypotenuse side is shown
it is immersed in water µ =
3 H K
, the change in power

in is (refractive index of lens material is 1.5)


(a) 2.55 D (b) 5.0 D
(c) + 7.5 D (d) – 7.5 D. [JIPMER 1993]
520 COMPREHENSIVE OBJECTIVE PHYSICS

796. The first focal length of a convex lens is 10 cm. An (a) x (b) x
object is placed at a distance of 15 cm from the first m1 − m2 m1 + m2
principal focus. The lens produces an image on the x x
(c) (d) .
other side at a distance of 20 cm from the second (m1 − m2 ) 2 (m1 + m2 ) 2
principal focus. Then the second focal length is
[WB JEE 1996]
(a) 30 cm (b) 10 cm
803. A ray of light is incident nor-
(c) 17.5 cm (d) 35 cm. [AFMC 1994] mally on the first refracting
A

797. An object is placed 1 metre in front of the curved face of the prism of refract- 90°
surface of a plano-convex lens whose plane surface is ing angle A. The ray of light
silvered. A real image is formed in front of the lens comes out at grazing emer-
at a distance of 120 cm. Then the focal length of the gence. If one half of the prism
lens is (shaded position) is knocked 90°
off, the same ray will
(a) 100 cm (b) 120 cm
(a) suffer a deviation of A/4
(c) 109.1 cm (d) 110.0 cm. Fig. 96
(b) not emerge out of the prism
[CMC Vellore 1994]
FG 1 sin A / 4IJ
798. The apparent thickness of a thick plano-convex lens
is measured once with the plane face upward and
(c) emerge at an angle of sin–1
H2 K
then with the convex face upwards. The value will be F1 I
(d) emerge at an angle of emergence sin G sec A / 2J .
H2 K
–1

(a) more in the first case


(b) same in the two cases [WB JEE 1997]
(c) more in the second cases 804. Two thin convex lenses of focal lengths f1 and f2 are
placed at a distance d between them. For the power
(d) can be any of the above depending on the value of its actual
of the combination to be zero, the separation d is
thickness. [EAMCET 1997]
(a) f1 – f2 (b) f1 + f2
799. A bi-convex lens (µ = 1.5) of focal length 0.2 m acts as
a divergent lens of power one dioptre when immersed (c) f1/f2 (d) ( f1 f2 ) .
in a liquid. The refractive index of the liquid is [EAMCET 1996]
(a) 1.33 (b) 1.67 805. The graph shows how the
(c) 1.25 (d) 1.2. [Orissa 1992] magnification m produced by
800. A concave lens made of water (µ = 1.33) is placed a thin convex lens varies with m
b
inside a glass slab (µ = 1.5) for an object placed image distance v. The focal
q
between the focus and twice the focus. The image length of the lens is
a c
formed is (a) b/c v
(a) virtual (b) real, inverted and magnified (b) b/ca Fig. 97
(c) virtual, inverted and magnified (c) bc/a (d) c/b. [DCE 1995]
(d) real, inverted and diminished. 806. The refractive index of the material of a prism is 2
801. An optical fibre consists of core of µ2 surrounded by and the angle of prism is 30°. One of its refracting
a cladding of µ1 < µ2. A beam of light enters from air faces is polished. The incident beam of light will
at an angle i with axis of fibre. The maximum value retrace back for angle of incidence
of i for which a ray can travel through the optical (a) 0° (b) 45°
fibre is (c) 60° (d) 90°. [DCE 1992]
(a) cos–1 µ 22 − µ 12 (b) sin–1 µ 22 − µ 12 807. A spherical surface of radius of curvature R sepa-
rates air (refractive index 1.0) from glass (refractive
(c) tan–1 µ 12 − µ 22 (d) sec–1 µ 12 − µ 22 . index 1.5). The centre of curvature is in glass. A point
[MNR 1996] object P placed in air is found to have a real image Q
802. In the displacement method, a convex lens is placed in the glass. The line PQ cuts the surface at a point O
between an object and a screen. The displacement and PO = OQ. The distance PO is equal to
between two positions of the lens is x. If the magnifi- (a) 5 R (b) 3 R
cations in the two positions are m1 and m2 , the focal (c) 2 R (d) 5 R. [IIT 1998]
length of the lens is
OPTICS 521

808. The cross-section of a glass prism has the form of an R 4 + R1 R 4 − R1


(a) (b)
isosceles triangle. One of the refracting faces is sil- R 1 + R4 − R2 + R3 R 3 + R2
vered. A ray of light falls normally on the other re- R4 + R1 R − R 2 + 4(R 2 − R 1)
(c) (d) 4 .
fracting face. After being reflected twice, it emerges (R 4 + R 3 ) − (R 1 + R 2 ) R 4 − R3
through the base of the prism perpendicular to it. [Bharati Vidyapeeth 2003]
The angles of the prism are 813. The focal length of a plano-convex lens, the convex
(a) 54°, 54°, 72° (b) 72°, 72°, 36° surface of which is silvered, is 0.3 m. If the refractive
(c) 45°, 45°, 90° (d) 57°, 57°, 76°. [DCE 1997] 7
index of the material of lens is , the radius of cur-
809. Two parallel light rays are inci- 4
dent at one surface of a prism of vature of the convex surface is

30°
refractive index 1.5 as shown in (a) 0.65 m (b) 0.91 m
Fig. 98. The angle between the (c) 1.05 m (d) 7.3 m. [Roorkee 1996]
emergent rays is nearly
30° 814. A cube made of a material of refractive index µ2 is
(a) 19° (b) 37° immersed in a liquid of refractive index µ1. A ray of
(c) 45° (d) 49°. light is incident on face AB
Fig. 98
at an angle i as shown. To- A
i
B
810. Two thin equi-convex lenses of focal lengths 10 cm tal internal reflection just
and 20 cm are placed inside a thin-walled glass box takes place on face BC. r
with curved sides, side by Then sin i is equal to m
side, such that these are
2
µ2 90° – r
tightly fitted inside. The (a)
µ1
glass is then filled with water
LMF µ2 I 2 OP
MNGH µ1 JK + 1PQ
and used as a lens. Determine f1 f2 D
a
C
(b)
the position of an object so Fig. 100
that an image twice the size
of the object is formed due F µ1 I 2 − 1 F µ2 I 2 − 1
to this lens combination. Fig. 99
(c) GH µ 2 JK (d) GH µ1 JK
µglass = 3/2 and µwater = 4/3
[Roorkee 1995]
(a) 10 cm, 15 cm (b) 12 cm, 4 cm
815. Two transparent slabs have identical geometrical
(c) 15 cm, 5 cm (d) 8 cm, 3 cm. dimensions. One is made of material A of refractive
[EAMCET 1993] index 1.5 and the other is made of two materials B
811. A converging lens placed between an object and and C, the ratio of their lengths being 1 : 2. The
screen forms sharp image of an object on a screen. A refractive index of C is 1.6. If monochromatic parallel
glass slab of refractive index µ is inserted between beams passing through the slabs have the same
number of waves, the refractive index of B is
the object and the lens. In order to obtain the sharp
(a) 1.4 (b) 1.3
image on the screen again, the lens has to be dis-
(c) 1.1 [CPMT 1997]
(d) 1.5.
placed by a distance x. The thickness of glass slab is
µ+1
816. An object and a screen are fixed at a distance d apart.
(a) µx/µ + 1 (b) When a convex lens of focal length f is moved be-
x
tween the object and the screen, sharp images of the
(c) x
RS µ + 1UV (d)
µx
. [JIPMER 2003] object are formed on the screen for two positions of
T µ W µ−1
the lens. The magnifications produced in the two
812. A microscope is focussed on a mark in the bottom of positions are m1 and m2 . Then
a beaker when partly filled with some liquid. The 1
(a) m1 = (b) | m1 | – | m2 | = 1
microscope reading is found to be R1. The travelling m2
microscope is now focussed for the surface of the (c) d ≤ 2f (d) d > 4f.
liquid and the reading is found to be R2. Some more [CML LDH 2003]
liquid is now poured in the beaker and the experiment 817. A bird in air looks at a fish vertically below it and
is repeated. The corresponding readings are now R3 inside water in a tank. If the distances of the fish as
and R4. Then the refractive index of liquid is estimated by bird is S1 and that of bird as estimated
by the fish is S2 then the refractive index of liquid µ
is
522 COMPREHENSIVE OBJECTIVE PHYSICS

S1 S1 + S2 824. An equilateral triangular prism is made of glass


(a) (b)
S2 S1 − S2 (µ = 1.5). A ray of light is incident normally on one of
S2 S1 + S 2 the faces. The angle between the incident and emer-
(c) (d) . gent ray is
S1 S1
[Haryana PMT 2003] (a) 60° (b) 90°
818. The maximum value of index of refraction of a mate- (c) 120° (d) 180°. [CMC LDH 1995]
rial of a prism which allows the passage of light 825. The size of the image of an object which is at infinity,
through it when the refracting angle of the prism is as formed by a convex lens of focal length 30 cm, is
A is 2 cm. If a concave lens of focal length 20 cm is placed
FG A IJ FG A IJ between the convex lens and the image at a distance
(a) 1 − sin
H 2K (b) 1 − cot
H 2K of 26 cm from the convex lens, calculate the new size
F AI
1 − tan 2 G J F AI
1 + cot 2 G J .
of the image.
(c)
H 2K (d)
H 2K (a) 1.25 cm (b) 2.5 cm
[BHU 2003] (c) 1.05 cm (d) 2 cm.
819. An object is placed to the left of a glass hemisphere with [IIT Screening 2003]
radius 1 m and index of refraction 1.5. Where is the 826. A ray of light is incident at
image of this object formed ? The distance of the object the glass-water interface at Air
from the plane surface of the hemisphere is 2 m.
an angle i. It emerges finally
(a) 4 m (b) 6 m mw = 4
parallel to the surface of wa- r 3
(c) 8 m (d) 12 m. [BHU 2003] ter. Then the value of µ g r
Water
820. A ray of light is incident at an angle i on one surface would be
of a prism of small angle A. It emerges normally from (a) (4/3) sin i
the opposite surface. If the refractive index of the Glass i
(b) 1/sin i
material of the prism is µ, the angle of incidence i is
nearly equal to (c) 4/3
Fig. 101
(a) A/µ (b) A/2µ (d) 1. [IIT Screening 2003]
(c) µA (d) µ A/2 . 827. The angle of prism is 60° and its refractive index is
1.5. There will be no emergent light if the angle inci-
[Bharati Vidyapeeth 2003]
dence on the first face is
821. A vessel is half filled with a liquid of refractive index
(a) equal to 30° (b) equal to 60°
µ. The other half of the vessel is filled with an immis-
cible liquid of refractive index 1.5 µ. The apparent (c) less than 27° (d) more than 30°.
depth of vessel is 50% of the actual depth. The value 828. Three glass prisms A, B
of µ is and C of same refractive in-
(a) 1.6 (b) 1.67 dex are placed in contact
B
(c) 1.5 (d) 1.4. with each other as shown P
A
[Bharati Vidyapeeth 1998] in Fig. 102 with no air gap C
822. A virtual image twice as long as the object is formed between the prisms. Mono-
O Q
by a convex lens when the object is 10 cm away from chromatic ray of light OP R
passes through the prism Fig. 102
it. A real image twice as long as the object will be
formed when it is placed at a distance from the lens assembly and emerges as QR. The conditions of mini-
(a) 40 cm (b) 30 cm mum deviation is satisfied in the prisms
(c) 20 cm (d) 15 cm. (a) A and C (b) B and C
823. A swimmer is swimming inside a tank of water. He (c) A and B (d) In all prisms A, B and C.
looks up at the sky through the water surface. The [CMC Vellore 1994]
surface is calm. Sky is bright due to day light. He 829. A boy is looking vertically downwards in a tank of
can see water and his eyes are 1 m above the water surface.
(a) a small illuminated patch directly above his head whose He sees a fish at a depth 3 m from the water surface.
angular size is independent of the depth of the swimmer If aµw = 4/3, the real depth of the fish from the water
inside the tank
surface is
(b) a small illuminated patch directly above his head whose
(a) 4 m (b) 16/3 m
angular size depends upon the depth of the swimmer
(c) 5 m (d) 4/3 m.
(c) Nothing but darkness outside the tank
[CMC Vellore 1992]
(d) The entire top surface of the water illuminated.
OPTICS 523
V
DISPERSION AND ABERRATIONS V
R R
830. A parallel beam of white light falls on a convex lens.
Images of blue, yellow and red light are formed on
other side of the lens at distances of 20 cm, 20.5 cm (a) (b)
and 21.4 cm respectively. The dispersive power of
the material of the lens will be
(a) 14/205 (b) 9/200
(c) 5/214 (d) 619/200. R V
831. In a direct vision spectroscope, one uses a flint and a V
R
(c ) (d )
crown glass prism combination to get zero mean
deviation but a finite dispersion, Which of the follow- Fig. 104
ing is an acceptable diagrammatic representation ? [National Standard Exam. in Physics 1992]
[C = crown, F = flint, V = violet, R = red] 834. A flint glass prism and a crown glass prism of angles
A′ and A respectively are to be combined in such a
R manner that there is dispersion without deviation.
(a) F For this to occur, the ratio A′/A must be
V (µ y ′ − 1)
C (µ y − 1)
(a) – (b) –
(µ y ′ − 1) (µ y − 1)
(c) (µ y ′ − 1) (d) (µy – 1).
835. A ray of light passes through an equilateral prism
V
F such that the angle of incidence is equal to the angle
(b)
C R of emergence and the latter is equal to 1/2 of the
angle of prism. The angle of deviation is
(a) 20° (b) 0°
R (c) 39° (d) 45°.

(c)
C 836. A thin prism P1 with angle 4° and made from glass
V
F of refractive index 1.54 is combined with another thin
prism P2 made from glass of refractive index 1.72 to
produce dispersion without deviation. The angle of
V
the prism P2 is
(a) 2.6° (b) 3°
C
(d) R (c) 4° (d) 5.33°. [IIT 1990]
F
837. Which of the following diagrams is a correct presen-
tation of deviation and dispersion of light by prism ?
Fig. 103

[National Standard Exam. in Physics 1990] R


R
832. A prism can produce a minimum deviation δ in a
V
light beam. If three such prisms are combined, the V
(a) (b)
minimum deviation that can be produced in this
beam is
(a) 0 (b) δ R
R
(c) 2δ (d) 3δ.
V
833. Which of the following diagrams shows correctly the V
(c ) (d )
dispersion of white light by a prism ?
Fig. 105

[National Standard Exam. in Physics 1994]


524 COMPREHENSIVE OBJECTIVE PHYSICS

838. A ray of light passes through an equilateral prism (a) red (b) orange
such that the angle of incidence is equal to the angle (c) pink (d) none of these.
of emergence and the latter is equal to (3/4)th the [CPMT 1988]
angle of the prism. The angle of deviation will be
845. If the refractive indices of crown glass for red, yellow
(a) 45° (b) 39°
and violet colours are 1.5140, 1.5170 and 1.5318
(c) 20° (d) 30°. [MP PMT 1999] respectively and for flint glass these are 1.6434, 1.6499
839. A prism (µ = 1.5) has the refracting angle of 30°. The and 1.6852 respectively, then the dispersive powers
deviation of a monochromatic ray incident normally for crown and flint glasses are respectively
on its one surface will be (a) 0.034 and 1.0 (b) 0.034 and 0.064
(sin 48°36′ = 0.75) (c) 0.064 and 0.034 (d) 1.3 and 0.064.
(a) 18° 36′ (b) 20° 30′
846. For a prism of refractive index 3 , the angle of the
(c) 18° (d) 22° 1′.
prism is equal to the angle minimum deviation. The
[MP PMT/PET 1988]
value of the angle of the prism is
840. The refractive index of a prism for a monochromatic
(a) 45° (b) 30°
wave is 2 and its refracting angle is 60°. For mini- (c) 60° (d) 50°. [Karnataka 1999]
mum deviation, the angle of incidence will be
847. The dispersion for a medium of wavelength λ is D.
(a) 30° (b) 45°
Then the dispersion for wavelength 2λ will be
(c) 60° (d) 75°.
(a) D/8 (b) D/4
[MP PMT 1989, 92 ; CPMT 1993]
(c) D/2 (d) D. [MP PET 1989]
841. A light ray is incident upon
a prism in minimum devia- 848. Angle of minimum deviation for a prism of refractive
tion position and suffers a index 1.5 is equal to the angle of prism. The angle of
deviation of 34°. If the prism is (cos 41° = 0.75)
shaded half of the prism is (a) 62° (b) 41°
knocked off, the ray will (c) 82° (d) 31°.
Fig. 106
(a) suffer a deviation of 34°
[MP PET/PMT 1988]
(b) suffer a deviation of 68°
(c) suffer a deviation of 17° (d) not come out of the prism. 849. To obtain a convex achromatic combination with two
lenses in contact, it is necessary to combine
[MP MPT 1992]
(a) two convex lenses of different materials
842. If a glass prism is dipped in water, its dispersive power
(b) one convex and the other concave but of same material
(a) increases (b) decreases
(c) a convex lens of higher dispersive power with a concave
(c) does not change
lens of lower dispersive power
(d) may increase or decrease depending on whether the angle
(d) a convex lens of lower dispersive power with a concave lens
of the prism is less than or greater than 60°.
of higher dispersive power.
[MP PMT 1995, modified]
850. A convex lens has a mean focal length of 20 cm. The
843. A thin prism P1 of angle of prism 4° and refractive
dispersive power of the material of the lens is 0.02.
index 1.525 is combined with another thin prism P2
Then the longitudinal chromatic aberration for an
of refractive index 1.70 for dispersion without devia-
tion. The angle of prism of P2 is object at infinity is
(a) 0.20 cm (b) 0.40 cm
(a) 5.33° (b) 4°
(c) 0.80 cm (d) 1.0 cm.
(c) 3° (d) 2.6°. [MP PET 1999]
851. The dispersive powers of the materials of the two
844. Fig. 107 shows the view through the
lenses are in the ratio 4 : 3. If the achromatic combi-
eye-piece of a prism spectrometer
nation of these two lenses in contact is a convex lens
with its slit illuminated by the
of focal length 60 cm, then the focal lengths of the
source of light emitting three
component lenses are
wavelengths corresponding to X G Y (a) – 20 cm and + 25 cm (b) 20 cm and – 25 cm
yellow (Y), green (G) and an
unknown colour (X). The colour at Fig. 107 (c) – 15 cm and + 20 cm (d) + 15 cm and – 20 cm.
X may be [CPMT 1989]
OPTICS 525

852. Consider the following two statements (a) White (b) Red coloured
(A) Line spectra contain information about atoms. (c) Violet (d) Yellow.
(B) Band spectra contain information about molecules. 860. A ray of light passes through an equilateral prism
(a) Both A and B are wrong (b) A is correct but B is wrong. such that the angle of incidence and the angle of
(c) B is correct but A is wrong.
emergence are both equal to 3/4th of the angle of
prism. The angle of minimum deviation is
(d) Both A and B are correct.
(a) 15° (b) 30°
853. Light is incident at an angle of 60° on a prism of
(c) 45° (d) 60°.
which the refracting angle of prism is 30°. The angle
between the incident and the emergent rays is also 861. A parallel beam of monochromatic light is incident
30°. The refractive index of material of prism will be at one surface of an equilateral prism. Angle of inci-
dence is 55° and angle of emergence is 46°. The angle
(a) 2 (b) 2 3
of minimum deviation will be
(c) 2 (d) 3. (a) less than 41° (b) equal to 41°
854. The difference between angle of minimum deviation (c) more than 41° (d) none of these.
of violet and red rays in the spectrum of white light 862. The refractive indices of flint glass for violet, yellow
from a prism is 2°. If angle of minimum deviation of and red colours are 1.632, 1.620 and 1.613 respec-
mean ray is 48°, the dispersive power of the material tively. If the mean focal length of the lens is 10 cm,
of the prism is the axial chromatic aberration is nearly
(a) 0.0832 (b) 0.0416 (a) 30 mm (b) 3 mm
(c) 24 (d) 48. [MP PMT 1990] (c) 0.3 mm (d) 6 mm.
855. The focal length of a thin convex lens for red and 863. It is desired to make a converging achromatic com-
blue rays are 100 cm and 96.8 cm respectively. The bination of mean focal length 50 cm by using two
dispersive power of the material of the lens is lenses of materials A and B. If the dispersive powers
(a) 0.968 (b) 0.98
of A and B are in ratio 1 : 2, then focal lengths of the
convex and the concave lenses are respectively
(c) 0.0325 [MP PET 1990]
(d) 0.325.
(a) 25 cm and 50 cm (b) 50 cm and 25 cm
856. A parallel beam of white light falls on a combination (c) 50 cm and 100 cm (d) 100 cm and 50 cm.
of a concave and a convex lens both of the same ma-
terial. Their focal lengths are 15 cm and 30 cm re- 864. White light is passed through a prism of angle 5°. If
spectively for the mean wavelength in white light. the refractive indices for the red and blue colours are
On a screen, a coloured pattern is formed with 1.641 and 1.659 respectively, what is the angle of dis-
persion between them ?
(a) violet at inner edge and red at outer edge
(a) 5° (b) 2°
(b) violet at outer edge and red at inner edge
(c) 0.09° (d) 0.9°.
(c) yellow at the outer edge
865. The angle of prism is 60°. What is the angle of inci-
(d) violet and red images coincide. dence for minimum deviation ? The refractive index
857. The minimum deviation produced by a hollow prism of material of prism is 2 .
filled with a certain liquid is found to be 30°. The (a) 45° (b) 60°
light ray is also found to be refracted at angle of 30°.
FG 2 IJ . [CPMT 1993]
The refractive index of the liquid is (c) 30° (d) sin–1
H 3K
(a) 2 (b) 3
866. A convex lens of focal length 15 cm is combined with
3 3 a concave lens of focal length 30 cm to obtain an
(c) (d) . [MP PET 1991]
2 2 achromatic combination. If the dispersive power of
858. Angle of glass prism is 60° and refractive index of the the material of the convex lens is 0.02, then the dis-
material of the prism is 1.414. What will be the angle persive power of the material of the concave lens, is
of incidence so that ray should pass symmetrically (a) 0.02 (b) 0.01
through prism ? (c) 0.04 (d) 2.02 .
(a) 38° 61′ (b) 35° 35′
867. Angle of a prism is 30° and its refractive index is 2
(c) 45° (d) 53° 8′.
and one of the surfaces is silvered. At what angle of
859. A convex lens of focal length 0.1 m is illuminated incidence, a ray should be incident on one surface so
with a parallel beam of white light. Then the image that after reflection from the silvered surface, it re-
obtained at a distance of 0.1 m from the lens is traces its path ?
526 COMPREHENSIVE OBJECTIVE PHYSICS

(a) 30° (b) 60° (a) 0.01 (b) 0.02


(c) 45° (d) sin–1 1.5 . (c) 1.005 (d) 0.995.
[MP PMT 1991} 874. The angle of prism is 60° and the refractive index of
A the material of prism is 1.5. If angles of incidence
868. For a prism, its refractive index is cot . Then mini- and emergence at first and second refracting faces
2
mum angle of deviation is are i1 and i2 , then for minimum deviation
(a) 180 – A (b) 180 – 2A (a) i1 = 0 (b) i1 < i2
A (c) i1 > i2 (d) i1 = i2.
(c) 90 – A (d) .
2
[All India PM/PD 1999] 875. In Q. No. 874, for maximum deviation
(a) i1 = 90° (b) i1 < i2
869. Deviation of 5° is observed from a prism whose angle
is small and whose refractive index is 1.5. The angle (c) i1 > i2 (d) i1 = i2.
of prism is 876. The angular dispersion produced by a prism
(a) 7.5° (b) 10° (a) increases if the average refractive index decreases
(c) 5° (d) 3.3°. [MP PET 1990] (b) increases if the average refractive index increases
870. Three prisms of crown glass, each have angle of prism (c) remains constant whether the average refractive index
9° and two prisms of flint glass are used to make increases or decreases.
direct vision spectroscope. What will be the angle of (d) has no relation with average refractive index.
flint glass prism if µ for flint is 1.60 and µ for crown 877. Two lenses in contact made of materials with disper-
glass is 1.53 ? sive powers in the ratio 2 : 1, behave as an achro-
(a) 11.9° (b) 16.0° matic lens of focal length 10 cm. The individual focal
(c) 15.3° (d) 9.11°. lengths of the lenses are
871. A ray of light is incident at an angle of 60° on one (a) 5 cm, – 10 cm (b) – 5 cm, 10 cm
face of a prism of angle 30°. The ray emerging out of (c) 10 cm, – 20 cm (d) – 20 cm, 10 cm.
the prism makes an angle of 30° with the incident 878. An achromatic convergent doublet of two lenses in
ray. The emergent ray is contact has a power of + 2 D. The convex lens has
(a) normal to the face through which it emerges power + 5 D. What is the ratio of the dispersive pow-
(b) inclined at 30° to the face through which it emerges ers of the convergent and divergent lenses ?
(c) inclined at 60° to the face through which it emerges (a) 2 : 5 (b) 3 : 5
(d) None of the above. [EAMCET 1990 ; MP PMT 1990] (c) 5 : 2 (d) 5 : 3. [MP PMT 1990]
872. The diagram shows part of a typical line emission 879. When a parallel beam of white light passes through a
spectrum. This spectrum extends through the vis- metal vapour, dark lines appear in the spectrum of
ible region of the electromagnetic spectrum into the the emergent light. This is principally because energy
ultraviolet region. is absorbed and
(a) is not re-radiated at all (b) is re-radiated as infra-red
(c) is re-radiated as ultra-violet
(d) is re-radiated gradually over a long period of time
X
(e) is re-radiated uniformly in all directions.
Fig. 108
OPTICAL INSTRUMENTS
Which statement is true for emission line X of the
spectrum ?
880. An astronomical telescope has an eye-lens of focal
(a) It has the longest wavelength and is at the ultraviolet end
of the spectrum. length 20 mm. In normal adjustment, when the final
(b) It has the highest frequency and is at the ultraviolet end of image of a distant object is at infinity, the separation
the spectrum. of the lenses is 500 mm. The angular magnification
(c) It has the lowest frequency and is at the red end of the of the telescope under these conditions is
spectrum.
(d) It has the shortest wavelength and is at the red end of the (a) 22 (b) 23
spectrum. (c) 24 (d) 25
873. The focal lengths of a thin convex lens for red and (e) 26.
blue colours are 100.5 cm and 99.5 cm. The disper-
[Northern Ireland Schools Exam. Council]
sive power of the lens is
OPTICS 527

881. In the previous question, to obtain the best view, the 888. The focal lengths of the objective and eye-lens of a
distance between the observer’s eye and the eye-lens microscope are 1.6 cm and 2.5 cm respectively. The
should be distance between the two lenses is 21.7 cm. If the
(a) zero (b) 19.2 mm final image is formed at infinity, the distance between
the object and the objective lens is
(c) 20.0 mm (d) 20.8 mm
(a) 1.8 cm (b) 1.70 cm
(e) 24.0 mm.
(c) 1.65 cm (d) 1.75 cm.
[London Schools Examination Department] 889. A compound microscope has an objective and eye-
882. An astronomical telescope consists of an objective of piece as thin lenses of focal lengths 1 cm and 5 cm
focal length 60 cm and an eye-piece of focal length respectively. The distance between the objective and
5 cm. It is focussed on a distant object such that the the eye-piece is 20 cm. The distance at which the ob-
rays emerging from the eye-lens are parallel. The ject must be placed in front of the objective if the
object subtends an angle of 2° at the objective. The final image is located at 25 cm from the eye-piece, is
angular width of the image is numerically
(a) 10° (b) 24° (a) 95/6 cm (b) 5 cm
(c) 48° (d) 54°. (c) 95/89 cm (d) 25/6 cm.
[CMC Vellore 1996] 890. In a compound microscope, the intermediate image
883. A telescope has an objective of focal length 50 cm and is
an eye-piece of focal length 5 cm. It is focussed for (a) virtual, erect and magnified
distinct vision on a scale 200 cm away from the ob- (b) real, erect and magnified
jective. Then the optical length of the telescope is (c) real, inverted and magnified
(a) 200/3 cm (b) 25/6 cm (d) virtual, erect and reduced.[IIT Screening 2000]
(c) 425/6 cm (d) 375/6 cm. 891. A telescope is adjusted for parallel rays. The distance
884. In Question No. 883, the linear magnification, m, between the objective and the eye-piece is observed to
produced by the telescope is numerically be 20 cm. The magnifying power of telescope is 9.
The focal lengths of the lenses are
1
(a) (b) 6 (a) 11 cm, 9 cm (b) 9 cm, 11 cm
3
(c) 2 (d) 16. (c) 10 cm, 10 cm (d) 18 cm, 2 cm.
885. The magnifying power of a telescope is 9. When it is [AFMC 1997]
adjusted for parallel rays, the distance between the 892. In representing the objective and eye-piece of a tel-
objective and the eye-piece is found to be 20 cm. The escope in diagrams, one has the options as follows
focal lengths of the lenses are (assuming a single lens presentation for the eye-piece)
(a) 18 cm, 2 cm (b) 11 cm, 9 cm P : objective : large aperture and thin
(c) 10 cm, 10 cm (d) 15 cm, 5 cm. Q : objective : large aperture and thick
[MP PMT 1986] R : eye-piece : large aperture and thick
886. For an astronomical telescope used in normal ad- S : eye-piece : small aperture and thick
justment, objective and the eye-piece are separated The best representation would be with
by a distance of 55 cm. If the magnifying power of (a) P and R (b) P and S
the telescope is 10, the power of the objective is (c) Q and R (d) Q and S.
(a) 5 D (b) 50 D
[National Standard Exam. in Physics 1995]
(c) 2 D (d) 0.5 D.
893. The focal length of a simple convex lens used as a
887. An astronomical telescope has an angular magnifi- magnifier is 10 cm. For the image to be formed at a
cation of magnitude 5 for distant objects. The sepa- distance of distinct vision D = 25 cm, the object must
ration between the objective and the eye-piece is be placed away from the lens nearly at a distance of
36 cm and the final image is formed at infinity. The (a) 5 cm (b) 7 cm
focal lengths of the objective fo and fe of the eye-piece
(c) 8 cm (d) 16 cm. [CPMT 1996]
are
(a) fo = 45 cm and fe = – 9 cm (b) fo = 50 cm and fe = 10 cm
894. The focal lengths of the objective and eye-lens of a
telescope are 100 cm and 10 cm respectively. If the
(c) fo = 7.2 cm and fe = 5 cm (d) fo = 30 cm and fe = 6 cm.
final image is formed at the least distance of distinct
[CMC Ludhiana, 1996 ; IIT 1989] vision, the magnifying power of the telescope is
528 COMPREHENSIVE OBJECTIVE PHYSICS

(a) 14 (b) 10 903. In compound microscope, magnifying power is 95 and


(c) 16 (d) 12. 1
the distance of object from objective lens is cm.
895. In the above question, the distance between the two 3.8
lenses is 1
The focal length of objective lens is cm. What is
(a) 90 cm (b) 110 cm 4
(c) 107.14 cm (d) 116.28 cm. the magnification of eye-piece ?
(a) 5 (b) 10
896. Numerical aperture of an oil immersion objective using
oil of refractive index 1.5 is 0.75. Then, its numerical (c) 100 (d) 200. [CBSE PMT 1999]
aperture, when kept in air is 904. The focal lengths of the
(a) 0.5 (b) 0.25 50 cm
objective and the eye
(c) 0.3 (d) 1. piece of a smaller astro-
[Karnataka CET 1998] nomical telescope are 50
897. The resolution limit of eye is 1′. At a distance Y km cm and 5 cm respec-
from eye, two trees stand with a lateral separation of tively. When telescope is
3 m. For two trees to be just resolved by naked eye, Y directed towards the sun, O E
should be the final image is formed Fig. 109
(a) 5 km (b) 10 km at 25 cm from the eye
(c) 15 km (d) 12 km. piece. If the diameter of the sun subtends an angle of
898. A simple telescope, consisting of an objective of focal 32′ at the objective, the angular magnification and
length 60 cm and a single eye-lens of focal length 5 actual size of image are
cm is focussed on a distant object in such a way that (a) 11, 2.11 cm (b) 12, 2.79 cm
parallel rays emerge from the eye-lens. If the object (c) 10, 2.97 cm (d) 14, 1.79 cm.
makes an angle of 2° at the objective, the angular 905. An object is placed at a distance u from a simple
width of the image is microscope of focal length f. The angular magnifica-
(a) 10° (b) 24° tion obtained depends
(c) 50° (d) 48°. (a) on f but not on u (b) on u but not on f
899. Two points, separated by a distance of 0.1 mm, can (c) on f as well as u (d) neither on f nor on u.
just be inspected on a microscope when light of wave-
length 6000 Å is used. If the light of wavelength 4800 Å 906. In a compound microscope, the focal lengths of ob-
is used, the limit of resolution is jective and eye-lenses are 1.2 cm and 3 cm respec-
(a) 0.8 mm (b) 0.08 mm
tively. If the object is put 1.25 cm away from the
objective lens and the final image is formed at infin-
(c) 0.1 mm (d) 0.04 mm.
ity, the magnifying power of the microscope is
900. Four lenses of focal lengths ± 15 cm and ± 150 cm are
(a) 150 (b) 200
available for making a telescope. To produce the larg-
est magnification, the focal length of the eye-piece (c) 250 (d) 400.
should be 907. Four convergent lenses have focal lengths 100 cm,
(a) – 150 cm (b) – 15 cm 10 cm, 4 cm and 0.3 cm. A telescope with maximum
(c) + 15 cm (d) + 150 cm. possible magnification will choose among these the
lenses of focal lengths
[All India PM/PD 1994]
(a) 100 cm, 4 cm (b) 100 cm, 0.3 cm
901. An astronomical telescope is focussed on a distant
(c) 10 cm, 0.3 cm (d) 10 cm, 4 cm.
star. If the distance between the objective lens and
the eye piece is 2.5 m and the power of the eye-piece 908. A microscope and a telescope each consist of two con-
is 4 D, verging lenses. In which one of the following ways is
(a) the magnifying power of the telescope is 70 the telescope similar to the microscope when both
are in normal adjustment ?
(b) the magnifying power is 40
(a) Each has a long focal length objective lens.
(c) the power of the objective lens is 0.444 D
(b) In each, the final image is inverted and virtual.
(d) the focal length of the objective is 25 cm.
(c) In each, the separation of the lenses is equal to the sum of
902. To increase the angular magnification of a simple their focal lengths.
microscope, one should increase (d) Each produces an intermediate image which is magnified
(a) the focal length of the lens and inverted.
(b) the power of the lens (c) the aperture of the lens (e) In each, the final image is in the focal plane of the instru-
(d) the object size. ment’s eye-piece.
OPTICS 529
909. The focal lengths of the objective and the eye-piece of 916. The focal lengths of objective and eye-lens of an as-
a compound microscope are 2.0 cm and 3.0 cm re- tronomical telescope are respectively 2 metre and
spectively. The distance between the objective and 5 cm. Final image is formed at (1) least distance of
the eye-piece is 15.0 cm. The final image formed by distinct vision (2) infinity. Magnifying power in two
the eye-piece is at infinity. The two lenses are thin. cases will be
The distances in cm of the object and the image pro- (a) – 48, – 40 (b) – 40, – 48
duced by the objective, measured from the objective (c) – 40, + 48 (d) – 48, + 40.
lens, are respectively
[MP PMT 1988]
(a) 2.4 and 12.0 (b) 2.4 and 15.0
917. A refracting astronomical telescope has several in-
(c) 2.3 and 3.0 (d) 2.3 and 12.0. [IIT 1995] terchangeable eye-pieces. In order to increase the in-
910. The average distance between the earth and moon is strument’s magnifying power, the operator should
38.6 × 104 km. The minimum separation between the replace the eye-piece with another of
two points on the surface of moon that can be resolved (a) shorter focal length, moving it closer to the objective lens
by a telescope whose objective lens has a diameter of (b) shorter focal length, moving it further from the objective
5 m with λ = 6000 Å is lens
(a) 5.65 m (b) 28.25 m (c) shorter focal length, without changing its distance from
(c) 11.30 m (d) 56.51 m. the objective lens
[MP PMT 1993] (d) longer focal length, moving it closer to the objective lens.
911. An astronomical telescope, having an objective lens (e) longer focal length, moving it further from the objective
of focal length + 80 cm, is focussed on the moon. In lens.
order to focus on an object 40 metre distant, the eye-
piece must be drawn out HUMAN EYE
(a) 3.26 cm (b) 2.34 cm
(c) 1.63 cm (d) 5.53 cm.
918. A person can see object only at distance greater than
40 cm. He is advised to use lens of power
912. The diameter of moon is 3.5 × 103
km and its dis-
(a) – 2.5 D (b) + 2.5 D
tance from the earth is 3.8 × 105 km. The focal lengths
(c) – 6.25 D [CPMT 1993]
(d) + 1.5 D.
of the objective and eye-piece are 4 metre and 10 cm
respectively. The diameter of the image of the moon 919. A man uses spectacles having concave lens of focal
will be approximately length 50 cm. He can see objects lying at 25 cm clearly
(a) 2° (b) 21° by using the spectacles. How far a book must be kept
from the eye-lens if he does not use his spectacles ?
(c) 40° (d) 50°. [CPMT 1991]
(a) 33.3 cm (b) 50 cm
913. The focal length of the eye-lens in Huygens’ eye-piece
(c) 25 cm (d) 50/3 cm.
is f. Then the focal length of the field-lens and the
separation between the two are respectively 920. Two camera lenses have focal lengths of 25 cm and
(a) 3f and f (b) f and 2f/3 15 cm. The first lens has a face diameter of 5 cm. For
(c) 3f and 2f (d) 2f and 3f. equal exposure time, the diameter of the second lens
914. An astronomical telescope has a converging eye-piece is
of focal length 5 cm and objective of focal length (a) 5 cm (b) 3 cm
80 cm. When the final image is formed at the least (c) 8.33 cm (d) 20 cm.
distance of distinct vision (25 cm), the separation be- 921. A hypermetropic person having near point at a dis-
tween the two lenses is tance of 0.75 m puts on spectacles of power 2.5 D.
(a) 75.0 cm (b) 80.0 cm The near point now is at
(c) 84.2 cm (d) 85.0 cm. (a) 0.75 m (b) 0.83 m
915. An object is viewed through a compound microscope (c) 0.26 m (d) 0.26 cm.
and appears in focus when it is 5 mm from the objec-
922. A hypermetropic person has to use a lens of power
tive lens. When a sheet of transparent material 3 mm
+ 5 D to normalise his vision. The near point of the
thick is placed between the objective and the micro-
hypermetropic eye is
scope, the objective lens has to be moved 1 mm to
(a) 1 m (b) 1.5 m
bring the object back into the focus. The refractive
index of the transparent material is (c) 0.5 m (d) 0.66 m.
(a) 1.5 (b) 1.6 923. A hypermetropic patient has near point at 50 cm.
(c) 1.8 (d) 2.0. The dioptric power for the corrective lens is
530 COMPREHENSIVE OBJECTIVE PHYSICS

(a) + 0.5 (b) – 0.5 f


932. With diaphragm of the camera lens set at , the
(c) + 2.0 (d) – 2.0. 2
924. With diaphragm of the camera lens set at f/2, the 1
correct exposure time is , then with diaphragm
correct exposure time is 1/100 second. Then with 100
diaphragm set at f/8, the correct exposure time is f
set at , the correct exposure time is
(a) 1/100 second (b) 1/400 second 4
(c) 1/200 second (d) 16/100 second. 1 1
(a) s (b) s
925. A certain far-sighted person cannot see objects closer 100 400
to the eye than 100 cm. The power of the lens which 1 4
(c) s (d) s.
will enable him to read at a distance of 25 cm will be 200 100
(a) 3 dioptre (b) 1 dioptre 933. A man requires a lens of power 2 D to read a book
(c) 4 dioptre (d) 2 dioptre. held at a distance of 25 cm from the eye. Four years
later, the man wearing same glass can read book
[MP PMT 1990]
clearly if it is at a distance of 0.4 m from the eye. The
926. A locality is photographed from an aeroplane, flying
power of the lens, man should use now, so that he
at a height of 2000 m with a camera whose focal length can again read clearly a book held at 25 cm is
is 50 cm. The ratio of the linear size of the image to
(a) + 4.5 D (b) – 4.5 D
the size of the object is nearly
(c) – 3.5 D (d) + 3.5 D.
(a) 1 : 10000 (b) 1 : 40
(c) 1 : 4000 (d) 1 : 1000. 934. The far point of a myopic eye is 150 cm. Calculate the
power of the lens required to correct the defect of eye.
927. A person cannot see the objects clearly placed at dis-
(a) – 0.67 D (b) + 0.66 D
tances more than 40 cm. He is advised to use lens of
power (c) – 0.33 D (d) + 1 D.
(a) – 2.5 D (b) + 2.5 D 935. A camera with a lens of focal length f takes a picture
(c) – 6.25 D (d) + 1.5 D. [CPMT 1993] of a boy. The image on the film has a height I and the
928. The focal length of a normal eye-lens is about film is at a distance v behind the lens. The lens-boy
(a) 1 mm (b) 2 cm distance and height of body are given by
(c) 25 cm (d) 1 m. vf If − vf – If
(a) , (b) ,
929. A person is not able to see objects farther than 80 cm f −v f −v f −v f −v
clearly, while another person is not able to see ob- vf If vf If
(c) , (d) , .
jects beyond 120 cm, clearly. The powers of the lenses f +v f +v f +v f +v
used by them for correct vision are in the ratio.
(a) 2 : 3 (b) 3 : 2 INTERFERENCE
(c) 1 : 2 (d) 2 : 1.
936. In a YDSE bi-chromatic light of wavelengths 400 nm
930. The maximum focal length of the eye-lens of a person and 560 nm are used. The distance between the slits
is greater than its distance from the retina. The eye is 0.1 mm and the distance between the plane of the
is
slits and the screen is 1 m. The minimum distance
(a) always strained in looking at an object
between two successive regions of complete darkness
(b) strained for objects at large distances only is
(c) strained for objects at short distances only
(a) 4 mm (b) 5.6 mm
(d) unstrained for all distances.
(c) 14 mm (d) 28 mm
931. When a simple camera is focussed on a distant ob-
[IIT Screening 2004]
ject, the distance from the lens to the film is found to
be 40.0 mm. In order to focus an image on the film 937. In Young’s experiment, the slit widths are in the ratio
for an object which lies 0.54 metre in front of the 1 : 9. The ratio of the intensity at minima to that at
lens, the required displacement of the lens is maxima is
(a) 0.2 mm away from the film (a) 1 : 4 (b) 1 : 3
(b) 0.2 mm towards the film (c) 1 : 9 (d) 1 : 1.
(c) 1.6 mm towards the film
938. In an interference experiment, we get 60 fringes in
(d) 3.2 mm away from the film.
the field of view of monochromatic light of wavelength
4000 Å. If we use monochromatic light of wavelength
OPTICS 531

6000 Å, then the number of fringes obtained in the 945. In a biprism experiment, by using light of wavelength
same field of view is 5000 Å, 5 mm wide fringes are obtained on a screen
(a) 1.5 (b) 90 1.0 m away from coherent sources. The separation
(c) 40 (d) 60. between two coherent sources is
939. The Young’s double slit experiment is performed with (a) 1.0 mm (b) 0.1 mm
blue light and green light of wavelengths 4360 Å and (c) 0.05 mm (d) 0.01 mm.
5460 Å respectively. If y is the distance of 4th maxima
[MP PMT 1998]
from the central one, then
(a) yb = yg (b) yb > yg 946. In a Young’s double slit experiment, the fringe width
is found to be 0.4 mm. If the whole apparatus is im-
yb 5460
(c) yb < yg (d) = . mersed in water of refractive index 4/3 without dis-
y g 4360
turbing the geometrical arrangement, the new fringe
940. The distance between two slits is 1 mm, wavelength width will be
of light used is 7000 Å and distance between slit and
(a) 0.30 mm (b) 0.40 mm
screen is 1 m. Then distance between 3rd black and
5th bright fringe is (c) 0.53 mm (d) 450 micron.
(a) 1.75 cm (b) 1.75 mm [CBSE PMT 1990]
(c) 1.05 mm (d) 0.875 mm. 947. Fig. 110 shows a double
941. A Young’s Double slit experiment is performed using slit experiment. P and Q nλ X
light of wavelength λ = 5000 Å, which emerges in are the slits. The path P
phase from two slits 3 × 10–7 m apart. A transparent lengths PX and QX are
(n + 2)λ
sheet of thickness t = 1.5 × 10–7 m is placed over one nλ and (n + 2) λ respec-
of the slits. The refractive index of the material of
tively, where n is a Q
this sheet is µ = 1.17. The central maximum of the
whole number and λ is
interference pattern appears at
the wavelength. Taking Fig. 110
(a) 3.50° (b) 8.84°
the central fringe as
(c) 8.48° (d) 4.88°.
zero, what is formed at
942. In two separate setups of the Young’s double slits
X?
experiment, fringes of equal width are observed when
lights of wavelengths in the ratio 1 : 2 are used. If the (a) First bright (b) First dark
ratio of the slit separation in two cases is 2 : 1, the (c) Second bright (d) Second dark.
ratio of distances, between the plane of slits and the
948. Two slits, 4 mm apart, are illuminated by light of
screen, in the two setups is
wavelength 6000 Å. What will be the fringe width on
(a) 4 : 1 (b) 1 : 1
a screen placed 2 m from the slits ?
(c) 1 : 4 (d) 2 : 1.
(a) 0.12 mm (b) 0.3 mm
943. The two coherent sources with intensity ratio β pro-
duce interference. The fringe visibility will be (c) 3.0 mm (d) 4.0 mm.

2 β [MP PET 2000]


(a) (b) 2β
1+ β 949. A parallel air film is formed between two glass plates.
2 β If the film thickness is 0.45 × 10–6 m, the film will be
(c) (d) .
(1 + β) 1+β best reflector for visible light in the neighbourhood
944. In a Fresenel Biprism Experiment, the distance be- of
tween the source and the screen is D and that be- (a) 9000 Å (b) 6750 Å
tween source and biprism is a. The wavelength of (c) 6000 Å (d) 4500 Å.
light used is λ. The fringe width is β and refracting
angle biprism is A. The refractive index of material [National Standard Exam. in Physics 1994]
of biprism is 950. If the source of light used in a Young’s double slit
Dλ Dλ experiment is changed from red to violet,
(a) 1 + (b) 1 −
2aAβ 2aAβ
(a) the fringes will become brighter
Dλ aA
(c) 1 − (d) 1 + . (b) consecutive fringes will come closer
aAβ Dλβ
532 COMPREHENSIVE OBJECTIVE PHYSICS

(c) the intensity of minima will increase fringe of longer wavelength coincides with (n + 1)th
(d) the central bright fringe will become a dark fringe. bright fringe of shorter wavelength. The value of n is
951. In Young’s experiment, the ratio of maximum and (a) 1 (b) 2
minimum intensities in the fringe system is 9 : 1. (c) 3 (d) 4.
The ratio of amplitudes of coherent sources is 957. In a Young’s experiment on interference, the distance
(a) 9 : 1 (b) 3 : 1 between the slits is d. The screen is at a distance D
(c) 2 : 1 (d) 1 : 1. from the slits. If a bright fringe is formed, opposite
to a slit, on the screen then the order of fringe is
952. In the Young’s double slit experiment, a mica strip of
thickness t and refractive index µ is introduced in d2 2λD
(a) (b)
the ray from first source S1. By how much distance 2λD d2
fringe pattern will be displaced ? d 2d
(c) (d) .
λD λD
d D
(a) (µ − 1)t (b) (µ − 1)t 958. An interference is observed due to two coherent
D d
sources A and B separated by a distance 4λ along Y-
d D axis where λ is wavelength of source. A detector D is
(c) (d) (µ − 1) .
(µ − 1)D d moved along the positive X-axis. The number of points
on the X-axis excluding the points x = 0 and x = ∞ at
[Rajasthan PMT 1996] which maximum will be observed is
953. One slit of a double slit experiment is covered by a (a) 3 (b) 4
thin glass plate of refractive index 1.4, and the other (c) 5 (d) 6.
by a thin glass plate of refractive index 1.7. The point
959. Sodium light (λ = 6 × 10–7 m) is used to produce
on the screen where the central maximum fell before
interference pattern. The observed fringe width is
the glass plates were inserted is now occupied by what 0.12 mm. What is the angle between the two interfer-
had been the fifth bright fringe before. Assume the ing wave trains ?
plates to have the same thickness t and λ = 480 nm. (a) 3 × 10–3 rad (b) 4 × 10–3 rad
Then the value of t is
(c) 5 × 10–3 rad (d) 6 × 10–3 rad.
(a) 12.4 µm (b) 14.8 µm
960. In a Young’s double slit experiment, the fringe pat-
(c) 8 µm (d) 3.2 µm. tern is observed on a screen placed at a distance D.
954. A parallel beam of light of wavelength greater than The slits are separated by d and are illuminated by
3500 Å falls on a double slit in a Young’s double slit light of wavelength λ. The distance from the central
experiment. It is found that wavelengths 4000 Å and point where the intensity falls to half the maximum
6000 Å are absent at a distance of 32 mm from the is
position of central maximum and the order of the λD λD
(a) (b)
interference at this point for the two wavelengths 3d 2d
differs by 8. If the distance between the slit and the λD λD
(c) (d) .
screen is 1 m, the separation between the two slits is d 4d
(a) 0.2 mm (b) 0.3 mm 961. Two coherent light waves propagate with a diver-
(c) 0.12 mm (d) 0.18 mm. gence angle θ < < 1. Waves have equal amplitude.
Distance between neighbouring maxima is
955. In Young’s experiment, the source slit S and the two
slits A and B are horizontal. With slit A above B, the λ
(a) λθ (b)
fringes are observed on a vertical screen K. The opti- θ
cal path length from S to B is increased very slightly, (c) λθ 2 (d) λ sin θ.
without changing the optical path length from S to 962. The intensity of light from one source is double of
A. Consequently, the fringe system on K the other coherent source in a double slit experiment.
(a) moves horizontally to the right The ratio of destructive to constructive interference
in the obtained pattern is
(b) moves horizontally to the left
1
(c) moves vertically downward (a) 34 (b)
34
(d) moves vertically upward.
1
(c) 17 (d) .
956. In Young’s double slit experiment, two lights of wave- 17
length 4800 Å and 6000 Å are used. The nth bright
OPTICS 533
963. In Young’s double slit experiment, carried out with wedge will 10th dark fringe be observed by reflected
light of wavelength λ = 5000 Å, the distance between light ?
the slits is 0.2 mm and the screen is at 200 cm from (a) 0.1 mm (b) 0.2 mm
the slits. The central maximum is at x = 0. The third (c) 0.3 mm (d) 0.4 mm.
maximum (taking central maximum as zeroth maxi- 970. White light may be considered to be a mixture of
mum) will be at x equal to waves of λ ranging between 3900 Å and 7800 Å. An
(a) 1.67 cm (b) 1.5 cm oil film of thickness 10,000 Å is examined normally
(c) 0.5 cm (d) 5.0 cm. by the reflected light. If µ = 1.4, then the film appears
[CBSE PMT 1992] bright for
964. A wedge-type air film is formed between the top face (a) 4000 Å, 4667 Å, 5600 Å, 7000 Å
of a crystal and a plane glass plate held a little above (b) 4308 Å, 5091 Å, 6222 Å
it. On shining the film with light of wavelength 6 × (c) 4000 Å, 5091 Å, 5600 Å
10–5 cm a number of interference fringes are observed. (d) 4667 Å, 6222 Å, 7000 Å.
On heating the crystal, the fringes start shifting. If
971. In Fig. 111, P and Q are identical radiators to f waves
we can detect a shift of one-fifth of a fringe-width,
the least count of measuring change in crystal height that are in phase and of the P
is, in 10–5 cm units, same wavelength λ. The radia-
(a) 6 × 4/5 (b) 6 × 2/5 tors are separated by dis- d
(c) 6/5 (d) 6/10. tance d = 3.00 λ. What is the
largest distance from P, along
[National Standard Exam. in Physics 1992]
the x-axis, for which fully de- Q
965. In Young’s double slit experiment, the phase differ- Fig. 111
structive interference occurs ?
ence between the light waves reaching third bright
(a) 8.75 λ (b) 2.5 λ
fringe from the central fringe will be
(a) Zero (b) 2π (c) 4.5 λ (d) 11.25 λ.

(c) 4π (d) 6π. [MP PMT 1994] 972. In an interference pattern, at a point, we observe the
16th order maximum for λ 1 = 6000 Å. What order
966. In a Young’s experiment λ = 4000 Å fringes observed will be visible here if the source is replaced by light of
have a width β. The light illuminating the setup now wavelength λ2 = 4800 Å ?
has λ = 6000 Å and the separation between the inter- (a) 16 (b) 18
fering sources is halved. What is the ratio of the dis-
(c) 20 (d) 24.
tance between the screen and the interfering sources
before and now if the fringe width remains unaltered ? 973. The path difference between two interfering waves at
a point on screen is 70.5 times the wavelength. The
(a) 1/3 (b) 3/1
point is
(c) 3/4 (d) 2/3. [IIT 1996] (a) dark (b) bright
967. In a certain double slit experimental arrangement, (c) not possible (d) green in colour.
interference fringes of width 1.0 mm each are ob- 974. What is the difference between B P
served when light of wavelength 5000 Å is used. Keep- PA and PB ? D
A
ing the set up unaltered, if the source is replaced by B
λ D
another source of wavelength 6000 Å, the fringe width (a) (b) λ B
2
will be B
(a) 0.5 mm (b) 1.0 mm 3λ
(c) (d) 2λ.
2 Fig. 112
(c) 1.2 mm (d) 1.5 mm.
968. In Young’s double slit experiment, using sodium 975. Green light of wavelength 5100 Å from a narrow slit
is incident on a double slit. If the overall separation
light (λ = 5898 Å), 92 fringes are seen. If given colour
of 10 fringes on a screen 3 m away is 3 cm, then the
(λ = 5461 Å) is used, how many fringes will be seen ?
double slit separation is
(a) 62 (b) 67
(a) 0.051 mm (b) 0.51 mm
(c) 85 (d) 99.
(c) 5.1 mm (d) 5.1 cm.
[Rajasthan PET 1996] 976. In a Young’s interference experiment, the nth maxi-
969. A glass wedge of angle 0.01 radian and µ = 1.5 is mum with wavelength λ 1 is at a distance y1. With
illuminated by monochromatic light of wavelength λ
6000 Å falling normally on it. At what distance from wavelength λ2, it is at a distance y2. Then, 1 =
λ2
534 COMPREHENSIVE OBJECTIVE PHYSICS

y1 y2 (a) 2I (b) 4I
(a) (b)
y2 y1 (c) 5I (d) 7I.
2 [IIT Screening 2001]
y12 y
(c) (d) 22 . 982. In a Young’s double slit experiment, 12 fringes are
y2 2 y1
observed to be formed in a certain segment of the
977. In a Young’s experiment, the distance of the second screen when light of wavelength 600 nm is used. If
dark band is observed to be at a distance of 0.3 cm the wavelength of light is changed to 400 nm, number
from the central bright band. The distance of the 4th of fringes observed in the same segment of the screen
bright band from the centre is is given by
(a) 0.2 cm (b) 0.4 cm (a) 12 (b) 18
(c) 0.6 cm (d) 0.8 cm. (c) 24 (d) 30.
978. A thin wedge-shaped air film is trapped between two [IIT Screening 2001]
glass plates, as shown in Fig. 113, and a parallel beam 983. A beam of light consisting of two wavelengths 6500 Å
of monochromatic light is incident normally on it. and 5200 Å, is used to obtain interference fringes in
The fringes observed are Young’s double slit experiment. Suppose the mth
bright fringe due to 6500 Å coincides with nth bright
fringe due to 5200 Å at a minimum distance from
the central maximum. Then
(a) m = 4 ; n = 5 (b) m = 5 ; n = 4
(c) m = 8 ; n = 10 (d) m = 10 ; n = 8.
[Karnataka CET 2001]
Glass
θ plates 984. In the set up shown in Fig. 115, both tubes T1 and T2
contain air having refractive index µ and a maxima
Fig. 113 is observed at O. The fringe system displaces upwards
(a) of irregular shape (b) straight by n = 15 fringes when T1 is filled with ammonia.
Length of tube is l and wavelength of light is λ. Re-
(c) circular (d) hyperbolic.
fractive index of ammonia is
979. In Young’s double slit experiment on interference,
the ratio of the intensities of a bright band and a
dark band is 16 : 1. The ratio of the amplitudes of the T1 S1
interfering waves is
1
(a) (b) 4 O
4
5 T2 S2
(c) (d) 16.
3
[Karnataka CET 1998] Slits
980. A thin slice is cut out of a glass Fig. 115
cylinder along a plane parallel
to its axis. The slice is placed 15λ 15λ
(a) µ + (b) µ −
on a flat glass plate as shown l l
in Fig. 114. The observed in- λ FG λ IJ .
terference fringes from this
(c) 15µ +
l
(d) 15 µ +
H l K
combination shall be Fig. 114
985. In Young’s double slit experiment, the two slits are
(a) straight (b) circular at a distance d apart. Interference pattern is observed
(c) equally spaced on the screen at a distance D from the slits. At a
(d) having fringe spacing which increases as we go outwards point on the screen directly opposite one of the slits,
[IIT Screening 1999] a dark fringe is observed. The wavelength of wave is
nearly
981. Two beams of light having intensities I and 4I inter-
d D
fere to produce a fringe pattern on a screen. The phase (a) (b)
D d
difference between the beams is π/2 at point A and π
at point B. Then the difference between the resultant d2 D
(c) (d) .
intensities at A and B is D d2
OPTICS 535
986. Fig. 116 here shows P and Q as two equally intense 989. In Fresenel biprism experiment, the distance between
coherent sources emitting radiations of wavelength the source and the screen is 1 m and that between
20 m. The separation PQ is 5 m, and phase of P is the source and the biprism is 10 cm. The wavelength
ahead of the phase of Q by 90°. A, B and C are three of light used is 6000 Å. The fringe width obtained is
distant points of observation equidistant from the 0.03 cm and the refracting angle of biprism is 1°. The
mid-point of PQ. The intensity of radiations at A, B, refractive index of the material of the biprism is
C will bear the ratio (a) 1.531 (b) 1.573
B (c) 1.621 (d) 1.732.
990. In a Young’s double slit experiment, the separation
between slits is 2 × 10 –3 m whereas the distance of
screen from the plane of slits is 2.5 m. Light of wave-
lengths in the range 2000 to 8000 Å is allowed to fall
on the slits. The wavelengths in the visible region
C P Q A that will be present on the screen at 10 –3 m from
Fig. 116 central maximum is
(a) 3500 Å (b) 4000 Å
(a) 0 : 1 : 4 (b) 4 : 1 : 0
(c) 6000 Å (d) 5000 Å.
(c) 0 : 1 : 2 (d) 2 : 1 : 0.
991. To obtain the central maximum at the centre, a mica
[National Standard Exam. in Physics 1994] sheet of refractive index 1.5 is introduced. Which of
987. Two coherent sources separated by distance d are the following is correct ?
radiating in phase having wavelength λ. A detector
moves in a big circle around the two sources in the d
S1
plane of the two sources. The angular position of
n = 4 interference maxima is given as S
d
−1 nλ 4λ
(a) sin (b) cos−1
d d
d λ S2
(c) tan −1 (d) cos −1 .
4λ 4d
988. Two coherent monochromatic waves of equal ampli- D(>>d)
tude are brought together to interfere on a screen.
Which one of the following graphs correctly repre- Fig. 118
sents the variations of intensity with position x across
(a) The thickness of sheet is 2( 2 – 1)d in front of S1
the pattern of fringes ?
(b) The thickness of sheet ( 2 + 1)d in front of S2
Intensity

Intensity

(c) The thickness of sheet is (2 2 d – 1) in front of S2


(d) The thickness of sheet is (2 2 – 1)d in front of S1.
O x O x 992. A biprism experiment is set up with edge of biprism
vertical. When half the area on the right hand side of
the biprism is covered with an opaque material, then
(a) (b)
we shall see
(a) fringes only in the left half of the field view
(b) fringes only in the right half of the field view
Intensity

Intensity

(c) intensity of the bright bands will be halved


(d) uniform illumination.
O x O x [National Standard Exam. in Physics 2002]
993. In a certain region A to B in a thin film, we get 10
fringes in the reflected beam of λ = 4200 Å. How many
(c ) (d )
fringes will be observed in the same region with
Fig. 117 λ = 6000 Å ?
536 COMPREHENSIVE OBJECTIVE PHYSICS

(a) 3/2 (b) 7 999. The central fringe of the interference pattern produced
(c) 15 (d) 24. by the light of wavelength 6000 Å is found to shift to
994. The distance between two slits is 0.03 cm. A screen is the position of 4th bright fringe after a glass sheet of
placed at a distance of 1.5 m where an interference refractive index 1.5 is introduced. The thickness of
pattern is observed in which fourth bright fringe is glass sheet would be
at 1 cm from the central maximum. The wavelength (a) 4.8 µm (b) 8.23 µm
of light used will be (c) 14.98 µm (d) 3.78 µm.
(a) 7000 Å (b) 6000 Å 1000. In Young’s double slit experiment, the interference
(c) 5000 Å (d) 4500 Å. pattern is observed to have an intensity ratio between
995. Coherent light is incident on two fine parallel slits S1 bright and dark fringes as 9 : 1. This implies that
and S2. A dark fringe occurs when the phase differ- (a) the intensities at the screen due to the two slits are 4 units
ence between the two waves from S1 and S2 is and 5 units.

FG n + 1IJ π rad (b) the amplitude ratio is 9 : 1


(a)
H 2 K (b) 2nπ rad (c) the amplitude ratio is 3 : 1

F 2n + 1IJ π rad
(c) G
(d) the intensities at the screen due to the two slits are 4 units
H 2 K (d) (2n + 1) π rad. and 1 units. [Pb. PMT 1996]
1001. A thin sheet of glass (µ = 1.5) of thickness 6 micron
996. A source emits electromagnetic waves of wavelength introduced in the path of one of interfering beams in
3 m. One beam reaches the observer directly and the a double slit experiment shifts the central fringe to a
other after reflection from a water surface, travelling position previously occupied by fifth bright fringe.
1.5 m extra distance and with intensity reduced to Then the wavelength of light used is
1/4, as compared to intensity due to the direct beam (a) 6000 Å (b) 3000 Å
alone. The resultant intensity will be (c) 4500 Å (d) 7500 Å.
(a) (1/4) fold (b) (3/4) fold 1002. In a double slit experiment, the separation between
(c) (5/4) fold (d) (9/4) fold. the slits is d and distance of the screen from slits is
[National Standard Exam. in Physics 1998] D. If the wavelength of light used is λ and I is the
intensity of central bright fringe, then intensity at
997. Fig. 119 shows two coherent distance x from central maximum is
sources S1 and S2 emitting at
F π2 xd I FG πxd IJ
wavelength λ. The separation
S1S2 is 1.5λ and S1 is ahead of
S1
θ
(a) I cos2 GH λD JK (b) I2 sin2
H 2λD K
phase by π/2 relative to S2 . Two FG πxd IJ FG πxd IJ .
H λD K H λD K
S2 (c) I cos2 (d) I sin2
of the maxima must occur in
Fig. 119
directions given by sin–1. 1003. A ray of light of intensity I is incident on a parallel
1 1 glass slab at a point A as shown. It undergoes partial
(a) 0, (b) 1,
2 2 reflection and refraction. At each reflection, 25% of
1 1 1 1 incident, energy is reflected. The rays AB and A′B′
(c) , − (d) , − .
6 2 2 6
Imax.
[National Standard Exam. in Physics 1993] undergo interference. The ratio is
Imin.
998. In Fig. 120, two point
sources S1 and S2 emit P B B¢
waves of same wave- I
length ; S 2 lags in A
phase behind S1 by 2πp
A A¢
radian. If there is a
S1
maximum in the di- S2
rection P 1 , the dis- Fig. 120
tance S1 A must (with
n an integer) be C
(a) (n – p)λ (b) (n + p)λ
(c) nλ + p (d) (n – 1/2) λ + p.
[National Standard Exam. in Physics 1996]
Fig. 121
OPTICS 537
(a) 4 : 1 (b) 8 : 1 used. What will be the distance between two consecu-
(c) 7 : 1 (d) 49 : 1. tive bright bands if blue light of wavelength 4500 Å is
used ?
1004. In a Young’s double slit interference experiment, one
(a) 0.3 mm (b) 0.4 mm
slit happens to have its width 4 times that of the
(c) 0.6 mm (d) 0.8 mm.
other. Assuming that intensity of outcoming light is
proportional to slit width, the ratio of the maximum 1010. A person sets up Young’s experiment using a sodium
to minimum intensity in the interference pattern is lamp and placing two slits 1 metre from a screen.
The person is not sure of slit separation and he var-
5
(a) 9 (b) ies the separation and finds that the interference
3 fringes disappear if the slits are too far apart. The
25
(c) 3 (d) . 1°
9 angular resolution of his eye is . How far apart
[National Standard Exam. in Physics 1992] 60
are the slits when he just cannot see the interference
1005. The slits in a Young’s double slit experiment have pattern ?
equal widths and the source is placed symmetrically
[λ = 5890 Å]
relative to the slits. The intensity at the central fringe
(a) 5 mm (b) 4.01 mm
is I0 . If one of the slits is closed, the intensity at this
(c) 2.025 mm (d) 3.025 mm.
point will be
1011. In Young’s double slit experiment, the two slits act
I
(a) I0 (b) 0 as coherent sources of equal amplitude A and wave-
4
length λ. In another experiment with the same setup,
I
(c) 0 (d) 4I0. [MP PMT 1999] the two slits are sources of equal amplitude A and
2 wavelength λ but are incoherent. The ratio of the in-
1006. Fig. 122 shows a tensity of light at the mid-point of the screen in the
double slit experi- Y
first case to that in the second case is
ment. P and Q are nλ (a) 1 : 1 (b) 2 : 1
the two coherent (c) 4 : 1 (d) none of these.
P
sources. The path 1012. Fig. 123 shows Young’s double slit apparatus. S1 and

lengths PY and QY +4 S2 are coherent sources emitting light of wavelength
(n
are nλ and (n + 4) λ λ. Light waves are
Q
respectively where n emitted from S 1 P
is a whole number and S2 in phase. A
and λ is wavelength. point P on the
S1
Taking the central screen corresponds
Fig. 122 to position of 5th
bright fringe as
zero, what is formed at Y ? maximum. What is S2
the phase difference
(a) First Bright (b) First Dark
between waves when Fig. 123
(c) Fourth Bright (d) Second Dark.
they arrive at P ?
1007. In a biprism experiment, the distance of 20th bright (a) 5 π (b) 9 π
band from the centre of interference pattern is 8 mm.
(c) 10 π (d) 15 π.
The distance of 30th bright band from the centre is
1013. A student is
(a) 10 mm (b) 12 mm
asked to measure
(c) 14 mm (d) 16 mm.
the wavelength
1008. Two coherent waves of intensities I and 4I interfere of monochro-
S2
at a point. The resultant intensity is 3I. Then the matic light. He L S1 5 cm M
phase difference between the two waves at the point sets up the appa- S3
is ratus shown in
(a) 0° (b) 60° Fig. 124. S 1 , S 2
(c) 90° (d) 120°. and S3 are nar- 60 cm
1009. Distance between two consecutive dark bands is row parallel slits.
10 cm
0.4 mm when yellow light of wavelength 6000 Å is L is a sodium
Fig. 124
538 COMPREHENSIVE OBJECTIVE PHYSICS

lamp. M is micrometer eye-piece. The student fails to 1019. In the Young’s slits arrangement shown, a pattern of
observe interference fringes. You would advise him equally spaced, parallel fringes appears on a screen
to placed at S.
(a) decrease the distance between S2 and S3 Which quantity, if increased, would cause the sepa-
(b) increase the widths of the slits ration of the fringes to increase ?
(c) replace L by a source of white light
(d) replace M by a telescope. Double
Single
1014. What is the least thickness of a soap film which will slit
slit
S
appear black when viewed with sodium light (λ =
589.3 nm) reflected perpendicular to the film ? Given :
y
µ of soap solution = 1.38.
(a) 213.5 nm (b) 428 nm d
Light
(c) 617 nm (d) 10000 Å. y
source
1015. Under which of the following sets of conditions will
the separation of the bright fringes of a double-slit
interference pattern be greatest ?
distance distance wavelength
x l
between from slits of source
slits to screen
(a) small small short Fig. 125

(b) small large short (a) x (b) y


(c) small large long (c) d (d) l.
(d) large small short 1020. What is the angular separation between the consecu-
(e) large small long. tive bright fringes in a Young’s double slit experi-
ment with blue-green light of wavelength 500 nm.
1016. What is the approximate minimum thickness of a
The separation between the slits is 2 × 10–3 m.
film which will strongly reflect the light of wavelength
(a) 0.014° (b) 0.14°
589 nm. The refractive index of the film is 1.25.
(c) 0.28° (d) 0.64°.
(a) 9 nm (b) 109 nm
1021. When a two slit arrangement was set up to produce
(c) 111 nm (d) 118 nm.
interference fringes on a screen using a monochro-
1017. An oil film of thickness 10,000 A and refractive index
matic source of green light, the fringes were found to
1.4 is examined by reflection of white light along the
be too close together for convenient observation. In
normal. The dominant wavelengths in the reflected
which of the following ways would it be possible to
light will be around
increase the separation of the fringes ?
(a) 4300 Å, 5100 Å, 6200 Å
(a) Decrease the distance between the screen and the slits
(b) 4100 Å, 4700 Å, 5600 Å, 7000 Å
(b) Increase the distance between the source and the slits
(c) 4700 Å, 7000 Å (d) 4000 Å, 5600 Å.
(c) Have a larger distance between the two slits
[National Standard Exam. in Physics 1993] (d) Increase the width of each slit
1018. Fringes of separation y are observed in a plane (e) Replace the light source with a monochromatic source of
1.00 m from a Young’s slit arrangement illuminated red light.
by yellow light of wavelength 600 nm. 1022. In a Fresnel biprism experiment, the two positions of
At what distance from the slits would fringes of the lens give separation between the slits as 16 cm and
same separation y be observed when using blue light 9 cm respectively. What is the actual distance of sepa-
of wavelength 400 nm ? ration ?
(a) 0.33 m (b) 0.67 m (a) 12.5 cm (b) 12 cm
(c) 0.75 m (d) 1.50 m. (c) 13 cm (d) 14 cm.
[CBSE PMT 1996]
OPTICS 539
1023. In the ideal double-slit experiment, when a glass-plate 1027. A double slit experiment is performed with light of
(refractive index 1.5) of thickness t is introduced in wavelength 500 nm. A thin film of thickness 2 µm
the path of one of the interfering beams (wavelength and refractive index 1.5 is introduced in the path of
λ), the intensity at the position where the central the upper beam. The location of the central maxi-
maximum occurred previously remains unchanged. mum will
The minimum thickness of the glass plate is (a) remain unshifted
(a) 2λ (b) 2λ/3 (b) shift downward by nearly two fringes
(c) λ/3 (d) λ. [IIT 2002] (c) shift upward by nearly two fringes
1024. Two identical narrow slits S1 and S2 are illuminated (d) shift downward by 10 fringes. [AIIMS 2003]
by light of wavelength λ from a point source P. 1028. In Young’s double slit experiment, let S1 and S2 be
the two slits. A thin film of transparent material of
thickness t and of refractive index µ is placed in front
S1 of slit S1. If β is the fringe width, on introduction of
l1 the film, the central maximum will shift towards
l3
β β
P (a) A by µ t (b) B by µ t
λ λ
β β
l2 Q (c) A by (µ − 1) t (d) B by (µ − 1) t .
S2
l4 λ λ

DIFFRACTION
1029. Light of wavelength 520 × 10–9 m falls normally on a
Fig. 126
plane diffraction grating having 5 × 103 lines per cm.
If, as shown in Fig. 126, the light is then allowed to The maximum number of images seen is
fall on a screen and if n is a positive integer, the (a) 5 (b) 50
condition for destructive interference at Q is that (c) 4 (d) 121.
(a) (l1 – l2) = (2n + 1) λ/2 (b) (l3 – l4) = (2n + 1)λ/2 1030. Light of wavelength 5000 Å is incident normally on a
(c) (l1 + l3) – (l2 + l4) = nλ slit. The first minimum of the diffraction pattern is
(d) (l1 + l3) – (l2 + l4) = (2n + 1)λ/2. formed at a distance of 5 mm from central maxi-
1025. A converging lens of long focal length is placed on a mum. The screen is situated at a distance of 2 m
plane mirror and illuminated normally with mono- from the slit. The slit width is
chromatic light so that interference fringes (Newton’s (a) 0.2 mm (b) 0.8 mm
rings) are visible. (c) 0.4 mm (d) 2.0 mm.
If the mirror is gradually separated from the lens, 1031. A parallel monochromatic beam of light is incident
(a) the ring system is unchanged normally on a narrow slit. A diffraction pattern is
(b) the central spot is unchanged but the distances between formed on a screen placed perpendicular to the di-
the other rings increase until they vanish from the field of rection of the incident beam. At the first minimum
view. of the diffraction pattern, the phase difference between
(c) the central spot is unchanged but the distances between the rays coming from the two edges of the slit is
the other rings decrease until they merge. (a) 0 (b) π/2
(d) new rings ‘appear’ from the centre (c) π (d) 2π. [IIT 1998]
(e) rings ‘disappear’ at the centre. 1032. In a Fraunhoffer diffraction experiment at a single
1026. In Fig. 127, CP represents a Q O R slit using light of wavelength 400 nm, the first mini-
wavefront and AO and BP, the θθ
mum is formed at an angle of 30°. Then the direction
corresponding two rays. Find the d θ of the first secondary maximum is
condition on θ for constructive in- C
−1 FG 4 IJ
terference at P between the ray BP
and reflected ray OP.
A
P
(a) tan H 3K (b) 60°

−1 F 3 I FG 3 IJ
(c) sin GH JK
−1
(a) cos θ = 3λ/2d
(b) cos θ = λ/4d
B
Fig. 127
4
(d) tan H 4K
[Karnataka CET 1997]
(c) sec θ – cos θ = λ/d (d) sec θ – cos θ = 4λ/d.
[IIT Screening 2003]
540 COMPREHENSIVE OBJECTIVE PHYSICS

1033. The distance between the first and the sixth minima 1039. A beam of monochromatic light of wavelength λ falls
in the diffraction pattern of a single slit is 0.5 mm. normally on a diffraction grating of line spacing d. If
The screen is 0.5 m away from the slit. If the wave- θ is the angle between the second-order diffracted
length of light used is 5000 Å, then the slit width will beam and the direction of the incident light, what is
be the value of sin θ ?
(a) 5 mm (b) 2.5 mm
(a) λ/d (b) d/λ
(c) 1.25 mm (d) 1.0 mm.
(c) 2λ/d (d) 2d/λ
1034. A slit 5 cm wide is irradiated normally with (e) d/2λ.
microwaves of wavelength 1.0 cm. Then the angular
spread of the central maximum on either side of 1040. A parallel beam of monochromatic light of wavelength
incident light is nearly λ is incident normally on a diffraction grating G.
(a) 1/5 radian (b) 4 radian The angle between the directions of the two second-
(c) 5 radian (d) 6 radian. order diffracted beams at P1 and P2 is α, as shown in
the diagram below.
1035. In a diffraction grating experiment, the first order
image of the 435.8 nm blue light from a commercial
P1
mercury vapour discharge lamp occurred at an an-
gle of 15.8°. A first order red line was also observed
at 23.7°, thought to be produced by an impurity in
the mercury.
G
The wavelength of red lines of various elements are
listed below. Which element is the impurity in the
α
mercury lamp ?
element wavelength/nm
(a) zinc 636.0
(b) cadmium 643.3
(c) hydrogen 656.3
(d) neon 670.8 P2
(e) caesium 697.8.
1036. A narrow slit of width 1 mm is illuminated by Fig. 128
monochromatic light of wavelength 600 nm. The What is the spacing of the lines on the grating ?
distance between the first minima on either side on a
4λ 2λ
screen at a distance of 2 m is (a) (b)
sin α sin α
(a) 1.2 cm (b) 1.2 mm
λ 2λ
(c) 2.4 cm (d) 2.4 mm. [IIT 1994] (c) (d)
sin α sin (α/2)
1037. Monochromatic light of wavelength λ is incident nor-
λ
mally on a diffraction grating consisting of alternate (e) .
sin (α/2)
opaque strips of width a and transparent strips of
width b. The angle between the emerging zero-order 1041. Consider Fraunhoffer diffraction pattern obtained
and first-order spectra depends on with a single slit illuminated at normal incidence. At
(a) a, b and λ (b) a and λ only the angular position of the first diffraction minimum
(c) b and λ only (d) a and b only the phase difference (in radian) between the wavelets
(e) λ only. from the opposite edges of the slit is
1038. When monochromatic light of wavelength 5.0 × 10–7 π π
(a) (b)
m is incident normally on a plane diffraction grat- 4 2
ing, the second-order diffraction lines are formed at (c) π (d) 2π. [IIT 1995]
angles of 30° to the normal to the grating. 1042. Light of wavelength λ is incident normally on a dif-
What is the number of lines per millimetre of the fraction grating for which the slit spacing is equal to
grating ? 3λ.
(a) 250 (b) 500 What is the sine of the angle between the second order
(c) 1000 (d) 2000 maximum and the normal ?
(e) 4000.
OPTICS 541
1 1 1047. Monochromatic light of wavelength λ is incident
(a) (b)
6 3 normally on a single slit RS of width a. The diffraction
2 pattern is formed on a screen PP′. The first minimum
(c) (d) 1
3 of this pattern makes an angle θ with the direction of
3 the incident light, as shown in Fig. 130.
(e) .
2
1043. A monochromatic plane wave of speed X
P
Monochromatic
c and wavelength λ is diffracted at a P light
small aperture. The diagram illus- R
trates successive wavefronts. θ
After what time will some portion S
of the wavefront XY reach P ?
3λ 2λ Y
(a) (b) P′
2c c
3λ 4λ Fig.129 Diffraction
(c) (d) pattern
c c
6λ Fig. 130
(e) .
c
Description for Questions 1044 and 1045 . Which one of the following gives the correct expres-
Theory of diffraction of waves tells that any beam sions both for the path difference (SP—RP) and for
limited in transverse width by an amount d shows sin θ ?
an extra angular spread λ/d besides the spread oth- (SP—RP) sin θ
erwise expected. Here λ is the wavelength of the waves. (a) λ/2 λ/2a
Typical examples are a telescope lens of aperture dia (b) λ/2 λ/a
d receiving light, or a circular disc of dia d oscillat- (c) λ/2 2λ/a
ing normal to itself and emitting sound waves ; in (d) λ λ/2a
each case angular spread λ/d occurs besides what (e) λ λ/a.
the geometry would suggest. 1048. Fig. 131 here shows a wavefront of light of wave-
Now answer questions 1044 and 1045 : length λ incident normally on a slit of width d. For a
distant observation point at inclination θ, the dis-
1044. A coastal guardship uses 50 cm dia disc to send sound
turbances from the ends A and B are represented by
waves of λ = 2 mm to make undersea studies. At
the ‘amplitude vectors’ (phasors) P and Q respectively.
200 m distance, the accuracy with which it can lo-
The angle ∆φ is given by
cate objects is nearest to
(a) 5 m (b) 1 m w

(c) 0.2 m (d) 0.1 m.


Q
[National Standard Exam. in Physics 1992]
A
1045. A torch uses a filament of dia 1.5 mm at the focus of d θ O ∆φ
a lens of dia 3.0 cm and focal length 15 cm. It sends B
a ‘parallel’ beam. But this has a finite spread due to
P
(i) finite size of the filament, and (ii) diffraction. If λ
may be taken as 6 × 10–5 cm, the ratio of spread due w
to (i) to that due to (ii) is nearest to
Fig. 131
(a) 500 (b) 50
F dI
(a) GH JK sin θ
FG 2πd IJ cos θ
(c) 5 (d) 1/5. λ
(b) HλK
[National Standard Exam. in Physics 1992] 2π 2π
(c) d sin θ (d) d tan θ .
1046. A slit of width d is illuminated by white light. The λ λ
first minimum (λ = 5900 Å) will fall at θ = 30°, if d [National Standard Exam. in
equals Physics 1990, Modified]
(a) 2950 Å (b) 5.9 × 10–4 mm 1049. An astronaut is looking down on earth’s surface from
(c) 1.18 micron (d) 2.34 × 10–4 cm. a space shuttle at an altitude of 400 km. Assuming
542 COMPREHENSIVE OBJECTIVE PHYSICS

that the astronaut’s pupil diameter is 5 mm and the I0


I0
wavelength of visible light is 500 nm, the astronaut (a) (b)
2 2
will be able to resolve linear objects of the size of
about (c) I0 2 (d) 2I0 .
(a) 0.5 m (b) 5 m 1057. A clear sheet of polaroid is placed on the top of a
(c) 50 m [AIIMS 2003]
(d) 500 m similar sheet so that axes make an angle of 30° with
1050. A telescope has an objective lens of 10 cm diameter each other. The ratio of intensity of the emergent
and is situated at a distance of one kilometre from light to that of unpolarised incident light is
two objects. The minimum distance between these (a) 1 : 3 (b) 1 : 4
two objects, which can be resolved by the telescope, (c) 3 : 4 (d) 3 : 8.
when the mean wavelength of light is 5000 Å, is of 1058. Two polaroids are aligned with their axes of trans-
the order of mission making an angle of 45°. They are followed
(a) 5 m (b) 5 mm by a third polaroid whose axis of transmission makes
(c) 5 cm (d) 0.5 m an angle of 90° with the first polaroid. What fraction
[All India PM/PD 2004] of the maximum possible light (if all polaroid were at
the same angle) passes through all three ?
1051. A slit is illuminated by white light. The first mini-
1
mum for red light falls at an angle of 30°. If wave- (a) 1 (b)
length of red light is 6500 Å, then the slit width is 2
1 1
(a) 0.3 micron (b) 1.3 micron (c) (d) .
(c) 2 m (d) 1 mm. 4 16
1059. A beam of unpolarised light having flux 10 –3 watt
1052. Light of wavelength 6000 Å is incident normally on a
falls normally on a polariser of cross sectional area
slit. The emergent beam forms a diffraction pattern
3 × 10–4 m 2. The polariser rotates with an angular
on a screen 2.5 m away. If the width of the slit is
frequency of 31.4 rad/s. The energy of light passing
6 mm, then the width of the central maximum is
through the polariser per revolution will be
(a) 0.3 mm (b) 0.5 mm
(c) 0.7 mm (d) 0.9 mm. (a) 10–4 joule (b) 10–3 joule
1053. Light is incident normally on a diffraction grating (c) 10–2 joule (d) 10–1 joule.
through which the first order diffraction is seen at 1060. Two tourmaline crystals are oriented with their
32°. The second order diffraction will be seen at crystallographic axes making an angle of 30°. What
(a) 48° (b) 84° percentage of the incident unpolarised light would
(c) 80° pass through the system ?
(d) There is no second order diffraction in this case. (a) 25 (b) 37.5
1054. Green light of wavelength 5400 Å is diffracted by a (c) 45 (d) 75.
grating ruled 2000 lines/cm. The angular deviation
1061. If an unpolarised light beam of intensity 4a2 passes
of the third order image is
through a polaroid, then the intensity of the emer-
(a) sin–1 (0.324) (b) cos–1 (0.324)
gent plane polarised light is
(c) tan–1 (0.324) (d) 82°.
(a) a (b) a2
1055. Light of wavelength 6328 Å is incident on a slit hav-
(c) 2a2 (d) 3a2.
ing a width of 0.2 mm. The width of central maxi-
mum, measured from minimum to minimum of the 1062. A light beam is incident on a rectangular glass plate
diffraction pattern on a screen 9 m away will be nearly (µ = 1.54.). The re-
A B
flected light OB
(a) 0.36° (b) 0.18°
passes through a Air
(c) 0.72° (d) 0.09°.
nicol prism. On ob- 29°

POLARISATION serving the trans- O


Glass
mitted light while
1056. A beam of unpolarised light passes through a tour- rotating the prism, Fig.132
maline crystal A and then it passes through a second it is seen that
tourmaline crystal B oriented so that its principal (a) intensity of light reduces to zero.
plane is parallel to that of A. The intensity of emer- (b) intensity of light decreases and then increases.
gent light is I0. Now B is rotated by 45° about the (c) there is no change of intensity of light.
ray. The emergent light will have intensity (d) intensity of light reduces to zero slowly and then starts to
increase.
OPTICS 543

1063. Light is incident from vacuum on the surface of a (a) 1, 3 only (b) 2, 4 only
medium of reflective index µ. If the angle of incidence (c) 3, 4 only (d) all four.
obeys the relation i = tan–1 (µ), the angle between the [National Standard Exam. in Physics 1993]
reflected and refracted rays is 1069. An unpolarised beam of light is incident on a group
(a) 30° (b) 45° of four polarising sheets which are arranged in such
a way that the characteristic direction of each polar-
(c) 90° (d) 135°.
ising sheet makes an angle of 30° with that of the
1064. Light from a denser medium 1 passes to a rarer me- preceding sheet. The % age of incident light trans-
dium 2. When the angle of incidence is θ, the reflected mitted by first polariser will be
and refracted rays are mutually perpendicular. The (a) 100% (b) 50%
critical angle will be (c) 25% (d) 12.5%.
(a) sin–1 (cot θ) (b) sin–1 (tan θ) 1070. Six polarisers are arranged in series with each
(c) sin–1 (cos θ) (d) sin–1 (sec θ). polariser making an angle 30° with the preceding one.
[National Standard Exam. in Physics 2000] If unpolarised light of intensity I0 is incident upon
the first polariser, what is the intensity of light emerg-
1065. A plane polarised beam of in- E ing from the final polariser ?
tensity I is incident on a P 243
9 3 I0 I0
polariser with the electric vec- (a) (b)
132 1024
tor inclined at 30° to the Optic A 243
axis of the polariser. Light com- (c) I0 (d) Nearly zero.
2048
ing out of the polariser passes 30°
30° 1071. Unpolarised light of intensity 32 W m –2 passes
through an analyser whose op- through three polarisers such that the transmission
tic axis is inclined at 30° to that axis of the last polariser is crossed with the first. If
Fig. 133 the intensity of emerging light is 3 W–2 m2, what is
of polariser. Intensity of light
coming out of the analyser is the angle between the transmission axes of the first
two polarisers ?
9 3
(a) I (b) I (a) 30° (b) 45°
16 4
(c) 60° (d) 90°.
1 3
(c) I (d) I. 1072. Four polaroids are placed such that the optic axis of
4 2 each is inclined at an angle of 30° with the optic axis
[National Standard Exam. in Physics 1997] of the preceding one. If unpolarised light of intensity
1066. Suppose θ is the polarising angle for a transparent I0 falls on the first polaroid, then the light transmitted
medium and the speed of light in that medium is v. from the fourth is
Then according to Brewster law 1
(a) I0 (b) I0
(a) θ = cot–1 (v/c) (b) θ = cos–1 v 16
(c) θ = sin–1 v (d) θ = cosec–1 v. (c) 0.21 I0 (d) 0.93 I0.

[Karnataka CET 1987] 1073. Linearly polarised light is incident at Brewster angle
on the surface of a medium. If the incident beam is
1067. A ray of light is incident on the surface of a glass polarised parallel to the plane of incidence, then the
plate of refractive index 1.55 at the polarising angle. parallel component of light is
The angle of refraction is
(a) partly refracted (b) completely refracted
(a) 57° 11′ (b) 32° 49′
(c) completely reflected (d) completely absorbed.
(c) 147° 11′ (d) 0°.
1068. A beam of light is passed through a polariser and an 1074. Refractive indices of a crystal for ordinary and spe-
analyser. The latter is then set for maximum inten- cial rays are 1.5442 and 1.5533 respectively. The thick-
sity of transmission. To reduce the intensity to 14 1
ness of a wave plate of the crystal for light of wave-
fraction, one tries four ways of turning the analyser 4
(A) and polariser (P) length 5893 Å is
(a) 1.02 × 10–4 m (b) 1.00 × 10–6 m
1. A + 20°, P – 40°, 2. A + 75°, P + 15°,
(c) 1.16 × 10–5 m (d) 1.61 × 10–5 m.
3. A – 60°, P – 0°, 4. A 0°, P – 60°.
Here + and – stand for clockwise and anti-clockwise 1075. A ray of light strikes a glass plate at an angle of 60°.
turning. Which one will work ? If the reflected and refracted rays are perpendicular
to each other, then the index of refraction of glass is
544 COMPREHENSIVE OBJECTIVE PHYSICS

3 velocity of sound, then the velocity of the car, in the


(a) (b) 3
2 same velocity units, will be
3 (a) ν/3 (b) ν/4
(c) (d) 3.
2
(c) ν/2 (d) ν/ 2
DOPPLER EFFECT [All India PM/PD 2004]
1076. A rocket is going away from earth at a speed 0.2 c, 1084. A star emits light of 5500 Å wavelength. It appears
where c = speed of light. It emits signals of frequency blue to an observer on earth. It means
4 × 107 Hz. Which frequency will be observed by an (a) star is moving away from earth
observer on earth ? (b) star is stationary (c) star is moving towards earth
(a) 4 × 106 Hz (b) 3.2 × 107 Hz (d) none of the above.
(c) 3 × 106 Hz (d) 5 × 107 Hz. 1085. The wavelength of light received from a galaxy is
1077. A rocket is moving away from the earth at a speed of 0.4% greater than that received from an identical
6 × 107 m s–1. The rocket has blue light in it. What source on the earth. The velocity of galaxy relative to
will be the wavelength of light recorded by an observer earth is
on the earth ? (Given : wavelength of blue light = (a) 1.2 × 105 m s–1 (b) 1.2 × 104 m s–1
4600 Å.) (c) 1.2 × 106 ms –1
(d) 1.2 × 107 m s–1.
(a) 4600 Å (b) 5520 Å
1086. A radar sends a radio signal of frequency 9 × 109 Hz
(c) 3680 Å (d) 3920 Å. towards an aircraft approaching the radar. If the re-
1078. A light source approaches the observer with velocity flected wave shows a frequency shift of 3 × 103 Hz,
0.8 c. The doppler shift for the light of wavelength the speed (in m s –1) with which the aircraft is ap-
5500 Å is proaching the radar, is
(a) 4400 Å (b) 1833 Å (Given : Velocity of the radio signal is 3 × 108 m s–1)
(c) – 3667 Å (d) 7333 Å. [MP PET 1996] (a) 150 (b) 100
1079. A rocket is going away from earth at a speed of (c) 50 (d) 25 [EAMCET 2003]
106 m s–1. If wavelength of light wave emitted by it
be 5700 Å, what will be its Doppler’s shift ? MISCELLANEOUS
(a) 200 Å (b) 19 Å
(c) 20 Å (d) 0.2 Å. 1087. Air has refractive index 1.0003. The thickness of air
1080. The wavelength of light observed on earth, from a column, which will have one more wavelength of
moving star is found to decrease by 0.05%. Relative yellow light (6000 Å) than in the same thickness of
to earth, the star is vacuum is
(a) moving away with a velocity of 1.5 × 105 m s–1. (a) 2 mm (b) 2 cm
(b) moving closer with a velocity of 1.5 × 105 m s–1.
(c) 2 m (d) 2 km.
(c) moving away with a velocity of 1.5 × 104 m s–1.
(d) moving closer with a velocity of 1.5 × 104 m s–1. 1088. 80 g of impure sugar when dissolved in a litre of
[MP PMT 1998] water gives an optical rotation of 9.9° when placed in
a tube of length 20 cm. If the specific rotation of sugar
1081. If a star is moving towards earth, then the lines are
shifted towards is 66°, then concentration of sugar solution will be
(a) red (b) infra-red (a) 80 g/litre (b) 75 g/litre
(c) blue (d) green. [AIIMS 1997] (c) 65 g/litre (d) 50 g/litre.
1082. A star emitting radiation of wavelength 5000 Å is 1089. According to modern theory for nature of light, the
approaching earth with a velocity of 1.5 × 106 m s–1. light has
The change in wavelength of radiation as received (a) wave nature only (b) particle nature only
on earth is
(c) both wave and particle (dual) nature
(a) 25 Å (b) zero
(c) 100 Å (d) 2.5 Å. (d) neither particle nor wave nature. [MP PMT 1998]
[CBSE PMT 1995] 1090. In Foucault’s rotating mirror experiment to deter-
mine the velocity of light in air, the distance of the
1083. A car is moving towards a high cliff. The car driver
rotating mirror from the concave mirror is 6000 m.
sounds a horn of frequency f. The reflected sound
The reflected ray is displaced through an angle of
heard by the driver has a frequency 2f. If v be the
OPTICS 545
48π × 10–3 radian. If the velocity of light in air is 1097. Light travels faster in air than in glass according to
3 × 108 ms–1, the number of rotations made by the (a) Wave theory of light (b) Corpuscular theory of light
rotating mirror in one minute are (c) Both 1 and 2 (d) Neither 1 nor 2.
(a) 3,000 (b) 9,000 [CMPT 1992]
(c) 18,000 (d) 36,000 [EAMCET 2003] 1098. Wavelength of light of frequency 100 Hz is
1091. The specific rotation of a liquid of length 10 cm, con- (a) 2 × 106 m (b) 3 × 106 m
centration 2 g cm–3 is 40°. The angle of rotation is (c) 4 × 106 m (d) 5 × 106 m.
(a) 10° (b) 70° 1099. A circular disc of which 2/3 part is coated with yel-
(c) 80° (d) 90°. low and 1/3 part is with blue is rotated about its
[Karnataka CET 1984] central axis with high velocity. Then it will be seen
as
1092. Time taken by sunlight to pass through a window of
3 (a) green (b) brown
thickness 4 mm whose refractive index is is (c) violet (d) white.
2
(a) 2 × 10–4 s (b) 2 × 108 s 1100. Waves from two different sources overlap near a par-
(c) 2 × 10–11 s (d) 2 × 1011 s. ticular point. The amplitude and the frequency of
1093. The ability of an optical instrument to show the im- the two waves are the same. The ratio of intensities
ages of two adjacent point objects as separate is called when the two waves arrive in phase to that when
(a) dispersive power (b) magnifying power they arrive 90° out of phase is
(c) resolving power (d) none of these. (a) 1 : 1 (b) 2 :1
1094. Velocity of light cannot be determined by (c) 2 : 1 (d) 4 : 1.
(a) Fizeau’s toothed wheel method
1101. Two points separated by a distance of 0.1 mm can
(b) Foucault’s Rotatory mirror method just be inspected in a microscope when light of
(c) Nicol Rotatory mirror method wavelength 6000 Å is used. If the light of wavelength
(d) Fresnel’s biprism. [CPMT 1992] 4800 Å is used, then the limit of resolution will be
1095. What should be refractive index of a transparent (a) 0.08 mm (b) 0.10 mm
medium to be invisible in vacuum ? (c) 0.12 mm (d) 0.80 mm.
(a) 1 (b) < 1 1102. A light wave of frequency 5 × 10 4 Hz enters a me-
(c) > 1 (d) none of these. dium of refractive index 1.5. In this medium, the
speed of light wave is
1096. The specific rotation of an optically active substance
of length l cm and concentration of c with an optical (a) 6 × 1010 cm s–1 (b) 4 × 1010 cm s–1
rotation θ is given by (c) 3.5 × 1010 cm s–1 (d) 2 × 1010 cm s–1.
(a) l c/10θ (b) θl c/10 1103. In Q. 1102, the wavelength of light wave in the
(c) 10θ/lc (d) 10/θ/lc. medium is
[Karnataka CET 1984] (a) 4000 Å (b) 5000 Å
(c) 6000 Å (d) 8000 Å.

Answers (Set II)


581. (b) 582. (a) 583. (b) 584. (b) 585. (a) 586. (b) 587. (c) 588. (b)
589. (c) 590. (a) 591. (b) 592. (a) 593. (b) 594. (a) 595. (a) 596. (b)
597. (c) 598. (b) 599. (c) 600. (c) 601. (d) 602. (d) 603. (b) 604. (c)
605. (b) 606. (c) 607. (c) 608. (c) 609. (b) 610. (d) 611. (c) 612. (b)
613. (d) 614. (d) 615. (d) 616. (d) 617. (d) 618. (d) 619. (a) 620. (d)
621. (a) 622. (b) 623. (d) 624. (b) 625. (e) 626. (c) 627. (a) 628. (b)
629. (a) 630. (c) 631. (a) 632. (c) 633. (c) 634. (a) 635. (a) 636. (b)
637. (c) 638. (d) 639. (d) 640. (a) 641. (d) 642. (c) 643. (d) 644. (b)
645. (a) 646. (c) 647. (a) 648. (d) 649. (c) 650. (b) 651. (c) 652. (a)
653. (d) 654. (c) 655. (a) 656. (c) 657. (c) 658. (b) 659. (c) 660. (c)
661. (d) 662. (a) 663. (c) 664. (b) 665. (a) 666. (a) 667. (d) 668. (c)
546 COMPREHENSIVE OBJECTIVE PHYSICS

669. (b) 670. (b) 671. (c) 672. (a) 673. (a) 674. (c) 675. (b) 676. (a)
677. (b) 678. (b) 679. (d) 680. (c) 681. (a) 682. (b) 683. (c) 684. (a)
685. (c) 686. (a) 687. (b) 688. (b) 689. (c) 690. (c) 691. (c) 692. (b)
693. (b) 694. (a) 695. (a) 696. (b) 697. (d) 698. (c) 699. (a) 700. (b)
701. (b) 702. (c) 703. (b) 704. (c) 705. (c) 706. (b) 707. (b) 708. (b)
709. (a) 710. (a) 711. (d) 712. (b) 713. (c) 714. (a) 715. (b) 716. (d)
717. (d) 718. (b) 719. (c) 720. (b) 721. (c) 722. (c) 723. (a) 724. (b)
725. (c) 726. (c) 727. (a) 728. (b) 729. (c) 730. (b) 731. (b) 732. (d)
733. (a) 734. (d) 735. (b) 736. (b) 737. (b) 738. (a) 739. (b) 740. (b)
741. (a) 742. (d) 743. (a) 744. (c) 745. (a) 746. (d) 747. (d) 748. (b)
749. (b) 750. (d) 751. (a) 752. (c) 753. (d) 754. (d) 755. (b) 756. (c)
757. (d) 758. (c) 759. (d) 760. (c) 761. (a) 762. (c) 763. (d) 764. (c)
765. (b) 766. (b) 767. (b) 768. (b) 769. (b) 770. (d) 771. (d) 772. (a)
773. (c) 774. (c) 775. (c) 776. (d) 777. (d) 778. (a) 779. (a) 780. (a)
781. (a) 782. (b) 783. (b) 784. (b) 785. (a) 786. (c) 787. (c) 788. (a)
789. (b) 790. (a) 791. (c) 792. (c) 793. (b) 794. (d) 795. (d) 796. (a)
797. (c) 798. (c) 799. (b) 800. (b) 801. (b) 802. (a) 803. (d) 804. (b)
805. (d) 806. (b) 807. (a) 808. (b) 809. (b) 810. (c) 811. (d) 812. (d)
813. (c) 814. (d) 815. (b) 816. (a) 817. (c) 818. (d) 819. (c) 820. (c)
821. (b) 822. (b) 823. (a) 824. (c) 825. (b) 826. (b) 827. (c) 828. (c)
829. (a) 830. (a) 831. (c) 832. (b) 833. (b) 834. (a) 835. (b) 836. (b)
837. (c) 838. (d) 839. (a) 840. (b) 841. (c) 842. (b) 843. (c) 844. (d)
845. (b) 846. (c) 847. (a) 848. (c) 849. (d) 850. (b) 851. (d) 852. (d)
853. (d) 854. (b) 855. (c) 856. (a) 857. (a) 858. (c) 859. (d) 860. (b)
861. (a) 862. (b) 863. (a) 864. (c) 865. (c) 866. (c) 867. (c) 868. (b)
869. (b) 870. (a) 871. (a) 872. (b) 873. (a) 874. (d) 875. (a) 876. (b)
877. (a) 878. (b) 879. (e) 880. (c) 881. (d) 882. (b) 883. (c) 884. (c)
885. (a) 886. (c) 887. (d) 888. (d) 889. (c) 890. (c) 891. (d) 892. (b)
893. (b) 894. (a) 895. (c) 896. (a) 897. (b) 898. (b) 899. (b) 900. (c)
901. (c) 902. (b) 903. (a) 904. (b) 905. (c) 906. (b) 907. (a) 908. (b)
909. (a) 910. (d) 911. (c) 912. (b) 913. (c) 914. (c) 915. (a) 916. (a)
917. (a) 918. (d) 919. (d) 920. (b) 921. (c) 922. (a) 923. (c) 924. (d)
925. (a) 926. (c) 927. (a) 928. (b) 929. (b) 930. (a) 931. (d) 932. (d)
933. (d) 934. (a) 935. (a) 936. (d) 937. (a) 938. (c) 939. (c) 940. (b)
941. (d) 942. (a) 943. (a) 944. (a) 945. (b) 946. (a) 947. (c) 948. (b)
949. (c) 950. (b) 951. (c) 952. (b) 953. (c) 954. (b) 955. (c) 956. (d)
957. (a) 958. (a) 959. (c) 960. (d) 961. (b) 962. (b) 963. (b) 964. (d)
965. (d) 966. (b) 967. (c) 968. (d) 969. (b) 970. (b) 971. (a) 972. (c)
973. (a) 974. (d) 975. (b) 976. (a) 977. (d) 978. (b) 979. (c) 980. (a)
981. (b) 982. (b) 983. (a) 984. (a) 985. (c) 986. (d) 987. (b) 988. (c)
989. (b) 990. (b) 991. (a) 992. (d) 993. (b) 994. (c) 995. (d) 996. (d)
997. (d) 998. (b) 999. (a) 1000. (d) 1001. (a) 1002. (c) 1003. (d) 1004. (a)
1005. (b) 1006. (c) 1007. (b) 1008. (d) 1009. (a) 1010. (c) 1011. (b) 1012. (c)
1013. (a) 1014. (a) 1015. (c) 1016. (d) 1017. (a) 1018. (d) 1019. (d) 1020. (a)
1021. (e) 1022. (b) 1023. (a) 1024. (d) 1025. (e) 1026. (b) 1027. (c) 1028. (c)
1029. (c) 1030. (a) 1031. (d) 1032. (c) 1033. (b) 1034. (a) 1035. (b) 1036. (d)
1037. (a) 1038. (b) 1039. (c) 1040. (d) 1041. (d) 1042. (c) 1043. (c) 1044. (b)
1045. (b) 1046. (c) 1047. (e) 1048. (c) 1049. (c) 1050. (b) 1051. (b) 1052. (b)
1053. (d) 1054. (a) 1055. (a) 1056. (a) 1057. (d) 1058. (c) 1059. (a) 1060. (b)
1061. (c) 1062. (b) 1063. (c) 1064. (b) 1065. (a) 1066. (a) 1067. (b) 1068. (d)
OPTICS 547

1069. (b) 1070. (c) 1071. (a) 1072. (c) 1073. (b) 1074. (d) 1075. (b) 1076. (b)
1077. (b) 1078. (a) 1079. (b) 1080. (b) 1081. (c) 1082. (a) 1083. (a) 1084. (c)
1085. (c) 1086. (c) 1087. (a) 1088. (b) 1089. (c) 1090. (d) 1091. (c) 1092. (c)
1093. (c) 1094. (d) 1095. (a) 1096. (c) 1097. (a) 1098. (b) 1099. (a) 1100. (c)
1101. (a) 1102. (d) 1103. (a).

Solutions (Set II)


I I 40 cos 60°
581. × t1 = × t2 589. E= lux
r12 r22 2×2
I I = 5 lux
×3= × t2
5×5 10 × 10
100 lm
10 × 10 × 3 590. Illumination = = 200 lux
or t2 = s = 12 s 0.5 m × 1 m
5×5
1 16 250 cos θ 250 cos θ′
582. = 591. E= +
x 2
(100 − x)2 ( 2 × 2.5)2 FH 3.52 + 2.52 IK 2

1 4
or =
x 100 − x
250 × 2.5 250 × 3.5
or 5x = 100 or x = 20 cm E= +
3
( 2 × 2.5) (3.52 + 2.52 )3 / 2
I cos 60°
583. 5 × 10−4 =
200 × 200
or I = 5 × 10–4 × 4 × 104 × 2 q
= 40 cp. q
2

Luminous flux
×

3.
584. Power of lamp =
2.

2
5
5

Luminous efficiency 2.5

×
m

3.5

2.
2
5
4 πI 4 π × 30 120π

m
= = = = 24π watt.
5 5 5
3I 2I 2.5
585. =
1.2 × 1.2 (1.2 − x)2 2.5
586. Luminous flux in lumen
Fig. 134 Fig. 135
3 × 0.685
= lm
1.50 × 10−3 625 875
=+ = 14.14 + 10.99 = 25.13 lux
= 1.370 × 103 lm. 44.2 79.6
25 592. The fluorescent tube is to be regarded as a line source.
587. Light falling in 8 second = × 8. The wavefront is cylindrical. The flux 4πI passes
162
through the curved surface of area 2πrl.
In the second case, total amount of light falling in t
4πI 2I
40 Illuminance = =
second = ×t 2πrl rl
242
Equating, we get the result. 2 × 100
= lux = 40 lux
588. In the given problem, it is understood that the light 5×1
I 593. The image clearly coincides with the source. If E and
beam is perpendicular to the surface. So, E = 2 .
r E′ represent illuminance with and without mirror
Again, E is given to be constant. So, I ∝ r2. respectively, then
I1 4×4 1
= =
I2 12 × 12 9
548 COMPREHENSIVE OBJECTIVE PHYSICS

1500 1500
598. =
4π 4 × 3.142
= 119.35 ≈ 120 cd
599. Power of lamp
luminous flux
r r =
luminous efficiency
or luminous flux
= 12 × 10 = 120 lumen
Luminous flux over a solid angle of π steradian
Fig. 136 120
lumen = 30 lumen
=
2 4
E 2I/r 2
= = I I I
E′ I/r 2 1 600. E1 = 2 , E2 = 2 + 2
r r 9r
594. Note that the beam of light is parallel.
E1 I 9r2 E2 10
I cos θ = 2 × or =
595. Eedge = 2 2 E2 r 10 I E1 9
r +h
I h θ I 50
= × 601. × 20 = 2 × 10
4
√r

r 2 + h2 1 h
2

r 2 + h2 h
+h

or I = 100 cd d
2

Ih
= I I I
( r 2 + h2 ) 3 / 2 602. + cos 45° = h
√2 h
r
h2 2 h2 (h − d)2
100 × 2 45°
Now, Eedge =
9 LM
3/ 2
OP Fig. 137
1+
1
=
h2
4
+4
N Q 2 2 ( h − d )2
Fig. 138
=
200 × 8
=
1600 FG h−d IJ 2 2 2 2.828
(25)3 / 2 125
or
H h K = =
2 2 + 1 3.828
100 25 h−d
Again, = 2 or h′ = 1 m or =
22 h′ 0.74 = 0.86
h
25 × 1 25 × 8 200
Now, E′ edge = = = d
9 FG
3 /2
(13)3/2
(IJ
13)3 / 2 or
h
= 1 – 0.86 = 0.14 or d = 0.14 h
4
+1
H K I1 64
200 1600 603. =
Again, = x× I2 100
(13)3 / 2 125
I1 64
125 125 1– = 1−
or x= = I2 100
8 (13)3 / 2 374.97
125 1 I2 − I1 36 I2 − I1
≈ = or = or × 100 = 36
375 3 I2 100 I2
Ecentre I (r 2 + h2 )3 / 2 I I cos 60°
596. = 2 × 604. × 12 = t or t = 24 s
2
Eedge h Ih r r2
FG 1 + 1IJ 3/2
I
(r + h ) 2 2 3/ 2 H4 K F 5I
= G J
3/2 605. 40 =
1× 1
or I = 40 cd
=
h3
=
13 H 4K 40 × 1 40 20
36 80 Again, E= 2 2 3/2
= =
597. t1 = × 10 (1 + 1 ) 8 2
3×3 2×2
= 10 2 = 10 × 1.414 = 14.14 lux.
200 × 9
or t1 = s = 50 s
36
OPTICS 549
606. Let us consider illuminance under B1 B2 B3
2E
B2. 2.5 m 2.5 m
615. Force = rate of change of momentum =
c
200 200 × 3
E = 2 +2× 3m
2 × 10
3 [2.52 + 32 ]3 / 2 = N = 0.67 × 10–7 N = 6.7 × 10–8 N
200 1200 3 × 108
= + Fig. 139
9 59.55 1 1 1
= 22.22 + 20.15 616. + =
u v f
= 42.37 ≈ 42 lux.
607. Refer to illustrations. 1 1 1 f − x2 + f − x1 1
+ = or =
2 2 3/ 2 f − x1 f − x2 f ( f − x1)( f − x2 ) f
Ecentre I (r + h )
608. = 2 ×
Eedge h Ih or f 2 – fx2 – fx1 + x1x2 = 2f2 – f(x1 + x2)

(r 2 + h2 )3 / 2 F1 + r I
2 3 /2 or f 2 = x1x2 or f = x1 x2
=
h3
= GH h JK
2 This is Newton’s mirror formula.

LM 64 OP 3/ 2
+ Never give sign to the unknown quantity.
= 1+
N 64 Q =2 2

0.5 1 1 1
609. ∆ω = 617. + =
5×5 − 20 v 20
Luminous flux = I ∆ω
0.5 1 2 1
or = = or v = 10 cm.
= 500 × lumen = 10 lumen v 20 10
5×5
610. The total solid angle is 4π. For a cone of semi-angle θ,
618.
the solid angle is πθ2. If these are clear, the rest follows.
Laser intensity is 4/θ2 times larger on this account : 60°
directionality of laser versus isotropicity of ordinary

30 °
°
60°

30
light.

611. 40 40
2
×3= t
25 502 30°

°
40 × 3 × 50 × 50 60° 60° 30
or t= s = 12 s
25 × 25 × 40
Fig. 140
612. Intensity ∝ 1/d2
∴ Iedge (r2 + h2) = Icentre (h2). δ = 2 (180° – 2 × 30°)
613. If r is the initial distance, then the new distance is = 2 (180° – 60°) = 240°
3r f
. 619. m=
4 f −u
I′ 16 FG I′ − 1IJ × 100 = FG 16 − 1IJ × 100
I
=
9 HI K H9 K 2=
− 0.2
− 0.2 − u
700
= = 77.77 0.2
9 or 2= or 0.4 + 2u = 0.2
0.2 + u
I′ 16
614. = or 2u = 0.2 – 0.4 = – 0.2
I 9
or u = – 0.1 m.
I′ 16 620. Reflections are not affected by medium.
− 1= −1
I 9 621. In the case of plane mirror, the size of image is the
I′ − I 7 I′ − I 700 same as the size of object.
or = or × 100 = = 77.77.
I 9 I 9
550 COMPREHENSIVE OBJECTIVE PHYSICS

622. Now, u = – 20 cm, v = 4 cm 1 −12


− =
4 −12 − u
1 12
8 cm − =
O I 4 12 + u
or 12 + u = – 48 or u = – 60 cm.
4 cm
12 cm
f
627. m=
f −u
− 20
20 cm −2=
− 20 − u
12 cm
20
−2=
Fig. 141 20 + u
1 1 1 or 20 + u = – 10 or u = – 30 cm.
= + = − 1+ 5 = 4 = 1
f − 20 4 20 20 5 −20 20
628. 2= or 2 =
or f = 5 cm −20 − u 20 + u
or 20 + u = 10 or u = (10 – 20) cm
1 1 1
623. + = or u = – 10 cm.
f v f
− 1 20
2 629. = or 20 – u = 40
2 20 − u
1 1 2 3 f
or = + = or v= or u = (20 – 40) cm = – 20 cm.
v f f f 3
630. (180° – 2i1) + (180° – 2i2)
30° 1 22 = 300°
624. θ = 30′ = = × rad
60 2 7 × 180 or 60° = 2 (i1 + i2)
i2 i2
22 or i1 + i2 = 30° ...(i) 90° –i2
= rad
14 × 180 Again, θ + (90° – i2) i1
i1
I O + (90° – i1)
= q 90° – i1
f u = 180°
22 Fig. 142
I = θf = × 200 cm or θ = i1 + i2
14 × 180 from = n (i), θ = 30°
= 1.746 cm = 1.75 cm.
360°
625. Let p be the momentum of a photon arriving at the Now, n= − 1 = 12 – 1 = 11
mirror, we then have 30°
h −9 −9
p= . 631. m= = or m = – 3
λ −9 − (−12) 3
The change in momentum for a photon = p – (– p) =
I
2h or = − 3 or I = – 3 × 3 cm or I = – 9 cm.
2p = . O
λ
Force = rate of change of momentum of the photons The negative sign indicates that the image is real.
FG IJ
2h 632. m=
15
=
15 3
=
= n
H K
λ
where n is the rate at which the photons
15 − (−10) 25 5
2nh I 3
hitting the mirror = . =
λ 5 5
f I = 3 cm
626. m=
f −u The positive sign indicates that the image is virtual.
OPTICS 551

633. 637.

q
R q
2 q
p–
q 2q
x

Fig. 143
Fig. 145
x R x R 2θ + 45° = 180° or 2θ = 135° or θ = 67.5°
= or =
sin θ sin(π – 2θ) sin θ sin2θ
638.
x R R
or = or x=
sin θ 2 sin θ cos θ 2 cos θ

+ A common misconception
R
f = is valid under all situations! This I1
2
is incorrect. This relation is true only if the aper-
ture of the mirror is small. f f

You might have thought (a) to be the cor- Fig. 146


rect option. But (c) is the correct option.
I1 would act as an object for the plane mirror. Clearly,
It may further be added here that as aper- the image is formed at a distance f in front of the
ture becomes smaller and smaller, θ also becomes mirror.
R 639. When there is no water in the C
smaller and smaller. In the limiting case, x = . mirror, the rays of light are in-
2
cident normally on the mirror
and retrace their path. So, we
634. Minimum length of plane mirror
get an image coincident with
1 1 the object as shown in Fig. 147
× height of man = × 180 cm = 90 cm
=
2 2 When the mirror is filled with
635. Height of lower edge of the mirror water, then the equivalent fo- O
cal length F is given by Fig. 147
1
= [Height of eyes from the floor] 1 1 1 1
2 = + +
F fwater lens fconcave mirror fwater lens
1 1 1
=
× 170 cm = 85 cm 1
2 or =2× +
F fwater lens fconcave mirror
636. Note that the distance of image
1 FG IJ
1 1
from the man is 10 m.
2m
or
F
= 2(µ − 1)
H K
R
+
R
2m θ 2
Now, θ= = 0.2 radian.
10 m 10 m
1 2 (µ − 1) 2 1 2µ R
Fig. 144
or = + or = or F =
F R R F R 2µ
Clearly, focal length of the new optical system is less
than the original focal length. So, the object is effec-
tively at a distance greater than twice the focal length.
So, the real image will be formed between F and 2F.
552 COMPREHENSIVE OBJECTIVE PHYSICS

360° 10 mm
640. n= =5 tan 60° =
72° x
10 mm 10
360 3= or x= mm
Note that is odd and object lies asymmetrically. x 3
θ
20
2x = mm.
641. 3
Number of reflections
2000 × 3
= = 100 × 1.732 = 173.2.
20
O I
1 π
3m 3m 646. θ= × radian
2 180
Diameter of image
4.5 m =θ
focal length
7.5 m
or Diameter of image
Fig. 148
1 π 15
×
= × × 100 cm = 6.55 cm.
642. v cos 45° = 20 2 2 180 2
v 1 1 1
= 20 2 647. + =
or or v = 40 cm s–1 20 v 20
2
1
The velocity of the image formed by the roof is the or =0 ⇒ v = ∞ .
same as the velocity of insect. v
648. ∆s IAB and ICD are similar.
So, (c) is the correct choice.
C
643. A thick glass mirror produces a number of images.
There is an apparent shift of actual silvered surface
towards the unsilvered face. A
Effective distance of the reflecting surface from
F
I E
d 3 O
unsilvered face = = cm = 2 cm.
µ 3/2
B
Distance of point object from effective reflecting sur-
face = 9 cm + 2 cm = 11 cm.
D
Distance of image from point object = 11 cm + 11 cm
Fig. 150
= 22 cm.
Distance of image from unsilvered face = (22 – 9) cm CD IE 3L
= 13 cm. = = =3
AB IF L
CD = 3AB = 3d
+ The second image is the brightest because 649.
this is produced as a result of first reflection at
the silvered face. θ
90 – θ
1 1 1 1 1 θ 90
644. + = or = or f = 10 cm. –θ
20 20 f f 10
645.
90 – θ

–θ
90

30° 10 mm θ θ θ
60°
Fig. 151
x
Fig. 149 θ + θ + θ = 180° or 3θ = 180° or θ = 60°
OPTICS 553
650. Clearly, the given mirror is a convex mirror.
v1 −3 FG IJ FG − 1IJ 2 = –1 m
m=
f or I1 = −
u
O=−
2 H KH 3K
f −u
I2 v
Again, =− 2
1 18 O v
= or 3 × 18 = 18 – u
3 18 − u
FG − 5 IJ FG 1 IJ 2 = – 0.5 m.
or u = –2 × 18 cm or u = – 36 cm. or I2 = −
H 4 K H − 5K
652. 654. Due to glass slab, increase in path
= (µ – 1) t = (1.5 – 1) 3 cm = 1.5 cm
P I
∴ u′ = – (21 + 1.5) cm
or u′ = – 22.5 cm
1 1 1
O Now, + =
− 22.5 v − 5
15 cm 1 1 1 1 5 − 22.5
20 cm or = − or =
20 cm v 22.5 5 v 22.5 × 5
25 cm 22.5 × 5
or v=− cm or v = – 6.43 cm.
17.5
Fig. 152
Geometrical distance
Clearly, the distance of I from P is 15 cm. = – (6.43 – 1.5) cm = – 4.93 cm.
Now, u = – 25 cm, v = 15 cm,
1 1 1 1 1 1
f = ? 655. + = or = +
− 900 v 30 v 900 30
1 1 1 1 1 1
+ = or = + 1 1 + 30 900
u v f f − 25 15 or = or v= cm ≈ 30 cm.
v 900 31
1 −3+5 75
or
f
=
75
or f = cm = 37.5 cm + Do not be confused by some useless data
2
in the problem.
653. For A
u = – 3 m, v1 = ?, f = – 1 m
656.
1 1 1 1 1 1 2
= − = − = − 1= −
v1 f u −1 −3 3 3 –θ
90°
20°
3
or v1 = − m
2 90°
–θ

For B
90°

1 1 1 1 1 4
= − or = − 1= −
v2 − 1 − 5 v2 5 5
θ 90°
5
or v2 = − m
4 Fig. 153
3 FG 5IJ 90 – θ + 90 + 20 + 90 – θ = 180°
Now, v1 − v2 = −
2
− −
H 4 K or –2θ + 110° = 0 or θ = 55°
3 5 1
=− + =− m = – 0.25 m
2 4 4
I1 v
Again, =− 1
O u
554 COMPREHENSIVE OBJECTIVE PHYSICS

660. A′ A
657.

C M
I3 I2 8 cm 2 cm I1
G F
O E
8 cm 2 cm
10 cm
D
12 cm

B′ B
N
Fig. 156
M1 M2
As ECD and EA′ B′ are similar.
Fig. 154 CD EF EF 1
= or CD = × A ′ B′ = AB
I1 is the image, of O, formed by M2. It is at a distance A ′ B′ EG EG 3
of 2 cm from M2. 1
= × 12 m = 4 m
3
I2 is the image of O formed by M1. Its distance from
M1 is 8 cm. Distance from M2 is 18 cm. 1.6
661. f = m = 0.8 m, u = − 1 m
2
I3 is the image of I1 formed by M1. Distance of I1 from
1 1 1 10 18 9
M1 is 12 cm. Distance of I3 from M1 is 12 cm. Dis- = − = + 1= =
tance of I3 from M2 = (12 + 10) cm = 22 cm. v 0.8 − 1 8 8 4
4
1 1 1 or v= m
658. + = 9
u v f 4
v 4
du dv dv du 662. m=– =− 9 =
− 2
− 2
=0 or − 2
= 2 u −1 9
u v v u
I f
2 663. =
v O u
or dv = − du
u2 f 6/2
I= ×O = × 864100 m
10 × 10 u 92900000
=− × 9 m s –1 = – 1 m s–1 = 0.0279 m = 27.9 mm ≈ 28 mm.
30 × 30
f
664. m=
659. M2 f −u

1 f
60° = or f − u = nf
30° n f −u
60° 30°
or – u = nf – f = (n – 1) f

665.
i
i
i i – i
90°
60° 90° – i
M1
90°
θ
90° –

Fig. 155 θ
θ
60° + 60° + 90° – i = 180°
or i = 30°
Fig. 157
OPTICS 555
90° + 90° – i + 90° – θ = 180° sin i tan i x 15 24
673. µ= ≈ = =
or i + θ = 90° or θ = 90° – i sin r tan r x 24 15
1 1 1 1 1
666. = + = + C
f u v − 0.2 − 0.3 r
− 0.3 − 0.2 − 0.50
= =
0.06 0.06
0.06 6
or f =− m =− × 1000 mm = – 120 mm.
0.50 50
FG f IJ 2 i
667. Length of image =
H f − uK b
r
1 1 1
+ + =
u v f x
du dv P
− 2 − 2 =0
u v Fig. 158
dv du 2
dv v 674. The angle subtended at the centre of curvature is
or − 2 = 2 or =− 2 clearly one radian. So, the field of view is 2 radian.
v u du u
675. For upright portion
668. f = – 10 cm, O = 5 cm,
− 10
u = – 100 cm, I = ? f −5
m= = 2 =
I f − 10
= f −u − (− 20) − 5 + 20
O f −u 2
− 10 − 10 −5 1
I= ×5 = × 5 cm = =−
− 10 − ( − 100) 90 15 3
= – 0.55 cm FG 1IJ 2
1
669. Image will still be formed on the white screen but
For horizontal portion, magnification is −
H 3K i. e.
9
.
with less intensity.
−13
1 1 1 1 1 1 Required ratio is = − 3: 1
670. + = or =− + 19
u v f v u f
f
Now, compare with y = mx + c. 676. m=
f −u
1 1 1
671. + = −f
u v f 2= or −2 f − 2u = − f
− f −u
1 1 1 1 2 f
+ = or = or v= −f
−f v f v f 2 or 2f + 2u = f or 2u = – f or u =
2
672. For concave mirror
1 30
15 677. = or 120 = 30 – u
u=− cm, v = ? 4 30 − u
2
or u = (30 – 120) cm = – 90 cm
10
f =− cm = – 5 cm 360°
2 678. =4
90°
1 1 1 1 1
= − = − n=4–1=3
v f u − 5 − 15/2
679. Let u=–x
1 2 −1
=− + = 1 1 1
5 15 15 + =–
− x − x − 10 12
or v = – 15 cm
1 1 1
Clearly, the position of the final image is on the pole or + =
of the convex mirror. x x + 10 12
556 COMPREHENSIVE OBJECTIVE PHYSICS

x + 10 + x 1 1.5 R
or = or v= = 6R
x( x + 10) 12 0.25
2 x + 10 1
or 2
=
x + 10 x 12
or x2 + 10x = 24x + 120
or x2 – 14x – 120 = 0
x2 – 20x + 6x – 120 = 0
or x(x – 20) + 6(x – 20) = 0 4R
or x + 6 = 0 or x – 20 = 0
6R
or x = – 6 or x = 20
∴ u = – x = 6 or – 20 cm Fig. 159
− 30 µ 2 µ1 µ − µ2
Now, m = = − 15
. Again, + = 1
− 20 −u v R
680. Think in terms of rectangular hyperbola. 1.5 125
. 1.25 − 15
.
I f + =
= − 4R v −R
681.
O f −u 1.25 1 1.5 2.5
f or = + =
I= × 4 cm = 2 cm v 4R 4R 4R
f − ( − f) 1.25 × 4R 5R
or v = =
682. nth order image formed by one mirror is at a distance 2.5 2.5
a or v = 2R
of (2n – 1) from this mirror. Distance from centre = 3R.
2
nth order image formed by the second mirror is at a 688. i > ic
a 45° > ic
distance of (2n – 1) from this mirror. or sin 45° > sin ic
2
a a 1
Required distance is (2n − 1) + a + (2n − 1) = 2na. or > sin ic
2 2 2
360 360 1 1
683. n= −1 or 3= −1 or > or 2 < n
θ θ 2 n
360 or n> 2
or =4 or θ = 90°
θ 689. The effective focal length is given by
684. λR > λG > λV
1 2 1
µR < µG < µV = +
F fl fm
(ic)R > (ic)G > (ic)V
It follows from VIBGYOR that the critical angles for 1 FG
1 1 1 IJ
yellow, orange and red are greater than the critical
angle for green. This decides in favour of (a).
But
fl
= (1.5 − 1)
H +
∞ 30
=
60 K
685. The rays from the object fall normally on the surface or 2 1
= Fig. 160
of the sphere and emerge undeviated. fl 30
686. For the case of concave lens, n2 > n. Again, R = 30 cm
For the case of convex lens of smaller power, n1 < n or
n > n1. R
fm = = 15 cm
2
687. µ1 µ2 µ2 − µ1
+ =
−u v R 1 1 1
Now, = +
1.25 1.5 1.5 − 1.25 1.5 0.25 F 30 15
+ = or =
− (− ∞) v R v R 1 1+ 2 3 1
or = = = or F = 10 cm
F 30 30 10
OPTICS 557
To have a real image of the size of the object, the 1 1
object must be placed at the centre of curvature of the 697. sin ic = = or ic = 48.6°
µ 1.33
equivalent mirror. Angle with horizon = 90° – 48.6° = 41.4°
So, the required distance is 2 × 10 cm i.e. 20 cm.
698. µ g sin θ c = µ 1 sin 90°
690. or µ g sin θ c = 1
When water is poured,
O 20 cm
40 cm µ w sin r = µ g sin θ c
5 cm
h or µ w sin r = 1
Again, µ a sin θ = µ w sin r

Fig. 161 or µ a sin θ = 1


or sin θ = 1 or θ = 90°
h 5 1
= = 699. Clearly, µ = 1.5
20 40 8
20 5 FG 1 IJ FG 1 IJ 2 FG IJ
or h=
8
cm =
2
cm = 2.5 cm. Apparent shift = t 1 −
H µ K H
= 4.5 1 −
1.5 K
= 4.5 1 −
3 H K
sin r ′ = 1.5 cm
691. µ=
sin i sin 60°
700. = 3
But r′ + r = 90° sin r1
or r′ = 90° – r r′ sin 60°
or sin r1 =
r′ = 90° – i 3
cos i i r 3 1 i1 –
∴ µ= or sin r1 = × r1
sin i 2 3
1 1 1
or = = d
sin ic tan i 2
Fig. 162 or r1 = 30°
or ic = sin −1(tan i)
d
n2 v1 νλ 1 Now, sin (i1 – r1) = Fig. 163
692. 1 n2 =
= = 5
n1 v2 νλ 2 or d = 5 sin (i1 – r1)
t 5
λ1 x y or d = 5 sin (60° – 30°) = 5 sin 30° = cm
= = = 2
t
λ2
y
x
FG 1 IJ
693. Power doubled, focal length halved. Now, think of 2 f.
701. Apparent shift = t 1 −
H µK
694. Symmetry considerations lead us to the right choice. 1 F 1 IJ
= t G1 −
1 FG2 IJ
1 FG IJ FG 1 − 1 IJ
n 8 H 3 2K or
8
= t 1−
H3 K
695.
f
=
1H K HR R K
−1
1 2
or
1 t
= or t =
3″
1 Fn I F 1 − 1 IJ
= G − 1J G
8 3 8
f
l H n′ K HR R K 1 2 702. sin ic =
1
2
or ic = 45°
fl (n − 1)n′ fn′ (n − 1)
Dividing, = or fl = – Now, 75° = r + 45° 75°
f n − n′ n′ − n or r = 30°
i
aµo 1.45 ic
= a µ o × wµ a = sin i r
696. wµo = = 1.09 Now, = 2
a µw 1.33 sin 30°
Again, oµw = a µw × o µ a 1 1
or sin i = 2 × =
2 2
a µw 1.33
= = = 0.92 ∴ i = 45° Fig. 164
aµo 1.45
558 COMPREHENSIVE OBJECTIVE PHYSICS

703. Apparent depth −R


Now, − 60 =
6 4 2 (µ − 1)
= + = 4 + 3 = 7 cm or 120 (µ – 1) = R ...(1)
1.5 4/3
1 1 1 1
f Again, = + +
704. m= F fl fl fm
f +u
1 2 1
f or = +
m= F fl R 2
f + −( x + f )
f 1 FG IJ
1 2
m=
−x
or
F H K
= 2 (µ − 1) R + R

1 2
705. r or = (µ − 1 + 1)
F R
ic ic or R
F= Fig. 168

h
−R
ic Now, − 20 =

or 40 µ = R ...(2)
Dividing (1) by (2), we get
Fig. 165 120 (µ − 1) R
= =1
r 40µ R
tan ic = or 120 (µ – 1) = 40 µ
h
1 or 120 µ – 40 µ = 120
Also sin ic = µ or 80 µ = 120
µ
1
1 120 3
tan ic = or µ= = = 1.5
2
ic 80 2
µ −1
µ
2
–1
r 1 30 cm
∴ = 707.
h 2 Fig. 166
µ −1
h
or r=
2
µ −1 20 cm

2
3.14 × 2.75 × 2.75
Required area = πr2 = πh
O I
=
2
µ −1 16
−1
9
3.14 × 2.75 × 2.75 × 9 2 15 cm 10 cm
= m
7
= 30.5 m2 Fig. 169
1 1 1 1 1 1 1
706. = + + − =
F fl fl fm v −15 10
1 2 1 1 1 1 1 3−2
= + or = − or =
F fl fm v 10 15 v 30
or v = 30 cm
1 1 1 FG IJ
or
F
= 2 (µ − 1)
R
+
∞ H K Clearly, the rays coming from the convex lens should
fall normally on the convex mirror. In other words,
R the rays should be directed towards the centre of cur-
or Fig. 167
F= vature of the convex mirror.
2 (µ − 1)
∴ 2f = 20 cm or f = 10 cm.
OPTICS 559

708. 715. Power of combination = + 2 D – 1 D = + 1 D


60° 30° 100
° = 1 or f in cm = 100
f in cm
60

So, the system behaves as a convergent lens of focal


length 100 cm.
716. Deviations at A and B are equal in magnitude and
have the same order.
Fig. 170
Refracted
Clearly, ic ≤ 60° ray
A d1 a b
So, maximum possible value of ic is 60°. a
B
b b d2
y
1 e n a
t r Em
erg
incid
Now lµg = sin i ray ent
c
µg 1
=
µl sin ic Fig. 173

3 ∴ Total deviation = (α − β) + (α − β) = 2(α − β)


or µl = µ g sin ic = 1.5 sin 60° = 1.5 ×
2 717. The given system is equivalent to a combination of
= 1.5 × 0.866 = 1.299 = 1.3 (i) convex lens made of material of refractive index
709. 1.5 and (ii) concave lens made of material of refrac-
tive index 1.6.
2 1 100 1
Power of convex lens = (1.5 − 1) = ; ∴ =
R R 20 R
1
or R= m, Power for concave lens
5
5 cm 5 cm = – (1.6 – 1) × 2 × 5 = – 6 D
Power of combination = 5 D – 6 D = – 1 D
Fig. 171 100
Focal length of combination = − 1 cm = – 100 cm
The focal length of the lens is clearly 10 cm.
710. Apparent shift 718. Velocity of light in medium
R. D. FG 1 IJ µ−1
=
3 cm
= 1.5 × 1010 cm s–1
= R.D. – A.D. = R.D. –
µ H
= 1−
µ K
R.D. =
µ
d. 0.2 × 10 −9 s
1 8
20 4
c 3 × 10 10 cm s−1
711. sin ic = = 2 × 10 = =
µ=
v
= = 2,
µ 2.5 × 108 25 5 1.5 × 10 10 cm s−1

FG 4 IJ Also
1
= sin θC or θC = 30°
ic = sin −1
H 5K 2
0.5 1 1 FG IJ
712. The angular range is clearly twice the critical angle.
r
719. In P, power =
R
× 2 + 0.6 − −
R R H K
713. tan ic = r
1 1.2
h = − , which is negative. So, P is a divergent com-
or r = h tan ic R R
h bination.
or rαh ic
ic 0.5 1
h is doubled, r is doubled. In Q, power = × 2 − 0.6 × , which is positive. So,
R R
Q is a convergent combination.
Fig. 172 1 1 1 d
720. = + −
714. The central ray goes undeviated. F f1 f2 f1 f2
So, µ2 = µ1. P = P1 + P2 – dP1P2
Also, µ3 < µ2. 1
=4+6– ×4×6=6D
6
560 COMPREHENSIVE OBJECTIVE PHYSICS

1 1 FG IJ 729. Wavelength is reduced by a factor of µ.


721. f = – 1 m, m =
y
, u = ?; u =
m
−1 f
H K 730. f =
D2 − d2
Substituting values, u = – (y – 1) metre 4D
722. O= I1 I2 4D
P=
D2 − d 2
723. Optical separation = d – (f1 + f2) = 25 – (10 + 15) = 0.
4×1 4 4
P= D = D = D
724. 1 − 0.4 × 0.4 1 − 0.16 0.84
= 4.76 D ≈ 5 D
x 731. r = h tan ic
h
or r=
y µ2 − 1 µ
y m
4 1
=
25
−1 ic
9
4×3 µ2 – 1
Fig. 174 = =3m
16 Fig. 176
725. D=2×3m=6m
FG 1 IJ = b1.6 − 1g FG 100 IJ
x
mx 732. P = (µ – 1) H RK H 15 K
6 100 60
= × = D=4 D
10 15 15
y 1 FG
1 1 IJ
F
733.
f
= (1.6 – 1)
H−
4 3 K R1
1 3−4
= 0.6
Fig. 175 f 12 C2 C1
726. Thickness 1 0.6 × 1
=–
= 6 × 1.5 + 4 × 1.5 = (9 + 6 ) cm = 15 cm f 12 R2

F I
727.
1
2
GH
= (1.5 − 1)
1

1
R 1 R2
JK =–
1
20 Fig. 177
1 F 1.5
=G
I F 1 − 1 IJ
− 1J G
or f = – 20 cm
f H 1.25 K HR R K F I
1 f (1.5 − 1) (1.25)
1 2
734.
1
f
= (1.63 − 1)
2
RA
GH JK
× =
2 1 1.5 − 1.25 1 2
= (µB − 1)
f 0.5 × 1.25 f RB
or =
2 0.25 2 2
Equating, (µB – 1) = 0.63 ×
1.25 RB RA
or f = cm = 5 cm .
0.25 2 (µ B − 1) 0.63 × 2
or =
d RB 0.9 R B
728. Apparent depth of A=
1.3 0.63 0.63
d or µB – 1 = or µB = 1 +
Apparent depth of B= 0.9 0.9
1.6 or µB = 1.7
d 1.6 1.6 sin i sin i
Ratio of apparent depths is × or
1.3 d 1.3 735. µ= ; µ=
sin r i
sin
2
OPTICS 561

i i sin 2A 2 sin A cos A


2 sin cos =µ or =µ
2 2 sin A sin A
µ= i
sin or µ = 2 cos A
2
i µ i
= cos−1
µ FG IJ 741. D ≥ 4f
or cos
2
=
2
or
2 2 H K or 4f ≤ D or f ≤
D
4
FG µ IJ
H 2K
−1
or i = 2 cos D
Clearly, the greatest value of f is .
4
1 v
736. sin ic = =
µ c FG 1 IJ = 1
or sin ic =
νλ ′
or sin ic =
4000
742. (µ − 1)
H RK R
νλ 6000 or µ – 1 = 1 or µ = 2
2 −1 FG 2 IJ 743. The refractive index, 1n2 (from medium 1 to medium
or sin ic =
3
or ic = sin
H 3K 2) for two given media 1 and 2 is given by
1 v speed of light in medium 1(c1)
737. sin 30° =
= n =
1 2
µ c speed of light in medium 2(c2 )
1
or v = × 3 × 108 m s−1 or v = 1.5 × 108 m s–1 wavelength of light in medium 1 (λ 1)
2 =
wavelength of light in medium 2 (λ 2 )
738. When convex lens is surrounded by denser medium,
it behaves like a diverging lens. n2 5
Now, =
1n2 =
n1 4
739.
60° c1 = 2.0 × 108 m s–1
λ1 = 500 nm
30°
5 2.0 × 108 500
60° ∴ = =
60° 4 c2 λ2
30°

Hence, λ2 = 400 nm
60° c2 = 1.6 × 108 m s–1
744. Clearly, r = 60° – 15° or r = 45°
sin 60° 3 2
Fig. 178 Now, µ= or µ= ×
sin 45° 2 1
At the second surface, the angle of incidence is greater
than critical angle. So, light is totally reflected at the 3
or µ= = 1.2247
second surface. As is clear from the ray diagram, the 2
light emerges out perpendicularly from the third sur-
1
face. 745. µ= =2
sin 30°
740. Using Snell’s law,
A
2 sin i = 1 × sin 90°
A
°– 1
90
sin i =or i = 30°
2A A 2
746. If the refractive index of the material of the lens is
greater than the refractive index of the surrounding
medium, then a concave lens would behave as a con-
cave lens.

Fig. 179
562 COMPREHENSIVE OBJECTIVE PHYSICS

747.
10 cm (20) (30)
⇒ v=−
5 cm 5 cm 20 + 30
= – 12 cm (to the left of the diverging lens.)
750.

Screen

to ¥
Triangle
Fig. 180 aperture
20 cm 10 cm
At 5 cm from the first lens, the second lens has a f
virtual object (image of the first lens) at its focal length.
Fig. 183
The emergent rays are therefore parallel.
The image on the screen is real and inverted.
< 5 cm > 5 cm The size of the image on the screen has aperture size
given by
FG 10 IJ = 0.5 cm
.
size = 10
H 20 K
Divergent light beam

Hence, the path of light on the screen is best repre-


sented in diagram d.
751. A diverging lens is ruled out because both x and y are
positive values. Both x and y equal 20 cm at their
smallest sum, which occurs when
x + y = 40 cm = 4f
∴ f = 10 cm
Fig. 181 This indicates a converging lens of focal length
= 10 cm
When the second lens is closer than 5 cm to the first
lens, its object is outside the focal length of the di- 100 100
752. P= − = (10 – 4) D = 6 D
verging second lens. This produces a virtual image 10 25
outside 2f of the second lens. The emergent rays are f
therefore divergent. 753. m=
f +u
749. f 1
m= =
f + mf 1+ m
754. Diminished erect image is produced only by a con-
cave lens.
755. Power of liquid lens
FG 2 IJ = 6 × 10 = 6 D
= (16
. − 1)
H 0.20 K 10
Power of concave lens
Fig. 182
2
When an object is placed between 2f and f (focal length) = − (1.5 − 1) = – 0.5 × 10 D
0.20
of the diverging lens, the image is virtual, erect and =–5D
diminished as shown in the graph. To compute the
Total power of two concave lenses = – 10 D
distance of the image from the lens, we apply
Power of system
1 1 1 1 1 1
= − ⇒ = − = – 10 D + 6 D = – 4 D
f v u − 20 v 30
OPTICS 563

1 1 1 1 5−6
Focal length = m = – 0.25 m Now, = − =
−4 F 24 20 120
sin r 1 or F = – 120 cm
= tan 30° =
756.
sin i 3
762.
1 FG 2 IJ
sin i f
= 0.5
H RK
or = 3 or µ =
sin r
3
1 FG 15. − 1IJ FG 2 IJ 1 2 2
So, speed of light in Y is 3 times less. f′
=
H 13. K H R K or
f′
=
13 R
F3 I 1 f′ 0.5 × 13 13
1 G JF 2I
= 2 − 1J G J
Now, × = or f′ = f
f G4
757. f 1 2 4
GH 3 JK H 30 K 763.
1 1 1
+ =
F3 3 I F 1 I
= G × − 1J G J
1 1
u v f
H 2 4 K H 15 K =
8
×
15 or
v
+ 1=
v
or
1
+ 1=
x
or f = 120 cm = 1.20 m u f y f
So, the graph cannot be a straight line. So, (a) is not
758. O= 16 × 9 cm = 12 cm
true.
1 FG 3 − 1IJ FG 2 IJ Graph II indicates that increase in u increases v which
12 H 2 K H RK
759. =
is not true. So, (b) is not true.
1 1 1
F3 I Again, + =
1 G2
= G − 1J G J
JF 2I u v f
f
GH 54 JK H R K v v
+ =
u v
v
f
x
Dividing, f = 1 R y+1=
12 0.2 × 2 f
R Graph should be linear. So, (c) is ruled out.
f 1 120 1 1 1
or = or f = cm = 30 cm Again, + =
12 0.4 4 u v f
3500 1 1 1
760. sin ic = = If = y, =x and
7000 2 u v
ic = 30° then we get a straight line. So, (d) is true.
761. Since the refractive index of the liquid is greater than 764.
the refractive index of glass therefore the focal length 1 F 1 − 1I
of the system has to be negative. So, options b and d
are excluded. f
GH R R JK
= (µ − 1)
1 2

1 F3 I F 1 1 IJ
= G − 1J G −
1 2
f H2 K H ∞ −20 K
Now, = (1.5 − 1)
24 R
or R = 24 cm 1 1 1
Again, for liquid concave lens, = × or f = 40 cm
f 2 20 Fig. 184
1 2 FG IJ 1
f
= − (1.6 − 1)
24 H K 765. vm =
2
c

1 3 1 c c
=− × µ= = =2 1
vm 1 or =2
f 5 12 c sin ic
2
1 1 1
or =− or f = – 20 cm sin ic =
f 20 or or ic = 30°
2
564 COMPREHENSIVE OBJECTIVE PHYSICS

1 FG 2 IJ 1 1 f1 1 600 − f12
766.
10
= (1.5 − 1)
H RK or
60
=
f1

600
or
60
=
600 f1
1 1 2 or 600f1 = 36000 – 60 f12
or = × or R = 10 cm
10 2 R
or 60f12 + 600f1 – 36000 = 0
1 1 1 1
Again, = + + or f12 + 10 f1 – 600 = 0
F fl fl fm
1 2 2 or f12 + 30f1 – 20f1 – 600 = 0
= +
F fl R or f1(f1 + 30) – 20 (f1 + 30) = 0
1 2 2 4 2 or f1 = 20 cm or – 30 cm
or = + = =
F
10 10 10 5 600 − 600
Now, f2 = − cm or
5 20 − 30
or F= cm = 2.5 cm
2 or f2 = – 30 cm or 20 cm
The combination behaves like a concave mirror of focal 770. Angular radius of image
length 2.5 cm. λ
Clearly, u = 2 × 2.5 cm = , diameter = 10–5 radian,
aperture
or u = 5 cm
Diameter of image is 2f times.
767. m = 30
771. Clearly, power of system is zero.
v + u = xu
1 1 5
v ∴ 0= + −
or + 1= x or m + 1 = x 20 f 20 f
u
1 15
or x = 31 or − = or f = –15 cm
20 20 f
c
768. µ=
v 772. O= I1 I 2
λ O2 = I1 I2
Also, µ=
λ′
O2 = A1 A2 = A1 A2
c λ
Equating, = 773. A = r1 + r2
v λ′
v λ′ λ′ 30° = r1 + 0
or = or v = c or r1 = 30°
c λ λ
4500 sin i 30°
= × 3 × 108 m s–1 Now, = 2
6000 sin 30°
r1
= 2.25 × 108 m s–1 1
or sin i = 2×
1 1 1 2
769. = + ...(1) 1
60 f1 f2
or sin i =
1 1 1 10 2
= + − Fig. 185
or i = 45°
30 f1 f2 f1 f2
1 1 10 10 1 1
FG IJ 1
or
30
=
60

f1 f2
or
f1 f2
=
60

30
774. 1= 3 1−
H µK
1 1 1 1 2 3
10 1− 2 1 or 1− = or = 1− = or µ =
or = =− or f1 f2 = – 600 µ 3 µ 3 3 2
f1 f2 60 60
1 3
600 Now, =
or f2 = – sin ic 2
f1
2
ic = sin −1
FG 2 IJ
From (1),
1
=
60 f1
1
+

1
600
or sin ic =
3
or
H 3K
f1 or ic = sin–1 (0.67)
OPTICS 565

1 r 0.8 × 10−14
775. sin ic = Again, µ2 = 1.20 +
µ
e500 × 10 j
2
−9
1
sin ic = h ic
0.8
43 or µ2 = 1.20 +
ic 25 θ
3 or µ2 = 1.20 + 0.032
or sin ic =
4 or µ2 = 1.232
ic = sin −1
FG 3 IJ Fig. 186
1
θ
or
H 4K or
sin ic ′
= 1.232
776. Convex mirror and concave lens do not form real im-
age. For concave mirror, if image distance is greater 1
or sin ic ′ = = 0.81
than object distance, the image will be enlarged. So, 1.232
only convex lens can be used for the purpose. or ic = sin–1 (0.81)
20.06 µ 20.06 × 1.47 = 54.26°
777. − = 1. 07 × 10 −8 Now, sin θ = 0.8 Fig. 187
3 × 1083 × 108
3 × 1. 07 or θ = 53.13°
3.21
or µ − 147
. = or µ = 1.47 + This angle is clearly greater than critical angle corre-
20 . 06 20.06
sponding to wavelength 400 nm. So, light of 400 nm
or µ = 1.47 + 0.16 or µ = 1.63
wavelength undergoes total internal reflection.
778. Wavelength in vacuum, λ
781. (90° – ic) + (90° – ic) + 90°
ic
3 × 108 = 180°
= 14
× 1010 Å = 0.6 × 104 Å ic
5 × 10 or 2 ic = 90° – ic
9 0°
ic
= 6000 Å or ic = 45° 90° – i c
λ 1
Now, µ= µ=
λ′ sin 45° Fig. 188
λ 6000 1
or λ′ = = Å = 4000 Å = = 2
µ 1.5 1

779. FG 2 IJ 2
5 = (15
H RK
. − 1)
782.
I 1
= ×
π

F 15. − 1IJ FG 2 IJ
–1= G
f 2 180

H n K H RK or I=
π
360
× 100 cm = 0.873 cm
0.5 n
Dividing, – 5 = 1 sin i
1.5 − n 783. = r′
or – 7.5 + 5n = 0.5n or – 7.5 = – 4.5n µ sin r ′ 90°
But r + r′ = 90°
75 5
or n= = or r′ = 90° – r i r
45 3
sin r′ = sin (90° – r)
0.8 × 10−14
780. µ1 = 1.20 + = cos r
e400 × 10 j −9 2
or sin r′ = cos i
Fig. 189
0.8 × 10−14 1 sin i
or µ1 = 1.20 + ∴ =
400 × 400 × 10−18 µ cos i
0.8 1
or µ1 = 1.20 + or µ1 = 1.20 + 0.05 or = tan i or sin ic = tan i = tan r
16 µ
or µ1 = 1.25 or ic = sin–1 (tan r)
1 c c
= 1.25 µ= = =2
sin ic 784.
v c2
1
or sin ic = = 0.8 or ic = 53.13°
1.25
566 COMPREHENSIVE OBJECTIVE PHYSICS

r
1 1 1
=2 or sin ic = or ic = 30°
sin ic 2 µ
∴ r=h h ic ic
785. Effective height of the bird as seen by the fish, Y 1
1− 2
= y + µ y′ µ
Bird
Fig. 191
h
dY dy dy′ y′ =
= +µ
dt dt dt µ2 − 1
4 dy′ 12 12 × 3
= = cm
9 = 3+ y 16 7
3 dt −1
9
dy′ 6 × 3
F I
Fish
18
or = =
dt 4 4 Fig. 190 788. −
1
40
= (1.5 − 1)
1

1
R1 R 2
GH JK
9
= 4.5 m s–1
= 1 1 1
2 − =−
R 1 R2 20
∴ Actual velocity of bird
1 FG
1.5 IJ FG − 1 IJ
786. When
= 4.5 m s–1
n = 1,
Now,
f
=
2 H−1
K H 20 K
0.5 F 1 I
µ (1) = µ 0 −
µ0 or
1
f
=− G− J
2 H 20 K
4 × 1 − 18
1 1
µ (1) > µ0 or = or f = 80 cm
f 80
When n = 2,
It behaves like a convex lens of focal length 80 cm.
µ0
µ (2) = µ0 − 789. Apparent height of object
4 × 2 − 18
4
µ (2) > µ0 = × 24 cm = 32 cm
3
When n = 3,
µ0
+ Note that when fish looks at the object in air,
µ (3) = µ0 − AD
4 × 3 − 18 µ= or AD = µ × RD
µ(3) > µ0 RD
When n = 4,
1 2
µ0 790. sin ic = =
µ (4) = µ0 − 1.5 3
4 × 4 − 18 2 FG IJ
µ (4) > µ0 or ic = sin −1
3 H K
= sin–1 (0.6667) = 41.8°
When n=5 The angle of incidence on face AC is 45° which is
µ0 clearly greater than 41.8°.
µ (5) = µ0 −
5 × 4 − 18 FG 200 IJ
µ (5) < µ0 791. P = (1.5 – 1)
H5K = 20 D
Clearly, the total internal reflection shall take place
P′ = (1.5 – 1) GH
F 200 IJ
6 K
at the top of a layer having n = 4. = 16.67 D
r
787. tan ic = or r = h tan ic Decrease in power
h
= 20 D – 16.67 D
sin ic sin ic
or r=h or r = h = 3.33 D
cos ic 1 − sin 2 ic f
792. m=
1 f +u
But sin ic =
µ 1 f
Now, =
2 f +u
OPTICS 567
Now, 2f = f + u or u = f 1 6+5 11
=− =−
f F 600 600
Again, 2=
f + (u − 30) 1 2 1 2
Now, = + =
f f F fl fm fl
or 2= or 2 =
f + ( f − 30) 2 f − 30 or fl = 2F
or 4f – 60 = f 600
or fl = 2 ×
or 3f = 60 or f = 20 cm 11
793. Note that two refractive indices are involved. [Negative sign is not to be used here]
1 1 1
794. − = 1200
v u f or fl = = 109.1 cm
11
1 1 1
− = 798. µ2 µ1 µ1 − µ 2
v −15 10 + =
−u v R
1 1 1 1 −2+3 In the first case,
or =− + or =
v 15 10 v 30
µ 1 µ − µ2
or v = 30 cm + = 1 =0
−d −v ∞
1 µ
or =
−v d
+ F1 d
v=−
or
µ
Fig. 194
In the second case,
Fig. 192 µ 1 1− µ
+ =
F1 I − d − v′ −R
795. 10 = (1.5 – 1) GH R −
1
R2
JK 1
= −
µ 1− µ
1 or
1 1 10 − v′ d R
or − = = 20 m 1 1 µ −1
R 1 R 2 0.5 or = +
FG 1.5 − 1IJ 20 D − v′ − v R
Again, P=
H 4/3 K Clearly,
1
>
− v′ − v
1

P = GH
F 4.5 − 4 IJ 20 D = 5 × 0.5 D
4 K
or 1 1 Fig. 195
or <
v′ v
= 2.5 D or v′ > v
Change in power
1 2FG IJ
= (2.5 – 10) D
= – 7.5 D
799.
0.2
= (1.5 − 1)
R H K
R = 0.2 m
796. f1 f2 = x1 x2
x1 x2 15 × 20 FG 15. − 1IJ FG 2 IJ
f2 =
f1
=
10
cm = 30 cm –1=
H µ K H RK
797. u = –100 cm
v = – 120 cm F 1.5 − 1IJ FG 2 IJ
–1= G
F=?
or
H µ K H 0.2 K
1 1 1 1 1.5 1.5 1 9
= + or − = − 1 or = 1− =
F u v 10 µ µ 10 10
1 1
= + 15
− 100 − 120 or µ= = 1.67
9
Fig. 193
568 COMPREHENSIVE OBJECTIVE PHYSICS

800. Since the surrounding medium is denser than the A


sin
material of the lens therefore the concave lens shall 2 = sin A
or A/2
behave like a convex lens. sin r
A/2
801. A r
sin
or 2 = 2 sin A cos A
sin r 2 2
90 – r
1
or sin r = A Fig. 198
r 2 cos
i 2
1 A
or sin r = sec
2 2
−1 1 FG
A IJ
Fig. 196
or r = sin
2
sec
H
2 K
Clearly, (90 – r) ≥ ic 1 1 d
804. P= + −
or sin (90° – r) ≥ sin ic or cos r ≥ sin ic f1 f2 f1 f2
1 When P = 0, then
or 1 − sin 2 r ≥
1µ 2
1 1 d
+ = or d = f1 + f2
f1 f2 f1 f2
1 µ2
or (1 – sin2 r) ≥ or (1 – sin2 r) ≥ 1
FG µ IJ2
2
µ 22 805. m=
f −v

Hµ K 1
f
v
sin i sin i or mf = f – v or m = 1 −
But = µ2 or sin r = f
sin r µ2
F 1 − sin i I ≥ µ
2 2 or
v
m= −+1
∴ GH µ JK µ2
2
1

2
2 or 2
µ2 − sin i ≥ 2
µ 12 f
For the case of real image, magnification is negative
or – sin2 i ≥ µ12 – µ22 or sin2 i ≤ µ22 – µ12 v
∴ – m = − v + 1 or m = − 1
or sin i ≤ µ2 − 2
µ 12 or i ≤ sin −1
µ2 −2
µ 12 f f
Comparing with y = mx + c, we find that reciprocal of
Clearly, the maximum value of i is sin −1 µ 22 − µ 12 . focal length f equals the slope of the graph.
v u v2 − u2 v+u FG IJ 1
= tan θ =
b c
802. m1 – m2 = −
u v
=
uv
= (v − u)
uv H K ∴
f c
or f =
b
1 1 1 v+u 806. A = r1 + r2
But = + = ∴ 30° = r1 + 0°
f u v uv 30°
or r1 = 30°
v−u
∴ m1 – m2 = or m1 – m2 = x sin i
f Now, = 2
f sin 30°
i 30°

x 1
or f= or sin i = 2×
m1 − m2 2
1
sin A 1 or sin i =
803. = 2 Fig. 199
sin 90° µ A
or i = 45°
1 807.
sin A = µ2 = 1.5
µ A 90°
sin A 2 1
= P x O x
Q
sin r µ Fig. 197
µ1 = 1

Fig. 200
OPTICS 569

µ1 µ2 µ2 − µ1 or r = 48.6°
+ = (iii) θ = r – i = 18.6°
−u v R
Required angle = 2 × 18.6° = 37.2°
1.5 1.5 − 1
1
+ = 1 3FG IJ FG 2 IJ
− ( − x)
1 3 1
x R 810.
f1
=
2 H
−1
K H RK
+ = 1 1
x 2 x 2R or = or R = 10 cm
5 1 f1 R
= or x = 5R
2 x 2R 1 3FG 2 IJ
a
f2
=
2 H
−1
R′ K
808. or R′ = f2 = 20 cm
FG IJ FG 1 − 1 IJ
25 b A 90° – i
1 4
Now, =
H
−1
K H −10 20 K
i c
i f3 3
90° – 2i
1 1 2+1 1
d i¢ or =− =−
i¢ f3 3 20 20
90° – q 1 1 1 1
Now, = + +
F f1 f2 f3
q q 1 1 1 1
or = + − or F = 10 cm
F 10 20 20
For real image
m=–2
Fig. 201
f
From ∆ abc, A + 90° + 90° – i = 180° Now, m=
f +u
or i=A 10
Now, complementary angles at point d. θ = 2 i –2=
10 + u
∴ θ = 2A
or 10 + u = – 5 or u = –15 cm
only (b) satisfies this.
For virtual image
m =+2
809. 30° 10
i r Now, +2= or 10 + u = 5
i θ 10 + u
or u = 5 – 10 or u = – 5 cm
θ
t FG 1IJ
H K
θ 811. t− =x or t 1− =x
µ µ
µx
or t=
µ−1
812. The following arguments shall lead us to the right
choice.
Fig. 202 (i) In the first case, the apparent depth is R2 – R1. So,
Following arguments lead us easily to the right choice. real depth is µ(R2 – R1).
(i) Angle between any two lines is the same as the (ii) Depth of additional liquid poured is R4 – R2.
angle between their perpendiculars. (iii) Total real depth is (R4 – R2) + µ (R2 – R1)
∴ i = 30° (iv) Total apparent depth = R4 – R3
1 sin 30° 1.5
= 0.75 Total real depth
(ii) = or sin r = (v) µ =
1.5 sin r 2
Total apparent depth
570 COMPREHENSIVE OBJECTIVE PHYSICS

1 2 1 µ 3.2
813. = + or = 1.5 −
F fl fm 3 3
1 µ−1LM 2 OP or
µ 4.5 − 3.2 1.3
= = or µ = 1.3
F
=2
R N
+
R Q 3
v
3 3
u
1 2µ 816. m1 = and m2 =
= u v
F R
1
R Clearly, m1 =
F= m2
2µ Fig. 203
y2
R×4 817. Distance as estimated by the bird, S1 = y1 +
0.3 = µ
2×7
Distance as estimated by the f is h, S2 = y2 + µy1
0.3 × 14 4.2
or R= m= m = 1.05 m S2 y2 + µy1
4 4 Now, =
S1 y
814. 90 – r = ic y1 + 2 B
µ µy1
sin (90 – r) = sin ic y1
or cos r = sin ic S 2 µy2 + µ 2 y1
or =
1 1 S1 µy1 + y2 y2
or 1 − sin 2 r = or 1 − sin 2 r = S2 µ( y2 + µy1) µ
1 µ2
µ2
or = y2
µ1 S1 y2 + µy1 F
F
µ 12 S2
or 1 – sin2 r = or µ= Fig. 204
µ 22 S1
sin i µ 818. Clearly, A = ic + ic = 2 ic
But = 1µ 2 = 2 A A
sin r µ1 or ic =
2
or sin r = µ 1 sin i
µ2 1 ic ic
µ=
sin ic
µ 12 sin 2 i µ 12
∴ 1− = 2 1
µ 22 µ2 or µ=
A Fig. 205
µ 2 sin2 i µ2 sin
or − 1 2 = − 1 + 12 2
µ2 µ2
A A
µ 12 sin2 i µ 12 sin2 + cos2
or = 1 − 2 2
µ 22 µ 22 or µ=
A
sin 2
µ 22 2
or sin2 i = −1
µ 12
A
1 + cot 2
Fµ I 2 or µ=
2
or sin i = GH µ JK
2
1
−1
819.

815. l = n λ′ =
µ
or µl=nλ
In the given problem, nλ = constant I1 O I
∴ µ l = constant
l 2l
∴ 1.5 l = µ + 1.6
3 3
µ 3.2 Fig. 206
or 1.5 = +
3 3
OPTICS 571

Refraction at plane surface


823.
µ1 µ2 µ2 − µ1
+ =
− u v1 R
1 1.5 1.5 − 1
+ =
− (− 2) v1 ∝
1.5 1 ic ic
or =− or v1 = – 3 m
v1 2
Due to refraction at plane surface, the image I1 would
be formed to the left of the plane surface at a distance
of 3 m. Distance of I1 from the curved surface is 4 m. Fig. 209
Refraction at curved surface Angular size is 2ic. The value of ic depends upon
µ2 µ µ − µ2 refractive index of water and not on the depth of
+ 1= 1
−u v R swimmer inside the tank.
1.5 1 1 − 1.5 1 2
+ = 824. sin ic = =
− (− 4) v 1 1.5 3
or sin ic = 0.6667 or ic = sin–1 (0.6667)
1.5 1 1 1.5
or + = 0.5 or = 0.5 − or ic = 41.8°
4 v v 4
The light would be incident on the second face at an
1 1 3 4−3
or = − = or v=8m angle of 60° which is greater than ic.
v 2 8 8
sin i
820. =µ A
sin A 60°

If i and A are small, then i


A
i
=µ 60°
A
or i=µA
°
821. Total apparent depth 60
Fig. 207
d d
= +
2µ 2 (15
. µ)
d Fig. 210
1 d d 2
Now, d= + Clearly, the angle between incident ray and emer-
2 2µ 3µ
gent ray is 120°.
1 2
1= +
d/2
or 1 1 1
µ 3µ 825. − = 26 cm 4 cm
3+2 v 4 − 20
or 1= Fig. 208
3µ 1 1 1
or = −
5 v 4 20
or 3µ = 5 or µ = = 1.67
3
4 1 30 cm
f = =
822. m= 20 5 Fig. 211
f +u
or v = 5 cm
f
Now, +2= I v 5 5
f − 10 Now, = = or I = × 2 cm = 2.5 cm
O u 4 4
or 2f – 20 = f or f = 20 cm
826. Using Snell’s law,
20 µg sin i = (1) sin 90°
Again, − 2 = or – 40 – 2u = 20
20 + u
1
or µg =
or – 2u = 20 + 40 or u = – 30 cm sin i
572 COMPREHENSIVE OBJECTIVE PHYSICS

827. Use A = r1 + r2 or e = 48° 36′


1 δ = i + e – A = 0 + 48° 36′ – 30° = 18° 36′
and µ=
sin ic FG A + δ IJ
H 2 K
m
sin
828. Minimum deviation, the refracted ray is parallel to 840. µ=
base. A
sin
4 2
829. µ =
a w3 FG
60° + δ m IJ
4 Real depth 2=
sin
2H K
= 60°
3 3 sin
∴ Real depth = 4 m 2
1 FG
60° + δ m IJ
830. ω=
fr − fb 21.4 − 20
f
=
20.5
=
1.4
=
14
20.5 205 2
= sin
2H K
831. Flint has larger refractive index. So, its prism must FG 60° + δ IJ
or sin 45° = sin
H 2 K
m
be of smaller refracting edge. Also, dispersion favours
flint. δm = 30°
833. Figs. (a) and (d) depict dispersion at one face only. So, Now, i + e = A + δm
these are wrong. In Fig. (c), emergent rays are paral-
A + δm
lel. So, this is wrong. or 2i = A + δm or i =
2
834. δ = (µ – 1) A ; δ′ = (µ′ – 1)A′
For total deviation to be zero, 60 + 30 90
= = = 45°
δ + δ′ = 0 2 2
841. A halved, δ halved.
∴ (µ – 1) A + (µ′ – 1) A′ = 0
843. See solution 836.
or (µ′ – 1) A′ = – (µ – 1) A
844. The unknown colour X can be any of the three :
A′ µ−1 violet, indigo, blue.
or =−
A µ′ − 1
µv − µr
1 1 845. Use ω =
835. i+e=A+δ; A+ A=A+δ µ−1
2 2
FG A + δ IJ
or δ = 0°
H 2 K
m
sin
4° 1.72 − 1 846. µ=
=− 4° 0.72 A
836. or =− = – 1.33 sin
A′ 1.54 − 1 A′ 0.54 2
4° A+A FG IJ
or A′ = –
1.33
= − 3°
3=
sin
2 H K
837. Deviation should take place at each face. Dispersion A
sin
takes place at first face only. 2
838. A + δ = i + e = 2e A A
2 sin cos
FG 3 AIJ − A or 3=
sin A
or 3= 2 2
or δ = 2e – A or δ = 2 H4 K sin
A
2
sin
A
2
3A
or δ= −A A A 3
2 or 2 cos = 3 or cos =
2 2 2
A 60°
or δ= = = 30° A
2 2 or cos = cos 30° or A = 60°
sin 30° 1 2 30° 2
839. = =
sin e 1.5 3 30° e 1
847. Dispersion ∝
3 3 λ3
or sin e = × sin 30° =
2 4 848. δm = A
or sin e = 0.75 Fig. 212 µ = 1.5
OPTICS 573

FG A + A IJ Angular dispersion 2°
1.5 =
sin
H 2 K 854. Dispersive power =
Deviation
=
48°
= 0.0416
A 855. ωfy = fr – fv
sin
2
fr − fv f − fv 100 − 96.8 3.2
A A ω= = r = =
2 sin cos fy fr + fv 100 + 96.8 98.4
3 2 2
= 2 2
2 A
sin = 0.0325
2
A 3 A 857. A = r1 + r2 = 30° + 30° = 60°
or cos= = 0.75 or
2 4 2
= cos −1 (0.75)
FG A + δ IJ FG 60° + 30° IJ
A
= 41°
sin
H 2 K
m
=
sin
H 2 K
or
2
or A = 82° Now, µ=
F AI
sin G J
sin
60°
849. ω1P1 + ω2P2 = 0 H 2K 2
Clearly, if power is more, dispersive power is less. sin 45° 1 2
Note that the achromatic combination shall be con- = = × = 2
sin 30° 2 1
vex if the convex lens has larger power.
858. A = r1 + r2 = 2r
850. Longitudinal chromatic aberration = ω f
A 60°
= 0.02 × 20 cm = 0.40 cm or r= = = 30°
2 2
ω1 ω2 sin i 1 1
851. + =0 Now, = 2 or sin i = 2=
f1 f2 sin 30° 2 2
ω1 ω or i = 45°
=− 2 860. i + e = A + δm
f1 f2
or δm = i + i – A = 2i – A
f2 ω 3 1 3
or =− 2 =− or =− 3A 3A A 60°
f1 ω1 4 f1 4 f2 =2× –A = −A= = = 30°
4 2 2 2
1 1 1 861. A+δ=i+e
Now, = +
60 f1 f2 or δ = i + e – A or δ = 55° + 46° – 60°
1 3 1 1 1 3 or δ = 41°
or =− + or = − Clearly, δm < 41°
60 4 f2 f2 60 f2 4 f2
862. µv = 1.632, µ = 1.620,
1 4−3 1
or = = or 4f2 = 60 µr = 1.613, fr – fv = ω f
60 4 f2 4 f2
or f2 = 15 cm
µv − µ r 1.632 − 1.613
= f = × 10
µ−1 1.620 − 1
1 3 1
Again, =− =−
f1 4(15) 20 0.019
= × 10 cm = 0.3 cm = 3 mm
0.620
or f1 = – 20 cm
853. i+e=A+δ ω1 1
863. =
60° + e = 30° + 30° or e = 0° ω2 2
So, light emerges out normally. f1 ω 1
Now, =− 1 =− or f2 = – 2f1
∴ r2 = 0 f2 ω2 2
∴ r1 = A = 30°
Now, 1 1 1
= +
sin 60° F f1 f2
Now, µ=
sin 30°
1 1 1
= +
3 2 50 f1 − 2 f1
or µ= × or µ = 3
2 1 1 −2+1 1
or = =
50 − 2 f1 2 f1
574 COMPREHENSIVE OBJECTIVE PHYSICS

or 2f1 = 50 or f1 = 25 cm 869. δ = (µ – 1)A


Again, f2 = –2 × 25 cm = – 50 cm 5° = (1.5 – 1)A or δ = 10°
864. Φ = (µb – µr) A = (1.659 – 1.641) 5° 870. Total deviation should be zero.
= 0.018 × 5° = 0.09° ∴ 2A2 (1.6 – 1) = 3 × 9 (1.53 – 1)
FG A + δ IJ 27 × 0.53
H 2 K
m
sin or A2 = = 11.9°
2 × 0.6
865. µ=
A 871. A+δ=i+e
sin
2 or e = A + δ – i = 30° + 30° – 60°
FG
60° + δ m IJ = 60° – 60° = 0°
2=
sin
2 H K Clearly, the emergent ray is normal to the face through
sin 30° which it emerges.
1 FG IJ δm 872. The emission line spectrum from left to right is in the
2
= sin 30° +
H K 2 increasing wavelength. The higher the wavelength,
the smaller the frequency of the emission spectrum.
sin 45° = sin FG 30° + δ IJ
δm
= 15° Thus the left-most line represents the ultraviolet emis-
or
H 2 K
or
m
2 sion while the right-most line is associated with the
or δm = 30° red light emission. Hence emission line X has the high-
ω2 ω 1 est frequency (thus shortest wavelength) and is at
866. + =0 the ultraviolet end of the spectrum.
f2 f1
873. f = fr fb = 100.5 × 99.5 cm
ω2 ω ω 1 f2
or =− 1 or ω2 = −
f2 f1 f1 = 9999.75 cm = 99.999
≈ 100 cm
0.02 × (− 30)
or ω2 = − or ω2 = 0.04 Again, ω f = fr – fb
15
867. A = r1 + r2 fr − fb
ω=
30° = r1 + 0 or r1 = 30° f
sin i sin i 100.5 − 99.5 1
Now, =µ or = 2 or ω= = = 0.01
sin r1 sin 30° 100 100
874. For minimum deviation, the angle of incidence should
1
or sin i = or i = 45° be equal to angle of emergence.
2 875. A = r1 + r2
FG A + δ IJ 60° = 0 + r2
H 2 K
m
sin
868. µ= r2 = 60°
A
sin 1 2
2 Again, sin ic = =
FG A + δ IJ 1.5 3
FG IJ
H 2 K
m
sin 2
cot
A
2
=
A
or ic = sin −1
3 H K
= 41.8°
sin Light at the other face would be incident at an angle
2
greater than critical angle. So, It would suffer total
A sin A + δ m FG IJ internal reflection producing maximum deviation.
or
cos
2 = 2 H K 876. Φ = (µv – µr)A
A A 877. For achromatism,
sin sin
2 2
ω1 ω2
F π A I F A + δ IJ
sin G − J = sin G f1
+
f2
=0
H2 2K H 2 K
m
or
ω1 ω f2 ω 1
=− 2 or =− 2 =−
π A A δm π δ f1 f2 f1 ω1 2
or − = + or −A= m
2 2 2 2 2 2 or f1 = – 2f2
or δm = π – 2A
OPTICS 575

1 1 1 1 4−1 3 200
Now, = + or = = or v0 = cm
F f1 f2 v0 200 200 3
1 1 1 1 1 1 For eye-piece,
or = + or = −
10 − 2 f2 f2 10 f2 2 f2 1 1 1
− =
1 1 ve ue fe
or = or f2 = 5 cm
10 2 f2 1 1 1
− =
Now, f1 = – 2 (5) cm = – 10 cm − 25 ue 5
878. The condition for achromatism is : 1 1 1 1 − 1−5
or =− − or =
ω1P1 + ω2P2 = 0 ue 25 5 ue 25
ω1P1 = – ω2P2 25
or ue = – cm
ω1 P 6
or =− 2
ω2 P1 200 25
Now, L = v0 + |ue| = +
Now, P1 + P2 = 2 D 3 6
or 5 + P2 = 2 or P2 = – 3 D 400 + 25 425
= cm = cm
ω1 −3 3 6 6
∴ =− =
ω2 5 5 v0 ve
879. When the parallel beam of white light passes through 884. M = M0 × Me = ×
u0 ue
the metal vapour, energy is absorbed by the metal if
the photon energy hf is sufficient to excite the atom to 200 25 × 6
= × =2
one of its higher levels. When the atom returns to its 3 × 200 25
ground state, it emits the same wavelength as the
fo
photon but equally in all directions. So the intensity 885. M=
of the radiation in the incident (original) direction is fe
reduced. A dark line is thus seen whose wavelength fo
is that of the absorbed photon. 9= or fo = 9fe
fe
880. fo + fe = 500 mm Also, L = fo + fe or 20 = fo + fe
or fo = (500 – 20) mm = 480 mm or 20 = 9fe + fe or 20 = 10 fe
fo 480 or fe = 2 cm
M= = = 24 .
fe 20 ∴ fo = 9 × 2 cm = 18 cm
881. The best position for the eye is eye-ring. All the light 886. fo + fe = 55
that enters the objective from the object must pass
out through the eye-ring. Usually, the telescope is
fo
m = 10 ; = 10 or fo = 10fe
designed to make the eye-ring diameter no larger than fe
1 1 1 Now, 10 fe + fe = 55
the eye-pupil width. Use − = for the eye-piece, or 11fe = 55 or fe = 5 cm
v u f
fo = 10 × 5 cm = 50 cm
with the objective acting as the object.
100
1 1 1 480 Power of objective = D =2D
= − = 50
v 20 500 20 × 500
887. Same as previous question.
1000 888. L = v0 + fe
or v = mm = 20.8 mm.
48 21.7 = v0 + 2.5
fo β
882. M= ,M= 1 1 1
fe α or v0 = 19.2 cm or − =
19.2 u0 1.6
fo 60
∴ β= α= 2° = 24°
fe 5 1 10 10 1 120 − 10 110
or − = − or − = =
883. For objective u0 16 192 u0 192 192
1 1 1 1 1 1 192
− = or = − or u0 = − cm = – 1.75 cm
v0 − 200 50 v0 50 200 110
576 COMPREHENSIVE OBJECTIVE PHYSICS

889. For the eye-piece l


897. θ=
ve = – 25 cm, fe = 5 cm r 3
1 1 1 FG 1 IJ π = 3 × 1 θ
Y km 1000 km
− =
− 25 ue 5 H 60 K 180 1000 Y
Fig. 213
1 1 1 1 − 1− 5
or =− − or =
ue 25 5 ue 25
3 60 × 180 × 7
25 or Y= × km
or ue = − 1000 22
6
= 10.3 km ≈ 10 km
25
Now, v0 = L – |ue| = 20 – fo 60
6 898. m= = = 12
fe 5
120 − 25 95
= cm = cm β β
6 6 = 12 or = 12 or β = 24°
α 2°
1 1 1 1 6 89
Now, − = or = − 1= − λ 2 4800
95 6 u0 1 u0 95 95 899. = = 0.8
λ1 6000
95 95 New resolution limit = 0.8 × 0.1 mm = 0.08 mm
or u0 = – cm or |u0| = cm
89 89
fo + (i) Resolution limit α wavelength.
891. = 9, fo + fe = 20 cm ; 10 fe = 20,
fe 1
(ii) Resolving power α
fe = 2 cm, Also, fo + 2 = 20 or fo = 18 cm wavelength
892. Telescope objective large aperture and large F (hence fo
thin) ; for the eye-piece, small aperture and small f 900. M=
fe
(hence thick). Only P and S fit them.
Smallest value of fe, largest value of M.
1 1 1
893. Use − = ; f = 10 cm, v = – 25 cm. 901. fo + fe = 2.5 m
v u f
1
f fFG IJ fe = m = 0.25 m
894. M = – o 1+ e
fe D H K 4
fo = (2.5 – 0.25) m = 2.25 m
100 FG
10 IJ 1
M=–
10
1+
25H K
= – 10 (1 + 0.4) Po =
2.25
D = 0.444D
= – 10 × 1.4 = – 14 D
902. M = 1+
fe D f
895. L = fo +
fe + D Lesser the focal length, more the power, higher the
10 × 25 angular magnification.
L = 100 +
10 + 25 1 1 1
903. For the objective, − =
250 v0 − 1 / 3.8 1 / 4
L = 100 +
35 1 1 1
= 100 + 7.14 = 107.14 cm. or + 3.8 = 4 or = 0.2 =
v0 v0 5
896. Numerical aperture of objective = µ sin θ
or v0 = 5 cm
0.75 = 1.5 sin θ
and numerical aperture in air = 1 sin θ 5
Now, Mo = = − 19
1
numerical aperture in air 1 −
∴ = 3.8
0.75 1.5
Again, M = Mo × Me
or numerical aperture in air
95
0.75 – 95 = – 19 × Me or Me = =5
= = 0.5 19
1.5
OPTICS 577

FG IJ
fo f 908. The wrong statements a, c, d and e are amended as
904. M=–
H K
fe
1+ e
D follows :

50 F 5I
(a) The telescope has a long focus objective.
M=–
5
G
H 1+ J
25 K
(b) The final image in the telescope is at infinity and is consid-
ered virtual.
50 30 (c) The separation is more than the focal sum in the micro-
M=– × = − 12
5 25 scope.
(d) The telescopic intermediate image is not magnified.
50 cm
(e) The final image is at infinity in the telescope, and at the
near point of the eye in the microscope.
909. Since the final image is formed at infinity therefore
a = 32¢ B² B¢ the image produced by the objective must be at a dis-
32¢ b tance of 3 cm from the eye-piece. The distance of this
image from objective lens is therefore (15 – 3) cm i.e.,
12 cm.

Again,
1 1 1
− =
12 u0 2

1 1 1 1− 6 − 5
25 cm
or = − = =
u0 12 2 12 12
Fig. 214 12
or u0 = − cm = – 2.4 cm
β 5
M= 1.22 λ
α
910. dθ =
tan β A ″ B″ /25 a
M= =
tan α A ′ B′ /50 d 1.22 × 6000 × 10 −10
=
A ″ B″ 50 38.6 × 104 × 103 5
12 = ×
25 A ′ B′ 1.22 × 6 × 38.6
or d= m = 56.51 m
32° 5
or A′′ B′′ = 6A′ Β′ = 6 × 50 × tan
60 911. In the first case, the image will be formed at a dis-
= 300 × tan (0.5333) tance of 80 cm from the objective.
300 × 9.308 × 10–3 = 2.7924 cm In the second case,
1 1 1 1 1 1 1 −1 1
906. − = − = or = +
v0 − 1.25 1.2 v0 (− 40) 0.8 v0 40 0.8
1 1 1 1 1.25 − 1.2 1 − 0.8 + 40
or = − or = or =
v0 1.2 1.25 v0 1.2 × 1.25 v0 32
1 0.05 32
= 150 or v0 = m or v0 = 0.8163 m
or or v0 = cm = 30 cm 39.2
v0 150
. 5
or v0 = 81.63 cm
v0 30
Mo = = Clearly, the eye-piece shall have to be drawn out
u0 − 1.25 through a distance of (81.63 – 80) i.e., 1.63 cm.
Mo = – 24
3.5 × 103 3.5 180°
912. α= radian = ×
D 25 3.8 × 105 3.8 × 100 π
Me = =
fe 3 3.5 × 180 × 7°
=
25 38 × 100 × 22
M = – 24 ×
3 fo 400
Also, M= = = 40 ;
M = – 200 fe 10
907. Order 4 cm for eye-piece always. 40 × 35 × 180 × 7°
For maximum F/f the choice is (a). β= = 21.1° ≈ 21°.
35 × 100 × 22
578 COMPREHENSIVE OBJECTIVE PHYSICS

918. The person suffers from hypermetropia. He should


914. Separation = fo + fe D
use convex lens of such a focal length that virtual
fe + D image is formed at 40 cm when the object is at a dis-
5 × 25 125 tance of 25 cm.
= 80 + = 80 +
5 + 25 30 100 100 1
− = or P = – 2.5 + 4 = 1.5 D
= 84.16 cm = 84.2 cm. − 40 − 25 f
FG 1 IJ 919. The person is suffering from short-sightedness or
915.
H µK
Shift = t 1 − myopia.
1 1 1
F I
1 = 3 G 1 − 1J or
1
= 1−
1
= −
f v u
H µK 3 µ 1
= −
1 1
or
1
=−
1

1
=
−1 − 2
1 1 2 3 − 50 d − 25 d 50 25 50
or = 1− = or µ = = 1.5 50
µ 3 3 2 or d=– cm
3
fo FG f IJ f2
916. (1) M=–
fe H
1+ e
D K 920. t ∝ 2
d

200 FG
5 IJ t is same.
M=–
5
1+
25H K d22 d12
= 2
f22 f1
FG 1 IJ 6 d1
M = – 40 1 +
H 5 K
= – 40 ×
5
= – 48
d2 =
f1
f2 = 5 × 15 = 3 cm
25
fo 200 1 1 1
(2) M= =− = – 40 921. = −
fe 5 f v u

917. Refer to diagram showing telescope in normal adjust- 1 1 1 100 25


2.5 = − or =− −
ment. − 0.75 u u 75 10
1 4 5 1 − 8 − 15 23
f1 f2 or =− − or = =−
eye-piece u 3 2 u 6 6
6
or u=– m = – 0.26 m
23
a
b 1 1
922. f= = m = 20 cm
F0 P 5
1 1 1
Now, − =
v u 20
top part 1 1 1 1 1 1
of or − = or = −
final v − 25 20 v 20 25
object (f2)
lens image 1 5−4 1 1
or = or =
v 100 − d 100
Fig. 215 or d = – 100 cm = – 1 m.
Telescope in normal adjustment = Final image at infinity 1 1 1
923. = −
f1 β f v u
M (angular magnification) = =
f2 α 1 1 1 1 1 1
= − or =− +
and separation of (distance between) lenses = f1 + f2 f − 50 − 25 f 50 25
Hence, to increase the magnifiying power (M) of the
1 − 1+ 2
instrument, eye-piece should be replaced by another = or f = 50 cm
shorter focal length (f 2) and moved closer to the f 50
objective lens. 100
P in dioptre = =2
50
OPTICS 579

f2 1 f1 1 − 80 FG IJ
924. t∝
d2 f2
×
1
=−
120 1 H K
“f” f f1 2
means that the diameter of aperture is . =
2 2 f2 3
1 f2 P1 3
Now, ∝ =
100 fFG IJ
2
P2 2
2 H K 1

1
=
1
1 931.
v − 54 4
or ∝4 ...(1)
100 1 1 1 1 50
f2 = − or =
Again, t∝ v 4 54 v 4 × 54
fFG IJ
2
4 × 54
8 H K or v=
50
cm or v = 4.32 cm
or t ∝ 64 ...(2) = 43.2 mm
Dividing (2) by (1), So, displacement is 3.2 mm away from the film.
64
= 16 or t =
16
s 1 f2 1
100 t = ∝ ∝4
4 100 932.
100 f FG IJ
2 or
100
...(1)

925.
1 1 1
= − 2 H K
f v u
f2
1 1 1 Again, t∝ or t α 16 ...(2)
= − or
1
=
1

1
FG f IJ 2
f − 100 − 25
1 4−1
f
1
25 100
3
H 4K
or = or = t 16
f 100 f 100 = =4 4
1 4 or t = s
100 100
or f= cm 100
3
1
100 × 3 933. f = m = 50 cm
P= =3D 2
100
v f 50 1 1 1 1 1 1
m= = = 1 Now, = − or = −
926. = 50 v − 25 v 50 25
u u 2000 × 100 4000
1 1 1 1 1− 2
927. = − or = or v = – 50 cm
f − 40 − ∞ v 50
or f = – 40 cm This is the near point of the eye.
100 100 1 1 1 1 1 1
P= =− D = − 2.5 D Again, = − or = −
f 40 50 v − 40 v 50 40
928. The shape (curvature) and therefore the focal length 1 4−5
of the lens can be modified somewhat by the ciliary or = or v = – 200 cm
v 200
muscles. As an example, when the muscle is relaxed,
This is the new near point.
the focal length is about 2.5 cm and (for a normal eye)
objects at infinity are in sharp focus on the retina. Again, 1 1 1
= −
When the object is brought closer to the eye, in order f − 200 − 25
to maintain the same image–lens distance (≈ 2.5 cm),
the focal length of the eye-lens becomes shorter by the 1 1 1 1 − 1+ 8 7
or =− + or = =
action of the ciliary muscles. This property of the eye f 200 25 f 200 200
is called accomodation. 200
1 1 1 or f = cm
= − 7
929.
f1 − 80 − ∞ 100 × 7 D
P= = 3.5 D
1 1 1 200
= −
f2 − 120 − ∞
580 COMPREHENSIVE OBJECTIVE PHYSICS

1 1 1 (2 × 3 − 1) Dλ
934. = − or f = – 150 cm 940. y3 ′ =
f − 150 − ∞ 2d
100 2 5Dλ
D = − D = – 0.67 D y5 =
P= d
− 150 3
Now, y5 – y3′
1 1 1 −1 1 1
935. − = or = − 5Dλ 5Dλ Dλ FG 5 IJ
v u f
−1 v− f
u f v
− fv
=
d

2d
=
d
5−
H 2 K
or = or u= 5Dλ 5 × 1 × 7000 × 10−10
u fv v− f = = m
2d 2 × 1 × 10−3
fv
or u= = 17.5 × 10–4 m = 1.75 mm.
f −v
941. d sin θ = nλ = (µ – 1)t
I v v
= = (µ − 1)t
O u fv or sin θ =
f −v d
I f −v O f (1.17 − 1) 1.5 × 10−7
or = or = = = 0.085
O f I f −v 3 × 10−7
or θ = sin–1 (0.085) = 4.876°
If
or O= D1λ 1
f −v 942. β=
936. Let nth minima of 400 nm coincide with mth minima d1
of 560 nm. Then D λ
β= 2 2
(2n – 1) 400 = (2m – 1) 560 d2


2n − 1 7 14 21
= = = Now, D1λ 1 = D2λ 2
2m − 1 5 10 15 d1 d2
i.e. 4th minima of 400 nm coincides with 3rd minima F IFλ I = 2 × 2 =4
of 560 nm. or D1
D2
d
= 1
d2
GH JK GH λ JK 1 1
2
1
The location of this minima is
I1 a12 a1
7(1000)(400 × 10−6 ) 943. β= = or = β
mm = 14 mm I2 a22 a2
2 × 0.1
Next, 11th minima of 400 nm will coincide with 8th Fringe visibility
minima of 560 nm. Imax. − Imin. (a + a2 )2 − (a1 − a2 )2
= = 1
The location of this minima is Imax. + Imin. (a1 + a2 )2 + (a1 − a2 )2
21(1000)(400 × 10 −6 ) 4 a1a2 2a a
mm = 42 mm = 2 2
= 2 1 22
2 × 0.1 2 (a1 + a2 ) a1 + a2
∴ Required distance = 28 mm a1
2
I1 1 a1 1 a2 2 β
937. = ; = = =
I2 9 a2 3 a12 β+1
+1
Imin. (3 − 1)2 4 1 a22
= = =
Imax. (3 + 1)2 16 4 944. δ = (µ – 1) A a δ
δ
d d
Also, δ=
+ Intensity is proportional to width of slit. 2a
2

∴ d
60 × 4 = (µ − 1)A
938. n′ × 6000 = n × 4000 or n′ = = 40 2a
6 or d = 2a (µ – 1)A Fig. 216
nDλ
939. yn = Dλ
d β=
d
Lesser the value of λ, lessr the value of yn.
OPTICS 581

Dλ Dλ yd yd
or β= or 2 aβA = 952. − (µ − 1) t = nλ or = (µ − 1) t + nλ
2a(µ − 1) A µ−1 D D
Dλ Dλ For n = 0,
or µ–1= or µ = 1 +
2 aβA 2 aAβ D(µ − 1)t
y=
Dλ Dλ d
945. β= or d= 953. Net path difference introduced
d β
= (1.7 – 1)t – (1.4 – 1)t = 0.3t
1 × 5000 × 10 −10 When the path difference is λ, the shift on the screen
= m = 1000 × 10–7 m
5 × 10 −3
= 10–4 m = 10–4 × 103 mm = 0.1 mm is

d
b = βg .
4 β
946. λ is reduced by a factor of . When path difference is 0.3t, shift on screen is × 0.3t
3 λ
4
So, β is reduced by a factor of . β 5λ
3 But × 0.3t = 5β or t =
λ 0.3
β 3
β′ = = β = 3 × 0.4 mm 5 × 480 × 10−9
4/3 4 4 = m = 8000 × 10–9 m
0.3
= 0.3 mm = 8 × 10–6 m = 8 µm
947. Note that the path difference is 2λ.
(2n + 1)Dλ
−10 954. yn =
Dλ 2 × 6000 × 10 2d
948. β= = m
d 4 × 10 −3 2dyn 2dyn
or 2n + 1 = or 2n = −1
=3× 10–4
m =3× 10–4 × 103 mm Dλ Dλ
= 0.3 mm 2dyn 1 dyn 1
or n= − or n = −
λ 4 µt 2Dλ 2 Dλ 2
949. 2 µt = (2n + 1) or λ =
2 2n + 1 dyn 1
Now, n1 = −
Dλ 1 2
4 × 1 × 0.45 × 10 −6 × 1010
or λ= Å
2n + 1 dyn 1
n2 = −
Dλ 2 2
1.8 × 104 18000
= Å = Å dyn dyn
2n + 1 2n + 1 n1 – n2 = −
Dλ 1 Dλ 2
Taking n = 1,
18000 or n1 – n2 = dyn 1 − 1
LM OP
λ=
3
Å = 6000 Å
D λ1 λ2 N Q
950. β=

or 8 =
d × 32 × 10 −3 1010

LM
1010 OP
d
λ smaller, β less.
1 MN
4000 6000 PQ
951.
Imax. b a + a2
= 1
g 2
or d=
8 × 4000 × 6000
2000 × 1010 × 32 × 10−3
m
I min. b a1 − a2 g 2
= 3 × 106 × 10–10 m = 3 × 10–4 m
9 ba +a g
2 = 3 × 10–4 × 103 mm = 0.3 mm
= 1 2 a1 + a2 3
1 ba −a g
2 or = yd yd
1 2 a1 − a2 1 955. + (µ − 1)t = nλ or = − (µ − 1)t + nλ
D D
or a1 + a2 = 3a1 – 3a2 or 4a2 = 2a1
When n = 0,
a1 4 2
or = = D(µ − 1)t
a2 2 1 y=–
d
582 COMPREHENSIVE OBJECTIVE PHYSICS

The negative sign indicates that the fringe system


φ 1 π
moves down. or cos = = cos 45° = cos
2 2 4
956. (n + 1) 4800 = n × 6000
n+1 1 1 π λ
or = 1.25 or = 0.25 = or n = 4 or φ= . Corresponding path difference is . So,
n n 4 2 4
957. Path difference β Dλ
D the required shift on the screen is i.e., .
2
= D +d −D 2 4 4d
d/2
Dλ λ λ
F dI
= DG1 +
2 1/2 961. β= = =

H D JK
d d/D θ
–D
2
a22
FG a + 1IJ 2

L 1d OP − D d
1

= D M1 +
2 2 Fig. 217 962. Imax.
= 1
b
a + a2 g 2
=
Ha K
2

MN 2 D 2
PQ = 2D I min. b
a1 − a2 g 2
a22
FG a − 1IJ
1
2

d2 d2
Ha K
2
Now, nλ = or n =
2D 2Dλ FG a 1 IJ 2

958. For maximum intensity, Y


=
Ha 2
+1
K
AD – BD = nλ FG a 1 I
− 1J
2

Ha K
A
or AD = nλ + BD
2
or AD = nλ + xn 4λ
Now, AD2 = AB2 + BD2 I1 2 a1
I1 = 2I2, = , = 2
or (nλ + xn)2 = (4λ)2 + xn2 B D X I2 1 a2

or n2λ2 + xn2 + 2nλ xn = 16 λ2 + xn2


Fig. 218
∴ Imax.
=
e 2 + 1j 2

=
5.827
= 34
I min. e 2 − 1j 0.171
(16 − n2 ) λ2
or 2nλxn = (16 – n2)λ2 or xn = I min. 1
2nλ or =
Imax. 34
15λ2
For n = 1, x1 = = 7.5λ 963.
nDλ 3 × 200 × 5000 × 10 −8 cm
2λ yn = =
d 0.2 × 10 −1
(16 − 4)λ2 = 1.5 cm
For n = 2, x2 = or x2 = 3λ
2×2×λ
1
(16 − 9)λ2 7λ 964. Air film ; So 2∆t = × 6 × 10−5 cm
For n = 3, x3 = = 5
6λ 6
For n = 4, xn = 0 6
That gives ∆t =
in units given.
Thus, there are only three points where maximum is 10
observed. 965. Path diff. 3λ corresponds to phase difference 6π.
Dλ Also, φ = 3nπ
959. β=
d Here, n = 3. ∴ φ = 6π
D × 6 × 10−7 D 0.12 × 10−3 Dλ
0.12 × 10–3 = or = 966. β=
d d 6 × 10−7 d
d 6 × 10 −7 D1λ 1 D2λ 2
or θ= = = 50 × 10–4 =
D 0.12 × 10−3 d1 d2
= 5 × 10–3 radian.
D1 λ 2 d1 D1 λ 2 d1
φ = × or = ×
960. I = I0 cos2 D2 d2 λ 1 D2 λ 1 d2
2
I0 φ 6000 2 3
= I0 cos2 = × =
2 2 4000 1 1
OPTICS 583

967. β′ λ′ 6000 6 λ
= = = 973. ∆ = 70.5 λ = 141
β λ 5000 5 2
6 λ
or β′ = × 1 mm = 1.2 mm Path difference is odd multiple of . So, the point is
5 2
dark.
968. n × 5461 = 92 × 5898
974. Shift on screen = 2 β
92 × 5898
or n= ≈ 99 So, corresponding change in path difference = 2λ
5461
969. 2µt = nλ 3
975. β= = 0.3 cm
t 10
nλ a Dλ
t= Now, β=
2µ x d
Fig. 219 300 × 5100 × 10−8
10 × 6000 × 10 −10
= m ∴ 0.3 =
2 × 1.5 d
= 2 × 104 × 10–10 m = 2 × 10–6 m 300 × 5100 × 10 −8
or d= cm
t t 2 × 10−6 0.3
Now, α= or x = = = 51 × 10–3 cm = 51 × 10–2 mm
x α 0.01
= 2 × 10–4 m = 2 × 10–4 × 103 mm = 0.51 mm
= 0.2 mm nDλ 1
976. y1 =
λ d
970. 2µt = (2n + 1) λ1 y1
2 nDλ 2 =
y2 = or
4µt d λ 2 y2
λ=
2n + 1 (2n + 1)Dλ
977. y n′ =
4 × 1.4 × 10,000 2d
λ= Å
2n + 1 (2 × 1 + 1)Dλ Dλ 0.6
0.3 = or = = 0.2 cm
56000 2d d 3
λ= Å
2n + 1 nDλ
Now, yn = = 4 × 0.2 cm = 0.8 cm
λ = 56000 Å, 18666.7 Å, 11200 Å, 8000 Å, 6222.2 Å, d
5091 Å, 4307.7 Å out of these wavelengths only the Imax. 16
last three belong to visible region. 979. =
I min. 1
λ
971. Path difference = x 2 + d2 − x = a1 + a2 4
2 =
x
a1 − a2 1
λ
Now, x 2 + d2 − x = 2a1 5
2 = a1 5
or
=
λ 2a2 3 a2 3
or x 2 + d2 = x +
2 d 980. Straight fringes are obtained because locus of same
2 2
2 x +d path difference gives lines which are parallel to the
λ
or x2 + d2 = x2 + + xλ axis of the cylinder.
4
981. IA = I1 + I2 + 2 I1 I2 cos φ
λ2
or d2 − = xλ π
4 Fig. 220 IA = I + 4I + 2 I × 4I cos
2
d2 λ
or x= − or 9λ – 0.25λ = 8.75λ IA = 5I
λ 4
Again, IB = I + 4I + 2 I × 4I cos π = 5I – 4I = I
16 × 6000
972. n × 4800 = 16 × 6000 or n = = 20 982. n × 400 = 12 × 600
4800
12 × 600
or n= = 18
400
584 COMPREHENSIVE OBJECTIVE PHYSICS

nDλ π
983. yn = difference between waves arriving at B is . Now,
d 2
nλ = constant π
I = 4I0 cos2 = 2I0. In the direction of C, the distur-
∴ m × 6500 = n × 5200 4
π π
m 5200 4 bances have a phase difference of + i.e. π. Since
= = 2 2
n 6500 5 the disturbances are 180° out of phase therefore the
yd intensity at C is zero.
984. + (µ – µ′)l = 0
D 987. ∆ = nλ

or
yd
D
= – (µ – µ′)l or y =
D
d
b
µ′ − µ l g d cos θ = nλ
θ
nλ δ2 δ1
cos θ =
or 15β =
D
d
b
µ′ − µ l g or
15Dλ
d
=
D
d
µ′ − µ l b g d

15λ or cos θ =
or 15λ = (µ′ – µ)l or µ′– µ = d Fig. 222
l
FG 4λ IJ
or µ′ = µ +
15λ
l
or θ = cos–1 HdK
988. Refer to sin2 term in intensity equation.
D2 + d2 − D
D
985. Path difference =
989. Refer to sol. 944
For dark fringe,

λ µ = 1+
D2 + d 2 − D = d 2 aAβ
2 2 2
D +d
λ 1 × 6000 × 10 −10
or D2 + d 2 = D + µ = 1+
π
2 2 × 10 × 10 −2 × × 0.03 × 10−2
Fig. 221 180
2 λ2
or D2 + d2 = D + + Dλ 6 × 10−4 × 7 × 180
4 or µ = 1+

or
LM
Dλ = d2 Neglecting
λ2 OP or λ =
d2
2 × 22 × 0.03
µ = 1 + 0.5727 or µ = 1.5727 = 1.573
MN 4 PQ D
or
nDλ
Time-saving solution 990. yn =
d
λ
D2 + d 2 − D = n × 2.5 × λ
2 10–3 =
2 × 10−3
eD + d j
2 2 1/2
− D=
λ
2
10 − 3 × 2 × 10 −3
F dI
D G1 +
2 1/2
λ F 1 d2 I or λ= m

H D JK
λ n × 2.5
2
−D=
2
or D 1+GH 2 D2 JK
−D=
2
0.8 × 10−6
2 2
d or λ = m
or 1 d λ or λ = n
=
2 D 2 D
20
0.8 × 10 −6 × 1010 Å
986. Separation = 5 m ; λ = 20 m ; So, separation is m or λ=
4 n
λ π 0.8 × 104 8000
i.e. , P is ahead of Q by a phase of i.e. ahead in or λ= Å or λ =
4 2 n n
λ For n= 1, λ = 8000 Å
path by . So, in the direction A, intensity is largest
2 For n= 2, λ = 4000 Å
(4I0). Thus, options (a) and (c) are eliminated. Phase
For n= 3, λ = 2666.7 Å
Clearly, 4000 Å is the right choice.
OPTICS 585

FG 2π IJ S
991. d 998. Data require HλK 1 A – 2πp = 2πn.
That leads to S1 A = (n + p)λ
d β
999. 4β = (µ – 1) t
2d λ
4λ 4 × 6000 × 10 −10
or t= = m
µ−1 1.5 − 1

Fig. 223
= 48 × 0–7 m = 4.8 × 10–6 m = 4.8 µm

2 d – d = (µ – 1)t 1000.
ba
1 g
+ a2
2
=
9
or
a1 + a2 3
=

or d( 2 – 1) = (1.5 – 1)t or t = 2( 2 – 1)d


ba
1 −a g
2
2
1 a1 − a2 1

2 a1 4 a1 2
992. Only one image of slit, no interference. or = or =
2a2 2 a2 1
993. n × 6000 = 10 × 4200
10 × 4200 I1 4
or n= or n = 7 or =
6000 I2 1
nDλ β (µ − 1)t
994. yn = 1001. (µ – 1) t = 5β or λ =
d λ 5
4 × 1.5 × 100 × λ
1= (1.5 − 1) 6 × 10−6 3
0.03 = = × 10–6 m
5 5
0.03 0.03
or λ= cm or λ = × 108 Å = 0.6 ×10–6 m = 0.6 × 10–6 × 1010
600 600
or λ = 5000 Å = 6000 Å
995. Phase difference is odd multiple of π. 1002. For the central bright fringe

λ I = I1 + I 1 + 2 I1 I2 cos 0°
996. Extra distance travelled = . Due to reflection from
2 I
or I = 2I1 + 2I1 = 4I1 or I1 =
λ 4
denser medium, extra path travelled is . Total path
2 Now, for distance x on screen, from central maximum,
difference is λ. Corresponding phase difference is 2π.
xd
Let I be intensity of the direct beam. Then, intensity path difference =
D
I
of reflected beam is . 2π xd
4 Corresponding phase difference, φ = .
λ D
I I Now,
Now, I′ = I + + 2 I × cos 2π
4 4
I I I I 2π xd
I′ = + +2 × cos
5I 2I 4 4 4 4 λ D
or I′ = + ×1
4 2 I I 2πxd
or I′ = + cos
9I 2 2 λD
I′ =
4 FG
I 2πxd IJ
9
or I′ =
2 H 1 + cos
λD K
So, the resultant intensity is . IF IJ
I′ = GH 2 cos
4 πxd πxd
K
2
or or I′ = I cos2
π 2 λD λD
997. Phase lag of light from S2 is 3π sin θ +
. For maxima 1
2 1003. Intensity of AB, I1 = I
this is n . 2π. In the range available we have 4
−5 −1 1 3
sin θ = , and . I
6 6 2 Intensity of AC =
4
586 COMPREHENSIVE OBJECTIVE PHYSICS

FG IJ = 3 I
1 3 β 4500 45
Intensity of CA′ = H K 16
4 4
I =
0.4 6000
or β =
60
× 0.4 mm = 0.3 mm.

3F 3 I β Dλ λ λ
= G IJ =
9 1010. θ= = = or d =
Intensity of A′ B′, I2 H
4 16 K 64
I D dD d θ

1 5890 × 10 −10
I I1 1 64 16 =
Now, I1 4 or 1 π
= = × = ×
I2 9 I2 4 9 9 60 180
I
64
a1 4 5890 × 10 −10 × 60 × 180
or = = m
a2 3 π

a12
FG a IJ 2 = 0.00202 m = 0.202 cm.
b g
1
Imax. a + a2
= 1 =
2
Ha 2
+1
K 1011. In the first case, the intensity is 4I and in the second
case, the intensity is 2I. So, the ratio is 4I : 2I i.e.
Imin. (a1 − a2 )2
a12
FG a
1
− 1J
I 2
2 : 1.
Ha2 K 1012. Path difference corresponding to 5th maximum is 5λ.
FG 4 + 1IJ 2
The corresponding phase difference is 10π.

=
H3 K =
49 Dλ
FG 4 − 1IJ 2 1 1013.
d
β=
H3 K For less d, β more
I1 4
1004. = nλ
I2 1 1014. 2µt = nλ or t=

a12 4
= λ 589.3
a22 1 tleast = = nm = 213.5 nm
2µ 2 × 1.38
a1 2
= 1015. It can be shown that the separation of the bright
2 1 fringes of a double-slit interference pattern is given
2 by
Imax. a1 + a2
d i 32 9
= = =
Imin. b
a1 − a2
2
g 12 1
1005. I0 ∝ (a + a)2 or I0 ∝ 4a2 A
Bright yn + 1
fringes
Again, I ∝ a2 yn
d
I0 I0
=4 or I =
I 4 B

1007. y ∝ n L
Screen

8 ∝ 20
y30 ∝ 30 Fig. 224
y30 30 3
∴ = = or y 30 = 12 mm. Lλ
8 20 2 yn +1 – yn =
d
1008. 3I = I + 4I + 2 I × 4I cos φ
where L is the distance from slits to screen.
or – 2I = 2 × 2I cos φ d is the distance between slits.
1 λ is the wavelength of source.
or cos φ = – or φ = 120°.
2 Thus, separation can be increased by increasing L,
1009. 0.4 ∝ 6000 decreasing d or using a source of longer wavelength
β ∝ 4500 λ.
OPTICS 587

λ (iv) replace the source with one of lower frequency or


1016. 2µd = longer wavelength (e.g. red light as suggested in
2
the options).
λ 589 Note : Changing the distance between the source
or d= = nm = 117.8 nm ≈ 118 nm
4µ 4 × 1.25 and the slits does not increase the separation of
the fringes.
1017. Path difference = 2µt
FG n + 1IJ 1022. O= I1 I2
= 28000 Å. This is to equal H 2K λ considering
O= 16 × 9 cm
Stokes phase change π . (b), (c), (d) contain 4000 Å, or
7000 Å giving nλ. So, only (a). O = 12 cm
λl β
1018. Fringe separation y = 1023. (µ − 1) t = nβ
d λ
where λ = wavelength of light, (µ − 1)t nλ
l = distance from the slit to screen, or =n or t=
λ µ−1
d = separation of the slit.
For minimum value of t, n should be minimum.
For same fringe separation, λyly = λblb
λ λ
⇒ distance from the slit, ∴ tmin. = = = 2λ
µ − 1 1.5 − 1
F λ I l = FG 600 IJ (1.00) = 1.50 m.
lb = GH λ JK H 400 K
y

b
y 1024. Path difference should be an odd multiple of
λ
2
.
1019. It can be shown that the separation of the fringes of a 1025. The forming of Newton’s rings is due to the interference
double-slit interference pattern is given by of light between converging lens and the flat plane
λl mirror. When the mirror is gradually separated from
fringe separation = the lens, the path difference of the light between
d
To increase fringe separation, one could converging lens and the flat plane mirror increases.
Hence, the bright and dark rings, though concentric
(i) increase wavelength of light or decrease its
frequency. with the point of contact, T, shrink towards their
centre T as their redii decrease. Eventually, the
(ii) increase the distance l or/and
Newton’s rings disappear at the point of contact, T.
(iii) decrease the distance d between slits. O
1026. The following arguments shall
−9
λ 500 × 10 180 × 7° lead us to the right choice.
1020. = × = 0.0143°
d 2 × 10 −3 22
d
θθ
(i) cos =
1021. OP
d
d
A or OP = cos θ = d sec θ
θ
d (ii) Treat OC as the component
of OP C
B P
OC = OP cos 2θ A
= d sec θ cos 2 θ Fig. 226
L
(iii) Path difference = (d sec θ + d sec θ cos 2θ)
Fig. 225 Corresponding phase difference
To increase the separation of the fringes, we can 2π
= (d sec θ + d sec θ cos 2θ) .
(i) increase the distance (L) between the screen and λ
the slits. (iv) Effective phase difference
(ii) decrease the distance (d) between the two slits. 2π
= (d sec θ + d sec θ cos θ) +π
(iii) decrease the width of each slit. λ
588 COMPREHENSIVE OBJECTIVE PHYSICS

(v) For constructive interference 1031. The phase difference between the wavelets from the

2π top and bottom edges of the slit is d sin θ, where d
(d sec θ + d sec θ cos 2θ) + π = (2n)π λ
λ is the width of the slit.
LM d + d (2 cos 2
θ − 1)
OP 2π The first minimum of the diffraction pattern occurs
or
N cos θ cos θ Q λ
+ π = (2n)π
at sin θ = .
λ
d
2π 2π λ
or (2 d cos θ) + π = 2nπ ∴ Required phase difference is d i.e. 2π.
λ λ d
400 × 10−9
4 d cos θ 1032. sin 30° =
or + 1 = 2n d
λ
1 400 × 10−9
4 d cos θ or = or d = 800 × 10–9 m
or = (2n − 1) 2 d
λ 3λ
(2n − 1)λ Again, sin θ =
2d
or cos θ =
4d 3 × 400 × 10−9 3
λ sin θ = or sin θ =
If n = 1, cos θ = 2 × 800 × 10−9 4
4d
FG IJ
3
1027.
β
(µ – 1)t
λ
= nβ or θ = sin–1 H K
4
5Dλ 5Dλ
(µ − 1)t (1.5 − 1) 2 × 10 −6 1033. y= or d =
or = n or n = d y
λ 500 × 10−9
1 5 × 0.5 × 5000 × 10−10
= × 10 = 2 = m
5 0.5 × 10−3
So, the central maximum shall shift upward by = 25 × 10–4 m = 25 × 10–1 mm
2 fringes. = 2.5 mm
1028. When path difference changes by λ, the shift of cen- λ 1
tral maximum will be equal to one fringe width β. 1034. Angular spread on either side = = radian.
d 5
β 1035. The first order image of the 435.8 nm (λ1) blue light
Shift corresponding to unit path difference is . Shift
λ occurred at an angle of 15.8° (θ1). If d is the grating
β spacing, then
corresponding to path difference (µ – 1) t is (µ – 1) t.
λ 435.8 nm
1029. (a + b) sin θn = nλ d sin θ1 = λ1 or d = = 1600.56 nm
sin 15.8°
Here n is the order of the spectrum and (a + b) is the Thus, the wavelength (λ2) of red line of the impurity
grating element. in the mercury is
For maximum value of n, sin θn = 1 d sin θ2 = λ2 where θ2 = 23.7°.
a+b ⇒ λ2 = (1600.56) (sin 23.7°) = 643.3 nm
∴ (a + b) = nλ or n =
λ Hence, the impurity is cadmium and the associated
1 1 red line’s wavelength is 643.3 nm
Now, n = ×
5000 520 × 10−9 × 102 1036. d sin θ = λ, dθ = λ,
A
104 λ
= = 3.846 ≈ 4 θ=
2600 d
nDλ 600 × 10−9 θ
1030. yn = θ=
B
d −3 D
1 × 10
1 × 2 × 5000 × 10−10 Fig. 227
5 × 10–3 = = 6 × 10–4 radian
d
Distance between minima = 2AB
−6
10 1 = 2D tan θ = 2Dθ = 2 × 2 × 6 × 10–4 m
or d= = m
5 × 10−3 5000 = 24 × 10–4 m = 2.4 mm.
1
= mm = 0.2 mm.
5
OPTICS 589
1037. The spacing of the slits on a diffraction grating d is
P
given by
A
d=a+b
O
The angle θ between the emerging zero-order and first- B O′
order spectra is given by
λ λ λ FG IJ
sin θ = =
d a+b
or θ = sin–1
a +b H K
Hence, θ is dependent on a, b and λ. Fig. 229
1038. When monochromatic light passes through a plane For P to be a minimum, the phase difference of sec-
diffraction grating, bright or principal maxima are ondary waves from O and A (or B) is π (out of phase).
obtained when Hence, the phase difference between the wavelets from
d sin θ = nλ the opposite edges is 2π.
d = spacing of lines. 1042. The sine of the angle between the second order maxi-
where λ = wavelength of light. mum and the normal is given by
n = order of bright image. d sin θ = nλ
We have θ = 30°, n = 2, λ = 5.0 × 10–7 m 2λ 2λ 2
sin θ = = =
∴ d=
e
2 5.0 × 10 −7
j = 2 × 10
−6
m
or
d 3λ 3
[∵ n = 2, d = 3λ]
1043. Distance between successive wavefronts is a wave-
sin 30°
length λ. Thus between wavelength XY and point P,
Hence number of lines per millimetre of the grating the path difference is 3λ.
is

1 1 Time taken from plane XY to reach P is thus t =
× 10−3 = × 106 × 10−3 = 500 c
d 2
2
1039. 1044. Angular spread ≈ 2 × rad
500
4
Linear spread = × 200 m = 1.6 m
θ
500
This is nearest to 1m.
1.5
1045. Angle subtended by filament = = 10−3
150
Fig. 228 6 × 10 −5
Spread due to diffraction = = 2 × 10–5 rad
In general, the angle of diffraction θ of the nth-order 3
diffraction beam is related to the spacing d by So, ratio is 50.
d sin θ = nλ 1046. d sin 30° = 5900 × 10–10
2λ or d = 2 × 5900 × 10–10 m
∴ sin θ = if n = 2.
d = 118 × 10–8 m = 1.18 × 10–6 m
1040. If d is the spacing of the lines on the grating, then the
= 1.18 micron.
angle θ of the nth order diffracted beams measured
1047.
with respect to the line normal to the grating is given
by P
d sin θ = nλ R
α
Now, θ= and n = 2. A θ
2
FG IJ
a d
α 2λ B
∴ d sin
H K
2
= 2λ or d =
α FG IJ
H K
S
sin P′
2
1041. The diagrammatic representation of the given prob-
lem is shown in Fig. 229. Fig. 230
590 COMPREHENSIVE OBJECTIVE PHYSICS

Let the centre of the slit RS be d. The first dark band 1053. 2d sin θ = nλ
will be formed at an angle θ to the incident beam if 2 × d sin 32° = λ
the path difference of light from the strip just below R Also, 2 × d sin θ = 2λ
λ
and the strip-just below d is , where λ is the sin θ 2λ
2 Dividing, = or sin θ = 2 sin 32°
wavelength of the light. This happens for all pairs of sin 32° λ
corresponding strip in Rd and dS because the path
= 2 × 0.529 = 1.059
λ
difference of exists. Hence, there is no light in But sin θ cannot be greater than 1.
2
direction θ when So, there is no second order diffraction.
1054. (a + b) sin θn = nλ
λ
dA = or SB = 2dA = λ 1
2 sin θn = 3 × 5400 × 10–8
i.e., SP – RP = SB = λ 2000
SB λ or sin θn = 2000 × 3 × 5400 × 10–8
Now, sin θ = sin ∠dRA = = or sin θn = 0.324
RS a
1048. In diffraction/interference theory, two contributions or θn = sin–1 (0.324)
like A cos ωt and B cos (ωt + α) are represented by two
2λ 2 × 6328 × 10−10
vectors of lengths A and B with angle α to get the 1055. sin θ = =
resultant amplitude with the correct phase. Since the d 0.2 × 10 −3
disturbances are along the same line, these are not = 6328 ×10–6
vectors in the ordinary sense of the term. These are or θ = sin–1 (0.006328) = 0.36°
called phase vectors or phasors. Phase difference
I0
2π 1056. Required intensity is I0 cos2 45° or .
= (d sin θ). Moreover, anticlockwise is taken as 2
λ
I0 I 3
+ ve. 1057. I= cos2 30° = 0 ×
2 2 4
1049. Resolving power of the eye is given by
3 I 3
λ = I0 or =
R = 1.22 ×D [d is diameter of eye lens] 8 I0 8
d
500 × 10 −9 × 4 × 105 I0
= 1.22 × m 1058. I1 = I0 cos2 45° =
5 × 10 −3 2
= 48.8 m ≈ 50 m I0 I
Again, I2 = cos2 45° = 0
1.22λ 2 4
1050. dθ =
a 2π 2 × 3.14
1059. T= = s
y ω 31.4
Also, dθ =
D = 0.2 s
y 1.22λ 1.22 λ D Energy transmitted/revolution
∴ = or y =
D a a FG I AIJ 0.2
H2 K
0
1051. d sin θ = nλ = (IA)T =
nλ 1 × 6500 × 10 −10
d= = 10−3 × 0.2
sin θ sin 30° = = 10–4 J
2
= 130 × 10–8 m = 130 × 10–8 × 106 micron
I 2
= 1.3 micron 1060. I = 0 cos 30°
2
2Dλ I 3
1052. Width of central maximum = I = 0 ×
d 2 4
2 × 2.5 × 6000 × 10 −10 3I0
= m I =
6 × 10 −3 8
= 5 ×10–4 m = 5 ×10–4 × 103 mm I
= 0.375
= 5 ×10–1 mm = 0.5 mm. I0
OPTICS 591

I
× 100 = 0.375 × 100 = 37.5 I0 FG IJ
3
5

I0
I =
2
×
4H K
1061. Half the intensity is transmitted. 243 I0
1063. µ = tan i I =
2048
sin i I0
µ = 1071. I= cos2 θ cos2 (90 – θ)
cos i 2
sin i 32
Also, µ= or 3 = cos2 θ sin 2 θ
sin r 2
FG π − iIJ 3
∴ sin r = cos i or sin r = sin H2 K or
16
= cos2 θ sin2 θ

π π 3
or r= − i or i + r = = 90° or = 4 cos2 θ sin2 θ
2 2 4
sin (90 − θ) 3
90° – θ or = sin2 2θ
1064. 2µ1 = sin θ 4
medium 2
2µ1 = cot θ 3
1
medium 1 or sin 2θ = or 2θ = 60°
2
2µ1 =
θ θ
sin c or θ = 30°
∴ c = sin–1 (tan θ). I0
1072. I= cos2 30° × cos2 30° × cos2 30°
Fig. 231 2
I0 FG IJ
3
3
1065. Amplitude decreases by a factor of cos θ i.e., by a factor
3 9
I =
2
×
4H K
of . So, intensity decreases by a factor . 27 I0
4 16 I =
128
1066. µ = tan θ
I = 0.21 I0
c v
= tan θ or cot θ = λ
v c 1074. t =
FG v IJ 4(µ′ − µ)
or θ = cot–1 H cK t=
5893 × 10 −10
= 1.61 × 10–5 m.
1067. tan i = 1.55 or i = tan–1 (1.55) = 57.17° 4(1.5533 − 1.5442)
1075. µ = tan 60°
sin 57.17° 0.84
= 1.55 or = 1.55
sin r sin r µ = 3
0.84 ∆ν v v
or sin r = = 0.5419 1076. = or ∆ν = ν
1.55 ν c c
or r = sin–1 (0.5419) = 32.81° 0.2c
= × 4 × 107 Hz = 0.8 ×107 Hz
= 32° 48.6′ ≈ 32° 49′. c
I0 ∴ ν′ = ν – ∆ν = 4 × 107 – 0.8 × 107
1068. In (1), θ = 60°, I = I0 cos2 60° = .
4 = 3.2 × 107 Hz.
In (2), θ = 75° – 15° = 60°, ∆λ v
1077. =
I λ c
I = I0 cos2 60° = 0 . In (3), θ = 60°,
4 ∆λ 6 × 107
= = 0.2
I0 I λ 3 × 108
I= . In (4), θ = 60°, I = 0 .
4 4 or ∆λ = 0.2 × 4600 Å = 920 Å
λ′ = 4600 Å + 920 Å = 5520 Å
1070. I=
I0
2
ecos2 30° j 5
v 0.8c
1078. ∆λ = λ = 5500 × = 4400 Å.
c c
592 COMPREHENSIVE OBJECTIVE PHYSICS

v 10 6 1 ∆ν
1079. ∆λ = λ = × 5700 Å = 19 Å. 1086. va = × ×c
c 3 × 10 8 2 ν
∆λ 0.05 1 3 × 103
1080. v = c = × 3 × 108 m s–1 = 1.5 × 105 m s–1. = × × 3 × 10 8 ms–1 = 50 m s–1
λ 100 2 9 × 109
1081. Apparent frequency increases. Apparent wavelength
decreases. 6000
1087. d = n × 6000 = (n + 1)
1.0003
1.5 × 10 6
1082. ∆λ = × 5000 Å = 25 Å. n+1 1
3 × 108 or = 1.0003 or = 0.0003
n n
1083. Firstly, the car is the source and at the cliff, one ob-
6000 6000 × 10−10
serves f. Now, d= Å = × 103 mm
Secondly, the cliff is now source. Its 0.0003 3 × 10−4
emitted frequency is f ′ and ob- Source = 2 mm.
server is now the driver who ob- 1088. θ 9.9 ∵ 1 dm = 10 cm
c= =
serves f ′. ls 20
× 66
As source and observer are moving Observer 10
in same direction, = 0.075 g cm–3 = 75 g litre–1.
v + v0 Fig. 232 4πnd cθ
∴ f′= f 1090. c = or n =
v − vs θ 4 πd
v + v0 3 × 10 8 × 48 π × 10 −3
or 2f = f or n= × 60
v − vs 4 π × 6000
or 2v – 2v0 = v + v0 [As vs = v0] = 36000 rotations/min.
v θ
or v0 = 1091. 2= or θ = 80°
3 1 × 40°
1084. Since wavelength of blue light is less than 5500 Å
c 3 × 108
therefore apparent frequency increases. So, the star 1092. vg = = m s–1 = 2 × 108 m s–1
is moving towards the earth. µ 3/2
0.4 x 4 × 10−3
1085. vs = × 3 × 108 m s–1 = 1.2 × 106 m s–1 t= = s = 2 × 10–11 s.
100 vg 2 × 108

KNOWLEDGE PLUS
In each question, a statement of assertion (A) is given and a corresponding statement of reason (R) is given just
below it. Of the statements, mark the correct answer as :
(A) If both A and R are true and R is the correct explanation of A.
(B) If both A and R are true but R is not the correct explanation of A.
(C) If A is true but R is false.
(D) If both A and R are false.
(E) If A is false but R is true.
1. Assertion (A). Plane polarised light is passing through a rotating polaroid. On viewing through the polaroid, we
find that the intensity of light is twice maximum and twice zero in one complete rotation.
Reason (R). In Huygens’ wave theory, the locus of all points in the same state of vibration is called a wavefront.
[HPMT 1997]
2. Assertion (A). Optically active substances are those substances which rotate the plane of polarisation of polarised
light.
Reason (R). Intensity of light changes when the light is polarised.
3. Assertion (A). If light is polarised by reflection, then the angle between reflected and refracted rays is 180°.
Reason (R). Brewster’s law : µ = tan ip.
Key 1. (B) 2. (B) 3. (E).
OPTICS 593

MCQs
SET III

with
More than one correct alternative

Average time allowed per question is 50 seconds.

1104. When a ray of light enters a denser medium, (a) I (θ) = I0 for θ = 0° (b) I (θ) = I0/2 for θ = 30°
(c) I (θ) = I0/4 for θ = 90°
(a) its frequency increases
(d) I (θ) is constant for all values of θ.
(b) its wavelength decreases 1108. Huygens’ principle of secondary wavelets may be used
(c) its velocity decreases to
(d) it may suffer a total internal reflection. (a) find the velocity of light in vacuum
[National Standard Exam. in Physics 1999] (b) explain the particle behaviour of light
(c) find the new position of a wavefront
1105. If polaroids are to be used to avoid glares of oncom-
(d)explain Snell’s law.
ing light, then
1109. The focal length of the objective of a compound
(a) visibility will decrease microscope is f0 and its distance from the eye-piece is
(b) transmittivity of windshield will decrease L. The object is placed at a distance u from the
(c) vehicles will move slowly objective. For proper working of the instrument
(a) L < u (b) L > u
(d) cost will increase.
(c) f0 < L < 2f0 (d) L > 2f0.
[National Standard Exam. in Physics 1999]
1110. Which of the following is correct ?
1106. White light is used to illuminate the two slits in a (a) The number of images observable between two parallel
Young’s double slit experiment. The separation plane mirrors is infinite.
between the slits is b and the screen is at a distance (b) Field of view is more for convex mirror as compared to
d (≥ b) from the slits. At a point on the screen directly plane mirror and concave mirror.
in front of one of the slits, certain wavelengths are (c) A diminished virtual image cannot be produced by plane
missing. Some of these missing wavelengths are mirror or concave mirror.
(a) λ = b2/d (b) λ = b2/3d (d) A diminished virtual image can be obtained by a convex
mirror.
(c) λ = 2b2/d (d) λ = 2b2/3d. [IIT 1984]
1111. Two convex lenses of focal lengths 10 cm and 20 cm
1107. In an interference arrangement respectively placed coaxially are separated by some
similar to Young’s double slit S1 distance d. The whole system behaves like a concave
experiment, the slits S1 and S2 d/2 lens. The possible value (s) of d is/are
are illuminated with coherent θ (a) 25 cm (b) 20 cm
microwave sources, each of fre- (c) 40 cm (d) 45 cm.
quency 106 Hz. The sources are d/2 1112. A convex lens forms a real image of a point object
synchronised to have zero phase S2 placed on its principal axis. If the upper half of the
difference. The slits are sepa- lens is covered, then
rated by distance d = 150 m. The Fig. 233
(a) the image will be shifted downward
intensity I (θ) is measured as a function of θ, where θ (b) the image will be shifted upward
is defined as shown. If I0 is maximum intensity, then (c) the image will not be shifted
I (θ) for 0 ≤ θ ≤ 90° is not given by (d) the intensity of the image will decrease.
594 COMPREHENSIVE OBJECTIVE PHYSICS

1113. In an experiment with a lens, the object distance u object 20 cm high is located at a distance of 4 m from
versus image distance v data were obtained. Which the convex lens. Which of the following is correct ?
of the following graphs will be linear ? (a) The real inverted image of height 3.8 cm is formed behind
the concave lens.
(a) 1/v versus 1/u (b) uv versus (v – u)
(b) The distance of the image from the convex lens is nearly
(c) v/u versus v (d) v versus u. 63 cm.
[National Standard Exam. in Physics 1990] (c) The distance of the image from the convex lens is 17 cm.
1114. Which of the following quantities increases when (d) The virtual and erect image of height 7.5. cm is formed in
front of concave lens at a distance of 37 cm from the con-
wavelength is increased ? Consider only the
vex lens.
magnitudes.
1119. How much water should be filled in a container, 21
(a) The power of a converging lens
cm in height, so that it does not appear to be half-
(b) The focal length of a converging lens
filled when viewed from the top of the container ?
(c) The power of a diverging lens Given : refractive index of water = 4/3.
(d) The focal length of a diverging lens. (a) 18.0 cm (b) 10.5 cm
1115. A narrow beam of white light goes through a slab (c) 12.0 cm (d) 14.0 cm.
having parallel faces. 1120. A liquid of refractive index
(a) The light never splits in different colours. 1.62 is placed between two
(b) The emergent beam is white.
plano-convex identical
lenses, the medium of
(c) The light inside the slab is split into different colours.
which has refractive index
(d) The light inside the slab is white. 1.54. Two possible arrange-
1116. A convex lens forms a real image, on a screen, of an ments P and Q are shown. P Q
object, the magnification being 0.5. The object and Which of the following is in-
Fig. 235
correct ?
the image distances are so changed that a real im-
(a) The system divergent in P and convergent in Q.
age equal in size to the object is formed. If the dis-
(b) The system is convergent in P and divergent in Q.
placement of object is 20 cm, then
(c) The system is convergent in both.
(a) the focal length of the lens is 20 cm
(d) The system is divergent in both.
(b) the initial distance of the object from the lens is 60 cm
1121. An optical instrument used for angular magnifica-
(c) the initial distance of the image from the lens is 30 cm tion has a 25 D objective and a 20 D eye-piece. The
(d) the final distance of the image from the lens is 25 cm. tube length is 25 cm when the eye is least strained.
1117. When light incident in a medium at an angle i is Which of the following is correct ?
refracted into a second medium at an angle r, the (a) The instrument is a telescope.
graph of sin i vs sin r is as shown in Fig. 234. Which (b) The instrument is a microscope.
of the following is correct ? (c) The angular magnification produced is 10.
(a) The velocity of light in sec- (d) The angular magnification produced is 20.
1122. The brightness-producing capacity of a source
ond medium is 3 times
(a) does not depend on its power
the velocity of light in the
(b) does not depend on the wevelength emitted
Sin r

first medium.
(b) The velocity of light in the (c) depends on its power
first medium is 3 times (d) depends on the wavelength emitted.
the velocity of light in sec- 1123. A Young’s double slit experiment is performed with
30°
ond medium. O Sin i white light.
(c) The critical angle is given (a) The central fringe will be white.
Fig. 234
1 (b) There will not be a completely dark fringe.
by sin i c =
3 (c) The fringe next to the central will be red.
(d) The fringe next to the central will be violet.
1
(d) The critical angle is given by sin ic = . 1124. When we see an object, the image formed on the
2
retina is
1118. A convex lens of power 2 D and a concave lens of
(a) real (b) virtual
power 1.5 D are spaced apart coaxially by 40 cm. An
(c) erect (d) inverted.
OPTICS 595
1125. An object is placed at the principal focus of a conver- (a) frequency of light remains constant.
gent lens and moves away from the lens slowly. Its (b) velocity of light increases by 1.5 times.
distance as measured from the principal focus is x.
(c) frequency of light increases by 1.5 times.
The image distance is measured from the second fo-
cus of the lens, and called x′. In the graph (Fig. 236) (d) wavelength (λ) of light decreases by 1.5 times.
the ordinate could mean : 1130. A planet is observed by an astronomical refracting
telescope having an objective of focal length 16 m
and an eye-piece of focal length 2 cm. Which of the
following is correct ?
A B C
(a) The distance between the objective and the eye-piece is
16.02 m
(b) The angular magnification of the planet is–800.
O x O x O x (c) The image of the planet is inverted
(d) The objective is larger than the eye-piece. [IIT 1992]
1131. Which of the following form (s) a virtual and erect
E
image for all positions of the object ?
D
(a) convex lens (b) concave lens
(c) convex mirror (d) concave mirror.[IIT 1996]
O x O x 1132. Two lenses, one concave and the other convex of same
power, are placed such that their principal axes coin-
Fig. 236 cide. If the separation between the lenses is x, then
(a) x′ for curve B (b) xx′ for curve E (a) real image is formed for x = 0 only
(c) magnification for curve D (b) real image is formed for all values of x
(d) reciprocal magnification for curve A. (c) system will behave like a glass plate for x = 0
[National Standard Exam. in Physics 1996] (d) virtual image is formed for all values of x other than zero.
1126. In which of the following the final image is erect ? 1133. Which of the following is correct ?
(a) Simple microscope (b) Compound microscope (a) Interference of light is evidence for transverse wave char-
(c) Astronomical telescope (d) Galilean telescope. acter of light
1127. A light wave can travel (b) Sunlight reflected from calm water surface at a slant an-
gle would show partial polarisation
(a) in vacuum (b) in vaccum only
(c) Rainbow appears because of scattering of light
(c) in a material medium (d) in a material medium only.
(d) The scheme of a source of light S and two narrow and
1128. Mark the correct options.
near slits A, B shown below is not adequate to obtain
(a) The luminous efficiency of a monochromatic source is al- interference on the right side of A, B.
ways greater than that of a white light source of same
power.
(b) The luminous efficiency of a monochromatic source of
wavelength 555 nm is always greater than that of a white
light source of same power. A
(c) The illuminating power of a monochromatic source of
S
wavelength 555 nm is always greater than that of a white B
light source of same power.
(d) The illuminating power of a monochromatic source is al-
ways greater than that of a white light source of same
power. Fig. 237
1129. When a beam of light with wavelength, λ = 6000 Å, [National Standard Exam. in Physics 1992]
travelling in air, enters a glass medium whose 1134. A ray of light passes from a denser to a rarer medium.
refractive index is 1.5, then At the surface of separation the angle of incidence is
596 COMPREHENSIVE OBJECTIVE PHYSICS

i and the angle between reflected and refracted rays


is of 90°. If the angles of reflection and refraction are
r and r′ respectively, then the value of the critical P Q
angle is
(a) sin–1 (tan r′ ) (b) sin–1 (tan r)
(c) sin–1 (tan i) (d) tan–1 (sin i) [IIT 1994]
1135. A ray of light travelling in a transparent medium
falls on a surface separating the medium from air, at R
an angle of incidence of 45°. The ray undergoes total
internal reflection. If n is the refractive index of the
Fig. 238
medium with respect to air, select the possible value (s) of
n from the following. [National Standard Exam. in Physics 1991]
(a) 1.3 (b) 1.4 1140. In the Young’s Experiment regarding intereference,
(c) 1.5 (d) 1.6 [IIT 1998] one of the slit is covered by a thin film that absorbs
50% of the incident light. Then
1136. Polarised light
(a) the bandwidth in the interference pattern increases
(a) is a different form of light
(b) the contrast between the bright and dark bands increases
(b) can show interference pattern
(c) the ratio of brightness of a bright band to that of a dark
(c) has direction of vibration restricted in some way band is about 36 : 1
(d) travels in any medium with a velocity slightly more than (d) there will be a shift in the position of the central bright
that for unpolarised light. band. [National Standard Exam. in
Physics 1997]
1137. A plane mirror reflecting a ray of incident light is
rotated through an angle θ about an axis through 1141. Identify the correct statement (s).
the point of incidence in the plane of the mirror per- (a) If the incident rays are converging, we have a real object.
pendicular to the plane of incidence. Then (b) If the final rays are converging, we have a real image.
(a) the reflected ray does not rotate. (c) If the incident rays are diverging, we have a real object.
(b) the reflected ray rotates through angle θ. (d) If the final rays are diverging, we have a real image.

(c) the reflected ray rotates through an angle 2θ. 1142. In case of spherical mirrors, indicate the quantities,
which are independent of the paraxial ray assumption.
(d) the incident ray is fixed.
(a) Pole (b) Focus
1138. Which of the following is correct regarding a convex
(c) Radius of curvature (d) Principal axis.
lens ?
1143. If white light is used in a Young’s double-slit experi-
(a) The object or its image which is closer to the lens is always
ment,
smaller.
(a) bright white fringes are formed at the centre of the screen
(b) The real object and its real image always lie on different
(b) fringes of different colours are observed clearly only in
sides of the lens.
the first order
(c) It never forms real image of a virtual object. (c) the first-order violet fringes are closer to the centre of the
(d) The object and its real image cannot come closer to 4f. screen than the first-order red fringes

1139. Interference fringes with light of wavelength λ fall- (d) the first-order red fringes are closer to the centre of the
screen than the first-order violet fringes.
ing along near normal to an air film are observed to
be as shown in Fig. 238. One of the faces of the film is 1144. In a single slit Fraunhofer diffraction experiment
setup,
known to be plane. From the fringes, one may con-
(a) if the distance between the screen and the slit is increased,
clude that
the angular width of the central maximum does not change
(a) the other face is also a plane face.
(b) if the wavelength is reduced, the angular width of maxima
(b) the other face is a cylindrical one. increases
(c) the air thickness difference between points P and Q can (c) if the wavelength is reduced, the minima shift toward the
only be 2λ. central maxima
(d) the air thickness difference between points Q and R is λ/2. (d) at the position of minima, the waves from the two ends of
the slit interfere constructively.
OPTICS 597
1145. Which of the following quantities related to a lens (a) Fig. 239 (I) shows deviation without dispersion
depend on the wavelength or wavelengths of the inci- (b) Fig. 239 (II) is for showing dispersion without deviation
dent light ? (c) In Fig. 239 (I), prism P is of flint glass and Q of crown glass
(a) Power (b) Focal length (if these two are the only options)
(c) Chromatic aberration (d) Radii of curvature. (d) In Fig. 239 (III) a transverse screen at P would show violet
1146. A diminished image of an object is to be obtained on at centre, red outside.
a screen 1.0 m from it. This can be achieved by ap- [National Standard Exam. in Physics 1992]
proximately placing
1149. P1 and P2 are identical prisms arranged as shown in
(a) a convex mirror of suitable focal length
Fig. 240. A ray of white light incident on one face of
(b) a concave mirror of suitable focal length
P1 undergoes dispersion and falls on one face of P2.
(c) a convex lens of focal length less than 0.25 m
Then
(d) a concave lens of suitable focal length. [IIT 1995]
1147. By properly combining two prisms made of different
materials, it is possible to Red
(a) have dispersion without average deviation
White
(b) have deviation without dispersion
Violet P2
(c) have both dispersion and average deviation
(d) have neither dispersion nor average deviation.
P1
1148. Which of the following is correct ?

Red Fig. 240


Q
White
P (a) light emerging from P2 will be white
Violet
(b) in the light emerging from P2, dispersion will be greater.
(I)
(c) the direction of light emerging from P2 will be parallel to
Red
White Q the direction of ray incident on P1
P Violet (d) the ray emerging from P2 will be white even if prisms P1
and P2 have identical geometry but different materials.
(II)
[National Standard Exam. in Physics 2000]
White

P Q
(III)

Fig. 239

Answers (Set III)


1104. (b), (c) 1105. (a), (b), (d) 1106. (a), (b) 1107. (a), (b), (c), (d) 1108. (c), (d)
1109. (b), (d) 1110. (a), (b), (c), (d) 1111. (c), (d) 1112. (c), (d) 1113. (a), (b), (c)
1114. (b), (d) 1115. (b), (c) 1116. (a), (b), (c) 1117. (b), (c) 1118. (a), (b)
1119. (a), (b), (d) 1120. (b), (c), (d) 1121. (b), (d) 1122. (c), (d) 1123. (a), (b), (d)
1124. (a), (d) 1125. (b), (c), (d) 1126. (a), (d) 1127. (a), (c) 1128. (b), (c)
1129. (a), (d) 1130. (a), (b), (c), (d) 1131. (b), (c) 1132. (b), (c) 1133. (b), (d)
1134. (b), (c) 1135. (c), (d) 1136. (b), (c) 1137. (c), (d) 1138. (a), (b), (d)
1139. (b), (d) 1140. (c), (d) 1141. (b), (c) 1142. (a), (c), (d) 1143. (a), (b), (c)
1144. (a), (c), (d) 1145. (a), (b), (c) 1146. (b), (c) 1147. (a), (b), (c) 1148. (a), (b), (d)
1149. (a), (c).
598 COMPREHENSIVE OBJECTIVE PHYSICS

Solutions (Set III)

1104. When light enters a denser medium, both the wave- φ


length and velocity are reduced by a factor of µ. For θ = 90°, I = I0 cos2
2
1106. Path difference s
d

1
But φ= d
= d2 + b2 – d λ
b
= (d2 + b2)1/2 – d 2
d +b
2 πd
∴ I = I0 cos2
F bI
s2
2 1/2 λ
GH d JK – d
= d 1+ 2 Fig. 241 π × 150
or I = I0 cos2
LMF b I OP
2
1/2 3 × 108 / 106

= d MG 1 + d J
HN K − 1PQ
2
or I = I0 cos2
π
2
or I = 0

L b − 1OP = b
= d M1 +
2 2 For θ = 0°, I = I0
MN 2d PQ 2d
2 Again, intensity is dependent on angle θ.

b2 λ 1 1 1 d 1 3 d
For darkness, = (2n – 1) 1111. = + − or = −
2d 2 F 10 20 200 F 20 200
For F to be negative,
b2 b2
or (2n – 1) λ = or λ = d 3
d (2 n − 1) d
> or d > 30 cm.
2 200 20
b
For n = 1, λ = 1 v
d 1116. – =
2 2 u
b
For n = 2, λ = . u
3d or u = – 2 v or v = –
φ 2
1107. I = I0 cos2 1 1 1 −2 1 1
2 Now, − − = or − =
2π yd 2π d D tan θ u u f u u f
Here, φ= or φ = −
λ D λ D 2
2π 3 1
or φ= d tan θ or – = or u = – 3f
λ u f
π d tan θ
∴ I = I0 cos2 In the second case, magnification is 1. Clearly, the
λ
y object distance is ‘– 2f ′.
c
But d = 150 m and λ = Clearly, the displacement of the object is f.
ν θ
D ∴ f = 20 cm so (a) is OK.
3 × 108
or λ = m Fig. 242 Now, initial distance of object from the lens
106
= – 60 cm
= 300 m
So, (b) is OK
π × 150 × tan θ
∴ I = I0 cos2 Initial distance of image from the lens
300
π tan θ u − 60
or I = I0 cos2 =– =– cm = 30 cm
2 2 2
π So, (c) is OK
For θ = 30°, I = I0 cos2 tan 30°
2 Final distance of image from the lens = 2f
π = 2 × 20 cm = 40 cm
= I0 cos2 = I0 cos2 52° = 0.4 I0
2 3 So, (d) is NOT OK.
OPTICS 599

sin r 1 FG 300 + 40IJ cm = 300 + 520 cm


1117. tan 30° =
sin i
or tan 30° =
sin i / sin r
= H 13 K 13
1 1 820
or = or µ =
1 2 3 = cm = 63.08 cm.
3 µ 13
v1 3h
= 3 ∴ v1 = 3 v2 1119. Clearly, 21 – h =
v2 4 21 – h

So, velocity of light in first medium is 3 times the 3h 3h


or 21 = h + 4
velocity of light in second medium. 4 h
1 1 7h
Again, sin ic = = . or 21 =
µ 3 4 Fig. 243
1118. f = 50 cm or h = 12 cm.
1 1 1 So, 12 cm water column is to be ruled out.
− =
v u f 1120. P α (µ – 1)
1 1 1 µ for liquid greater than µ for material of lens.
− =
v − 400 50 So, the concave liquid lens dominates in arrangement
1 1 1 P. So, P behaves as a divergent system.
= – In arrangement Q, the liquid lens is plano-concave.
v 50 400
So, its power is half of the power of the liquid concave
1 7 400
= or v = cm lens in arrangement P. Now, the power of the convex
v 400 7 lens dominates. So, the system is convergent in Q.
400 1
Again, m = =– 100
7 (− 400) 7 1121. fo = cm = 4 cm
25
I1 1 20 100
=– or I1 = – cm
20 7 7 fe = cm = 5 cm
20
The image formed by the convex lens would act as an The focal lengths of the objective and eye-piece are
object for the concave lens. Distance of this image small. Moreover, the focal length of the eye-piece is
FG 400 − 40IJ cm greater than the focal length of the objective.
from the concave lens = H7 K So, the given instrument is a compound microscope.
400 − 280 120 So, (b) is OK
= cm = cm Again, the eye is least strained when the image is at
7 7
infinity. Clearly, the image formed by the objective is
1 1 1 at the principal focus of the eye-piece i.e. at a distance
Again, − =
v u f of 5 cm from the eye-piece. The distance from the ob-
1 1× 7 1.5 −3 jective is 20 cm.
15
– =– =– =
v 120 100 1000 200 1 1 1 1 1 1
Now, – = or – = –
1 3 7 − 9 + 35 20 u0 4 u0 4 20
=– + =
v 200 120 600
1 1
600 300 or – = or u0 = – 5 cm
or v= cm = cm. u0 5
26 13
300/13 v0 20
I′ Mo = = =–4
Now, = u0 −5
20 120/7

7 D 25
20 300 7 Me = = =5
× fe 5
or I′ = – × cm = – 3.8 cm.
7 13 120 M = M0 × Me = – 4 × 5 = – 20
The negative sign indicates that the image is real.
So, (d) is OK.
Distance of image from convex lens
600 COMPREHENSIVE OBJECTIVE PHYSICS

1124. Note that the image formed on the retina is inverted 1 1 1 x


and highly diminished but the brain interprets the 1132. = − –
F f1 f2 f1 (− f2 )
signals in such a way that we see an erect and actual
size image. 1 x 1 x f12
or = or = 2 or F =
1125. Newton’s formula F f1 f2 F f1 x
f 2 = xx′ F > 0 for all values of x.
f2 So, real image is formed for all values of x. Thus, (b)
x′ =
x is correct.
So, (a) is incorrect and (b) is correct. Again, for x = 0, F = ∞
Again, magnification decreases non-linearly with in- So, the system behaves like a glass plate.
crease in x. So, (c) is correct. Also, (d) is correct.
Thus, (c) is correct.
[Curves A and D are schematic.]
1130. L = fo + fe 1133. (a) No ; Interference occurs for longitudinal waves too
So, (a) is correct (b) Yes ; it contains more of horizontal E vibrations
f0 (c) No ; the origin is total internal reflection + mini-
Again, M=– mum deviation condition.
fe
So, (b) is correct (d) true, it is not adequate since A, B are not coherent.
Image formed by an astronomical telescope is inverted sin i
1
So, (c) is correct. 1134. =
µ sin r′
Objective has to be of larger aperture so that light-
gathering power is good. r′ + r = 90°
r′ + i = 90°
1131. or r′ = 90° – i
1 sin i
=
µ sin (90° − i)

sin i Fig. 246


= = tan i
cos i
O I F
1
Again, = sin ic
µ
∴ sin ic = tan i or ic = sin–1 (tan i)
Also ic = sin–1 (tan r)
Fig. 244 choice (b) Note that the angle of incidence is equal to angle of
reflection.
1135. µ1 sin θ1 = µ2 sin θ2
µ1
µ1 sin 45° = 1 × sin θ2 or sin θ2 =
2
For total internal reflection, θ2 ≥ 90°
or sin θ2 ≥ sin 90°
O I F C
µ1
or sin θ2 ≥ 1 or ≥ 1 or µ1 ≥ 2
2
Clearly, (c) and (d) are the possible right choices.

Fig. 245 choice (c)


OPTICS 601
1139. (a) Fringes not equidistant, statement is False 1148. (a) yes. R, V parallel (no dispersion), but turned clock-
(b) For plane and cylindrical faces pair, the fringes wise w.r.t. W
should contract as we go out. This is OK. True (b) yes. Mean (of V, R) parallel to W, but finite spread
between V, R
λ
(c) Each step means ∆t = in magnitude. But 4 steps (c) no. Deviation shown is favouring P, which must
2
be of crown glass (For equal dispersion, flint gives
do not necessarily mean ∆t = 2λ. It could be zero to
more deviation).
2λ (max). False
(d) true ; f is shorter for violet.
λ 1149. Since two prisms are identical and parallel therefore
(d) One step of . True
2 the dispersion due to P1 is cancelled by P2. The light
1140. Bandwidth is independent of intensity emerging from P2 will be white and the emergent ray
Contrast is less when amplitudes are different. Ratio will be parallel to the incident ray.
(1.4 + 1)2
of amplitudes 1 : 2 . Intensity ratio is =
(1.4 − 1)2
36. Central fringe will shift due to additional optical
path.

KNOWLEDGE PLUS
l A ray of light is incident on an equilateral glass prism placed on a horizontal table. For minimum deviation,
which of the following in true ?
(a) PQ is horizontal (b) QR is horizontal
(c) RS is horizontal Q R

(d) Either PQ or RS is horizontal. [IIT Screening 2004]


S
Sol. In the minimum deviation position, the refracted ray is parallel to the base of the P
prism. Fig. 247
So, (b) is the right choice.
l In a vessel of depth 15 cm, liquid is poured till the liquid level appears to be at half the depth, the liquid level
is 5 cm from the top. Calculate the refractive index of the liquid.
(a) 1.33 (b) 3.0 (c) 1.5 (d) 1.7 (e) 2 [Kerala PMT 2003]
Real depth 10 20 4
Sol. µ= = = = = 1.33
Apparent depth 15 / 2 15 3
So, (a) is the right choice.
602 COMPREHENSIVE OBJECTIVE PHYSICS

SET IV MCQs
based on
TYPICAL NUMERICAL BANK
(Exclusively for Engineering Entrance Tests)

Average time allowed per question is 60 seconds.

1150. A 60 watt bulb is hang over the centre of a table 4′ × 4′ (a) 16 s (b) 36 s
at a height of 3′. The ratio of the intensities of illumi- (c) 46 s (d) 24 s. [DCE 2003]
nation at a point on the centre of the edge and the 1154. An isotopic point source of light is suspended h me-
corner of the table is tre vertically above the centre of a circular table of
17 2 radius r metre. Then the ratio of illuminance at the
(a) (b) centre to that at the edge of the table is
13 1
F r2 I F h2 I
F 17 I
(c) GH JK × 60
F 17 I 3 / 2 .
(d) G J
GH h2 JK
(a) 1 + GH
(b) 1 +
r2
JK
13 H 13 K
F r2 I 3/ 2
(c) G 1 +
F h2 I 3 / 2 .
H h2 JK
(d) G 1 +
H r 2 JK
1151. Fig. 248 shows a lamp of luminous intensity 72
candela hanging at a height of 1.20 m over a table
and at a distance of
1155. In Question Number 1154 for the illuminance at the
0.80 m from a wall.
On the wall, there edge of the table to be maximum, the height h from
hangs a small mirror
0.8 m the table is
1.20 m
so that the distance of r
(a) 2r (b)
the mirror from the 1m 2
centre of the lamp is r
1 m. The approximate (c) (d) 2 (r).
2
illuminance on the 1156. What path the ray of Q
table under the lamp light PQ would follow
will be Fig. 248
after getting reflected
(a) 30 lx (b) 40 lx from the second mir-
(c) 50 lx (d) 61 lx. ror ?
1152. In the above question how will be illuminace change (a) Parallel to PQ
(b) Perpendicular to PQ
if the mirror is removed ? P
(c) Parallel to first mirror
(a) It will decrease by 31.5 lx (b) It will increase by 31.5 lx
(d) Data is inadequate. Fig. 249
(c) It will decrease by 10.8 lx (d) It will decrease by 2.1 lx.
1153. The exposure time required for photographing an 1157. Two mirrors, each 1.6 m long, are facing each other.
object illuminated by a 100 W bulb at a distance The distance between the mirrors is 20 cm. A light
of 1 m is 8 s. What is the exposure time if two 100 W ray is incident on one end of one of the mirrors at an
bulbs at distances of 3 m and 4 m from the object angle of incidence of 30°. How many times is the ray
being illuminated are used if the total luminous reflected before it reaches the other end ?
energy flux over photographic plate remains (a) 1 (b) 3
unchanged ? (c) 7 (d) 14.
OPTICS 603

1158. Refer to Fig. 250. The angle from the object, then the focal length of the convex
at which the ray is incident 60° mirror is
on the second mirror is (a) 10 cm (b) 15 cm
(a) 10° (c) 20 cm (d) 40 cm.
50°
(b) 20° 1165. An illuminated object is placed at a distance D from
(c) 30° a screen. When a convex lens is placed at position A,
(d) 40°. a sharp magnified image is formed on the screen.
Fig. 250 When the lens is shifted by a distance d to another
1159. In the previous question, what is the angle at which position B, a sharp but diminished image is formed
the ray is incident on the first mirror after being on the screen. The ratio between the sizes of the two
reflected from the second mirror ? images will be
(a) 30° (b) 0° D2 (D + d ) 2
(a) (b)
(c) 70° (d) infinity. d2 (D − d ) 2
1160. A point object is placed at a distance of 500 mm from
D FG D + d IJ .
a concave mirror length 200 mm. If the object is moved
towards the mirror by 100 mm, the image moves by
(c)
d
(d)
H D − dK
a distance of 1166. For a medium in the form
(a) 67 mm towards mirror of sphere, rays starting
(b) 67 mm away from the mirror from one end of a diameter
in a small cone emerge
(c) 10 mm towards mirror
from the opposite surface as
(d) 10 mm away from the mirror. a parallel beam. The refrac-
1161. A candle flame 3 cm high is placed at distance of 3 m tive index of material of the
from a wall. How far from the wall must a concave sphere is Fig. 251
mirror be placed in order that it may form an image (a) 3/2 (b) 2/3
of the flame 9 cm high on the wall ?
(c) 1/2 (d) 2/1.
(a) 225 cm (b) 300 cm
[National Standard Exam. in Physics 1990]
(c) 450 cm (d) 150 cm.
1162. A concave mirror of radius of curvature 60 cm is 1167. A ray of light is incident on a convex lens of focal
placed at the bottom of a tank containing water upto length 80 cm. The incident light is parallel to the
a height of 20 cm. The mirror faces upwards with its principal axis and is at a height of 10 cm above the
axis vertical. Solar light falls normally on the sur- principal axis. The deviation produced by the lens is
face of water and the image of the sun is formed. If a ...... Given : lens is very thin.
4 1
µw = , then with the observer in air, the distance of (a) (b) 8
3 8
the image from the surface of water is
(c) 80 × 10 (d) 45°
(a) 30 cm (b) 10 cm
1168. A double convex lens has a focal length of 25 cm in
(c) 7.5 cm above (d) 7.5 cm below.
air. When it is dipped into a liquid of refractive index
1163. A concave mirror forms on a screen a real image of
4
twice the linear dimensions of the object. Object and , its focal length increases by 75 cm. The refractive
3
screen are then moved until the image is three times
index of the material of the lens is
the size of the object. If the shift of the screen is 25 cm,
then the focal length of the mirror is 4 3
(a) (b)
(a) 5 cm (b) 16.66 cm 3 4
(c) 25 cm (d) 37.5 cm. 25 × 3 × π × 2
(c) 1.5 (d) .
1164. A luminous object is placed 20 cm from the surface .4 × 75
of a convex mirror and a plane mirror is set so that
1169. Fig. 252 shows a glass prism of refractive index 1.5
the virtual images formed in the two mirrors coin-
cide. If the plane mirror is at a distance of 15 cm immersed in water. A ray of light is shown to be
incident on the face AC. After reflection from AC, it
604 COMPREHENSIVE OBJECTIVE PHYSICS

meets BC normally. Given :


refractive indices of glass and B A α′ B
θ
3 4
water are 2 and 3 respec-
α
tively. Now, sinθ r′ A
(a) = 1
r
(b) = critical angle C
C O
8
(c) > Fig. 252
9
2
(d) < . [IIT 1981]
3
Fig. 253
1170. A small luminous object is placed at a distance of 24 cm
from a convex lens of focal length 15 cm on its prin- µRr
cipal axis. A concave lens of focal length 30 cm is (a)
R−r
(b) µ r
placed coaxially on the other side of the convex lens
at a distance of d cm from it. If a real image is formed (c) µ (R – r) (d) µR.
by the combination at a point 80 cm from the convex 1175. A luminous object and a screen are fixed distance D
lens, the value of d is apart. A converging lens of focal length f capable of
(a) 10 cm (b) 15 cm forming a real image of the object on the screen will
(c) 20 cm (d) 25 cm. do so for two positions of the lens that are separated
1171. The diameter of the face of a plano-convex lens is D. by distance d such that
The maximum thickness of the lens is t. Then the
(a) d = D(D − 4 f ) (b) d = D (D + 4f)
radius of curvature of the convex surface is nearly
(c) d = D(D + f ) (d) d = D(2D + f ) .
D2
(a) D2 (b)
8 1176. The plane face of a plano-convex lens is silvered. The
system would now behave as a
D2 D2 (a) concave mirror (b) convex mirror
(c) (d) .
8t 2t
(c) concavo-convex mirror (d) convexo-concave mirror.
1172. The diameter of a plano-convex lens is 6 cm and thick- 1177. In the previous question, the radius of the mirror is
ness at the centre is 3 mm. If the speed of light in the [Given : r = radius of curvature of the spherical sur-
material of the lens is 2 × 108 m s–1, then the focal face].
length of the lens is
(a) r µ (b) r (µ – 1)
(a) 10 cm (b) 15 cm
r r
(c) 20 cm (d) 30 cm. (c) (d)
µ−1 µ
1173. Viewed normally through the flat surface, the greatest 1178. Two thin convex y
thickness of a plano-convex lens appears to be 2 cm lenses of focal
and when viewed through its curved surface, it lengths f1 and f2
20 are separated by
appears to be cm. The actual thickness of the
9 a horizontal dis- O
x

lens is 3 cm. The refractive index of the material of tance d (where d


the lens is < f1 , d < f2) and
their centres are
27 3
(a) (b) displaced by a d
20 2
vertical separa- Fig. 254
10 20 tion ∆ as shown in the Fig. 254.
(c) (d) .
7 9
Taking the origin of coordinates, O, at the centre of
1174. A hollow sphere of glass of refractive index µ has the first lens, the x and y coordinates of the focal
external radius R and internal radius r. Then appar- point of this lens system, for a parallel beam of rays
ent radius r′ is coming from the left, are given by
OPTICS 605

f1 f2 f1( f2 + d) ∆2
(a) X = ,y=∆ (b) X = ,Y= A D
f1 + f2 f1 + f2 − d f1 + f2

f1 f2 + d( f1 − d) ∆( f1 − d) αmax n1 n2
(c) X = , Y=
f1 + f2 − d f1 + f2 − d
f1 f2 + d( f1 − d) B C
(d) X =
f1 + f2 − d
, Y = 0. [IIT 1993]

1179. A diverging beam of light from a point source S hav- Fig. 256
ing divergence angle α, falls symmetrically on a glass
slab as shown in Fig. 255. The angles of incidence of LM n1 cos F sin−1 n2 I OP LMn cos F sin−1 1 I OP
(a) sin–1
MN n2 GH n1 JK PQ (b) sin MN 1 GH n2 JK PQ
–1
the two extreme rays are equal. If the thickness of
the glass slab is t and the refractive index is n, then
the divergence angle of the emergent beam is F n1 I F n1 I .
S
(c) sin–1 GH n2 JK (d) sin –1
GH n2 JK
[IIT Screening 2000]
α
1182. A concave lens of glass, refractive index 1.5, has both
surfaces of same radius of curvature R. On immersion
in a medium of refractive index 1.75, it will behave
as a
(a) convergent lens of focal length 3.5 R
i i (b) convergent lens of focal length 3.0 R
(c) divergent lens of focal length 3.5 R
(d) divergent lens of focal length 3.0 R.
n t
[IIT Screening 1999]
1183. A long cylindrical tube containing water is closed by
Fig. 255 an equiconvex lens of focal length 10 cm in air. A
(a) zero (b) α point source is placed along the axis of the tube
outside it at a distance of 21 cm from the lens. If the
FG 1 IJ FG 1 IJ . refractive index of the material of the lens is 1.5 and
(c) sin–1 H nK (d) 2 sin–1 H nK 4
that of water is , then the image is formed at what
[IIT Screening 2000] 3
distance inside the tube ?
1180. A film of air is enclosed between a pair of thin
(a) 30 cm (b) 40 cm
microscope slides and the combination is then
(c) 50 cm (d) 70 cm.
inserted in water. A ray of white light is projected
through water and the light reflected by thin film of 1184. µ for the material of a 60° prism is 1.5. Given :
air sandwiched between the two slides is received on 2 3
a screen. If the angle of incidence of the ray on the sin 42° ≈ and sin 49° ≈ . The angle of incidence
3 4
film is gradually decreased from 90°, the reflected for minimum deviation is nearly
light will (a) 30° (b) 49°
(a) turn red and then vanish (c) 38° (d) 28°.
(b) remain white and then vanish
1185. In Q. 1184, the angle of minimum deviation is nearly
(c) remain white at all angles of incidence (a) 30° (b) 49°
(d) turn blue and then vanish. (c) 38° (d) 28°.
1181. A rectangular glass slab ABCD, of refractive index 1186. In Q. 1184, the angle of incidence for maximum
n1, is immersed in water of refractive index n2 (n1 > deviation is nearly
n2 ). A ray of light is incident at the surface AB of the (a) 30° (b) 49°
slab as shown in Fig. 256. The maximum value of
(c) 128° (d) 28°.
the angle of incidence αmax such that the ray comes
out only from the other surface CD is given by 1187. An isosceles prism of angle 120° has a refractive in-
dex of 1.44. Two parallel monochromatic rays enter
606 COMPREHENSIVE OBJECTIVE PHYSICS

the prism parallel to each other in (c) the fringe pattern moved towards X
air as shown in Fig. 257. The rays (d) the fringe pattern moved towards Y
emerging from the opposite faces (e) the separation of the fringes decreased in the region OY
120°
(a) are parallel to each other but was unchanged in the region OX.
(b) are diverging 1190. White light covers the range of wavelengths from 400
(c) make an angle 2(sin–1 (0.72) – 30°) nm to 700 nm. A diffraction grating with 6 × 105
with each other lines per metre is placed at right angles to a ray of
(d) make an angle 2 sin–1 (0.72) with Fig. 257 white light and produces the first and second order
each other. [IIT 1995] spectra shown in Fig. 260. The figure is not drawn to
1188. A glass sphere of radius r = 5 × 10–2 m has a small scale.
bubble 2 × 10–2 m from its centre. The bubble is viewed
along a diameter of the sphere from the side on which
Grating Second
it lies. Refractive index of glass is 1.5. Distance from Red
order
surface at which the bubble will appear is Blue spectrum

Red
β First
White Blue
light α order
spectrum

X
Observer’s
eye
r

Fig. 260
Fig. 258
The angle between the red and blue ends of the spec-
(a) 2.5 cm (b) 5.2 cm
trum is α for the first order spectrum and β for the
(c) – 5.2 cm (d) – 2.5 cm.
second order spectrum. How do α and β compare ?
1189. The diagram below (Fig. 259) illustrates an experi-
(a) α < 21 β (b) α = 2 β
mental arrangement that produces interference
fringes with a double slit. (c) α = 21 β (d) α > 2 β
X (e) α = β.
1191. A narrow beam of monochromatic light falls at nor-
mal incidence on a diffraction grating. Third-order
S1 diffracted beams are formed at angles of 45° to the
original direction. What is the highest order of dif-
O
S2
fracted beam produced by this grating ?
(a) 3 rd (b) 4 th
Monochromatic (c) 5 th (d) 6th.
Screen
source
Y
1192. Plane-polarised radio-waves are transmitted by a
Thin glass plate vertical aerial. The amplitude of the waves is A when
they reach a receiving aerial which is tilted from the
Fig. 259 vertical at an angle θ in the plane perpendicular to
the direction of arrival. The power delivered by the
When slit S2 was covered with a very thin plate of aerial to the receiver is proportional to
glass as shown, (a) A2 cos2 θ (b) A cos θ
(a) the separation of the fringes increased (c) zero (d) A sin θ
(b) the separation of the fringes decreased (e) A2 sin2 θ.
OPTICS 607

Answers (Set IV)


1150. (d) 1151. (d) 1152. (c) 1153. (c) 1154. (c) 1155. (c) 1156. (a) 1157. (d)
1158. (a) 1159. (c) 1160. (b) 1161. (c) 1162. (c) 1163. (c) 1164. (c) 1165. (b)
1166. (d) 1167. (a) 1168. (c) 1169. (c) 1170. (c) 1171. (c) 1172. (d) 1173. (b)
1174. (b) 1175. (a) 1176. (a) 1177. (c) 1178. (c) 1179. (b) 1180. (d) 1181. (a)
1182. (a) 1183. (d) 1184. (b) 1185. (c) 1186. (d) 1187. (c) 1188. (d) 1189. (d)
1190. (a) 1191. (b) 1192. (a).

Solutions (Set IV)

1150. Refer to Synopsis for the formula d LM Ih OP


LM h + l OP
2
2 3/ 2
1155.
N
dh [ r 2 + h2 ]3 / 2 Q =0
r
= M
MM h + l PPP
EC 2 On simplification, h =

EC 2 2
2
N 4Q 1156. Q

LM 9 + 16 OP 3/ 2

= M
L 17 O 3/ 2 i i

16 P = M P
2
MN 9 + 4 PQ N 13 Q
i
72 72 × 1.20
1151. E = + 90° – i
1.20 2
[1.22 + 1.62 ]3/2 1.6 m P
90° – i
86.4
= 50 + i
8
1.20 m

2
= 50 + 10.8 1.6
2 +
= 60.8 ≈ 61 lx 1.2
q
Fig. 261
Fig. 262
1152. When the mirror is removed, the contribution of the
‘‘image’’ will disappear. x
1157. tan 30° = cm
1153. If the two photographic prints have to receive the 20
same luminous energy flux,
1
I1 I2 or x = 20 × cm
3
× t1 = × t2
r12 r2 2
20
100 FG 100 + 100 IJ 100 × 8 Distance covered in one reflection = cm
12
×8= H3 4 K
2 2 t2 or t2 =
100 100
+ Total number of reflections
3

9 16
100 × 8 × 9 × 16 160 3
t2 = s = 46 s. = = 13.856 ≈ 14.
2500 20
608 COMPREHENSIVE OBJECTIVE PHYSICS

1 1 1
x = +
v 500 − 200

1 1 1 2−5
= – =
30° v 500 200 1000
20 cm
1000
v= – cm
3
30° Again, u′ = – 400 mm
f = – 200 mm
v′ = ?
1 1 1
+ =
Fig. 263 v′ u′ f

1158 & 1159. Now, θ + 40° + 60° = 180° 1 1 1 1 1 1


+ = or =– +
θ = 80° v′ − 400 − 200 v′ 200 400
Again, θ + θ′ = 90°
1 −4+2
or θ′ = 90° – 80° = 10° or =–
v′ 800
800
or v′ = – cm = – 400 cm
2
°
30 30° FG 1000IJ
Increase in image distance = (– 400) cm – − H 3 K
cm

θ′ θ B 1000 200
= – 400 = – mm
3 3
40° θ′
A = – 66.67 mm. = – 67 mm.
50°
v
50° 1161. m=–
u
θ″ − ( x + 300) x + 300
–3=– or –3=–
C
i
−x x
or – 3x = – x – 300 or – 2x = – 300
or x = 150 cm
Fig. 264 Distance of wall from the mirror = (150 + 300) cm =
450 cm.
In ∆ABC, θ″ + 140° + 20° = 180° 1162. Focal length of concave
θ″ = 180° – 160° I1
mirror = 30 cm
θ″ = 20° In the absence of mirror, I
Now, i + θ″ = 90° the image would be formed
or i = 90° – 20° = 70°. at a distance of 30 cm
1160. u = – 500 mm from the mirror or at a dis- 30 cm
tance of 10 cm from the
f = – 200 mm
water surface.
v = ?
3 v
1 1 1 Now, =
+ = 4 10
v u f Fig. 265
30
1 1 1 or v= cm
=– + 4
v u f
= 7.5 cm
OPTICS 609

v µ2 µ µ − µ2
1163. m=– + 1 = 1
u −u v R
v µ 1 1− µ
–2=– Substituting values, + =
u − (− 2R) ∞ −R
v µ 1− µ
or v = 2u or u = or = or – µ = 2 – 2µ or µ = 2.
2 2 −1
1 1× 2 1 1 3 h 10 cm 1
= + or = ...(1) 1167. δ= = =
f v v f v f 80 cm 8
Again, – 3 = –
v + 25 1
∝ (µ – 1) ;
1
α FG 3µ − 1IJ
u
1168.
25 100 H4 K
v + 25
or –3=– or v + 25 = 3 u 1169. The incident ray in the prism has also an angle of
u incidence θ. This follows from geometrical construc-
1 1 3
∴ = + µw
f v + 25 v + 25 tion. Clearly, θ > ic or sin θ > sin ic or sin θ > .
µg
1 4
or = ...(2) 1170. For convex lens
f v + 25
From (1) and (2), 1 1 1
− =
v − 24 15
3 4
= 1 1 1 8−5 1
v v + 25 or = – = =
3v + 75 = 4v or v = 75 cm v 15 24 120 40
1 or v = 40 cm
4
From = n(2), f = or f = 25 cm. For concave lens
100
1164. 1 1 1
– =
80 − d 40 − d − 30

40 − d − 80 + d 1
=−
(80 − d)(40 − d) 30

I
− 40 −1
O =
80 × 40 − 120d + d 2 30
2
d – 120d + 3200 = 1200
or d2 – 120d + 2000 = 0
15 cm 5 cm
10 cm or d2 – 100 d – 20d + 2000 = 0
20 cm or d(d – 100) – 20 (d – 100) = 0
Fig. 266
or d = 20 cm or 100 cm
Now, u = – 20 cm, v = 10 cm, f = ? The given data eliminates the possibility of 100 cm
1 1 1 distance.
= +
f v u FG D IJ FG D IJ
1
=
1
+
1
or
1
=
2−1
1171. (2R – t)t = H 2K H 2K
f 10 − 20 f 20 D2
or 2Rt – t2 = 2R – t
or f = 20 cm 4
Neglecting t2,
I1 v I2 u I1 v
2
FG D + d IJ 2
D2
1165. = , = ,
O u O v I2 u
= 2
H D − dK 2Rt =
4
D/2
t
D/2

1166. This problem deals with refraction from denser to rarer D2


medium at a curved surface. or R=
8t Fig. 267
610 COMPREHENSIVE OBJECTIVE PHYSICS

1172. (2R – 0.3) 0.3 = 3 × 3 A parallel beam is incident on lens O. It would have
2R – 0.3 = 30 formed image at its focus at point A. For lens O1, A
2R = 30 + 0.3 acts as a virtual object, at a distance ( f1 – d) from
2R = 30 cm
2R – 0.3 lens O1. The final image is formed at B. For lens O1,
R = 15 cm u = + ( f1 – d), v = ?
1 FG IJ
1 3 cm 3 cm 1 1 1
f
= (1.5 – 1) H K
15 0.3
cm
Now,
v

f1 − d
=
f2
or f = 30 cm 1 1 1 f2 + ( f1 − d)
3 Fig. 268 or = + =
v f2 ( f1 − d) f2 ( f1 − d)
1173. µ= = 1.5
2 f2 ( f1 − d)
∴ v =
( f1 + f2 − d)
 Note that some useless data is there.

1174. r = R sin α
r′ (= BC) = R sin α′
O1 A1 B1
Now, sin α′ = µ sin α y
r′ r O B y
=µ or r′ = µr
R R A

D−d D+d v
1175. u = and v =
2 2
1 1 d
1 (f1 – d)
Now, − = f1
v u f x
2 2 1
– = Fig. 270
D+d − (D − d) f
2 2 1 Now X=d+v
+ =
D+d D−d f f2 ( f1 − d) d( f1 + f2 − d) + f2 ( f1 − d)
=d+ =
1 D− d+D+ d ( f1 + f2 − d) ( f1 + f2 − d)
or =2
f (D + d)(D − d)
df1 − d2 + f1 f2 d( f1 − d) + f1 f2
1 4D = =
or = 2 or D2 – d2 = 4Df f1 + f2 − d f1 + f2 − d
f D − d2
Transverse distance of A from lens O1 is ∆. This is to
or d2 = D2 – 4Df or d2 = D(D – 4f ) be treated as size of object. The final image formed is
or d = D(D − 4 f ) at a distance y from the axis of lens O1. If magnifica-
tion is m.
1 1 1 1
1176 & 1177. = + + v y
F fl fl fm m= =
u (− ∆)
1 1 FG IJ + 1
F
= 2(µ – 1)
r HK ∞ y=–∆
v
u
=–∆
f2 ( f1 − d)
( f1 + f2 − d)( f1 −d)
r
or F= f2
2(µ − 1) =–∆
f1 + f2 − d
Radius of curvature of the mirror
∆ f2
2r r Fig. 269 Now Y=∆–|y|=∆–
= 2F = = f1 + f2 − d
2(µ − 1) µ−1
1178. The final image is formed at point B having co-ordi- =∆
LM1 −
f2
=
OP
∆( f1 − d)
.
nates (X, Y). N f
1 + f2 − d Q
f1 + f2 − d
1179. Emergent rays are parallel to incident rays.
OPTICS 611
1180. When the angle of incidence is decreased from 90°, 1 1 1
the critical angle of incidence for red will be reached Adding, = −
v 2R 28
first.
When the angle of incidence is further decreased, a 1 FG
1 1 IJ
stage is reached when the angle of incidence becomes
Again,
10
= (1.5 – 1) +
H
R R K
equal to the critical angle for blue. So, it is the blue
colour which is finally seen after which the light or R = 10 cm
vanishes. 1 1 −1
∴ =
sin α max n1 v 20 28
1181. =
sin r n2 or v = 70 cm
π
But, r + θc =
2
A+D Fig. 272
θc sin
1184 to 1186. µ= 2
A
sin
r 2
αmax
A+D A
or sin = µ sin
2 2
A+D
Here = Angle of incidence
2
Fig. 271 A+D 3 60° FG IJ
3 3 1

or r =
π
– θc
∴ sin
2
=
2
sin
2
=
2H K
sin 30° =
2
×
2
2 A+D 3
FG π − θ IJ sin
2
=
4
or sin r = sin
H2 K c = cos θc

A+D ~
49°
sin α max n1 n1 2
∴ = or sin αmax = cos θc i.e. Angle of incidence ≈ 49°
cos θ c n2 n2

αmax = sin–1
LM n 1 O
cos θ P
A + D ≈ 98°
60° + D ≈ 98°
or
Nn 2
c
Q D ≈ 38°
n2 Fn I
But sin θc =
n1
or θc = sin–1 GH n JK
2 i.e. Angle of minimum deviation = 38°
1 π π
LM n F F n I I OP . For maximum deviation, either i1 =
2
or i2 =
2
∴ αmax = sin –1 cos G sin
MN n
1
2H
−1
GH n JK JK PQ
2
1 Let i2 =
π
2
1 F 1.5 − 1IJ FG − 2 IJ = 2
= G Then r2 = Critical angle = c
1182.
fm H 1.75 K H R K 7R r2 = 42°
7R But r1 + r2 = 60°
or fm = = 3.5 R
2 ∴ r1 = 18°
Clearly, the concave lens would behave as a conver-
gent lens of focal length 3.5 R. sin i1
= µ or sin i1 = µ sin r1
sin r1
1183. 1.5 1 1.5 − 1
− = 3 3
v1 u R = sin 18° = (0.31)
2 2
1.33 1.5 1.33 − 1.5 or sin i1 = 0.466 or i1 ~ 28°.
− =
v v1 −R
612 COMPREHENSIVE OBJECTIVE PHYSICS

1187. Fig. 273 shows A Similarly, the angle of diffraction θBn for the blue light
∠i = ∠A = 30° i r is given by
i
δ
According to Snell’s law d1 sin θBn = n λB
120°
n1 sin i = n2 sin r θ where λB = wavelength of blue light (450 to 500 nm)
≈ 500 nm
or sin r = 1.44 sin 30° i δ
FG IJ
λB
1
= 1.44 × = 0.72 B
∴ θB1 = sin–1
dH K= sin–1 (0.30) ≈ 17°
2
Fig. 273 FG 2λ IJ
HdK
B
∠δ = ∠r – ∠i θB2 = sin–1 = sin–1 (0.60) ≈ 37°
= sin–1 (0.72) – 30°
α and β can now be determined as follows :
∠θ = 2δ = 2 {sin–1 (0.72) – 30°}.
α = θR1 – θB1 = 25° – 17° = 8°
1 1.5 1 − 1.5
1188. − = β = θR2 – θB2 = 57° – 37° = 20°
v −3 −5 1
or simplification, v = – 2.5 cm. hence, 2α = 16° < 20° = β or α < β.
2
1189. With no glass plate, the path difference between waves 1191. When monochromatic light passes through a plane
reaching O from S1 and S2 is zero, i.e. S1O = S2O, so diffraction grating, bright or principal maxima are
there is a bright fringe at O. The refractive index for obtained from
glass is about 1.5. If the light rays from S2 travel a d sin θ = nλ
length of d in the glass, then the light rays from S1
where d = spacing of lines.
have to travel an equivalent path length of 1.5 d in
λ = wavelength of light.
x n = order of bright image.
n = 3 occurs at 45°.
Given
S1 λ 1
∴ d sin 45° = 3λ or = sin 45°
d 3
FG IJ
O
S2 λ n n
Thin glass
Plate
O′ hence sin θ = n
d
=
H K
3
sin 45° =
3 2
Screen Since maximum angle allowed is 90°, the highest or-
y der of diffracted beam is thus given by
Fig. 274 n = truncated integer of 3 2 = 4.
air in order to arrive at the screen in phase as indi- i.e. highest order nmax = 4.
cated in the diagram. Thus the path difference be- 1192.
tween S1O′ and S2O′ is 0.5 d in order that the light
rays arrive at O′ in phase. This principle applies to
other fringes on the screen. Hence, the fringe pat- A
tern moves towards Y.
1190. The angle of diffraction θRn for nth order diffraction Acos θ
pattern of the red light is given by θ

d sin θRn = n λR
1
where d= m
6 × 105
λR = wavelength of red light (650 to 700 nm) Fig. 275
≈ 7.0 × 10–7 m The amplitude of the plane-polarised radio-waves at
FG λ IJ = sin (0.42) ≈ 25° the receiving aerial is reduced by a factor of cos θ as
H dK
R
∴ θR 1 = sin–1 –1 depicted in the diagram. Since intensity or power is
proportional to the square of the amplitude at receiv-
FG 2λ IJ = sin (0.84) ≈ 57° ing end, the power delivered by the aerial to the re-
H dK
R –1
θR 2 = sin–1 ceiver is proportional to
A2 cos2 θ.
OPTICS 613

SELF-EVALUATION TEST I
Based on UNIT XV
[Expected Questions for Forthcoming Examinations]
1. A ray of light travelling inside a rectangular glass contrast between the maximum and minimum
block of refractive index 2 is incident on the glass- intensity is good when
air surface at an angle of incidence of 45°. The refrac- (a) I1 is much greater than I2
tive index of air is 1. Under these conditions, the ray (b) I1 is much smaller than I2
(a) will emerge into the air without any deviation (c) either I1 or I2 is zero (d) I1 = I2.
(b) will be reflected back into the glass 9. The velocity of light emitted by a source S observed
(c) will be absorbed by an observer O, who is at rest with respect to S is
(d) will emerge into the air with an angle of refraction equal c. If the observer moves towards S with velocity v,
to 90°. the velocity of light as observed will be
2. A candle is placed in front of a concave mirror. The (a) c + v (b) c – v
image of candle is formed on a wall at a distance of
v2
2 m from the candle. The size of the image is 6 times (c) c (d) 1− 2 .
that of the candle. The distance of the candle from c
the mirror is 10. A film projector magnifies a 3 cm × 2 cm film strip
(a) 1.6 m (b) 2.4 m on a screen. If the linear magnification is 10, the area
(c) 2 m (d) 0.4 m. of the magnified film on the screen is
3. In Q. No. 2, the focal length of the mirror used is (a) 6 cm2 (b) 60 cm2
(a) 0.343 m (b) 0.48 m (c) 600 cm2 (d) 6000 cm2.
(c) 3.43 cm (d) 4.8 cm. [MP PMT 1996]
4. The two adjacent walls and the ceiling of a rectangu- 11. A locality is photographed from an aeroplane, flying
lar room are mirror-surfaced. The number of images at a height of 2000 m, with a camera whose focal
of himself that an observer sees is length is 50 cm. The ratio of the linear size of the
(a) 2 (b) 4 image to the size of the object, is nearly
(c) 6 (d) 8. (a) 1 : 40 (b) 1 : 1000
5. A sound wave travels from air to water. The angle of (c) 1 : 4000 (d) 1 : 10000.
incidence is α1 and the angle of refraction is α2. As- 12. We cannot see the shadow of the bird flying high in
suming Snell’s law to be valid. the air because
(a) α2 < α1 (b) α2 > α1 (a) the umbra of the image becomes very large and faint
(c) α2 = α1 (d) α2 = 90°. (b) the umbra vanishes altogether and penumbra becomes
[National Standard Exam. in Physics 1999] too faint to be observed
(c) the shadow changes its position very quickly as the bird
6. The apparent wavelength of the light from a star
flies
moving away from the earth is 0.01% more than its
(d) the umbra and penumbra overlap.
real wavelength. Then the velocity of star is
(a) 60 km s–1 (b) 15 km s–1
13. A clock hung on a wall has marks instead of numbers
on its dial. On the opposite wall, there is a mirror,
(c) 150 km s–1 (d) 30 km s–1.
and the image of the clock in the mirror, if read,
7. An aeroplane is flying at a height 1500 m. It has a indicates the time as 8.20. What is the time on the
camera having convex lens of focal length 45 cm and clock ?
photographic plate 30 cm × 30 cm. How much area (a) 3.40 (b) 4.40
on the ground can be photographed at one time ? (c) 5.20 (d) 4.20.
(a) 103 m2 (b) 105 m2
14. Which of the following can produce a virtual image
(c) 104 m2 (d) 106 m2. larger than the object ?
8. If the intensities of the two interfering beams in (a) convex mirror (b) concave mirror
Young’s double slit experiment be I1 and I2 , then the (c) plane mirror (d) concave lens.
614 COMPREHENSIVE OBJECTIVE PHYSICS

15. Which of the following can produce a parallel beam (b) the wavelengths are not equal.
of light if a point source of light is given ? (c) the amplitudes are not equal.
(a) convex mirror (b) concave mirror (d) none of the above.
(c) concave lens 22. A laser beam may be used to measure very large
(d) two plane mirrors inclined at 90° to each other. distance because
16. Which of the following graphs depicts correctly the (a) it is unidirectional (b) it is coherent
variation of magnification with object distance for a (c) it is monochromatic (d) it is not absorbed.
concave mirror ?
23. A piece of green glass when heated in dark will appear
(a) (b) to be
(a) red (b) green

1 (c) white (d) black.


1
m m 24. Light appears to travel in straight lines since
(a) it is not absorbed by the atmosphere
(b) it is reflected by the atmosphere
f 2f 3f f 2f 3f (c) its wavelength is very small
u f
(d) its velocity is very large.
25. The refractive indices of glass and water w.r.t. air
(c) (d) are 3/2 and 4/3 respectively. The refractive index of
glass w.r.t. water will be
(a) 8/9 (b) 9/8
m
m (c) 7/9. (d) none of these.
[MP PET 2000]
2f 26. In the interference pattern, energy is
u u
(a) created at the position of maxima
(b) destroyed at the position of minima
Fig. 276
(c) conserved but is redistributed
17. Focal length of a convex lens will be maximum for (d) none of the above.
(a) blue light (b) yellow light
27. How does the intensity (I) of a wave depend on dis-
(c) green light (d) red light. tance (r) from a line source ?
18. According to corpuscular theory of light, the speed (a) I ∝ r–1 (b) I ∝ r–2
of light is (c) I ∝ r (d) I ∝ r1/2 .
(a) greater in denser media than the rarer media
28. A beam of monochromatic blue light of wavelength
(b) less in denser media than the rare media 4200 Å in air travels in water, refractive index = 4/3.
(c) the same in all media Its wavelength in water will be
(d) independent of the media but depends on the temperature. (a) 2800 Å (b) 5600 Å
19. Doppler shift in frequency does not depend upon (c) 3150 Å (d) 4000 Å.
(a) the frequency of the wave produced 29. Light from a sodium lamp passes through a single
(b) the velocity of the source narrow slit and then through two closed parallel
(c) the velocity of the observer narrow slits. If you look through the double slit
(d) distance from the source to the observer. towards the sodium lamp, you will see
(a) a continuous yellow band (b) a continuous black band
20. Longitudinal waves do not exhibit
(c) alternate black bands and yellow bands
(a) Refraction (b) Reflection
(c) Diffraction (d) Polarisation. (d) the colours of the rainbow.

21. In Young’s double slit experiment, if the two slits are 30. The critical angle for total internal reflection of light
illuminated with separate sources, no interference going from medium I to medium II is given by the
pattern is observed because 5
relation tan i c = . The refractive index of the I
(a) there will be no constant phase difference between the 9
two waves. medium with respect to the medium II is
OPTICS 615
(a) 1.89 (b) 1.67 38. Light incident on an air-glass boundary is partially
reflected, and partially refracted. A person measures
156 106
(c) (d) . the angles of reflection (r) and refraction (r′) for vari-
0.5 5
ous angles of incidence (i). The graph will not be a
31. Which of the following happens when a monochro- straight line if he plots
matic light wave passes from air to glass ? (a) i versus sin r′ (b) i versus r
(a) Both the frequency and wavelength decrease.
(b) Frequency increases and wavelength decreases. (c) sin i versus sin r (d) sin r versus sin r′.
(c) Frequency remains the same but wavelength decreases. 39. A photon of light enters a block of glass after travel-
(d) Frequency and wavelength are unchanged. ling through vacuum. The energy of the photon on
32. The velocity of light is minimum in entering the glass block
(a) vacuum (b) glass (a) increases because its associated wavelength decreases.
(c) diamond (d) water. (b) decreases because the speed of the radiation decreases
33. The difference between light waves and sound waves (c) stays the same because the speed of the radiation and the
is associated wavelength do not change
(a) light waves in air are transverse while sound waves in air (d) stays the same because the frequency of the radiation
are longitudinal. does not change.
(b) light waves in air are longitudinal while sound waves in (e) stays the same because, although the speed of the
air are transverse. radiation and its wavelength both increase, they do so by
(c) light waves in air may be longitudinal or transverse but the same factor.
sound waves in air are always longitudinal. 40. Michelson-Morley’s experiment was designed to test
(d) light waves in air are transverse only while sound waves the validity of
in air may be longitudinal or transverse. (a) ether hypothesis (b) length contraction
34. A beam of light is partially reflected and partially (c) mass variation (d) time dilation.
refracted from a surface. The angle between reflected
41. A small angle prism has a prism angle A = 4° and
and the refracted light is 90°. The angle of refraction
is 30°. Then angle of incidence must be 3
refractive index = . It is placed with its base hori-
(a) 60° (b) 78° 2
(c) 75° (d) 50°.
zontal in front of a vertical mirror. A horizontal ray
of light passes through the prism and is reflected
35. A glass prism has refractive index 1.5 and the re-
back from the mirror. By what angle the mirror
fracting angle is 90°. If a ray falls on it at an angle of
incidence of 30°, then the angle of emergence will be should be rotated so that the reflected ray becomes
(a) 60° (b) 30°
horizontal ?
(c) 45° (a) 1° (b) 4°

(d) the ray will not emerge out of this prism. (c) 6° (d) 8°.

36. Light in air is incident on the surface of a liquid tank. 42. An eye specialist prescribes spectacles having a com-
The reflected and refracted rays are mutually bination of convex lens of focal length 40 cm in con-
perpendicular. If the angle of incidence is 60°, the tact with a concave lens of focal length 25 cm. The
refractive index of the liquid is power of this lens combination in diopter is
(a) 1.33 (b) 1.73 (a) + 1.5 (b) – 1.5
(c) 1.5 (d) 1.4. (c) + 6.67 (d) – 6.67.
37. A beam of light is converging towards a point I on a [IIT 1997 (Cancelled paper)]
screen. A plane parallel plate of glass whose thick-
43. An equi-convex glass lens (Fig. 277 A) has a focal
ness in the direction of the beam = t, refractive index
= µ, is introduced in the path of the beam. The con- length f and power P. It is cut into two symmetrical
vergence point is shifted by halves (Fig. 277 B) by a plane containing the principal
FG 1 IJ away FG 1 IJ away axis. The two parts are as shown in (Fig. 277 C).
H µK
(a) t 1 −
H µK
(b) t 1 + Then the power of the each part is

F 1I
(c) t G 1 − J nearer
F 1I
(d) t G 1 + J nearer.
H µK H µK
616 COMPREHENSIVE OBJECTIVE PHYSICS

(a) 0.7 m (b) 0.25 m


(c) 1 m (d) 2 m.
46. In Question Number 45, if a drop of water is placed
between the lens and mirror and again image formed
is to coincide with the pin itself, the position of the
pin
(a) remains unchanged (b) must be lowered
A B C (c) must be raised up (d) may be raised up or lowered.
Fig. 277 47. A particle executes simple harmonic motion of
(a) P (b) P/2 amplitude 1 cm along the principal axis of a convex
lens of focal length 12 cm. The mean position of
(c) 2P (d) Zero.
oscillation is at 20 cm from the lens. The amplitude
44. A ray incident at a point at an angle of incidence of of oscillation of the image of the particle is
60° enters a glass sphere of refractive index 3 . It (a) 1 cm (b) 2 cm
suffers both reflection and refraction at the farther (c) 2.25 cm (d) 2.75 cm.
surface of the sphere. At this surface, the reflected
48. A point object O is
and refracted rays are inclined to each other at an
placed at a distance of
angle of
0.3 m from a convex
(a) 45° (b) 90°
lens of focal length 0.2 0.0005 m
(c) 135° (d) 180°. m cut into two halves, O 0.0005 m
45. A convex lens is placed over a plane mirror as shown, each of which is dis-
in Fig. 278. When a pin is placed as shown, the im- placed by 0.0005 m as
age formed coincides with the pin itself. The focal shown in Fig. 279. The
length of lens is image distance is
Fig. 279
(a) 0.2 m
(b) 0.4 m
(c) 0.6 m (d) 0.8 m.
49. In the previous question, the number of images is
(a) 1 (b) 2
0.7 m
(c) 4 (d) 6.
50. In Q 48, the distance between the images is
(a) 0.001 m (b) 0.002 m
(c) 0.003 m (d) 0.004 m.

Fig. 278

Answers
1. (d) 2. (d) 3. (a) 4. (c) 5. (b) 6. (d) 7. (d) 8. (d)
9. (c) 10. (c) 11. (c) 12. (b) 13. (a) 14. (b) 15. (b) 16. (b)
17. (d) 18. (a) 19. (d) 20. (d) 21. (a) 22. (a) 23. (a) 24. (c)
25. (b) 26. (c) 27. (a) 28. (c) 29. (a) 30. (d) 31. (c) 32. (c)
33. (a) 34. (a) 35. (d) 36. (b ) 37. (a) 38. (a) 39. (d) 40. (a)
41. (a) 42. (b) 43. (a) 44. (b) 45. (a) 46. (c) 47. (c) 48. (c)
49. (b) 50. (c)
OPTICS 617

Solutions
1. Using Snell’s law, 3
or m=
2 sin 45° = 1 sin θ or sin θ = 1 or θ = 90°. 10000
30 3
2. m=– v Now, = or O = 105 cm
u O 10000
or O = 103 m.
– 6 = – v or v = 6u Required area = 103 m × 103 m = 106 m2
u
Clearly, 5u = 2 or u = 0.4 m. 9. The velocity of light does not depend upon the velocity
of the observer.
10. Area of film = (10 × 3) × (10 × 2) cm2
= 30 × 20 cm2 = 600 cm2
f f
11. m= ≈
f −u −u
50 1
= = .
2000 × 100 4000
u 13. The required time can be obtained by subtracting the
given time from 12 h 0 minute.
F I
6u
1
Fig. 280 17.
f
= (µ – 1)
1

1
R1 R 2
GH JK
1 1 1
3. + = b
u v f µ=a+
λ2
1 1 1 1 10 10
+ = or =– – 1
− 0.4 − 2.4 f f 4 For red light, λ maximum ; µ minimum ; mini-
24 f
1 10 + 60 mum, f maximum.
or =–
f 24 wµ
µg 3/2 9
25. g = = =
24 µw 4/3 8
or f =– m = – 0.343 m.
70 λ 4200
4. One image is produced by each of the plane mirrors 28. λ′ = = Å
µ 4/3
and one by each of the three pairs of combination of
12600
two mirrors each. So, total number images is 3 + 3 = Å = 3150 Å
i.e. 6. 4
5. Sound travels faster in water as compared to that in 5
30. tan ic = 6
air. 9 10
2 or
So, α2 is greater. 5
sin ic = 2 +5
5
λ′ − λ v ∆λ v 106 9
6. = or =
λ c λ c 1
µ= ic
0.01 v sin ic
or = or v = 10–4 × 3 × 108 m s–1 9
100 c
106 Fig. 281
or v = 3 × 104 m s–1 = .
or v = 30 × 103 m s–1 = 30 km s–1 5

f f 31. Frequency is a characteristic of the source of light


45
7. m= ≈ = and not medium.
f −u −u 1500 × 100
618 COMPREHENSIVE OBJECTIVE PHYSICS

c
32. µ=
v
c
v=
µ
Higher the value of µ, lesser is the value of v. 2°

33. Electromagnetic waves are transverse in nature.



sin i
34. =µ
sin 30°
Also, tan i = µ
Fig. 283
sin i 1
∴ tan i = or =2 When mirror is turned through a certain angle θ, the
sin 30° cos i reflected ray turns through twice that angle. So, to
1 turn the reflected ray through 2°, the mirror should
or cos i = or i = 60°
2 be turned through 1°.
sin 30° 3 42. Power of convex lens
35. = 1.5 =
sin r1 2 100
= = 2.5 D
1 40
or sin r1 = or r1 = 19.47° ≈ 19.5° Power of concave lens
3
100
=–=–4D
25
Power of combination
90°
= 2.5 D – 4 D = – 1.5 D
43. Neither the refractive index nor the radii of curva-
30° r1 r2 ture change. So, power remains unaffected.
sin 60°
44. = 3
sin r1
sin 60° 1
or sin r1 = =or r1 = 30°
3 2
Fig. 282 Again, r2 = 30° [Equal sides, opposite angles equal]
Now, r2 + r1 = 90° or r2 = 90° – r1
or r2 = 90° – 19.5° = 70.5°

1 1 2 e
Again, sin ic = = = or ic = 41.8° 60° r1 r2
1.5 15 3 r2 θ
10
r2 > ic
So, the light would suffer total internal reflection and
not emerge out of the prism.

36. µ = tan 60° = 3 = 1.732


Fig. 284
39. When light enters glass after travelling through a
vacuum, the speed of the radiation and its wavelength sin 30° 1
Now, =
both decrease. Frequency of the radiation remains sin e 3
unchanged. Hence, the energy of the photon remains
unchanged since it is frequency-dependent. 3
or sin e = or e = 60°
FG 3 − 1IJ 2
41. δ = (µ – 1) A =
H2 K 4° = 2° Angle θ between reflected and refracted rays
= 180° – (30° + 60°) = 90°.
OPTICS 619

45. The rays of light, after refraction through the lens, 48 to 50. As shown in Fig. 285, two images are formed.
must fall normally on the mirror. In other words, the
rays of light should come out of the lens as parallel
beam. Clearly, the pin must be at the principal focus I1
of the lens.
C1
46. In the presence of water, the power of the lens de-
creases and the focal length increases. O
d
C2
1 1 1
47. − =
v u f
I2
dv F du IJ
– G−

v2 H uK 2
=0 u v

dv du Fv I
2 Fig. 285
or –
v2
=–
u 2
or dv = du GH u JK
2
1 1 1
Now, − =
1 1 1 v u f
Again, − =
v − 20 12 1 1 1 1 1 1
or – = or = –
1 1 1 5−3 2 v − 0.3 0.2 v 0.2 0.3
or = − = =
v 12 20 60 60 1 0.3 − 0.2
or = or v = 0.6 m
or v = 30 cm v 0.2 × 0.3

FG 30 IJ 2
9
Using triangles OC1C2 and OI1I2, we get
Now, dv = 1
H 20 K =
4
cm = 2.25 cm. d
c1c2
=
u+v
u
d 0.3 + 0.6
or =
2 × 0.0005 0.3
or d = 3 × 2 × 0.0005 = 0.003 m.

KNOWLEDGE PLUS
l A beam of light composed of red and green rays is incident obliquely at a point on the face of a rectangular glass
slab. When coming out on the opposite parallel face, the red and green rays emerge from
(a) two points propagating in two different parallel directions
(b) one point propagating in two different directions
(c) one point propagating in the same direction
(d) two points propagating in two different non parallel directions. [All India PM/PD 2004]
Sol. Refractive index of glass is different for different colours.
So, (a) is the right choice.
620 COMPREHENSIVE OBJECTIVE PHYSICS

SELF-EVALUATION TEST II
Based on UNIT XV

DIRECTIONS :
(i) MCQs 1 to 24 have one correct alternative.
(ii) MCQs 25 to 30 have more than one correct alternative.
(iii) MCQs 31 to 35 have one or more than one correct alternative.

1. In a river 2 m deep, a water level measuring post 5. Fig. 287 on the left below shows two coherent sources
embedded into the river stands vertically with 1m of S 1, S2 with separation equal to the wavelength of
it above the water surface. If the angle of inclination emission. They emit in the same phase. Fig. 288 show
of the sun above the horizon is 30°, the length of the four polar graphs in which the intensity in direction
shadow of the post on the bottom surface of the river θ is shown by the length of radius vector in that di-
is : (Given : µ for water = 4/3). rection. Which of the polar graphs represents
schematically the correct intensity distribution ?

30°

1m
q q

2m
q (a) (b)
S1 S2
l

Fig. 287 q
q
Fig. 286
(a) 1.44 m (b) 2.44 m
(c) 3.44 m (d) 4.44 m.
(c) (d )
2. The principal section of glass prism is an isosceles
∆PQR with PQ = PR. The face PR is silvered. A ray Fig. 288
is incident perpendicularly on face PQ and after two
reflections, it emerges from base QR, normal to it. [National Standard Exam. in Physics 1991]
The angle QPR of prism is 6. The distance between two point sources of light is
(a) 6° (b) 16° 24 cm. Where should a convex lens of focal length 9
(c) 26° (d) 36°. cm be placed so that the images of both sources are
3. In a circularly polarised light, the location of the head formed at the same point ?
of the electric vector always lies on a (a) 18 cm or 6 cm (b) 10 cm or 14 cm
(a) circle (b) ellipse
(c) 16 cm or 8 cm (d) 9 cm or 15 cm.
(c) helix (d) parabola.
7. A convex lens forms a real image of a point object
4. Suppose all the electric vectors of all the wave trains
placed at a distance of 0.5 m. The image is also at a
in a given beam of light oscillate in a definite orienta-
distance of 0.5 m from the lens. Mid-way between
tion. Such a light beam is
the lens and the image a convex mirror is placed. It
(a) yellow (b) green
is found that there is no parallax between the object
(c) helix (d) linearly polarised.
OPTICS 621

and its image. The focal length of the convex mirror 2 3


(a) sin r = (b) sin r =
is 3 2
(a) 1 m (b) 0.5 m 2 3
(c) sin r = (d) sin r = .
(c) 0.25 m (d) 0.125 m. 13 13
8. A convex lens of focal length 40 cm is held coaxially 15. A convex lens of focal length 10 cm forms a real im-
and 12 cm, above a concave mirror of focal length age of an object placed at a distance of 20 cm from it.
18 cm. A luminous point object placed y cm above Mid-way between the convex lens and the position of
the lens on its axis gives rise to an image coincident the image, a thin concave lens is introduced. The
with itself. Then y is equal to image formed now is at a distance of 5 cm away from
(a) 15 cm (b) 16 cm the earlier position. The focal length of concave lens
(c) 36 cm (d) 40 cm. is
(a) – 2.5 cm (b) – 5 cm
9. A plano-convex lens, when silvered on the plane side,
behaves like a concave mirror of focal length 30 cm. (c) – 10 cm (d) – 30 cm.
However, when silvered on the convex side, it be- 16. A convex lens and a conave lens having focal lengths
haves like a concave mirror of focal length 10 cm. 30 cm and 20 cm are placed 10 cm apart. The dis-
The refractive index of the material is tance of a source of light at which it must be placed
(a) 3.0 (b) 1.5 for this system to give a parallel beam of rays is
(a) 10 cm (b) 20 cm
(c) 1.0 (d) 2.0.
(c) 30 cm (d) ∞.
10. In the previous question, the true radius of the spheri-
17. A short linear object of length b lies along the axis of
cal surface of the plano-convex lens is
a concave mirror of focal length f at a distance u
(a) 30 cm (b) 10 cm from the pole of the mirror. The size of the image is
(c) 20 cm (d) 60 cm. nearly equal to
11. A convex lens of focal length 20 cm is placed in front FG u − f IJ 1/2 FG u − f IJ
of a convex mirror of radius of curvature 15 cm. A
luminous object is placed in front of the convex lens
(a) b
H f K (b) b
H f K
at a distance of 40 cm from it. Then the distance F f IJ 1/2
(c) b G
F f IJ 2 .
(d) b G
between the convex mirror and the convex lens to Hu− f K Hu− f K
get an inverted image of the luminous object, coinci- 18. For a prism allowing grazing incidence and grazing
dent with the object is emergence, the angle of prism is equal to
(a) 40 cm (b) 25 cm (a) critical angle (b) twice the critical angle
(c) 15 cm (d) 35 cm. (c) 0 (d) ∞.
12. In the previous question, the distance between the 19. A ray of light will not emerge out of a prism (refract-
convex mirror and the convex lens to get an upright ing angle A), whatever may be the angle of incidence,
image of the luminous object coincident with the ob- if refractive index µ is greater than
ject, is A A
(a) sin (b) cos
(a) 40 cm (b) 25 cm 2 2
A A
(c) 15 cm (d) 35 cm. (c) sec (d) cosec
2 2
13. A thin equiconvex lens has focal length 10 cm and 20. If A is the refracting angle of the prism and µ is the
refractive index 1.5. One of its faces is now silvered refractive index of the material of the prism, then
and for an object placed at distance u in front, the the light will emerge out of the prism only if angle of
image coincides with the object. The value of u is incidence is greater than
(a) 10 cm (b) 5 cm 2
(a) sin–1 [ µ − 1 sin A – cos A]
(c) 20 cm (d) 15 cm.
14. Light entering an air-glass (µ = 1.5) boundary is (b) sin–1 [ µ 2 − 1 cos A – sin A]
partly reflected and partly refracted. If the incident 2
(c) sin–1 [ µ + 1 sin A + cos A]
and reflected rays are at right angles to each other,
the angle of refraction r is given by (d) sin–1 [ µ 2 + 1 cos A + sin A].
622 COMPREHENSIVE OBJECTIVE PHYSICS

21. A ray of light passing through a prism having µ = 2 24. P is a small-angled


suffers minimum deviation. It is oberved that the prism of angle 3° made
angle of incidence is double the angle of refraction of a material of refrac- 3°
within the prism. The refracting angle of the prism tive index 1.5. A ray of
is light is incident as
shown in Fig. 291 M is
(a) 30° (b) 60°
a plane mirror. The an-
(c) 90° (d) 115°. gle of deviation for the
22. Consider the arrangement shown in Fig. 289. Through ray reflected from the
what angle the vertical mirror M should be rotated mirror M with respect P M
so that the image of point source of light S is formed to the incident ray is Fig. 291
on S itself ? (a) 4.5° (b) 175.3°
(c) 177° (d) 178.5° .
25. Which of the following properties of light conclusively
support wave theory of light ?
µ = 1.5 4°
S (a) Light obeys laws of reflection
(b) Speed of light in water is smaller than the speed in vacuum.
(c) Light shows interference.
88° 88°
(d) Light shows photoelectric effect.
26. Two monochromatic coherent point sources S 1 and
S2 are separated by a distance L. Each source emits
M light of wavelength λ, where L >> λ. The line S1 S2
when extended meets a screen perpendicular to it at
Fig. 289
point A. Select the correct statement (s) from the fol-
(a) 1° (b) 2° lowing.
(c) 4° (d) not possible. (a) The interference fringes on the screen are circular in shape
23. Fig. 290 shows an apparatus for measuring the speed (b) The interference fringes on the screen are straight lines
of light, c. Light is incident at P on one face of an perpendicular to line S1S2A
octagonal mirror M and then traverses a path PQRS, (c) The point A is on intensity maximum if L = nλ (n = integer)
of total length L, before being reflected again from (d) The point A is always an intensity maximum for any sepa-
another face of M. ration L.
27. Four light waves are represented by
(i) y = a1 sin ωt (ii) y = a2 sin (ωt + φ)
(iii) y = a1 sin 2ωt. (iv) y = a2 sin 2(ωt + φ).
S
R Interference fringes may be observed due to superpo-
sition of
M
Q (a) (i) and (ii) (b) (i) and (iii)
P (c) (ii) and (iv) (d) (iii) and (iv).
28. Mark the correct options
(a) Luminous flux and radiant flux have same dimensions.
(b) Luminuous flux and luminous intensity have same di-
Fig. 290 mensions.
M is rotated about its axis of symmetry as indicated (c) Radiant flux and power have same dimensions.
and its angular velocity is adjusted until the light (d) Relative luminosity is a dimensionless quantity.
emerges in the same direction as when M is station- 29. A room is illuminated by an extended source. The
ary. What is the smallest angular velocity at which illuminance at a particular portion of a wall can be
this condition is achieved ? increased by
(a) πc/8L (b) πc/4L (a) moving the source (b) rotating the source
(c) πc/2L (d) πc/L (c) bringing some mirrors in proper positions
(e) 2πc/L. (d) changing the colour of the source.
OPTICS 623
30. A telescope and a microscope do not differ in respect 33. Parallel water waves of wavelength 10 m strike a
of straight sea wall. The wavefronts make an angle of
(a) Effective focal length of the system 30° with the wall as shown.
(b) Method of focussing (c) Linear magnification
(d) None of the above. Wavefronts 10 m
[National Standard Exam. in Physics 1999]
31. An astronomical telescope has an angular magnifi-
cation of magnitude 5 for distant objects. The sepa-
ration between the objective and the eye-piece is 36 30° 30°
cm and the final image is formed at infinity. The
focal length fo of the objective and the focal length fe
of the eye-piece are
(a) fo = 45 cm and fe = – 9 cm (b) fo = 50 cm and fe = 10 cm Fig. 293
(c) fo = 7.2 cm and fe = – 5 cm (d) fo = 30 cm and fe = – 6 cm.
What is the difference in phase at any instant be-
[IIT 1989] tween the waves at two points 5 m apart along the
32. A parallel beam of monochromatic light, initially trav- wall ?
elling upwards, enters region of the atmosphere in (a) 45° (b) 55°
which the refractive index increases steadily with (c) 90° (d) 180°.
height. The path of the beam is represented by
34. In dispersion without deviation,
Height Height Height (a) the deviation produced by one is crossed by the other
prism.
(b) the dispersion produced by one prism is crossed by the
other prism.
(c) the resultant deviation is zero
(d) none of the above.
35. A planet is observed by an astronomical refracting
Horizontal distance Horizontal distance Horizontal distance
(a ) (b) (c) telescope having an objective of focal length 16 m
Height Height
and an eye-piece of focal length 2 cm. Mark the correct
statement.
(a) The objective is larger than eye-piece.
(b) The image of the planet is inverted.
(c) The angular magnification is 800.
(d) The distance between the objective and the eye-piece is
16.02 m.
Horizontal distance Horizontal distance
(d ) (e)

Fig. 292

Answers
1. (c) 2. (d) 3. (c) 4. (d) 5. (c) 6. (a) 7. (d) 8. (a)
9. (b) 10. (a) 11. (b) 12. (a) 13. (b) 14. (c) 15. (d) 16. (d)
17. (d) 18. (b) 19. (d) 20. (a) 21. (c) 22. (b) 23. (b) 24. (d)
25. (b), (c) 26. (a), (c) 27. (a), (d) 28. (b),(c),(d) 29. (a),(b),(c),(d) 30. (a), (c) 31. (a), (d) 32. (c)
33. (c) 34. (a), (c) 35. (a),(b),(c),(d).
624 COMPREHENSIVE OBJECTIVE PHYSICS

Solutions
1. α + β + β = 180°
30° or 2β + α = 180°
A
Now 90° – 2α + β + 90° = 180°
60° 1 m
60° or β = 2α
B
∴ 5α = 180° or α = 36°.
x2 C
r 5. Polar graphs for intensity distribution in interference.
π
2m 2m Maxima along θ = 0 and , only choice (c).
2

E
6.
D
x1 x2
I 1, I 2
Fig. 294
Using Snell’s law, S1 S2
x2
4 sin 60° 3
= =
3 sin r 2 sin r
x1 x
3 3
sin r = Fig. 296
8
Clearly, one image is real and the other image is
27 37
cos r = =1− virtual.
64 8
1 1 1
3 3 8 3 3 Using lens equation − = , we get
tan r = × = v u f
8 37 37
Now, x1 = 2 tan r 1 1 1 1 1 1
– = and − =
3 x − x1 f x x2 −f
2×3 3 6 3
= = =6 = 1.71 m
37 37 37 1 1 2
Subtracting, + =
Again, x2 = 1 tan 60° = 3 = 1.73 m x1 x2 f
Length of shadow = x1 + x2 = 3.44 m. 1 1 2
2. or + =
P x1 24 − x1 9

α
24 2
or =
24 x1 − x12 9
α
α or 108 = 24x1 – x12
90° – 2α or x12 – 24x1 + 108 = 0
or x12 – 18x1 – 6x1 + 108 = 0
2α or x1 (x1 – 18) – 6 (x – 18) = 0
90° – 2α or (x1 – 6)(x1 – 18) = 0

or x1 = 6 cm or 18 cm.
7. The ray coming from the lens and incident on the
β β mirror must retrace its path so that there is no paral-
Q R
lax between the object and the image formed by con-
vex mirror. This is possible only if 2f = 0.25 m
Fig. 295
OPTICS 625

1 1 1 1
Case II. = + +
F fl fl fm
1 2 1
or = +
F fl fm
1 FG µ − 1IJ + 2
or
F
=2
H R K R
0.25 m 1 2 Fig. 300
or = (µ – 1 + 1)
F R
Fig. 297
or f = 0.125 m. R R
C or F = or 2µ =
8. For image to be coincident 2µ F
with the object, the light
−R R
after refraction from the or 2µ = or 2µ = ...(ii)
lens, must fall normally − 10 10
on the mirror. The rays Dividing (i) by (ii), we get
36 cm

of light reflected from the y


2(µ − 1) R 10 1
mirror, if produced back- = × =
wards, should meet at the 2µ 30 R 3
centre of curvature C of 12 cm or 6(µ – 1) = 2µ or 6µ – 6 = 2µ
the mirror.
3
For the convex lens, or 4µ = 6 or µ =
2
u = – y,
Fig. 298
v = – [36 – 12] cm 3 R
From = n (ii), 2 × =
= – 24 cm 2 10
f = 40 cm or R = 30 cm
1 1 1 11. In this problem, the rays must fall normally on the
Now, – = convex mirror so that they retrace their path. This is
v u f
possible if the rays, after refraction from the lens, are
1 1 1 headed towards the centre of curvature C of the mirror.
− =
− 24 − y 40
40 cm
1 1 1 1 3+5
or = + or =
y 40 24 y 120
120
or y = cm = 15 cm.
8
1 1 1 1
9 & 10. Case I. = + + O C
F fl fl fm

1 2
or =
F fl
15 cm
1 L µ − 1OP
=2 M
40 cm
or
F N R Q Fig. 301
R Now, note that ‘‘u = 2f ’’ clearly, the distance between
or F= Fig. 299
2(µ − 1) the lens and the mirror
−R = (40 – 15) cm = 25 cm
– 30 =
2(µ − 1) 12. In this problem, the image formed by the lens should
R bt at the pole of the mirror.
or 2(µ – 1) = ...(i)
30
626 COMPREHENSIVE OBJECTIVE PHYSICS

1 10 − 15 1 5
or = or =–
f 150 f 150
or f = – 30 cm
16. f1 = 30 cm, f2 = – 20 cm
d = 10 cm
1 1 1 1
= + −
F f1 f2 f1 f2
1 1 1 10
Fig. 302 = – +
F 30 20 30 × 20
1 FG IJ
2 1
=
1 1 1
13.
f
= (µ – 1)
RH K F

30 20 60
+

= (1.5 – 1) FG IJ
1 2−3+1
1 2 or = =0
10 H RK F 60
So, power of system is zero.
or R = 10 cm
17. Size of image
1
F
=
1
+
1
10 10 10
+
2
Fig. 303 FG f IJ 2

1 2 1 1 1
= m2 × length of object =
Hu− f K b
or = + = + or F = 2.5 cm
F 10 5 5 5 18.
The image would coincide with the object if the object A
distance is equal to 2F.
i.e. 2 × 2.5 cm or 5 cm. 90° ic 90°
14. µ = tan i ic

3 sin i Ö3
= 3
2 sin r
3 i
or sin r = sin i 2 Fig. 306
2
A = r1 + r2
3 Fig. 304
Now, tan i = A = ic + ic
2
A = 2ic
3
sin i = 19. Let us consider the maximum value of i. This is clearly
13
90°.
2 3 2 So, r1 = ic Also, r2 > ic
∴ sin r = × = A
3 13 13 Clearly, r1 + r2 > 2ic
15. 10 cm or A > 2ic
A i
or > ic r1
2 r2
A
I
I′ or sin > sin ic
2
B C
A 1
15 cm or sin > Fig. 307
2 µ
20 cm
1 A 1
Fig. 305 or < sin or µ >
µ 2 A
For concave lens, sin
2
u = 10 cm, v = 15 cm, f = ? A
or µ > cosec .
1 1 1 1 1 1 2
Now, = − or = −
f v u f 15 10
OPTICS 627
20. We do not want total internal reflection at AC. 23. The light reflected from P now arrives at the surface
So, r2 < i2 A S in the time taken for the octagonal mirror M to
rotate through 45° (π/4) if M is rotating at its small-
Now, r1 + r2 = A
est angular velocity, ω0.
or r2 = A – r1
L
or ic > A – r1 i
r1 s.
Time taken for light to travel from P to S =
r2 c
or r1 > A – ic
Time taken by M to rotate through an angle of π/4
Now, sin r1 > sin (A – ic)
But 1 × sin i = µ × sin r1 B C π/4 π
Fig. 308 = =
ω0 4 ω0
sin i
or sin r1 =
µ ∴ Smallest angular velocity, ω0 is given by
sin i π L πc
∴ > sin (A – ic) = or ω0 = rad s–1.
µ 4ω 0 c 4L
or sin i > µ sin (A – ic) 24. δ = (µ – 1) A = (1.5 – 1) 3° = 1.5°
or sin i > µ [sin A cos ic – cos A sin ic]

or sin i > µ [sin A 1 − sin2 ic – cos A sin ic]

or
LM
sin i > µ sin A 1 −
1
− cos A
1 OP
MN µ2 µ PQ a b 1.5°
2
sin i > [ µ − 1 sin A – cos A]
2
or i > sin–1 [ µ − 1 sin A – cos A] 1.5°
21. δm = i + e – A 1.5°

= 2i – A = 2(2r) – A = 2A – A = A
FG A + δ IJ
H 2 K
m
sin
Now, µ= Fig. 309
A
sin
2 Angle with reference to incident light ab
FG
A+A IJ = 180° – 1.5° = 178.5°
or µ=
sin
H 2 K 27. The phase difference between (i) and (ii) is time-inde-
A pendent. The phase difference between (iii) and (iv) is
sin
2 also time-independent. The phase difference between
sin A A (i) and (iii) and also between (ii) and (iv) is time de-
or µ= or µ = 2 cos
A 2 pendent. So, coherence is not possible.
sin
2 28. Luminous intensity is luminous flux per unit solid
A A 1 angle.
or 2 = 2 cos or cos =
2 2 2 Total radiant flux is the total energy of radiation emit-
A ted by a source per unit time. Power is energy / time.
or = 45° or A = 90°.
2 Relative luminosity of a wavelength is the ratio of
22. δ = (µ – 1) A = (1.5 – 1) 4° = 2° luminous flux of a source of given wavelength and
The angle between reflected ray and incident ray is luminous flux of a 555 nm source of the same power.
4°.
fo
When a mirror turns through an angle θ, the reflected 31. M= and L = fo + fe
ray turns through twice that angle. fe
∴ 2θ = 4° or θ = 2° Both the equations are satisfied by (a) and (d)
628 COMPREHENSIVE OBJECTIVE PHYSICS

32. Light beam entering a denser Thus for a separation of 5 m


medium will be refracted at along the sea wall, the phase d
an angle r smaller than the r difference is thus
30°
angle of incidence i. Hence the
5 cm
light beam moves towards the i
vertical axis as it travels up- 2.5 Fig. 311
φ= × 360° = 90°.
wards, into a region of atmos- 10
phere in which the refractive 35. (a) The size of the objective of a telescope is larger
index increases steadily with Fig. 310 than the size of the eye-piece.
height. (b) The telescope produces inverted image
33. Distance d = 5 sin 30° fo 16
(c) M= = = 800
= 2.5 m fe 0.02
Wavelength of 10 m corresponds to 360°. (d) L = fo + fe = (16 + 0.02) m = 16.02 m.

KNOWLEDGE PLUS

l The refractive index of an equilateral prism is 3 . The angle of incidence at which the deviation is minimum is
(a) 30° (b) 45° (c) 60° (d) 75° (e) 15° [Kerala PMT 2003]
FG A + δ IJ
H 2 K
m
sin
Sol. For minimum deviation, µ = A
sin
2
FG
60° + δ m IJ
sin
H2 K
3 60° + δ m FG IJ
3=
sin
60° or
2
= sin
2 H K
2
FG 60° + δ IJ 60° + δ m δ +A
H 2 K
m
or sin 60° = sin or 60° = or δm = 60° = 2i – A or i = m = 60°
2 2
So, (c) is the right choice.

l One of the refracting surfaces of a prism of refractive index 2 is silvered. The angle of the prism is equal to the
critical angle of a medium of refractive index 2. A ray of light incident in the unsilvered surface passes through
the prism and retraces its path after reflection at the silvered face. Then the angle of incidence on the unsilvered
surface is
(a) 0° (b) 30° (c) 45° (d) 60° [EAMCET 2003]
FG 1 IJ = sin FG 1IJ = 30°
Sol. Critical angle = sin −1
H 2K
−1
H µK
∴ Angle of prism = 30°
A = r1 + r2 or 30° = r1 + 0 or r1 = 30°
sin i sin i 1
µ= or 2= or sin i = or i = 45°
sin ri sin 30° 2
So, (c) is the right choice.
UNIT XVI

DUAL NATURE OF MATTER AND RADIATIONS

l Photo-electric effect l Einstein’s photo-electric equation l Particle nature of light


l Photo-cells l Matter waves l Wave nature of particles
l de-Broglie relation l Davisson and Germer experiment l de-Broglie

UNIT DETAILS

1. Synopsis Points 10 Plus

2. Illustrations 10 Plus

3. MCQs from Competitive Examinations 155 Plus

4. Self-Evaluation Tests 2

5. Total Number of MCQs 285 Plus

6. Total Number of Solutions 255 Plus


CHAPTER 16

DUAL NATURE OF MATTER AND RADIATIONS

SYNOPSIS

1. EFFECT OF INTENSITY OF THE INCIDENT 3. EFFECT OF FREQUENCY ON STOPPING PO-


RADIATION ON PHOTOELECTRIC CURRENT TENTIAL
The photoelectric current varies directly as the inten- Y
sity of the incident radiation.

Stopping potential, V 0
Y
ν0
O X
Frequency (ν)
current (in µA)
Photoelectric

W0
e
Y′
Fig. 3
O Light intensity, I X 4. LAWS OF PHOTOELECTRIC EMISSION
(i) For a given material, there is a certain minimum
Fig. 1 frequency of radiation known as the threshold frequency,
such that if the incident radiation has a frequency below
2. EFFECT OF POTENTIAL this threshold, no photoelectric emission will ever take place,
however intense the radiation may be and for whatever length
of time we may expose the material to this radiation.
I (in µA)

(ii) For the incident frequency ν greater than the


B threshold frequency ν0, the photoelectric current is di-
A rectly proportional to the intensity of radiation till a
saturation is reached. (There is no threshold intensity). Even
the feeblest incident radiation will emit electrons, however
few in number, provided the frequency is greater than thresh-
old frequency.
O
– V0 +V (iii) For the incident frequency greater than the
threshold frequency, the maximum kinetic energy of the
Fig. 2 photoelectrons is directly proportional to only the
frequency of incident radiation and it is independent of
both the intensity of radiation and the duration of time of
exposure.

631

C-11\IITS\C16-1
632 COMPREHENSIVE OBJECTIVE PHYSICS

(iv) The photoelectric emission is an instantaneous the velocity of light. Further, according to quantum theory,
process. The time interval between the falling of radiation E = hν, where ν is the frequency of radiation and ‘h’ is Planck’s
on the photoelectric surface and the ejection of photoelec- constant. Combining the two, we get
trons is of the order of 10–9 second. E = hν = mc2
5. EINSTEIN’S PHOTOELECTRIC EQUATION But c = νλ for waves. Thus for a quantum of light
(photon), we get
1
mvmax 2 = hν – φ 0 c h
2 E=h. = mc2 ; giving us λ =
λ mc
Discussion. (i) If ν < ν0 then v2 is a negative quan-
The product of mass and velocity is the momentum p.
tity. So, the velocity v of the emitted photoelectron is imagi-
nary. Thus, no photoelectric emission is possible in this case. h
∴ λ=
From here, we get concept of threshold frequency as that p
frequency below which no photoelectric emission is possible.
Thus a quantum (particle) having a momentum p is
(ii) If ν > ν0 then v2 is a positive quantity. So, photo-
associated with a wavelength λ.
electric emission shall take place. Higher the intensity of
incident light, greater will be the number of available pho- This relation was for a photon and it proved true ex-
tons and consequently, greater will be the number of emit- perimentally. It now occurred to de-Broglie that what is true
ted photoelectrons. So, the photoelectric current will be in- for a photon, may also be true for a material particle of mass
creased. m moving with velocity v (not c).
(iii) K.E. of emitted electron = h(ν – ν0) h
So, the K.E. of the photoelectron is directly propor- λ= (∵ p = mv)
mv
tional to the frequency of incident radiation. The frequency
should, of course, be greater than threshold frequency. This means that a material particle of mass m mov-
c ing with a velocity v, can be considered as a wave of length λ,
(iv) c = νλ or =ν
λ given by the above relation. This is de-Broglie wave equa-
1 hc tion. It firmly establishes the aspect of unity of matter ;
∴ mvmax 2 = – φ0
2 λ since the momentum p can be associated only with a parti-
If λ is decreased, the kinetic energy shall increase. cle, while λ is associated only with a wave.
Thus, smaller the wavelength λ, greater will be the kinetic Discussion
energy and hence greater will be the velocity of emitted elec- 1
tron. (i) λ α i.e. if v = 0, λ = ∞.
v
(v) Photoelectric emission is a ‘knock-out’ process in
Thus, the matter waves are associated with mate-
which a single photon of energy hν knocks out an electron
rial particles only if they are in motion.
1
with energy mvmax 2 . Increase in the intensity of incident 1
2 (ii) λ α . Smaller the mass of the particle, higher is
m
radiation simply means an increase in the number of incident the wavelength associated with it.
photons. Thus, many photons of the same frequency will
1
simply knock out as many electrons of the same kinetic (iii) λ α . Larger the momentum of the particle,
energy. As a result, there is an increase in photoelectric p
shorter is the wavelength.
current. But the photoelectrons, though increased in number,
will all have the same energy as given by Einstein’s equation. (iv) Wavelength associated with a material particle is
independent of the charge of the particle.
(vi) An increase in the frequency of the incident radia-
tion increases the energy of the emitted electrons as given 7. BOHR’S POSTULATE OF MOMENTUM OF STA-
by Einstein’s equation. An increase in frequency does not TIONARY NON-RADIATING ORBITS
increase the number of photoelectrons. This means that the 2πr = nλ
photocurrent willl not increase because the photocurrent
depends simply upon the number of photoelectrons. Thus, h
But λ=
the increase in frequency has no effect on the magnitude of mv
photocurrent. nh h
∴ 2πr = or mvr = n
6. DE-BROGLIE WAVE EQUATION mv 2π
According to Einstein relation of mass-energy equiva-
Fig. 4
lence, E = mc2, where m is the mass of the particle and c is
DUAL NATURE OF MATTER AND RADIATIONS 633

8. DE-BROGLIE WAVELENGTH OF AN ACCELER- In the given experiment,


ATED ELECTRON V = 54 volt
Consider an electron accelerated through a potential
6.63 × 10 −34
difference of V volt. According to work-energy principle, work ∴ λ= −31
done (eV joule) on the electron must be equal to the gain of (2 × 9 × 10 × 1.6 × 10−19 × 54)1/ 2
FG 1 mv IJ of the electron. = 1.67 Å …(2)
H2 K
kinetic energy 2
It is clear from (1) and (2) that the wavelength deter-
mined by two different methods comes out to be the same.
1 2 eV This confirms that the electrons are diffracted in the same
∴ eV = mv2 or v =
2 m way as de-Broglie waves.
Now, de-Broglie wavelength associated with moving
electron is given by : 10. THOMSON’S EXPERIMENT ON ELECTRON
DIFFRACTION
h h h
λ= = =
mv 2eV 2meV Electron diffraction
m
m rings

Putting h = 6.62 × 10–34 J s, m = 9.1 × 10–31 kg and G


e = 1.6 × 10–19 C, we get
F
6.62 × 10–34
λ= m
2 × 9.1 × 10 –31 × 1.6 × 10–19 × V Thin
platinum
12.27 × 10 –10
Photographic
12.27 foil
λ= m or λ= Å Electron gun plate
V V
9. DAVISSON AND GERMER EXPERIMENT Fig. 7

It has been observed 11. DE-BROGLIE WAVES ASSOCIATED WITH


that the scattered electron cur- ATOMS AND MOLECULES
rent (or detector current) is F The maximum intensity was obtained at angle θ given
maximum when the following by
relation given by Bragg for X-
ray diffraction is satisfied.
D 2d sin θ = nλ
2d sin θ = nλ The de-Broglie wavelength of helium molecule was
θ calculated. An excellent agreement was noted between the
where n is order of diffraction,
d is the atomic spacing be- observed and calculated values.
tween successive crystal C 12. PRACTICAL APPLICATION OF DE-BROGLIE
planes and θ is the angle at WAVES (THE ELECTRON MICROSCOPE)
which strong reflection takes The wavelength of de-Broglie waves associated with
place. Fig. 5
an electron is given by
The value of d, as de-
termined by X-ray reflection h
λ=
by nickel crystal, comes out to 2meV
Detector current

be 0.91 Å. The value of θ comes Substituting the values of h, m and e, we get


out to be 65°.
For n = 1, 12.3
λ= Å
λ = 2 d sin θ V
= 2 × 0.91 × sin 65° 54 V
where V is the voltage through which an electron is acceler-
= 1.65 Å ...(1) Voltage, V ated. If an electron has an energy of 300 eV, i.e., it is acceler-
We know that Fig. 6 ated through a potential difference of 300 volt, then

h 12.3
λ= Å = 0.71 Å
λ= 2 me V 300
634 COMPREHENSIVE OBJECTIVE PHYSICS

13. POINTS TO REMEMBER h h h


(i) The photoelectric current is inversely proportional λ= = =
p 2mE 2mqv
to the square of the distance of the light source
from the photo-sensitive surface. 12.27
In the case of electron, λ= Å
(ii) The stopping potential depends upon the frequency V
of the incident light and the nature of the mate- 0.286 Å
rial. In the case of proton, λ=
V
(iii) The stopping potential does not depend upon the
intensity of incident light. 0.202
In the case of deuteron, λ= Å
V
1
(iv) eVs = mv2max
2 In the case of α–particle, λ = 0.101 Å
V
1
(v) mv2max = hν– φ0 (xvi) Formula for wavelength of thermal neutrons is :
2
h
λ=
hν φ0 2mkT
(vi) eVs = hν– φ0 or Vs = –
e e
h ILLUSTRATIONS
So, Vs – ν graph is a straight line having slope
e Illustration 1. Two identical photocathodes receive
– φ0 light of frequencies f1 and f2. If the velocities of the photo
and intercept ‘ ’ electrons (of mass m) coming out are respectively are v1
e
and v2 , then
2h(ν – ν0 ) 2hc λ 0 − λ
(vii) vmax =
m
=
m λ λ0 LM 2h ( f OP 1/ 2

(viii) Energy of photon, E = hν =


hc
(a) v1 – v2 =
N m2h 1 – f2 )
Q
λ (b) v12 – v22 = ( f1 – f2 )
m
E hν
(ix) Mass of photon, m = 2 = 2 =
h
(c) v1 + v2 =
2hLM 1/ 2
OP
c c λc
mc2 E hν h
m N
( f1 + f2 )
Q
(x) Momentum of photon, p = mc = = = = 2h
c c c λ (d) v1 + v2 =
2 2
( f1 + f2 ) [AIEEE 2003]
m
h hc h Sol. According to Einstein’s photoelectric equation,
(xi) Wavelength of photon, λ = = =
p E mc
1
hc m v2max = hν – φ0
(xii) The energy of a photon is given by E = hν = 2
λ
In the given problem
where h(= 6.62 × 10–34 J s) is Planck’s constant, ν
1
is the frequency of the photon, λ is the wavelength m v12 = hf1 – φ0
of the photon and c is the velocity of light. 2
1
(xiii) Eλ = hc m v22 = hf2 – φ0
2
6.6 × 10–34 × 3 × 108 × 1010 1
Eλ (in Å eV) = Now, m (v12 – v22) = h (f1 – f2)
1.6 × 10–19 2
= 12375 2h
or v12 – v22 = (f1 – f2)
12375 m
Wavelength (in Å) =
Energy in eV So, (b) is the right choice.
(xiv) Wavelength associated with moving particle, Illustration 2. When green light is incident on a
certain metal surface, electrons are emitted but no electrons
h h h
λ= = = are emitted by yellow light. If red light is incident on the
p mv 2mE same metal surface, then
(xv) When a charge q is accelerated through a poten- (a) more energetic electrons will be emitted
tial difference V, the wavelength associated with
(b) less energetic electrons will be emitted
the charged particle is given by
DUAL NATURE OF MATTER AND RADIATIONS 635

(c) emission of electrons will depend on the intensity h φ


of light Sol. eVs = hν – φ0 or Vs = ν– 0
e e
(d) no electrons will be emitted. [MP PET 2000]
Comparing with y = mx + c,
Sol. Red light photons are weaker than yellow light We find that
photons. So, no electrons would be emitted with red light.
h
tan θ =
V I B G Y O R e
✔ ✕ ✕ ✕ So, (a) is the right choice.
Illustration 6. The energy that should be added to
So, (d) is the right choice. an electron to reduce its de-Broglie wavelength from 10–10 m
Illustration 3. Photons of energy 6 eV are incident to 0.5 × 10–10 m will be
on a metal surface whose work function is 4 eV. The mini- (a) four times the initial energy
mum kinetic energy of the emitted photo-electrons will be (b) equal to the initial energy
(a) 0 eV (b) 1 eV (c) twice the initial energy
(c) 2 eV (d) 10 eV. [MP PET 2001] (d) thrice the initial energy. [KCET 2000]
Sol. Electrons are emitted with kinetic energies vary-
ing from 0 eV to (6–4) eV i.e. 2 eV. So, the minimum kinetic h
Sol. λ=
energy is zero. 2mE
So, (a) is the right choice. λ would be halved if E becomes four times. Clearly,
Illustration 4. For the photoelectric effect, the maxi- the energy to be added is thrice the initial energy. .
mum kinetic energy Ek of the emitted photo-electrons is So, (d) is the right choice.
plotted against the frequency ν of the incident photons as
Illustration 7. In photoelectric effect, the number
shown in the Fig. 8. The slope of the curve gives
of electrons ejected per second is
(a) proportional to the wavelength of light
(b) proportional to the intensity of light
(c) proportional to the work function of the metal
Ek (d) proportional to the frequency of light.
[KCET 2003]
Sol. Photoelectric current is proportional to the in-
ν tensity of light.
So, (b) is the right choice.
Fig. 8
Illustration 8. Sodium and copper have work func-
(a) charge of the electron tions 2.3 eV and 4.5 eV respectively. Then the ratio of the
(b) work function of the metal wavelengths is nearest to
(c) Planck’s constant (a) 1 : 2 (b) 4 : 1
(d) ratio of the Planck’s constant to electronic charge.
(c) 2 : 1 (d) 1: 4 [AIEEE 2002]
[MP PET 2001]
Sol. Ek = hν – φ0 hc
Sol. φ0 = hν0 =
Comparing with y = mx + c, we find that the slope of λ0
the curve gives Planck’s constant 1
So, (c) is the right choice. λ0 α
φ0
Illustration 5. In a photoelectric experiment, the
stopping potential Vs is plotted against the frequency ν of λ1 φ02 λ 1 4.5
= or = ≈ 2
incident light. The resulting curve is a straight line which λ 2 φ01 λ 2 2.3
makes an angle θ with the ν–axis. Then tan θ will be equal So, (c) is the right choice.
to (φ= work function of surface)
Illustration 9. There are n1 photons of frequency ν1
(a) h/e (b) e/h in a beam of light. In an equally energetic beam, there are
(c) – φ/e (d) eh/φ. [MP PMT 2000] n2 photons of frequency ν2. Then the correct relation is
636 COMPREHENSIVE OBJECTIVE PHYSICS

n1 ν1 n1 Illustration 12. If the frequency of light incident on


(a) = (b) =1
n2 ν 2 n2 a metallic plate be doubled, will the kinetic energy of elec-
trons be also doubled?
n1 ν1 2 n1 ν 2
(c) = (d) = . [KCET 2003] Sol. E = hν – φ0; E′ = 2 hν – φ0
n2 ν 2 2 n2 ν1
E′ 2 hν – φ0 E′
n1 ν2 = ; Clearly > 2 or E′ > 2E
Sol. n1 hν1 = n2 hν2 or = E h ν – φ0 E
n2 ν1
So, (d) is the right choice. So, the kinetic energy will be more than doubled.
Illustration 10. A photoelectric cell is illuminated Illustration 13. Prove that
by a point source of light 1 m away. When the source is the product of the slope of ν – Vs
graph and electronic charge gives
shifted to 2 m, then
the value of Planck’s constant. Vs
(a) each emitted electron carries one quarter of the ∆Vs
Sol. We know that θ
initial energy ∆ν
(b) number of electrons emitted is half the initial 1
m vmax2 = hν – W0
number 2
ν0 ν
(c) each emitted electron carries half the initial energy 1
But m vmax2 = eVs Fig. 9
(d) number of electrons emitted is a quarter of the 2
initial number. [All India PM/PD 2003] ∴ eVs = hν – W0 ... (1)
Sol. When distance is doubled, the intensity is re- Let the stopping potential be increased by an amount
duced by a factor of 4. So, the number of electrons becomes a ∆Vs when frequency is increased by an amount ∆ν.
quarter of the initial number. Then,
So, (d) is the right choice. e (Vs + ∆Vs) = h (ν + ∆ν) – W0 ...(2)
Illustration 11. Find the number of photons in 6.62 J Subtracting (1) from (2), we get
of radiation energy of frequency 1012 Hz. ∆Vs
e∆Vs = h∆ν or e =h
E 6.62 ∆ν
Sol. n= = = 1022
hν 6.62 × 10 –34 × 1012 or e × slope of ν – Vs graph = Planck’s constant.
DUAL NATURE OF MATTER AND RADIATIONS 637

QUESTION BANK

MCQs
SET I

based on
Memory Work, Hard Facts, Important Terms,
Important Formulae etc.

Average time allowed per question is 20 seconds.

1. A radiation of energy E falls normally on a perfectly 7. When a beam of light is used to determine the posi-
reflecting surface. The momentum transferred to the tion of an object, the maximum accuracy is achieved
surface is if the light is
(a) Ec (b) 2E/c (a) polarised (b) of longer wavelength
(c) E/c (d) E/c 2
[AIEEE 2004] (c) of shorter wavelength (d) of high intensity
2. According to Einstein’s photoelectric equation, the [AIIMS 2003]
plot of the kinetic energy of the emitted photo elec- 8. The satellites in the orbit are coated with platinum
trons from a metal Vs the frequency, of the incident due to its
radiation gives a straight line whose slope (a) low work function (b) high work function
(a) depends both on the intensity of the radiation and the metal (c) low melting point (d) low cost.
used
9. The work function of a metal is 1.65 eV. The thresh-
(b) depends on the intensity of the radiation
old wavelength is
(c) depends on the nature of the metal used
(a) 2000 Å (b) 8000 Å
(d) is the same for all metals and independent of the intensity
of the radiation [AIEEE 2004] (c) 4000 Å (d) 7500 Å.

3. The phenomenon inverse to photoelectric effect is : 10. Of the following properties, the photon does not
(a) Compton effect (b) Raman effect
possess
(a) rest mass (b) momentum
(c) pair production (d) production of X-rays
(e) production of electrons from a heated filament (c) energy (d) frequency. [BHU 1997]
[Kerala PMT 2003] 11. Photoelectric effect was discovered by
4. What wavelength must electromagnetic radiation (a) Hertz (b) Hallwach
have if a photon in the beam is to have the same (c) Lenard (d) Millikan. [BHU 2003]
momentum as an electron moving with speed v ? 12. If the distance of 100 watt lamp is increased from a
h photo cell, the saturation current ‘i’ in the photocell
(a) (b) mvh
mv varies with the distance d as
(c) mvr (d) Not possible. (a) i α d2 (b) i α d
5. The numerical value of hc is 1 1
(c) i α (d) i α . [AFMC 2000]
(a) 12.4 eVÅ (b) 12375 eVÅ d d2
(c) 931 MeVÅ (d) 1.6 eVÅ. 13. A cathode ray tube is operated at 10 kV. What will be
6. In a positive ray discharge tube, the pressure is of the energy of cathode rays in eV ?
the order of (a) 10 keV (b) 100 keV
(a) 10–3 cm of mercury (b) 10–7 cm of mercury (c) 1 keV (d) 0.1 keV.
(c) 1 cm of mercury (d) 0.1 cm of mercury. 14. It takes 4.2 eV to remove one of the least tightly bound
electrons from a metal surface. When UV photons of
638 COMPREHENSIVE OBJECTIVE PHYSICS

a single frequency strike a metal, electrons with ki- (a) 4 hν0 (b) 3 hν0
netic energies ranging from 0 to 2.6 eV are ejected. hν0 hν0
The energy of incident photon is (c) (d) .
4 2
(a) 2.6 eV (b) 6.8 eV 24. A photosensitive plate is illuminated by green light
(c) 13.6 eV (d) 13.6/4 eV. and photoelectrons are emitted with maximum kinetic
15. Light of wavelength 6200 Å falls on a metal having energy 4 eV. If the intensity of the incident radiation
photoelectric work function 2 eV. What is the value is reduced to one-fourth of the original value, then
of stopping potential ? the maximum K.E. of the photoelectrons will be
(a) 0 V (b) 2 V (a) 0.1 eV (b) 1 eV
(c) 3 V (d) 1.2 heV. (c) 4 eV (d) 16 eV.
16. Light of frequency 4.5 times the threshold frequency 25. According to wave theory of light, photo-emission
is incident on photo-sensitive material. If the fre- would take place at a frequency
quency is halved and intensity is doubled, then the
(a) < ν0 (b) > ν0
photo-current becomes
(a) quadrupled (b) doubled (c) ν0 (d) Any of these.

(c) halved (d) zero. 26. Which of the following is true for discharge of elec-
tricity through a gas ?
17. The free electrons from a metal surface can be taken
out by the process (a) All positive particles have same charge.
(a) Thermionic emission (b) Field emission (b) All negative particles have same charge.
(c) Photoelectric emission (d) Any of the above method. (c) All positive particles in a given gas have always the same
18. Dual nature of radiation is shown by mass.
(a) Diffraction and reflection (b) Refraction and diffraction (d) All positive particles have the same speed.
(c) Photoelectric effect alone [National Standard Exam. in Physics 1995]
(d) Photoelectric effect and diffraction. [MP PET 1991] 27. Consider light of given intensity and frequency fall-
19. In a photoelectric cell, illuminated with a certain ra- ing on a substance that emits photoelectrons. The
diation, the minimum negative anode potential with intensity is decreased to one-third its value and the
respect to emitting metal required to stop the elec- frequency increased by three times. Consequently the
tron is 2 V. The maximum kinetic energy of photo- velocity of the photoelectrons will
electrons is (a) increase (b) decrease
(a) 0 eV (b) 1 eV (c) increase or decrease depending on the exact values of the
(c) 2 eV [AFMC 2003]
(d) 4 eV. new intensity and frequency
20. Neglecting variation of mass with velocity, the wave- (d) remain the same.
length associated with an electron having a kinetic
28. The mass of a photon at rest is
energy E is proportional to
(a) 1 a.m.u. (b) 1.67 × 10–35 kg
(a) E1/2 (b) E
(c) 9 × 10–31 kg (d) Zero.
(c) E–1/2 (d) E–2 [CMC LDH 2003]
29. For a given speed, which of the following has the
21. Light of wavelength λ falls on a metal having work
shortest de-Broglie wavelength ?
function hc/λ0. Photoelectric effect will take place only
(a) electron (b) proton
if
(a) λ ≥ λ0 (b) λ ≥ 2λ0 (c) deutron (d) α-particle.

(c) λ ≤ λ0 (d) λ < λ 0/2. 30. Consider electrons and protons accelerated in a
vacuum tube across the same potential difference.
22. The energy of a photon corresponding to visible light
of maximum wavelength is nearly Which of the following is true ?
(a) 1 eV (b) 1.6 eV (a) protons have smaller momentum

(c) 3.2 eV (d) 7 eV. (b) electrons have smaller velocity

23. The photoelectric threshold frequency of a metal is (c) protons have larger kinetic energy
ν0 . When light of frequency 4ν 0 is incident on the (d) protons and electrons have same kinetic energy.
metal, the maximum kinetic energy of the emitted [National Standard Exam. in Physics 1995]
photoelectrons is
DUAL NATURE OF MATTER AND RADIATIONS 639
31. The velocity of different ions of same mass and charge 39. For a given kinetic energy, which of the following
in a positive ray will vary because has the smallest de-Broglie wavelength ?
(a) they are accelerated to different extent (a) electron (b) proton
(b) some ions are attracted and others are retarded (c) α-particle. [MP PMT 1999]
(c) they have different initial velocities 40. The slope of graph showing variation of frequency of
(d) they are accelerated through different distances. incident light with stopping potential for a given
32. Which of the following is correct ? surface will be
(a) A point mass, whether at rest or in unifrom motion, can- h
(a) h (b)
not be accompanied by matter waves. e
(b) Only a charged particle at rest is accompanied by matter (c) eh (d) e.
waves. 41. If in a photoelectric experiment, the wavelength of
(c) Only an accelerated charged particle is accompanied by incident radiation is reduced from 6000 Å to 4000 Å,
matter waves. then
(d) A moving particle, whether charged or uncharged, is al- (a) stopping potential will decrease.
ways accompanied by matter waves. (b) stopping potential will increase.
33. Graph of maximum kinetic energy of the photo-elec- (c) kinetic energy of emitted electrons will decrease.
trons against ν, the frequency of the radiation inci-
(d) the value of work function will decrease.
dent on the metal, is a straight line of slope equal to
(a) work function (b) stopping potential 42. An electron is moving on a circular path of radius r
with speed v in a transverse field B. e/m for it will be
h
(c) (d) h. v B
e (a) (b)
Br rv
34. Mass of a photon of frequency ν is given by vr
hν h (c) Bvr (d) . [MP PMT 1999]
(a) m = (b) m = B
c λ 43. An electron and a proton enter a magnetic field per-
hν hν2 pendicularly. Both have same kinetic energy. Which
(c) m = 2 (d) m = .
c c of the following is true ?
[Karnataka 1999] (a) Trajectory of electron is less curved.
35. A photon behaves as if it had a mass equal to (b) Trajectory of proton is less curved.
hν hν (c) Both trajectories are equally curved.
(a) (b)
c c2 (d) Both move on straight line path. [MP PET 1999]
c2 44. Which of the following graphs correctly represents
(c) (d) hνc.
hν the variation of particle momentum with associated
36. A point source of light is used in a photoelectric ef-
de-Broglie wavelength ?
fect. If the source is removed farther from the emit-
ting metal, the stopping potential
(a) will increase (b) will decrease
(c) will remain constant
p p
(d) will either increase or decrease. [JIPMER 2003]
37. A photo-sensitive material would emit electrons if
excited by photons beyond a threshold. To overcome l l
threshold, one would increase (a) (b)
(a) frequency of light (b) intensity of light
(c) wavelength of light
(d) voltage applied to light source.
38. An electron and a proton have the same de-Broglie p p
wavelength. Then the kinetic energy of the electron
is
(a) Zero (b) infinity l l
(c) (d )
(c) equal to the kinetic energy of the proton
Fig. 10
(d) greater than the kinetic energy of the proton.
640 COMPREHENSIVE OBJECTIVE PHYSICS

45. An electron is moving with velocity v. It enters a 51. A tiny particle of mass 10–13 kg moving with a veloc-

region of uniform magnetic flux density B , and uni- ity of 10 cm s –1 is associated with a wave of wave-
→ → → length
form electric field E . E and B are mutually perpen-
(a) 6.62 × 10–18 cm (b) 6.62 × 10–12 cm
dicular. The velocity v will not remain constant un-
→ (c) 6.62 × 10 cm –2
(d) 6.62 × 10–34 cm.
less v is
→ → 52. The photo-electrons emitted from a surface of sodium
(a) perpendicular to E and parallel to B
→ → metal are such that
(b) perpendicular to B (c) parallel to E (a) they all are of the same frequency
→ → B
(d) perpendicular to E and to B and of magnitude (b) they have the same kinetic energy
E
→ → E (c) they have the same de-Broglie wavelength
(e) perpendicular to E and to B and of magnitude . (d) they have their speeds varying from zero to a certain maxi-
B
[London Schools Examination Department] mum. [MP PMT 1992]
46. In Compton scattering relative to original photon, 53. The study of discharge of electricity through gases
the photon scattered from a collision with a free elec- led to the discovery of
tron has higher (a) structure of atom (b) spectral lines
(a) energy (b) momentum (c) proton (d) electron.
(c) frequency (d) wavelength.
54. The photoelectric effect is based on the conservation
[MANIPAL 1998]
of
47. Emission of electrons from a metal plate illuminated (a) mass (b) energy
with monochromatic electromagnetic radiation will
(c) momentum (d) angular momentum.
always take place provided
(a) the radiation is sufficiently intense. [MAHE 1997]
(b) the work function of the plate is less than the energy of a 55. An ultraviolet light source causes the emission of
single photon and the plate is uncharged. photoelectrons from a zinc plate. A more intense
(c) the wavelength of the radiation exceeds a minimum value. source of the same wavelength would give
Maximum No. of electrons/
(d) the plate is always negatively charged.
energy/electron second
(e) the plate is freshly cleaned.
(a) more the same
[London Schools Examination Department]
(b) the same more
48. The stopping potentials are V1 and V2 with incident (c) the same the same
lights of wavelengths λ1 and λ 2 respectively. Then (d) more more
V1 – V2 = (e) less more
hc λ 1λ 2 hc 1F 1 I [Scottish Examination Board]
(a)
e λ1 – λ2
(b) GH

e λ1 λ2 JK 56. Threshold wavelength for a metal is 5200 Å. Photo-
electrons will be ejected if it is irradiated by a light
he 1F 1 I he from
(c) GH

c λ1 λ2
JK (d)
cλ 1 λ 2
(λ 1 – λ 2 ) .
(a) 50 watt infrared lamp (b) 1 watt infrared lamp
49. The de-Broglie wavelength λ of a particle with mass (c) 50 watt ultraviolet lamp (d) 0.5 watt infrared lamp.
m and kinetic energy E is given by [IIT 1982 ; MP PMT 1992]
2mE 57. A photon of frequency ν has a momentum associated
(a) λ = h 2mE (b) λ =
h with it. If c is the velocity of light, this momentum is
h hE hν hν
(c) λ = (d) λ = (a) (b)
2 c
2mE 2mE c
ν
[AIIMS 2001] (c) (d) hνc.
c
50. The photoelectric effect can be understood on the basis
of 58. The momentum of an electron of wavelength 100 Å
(a) wave theory of light only is
(b) electromagnetic theory of light only (a) 6.6 × 1026 kg m s–1 (b) 6.6 × 10–23 kg m s–1
(c) quantum theory of light only
(c) 6.6 × 10–26 kg m s–1 (d) 6.6 × 1023 kg m s–1.
(d) None of the above.
DUAL NATURE OF MATTER AND RADIATIONS 641

59. In Einstein’s photoelectric equation (Ek = hν – φ0), Ek frequency is halved and intensity is doubled, the
refers to photo current becomes
(a) kinetic energy of all the emitted electrons. (a) quadrupled (b) doubled
(b) maximum kinetic energy of the emitted electrons. (c) halved (d) zero. [EAMCET 1991]
(c) mean kinetic energy of the emitted electrons. 67. The best metal to be used for photoemission is
(d) minimum kinetic energy of the emitted electrons. (a) potassium (b) sodium
[All India PM/PD 1997] (c) cesium (d) lithium.
60. The electrons are emitted in the photoelectric effect [EAMCET 2001]
from a metal surface 68. If the frequency of light in a photoelectric experiment
(a) only if the frequency of the incident radiation is above a is doubled, the stopping potential will
certain threshold value. (a) be doubled (b) be halved
(b) only if the temperature of the surface is high. (c) become more than double
(c) at a rate that is independent of the nature of the metal. (d) become less than double. [BHU 2002]
(d) with a maximum velocity proportional to the frequency of
69. The photons in a radio wave of wavelength 3 × 104 cm
the incident radiation. [MP PET 1992]
have an energy
61. If the wavelength of light incident on the surface of a (a) 6.6 × 10–28 J (b) 6.6 × 10–40 J
metal is decreased, the maximum velocity of emitted
(c) 19.86 × 10–34 J (d) 2.2 × 10–38 J.
photo-electrons will
(a) decrease (b) increase 70. Light of frequency ν is incident on a substance of
threshold frequency ν0 (ν 0 < ν). The energy of the
(c) not change
emitted photo-electron will be
(d) initially decrease and then increase.
(a) h (ν – ν0) (b) h/ν
62. Light of wavelength 4000 Å is incident on a metal
(c) he (ν – ν0) (d) h/ν0. [MP PET 2000]
plate whose work function is 2 eV. The maximum
71. A photon of energy hν is absorbed by a free electron
kinetic energy of the emitted photoelectrons would
of a metal having work function φ < hν
be
(a) The electron is sure to come out.
(a) 2.0 eV (b) 1.5 eV
(b) The electron is sure to come out with a kinetic energy
(c) 1.1 eV (d) 0.5 eV. [BHU 1996] hν – φ.
63. The de-Broglie wavelength of an electron in the nth (c) Either the electron does not come out or it comes out with
Bohr orbit is related to the radius R of the orbit as a kinetic energy hν – φ.
(a) n λ = n R (b) nλ = 3/2π R (d) It may come out with a kinetic energy less than hν – φ.
(c) n λ = 2π R (d) n λ = 4π R.
[EAMCET 2003]
[Bharati Vidyapeeth 2000]
72. The retarding potential for having zero photo-elec-
64. A source of light is placed at a distance of 1m from a
tron current
photo cell and cut off potential is found to be V0 . If
(a) is proportional to the wavelength of incident light
the distance is doubled, the cut off potential will be
(b) increases uniformly with the increase in the wavelength
V0
(a) 2 V0 (b) of incident light
2
V0 (c) is proportional to the frequency of incident light
(c) V0 (d) [CMC LDH 2000]
4 (d) increases uniformly with the increase in the frequency of
65. The equation E = pc is valid incident light wave. [MP PMT/PET 1988]
(a) for an electron as well as for a photon. 73. When light falls on a metal surface, the maximum
(b) for an electron but not for a photon. kinetic energy of the emitted photo-electrons depends
(c) for a photon but not for an electron. upon
(d) neither for an electron nor for a photon. (a) the time for which light falls on the metal
[Pb. PMT 2003] (b) frequency of the incident light
66. Light of frequency 1.5 times the threshold frequency (c) intensity of the incident light
is incident on photosensitive material. If the (d) velocity of the incident light. [MP PET 1992, 93]
642 COMPREHENSIVE OBJECTIVE PHYSICS

74. In a series of photoelectric y (e) the reduction in radiation intensity with distance from a
emission experiments on a cer- point source.
tain metal surface, possible re- 79. Which graph shows how the energy E of a photon of
lationships between the follow- light is related to its wavelength λ ?
ing quantities were investi-
gated : threshold frequency fo ,
frequency of incident light fo , E E
light intensity P, photocurrent 0
I, maximum kinetic energy of 0 x
photoelectrons Tmax. Fig. 11
Two of these quantities, when plotted as a graph of y
against x, give a straight line through the origin. O O
0 λ 0 λ λ
Which of the following correctly identifies x and y (a) (b)
with the photoelectric quantities ?
x y E E
(a) I fo
(b) f T max
(c) P I
(d) P T max
O O
75. J.J. Thomson’s cathode-ray tube experiment dem- 0 (c) λ 0 (d ) λ
onstrated that
Fig. 12
(a) cathode rays are streams of negatively charged ions.
(b) all the mass of an atom is essentially in the nucleus. 80. Which curve shows the relationship between the
(c) the e/m of electrons is much greater than the e/m of protons. energy E and the wavelength λ of a photon of electro-
magnetic radiation ?
(d) the e/m ratio of the cathode-ray particles changes when a
different gas is placed in the discharge tube.
[All India PM/PD 2003] E E E

76. A cesium photocell, with a steady potential differ-


ence of 60 V across it, is illuminated by a bright point
source of light 50 cm away. When the same light is
placed 1 m away, the photoelectrons emitted from O λ O λ O λ
(a) (b) (c)
the cell
(a) are one quarter as numerous E E
(b) are half as numerous
(c) each carry one quarter of their previous momentum
(d) each carry one quarter of their previous energy.
[KCET 2002] O (d ) λ O (e) λ
77. When visible light is incident on a metal surface, no
photoelectrons are emitted. If a second beam is to be Fig. 13
selected, it must be 81. Photon is the name given to
(a) IR radiation (b) Microwave
(a) an electron emitted from a metal surface by the action of
(c) Radio wave (d) UV radiation. light.
[KCET 2002] (b) a unit of energy.
78. Which of the following physical phenomena cannot (c) a positively charged atomic particle.
be described only by the wave theory of electromag-
(d) an electron emitted from a metal surface by the action of
netic radiation ?
heat.
(a) diffraction (b) interference
(e) a quantum of electromagnetic radiation.
(c) photoelectric emission (d) polarisation
DUAL NATURE OF MATTER AND RADIATIONS 643

82. The de-Broglie wavelength of an electron in the first (c) equal to twice the circumference of the first orbit.
Bohr orbit is (d) equal to the circumference of the first orbit.
(a) equal to one-fourth the circumference of the first orbit. [KCET 2000]
(b) equal to half the circumference of the first orbit.

Answers (Set I)
1. (b) 2. (d) 3. (d) 4. (a) 5. (b) 6. (a) 7. (c) 8. (b)
9. (d) 10. (a) 11. (a) 12. (d) 13. (a) 14. (b) 15. (a) 16. (b)
17. (d) 18. (d) 19. (c) 20. (c) 21. (c) 22. (b) 23. (b) 24. (c)
25. (d) 26. (b) 27. (a) 28. (d) 29. (d) 30. (d) 31. (c) 32. (d)
33. (d) 34. (c) 35. (b) 36. (c) 37. (a) 38. (d) 39. (c) 40. (b)
41. (b) 42. (a) 43. (b) 44. (d) 45. (e) 46. (d) 47. (b) 48. (b)
49. (c) 50. (c) 51. (a) 52. (d) 53. (d) 54. (b) 55. (b) 56. (c)
57. (b) 58. (c) 59. (b) 60. (a) 61. (b) 62. (c) 63. (c) 64. (c)
65. (c) 66. (d) 67. (c) 68. (c) 69. (a) 70. (a) 71. (d) 72. (d)
73. (b) 74. (c) 75. (c) 76. (a) 77. (d) 78. (c) 79. (d) 80. (e)
81. (e) 82. (d).

Solutions (Set I)
E 13. W = qV
1. Pi =
c 14. Energy of incident photon
If the surface is perfectly reflecting, then = (4.2 + 2.6) eV = 6.8 eV
E 15. Energy corresponding to 6200 Å
Pf =
c
12375
2E = eV = 1.996 eV ≈ 2eV
Change = 6200
c Clearly, the electron is emitted with zero kinetic
2. According to Einstein’s photoelectric equation energy. So, the stopping potential is zero.
KEmax = hν – φ0
16. Photoelectric current is directly proportional to
Compare it with straight line equation, y = mx + c intensity of incident light.
Clearly, slope of the line in the graph is h, the Planck’s
constant  In these problems, ensure that when the
7. The maximum accuracy in finding the position of an frequency is reduced, it does not fall below
object using a light beam is achieved, when the light the threshold frequency.
beam used is of shorter wavelength. If the wavelength
is longer, the spread in the wave will be more. h h h
20. λ= = =
9. Wavelength in Å mv p 2mE
12375 12375 ∴ λ∝E –1/2
.
= = = 7500 Å
Energy in eV 1.65 1
23. mv2max. = 4hν0 – hν0 = 3 hν0
12. Photoelectric current is directly proportional to 2
intensity of incident light. Also, intensity of light is 24. Intensity has no effect on maximum kinetic energy of
inversely porportional to square of distance. photoelectrons.
644 COMPREHENSIVE OBJECTIVE PHYSICS

30. In vacuum, no collisions are supposed to occur. So, wrongly, you will arrive at the wrong choice. For ex-
the kinetic energies of protons and electrons are the ample, a freshly cleaned plate will only exhibit photo-
same. So, (d) true. (c) is then clearly invalid. For (a) electric emission if the radiation frequency is greater
than a certain minimum value. So, if the radiation
2E
and (b) use p = 2mE and v = . frequency is less than the minimum value (threshold
m frequency), then emission does not occur. So, (e) is
1 out.
33. mv2max = hν – φ0
2 48. eVs = hν – φ0
Compare with y = mx – c. hν
Vs = – φ0
36. When the position of source of light is changed, the e
intensity of light incident on the metal will change. hc
Vs = – φ0
Intensity has no effect on stopping potential. eλ
37. For photoelectric effect, the frequency of incident light hc
V1 = – φ0
should be greater than threshold frequency. eλ 1
h hc
38. λ= V2 = – φ0
2mE eλ2
mE = constant hc 1 F 1 I
Mass of electron is less; E more
V1 – V2 = GH

e λ1 λ2
JK
1 h h
39. λ∝ 49. λ= =
m mv p
h h
40. eVs = hν – φ0 or Vs = ν – φ0 λ=
e 2mE
Compare with y = mx – c.
h
41. Wavelength reduced; frequency increased, stopping 51. λ=
mv
potential increased .
6.62 × 10 –34
mv2 e v = × 100 cm
42. Bev = or = 10 –13 × 10 × 10 –2
r m rB
mv = 6.62 × 10–18 cm
43. r= 55. Intensity of incident light is energy per second per
Be
p unit area of the beam. Increased intensity of the same
2mE k
r= or r = wavelength means more photons of the same energy.
Be Be
r∝ m 12375
56. φ0 = eV = 2.3798 eV ≈ 2.4 eV
Proton has greater mass; larger radius of trajectory; 5200
less curved. Energy of infrared lamp is less than 2.4 eV.

 Larger the curvature, lesser is the 57. E = mc2 = hν or mc =
c
radius of curvature.
58. h
λ=
h p
44. λ=
p 6.6 × 10 –34
h
1 p= = kg m s–1
λ∝ λ 100 × 10 –10
p = 6.6 × 10–26 kg m s–1
So, graph between λ and p is a rectangular hyperbola. 62. Energy corresponding to 4000 Å
45. Take the help of Fleming’s Left Hand rule. 12375
= eV = 3.09 eV = 3.1 eV
46. Reduced energy, higher wavelength. 4000
47. The key to the question is in the words “...... will al- Maximum kinetic energy of emitted photoelectrons
ways take place provided”. You may well know all the = (3.1 – 2) eV = 1.1 eV
relevant facts. But if you interpret the question
DUAL NATURE OF MATTER AND RADIATIONS 645
63. Only those orbits are allowed for which the circum- The rate of emission of photoelectrons (i.e. photocurrent)
ference is an integral multiple of λ. depends proportionally on the rate of incident photons.
2πR = nλ 76. When distance is doubled, intensity becomes one-
64. When distance is increased, the intensity is reduced. fourth. So, the number of photoelectrons becomes one-
But intensity has no effect on cut-off potential. fourth.
66. When the given frequency is halved, the frequency 77. UV radiation is more energetic than visible light.
becomes less than threshold frequency. So, no photo- 78. The emission of the photoelectrons from the metal
emission.
surface can only be explained by Einstein’s particle
67. Low work function (photon) theory.
68. eVs = hν – φ0
hc
eVs′ = 2hν – φ0 79. Photon energy, E = hf = in the usual notation,
λ
Vs ′ 2hν – φ 0
= 1
Vs hν – φ0 hence E ∝ .
λ
Clearly, Vs′ > 2 Vs
80. Each photon has associated with it an energy E given
69. λ = 3 × 104 cm = 3 × 104 × 108 Å
by
= 3 × 1012 Å
12375 hc
Now, energy in eV = E = hf =
3 × 10 12 λ
energy in joule where f = frequency of the electromagnetic wave
12375 × 1.6 × 10 –19 radiated.
= = 6.6 × 103 × 10–31
3 × 10 12 λ = wavelength of the electromagnetic
= 6.6 × 10–28 J radiation.

1 1
70. mv2max = hν – hν0 = h (ν – ν0) Thus E∝ since h and c are constants.
2 λ
71. Maximum kinetic energy is hν – φ. 81. By quantum theory of radiation, electromagnetic
74. radiation is assumed to consist of particles called
Photocurrent I
photons, each of which carries a packet of quantum
of energy and the amount of energy carried was equal
to hf, where f is the frequency of the radiation and h
is the Planck’s constant.
Light intensity P Thus, photon is the name given to a quantum of
O electromagnetic radiation.
Fig. 14

KNOWLEDGE PLUS
l A photon of energy 4 eV is incident on a metal surface whose work function is 2 eV. The minimum reverse
potential to be applied for stopping the emission of electron is :
(a) 2 V (b) 4 V (c) 6 V (d) 8 V [AIIMS 2004]
1
Sol. mv2 max = hν – φ0 = (4 – 2) eV = 2 eV
2
When stopping potential is applied, no electron will reach the cathode and the current will becomes zero
1
∴ mv2 max
eV0 = or eV0 = 2 eV ⇒ Stopping potential = 2 V
2
So, (a) is the right choice.
646 COMPREHENSIVE OBJECTIVE PHYSICS

MCQs
based on
SET II
Important Basic Concepts
+
Application of Formulae
+
Brain Teasers

Average time allowed per question is 50 seconds.

83. A proton and an α-particle are accelerated through (a) 0.95 × 107 m s–1 (b) 1.9 × 107 m s–1
the same potential difference. The ratio of de-Broglie (c) 3.78 × 107 m s–1 (d) 5.67 × 107 m s–1.
wavelength of the proton to that of the α-particle will 89. In an experiment on photo-electric effect, stopping
be potential is 1.0 V when light of wavelength 6520 Å is
(a) 2 : 1 (b) 1 : 1 incident on the emitting surface. The stopping po-
tential is 2.9 V for light of wavelength 3260 Å. The
(c) 1 : 2 (d) 2 2 : 1 work function of the metal is
[National Standard Exam. inPhysics 1989] (a) 0.9 eV (b) 1.9 eV
(c) 5.8 eV
84. Light of wavelength λ strikes a photoelectric surface
and electrons are ejected with kinetic energy K. If K (d) Cannot be deduced from the given data.
is to be increased to exactly twice its original value, [National Standard Exam. in Physics 1997]
the wavelength must be changed to λ′ such that 90. The de-Broglie wavelength of an electron of speed
0.5 km s–1 is
λ λ
(a) λ′ < (b) λ′ > (a) 1.5 × 10–6 m (b) 1.5 × 10–8 m
2 2
(c) 1.5 × 10–10 m (d) 1.5 × 10–12 m.
λ λ
(c) λ > λ′ > (d) λ′ = 91. In Millikan’s oil drop experiment, an oil drop of
2 2
radius r and charge Q is held in equilibrium between
85. Lights of two different frequencies whose photons have the plates of a charged parallel plate capacitor when
energies 1eV and 2.5 eV respectively, successively il- the potential difference is V. To keep a drop of radius
luminate a metal of work function 0.5 eV. The ratio 2r and charge 2Q in equilibrium between the plates,
of maximum speeds of the emitted electrons will be the potential difference V′ required is
(a) 1 : 5 (b) 1 : 4 (a) 8 V (b) 4 V
(c) 1 : 2 (d) 1 : 1. (c) 2 V (d) V.
86. If E1, E2 and E3 are the respective kinetic energies of 92. The de-Broglie wavelength of a 1 kg object moving
an electron, an alpha particle and a proton, each hav- with speed of 1 m s–1 is
ing the same de-Broglie wavelength, then
(a) 6.6 × 10–34 m (b) 6.0 × 10–34 m
(a) E1 > E3 > E2 (b) E2 > E3 > E1
(c) 6.6 × 10–10 m (d) 6.0 × 10–10 m.
(c) E1 > E2 > E3 (d) E1 = E2 = E3.
93. The diagram shows a charged oil drop between two
87. If h is Planck’s constant, then the momentum of a horizontal plates connected to a high voltage source
photon of wavelength 0.01 Å is V. Which of the following statements is/are correct ?
(a) 10–2 h (b) h 1. If the drop is stationary, then it must carry a
(c) 102 h (d) 1012 h. positive charge.
88. An electron is accelerated by a potential difference of 2. If the drop is falling at ‘terminal velocity’, then a
1000 volt. Its velocity will be resultant force must be acting on it.
DUAL NATURE OF MATTER AND RADIATIONS 647

2me
(a) 1 (b)

+ me
me
(c) (d) . [MNR 1994]
mα 2mα
+
98. When radiation is incident on a photoelectron emit-
Oil drop V
ter, the stopping potential is determined to be 9 V.
– Assuming the charge to mass ratio e/m for electron

to be 1.8 × 10 11 C kg –1, the maximum velocity of
ejected electron is found to be in m s–1
(a) 1.8 × 106 (b) 6 × 105
(c) 8 × 105 (d) 10 × 105.

Fig. 15 99. We wish to see inside an atom. Assuming the atom


to have a diameter of 100 pm, this means that one
3. If the drop is stationary and then it suddenly starts must be able to resolve a width of say 10 pm. If an
to rise, then it must have gained extra electrons. electron microscope is used, the minimum electron
(a) 1 only (b) 2 only energy required is about :
(c) 3 only (d) 1 and 2 only (a) 1.5 keV (b) 15 keV
(e) 2 and 3. [Oxford Local Examinations] (c) 150 keV (d) 1.5 MeV [AIIMS 2004]
94. Fig. 16 shows the varia- 100. What will be the frequency of the photons having
tion of photocurrent with c energy 75 eV ?
anode potential for a (a) 1.81 × 10–15 Hz (b) 18.2 × 1015 Hz
current
Photo

b a
photo-sensitive surface for (c) 181 × 10–15 Hz (d) None of the above.
three different radiations.
101. A metal surface of work function 1.07 eV is irradi-
Let I a , I b and I c be the
ated with light of wavelength 332 nm. The retarding
intensities and fa , fb and O Anode potential potential required to stop the escape of photo-elec-
fc be the frequencies for
Fig. 16 trons is
the curves a, b and c re-
(a) 4.81 eV (b) 3.74 eV
spectively.
(a) fa = fb and Ia ≠ Ib (b) fa = fc and Ia = Ic (c) 2.66 eV (d) 1.07 eV. [MP PMT 1992]

(c) fa = fb and Ia = Ib (d) fb = fc and Ib = Ic 102. The mass of a particle is 400 times than that of an
electron and the charge is double. The particle is ac-
[IIT Screening 2004]
celerated by 5 V. Initially the particle is at rest. Then
95. The energy of a photon is equal to the kinetic energy its final kinetic energy will be
of a proton. The energy of the photon is E. Let λ1 be (a) 5 eV (b) 10 eV
the de-Broglie wavelength of the proton and λ2 be the
(c) 100 eV (d) 2000 eV.
wavelength of the photon. The ratio λ 1 /λ 2 is
[MP PMT 1990]
proportional to
(a) E° (b) E1/2 103. 5 V of stopping potential is needed for the photo elec-
trons emitted out of the surface of work function
(c) E–1
(d) E–2
2.2 eV by the radiation of wavelength
[IIT Screening 2004]
(a) 1719 Å (b) 3444 Å
96. A photon of energy 3.4 eV is incident on a metal having (c) 861 Å (d) 3000 Å.
work function 2 eV. The maximum K.E. of photo-
104. The work function of tungsten is 4.5 eV. The thresh-
electrons is equal to
old frequency is
(a) 1.4 eV (b) 1.7 eV
(a) 1.09 × 1015 Hz (b) 0.91 × 1015 Hz
(c) 5.4 eV (d) 6.8 eV. [MP PET 1991] (c) 1.09 × 10–15 Hz (d) 1 × 1016 Hz.
97. The ratio of momenta of an electron and an α-parti- 105. If a surface has work function of 3.00 eV, the longest
cle which are accelerated from rest by a potential wavelength of light which will cause the emission of
difference of 100 V is electrons is
648 COMPREHENSIVE OBJECTIVE PHYSICS

(a) 4.876 × 10–7 m (b) 5.998 × 10–7 m (a) 0.3 V (b) 0.6 V
(c) 4.125 × 10–7 m (d) 6.847 × 10–7 m. (c) 1.2 V (d) 2.4 V.
[AMU 2003] [MP PMT 1994]
106. A proton when accelerated through a potential dif- 113. Work function of nickel is 5.01 eV. When ultraviolet
ference of V volt has a wavelength λ associated with radiation of wavelength 2000 Å is incident on it,
it. An α-particle, in order to have the same λ,must be
electrons are emitted. What will be the maximum
accelerated through a potential difference of
velocity of emitted electrons ?
(a) V volt (b) 4V volt
(a) 3 × 108 m s–1 (b) 6.46 × 105 m s–1
V
(c) 2V volt (d) volt. (c) 10.36 × 105 m s–1 (d) 8.54 × 106 m s–1.
8
114. In Fig. 17, A and B are two horizontal metal plates
107. An electron of mass m when accelerated through a
separated by insulating plugs. The region between A
potential difference V has de-Broglie wavelength λ .
The de-Broglie wavelength associated with a proton and B is evacuated. An electron of mass ‘m’ remains
of mass M accelerated through the same potential at rest inside the chamber. If a proton of mass “M”
difference will be replaces electron under similar conditions, then the
proton
m m
(a) λ (b) λ
M M

M M
(c) λ (d) λ . A
m m
[All India PM/PD 1995]
108. The momentum of a photon is 2 × 10–16 g cm s–1. Its e V
energy is
(a) 0.61 × 10–26 erg (b) 2.0 × 10–26 erg
B
(c) 6 × 10–6 erg (d) 6 × 10–8 erg.
109. The kinetic energy of the electron is E when the inci-
dent wavelength is λ . To increase the K.E. of the
electron to 2E, the incident wavelength must be Fig. 17
(a) 2 λ (b) λ/2 (a) continues to remain at rest
hcλ hcλ eE
(c) (d) . (b) acquires an acceleration in the upward direction
E λ + hc Eλ − hc m
110. The radiations of 500 nm liberate photo-electrons from FG Mg – eE IJ in the downward
the cesium surface of work function 1.8 eV. The en-
(c) acquires an acceleration
H M K
direction
ergy of electron is F m + M IJ g in the downward
(d) acquires an acceleration GH
(a) 3.28 eV (b) 1.36 eV M K
(c) 0.68 eV (d) 0.6 eV. direction.
[National Standard Exam. in Physics 1997]
111. The work function of a metal is 1.6 × 10–19 J. When
the metal surface is illuminated by the light of wave- 115. The work functions for tungsten and sodium are
4.5 eV and 2.3 eV respectively. If the threshold
length 6400 Å, then the maximum kinetic energy of
wavelength λ for sodium is 5460 Å, the value of λ for
emitted photo-electrons will be (Planck’s constant
tungsten is
h = 6.4 × 10–34 Js)
(a) 5893 Å (b) 10683 Å
(a) 15 × 10–19 J (b) 2.8 × 10–19 J
(c) 2791 Å (d) 528 Å . [MP PET 1990]
(c) 1.5 × 10–19 J (d) 1.5 × 10–19 eV.
116. Light of wavelength 4000 Å falls on a photosensitive
[MP PMT 1989] metal and a negative 2 V potential stops the emitted
112. When the light source is kept 20 cm away from a electrons. The work function of the material (in eV)
photo cell, stopping potential 0.6 V is obtained. When is approximately
source is kept 40 cm away, the stopping potential (h = 6.6 × 10–34 Js, e = 1.6 × 10–19C, c = 3 × 108 m s–1
will be
DUAL NATURE OF MATTER AND RADIATIONS 649
(a) 1.1 (b) 2.0 B – into the
plane of paper
(c) 2.2 (d) 3.1 . [MP PET 1995]
117. The radio transmitter operates on a wavelength of
1500 m at a power of 389.6 kilowatt. Then the fre-
quency corresponding to this wavelength is
(a) 500 kHz (b) 200 kHz
–e v
(c) 100 kHz (d) 400 kHz.
[All India PM/PD 2002] E

118. The K.E. of the photoelectrons is E when the inci-


dent wavelength is λ/2. The K.E. becomes 2 E when
the incident wavelength is λ/3. The work function of
the metal is Fig. 18
hc 2 hc
(a) (b) (a) continue to travel undeflected in their original direction
λ λ
(b) be deflected upwards out of the plane of the diagram
3 hc hc
(c) (d) . (c) be deflected downwards into the plane of the diagram
λ 3λ
(d) be deflected upwards in the plane of the diagram
119. Light from a hydrogen discharge tube is made inci-
(e) be deflected downwards in the plane of the diagram.
dent on the cathode of a photoelectric cell. The work
[Cambridge Local Examinations Syndicate]
function of the cathode surface is 3.1 eV. In order to
123. Given that a photon of light of wavelength 10,000 Å
reduce the photoelectric current to zero value, the
has an energy equal to 1.23 eV. When light of wave-
minimum potential applied to anode with respect to
length 5000 Å falls on a photoelectric cell, the satura-
cathode should be
tion current is 0.40 × 10–6 ampere and the stopping
(a) – 3.1 volt (b) + 10.5 volt potential is 1.36 volt. Then the work fucntion is
(c) – 16.7 volt (d) – 10.5 volt. (a) 0.43 eV (b) 1.10 eV
[CMC Vellore 2003] (c) 1.36 eV (d) 2.47 eV.

120. Silver has a work function of 4.7 eV. When ultraviolet 124. Ultraviolet radiations of 6.2 eV falls on an aluminium
surface (work function 4.2 eV). The kinetic energy in
light of wavelength 100 nm is incident upon it, a
joule of the fastest electron emitted is nearly
potential of 7.7 volt is required to stop the
(a) 3.2 × 10–21 (b) 3.2 × 10–19
photoelectrons from reaching the collector plate. The
(c) 3.2 × 10–17 (d) 3.2 × 10–15.
potential required to stop photo electrons when light
[MP PET 1990]
of wavelength 200 nm is incident upon silver is
125. Work function of a metal is 2.51 eV. Its threshold
(a) 1.5 V (b) 1.85 V
frequency is
(c) 1.95 V (d) 2.37 V. [Manipal 2002] (a) 5.9 × 1014 Hz (b) 6.5 × 1014 Hz
121. Let nr and nb be respectively the number of photons (c) 9.4 × 1014 Hz (d) 6.08 × 1014 Hz.
emitted by a red bulb and a blue bulb of equal power Background for Questions 126 to 129
in a given time. Then For photons of wavelength λ in vacuum, the linear
(a) nr = nb (b) nr < nb momentum p is given by
(c) nr > nb p = h/λ
(d) The information is insufficient to get a relation between nr De-Broglie gave the idea that for material particles
and nb [AIIMS 2002] of momentum p, there should be an associated wave
122. Electrons travelling at a velocity of 2.4 × 106 m s–1 whose wavelength is given by λ = h/p.
enter a region of crossed electric and magnetic fields For photons, the energy is given by
as shown in Fig. 18. If the electric field is 3.0 × 106 E = hν
V m–1, and the flux density of the magnetic field is whereas the kinetic energy of particles (with speed
1.5 T, the electrons upon entering the region of the well below c) is related to p through
crossed fields will E = p2/2m.
Now answer the questions given below.
650 COMPREHENSIVE OBJECTIVE PHYSICS

126. λ is proportional to 1
(a) (b) 1
(a) 1/E for both photons and particles 2
(c) 2 (d) 2
(b) 1/E for photons : 1/ E for particles
(e) 4. [Southern Universities Joint Board for
(c) 1/ E for both photons and particles
School Examinations Bristol]
(d) 1/ E for photons ; 1/E for particles. 133. A monochromatic light is used in an experiment on
127. If the de-Broglie wavelength is λo for protons acceler- photoelectric effect. The stopping potential
ated through 100 V, that for alpha particles acceler- (a) is related to the mean wavelength.
ated through the same voltage will be (b) is related to the longest wavelength.
(a) λo (b) λo/2 (c) is related to the shortest wavelength.
(d) is not related to the wavelength.
(c) λo/2 2 (d) λo/4.
134. The collector plate in an experiment on photoelectric
128. If λo stands for mid-wavelength in the visible region, effect is kept vertically above the emitter plate. Light
the de-Broglie wavelength for 100 V electrons is source is put on and a saturation photocurrent is
nearest to recorded. An electric field is switched on which has a
vertically downward direction.
(a) λo/5 (b) λo/50
(a) The photocurrent will increase.
(c) λo/500 (d) λo/5000.
(b) The kinetic energy of the electrons will increase.
129. For a particle of mass m enclosed in a one-dimen-
(c) The stopping potential will decrease.
sional box of length L, the de-Broglie concept would
(d) The threshold wavelength will increase.
lead to stationary waves, with nodes at the two ends.
135. Four elements A, B, C, D, have work functions 2,
The energy values allowed for such a system (with n
2.4, 2.8, 3.2 eV. Light of wave length 4000 Å is incident
as integers) will be
on them. The elements which emit photo electrons
h2 h2 are
(a) 2
n2 (b) n2 (a) A, B, C, D (b) A, B, C
8mL 4 mL
(c) A, B (d) A.
h h2 2 136. Two metal plates having a potential difference of 800 V
(c) n (d) n .
4 mL 4 mL2 are 0.02 m apart horizontally. A particle of mass
[National Standard Exam. in Physics 1991] 1.96 × 10–15 kg is suspended in equilibrium between
the plates. If e is the elementry charge, then charge
130. The threshold frequency for photoelectric effect cor- on the particle is
responds to a wavelength of 5000 Å. Its work func- (a) e (b) 3e
tion is
(c) 6e (d) 8e. [MP PET 1999]
(a) 1 Joule (b) 4 × 10–19 joule
137. An electron accelerated under a potential difference
(c) 3 × 10–19 joule (d) 2 × 10–19 joule. V volt has a certain wavelength λ . Mass of the proton
131. The threshold frequency for certain metal is ν0. When is 2000 times the mass of the electron. If the proton
light of frequency 2ν0 is incident on it, the maximum has to have the same wavelength λ , then it will have
velocity of photoelectrons is 4 × 106 m s–1. If the fre- to be accelerated under a potential difference of
quency of incident radiation is increased to 5ν0, then (a) V volt (b) 2000 V volt
the maximum velocity of photoelectrons will be V
(c) volt (d) 2000 V volt.
2000
(a) 4/5 × 106 m s–1 (b) 2 × 106 m s–1
[Himachal PMT 2002]
(c) 8 × 106 m s–1 (d) 2 × 107 m s–1.
138. In a photoelectric experiment, electrons are ejected
132. A beam of α-particles and protons of the same kinetic
from metals X and Y by light of frequency f. The
energy E enters a uniform magnetic field at right potential difference V required to stop the electrons
angles to the field lines so that the particles bend on is measured for various frequencies. If Y has a greater
circular paths. The ratio of path radius for α-particles work function than X, which graph illustrates the
and path radius for protons is equal to expected results ?
DUAL NATURE OF MATTER AND RADIATIONS 651
If the experiment were repeated with light of twice
V V V
X X Y the intensity but the same wavelength, which of the
X graphs below would best represent the new relation
Y Y
between I and V ? (In these graphs, the result of the
0 0 0 original experiment is indicated by a broken line.)
0 f 0 f 0 f
(a) (b) (c)
I I
V V
Y Y
X X

0 0
0 f 0 f O V O V
(d ) (e)
(a) (b)

Fig. 19 I

139. Two beams P and Q of light of the same wavelength


fall upon the same metal surface causing
photoemission of electrons. The photoelectric current
produced by P is four times that produced by Q.
Which of the following gives the ratio O V
(c)
wave amplitude of beam P
? I I
wave amplitude of beam Q
1 1
(a) (b)
4 2
(c) 2 (d) 4
(e) 16.
O V O V
140. Light of wavelength 5000 Å and intensity 39.8 Wm–2 (d ) (e)
is incident on a metal surface. If only 1% photons of
incident light emit photoelectrons, then the number Fig. 21
of electrons emitted per second per unit area from
142. In a photoelectric cell, the wavelength of incident light
the surface will be nearly
is changed from 4000 Å to 3600 Å. The change in
(a) 1018 (b) 1020
stopping potential will be
(c) 1022 (d) 1024. [MNR 2002] (a) 0.14 V (b) 0.24 V
141. A metal surface in an evacuated tube is illuminated (c) 0.35 V (d) 0.44 V. [BHU 2001]
with monochromatic light causing the emission of
143. If the de-Broglie waves associated with each of the
photoelectrons which are collected at an adjacent elec-
following particles are to have the same wavelength,
trode. For a given intensity of light, the way in which
which particle must have the smallest velocity ?
the photocurrent I depends on the potential differ-
(a) proton (b) α-particle
ence V between the electrodes is as shown in Fig. 20
(c) 2 H nucleus (deuteron) (d) electron
1
I
(e) neutron.
144. The threshold frequency for a certain metal is ν0 .
When frequency of incident radiation is 2ν 0 , the
maximum velocity of photoelectrons is 3 × 106m s–1.
If the frequency of incident radiation is increased to
10 ν0 , then the maximum velocity of photoelectrons
will be
V (a) 3 × 106 m s–1 (b) 6 × 106 m s–1
O (c) 9× 106 m s–1 (d) 12 × 106 m s–1.

Fig. 20
[UGET 1998]
652 COMPREHENSIVE OBJECTIVE PHYSICS

145. In Q. 144, if Vs is the stopping potential in the first 150. When a metallic surface is illuminated by light of
case, then the stopping potential in the second case frequency 8 × 1014 Hz, a photoelectron of energy 0.5 eV
will be is emitted. When the same surface is illuminated by
(a) Vs (b) 3 Vs light of frequency 12 × 10 14 Hz, photoelectron of
(c) 9 Vs (d) 12 Vs. [EAMCET 2001] maximum energy 2 eV is emitted. The work function
is
146. A stopping potential of 0.82 volt is required to stop
(a) 0.5 eV (b) 1.5 eV
the photoelectrons emitted from a metallic surface
by light of wavelength 4000 Å. The stopping poten- (c) 2.5 eV (d) 3.5 eV.
tial for wavelength 3000 Å will be [Haryana PMT 2002]
(a) 1.85 V (b) 2.85 V 151. In Q. 150, the value of Planck’s constant is
(c) 3.0 V (d) 4.1 V. [Pb. PMT 2002] (a) 6.0 × 10–34 J s (b) 6.3 × 10–34 J s
147. An ultraviolet radiation source causes the emission (c) 6.4 × 10–34 J s (d) 6.6 × 10–34 J s.
of photoelectrons from a zinc plate. 152. In a photoelectric experiment, the potential difference
How would the maximum kinetic energy Ek of the V that must be maintained between the illuminated
photoelectrons and the number of photoelectrons surface and the collector so as just to prevent any
emitted per second n be affected by substituting a
electrons from reaching the collector is determined
more intense source of the same wavelength ?
for different frequencies f of the incident illumination.
Ek n
The graph below is obtained.
(a) decreased increased
(b) unchanged unchanged V
(c) unchanged increased
V1
(d) increased unchanged
(e) increased increased.
148. Light quanta of energy 3.5 × 10–19 J fall on the cath-
ode of a photocell. The current through the cell is
just reduced to zero by applying a stopping potential
of 0.25 V.
0
What is the work function energy of the cathode ? f0 f1 f
(a) 2.9 × 10–19 J (b) 3.1 × 10–19 J
Fig. 23
(c) 3.5 × 10–19 J (d) 3.9 × 10–19 J
(e) 6.4 × 10–19 J. What is the maximum kinetic energy of the electrons
emitted at frequency f1 ? [The symbol e represents
149. In an experiment on the photoelectric effect, an evacu-
ated photocell with a pure metal cathode is used. the charge on an electron and h is the Planck con-
Which graph best represents the variation of y, the stant]
minimum potential difference needed to prevent cur- (a) hf1 (b) V1 (f1 – fo)
rent from flowing, when x, the frequency of the inci- (c) h (f1 – fo) (d) V1
dent light, is varied ?
(e) eV1 (f1 – fo).

y y y 153. If stopping potentials corresponding to wavelengths


4000 Å and 4500 Å are 1.3 V and 0.9 V respectively,
then the work function of the metal is
O O O
(a) 0.3 eV (b) 1.3 eV
O x O x O x (c) 2.3 eV (d) 5 eV. [DCE 2003]
(a) (b) (c)
y y
154. The minimum intensity of light to be detected by
human eye is 10–10 Wm–2. The number of photons of
wavelength 5.6 × 10–7m entering the eye with pupil
area 10–6 m2 per second for vision is nearly
O O
(a) 100 (b) 200
O (d ) x O (e) x
(c) 283 (d) 400. [CMC LDH 2000]
Fig. 22
DUAL NATURE OF MATTER AND RADIATIONS 653

155. A student connects a photocell to supply and finds (a) h 2m E (b) 2mE /h
that when the cell is exposed to monochromatic (c) h/ mE (d) h/ 2mE
radiation, a current flows only when the potential
(e) h 2/ mE .
difference across the cell is less than 1.6 V. What is
the maximum energy of the emitted electrons ? 161. In Davisson-Germer experiment, the correct relation
between angle of diffraction φ and glancing angle θ is
(a) 1.0 × 10–19 J (b) 2.6 × 10–19 J
φ φ
(c) 4.8 × 10–11 J (d) 1.6 J (a) θ = 90° – (b) θ = 90° +
2 2
(e) 1.0 × 10+19 J. φ
156. Which one of the following graphs could represent (c) θ = (d) θ = φ. [JIPMER 2002]
2
the distribution P(E) of energies E of α-particles emit- 162. If E and λ represent the energy and wavelength re-
ted from a given source ? spectively of an electron, then the graph between log
λ and log E will have
P(E) P(E) P(E) (a) positive slope (b) negative slope
(c) zero slope (d) infinite slope.
[Bharati Vidyapeeth 2002]
0 0 0 163. The energy of an electron of de-Broglie wavelength
0 E 0 E 0 E 1 Å is
(a) (b) (c)
(a) 150.5 eV (b) 15.05 eV
P(E) P(E) (c) 1.505 eV (d) 0.1505 eV.
[CMC Vellore 2002]
164. A beam of monochromatic radiation falls on to a metal
0 0 X and photoelectrons are emited. The rate of emis-
0 (d ) E 0 (e) E sion of photoelectrons will be doubled if
(a) a beam of double the intensity is used
Fig. 24
(b) radiation of double the frequency is used
157. The de-Broglie wavelength associated with a nitrogen
(c) radiation of double the wavelength is used
molecule at atmospheric pressure and temperature
27°C will be nearly (d) the thermodynamic temperature of the metal is doubled
(a) 0.1 Å (b) 0.2 Å (e) a metal with a work function half that of X is substituted
for X.
(c) 0.3 Å (d) 0.4 Å.
158. The de-Broglie wavelength of neutrons in thermal 165. The work function for aluminium is 4.125 eV. The
equilibrium is cut-off wavelength for photoelectric effect for Alu-
minium is
30.8 3.08
(a) Å (b) Å (a) 300 nm (b) 200 nm
T T
(c) 420 nm (d) 150 nm. [KCET 2001]
0.308 0.0308
(c) Å (d) Å. [BHU 2001] 166. Two radiations containing photons of energy twice
T T
and five times the work function of a metal are inci-
159. The intensity of a beam of monochromatic light is dent successively on the metal surface. The ratio of
doubled. Which one of the following represents the the maximum velocities of the emitted electrons in
corresponding change, if any, in the momentum of the two cases will be
each photon of the radiation ?
(a) 1 : 3 (b) 1 : 4
(a) increased fourfold (b) doubled
(c) 1 : 2 (d) 1 : 1. [KCET 2000]
(c) the same (d) halved
167. A particle of mass M at rest decays into two masses
(e) reduced fourfold.
m1 and m2 with non-zero velocities. The ratio of de-
160. What is the de-Broglie wavelength of a particle of
λ1
mass m and kinetic energy E ? Broglie wave lengths of the particles is
λ2
[h is the Planck constant.]
654 COMPREHENSIVE OBJECTIVE PHYSICS

m2 m1 176. What is the ratio of the wavelengths of a photon and


(a) (b) that of an electron of the same energy ?
m1 m2
2m 2m
m1 (a) c (b)
(c) (d) 1 : 1. [KCET 2003] E E
m2
m
168. What is the energy of a proton of wavelength 0.4 Å ? (c) (d) mE . [Manipal 2000]
E
(a) 0.51 eV (b) 1.51 eV
177. If λ1 and λ 2 denote the wavelengths of de-Broglie
(c) 10.51 eV (d) 100.51 eV.
waves for electrons in the first and second Bohr orbits
169. The de-Broglie wavelength associated with a neutron
λ1
of energy 1 keV is in hydrogen atom, then is equal to
λ2
(a) 4.1 × 10–3 Å (b) 7.1 × 10–3 Å
(c) 9.1 × 10–3 Å (d) 19.1 × 10–3 Å. 2 1
(a) (b)
1 2
170. The ratio of the velocities of a proton and α-particle
is 4 : 1. The ratio of their de-Broglie wavelengths is 1 4
(c) (d) . [MNR 1997]
4 1
(a) 1 : 1 (b) 1 : 2
178. The de-Broglie wavelength of an electron is same as
(c) 2 : 1 (d) 4 : 1. [AFMC 2001]
that of a photon. If the velocity of electron is 50% of
171. An electron and a photon possess the same de-Broglie the speed of light, then ratio of kinetic energy of pho-
wavelength. If Ee and Eph are respectively the ener-
ton to the kinetic energy of electron is
gies of electron and photon and v and c are their
(a) 0.5 (b) 1.5
Ee
respective velocities, then = (c) 2.5 (d) 4.0. [JIPMER 2001]
E ph
179. Radiations of wavelength 1000 Å fall on a metal plate.
v v If h = 6.62 × 10–34 Js and work function of the plate is
(a) (b)
c 2c negligible, then the kinetic energy of the photoelec-
v v trons emitted is
(c) (d) .
3c 4c (a) 124.1 eV (b) 12.41 eV
[Bharati Vidyapeeth 2003] (c) 1.241 eV (d) 0.1241 eV .
172. In Q. 171, if the velocity of electron is 25% of the [All India PM/PD 2001]
Ee 180. The frequency of the incident light falling on a pho-
velocity of photon, then E = tosensitive metal plate is doubled. The kinetic en-
ph
ergy of the emitted photo electron is
(a) 1 : 2 (b) 1 : 4
(a) unchanged (b) doubled
(c) 1 : 8 (d) 1 : 16.
(c) less than doubled (d) more than doubled.
173. An electron and a photon, each has a wavelength of
1.2 Å. What is the ratio of their energies ? [JIPMER 1995]
(a) 1 : 10 (b) 1 : 102 181. Photons with a frequency of 4 × 104
Hz will produce
(c) 1 : 103 (d) 1 : 104. [JIPMER 1997] photoelectrons with a maximum kinetic energy of
174. What is the wavelength of a photon of energy 1 eV ? (a) 2.65 × 10–19 J (b) 3.65 × 10–19 J

(a) 12.4 × 103 Å (b) 2.4 × 103 Å (c) 4.65 × 10–19 J (d) 2.65 × 1019 J.
[CPMT 2002]
(c) 0.4 × 102 Å (d) 1000 Å.
[CMC Vellore 2001] 182. An important spectral emission line has a wavelength
of 21 cm. The corresponding photon energy is
175. The distance between two consecutive atoms of the
crystal lattice is 1.227 Å. The maximum order of dif- (a) 5.9 × 10–11 eV (b) 5.9 × 10–8 eV
fraction of electrons accelerated through 104 volt is (c) 5.9 × 10–6 eV (d) 5.9 × 10–4 eV .
(a) 1 (b) 5 [CMC Vellore 1999]
(c) 10 (d) 20.
DUAL NATURE OF MATTER AND RADIATIONS 655

183. If the energy of a photon corresponding to a wave- (a) 400 pm (b) 200 pm
length of 6000 Å is 3.32 × 10–19 J, the photon energy pm
pm
(in J) for a wavelength of 4000 Å will be (c) (d) . [CMC LDH 2002]
200 400
(a) 4.98 × 10–19 (b) 4.98 × 10–16
186. A 20 amu atom emits photon of 6.6 Å while making a
(c) 49.8 × 10–19 (d) 2.21 × 10–19 . [BHU 1999]
transition from excited state to ground state. The
184. The de-Broglie wavelength of helium atom at a tem- recoil energy of the atom will be
perature of 89°C, will be (a) 1.5 × 10–23 J (b) 3.5 × 10–23 J
(a) 0.47 Å (b) 0.58 Å
(c) 5.1 × 10–23 J (d) 7.5 × 10–23 J.
(c) 0.66 Å (d) 0.73 Å. [BHU 2000]
[Bharati Vidyapeeth 1999]
185. If the momentum of electron is changed by pm , then
the de-Broglie wavelength associated with it changes
by 0.50%. The initial momentum of electron will be

Answers (Set II)


83. (d) 84. (c) 85. (c) 86. (a) 87. (d) 88. (b) 89. (a) 90. (a)
91. (b) 92. (a) 93. (c) 94. (a) 95. (b) 96. (a) 97. (d) 98. (a)
99. (b) 100. (b) 101. (c) 102. (b) 103. (a) 104. (a) 105. (c) 106. (d)
107. (b) 108. (c) 109. (c) 110. (c) 111. (c) 112. (b) 113. (b) 114. (d)
115. (c) 116. (a) 117. (b) 118. (a) 119. (b) 120. (a) 121. (c) 122. (e)
123. (b) 124. (b) 125. (d) 126. (b) 127. (c) 128. (d) 129. (a) 130. (b)
131. (c) 132. (b) 133. (c) 134. (b) 135. (b) 136. (b) 137. (c) 138. (a)
139. (c) 140. (a) 141. (b) 142. (c) 143. (b) 144. (c) 145. (c) 146. (a)
147. (c) 148. (b) 149. (b) 150. (c) 151. (a) 152. (c) 153. (c) 154. (c)
155. (b) 156. (c) 157. (c) 158. (a) 159. (c) 160. (d) 161. (a) 162. (b)
163. (a) 164. (a) 165. (a) 166. (c) 167. (d) 168. (a) 169. (c) 170. (a)
171. (b) 172. (c) 173. (b) 174. (a) 175. (c) 176. (a) 177. (b) 178. (d)
179. (b) 180. (d) 181. (a) 182. (c) 183. (a) 184. (c) 185. (b) 186. (a).

Solutions (Set II)


h hc
83. λp = = 2E + φ0
2m peV λ′

h F
λ ′ E + φ0 I
λα =
2mα (2e)V
Dividing, GH
λ 2E + φ0
JK or
λ′
λ
<1

h ∴ λ′ < λ or λ > λ′ ... (i)


λα =
2(4m p ) (2e)V LM OP
λp λp Also,
λ ′ 1 E + φ0
=
λ 2 E + φo
MM PP
λα
= 8 or
λα
=2 2
2 N Q
hc λ′ 1 λ
84. = E + φ0 or > or λ′ > ...(ii)
λ λ 2 2
656 COMPREHENSIVE OBJECTIVE PHYSICS

It follows from (i) and (ii) that h hc


95. λ1 = and λ2 =
λ 2 mE E
λ > λ′ > .
2 λ1 h E
= ×
1 λ2 2mE hc
85. mvmax.2 = hν – φ0
2 λ1 E λ1
or ∝ or ∝ E1/2
2 ( hν – φ0 ) λ2 E λ2
or vmax. =
m 96. Maximum kinetic energy
1 = (3.4 – 2) eV = 1.4 eV
2 (1 – 0.5)
Now, v= = h
m m 97. λ=
2meV
2 (2.5 – 0.5) 2
and v′ = =
m m p = 2meV
v 1 pe 2meeV
= =
v′ 2 pα 2mα (2e)V
86.
h
λ= pe me
2mE =
λ is constant pα 2mα
∴ mE = constant 1
98. mv2max = eVs
For electron, m is least, E is maximum. For α-parti- 2
cle, m is maximum and E2 is minimum.
2eVs 11
h vmax = = 2 × 1.8 × 10 × 9 m s–1
87. p= = 1012 h m
10–2 × 10 –10 = 1.8 × 106 m s–1
2eV 2 × 1.6 × 10–19 × 1000 h h h
88. v= = m s–1 99. λ = = or v =
m 9 × 10 –31 p mv mλ
7 –1
= 1.9 × 10 m s 6.63 × 10 −34
or v= m s–1
89 . hν1 = 1.0 eV + W and hν2 = 2hν, 9.11 × 10 −31 × 10 −11
= 2.9 eV × W, 2.9 + W = 2(1 + W) = 7.28 × 107 m s–1
W = 0.9 eV.
1
Kinetic energy = mv2
h 2
90. Use λ =
mv
1
3 = × 9.11 × 10−31 × (7.28 × 107 )2 J
r 2
91. Vα ; Both r and Q are doubled ; So, V is increased
Q = 241.41 × 10–17 J ≈ 15 keV
by a factor of 4. 100. E = hν
h E
92. λ= ν=
mv h
h
λ= m 75 × 1.6 × 10 –19
1× 1 ν= Hz = 18.2 × 1015 Hz
6.6 × 10–34
λ = h metre
101. Energy in eV corresponding to 332 × 10–9m or 3320
93. If the oil drop suddenly gains electrons, then the up- Å
ward electric force shall exceed the gravitational force.
94. The stopping potential for curves ‘a’ and ‘b’ is same. 12375
= = 3.73 eV
∴ fa = fb 3320
Also saturation current is proportional to intensity. 1
mvmax2 = (3.73 – 1.07) eV = 2.66 eV
∴ Ia < Ib 2
DUAL NATURE OF MATTER AND RADIATIONS 657
102. W = qV = (2e)5V = 10 eV Energy of electron
103. hν = 5 eV + 2.2 eV = 7.2 eV = (2.475 – 1.8) eV = 0.675 eV = 0.68 eV
12375 12375
7.2 = 111. φ0 = 1 eV, hν = eV = 1.93 eV
λ in Å 6400

12375 1
or λ in Å = ≈ 1719 mv2max = hν – φ0
7.2 2
= (1.93 – 1) eV = 0.93 eV
φ0
104. φ0 = hν0 or ν0 = = 0.93 × 1.6 × 10–19 J = 1.5 × 10–19 J
h
112. Intensity of light has no effect on stopping potential.
4.5 × 1.6 × 10 –19 12375
or ν0= Hz = 1.09 × 1015 Hz
6.6 × 10–34 113. Energy corresponding to 2000 Å = eV = 6.2 eV
2000
12375 Maximum kinetic energy
105. λ in Å = = 4125
3 = (6.2 – 5.01) eV = 1.19 eV
λ in metre = 4125 × 10–10 m = 4.125 × 10–7 m
1
h Now, × 9.1 × 10–31 × v2max.
106. λ = ; In the given problem, mq V = constant 2
2 mqV = 1.19 × 1.6 × 10–19
V 1.19 × 1.6 × 10 –19 × 2
∴ 4 × 2 V′ = 1 × 1 × V or V′ = or v2max. =
8 9.1 × 10 –31
h or v2max. = 0.418 × 1012 = 41.8 × 1010
107. λ=
mv or vmax. = 6.46 × 105 m s–1
h 114. Electron at rest gives mg = eE
λ= h
or λ′ = Downward force on proton
2meV 2MeV
= Mg + eE = Mg + mg
λ′ m m (M + m) g
= or λ′ = λ
λ M M = (M + m)g ; a =
M
108. E = pc hc
E = 2 × 10–16 × 3 × 1010 erg 115. φ0 = hν0 =
λ0
= 6 × 10–6 erg
or φ0λ0 = constant
hc
109. E= – φ0 2.3 × 5460
λ 4.5 × λ = 2.3 × 5460 or λ = Å
4.5
hc 2hc hc = 2790.7 Å ≈ 2791 Å.
2E = – φ0 or – 2φ0 = – φ0
λ′ λ λ′ 12375
116. – φ0 = 2 eV
4000
hc 2hc hc 2hc hc
or = – φ0 or = +E– or φ0 = (3.1 – 2) eV = 1.1 eV
λ′ λ λ′ λ λ
c
hc hc hc Eλ + hc 117. c = νλ , ν =
or =E+ or = λ
λ′ λ λ′ λ
3 × 108 1
hcλ ν= Hz = × 106 Hz
λ′ = 1500 5
or
Eλ + hc = 0.2 × 106 Hz = 2 × 105 Hz
110. λ = 500 nm = 500 × 10–9 × 1010 Å = 200 × 103 Hz = 200 kHz
= 5000 Å hc
118. E= – φ0
Energy corresponding to 5000 Å λ/2
12375 hc FG
2hc 3hc IJ
=
5000
eV = 2.475 eV 2E =
λ/3
– φ0 or 2
λ H– φ0 =
λ K
– φ0
658 COMPREHENSIVE OBJECTIVE PHYSICS

(a) and (b)


or 4hc – 2φ = 3hc – φ or φ = hc .
0 0 0
λ Clearly, (b) is the right choice.
λ λ
127. De-Broglie waves : λ vs mass
119. The energy of photons from hydrogen discharge tube
For alpha, mass 4-fold and E 2-fold (charge 2e). Hence
is 13.6 eV.
eVs = (13.6 – 3.1) eV = 10.5 eV 8 fold smaller λ
∴ Vs = +10.5 volt 128. Order of magnitude calculation is enough.
120. λ = 100 nm = 1000 Å (2mE)1/2 = (2 × 9 × 10–31 × 100 × 1.6 × 10–19)1/2
≈ 5 × 10–24.
12375
Energy corresponding to 1000 Å = eV And h ≈ 6 × 10–34 Js
1000
So, λ ≈ 10–10 m. Mid-visible has
= 12.375 eV
λ0 ≈ 5000 × 10–10 m
Now, 7.7 = 12.375 – φ0
129. One-dimensional potential well
or φ0 = 12.375 – 7.7 = 4.675 eV
2L
In the second case, energy corresponding to 2000 Å λ= ,
n
12375
= eV = 6.1875 eV nh p2
2000 p= , E= . This gives (a).
2L 2m
Now, eVs′ = (6.1875 – 4.675) eV
= 1.5125 eV 12400
130. φ0 = eV = 2.48 eV
or Vs′ = 1.5 volt 5000
121. p = nhν = 2.48 × 1.6 × 10–19 J ≈ 4 × 10–19 J
⇒ nν = constant 131. In the first case,
For red light, ν less and n more. 1
mv2max. = 2hν0 – hν0 = hν0
122. It is important to understand that all forces and de- 2
flections occur in the plane of paper and that at a In the second case,
certain velocity, the deflection will be zero. For veloci- 1
ties other than this, the deflection is determined by mv′ 2max. = 5hν0 – hν0 = 4hν0
2
whether the magnetic force is greater or less than the
Clearly, vmax is doubled.
electric force.
132. The centripetal force on each particle is provided by
123. When λ is halved, energy is doubled.
the magnetic field.
So, energy corresponding to 5000 Å is 2 × 1.23 eV
or 2.46 eV. hc
133. eVs = hνmax. =
Now, φ0 = (2.46 – 1.36) eV = 1.1 eV λ min

1 134. The electrons are accelerated in a direction opposite


124. mv2max. = (6.2 – 4.2) eV to the direction of electric field.
2
= 2 eV = 2 × 1.6 × 10–19 J = 3.2 × 10–19 J 12375
135. Energy corresponding to 4000 Å = eV
125. φ0 = hν0 4000
= 3.1 eV
φ0 2.51 × 1.6 × 10 –19 The work functions of only A, B and C are below
or ν0 = = Hz
h 6.6 × 10 –34 3.1 eV.
= 6.08 × 1014 Hz 136. qE = mg
126. De-Broglie waves : λ vs energy. For particles, qV mgd
= mg or q =
h h d V
λ= = .(b) and (c).
p 2mE 1.96 × 10 –15 × 9.8 × 0.02
= C
c hc 800
For photons, λ = =
ν E = 4.8 × 10–19 C = 3 × 1.6 × 10–19 C = 3 e.
DUAL NATURE OF MATTER AND RADIATIONS 659

h Vs is called the stopping potential which is the value


137. λ= of the negative potential difference which just stops
2meV
the electrons with maximum energy from reaching
⇒ mV = constant the anode.
Now, 2000 × V′ = 1 × V
Since the light is of the same frequency (intensity is
V doubled but the same wavelength of light), the
V′ = volt
2000 stopping potential Vs is the same even though the
138. From the Einstein’s photoelectric theory, we have intensity is doubled.
eV = hf – φ The number of photoelectrons emitted per unit area
where V is the potential difference and of cross-section of the beam of light per unit time is
proportional to the intensity of light.
φ is the work-function of metal.
Thus, the current I measured is doubled as the
φY > φX , fY > fX
intensity of light is doubled. The graph of I versus
fY is the threshold frequency of Y. potential difference V is thus best represented in graph
fX is the threshold frequency of X. b.
h 142. eVs = hν – φ0
Furthermore, V versus f has a constant slope of .
e eVs′ = hν′ – φ0
139. The photoelectric current detected is proportional to FG 12375 – 12375 IJ eV
the number of electrons emitted, which in turn, is
e(Vs′ – Vs) = hν′ – hν =
H 3600 4000 K
directly proportional to the intensity of the light beam. ∴ Vs′ – Vs = 3.44 – 3.09 = 0.35 V
If I is the current detected and A is the amplitude of 143. If the de-Broglie waves have the same wavelength λ,
the light beam, we have A2 α I since intensity is pro- then the momentum p of the particles must be the
portional to square of light amplitude. same.
wave amplitude of beam P h
⇒ p = mv =
wave amplitude of beam Q λ

F current produced by P IJ
1 Where m is the mass of the the particle(s)
= G
2

H current produced by Q K v is the velocity of the particle(s).


α-particle has the largest mass number and hence its
F 4I = 2
= G J
1/2 mass m is the largest among the rest. Since the ve-
H 1K locity v is inversely proportional to the mass m for
the same momentum p, the α-particle thus has the
140. E = nhν
smallest velocity.
nhc
Ε= 1
λ 144. & 145. mv2max. = 2hν0 – hν0 = hν0
2
Eλ 1
or n= Again, mv′2max. = 10hν0 – hν0 = 9hν0
hc 2
n 39.8 × 5000 × 10 –10 Clearly, vmax. is increased by a factor of 3 and stop-
Now, n′ = = ping potential by factor of 9.
100 100 × 6.6 × 10 –34 × 3 × 108
≈ 1018 12375
146. 0.82 eV = – φ0
141. By Einstein’s particle (photon) theory, the energy of a 4000
photon is proportional to its frequency and is inde- ∴ φ0 = (3.09 – 0.82) eV = 2.27 eV
pendent of the light intensity. If the quantum of en- 12375
ergy hf in the light incident on the metal and the Again, eVs = – 2.27
3000
work function of the metal is W0, then the maximum
Vs = (4.125 – 2.27) volt = 1.85 volt
kinetic energy Emax. is given by
147. Using Einstein’s photo theory, the maximum kinetic
Emax. = eVs = hf – W0 ...(1) energy Ek of the emitted electrons is related to the
660 COMPREHENSIVE OBJECTIVE PHYSICS

frequency f and hence the wavelength λ of the incident of the negative potential difference which just stops
light by the equation the electrons with maximum energy from reaching
A. V is called the stopping potential. Therefore,
hc
Ek = hf – w0 = – w0 ...(1) eV = Emax. …(2)
λ
From (1) and (2), we have
where c = speed of light.
eV = Emax. = h(f – fo)
w0 = work function of the material = mini-
mum kinetic energy needed to free an h h
V= (f – f0) or y = (x – x0)
electron out of the material. e e
For incident wave of the same wavelength, the maxi- RSVf UV is replaced by RS xyUV .
mum kinetic energy Ek of the photoelectrons remains
where
TW TW
unchanged as easily deduced from equation (1). The variation of V (or y) is thus a straight line of
The higher the intensity of the incident wave, the h
gradient , when it is plotted against f (or x). The
more the number of photons incident on the zinc plate e
surface ; and hence more photoelectrons would be emit- curve has a minimum value of zero at frequency f (or
ted. Thus the number of photoelectrons emitted per x) at f0. It is best represented in graph b.
second n would be increased for a more intense source. 150. 8 × 1014 h = φ0 + 0.5
148. The Einstein’s photoelectric equation is 12 × 1014 h = φ0 + 2
Emax. = hf – W0 12 φ +2
Dividing, = 0
where E max. is the maximum kinetic energy of 8 φ 0 + 0.5
electrons 3 φ +2
= 0
hf is the incident photon energy 2 φ0 + 0.5
W0 is the work function of the photocell 3φ0 + 1.5 = 2φ0 + 4
If Vs is the stopping potential necessary to stop the or φ0 = 2.5 eV
electrons with maximum energy from reaching the 2.5 + 0.5
anode, then 151. h= × 1.6 × 10–19 Js
8 × 1014
eVs = Emax. = 6 × 10–34 Js
From the equations, we may write 152. The work function φ of the metal is given by
eVs = hf – W0 φ = hf0
⇒ (1.6 × 10–19) (0.25) = 3.5 × 10–19 – W0 where f0 is the threshold frequency of the metal below
∴ W0 = 3.5 × 10–19 – 0.4 × 10–19 which no emisson of electron is obtained.
= 3.1 × 10–19 J The Einstein’s photoelectric equation is given by
149. By Einstein’s particle (photon) theory, the maximum eV = Emax. = hf – φ
kinetic energy Emax. of the emitted electrons from the = hf – hf0 = h(f – f0)
cathode is proportional to the frequency f of the light. where Emax. = maximum kinetic energy of electrons
This is expressed in the Einstein’s photoelectric equa- V = stopping potential for the electrons
tion below Therefore, eV1 = Emax. = h(f1 – f0)
Emax. = hf – φ0 = hf – hf0 hc
hc
= h(f – f0) ....(1) 153. eVs = – φ0 or eVs + φ0 =
λ λ
where h is the Planck’s constant, hc
or λ=
φ0 is the work function of the metal and is related to eVs + φ0
the threshold frequency fo by φ0 = hfo . It is the mini- hc
Now, λ1 =
mum amount of work or energy necessary to take a eVs1 + φ 0
free electron out of the metal against the attractive hc λ 2 eVs1 + φ0
forces of surrounding positive ions. λ2 = or =
eVs2 + φ 0 λ 1 eVs2 + φ0
At a particular negative potential difference V applied
. + φ0
4500 13 9 1.3 + φ0
to the anode A, the current becomes zero. This is value = or =
4000 0.9 + φ0 8 0.9 + φ0
DUAL NATURE OF MATTER AND RADIATIONS 661
or φ0 = 8 × 1.3 – 9 × 0.9 or φ0 = (10.4 – 8.1) eV 159. If the intensity of a beam of monochromatic light is
= 2.3 eV doubled but its frequency f and hence its wavelength
154. P = nhν λ remains unchanged, then the number of photons
per unit area of cross-section of the beam of light per
P Pλ IAλ unit time is also doubled. But the momentum P of
n= or n = or n =
hν hc hc each photon of the rediation is the same and is given
by
10 –10 × 10 –6 × 5.6 × 10 –7
= = 282.8 ≈ 283 h
6.6 × 10 –34 × 3 × 10 8 P=
155. The value of 1.6 V is the stopping potential Vs which λ
just stops the electrons with maximum energy Emax. 160. By de-Broglie’s theory, the wavelength λ of a particle
from being detected as current flow. Thus of mass m and momentum P is given by
Emax. = eVs = (1.6 × 10–19) (1.6) h h
= 2.56 × 10–19 J = 2.6 × 10–19 J λ= =
P mv
156. By de-Broglie’s theory and quantum theory of energy,
where ν is the velocity of the particle
the momentum p and E of α-particles emitted from a
source are given by Now, the kinetic energy E of the particle is related to
h its mass m and speed ν by the equation below.
E = hf ; p =
λ 1
hc E= mv2
Hence, E = hf = = pc = mc2 2
λ
⇒ 2mE = (mv)2
where m is the mass of the α-particles
Thus, the momentum p is related to energies E or ∴ P = mv = 2mE
mass m by Hence, the wavelength λ is related to mass m and
E kinetic energy E by
p = = mc
c h
λ=
which is proportional to the mass of the α-particles. 2mE
The distribution p of energies E of α-particles emitted
from a given source is thus best represented in graph
161. 2θ + φ = 180°
c.
or 2θ = 180° – φ
157. h θ
λ= φ φ
3mkT or θ = 90° –
2 θ
6.62 × 10–34
= m
3 × 28 × 1.67 × 10 –27 × 1.38 × 10–23 × 300 h Fig. 25
162. λ=
2mE
6.62 × 10 –34 6.62 × 10 −34 1
= m = m log λ = log h – log (2mE)
10 –25 58075.9 241 × 10 −25 2
= 0.02746 × 10–9 m Clearly, the straight line has a negative slope.
= 0.2746 × 10–10 m = 0.27 Å ≈ 0.3 Å 12.27
163. λ= Å
158. h V
λ=
2mkT 12.27 × 12.27
V= = 150.5 volt
6.62 × 10 –34 1
= m
2 × 1.67 × 10–27 × 1.38 × 10–23 T So, energy is 150.5 eV.
164. By Einstein’s particle (photon) theory, the number of
6.62 × 10 –34 3.079
photoelectrons emitted per unit area of cross-section
= −25
m= × 10–9 m
2.15 × 10 T T of the beam of light per unit time is proportional to its
30.79 30.8 intensity. Thus, the rate of emission of photoelectrons
= Å≈ Å will be doubled if the beam of light has its intensity
T T
doubled.
662 COMPREHENSIVE OBJECTIVE PHYSICS

hc hc ∴ LM 2E OP E
e
E ph2
165. φ0= hν0 =
λ0
or λ0 =
φ0
2
Nv Q
2 e =
c2
2
or
12375
× 10–10 × 109 nm 4E e2 E ph E e2 v2
λ0 = or = or =
4.125 v2 c2 E ph2 4 c2
12375 Ee v
or λ0 = nm = 300 nm or =
41.25 E ph 2 c
1 c
166. 2 φ0 = φ0 + mv12
2 172. Ee 4 1
= =
1 E ph 2c 8
or φ0 = mv12 ...(1)
2 h2
1 Ee 2 h2 λ h
Also, 5 φ0 = φ0 + mv22 173. = 2mλ = × =
2 E ph hc 2mλ 2
hc 2mλc
1 λ
or 4 φ0 = mv22 ...(2)
2 6.6 × 10 –34
=
Dividing (1) by (2), 2 × 9.1 × 10 –31 × 1.2 × 10–10 × 3 × 108

v12 φ0 1 1
v1 1 = 0.1 × 10–1 =
2
= = or = 100
v2 4 φ0 4 v2 2
mc2 E hν hν h
h 174. p = mc = = = = =
167. λ= c c c νλ λ
mv
E h hc
Here 0 × M = m1v1 + m2v2 Now, = or E=
c λ λ
Clearly, m1v1 = – m2v2
In magnitude, mv = constant hc 6.6 × 10 –34 × 3 × 108
or λ= = × 1010 Å
λ1 1
E 1 × 1.6 × 10 –19
∴ = .
λ2 1 = 12.375 × 103 Å
175. d sin θ = nλ
0.286
168. λ= Å d sin θ
E(in eV) n=
λ
0.286
E(in eV) = d d 1.227 104
0.4 nmax. == = V= = 10
λ 12.27 12.27 10
E(in eV) = 0.51
V
0.286 hc
169. λ= Å
E(in eV) λ ph E c 2m
176. = = 2mE = c
0.286 λe h E E
λ= Å ≈ 9.1 × 10–3 Å 2mE
1000
λp 13.6
mα vα 4 1 177. Energy of electron in nth orbit, En = – 2 eV
170. = = × = 1: 1 n
λα m pvp 1 4
For first Bohr orbit,
h hc n=1
171. λ= =
2mE e E ph E1 = – 13.6 eV
For second Bohr orbit,
E2 ph n=2
or 2mEe =
c2
13.6 λ1 E2
E2 = – eV or =
1 2E 4 λ2 E1
But Ee = mv2 or m = 2 e
2 v
DUAL NATURE OF MATTER AND RADIATIONS 663

λ1 1 hc
= λ1 1 183. E1 =
λ2 4 ∴ = λ1
λ2 2
hc
E ph 2c 2c E2 =
178. = = = 4. λ2
Ee v c/2
E2 λ 1
hc 1 =
179. = mv2 E1 λ 2
λ 2
180. Ek = hν – φ0 or Ek′ = 2hν – φ0 λ1
E2 = × E1
FG φ0 IJ λ2
Ek ′ 2hν – φ0
= =
H
2 hν –
2 K 184. λ=
h
Ek hv – φ0 hν – φ0 3mkT
Ek ′ ∆pm
Clearly >2 or Ek′ > 2Ek 185. p= λ
Ek ∆λ
2
181. hν = (Ek)max. p2 h
186. E= =
hc 2m 2mλ2
182. E = hν =
λ

KNOWLEDGE PLUS
l The work function of a substance is 4.0 eV. The longest wavelength of light that can cause photoelectron emission
from this substance is approximately :
(a) 310 nm (b) 400 nm (c) 540 nm (d) 220 nm. [AIEEE 2004]
hc hc
Sol. =W or λlongest =
λ W
6.63 × 10−34 × 3 × 108
or λlongest = m ≈ 310 nm
4.0 × 1.6 × 10−19
So, (a) is the right choice.

l The work functions for metals A, B and C are respectively 1.92 eV, 2.0 eV and 5 eV. According to Einstein’s
equation, the metals which will emit photo-electrons for a radiation of wavelength 4100 Å is/are :
(a) None (b) A only (c) A and B only (d) All the three metals.
[All India PM/PD 2005]
12400
Sol. Energy corresponding to 4100 Å = eV = 3.02 eV
4100
Since work functions for metals A and B are less than 3.02 eV therefore A and B will emit photo-electrons.
So, (c) is the right choice.

l In the case of photon,


(a) it energy is proportional to the square of its velocity.
(b) it loses energy upon entering a denser a medium.
(c) collision with another photon is not possible.
(d) its speed can changed but cannot become equal to zero. [National Standard Exam. is Physics 2005]
Sol. Factual information.
So, (c) and (d) is the right choice.
664 COMPREHENSIVE OBJECTIVE PHYSICS

SET III MCQs


with
More than one correct alternative

Average time allowed per question is 50 seconds.

187. Fig. 26 shows the results of an experiment involving (c) Photoemission occurs when light of appropriate frequency
photoelectric effect. The graphs A, B, C, D relate to but minimum intensity is incident.
light beam having different wavelengths. (d) The charge of photoelectrons is quantised.
190. A source of light is at a distance “s” from metal plate.
i (Current) The plate emits electrons having stopping potential
“V”. Then the wrong statements are
A
(a) V decreases as s increases
B (b) V decreases as s decreases
C (c) V increases when frequency of light is increased
D (d) V alters if metal is changed.
191. Photoelectric effect support quantum nature of light
because
O V (a) there is minimum frequency of light below which no photo-
Retarding Accelerating
electrons are emitted
(b) electric charge of photoelectrons is quantised
Fig. 26
(c) the maximum kinetic energy of photoelectrons depends
(a) Beam B has highest frequency. only on the frequency of light and not on its intensity
(b) Beam C has longest wavelength.
(d) even when the metal surface is faintly illuminated, the
(c) Beam A has the highest rate of photoelectric emission. photoelectrons leave the surface immediately.
(d) Photoelectrons ejected by beam B have the highest [IIT 1987]
momentum.
192. Fig. 27 shows a Photo cell
Light
188. When a monochromatic point source of light is at a
photo cell circuit.
distance of 0.2 m from a photoelectric cell, the cut-off
voltage and the saturation current are respectively The cathode of the C

0.6 V and 18.0 mA. If the same source is placed 0.6 m photo cell is illumi-
A
away from the photocell, then nated by a mono-
(a) the stopping potential will be 0.2 V chromatic light. If
(b) the stopping potential will be 0.6 V the intensity is kept
Fig. 27
(c) the saturation current will be 2.0 m A constant and the fre-
quency of the incident light is increased, then the
[IIT 1992]
(d) the saturation current will be 6.0 m A.
(a) photo electric current in the circuit increases.
189. Which one of the following statements is not in sup-
port of the quantum nature of light ? (b) photo electric current in the circuit decreases.
(a) There is a minimum frequency of light below which no (c) maximum kinetic energy of the photo electrons increases
photoelectrons are emitted. (d) photo electric current in the circuit can be reduced to zero,
(b) The maximum K.E. of photoelectrons depends only on the when the polarity of the terminals is reversed.
frequency of light and not on the intensity.
DUAL NATURE OF MATTER AND RADIATIONS 665
193. Which of the following phenomena can be explained (a) the work function of A is 2.25 eV
only on the basis of quantum theory of light ? (b) the work function of B is 4.20 eV
(a) Energy spectrum of black body radiation (c) TA = 2.00 eV (d) TB = 2.75 eV. [IIT 1994]
(b) Atomic spectra (c) Photoelectric effect
195. Which of the following statement(s) is/are true ?
(d) Doppler effect.
(a) Maximum velocity of photoelectrons depends on frequency
194. When photons of energy 4.25 eV strike the surface of and intensity of the incident light
a metal A, the ejected photoelectrons have maximum (b) Maximum velocity of photoelectrons depends only on the
kinetic energy TA eV and de-Broglie wavelength λA. frequency of the incident light.
The maximum kinetic energy of photoelectrons lib- (c) Photoelectric current increases with increase in intensity
erated from another metal B by photons of energy of incident light.
4.70 eV is TB = (TA – 1.50)eV. If the de-Broglie wave- (d) Photoelectric current is independent of the intensity of
length of these photoelectrons is λB = 2λA, then the incident light.

Answers (Set III)

187. (a), (b), (c), (d) 188. (b), (c) 189. (a), (d) 190. (a), (b) 191. (a), (c), (d) 192. (c), (d) 193. (a), (b), (c) 194. (a), (b), (c)
195. (b), (c).

Solutions (Set III)


187. Intercept on V-axis is proportional to frequency. Maxi- 194. 4.25 = φA + TA
mum intercept is for beam B. So, B has highest fre- 4.70 = φB + TB
quency. So, (a) is OK. C has minimum intercept, mini- 4.70 = φB + TA – 1.50
mum frequency and hence maximum wavelength. So,
6.20 = φB + TA
(b) is OK. The highest peak is for beam A. So, (c) is
∴ φB – TB = 1.95

OK. Momentum of photon = , Momentum propor- λA TB
c Again, =
tional to frequency. λA TA
188. The stopping potential is independent of the internsity λA TB
of incident light. So, it remains unchanged. Since dis- =
2λ A TA
tance has been increased by a factor of 3 therefore the
intensity is reduced by a factor of 9. TB 1
= or TA = 4TB
191. There exists a threshold frequency. Moreover, photo- TA 4
electric effect is instantaneous. Also,TA – TB = 1.50.
666 COMPREHENSIVE OBJECTIVE PHYSICS

SET IV MCQs
based on
TYPICAL NUMERICAL BANK
(Exclusively for Engineering Entrance Tests)

Average time allowed per question is 60 seconds.

196. A stream of photons is impinging normally on a com- 199. A radio transmitter operates at a frequency of 880
pletely absorbing screen in vacuum. If irradiance (en- kHz and a power of 10 kW. The number of photons
ergy/area/time) is I, then the pressure on the screen emitted per second is
is (a) 0.075 × 10–34 (b) 1.71 × 1031
I c (c) 13.27 × 1034 (d) 1327 × 1034.
(a) (b)
c I 200. What wavelength is corresponding to a beam of
(c) cI (d) zero . electrons whose kinetic energy is 100 eV ?
197. How many red photons (wavelength λ) must strike a (h = 6.6 × 10–34 Js, 1 eV = 1.6 × 10–19 J,
totally reflecting screen per second at normal inci- me = 9.1 × 10–31 kg)
dence, if the exerted force is to be 1N ?
(a) 4.8 Å (b) 3.6 Å
λ 2λ (c) 1.2 Å (d) 2.4 Å . [MP PET 1991]
(a) (b)
h h 201. Light of wavelength 0.6 µm from a sodium lamp falls
λ on a photo cell and causes the emission of photoelec-
(c) (d) infinity. tron for which the stopping potential is 0.5 volt. With
2h
light of wavelength 0.40 µm from a mercury vapour
198. When a certain matallic surface is illuminated with
lamp, the stopping potential is 1.5 volt. Then the work
light of wavelength λ , the stopping potential is 3V0 .
When the same surface is illuminated with light of function in electron volt of the photocell surface is
(a) 0.75 eV (b) 1.5 eV
wavelength 2λ, the stopping potential is V 0. The
threshold wavelength for this surface is (c) 3 eV (d) 02.5 eV.
(a) 6λ (b) 4λ 202. If 5% of energy supplied to a bulb is radiated as vis-
ible light, how many quanta are emitted per second
λ by 100 watt lamp ? Assume wavelength of visibile
(c) (d) 8λ.
4 light as 5.6 × 10–5 cm
(a) 1.4 × 10–19 (b) 2.0 × 10–4
(c) 2.0 × 10 4
(d) 1.4 × 1019.

Answers (Set IV)


196. (a) 197. (c) 198. (b) 199. (b) 200. (c) 201. (b) 202. (d).
DUAL NATURE OF MATTER AND RADIATIONS 667

Solutions (Set IV)

energy [ML2T – 2 ] 199. P = nhν


196. = 2
area × time × velocity [L ] [T] [LT –1 ] h
200. λ=
–2
[MLT ] 2 mE k
= [ML–1T–2] =
[L2 ] hc
201. Use – φ0= eVs
= [Pressure] . λ
nhc 5 5λ
197. 2nh λ 202. = × 100 = 5 or n=
= 1 or n = λ 100 hc
λ 2h
198. hν = φ0+ eV
hc hc hc hc
= + e(3V0) and = + eV0
λ λ0 2λ λ 0
On solving, λ0 = 4λ.

KNOWLEDGE PLUS
O According to Einstein’s photoelectric equation, the graph between the kinetic energy of photoelectrons ejected and
the frequency of incident radiation is

(a) (b) (c) (d)


Kinetic energy
Kinetic energy

Kinetic energy

Kinetic energy

Frequency Frequency Frequency Frequency

Fig. 28

[All India PM/PD 2004]


Sol. Kinetic energy of photoelectron ejected is given by,
K.E. = hν – W
= hν – hν0
where work function depends on the type of material.
If the frequency of incident radiation is greater than ν0 , only then the ejection of photoelectrons starts. After that
as frequency increases, kinetic energy also increases.
So, (c) is the right choice.
668 COMPREHENSIVE OBJECTIVE PHYSICS

SELF-EVALUATION TEST I
Based on UNIT XVI
[Expected Questions for Forthcoming Examinations]

1. In the photo-ionisation of atomic hydrogen, what will 8. A point source

Current
be the maximum K.E. of the ejected electron when a causes photoelectric a
photon of wavelength λ is absorbed by the atom ? effect from a small
hc hc metal plate. Which of b
(a) (b) − 13.6 eV the curves in Fig. 29
λ λ c
(c) 13.6 eV (d) Ec 2 MeV. may represent the d
saturation photo-
2. An electron is accelerated through a potential differ-
current as a function
ence of V volt. It has a wavelength λ associated with Distance
of the distance be-
it. Through what potential difference an electron must
tween the source and Fig. 29
be accelerated so that its de-Broglie wavelength is
the metal ?
the same as that of a proton ? Take mass of proton to
be 1837 times larger than the mass of electron. (a) A (b) B
(a) V volt (b) 1837 V volt (c) C (d) D. [AMU 2003]
V 9. For some material, the threshold wavelength is 400
(c) volt (d) 1837 V volt. nm. The work function is
1837
3. The electric eye is (a) 3.1 eV (b) 3.1 J
(a) burglar alarm (b) fire alarm (c) 6.2 eV (d) 1.8 eV. [AMU 1996]
(c) television (d) photo electric cell. 10. Let p and E denote the linear momentum and energy
4. The energy of a photon of wavelength λ is given by respectively of a photon. If the wavelength is
(a) hλ (b) chλ decreased,
(c) h/λ (d) hc/λ. (a) both p and E increase
(b) p increases and E decreases
5. When green light is incident on the surface of metal,
(c) p decreases and E increases
it emits photo-electrons but there is no such emis-
sion with yellow colour light. Which one of the col- (d) both p and E decrease. [AIIMS 2000]
ours can produce emission of photoelectrons ? 11. The rest mass of the photon is
(a) Orange (b) Red (a) 0 (b) ∞
(c) Indigo (d) none of the above. (c) between 0 and ∞ (d) equal to that of an electron.

6. The maximum wavelength of radiation that can pro- [MP PET 1994]
duce photoelectric effect in a certain metal is 200 nm. 12. Assuming photoemission to continue to take place,
The maximum kinetic energy acquired by electron the factor by which the maximum velocity of the emit-
due to radiation of wavelength 100 nm will be ted photoelectrons changes when the wavelength of
the incident radiation is increased four times, is
(a) 12.4 eV (b) 6.2 eV
1
(c) 100 eV (d) 200 eV. [MP PMT 1994] (a) 4 (b)
4
7. A cesium photocell with a steady potential difference 1
(c) 2 (d) .
of 60 V across it is illuminated by a small bright 2
light placed 1 m away. When the same light is placed [Assume work function to be negligible]
2 m away, the electrons crossing the photocell 13. The photocurrent in an experiment on photoelectric
(a) each carry one-quarter of their previous momentum effect increases if
(b) each carry one-quarter of their previous energy (a) the intensity of the source is increased.
(c) are one-quarter as numerous (b) the exposure time is increased.
(d) are half as numerous. (c) the intensity of the source is decreased.
(d) the exposure time is decreased.
[Bharati Vidyapeeth 2001]
DUAL NATURE OF MATTER AND RADIATIONS 669

14. The photoelectrons emitted from a given cathode on 21. The work function of a metal is hν 0. Light of fre-
the incidence of a given monochromatic beam of light quency ν falls on this metal. The photoelectric effect
have will take place only if
(a) an energy spread with a lower limit. (a) ν ≥ ν0 (b) ν > 2ν0
(b) an energy spread with an upper limit. (c) ν < ν0 (d) ν < ν0 / 2.
(c) an energy spread with no sharp limits. 22. The energy of a photon is 3 × 10–19 J. Its momentum
(d) a definite energy only. is
[National Standard Exam. in Physics 1990] (a) 10–27 kg m s–1 (b) 9 × 10–11 kg m s–1
15. Light from a hydrogen discharge tube is incident on (c) 10–11 kg m s–1 (d) 3 × 10–7 kg m s–1.
the cathode of a photoelectric cell. The work function 23. In a dark room of photography, generally red light is
of the cathode surface is 4.2 eV. In order to reduce used. The reason is
the photocurrent to zero, the voltage of the anode (a) most of the photographic films are not sensitive to red
relative to the cathode must be made light
(a) – 4.2 V (b) – 9.4 V (b) the frequency for red light is low and hence the energy hν
of photons is less
(c) – 17.8 V (d) + 9.4 V.
(c) (a) and (b) both
[National Standard Exam. in Physics 1990]
(d) none of the above.
16. In a Millikan oil-drop experiment, a charged oil drop
24. X-rays are used to irradiate sodium and copper sur-
falls at speed 10 units without field, and rises at speed
faces in two separate experiments and stopping po-
6 units under a vertical electrical field E. Under a
tential determined. The stopping potential is
field E/2, its speed will be
(a) equal in both cases (b) greater for sodium
(a) 18 units (b) 16 units
(c) greater for copper (d) infinite in both cases.
(c) 14 units (d) 12 units.
25. The strength of a photoelectric current depends upon
[National Standard Exam. in Physics 1994] (a) frequency of incident radiation.
17. The photoelectric threshold for some material is 200 (b) intensity of incident radiation.
nm. The material is irradiated with radiations of
(c) angle of incident radiation.
wavelength 400 nm. The maximum kinetic energy of
(d) distance between anode and cathode.
the emitted photoelectrons is :
26. The idea of the quantum nature of light has emerged
(a) 2 eV (b) 1 eV
in an attempt to explain
(c) 0.5 eV (d) There is no photoemission.
(a) the thermal radiations of a black body.
18. The human eye can barely detect a yellow light (b) the interference of light.
(λ = 6000 Å) that delivers 1.7 × 10–18 W to the retina. (c) radioactivity. (d) thermionic emission.
The number of photons per second falling on the eye
27. The kinetic Energy of most energetic electrons emitted
is nearest to
from a metallic surface is doubled when the
(a) 5 × 109 (b) 500 wavelength λ of the incident radiation is changed
(c) 50 (d) 5. from 400 nm to 310 nm. The work function of the
[National Standard Exam. in Physics 1996] metal is
19. The momentum of a photon is p. The frequency asso- (a) 0.9 eV (b) 1.7 eV
ciated with it is given by (c) 2.2 eV (d) 3.1 eV.
(a) pc/h (b) ph/c 28. Stopping potential depends
(c) hc/p (d) h/pc. (a) only upon the energy of incident photon.
[National Standard Exam. in Physics 1989] (b) only on the work function of the metal.
20. Sodium surface is illuminated by ultraviolet and vis- (c) on the difference in energy of incident photon and work
function of metal.
ible radiation successively and the stopping poten-
(d) on the sum of energy of incident photon and work func-
tial determined. This stopping potential is tion of metal.
(a) equal in both cases. 29. How many photons are emitted per second by a
(b) more with ultraviolet light. 5 mW laser source operating at 632.8 nm ?
(c) more with visible light (a) 1.6 × 1016 (b) 1.6 × 1013
(d) varies randomly. (c) 1.6 × 1010 (d) 1.6 × 103.
670 COMPREHENSIVE OBJECTIVE PHYSICS

30. Fig. 30 shows the 37. Two identical metal plates show photoelectric effect.
2
plot of the stopping 1.656 Light of wavelength λA falls on plate A and λB falls on
potential versus plate B. λA = 2λB. The maximum K.E. of the photo-
Vs
the frequency of the 1
35 electrons are KA and KB respectively. Which one of
(in volt)
the following is true ?
light used in an
(a) 2KA = KB (b) KA = 2KB
experiment on 1 2 3 4 5
photoelectric effect. ν (c) KA < KB/2 (d) KA > 2KB.
14
h (x10 Hz) [All India PM/PD 1993]
The ratio is Fig. 30
e 38. An electron of charge ‘e’ coulomb passes through a
(a) 10–15 V s (b) 2 × 10–15 Vs potential difference of V volt. Its energy in ‘joule’ will
(c) 3 × 10–15 V s (d) 4.14 × 10–15 V s. be
31. In the previous question, the work function is (a) V/e (b) eV
(a) 00.212 eV (b) 0.313 eV (c) e/V (d) V. [MP PET 2000]
(c) 0.414 eV (d) 0.515 eV. 39. An electron is accelerated through a potential differ-
32. The use of photoelectric cells in cinema depends upon ence of 200 volt. If e/m for the electron be 1.6 × 1011
the fact that the number of electrons produced is pro- coulomb/kg, then the velocity acquired by the elec-
portional tron will be
(a) directly to the wavelength of light (a) 8 × 106 m s–1 (b) 8 × 105 m s–1
(b) inversely to the wavelength of light (c) 5.9 × 106 m s–1 (d) 5.9 × 105 m s–1.
(c) to the intensity of light (d) to the charge of electron. [MP PET 2000]
33. If the wavelength of incident radiation in a photo- 40. In Millikan’s experiment, an oil drop having charge
electric experiment is decreased from 6000 Å to q gets stationary on applying a potential difference V
4500 Å, in between two plates separated by a distance ‘d’.
(a) the photoelectric current may stop The weight of the drop is
(b) the photoelectric current will increase d
(a) q Vd (b) q
(c) the stopping potential will decrease V
q V
(d) the stopping potential will increase. (c) (d) q . [MP PMT 2001]
Vd d
34. The variation of photoelectric current given by the
41. Electron volt is a unit of
photocell with the intensity of light is given by a graph
(a) potential (b) charge
which is
(a) a parabola (c) power (d) energy.

(b) a straight line with positive slope passing through the origin [MP PMT 2001]
(c) a straight line with positive slope with intercept on cur- 42. Light of frequency ν is incident on a certain photo-
rent axis electric substance with threshold frequency ν0. The
(d) a straight line with negative slope. work function for the substance is
35. In order to increase the kinetic energy of ejected pho- (a) hν (b) hν0
toelectrons, there should be an increase in (c) h (ν – ν0) (d) h (ν + ν0).
(a) intensity of radiation (b) wavelength of radiation [MP PMT 2001]
(c) frequency of radiation 43. A particle of mass ‘m’ and charge ‘q’ is accelerated
(d) both the wavelength and intensity of radiation. through a potential difference of V volt. Its energy
[MP PMT 1987] will be
(a) qV (b) mqV
36. When a photon of light falls on a metal, it is absorbed
through a distance before ejection of photoelectron. F qI
(c) GH JK V (d)
q
. [MP PET 2001]
This distance is given by m mV
(a) charge on an electron 44. The metal surface is illuminated by a light of given
intensity and frequency to cause photo emission. If
(b) charge to mass ratio of electron
the intensity of illumination is reduced to one-fourth
(c) work function of the emitter
of its original value, then the maximum kinetic en-
(d) Planck’s constant. ergy of emitted photoelectrons will be
DUAL NATURE OF MATTER AND RADIATIONS 671
1 (a) V (b) V
(a) th of original value (b) unchanged
16
(c) twice the original value (c) V3 (d) V7/2.
(d) four times the original value. [MP PMT 1990] 48. The maximum velocity of electrons emitted from a
45. In photoelectric effect, metal surface is v. What would be the maximum ve-
locity if the frequency of incident light is increased
(a) light energy is converted into heat energy.
by a factor of 4 ?
(b) light energy is converted into electric energy.
(a) 2v (b) > 2v
(c) light energy is converted into sound energy.
(c) < 2v (d) between 2v and 4v.
(d) electric energy is converted into light energy.
49. The work function of a substance is 4 eV. What is
46. A photon of light enters water after travelling through the approximate longest wavelength of light that can
vacuum. The energy of the photon on entering water cause photo-emission ?
(a) remains the same because frequency does not change (a) 309 nm (b) 209 nm
(b) remains the same because the associated wavelength does (c) 109 nm (d) 9 nm.
not change
50. If the energy of a photon is 25 eV and the work func-
(c) increases because associated wavelength decreases
tion of the material is 7 eV, then the value of stop-
(d) decreases because speed decreases. ping potential is
47. The potential difference applied to an X-ray tube is (a) 3 V (b) 9 V
V. The ratio of the de Broglie wavelength of electron (c) 18 V (d) 27 V.
to the minimum wavelength of X-ray is directly pro-
portional to

Answers
1. (b) 2. (c) 3. (d) 4. (d) 5. (c) 6. (b) 7. (c) 8. (d)
9. (a) 10. (a) 11. (a) 12. (d) 13. (a) 14. (b) 15. (b) 16. (a)
17. (d) 18. (d) 19. (a) 20. (b) 21. (a) 22. (a) 23. (c) 24. (b)
25. (b) 26. (a) 27. (c) 28. (c) 29. (a) 30. (d) 31. (c) 32. (c)
33. (d) 34. (b) 35. (c) 36. (c) 37. (c) 38. (b) 39. (a) 40. (d)
41. (d) 42. (b) 43. (a) 44. (b) 45. (b) 46. (a) 47. (b) 48. (b)
49. (a) 50. (c).

Solutions
1. Ek = hν – 13.6 eV. 6. λ0 = 2000 Å
h 1 12375 12375
2. λ= mv2 max. = − = 12.375 – 6.187
2mqV 2 1000 2000
mV = constant = 6.188 eV ≈ 6.2 eV
V 7. Intensity and hence photoelectric current is inversely
1837 V′ = 1 × V or V′ = volt proportional to square of distance.
1837
4. E = hν. 8. Saturation current is inversely proportional to the
square of distance.
hc
E= 9. λ0 = 4000 Å
λ
12375
5. V I B G Y O R φ0 = eV = 3.09 eV = 3.1 eV.
4000
      
672 COMPREHENSIVE OBJECTIVE PHYSICS

h 6.2 – 2φ0 = 3.99 – φ0


10. p=
λ or φ0 = 6.2 – 3.99 = 2.21 eV
hc
Also, E= hc 6.6 × 10−34 × 3 × 108
λ 29. E= = J
λ 632.8 × 10−9
So, if λ is decreased, both p and E increase.
= 3.14 × 10–19 J
hc
12. Ek = Number of photons/s
λ
hc 5 × 10 −3
E′k = = = 1.6 × 1016
4λ 3.14 × 10 −19
E′ k 1 30. eVs = hν – φ0
v′max. 1
= or = h φ0
Ek 4 vmax. 2 Vs = ν −
e e
13. The photoelectric current is directly proportional to
intensity of incident light. h 1.656
Now, = slope = = 0.414 × 10–14 Vs
14. hν – φ0 is an upper limit for energy. There is a spread
e 4 × 1014
because electrons may be ejected from different levels = 4.14 × 10–15 Vs.
below top. 31. φ0 = hν0
15. H discharge tube means max. hν = 13.6 eV. Work or φ0 = e × 4.14 × 10–15 × 1 × 1014 J
function 4.2 V. So, (13.6 – 4.2) V or 9.4 V. So, required = 0.414 eV.
voltage is – 9.4 V. hc
16. Effect of +E equivalent to 10 – (– 6) 37. KA = – φ0
λA
E hc
will be 8 units down.
= 16 units up. So effect of – KB = − φ0
2 λB
That gives 10 + 8 = 18 down.
17. λ should not exceed λ0. But λA = 2λB
hc
∴ KA = − φ0
W 1.7 × 10 −18 × 6000 × 10 −10 2λ B
18. n= =
hc
λ
6.6 × 10 −34 × 3 × 108
KA =
LM
1 hc OP
φ
− φ0 – 0 or KA =
K B φ0

=
10−24
≈5 N
2 λB 2Q 2

2
2 × 10 −25 KB
∴ KA < .
mc2
E hν 2
19. p = mc = = =
c c c
pc + Note the special technique used in the
or ν =
h solution of this problem.
20. The frequency of ultraviolet radiation is more than
W
the frequency of visible radiation. 38. V=
q
hν W = qV = eV.
22. p=
c
3 × 10 −19 1 2eV
p= kg m s–1 39. mv2 = eV or v =
3 × 108 2 m
p = 10–27 kg m s–1 = 2 × 1.6 × 1011 × 200 m s–1
24. Sodium has low work function. So, maximum kinetic
= 8 × 106 m s–1.
energy is more in the case of sodium. Thus, stopping
potential is more for sodium. qV
40. mg = qE =
12375 d
27. Ek = – φ0 = 3.1 – φ0 41. It is the energy acquired by an electron when acceler-
4000 ated through a potential difference of 1 volt.
12375
2Ek = – φ0 = 3.99 – φ0 42. φ0 = hν0.
3100
DUAL NATURE OF MATTER AND RADIATIONS 673

W v′ 2 4 hν − φ 0
43. V= or W = qV. =
q v2 hν − φ 0
44. The maximum kinetic energy does not depend upon
intensity. v′ 2 4 [ hν − φ0 ] + 3φ0
2 =
46. E = hν v hν − φ0
h hc v′ 2 3φ0
47. λ= ; = eV or =4+
2meV λ min. v 2
hν − φ0
hc h λ
λ× = eV or ∝ V Clearly, v′ > 2v.
λ min. 2meV λ min.
1 12375
48. mv2 = hν – φ0 49. Å = 3094 Å = 309.4 nm.
2 4
1 50. eVs = 25 eV – 7 eV or Vs = 18 volt.
mv′ 2 = 4hν – φ0
2

KNOWLEDGE PLUS
l If an electron and a photon propagate in the form of waves having the same wavelength, it implies that they have
the same:
(a) energy (b) momentum (c) velocity (d) angular momentum. [AIIMS 2003]
h
Sol. de-Broglie wavelength, λ =
p
As electron and photon are having the same wavelength λ and h is a constant, therefore momentum of both of them
will be the same.
So, (b) is the right choice.

l For a given kinetic energy which of the following the smallest de-Broglie wavelength ?
(a) electron (b) proton (c) neutron
(d) deuteron (e) α-particle. [Kerala PMT 2003]
h h
Sol. de-Broglie wavelength, λ = =
p 2mE
1
For a given kinetic energy, λ ∝
m
As α-particle has largest mass (out of five given particles), therefore it has the smallest de-Broglie wavelength.
So, (e) is the right choice.

1
l A photocell is illuminated by a small bright source placed 1 m away. When the same source of light is placed m
2
away, the number of electrons emitted by photocathode would
(a) increase by a factor of 2 (b) decrease by a factor of 2.
(c) increase by a factor of 4 (d) decrease by a factor of 4. [AIEEE 2005]
1
Sol. Intensity ∝
(distance)2
Since distance is halved therefore intensity is increased by a factor of 4. Moreover photoelectric current and hence
number of photoelectrons is proportional to intensity.
So, (c) is the right choice.
674 COMPREHENSIVE OBJECTIVE PHYSICS

SELF-EVALUATION TEST II
Based on UNIT XVI

DIRECTIONS :
(i) MCQs 1 to 24 have one correct alternative.
(ii) MCQs 25 to 30 have more than one correct alternative.
(iii) MCQs 31 to 35 have one or more than one correct alternative.

1. A homogeneous ball (mass = m) of ideally black ma- (a) λe = λp (b) λe < λp


terial at rest, is illuminated with a radiation having (c) λ e > λp
a set of photons (wavelength = λ ) each with same (d) The relation between λe and λp depends on the accelerat-
momentum and same energy. The rate at which pho- ing potential difference.
tons fall on the ball is n. The linear acceleration of
the ball is 4. The momentum of the photon of wavelength 5000 Å
will be
mλ nh
(a) (b) (a) 1.3 × 10–27 kg m s–1 (b) 1.3 × 10–28 kg m s–1
nh mλ
nh 2πmλ (c) 4 × 1029 kg m s–1 (d) 4 × 10–18 kg m s–1.
(c) (d) .
(2π) (mλ) nh 5. The eye can detect 5 × 104
photons/m2s of green light
2. The photo-electric cell is connected to a source of vari- (λ = 5000 Å), while ear can detect 10–13 watt/m2. As a
able potential difference, connected across it and the power detector, which is more sensitive and by what
photo-electric current resulting (µA) is plotted against factor ?
the applied potential difference (V). The graph in the (a) Eye is more sensitive and by a factor of 5.00.
broken line represents one for a given frequency and
(b) Ear is more sensitive by a factor of 5.00.
intensity of the incident radiation. If the frequency is
increased and the intensity is reduced, which of the (c) Both are equally sensitive.
following graphs of unbroken line represent the new (d) Eye is more sensitive by a factor of 10–1.
situation ? 6. The velocity of an electron of energy 26 eV will be
(a) 106 m s–1 (b) 6 × 106 m s–1
Current (µA)
(c) 3 × 10 m s6 –1
(d) 3 × 1010 m s–1.
7. An AIR station is broadcasting the waves of wave-
A
B length 300 m. If the radiating power of the transmit-
C ter is 10 kW, then the number of photons radiated
D per second is
E
(a) 1.5 × 1029 (b) 1.5 × 1031
(c) 1.5 × 10 33
(d) 1.5 × 1035.
[MP PET 1989]
P.D.(V)
8. The work function of a photoelectric material is 3.3 eV.
Fig. 31 The threshold frequency will be equal to
(a) 8 × 1014 Hz (b) 8 × 1010 Hz
(a) A (b) B
(c) 5 × 10 Hz
20
(d) 4 × 1014 Hz.
(c) C (d) D
(e) E.
9. Photoelectric work function of a metal is 1 eV. Light
of wavelength λ = 3000 Å falls on it. The photoelec-
3. A proton and an electron are accelerated by the same
trons come out with velocity
potential difference. Let λ e and λ p denote the de-
Broglie wavelengths of the electron and the proton (a) 10 m s–1 (b) 103 m s–1
respectively. (c) 104 m s–1 (d) 106 m s–1.
DUAL NATURE OF MATTER AND RADIATIONS 675

10. If a surface has a work function 4.0 eV, what is the This means that
maximum velocity of electrons liberated from the sur- (a) the emission of photoelectrons is stopped
face when it is irradiated with ultra violet radiation (b) the photoelectrons are emitted but are reabsorbed by the
of wavelength 0.2 µm ? emitter metal
(a) 4.4 × 105 m s–1 (b) 8.8 × 107 m s–1 (c) the photoelectrons are accumulated near the collector
(c) 8.8 × 10 m s
5 –1
(d) 4.4 × 107 m s–1. plate
11. The photoelectric threshold wavelength for a metal (d) the photoelectrons are dispersed from the sides of the
surface is 6600 Å. The work function for this is apparatus.
(a) 1.87 V (b) 1.87 eV 17. Given h = 6.6 × 10 –34 Js. The momentum of each
(c) 18.7 eV (d) 0.18 eV. [MP PET 1991] photon in a given radiation is 3.3 × 10–29 kg m s–1.
The frequency of radiation is
12. An image of the sun is formed by a lens, of focal
length of 30 cm, on the metal surface of a photoelec- (a) 3 × 10 Hz (b) 6 × 1010 Hz
tric cell and a photoelectric current I is produced. (c) 7.5 × 1012 Hz (d) 1.5 × 1013 Hz.
The lens forming the image is then replaced by an- 18. The radio transmitter operates on a wavelength of
other of the same diameter but of focal length 15 cm. 1500 m at a power of 400 kilowatt. The energy of
The photoelectric current in this case is radio-photon in joule is
I (b) I (a) 1.32 × 10–24 J (b) 1.32 × 10–28 J
(a)
2 (c) 1.32 × 10–26 J (d) 1.32 × 10–32 J.
(c) 2I (d) 4I.
19. The work function of a metallic surface is 5.01 eV.
13. A modern 200 watt sodium street lamp emits yellow The photo-electrons are emitted when light of wave-
light of wavelength 0.6 µm. Assuming it to be 25% length 2000 Å falls on it. The potential difference ap-
efficient in converting electrical energy to light, the
plied to stop the fastest photo-electrons is [h = 4.14 ×
number of photons of yellow light it emits per second 10–15 eVs]
is
(a) 1.2 V (b) 2.24 V
(a) 6.2 × 1020 (b) 3 × 1019
(c) 3.6 V (d) 4.8 V.
(c) 1.5 × 1020 (d) 6 × 1018.
20. What is the angular momentum of an electron of de-
14. In a photoemissive cell, with exciting wavelength λ,
Broglie wavelength λ ? Given : r is the radius of orbit.
the fastest electron has speed V. If the exciting wave-
length is changed to 3λ/4, the speed of the fastest rh 2rh
(a) (b)
emitted electrons will be λ λ
(a) V (4/3)1/2 (b) V (3/4)1/2 3rh 4rh
(c) (d) .
(c) less than v (4/3)1/2 (d) greater than V(4/3)1/2. λ λ
[National Standard Exam. in Physics 1994] 21. A photon of wavelength 0.1 Å is emitted by a helium
15. Photo-cells have three kinds atom. The recoil energy of the atom as a consequence
of the emission of photon is
1. photoemissive cells
(a) 0.05 eV (b) 1.05 eV
2. Photovoltaic cells
(c) 2.05 eV (d) 3.05 eV.
3. photoconductive cells
22. A monochromatic source of light is placed at a large
Which of the following statements is true ? distance d from a metal surface. Photoelectrons are
(a) 1 and 2 need vacuum for functioning. ejected at rate n, the kinetic energy being E. If the
(b) 1 and 3 need external voltage for working. source is brought nearer to distance d/2, the rate
(c) only 1 and 3 are useful for light intensity measurements. and kinetic energy per photoelectron become nearly
(d) both 2 and 3 are used for solar cells. (a) 2n and 2E (b) 4n and 4E
[National Standard Exam. in Physics 1993] (c) 4n and E (d) n and 4E.
16. When stopping potential is applied in an experiment [National Standard Exam. in Physics 1992]
on photoelectric effect, no photocurrent is observed.
676 COMPREHENSIVE OBJECTIVE PHYSICS

23. In a Millikan oil-drop experiment, one of the drops The two particles
falls at speed v without field and rises at speed 2v (a) move with the same speed
with field E applied. If the field is made E/2, the (b) move with the same linear momentum
drop will (c) move with the same kinetic energy
(a) fall with speed v/4 (b) rise with speed v/2 (d) have fallen through the same height.
(c) rise with speed 3v/2 (d) remain steady.
28. In Millikan’s experiment on photoelectric effect, the
[National Standard Exam. in Physics 1995] stopping voltage (V) was measured for light of differ-
24. Which of the following has greatest associated pho- ent wavelengths λ. Which of the following are true ?
ton energy ? (a) V versus 1/λ graph was a straight line
(a) Yellow light from a sodium vapour lamp. (b) For different surfaces, the straight lines came out as
(b) A γ-ray emitted by a radioactive nucleus. parallel

(c) A radiowave emitted by the antenna of a commercial radio (c) The straight line always had a positive intercept on V axis
station. (d) The slope of the straight line gave hc/e.
(d) A microwave beam emitted by airport traffic control radar. [National Standard Exam. in Physics 1991]
25. In Millikan’s experimental graphs of Fig. 32 regard- 29. If the wavelength of light in an experiment on photo-
ing photoelectric effect, which of the following state- electric effect is doubled,
ments holds true ? (a) the photoelectric emission will not take place
(b) the photoelectric emission may or may not take place
(a) x-axis shows wavelength of light used
(c) the stopping potential will increase
(b) y-axis shows the kinetic energy of the slowest among the
electrons ejected (d) the stopping potential will decrease.

(c) the intercept on the x-axis is proportional to the work- 30. Light from a monochromatic source is incident nor-
function of the cathode mally on a small photo sensitive surface S having
work function Φ. If power of the source is W and a is
(d) the two graph lines for different cathodes are always
the distance between the source and S, then
parallel.
(a) the number of photons striking the surface per unit time
FG WλS IJ
Ek
will be H
4 πhca2 K
(b) the maximum energy of the emitted electrons will be
? 1
ν (hc – λΦ)
λ
(c) the stopping potential needed to stop the most energetic
e
photons will be (hc – λΦ)
λ
Fig. 32 (d) photo emission occurs only if 0 ≤ λ ≤ hc / Φ.
[National Standard Exam. in Physics 2002]
[National Standard Exam. in Physics 1995]
31. Electrons are emitted from a metal plate when yel-
26. Which of the following is correct ? low light is incident on its surface. Then
(a) The number of photo-electrons emitted increases if the (a) electrons will be certainly emitted from it if red light is
intensity of incident light is increased. incident on its surface
(b) The maximum kinetic energy of electrons emitted in pho- (b) electrons will be certainly emitted from the plate when
toelectric effect increases with the increase in the fre- violet light is incident on its surface
quency of incident light.
(c) when blue light is incident, electrons will be emitted and
(c) The graph between the frequency of incident light and maximum kinetic energy of emitted electrons will be
the stopping potential is a straight line. greater
(d) The photo-electrons emitted from metal surface are such (d) when blue light is incident, electrons will be emitted and
that their velocity lies between zero and a finite maximum. maximum kinetic energy of emitted electrons will be
27. In which of the following situations the heavier of smaller.
the two particles has smaller de-Broglie wavelength ? [National Standard Exam. in Physics 2000]
DUAL NATURE OF MATTER AND RADIATIONS 677

32. Shining light of wavelength λ and intensity I on a 34. Which one of the following is/are true for cathode
surface S produces photoelectrons at rate R and with rays ?
maximum kinetic energy E. Consider the following (a) They are electromagnetic waves.
statements for the effect of changing one parameter (b) They travel in straight line.
at a time (c) They have kinetic energy.
1. doubling I always doubles R
(d) They produce fluorescence in certain substances.
2. doubling I does not change E at all
35. The maximum kinetic energy of photoelectrons ejected
3. making λ half always makes E more than 2-fold
from a photometer when it is irradiated with radia-
The true statements are
tion of wavelength 4000 Å is 1 eV. If the threshold
(a) 1 and 2 only (b) 2 and 3 only
energy of the surface is 1.9 eV, then
(c) 1 and 3 only (d) all three.
(a) maximum kinetic energy will increase if the intensity of
[National Standard Exam. in Physics 1991] radiation is increased.
33. When the intensity of a light source is increased
(b) the maximum kinetic energy of photoelectrons, when it is
(a) the number of photons emitted by the source in unit time
irradiated with 50000 Å photons, will be 0.42 eV.
increases
(c) the maximum kinetic energy of photoelectrons, when it is
(b) the total energy of the photons emittd per unit time
irradiated with 5000 Å photons, will be 0.42 eV.
increases
(d) the longest wavelength which will eject the photoelec-
(c) more energetic photons are emitted
trons from the surface is nearly 6500 Å.
(d) faster photons are emitted.

Answers
1. (b) 2. (d) 3. (c) 4. (a) 5. (a) 6. (c) 7. (b) 8. (a)
9. (d) 10. (c) 11. (b) 12. (b) 13. (c) 14. (d) 15. (b) 16. (b)
17. (d) 18. (b) 19. (a) 20. (a) 21. (c) 22. (c) 23. (b) 24. (b)
25. (c), (d) 26. (a), (b), (c), (d) 27. (a), (c), (d) 28. (a), (b), (d) 29. (b), (d) 30. (a), (b), (d) 31. (b), (c) 32. (d)
33. (a), (b) 34. (b), (c), (d) 35. (c), (d).

Solutions
1. Momentum imparted per unit time = np 6.6 × 10 −34
p= kg m s–1
nh 5000 × 10 −10
=
λ = 1.32 × 10–27 kg m s–1
nh
Acceleration = 5. Energy received by the eye,

2. Intensity reduced ; current reduced. nhc
E=
Frequency increased ; stopping potential increased. λ

3.
h 5 × 104 × 6.6 × 10 −34 × 3 × 108
λ= =
2 mqV 5000 × 10−10
1 = 0.2 × 10–13 Wm–2
λ∝
m 1
So, eye is more sensitive by a factor = 5.00.
Lesser mass, larger λ. 0.200
h 1
4. p= 6. mv2 = 26 × 1.6 × 10–19
λ 2
678 COMPREHENSIVE OBJECTIVE PHYSICS

2 × 26 × 1.6 × 10 −19 4
v2 = 14. hν is made fold.
9.1 × 10 −31 3
v = 9.14 × 1012 or v = 3 × 106 m s–1
2 4
So, (hν – φ0) greater than times.
7. If n be the number of photons/s, then 3
nhc 15. Only 2 used for solar cells. Only 1 needs vacuum. All
= 10 × 103 three may be used for 1 measurement. That leaves
λ
(b) true.
or n= 10 × 103 × 300 hν pc
17. p= or ν =
6.6 × 10−34 × 3 × 108 c h
3 × 106 1 3.3 × 10−29 × 3 × 108
= = × 1032 = 1.5 × 1031 or ν= Hz
6.6 × 3 × 10−26 6.6 6.6 × 10−34
φ0 = 1.5 × 1013 Hz
8. φ0 = hν0 or ν0 =
h 12375
18. Energy corresponding to 1500 m = eV
1500 × 1010
3.3 × 1.6 × 10−19
= Hz = 8 × 1014 Hz = 8.25 × 10–10 eV
6.6 × 10−34
= 8.25 × 10–10 × 1.6 × 10–19 J
1 hc
9. mv2 max.= − φ0 = 1.32 × 10–28 J
2 λ
19. eV = hν – φ0
1
mv2max. = 12375 eV − 1 eV
2 FG 12375 − 5.01IJ
1
3000 eV = H 2000 K eV
mv2 max. = 3.125 × 1.6 × 10–19 or V = (6.1875 – 5.01) eV
2
= 1.18 V ≈ 1.2 V
2 × 3.125 × 1.6 × 10−19 rh
vmax. = ≈ 106 m s–1 20. L = mvr = pr =
9.1 × 10−31 λ
10. Energy corresponding to 0.2 × 10–6 m or 0.2 × 10–6 × p2 h2
21. E= =
1010 Å 2m 2mλ2
12375 (6.62 × 10 −34 )2 1
or 2000 Å = = 6.1875 eV = × eV
2000 2 × 4 × 1.67 × 10 −27 −10 2
× (0.1 × 10 ) 1.6 × 10−19
1 43.82 × 10 −68
mv2 max. = (6.1875 – 4) eV = = 2.05 eV
2 21.376 × 10 −68
2 × 2.1875 × 1.6 × 10 −19 22. Kinetic energy is same, that settles for (c).
or v2max. =
9.1 × 10 −31 Intensity 4-fold, so n 4-fold.
or vmax. = 0.769 × 1012 = 0.876 × 106 23. Field change 0 to E makes v change from –v to +2v,
= 8.76 × 105 m s–1. E
total change + 3v. So field change 0 to will mean
2
12375 + 3v
11. φ0 = eV = 1.875 eV v
6600 velocity change i.e. from – v to + .
2 2
12. In both the cases, the intensity is same. 25. (a) No. It shows ν (b) No, it is for the fastest electrons
25 ejected (c) Yes. W = hν′ with ν′ as intercept (d) Yes,
13. Effective power = × 200 W = 50 W
100 the slope is h (Planck’s constant).
nhc hc
Now, 50 = nhν = 28. (a) eV = − φ 0 . Hence true
λ λ
50λ (b) W changes; slope same. True
n=
hc W
(c) Intercept will be − . False
50 × 0.6 × 10−6 e
n= = 1.5 × 1020.
6.6 × 10−34 × 3 × 108
DUAL NATURE OF MATTER AND RADIATIONS 679

hc 31. ν of red < ν of yellow.


(d) Slope equals . True.
e (a) is not true.
29. If wavelength exceeds threshold wavelength, then there ν of violet > ν of yellow
would be no photo-emission. Also, due to decrease in (b) is true and (c) is true
frequency, the stopping potential decreases.
Hence (d) is not true.
30. Energy received at photo sensitive surface per unit
32. Doubling I doubles the number of electrons but does
WS not alter the maximum E. (1, 2 true). Making λ
time = .
4 πa2 half, doubles hν, and hence more than doubles
∴ number of photons = energy per unit time (hν – W) (3 true).
÷ energy of a photon.

KNOWLEDGE PLUS
l The de-Broglie wavelength of a particle moving with a velocity 2.25 × 108 m s–1 is equal to the wavelength of a
photon. The ratio of kinetic energy of the particle to the energy of the photon is
Given : (velocity of light is 3 × 108 m s–1)
(a) 1/8 (b) 3/4 (c) 5/8 (d) 7/8 [EAMCET 2003]
E1 hv λ v E1 2.25 × 10 8 225 3
Sol. = × = or = = =
E2 λ hc c E2 3 × 108 300 4
So, (b) is the right choice.

l If the kinetiic energy of a free electron doubles, its de-Broglie wavelength changes by the factor
1 1
(a) 2 (b) (c) 2 (d) [AIEEE 2005]
2 2
h h h
Sol. λ= = =
mv p 2m E k
1
λ∝
Ek
1
λ′ ∝
2Ek
λ′ 1
=
λ 2
So, (b) is the right choice.

l A photosensitive metallic surface has work function, hν0 . If photons of energy 2hν0 fall on this surface, the
electrons come out with a maximum velocity of 4 × 106 m s–1. When the photon energy is increased to 5 h ν0 , then
maximum velocity of photoelectrons will be :
(a) 2 × 107 m s–1 (b) 8 × 106 m s–1 (c) 2 × 106 m s–1 (d) 8 × 105 m s–1 [All India PM/PD 2003]
1
Sol. mv2max. = hν – φ0
2
1
m × 4 × 106 × 4 × 106 = 2hν0 – hν0 = h ν0
2
1 1 1
Again, m × v2max. = 5hν0 – hν0 = 4hv0 or m × v2max. = 4 × m × 4 × 106 × 4 × 106
2 2 2
or vmax. = 8 × 106 m s–1
So, (b) is the right choice.
UNIT XVII

ATOMIC NUCLEUS

l Alpha-particle scattering experiment l Size of the nucleus l Composition of the


nucleus—protons and neutrons l Nuclear stability l Radioactivity l Alpha, Beta and Gamma
particles/rays and their properties l Radioactive decay law l Simple explanation of α-decay,
β-decay and γ-decay l Mass-energy relation l Mass defect l Binding energy per nucleon and
its variation with mass number l Nature of nuclear forces l Nuclear reaction l Nuclear
fission and Nuclear fusion

UNIT DETAILS

1. Synopsis Points 30 Plus

2. Illustrations 15 Plus

3. MCQs from Competitive Examinations 230 Plus

4. Self-Evaluation Tests 2

5. Total Number of MCQs 345 Plus

6. Total Number of Solutions 315 Plus


CHAPTER 17

ATOMIC NUCLEUS

SYNOPSIS

1. RUTHERFORD’S ALPHA SCATTERING Thomson model of atom and it ultimately led to the rejection
EXPERIMENT of Thomson model. The backward scattering led Rutherford
to conclude that the whole of the positive charge and nearly
the entire mass of the atom were concentrated in an ex-
Fluorescent screen

tremely small central core. This ‘core’ was named as ‘nu-


cleus’ of the atom.
The graph between the angle of scattering and the
number of α-particles scattering in the corresponding direc-
Gold foil tion is as shown in Fig. 2. Rutherford concluded that the
Lead box
number N of particles scattered at an angle θ is such that
1
Fig. 1
N∝
FG θ IJ
(i) Most of the α-particles either passed straight
sin 4
H 2K
through the gold foil or were scattered by only small angles, Rutherford had analytically calculated the following
of the order of a few degrees. This observation led to the relation between the impact parameter b and the scattering
conclusion that an atom has a lot of empty space in it. angle θ.

Z e2 cot θ / 2
b=
1FG IJ
4 πε 0
2 H
mvi2
K
No. of α-particles scattered

2. DISTANCE OF CLOSEST APPROACH


At this distance, the kinetic energy of the α-particle
is transformed into electrostatic potential energy. If Z be the
atomic number of the nucleus, then

1 1 (Ze) (2e)
LM ∵ Electrostatic OP
2
mu2 =
4 πε 0 r0 MNP.E. = 4πε1 . q rq PQ
0
1 2

Scattering angle
1 4Ze2
or r0 =
Fig. 2 4 πε 0 mu2

(ii) A few α-particles were deflected through 180° i.e. 1 2Ze2 1 2Ze2
Again, r0 = = .
they were scattered in the backward direction. This back- 4 πε0 1 mu2 4 πε0 E k
ward scattering of α-particles was inconsistent with the 2

683

C-11\IITS\C17-1
684 COMPREHENSIVE OBJECTIVE PHYSICS

3. BOHR’S THEORY OF HYDROGEN ATOM This is the radius of the lowest orbit and is known as
The electrostatic force Bohr radius.
of attraction between the nu- Velocity of Electron
cleus and the electron is
nh 1 n2 h2
given by –e v= and r =
2πmr k 4 π 2me2
e2 F +e
nh × k × 4π 2me2
F=k 2πe2
r2 ∴ v= or v=k
2πm × n2 h2 nh
This force provides the v r
necessary centripetal force to 2 πe2
the electron. In general, vn = k
nh
mv2 e2 Fig. 3
FG IJ e2 F IF IJ
JK GH
=k 2 2π 1 1

r r or vn =
1
n
(ke2)
hH K or vn =
n 4 πε 0 h/2π GH K
where m is the mass of the electron and v is the velocity of
Let us now estimate how close the speed of electron in
the electron. Since the nucleus is nearly 2000 times heavier
hydrogen atom is to the speed of light in vacuum. Multiply-
than the electron therefore it is reasonable to assume that
ing the numerator and denominator of the above equation
the nucleus remains at rest.
by c, we get
ke2
∴ mv2 =
r
...(1)
F I
GG e 2
J
FG IJ JJJ
c
nh vn =
But mvr = (Bohr’s postulate) n 4πεGG h
H K K

H
0

c

nh The parenthesis of the above equation gives a dimen-


or v=
2πmr sionless constant. This is known as fine structure constant
FG nh IJ 2
ke2 α. Its value is
1
.
From eqn. (1), m
H 2πmr K =
r 137
1 n 2 h2 1 c
On simplification, r= . ∴ v=
k 4 π 2 me2 137 n
So, the speed of the electron in the innermost orbit
1 n2 h2 1
In general, rn = . 2 2
k 4 π me (n = 1) is of the speed of light in vacuum. The orbital
137
speeds in the outer orbits fall by a factor of n.
This equation gives the radii of the permitted orbits.
The radii of the permitted orbits are proportional to n2 (square Energy of Electron. An electron revolving around
of principal quantum number). For n = 1, we get the small- the nucleus possesses both the kinetic energy Ek and poten-
est orbit. This represents the normal state of the hydrogen tial energy Ep.
atom. ∴ Total energy of electron, E = Ek + Ep
1 h2 1 2
Radius of hydrogen atom, r1 = . 2 2 Now, mv2 = ke
Ek =
k 4 π me 2 2r
h = 6.63 × 10–34 J s, [From Eq. (1)]

F 1 I The potential energy Ep of electron in the field of nu-


k = 9 × 109 Nm2 C–2 = GH 4 πε 0
JK cleus is given by
(e) (e) ke2
m = 9 × 10–31 kg, e = 1.6 × 10–19 C Ep = – k =–
r r
(6.63 × 10−34 )2
∴ r1 = m ke2 ke2 ke2
9 × 109 × 4 × (3.142)2 × 9 × 10−31 × (1.6 × 10 −19 )2 ∴ E= − =−
2r r 2r
= 0.53 × 10–10 m = 0.53 Å (1 Å = 10–10 m)
ATOMIC NUCLEUS 685

r=
1 n2 h2
2 π 2me4 F1 1 I
But
k 4 π 2me2 or W = k2
h 2 GH n 1
2

n2 2 JK ...(3)

ke2 F4π 2me2 I


∴ E=–
2
kGHn 2 h2
JK Note 1. If instead of considering the nucleus of hy-
drogen, we consider a nucleus of atomic number Z, then
equation (2) becomes
2π 2me4
or E = – k2
n2 h2 2π 2Z2me4
E n = − k2
The negative sign indicates that the electron is bound n2 h2
Equation (3) becomes
to the nucleus and some work is required to be done to sepa-
rate it from the nucleus. An electron has lesser energy in an 2 π 2Z2me4 F1 1 I
inner orbit than in an outer orbit. When the electron is in W = k2
h2
GH n 1
2

n22
JK
the innermost orbit, it has least energy. The energy state
corresponding to n = 1 is known as the ground state. Since E 4. ORIGIN OF SPECTRAL LINES
depends upon n therefore energy of the hydrogen atom may
According to Bohr’s frequency condition,
as well be denoted by En.
hν = En2 − En1
2π 2me4
En = – k2 ...(2) 2 π 2me4 F 2π 2me4 I
n2 h2 or hν = − k 2
n2 h 2 2 GH
− − k2
n12 h2
JK
We can also express energy in terms of fine structure
constant α. Multiplying numerator and denominator by c2, 2π2 me4 2π2 me4
or hν = k2 − k2
we get 2 2
n1 h n22 h2

2π 2me 4 2π 2me4 F1 1 I
En = – k2c2
n 2h 2 c 2
or ν = k2
h 3 GH n 1
2

n2 2 JK
F I 2 If c is the velocity of light in vacuum and λ is the

1 mc2 GG e JJ 2
wavelength of radiation emitted, then
F1 I
GGH 4πε FGH 2hπ IJK c JJK 2π 2me4
or En = – c 1
2 n2
0 λ
= k2
h3
GH n 1
2

n K
J 2
2 [∵ ν = c/λ]

1 mc2 2 1 2π 2me4 F1 1 I
or En = –
2 n2
α or
λ
= k2
ch3
GH n 1
2

n K
J 2
2

It was shown by Sommerfield that when the restric-


1
tion of circular orbits is relaxed, the above equation contin- Here, is the wave number. It is defined as the
ues to hold even for elliptical orbits. λ
number of waves in unit distance. It is denoted by ν .
1
The value of mc2α2 is 13.6 eV. 2π 2me 4 F1 1 I
2
13.6
ν = k2
ch 3 GH n 1
2

n2 2 JK
∴ En = – 2
eV
n
F 2π 2 me4 I is known as Rydberg constant
If W is the work done in moving an electron from a
lower energy level (n1) to higher energy level (n2), then
GH
The term k2
ch 3 JK
W = E n2 − E n1 after the name of the Swedish spectroscopist J. Rydberg. It
is denoted by R. Its value is 1.0974 × 107 m–1. It agrees
where En2 and En1 are the energies corresponding to quan- closely with the experimental value.
tum number n2 and n1 respectively.
F1 1 I
LM OP − LM− k OP
∴ ν=R GH n 2

n22
JK
2 π 2 me4 2π 2 me4 1
W = − k2 2
MN 2 2
n2 h PQ MN n12 h2 PQ The above relation, known as Rydberg formula for
the spectrum of the hydrogen atom, is extremely useful in
686 COMPREHENSIVE OBJECTIVE PHYSICS

understanding the origin of spectral lines. Various spectral Balmer series is so named because it was discovered
lines of different frequencies are produced for different val- by Balmer in 1885. The first four lines of Balmer series lie in
ues n1 and n2. The spectrum of the hydrogen atom was de- the visible region of the spectrum. As many as twenty-nine
termined first in the late nineteenth century. At that time, lines belonging to Balmer series have been detected. This
the few lines in the visible region were noted by Balmer, a
series goes to the ultraviolet region of the spectrum.
school teacher in Switzerland.
(iii) Paschen series. The spectral lines of this series
(i) Lyman series. The spectral lines of this series
correspond to the transition of an electron from some higher
correspond to the transition of an electron from some higher
energy state to the innermost orbit (n = 1). energy state to an orbit having n = 3.
For Lyman series, n1 = 1 and n2 = 2, 3, 4, .... For Paschen series, n1 = 3, n2 = 4, 5, 6, .....
The wave numbers and the wavelengths of the spec- The wave numbers and the wavelengths of the spec-
tral lines constituting the Lyman series are given by tral lines constituting the Paschen series are given by

F I 1 F1 1 I
ν=
1
λ
1 1
=R 2 − 2
1 n2
GH JK ν=
λ GH
=R 2 − 2
3 n2
JK
Paschen series is so named because it was discovered
Lyman series was first predicted by Bohr. This series
by Paschen. Just like other series, this series was first pre-
lies in the ultraviolet region of the spectrum. It is not visible
with naked eye. But it can be photographed. This series was dicted by Bohr. Paschen series lies in the infrared region of
first photographed by T. Lyman of Harvard University in the spectrum and is invisible.
1916. (iv) Bracket series. The spectral lines of this series
(ii) Balmer series. The spectral lines of this series correspond to the transition of an electron from a higher
correspond to the transition of an electron from some higher energy state to the orbit having n = 4.
energy state to an orbit having n = 2. For this series, n1 = 4 and n2 = 5, 6, 7, ....
For Balmer series, n1 = 2, n2 = 3, 4, 5, .... The wave numbers and the wavelengths of the spec-
The wave numbers and the wavelengths of spectral tral lines constituting the Bracket series are given by
lines constituting the Balmer series are given by
F I 1 F1 1 I
1 1 1
ν= =R 2 − 2 GH JK
ν=
λ GH
=R 2 − 2
4 n2
JK
λ 2 n2
This series lies in the infrared region of the spectrum.
(v) Pfund series. The spectral lines of this series
correspond to the transition of electron from a higher energy
state to the orbit having n = 5.
s

For this series, n1 = 5 and n2 = 6, 7, 8, ...


serie

The wave numbers and the wavelengths of the spec-


an

tral lines constituting the Pfund series are given by


Lym

erie
s 1 F1 1 I
Balm
er s ν=
λ GH
=R 2 − 2
5 n2
JK
This series lies in the far infrared region of the
n=1 spectrum.
Pa
sch (vi) Humphery series. For this series, n1 = 6 and
n=2 en
ser
ies n2 = 7, 8, ...
n=3
All the above spectral series have been shown in
Br
ack

n=4 Fig. 4. This figure is called Kossel diagram.


et

The impressive numerical agreement between the


ser
Pfund

n=5
ies

theoretically calculated values of the spectral frequencies of


the hydrogen atom and the experimentally observed spec-
series

n=6
trum speaks of the brilliant success of Bohr’s work. Niels
n=7 Bohr was awarded the 1922 Nobel Prize for Physics for this
Fig. 4
work.
ATOMIC NUCLEUS 687

5. ENERGY LEVELS OF HYDROGEN Continuum

2π 2me4 0 n=¥
We know that En = – k2 n=7
n2 h2 n=6
0.54 n=5
Now, k = 9 × 109 N m2 C–2, Pfund
series
m = 9 × 10–31 kg, e = 1.6 × 10–19 C, 0.85 n=4
Bracket
h = 6.6 × 10–34 J s series

(9 × 109 )2 × 2 × (3.142)2 × 9 × 10−31 – 1.51 n=3


Paschen
× (1.6 × 10−19 ) 4 series
∴ En = – joule
n (6.6 × 10−34 )2
2

W (Energy in eV)
2.15 × 10−18
or En = – joule – 3.4 n=2
n2 Balmer
series
2.15 × 10 −18
or En = – eV
n2 × 1.6 × 10−19
[∵ 1 eV = 1.6 × 10–19 J] Thick arrows show
series limits.
13.6
or En = – eV
n2
This equation gives the binding energy of the electron
in the nth orbit of hydrogen atom. The negative sign signi-
fies that the electron is bound to the nucleus.
For n = 1 (K shell), we get the energy E1 of the ground
state. 1.6 n=1
Lyman
series
13.6
∴ E1 = – eV = – 13.6 eV
12 Fig. 5

This ground state energy of the hydrogen atom gives 6. ENERGY QUANTISATION
the ionisation energy of the atom. This is the minimum The process of restricting the possible values
amount of energy required to remove an electron from n = 1 of a physical quantity to a set of discrete values is
to n = ∞. For hydrogen atom, its value is 13.6 eV. Thus, if an called quantisation. We know that a stretched string can
energy of 13.6 eV is given to the electron in the hydrogen oscillate only in definite stationary states whose frequencies
atom, then the electron will be knocked out completely from are determined by elastic properties and boundary condi-
the atom. In other words, atom will be ionised. It may be tions. Since both ends of a stretched string clamped at its
ends must remain forever at rest therefore the wavelengths
clearly noted that the ionisation energy (13.6 eV) is only
of the allowed stationary states are related to the length of
numerically equal to the ground state energy (– 13.6 eV).
the string. The allowed wavelengths are given by
Normally, the hydrogen atom will be in its ground
2L
state. It is nearest to the nucleus and has size given by Bohr λ=
radius. When the hydrogen atom receives energy by proc- n
esses such as electron collisions, the electron can make a where L is the length of the string and n = 1, 2, 3, 4, .....
transition to states with higher energy which are therefore In order to understand discretisation in quantum
called the excited states. physics, let us make use of de-Broglie hypothesis i.e., parti-
For n = 2 (L shell), we get energy E2 of the atom when cles are wavelike. Let us postulate that just like a vibrating
the electron is in the second orbit. string, a free particle of mass m confined to a line of length L
can have only those values of the momentum for which the
13.6
E2 = –
22
eV = –
13.6
4
eV = – 3.4 eV
de-Broglie wavelength belongs to the set λ =
LM 2L OP
N n Q
. It follows

from here that


688 COMPREHENSIVE OBJECTIVE PHYSICS

h h (iii) The nuclear density is not uniform throughout


p= or p = n the nucleus. It has maximum value at the centre and de-
2L 2L
n creases gradually as we move away from the centre of the
In terms of momentum, the energy of a free particle nucleus.
is given by (iv) The nuclear radius is the distance from the cen-
tre of the nucleus at which the density of nuclear matter
p2 n2 h2 decreases to one-half of its maximum value at the centre.
E= or E =
2m 8mL2
9. ISOTOPES
By assigning the values 1, 2, 3, 4, ....... to ‘n’, we get
the stationary states in ascending order of energy. Accord- These are the atoms of the same element having the
ing to Bohr, a quantum system can only be in one of the same atomic number but different atomic weights. Such
stationary states. The stationary state of the lowest energy nuclides having same number of protons but different number
is called the ground state. For the quantum system under of neutrons are called isotopes. In other words, they have
same atomic number Z, but different mass number A.
h2
discussion, the stationary state with energy is the The number of orbital electrons is the same in isotopes
8mL2
ground state. of elements. This explains as to why they possess identical
7. SIZE OF NUCLEUS chemical properties. Thus, such atoms have the same (iso)
place (tope) in the periodic table. On account of the different
It has been confirmed by various experiments that
the nucleus does not have a sharp or well-defined boundary. masses, isotopes possess different physical behaviour and
properties.
However, the nuclear radius R can be given by
R = R0 A1/3 The isotopes of an element may have same name or
different names. As an example, the three isotopes 1H1, 1H2,
where R0 (= 1.2 × 10–15 m) is a constant which is the same
for all nuclei and A is the mass number of the nucleus. and 1H3 of hydrogen are known as Hydrogen, Deuterium
and Tritium. All the three names are different. On the other
The unit of nuclear radius is fm. 1 fm = 10–15 m
hand 17 Cl37 and 17Cl35 are merely known as isotopes of
The nuclear radii range from 1 fm to 10 fm.
chlorine.
4 4
Nuclear volume, V = πR3 = π (R0 A1/3)3 Nearly all the known elements have one or more
3 3 isotopes. While Hg (mercury) has nine isotopes, tin has as
4 many as ten isotopes (largest number).
= π R03 A
3 The isotopes can occur either naturally or can be
It is clear from here that the nuclear volume is pro-
produced artificially in the laboratory.
portional to mass number.
10. ISOBARS
8. NUCLEAR DENSITY
Isobars are atoms of different elements having the
Consider a nucleus of mass number A and radius R.
same atomic mass but different atomic number. Since Z
Mass of nucleus = A amu = A × 1.66 × 10–27 kg
numbers of isobars are different therefore they do not occupy
Volume of nucleus
the same place in periodic table. Also, for the same reason,
4 4 22 their chemical properties are widely different from each other.
= πR03A = × × (1.2 × 10–15)3 A m3
3 3 7 The light nuclei have unstable isobars. However, heavy
= 7.24 × 10–45 A m3 nuclei have stable isobars and these occur in pairs.
There is a special property of isobars. It is that the
A × 1.66 × 10 −27
Density of nucleus, ρ = kg m–3 number of protons in one isobar equals the number of protons
7.24 × 10 −45 × A in another and vice versa. Such isobars are called mirror-
= 2.29 × 1017 kg m–3 nuclides of each other.
Discussion. (i) The nuclear density does not depend Examples of isobars. (i) 32Ce76 and 34Se76 ; (ii) 18Ar40
upon mass number. So, we can safely conclude that all nuclei and 20Ca40 ; (iii) 26Fe58 and 27Ni58.
possess nearly the same density.
11. ISOTONES
(ii) The nuclear density has extremely large value.
Such high densities are found in white dwarf stars which The atoms whose nuclei have the same number of
contain mainly the nuclear matter. neutrons are called isotones.
ATOMIC NUCLEUS 689

In this case, (A – Z) = N is the same. For example, the neutron is equal to the average kinetic energy of hydrogen
Cl 37 and 39 are isotones. This is because number of
17 19 K molecules in the moderator. Thus, the neutrons are now in
neutrons in chlorine (= 37 – 17 = 20) and number of thermal equilibrium with the molecules of the moderator.
neutrons in potassium (= 39 – 19 = 20) are same. These are termed as ‘thermal neutrons’ . Obviously, these
12. PROPERTIES OF NEUTRONS are quite slow neutrons having a low energy. These are used
Among its properties, the following may be listed : to split up heavy nuclei like uranium etc.
(i) Neutrons are fundamental constituents of a nu- 14. MASS DEFECT
cleus (all except hydrogen). Inside a nucleus, neutrons stay ∆m = [Zmp + (A – Z)mn] – M
for ever ; but as a projected particle outside it, it exists for a
As an example, let us consider the mass defect in the
short time only. The ‘half-life’ of a neutron is about 13 min-
case of deuteron. It is an isotope of hydrogen. It contains one
utes. A neutron, which is free to move, decays into a proton,
proton and one neutron.
an electron and a new particle called ‘anti-neutrino’. Thus,
1
mp = mass of proton = 1.007825 amu
0n → 1H1 + –1 e
+ ν. 0
mn = mass of neutron = 1.008665 amu
(ii) In nuclei of heavier elements, the number of neu-
mp + mn = 2.016490 amu
trons is greater than the number of protons. It is this abun-
dance of neutrons which makes the elements stable. If, how- M = mass of deuteron = 2.014103 amu
ever, the number of protons increases, due to Coulomb re- Mass defect, ∆m = mp + mn – M = 0.002387 amu
pulsion, the stability of that nucleus decreases. 15. BINDING ENERGY
(iii) Since neutrons are uncharged particles, therefore The binding energy is related to mass defect by Ein-
these are neither affected by external magnetic or electric stein’s mass-energy relation. If ∆m be the mass defect, then
fields nor by the presence of protons when they enter or Binding energy = ∆m c2 where c is the speed of light.
penetrate the nucleus. Thus, neutrons are almost ideal In the case of deuteron,
‘bullets’ for performing experiments on artificial
Binding energy = 0.002387 × 931 MeV = 2.22 MeV
transmutations.
Thus, an energy of 2.22 MeV is required to separate
(iv) Depending upon their speed, neutrons are put in by an infinite distance a neutron from proton. This has been
two categories, (a) fast neutrons and (b) slow neutrons. Both confirmed experimentally.
are fully capable of penetrating a nucleus and causing artifi-
16. NUCLEAR FORCES
cial disintegration in the nucleus. An example of a high speed
neutron is (i) Nuclear forces are ordinarily attractive.
1 198 198 (ii) Nuclear forces are charge-independent.
0n + 80Hg → 79Au + 1H1
(iii) Nuclear forces are short-range forces.
Here mercury is converted into gold by artificial trans-
(iv) Nuclear forces are spin-dependent.
mutation, on being bombarded with high speed neutrons.
Proton is given out. An example of a slow speed neutron is (v) Nuclear forces show saturation properties.
1 (vi) Nuclear forces are non-central forces.
0n + 5B10 → 3Li7 + 2He4
Boron bombarded with neutron gives lithium and Force Relative strength
α-particle is given out. Gravitational force 1
13. THERMAL NEUTRONS Coulomb’s force 1036
Fast neutrons can be converted into slow neutrons by
Nuclear force 1038
letting them pass through materials, which decrease their
speed. Such materials are known as ‘moderators’. Essentially,
17. BINDING ENERGY CURVE
these moderators are rich in hydrogen atoms-like paraffin,
Fig. 6 shows binding energy curve. The average bind-
deuterium oxide (heavy water) etc. This slowing down of the
ing energy per nucleon is plotted against mass number for
speed takes place on the well-known laws of kinematics. When naturally occurring nuclei.
two particles of the same mass collide (here neutron and
Fig. 7 shows binding energy curve for light nuclei on
proton in the hydrogen of the moderator), their velocities are a magnified scale.
interchanged. Thus, the speed of a neutron is slowed down,
Following are the special features of binding energy
while the speed of the proton or the hydrogen in the moderator curve.
is increased. This process goes on till the kinetic energy of
690 COMPREHENSIVE OBJECTIVE PHYSICS

(i) The binding energy per nucleon of very light strongly bound) as compared to their neighbours. It is
nuclides such as 1H2 is very small. interesting to note that the mass numbers of these nuclides
are multiples of 4 and they contain equal number of protons
Region of
max. stability and neutrons. Each of these nuclei can be formed by adding
an α-particle to the preceding nucleus.
Fusion Fission
(iv) After mass number 20, there is a gradual increase
in binding energy per nucleon. The maximum value is
9
8.8 reached at A = 56. This value is 8.8 MeV. Clearly, the iron
nucleus (26Fe56) is the most stable.
e
8 cd
b
a
Average B.E./Nucleon (in MeV)

7 (v) The binding energy per nucleon of nuclides having


mass numbers ranging from 40 to 120 are close to the maxi-
6
a 4 mum value. So, these elements are highly stable and non-
He
5
8 radioactive.
b Be
4
c 12 (vi) Beyond A = 120, the value decreases and falls to
C
3 d 16 7.6 MeV for Uranium. This decrease is primarily due to re-
O
e 20 pulsion among protons whose number increases in heavy
2 Ne
nuclides.
1 238 (vii) Beyond A = 238, the binding energy per nucleon
shows a rapid decrease with increase in mass number. This
0 40 56 80 120 160 200 240
makes elements beyond Uranium (trans-uranium elements)
Mass number (A)
quite unstable and radioactive. They rarely exist.
Fig. 6 (viii) The fact that the binding energy curve
“droops” at both high and low mass numbers has
very important practical consequences.
The drooping of the binding energy curve at high mass
numbers tells us that nucleons are more tightly bound when
they are assembled into two middle-mass nuclei rather than
into a single high-mass nucleus. In other words, energy can
be released in the nuclear fission of a single massive nucleus
into two smaller fragments.
The drooping of the binding energy curve at low mass
numbers, on the other hand, tells us that energy will be
released if two nuclei of small mass numbers combine to
form a single middle-mass nucleus. This process, the re-
verse of fission, is called nuclear fusion. It occurs inside our
Sun and other stars and is the mechanism by which the
Sun generates the energy it radiates to us.
18. SODDY’S DISPLACEMENT LAW OF RADIOAC-
TIVE TRANSFORMATIONS
Fig. 7 1. When a nucleus ejects an α-particle, the mass
becomes less by 4 units and charge decreases
(ii) Initially, there is a steep rise in curve. This indi- by 2 units. Thus, the nucleus ZYA on emission of
cates a rapid rise in the value of binding energy per nucleon. α-particle gets transformed into a new nucleus
A–4.
(iii) Between mass numbers 4 and 20, the curve shows Z–2Y
cyclic recurrence of peaks corresponding to 2He4, 4Be8, 6C12, ∴ A α A–4
ZY → Z–2Y
8
O16 and 10Ne20. This shows that the binding energy per
Thus, the substance shifts or is displaced from its
nucleon of these nuclides is greater than those of their
original position in the periodic table, two steps
immediate neighbours. So, these are more stable (or more
backwards.
ATOMIC NUCLEUS 691

2. When a nucleus ejects a β-particle, the mass re- N


mains unchanged and the charge increases by 1 or = e– λ t ;
N0
unit. So a material ZYA on emission of β-particle
gets transformed into a new nucleus as Z+1YA. or N = N0 e– λ t; as shown in Fig. 8.
Thus the original substance shifts or is displaced 20. RADIOACTIVE DISINTEGRATION
one step higher in the periodic table. CONSTANT λ
A A According to the laws of radioactive decay, we have
Thus ZY → Z+1Y
β
dN
3. When a nucleus emits γ-rays, the mass or the = – λ dt
charge or the position of the nucleus in the peri- N
odic table are not affected. Only some energy is dN
If dt = 1 second, then =–λ
radiated and the original nucleus shifts from higher N
energy level to lower energy level. Thus, λ may be defined as the relative number of at-
19. RUTHERFORD AND SODDY’S LAWS OF RA- oms decaying per second.
DIOACTIVE DECAY Again, since N = N0 e– λ t
1. The disintegration of radioactive material is purely 1 N
a random process and it is merely a matter of and if , we get N = N0 e–1 = 0
t=
λ e
chance, which nucleus will suffer disintegration,
Thus, λ is also defined as the reciprocal of the time
or decay first.
2. The rate of decay is completely independent of the N 1
when falls to .
physical composition and chemical condition of the N0 e
material.
21. HALF-LIFE PERIOD
3. The rate of decay is directly proportional to the
quantity of material actually present at that in- Consider the situation when the decaying material is
stant. Thus as the decay goes on, the original ma- 1
terial goes on decreasing in quantity and the rate reduced to exactly of its original quantity. The time taken
2
of decay consequently goes on decreasing.
FG 1 IJ
Thus from the third law, if N is the number of radio-
active atoms present at any instant, then the rate of H
for this decay N/N 0 =
2 K
is called the half-life period of the

decay, material. It is defined as the time required for the dis-


appearance of half of the amount of the radioactive
N0
dN N substance originally present.
− ∝N
dt If T represents the half-life period, then
dN N 1
or − = λN, N0/2 = = e– λT or eλT = 2
dt N0 2
where λ is the decay N0/4 ∴ λT = loge 2 = 0.6931
constant or the disin- N0/8
N0/16 0.6931 0.6931
tegration constant. 0 T 2T 3T 4T ∴ T= or λ=
λ T
Time in half-lives
dN Combining these relations, we obtain
∴ = – λN Fig. 8
dt
N N0
= e− λt or = e λt
dN N0 N
On re-arranging, = – λ dt
N
On integration, loge N = – λ t + C N0 N 0 0.6931
∴ loge = λt or 2.303 log10 = t
where C is the integration constant. N N T
If at t = 0, we had N0 atoms, loge N0 = 0 + C 2.303 N
or t= T log10 0
Thus, we get loge N – loge N0 = – λ t 0.6931 N
N N0
or loge =–λt or t = 3.323 T log10
N0 N
692 COMPREHENSIVE OBJECTIVE PHYSICS

This relation shows that a material with a half-life The activity of a radioactive substance is said
period T changes in quantity from N0 to N in time t. to be one curie if it undergoes 3.7 × 1010 disintegrations
per second.
22. FORMULA FOR NUMBER OF ATOMS LEFT
BEHIND AFTER n HALF LIVES 1 curie = 3.7 × 1010 disintegrations/s
Let N 0 be the number of atoms of a radioactive Smaller units are millicurie and microcurie.
substance in the beginning. After time T, the number of 1 millicurie = 3.7 × 107 disintegrations/s
N
atoms left will be . After a time 2T, the number of atoms 1 microcurie = 3.7 × 104 disintegrations/s
2
1 N0 FG IJ . After a time 3T, the number
1
2
(ii) The rutherford (Rd). The activity of a radio-
left will be
2
×
2 H K
i.e., N0
2 active substance is said to be one rutherford if it
undergoes 106 disintegrations per second.
1 F 1I F 1I 2
× N G J i.e., N G J . Proceeding
3
of atoms left will be
2 H 2K 0 H 2K 0
1 rutherford = 106 disintegrations s–1
Smaller units are millirutherford and microrutherford.
in the same way, the number of atoms left behind after n
1 millirutherford = 103 disintegrations s–1
FG 1IJ n
half-lives will be N0
H 2K . 1 microrutherford = 1 disintegration s–1
(iii) The becquerel (Bq). The activity of a radioac-
FG 1IJ n tive substance is said to be one becquerel if it under-
∴ N = N0
H 2K goes 1 disintegration per second.
1 becquerel = 1 disintegration s–1
If t is the time corresponding to n half-lives, then Relation between different units
t 1 curie = 3.7 × 104 rutherford = 3.7 × 1010 becquerel.
t = n T or n=
T 25. ALPHA DECAY

FG 1IJ t/ T All nuclei with Z > 83 and A > 209 spontaneously


∴ N = N0
H 2K transform themselves into lighter ones through the emis-
sion of one or more alpha particles which are 2He4 nuclei.
Since an alpha particle consists of two protons and two neu-
23. AVERAGE OR MEAN LIFE OF RADIOACTIVE
ELEMENTS trons therefore an alpha decay reduces the atomic number
by two and atomic weight by four. Transformation of ZXA
It is the average time for which an atom of a
nucleus into Z–2YA–4 nucleus by an alpha decay is expressed
radioactive substance exists. It is given by the average
of the life span of all the atoms of the radioactive element. It by the following equation :
A A–4 + He4
is denoted by τ. ZX → Z–2 Y 2

1 The energy Q released in the process can be obtained


τ= from Einstein’s mass-energy relation. It is given by
λ
Q = (mX – mY – mHe) c2
So, the average life of a radioactive element is the
reciprocal of its decay constant. This energy is shared both by the daughter nucleus
YA–4 and the alpha particle He4.
Z–2 2
0.693
Note. Half-life = = 0.693 τ Speed of α-particles
λ
Transformation of the ZXA nucleus into the Z–2YA–4
24. UNITS OF RADIOACTIVITY nucleus by an alpha decay can be expressed by the equation
The activity of a radioactive sample is generally ex- A A–4 + He4
ZX → Z–2 Y 2 ...(1)
pressed in terms of its rate of decay. In other words, the The energy, Q, released in this process, can be obtained
activity of a radioactive sample is expressed in terms of the from Einstein’s mass-energy relation.
number of disintegrations per unit time. The radioactivity
It is given by the equation
is measured in the following three units :
Q = (mX – mY – mHe)c2 ...(2)
(i) The curie (Ci). This was originally defined as the
This energy is shared by the daughter nucleus,
activity of 1 g of radium in equilibrium with its by-products. A–4
Z–2 Y and the alpha-particle, 2He4.
But it is now defined as under :
ATOMIC NUCLEUS 693

As the parent nucleus ZXA is at rest before it undergoes of conservation of energy itself. We cannot even say that the
alpha-decay, alpha-particles are emitted with fixed energy, ‘energy difference’ is taken away by γ-rays because there is
which can be calculated by applying the principles of no emission of γ-radiation in the example under considera-
conservation of energy and momentum. Let vHe and vY be tion. In addition to this difficulty, even the law of conserva-
the velocities of the alpha-particle and the daughter nucleus, tion of momentum seems to be violated.
A–4 . The principle of conservation of momentum gives
Z–2Y All these difficulties were overcome in 1933 by
mYvY = mHe vHe ...(3) Wolfgang Pauli who proposed that a second particle is also
By equating the sum of kinetic energies of the nu- emitted and assigned theoretically the following properties
cleus Y and the alpha particle to the energy released in the to this particle :
alpha-decay, we have another equation (i) zero rest mass (ii) zero charge
1 1 1
m v2 + m v2=Q ...(4) (iii) a spin equal to .
2 He He 2 y y 2
By substituting for vy from eq. (3) in eq. (4), you can Enrico Fermi developed the theory of this new parti-
easily obtain cle and called it neutrino. It was later on found that two
1 mY kinds of neutrinos are involved in beta decay, the neutrino
m v2 = Q ...(5) and the antineutrino. In the ordinary beta decay, it is an
2 He He m y + mHe
antineutrino that is emitted.
− A – 4 amu and mHe ~
If we substitute my ~ − 4 amu in n → p + e + ν
eq. (5), the kinetic energy carried by the alpha-particle can
be approximated by the relation Note. The process of beta-decay is a characteristic of nuclei
having a large proportion of neutrons.
1 ( A − 4)
KEHe = m v2 ~ Q ...(6) 27. GAMMA DECAY
2 He He − A
Just like an excited atom, an excited nucleus can make
In the decay of 86Rn222, Q = 5.587 MeV and KEHe a transition to a state of lower energy by emitting a photon.
= 5.486 MeV. The velocity of the alpha-particle emitted by
222 can be easily estimated from its kinetic energy. The energies of the atomic states of hydrogen are of the order
86Rn of electron volts. So, the wavelength of light emitted in atomic
2 × 5.486 × 1.6 × 10 −13 m s–1 transitions correspond to photons having energy of the order
vHe = of electron volts. On the other hand, the energies of the
4.00 × 1.66 × 10−27
nuclear states are of the order of million electron volts. So,
= 1.63 × 107 m s–1 the photons emitted by nuclei can have energy of the order of
26. BETA DECAY several million electron volts. The wavelength of photons of
such energy is a fraction of an angstrom. The short
It is a process in which an electron is emitted by a
wavelength electromagnetic waves emitted by nuclei are
nucleus. It is difficult to understand this process because
called the gamma rays. Most radio isotopes, after an alpha-
there are strong arguments against the presence of electron
decay or a beta-decay, leave the daughter nucleus in an
in the nucleus. However, this difficulty is solved if we as-
excited state. An excited nucleus is denoted by an asterisk
sume that (i) a neutron inside the nucleus breaks up into a
after its usual symbol. Thus, 38Sr87*, refers to 38Sr87 in an
proton and an electron (ii) the electron is ejected from the
excited state.
nucleus immediately after its creation. In other words, we
have assumed that the neutron within the beta-emitting Excited nuclei return to their ground states by emit-
nucleus is radioactive just like a free neutron. ting photons whose energies correspond to the energy differ-
ences between the various initial and final states in the tran-
If we study the energy distribution of electrons emit-
sitions involved. The photons emitted by nuclei range in en-
ted in the β-decay of 83Bi210, it is observed that the energy
ergy upto several MeV. These are traditionally called gamma
varies between zero and 1.17 MeV. It is of course very rare
rays.
that the emitted electron has an energy of 1.17 MeV. Most of
the electrons have an energy of 0.15 MeV. This presents Fig. 9 (a) shows the beta decay of 12Mg27 to 13Al27. The
another serious difficulty in the proper understanding of the half life of the decay is 9.5 min and it may take place to
process of beta-decay. For every 89Bi210 decay, the same mass either of the two excited states of 13Al27. The resulting 13Al27*
vanishes but very few emitted electrons possess an energy of nucleus then undergoes one or two gamma decays to reach
1.17 MeV. This creates doubts about the validity of the law the ground state.
694 COMPREHENSIVE OBJECTIVE PHYSICS

The fission is accompanied by the release of three neu-


27 60
12Mg 27Co trons and radiation energy in the form of γ-rays. The reac-
tion is represented as
1 235 → ( U236) → 141 + 92 + 3 n1 + γ
0n + 92 U 92 56Ba 36Kr 0
β β β
Energy released in fission. The fission fragments
Barium, Krypton and neutrons are released with high ve-
1.015 MeV locities. Also energy is released in the form of γ-rays. An
γ Eγ = 1.17 MeV
estimate can be made as in the example given below :
0.834 MeV
Before the reaction :
γ γ Mass of 235
92 U = 235.0439 amu
1.33 MeV

0 Mass of 0n1 = 1.0087 amu


27 60
13Al 28Ni Total mass = 236.0526 amu ...(i)
After the reaction :
Fig. 9 (a) Fig. 9 (b) 141
Mass of 56Ba = 140.9129 amu
Let us now consider another example. It is the decay 92
Mass of 36Kr = 91.8973 amu
of 27Co60. By beta emission, the 27Co60 nucleus is first trans-
formed into an excited 28Ni60 nucleus which in turn reaches Mass of three 0n1 = 3.0261 amu
the ground state by emitting photons of energies 1.17 MeV Total mass = 235.8363 amu ...(ii)
and 1.33 MeV. Fig. 9 (b) shows the process through an en- Mass defect = 0.2163 amu [(i) – (ii)]
ergy level diagram Since 1 amu = 931 MeV,
As an alternative to gamma decay, an excited nucleus ∴ The energy released
in some cases may return to its ground state by giving up its = 931 × 0.2163 = 201.37 MeV ≈ 200 MeV
excitation energy to one of the orbital electrons around it.
This is a huge figure. Calculations reveal that 235 g
This process is known as internal conversion. It can be
of Uranium, on complete fission, releases energy equivalent
regarded as a kind of photoelectric effect in which a nuclear
to the burning of about 600 tonnes of coal. However, this 200
photon is absorbed by an atomic electron. The internal
MeV consists of K.E. of fission fragments of released neu-
conversion in-fact represents a direct transfer of excitation
trons and of the γ rays.
energy from a nucleus to an electron. The emitted electron
has a kinetic energy equal to the lost nuclear excitation energy 29. NUCLEAR FUSION
minus the binding energy of the electron in the atom. We know ‘fission’ to be a process in which a heavy
Most excited nuclei have very short half-lives against nucleus breaks up into two lighter nuclei. Fusion, on other
gamma decay. But a few remain excited for as long as sev- hand, is the reverse of fission. Thus, fusion is a process in
eral hours. A long-lived excited nucleus is called an isomer which lighter nuclei merge into one another to form a heavier
of the same nucleus in its ground state. The excited nucleus nucleus. As in fission, fusion also is accompanied by a re-
87
38Sr * has a half-life of 2.8 hour and is accordingly an iso-
lease of energy.
mer of 38Sr87. The binding energy per nucleus thus formed is greater
28. NUCLEAR FISSION than the binding energy per nucleon of the lighter elements,
which fuse to form the single nucleus. Taking an example,
γ γ let us consider the fusion of the deuterium nuclei to form a
141
56Ba single helium nucleus.
We know that mass of a deuteron = 2.01471 amu
235 n
92U ∴ Mass of two deuterons = 4.02942 amu
n Mass of α-particle (i.e., a Helium nucleus)
n
n = 4.00388 amu
92
∴ ∆m, mass defect = 0.02554 amu
36Kr Since 1 amu = 931 MeV,
γ γ
∴ The energy liberated = 0.02554 × 931 MeV
Fig. 10 = 23.78 MeV ≈ 24 MeV
ATOMIC NUCLEUS 695

Thus, a single helium nucleus formed out of fusion of 1


two deutrons (i.e., deuterium nuclei) releases 24 MeV energy. or f∝ 3/2
r
In case a large number of helium nuclei are fused, we readily
see that a tremendous amount of energy is released. 2.
2
30. SOURCE OF ENERGY IN THE SUN AND THE r∝n
16
STARS
(i) Proton-Proton Cycle. The interior of Sun is at
r 12
about 27 million K. The thermo-nuclear reactions taking
place are as follows :
1 8
1H + 1H1 → 1H2 + 1e0 + 0.4 MeV ...(i)
1
1H + 1H2 → 2He3 + 5.5 MeV ...(ii)
3 3 4 1
2He + 2He → 2He + 2 1H + 12.9 MeV ...(iii) 4

If we imagine that the reactions (i) and (ii) occur twice,


then
1 2 3 4 n
41H1 → 2He4 + 21e0 + 24.7 MeV
Fig. 11
Thus at that high temperature available in the core
of the Sun, four protons fuse into a Helium nucleus with the
release of two positrons and 24.7 MeV of energy. 3. nh
L=
(ii) Carbon-Nitrogen Cycle. It is understood that 4 2π

for stars whose interior temperatures are greater than that


of Sun, a Carbon-Nitrogen cycle takes place to produce the L
3
desired thermonuclear fusion reactions. It takes place as
indicated below :
12
2
6C + 1H1 → 7N13 + hν (energy)
13 → C13 + e0 (positron)
7N 6 1
13 1
6C + 1H1 → 7N14 + hν (energy)
14 + 1H1 → 8O15 + hν (energy)
7N
15 → N15 + e0 (positron)
8O 7 1 + energy 1 2 3 4 n
H 1+ N15 → C12 + He4
1 7 6 2
Fig. 12
The net result of this complete cycle of reactions is
that four protons fuse into one helium nucleus with the emis-
sion of positrons and release of energy.
4. 1 2 3 4 n
31. IMPORTANT POINTS TO REMEMBER AND
BASIC CONCEPTS
1. Relation between frequency and radius of orbit 4
E
2 2 2
mv ke ke
= 2 or mv2 =
r r r 8

ke2 ke2
or v2 = or (rw)2 = 12
mr mr
14 1
ke2 ke2 E∝ –
or w2 = or (2πf 2) = 16 n2
mr 3 mr 3
ke2 1
or 4λ2f 2 = or f 2 ∝
mr 3 r3 Fig. 13
696 COMPREHENSIVE OBJECTIVE PHYSICS

5.

Sr. No. Name of the Region Lower Upper Longest wavelength in terms Value of longest wavelength
spectral series state state of R λmax.
Series limit in terms Value of shortest wavelenght
of R λmin.

4 22
1. Lyman Ultra violet 1 2 = 1212.1 Å (121.2 nm)
3R 3R

1 12
∞ = 909.1 Å (90.9 nm)
R R

36 6 2
2. Balmer Visible 2 3 = 6545.4 Å (654.5 nm)
5R 5R

4 22
(Broadly speaking) ∞ = 3636.4 Å (363.6 nm)
R R
144 122
3. Paschen Infra red 3 4 = 18701.3 Å (1870.1 nm)
7R 7R
9 32
∞ = 8181.8 Å (818.2 nm)
R R

400 202
4. Brackett Infra red 4 5 = 40404.0 Å (4040.4 nm)
9R 9R
16 42
∞ = 14545.4 Å (1454.5 nm)
R R

900 302
5. Pfund For Infra red 5 6 = 74380.0 Å (7438.0 nm)
11R 11R

25 52
∞ = 22727.2 Å (2272.7 nm)
R R

Note : All values have been calculated by taking R = 1.1 × 107 m–1.

6. Quantum Numbers. Quantum numbers are the (iii) The total number of possible values is n.
numbers required to completely specify the position, (iv) It is also sometimes called azimuthal quantum
orbital angular momentum and configuration of elec- number.
trons in the presence of an external magnetic field (v) It determines the orbital angular momentum of
and electron-spin. the electron and also the shape of the orbit.
If spin-orbit interaction is not considered, then in the (vi) It gives the number of subshells associated with
presence of strong magnetic field, following four quan- the orbit i.e. the number of subshells associated
tum numbers are considered : with a particular value of n.
Principal quantum number (n) (vii) For n = 1, l has only one value i.e. l = 0.
(i) It is associated with radius, size of orbit and also For n = 2, l has two values l = 0, 1.
the energy levels of electron. For n = 3, l has three values l = 0, 1, 2.
(ii) Its value can be 1, 2, 3, …... ∞ (viii) The various subshells or values of l are also desig-
Orbit angular momentum quantum number (l) nated by letters s, p, d, f, ...... . For l = 4 and
(i) It is associated with energy levels, shapes etc. of higher values, the letters follow alphabetical or-
orbitals. der after f i.e. l = 4 is designated as g, l = 5 is
(ii) Its value can be 0, 1, 2, 3, ...... (n – 1). designated as h and so on.
ATOMIC NUCLEUS 697

(ix) The magnitude L of the orbital angular momen- The probability that an active nucleus decays by the

tum L of an electron in an atom is quantised i.e. first process in a time interval it is λ1dt.
The probability that an active nucleus decays by the
it can have only certain values. These values are
second process is λ2dt.
h The probability that the active nucleus either decays
L = l (l + 1) .
2π by the first process or by the second process is λ1dt + λ2dt.
Magnetic quantum number (me ) If λ is the effective decay constant, then
(i) It is associated with the orientation of electron λ dt = λ1 dt + λ2 dt
orbits in the presence of magnetic field. In the or λ = λ1 + λ2
presence of magnetic field, only those orientations log e 2 log e 2 log e 2 1 1 1
of the electron orbits are possible for which me = l ∴
= + or = +
t t1 t2 t t1 t2
cos θ is a whole number. 1 t2 + t1
or = or t = t1t2
(ii) It can have values from – l to l, including 0. t t1t2 t1 + t2
(iii) The total number of values is (2l + 1).
Magnetic spin angular momentum quantum number  FOOD FOR THOUGHT !
(ms or s)
Can you convert the above relation into
(i) It represents the clockwise or anticlockwise spin a relation for mean lives ?
of the electron in the presence of magnetic field. In
other words, it represents the direction of spin of 0.693 τ 1 × 0.693 τ2
0.693 τ =
the electron in the direction of magnetic field or 0.693 τ 1 + 0.693 τ2
opposite to it. τ τ
or τ= 1 2 .
(ii) It has only two possible values. τ 1 + τ2
If the spin-orbit interaction is considered, then in
addition to principal quantum number and orbital ILLUSTRATIONS
angular momentum quantum number, we also Illustration 1. A nucleus with Z = 92 emits the fol-
consider the following two quantum numbers. lowing in a sequence :
Total angular momentum quantum number (J) α, β–, β–, α, α, α, α, α, β–, β–, β+, α, β+, α.
(i) It is associated with the total angular momentum The Z of the resulting nucelus is.
(orbital and spin) of the electron. (a) 74 (b) 76
(ii) Its values are l ± s. (c) 78 (d) 82. [AIEEE 2003]
Total magnetic quantum number (mj) Sol. Decrease in Z due to emission of 8 α-particles = 16
(i) It is associated with total angular momentum of Increase in Z due to emission of 4 β-particles = 4
electron in the direction of magnetic field. Decrease in Z due to emission of 2 positrons = 2
(ii) Its values are from – J to + J. ∴ Z of resultant nucleus = 92 – 16 + 4 – 2 = 78
(iii) Its maximum possible values are (2J +1). So, (c) is the right choice.
7. For hydrogen-like atoms Illustration 2. Nucleus A is converted into C
through the following reactions
(rn )H
(i) vn = Z(vn)H (ii) rn = A→B+α [ α – alpha particle
Z –
B→C+2β β– – electron]
(iii) En = Z2(En)H.
Then,
8. Effective half-life of a radioactive nucleus de- (a) A and B are isotopes (b) A and C are isobars
caying by two different processes
(c) A and B are isobars (d) A and C are isotopes.
Let t1 be the half-life for the first process and t2 the [Karnataka CET 2003]
half-life for the second process. Let t be the effective
Sol. A and C have the same charge number.
half-life of the nucleus.
So, (d) is the right choice.
log e 2
Decay constant for first process, λ 1 =
t1 Illustration 3. A radioactive substance decays to
1/16th of its initial activity in 40 days. The half-life of the
log e 2
Decay constant for second process, λ 2 = radioactive substance expressed in days is
t2
698 COMPREHENSIVE OBJECTIVE PHYSICS

(a) 2.5 (b) 5 (a) 109 K (b) 107 K


(c) 10 (d) 20. [AIIMS 2003] (c) 10 K
5
(d) 103 K. [AIEEE 2003]
A0 Sol. To initiate the reaction
Sol. A=
2t / T 3
1 1 kT = 7.7 × 10–14
= or 2t/T = 24 2
16 2t /T
3
t t 40 or × 1.38 × 10–23 T
or = 4 or T = = = 10 days 2
T 4 4
= 7.7 × 10–14
So, (c) is the right choice.
Illustration 4. When a U238 nucleus originally at 7.7 × 10 –14 × 2
or T= K = 3.7 × 109 K
rest decays by emitting an alpha particle having a speed ‘u’, 3 × 1.38 × 10 –23
the recoil speed of the residual nucleus is So, (a) is the right choice.
4u 4u Illustration 7. The mass of proton is 1.0073 u and
(a) – (b)
238 238 that of neutron is 1.0087 u (u = atomic mass unit). The
4u 4u
(c) – (d) . [AIEEE 2003] binding energy of 4 He is
234 234 2

Sol. The momentum of the system remains the same (a) 0.0305 J (b) 0.0305 erg
before and after decay. This is in accordance with the princi-
(c) 28.4 MeV (d) 0.061 u.
ple of conservation of momentum.
Given helium nucleus mass = 4.0015 u
Now, 4u + 234v = 238 × 0
[All India PM/PD 2003]
4u Sol. Mass defect
or 234v = – 4u or v= –
234 = 2 mP + 2 mN – mHe
So, (c) is the right choice. = 2 × 1.0073 + 2 × 1.0087 – 4.0015
Illustration 5. A radioactive sample at any instant
= 0.0305
has its disintegration rate 5000 disintegrations per minute.
After 5 minutes, the rate is 1250 disintegrations per minute. Binding energy
Then, the decay constant (per minute) is = (931 × mass defect) MeV
(a) 0.8 ln 2 (b) 0.4 ln 2 = (931 × 0.0305) MeV
(c) 0.2 ln 2 (d) 0.1 ln 2. [AIEEE 2003] = 28.4 MeV (∵ 1 amu = 931 MeV)
Sol. N = N0 e–λt So, (c) is the right choice.
dN Illustration 8. A hypothetical radioactive nucleus
= – λN0 e–λt
dt decays according to the following series
– 5000 = – λN0 e–λ × 0 α β– α γ
A → A1  → A2 → A3 → A4
– 1250 = – 1 N0 e–λ × 5
If the mass number and atomic number of A are re-
1250 5000 spectively 180 and 72, then the atomic number and mass
or e–5λ = or e5λ = =4
5000 1250 number of A4 will be respectively
or 5λ = loge 4 = 2 loge 2 (a) 69, 171 (b) 70, 172
2 (c) 68, 172 (d) 69, 172.
or λ= loge 2 = 0.4 loge 2
5 [Karnataka CET 2003]
So, (b) is the right choice. Sol. Two α-particles decrease mass number by 8. So,
Illustration 6. In the nuclear fusion reaction mass number is (180 – 8) = 172. Charge number is decreased
2H by 4. So, charge number is (72 – 4) or 68. Emission of β-
1
+ 31 H → 42 He + n, given that the repulsive potential en-
particle increases charge number by 1. So, the charge number
ergy between the two nuclei is ~ 7.7 × 10–14 J, the tempera- of A4 is (68 + 1) i.e. 69. Note that the emission of γ-ray changes
ture at which the gases must be heated to initiate the reac- neither mass number nor charge number.
tion is nearly
So, (d) is the right choice.
[Boltzmann’s constant k = 1.38 × 10–23 J/K]
ATOMIC NUCLEUS 699

Illustration 9. What fraction of a radioactive (a) 1 photon with energy 10.2 eV and an electron
material will get disintegrated in a period of two half-lives ? with energy 1.4 eV
(a) Whole (b) Half (b) 2 photon with energy 10.2 eV
(c) One-fourth (d) Three-fourth. (c) 2 photon with energy 1.4 eV
[MP PET 2000] (d) one photon with energy 3.4 eV and 1 electron with
energy 1.4 eV. [IIT Screening 2005]
N0 N0
Sol. N= = Sol. 10.2 eV photon on collision will excite H-atom
22 4
to first excited state but hydrogen atom will return to
1 3
Fraction disintegrated = 1 – = ground state before next collision. Second photon will provide
4 4
So, (d) is the right choice. ionisation energy to hydrogen atom. So, electron will be
ejected with energy 1.4 eV.
Illustration 10. Fig. 14 shows some energy levels
of an atom. If the transition E3 to E2 gives emission of So, (a) is the right choice.
ultraviolet radiation, which of the following transitions Illustration 13. If a star converts all of its Helium
could give infra red radiation ? into oxygen nucleus, find the amount of energy released
per nucleus of oxygen. He = 4.0026 amu, O = 15.9994 amu
E4 (a) 7.26 MeV (b) 7 MeV
E3 (c) 10.24 MeV (d) 5.12 MeV.
[IIT Screening 2005]
E2 Sol. E = [4 × 4.0026 – 15.9994] × 931 MeV
= [16.0104 – 15.9994] × 931 MeV
E1
= 0.011 × 931 MeV = 10.24 MeV
So, (c) is the right choice.
Fig. 14
Illustration 14. A singly ionised helium ion and a
(a) E4 to E3 (b) E3 to E1
hydrogen ion are accelerated from rest through a potential
(c) E4 to E2 (d) E2 to E1 . difference V to velocities uHe and uH respectively. The ratio
[National Standard Exam. in Physics 1989] uHe/uH is equal to
Sol. E4 – E3 = – 0.85 – (– 1.5) (a) 1/4 (b) 1/2
= (1.5 – 0.85) eV = 0.65 eV
(c) 2 (d) 2.
This energy is compatible with the infra red region of
the spectrum. [National Standard Exam. in Physics 2005]
So, (a) is the right choice. Sol. The given ions have the same charge and accel-
Illustration 11. Nuclear fusion is possible erate through the same potential difference. Both gain the
(a) only between light nuclei. same kinetic energy. It is to be noted that helium is four
(b) only between heavy nuclei. times as heavy as hydrogen ion.
(c) between both light and heavy nuclei. So, (b) is the right choice.
(d) only between nuclei which are stable against β-decay. Illustration 15. The radioactive emissions which
produce an isotope of the original nucleus are
[AIIMS 2003]
(a) one alpha and four beta
Sol. It follows from the binding energy curve that
nuclear fusion is possible only between light nuclei. (b) one alpha and two beta
So, (a) is the correct choice. (c) one alpha and one beta
Illustration 12. A photon of energy 10.2 eV collides (d) two alpha and one beta.
inelastically with a Hydrogen atom in ground state. After [National Standard Exam. in Physics 2005]
a certain time interval of few micro seconds another photon Sol. Emission of one alpha reduces atomic number
of energy 15.0 eV collides inelastically with the same by 2 whereas that of one beta increases if by one.
hydrogen atom, then the observation made by a suitable So, (b) is the right choice.
detector is
700 COMPREHENSIVE OBJECTIVE PHYSICS

Illustration 16. The intensity of gamma radiation (a) 5 fermi (b) 4 fermi
from a given source is I. On passing through 36 mm of lead, (c) 8 fermi (d) 6 fermi. [AIEEE 2005]
I
it is reduced to . The thickness of lead which will reduce Sol. R = R0 A1/3
8
the intensity to
I
will be R2 LM OP
125
1/3
5

(a) 12 mm
2
(b) 18 mm
R1
=
27N Q =
3
5 5
(c) 9 mm (d) 6 mm. or R2 =R1 = × 3.6 fm = 6 fm
3 3
[AIEEE 2005]
So, (d) is the right choice.
Sol. I′ = I e–µx
Illustration 19. A nuclear transformation is denoted
I
or eµx = by X(n, α) 73 Li . Which of the following is the nucleus of
I′
I element X ?
or µx = loge . 11
I′ (a) 4 Be (b) 95 Be
I×8 10
Now, 36 µ = loge = 3 loge 2 (c) 5 B (d) 12
C6 . [AIEEE 2005]
I
I×2 Sol. X + 0n1 → 3Li7 + 2He4
x µ = loge
I ⇒ X is 5B10
or x log e 2 1 So, (c) is the right choice.
= =
36 3 log e 2 3
Illustration 20. The dia-
36 gram shows the energy levels for n=4
or x= mm = 12 mm n=3
3 an electron in a certain atom.
So, (a) is the right choice. Which transition shown repre- n=2
Illustration 17. Starting with a sample of pure 66 Cu, sents the emission of a photon
7 with the most energy ?
of it decays into Zn in 15 minutes. The corresponding (a) II (b) I n=1
8 I II III IV
half-life is (c) IV (d) III.
1 Fig. 15
[AIEEE 2005]
(a) 7 minute (b) 5 minute
2
(c) 15 minute (d) 10 minute. 13.6 z2
Sol. En = – eV
[AIEEE 2005] n2

N0 k
Sol. N= or En = – eV
2 t/ T1 / 2 n2

N 1 k k k
= E1 = – eV, E2 = – , E4 = –
N0 215 / T1/ 2 1 4 16

7 1 k FG IJ
k
1– =
8 215 / T1/ 2
Now, E2 – E1 = –
4
– −
1H K
1 1 k 3k
= =k– =
2 3
2 15 / T1 / 2 4 4

15 k FG IJ
k
∴ T1/ 2 =
3
= 5 minute E4 – E2 = –
16
− −
H K
4
So, (b) is the right choice. k k 3k
= − =
Illustration 18. If radius of the 27 4 16 16
13 Al nucleus is
125
Clearly, (E2 – E1) > (E4 – E2)
estimated to be 3.6 Fermi then the radius of 52 Te nucleus
So, (d) is the right choice.
be nearly
ATOMIC NUCLEUS 701

QUESTION BANK

MCQs
SET I

based on
Memory Work, Hard Facts, Important Terms,
Important Formulae etc.

Average time allowed per question is 20 seconds.

1. If in a nuclear fusion process, the masses of the fus- (a) both kinetic energy (KE) and potential energy (PE) increase
ing nuclei be m1 and m2 and the mass of the result- (b) both KE and PE decrease
ant nucleus be m3 , then (c) PE increases, KE decreases
(a) m3 = | m1 – m2 | (b) m3 < (m1 + m2) (d) PE decreases, KE increases.
(c) m3 > (m1 + m2) (d) m3 = m1 + m2. [National Standard Exam. in Physics 1990]
[All India PM/PD 2004] 5. An alpha particle colliding with one of the electrons
2. The dependence of binding energy per nucleon, BN , in a gold atom loses
on the mass number A is represented by (a) most of its momentum
(b) about one-third of its momentum
(c) little of its energy
(d) most of its energy.
BN BN [National Standard Exam. in Physics 1996]
6. Mp denotes the mass of a proton and Mn that of a
A = 56 A A = 124 A neutron. A given nucleus, of binding energy B, con-
(a) (b)
tains Z protons and N neutrons. The mass M(N, Z)
of the nucleus is given by (Given : c is the velocity of
light.)
(a) M(N, Z) = NMn + ZMp + Bc2

BN (b) M(N, Z) = NMn + ZMp – B/c2


(c) M(N, Z) = NMn + ZMp + B/c2
(d) M(N, Z) = NMn + ZMp – Bc2.
A = 96 A
(c) [All India PM/PD 2004]
7. The Bohr model of atoms
Fig. 16
[AIIMS 2004] (a) uses Einstein’s photo-electric equation
(b) predicts continuous emission spectra for atoms
A
3. A nucleus, represented by the symbol Z X, has (c) predicts the same emission spectra for all types of atoms
(a) Z protons and A – Z neutrons. (d) assumes that the angular momentum of electrons is
(b) Z protons and A neutrons. quantised. [All India PM/PD 2004]
(c) A protons and Z – A neutrons. 8. When an electron-positron pair annihilates, the energy
(d) Z neutrons and A – Z protons. released is about
[All India PM/PD 2004] (a) 0.8 × 10 –13 J (b) 1.6 × 10 –13 J
4. When a hydrogen atom is raised from the ground (c) 3.2 × 10 –13
J (d) 4.8 × 10 –13 J.
state to an excited state, [AIIMS 2004]
702 COMPREHENSIVE OBJECTIVE PHYSICS

9. Fig. 17 represents in simplified form some of the en- 16. Which of the following is a correct statement ?
ergy levels of the hydrogen atom. The energy axis (a) Radioactive materials which emit beta particles contain
has a linear scale. electrons inside their nuclei.

E4
(b) When an atom emits an electron in ionisation, its atomic
number increases by 1.
E3
(c) In X-ray emission, there is wavelengthwise distribution of
intensity with a definite upper limit of wavelength beyond
E2
Increasing which the intensity is zero.
energy
(d) In a cathode ray tube, if the accelerating voltage is in-
creased, the sensitivity (transverse deflection per unit volt
on the deflecting plates) decreases.
[National Standard Exam. in Physics 1991]
E1 238 absorbs a neutron. The product emits an elec-
17. 92 U
Fig. 17
tron. This product further emits an electron. The
result is
If the transition of an electron from E4 to E2 were (a) Pu239 (b) Pu239
associated with the emission of blue light, which tran- 94 90

(c) Pu237
(d) Pu237.
sition could be associated with the absorption of red 93 94

light ? [National Standard Exam. in Physics 1988]


(a) E4 to E1 (b) E3 to E2 18. The velocity of an electron in the nth orbit of H atom
(c) E2 to E3 (d) E1 to E4. is given by
10. Carbon dating is best suited for determining the age c
(a) (b) nc
of fossils if their age in years is of the order of n
(a) 103 (b) 104 1 c n
(c) (d) c.
(c) 105 (d) 106. [AIIMS 2004] 137 n 137
19. The ratio between kinetic energy and total energy of
11. In nuclear reactors, in order to slow down the fast
the electron of hydrogen atom, according to Bohr’s
neutrons, the target material should contain
model, is
(a) heavy nuclei (b) light nuclei
(a) 1 : – 1 (b) 1 : 1
(c) intermediate nuclei (d) any nuclei.
(b) 1 : 2 (d) 2 : 1.
[National Standard Exam. in Physics 1998]
20. The activity of a radioactive element decreases to one-
12. The binding energy of the hydrogen atom (the en- third of the original activity A0 in a period of nine
ergy binding the electron to the nucleus) is years. After a further lapse of 9 years, its activity will
(a) 1 eV (b) infinite be
(c) 13.6 eV (d) zero. [BHU 1993] 2
(a) A0 (b) A
13. According to Bohr’s theory, the radius of orbit de- 3 0
scribed by principal quantum number n and atomic A0 A0
number Z is proportional to (c) (d) .
9 6
(a) Z2n2 (b) Z2/n2
21. The volume occupied by an atom is greater than the
(c) Z2/n (d) n2/Z. volume of the nucleus by a factor of about
14. The radius of Bohr’s first orbit is a0 . The electron in (a) 101 (b) 105
the nth orbit has a radius (c) 1010 (d) 1015.
(a) na0 (b) a0/n [All India PM/PD 2003]
(c) n2a0 (d) a0/n2. [Roorkee 1993] 22. As a result of a number of successive decay processes
15. Solar energy is mainly caused due to in a radioactive series, the nucleon number decreases
(a) burning of hydrogen in the oxygen by 4 while the proton number is unchanged. The par-
ticles emitted are
(b) fission of uranium present in the Sun
(a) 1 alpha-and 1 beta-particle
(c) fusion of protons during synthesis of heavier elements
(b) 1 alpha- and 2 beta-particles
(d) gravitational contraction. [All India PM/PD 2003]
ATOMIC NUCLEUS 703
(c) 2 alpha-and 1 beta-particle (c) more than zero
(d) 2 alpha-and 2 beta-particles (d) sometimes more and sometimes less than zero.
(e) 4 alpha-particles. [Bharati Vidyapeeth 1999]
[London Schools Examination Department] 30. Thermonuclear reactions are responsible for energy
23. The ratio of the mass defect of the nucleus to its mass production
number is maximum among following nuclei in (a) at the centre of the earth (b) inside the stars
(c) in volcanoes (d) in the modern spaceships.
(a) N14 (b) Si28
(c) Fe56 (d) U238. [Karnataka 1992]
[National Standard Exam. in Physics 1997] 31. Neutron decay in the free space is given as follows
0n → 1H + –1e + [ ]
1 1 0
24. An atom emits a spectral line of wavelength λ when
an electron makes a transition between levels of en- Then the parenthesis represents an
ergy E1 and E2. Which expression correctly relates λ , (a) photon (b) graviton
E1 and E2 ? (c) neutrino (d) antineutrino.
h 32. In which of the following transitions will the wave-
(a) λ = (E1 – E2) (b) λ = ch (E1 – E2) length be minimum ?
c
c ch (a) n = 5 to n = 4 (b) n = 4 to n = 3
(c) λ = (d) λ = (c) n = 3 to n = 2 (d) n = 2 to n = 1.
h(E1 − E2 ) E1 − E2
25. In Bohr’s model of hydrogen atom let PE represent [BHU 2000]
potential energy, and TE the total energy. In going A
33. A nucleus X emits an α-particle. The resultant nu-
to a higher orbit, Z

(a) PE increases, TE decreases cleus emits a β + particle. The respective atomic and
(b) PE decreases, TE increases mass number of the final nucleus will be
(c) PE increases, TE increases (a) Z – 3, A – 4 (b) Z – 1, A – 4
(d) PE decreases, TE decreases. (c) Z – 2, A – 4 (d) Z, A – 2.

[National Standard Exam. in Physics 1992] [MP PET 2000]


26. The electron in the lowest (n = 1) orbit in hydrogen 34. In an experiment to study the scattering of alpha
atom has energy – 13.6 eV. How much energy is re- particles as they pass through a gold foil, the alpha
quired to ionise a hydrogen atom which is already in particles with smallest aiming error are those de-
the first excited level ? flected by
(a) 3.4 eV (b) 10.2 eV (a) smallest angles (b) nearly 180°
(c) 13.6 eV (d) 1.9 eV. [MP PET 2000] (c) nearly 90° (d) nearly 45°.
27. The energy of an atom (or ion) in its ground state is [National Standard Exam. in Physics 1996]
– 54.4 eV. It may be 35. The mass and energy equivalent to 1 amu respec-
(a) hydrogen (b) deuterium tively are
(c) He+ (d) Li++. [CMC LDH 1998] (a) 1.67 × 10–27 gm, 9.30 MeV
28. In beta decay, (b) 1.67 × 10–27 kg, 930 MeV
(a) the daughter nucleus has one proton more than the par- (c) 1.67 × 10–27 kg, 1 MeV
ent nucleus.
(d) 1.67 × 10–34 kg, 1 MeV. [MP PMT 2001]
(b) the parent and the daughter nuclei have the same number
of protons. 36. Pick out the correct statement.
(c) the daughter nucleus has one neutron more than the (a) As the voltage applied to the X-ray tube increases, the
parent nucleus. intensity of X-ray increases.
(d) the daughter nucleus has one proton less than the parent (b) All the β active nuclei emit electrons.
nucleus. [MAHE 1994] (c) Infra red radiation is used in the study of molecular struc-
29. The total energy of an atomic electron is ture.
(a) zero (b) less than zero (d) Moderator is used in a reactor to reduce the rate of reac-
tion.
704 COMPREHENSIVE OBJECTIVE PHYSICS

37. During a negative beta decay, 44. In a beryllium atom, if a0 be the radius of the first
(a) an electron which is already present within the nucleus is orbit, then the radius of the second orbit will be
ejected. a0
(a) 2a0 (b)
(b) a neutron in the nucleus decays emitting an electron. 2
a0
(c) a part of binding energy of the nucleus is converted into (c) 4a0 (d) .
an electron. 4
45. The difference between U 235 and U 238 atom is that
(d) an atomic electron is ejected. [IIT 1987]
(a) U238 contains 3 more protons.
38. The series limit wavelength of the Lyman series for (b) U238 contains 3 more protons and 3 more electrons.
hydrogen atom is given by (c) U238 contains 3 more neutrons and 3 more electrons.
(a) 16/R (b) 9/R (d) U238 contains 3 more neutrons.
(c) 4/R (d) 1/R. 46. The velocity of electron in an atom according to Bohr’s
39. In a mean life of a radioactive sample, theory is
(a) about 1/3 of substance disintegrates. 1 nh
(a) (b)
(b) about 2/3 of the substance disintegrates. 2πmr 2πmr
(c) 2πmr (d) none of these.
(c) about 90% of the substance disintegrates.
47. The average number of prompt neutrons produced
(d) almost all the substance disintegrates.
per fission of U235 is
[MP PMT 2000] (a) more than 5 (b) 3 to 5
40. For principal quantum number n = 3, the possible (c) 2 to 3 (d) 1 to 2. [MP PMT 2000]
values of orbital quantum number ‘l’ are
48. A nuclear reaction given by ZXA → Z+1Y
A + –1e
0 + ν
(a) 1, 2, 3 (b) 0, 1, 2, 3
represents
(c) 0, 1, 2 (d) –1, 0, +1.
(a) β-decay (b) γ-decay
[MP PMT 2001]
(c) fusion (d) fission.
41. One reaction which might be used for controlled nu- [All India PM/PD 2003]
clear fusion is shown
7 49. In a cancer therapy unit, patients are given treat-
Li + 21 H → 2 ( 42 He) + X
3 ment from a certain radioactive source. This source
What is particle X ? has a half-life of 4 years. A particular treatment re-
(a) an α-particle (b) an electron quires 10 minutes of irradiation when the source is
first used.
(c) a neutron (d) a proton.
How much time is required for this treatment, using
42. In order to trace the line of a water-pipe buried 0.4 m
the same source, 2 years later ?
below the surface of a field, an engineer wishes to
(a) 7 minutes (b) 10 minutes
add a radioactive isotope to the water.
(c) 14 minutes (d) 20 minutes.
Which sort of isotope should be chosen ?
50. What was the fissionable material used in bomb
emitter half-life dropped at Nagasaki (Japan) in the year 1945 ?
(a) β a few hours (a) Uranium (b) Neptunium
(b) β several years (c) Beryllium (d) Plutonium. [MNR 1985]
(c) γ a few hours
51. The half-life period and the mean life period of a
(d) γ several year.
radioactive element are denoted respectively by T h
43. When it disintegrates, a certain radioactive nuclide and Tm , then
P emits γ-radiation and a single α-particle, forming a (a) Th = Tm (b) Th > Tm
daughter product Q. Which one of the following state- (c) Th < Tm (d) Th ≥ Tm.
ments is true ?
52. The ionisation energy of a hydrogen-like atom is 9
(a) P and Q are isotopes of the same element.
Rydberg. The radius of first orbit of this atom, as
(b) The mass number of P is one more than that of Q. compared to the radius of first orbit of hydrogen
(c) The mass number of P is one less than that of Q. atom, will be
(d) P has more protons in its nucleus than Q. (a) one-fourth (b) half
(e) The atomic number of P is less than that of Q. (c) one-third (d) three times. [BHU 1999]
ATOMIC NUCLEUS 705

53. The spectral series of hydrogen which lies entirely in 61. The ratio of wavelength of first line of Lyman series
the ultraviolet part is of doubly ionised lithium atom to that of the first
(a) Lyman (b) Balmer line of Lyman series of deuterium (1H2) will be
(c) Paschen (d) Pfund. [MP PET 2000] (a) 4 : 1 (b) 1 : 4

54. Pick out the statement which is true. (c) 9 : 1 (d) 1 : 9. [Manipal 1999]
(a) The energy released per unit mass is more in fission than 62. There are seven orbitals in a subshell. Then the value
in fusion. of l for it will be
(b) The energy released per atom is more in fusion than in (a) 4 (b) 3
fission. (c) 2 (d) 1. [Pb. PMT 2000]
(c) The energy released per unit mass is more in fusion and 63. Fig. 18 shows the principle of a simple form of mass
that per atom is more in fission. spectrometer. Ions are passed through narrow slits,
(d) Both fission and fusion produce same amount of energy S1 and S2 , and into a velocity selector. The selected
per atom as well as per unit mass. ions, after passage through the slit S3 , are deviated
[Karnataka CET 2001] by the uniform magnetic field.
55. In nuclear fission, the percentage of mass converted
S1 S2 S3
into energy is about Path of Region of
ions magnetic
(a) 10% (b) 0.01%
field
(c) 0.1% (d) 1%.
[Karnataka CET 2002] Velocity
selector
56. If E1 is the energy released per unit mass in nuclear
fusion and E2 that in nuclear fission, then
(a) E1 = E2 (b) E1 = – E2 P

(c) E1 > E2 (d) E1 < E2.


[Karnataka CET 2002] Fig. 18

57. The radioactive decay of the certain nuclide is gov- Which of the following quantities must be the same
erned by the following relationship for all ions arriving at point P ?
charge
dn (a) charge (b)
= – λn where λ = 2.4 × 10–8 s–1 mass
dt (c) kinetic energy (d) mass
What is the half-life of the nuclide ? (e) momentum.
(a) 2.9 × 107 s (b) 1.3 × 107 s
64. 238 U decays through a series of transformations to
(c) 1.2 × 10–8 s (d) 3.4 × 10–8 s 92

(e) 8.0 × 10–8 s. a final stable nuclide. The particles emitted in the
58. Which of the following is not a mode of radioactive successive transformations are
decay ? α β β α α
(a) positron emission (b) electron capture Which nuclide is not produced during this series of
(c) fusion (d) alpha decay. transformations ?
228 234
59. The energy required to knock out the electron in the (a) 88 Ra (b) 91Pa
third orbit of an atom is equal to 230 234
(c) 90 Th (d) 92 U .
13.6
(a) 13.6 eV (b) eV 65. The order of the size of nucleus and Bohr radius of
9
13.6 3 an atom respectively is
(c) – eV (d) – eV. (a) 10–14 m, 10–10 m (b) 10–10 m, 10–8 m
3 13.6
60. Which of the following is true ? (c) 10 –20
m, 10 –16
m (d) 10–8 m, 10–6 m.
(a) Lyman series is a continuous spectrum. [MP PMT 2001]
(b) Paschen series is a line spectrum in the infra red. 66. Two electrons in an atom are moving in orbits of
(c) Balmer series is a line spectrum in the ultraviolet. radii R and 9R respectively. The ratio of their fre-
(d) the spectral series formula can be derived from the
quencies will be
Rutherford model of the hydrogen atom. (a) 1 : 8 (b) 8 : 1
(c) 1 : 27 (d) 27 : 1. [BHU 1998]
706 COMPREHENSIVE OBJECTIVE PHYSICS

67. The value of wavelength of radiation emitted due to What fraction remains undecayed after 24 days ?
transition of electrons from n = 4 to n = 2 state in 1
(a) 0 (b)
hydrogen atom will be 128
5R 16 1 1
(a) (b) (c) (d)
36 3R 64 32
36 3R 1
(c) (d) . (e) . [London Schools Examination
5R 16 16
Department]
[All India PM/PD 2001]
75. The activity from a radioactive source is found to
68. Half-life of radioactive element depends upon fall by 0.875 of its initial activity in 210 s. What is the
(a) amount of element present half-life of the source ?
(b) temperature (c) pressure (a) 30 s (b) 60 s
(d) nature of element. [AMU 1993] (c) 70 s (d) 105 s
69. Fusion reaction takes place at about (e) 120 s.
(a) 3 × 10 2 K (b) 3 × 10 3 K 76. In which of the following systems will the wavelength
(c) 3 × 10 4 K (d) 3 × 10 6 K. corresponding to n = 2 to n = 1 be minimum ?
[CMC Vellore 1998] (a) hydrogen atom (b) deuterium atom

70. A radioactive isotope has a decay constant λ and a (c) singly ionised helium (d) doubly ionised lithium.
molar mass M. Taking the Avogadro constant to be [Himachal PMT 1997]
L, what is the activity of a sample of mass m of this 77. The passage of γ-ray photons through materials some-
isotope ? times results in pair production, i.e., the transfor-
λmL mation of a γ-ray photon into a positron and an elec-
(a) λmML (b)
M tron (each of mass m). What is the maximum wave-
λML mL
length of γ-ray photons for which pair production is
(c) (d) possible ?
m λM
1 1
mλ (a) (b)
(e)
ML
. 2mch 2mc2 h

71. The minimum wavelength in Lyman series is h h


(c) (d)
1 2 mc 2mc2
(a) (b) R
R h
(e) .
1 2mc3
(c) (d) Rc . (c = the speed of light ; h = Planck constant)
Rc
78. Fusion reaction takes place at high temperature
72. The energy of an electron in an excited hydrogen atom
because
is –3. 4 eV. Its angular momentum according to Bohr’s
(a) atoms are ionised at high temperature
theory will be
(b) molecules break-up at high temperature
(a) 2.11 × 10–34 J s (b) 2.11 × 10+34 J s
(c) nuclei break-up at high temperature
(c) 2.11 × 10–34 erg s (d) zero.
(d) kinetic energy is high enough to overcome repulsion be-
[Bharati Vidyapeeth 1998] tween nuclei.
73. A possible fuel for nuclear reactors is 79. In a working nuclear reactor, cadmium rods are used
(a) Lead (b) Graphite to
(c) Cobalt (d) Plutonium. (a) speed up neutrons (b) slow down neutrons
74. The half-life of a certain radioactive element is such (c) absorb some neutrons (d) absorb all neutrons.
7 80. Radioactive 14C dating was used to find the age of a
that of a given quantity decays in 12 days. wooden archaeological specimen. Measurements were
8
taken in three situations for which the following count
rates were obtained.
ATOMIC NUCLEUS 707
specimen count rate 81. A triton contains
1 g sample of living wood 80 counts per minute (a) 1 proton + 1 neutron (b) 1 proton + 2 neutrons
1 g sample of archaeological 35 counts per minute (c) 1 proton + 3 neutrons (d) 1 proton + 4 neutrons.

specimen 82. The maximum wavelength of Lyman series is


no sample 20 counts per minute 1 1
(a) (b)
If the half-life or 14C is known to be 5700 years, what R2 RC
was the approximate age of the archaeological speci- c 4
men ? (c) (d) .
R 3R
(a) 2500 years (b) 7000 years
(c) 11000 years (d) 13000 years
(e) 23000 years.

Answers (Set I)
1. (b) 2. (a) 3. (a) 4. (c) 5. (c) 6. (b) 7. (d) 8. (b)
9. (c) 10. (b) 11. (b) 12. (c) 13. (d) 14. (c) 15. (c) 16. (d)
17. (a) 18. (c) 19. (a) 20. (c) 21. (d) 22. (b) 23. (c) 24. (d)
25. (c) 26. (a) 27. (c) 28. (a) 29. (b) 30. (b) 31. (d) 32. (d)
33. (a) 34. (b) 35. (b) 36. (c) 37. (b) 38. (d) 39. (b) 40. (c)
41. (c) 42. (c) 43. (d) 44. (c) 45. (d) 46. (b) 47. (c) 48. (a)
49. (c) 50. (d) 51. (c) 52. (c) 53. (a) 54. (c) 55. (c) 56. (c)
57. (a) 58. (c) 59. (b) 60. (b) 61. (d) 62. (b) 63. (b) 64. (a)
65. (a) 66. (d) 67. (b) 68. (d) 69. (d) 70. (b) 71. (a) 72. (a)
73. (d) 74. (c) 75. (c) 76. (d) 77. (c) 78. (d) 79. (c) 80. (c)
81. (b) 82. (d).

Solutions (Set I)
1. When two light nuclei are combined to form a heavy 8. When an electron and positron combine, they annihi-
nucleus, there occurs a small mass defect. In other late each other and energy is released.
words, the mass of the heavy nucleus turns out to be E = mc2 = 2 × 9.1 × 10–31 × (3 × 108)2 J ≈ 1.6 × 10–13 J.
less than the sum of the masses of two light nuclei. 9. Frequency ν of electromagnetic radiation is related to
This small mass defect results in the release of a huge the energy change ∆E of an atom by the equation
amount of energy according to the relation ∆E = ∆m c2. hc
2. The binding energy per nucleon is maximum (≈ 8.8 ∆E = hν =
λ
MeV) for the nucleus having mass number 56.
h is the Planck constant.
So, this nucleus is most stable i.e., iron is the most
Here λ is the wavelength of the electromagnetic ra-
stable element of periodic table.
diation.
4. As r increases, potential energy increases (this part
c is the speed of light.
is widely understood). For kinetic energy remember
Blue light has a higher frequency than the red light
the extreme : at very high orbits kinetic energy tends
and hence if absorption of red light is to occur, the
to zero.
corresponding change in energy level should be smaller.
5. Due to large mass ratio, very little energy transfer
Thus the possible transitions associated with the ab-
(result of conservation of momentum and energy,
sorption of red light are either E2 to E3 or E3 to E4.
both.)
708 COMPREHENSIVE OBJECTIVE PHYSICS

(Absorption of electromagnetic radiation is always as-


FG IJ
9
A 1 T1/ 2
sociated with a rise from lower energy level to a higher
one.)
A′ = 0
3 2 H K
10. Cosmic ray neutrons in the atmosphere reacts with Dividing,
A′ × 3 1 A
the nuclei of nitrogen ( 14
7 N ) to produce radioactive = or A′ = 0 .
A0 3 9
isotope of carbon ( 14
6 C ). 22. Note that the nucleon number is the number of neu-
1 14 14 1
trons and protons. Also, the nucleon number is un-
0n + 7N → 6 C + 1H changed by beta-decay.
14
combines with oxygen to form CO2. Green plants 23. Binding energy per nucleon is maximum in Iron.
6C
make their food by taking CO2 in the presence of sun- 24. By quantum theory of radiation, the energy change
light (Photosynthesis). Animals eat plants and get this ∆E between energy levels is proportional to the fre-
14
quency of electromagnetic radiation f and is given by
carbon. 6C already absorbed by the plants and ani-
hc
mals starts decaying to 12 with half-life nearly 5700 ∆E = hf =
6C λ
years, when plants and animals die. The ratio 14C :
12C hc hc
in the specimens of wood or bone give us an esti- Hence λ= =
mate of their age. This method is most suitable to ∆E E1 − E2
determine age upto 5 × 104 years. 25. Potential energy = – C/r2 and total energy = – Rhc/n2.
With higher orbit both r and n increase. So both be-
 Geologists use Uranium dating whereas An- come less negative ; hence both increase.
thropologists use carbon dating. 13.6
26. Energy in first excited level = – eV = – 3.4 eV
22
11. For maximum loss of energy, the colliding particle
Clearly, the ionisation energy is 3.4 eV.
and target nuclei should have nearly equal mass.
27. Energy of an electron in the nth orbit of hydrogen like

13 & 14. rn =
LM
1 1 n2 h2 OP atom is En = Z2 (En)H
MN
Z K 4 π 2me2 PQ Z2 =
− 54.4
=4 or z = 2
2 − 13.6
n
or rn ∝
Z Clearly, ion is He+.
16. (a) False. Beats are generated by n to p conversion in 28. n → p + e + ν
the nucleus.
Clearly, the daughter nucleus has one more proton.
(b) False. Z is related to nuclear charge, not net charge
of atom/ion. 29. Electron is bound to the nucleus. So, total energy is
negative.
(c) Upper limit is for frequency, not wavelength. False.
(d) Faster electrons, less time through the plates, so 31. The emission of antineutrino is a must for the valid-
sensitivity falls. True. ity of different laws.
17. 92U238 + 0n1 → 92U239 32. Widest gap, maximum frequency, minimum wave-
→ –1e0 + 93Np239 → e– + 94Pu239 length.

ke2 ke2 33. AX


Z
→ 42 He + AZ −− 24 Y , A−4
Z−2
Y → e+ + A−4
Z−3
Y′
19. Ek = and E = −
2r 2r
Ek 1 + During β+ emission.
Now, = 1
E −1 → 0n1 + β+
1p
The proton changes into neutron. So, charge
A = A FG IJ
t
1 T1/2 number decreases by 1 but mass number re-
20.
H 2K0
mains unchanged.

F 1I
9
34. Smallest aiming error, largest interaction.
=A G J
A T1/ 2
H 2K
0
0 Hence, largest deflection (180°).
3
ATOMIC NUCLEUS 709

36. Wavelength and not the Intensity depends on volt- 45. U235 and U238 are isotopes of uranium. The difference
age. β decay includes positron emission. Moderator in it is only due to the number of neutrons; the number
does not slow down the rate of chain reaction. of protons are the same.
37. Since an electron cannot live inside the nucleus there- nh nh
fore it comes out of the nucleus. 46. mvr = or v =
2π 2πmr
1 1 LM 1 OP 1 47. During a fission event, in general, two or three neu-
38.
λ
=R 2 −
1 N ∞ Q or λ =
R
.
trons are emitted. If the total number of neutrons
emitted in a large number of events is divided by the
39. N = N0 e–λt
1
number of events, the average comes out to be around
−λ
λ 1 2.47. We say that on an average 2.47 neutrons are
N = N0 e = N
e 0 emitted in each fission event. As a round figure, we
N 1 may say 2.5 neutrons are emitted.
= = 0.3679 49. If N0 is the initial particles of the radioactive source,
N0 2.718
then
Fraction of the element that disappears
dN FG IJ
2
= (1 – 0.3679) = 0.63 ≈ .
initial activity
dt 0 H K
= − λN 0
3 where N = N0 e–λt ; t in years.
40. The orbital quantum number l has possible values 0, ln 2
1, 2, ......,(n – 1). The total number of possible values λ= ; half-life = 4 years.
4
is n.
Number of radioactive particles needed for the
If n = 3, then the possible values are 0, 1, 2. treatment is
41. By conservation of mass number and atomic number, FG dN IJ 10
the nuclear reaction equation may be re-written as N=
H dt K 0
×
4 × 365 × 24 × 60
7 Li
3
+ 21 H → 2 e j
4
2 He + 01 n 10λN0
= ...(1)
Thus particle X is a neutron. 4 × 365 × 24 × 60
42. A γ radioactive isotope of short half-life should be cho- Two years later, the activity remained is
sen. Lead water pipes can prevent the penetration of
FG dN IJ
γ rays and its half-life of a few hours can have no harm-
ful effects on humans and yet can be detected through
H dt K 2
= – λN2 = – λN0e –2λ.

the leakage of pipes. If t minutes is needed for the same treatment, then
43. The reaction can be represented by FG dN IJ t
AP
Z
→
A –4
Z–2
Q + 4
2
He + γ-radiation
N=
H dt K 2
×
4 × 365 × 24 × 60

We thus see that P has more protons and neutrons in t λ N 0 e −2 λ


= ...(2)
its nucleus than Q. In fact, the mass number of P is 4 4 × 365 × 24 × 60
more than that of Q and the atomic number of P is 2 1
ln 2
more than that of Q. Since the atomic numbers of P i.e., t = 10e2λ = 10e 2 = 10 2 = 14.1 minutes.
and Q are different, they are not isotopes of the same
50. The highly destructive nuclear bomb (or atom bomb)
element.
is based on the nuclear fission reactions of uranium

44. rn =
1
(rn )H or rn =
LM
1 1 n 2 h2 OP – 235 or plutonium – 239. In the nuclear bomb, the
Z MN
Z K 4 π 2me2 PQ fission reaction of uranium – 235 (or plutonium – 239)
is deliberately allowed to go out of control. This
1 4πε 0 n2 h2 ε 0 n2 h2 produces an enormous amount of energy in a very
or rn = or rn =
Z 4π 2 me2 Zπme2 short time. The atom bomb based on the fission of
2 uranium – 235 was dropped on city of Hiroshima in
or rn ∝ n or r2 n22 Z Japan on 6th August, 1945 during the second world
= × 12
Ζ r1 Z2 n1 war. Another atom bomb based on the fission of
r2 plutonium – 239 was dropped on the city of Nagasaki
or = 4 or r2 = 4ao. in Japan three days later on 9th August 1945.
r1
710 COMPREHENSIVE OBJECTIVE PHYSICS

α β β α α
Th Th 64. 238
U → 234 Q → 234 R → 234 S → 230 T → 226 Pb
51. = 0.693 or <1 or Th < Tm 92 90 91 92 90 88
Tm Tm
228 Ra
is not produced during this series of transfor-
52. EZ = EH Z2 88
mations.
9 = 1 × Z2 or Z = 3
1 1
f ∝ 3/ 2 or f ′ ∝
Now, rZ =
rH
Z
or rZ =
rH
3
66.
R 9R
3/ 2
b g
Dividing, f f
53. Lyman series lies in the ultraviolet region of the spec- = 93 / 2 or = 27
f′ f′
trum. It is not visible with naked eye. But it can be
1 LM1 1 OP 1 1 LM
1 OP
photographed. This series was first photographed by
T. Lyman of Harvard University in 1916.
67.
λ
=R
MN −
n12 n22 PQ or
λ
=R −
4 16 N Q
57. The half-life of the nuclide is related to the decay con- 1 L 4 − 1OP
=RM λ=
16
stant λ by the equation λ N 16 Q or
3R
ln 2 0.693 0.693 70. The given sample of radioactive isotope of mass m
T1/2 = = = = 2.9 × 107 s . mL
λ λ 2.4 × 10 −8 contains number of undecayed radioactive parti-
M
60. Paschen series lies in the infra red region of the spec- cles. From the law of radioactive decay, its activity A
trum and is invisible. is given by

LM OP FG mL IJ = − λmL
61. 1
λ
= RZ2
1
n12
1
− 2
MNn2 PQ
A=−λ
HMK M
LM OP 1 L 1 1O
= R M − P or 1
1
λ Li
1
1 2
1
= R × 32 2 − 2
N Q
71.
λ min. N1 ∞Q
2 λ min. =
R

1
= R × 12
LM 1 − 1 OP 72. L=n
h

λ D1 N1 2 Q
2 2

2 × 6.62 × 10 −34
λ Li 1 L= Js or L = 2.11 × 10–34 J s
Dividing, = 2 × 3.14
λ D1 9
1
74. The radioactive element has of a given quantity
62. m = 2l + 1 8
that remains after 12 days. After 24 days, or in addi-
7 = 2l + 1 or 2l = 6 or l = 3. tional 12 days, the fraction that remains undecayed
63. For a given magnetic field B and electric field E, the is
velocity of the ions pass through the slit S3 is of
preselected value given by FG 1IJ FG 1IJ 1
E H 8K H 8K or
64
v=
B 75. The activity from the redioactive source has fallen to
Trasversing at this speed, the ions arrive at point P (1 – 0.875) or 0.125 of its initial activity. This corre-
provided that FG 1IJ 3

mv2
sponds to
H 2K of its initial activity or the passage of
= qvB 3 half-lives. Hence, the half-life of the source is
R
1
where R is the radius associated with the circular (210 s) or 70 s
3
path taken by the ions
v q 1
⇒ 76. λ∝ for hydrogen-like atoms
= Z2
RB m
Z maximum, λ minimum.
Hence, for a given magnetic field B and the selected
velocity v, all ions will arrive at point P (same R) if 77. The energy of the positron and the electron E is re-
the quantity q/m or charge/mass is the same. lated to the frequency f of the γ-ray photon by the
equation
ATOMIC NUCLEUS 711

c 30 counts/min and another 5700 years to decay further


E = hf = h 1
λ
to (30) or 15 counts/min. Hence, the age of the
where λ is the wavelength of γ-ray proton. 2
By the Einstein’s mass-energy relation, we have archaeological specimen is 2(5700) or approximately
11000 years to the nearest thousands.
E = mc2 + mc2 = 2mc2

⇒ 2mc2 = h
c
82.
1
=R
1 LM 1
− 2
OP
λ λ n12
MN
n2 PQ
∴ λ=
hc
=
h
1 L 1 1 OP
=RM − 1 LM 1 OP
2 mc2 2 mc
80. Eliminate the background radioactivity, the actual
λ max. N1 2 Q 2 2 or
λ max.
= R 1−
N 4 Q
count rates for living wood is 60 counts/min while 4
or λ max. = .
that of the specimen is 15 counts/min. 3R
Since the half-life of 14C is 5700 years, it takes 5700
1
years to decay from 60 counts/min to (60) or
2

KNOWLEDGE PLUS
l The nuclei of which one of the following pairs of nuclei are isotones ?
(a) 34Se74, 31Ga71 (b) 20Ca40, 16S32 (c) 42Mo92, 40Zr92 (d) 38Sr84, 38
Sr86 [All India PM/PD 2005]
74 71
Sol. Same number of neutrons in 34 Se and 31Ga .
So, (a) is the right choice.

mass of fission products


l In any fission process the ratio is :
mass of parents nucleus
(a) Less than 1 (b) equal to 1
(c) depends on the mass of the parent nucleus (d) greater than 1 [All India PM/PD 2005]
Sol. Mass of fission products < mass of parent nucleus
So, (a) is the right choice.

l Fission of nuclei is possible because the binding energy of per nucleon in them :
(a) increases with mass number at low mass numbers.
(b) increases with mass number at high mass numbers.
(c) decreases with mass number at low mass numbers.
(d) decreases with mass number at high mass numbers. [All India PM/PD 2005]
Sol. For nuclei having A > 56, binding energy per nucleon gradually decreases.
So, (d) is the right choice.
712 COMPREHENSIVE OBJECTIVE PHYSICS

MCQs
based on
SET II
Important Basic Concepts
+
Application of Formulae
+
Brain Teasers

Average time allowed per question is 50 seconds.

83. If the electron in an hydrogen atom jumps from an 88. An electron jumps from the 4th orbit to the 2nd orbit
orbit with level ni = 3 to an orbit with level nf = 2, the of hydrogen atom. Given : the Rydberg’s constant
emitted radiation has a wavelength given by R = 105 cm –1. The frequency in Hz of the emitted
R 36 radiation will be
(a) λ = (b) λ =
6 5R 3 3
(a) × 105 (b) × 1015
6 5R 16 6
(c) λ = (d) λ = . [AIIMS 1997]
R 36 9 3
84. The energy change is greatest for a hydrogen atom (c) × 1015 (d) × 10 16 .
16 4
when its state changes from
[CMC Vellore 1997]
(a) n = 2 to n = 1 (b) n = 3 to n = 2
14 N is 14.003074 u
(c) n = 4 to n = 3 (d) n = 5 to n = 4. 89. Given that the atomic mass of 7
85. Two radioactive isotopes P and Q have half-lives of and that the sum of the atomic masses of 1H
and
1
10 minutes and 15 minutes, respectively. Freshly
13 C
prepared samples of each isotope initially contain the 6 is 14.011179u, it would be reasonable to suppose
same number of atoms as one another. After 30 that the nuclear reaction
the number of atoms of P 1
minutes, the ratio the number of atoms of Q will be 1
H + 13
7
C → 14
7
N

(a) 0.5 (b) 2.0 (a) can only happen if there is a net supply of energy.
(c) 1.0 (d) 3.0 (b) could not take place at all.
(e) 0.25. [Associated Examination Board, (c) could occur only in conditions of zero gravity, i.e. of ‘weight-
Hampshire] lessness’.
(d) must involve the emission of a further unchanged atomic
86. A certain radioactive element has half-life of 4 days.
particle.
The fraction of material that decays in two days is,
(e) will result in the emission of energy.
(a) 1/2 (b) 1/ 2 [Cambridge Local Exam. Syndicate]
(c) 2 (d) e j
2 −1 / 2. 90. The band spectra (characteristic of molecular spe-
cies) is due to emission of radiation in
[National Standard Exam. in Physics 1998]
(a) gaseous state (b) liquid state
87. In the disintegration series
(c) solid state (d) all of three states.
238 α β−
U → ZA Y ,
→ X  
92 [AFMC 1997]
The values of Z and A respectively will be 91. In Fig. 19, E 1 to E6 represent some of the energy
(a) 92, 236 (b) 88, 230 levels of an electron in the hydrogen atom
(c) 90, 234 (d) 91, 234. [MP PMT 2001]
ATOMIC NUCLEUS 713
E6 – 0.38 ev 97. The frequency of emission line for any transition in
– 0.54 ev
E5 positronium is x times the frequency for the corre-
– 0.85 ev
E4 sponding line in the case of H-atom, where x is
E3 – 1.5 ev (a) 2 (b) 1/ 2
E2 – 3.4 ev (c) 1/2 2 (d) 1/2 .
[National Standard Exam. in Physics 1993]
98. The orbiting speed vn of e– (the same as for e+) in the
E1 – 13.6 ev
nth orbit in the case of positronium is x-fold com-
Fig. 19
pared to that in nth orbit in hydrogen atom, where x
has the value
Which one of the following transitions produces a
(a) 1 (b) 2
photon of wavelength in the ultra-violet region of the
electromagnetic spectrum ? (c) 1/ 2 (d) 2.
[1 eV = 1.6 × 10–19 J.] [National Standard Exam. in Physics 1993]
(a) E2 — E1 (b) E3 — E2 99. If the wavelength of photon emitted due to transi-
(c) E4 — E3 (d) E5 — E4 tion of electron from third orbit to first orbit in a
(e) E6 — E5. hydrogen atom is λ, then the wavelength of photon
emitted due to transition of electron from fourth or-
92. The second line of Balmer series has wavelength
bit to second orbit will be
4861 Å. The wavelength of the first line of Balmer
series is 128 25
(a) λ (b) λ
(a) 1216 Å (b) 6563 Å 27 9
36 125
(c) 4340 Å (d) 4101 Å. (c) λ (d) λ.
7 11
93. The electron in a hydrogen atom jumps from ground
[Bharati Vidyapeeth 2002]
state to the higher energy state where its velocity is
reduced to one-third its initial value. If the radius of 100. The first spectral line of sodium atom is 5890 Å. The
the orbit in the ground state is r, the radius of new first excitation potential of sodium is
orbit will be (a) 3.7 V (b) 2.1 V
(a) 3r (b) 9r (c) 10.2 V (d) 4.1 V.
r r 101. The initial activity of a certain radioactive isotope
(c) (d) . [JIPMER 2003] was measured as 16000 counts per minute. Given
3 9
94. In the case of thorium (A = 232 and Z = 90), we ob- that the only activity measured was due to this iso-
tain an isotope of lead (A = 208 and Z = 82) after tope and that its activity after 12 hours was 2100
some radioactive disintegrations. The number of α counts per minute, its half-life, in hours, is nearest
and β particles emitted are respectively to
(a) 6, 3 (b) 6, 4 (a) 9.0 (b) 6.0
(c) 5, 5 (d) 4, 6. (c) 4.0 (d) 3.0

95. Which of the following parameters are the same for (e) 2.0. [Oxford Local Examinations]
all hydrogen-like atoms and ions in their ground 102. Transitions between three energy levels in a
states ? particular atom give rise to three spectral lines of
(a) radius of the orbit (b) speed of the electron wavelengths, in increasing magnitudes, λ 1, λ2 and
(c) energy of the atom λ 3. Which one of the following equations correctly
relates λ1, λ2 and λ3 ?
(d) orbital angular momentum of the electron.
(a) λ1 = λ2 – λ3 (b) λ1 = λ3 – λ2
[Pb. PMT 2002]
96. In the ground state of positronium, the e – to e + 1 1 1 1 1 1
(c) = + (d) = −
distance is x times the Bohr radius, where x has the λ1 λ2 λ 3 λ1 λ 3 λ 2
value 1 1 1
(a) 1 (b) 2 (e) = − .
λ1 λ 2 λ3
(c) 22 (d) 2 2 . 103. Which of the following is true when Bohr gave his
[National Standard Exam. in Physics 1993] model for hydrogen atom ?
714 COMPREHENSIVE OBJECTIVE PHYSICS

(a) It was not known that hydrogen lines could be explained E4


as differences of terms like R/n2 with R constant, n an E3
integer.
(b) It was not known that positive charge is concentrated in a E2
Energy
nucleus of small size.
(c) It was not known that radiant energy occurred in energy
bundles defined by hv with h a constant and v a frequency.
(d) Bohr knew terms like R/n2 and in the process of choosing
the allowed orbits to fit them he got ‘‘angular momentum E1
= n (h/2π)’’ as a deduction.
Fig. 20
[National Standard Exam. in Physics 1995]
(a) E4 to E1 (b) E3 to E1
104. The fission of a heavy nucleus gives, in general, two
smaller nuclei, two or three neutrons, some β-parti- (c) E3 To E2 (d) E1 to E3
cles and some γ-radiation. It is always true that the (e) E2 to E3.
nuclei produced 109. In radioactive de-
4
(a) have a total rest-mass that is greater than that of the cay, if ∆N is the
original nucleus. number of nuclei
(b) have large kinetic energies that carry off the greater part decaying in interval 3
of the energy released. ∆t, and N is the r
number of mean ac-
(c) travel in exactly opposite directions. 2
tive nuclei during
(d) have neutron-to-proton ratios that are too low for stability. that interval, the 1
t

(e) have identical neutron-to-proton ratios. graph between the


quantity r = (1/N) Fig. 21
105. If the series limit wavelength of the Lyman series for
hydrogen atom is 912 Å, then the series limit wave- ∆ N/∆t versus time t will have the shape
length for the Balmer series for the hydrogen atom (a) 1 (b) 2
is (c) 3 (d) 4.
(a) 912 Å/2 (b) 912 Å [National Standard Exam. in Physics 1995]
(c) 912 × 2 Å (d) 912 × 4 Å. [BHU 1997] 110. In a positive ray analysis, singly ionised particles and
106. A radioactive source contains two materials. One doubly ionised particles form identical parabola when
decays by the emission of an α-particle with a half- the magnetic fields are 5 T and 7.5 T respectively.
life of 4 days while the other emits β-particles with a The ratio of their masses is
half-life of 3 days. The initial corrected count rate is (a) 2 : 9 (b) 9 : 2
176 s–1 but this becomes 80 s–1 when a piece of tissue
(c) 2 : 3 (d) 3 : 2.
paper is placed between the source and the detector.
What will be the corrected count rate after 12 days 111. In a positive ray analysis experiment two similar pa-
without the tissue paper present ? rabolas are obtained for two different gases for the
(a) 5 s–1 (b) 11 s–1 electric fields of 300 V/m and 2000 V/m. If one is
singly ionised and the other is doubly ionised, then
(c) 12 s–1 (d) 17 s–1
the ratio of their masses is
(e) 22 s–1. [Scottish Examination Board]
(a) 3 : 1 (b) 2 : 3
107. What amount of original radioactive atom is left af-
(c) 1 : 3 (d) 3 : 2.
ter five half-lives ?
112. The wavelength of the first line of Balmer series is
(a) 3% (b) 5%
6563 Å. The wavelength of the first line of Lyman
(c) 10% (d) 20%. [AFMC 1994] series is
108. Fig. 20 represents in simplified form some of the (a) 1215 Å (b) 4861 Å
lower energy levels of the hydrogen atom. (c) 4340 Å (d) 4101 Å.
If the transition of an electron from E4 to E2 were 113. Consider the fission reaction
associated with the emission of blue light, which one 92 U
236 → X117 + Y117 + n1 + n1
0 0
of the following transitions could be associated with i.e. two nuclei of same mass number 117 are formed
the emission of red light ? plus two neutrons. The binding energy per nucleon
ATOMIC NUCLEUS 715
of X and Y is 8.5 MeV where as of U236 is 7.6 MeV. 116. The energy of an electron in excited Hydrogen atom
The total energy liberated will be about is – 3.4 eV. Then, according to Bohr’s Theory, the
(a) 2 MeV (b) 20 MeV angular momentum of this electron in Js is
(c) 200 MeV (d) 2000 MeV. (a) 2.11 × 10–34 (b) 3 × 10–34
114. In the previous question, the percentage of mass con- (c) 2 × 10–34 (d) 0.5 × 10–34.
verted during fission into energy is nearly
[National Standard Exam. in Physics 1999]
(a) 0.1% (b) 0.01%
117. A radioactive atom X emits a β-particle to produce
(c) 1% (d) 10%.
an atom Y which then emits an α-particle to give an
115. Fig. 22 shows five energy levels of an atom, one be-
atom Z
ing much lower than the other four. Five transitions
between the levels are indicated, each of which pro- 1. the atomic number of X is less than that of Z.
duces a photon of definite energy and frequency. 2. the atomic number of Y is less than that of Z.
E5 3. the mass number of X is the same as that of Y.
E4 (a) 1, 2, 3 correct (b) 1, 2 correct

E3 (c) 2, 3 correct (d) 1 correct


(e) 3 correct. [Northern Ireland Schools
Examination Council]
E2
118. In interpreting Rutherford’s experiments on the scat-
tering of alpha particle by thin foils, one must exam-
ine what were known factors, and what the experi-
ments settled. Which of the following is true in this
context ?
(a) The number of electrons in the target atoms (i.e., Z) was
settled by these experiments.

E1 (b) The validity of Coulomb’s law for distances as small as


10–13 m was known before these experiments.
Fig. 22
(c) The experiments settled that size of the nucleus could not
Which one of the spectra below best corresponds to be larger than a certain value.
the set of transitions indicated ? (d) The experiments also settled that size of the nucleus could
(Low) Frequency (High) not be smaller than a certain value.
[National Standard Exam. in Physics 1994]
119. Bombardment of a certain material with α-particles
(a) produces an emission which penetrates lead, ejects
protons from paraffin wax, and travels at speeds up
to 5 × 107 m s–1. What does this emission consist of ?
(a) X-rays (b) α-particles
(b)
(c) β-particles (d) neutrons
(e) ultra-violet light.
120. The ionisation potential for second He electron is
(c)
(a) 3.6 eV (b) 27.2 eV
(c) 54.4 eV (d) 100 eV.
121. A sample contains 16 g of a radioactive material, the
(d)
half-life of which is two days. After 32 days, the
amount of radioactive material left in the sample is
1
(a) less than 1 mg (b) g
(e) 4
1
(c) g (d) 1 g.
Fig. 23 2
716 COMPREHENSIVE OBJECTIVE PHYSICS

122. An atom makes a transition from a state of energy 128. If N0 is the original mass of the substance of half-life
E2 to one of lower energy E1. period t1/2 = 5 years, then the amount of substance
Which of the following gives the wavelength of the left after 15 years is
radiation emitted in terms of the Planck’s constant (a) N0/8 (b) N0/16
h and the speed of light c ? (c) N0/2 (d) N0/4. [AIEEE 2002]
E2 − E1 hc hc
(a) (b) − 129. The rest mass of the deuteron, 2 H , is equivalent to
hc E2 E1 1

hc hc hc an energy of 1876 MeV, the rest mass of a proton is


(c) − (d) equivalent to 939 MeV and that of a neutron to 940
E1 E2 E2 − E1
c
MeV.
(e)
b
h E2 − E1
.
g A deuteron may disintegrate to a proton and a neu-
tron if it
123. The angular momentum of electron in hydrogen atom
(a) emits a γ-ray photon of energy 2 MeV.
is proportional to
(b) captures a γ-ray photon of energy 2 MeV.
(a) r (b) 1/r
(c) emits a γ-ray photon of energy 3 MeV.
(c) r2 (d) 1/ r .
(d) captures a γ-ray photon of energy 3 MeV.
124. Listed below are five phenomena connected with pho- (e) emits a γ-ray photon of energy 4 MeV.
tons and/or charged particles
130. A newly prepared radioactive nuclide has a decay
1. alpha-particle emission constant λ of 10 –6 s–1.
2. beta-particle emission What is the approximate half-life of the nuclide ?
3. line emission spectra (a) 1 hour (b) 1 day
4. line absorption spectra (c) 1 week (d) 1 month.
5. electron diffraction. [Haryana PMT 2002]
Which of these phenomena gives direct evidence for 131. An energy level diagram for an atom is shown drawn
the existence of discrete electronic energy levels in to scale. The electron transitions give rise to the emis-
atoms ? sion of a spectrum of lines of wavelength, λ1 , λ2 , λ3 ,
(a) 1 and 5 only (b) 2 and 3 only λ4 , λ5 .
(c) 3 and 4 only (d) 2, 3, 4 and 5 only
(e) 1, 2, 3, 4 and 5.
125. A sample contains 10 mg of radio-active material of Energy
half-life 270 days. After 540 days the mass of radio-
active material left will be λ4 λ5
(a) 5 mg (b) 2.5 mg
(c) 1.25 mg (d) zero.
[National Standard Exam. in Physics 1991] λ1 λ2 λ3

126. Cobalt-57 is radioactive, emitting β particles. The half-


life for this is 270 days. If 100 mg of this is kept in an
open container, the mass of cobalt-57 after 540 days
will be Fig. 24
(a) 50 mg (b) 50 / 2 mg Which statement is correct ?
(c) 25 mg (d) zero. (a) It can be deduced from the energy level diagram that
[National Standard Exam. in Physics 1993] λ1 > λ2.
(b) It can be deduced from the energy level diagram that
127. The activity of a radioactive sample is 1.6 curie, and λ3 = λ1 + λ5.
its half-life is 2.5 days. Its activity after 10 days will (c) Of the five wavelengths, λ4 is the shortest.
be (d) The transition corresponding to wavelength λ3 represents
(a) 0.8 curie (b) 0.4 curie the ionisation of the atom.
(c) 0.1 curie (d) 0.16 curie. 132. As the electron in Bohr orbit of hydrogen atom passes
[MP PMT 2000] from state n = 2 to n = 1, the kinetic energy K and
potential energy U change as
ATOMIC NUCLEUS 717
(a) K two-fold, U also two-fold (a) 10.2 eV (b) 0 eV
(b) K four-fold, U also 4-fold (c) 3.4 eV (d) 6.8 eV. [AIEEE 2002]
(c) K four-fold, U two-fold 140. In the disintegration series
(d) K two-fold, U four-fold. 238 α β− A
U → X → U
92 Z
[National Standard Exam. in Physics 1991]
the values of Z and A respectively will be
133. The ratio of the speed of the electron in the first Bohr (a) 92, 236 (b) 88, 230
orbit of hydrogen and the speed of light is equal to
(c) 90, 234 (d) 91, 234. [MP PET 2001]
(where e, h and c have their usual meanings)
141. Samples of two radioactive nuclides, X and Y, each
(a) 2πhc/e2 (b) e2h/2πc
have equal activity A0 at time t = 0. X has a half-life
(c) e2c/2πh (d) 2πe2/hc.
of 24 years and Y a half-life of 16 years.
[MP PMT 2000] The samples are mixed together.
134. An electron has a mass of 9.1 × 10–31
kg. It revolves What will be the total activity of the mixture at t = 48
around the nucleus in a circular orbit of radius 0.529 years ?
× 10–10 metre at a speed of 2.2 × 106 ms–1. The mag- 1 1
(a) A0 (b) A0
nitude of its linear momentum in this motion is 12 4
(a) 1.1 × 10–34 kg m s–1 (b) 2.0 × 10–24 kg m s–1 3 3
(c) A0 (d) A 0 .
(c) 4.0 × 10–24 kg m s–1 (d) 4.0 × 10–31 kg m s–1. 16 8
142. Considering only alpha particles, electrons and pro-
135. The wavelength of first line of Lyman series in hy-
tons, which one of the following correctly names the
drogen atom is 1216 Å. The wavelength of first line
particles for which the magnitude of the specific charge
of Lyman series for 10 times ionised sodium atom
(Q/m) is greatest and least ?
will be
greatest Q/m least Q/m
(a) 0.1 Å (b) 1000 Å
(a) electron alpha particle
(c) 100 Å (d) 10 Å.
(b) alpha particle electron
[Bharati Vidyapeeth 2001] (c) electron proton
136. The half-life of a certain radioactive isotope is 32 hours. (d) proton electron
What fraction of a sample would remain after 16 (e) proton alpha particle.
hours ? 143. The wavelengths involved in the spectrum of deute-
(a) 0.25 (b) 0.71
rium
FH DIK are slightly different from that of hydro-
2
1
(c) 0.29 (d) 0.75
(e) 0.50. gen spectrum, because
(a) the attraction between the electron and the nucleus is dif-
137. Suppose two deuterons must get as close as 10 –14 m ferent in the two cases
in order for the nuclear force to overcome the repul-
(b) the size of the two nuclei are different
sive electrostatic force. The height of the electrostatic
(c) the nuclear forces are different in the two cases
barrier is nearest to
(d) the masses of the two nuclei are different.
(a) 0.14 MeV (b) 2.3 MeV
[AIEEE 2003]
(c) 1.8 × 10–9 MeV (d) 0.56 MeV.
144. A sample of radioactive element has a mass of 10 g at
[National Standard Exam. in Physics 1996]
an instant t = 0. The approximate mass of this ele-
138. The ionisation potential of H atoms is 13.6 V. The ment in the sample after two mean lives is
energy difference between n = 2 and n = 3 levels is (a) 1.35 g (b) 2.50 g
nearest to (c) 3.70 g (d) 6.30 g.
(a) 1.9 eV (b) 2.3 eV [All India PM/PD 2003]
(c) 3.4 eV (d) 4.5 eV. 145. If the binding energy of the electron in a hydrogen
[National Standard Exam. in Physics 1995] atom is 13.6 eV, the energy required to remove the
139. If 13.6 eV energy is required to ionise the hydrogen electron from the first excited state of Li++ is
atom, then the energy required to remove an elec- (a) 122.4 eV (b) 30.6 eV
tron from n = 2 is (c) 13.6 eV (d) 3.4 eV. [AIEEE 2003]
718 COMPREHENSIVE OBJECTIVE PHYSICS

146. Atomic mass number of an element is 232 and its (a) R/E (b) E/V
atomic number is 90. The end product of this radio- (c) RE [KCET 2002]
(d) VR.
active element is an isotope of lead (atomic mass 208 154. In a sample of radioactive material, what percentage
and atomic number 82). The number of α and β par- of the initial number of active nuclei will decay dur-
ticles emitted is ing one mean life ?
(a) α = 3, β = 3 (b) α = 6, β = 4 (a) 69.3% (b) 63%
(c) α = 6, β = 0 (d) α = 4, β = 6. (c) 50% (d) 37%. [KCET 2002]
147. An electron in H-atom makes a transition from n = 3 155. A radioactive sample consists of two distinct species
to n = 1. The recoil momentum of H-atom will be having equal number of atoms initially. The mean
(a) 6.45 × 10–27 N s (b) 6.8 × 10–27 N s life time of one species is τ and that of the other is 5τ.
(c) 6.45 × 10–24 Ns (d) 6.8 × 10–24 N s. The decay products in both cases are stable. A plot is
[BHU 1994] made of the total number of radioactive nuclei as a
function of time. Which of the following figures best
148. The frequency of revolution of electron in nth orbit is
represents the form of this plot ?
fn. If the electron makes a transition from nth orbit
(a) (b)
to (n–1)th orbit, then the relation between the fre- N N
quency (ν) of emitted photon and fn will be
(a) ν = fn2 (b) ν = fn
1
(c) ν = (d) ν = fn. [AFMC 2000]
fn
149. If n >> 1, then the dependence of frequency of a pho- τ t τ t
ton, emitted as a result of transition of electron from (c) (d)
N N
nth orbit to (n – 1)th orbit, on n will be
1 1
(a) ν ∝ (b) ν ∝ 2
n n
1 1
(c) ν ∝ (d) ν ∝ .
n3 ν3
[Haryana PMT 2001] τ t τ t

150. In which of the following systems will the radius of Fig. 25


the first orbit (n = 1) be minimum ? [IIT 2001]
(a) doubly ionized lithium (b) singly ionized helium 156. After an interval of one day, 1/16th initial amount
(c) deuterium atom (d) hydrogen atom. of a radioactive material remains in a sample. Then
[All India PM/PD 2003] its half-life is
(a) 6 hours (b) 12 hours
151. The radius of the Bohr orbit in the ground state of
hydrogen atom is 0.5 Å. The radius of the orbit of (c) 1.5 hours (d) 3 hours. [KCET 2000]]
the electron in the third excited state of He+ will be 157. Two radioactive materials X1 and X2 have decay con-
(a) 8 Å (b) 4 Å stants 10λ and λ respectively. If initially they have
(c) 0.5 Å (d) 0.25 Å. [MP PMT 2000]
the same number of nuclei, then the ratio of the
number of nuclei of X1 to that of X2 will be 1/e after
152. A hydrogen atom moving with velocity 4 m s–1 ab- a time
sorbs a photon of wavelength λ and stops. The value 1 1
of λ will be (a) (b)
10λ 11λ
(a) 1000 Å (b) 2000 Å 11 1
(c) (d) . [IIT 2000]
(c) 3000 Å (d) 4000 Å. 10λ 9λ
[All India PM/PD 2002] 158. The electron in a hydrogen atom makes a transition
from an excited state to the ground state. Which of
153. In the Bohr model of the hydrogen atom, let R, V the following statements is true ?
and E represent the radius of the orbit, the speed of (a) Its kinetic energy increases and its potential and total ener-
electron and the total energy of the electron respec- gies decrease.
tively. Which of the following quantities is propor- (b) Its kinetic energy decreases, potential energy increases
tional to the quantum number n ? and its total energy remains the same.
ATOMIC NUCLEUS 719
(c) Its kinetic and total energies decrease and its potential en- (a) nitrogen (b) carbon
ergy increases. (c) boron (d) oxygen. [IIT 1999]
(d) Its kinetic, potential and total energies decrease.
165. Three-fourths of the active nuclei present in a radio-
[IIT 2000] active sample decay in 3/4 s. The half-life of the sam-
159. The electron in a hydrogen atom makes a transition ple is
from n = n1 to n = n2 state. The time period of the (a) 1 s (b) 1/2 s
electron in the initial state is eight times that in the (c) 3/4 s (d) 3/8 s. [KCET 2001]
final state. The possible values of n1 and n2 are A
166. If m, mn and mp are the masses of Z X nucleus,
(a) n1 = 4, n2 = 2 (b) n1 = 8, n2 = 2
neutron and proton respectively
(c) n1 = 8, n2 = 1 (d) n1 = 6, n2 = 2.
(a) m = (A – Z) mn + Zmp (b) m < (A – Z) mn + Zmp
[KCET 2001] (c) m > (A – Z) mn + Zmp (d) m = (A – Z) mp + Zmn.
160. An electron revolving in an orbit of radius 0.5 Å in a [KCET 2003]
hydrogen atom executes 1016 revolutions per second. 215At
167. The half-life of is 100 µs. The time taken for the
The magnetic moment of electron due to its orbital
radioactivity of a sample of 215At to decay to 1/16th
motion will be
of its initial value is
(a) 1.256 × 10–23 A m2 (b) 653 × 10–26 A m2
(a) 400 µs (b) 6.3 µs
(c) 10–3 A m2 (d) 256 × 10–26 A m2.
(c) 40 µs (d) 300 µs. [IIT 2002]
[AMU 2001]
168. Which of the following processes represents a gamma-
161. Four lowest energy levels of H atom are shown in decay ?
Fig. 26. The number of possible emission lines would
(a) AXZ + γ → AXZ–1 + a + b (b) AXZ + 1n0 → A–3XZ–2 + c
be
(c) AXZ → AXZ + f (d) AXZ + e–1 → AXZ–1 + g .
n=4
[IIT 2002]
n=3
169. The transition from the state n = 4 to n = 3 in a
n=2 hydrogen-like atom results in ultraviolet radiation.
Infra red radiation will be obtained in the transition
(a) 2 → 1 (b) 3 → 2
n=1 (c ) 4 → 2 (d) 5 → 4. [IIT 2001]
Fig. 26 170. The total energy of electron is the ground state of
hydrogen atom is – 13.6 eV. The potential energy of
(a) 3 (b) 4 an electron in the ground state of Li2+ ion will be
(c) 5 (d) 6. [MP PMT 2001] (a) 122.4 eV (b) – 122.4 eV
162. Which stable nucleus has radius one-third of that of (c) 244.8 eV (d) – 244.8 eV. [BHU 1997]
Os189 ?
171. The radius of 29
Cu64 nucleus will be
(a) Be9 (b) Li7
(a) 1.2 × 10–15 m (b) – 2.4 × 10–15 m
(c) F19 (d) none of these.
(c) 3.6 × 10–15 m (d) 4.8 × 10–15 m.
[Pb. PMT 1998]
[CMC LDH 1996]
163. The half-life period of a radioactive element X is same 172. If mass of proton = 1.008 amu and mass of neutron
as the mean-life time of another radioactive element = 1.009 amu, then the binding energy per nucleon
Y. Initially both of them have the same number of for 4Be9 (mass = 9.012 amu.) will be
atoms. Then (a) 0.0672 MeV (b) 0.672 MeV
(a) X and Y have the same decay rate initially. (c) 6.724 MeV (d) 67.2 MeV.
(b) X and Y decay at the same rate always. [CMC LDH 2002]
(c) Y will decay at a faster rate than X. 173. The wavelength of radiation, required to excite the
(d) X will decay at a faster rate than Y. [IIT 1999] electron from first orbit to third orbit in a doubly
22Ne ionised lithium atom, will be
164. nucleus, after absorbing energy, decays into two
(a) 134.25 Å (b) 125.5 Å
α-particles and an unknown nucleus. The unknown
nucleus is (c) 113.7 Å (d) 110 Å. [BHU 2002]
720 COMPREHENSIVE OBJECTIVE PHYSICS

174. The force acting on the electron in a hydrogen atom 182. A sample of hydrogen is bombarded by electrons.
depends on the principal quantum number as Through what potential difference should the elec-
1 trons be accelerated so that second line of Balmer
(a) F ∝ n2 (b) F ∝
n2 series be emitted ?
1 (a) 2.55 V (b) 10.2 V
(c) F ∝ n4 (d) F ∝ 4 . [AFMC 1995] (c) 12.09 V (d) 12.75 V. [AMU 2002]
n
175. If the rest mass of electron or positron is 0.51 MeV, 183. The mass defect due to nuclear fission is 0.1%. The
then the kinetic energy of each particle in the electron- energy released on account of fission of 1 kg mass
positron pair production by a γ-photon of 2.42 MeV will be (in kWH)
will be (a) 2.5 × 10–7 (b) 2.5 × 107
(a) 0.3 MeV (b) 1.9 MeV (c) 2.5 × 109 (d) 2.5 × 105.
(c) 0.7 MeV (d) 1.5 MeV. [CMC LDH 2003]
[Manipal 1995] 184. The minimum energy to ionise an atom is the energy
176. In Rutherford scattering experiment, the number of required to
α-particles scattered at 60° is 5 × 106. The number of (a) add one electron to the atom.
α-particles scattered at 120° will be (b) excite the atom from its ground state to its first excited
3 state.
(a) 15 × 106 (b) × 106 (c) remove one outermost electron from the atom.
5
5 (d) remove one innermost electron from the atom.
(c) × 106 (d) none of these. (e) remove all the electrons from the atom.
9
[BHU 2003] 185. A stationary thoron nucleus (A = 200, Z = 90) emits
177. The energy released as a result of mutual interaction an alpha particle with kinetic energy Ea. What is the
of proton and antiproton will be kinetic energy of the recoiling nucleus ?
Ea Ea
(a) 1.862 eV (b) 1.862 KeV (a) (b)
108 110
(c) 1.862 MeV (d) 1.862 GeV.
Ea E
[CMC LDH 1994] (c) (d) a
55 54
178. The kinetic energy of an electron in the fourth orbit (e) Ea.
of lithium atom will be 186. The fraction of a radioactive material which remains
(a) 0.85 eV (b) 1.70 eV active after time t is 9/16. The fraction which remains
(c) 7.65 eV (d) 9.36 eV. active after time t/2 will be
[Haryana PMT 2003] 4 7
(a) (b)
179. The speed of electrons in the second orbit of Be3+ ion 5 8
3 3
will be (c) (d) . [JIPMER 2002]
5 4
c 2c
(a) (b) 187. The half-life of a radioactive substance, as compared
137 137 to its mean life, is nearly
3c 4c
(c) (d) . (a) 30% (b) 60%
137 137
(c) 70% (d) 100%.
[Himachal PMT 2000]
[Himachal PMT 2002]
180. The potential energy of an electron in the fifth orbit
of hydrogen atom is 188. The rate of decay of a radioactive element at any
(a) 0.54 eV (b) – 0.54 eV
instant 103 disintegrations per second. If the half-life
of the element is 1 second, then the rate of decay
(c) 1.08 eV (d) – 1.08 eV.
after one second will be
[Manipal 1996] (a) 500 per s (b) 1000 per s
181. If the radius of an orbit is r and the velocity of elec- (c) 250 per s (d) 2000 per s. [AMU 2003]
tron in it is v, the frequency of electron in that orbit
189. If R is the Rydberg constant for Hydrogen, the wave
will be
number of the first line in the Lyman series is

(a) 2πrv (b) R
vr (a) (b) 2R
2
vr v R 3R
(c) (d) . [Manipal 2003] (c) (d) . [KCET 2000]
2π 2πr 4 4
ATOMIC NUCLEUS 721
190. If elements with principal quantum number n > 4 (a) 449 years (b) 399 years
were not allowed in nature, the number of possible (c) 109 years (d) 9 years.
elements would have been [CMC Vellore 2003]
(a) 32 (b) 60
198. An archaeologist analyses a wooden sample of pre-
(c) 64 (d) 4. historic structure and finds that the ratio of C14 to
[KCET 2003 ; MP PET 1999 ; IIT 1983] ordinary carbon in it is only one-fourth of that in the
191. Half-life of a radioactive substance is 20 minutes. The cells of living plants. If the half-life of C 14 is 5700
time between 20% and 80% decay will be years, then the age of the wood will be
(a) 40 minutes (b) 20 minutes (a) 22,807 years (b) 5709 years

(c) 25 minutes (d) 30 minutes. [KCET 2003] (c) 2458 years (d) 11,403 years.

192. A nucleus with mass number 220 initially at rest emits [MNR 1999]
an α-particle. If the Q value of the reaction is 199. The percentage of quantity of a radioactive material
5.5 MeV, calculate the kinetic energy of the α-particle that remains after 5 half-lives will be
(a) 4.4 MeV (b) 5.4 MeV (a) 31% (b) 3.125%
(c) 5.6 MeV (d) 6.5 MeV. [IIT 2003] (c) 0.3% (d) 1%. [Pb. PMT 2003]
193. For an electron in the second orbit of Bohr Hydro- 200. A hydrogen-like atom emits radiations of frequency
gen atom, the moment of linear momentum is 2.7 × 1015 Hz when it makes a transition from n = 2
(a) πh (b) 2πh to n = 1. The frequency emitted in a transition from
h 2h n = 3 to n = 1 will be
(c) (d) . [KCET 2003] (a) 1.8 × 1015 Hz (b) 3.2 × 1015 Hz
π π
194. Let the potential energy of hydrogen atom in the (c) 4.7 × 105 Hz (d) 6.9 × 1015 Hz.
ground state be zero. Then its total energy in the [CMC Vellore 2002]
first excited state will be
201. Radiation from a radioac-
(a) 10.2 eV (b) 13.6 eV tive source enters an evacu-
(c) 23.8 eV (d) 27.2 eV. [KCET 2001] ated region in which there
195. The graph represents the decay of a newly-prepared is a uniform magnetic field
sample of radioactive nuclide X to a stable nuclide Y. perpendicular to the plane Y
The half-life of X is τ. The growth curve for Y inter- of the diagram. This region X
sects the decay curve for X after time T. is divided into two by a
sheet of aluminium about Aluminium
Number
of atoms X 1 mm thick. The curved,
Fig. 28
Y horizontal path followed by
the radiation is shown in Fig. 28.
Which of the following correctly describes the type of
radiation and its point of entry ?
type of radiation point of entry
0 T Time
(a) alpha X
Fig. 27 (b) alpha Y
What is the time T ? (c) beta X
(a) τ/2 (b) ln (τ/2) (d) beta Y
(c) τ (d) ln (2τ) (e) gamma X.
(e) 2τ. 202. A radioactive nucleus undergoes a series of decays
according to the scheme
196. The decay constant of radium is 4.28 × 10–4 per year.
Its half-life will be α β α γ
A → A1 → A2 → A3 → A4
(a) 2000 year (b) 1240 year If the mass number and atomic number of A are 180
(c) 63 year (d) 1619 year. and 72 respectively, then what are these numbers for
197. The mean lives for α- and β-decay of radioactive sub- A4 ?
stance are 1620 years and 405 years respectively. In (a) 172 and 69 (b) 174 and 70
3 (c) 176 and 69 (d) 176 and 70.
how much time will its th fraction will decay, if it
4
[CMC LDH 1993]
simultaneously emits α- and β-particles ?
722 COMPREHENSIVE OBJECTIVE PHYSICS

203. The intensity of γ-rays from a source (I0 ) reduces to 210. The wavelengths of first lines of Balmer series of hy-
I0/8 after passing through 48 mm thick sheet of lead. drogen, He+ and Be3+ are λ1 , λ2 and λ3 respectively.
The thickness of the sheet for obtaining intensity equal The ratio of these wavelengths will be
to I0/2 will be (a) 1 : 4 : 16 (b) 1 : 2 : 4
(a) 48 mm (b) 24 mm (c) 16 : 4 : 1 (d) 4 : 2: 1. [JIPMER 1998]
(c) 16 mm (d) 8 mm. [MNR 1996] 211. If there are N nuclear particles in a nucleus of radius
204. The orbital velocity of electron in the ground state is R, then the number of nuclear particles in radius 3R
v. If the electron is excited to energy state – 0.54 eV, will be
its orbital velocity will be (a) N (b) 2N
v (c) 27 N (d) 21/3 N. [Pb. CET 1998]
(a) v (b)
3 212. The mass numbers of helium and sulphur are 4 and
v v 32 respectively. The ratio of the nuclear radii of sul-
(c) (d) . [WB JEE 1994]
5 7 phur and helium will be
205. If 10% of a radioactive substance decays in every 5 (a) 32 (b) 8
years, then the percentage of the substance that will
have decayed in 20 years will be (c) 2 [Pb. PMT 1999]
(d) 2.
(a) 40% (b) 50% 213. At time t = 0, some radioactive gas is injected into a
(c) 65.6 % (d) 34.4%. [EAMCET 1992]
sealed vessel. At time T, some more of the same gas
is injected into the same vessel.
206. A stationary 238 U nucleus decays by α emission
Which one of the following graphs best represents
generating a total kinetic energy T
the variation of the logarithm of the activity A of the
238
U → 234
Th + 42 α gas with time t ?
92 90
What is the kinetic energy of the α particle ? In A In A
(a) slightly less than T/2
(b) T/2
(c) slightly less than T
(d) slightly greater than T.
207. The graph (Fig. 29)
T t T t
shows the number 3
(a) (b)
of particles N t 2 In A In A
In Nt

emitted per
1
second by a
0
radioactive source 0 20 40 60
as a function of Time, t
time t Fig. 29
T t T t
The relationship between Nt and t is (c) (d)
(a) Nt = 1000 e–20t (b) Nt = 20 e20t In A
(c) Nt = 3 e–0.05t (d) Nt = 20 e–0.05t
(e) Nt = 1000 e0.05t.
208. The mass defect for the nucleus of helium is 0.0303
amu. What is the binding energy per nucleon for
helium in MeV ?
(a) 28 (b) 7 T t
(e)
(c) 4 (d) 1.
209. The radioactivity of a certain radioactive element Fig. 30
drops to 1/64 of its initial value in 30 second. Its half
life is 214. The masses of neutron and proton are 1.0087 amu
(a) 2 second (b) 4 second and 1.0073 amu respectively. If the neutrons and
(c) 5 second (d) 6 second.
ATOMIC NUCLEUS 723
protons combine to form a helium nucleus (alpha 222. An α-particle of energy 5 MeV is scattered by a ura-
particles) of mass 4.0015 amu, the binding energy of nium nucleus at an angle of 180°. The distance of
the helium nucleus will be : (1 amu = 931 MeV) closest approach of α-particles to the nucleus will be
(a) 28.4 MeV (b) 20.8 MeV (a) 5.3 × 10–14 cm (b) 5.3 × 10–10 cm
(c) 27.3 MeV (d) 14.2 MeV. (c) 5.3 × 10–14 m (d) 5.3 × 10–10 m.
215. The masses of two radioactive substances are same 223. The minimum wavelength of Paschen series of hy-
and their half-lives are 1 year and 2 years respec- drogen atom will be
tively. The ratio of their activities after six years will (a) 18,700 Å (b) 970 Å
be (c) 1,022 Å (d) 8,182 Å.
(a) 1 : 4 (b) 4 : 1
[Haryana PMT 1998]
(c) 1 : 8 (d) 8 : 1.
224. The ratio of area of orbit of first excited state of elec-
216. An excited hydrogen atom returns to the ground tron to the area of orbit of ground level, for hydro-
state. The wavelength of emitted photon is λ . The gen atom, will be
principal quantum number of the excited state will (a) 16 : 1 (b) 4 : 1
be
(c) 8 : 1 (d) 2 : 1.
F λR IJ 1/2
(a) G
F λR + 1IJ 1/2
(b) G F RI
H λR − 1K H λR K 225. The graph of log GH R JK versus A (R = radius of a
0
L
(d) M
O 1/2 nucleus and A = mass number) is
N λR (λR + 1) PQ .
1
(c) λR (λR + 1) 1/2 (a) a circle (b) an ellipse
(c) a parabola (d) a straight line.
[Manipal 1998]
[AIIMS 1998]
217. The radius of the nucleus 8O16 is 3 × 10–15 m. The
density of this nucleus will be 226. An α-particle of energy 5 MeV is scattered through
(a) 2.35 × 10–17 kg m–3 (b) 3.35 × 1017 kg m–3 180° by a fixed uranium nucleus. The distance of clos-
est approach is of the order of
(c) 2.35 × 1017 kg m–-3 (d) 3.35 × 10–17 kg m–3 .
(a) 10–12 cm (b) 10–10 cm
[AMU 1998]
(c) 1 Å (d) 10–15 cm. [AIEEE 2004]
218. If the magnitude of energy of the electron in nth Bohr
orbit is En and Jn is its angular momentum, then 227. The equations
1 dN
(a) En ∝ Jn3 (b) En ∝ = − λN and N = N0 e–λt
−3 dt
Jn
1 describe how the number N of undecayed atoms in a
(c) En ∝ Jn2 (d) En ∝ . sample of radioactive material, which initially (at
Jn 2
t = 0) contained N 0 undecayed atoms, varies with
[CMC LDH 1997]
time t. Which one of the following statements about
219. What is the relationship between the decay constant λ is correct ?
λ and the half-life t1/2 of a radioactive isotope ? (a) λδt gives the fraction of atoms present which will decay in
(a) λ = t1/2 (b) λ = 1/t1/2 the next small time interval δt.
(c) λ = t1/2 ln2 (d) λ = (ln2 )/t1/2 (b) λ is the time needed for N to fall from N0 to the value N0/e.
1 (c) λ is equal to the half-life of the sample.
(e) λ = .
t1/2 ln2 (d) λ is the number of atoms left after a time equal to e second.
220. A nucleus disintegrates into two nuclear parts which (e) λ is the chance that any one atom will still be undecayed
have their velocities in the ratio of 2 : 1. The ratio of after one second.
their nuclear sizes will be 228. A nucleus of mass number A, originally at rest, emits
(a) 31/2 : 1 (b) 1 : 21/3 an α-particle with speed v. The daughter nucleus re-
(c) 21/3 : 1 (d) 1 : 31/2. [AIEEE 2004] coils with a speed.
221. In a radioactive substance at t = 0, the number of (a) 2v/(A + 4) (b) 4v/(A + 4)
atoms is 8 × 104. Its half-life period is 3 year. The (c) 4v/(A – 4) (d) 2v/(A – 4).
number of atoms 1 × 104 will remain after interval [AIIMS 2004]
(a) 9 years (b) 8 years 229. The maximum wavelength of Brakett series of hy-
(c) 6 years (d) 24 years. drogen atom will be
724 COMPREHENSIVE OBJECTIVE PHYSICS

(a) 404 Å (b) 4040 Å different wavelengths in the emission spectrum of


(c) 40404 Å (d) 40,400 Å. excited atoms. The value of λ will be
(a) 1212 Å (b) 912 Å
230. A hydrogen atom in the ground state is excited by
radiations of wavelength 975 Å. The energy state to (c) 947 Å (d) 1247 Å.
which the atom is excited, is [JIPMER 1999]
(a) 4 (b) 3 237. Energy E of a hydrogen atom with principal quan-
(c) 2 (d) 1. − 13.6
231. If the difference of energies of an electron in the sec- tum number n is given by E = eV . The energy
n2
ond and the fourth orbits of an atom is E, then the of a photon ejected when the electron jumps from n =
ionisation energy of that atom will be 3 state to n = 2 state of hydrogen is approximately
36 16 (a) 0.85 eV (b) 3.4 eV
(a) E (b) E
15 3
(c) 1.9 eV (d) 1.5 eV.
15 3
(c) E (d) E. [MNR 1997] [All India PM/PD 2004]
36 16
232. The fraction of atoms of radioactive element that de- 238. If the half-lives of a radioactive element for α and β
cays in 6 days is 7/8. The fraction that decays in 10 decay are 4 year and 12 years respectively, then the
days will be percentage of the element that remains after 12 year
will be
(a) 70/80 (b) 77/88
(a) 6.25% (b) 5.25%
(c) 31/32 (d) 35/36.
(c) 4.25% (d) 3.50. [AFMC 1999]
233. The energy of He+ ion in the first excited state will be
(a) – 13.6 eV (b) – 14.4 eV 239. After 280 days, the activity of a radioactive sample is
6000 dps. The activity reduces to 3000 dps after an-
(c) – 6.8 eV (d) – 27.2 eV.
other 140 days. The initial activity of the sample in
[CMC Vellore 2000] dps is
234. The binding energy per nucleon of deuteron ( 12 H ) and (a) 6000 (b) 9000
(c) 3000 (d) 24000.
helium nucleus ( 24 He)
is 1.1 MeV and 7 MeV respec-
tively. If two deuteron nuclei react to form a single [IIT Screening 2004]
helium nucleus, then the energy released is 240. An X-ray tube is operated at a constant potential dif-
(a) 23.6 MeV (b) 26.9 MeV ference and it is required to get X-rays of wavelength
(c) 13.9 MeV (d) 19.2 MeV.
not less than 0.2 nm. Then the potential difference in
kilovolt is
[AIEEE 2004]
[Given : h = 6.63 × 10–34 J s, e = 1.6 × 10–19 c, c = 3 ×
235. The half-life of radium is about 1600 years. Of 100 g 108 m s–1]
of radium existing now, 25 g will remain unchanged
(a) 24.8 (b) 12.4
after
(a) 6400 years (b) 2400 years
(c) 6.2 (d) 3.1. [EAMCET 2003]
(c) 3200 years (d) 4800 years. 241. A nucleus splits into two nuclear parts having radii
in the ratio 1 : 2. Their velocities are in the ratio
[All India PM/PD 2004]
(a) 8 : 1 (b) 6 : 1
236. Radiations of wavelength λ are incident on hydro-
gen atoms in the ground state. A fraction of these (c) 4 : 1 (d) 2 : 1. [EAMCET 2003]
radiations is absorbed by these atoms. There are ten

Answers (Set II)


83. (b) 84. (a) 85. (a) 86. (d) 87. (d) 88. (c) 89. (e) 90. (a)
91. (a) 92. (b) 93. (b) 94. (b) 95. (d) 96. (b) 97. (d) 98. (a)
99. (a) 100. (b) 101. (c) 102. (c) 103. (d) 104. (b) 105. (d) 106. (d)
107. (a) 108. (c) 109. (c) 110. (a) 111. (c) 112. (a) 113. (c) 114. (a)
115. (e) 116. (a) 117. (e) 118. (c) 119. (d) 120. (c) 121. (a) 122. (d)
ATOMIC NUCLEUS 725

123. (a) 124. (c) 125. (b) 126. (c) 127. (c) 128. (a) 129. (d) 130. (c)
131. (a) 132. (b) 133. (d) 134. (b) 135. (d) 136. (b) 137. (a) 138. (a)
139. (c) 140. (d) 141. (d) 142. (a) 143. (d) 144. (a) 145. (b) 146. (b)
147. (a) 148. (d) 149. (c) 150. (a) 151. (b) 152. (a) 153. (d) 154. (b)
155. (d) 156. (a) 157. (d) 158. (a) 159. (a) 160. (a) 161. (d) 162. (b)
163. (c) 164. (b) 165. (d) 166. (b) 167. (a) 168. (c) 169. (d) 170. (d)
171. (d) 172. (c) 173. (c) 174. (d) 175. (c) 176. (c) 177. (d) 178. (c)
179. (b) 180. (d) 181. (d) 182. (d) 183. (b) 184. (c) 185. (d) 186. (d)
187. (c) 188. (a) 189. (d) 190. (b) 191. (a) 192. (b) 193. (c) 194. (c)
195. (c) 196. (d) 197. (a) 198. (d) 199. (b) 200. (b) 201. (d) 202. (a)
203. (c) 204. (c) 205. (d) 206. (c) 207. (d) 208. (b) 209. (c) 210. (c)
211. (c) 212. (c) 213. (b) 214. (a) 215. (a) 216. (a) 217. (c) 218. (d)
219. (d) 220. (b) 221. (a) 222. (c) 223. (d) 224. (a) 225. (d) 226. (a)
227. (a) 228. (c) 229. (d) 230. (a) 231. (b) 232. (c) 233. (a) 234. (a)
235. (c) 236. (c) 237. (c) 238. (a) 239. (d) 240. (c) 241. (a).

Solutions (Set II)


83.
1 1
=R 2 − 2 =
LM
1 5R OP
;λ=
36 visible spectrum was the Balmer series and the infra
λ 2 3N 36 Q 5 R red was the Paschen series.
The ultraviolet series is obtained when the energy of
84. Widest energy gap.
the atom falls to the lowest energy level E1 corre-
85. 30 minutes is 3 half-lives for P and 2 half-lives for Q.
sponding to n = 1. The visible spectrum is obtained
N0 for energy falls to a higher level E2 corresponding to
86. In 1 half-life, the radioactive nuclei remaining is .
2 n = 2. The infrared spectrum is obtained for energy
N0 falls to the higher level E3 where n = 3.
In n half-lives, the radioactive nuclei remaining is n .
2 Thus, ultraviolet radiation could only be possible with
So, number of nuclei disintegrated in n half-lives is transition from E2 to E1 out of the five given transi-
F N I
GH JK 1
N0 − 0 . For n = , the fraction distintegrated is
2n 2
tions.
92. For first member,
FG 1 − 1 IJ . 1 LM1 1 OP 1 LM
1 1 OP
H 2K λ1
=R 2 − 2
N
2 3 Q or
λ1
=R −
4 9N Q
87. α decreases mass number by 4 and reduces charge
1 L 9 − 4 OP
=RM
36
number by 2. β-decay keeps mass number unchanged
and increases charge by 1. Clearly, (d) is the right
or
λ1 N 36 Q or λ1 =
5R
choice. For second member,
1 LM
1 1 OP 1 LM 1 1
=R 2 − 2
OP 1 1 1 LM OP
88.
λ
=R 2 − 2
2N 4 Q λ2 N2 4 Q or
λ2
=R −
4 16 N Q
or
ν L 1 1 OP
=RM −
LM 1 − 1 OP 1 L 4 − 1OP
=RM
1 3R
c N 4 16 Q or ν = cR
N 4 16 Q or
λ 2 N 16 Q
or =
λ 2 16
3 9 16
= 3 × 108 = × 1015 Hz.
× 105 (10–2)–1 × or λ2 =
16 16 3R
91. The wavelengths of the hydrogen spectrum could be
λ 1 36 3R λ1 27
arranged in a formula or series named after its Now, = × or =
discoverer. The ultraviolet was the Lyman series, the λ 2 5R 16 λ 2 20
726 COMPREHENSIVE OBJECTIVE PHYSICS

27 27
or
1
=R
1LM

1 OP
or
1 3
or λ1 =
20
× λ2 =
20
× 4861 Å = 6562.4 Å.
λ′ N
4 16 Q λ′
=R×
16
1 c 1 16
93. v= or v ∝ or λ′ = R
137 n n 3
λ ′ 16 8R 128
Since v is reduced to one-third therefore = × or λ′ = λ.
λ 3R 9 27
n =3
12375
Now, r ∝ n2. 100. = 2.1 eV
5890
94. Decrease in mass number = 232 – 208 = 24 So, first excitation potential is 2.1 V.
24 101. A = A0 e–λt ; 2100 = 16000 e–12λ ; e12λ = 7.6 ; 12λ = loge
Number of α-particles emitted = =6
4
2 1
Due to emission of 6 α-particles, decrease in charge 7.6 = 2 ; λ = = ;
number = 12. 12 6
But decrease in charge number is 8. Clearly, 4 β par- 0.6931 × 6
T= ≈ 4.
ticles are emitted. 1

95. L = mvr = m k
LM 2 π Ze 2 OP LM 1 n h OP
2 2
102. Let the three energy levels
be E1, E2 and E3. The wave-
E3
λ3
MN nh PQ MN k 4π mZe PQ
2 2 lengths λ1, λ2 and λ3 of the E2
spectral lines correspond to
nh the three energy transitions
= λ2 λ1
2π are depicted as shown. By
Clearly, L is constant. quantum theory, the energy
change E is directly related
K 1 FG IJ to the frequency f and hence E1
96. Bohr radius a0 =
m
. Changing m to µ = m leads
2 H K it is inversely proportional to Fig. 31
to modified value 2a0. (This is not from e– to centre of the wavelength λ of the elec-
mass, but from e– to e+ ). tromagnetic radiation as stated in the equation be-
97. Rydberg constant determines the frequencies. We low.
have R = K m. So modified R is half of H-atom.
hc 1
K . Since modified m is half, and modified r E = hf = or E∝
98. mvn2= n λ λ
rn (Given λ1 < λ2 < λ3)
is double, vn remains the same as in H-atom.
Thus, for the three wavelengths, we have

 Positronium is an atom in which an elec-


tron (e–) and a positron (e+ ) go round their cen-
R| E3 – E2 =
hc
λ3
...(1)

tre of mass. Bohr’s conditions hold for it, as ||


used in Hydrogen atom, but the mass me of the
electron is replaced by the modified mass
S| E2 – E1 =
hc
λ2
...(2)

me m p
|| hc
µ=
me + mp
, where mp is the positronium mass, T E3 – E1 =
λ1
...(3)

which is equal to me. With this, one may treat Now, E3 – E1 = (E3 – E2) + (E2 – E1)
the electron going round the positron, and ap- 1 1 1
hc hc hc
ply the equations used for hydrogen atom case. ⇒ = + ⇒ = + .
λ1 λ3 λ2 λ 1 λ 2 λ 3

1 1 LM1 OP 103. (a) No, since Balmer formula was known.


99.
λ 1 N
=R 2 − 2
3 Q (b) No, since Rutherford scattering experiment was
known.
1 L 1 O 8R
= R M1 − P = 9
λ N 9Q 9 or λ =
8R
(c) No, since Einstein photon theory was known.
(d) Bohr chose ‘allowed’ energy levels α′/n2 and these
1
=RM
L 1 − 1 OP led to angular momentum quantised as nh/2π as a
Again,
λ′ N2 4 Q
2 2
derivation.
ATOMIC NUCLEUS 727
104. The nuclear fission differs from other nuclear reactions Blue light falls in the Balmer series and it is obtained
in three respects when the atom makes a transition from E4 to E2.Red
(a) the nucleus is deeply divided into two large fission light falls also in the Balmer series and it has a lower
fragments or nuclei of roughly equal mass. The frequency compared to blue light. By quantum theory
nuclei or fission fragments fly apart at great speed of radiation, the energy change E is proportional to
thus possess large kinetic energies that carry off the frequency of electromagnetic radiation f by E =
the greater part of the energy released. hf. Thus, red light is associated with a smaller energy
change from a lower energy level (compared to E4) to
(b) the mass decrease is appreciable and hence large
the first excited state E2. Hence, the only possible
energy is released.
transition that results in the emission of red light is
(c) other neutrons, called fission neutrons, are emit- the E3 to E2 transition.
ted in the process. Small amount of energy is re-
109. r is for fractional decay rate, which by Rutherford
leased in the form of β and γ radiation.
law, is constant.
105. For Lyman series, the series limit wavelength is given
by e 2
110. B = constant
1 LM
1 1
= R 2 − 2 = R or λ =
OP 1
m
λ 1N ∞ Q R
or
m1
m ∝ eB2
e×5×5 1 2
For Balmer series, the series limit wavelength is given Now, = = =
by m2 2e × 7.5 × 7.5 3 × 1.5 9
1 1LM1
=R 2 − 2
OP = R 4 1 e
λ′ 2 N
∞ Q 4 or λ′ =
R
111.
E m
= constant

λ′ = 4
LM 1 OP e
Clearly,
NR Q or λ′ = 4λ. or m∝
E
106. Tissue paper is capable of absorbing α-particles. Now, m1 e 2000 1
decide what contribution to the initial corrected count ∴ = × = .
m2 3000 2e 3
rate of 176 s–1 is made by α-particles. Then determine
what the contribution will be from the α-particles af- 112. For first line of Balmer series
ter 12 days. Now, consider the contribution to the
1 LM
1 1
=R 2 − 2
OP 1 LM
1 1 OP
initial corrected count rate made by β-particles. What
contribution will β-particles make after 12 days ? Then λ 2N 3 Q or
λ
=R −
N
4 9 Q
add the two contributions (of each component after 12
or
1
=RM
L 9 − 4 OP = 5R or λ =
36
days) to give the final answer. Use the fact the 12
days is a whole number of half-lives in both cases ; λ N 36 Q 36 5R
after one half-life, the contribution drops to 50% ; 1 LM
1 1 OP
after two half-lives, the contribution drops to 25% of
the initial value, etc.
For first line of Lyman series
λ′ 1N
=R 2 − 2
2 Q
N FG IJ = FG 1IJ
1
n 5
1 1
= R 1−
1LM OP
107.
N0
=
H K H 2K
2
=
32 λ′ 4 N Q
N 100 1 3R 4
× 100 = ≈ 3. = or λ′ =
N0 32 λ′ 4 3R
108. The wavelengths of the hydrogen spectrum could be λ′ 4 5R 20
Now, = × =
arranged in a series named after its discoverer. The λ 3R 36 108
ultraviolet was the Hyman series obtained when
energy of the atom falls to the lowest energy level 20
λ′ = × 6563 Å = 1215.4 Å.
E1 (– 13.6 eV) corresponding to n = 1. The visible 108
spectrum known as the Balmer series is obtained for 115. The first three transitions from the left fall in the
energy falls to the first excited state E2 (– 3.4 eV) Lyman series of the hydrogen spectrum which corre-
corresponding to n = 2. The infra red spectrum known sponds to ultraviolet radiation.
as the Paschen series is obtained for energy falls to The fourth transition falls in the Balmer series of the
the second excited state E3 (– 1.5 eV) corresponding to spectrum which corresponds to the visible light
n = 3. emission.
728 COMPREHENSIVE OBJECTIVE PHYSICS

The last transition falls in the Paschen series which 9


corresponds to the infra-red radiation.
9
Be + 42 He → 12
C + 01 n 4 Be (α, n) 12
6 C
4 6
The frequency of electromagnetic radiation is related 120. Ionisation potential for He electron
to the energy change (which is represented by the = 22 × 13.6 eV = 54.4 eV.
line between energy levels) by E = hf. Thus, frequen-
cies of the last two transitions are closer to each other FG 1IJ 32 / 2
FG 1IJ 16
= 24 ×
1 1
on the extreme left of the frequency spectrum whereas
121. 16 ×
H 2K = 16 ×
H 2K 2 16
=
212
< 1 mg.
the frequencies of the first three transitions are closer
122. By quantum theory of radiation, the energy change E
to one another and falls on the right corner of the
between energy levels is proportional to the frequency
frequency spectrum. of electromagnetic radiation f and is given by the equa-
The spectrum of the five transitions is thus best rep- tion below
resented in diagram (e).
hc
E = hf =
1 λ
116. En = – 3.4 eV, En ∝
n2 where h is the Planck’s constant
E1 = – 13.6 eV ; c is the speed of light
Clearly n = 2 λ is the wavelength of the radiation
Angular momentum Now, energy change by transition from E 2 to E1 is
given by
nh 2h h
== = = 2.11 × 10–34 J s. E = E2 – E1 ...(1)
2π 2π π
From (1) and (2), we have
117. It is probably easier to consider the last two state-
hc
ments before the first one since they both involve com- E 2 – E1 =
parison between an atom and its daughter product. λ
Can you make your choice by considering 2 and 3 hc
⇒ λ=
only ? E2 − E1
118. (a) Z was taken from X-ray scattering experiments
nh
(b) validity not known earlier ; established by Ruther- 123. L=

ford experiments
But, n2 ∝ r or n ∝ r½
2
Ze ∴ L∝ r.
(c) yes, the experiments said r < set upper limit
E 124. Alpha (α) particle and beta (β) particle are actually
for r
helium nucleus and electron respectively. They are
(d) lower limit of r not set.
emitted by nucleus and hence is evidence that radio-
119. The emission consists of neutrons. When Beryllium activity is a nuclear process.
is bombarded by α-particles, it emits neutrons of high
Line spectrum consists of discrete lines, each corre-
speed and the neutrons are capable of penetrating
sponding to a wavelength. The wavelengths are char-
many centimetres of lead. These neutrons are capa-
acteristic of the element emitting the light. When the
ble of causing protons of energy to be shot out from
continuous-spectrum light from a carbon arc is seen
materials such as paraffin wax which contain hydro-
through tube containing sodium vapour, an absorp-
gen.
tion spectrum is observed. A series of dark lines cor-
Chadwick was able to account for all the observed respond to the wavelengths absorbed by the sodium
effects as stated above when the emission was first atom is observed. Both spectra are direct evidence of
thought to be very energetic gamma rays which could discrete electronic energy levels in atoms.
not be so because if it were so, energy and momentum
Diffraction of a beam of electrons by a metal crystal
were not conserved in the collision producing it. The
(nickel) shows de-Broglie’s idea that electrons have
Beryllium radiation consists of uncharged particles
wave properties.
with a mass almost the same as proton’s and is called
125. Time = 2 × half-life.
a neutron FG 1nIJ by him and the action of α-particles on
H0 K So, half of half left, 2.5 mg.
beryllium can be represented by 126. 2 half-lives, hence half of half left.
ATOMIC NUCLEUS 729

A0 A0 136. The radioactive decay constant λ is given by


127. A= or A=
2
10
2.5 24 ln 2 0.693
λ= = hr–1
T1/2 32
A0 1.6
or A= = curie = 0.1 curie. From the equation N = N0e–λt, the fraction of a sample
16 16
remaining after 16 hours is given by
128. N0 N0 N0
N= = = N − d 0.693 i ( 16)
2 15 / 5
2 8 3 = e −λt = e 32 = e–0.3465 = 0.71.
N0
129. Disintegration of deuteron to a proton and a neutron
137. Barrier height
can be represented by
1 e
2
H + Q → 11 H + 01 n = (in eV)
1 4 πε 0 re
The energy captured is the γ-ray photon of energy Eγ .
9 × 109 × 1.6 × 10−19
By conservation of energy, the energy of the γ-ray = eV = 1.4 × 105 eV
photon Eγ is given by 10−14
Eγ + 1876 = 939 + 940 FG 1 − 1 IJ eV = 1.9 eV.
⇒ Eγ = (939 + 940) – 1876 = 3 MeV.
138. 13.6
H2 3 K
2 2

130. Decay constant, λ = 10–6 s–1. 13.6


139. eV = 3.4 eV.
The half-life T1/2 is thus given by 4
0.693 0.693 140. New mass number = (238 – 4) = 234
T1/2 = = = 0.693 ×106 s
λ 10 −6 New charge number
= 192.5 hrs. = 8 days = 1.14 week = 92 – 2 + 1 = 91.
≈ 1 week. 141. Given
X has activity A0 at t = 0 and its half-life is 24 years.
hc
131. ∆E = hf = because photon energy is quantised. Y has activity A0 at t = 0 and its half-life is 16 years.
λ
From the diagram, 1
At t = 24 years ⇒ activity of X = A
hc hc 2 0
∆E2 > ∆E1 ⇒ > i.e. λ1 > λ2.
λ2 λ1 1
At t = 48 years ⇒ activity of X = A
132. r–1 becomes 4-fold. So, potential energy 4-fold. Since 4 0
KE 1
ratio remains constant, KE also 4-fold. At t = 16 years ⇒ activity or Y = A
PE 2 0
2
133. vn = k 2πe 1
nh At t = 32 years ⇒ activity of Y = A
4 0
Take k=1
1
At t = 48 years ⇒ activity of Y =
A
2πe2 8 0
vn =
nh Thus total activity of the mixture of X and Y at t = 48
years is
2πe2
v1 =
h 1 1 3
A 0 + A 0 = A0 .
2 4 8 8
v1 2πe
= . 142. Mass of a α-particle = 4.0015 u
c hc
134. Momentum = mv where u is the unified atomic mass unit of 166 ×
10–27 kg
= 9.1 × 10–31 × 2.2 × 106 kg m s–1
Mass of an electron = 0.5371 mu
= 2 × 10–24 kg m s–1.
Mass of a proton = 1.0073 u
1 Charge on a α-particle = 2e where e is the basic charge
135. λ∝
Z2 unit of 16 × 1019 C
1216 Charge on an electron = – e
Now, λNa = ≈ 10 Å. Charge on a proton =+e
11 × 11
730 COMPREHENSIVE OBJECTIVE PHYSICS

Thus, magnitude of specific charge 4 π 2 k2 me4


e FG e IJ or fn =
n3 h3
for electron is
0.5371 mu
or 1862
H uK 2 2π 2 me4 LM 1 − 1 OP
Again, hν = k
e F eI
0.9928 G J
h2 N (n − 1) n Q
2 2
for proton is
1.0073 u
or
H uK 2 π 2me4 LM n − (n − 1) OP
2 2

F eI
2
or ν= k
for α-particle is 2e or 0.4998 G J .
H uK
h 3
MN n (n − 1) PQ
2 2

144.
4.0015 u
Let t = 0 , M0 = 10 g or ν = k2
2π2me4 LM (2n − 1) OP
F 1I
h3 N n (n − 1) Q
2 2

= 2G J
t=2τ
H λK [given] If n is very large, then
2π 2me4 2n
−λ 2
di ν = k2 ×
Then from, M = M0 e–λt = 10 e λ
h 3
n4

F 1I
= 10 G J
2
or ν=
4 π 2 k2 me4
= fn
H eK = 1.35 g n3 h3

145. Energy of first excited state of helium 4π 2 k2me4


149. ν=
n3 h3
32 × 13.6
=− eV = – 30.6 eV 1
22 ν∝ .
n3
Energy required to remove electron
= – (– 30.6) eV = 30.6 eV. 1
150. Radius of first orbit, r ∝ . For doubly ionised lithium
Z
232 − 208
146. Number of α-particles emitted = =6 Z (= 3) will be maximum, hence for doubly ionised
4 lithium r will be minimum.
Decrease in charge number due to α-emission = 12. 151. For third excited state, n = 4
But decrease in charge number = (90 – 82) i.e., 8.
Clearly, 4 β-particles are emitted. n2
rn = r0
2
147. E 12.09 × 1.6 × 10 −19 N s
= 4×4
c 3 × 108 or r4 = 0.5 × Å = 4 Å.
2
= 6.45 × 10–27 N s 152. Using law of conservation of momentum,
13.6 13.6 FG IJ h
= mv or λ =
h
Note. E=–
9
− −
1 H K λ mv
13.6 8 × 13.6 6.62 × 10−34 × 1010
= 13.6 − = eV = 12.09 eV. = Å = 991 Å ≈ 103 Å.
9 9 1.67 × 10−27 × 4
ke2 ke2 1
148. mr ω2 = 2
or ω2 = 3
153. V∝ , R ∝ n2
r mr n
ke 2
ke2 ∴ V R ∝ n.
or 4π2fn2 = 3 or fn2 = 2 3 154. N = N0 e–λt
mr 4π mr
1
N = N0 e − λ d λ i
1 n2 h2
But, r=
k 4π 2me2 1
N= N0
ke2 (k × 4 π 2me2 )3 e
∴ fn2 =
4 π 2 m (n 2 h 2 ) 3 N 1
4 8 2 2 2 2 4 2 =
k e (4 π ) m k e (4 π ) m N0 e
or fn2 = or fn =
(n2 h2 )3 n3h3
ATOMIC NUCLEUS 731
Decay percentage 162. R = R0 A1/3
FG1 IJ 1 FG IJ R = R0 (189)1/3
= 1−
He K
× 100 = 1 −
2.718
× 100
H K R
= R0 (A)1/3
= (1 – 0.368) × 100 3
= 0.632 × 100 = 63.2.
(189)1/3 189
155. Both the species follow exponential decay, irrespec- Dividing, 3 = or 27 =
( A)1/ 3 A
tive of their life time.
189
N0 or A= =7
156. N= 27
2t/T
N0 N 0.63 1
= 0 163. =
16 2t / T λX λY
t or λX = 0.693 λY or λX < λY
2t/T = 16 = 24 or =4
T Since λN is the decay rate therefore Y will decay faster.
t 24 164. 10Ne22 + hν = 2 2He4 + 6X14
or T= = h=6h 14 is an isotope of carbon 14.
4 4 6X
157. N = N0 e–λt 1 1 3

N 165. = 3
or 22 = 2 4T
= N0 e −10 λt 4
e 24T

N e − λt 3
= − 10λt e10 λt or 8T = 3 or T =
s.
N e or e = 8
e eλt 166. Mass of nucleus is less than the sum of the masses of
or e = e9λt or 9λt = 1 the constituent particles.
1 1 1
or t = 167. = t
9λ 16
2 100
ke2 or
1 1
= t / 100 or 4 =
t
158. Ek = 4
2r 2 2 100
or t = 400 µs.
ke2
Ep = − 168. In γ-decay, neither A nor Z changes.
2r
169. Note that infrared radiation has lesser energy than
As the hydrogen atom makes transition to the ground
ultraviolet radiation.
state, r decreases.
Ek increases, Ep and E decrease. ke2
170. Ep = −
159. T2 ∝ r3 and r ∝ n2 r
∴ T2 ∝ n6 or T ∝ n3
ke2
T1 FG IJ
n
= 1
3 E=–
2r
T2 H K
n2 Ep = 2E
Ep = 2(– 13.6) eV = – 27.2 eV.
Fn I
8=G J 1
3
n1 Potential energy of electron in the ground state of Li2+
Hn K 2
or
n2
=2
ion is – 32 × 27.2 eV or – 244.8 eV.
160. M = IA = efπr 2 171. R = R0 A1/3
= 1.6 × 10–19 × 1016 × 3.14 × (0.5 × 10–10)2 A m2 R = 1.2 × 10–15 (64)1/3 m
= 1.256 × 10–23 A m2 R = 4.8 × 10–15 m.
172. Binding energy
n(n − 1) 4(4 − 1)
161. = =6 = [4 × 1.008 + 5 × 1.009 – 9.012] × 931
2 2
= [4.032 + 5.045 – 9.012] 931 MeV
= 0.065 × 931 MeV.
732 COMPREHENSIVE OBJECTIVE PHYSICS

0.065 × 931 1 c
B.E./nucleon = MeV v = 4× ×
9 137 2
= 6.724 MeV.
2c
173. Required energy v=
137
LMFG − 13.6 IJ − FG − 13.6 IJ OP × 9 13.6
=
NH 9 K H 1 K Q 180. E=–
52
eV

L
= M13.6 −
13.6 O E = – 0.544 eV
9 PQ
9 = 8 × 13.6 eV
N Ep = – 2 × 0.544 eV = – 1.088 eV.
12375 181. v = rω
Wavelength = Å = 113.7 Å v
8 × 13.6 v = r × 2πν or ν = .
2πr
mv2 182. 13.6 – 0.85 = 12.75 eV
174. F=
r So, 12.75 V is the required potential difference.
1
But v∝ and r ∝ n2
183. 0.1 1 × 3 × 108 × 3 × 108 kWh
n E= ×
100 103 × 3600
1
F∝ . = 2.5 × 107 kWh.
n4
184. The minimum energy to ionise an atom is the energy
2.42 − 1.02 required to excite an electron in the atom from its
175. = 0.7 MeV
2 ground state corresponding to n = 1, to its ionisation
1 level corresponding to n = ∞. If the electron is given
176. N∝ an amount of energy equal to the ionisation energy,
θ
sin4 the electron is just able to become a ‘free’ electron and
2
the atom is about to be ionised. When this occurs, one
θ 60° outermost electron from the atom is just about to be
sin4 1 sin 4
N2 2 N2 2 removed from the atom. Much higher energy is nec-
= or =
N1 sin 4 θ 2 5 × 106 sin 4 120° essary to free an innermost electron from the atom
2 2 because this will occur only after the outer electrons
are all removed.
or N2 sin 4 30°
6
= 185. The α-particle emitting radioactive gas, Thoron – 220,
5 × 10 sin 4 60°
decays to Radium – 216 and emits and α-particle. The
FG 1IJ FG 2 IJ
4 4
=
5
× 106 .
reaction can be represented by
or N2 = 5 × 106 ×
H 2K H 3 K 9 220
90
Th → 4
2
He + 216
66
Ra
177. ∆E = 2mpc2 By conservation of momentum, we have momentum
= 2 × 931 MeV = 1862 MeV of α-particle = momentum of recoiling nucleus Ra
= 1.862 × 109 eV = 1.862 GeV. vR ma 4 1
⇒ mava = mRvR ⇒ = = =
ke 2 va mR 216 54
178. (i) Ek = The kinetic energy of Ra, ER is related to the kinetic
2r
energy of alpha particle Ea by the equation below
ke2
E=– 1
2r ER
mR vR2
mR F I F v I =Fm I F m I
2 2

∴ Ek = – (– E) Ea
= 2
1
mava 2
=
ma GH JK GH m JK GH m JK GH m JK
R
a
R
a
a
R
Now, kinetic energy in fourth orbit of lithium atom 2
13.6 ma 1
= 9× eV = 7.65 eV.
16 = =
mR 54
LM 1 c OP Ea
179. v=Z
N 137 n Q ∴ ER =
54
.
ATOMIC NUCLEUS 733
will be shifted by 27.2 eV. So, the total energy in the
FG IJ
t
9 1 T first excited state will be (– 3.4 + 27.2) eV or 23.8 eV.
186.
16
=
2H K 195. By the law of conservation of mass, the total mass
N F 1I
t number is constant before and after the reaction. In
=G J
2T
N 0 H 2K other words, the total number of active atoms decay-
ing from nuclide X must be equal to the number of
F N I = FG 1IJ
2 t/ T
F NI 2 atoms in nuclide Y. That is
GH N JK H 2 K
0
or GH N JK
0
=
9
16
NY + NX = N
where N is the initial number of active atoms in
N 3 nuclide X,
or =
N0 4 N X is the number of active atoms that remain
in nuclide X
 Note the special technique used in solving and N Y is the number of atoms in nuclide Y.
a typical problem. At time T, the number of atoms in Y is equal to the
number of atoms remaining in nuclide X.
187. T = 0.693 τ 1
∴ NY = NX = N. But NX = Ne–λt
T 2
= 0.693
τ where λ is the decay constant of nuclide X and is given
Percentage is 69.3. ln 2
by .
A 2 N2 τ
188. =
A 1 N1 At t = T,
A2 1 1000 1
= or A2 = = 500 s–1 NY = NX = N = Ne–λT
3 2 2
10 2
1 LM
1 1
=R 2 − 2
OP ⇒ eλT = 2
189.
λ 1N 2 Q ⇒ λT = ln 2 or T =
ln 2

or
1 L 1O
= R M1 – P or
1 3R
λ
λ N 4Q λ
=
4
196. T1/2 =
0.693
year
3R 4.28 × 10 −4
wave number, ν =
4 = 1619.15 year ≈ 1619 year.
190. N = ∑2n2 N0
197. N = N0 e–λt or eλt =
N = 2 (12 + 22 + 32 + 42) N
= 2 (1 + 4 + 9 + 16) = 2 × 30 = 60.
N0 1 N
20 1 1 1 1 1 or λt = loge or t= log e 0
191. = or = or = N λ N
80 2 t/20 4 2t/ 20 22 2t/ 20
τα τβ 4
t or t = log e
or =2 or t = 40 minute. τ α + τβ 1
20
192. Applying conservation of momentum and conservation τ α τβ
of energy, or t = 2.303 × log 10 4
τ α + τβ
KEα = 5.4 MeV.
h τα τβ
193. L = 2× or t = 2 × 2.303 log 10 2
2π τα + τβ
h
L=. 2 × 2.303 × 0.3010 × 1620 × 405
π or t = years
1620 + 405
194. Potential energy of hydrogen atom in the ground state
is ‘– 27.2 eV’. If the potential energy in the ground = 449.2 years.
state is taken as zero, then the whole energy spectrum
734 COMPREHENSIVE OBJECTIVE PHYSICS

198. N = N0 e–λt The radius of circular orbit r of an β-particle in the


N0 presence of the magnetic field B is given by
N0
= e λt or λt = loge
N N mv
r=
1 N T N qB
or t = log e 0 or t = log e 0
λ N 0.693 N where m is the mass of the β-particle,
5700 4 q is the charge on the β-particle
= × 2.303 log 10
0.693 1 and v is the speed of the β-particle.
2 × 5700 × 2.303 The smaller the speed v, the smaller will be the ra-
= × log 10 2
0.693 dius of the circular orbit (r ∝ v). After the β-particles
2 × 5700 × 2.303 × 0.3010 pass through the aluminium sheet, its speed v is re-
= years duced and the radius of the orbit is also reduced.
0.693
= 11403 years. 202. Two α-particles reduce mass number by 8.
N 1 ∴ New mass number = 180 – 8 = 172
199. = 5T /T
N0 2 Emission of two α-particles reduces charge number
by 4. Emission of β particles increases charge number
N 1
= by 1.
N0 25
∴ New charge number = 72 – 4 + 1 = 69.
N 100 203. I = I0 e–µx
× 100 = = 3.125.
N0 32
I I0
or = e −µx or eµx =
200.
1
= Z2 R
LM1 1
− 2
OP I0 I
λ MN
n12
n2 PQ I0 1 I
log e 0
or µx = loge or x =
ν
=Z RM
L 1 − 1 OP
2
I µ I
c MN n n PQ
1
2
2
2
or x ∝ loge
I0

ν = cZ R M 2
L 1 − 1 OP I

MN n n PQ
1
2
2
2 Now, 48 ∝ loge 8
d ∝ loge 2

2.7 × 10 = cZ R M −
L 1 1 OP d log e 2
N1 2 Q
15 2
2 2 Dividing, =
48 log e 23

ν = cZ R M −2 L 1 1 OP 48
N1 3 Q
2 2 or d
=
48 3
1 or d =
3
mm = 16 mm.

1 13.6 13.6
1− 204. En = – or n2 = –
ν 9 = 8 × 4 = 32
= n2 − 0.54
2.7 × 1015 1 − 1 9 3 27 or n2 = 25.2 or n = 5 (nearly)
4
v
32 ∴ vn = .
or ν= × 2.7 × 1015 Hz = 3.2 × 1015 Hz. 5
27
201. γ-rays are electromagnetic waves of which no 205. N = N0 e–λt
deflection is observed when a beam of γ-rays is allowed N
to pass through a magnetic field. Since the presence = e− λt
N0
of magnetic field deflects the path of the radiation,
particularly near X, the radiation must either be α- N0
particles or β-particles. = eλ t
N
α-particles can be cut out by the thin aluminium sheet
about 1 mm thick which is not the case here. Hence, N0
λt = loge
we conclude that the radiation must be β-particles. N
ATOMIC NUCLEUS 735

1 N 1
t = log e 0 ⇒ ln Nt – ln 20 = – t [ln 20 = 3.00]
λ N 20
N0 Nt
t ∝ loge ⇒ ln = − 0.05 t
N 20
100 Hence, Nt = 20 e–0.05t.
5 ∝ loge
90 208. Binding energy/nucleon
N0 0.0303 × 931
20 ∝ loge = MeV = 7.05 MeV.
N 4
Dividing, A 1
209. =
A0 2t/ T
100
log 10 1 1 1 1
5 90 = 30 / T
= or =
20 N 64 2 26 230 / T
log 10 0
N 30 30
or =6 or T = s = 5 s.
N0 10 N0 FG IJ
10
4 T 6
or log 10
N
= 4 log 10
9
or
N
=
H K
9 210. Since λ ∝ 2 ,
Z
1

N 9FG IJ 4 1 1 1
∴ λ1 : λ2 : λ3 :: 2 : 2 : 2
or
N0
=
10 H K = 0.6561
1 2 4
1 1 1
Percentage of substance that decays or : : or 16 : 4 : 1.
1 4 16
= (1 – 0.6561) × 100 = 34.39.
211. Number of particles ∝ volume ∝ R3
206. Let the kinetic energy of the α-particle be Eα and that
of the thorium Th be Eth. N′ 3R FG IJ 3

The ratio of kinetic energies is N


=
R H K
= 27

1 N′ = 27 N.

mαvα 2
mα vα F IF I
2
212. R = R0 A1/3
= 2 = GH JK GH JK
...(1)
Eth 1
mthvth2 mth vth Rsulphur FG 32 IJ 1/3
2
By conservation of momentum the momentum of the Rhelium
=
H 4K
= (8)1/3 = 2.

α-particle and that of the recoiling thorium Th must 213. The rate at which the atoms decays from the radioac-
be equal. Thus tive gas is proportional to the number of atoms present
να m in the gas by the equation
mανα = mthνth or = th ...(2)
dN
νth mα = − λN
Subst. (2) into (1), we have dt
where λ is the radioactive decay constant
Eα mα F IFm I 2
mth 234
Eth
=
mth GH JK GH m JK th
α
=

=
4
= 58.5 From which we obtain
N = N0e–λt
Thus, the kinetic energy of the α-particle expressed and hence ln N = – λt loge N0
as the fraction of the total kinetic energy T is given where N0 is the initial number of atoms in the gas at
by t = 0.
58.5 58.5 It is a straight line of negative gradient with magni-
Eα = T= T = 0.98 T tude λ when ln N is plotted against t.
1 + 58.5 59.5
Thus, from t = 0 to t = T, the logarithmic of number
which is slightly less than T.
of atoms ln N decreases uniformly at the rate of λ for
207. The graph can be represented by the linear equation the time period from t = 0 to t = T. When more of the
as follows : same gas is injected into the vessel, the number of
ln N t − 3 0−3 atoms present increases instantaneously and the de-
=
t−0 60 − 0 cay of atoms takes place at the same rate λ which is
best represented in graph (b).
736 COMPREHENSIVE OBJECTIVE PHYSICS

214. Mass defect, ∆m −λt


⇒ 2 = e 1/ 2 ⇒ ln 2 = λt1/2
= (2 × 1.0073 + 2 × 1.0087) – 4.0015
ln 2
= (2.0146 + 2.0174 – 4.0015) i.e., λ= .
t1/2
= (4.0320 – 4.0015) amu
220. From conservation of momentum
= 0.0305 amu
Binding energy m1 v2 1
= =
= 0.0305 × 931 MeV = 28.3955 MeV. m2 v1 2
1 N0 m1 r13
215. N1 = N0 = As ∝
26 / 1 26 m2 r23
1 N0
Again, N2 = N0 = 1
26 / 2 23 ∴ Ratio of nuclear sizes = 1/3 .
3
2
Dividing, N 1 = 2 = 1 1 1 1
N 2 26 8 221. 1 × 104 = 8 × 104 t/ 3 or =
2 8 2t / 3
0.693 N
Now, A = λN = 1 1 t
T or = t/ 3 or 3=
F I FT I 2 3
2 3
∴ A 1 N 1 T2
A2
= ×
T1 N 2
=
N1
N2
GH JK GH T JK2
1
or t = 9 years.

1 2 1 1 2 Ze 2
=
× = . 222. r0 =
8 1 4 4 πε0 E k

216.
1 1
= R 1− 2
LM OP 9 × 109 × 2 × 92 (1.6 × 10−19 )2
λ n N Q =
5 × 106 × 1.6 × 10−19
m
1 1 1 1
or = 1− 2 or = 1− = 5.29 × 10–14 m.
λR n2
n λR
1 1 LM
1 OP 1 R
or
1
2
=
λR − 1
or n2 =
λR
λR − 1
223.
λ
=R 2 −
3 ∞N Q or =
λ 9
n λR
9 9
F λR IJ
n= G
1/ 2 or λ=
R
=
1.1 × 107
m
or
H λ R − 1K .

M 9 × 1010 9000
= Å = Å = 8181.8 Å
217. Density, ρ = 1.1 × 107 1.1
V
16 × 1.66 × 10 −27 ≈ 8182 Å.
= kg m −3 224. A ∝ r2
4
3
× 3.14 × 3 × 10 e j
−15 3
Also r ∝ n2
= 2.35 × 1017 kg m–3. ∴ A ∝ n4
1
218. En ∝ , Jn ∝ n A 1 24
n2 = = 16 .
A 2 14
1
∴ En ∝ . R
J n2 225. R = R0 A1/3 or = A 1/3
219. The activity of a radioactive isotope is given by R0
N = N0 e–λt R 1
log = loge A
where N0 = initial activity eR 0 3
λ = decay constant Comparing with y = mx, we find that the graph is a
When the activity is decayed to half of its initial value straight line.
of half-life t1/2. 226. At closest approach, all the kinetic energy of the α-
1 − λt particle will be converted into the potential energy of
N0 = N0 e 1 / 2
2 the system,
ATOMIC NUCLEUS 737

∴ 5MeV =
1 q1q2
229. 1 LM
1 1 OP
4 πε 0 r λ 4N
=R 2 − 2
5 Q
Z1 × Z2 e2 1
=RM
L 1 − 1 OP
5 × 106 × e = 9 × 109 ×
r λ N 16 25 Q
or r=
9 × 109 × 92 × 2 × 1.6 × 10−19 1
=RM
L 9 OP 16 × 25
5 × 106 λ N 16 × 25 Q or λ =
9R
or r = 5.3 × 10–14 m = 5.3 × 10–12 cm.
16 × 25 × 1010
dN or λ= Å = 40.4 × 103 Å.
227. From the equations = − λN and N = N0 e–λt , we 9 × 1.1 × 107
dt
may state that
230.
1 1 1
= 1.1 × 107 2 − 2
LM OP
(a) λδt gives the fraction of atoms present which will
decay in the next small time interval δt. This is
975 × 10 −10 1 n N Q
1 1
dN or 1− =
evident from the equation = – λN since for n2 975 × 1.1 × 10−3
N
small time interval δ t.
1 103
or = 1−
δN δN n2
975 × 1.1
= − λN ⇒ – = λδt
dt N
1 1
(b) t = is the time needed for N to fall from N0 to = 1 − 0.9324
λ n2
N
the value 0 . 1 1
e = 0.067599 or n2 = = 14.8
1 n2 0.067599
This is obtained by substituting t = into or n = 3.846 ≈ 4.
λ
–λt
N = N0e which gives E1 FG
E IJ E1 E1 3E1
N0
231. E4 – E2 = −
16 H
− − 1
4 K =
4

16
=
16
N = N0e–λ(1/λ) = N0 e−1 =
e 3E1 16E
or E= or E1 = .
(c) the half-life of the sample is given by 16 3
0.693 N
T1/2 = 232. = e − λt
λ N0
(d) the number of atoms left after a time equal to e
N0
seconds is given by = eλ t
N
N = N0e–λt ≠ λ
N0
(e) λ gives the fraction of atoms that has decayed af- loge = λt
N
δN
ter one second from the equation – = λδt for 1 1 N N0
N or t = log e 0 or t ∝ log e
δN λ N N
second λ = – .
N N0
A or t ∝ log 10
228. X → YA–4 + α N
(Parent nucleus) (Daughter nucleus)
8
As the parent nucleus is at rest, then according to 6 ∝ log10
1
conservation of momentum
0 = mYvY + mαvα N0
10 ∝ log10
or 0 = (A – 4)vY + 4v N

4v N0
or vY = – log 10
A−4 Dividing, N = 10
8 6
The negative sign indicates the recoil speed of daugh- log 10
1
ter nucleus.
738 COMPREHENSIVE OBJECTIVE PHYSICS

or N0 5 1 24 25
log 10 = log 10 23 = R or λ =
N 3 λ 25 24 R
N0 25 × 1010 25000
or log10 = log10 25
N or λ= Å = Å
24 × 1.1 × 10 7 24 × 1.1
N0 N 1 = 946.96 Å ≈ 947 Å.
or = 25 = 32 or =
N N 0 32 237. Energy of photon = E3 – E2
Fraction that decays = 1 −
1
=
31
. LM − 13.6 + 13.6 OP eV = 13.6 LM − 1 + 1 OP eV
13.6Z2
32 32 =
N 3 2
2 Q
2
N 9 4Q
233. E=− eV 5
n2 = 13.6 × eV = 1.9 eV.
36
13.6 × 2 × 2 4 × 12 48
E=− eV = – 13.6 eV. 238.
4 T= = =3
4 + 12 16
234. Binding energy per nucleon for 21 H is 1.1 MeV N 1 1 1
= = =
∴ Binding energy = 2.2 MeV N0 212 / 3 24 16
Binding energy per nucleon for 42 He is 7 MeV N 100
× 100 = = 6.25 .
∴ Binding energy = 28 MeV N0 16
Energy released = [28 – 2 × 2.2] MeV 239. The half-life of the given radioactive sample is 140
days. In 420 days, there will be three half-lives.
= 23.6 MeV.
Therefore, (d) is correct.
FG 1IJ n
N FG IJ
1
n
235. N = N0
H 2K or
N0
=
2H K 240. eV =
hc
λ
or V =
hc
λe
25 1FG IJ n
6.63 × 10 −34 × 3 × 108
100
=
2 H K or n = 2 or V=
0.2 × 10 −9 × 1.6 × 10 −19
volt » 6.2 kilo volt.

∴ Total time in which radium change to 25 g 241. From law of conservation of linear momentum
= 2 × 1600 years = 3200 years. v1 m2
=
n(n − 1) v2 m1
236. = 10
2 4
ρ × πr23 r3 8
On simplification, n = 5 v1 3
or = = 23 = .
Now,
1 LM
= R 1−
1 OP v2 ρ × 4 πr 3 r1 1
N Q
1
λ 25 3

KNOWLEDGE PLUS
l Energy levels A, B and C of a certain atom correspond to increasing values of Energy i.e. EA < EB < EC . If λ1 , λ2
and λ3 are wave lengths of radiations corresponding to transitions C to B, B to A and C to A respectively, which
of the following relations is correct ?
λ1λ 2
(a) λ3 = (b) λ3 = λ1 + λ2 (c) λ1 + λ2 + λ3 = 0 (d) λ32 = λ12 + λ22 [All India PM/PD 2005]
λ1 + λ 2
Sol. EC – EA = (EC – EB) + (EB – EA) ...(C)
hC hC hC
= + ...(B)
λ3 λ1 λ2
1 1 1
or = + ...(A)
λ3 λ1 λ2
1 λ + λ1 λ 1λ 2
or = 2 or λ3 =
λ3 λ 1λ 2 λ1 + λ2
So, (a) is the right choice.
ATOMIC NUCLEUS 739

SET III MCQs


with
More than one correct alternative

Average time allowed per question is 50 seconds.

242. When an electron in the hydrogen atom moves from (b) the thick concrete shield is used to slow down the speed of
the second orbit to the third orbit, fast-neutrons
(a) both kinetic and potential energy decrease (c) heavy water (or graphite) moderates the activity of the
reactor.
(b) P.E. increases and K.E. decreases
(d) out of U238 and U235 natural uranium has less than 1% of
3
(c) the angular velocity increases by a factor U 235.
2
[National Standard Exam. in Physics 1992]
27
(d) the period of revolution increases by a factor . 247. Let An be the area enclosed by the nth orbit in a hy-
8
[National Standard Exam. in Physics 1997] drogen atom. The graph of ln (An/A1) against ln (n)
243. When a photon stimulates the emission of another (a) will pass through the origin
photon, the two photons have (b) will be a straight line with slope 4
(a) same energy (b) same direction (c) will be a monotonically increasing non-linear curve
(c) same phase (d) same wavelength. (d) will be a circle.
244. A nitrogen nucleus 7N14 absorbs a neutron and can 248. 235
92 U is ‘α’(alpha) active. Then in a large quantity of
transform into lithium nucleus 3Li7 under suitable
the element
conditions, after emitting
(a) the probability of a nucleus disintegrating during one sec-
(a) 4 protons and 3 neutrons
ond is lower in the first half-life and greater in the fifth
(b) 5 protons and 1 negative beta particle half-life
(c) 2 alpha particles and 2 gamma particles (b) the probability of a nucleus disintegrating during one sec-
(d) 1 alpha particle, 4 protons and 2 negative beta particles ond remains constant for all time
(e) 4 protons and 4 neutrons. (c) every nucleus must integrate by the lapse of the average
245. It is observed that only 0.39% of the original radio- life of the population
active sample remains undecayed after eight hours. (d) quite an appreciable quantity of U235 will remain even
Hence after the average life
(a) the half-life that substance is 1 hour (e) the energy of the emitted ‘α’ particle is less than the disin-
(b) the mean life of the substance is 1/log 2 hour tegration energy of the U235 nucleus.
(c) decay constant of the substance is (log 2)/hour 249. According to Bohr’s theory of hydrogen atom, for
(d) if the number of radio active nuclei of this substance at a the electron in the nth permissible orbit
given instant is 108 then the number left after 30 min. 1
(a) linear momentum ∝
n
would be 2 × 10 9 .
(b) radius of orbit ∝ n
[National Standard Exam. in Physics 2002] 1
(c) kinetic energy ∝
246. In a Nuclear reactor n2
(a) the chain reaction is kept under control by rods of cad- (d) angular momentum ∝ n.
mium, whose going in reduces the rate [National Standard Exam. in Physics 2000]
740 COMPREHENSIVE OBJECTIVE PHYSICS

250. Which of the following products in a hydrogen atom 251. In case of radioactive radiations
are independent of the principal quantum number (a) some are not deviated by electric and magnetic fields
n? The symbols have their usual meanings (b) some carry negative charge
(a) vn (b) Er (c) all are electromagnetic waves
(c) En (d) vr. (d) all produce X-rays when suddenly stopped.
[National Standard Exam. in Physics 1999]

Answers (Set III)


242. (b), (d) 243. (a), (b), (c), (d) 244. (c), (d), (e) 245. (a), (b), (c) 246. (a), (d) 247. (a), (b) 248. (b), (d), (e) 249. (a), (c), (d)
250. (a), (b) 251. (a), (b).

Solutions (Set III)


242. Total energy should increase ; Kinetic energy decreases 249. Radius r ∝ n2
and potential energy increases. According to Kepler’s nh
law, T2 α r3 ; Also r α n2. This gives ratio of periods. (b) is not true mvr = and radius r ∝ n2

N
245. Use = e− λt . ∴ Linear momentum is proportional to
1
N0 n
246. (a) true, Cd absorbs neutrons (a) is true .
(b) no, concrete reflects, does not slow down 1
1
(c) ‘‘moderates the activity’’ is not correct. ‘‘moderator’’ En ∝ and kinetic energy = E
n2 2 n
in the sense of slowing the neutrons is different (c) is true
(d) true fact. (d) is Bohr’s postulate.
247. A = πrn2

KNOWLEDGE PLUS
l A radioactive material has a half-life of one hour. Then the fraction of the material that remains without disinte-
gration at the end of 4 hours is :
1 1 1 1
(a) 1 (b) (c) (d) (e)
2 4 8 16
[Kerala PMT 2003]
Solution. Number of half-lives, n = 4
N FG IJ = FG 1IJ
1
n 4
1
N0
=
H K H 2K
2
=
16
So, (e) is the right choice.
ATOMIC NUCLEUS 741

SET IV MCQs
based on
TYPICAL NUMERICAL BANK
(Exclusively for Engineering Entrance Tests)

Average time allowed per question is 60 seconds.

252. In hydrogen atom, the transition takes place from n = 3 259. The radius of hydrogen atom in the ground state is
to n = 2. If Rydberg’s constant is 1.09 × 107 per metre, 5.3 × 10 –11 m. When struck by an electron, its radius is
the wavelength of the light emitted is found to be 21.2 × 10 –11 m. The principal quantum
(a) 6606 Å (b) 4861 Å number of the final state will be
(c) 4340 Å (d) 4101 Å. (a) 1 (b) 2
253. An alpha particle of energy 5 MeV is scattered through (c) 3 (d) 4. [UGET 1999]
180° by a fixed uranium nucleus. The distance of the
260. The wavelength of the first line of Balmer series is 6563
closest approach is of the order of
Å. The Rydberg’s constant is
(a) 10 –15 cm (b) 10 –13 cm
(a) 1.09 × 105 per m (b) 1.09 × 106 per m
(c) 10 –12 cm (d) 10 –19 cm.
(c) 1.097 × 107 per m (d) 1.09 × 108 per m.
[JIPMER 1996]
261. A Hydrogen atom and a Li++ ion are both in the second
254. Certain radio-active substance reduces to 25% of its
excited state. If lH and lLi are their respective electronic
value in 16 days. Its half-life is
angular momenta, and EH and ELi their respective en-
(a) 32 days (b) 8 days
ergies, then
(c) 64 days (d) 28 days. [WB JEE 1997]
(a) lH > lLi and |EH| > |ELi |
255. The wavelength of radiation required to excite an elec-
(b) lH = lLi and |EH| < |ELi|
tron from first to third Bohr orbit in a doubly ionised
lithium atom will be (c) lH = lLi and |EH| > |ELi|
(a) 113.74 m (b) 113.74 cm (d) lH < lLi and |EH| < |ELi|. [IIT Screening 2002]
(c) 113.73 Å (d) 113.74 mm. [MNR 2000] 262. Imagine an atom made up of a proton and a hypotheti-
256. What is the age of an ancient wooden piece if it is known cal particle of double the mass of the electron but hav-
that the specific activity of C14 nuclide in it amounts to ing the same charge as the electron. Apply the Bohr atom
3 model and consider all possible transitions of this hy-
of that in freshly felled trees ? Given : the half-life of pothetical particle to the first excited level. The longest
5
14
C nuclide is 5570 years. wavelength photon that will be emitted has wavelength
(a) 1000 years (b) 2000 years λ (given in terms of the Rydberg constant R for the hy-
(c) 3000 years (d) 4000 years. drogen atom) equal to
257. A helium atom, a hydrogen atom and a neutron have 9 36
(a) (b)
masses of 4.003 u, 1.008 u and 1.009 u (unified atomic 5R 5R
mass units), respectively. Assuming that hydrogen at- 18 4
(c) (d) .
oms and neutrons can fuse to form helium, what is the 5R R
binding energy of a helium nucleus ? [IIT Screening 2000]
(a) 2.01 u (b) 2.014 u 263. The electric potential between a proton and an electron
(c) 1.017 u (d) 0.031 u r
is given by V = V0 ln r , where r0 is a constant. Assum-
(e) 0.0031 u. [London Schools Examination 0
Department] ing Bohr’s model to be applicable, write variation of rn
258. How many revolutions does an electron complete in one with n, n being the principal quantum number.
second in the first orbit of hydrogen atom ? (a) rn ∝ n (b) rn ∝ 1/n
(a) 6.67 × 10 15
(b) 100 (c) rn ∝ n2 (d) rn ∝ 1/n2 .
(c) 1000 (d) 1. [UGET 1998] [IIT Screening 2003]
742 COMPREHENSIVE OBJECTIVE PHYSICS

Answers (Set IV)


252. (a) 253. (c) 254. (b) 255. (c) 256. (d) 257. (d) 258. (a) 259. (b)
260. (c) 261. (b) 262. (c) 263. (a).

Solutions (Set IV)


252.
1 1 1
= 1.09 × 107 2 − 2
LM OP 257. Refer to the definition of mass defect. What is the struc-
λ 2 3 N Q ture of a helium nucleus in terms of numbers of neu-
trons and protons ? Remember that a hydrogen nu-
1
= 1.09 × 107
1 1

LM OP cleus is a single proton.
λ 4 9 N Q 258. mvr =
nh
1 5 2π
= 1.09 × 107 × m
λ 36 nh
mr2ω =
36 × 1010 2π
or λ= Å = 6606 Å.
5 × 1.09 × 107 nh nh
ω= 2 or 2πν =
1 (Ze) (e) 2 πmr 2 πmr 2
253. Use E =
4 πε 0 r0 nh
or ν=
1 1 4 π 2mr 2
254. = 16
4 nh
or ν=
16
2T
4 π 2m
LM
1 n2h2
2
OP
2T
= or
16
22 =2 or T = 8 days. MN
k 4π 2 me2 PQ
T
k2 (4 π 2 me2 )2 nh
255. For the case of hydrogen, or ν=
4 π 2 m (n2 h2 )2
13.6
E=– − ( − 13.6) 4 π 2mk2 e4
9 or ν=
FG 13.6 IJ n3h3
E = 13.6 −
H 9 K
eV = (13.6 – 1.51) eV For first orbit, n = 1
= 12.09 eV 4 π 2mk2 e4
∴ ν=
For lithium, ELi h3
= 9 × 12.09 eV = 108.81 eV 4(3.14)2 9.1 × 10 −31 (9 × 109 )2 (1.6 × 10−19 )4
=
Now, λ=
12375
Å = 113.73 Å (6.6 × 10−34 )23
108.81 = 6.67 × 10 rev s–1
15

3 259. rn ∝ n2
256. N = N0 e–λt
5 0 n′ 2 21.2 × 10 − 11 n′ 2
5 = or =4
eλt = n2 5.3 × 10 − 11 n2
3
n′ 2
5 5 or =4 or n′ = 2.
loge eλt = log e or λt = log e 12
3 3
1 5 1 1LM 1 OP
or t = log e
λ 3
260.
λ 2 N
=R 2 − 2
3 Q
T 0.693 1 L 1 1O
= R M − P or
1 5
=
0.693
× 0.5 T=
λ
or
λ N4 9Q λ
=R
36
5570 × 0.5 36 36
= years = 4018.7 years or R= = m–1
0.693 5λ 5 × 6563 × 10−10
≈ 4000 years.
ATOMIC NUCLEUS 743

36000 1 9−4 1 5R
= × 107 m–1 = 1.097 × 107 m–1. or =R or =
5 × 6563 λ 36 λ 36
h 36
261. L=n or λ=
2π 5R
L same. 1
Again, λ∝
2
Z × 13.6 m
(En)Li = –
n2 Mass of hypothetical hydrogen atom is double.
13.6 1 36 18
(En)H = – 2 ∴ λ′ = × =
n 2 5R 5R
Clearly | EH | < | ELi |. − du mv2
1 1 F1 I 263. |F| =
dr
=
r
262.
λ
=R −GH
n12 n22
JK v0 r
⇒ v= which is a constant
For longest wavelength, m
n1 = 2, n2 = 3 nh
mvnrn =
1 1 LM
1 OP 1 1 1LM OP 2π
λ
=R 2 − 2
2 3 N Q or
λ
=R
4

N
9 Q rn ∝ n .

KNOWLEDGE PLUS
In each question, a statement of assertion (A) is given and a corresponding statement of reason (R) is given just
below it. Of the statements, mark the correct answer as :
(A) If both A and R are true and R is the correct explanation of A.
(B) If both A and R are true but R is not the correct explanation of A.
(C) If A is true but R is false.
(D) If both A and R are false.
(E) If A is false but R is true.
1. Assertion (A). Outside the nucleus, neutron is an unstable particle. Its half-life is nearly 12 minutes.
Reason (R). Outside the nucleus, deuterons are not available.
2. Assertion (A). A neutron and a proton kept together can form a deuteron only if their spins are parallel to each
other.
Reason (R). Nuclear forces are spin-dependent.
3. Assertion (A). The positive value of packing fraction implies a large value of binding energy.
Reason (R). The divergence of the nuclear mass from the nearest whole number per nucleon is called packing
fraction.
Key 1. (C) 2. (A) 3. (E)

l The atomic number (Z) of an element whose kα wavelength is λ is 11. The atomic number of an element whose kα
wavelength is 4λ is equal to
(a) 6 (b) 11 (c) 44 (d) 4 [IIT Screening 2005]
1
Sol. For kα , ν ∝ (z – 1) or ∝ (z – 1)
λ
or λ∝ 1 or (z – 1)2 λ = constant
2
( z − 1)
(z – 1)2 (4λ) = (11 – 1)2 λ or z–1=5 or z = 6
So, (a) is the right choice.
744 COMPREHENSIVE OBJECTIVE PHYSICS

SELF-EVALUATION TEST I
Based on Unit XVII
[Expected Questions for Forthcoming Examinations]

1. A particle moving with a velocity 1/10th of that of light


will cross a nucleus in about
(a) 10–47 s (b) 10–21 s N D R
(c) 10–12 s (d) 10–8 s. N

2. Which of the following is never true for cathode rays ?


(a) They possess kinetic energy. t t t
(1) (2) (3)
(b) They are electromagnetic waves.
Fig. 32
(c) They cause fluorescence.
(d) They produce X rays on striking matter. [National Standard Exam. in Physics 1989]
[National Standard Exam. in Physics 1989] 8. A high energy α-particle collides with a 14 N nucleus to
7
3. A hydrogen atom in ground state absorbs 10.2 eV of
energy. The orbital angular momentum of the electron produce a 17 O
nucleus.
8
is increased by
(a) 1.05 × 10–34 Js (b) 2.11 × 10–34 Js
What could be the other products of this collision ?
(a) a γ-photon alone (b) a γ-photon and a β-particle
(c) 3.16 × 10–34 Js (d) 4.22 × 10–34 Js.
(c) a γ-photon and a neutron (d) a γ-photon and a proton.
4. Hydrogen (H), deuterium (D), singly ionised helium
(He+) and doubly ionised lithium (Li++)– all have one 9. Methods of recording electrical impulses from a Gei-
electron around the nucleus. For a given transition (say ger-Muller tube include (i) an amplifier and loud-
n = 2 to n = 1), let the wavelengths in the four cases be, speaker. (ii) a counter (scaler), (iii) a ratemeter. Which
respectively λ1 , λ2 , λ3 , λ4 . The expected relation approxi- one of the following statements is correct ?
mate) is (a) The loudness of an individual click from the loudspeaker
(a) λ1 ≈ λ2 ≈ 2λ3 ≈ 3λ4 (b) λ1 ≈ λ2 ≈ 4λ3 ≈ 9λ4 is a measure of the number of particles arriving per
second.
1 1 1
(c) λ1 ≈ λ ≈ λ ≈ λ (d) λ1 ≈ 2λ2 ≈ 2 2 λ3 ≈ 3 2 λ4. (b) The pitch of the signal in the loudspeaker may be used
2 2 2 3 4 4
to distinguish between the types of radiation producing
[National Standard Exam. in Physics 1993] the impulses.
5. In terms of Rydberg constant R, the shortest wavelength (c) The reading on the counter is the instantaneous value of
in Balmer series of hydrogen atom spectrum will have the number of electrical impulses per unit time.
wavelength (d) The reading on the ratemeter is the average value of the
(a) 1/R (b) 4/R number of electrical impulses per unit time.
(c) 3/2R (d) 9/R. (e) For a counter and a ratemeter both connected to the same
[National Standard Exam. in Physics 1994] G-tube, the relation between the readings c and r of the
counter and ratemeter respectively is c = dr/dt.
6. The ratio of minimum to maximum wavelength in
Balmer series is 10. In a laboratory experiment on emission from atomic
(a) 5 : 9 (b) 5 : 36 hydrogen in a discharge tube, only a small number of
lines are observed whereas a large number of lines are
(c) 1 : 4 (d) 3 : 4. [MP PET 2000]
present in the hydrogen spectrum of a star. This is be-
7. In a radioactive decay, let N represent the number of
cause in a laboratory
residual active nuclei, D the number of daughter nu-
(a) the amount of hydrogen taken is much smaller than that
clei, and R the rate of decay at any time t. Three curves
present in the star.
are shown in Fig. 32. The correct ones are
(a) 1 and 3 (b) 2 and 3 (b) the temperature of hydrogen is much smaller than that
of the star.
(c) 1 and 2 (d) all three.
ATOMIC NUCLEUS 745
(c) the pressure of hydrogen is much smaller than that of 18. The SI unit of radioactivity is
the star. (a) Roentgen (b) Rutherford
(d) the gravitational pull is much smaller than that in the (c) Curie (d) Becqueral. [KCET 2002]
star.
19. For uranium nucleus, how does its mass vary with vol-
11. A 238U nucleus emits alpha particles. In the lab the sepa- ume ?
ration between the alpha particle and the daughter nu- (a) m ∝ V (b) m ∝ 1/V
cleus becomes x in time t after the decay. If the decay
(c) m ∝ V (d) m ∝ V2.
were to occur in an aeroplane travelling with speed v
along the x-axis, the separation, as observed by a pas- [IIT Screening 2003]
senger in the plane will be 20. The mass number of a nucleus is
(a) x + vt (b) x – vt (a) always less than its atomic number

(c) x (d) none of these. (b) always more than its atomic number
(c) sometimes equal to its atomic number
[National Standard Exam. in Physics 1991]
(d) sometimes less than and sometimes more than its atomic
12. Two electrons are revolving around a nucleus at dis- number. [All India PM/PD 2003]
tances ‘r’ and ‘4r’. The ratio of their periods is
21. Three photons coming from excited atomic-hydrogen
(a) 1 : 4 (b) 4 : 1 sample are picked up. Their energies are 12.1 eV, 10.2
(c) 8 : 1 (d) 1 : 8. eV and 1.9 eV. These photons must come from
[National Standard Exam. in Physics 1998] (a) a single atom (b) two atoms
(c) three atoms
13. Which of the following cannot be emitted by radioac-
tive substances during their decay ? (d) either two atoms or three atoms.
(a) electrons (b) protons 22. The energy of an electron in nth orbit of the hydrogen
13.6
(c) neutrinoes (d) helium nuclei. atom is given by En = – eV .
n2
[AIEEE 2003]
The energy required to raise an electron from the first
14. Whenever a hydrogen atom emits a photon in the Balmer orbit to the second orbit will be
series (a) 10.2 eV (b) 12.1 eV
(a) it need not emit any more photon.
(c) 13.6 eV (d) 3.4 eV.
(b) it may emit another photon in the Paschen series.
23. In the above problem, the energy required to ionise hy-
(c) it must emit another photon in the Lyman series.
drogen atom is
(d) it may emit another photon in the Balmer series. (a) 10.2 eV (b) 13.6 eV
[KCET 2002] (c) 12.1 eV (d) 3.4 eV.
23
15. The sodium nucleus 11
Na contains 24. In question no. 22, the wavelength of the light emitted
(a) 11 electrons (b) 12 protons when the electron jumps from second to the first orbit is
(c) 23 protons (d) 12 neutrons. (a) 6563 Å (b) 4101 Å
(c) 4861 Å (d) 1212 Å.
[MP PET 2001]
25. The wavelength of the second line of Balmer series in
16. The order of the size of nucleus and Bohr radius of an
the hydrogen spectrum is 4861 Å. The wavelength of
atom respectively are the first line is
(a) 10–14 m, 10–10 m (b) 10–10 m, 10–8 m
27 20
(c) 10–20 m, 10–16 m (d) 10–8, 10–6 m. (a) × 4861 Å (b) × 4861 Å
20 27
[MP PET 2001]
(c) 20 × 4861 Å (d) 4861 Å.
17. A nucleus of mass 218 amu in free state decays to emit 26. If the mass of mu-meson is 207 times the mass of an
an alpha particle (mass 4 amu). The kinetic energy of electron, then the ratio of the radius of a mu-meson
the alpha particle is found to be 6.7 MeV. The recoil ground state orbit to that of corresponding electron or-
energy of the daughter nucleus (in Mev) is bit is nearly
(a) 6.7 × 4/214 (b) 6.7 × (4/214)2 1 FG 1 IJ 2
(c) 6.7 × 4/218 (d) 6.7 × (4/218)2 . (a)
207
(b)
H 207 K
{National Standard Exam. in Physics 1990] (c) 207 (d) (207)2.
746 COMPREHENSIVE OBJECTIVE PHYSICS

27. In the previous question, the ratio of the ionisation po- 35. If radiations of all wavelengths from ultraviolet to infra
tential of the mu-meson in its ground state level to that red is passed through hydrogen gas at room tempera-
of the corresponding electron is nearly ture, absorption lines will be observed in
1 1 (a) Balmer series (b) Lyman series
(a) (b) 2 (c) both Lyman and Balmer series
207 (207)
(c) 207 (d) (207)2. (d) Lyman, Balmer and Paschen series. [KCET 2001]
28. Which of the following atoms has the lowest ionisation 36. In the nuclear reaction 7N14 + 2He4 → X + 1H1 the
nucleus X is
potential ?
(a) 8N16 (b) 8N17
(a) 168 O (b) 147 N (c) 16 [KCET 2000]
(d) 8O17.
8O

(c) 133 Cs (d) 40 Ar . [AIEEE 2003] 37. Two alpha-particles with equal energies are fired
55 18
towards the nucleus of a gold atom.
29. The density of nucleus in kg/m3 is of the order of
Which diagram could represent their paths (in the plane
(a) 104 (b) 109 of the paper ?
(c) 1013 (d) 1017. [MP PMT 2000]
30. 1 amu is equivalent to A B
(a) 1.6 × 10–12 J (b) 1.6 × 10–19 J
(c) 1.5 × 10–10 J (d) 1.5 × 10–19 J.
[MP PET 2001] Gold nucleus Gold nucleus
31. A nucleus has a nucleon number A, a proton number Z,
and a binding energy B. The masses of the neutron and C D
proton are mn and mp , respectively, and c is the speed
of light. The mass of the nucleus is given by the expres-
sion
Gold nucleus Gold nucleus
(a) (A – Z)mn + Zmp – B/c2 (b) (A + Z)mn + Zmp + B/c2
(c) Amn + Zmp – B/c2 (d) Amn + Zmp + B/c2. Fig. 33
32. In order to trace the line of a water-pipe buried about (a) A (b) B
half a metre below the surface of a field, an engineer (c) C (d) D.
proposes to add a radioactive isotope to the water. Which
38. Which sketch graph best represents the variation of bind-
sort of isotope should he choose ? ing energy per nucleon with nucleon number ?
Emitter Half-life
(a) α a few hours Binding Binding
energy energy
(b) β a few hours per per
(c) β several years nucleon nucleon
(d) γ a few hours
Nucleon number Nucleon number
(e) γ several years.
(a) (b)
33. The electron emitted in beta radiation originates from
(a) inner orbits of atoms Binding Binding
(b) free electrons existing in nuclei energy energy
per per
(c) decay of a neutron in a nucleus nucleon nucleon
(d) photon escaping from the nucleus. [IIT 2001]
Nucleon number Nucleon number
34. An atom of mass number A and atomic number Z emits
(c) (d)
successively an α-particle, β-particle and γ-rays. The
mass number and atomic number of the end product Fig. 34
are 39. Fig. 35 shows the principle of a simple form of mass
(a) A, Z + 1 (b) A – 1, Z – 4 spectrometer. Ions are passed through narrow slits S1
(c) A – 4, Z – 1 (d) A – 4, Z + 2. and S2 and into a velocity selector. The selected ions,
[KCET 2000] after passage through the slit S3 ,are deviated by the
uniform magnetic field.
ATOMIC NUCLEUS 747

Path of S1 S2 S3 (a) 6563 Å (b) 3646 Å


Region of
ions magnetic (c) 3664 Å (d) 6536 Å.
field 44. Which one of the following statements is true of both α-
particles and X-rays ?
Velocity
selector (a) They cause ionisation of the air when they pass through
it.
(b) They can be detected after passing through a few milli-
P metre of aluminium.
(c) They can be deflected by electric fields.
Fig. 35 (d) They can be deflected by magnetic fields.
Which quantity must be the same for all ions arriving (e) They are used industrially for the photographic detec-
tion of flaws in metal castings.
at point P ?
(a) charge (b) mass
45. When the nucleus of an atom absorbs one of the atom’s
orbital electrons, the process is known as k-capture.
charge
(c) momentum (d) . Which equation (in which X denotes the appropriate
mass
particle) represents this process ?
40. Helium nuclei may result from the bombardment of
lithium nuclei with protons. The reaction can be repre- (a) 55 Fe + 0 X → 55 Mn
26 –1 25
sented by the following nuclear equation (b) 63 Ni → 0X + 63 Cu
28 –1 29
7 Li 1
+ p → 2 [ 42 He] + energy
3 1 (c) 105 B + 1 X → 7 U + 42 He
0 3
The speed of light is c, and the masses of the particles
are (d) 7 Li + 11 X → 8 Be .
3 4
lithium mL 46. A radioactive isotope decays by a one-state process into
helium mH a stable nuclide.
proton mP . Which graph could represent the activity A of the iso-
tope plotted against time t ?
What is the net energy released during such a reaction ?
(mL + mP ) − 2 mH
(a) [2mH – (mL + mP)] c2 (b) A A A
c2
(c) [(mL + mP) –2mH ] c2 (d) (2mH + mL + mP) c2
2mH − (mL + mP )
(e) .
c2
41. A collimated beam of monochromatic γ-rays of intensity
I0 penetrates a material and the intensity I varies with
depth x according to the relationship I = I0e–µx. Given
that a is the distance which causes the intensity to be
halved. Then
0 0 0
(a) µ = 1/a (b) µ = (ln 2)/a 0 t 0 t 0 t
(a) (b) (c)
(c) µ = (lg 2)/a (d) µ = 2/a
(e) µ = 1/2a.
A A
238 U 239 Np by β-emission is not possi-
42. The decay of 92 to 93
ble because
(a) β-decay only occurs in isotopes of low mass.
239
(b) 93 Np is not a stable isotope.
(c) mass number cannot increase in a decay process.
(d) atomic number cannot increase in a decay process.
(e) mass number and atomic number must both decrease in
a decay process. 0 0
43. The limit of Balmer series is 3646 Å. The wavelength of 0 (d) t 0 (e) t
first member of this series will be Fig. 36
748 COMPREHENSIVE OBJECTIVE PHYSICS

47. An event on a distant star causes the emission of a burst (c) As the photon moves out against strong gravitational at-
of radiation containing β-particles, γ-rays and light. traction, its energy decreases. So, frequency is expected
Which one of the following statements about the order to decrease. This is called gravitational shift.
in which these radiations arrive at, the Earth is (d) The neutrons emitted in fission need not be slowed down
correct ? to cause further fission.
(a) the light would arrive first 49. The ionising power is
(b) the γ-rays would arrive first
(a) same in all the three (b) maximum in α-particles
(c) the light and the γ-rays would arrive together, ahead of
(c) maximum in β-particles (d) maximum in γ-rays.
the β-particles
(d) the light and the β-particles would arrive together, ahead 50. The difference of energies between first two energy level
of the γ-rays of hydrogen atom is 10.2 eV. The same energy differ-
(e) all three would arrive together. ence for an atom with charge number 10 and mass
48. Which of the following is incorrect ? number 20 will be
(a) Fission yields greater energy per atom. (a) 2040 eV (b) 1020 eV
(b) Fusion yields greater energy per unit mass. (c) 510 eV (d) 102 eV.

Answers
1. (b) 2. (b) 3. (a) 4. (b) 5. (b) 6. (a) 7. (d) 8. (d)
9. (d) 10. (b) 11. (c) 12. (d) 13. (b) 14. (c) 15. (d) 16. (a)
17. (a) 18. (d) 19. (a) 20. (c) 21. (d) 22. (a) 23. (b) 24. (d)
25. (a) 26. (a) 27. (c) 28. (c) 29. (d) 30. (c) 31. (a) 32. (d)
33. (c) 34. (c) 35. (d) 36. (d) 37. (a) 38. (a) 39. (d) 40. (c)
41. (b) 42. (c) 43. (a) 44. (a) 45. (a) 46. (c) 47. (c) 48. (d)
49. (b) 50. (b).

Solutions
1 10− 14 FG 1 − 1 IJ c.
1.
10
× 3 × 108 =
t
5. Frequency = R
H2 ∞ K2 2

10 − 14 × 10 1 4
t= s= × 10–21 s. Hence, λ=
3 × 10 8 3 R
2. Cathode rays are streams of fast moving electrons. 1 F1 − 1I
3. – 13.6 + 10.2 = – 3.4 eV
6.
λ
∝ GH n n JK
1
2
2
2

− 13.6 13.6
= – 3.4 or n2 = =4 FG 1 − 1 IJ
n2 3.4
λ min. H2 3 K = 5.
2 2
or n=2
λ max.
=
FG 1 − 1 IJ 9
Increase in angular momentum =
h

H 2 ∞K
2

6.625 × 10− 34 7. N = N0e–λt , D = N0 – N and


= Js
2 × 3.14 dN R
R== – λN ⇒ = λ.
= 1.05 × 10–34 J s dt N
4. As an approximation only nuclear charge decides λ 8. From the laws of conservation of mass (nucleon
∝ Z –2. So, we expect λ2 = λ1, number) and charge (proton number), the reaction can
λ3 = λ1/4 and λ4 = λ1/9. be represented by
ATOMIC NUCLEUS 749

14
N + 42 He → 178 O + 11 H 13.6 13.6
7 22. E=– 2 – (– 13.6) = –
+ 13.6
2 4
Hence the emitted particle is a proton.
= (13.6 – 3.4)eV = 10.2 eV
9. The amplifier is installed to amplify the pulse voltage
23. Ionisation energy + (– 13.6eV) = 0
generated across the high resistor R in the tube due to
or Ionisation energy = 13.6 eV.
the sudded ‘discharge’ occcurs between the anode and
cothode in the tube. 24. 1 LM
1 1 OP
The counter (scaler) counts the actual number of pulses λ
=R −
1 4N Q
from the Geiger Miller tube. On the other hand a 1 3 4
=R× or λ =
ratemeter gives directly the average number of pulse λ 4 3R
per second or count rate from the tube. Only state-
4 × 1010
ment d gives corectly the function of one of the compo- or λ= Å = 1212 Å.
3 × 1.1 × 107
nents (ratemeter) in the Geiger-Miller tube.
11. Event and observer in same frame, hence no change in 1 LM
1 1
=R 2 − 2
OP
value.
25.
λ2 2 N4 Q
12. T2 ∝ R3 1 L 1 1 OP
=RM −
TR FG IJ
R
3/ 2
FG 1IJ 3/2 λ2 N 4 16 Q
T4R

4RH K =
H 4K 1 L3O
= R M P or λ2 =
16
λ 2 N 16 Q 3R

=
1
Again,
1
=RM
L 1 − 1 OP
8
λ 1 N2 3 Q2 2
14. For Balmer series, the electron is finally still in the
excited state. It has to reach the ground state by emit-
1 L 1 1O
=RM − P
ting a photon. λ1 N4 9Q
15. 23 – 11 = 12. 1 5R 36
= or λ1 =
17. Momentum conservation p(daughter) = p(alpha) in λ 1 36 5R
p2 λ 1 36 3R 27
magnitude. Since K.E = therefore the correct = × = × 4861 Å
2m λ 2 5R 16 20
choice is (a).
27
18. Becquerel is the SI unit of redioactivity. or λ1 =× 4861 Å.
20
1 Bq = 1 decay/s 1
26. r∝
1 Ci = 3.7 × 1010 Bq. m
19. Since nuclear density is constant therefore mass ∝ 27. Ionisation potential ∝ m
volume. 28. 133 Cs largest size ; Electrons in outermost orbit “maxi-
55
20. Mass number = atomic number + number of neutrons. mum” away from the nucleus. So, easy ionisation ;
In the case of hydrogen, the number of neutrons is zero. minimum potential.
So, mass number = atomic number 30. 1 amu = 1.66 × 10–27 kg
So, mass number is sometimes equal to atomic number. Now, E = 1.66 × 10–27 × 3 × 108 × 3 × 108 J
21. If the electron jumps from n3 level to n1 level, then = 1.49 × 10–10 J.
photon of energy 12.1eV is emitted. 31. The neutron number is A – Z. By considering the bind-
ing energy of the nuclide, we may write.
If the electron jumps from n3 level to n2 level, then 1.9
[(A – Z)mn + Zmp – mass of the-nucleus]c2 = B.
eV photon is emitted. If the electron jumps from n2
B
level to n1 level, then 10.2 eV photon is emitted. Clearly i.e., mass of the nucleus = (A – Z)mn + Zmp – 2 .
these transitions are possible in minimum two atoms c
32. Among the three types of radiation, γ-rays can
and maximum three atoms.
penetrate metal much better than the α-particles and
750 COMPREHENSIVE OBJECTIVE PHYSICS

β-particles. Emission of α-particles can easily be cut Path of S1 S2 S3 Region of


off by thin sheet of paper and β-particles can be cut icons magnetic
out by lead. γ-rays can only be cut off by very thick field
lead and also the abosorption of γ-rays in air is less
Velocity
appreciable compared with β-particles and α-particles. selector B1
Therefore γ-ray isotipe is most suitable for detecting
the location of the pipe.
P
The count rate of emission of γ-rays is proportional to
the rate of disintegration of the atoms, which in turns
is proportinal to the radioactivity decay constant λ. The Fig. 38
constant λ is inversely proportional to the half-life time 40. Mass defect of this nuclear reaction is (mL+ mP) – 2mH.
constant T1/2 by λ = 0.693/T1/2. Thus a higher count By Einstein’s mass-energy relation, the net energy re-
rate would be obtained if the half-life of emssion of λ- leased is thus
rays is only a few hours compared to several years. [(mL+mP) 2mH]c2.
34. Emission of α-particle gives A-4 and Z-2. Emission of β 41. If the distance a at which the intensity is reduced to
particle gives A-4 and Z-1. Emission of γ-ray has no half of the initial value I0, then
effect on A and Z. 1
37. The two alpha-particles are having the same speeds, I= I = I0e–µx
2 0
the one near to the gold nucleus experiences a larger ⇒ 2 = eµa ⇒ ln 2 = µa
repulsive force. ln 2
i.e. µ=
.
38. Typical variation of the binding energy per nucleon a
among the elements is given below. 42. The β-particle is a fast moving electron at high speed
and is denoted by 0θ. By laws of conservation of mass
MeV −1
4He 12C
and charge, the reaction must be such that
56Fe
238U 1. the total mass number is constant before and after
8 – 16O the reaction.
Binding energy
per nucleon, E

14N
6 2. the total charge number is constant before and af-
6Li ter the reaction.
4
We check that the mass number is 238 before the de-
2 2H 238
cay of 92 U takes place and increases to 239 after the
238
O 20 60 100 220 240 decay of 92 U has taken place which does not agree
Mass number, 4
with the law of conservation of mass. Hence, the decay
238 239
Fig. 37 of 92 U to 93 Np
by β-emission is not possible.
Variation of binding energy per nucleon with mass number.
43. 1
=R
1 1

LM OP
Excluding the nuclei lighter than 12C, we can see from
the diagram that the average binding energy per nu-
λ1 4 9 N Q
1 R×5 36
cleon. E/A, is fairly constant for the great majority of = or λ1 =
nuclei. The average value is about 8 MeV per nucleon. λ1 36 5R
The peak occurs at approximately the iron nucleus 56Fe
Again,
1
=R
1
=
R LM OP or λl =
4
which is therefore one of the most stable nuclei. λl 4 4 N Q R
39. The ions emitting from S3 must have the same veloc- λ1 36 R 9
E = × =
ity v = , after which they undergo circular path with λl 5 R 4 5
B
radius r given by 9
or λ1 =
× 3646 Å = 6562.8 Å.
mv m FG IJ FG v IJ = FG m IJ FG E IJ 5
r=
B1q
=
q H K H B K H q K H BB K
1 1
44. The following statements are true for α- particles or
X-rays or both.
ATOMIC NUCLEUS 751
(a) They cause ionisation of the air when they pass 46. Disintegration of the radioactive isotope occurs
through it. exponentially according to the equation
(b) only X-rays can be detected after passing through
a few millimetres of aluminium. dN
A= = − λN
(c) only α-particles can be deflected by electric fields. dt
(d) only α-particles can be deflected by magnetic fields. A is the activity or rate of decay.
(e) only X-rays are used industrially for the photo- where λ is the decay constant.
graphic detection of flaws in metal castings. N is the number of disintegrating atoms.
45. When the nucleus of an atom absorbs one of the at- When it becomes a stable nuclide, no further disinte-
om’s orbital electrons, its total mass (nucleon) number gration is possible and hence activity A has to be zero.
A is constant before and after the event occurs. The
proton number Z is reduced by 1 since absorption of 47. Both light and γ-rays travel at speed of light since they
an electron of its own has increased the neutron are electromagnetic waves. β-particles are fast mov-
number by 1. ing electrons at high speed near to speed of light. There-
Therefore, M and N are the atoms before and after the fore, both the light and the γ-rays would arrive together,
event, then the event can be written as ahead of the β-particles.
A 0 A 50. 100 × 10.2 = 1020 eV.
ZM + −1X → Z − 1N
which is best represented by equation in (a).

KNOWLEDGE PLUS
l Radioactive nuclei that are injected into a patient collect at certain sites within its body, undergoing radioactive decay
and emitting electromagnetic radiation. These radiations can then be recorded by a detector. This procedure provides
an important diagnostic tool called
(a) Gamma camera. (b) CAT scan.
(c) Radiotracer technique. (d) Gamma ray spectroscopy. [AIIMS 2003]
Solution. Radiotracer technique is being widely used for diagnostic tests. For example, to know the exact position
where the haemorrhage has taken place in the body, blood labelled with radioisotope chromium 51Cr is injected into
the patient. If there is no haemorrhage, the radioactivity is distributed throughout the circulatory system. But if
there is a haemorrhage, there will be a marked increase of radioactivity in the region where haemorrhage is taking
place.
So, (c) is the right choice.

l In the reaction 12 H + 13 H → 42 He + 10 n , if the binding energies of 2 3


1 H, 1 H and 4
2 He are respectively a, b and c (in
MeV), then the energy (in MeV) released in this reaction is :
(a) a + b – c (b) c – a – b
(c) c + a – b (d) a + b + c. [All India PM/PD 2005]
Solution. Factual information
So, (b) is the right choice.
752 COMPREHENSIVE OBJECTIVE PHYSICS

SELF-EVALUATION TEST II
Based on UNIT XVII

DIRECTIONS :
(i) MCQs 1 to 24 have one correct alternative.
(ii) MCQs 25 to 30 have more than one correct alternative.
(iii) MCQs 31 to 35 have one or more than one correct alternative.

1. The possible number of values of magnetic orbital quan- 6. A hydrogen atom moving with velocity u collides
tum number m can take are inelastically with another hydrogen atom at rest. Both
(a) 2l – 1 (b) 2l the atoms are in the ground state before collision. The
(c) 2l + 1 (d) 3l + 1. minimum value of u, so that one of the atoms get ex-
cited, will be
2. Electrons emitted by a hot filament pass down a tube
(a) 3.25 × 106 m s–1 (b) 9.25 × 105 m s–1
containing hydrogen and are then collected by an an-
ode which is maintained at a positive potential with (c) 6.25 × 10 m s
4 –1
(d) 6.25 × 103 m s–1.
respect to the filament. The gas near the anode is found 7. Masses to two isobars 29Cu64 and 30Zn64 are 63.9298u
to emit monochromatic ultra-violet radiation. The ra- and 63.9292u respectively. It can be concluded from
diation is monochromatic because these data that
(a) the nuclei emitting it are identical. (a) both the isobars are stable.
(b) the atoms emitting it each contain only one electron. (b) Zn64 is radioactive decaying to Cu64 through β-decay.
(c) the electrons gain only enough energy to raise the hy- (c) Cu64 is radioactive, decaying to Zn64 through γ-decay.
drogen atoms to their first excited state. (d) Cu64 is radioactive, decaying to Zn64 through γ-decay.
(d) the potential difference between the filament and the [IIT 1997]
anode is less than the ionisation potential of hydrogen.
8. The existence of energy levels within atoms can be dem-
(e) the energy of the electrons is less than the energy of a
quantum of light.
onstrated directly by observing that
(a) atoms can emit line spectra .
3. The radius of the shortest orbit in a one-electron system
(b) photoelectrons are only emitted for wavelengths greater
is 18 pm. It may be
than a critical wavelength .
(a) hydrogen (b) deuterium
(c) some α-particles are reflected back through very large
(c) He+ (d) Li++. angles by atoms in a solid .
4. The wavelength of radiation emitted is λ0 when an elec- (d) X-rays with frequencies up to a certain maximum are
tron in Hydrogen atom jumps from the third orbit to emitted by a target .
second. If in the Hydrogen atom itself, the electron jumps (e) atoms in a solid diffract electrons in the same way as
from fourth orbit to second orbit, the wavelength of crystals diffract X-rays .
emitted radiation will be 9. The wavelength of yellow line of sodium is 5896 Å. Its
16 20 wave number will be
(a) λ (b) λ (a) 50883 × 1010 per second (b) 16961 per cm
25 0 27 0
27 25 (c) 17581 per cm (d) 50883 per cm.
(c) λ (d) λ . [MP PET 2001]
20 0 16 0 [MP PET 2001]
5. The deviation of α-particles by thin metal foils through 10. In a mass-spectrometer, an ion of mass m and charge q
angles that range from 0° to 180° can be explained by enters a region of uniform magnetic field acting
(a) scattering from free electrons perpendicularly to the original line of flight. The
(b) scattering from bound electrons resulting path is
(c) diffuse reflections from the metals surface (a) circular and of radius proportional to m/q.
(d) scattering from small but heavy regions of positive charge (b) circular and of radius proportional to q/m.
(c) helical and of radius proportional to q/m.
(e) diffraction from the crystal lattice.
ATOMIC NUCLEUS 753
(d) parabolic with a displacement from the original path pro- 13. Binding energy per

Binding Energy/nucleon in MeV


portional to m/q. nucleon vs mass
8.5 Y
(e) linear and perpendicular to the original path. number curve for 8.0
X
W
11. Fig. 39 represents, drawn to scale, the energy levels for nuclei is shown in 7.5
an electron in a certain atom. Fig. 41 W, X, Y and 5.0 z
Z are four nuclei in-
dicated on the curve.
Energy E4 The process that
30 60 90 120
E3 would release en- Mass number of nuclei
E2 ergy is Fig. 41
(a) Y → 2 Z
(b) W → X + Z
(c) W → 2 Y (d) X → Y + Z.
[IIT Screening 1999]
257
E1 14. If the atom 100Fm follows the Bohr model and the
radius of 100Fm257 is n times the Bohr radius, then find
n
Fig. 39 (a) 100 (b) 200
The transition from E3 to E1 produces a green line. What (c) 4 (d) 1/4.
transition could give rise to a red line ? [IIT Screening 2003]
(a) E4 to E3 (b) E4 to E2 15. Which of the curves may I
(c) E4 to E1 (d) E3 to E2 represent the speed of IV
(e) E2 to E1. the electron in a hydro-
II
12. Fig. 40 shows a mass-spectrometer in which positive gen atom as a function
III
ions pass through slits S1 , S2 , S3 , before entering the of the principal quan-
main chamber. Between S2 and S3 they pass through tum number n ?
mutually perpendicular magnetic and electric fields, the (a) I (b) II
Fig. 42
intensities of which may be varied. (c) III (d) IV.
16. A star initially has 1040 deuterons. It produces energy
S1 via the processes
2 2 3 2 3 4
Magnetic 1H + 1H → 1H + p and 1H + 1H → 2He + n
S2 field 16
If the average power radiated by the star is 10 W, the
deuteron supply of the star is exhausted in a time of the
order of
(a) 106 s (b) 108 s
S3
(c) 1012 s (d) 1016 s.
Main The masses of the nuclei are as follows : M(H2) = 2.014
chamber
amu ; M(p) = 1.007 amu ; M(n) = 1.008 amu ; M(He4) =
4.001 amu. [IIT 1993]
17. The decay constant of a radioactive sample is λ. The
half-life and mean-life of the sample are respectively
given by
Fig. 40
1 (ln 2) (ln 2) 1
The purpose of the mutually perpendicular fields be- (a) and (b) and
λ λ λ λ
tween S2 and S3 is to 1 λ 1
(a) accelerate the ions to high velocity. (c) λ (ln 2) and (d) and . [III 1989]
λ (ln 2) λ
(b) eliminate stray electrons from the beam. 18. Naturally occurring gallium contains two isotopes
(c) select ions of a particular charge. 71 69
31 GA and 31Ga with relative abundance of 40% and
(d) select ions of a particular mass. 60% respectively. The average number of neutrons in a
(e) select ions of a particular velocity. gallium nucleus in the mixture is
754 COMPREHENSIVE OBJECTIVE PHYSICS

(a) 38 (b) 40 23. Fig. 44 shows an arrangement of two metallic half-cyl-


(c) 39 (d) 38.8 inders with a common axis with a number of slits S
(e) 78. that define a semicircular path of radius r the whole
being enclosed in a
19. A freshly prepared radioactive source of half-life 2 hr S2
vacuum vessel. The outer
emits radiation of intensity which is 64 times the per-
half-cylinder is at a posi-
missible safe level. The minimum time after which it
tive potential with re-
would be possible to work safely with this source is
spect to the inner one so r
(a) 6 h (b) 12 h
that a constant radial S S3
(c) 24 h (d) 128 h [IIT 1988] electric field is main-
1

20. The count rate of activity of a radioactive sample of a tained between them. A
very large population decreased from 1024 to 128 in 3 collimated beam of sin-
minutes. Then the rate of disintegration at the end of 5 gly charged positive ions
minutes is is injected at S1 Fig. 44
(a) 96 (b) 64 Given that the incident beam contains ions of different
(c) 48 (d) 32. masses and speeds, the beam which emerges at S3 con-
21. The elastic collision between an alpha-particle and a tains only ions that have the same
stationary helium atom was observed in a cloud cham- (a) mass (b) specific charge
ber. Given that all three tracks in each diagram lie in (c) speed (d) kinetic energy
the plane of the paper, which one of the following dia-
(e) momentum.
grams could illustrate the tracks obtained ?
24. Fig. 45 shows a graph of the binding energy per nu-
cleon for a number of naturally-occurring nuclides plot-
ted against their mass number.

9
a a
Binding energy per nucleon/MeV

8 27
Al
13
(a) (b) 7 238
23 U
Na 92
6 11

a a 4
(c) (d)
3
2
a
(e) 2
1 H
1

0 20 40 60 80 100 120 140 160 180 200 220 240


Fig. 43 Mass number
22. A head-on elastic collision between an α-particle and a
Fig. 45
stationary helium nucleus would be impossible to de-
tect in a cloud chamber because Which of the following statements is a correct deduc-
(a) non photons are emitted in the collision tion from the graph ?
(b) the helium nucleus is too light to deflect the α-particle 2
noticeably (a) Of the nuclides plotted, 1 H is the most stable.
(c) the track of the helium nucleus after the collision is (b) Energy will be released if a nucleus with a mass number
indistinguishable from that of the α-particle before the
greater than about 80 undergoes fusion with any other
collision
nucleus.
(d) details of the interactions of individual particles cannot
be detected in a cloud camber (c) Energy will be released if a nucleus with a mass number
less than about 80 undergoes fission as a result of parti-
(e) unchanged particles do not leave tracks in a cloud
cle bombardment
chamber.
ATOMIC NUCLEUS 755

(d) 27 Al will not spontaneously emit an alpha particle to 28. In a nuclear fission,
13
(a) in elements of high atomic mass number , energy is re-
23
become 11 Na . leased
(b) linear momentum and total energy are conserved, but
(e) 238
92
U is the stable end-point of a number of radioactive not angular momentum
series.
(c) linear momentum, angular momentum and total energy
25. A large population of radioactive nuclei starts are conserved
disintegrating at t = 0. At time t, if N = number of parent (d) the probability of neutron being absorbed by a fission-
nuclei present, D = the number of daughter nuclei able nucleus increases when the neutrons are slowed
present and R = rate at which the daughter nuclei are down.
produced, then the correct representation will be 29. Which of the following statements are correct ?
(a) Heavy nuclei have more number of protons than neu-
trons.
(b) Heavy nuclei have more number of neutrons than pro-
tons.
N D (c) Heavy nuclei are likely to undergo fission than fusion.
(d) Heavy nuclei are more likely to undergo fusion than
fission.
30. When the atomic number A of the nucleus increases,
t t (a) initially the neutron-proton ratio is constant = 1
(a) (b) (b) initially neutron-proton ratio increases and later decreases
(c) initially binding energy per nucleon increases and later
decreases
(d) the binding energy per nucleon increases when the
R neutron-proton ratio increases.
D
N 31. The probability of disintegration per second of a nucleus
in a given radio-active sample
(a) increases proportional to the life time lived by the nucleus
(b) decreases with the life time lived
t t
(c) (d) (c) is independent of the life time lived
(d) depends upon the total number of identical nuclei present
Fig. 46 in the sample.
32. Which of the following is correct for a nuclear reactor ?
26. Which of the following statement(s) is (are) correct ?
(a) A typical fission is represented by
(a) The rest mass of a stable nucleus is less than the sum of
the rest masses of its separated nucleons. 92
U235 + 0n1 → 56
B143 + 36Kr93 + energy
(b) Heavy water (D2O) is used as moderator in preference to
(b) The rest mass of a stable nucleus is greater than the
ordinary water (H2O) because H may capture neutrons,
sum of the rest masses of its separated nucleons.
while D would not do that.
(c) In nuclear fusion, energy is released by fusing two nu-
(c) Cadmium rods increase the reactor power when they go
clei of medium mass (approximately 100 amu).
IN, and decrease when they go OUTWARDS
(d) In nuclear fission, energy is released by fragmentation
(d) Slower neutrons are more effective in causing fission than
of a very heavy nucleus.
faster neutrons in the case of U(235).
27. The instability of the nucleus can be due to various
[National Standard Exam. in Physics 1993]
causes. An unstable nucleus emits radiations if possi-
ble to transform into less unstable state. Then the cause 33. An electron orbiting around the nucleus of an atom
and the result can be (a) has a magnetic dipole moment
(a) a nucleus of excess nucleons is α active (b) exerts an electric force on the nucleus equal to that on it
by the nucleus
(b) an excited nucleus of excess protons is β active

(c) does produce a magnetic induction at the nucleus


(c) an excited nucleus of excess protons is β+ active
(d) has a net energy inversely proportional to its distance
(d) a nucleus of excess neutrons is β– active.
from the nucleus.
756 COMPREHENSIVE OBJECTIVE PHYSICS

34. A negative charge is revolving around a fixed positive 35. Fig. 47 shows Bohr orbit of
charge in a circular orbit. If the classical idea of an electron in H-atom, the elec-
accelerating charge radiating energy is valid, then the tron going clockwise. The mag-
negative charge will netic field is normal to the or-
(a) spiral towards the positive charge, with increasing bit and into the plane of paper
kinetic energy (away from observer). Now,
(b) spiral towards the positive charge with potential energy which of the following is cor-
Fig. 47
decreasing at a faster rate than increase in its kinetic rect ?
energy (a) The orbital angular momentum is into the plane of pa-
(c) spiral away from the positive charge and finally escape per.
from the binding of the positive charge (b) The orbital magnetic moment is out of the plane of pa-
(d) revolve around the positive charge with increasing fre- per.
quency of revolution. (c) The orbital frequency in presence of this magnetic field
is higher than the normal value in Bohr orbit.
(d) The Bohr orbits of all the H-atoms in a sample would
align the same way in the given magnetic field.
[National Standard Exam. in Physics 1992]

Answers
1. (c) 2. (c) 3. (d) 4. (b) 5. (d) 6. (c) 7. (a) 8. (a)
9. (b) 10. (a) 11. (e) 12. (e) 13. (c) 14. (d) 15. (c) 16. (c)
17. (b) 18. (d) 19. (b) 20. (d) 21. (c) 22. (c) 23. (d) 24. (d)
25. (a), (b), (c), (d) 26. (a), (d) 27. (a), (c), (d) 28. (a), (c), (d) 29. (b), (c) 30. (a), (c) 31. (c), (d) 32. (b), (d)
33. (a), (b), (c), (d) 34. (a), (b), (d) 35. (a), (b), (c).

Solutions
2. Before Bohr’s theory of separate energy levels of the Thus monochromatic ultraviolet radiation is observa-
hydrogen atom, it was found that the wavelengths of tion because the electrons gain only enough energy to
the hydrogen spectrum could be arranged in a formula raise the hydrogen atoms to their first excited state
or series named after its discoverer. The visible spec- corresponding to n = 2.
trum was the Balmer series, the ultraviolet was the 3. Radius of shortest orbit of hydrogen atom
Lyman series and the infra red was the Paschen se- = 0.53 × 10–10 m = 53 × 10–12 m
ries.
= 53 pm
Bohr’s theory of energy levels accounted for all the se-
Radius of hydrogen-like atom
ries
(1) the ultraviolet series, for example, is obtained when Radius of hydrogen atom
=
energy of the atom falls to the lowest energy level Z
E1 (– 13.6 eV) correspond to n = 1. 53
∴ Z= ≈ 3.
(2) the visible spectrum is obtained for energy falls to 18
the first excited state E2 (– 3.4 eV) corresponding 1 LM
1 1OP
to n = 2. 4.
λ0 2N
=R 2 – 2
3 Q
(3) the infra red spectrum is obtained for energy falls
to the second excited state E3 (– 1.5 eV) correspond- 1 L 1 1 O 5R
=RM – P =
ing to n = 3. λ0 N 4 9 Q 36
ATOMIC NUCLEUS 757

36 v2
R= qBv = m
5λ0 r
1 1 LM
1 OP where v is the speed of the ion of mass m and charge q
Again,
λ
=R 2 – 2
2 4N Q mv m FG IJ FG v IJ which is directly proportional
1 36 4–1
∴ r=
qB
=
q H K H BK
= ×
λ 50λ 0 16 m
to .
1 36 × 3 q
= λ0
λ 5 × 16 11. Red line has a wavelength λt of about 0.7 µm which is
longer than that of green light λg at about 0.55 µm
1 27 20
= λ 0 or λ = λ0 c
λ 20 27 From f = where c is the speed of light, red line thus
5. The deviation of α-particles by thin metal foils through λ
angles that range from 0° to 180° was found in an ex- has a lower frequency ft compared to that of green line
fg. Since energy change E due to transition between
periment conducted by Geiger and Marsden in 1890 to
levels gives rise to electromagnetic radiation is
investigate the scattering of α-particles. The scatter-
proportional to its frequency f where E = hf, the energy
ing of α-particles is due to presence of small but highly
change for the red line must be smaller than that of
concentrated positive charge at the heart or centre of the green line. The possible transitions for the red line
the atom. Those particles very close to the nucleus are are the E2 to E1, E3 to E2,E4 to E2 and E4 to E3 to E3
deflected through a large angle since the repulsive force transitions.
is then very big. hc
6. Loss in energy in inelastic collision The energy change for the red line is Et =
λ
1 m1m2 (6.63 × 10 –34 ) (3 × 108 )
= (u1 – u2 )2
2 m1 + m2 =
0.7 × 10–6
1 m 2 mu2 = 2.84 eV
= u =
2 2 4 hc
The energy change for the green line is Eg =
λ
mu2
Now = 10.2 × 1.6 × 10–19 (6.63 × 10 –34 ) (3 × 108 )
4 =
0.55 × 10–6
4 × 10.2 × 1.6 × 10–19 = 3.62 eV
or u= –27 m s–1
1.67 × 10 Et 2.84
=
= 6.25 × 104 m s–1 Now E g 3.62 ≈ 0.8
7. 64 → 30Zn64 + –1e0
29Cu Thus, the line drawn on the diagram corresponds to
In beta decay, atomic number increases by 1 whereas the energy change Et for the red line should be about
atomic mass number remains unchanged. 0.8 times the length of the line drawn for the transi-
8. The fact that the emission spectrum of an atom has tion from E3 to E1 for the green line.
lines that are separated is experimental evidence for Thus, the red line must be produced by the transition
the existence of separate or ‘quantised’ energy levels from E2 to E1.
in atom.
12. The perpendicular fields of magnetic B and electric E
1 108 pull on the ions in opposite directions.
9. ν= = cm–1
5896 × 10 –8 5896 When equal, electric force qE = magnetic force Bqv
= 16960.6 cm–1 ≈ 16961 cm–1. E
or E = Bv ∴ v =
10. Since the force is always normal to the velocity of the B
ion and is constant, the resulting path is a circular This arrangement of perpendicular electric and mag-
one. The radius r of the curve is computed from the netic fields is therefore called a ‘velocity selector’ since
magnetic centripetal force. Thus it only allows ions through S3 which have the same
velocity V equals to E/B.
758 COMPREHENSIVE OBJECTIVE PHYSICS

13. For reaction W → 2Y 1024


λt = log e
Binding energy of W = 7.5 × 120 = 900 MeV 128
Binding energy of 2Y = 2(8.5 × 60) = 1020 MeV λt = loge 8
∴ Binding energy (2Y) > Binding energy (W) 1
t= loge 23
So, energy will be released. λ
T
m2 t= × 3 loge 2
14. rn = (0.53) Å log e 2
Z
t 3
m2 (0.53) Å or T= = min = 1 min
(rn)Fm = = 0.53 Å × n 3 3
100 N 1
Now, = t/ T
1 N0 2
m is 5 for Z = 100 ⇒ n =
4 N 1 1024 1024
= 5 / 1 or N = = = 32.
1 c 1 1024 2 25 32
15. v= or v ∝ .
137 n n 21. An α-particle is infact, a helium nucleus. Thus, when
16. Adding given equations, we get oblique collision occurs between the moving α-particle
2 2 2 4
1H + 1H + 1H → 2He + p + n
and a stationary helium nucleus, the helium atom will
Mass defect = 3 × 2.014 – 4.001 – 1.007 – 1.008 gain the velocity V cos θ from the α-particle.
= 0.026 amu V sin θ
Energy released by three deuterons α-particle α-particle
= 0.026 × 931 MeV V V
= 0.026 × 931 × 1.6 × 10–13 J θ θ
= 3.87 × 10–12 J
Energy released by 1040 deuterons,
He
3.87 × 10 –12 × 1040
He
E= ≈ 1028 V cos θ
3
If P is power and t is time
Fig. 48
E 1028
∴ E = Pt or t = = = 1012 s. On the other hand the α-particle loses all its energy in
P 1016
the direction parallel to the line joining the centres at
log e 2 the moment of impact and moves in the direction per-
17. T1/2 =
λ pendicular to that of the helium atom at a speed of V
1 sin θ. Thus, the track obtained is best represented in
and Tmean = .
λ diagram C.
71
18. Number of neutrons in 31Ga = 40 22. An α-particle is infact a helium nucleus. In head-on
Number of neutrons in Ga69 = 38 electric collision of an α-particle with a stationary
31
40% of 40 is 16 helium nucleus, all the energy of the α-particle is given
to the helium nucleus and the helium nucleus will
60% of 38 is 22.8
continue the track of the α-particle. Thus, the track of
Total is 38.8. the helium nucleus after the collision is
N FG IJ
1
t/ T indistinguishable from that of the α-particle before the
19.
N0
=
H K
2
collision, and hence the collision would be impossible
to detect in a cloud chamber.
1 F 1I FG 1IJ = FG 1IJ
t/T 6 t /T
=G J 23. The electric field E is acting perpendicular to the
64 H 2 K H 2K H 2K
or
direction of motion of the beam of ions entering at S1,
t passing through S2 and leaving from S1. Since the force
or =6 or t = 6 × 2 h = 12 h F on the beam of ions emerging at S3 is perpendicular
T
to the motion of the ions, no work is done on the beam
20. N = N0 e–λt
of ions and hence the force F cannot change the energy
128 = 1024 e–λt of the ions. Thus, the beams which emerges at S3
1024 = λt contains only ions that have the same kinetic energy.
e
128
ATOMIC NUCLEUS 759
24. From the graph, the heavier aluminium nucleus Al which is more
stable since it has a higher binding energy per
(a) 21 H has smallest binding energy per nucleon of 1 nucleon than that of sodium Na. The loss of
MeV and is not the most stable nuclide. The potential energy is released as kinetic energy of
maximum binding energy per nucleon is at about aluminium Al.
mass number A = 80. Nuclides at about A = 80 are Since aluminium is more stable than sodium Na,
thus most stable since they need most energy to spontaneous disintegration of aluminium to become
disintegrate. sodium Na with α-decay is not possible.
(b) A nucleus with mass number greater than about 238 U
(e) is unstable end-point of the number radio-
80 undergoes fission as a result of particle 92
bombardment will splits into two product nuclei of active series of mass number greater than about
roughly equal mass. Each nucleon, when the 80 since its binding energy per nucleon is the lowest.
nucleus disintegrates, becomes a member of smaller 32. (a) No. In a typical one some neutrons do come out,
nucleus, so the binding energy per nucleon averaging >2.
increases. The nucleons in the product nuclei are (b) True. Infact H could share momentum better than
even more tightly bound together so the binding D, but H captures neutrons.
energy increases and there is loss of potential
(c) The reverse is true. Cd absorbs neutrons. So, going
energy as nucleons are pulled together by the
IN decreases power.
nuclear force. The loss of potential energy is
released as the kinetic energy of the product nuclei. (d) Yes, that is why we go in moderators.
(c) When a nucleus with a mass number less than → →

about 80 undergoes fusion with any other nucleus, 35. (a) Yes, r × p is into plane of paper.
the binding energy per nucleon of the product (b) Yes, because of –ve charge sign.
nucleus is increased and there is again loss of → → → →
potential energy by the fusion process. Thus, energy (c) v × B is away from centre, q v × B is thus towards
is released as the kinetic energy of the product the centre. Hence, frequency increases.
nucleus when light nuclei are fused together. (d) No, there will be distribution, though more towards
(d) As explained in (c), the light sodium nucleus Na alignment.
can be fused together with the α-particle to produce

KNOWLEDGE PLUS
l Characteristic X-rays are produced due to :
(a) transfer of momentum in collision of electrons with target atoms.
(b) transition of electrons from higher to lower electronic orbits in an atom.
(c) heating of the target.
(d) transfer of energy in collision of electrons with atoms in the target. [AIIMS 2003]
Solution. When high energy electron hits the target and takes out an electron from inner orbit, there is a possibility
of an outer electron to fall down to fill up that vacancy. When such a transition of electron takes place from higher
energy level to lower energy level, the difference of energy between the two levels is given off in the form of radiation
known as characteristic X-rays.
So, (b) is the right choice.
UNIT XVIII

SOLIDS AND SEMICONDUCTOR DEVICES,


ELECTROMAGNETIC WAVES AND
PRINCIPLES OF COMMUNICATION
l Energy bands in solids (qualitative ideas only) l Difference between metals, insulators and
semiconductors using band theory l Intrinsic and extrinsic semiconductors l p-n
junction l Semiconductor diode-characteristics in forward and reverse bias l Diode as a
rectifier l Solar cell l Photo diode l LED l Zener diode as a voltage regulator
l Junction transistor l Transistor action l Characteristics of a transistor l Transistor as
an amplifier (Common emitter configuration) and oscillator l Logic gates (OR, AND, NOT, NAND
and NOR) l Elementary ideas about I.C. l Electromagnetic waves and their characteristics
(qualitative ideas only) l Transverse nature of electromagnetic waves l Electromagnetic spectrum
(Radio-microwaves, infra-red, optical, ultraviolet, X-rays, gamma rays) including elementary facts about
their uses l Propagation of electromagnetic waves in atmosphere l Elementary idea of analog
and digital communication l Need for modulation l Modulation—amplitude, frequency and pulse
modulation l Elementary idea about demodulation l Data transmission and retrieval—Fax and
Modem l Space communication—propagation of electromagnetic waves in atmosphere l Sky and
space wave propagation l Satellite communication l Applications in Remote sensing l Line
communication—2-wire lines, cables, telephone links l Optical communication (optical fibres,
lasers) l Elementary principle of light modulation

UNIT DETAILS

1. Synopsis Points 45 Plus

2. Illustrations 15 Plus

3. MCQs from Competitive Examinations 215 Plus

4. Self-Evaluation Tests 2

5. Total Number of MCQs 450 Plus

6. Total Number of Solutions 430 Plus


CHAPTER 18

SOLIDS AND SEMICONDUCTOR DEVICES,


ELECTROMAGNETIC WAVES AND
PRINCIPLES OF COMMUNICATION

SYNOPSIS

1. INTRINSIC SEMICONDUCTOR energy required to detach this fifth electron from the atom is
It is a pure semiconductor i.e. it is free from impuri- of the order of only 0.05 eV for silicon. It is considerably less
ties. In this semiconductor, the valence band and the conduc- than the energy required to break a covalent bond. This small
tion band are separated by nearly 1 eV. The energy gap in the amount of energy is easily provided by the thermal agitation
case of silicon is 1.1 eV. In the case of Germanium, it is 0.74 of the crystal. Thus, the fifth valence electron can move about
eV. In terms of energy bands, the position of a semiconductor almost as freely as an electron in a metal—almost free because
lies between a metallic conductor and an insulator. it is always having a weak interaction with the parent atom.
Silicon and Germanium are the best examples of semi- The force of attraction between the positively charged (+ 5)
conductors because these are the most widely used semicon- impurity ion and this mobile electron is weakened by the
ductors. dielectric constant of the medium. This means that such
electrons from impurity atoms will have energies slightly less
2. DOPING A SEMICONDUCTOR
than the energies of electrons in the conduction band. In the
Following are the three different methods of doping a
energy band picture, the energy state corresponding to the
semiconductor.
fifth valence electron is in the forbidden gap and is slightly
(i) The crystal is heated in an atmosphere containing below the conduction band. The energy level is indicated by
dopant atoms. The dopant atoms diffuse into the hot crystal. the dashed line in Fig. 2. This level is called the donor level.
(ii) The impurity atoms are added in the molten state
of semiconductor. Valence
electrons
(iii) The intrinsic semiconductor is bombarded by ions +4 +4 +4
of impurity atoms.
Covalent Free
bond
3. N-TYPE SEMICONDUCTOR electron

When an elemental semiconductor of Group IV such +4 +5 +4


as Si or Ge is doped with a pentavalent impurity (an element Pentavalent
of group V such as phosphorus, arsenic or antimony), we get impurity ion Silicon
ion
N-type semiconductor.
+4 +4 +4
Fig. 1 shows the crystal structure obtained when silicon
is doped with pentavalent impurity. Four of the five valence
electrons occupy covalent bonds. The fifth electron will be
loosely bound. It will be available as a carrier of current. The Fig. 1

763

C-11\IITS\C18-1
764 COMPREHENSIVE OBJECTIVE PHYSICS

When the fifth valence In a P-type semiconductor, holes are the majority
Conduction
electron is transferred to the cond- band charge carriers and the free electrons are the minority charge
uction band, the parent impurity carriers.
atom becomes positively charged Donor
Like in an N-type semiconductor, a P-type semicon-
immobile ion. In this way, each level ductor also satisfies the relation
impurity atom donates a free Valence pn = pi ni = ni2
electron to the semiconductor. Such band
For each impurity atom, there is a free hole in the va-
impurity elements are called lence band but there is no corresponding generation of free
donors. It is for this reason that N- Fig. 2 electron in the conduction band. So, p > pi but n < ni.
type semiconductor is sometimes
called donor-type semiconductor. 6. FORWARD-BIASED PN JUNCTION DIODE
While the holes of the P-side are repelled by the posi-
4. CARRIER CONCENTRATION IN N-TYPE SEMI- tive terminal of the battery, the electrons of the N-side are
CONDUCTOR repelled by the negative terminal of the battery.
If n and p represent the electron and hole concentra- P N
tions respectively in N-type semiconductor, then it can be
shown that – – – – – + + + + +

np = ni pi = ni2 – – – – – + + + + +

– – – – – + + + + +
where ni and pi are the intrinsic values.
– – – – – + + + + +
5. P-TYPE SEMICONDUCTOR
When an elemental semiconductor of Group IV such
as Si or Ge is doped with a trivalent impurity (an element of
Group III such as Indium, Boron or Gallium), we get P-type
+ –

semiconductor. Fig. 5

Thus, the potential barrier as well as the width of the


depletion region are reduced. Consequently, a large number
of majority charge carriers are able to diffuse across the
junction.
7. REVERSE-BIASED PN JUNCTION DIODE
P N

– – – – – + + + + +

– – – – – + + + + +

– – – – – + + + + +

– – – – – + + + + +

Depletion region
Fig. 3 Fig. 4

The acceptor impurity produces an energy level just


above the valence band. This energy level is shown by a dashed – +

line in Fig. 4. The energy difference between the acceptor Fig. 6


energy level and the top of the valence band is clearly much
While the holes of the P-side are attracted towards the
smaller than the band gap. Electrons from the valence band negative terminal of the battery, the electrons of the N-side
can therefore easily move into the acceptor level by being ther- are attracted towards the positive terminal of the battery.
mally agitated. Thus, the majority carriers are pulled away from the junction
SOLIDS AND SEMICONDUCTOR DEVICES, ELECTROMAGNETIC WAVES AND PRINCIPLES OF COMMUNICATION 765

thereby increasing the depletion region and the potential


LM exp F eV I − 1OP
MN GH nk T JK PQ
barrier. I = I0
The reverse-biased PN junction diode has an effective
capacitance called transition or depletion capacitance. While Here, I0 represents saturation current, k is Boltzmann’s
the P and N regions (low resistivity regions) of the reverse- constant, T is absolute temperature and n is the identity factor.
biased diode act as the plates of the capacitor, the depletion
For silicon, the value of n is more than one. However, for
region (high resistivity region) acts as the dielectric.
germanium, it is equal to one.
8. FORWARD-BIAS CHARACTERISTICS When the PN junction is forward-biased, V is very
Fig. 7 shows the silicon- Y large. So, the unity in paranthesis can be neglected. In this
diode characteristic when the case, the current is given by
diode is forward-biased. It is
Forward current (mA)

clear from the graph that the FG eV IJ .


diode current is very small for
the first few tenths of a volt.
I = I0 exp
H nk T K
I When the PN junction is reverse-biased, V is negative.
This is because the diode does Knee
not conduct well until the point So, the exponential term can be neglected as compared to 1.
VK
external voltage overcomes the So, the current in the reverse bias is given by I = I0.
barrier potential. As we O 0.7 V X This shows as to why the reverse current is independ-
approach 0.7 V, larger number Forward bias V
ent of the applied voltage. I0 is generally referred to as re-
of the free electrons and holes
Fig. 7 verse saturation current.
start crossing the junction.
Above 0.7 V, even a small increase in voltage produces a large For Ge at room temperature (300 K),
increase in current. The voltage at which the current I = I0 exp 38.7.
starts to increase rapidly is called the cut-in or knee
11. RESISTANCE OF A DIODE
voltage (V0) of the diode. While for a silicon diode, it is
nearly 0.7 V, for a germanium diode, it is nearly 0.3 V. The V
(i) Static or DC resistance =
forward current which is due to the flow of majority charge I
carriers is typically in the range of mA.
(ii) The dynamic or AC resistance of a junction
9. REVERSE-BIAS CHARACTERISTICS diode is the ratio of small change in voltage to the small
Fig. 8 shows the Reverse bias change in current.
reverse-bias characteristics. Vz –V
∆V
In the reverse-bias, the rd =
Zener ∆I
Reverse current (µA)

diode current (called reverse voltage


current) is very small (only Beyond the knee point, the forward characteristic is
I
a few µA for germanium nearly straight. So, above the knee point, the dynamic resist-
breakdown
Crystal

diode and only a few nA for ance of the diode is almost independent of the applied volt-
silicon diode). It remains age. The corresponding region of the forward characteristic is
small and approximately called linear region.
constant for all voltages less
than the breakdown voltage 12. LIMITATIONS OF A DIODE
V z . At breakdown, the Fig. 8 (i) A diode merely converts AC into DC and its input
current increases rapidly for and output circuits are not separate.
small increases in voltage. (ii) A diode cannot give amplification i.e., diode cannot
This is because the covalent bonds near the junction increase the amplitude of the signal.
break down and a large number of electron-hole pairs are
liberated. The breakdown voltage is also called Zener voltage 13. ZENER DIODE
(after the American physicist, Clarence Zener). Zener diode is a PN P N
10. RELATION BETWEEN V AND I FOR A PN junction diode specially de-
JUNCTION DIODE—A QUANTITATIVE STUDY signed to work only in the re-
The current I flowing in a PN junction diode is related verse breakdown region. Fig. 9
to the voltage V applied across the diode by the following Fig. 9 shows the symbolic repre-
relation. sentation of a Zener diode.
766 COMPREHENSIVE OBJECTIVE PHYSICS

Zener diode is made from highly doped P and N type The current is carried by holes inside the transistor and
semiconductors. Its power rating is kept high so that it is not by electrons in the external circuit.
damaged by the large reverse current. The voltage across such In the emitter-base external circuit, the flow of elec-
a diode remains constant. trons is from P to N. So, the conventional current IE flows
The Zener diodes are used for making constant volt- from N to P. In the collector-base external circuit, the flow of
age power supplies. An experimental arrangement employ- electrons is from base to collector. So, the conventional cur-
ing Zener diode is shown in Fig. 10. The Zener diode is con- rent IC flows from collector to base. The difference between IE
nected to fluctuating voltage supply through a resistor R. This and IC is the base current IB. It flows in the direction of IE.
resistor is called dropping resistor. The constant voltage out- The relation between the three currents is as under :
put is taken across a load resistance RL connected in parallel
IE = IB + IC
with the diode.
R 15. WORKING OF NPN TRANSISTOR

Emitter Collector
junction junction
N N P N
NPN
Fluctuating Constant E C
RL voltage supply
voltage supply
P
B
IE IB IC
IE IC
IB

Fig. 10
– + – + – + – +
VEB VCB VEB VCB
14. WORKING OF PNP TRANSISTOR
The emitter-base junction is always forward-biased with (a) (b)
the help of voltage VEB. The collector-base junction is always
reverse biased with the help of voltage VCB. Fig. 12

The holes in the emitter are pushed into the base by The working of the NPN transistor can be studied with
the electrostatic influence of the positive terminal of the the help of the circuit shown in Fig. 12. While Fig. 12 (a) gives
battery of voltage VEB. Since the base is thin and only slightly the block diagram, Fig. 12 (b) is the equivalent circuit in which
doped therefore only a few holes (say, 1%) combine with the symbol of NPN transistor has been used. The emitter
electrons in the base. So, the base current IB is small. junction is forward-biased with the help of the emitter-base
battery of voltage VEB. The collector junction is reverse biased
Emitter
junction
Collector with the help of collector-base battery of voltage VCB.
junction
P N P The electrons in the emitter are pushed into the base.
PNP Since the base is thin and only slightly doped therefore a very
E C
small fraction (say, 1%) of the incoming electrons combine
e with the holes. Moreover, the electrons are rushing towards
B the collector.
IE IC
IE IC IB The electrons collected by the collector move towards
IB
the positive terminal of the collector-base battery. The defi-
e
+ – + – ciency of these electrons is made up by the electrons released
+ – + – VEB VCB from the negative terminal of the emitter-base battery. Thus,
VEB VCB
we conclude that the current is carried by electrons both in the
(a) (b) external circuit as well as inside the transistor.

Fig. 11 IE = IC + IB
SOLIDS AND SEMICONDUCTOR DEVICES, ELECTROMAGNETIC WAVES AND PRINCIPLES OF COMMUNICATION 767

16. TRANSISTOR AS COMMON BASE AMPLIFIER If we assume that 5% of emitter current appears as
(i) NPN Transistor as Common Base Amplifier base current due to electron-hole combination in base, then
95% of the emitter current flows as collector current.
NPN IB = 5% of IE = 0.05 IE
IE E C IC
and IC = 95% of IE = 0.95 IE
IC
B

ICRL
The collector current IC flowing through the load re-
RL sistance RL produces a potential drop ICRL across the load
VCB
+
IC resistance RL. This voltage drop ICRL is clearly in opposition
IB +
Input to the applied voltage VCC.
VCC
Output signal
signal ∴ Net collector voltage, VCB = VCC – ICRL ...(1)

IC (Amplified)
– +
When the input signal (signal to be amplified) is fed to
IE IC
VEE the emitter-base circuit, it will change the emitter voltage
and hence the emitter current. This, in turn, will change the
Fig. 13 collector current. This will vary the collector voltage VCB in
accordance with equation (1). This variation of collector volt-
IB = 5% of IE = 0.05 IE
age will appear as an amplified output.
and IC = 95% of IE = 0.95 IE
Phase relationship between the output and in-
The collector current IC flowing through the load re- put signals
sistance RL produces a potential drop ICRL across the load
When PNP transistor is used as common base
resistance RL. This voltage drop ICRL is clearly in opposition amplifier, the output and input signals are in the same
to the applied voltage VCC. phase. The output signal (amplified signal) follows exactly
∴ Net collector voltage, VCB = VCC – ICRL ...(1) all the variations of the input signal.
When the input signal (signal to be amplified) is fed to
17. VARIOUS GAINS IN COMMON BASE
the emitter-base circuit, it will change the emitter voltage AMPLIFIER
and hence the emitter current. This, in turn, will change the
(1) Current amplification factor or current gain
collector current. This will vary the collector voltage VCB in
Its value depends upon whether the current is direct
accordance with equation (1). This variation of collector volt-
current or alternating current.
age will appear as an amplified output.
(i) dc current gain (α αdc)
Phase relationship between the output and in-
It is the ratio of the collector current (I C) to the
put signals
emitter current (IE) at constant collector voltage.
When NPN transistor is used as common base
FI I
amplifier, the output and input signals are in the same
phase. The output signal (amplified signal) follows exactly
αdc = GH I JK
C
E V
CB
all the variations of the input signal. The d.c. current gain is always less than one. Its value
(ii) PNP Transistor as Common Base Amplifier generally varies from 0.95 to 0.98.
(ii) ac current gain (α αac)
IE PNP It is the ratio of change in collector current (∆ ∆IC)
IC
E C to the change in emitter current (∆ ∆IE) at constant col-
+
lector voltage.
B RL ICRL
F ∆I I
IB
VCB

– ∴ αac = GH ∆I JK
C
E V
Input CB
VCC
signal Output signal (2) ac voltage gain
It is the ratio of change in output voltage to the
+
IC (Amplified)
IE + – I IC change in input voltage. It is denoted by Av.
VEE E
∆VCB
ac voltage gain, Av =
Fig. 14 ∆Vi
Using Kirchhoff’s first law, IE = IB + IC. Let R 0 and R i represent the output and input
resistances respectively of the amplifier circuit.
768 COMPREHENSIVE OBJECTIVE PHYSICS

∆IC F I FR I and hence the emitter current. This, in turn, will change the
Then, Av =
∆IC × R o
∆IE × R i
or Av =
∆IE
GH JK GH R JK
o
i
collector current. This will vary the collector voltage VCB in
accordance with equation (1). This variation of collector voltage
or Av = αac × Resistance gain will appear as an amplified output.
(3) ac power gain Phase relationship between the output and input
It is the ratio of change in output power to the signals
change in input power. When NPN transistor is used as common emitter
ac power gain change in output power amplifier, the output and input signals are 180° out of
=
change in input power phase.

∆VCB × ∆IC ∆VCB F I F ∆I I (ii) PNP Transistor as Common Emitter Ampli-


=
∆Vi × ∆IE
=
∆Vi
GH JK GH ∆I JK
C
E
fier

= Av × αac IC

∆I2C × R o F ∆I I 2
Ro PNP +
Again, power gain =
∆I2E × Ri
= GH ∆I JK
C
E
×
Ri
B
C

E RL I CR L
= α2ac × resistance gain VCE
Since the current gain is less than one therefore the –

IE
power gain is less than the voltage gain. Input VCC
Output signal
signal (Amplified)
18. TRANSISTOR AS COMMON EMITTER AMPLI-
+
FIER
IC
(i) NPN Transistor as Common Emitter Amplifier
– + IB
VBB
IC
NPN – Fig. 16
IB C
B
I CR L Using Kirchhoff’s first law, IE = IB + IC .
RL
E If we assume that 5% of emitter current appears as
VCE + base current due to electron-hole combination in base, then
+
IE 95% of the emitter current flows as collector current.
Input Output signal
signal
VCC
(Amplified) IB = 5% of IE = 0.05IE

and IC = 95% of IE = 0.95IE
+ – IB IC The collector current IC flowing through the load re-
VBB
sistance RL produces a potential drop ICRL across the load
resistance RL. This voltage drop ICRL is clearly in opposition
Fig. 15
to the applied voltage VCC .
Using Kirchhoff ’s first law, IE = IB + IC. ∴ Net collector voltage, VCE = VCC – ICRL ...(1)
If we assume that 5% of emitter current appears as When the input signal (signal to be amplified) is fed to
base current due to electron-hole combination in base, then the base-emitter circuit, it will change the base voltage and
95% of the emitter current flows as collector current. hence the base current. This, in turn, will change the collec-
IB = 5% of IE = 0.05 IE tor current. This will vary the collector voltage VCE in accord-
and IC = 95% of IE = 0.95 IE ance with equation (1). This variation of collector voltage will
The collector current IC flowing through the load re- appear as an amplified output.
sistance RL produces a potential drop ICRL across the load Phase relationship between the output and in-
resistance RL. This voltage drop ICRL is clearly in opposition put signals
to the applied voltage VCC. When PNP transistor is used as common emitter
∴ Net collector voltage, VCE = VCC – ICRL ...(1) amplifier, the output and input signals are 180° out of
When the input signal (signal to be amplified) is fed to phase.
the base-emitter circuit it will change the base-emitter voltage
SOLIDS AND SEMICONDUCTOR DEVICES, ELECTROMAGNETIC WAVES AND PRINCIPLES OF COMMUNICATION 769

19. VARIOUS GAINS IN COMMON EMITTER Since the current gain is larger than one therefore the
AMPLIFIER power gain is also larger than the voltage gain.
(1) Current amplification factor or current gain 20. RELATION BETWEEN α AND β
Its value depends upon whether the current is direct (i) for ac values
current or alternating current. In both the common base and common emitter circuits,
(i) dc current gain (β
βdc) the emitter current is the sum of the collector current and the
base current.
It is the ratio of the collector current (IC) to the
∴ IE = IB + IC
base current (IB) at constant collector voltage. or ∆IE = ∆IB + ∆IC or ∆IB = ∆IE – ∆IC
FI I ∆IC
βdc = GH I JK
C
B V
∆I
βac = C =
∆IC
=
∆IE
=
αac
CE
The dc current gain is much larger than one. Its value ∆IB ∆IE − ∆IC 1 − ∆IC 1 − αac
generally varies from 15 to 50. ∆IE
(ii) ac current gain (ββac) (ii) for dc values
It is the ratio of change in collector current (∆ ∆IC) IC IC I /I α dc
βdc = = = C E =
to the change in base current (∆ ∆IB) at constant collector I
IB IE − IC 1 − C 1 − αdc
voltage. IE
F ∆I I
∴ βac = GH ∆I JK
C
B V
21. HOW TO CONVERT A NUMBER FROM DECI-
MAL SYSTEM TO BINARY SYSTEM ?
CE
For converting a decimal number to a binary number,
(2) ac voltage gain
we use a method called double method. This method is
It is the ratio of change in output voltage to the summed up below :
change in input voltage. It is denoted by Av. Go on dividing the number and the successive quotients
by 2 till the quotient is zero. Write the remainders obtained in
∆VCE
ac voltage gain, Av = successive divisions in the reverse order. This combination of
∆Vi remainders represents the required number in the binary
Let R 0 and R i represent the output and input system.
resistances respectively of the amplifier circuit. Illustration 1. Let us find the binary equivalent of (8)10 .
∆IC × R o
Then Av = 2 8
∆IB × R i
F ∆I I F R I
=G
2 4–0 The remainders in reverse order are 1, 0, 0 and 0.

H ∆I JK GH R JK
C o
or Av 2 2–0 So, the required binary number is 1 0 0 0.
B i
2 1–0
or Av = βac × Resistance gain
(3) ac power gain 0–1
It is the ratio of change in output power to the
change in input power. ∴ (8)10 = (1 0 0 0)2
This equation could also be written as :
ac power gain =
change in output power
(8)decimal = (1 0 0 0)binary
change in input power
Illustration 2. Let us find the binary equivalent of
∆VCE × ∆IC ∆VCE F I F ∆I I
=
∆Vi × ∆IB
=
∆Vi
GH JK GH ∆I JK
C
B
(23)10 .

2 23
= A0 × βac
2 11 – 1
∆I2C × R o
Again, power gain = 2 5–1 The remainders in reverse order are 1, 0, 1, 1 and 1.
∆I2B × Ri
FG ∆I IJ
C
2
Ro
2 2–1 So, the required binary number is 1 0 1 1 1.
=
H ∆I K
B Ri
× 2 1–0 ∴ (23)10 = (1 0 1 1 1)2

= β 2ac × resistance gain 0–1


770 COMPREHENSIVE OBJECTIVE PHYSICS

22. HOW TO WRITE THE DECIMAL EQUIVALENT OF When count at a particular place increases to highest
A GIVEN BINARY NUMBER ? digit of the system, 1 is to be added to the next place. This 1
In order to understand this conversion, let us first which is to be added to the next place is called carry.
refresh our memory about the ‘weightage’ that we give to Let us now apply the concept of carry to binary num-
different digits used to represent a number in the decimal bers. When 1 is added to 1, the result increases beyond the
system. As an illustration, let us consider a number 3425. highest available digit i.e., 1 in the binary system. So, we write
The following table gives the weightage of different digits in 0 and carry 1 to the next column.
this number. ∴ 1 + 1 = 10
Let us now learn the technique of finding the binary
digit weightage
equivalent of a fractional decimal number. The given frac-
5 100 Clearly, tional decimal number is to be continuously multiplied by 2,
2 101 3425 = 3 × 103 + 4 × 102 + 2 × 101 + 5 × 100 recording each time a carry in the integer position.The process
4 102 is to be continued till we get either enough bits of binary equiva-
lent or zero.This carries in the forward order would give the
3 103
binary equivalent.
As is clear from the above table, a digit in ‘unit place’ Illustration 5. Let us determine the binary equivalent
has a lesser weightage than a digit in the ‘tens place’ and so of (0.625)10
on. 0.625 × 2 = 1.25 = 0.25 with a carry of 1
Let us now turn our attention to the weightage of dif- 0.25 × 2 = 0.50 = 0.50 with a carry of 0
ferent digits in a binary number. In a binary number, as we
0.50 × 2 = 1.00 = 0 with a carry of 1
proceed from right to left, the magnitude of the digit
So, the binary equivalent is (0.101)2.
as converted to decimal system increases by a factor of
Note. While the point used in decimal fraction is called deci-
2 for each digit.
mal point, the point used in binary fraction is called binary point.
Illustration 3. A binary number 1 0 1 1 1 means
1 × 20 + 1 × 21 + 1 × 22 + 0 × 23 + 1 × 24 24. THE OR GATE
So, it would be written as 1 + 2 + 4 + 0 + 16 = 23 in the The truth table is as under :
base of ten.
A B Y
∴ (1 0 1 1 1)2 = (23)10
0 0 0
Note. In a binary number, the first digit from the extreme
right is called the least significant bit (LSB). Similarly, the first 0 1 1
digit from the extreme left is called the most significant bit (MSB). 1 0 1
Illustration 4. A binary number 1 1 0 0 1 1 0 1 means 1 1 1
1 × 20 + 0 × 21 + 1 × 22 + 1 × 23 + 0 × 24
The OR operation is denoted by + in Boolean algebra.
+ 0 × 25 + 1 × 26 + 1 × 27.
∴ A+B=Y
So it would be written as 1 + 0 + 4 + 8 + 0 + 0 + 64 +
128 = 205. It is read as ‘Y equals A OR B’.
The OR gate can
23. HOW TO CONVERT A FRACTIONAL DECIMAL A
have any number of
NUMBER INTO A BINARY NUMBER ?
inputs and one output.
In order to understand the method of this conversion, However, for the sake of Y
B
let us first understand the mathematical technique of “carry” simplicity and conveni-
in the decimal number system. Suppose the speedometer of ence, we are considering
your scooter reads 00999. When the scooter covers the next a two-input OR gate. The Fig. 17
kilometre, the reading is 01000. Clearly, when 1 is added to 9 logic symbol is shown in
in the unit place, we get zero in the unit place. Now, 1 is to be Fig. 17.
added to 9 in the tens place. So, we conclude the following :
If either input A or B or both are high (1), then the OR
gate gives a high output, otherwise the output is low (0).
SOLIDS AND SEMICONDUCTOR DEVICES, ELECTROMAGNETIC WAVES AND PRINCIPLES OF COMMUNICATION 771

25. THE AND GATE Truth Table


The truth table is as under : A B Y′ Y

A B Y 0 0 0 1
0 0 0 0 1 0 1
0 1 0
1 0 0 1
1 0 0
1 1 1 0
1 1 1
The truth table A
The AND operation is denoted by ( . ) in Boolean corresponding to the Y
Algebra. NAND gate can be ob- B
∴ A.B=Y tained by logically using
Fig. 21
It is read as ‘Y equals A AND B’. the truth table of AND
The AND gate and NOT gates.
A
can have any number of The logic symbol of NAND gate is shown in Fig. 21.
inputs but only one out- Notice how the symbol is the same for the AND gate but with
put. For the sake of sim- Y a small circle at the output. This small circle is always taken
plicity and convenience, B to indicate a NOT or INVERTER operation.
we shall consider a two- Fig. 18 The NAND gate is expressed as
input AND gate. The Y = A . B or simply AB and is read as NOT A AND
logic symbol is shown in Fig. 18.
B.
An AND gate gives a high output (1) if input A and
The truth table for NAND gate :
input B are both high (1) ; otherwise the output is low.
An AND gate gives a high output (1) if input A and Input Input Output
input B are both high (1) ; otherwise the output is low. A B Y
26. THE NOT GATE 0 0 1
The truth table is as under :
0 1 1
Input (A) Output (Y)
1 0 1
0 1
1 1 0
1 0
The NAND gate is a very useful gate in itself. But it is
The function of the NOT widely used because all other gates can be constructed using
(inverter) gate can be repre- only NAND gates. As an example, if a NOT gate is needed, it
sented briefly by the Boolean ex- A Y can be made from a NAND gate in two ways :
pression as
A =Y + V(1)
Fig. 19
where A means A (bar). A
In words “not A equals Y”. Y
B
27. THE NAND (NOT AND) GATE
Fig. 22
A
(a) One input such as A is permanently tied (connected)
Y´ Y high so that A is 1 in the truth table. In this case, the only
B lines possible are lines 3 and 4, so if B is high, the output Y is
low (0) and if B is low, Y is high (1).
Fig. 20
772 COMPREHENSIVE OBJECTIVE PHYSICS

(b) Both inputs may be tied (connected) together so Truth table of NOR gate
that inputs A and B must both be high or low together (Fig. 23).
A B Y
This means that only lines 1 and 4 of the truth table are pos-
sible and so, if A and B are both high, Y is low. If A and B are 0 0 1
both low, Y is high. However, this method is not a good elec- 0 1 0
tronic practice and should not be used.
1 0 0
A 1 1 0
Y
B 29. EOR GATE OR XOR GATE
This EOR gate is an abbreviation of exclusive OR gate.
Fig. 23 Its symbol is shown in Fig. 26. Its truth table is as follows :
28. THE NOR (NOT OR) GATE A B Y
It is equivalent to an OR gate followed by a NOT gate
0 0 0
(INVERTER), i.e., all the outputs of the OR gate are inverted
(changed from 0 to 1 or 1 to 0). So, the truth table for the NOR 0 1 1
gate is opposite to the OR gate.
1 0 1
A 1 1 0
Y´ Y
The EOR gate gives a high output if either input A or
B
input B but not both are high. Otherwise, it gives a low input.
The output Y given by the EOR gate is given by
Fig. 24 Y=A B + A B
When the output Y′ of an OR gate is connected to the
A
input of a NOT gate, the circuit is termed as the NOR gate.
The truth table of a NOR gate is obtained from the truth tables
of OR and NOT gates as shown. Y
Truth table B

A B Y′ Y
Fig. 26
0 0 0 1
30. AMPERE—MAXWELL’S CIRCUITAL LAW

z
0 1 1 0
→ → dφE
1 0 1 0 B . dl = µ 0I + µ 0ε 0
dt
1 1 1 0
31. MAXWELL’S EQUATIONS
The logic symbol of a NOR gate is shown in Fig. 25.
I. Maxwell’s first equation z →
E . dS =
→ q
ε0

II. Maxwell’s second equation z S


→ →
B . dS = 0

Fig. 25
III. Maxwell’s third equation
z C
→ →
E . dl = −
d
dt z
S
→ →
B . dS

The NOR gate is expressed as


Y = A + B and is read as A or B negated.
IV. Maxwell’s fourth equation
z → → LM
N
B . dl = µ 0 I + ε 0
dφ E
dt
OP
Q
SOLIDS AND SEMICONDUCTOR DEVICES, ELECTROMAGNETIC WAVES AND PRINCIPLES OF COMMUNICATION 773

32. THE SPECTRUM OF ELECTROMAGNETIC where µ0 and ε0 are the permeability and permit-
RADIATION tivity respectively of free space.
6. Speed of electromagnetic waves in a material me-
Frequency Name of Photon Wavelength, dium is given by :
Hz Radiation Energy, eV m
1
v=
µε
22
10
7 –13
21 10 10
10
Gamma Rays 6 –12
where µ and ε are the absolute permeability and
10 10
the absolute permittivity respectively of the me-
20
10
dium.
5 –11
10
19 10 10
X-Rays
10
18 10
4
10
–10
7. Electromagnetic waves obey the principle of su-
17 10
3
10
–9
per-position.
10
16 10
2
10
–8 8. Electromagnetic waves carry energy as they propa-
10 Ultraviolet
Visible
–7 gate through space. This energy is divided equally
10
15 10 10 → →
0 –6 between E and B .
10
14 10 10
–1 –5 9. Electromagnetic waves can transfer energy as well
13 10 10
10 Infrared
–2 –4
as momentum to objects placed in their path.
10
12 10 10 → →
–3 –3 10. The relative magnitudes of E and B in empty space
11 10 10
10
–4 –2
are related by
10
10 Microwave 10 10
E
c=
–5 –1
10
9 10 10 .
–6
10
0 B
10
8
TV, FM 10
7 10
–7
10
1 34. ELEMENTS OF A COMMUNICATION SYSTEM
10
6 10
–8
10
2 Fig. 28 depicts the elements of a communication sys-
10 Standard Broadcast
–9 3 tem. There are three essential parts of any communication
5 10 10
10
Radio Frequency –10 4
system, the transmitter, transmission channel and receiver.
10
4 10 10 Each part plays a particular role in signal transmission, as
10
3 10
–11
10
5
follows :

Input Transmitted Received Output


Fig. 27 signal signal signal signal
Source Transmission
33. PROPERTIES OF ELECTROMAGNETIC WAVES Transmitter Receiver Destination
channel
1. Electromagnetic waves are produced by acceler-
ated charges.
Noise, interference
2. Electromagnetic waves propagate in the form of
and distortion
varying electric and magnetic fields. The fields are
not only perpendicular to each other but also to
the direction of propagation of the wave. This prop- Fig. 28
erty can be summarised by saying that the elec-
The transmitter processes the input signal to
tromagnetic waves are transverse waves.
produce a suitable transmitted signal suited to the charac-
3. Electromagnetic waves do not require a material
teristics of the transmission channel. Signal processing for
medium for their propagation. This is very much
transmission almost always involves modulation and may
unlike the mechanical waves which cannot
propagate without a material medium. also include coding.
4. The velocity of electromagnetic waves in free space is The transmission channel is the electrical medium
equal to the velocity of light in free space. that bridges the distance from source to destination. It may
5. Speed of electromagnetic waves in free space is be a pair of wires, a coaxial cable, or a radio wave or laser
given by : beam. Every channel introduces some amount of transmis-
sion loss or attenuation. So, the signal power progressively
1
c= decreases with increasing distance.
µ 0 ε0
774 COMPREHENSIVE OBJECTIVE PHYSICS

The receiver operates on the output signal from the The audio frequencies range from 20 Hz to 20 kHz.
channel in preparation for delivery to the transducer at the Suppose a frequency of 20 kHz is to be radiated directly into
destination. Receiver operations include amplification to com- space. For this,
pensate for transmission loss. These also include demodula- 3 × 108
Length of antenna = m = 15000 m = 15 km.
tion and decoding to reverse the signal-processing performed 20 × 103
at the transmitter. Filtering is another important function This is too long an antenna to be constructed practi-
at the receiver. cally. So, it is impracticable to radiate audio signal directly
Fig. 28 represents one-way or simplex (SX) trans- into space.
mission. Two-way communication of course requires a Let us now calculate the length of the antenna if a
transmitter and receiver at each end. A full-duplex (FDX) carrier wave of, say, 1000 kHz is used to carry the signal.
system has a channel that allows simultaneous trans- 3 × 108
mission in both directions. A half-duplex (HDX) system allows Length of antenna = m = 300 m
106
transmission in either direction but not at the same time. An antenna of 300 m length can be easily constructed.
35. ADVANTAGES AND DISADVANTAGES OF 2. Wireless communication
DIGITAL COMMUNICATION SYSTEMS OVER One desirable feature of radio transmission is that it
ANALOG COMMUNICATION SYSTEMS should be carried without wires (i.e.,) radiated into space. At
(a) A common format for encoding different kinds of audio frequencies, radiation is not practicable because the
message signals (e.g., speech signal, video signal, computer efficiency of radiation is poor. However, efficient radiation
data etc.) for the purpose of transmission. of electrical energy is possible at high frequencies (> 20
(b) An improved security of message. kHz). For this reason, modulation is always done in commu-
(c) Increased immunity to noise and external inter- nication systems.
ference. 3. Operating range
(d) Flexibility in configuring digital communication The energy of a wave depends upon its frequency. The
systems. greater the frequency of the wave, the greater is the energy
Disadvantages possessed by it. As the audio signal frequencies are small,
these cannot be transmitted over large distances if radiated
(a) Increased transmission bandwidth
directly into space. The only practical solution is to modu-
(b) Increased system complexity. late a high frequency carrier wave with audio signal and
36. UNDESIRABLE EFFECTS IN THE COURSE OF permit the transmission to occur at this high frequency (car-
SIGNAL TRANSMISSION rier frequency).
(a) Attenuation 38. AMPLITUDE MODULATION
(b) Distortion In amplitude modulation, only the amplitude of the
(c) Interference carrier wave is changed in accordance with the intensity of
(d) Noise. the signal. However, the frequency of the modulated wave
37. MODULATION remains the same as the carrier frequency. Fig. 29 shows
The velocity of electromagnetic waves is 3 × 108 m s–1. the principle of amplitude modulation, (i) shows the audio
On the other hand, the velocity of sound waves is 332 m s–1. electrical signal, whereas (ii) shows the carrier wave of con-
Clearly, sound waves cannot be used to transmit intelligence stant amplitude and (iii) shows the amplitude modulated
to far off places. Only the electromagnetic waves can do the wave.
job. Note that the amplitude of both positive and negative
Modulation is extremely necessary in communication half cycles of carrier wave are changed in accordance with
system due to the following reasons : the signal. For instance, when the signal is increasing in
1. Practical Antenna Length (L) the positive sense, the amplitude of carrier wave also
When free space is the communication channel, an- increases. On the other hand, during negative half cycle of
tennas radiate and receive the signal. Theory shows that the signal, the amplitude of carrier wave decreases. Amplitude
the antennas operate effectively only when their dimensions modulation is done by an electronic circuit called modulator.
are of the order of the magnitude of wavelength of the signal The following points are worth noting in amplitude
being transmitted. modulation :
v 3 × 108 (i) The amplitude of the carrier wave changes accord-
Now, L=λ= = Hz ing to the intensity of the signal.
ν ν
SOLIDS AND SEMICONDUCTOR DEVICES, ELECTROMAGNETIC WAVES AND PRINCIPLES OF COMMUNICATION 775

(ii) The amplitude variation of the carrier wave is at The amplitude of the carrier remains unchanged at
the signal frequency fs . all times. In other words, the amplitude of the modulated
(iii) The frequency of the amplitude modulated wave re- wave remains the same as the amplitude of the carrier wave.
mains the same, i.e., carrier frequency, fc . The frequency of the carrier is made to fluctuate sym-
metrically above and below its unmodulated frequency. As
es es an example, a carrier frequency, of 1000 kHz may be caused
to swing between 925 kHz and 1075 kHz or any other amount
t t chosen in accordance with the signal voltage. In frequency
modulation, the deviation of the carrier frequency from its
average value is proportional to the instantaneous amplitude
(i) Signal (ii) Carrier of the modulating signal. When the signal voltage is zero,
the carrier frequency is unchanged. When the signal
e
approaches its positive peaks, the carrier frequency is
increased to maximum as indicated by the closely spaced
cycles. However, during the negative peaks of signal, the
t carrier frequency is reduced to minimum as shown by widely
spaced cycles.
41. NECESSITY OF FREQUENCY MODULATION
1. Various electrical machines and noises cause am-
(iii) AM Wave plitude disturbance in the transmission of ampli-
tude-modulated wave. This makes the reception
Fig. 29
noisy. So, there is a need for different type of modu-
39. DISADVANTAGES OF AMPLITUDE MODU- lation which can reduce the noise factor. Frequency
LATION modulation (FM) was proposed as a means of im-
(i) Low efficiency proving the signal-to-noise ratio of a radio system.
(ii) Noisy reception The first practical system was put forward in 1936
(iii) Small operating range as an alternative to AM in an effort to make radio
(iv) Reproduction is not of high fidelity. transmissions more resistant to noise.
40. FREQUENCY MODULATION 2. Fidelity or audio quality of amplitude modulated
transmission is poor. This type of transmission is
also not good for musical programmes. There is a
Modulating need to eliminate amplitude-sensitive noise. This
Wave is possible if we eliminate amplitude variation. In
other words, there is a need to keep the amplitude
of the carrier constant. This is precisely what we
Fig. 30
do in frequency modulation.
42. PULSE MODULATION
Frequency Pulse-amplitude modulation (PAM). The
Modulation modulating signal is sampled at the basic rate, usually 1/2
fm , where fm is the maximum value. The amplitude of the
Fig. 31
modulation determines the amplitude of the transmitted
pulse. The band width will be great for exact pulse waveform,
but may be reduced by a low-pass filter, thereby somewhat
rounding the pulses. As long as the amplitude at the pulse
intervals is preserved, no distortion will be introduced.

Fig. 32
776 COMPREHENSIVE OBJECTIVE PHYSICS

transmit and receive electronically transmitted data. It is


the key that unlocks the world of the internet and its World
Modulation Wide Web, commercial online services, electronic mail (E-
mail), and bulletin board systems (BBSes).
Types of Modems
(i) External modem (ii) Internal modem
(iii) PC Card modem
PAM 44. IMPORTANT TERMS USED IN THE STUDY OF
SKY WAVE PROPAGATION
(i) The critical frequency (fc) for a given layer is
the highest frequency that will be returned down to Earth
by that layer after having been beamed straight up at it. It
is important to realise that there is such a maximum, and it
PPM is also necessary to know its value under a given set of
conditions. This value changes with these conditions.

Projected
path
PDM

Ionised layer
PCM

Fig. 33 Actual
Virtual
Actual height
Pulse-position modulation (PPM). The timing path height
or position of the pulse is varied around a fixed mean value
and conforms to the signal amplitude at the time of sam-
Ground surface
pling. A positive signal may set the pulse ahead, a negative
signal will set the pulse behind the reference time. In recep- Fig. 34
tion, the pulse shape received is not important. The time of
arrival is the only information necessary for interpretation A wave will be bent downward provided that the rate
of the signal. of change of ionisation density is sufficient.
(ii) The maximum usable frequency, or MUF, is
Pulse-duration modulation (PDM). It is also
also a limiting frequency, but this time for some specific
referred to as pulse-width modulation. It varies the pulse
angle of incidence other than the normal. In fact, if the angle
length or duration around a fixed value. This may be done
of incidence is θ, it follows that
by shifting either the leading or trailing edge, or both, as a
function of the sampled amplitude. Greater channel band critical frequency
MUF = = fc sec θ
width is required than for PAM. Design must be co-ordinated cos θ
to prevent overlap of the variable length pulses in multiplex This is the so-called secant law, and it is very useful
use. The reception of such a wave is simple. The information in making preliminary calculations for a specific MUF.
can be recovered from the demodulated pulses by passing Strictly speaking, it applies only to a flat Earth and a flat
them through a low-pass filter. reflecting layer.
Pulse-code modulation (PCM). It translates the (iii) The skip distance is the shortest distance from
sampled amplitude data into a code. The code is transmitted a transmitter, measured along the surface of the Earth, at
as a succession of pulses and spaces. The system enjoys which a sky wave of fixed frequency (more than fc) will be
greater freedom from noise and interfering signals than is returned to Earth.
possible with most other pulse systems.
45. HEIGHT OF TRANSMITTING ANTENNA
43. MODEM
A modem (a modulator/demodulator) lets you connect If the broadcast is made from a height h above the
your computer to a standard telephone line so that you can ground, no reception by direct signals is possible beyond the
SOLIDS AND SEMICONDUCTOR DEVICES, ELECTROMAGNETIC WAVES AND PRINCIPLES OF COMMUNICATION 777

points A and B. The distance Illustration 7. The electrical circuit used to get
P
( or ) upto which signals smooth DC output from a rectifier circuit is called
can be received can be calculated (a) Filter (b) Oscillator
in terms of h and the radius R (c) Logic gates (d) Amplifier.
of the Earth. h [Karnataka CET 2000, 2003]
In the right-angled trian- Sol. ‘Filter’ filters out the fluctuations in the output
gle CBP, O of the rectifier.
CP2 = CB2 + BP2 So, (a) is the right choice.
But CP = R + h Illustration 8. In the case of constants α and β of a
R
and PB ≈ OB = d A B transistor
R
∴ (R + h)2 = R2 + d2 (a) α = β (b) β < 1 α > 1
or R2 + h2 + 2Rh = R2 + d2
C (c) αβ = 1 (d) β > 1 α < 1.
or d2 = h2 + 2Rh Fig. 35 [Karnataka CET 2003]
It is clear from this equation that if h is large, d will Sol. The value of β may range from 15 to 50. On the
other hand, the value of α varies from 0.95 to 0.98.
be large. This explains as to why the television broadcasts
are made from tall antennas. So, (d) is the right choice.
Illustration 9. A n-p-n transistor conducts when
46. RADIO FREQUENCY BANDS (a) both collector and emitter are positive with respect
Band Frequency Wavelength
to the base
range range (b) collector is positive and emitter is negative with
respect to the base
Extremely low frequency (ELF) < 3 kHz > 100 km
(c) collector is positive and emitter is at same pot-
Very low frequency (VLF) 3-30 Hz 10-100 km ential as the base
Low frequency (LF) 30-300 kHz 1-10 km (d) both collector and emitter are negative with
respect to the base. [All India PM/PD 2003]
Medium frequency (MF) 300 kHz-3 MHz 100 m-1 km
Sol. Only a positive collector can collect free electrons
High frequency (HF) 3-30 MHz 10-100 m from the emitter.
Very high frequency (VHF) 30-300 MHz 1-10 m So, (b) is the right choice.
Illustration 10. In a n-p-n transistor circuit, the
Ultra high frequency (UHF) 300 MHz-3 GHz 10 cm-1 m
collector current is 10 mA. If 90% of the electrons emitted
Superhigh frequency (SHF) 3-30 GHz 1-10 cm reach the collector, the emitter current (I E) and base cur-
Extremely high frequency (EHF) 30-300 GHz 1 mm-1 cm rent (IB) are given by
(a) IE = 11 mA ; IB = 1 mA
ILLUSTRATIONS (b) IE = 1 mA ; IB = 11 mA
Illustration 6. Suppose the base J1 J2
(c) IE = 9 mA ; IB = – 1 mA
of the PNP transistor is left unconnected.
In which direction would the current flow P N P
P
(d) IE = – 1 mA ; IB = 9 mA.
in the transistor ? [Karnataka CET 2001]
(a) Towards J1 100
Sol. IE = × 10 mA ≈ 11 mA
(b) Towards J2 90
(c) Away from J2 Now, IB = IE – IC = (11 – 10) mA
Fig. 36
= 1 mA
(d) The current would not flow.
So, (a) is the right choice.
Sol. With the base unconnected, there would be no
current flowing through the transistor in any direction. This Illustration 11. If a full wave rectifier circuit is op-
is because if we tend to flow the current from E to C, the erating from 50 Hz mains, the fundamental frequency in
junction J2 is reverse-biased. If we tend to flow the current the ripple will be
from C to E, the junction J1 would become reverse-biased. (a) 25 Hz (b) 50 Hz
So, (d) is the right choice. (c) 70.7 Hz (d) 100 Hz.
[All India PM/PD 2003 ; AIEEE 2005]
778 COMPREHENSIVE OBJECTIVE PHYSICS

Sol. In a full wave rectifier, the fundamental fre- 10


quency in ripple is twice of input frequency. (a) V (b) 10 V
π
So, (d) is the right choice.
Illustration 12. The difference in the variation of 20
(c) V (d) 10/ 2 V.
resistance with temperature in a metal and a semiconduc- π
tor arises essentially due to the difference in the [All India PM/PD 2004]
(a) variation of scattering mechanism with
Vm 10
temperature Solution. Vdc = =
π π
(b) crystal structure So, (a) is the right choice.
(c) variation of the number of charge carriers with Illustration 16. An electromagnetic wave of fre-
temperature quency ν = 3.0 MHz passes from vacuum into a dielectric
(d) type of bonding. [AIEEE 2003] medium with permitivity ε = 4.0.
Sol. In the case of metals, the number of collisions Then
increase with the increase in temperature. So, resistance (a) wavelength is halved and frequency remains unchanged
increases. In the case of semiconductors, the number of (b) wavelength is doubled and frequency becomes half
charge carriers increases with increase in temperature. So, (c) wavelength is doubled and the frequency remains un-
(c) is the right choice. changed
Illustration 13. The length of a half wave dipole at (d) wavelength and frequency both remain unchanged.
30 MHz is
[All India PM/PD 2004]
(a) 10 m (b) 50 m
Solution. Energy (hν) is conserved in refraction. So,
(c) 5 m (d) 100 m. ν remains unchanged.
[JIPMER 2003] Speed, v = νλ or v ∝ λ
λ C 8
3 × 10 Since v is halved, therefore λ is halved.
Sol. L= = = m=5m
2 2 ν 2 × 30 × 106 So, (a) is the right choice.
So, (c) is the right choice. Illustration 17. The electrical conductivity of a semi-
Illustration 14. A Ge specimen is doped with Al. conductor increases when electromagnetic radiation of wave-
The concentration of acceptor atoms is ~ 1021 atoms/m3. length shorter than 2480 nm is incident on it. The band gap
Given that the intrinsic concentration of electron-hole pairs in (eV) for the semiconductor is
is ~ 1019/m3, the concentration of electrons in the specimen (a) 0.7 eV (b) 0.5 eV
is : (c) 2.5 eV (d) 1.1 eV.
(a) 1017/m3 (b) 1015/m3 [AIEEE 2005]
(c) 104/m3 (d) 102/m3. [AIIMS 2004]
12375
Solution. ni2 = nenh Sol. E= eV
λ(in Å )
(1019)2 = ne × 1021
or ne = 1017/m3 12375
= eV = 0.5 eV
So, (a) is the right choice. 24800
Illustration 15. The peak voltage in the output of a So, (b) is the right choice.
half-wave diode rectifier fed with a sinusoidal signal with-
out filter is 10 V. The dc component of the output voltage is
SOLIDS AND SEMICONDUCTOR DEVICES, ELECTROMAGNETIC WAVES AND PRINCIPLES OF COMMUNICATION 779

QUESTION BANK

MCQs
SET I

based on
Memory Work, Hard Facts, Important Terms,
Important Formulae etc.

Average time allowed per question is 20 seconds.

1. Which logic gate is represented by the following com- (c) the depletion region is reduced and barrier height is
bination of logic gates ? increased.
(d) both the depletion region and barrier height are increased.
A [AIEEE 2004]
y 6. Of the diodes shown in Fig. 38, which one is reverse
biased ?
B
(a) (b) +5V

Fig. 37 R R

(a) OR (b) NAND – 10 V


(c) AND (d) NOR. [AIIMS 2004]
2. The output of OR gate is 1
(a) if either or both inputs are 1. (c) + 10 V (d)
– 12 V
(b) only if both inputs are 1. R
(c) if either input is zero. R
–5V
(d) if both inputs are zero. [All India PM/PD 2004] +5V
3. The manifestation of band structure in solids is due
Fig. 38
to
[All India PM/PD 2004]
(a) Bohr’s correspondence principle.
7. Zener diode is used for
(b) Pauli’s exclusion principle.
(a) Amplification
(c) Heisenberg’s uncertainty principle. (b) Producing oscillations in an oscillator
(d) Boltzmann’s law. [AIEEE 2004] (c) Stabilisation (d) Rectification.
4. In semiconductors at a room temperature, [All India PM/PD 2005]
(a) the valence band is completely filled and the conduction 8. When npn transistor is used as an amplifier,
band is partially filled. (a) electrons move from collector to base.
(b) the valence band is completely filled. (b) holes move from emitter to base.
(c) the conduction band is completely empty. (c) electrons move from base to collector.
(d) the valence band is partially empty and the conduction band (d) holes move from base to emitter. [AIEEE 2004]
is partially filled. [All India PM/PD 2004] 9. Consider the following statements A and B and iden-
5. When p-n junction diode is forward biased then tify the correct answer.
(a) both the depletion region and barrier height are reduced. A. A zener diode is always connected in reverse bias
(b) the depletion region is widened and barrier height is B. The potential barrier of a p-n junction lies between
reduced. 0.1 V to 0.3 V approximately.
780 COMPREHENSIVE OBJECTIVE PHYSICS

(a) A and B are correct.


(b) A and B are wrong.
(c) A is correct, but B is wrong.
Load Load
(d) A is wrong, but B is correct. [EAMCET 2003]
10. When a PN junction diode is forward biased, the flow
of current across the junction is mainly due to
(d) (e)
(a) drift of charges. (b) diffusion of charges.
(c) both drift and diffusion of charges. Fig. 39
(d) depends on the nature of the material.
16. An NPN transistor is biased to work as an amplifier.
11. A PN junction diode cannot be used Which of the following statements is not correct ?
(a) as rectifier. (a) The electrons go from base region to collector region.
(b) for converting light energy to electrical energy. (b) The electrons go from emitter region to base region.
(c) for getting light radiation. (c) The electrons go from the collector region to base region.
(d) for increasing the amplitude of an ac signal. (d) The holes go from base region to emitter region.
[All India PM/PD 1999] [CMC Vellore 1997]
12. To use a transistor as an amplifier 17. A semiconductor is known to have an electron con-
(a) emitter-base junction is forward biased and collector-base centration of 8 × 1013/cm3 and hole concentration of
junction is reverse biased. 5 × 1012/cm3. The semiconductor is
(b) both junctions are forward biased. (a) N-type. (b) P-type.
(c) both junctions are reverse biased. (c) intrinsic. (d) insulator.
(d) it does not matter how the transistor is biased, it always [Pb. CET 1992]
works an as amplifier.
18. The energy gap is a measure of the strength of
13. When an impurity is doped into an intrinsic semi-
(a) covalent bond. (b) semiconductor.
conductor, the conductivity of the semiconductor
(a) increases. (b) decreases. (c) current. (d) potential difference.

(c) remains the same. (d) becomes zero. 19. Diffusion current in a p-n junction is greater than
[AIIMS 1998] the drift current in magnitude
(a) if the junction is forward-biased.
14. The Boolean expression for AND gate is
(a) A + B = Y (b) A . B = Y (b) if the junction is reverse-biased.
(c) if the junction is unbiased.
(c) A = Y (d) A . B = Y. [AMU 2000]
(d) in no case. [Bharati Vidyapeeth 1999]
15. A capacitor is to be used to provide smoothing for a
20. The possible number of crystal lattices is
half wave rectifier. In which of the following diagrams
is capacitor correctly connected ? (a) 14 (b) 7
(c) 5 (d) 3.
21. A crystal lattice is
(a) a random arrangement of atoms in a crystal.

Load Load (b) an ordered arrangement of atoms inside a crystal.


(c) a random arrangement of molecules but orderly arrange-
ment of atoms.
(a) (b) (d) a piece of crystal.
22. Which of the following is based on photovoltaic effect ?
(a) Solar cell (b) Zener diode

Load (c) Light-emitting diode (d) PNP transistor.


23. Which of the following makes use of gallium arsenide ?
(a) Zener diode (b) PN junction diode
(c) (c) NPN transistor (d) Light-emitting diode.
SOLIDS AND SEMICONDUCTOR DEVICES, ELECTROMAGNETIC WAVES AND PRINCIPLES OF COMMUNICATION 781

24. Which of the following is required for making con- (a) I2


stant voltage power supply ?
(a) P-type semiconductor (b) N-type semiconductor
(c) PNP transistor (d) Zener diode. 0 t
25. Electric conduction in a semiconductor takes place (b) I2
due to
(a) electrons only. (b) holes only.
(c) both electrons and holes. (d) neither electrons nor holes. 0 t
[Pb. PMT 2000] (c) I2

26. Silicon is a semiconductor. If a small amount of As


t
is added to it, then its electrical conductivity
(a) decreases. (b) increases.
0
(c) remains unchanged. (d) becomes zero. (d) I2
[MP PMT 1996] Zero at all times
0
27. The Boolean expression for NAND gate is t
(a) A + B = Y (b) A . B = Y (e) I2

(c) A = Y (d) A . B = Y .
0
[Himachal PMT 1998] t
28. Fig. 40 shows a full-wave rectifier with a smoothing
capacitor. Fig. 42
29. If ne and nh are the number of electrons and holes in
a semiconductor heavily doped with phosphorus, then
Input (a) ne >> nh (b) ne << nh
(c) ne ≤ nh (d) ne = nh.
I1 [MP PMT 2000]
I2 30. Which of the following physical properties is the same
along all directions in an isotropic solid ?
I3 (a) Refractive index (b) Electrical conductivity
(c) Thermal conductivity (d) All of these.
Load
31. S m–1 is the unit of
Fig. 40 (a) hole mobility (b) electron mobility
(c) electrical conductivity (d) none of these.
The currents I1 and I3 vary with time as shown in
Fig. 41. Which of the graphs a – e shown in Fig. 42 32. On increasing the reverse bias to a large value in a
best represents the variation with time of the current P-N junction diode, current
I2 ? (a) increases slowly (b) remains fixed
(c) suddenly increases (d) decreases slowly.
I1 [MP PMT 1994]
33. In an insulator, the forbidden energy gap between
the valence band and conduction band is of the order
of
0 t
(a) 1 MeV (b) 0.1 MeV
I3 (c) 1 eV (d) 5 eV. [MP PET 1996]
34. P-N junction is said to be forward biased, when
(a) the positive pole of the battery is joined to the P-semicon-
0 t ductor and negative pole to the N-semiconductor.
(b) the positive pole of the battery is joined to the N-semicon-
Fig. 41 ductor and negative pole to the P-semiconductor.
782 COMPREHENSIVE OBJECTIVE PHYSICS

(c) the positive pole of the battery is connected to N-semicon- Which set of diagrams shows these traces most accu-
ductor or P-semiconductor. rately ?
(d) a mechanical force is applied in the forward direction.
c.r.o.1 c.r.o.2 c.r.o.3
[MP PET 1994]
35. The valency of an impurity element added to germa-
nium crystal in order to convert in into P-type semi- A
conductor is
(a) 6 (b) 5
(c) 4 (d) 3. B
[Himachal PMT 2000]
36. The majority and minority charge carriers in N-type
semi-conductors are respectively C
(a) electrons and electrons. (b) electrons and holes.
(c) holes and electrons. (d) holes and holes.
D
37. The diffusion current in a p-n junction is
(a) from the n-side to the p-side.
(b) from the p-side to the n-side.
Fig. 44
(c) from the n-side to the p-side if the junction is forward-
biased and in the opposite direction if it is reverse-biased. (a) A (b) B
(d) from the p-side to the n-side if the junction is forward- (c) C (d) D.
biased and in the opposite direction if it is reverse-biased.
[Haryana PMT 1997] 40. An intrinsic semiconductor at absolute zero of tem-
perature behaves as
38. For germanium crystal, the forbidden energy gap in
joule is (a) an insulator. (b) a metallic conductor.
(a) 1.12 × 10–19 (b) 1.76 × 10–19 (c) a superconductor. (d) a semiconductor.
(c) 1.6 × 10–19 [MP PET 2000]
(d) Zero. [National Standard Exam. in Physics 1989]
39. Fig. 43 shows three circuits used to rectify an alter- 41. Iron and silicon wires are heated from 30°C to 50°C.
nating current. A cathode ray oscilloscope (c.r.o.) with The correct statement is that
fixed settings of time-base and Y-gain is connected in
(a) resistance of both wires increases.
turn as shown and produces a different trace in each
case. (b) resistance of both wires decreases.
(c) resistance of iron wire increases and that of silicon wire
decreases.
(d) resistance of iron wire decreases and that of silicon wire
c.r.o.
1 increases. [MP PET 1995]
42. The electrical resistance of the following decreases
with rise in temperature
(a) Metals (b) Semiconductors
(c) Gold (d) Constantan.
c.r.o.
2 [MP PET 1994]
43. A P-N junction has a thickness of the order of
(a) 1 cm (b) 1 mm
(c) 10–6 cm (d) 10–12 cm. [BITS 1990]
44. Platinum and silicon are heated upto 250°C and after
then cooled. In the process of cooling,
(a) resistance of platinum will increase and that of silicon will
c.r.o decrease.
3
(b) resistance of silicon will increase and that of platinum will
decrease.
Fig. 43
SOLIDS AND SEMICONDUCTOR DEVICES, ELECTROMAGNETIC WAVES AND PRINCIPLES OF COMMUNICATION 783

(c) resistance of both will decrease. 52. Fig. 45 shows a bridge


(d) resistance of both will increase. [MP PMT 1994] rectifier with a X
sinusoidal alternating P Q
45. The following Truth Table voltage applied to it, the
A B Y output terminals P and
Q being joined together
0 0 1
by a load resistor.
1 0 0 If the diode X were re- Fig. 45
0 1 0 moved leaving a break in
the circuit, which trace would be seen on a cathode-
1 1 0
ray oscilloscope connected across PQ ?
is for
(a) OR gate (b) AND gate (a) (b)
(c) NOT gate (d) NOR gate.
[CMC LDH 2000] (c) (d)
46. The depletion layer in the p-n junction region is caused
by Fig. 46
(a) diffusion of carriers. (b) drift of electrons.
53. Which statement is correct ?
(c) drift of holes. (d) migration of impurity ions.
(a) N-type germanium is negatively charged and P-type
[All India PM/PD 1994] germanium is positively charged.
47. The resistance of a reverse-biased p-n junction diode (b) Both N-type and P-type germanium are neutral.
is about (c) N-type germanium is positively charged and P-type
(a) 1 ohm (b) 102 ohm germanium is negatively charged.
(c) 103 ohm (d) 106 ohm. (d) Both N-type and P-type germanium are negatively
[MP PMT 2000] charged. [MP PMT 1994]
.
48. If A = 1 and B = 0, then A A + B in the Boolean 54. An example of a semiconductor is
expression is equal to (a) Platinum (b) Germanium
(a) B (b) A . B (c) Quartz (d) Mica. [BIT 1983]
(c) A (d) B . B.
[JIPMER 2002] 55. A semiconductor device is connected in a series cir-
49. In an experiment, the saturation in the plate current cuit with a battery and a resistance. A current is
in a diode is observed at 240 V. But, a student still found to pass through the circuit. If the polarity of
wants to increase the plate current. It can be done, if the battery is reversed, the current drops almost to
(a) the plate voltage is increased further. zero. The device may be
(b) the plate voltage is decreased. (a) a P-type semiconductor. (b) an N-type semiconductor.
(c) the filament current is decreased. (c) a P-N junction. (d) an instrinsic semiconductor.
(d) the filament current is increased. [MNR 1994] [MP PET 1995]
50. The drift current in a p-n junction is 56. To a germanium sample, traces of gallium are added
(a) from the n-side to the p-side. as an impurity. The resultant sample would behave
like
(b) from the p-side to the n-side.
(a) a conductor (b) a p-type semiconductor
(c) from the n-side to the p-side if the junction is forward-
(c) an n-type semiconductor (d) an insulator.
biased and in the opposite direction if it is reverse-biased.
(d) from the p-side to the n-side if the junction is forward- [AIIMS 2003]
biased and in the opposite direction if it is reverse-biased. 57. In Fig. 47, the input is across the B
[CMC LDH 1997] terminals A and C and the output
is across B and D. Then the out-
51. The band gap in Germanium and Silicon in eV re- A
put is C
spectively is
(a) zero.
(a) 0.7, 1.1 (b) 1.1, 0.7
(b) same as the input. D
(c) 1.1, 0 (d) 0, 1.1.
(c) full wave rectified. Fig. 47
[MP PMT 2001 ; MP PET 2001]
(d) half-wave rectified. [All India PM/PD 1994]
784 COMPREHENSIVE OBJECTIVE PHYSICS

IC
58. In the study of transistor as amplifier, if α = and
IE
IC
β= , where IC, IB and IE are the collector, base and
IB (a) (b)
emitter currents, then
1− α 1+ α
(a) β = (b) β =
α α
α α
(c) β = (d) β = . (c) (d)
1− α 1+ α
Fig. 50
[All India PM/PD 2000]
63. Repetitive use of which of the following can help in
59. In an N-type semiconductor, donor valence band is
making many digital circuits ?
(a) above the conduction band of the ho

You might also like